You are on page 1of 541

Editors : 

Sc hrie r,  Robe rt W .


Title :  Inte rna l Me dic ine  Ca s e book ,  The : Re a l P a tie nts ,  Re a l Ans w e rs ,
3rd Edition
Copy r i ght  ©2007  Li ppi ncott  Wi l l i am s  &  Wi l k i ns

>  Fr o nt  o f   B o o k   >  Ed ito r s

Editor
Robe rt W .  Sc hrie r MD
Pr of essor
Depar tm ent  of   Medi ci ne,   U ni v er si ty   of   Col or ado  School   of   Medi ci ne,   Denv er ,
Col or ado

Secondary Editors
Sonya  Se iga fus e
Acqui si ti ons  Edi tor

Fra nny Murphy
Dev el opm ental   Edi tor

Na nc y W inte r
Managi ng  Edi tor

Nic ole  W a lz
Pr oject  Manager

Ka thy Brow n
Manuf actur i ng  Manager

Kimbe rly Sc honbe rge r


Mar k eti ng  Manager

Doug Smoc k
Cr eati v e  Di r ector

Ca ndic e  Ca rta ­Mye rs


Cov er   Desi gner

La s e rw ords  P riva te  Limite d,  Che nna i,  India


Pr oducti on  Ser v i ces

RR Donne lle y Cra w fords ville


Pr i nter

Contributing Authors
Contributing Authors
Toma s  Be rl MD
Pr of essor   of   Medi ci ne
Di v i si on  of   Renal   Di seases  and  Hy per tensi on,   Depar tm ent  of   Medi ci ne,
U ni v er si ty   of   Col or ado  School   of   Medi ci ne,   Denv er   and  Aur or a,   Col or ado

W illia m R.  Brow n MD


Pr of essor   of   Medi ci ne
Di v i si on  of   Gastr oenter ol ogy ,   Depar tm ent  of   Medi ci ne,   U ni v er si ty   of
Col or ado  School   of   Medi ci ne,   Denv er   and  Aur or a,   Col or ado

Staf f   Gastr oenter ol ogi st,   Denv er   Heal th  Medi cal   Center ,   Denv er ,   Col or ado

He nry N.  Cla ma n MD


Pr of essor   of   Medi ci ne
Di v i si on  of   Al l er gy   and  Cl i ni cal   Im m unol ogy ,   Depar tm ent  of   Medi ci ne,
U ni v er si ty   of   Col or ado  School   of   Medi ci ne,   Denv er   and  Aur or a,   Col or ado

Ste phe n C.  Dre s k in MD


Pr of essor   of   Medi ci ne
Di v i si on  of   Al l er gy   and  Cl i ni cal   Im m unol ogy ,   Depar tm ent  of   Medi ci ne,
U ni v er si ty   of   Col or ado  School   of   Medi ci ne,   Denv er   and  Aur or a,   Col or ado

Ste phe n C.  Dre s k in MD


Di r ector
Al l er gy ,   Asthm a,   and  Im m unol ogy   Pr acti ce,   U ni v er si ty   of   Col or ado  Hospi tal ,
Aur or a,   Col or ado

Robe rt W .  Ja ns on MD


Pr of essor   of   Medi ci ne
Di v i si on  of   Rheum atol ogy ,   Depar tm ent  of   Medi ci ne,   U ni v er si ty   of   Col or ado
School   of   Medi ci ne,   Denv er   and  Aur or a,   Col or ado

Robe rt W .  Ja ns on MD


Chi ef
Rheum atol ogy   Secti on,   Denv er   Veter ans  Adm i ni str ati on  Medi cal   Center ,
Denv er ,   Col or ado

JoAnn Linde nfe ld MD


Pr of essor   of   Medi ci ne
Di v i si on  of   Car di ol ogy ,   Depar tm ent  of   Medi ci ne,   U ni v er si ty   of   Col or ado
School   of   Medi ci ne,   Denv er   and  Aur or a,   Col or ado
Ja ne  E.  B.  Re us c h MD
Pr of essor   of   Medi ci ne
Di v i si on  of   Endocr i nol ogy ,   Di abetes,   and  Metabol i sm ,   Depar tm ent  of
Medi ci ne,   U ni v er si ty   of   Col or ado  School   of   Medi ci ne,   Denv er   and  Aur or a,
Col or ado

Ja ne  E.  B.  Re us c h MD


Staf f
Denv er   Veter ans  Adm i ni str ati on  Medi cal   Center ,   Denv er ,   Col or ado

La ure nc e  Robbins  MD


Associ ate  Pr of essor   of   Medi ci ne
Di v i si on  of   Ger i atr i cs,   Depar tm ent  of   Medi ci ne,   U ni v er si ty   of   Col or ado
School   of   Medi ci ne,   Denv er   and  Aur or a,   Col or ado

La ure nc e  Robbins  MD


Associ ate  Chi ef   of   Staf f
Ger i atr i cs  and  Ex tended  Car e  Depar tm ent,   Easter n  Col or ado  Heal th  Car e
Sy stem ,   Depar tm ent  of   Veter ans  Af f ai r s,   Denv er ,   Col or ado

Ire ne  E.  Sc ha ue r MD


Instr uctor
Di v i si on  of   Endocr i nol ogy ,   Di abetes,   and  Metabol i sm ,   Depar tm ent  of
Medi ci ne,   U ni v er si ty   of   Col or ado  School   of   Medi ci ne,   Denv er   and  Aur or a,
Col or ado

Robe rt T.  Sc hoole y MD


Pr of essor   of   Medi ci ne
Head  Di v i si on  of   Inf ecti ous  Di seases,   Depar tm ent  of   Medi ci ne,   U ni v er si ty   of
Cal i f or ni a  School   of   Medi ci ne,   San  Di ego,   Cal i f or ni a

Ma rvin I.  Sc hw a rz MD


Jam es  C.   Cam pbel l   Pr of essor   of   Medi ci ne
Di v i si on  of   Pul m onar y   Di seases  and  Cr i ti cal   Car e  Medi ci ne,   Depar tm ent  of
Medi ci ne,   U ni v er si ty   of   Col or ado  School   of   Medi ci ne,   Denv er   and  Aur or a,
Col or ado

P a ul A.  Se ligma n MD


Pr of essor   of   Medi ci ne
Di v i si on  of   Hem atol ogy ,   Depar tm ent  of   Medi ci ne,   U ni v er si ty   of   Col or ado
School   of   Medi ci ne,   Denv er   and  Aur or a,   Col or ado

Simon Sha k a r MD
Assi stant  Pr of essor   of   Medi ci ne
Di v i si on  of   Car di ol ogy ,   Depar tm ent  of   Medi ci ne,   U ni v er si ty   of   Col or ado
School   of   Medi ci ne,   Denv er   and  Aur or a,   Col or ado

Is a a c  Te ite lba um MD


Pr of essor   of   Medi ci ne
Di v i si on  of   Renal   Di seases  and  Hy per tensi on,   Depar tm ent  of   Medi ci ne,
U ni v er si ty   of   Col or ado  School   of   Medi ci ne,   Denv er   and  Aur or a,   Col or ado

Is a a c  Te ite lba um MD


Di r ector
Acute  and  Hom e  Di al y si s  Pr ogr am s,   U ni v er si ty   of   Col or ado  Hospi tal ,   Denv er
and  Aur or a,   Col or ado

Rona ld Zolty MD
Assi stant  Pr of essor   of   Medi ci ne
Montef i or e  Medi cal   Center ,   Di v i si on  of   Car di ol ogy ,   Depar tm ent  of   Medi ci ne,
Br onx ,   N ew   Yor k
Editors :  Sc hrie r,  Robe rt W .
Title :  Inte rna l Me dic ine  Ca s e book ,  The : Re a l P a tie nts ,  Re a l Ans w e rs ,
3rd Edition
Copy r i ght  ©2007  Li ppi ncott  Wi l l i am s  &  Wi l k i ns

>  Fr o nt  o f   B o o k   >  P r e f a c e

Preface

The  basi s  f or   an  i nter nal   m edi ci ne  casebook   or i gi nated  sev er al   y ear s  ago,
bor n  of   m y   conv i cti on  that  house  staf f   and  students  ar e  sti m ul ated  and
m oti v ated  best  w hen  thei r   l ear ni ng  i s  f ocused  on  r eal   pati ents.   Steps  w er e
tak en,   ther ef or e,   to  augm ent  the  pati ent­or i ented  i nstr ucti on  of   our   house
staf f   and  students.   Thi s  w as  accom pl i shed  i n  l ar ge  par t  by   our   i n­house
publ i cati on  of   a  l ar ge  num ber   of   pati ent  cases  cov er i ng  a  v ar i ety   of   di seases
that  f or m ed  the  cur r i cul um   f or   m edi cal   students  and  the  house  staf f   r otati ng
thr ough  the  m edi cal   w ar ds  and  tak i ng  subspeci al ty   el ecti v es.   These  pati ent
cases  becam e  the  basi s  f or   the  f i r st  edi ti on  of   The  Inter nal   Medi ci ne
Casebook :  Real   Pati ents,   Real   Answ er s.   Si nce  the  f i r st  tw o  edi ti ons  hav e
been  so  w el l   r ecei v ed,   I  am   ev en  m or e  conv i nced  of   the  need  f or   such  a
pati ent­or i ented  educati onal   tool .

In  thi s  thi r d  edi ti on  of   the  Casebook ,   w e  hav e  r ev i sed  and  updated  the
pati ent­or i ented  cases,   w hi ch  cov er   appr ox i m atel y   90  ar eas  of   i nter nal
m edi ci ne.   Fi r st,   the  per ti nent  aspects  of   the  pati ent's  hi stor y   and  phy si cal
ex am i nati on  ar e  pr esented.   Questi ons  ar e  then  posed  about  the  appr opr i ate
di agnosti c  w or k up  and  tr eatm ent.   Thi s  sets  the  stage  f or   a  Socr ati c
appr oach  to  l ear ni ng  betw een  the  attendi ng  phy si ci an  and  the  house  of f i cer
or   m edi cal   student.   The  f or m at  of   the  Casebook   al so  l ends  i tsel f   to  sel f ­
i nstr ucti on  w i th  thi s  questi on­and­answ er   appr oach.

The  Inter nal   Medi ci ne  Casebook :  Real   Pati ents,   Real   Answ er s  has  pr ov ed  to
be  an  i nv al uabl e  l ear ni ng  tool   to  students  and  house  of f i cer s,   and  al so  a
teachi ng  ai d  to  any one  i nv ol v ed  i n  the  educati on  of   f utur e  phy si ci ans.   I  am
gr atef ul   f or   the  ex per ti se  and  suppor t  of   the  author s,   each  of   w hom   i s  an
em i nentl y   qual i f i ed  educator   and  hi ghl y   r egar ded  l eader   i n  hi s  or   her   f i el d.   I
al so  thank   Jan  Dar l i ng  f or   her   edi tor i al   assi stance.
R.   W.   S.
Editors :  Sc hrie r,  Robe rt W .
Title :  Inte rna l Me dic ine  Ca s e book ,  The : Re a l P a tie nts ,  Re a l Ans w e rs ,
3rd Edition
Copy r i ght  ©2007  Li ppi ncott  Wi l l i am s  &  Wi l k i ns

>  T a b le   o f   C o nte nts   >  C ha p te r   1  ­  A lle r g y   a nd   C linic a l  I m m uno lo g y

Chapter 1
Allergy and Clinical Immunology

Ste phe n C.  Dre s k in


He nry N.  Cla ma n

Anaphylaxis
1.   What  i s  the  cl i ni cal   pr esentati on  i n  a  ty pi cal   case  of   anaphy l ax i s?

2.   What  i s  the  under l y i ng  pathophy si ol ogi c  pr ocess?

3.   What  condi ti ons  shoul d  be  consi der ed  i n  the  di f f er enti al   di agnosi s?

P. 2

Discussion
1.   What  i s  the  cl i ni cal   pr esentati on  i n  a  ty pi cal   case  of   anaphy l ax i s?

In  the  m ost  sev er e  cases,   the  cl i ni cal   pr esentati on  consi sts  of   sudden
hy potensi on  w i th  or   w i thout  cutaneous  si gns,   br onchospasm ,   or
l ar y ngeal   obstr ucti on.   Pati ents  occasi onal l y   r epor t  a  “sense  of
i m pendi ng  doom . â€​   Thi s  m ay   occur   w i thi n  m i nutes  of   the  i ngesti on  of   a
speci f i c  f ood,   i njecti on  of   an  anti gen  (e. g. ,   an  anti bi oti c),   or   an  i nsect
sti ng,   and  m ay   be  f atal .   A  l ess  r api d  onset  can  begi n  w i th  ur ti car i a,
angi oedem a,   shor tness  of   br eath,   hoar seness,   and  m oder ate
hy potensi on.   If   the  of f endi ng  substance  has  been  i ngested,   ther e  can  be
abdom i nal   cr am ps,   v om i ti ng,   and  di ar r hea.   The  di agnosi s  of
anaphy l ax i s  shoul d  be  easi l y   m ade  i f   the  sy m ptom s  descr i bed  appear
ov er   the  cour se  of   m i nutes  up  to  an  hour .   The  bl ood  pr essur e  need  not
dr op—that  i s,   ther e  can  be  anaphy l ax i s  w i thout  shock .

2.   What  i s  the  under l y i ng  pathophy si ol ogi c  pr ocess?

Mast  cel l s  and  basophi l s  ar e  acti v ated  w hen  an  anti gen  (e. g. ,   peni ci l l i n)
com bi nes  w i th  the  anti gen­com bi ni ng  si te  of   i m m unogl obul i n  E  (IgE)
anti bodi es  that  ar e  bound  to  FcεRI,   the  hi gh  af f i ni ty   r eceptor   f or   IgE.
Vasoacti v e  m edi ator s  such  as  hi stam i ne,   l euk otr i ene  C4  (LTC4),   and
pr ostagl andi n  D 2   (PGD 2 )  r api dl y   enter   the  ci r cul ati on.   In  som e
ci r cum stances,   m ast  cel l s  ar e  acti v ated  by   non­IgE  m echani sm s,   such
as  m ay   be  tr i gger ed  by   r adi ocontr ast  dy e  i njecti ons  or   by   nonster oi dal
anti i nf l am m ator y   dr ugs  (N SAIDs);  thi s  i s  cal l ed  an  anaphy l actoi d
r eacti on,   but  the  basi c  phy si ol ogi c  char acter i sti cs  and  tr eatm ent  ar e
other w i se  si m i l ar   to  those  of   IgE­m edi ated  anaphy l ax i s.

3.   What  condi ti ons  shoul d  be  consi der ed  i n  the  di f f er enti al   di agnosi s?

The  di f f er enti al   di agnosi s  l i st  i s  not  l ong.   Col l apse  due  to  septi c  shock ,
car di ac  ar r hy thm i a,   or   asy stol e  m ust  be  consi der ed.   The  m ost  com m on
sour ce  of   er r or   i s  f ai l i ng  to  r ecogni ze  v asov agal .   In  such  a  si tuati on,
the  pati ent's  pul se  i s  sl ow   and  ther e  i s  no  ur ti car i a,   edem a,   or
dy spnea.   The  pul se  i s  al w ay s  r api d  i n  the  setti ng  of   anaphy l ax i s  unl ess
the  pati ent  i s  tak i ng  a  β­adr ener gi c  bl ock er   or   ther e  i s  an  under l y i ng
conducti on  def ect.   Pati ents  w i th  hy per v enti l ati on  do  not  w heeze  or   hav e
hy potensi on.   How ev er ,   deter m i ni ng  the  cause  of   the  anaphy l acti c
epi sode  can  be  di f f i cul t  because  antecedent  ev ents  ar e  not  al w ay s  cl ear
and  som e  epi sodes  w i l l   r em ai n  i di opathi c.

Case
An  18­y ear ­ol d  w om an  i s  seen  i n  a  l ocal   Em er gency   depar tm ent  (ED)
com pl ai ni ng  of   acute  shor tness  of   br eath,   sw el l i ng,   and  a  pr ur i ti c  r ash.
Thr ee  hour s  bef or e  her   sy m ptom s  began,   she  had  a  “sti r   f r y â€​   contai ni ng
tof u  w hi ch  she  had  nev er   eaten  bef or e.   Thi r ty   m i nutes  bef or e  her   ar r i v al   i n
the  ED,   she  w as  at  the  gy m   w her e  she  under took   her   usual   br i ef   (1  m i nute)
w ar m ­up  and  began  r unni ng.   Wi thi n  10  m i nutes  she  f el t  f l ushed,   i tchy ,   and
shor t  of   br eath,   and  noted  the  sensati on  of   an  enl ar gi ng  l um p  i n  her   thr oat.
Her   boy f r i end  dr ov e  her   to  the  ED  w her e  she  w as  ex am i ned  i m m edi atel y .
She  r epor ts  that  she  has  nev er   ex per i enced  si m i l ar   sy m ptom s.   She  appear s
anx i ous  and  di aphor eti c;  her   v i tal   si gns  ar e  r em ar k abl e  f or   a  r espi r ator y
r ate  of   32  per   m i nute,   a  pul se  r ate  of   108  per   m i nute,   and  a  bl ood  pr essur e
of   85/50  m m   Hg.   She  i s  noted  to  be  di f f usel y   f l ushed,
P. 3
and  car ef ul   ex am i nati on  of   her   sk i n  r ev eal s  m ul ti pl e  ur ti car i al   l esi ons  on
her   f ace  and  tr unk .   Her   uv ul a  i s  sw ol l en  and  i s  par ti al l y   obstr ucti ng  her
poster i or   phar y nx .   Inspi r ator y   str i dor   i s  noted  ov er   her   thr oat  and  r adi ates
to  both  l ung  f i el ds.   The  r em ai nder   of   her   ex am i nati on  i s  nor m al .

1.   What  m i ght  be  the  cause  or   causes  of   her   r eacti on?


2.   How   shoul d  she  be  tr eated?
3.   What  f ol l ow ­up  shoul d  be  r ecom m ended?

Case Discussion
1.   What  m i ght  be  the  cause  or   causes  of   her   r eacti on?

Ther e  ar e  a  num ber   of   f actor s  that  m i ght  hav e  tr i gger ed  thi s  r eacti on.
Al though  the  l i st  of   potenti al   causes  of   IgE­m edi ated  anaphy l ax i s  i s
l ong  and  conti nues  to  ex pand,   agents  that  deser v e  speci al   m enti on
i ncl ude  v enom ous  i nsect  sti ngs,   f oods,   i njecti on  of   al l er gy   ex tr acts
(al l er gy   shots),   l atex ,   and  m edi cati ons  of   any   ty pe  but  par ti cul ar l y
anti bi oti cs  and  heter ol ogous  pr otei ns.   N on–IgE­m edi ated  anaphy l ax i s
i s  m ost  of ten  caused  by   r adi ocontr ast  m edi a,   opi oi ds,   N SAIDs,   and
phy si cal   sti m ul i   such  as  ex er ci se­i nduced  and  f ood­r el ated  anaphy l ax i s.

IgE­m edi ated  al l er gi es  to  v enom s  or   f oods  ar e  com m on  causes  of
anaphy l ax i s  i n  al l   age­gr oups.   In  the  absence  of   a  k now n  sti ng  or
i njecti on  of   al l er gen,   f oods  and  m edi cati ons  shoul d  be  i m m edi atel y
consi der ed.   The  f oods  m ost  com m onl y   i m pl i cated  i n  chi l dr en  ar e  m i l k ,
eggs,   and  peanuts  and  i n  adul ts  ar e  peanuts,   tr ee  nuts,   shel l f i sh,   and
f i n  f i sh  but  v i r tual l y   any   f ood  i s  a  potenti al   cause  i n  a  sensi ti zed
per son.   Ty pi cal   r eacti ons  begi n  w i thi n  m i nutes  but  m ay   occur   af ter
sev er al   hour s.

Most  cases  of   dr ug­i nduced  anaphy l ax i s  ar e  IgE­m edi ated  and  ar e  of ten
due  to  peni ci l l i n  anti bi oti cs,   al though  al m ost  any   dr ug  can  be  eti ol ogi c.
In  the  sur gi cal   setti ng,   anaphy l acti c  r eacti ons  ar e  m ost  of ten  due  to
m uscl e  r el ax ants  and  l atex   but  can  al so  be  due  to  hy pnoti cs,
anti bi oti cs,   opi oi ds,   col l oi ds,   and  other   agents.   Aspi r i n  and  N SAIDs  ar e
potent  i nhi bi tor s  of   the  cy cl oox y genase  pathw ay   of   ar achi doni c  aci d
m etabol i sm ,   r epor tedl y   causi ng  ser i ous  r eacti ons  i n  up  to  10%  of
i ndi v i dual s  w i th  asthm a  and  i n  1%  of   the  gener al   popul ati on.   In
asthm ati c  pati ents,   the  r eacti on  consi sts  of   sev er e  br onchospasm ;
i ndi v i dual s  w ho  do  not  hav e  asthm a  m ay   hav e  ur ti car i a,   angi oedem a,
and  anaphy l ax i s.   Reacti ons  to  these  dr ugs  ar e  usual l y   not  m edi ated  by
IgE.   The  m echani sm   r esponsi bl e  f or   causi ng  them   i s  poor l y   under stood,
but  i s  possi bl y   r el ated  to  the  i nhi bi ti on  of   cy cl oox y genase  and  the
shi f ti ng  of   ar achi donate  m etabol i sm   to  the  l i pox y genase  pathw ay .
Ther e  ar e  no  i m m unol ogi c  tests  that  can  detect  thi s  sensi ti v i ty ,   and
chal l enge  tests,   w hi ch  r equi r e  the  use  of   str i ct  pr ecauti ons,   r em ai n  the
onl y   r el i abl e  m ethod  to  i denti f y   aspi r i n­  and  N SAID­sensi ti v e  pati ents.
How ev er ,   as  w i th  any   dr ug,   IgE­m edi ated  r eacti ons  can  al so  occur .

Ex er ci se  can  cause  a  l i m i ted  ar r ay   of   sy stem i c  r eacti ons,   par ti cul ar l y   a


ty pe  of   ur ti car i a  (hi v es)  cal l ed  chol i ner gi c  ur ti car i a,   and  r ar el y
ex er ci se­i nduced  anaphy l ax i s.   An  unusual   sy ndr om e  of   ex er ci se­
i nduced  and  f ood­r el ated  anaphy l ax i s  i s  not  uncom m on  w her ei n  the
pati ent  can  eat  a  speci f i c  f ood  and  ex er ci se  w i thout  pr obl em s.   But  i f
the  f ood  i s  eaten  w i thi n  as  m any   as  sev er al   hour s  bef or e  ex er ci se,
P. 4
the  pati ent  w i l l   ex per i ence  anaphy l ax i s.   Sy stem i c  m astocy tosi s  i s  a
r ar e  di sease  char acter i zed  by   a  “gai n  of   f uncti onâ€​   m utati on  i n  the
r eceptor   f or   stem   cel l   f actor   and  a  r esul tant  ov er gr ow th  of   m ast  cel l s.
Ty pi cal l y ,   these  pati ents  hav e  f r equent  m ani f estati ons  of   m ast  cel l
degr anul ati on  such  as  f l ushi ng  and  hi v es  but  can  pr esent  w i th  i sol ated
anaphy l ax i s.   Fi nal l y ,   a  f ew   pati ents  hav e  i di opathi c  anaphy l ax i s,   that
i s,   they   hav e  one  or   m or e  epi sodes  of   anaphy l ax i s  that  r em ai n
unex pl ai ned  af ter   a  thor ough  ev al uati on.

In  thi s  par ti cul ar   case,   ther e  w as  no  hi stor y   of   i ngesti on  of   a
m edi cati on  and  no  ex posur e  to  a  v enom ous  i nsect.   How ev er ,   m any   of
the  other   di agnoses  m enti oned  ar e  possi bl e  ex pl anati ons  f or   the
pati ent's  epi sode  and  the  tr ue  eti ol ogy   m ay   onl y   becom e  appar ent  ov er
ti m e.   It  i s  i m por tant  to  consi der   an  al l er gi c  r eacti on  to  soy   because
tof u  w as  i ngested  shor tl y   bef or e  the  anaphy l acti c  epi sode,   w as  a  new
f ood  f or   her ,   and  ther e  i s  adequate  oppor tuni ty   to  becom e  sensi ti zed  to
soy   thr ough  a  num ber   of   com m on  f oods.   Of   cour se  a  “sti r   f r y â€​
contai ns  m any   f oods  i n  addi ti on  to  the  tof u  and  any   of   these  ar e
potenti al   cul pr i ts.   In  the  ev ent  that  the  r esul ts  of   al l   di agnosti c
ev al uati ons  ar e  negati v e,   thi s  epi sode  m ay   i ndeed  be  i di opathi c.

2.   How   shoul d  she  be  tr eated?

Ther e  i s  a  gener al l y   accepted  pr otocol   f or   tr eati ng  anaphy l acti c


sy ndr om es.   Epi nephr i ne  i s  the  m ai nstay   of   tr eatm ent  i n  anaphy l ax i s;  i t
acts  by   i nhi bi ti ng  m edi ator   r el ease  f r om   m ast  cel l s  and  basophi l s,
r el ax i ng  br onchi al   sm ooth  m uscl e,   and  bol ster i ng  bl ood  pr essur e.
Of ten,   al l   of   the  si gns  and  sy m ptom s  of   anaphy l ax i s  r esol v e  com pl etel y
w i thi n  m i nutes  of   a  si ngl e  i njecti on  of   epi nephr i ne  but  the  under l y i ng
pathophy si ol ogi c  ev ents  per si st  and  sy m ptom s  r ecur   af ter   the
epi nephr i ne  i s  m etabol i zed.   As  soon  as  the  di agnosi s  of   anaphy l ax i s  i s
str ongl y   enter tai ned,   star ti ng  dose  of   0. 3m L  of   a  1:1, 000  sol uti on  of
epi nephr i ne  shoul d  be  gi v en  i ntr am uscul ar l y ,   pr ef er abl y   i n  the  thi gh,
f or   an  adul t  and  0. 01  m g/k g  f or   a  chi l d.   If   no  untow ar d  si de  ef f ects
occur ,   the  pati ent  m ay   r ecei v e  r epeated  doses  ev er y   10  m i nutes  unti l
the  sy m ptom s  i m pr ov e.   Rar el y ,   1  to  2  m L  of   a  1:10, 000  di l uti on  of
epi nephr i ne  i s  gi v en  by   i ntr av enous  (IV)  r oute  but  thi s  shoul d  be
av oi ded  i f   possi bl e  because  i t  m ay   cause  potenti al l y   f atal   car di ac
ar r hy thm i as.   In  a  pati ent  w hose  pr i m ar y   si te  of   i nv ol v em ent  i s  the
upper   ai r w ay   (such  as  the  one  descr i bed  her e),   i nhal ed  2%  r acem i c
epi nephr i ne  i s  a  v al uabl e  adjunct  to  par enter al   ther apy   (at  a  dose  of
0. 5  m L).   In  addi ti on,   an  H 1   antagoni st  (e. g. ,   di phenhy dr am i ne,   50  m g)
shoul d  be  gi v en  sl ow l y   by   the  IV  r oute.   Recent  ev i dence  show s  that  the
addi ti on  of   an  H 2   bl ock er ,   such  as  r ani ti di ne  150  m g  by   sl ow   IV  i nf usi on
l eads  to  m or e  r api d  r esol uti on  of   acute  al l er gi c  ev ents.   If   the  pati ent
has  sev er e  ai r w ay   obstr ucti on  at  pr esentati on  (cy anosi s)  or   the
obstr ucti on  w or sens  despi te  the  pr om pt  use  of   epi nephr i ne,
endotr acheal   i ntubati on  shoul d  be  per f or m ed  pr om ptl y   usi ng  a  sm al l ­
bor e  tube  (no.   4  or   5).   If   thi s  i s  not  possi bl e  because  of   the  degr ee  of
edem a,   cr i cothy r otom y   shoul d  be  per f or m ed.   In  the  ex am pl e  gi v en,
sem i el ecti v e  i ntubati on  w as  chosen  by   a  sk i l l ed  ED  phy si ci an  to  av oi d
the  com pl i cati ons  associ ated  w i th  em er gency   i ntubati on  of   pati ents  w i th
sw el l i ng  of   the  ai r w ay .   The  pati ent  w as  ex tubated  af ter   8  hour s  w i thout
sequel ae.
Anaphy l ax i s  i n  pati ents  w ho  ar e  tak i ng  β­adr ener gi c  antagoni sts  m ay
be  par ti cul ar l y   di f f i cul t  to  tr eat.   If   w heezi ng  does  not  r espond  to  i ni ti al
epi nephr i ne  or   i nhal ed
P. 5
β­agoni st  ther apy ,   gl ucagon  shoul d  be  adm i ni ster ed.   Pati ents  w i th
hy potensi on  r ef r actor y   to  tr eatm ent  w i th  subcutaneous  epi nephr i ne,
anti hi stam i nes,   and  par enter ­al   f l ui ds  m ay   r equi r e  par enter al
dopam i ne,   nor epi nephr i ne,   and  gl ucagon  ther apy .

In  pati ents  w ho  hav e  si gni f i cant  sy m ptom s  af f ecti ng  any   tar get  or gan
sy stem ,   IV  cor ti coster oi ds  (Sol u­Medr ol ,   1  m g/k g)  shoul d  be  gi v en
i m m edi atel y ,   f ol l ow ed  by   an  or al   dose  6  hour s  l ater .   Pati ents  w ho  hav e
a  pr ol onged  cl i ni cal   cour se  shoul d  conti nue  to  r ecei v e  cor ti coster oi ds
ev er y   6  hour s.

Ini ti al   ther apy   consi sti ng  of   the  i nter v enti ons  just  descr i bed  br i ngs
about  com pl ete  and  sustai ned  r el i ef   of   the  si gns  and  sy m ptom s  of
anaphy l ax i s  i n  50%  of   pati ents.   How ev er ,   one  f our th  of   the  pati ents
r em ai n  par ti al l y   r esi stant  to  ther apy   f or   sev er al   hour s  and  occasi onal l y
f or   sev er al   day s  (pr otr acted  anaphy l ax i s).   The  r em ai ni ng  25%  of   the
pati ents  r espond  to  i ni ti al   ther apy ,   but  af ter   a  v ar i abl e  i nter v al   (up  to
8  hour s)  w i thout  si gns  or   sy m ptom s,   they   ex per i ence  r ecur r ence  of
l i f e­thr eateni ng  com pl i cati ons  (bi phasi c  anaphy l ax i s).   Ther e  i s  no
r el i abl e  w ay   to  pr edi ct  w hi ch  pati ents  w i l l   hav e  such  a  r el apse.
Ther ef or e,   al l   pati ents  w i th  anaphy l ax i s  shoul d  be  obser v ed  by   m edi cal
per sonnel   f or   at  l east  8  hour s  af ter   the  onset  of   the  epi sode.

3.   What  f ol l ow ­up  shoul d  be  r ecom m ended?

Al l   pati ents  w i th  a  hi stor y   of   anaphy l acti c  r eacti on,   no  m atter   w hat  the
cause,   shoul d  be  gi v en  pr el oaded  sy r i nges  contai ni ng  epi nephr i ne  f or
sel f ­adm i ni str ati on  (e. g. ,   Epi Pen  0. 3  m g  or   Epi Pen  Jr   0. 15  m g  Auto­
i njector ,   Dey   Inc. ,   N apa,   Cal i f or ni a)  and  al so  di phenhy dr am i ne  (25  m g
capsul es).   Bef or e  they   l eav e  the  ED,   the  pati ent  and  f am i l y   m em ber s
shoul d  be  tr ai ned  on  how   to  adm i ni ster   the  epi nephr i ne  and
di phenhy dr am i ne  and  tol d  to  cal l   911  or   pr oceed  i m m edi atel y   to  a
near by   ED.   Another   i m por tant  gener al   m easur e  to  be  i m pl em ented  i s
the  r epl acem ent  of   β­bl ock i ng  dr ugs  w i th  a  sui tabl e  al ter nati v e
m edi cati on  i f   possi bl e  i n  those  i ndi v i dual s  w ho  ar e  at  possi bl e  r i sk   f or
f ur ther   epi sodes  of   anaphy l ax i s.

Af ter   acute  tr eatm ent  of   the  anaphy l acti c  epi sode,   the  m ost  l i k el y
cause  of   the  r eacti on  shoul d  be  deter m i ned  so  that  r ecom m endati ons
on  f utur e  av oi dance  can  be  m ade.   In  the  pati ent  descr i bed  her e,   f ood
sk i n  tests  to  al l   the  i ngr edi ents  i n  the  m eal   pr ecedi ng  her   r eacti on
shoul d  be  per f or m ed  by   an  al l er gi st/i m m unol ogi st.   Soy   i s  par ti cul ar l y
suspi ci ous  as  a  possi bl e  cause  of   her   anaphy l ax i s.   If   the  sk i n  test
r esul ts  ar e  posi ti v e  to  one  or   m or e  f oods,   car ef ul l y   m oni tor ed,   gr aded,
doubl e­bl i nded,   pl acebo­contr ol l ed  (DBPC)  f ood  chal l enges  can  be
per f or m ed  to  i denti f y   the  eti ol ogi c  agent.   In  the  ev ent  that  the  f ood
chal l enge  r esul ts  ar e  negati v e,   i t  i s  not  necessar y   f or   the  pati ent  to
av oi d  soy   or   any   of   the  other   i m pl i cated  f oods.   In  pr acti ce,   m ost
pati ents  just  av oi d  the  suspect  f ood  or   f oods.   Thi s  i s  subopti m al
because  ther e  m ay   be  unnecessar y   changes  i n  l i f esty l e  and  nutr i ti on
m ay   be  com pr om i sed.   Pati ents  w i th  a  hi stor y   of   f ood­i nduced
anaphy l ax i s  shoul d  al w ay s  car r y   epi nephr i ne  and  di phenhy dr am i ne
(Benadr y l )  because  of   the  possi bi l i ty   of   i nadv er tent  ex posur e.   Pati ents
w i th  r ecur r ent  epi sodes  of   i di opathi c  anaphy l ax i s  hav e  been  show n  to
benef i t  f r om   dai l y   adm i ni str ati on  of   anti hi stam i ne  and  or al
cor ti coster oi d  ther apy .   Thi s  ty pe  of   ther apy ,   how ev er ,   i s  unw ar r anted
i n  pati ents  w ho  hav e  k now n  av oi dabl e  causes  of   anaphy l ax i s.

The  pati ent  descr i bed  her e  had  a  posi ti v e  sk i n  test  to  cel er y   and  a
negati v e  test  to  soy .   The  m ost  l i k el y   di agnosi s  i n  her   case  i s  ex er ci se­
i nduced  and  f ood­r el ated
P. 6
anaphy l ax i s.   So,   she  w as  adv i sed  to  av oi d  ex er ci si ng  f or   6  hour s  af ter
eati ng  cel er y ,   to  car r y   her   epi nephr i ne  and  di phenhy dr am i ne,   to  nev er
ex er ci se  al one,   and  to  nev er   ex er ci se  i n  r em ote  setti ngs.

Suggested Readings
Canter   LM.   Anaphy l actoi d  r eacti ons  to  contr ast  m edi a.   Al l er gy   Asthm a
Pr oc  2005;26:199.

Li eber m an  P.   Bi phasi c  anaphy l acti c  r eacti ons.   Ann  Al l er gy   Asthm a


Im m unol   2005;95:217.

Li eber m an  P.   Anaphy l ax i s.   Med  Cl i n  N or th  Am   2006;90:77.

Si cher er   SH,   Sam pson  HA.   Food  al l er gy .   J  Al l er gy   Cl i n  Im m unol


2006;117:S470.

Si m ons  FE.   Anaphy l ax i s,   k i l l er   al l er gy :  l ong­ter m   m anagem ent  i n  the


com m uni ty .   J  Al l er gy   Cl i n  Im m unol   2006;117:367.

Wi ener   ES,   Bajaj  L.   Di agnosi s  and  em er gent  m anagem ent  of   anaphy l ax i s


i n  chi l dr en.   Adv   Pedi atr   2005;52:195.

Angioedema
1.   What  ar e  the  cl i ni cal   pi ctur es  associ ated  w i th  angi oedem a?
2.   What  pathophy si ol ogi c  pr ocesses  under l i e  angi oedem a?

3.   How   i s  her edi tar y   angi oedem a  (HAE)  di agnosed?

Discussion
1.   What  ar e  the  cl i ni cal   pi ctur es  associ ated  w i th  angi oedem a?

Angi oedem a  can  pr esent  w i th  sev er al   di f f er ent  cl i ni cal   pi ctur es.   It  can
i ncl ude  an  ex agger ated  f or m   of   ur ti car i a,   w i th  i tchi ng  and  sw el l i ng  of
sof t  ti ssues  that  can  ar i se  any w her e  i n  the  body   and  appear   w i thi n  a
f ew   m i nutes  or   ov er   the  cour se  of   hour s.   Al ter nati v el y ,   i t  m ay   i nv ol v e
the  br onchi al   m ucosa  or   the  v ocal   cor ds,   l eadi ng  to  ai r w ay   obstr ucti on.
Other   f or m s  of   angi oedem a  do  not  i ncl ude  i tchi ng  or   ur ti car i a.   They
m ay   be  l ocal   or   m ay   r esul t  f r om   tr aum a.   In  these  cases,   the  sw ol l en
ti ssues  m ay   hur t,   but  do  not  i tch.

2.   What  pathophy si ol ogi c  pr ocesses  under l i e  angi oedem a?

In  angi oedem a  that  coex i sts  w i th  ur ti car i a,   the  under l y i ng  m echani sm
i s  the  sam e  as  that  i n  anaphy l ax i s—  the  acti v ati on  of   m ast  cel l s  w i th
the  r el ease  of   m ast  cel l   m edi ator s,   such  as  hi stam i ne.   In  the  setti ng  of
angi oedem a  w i thout  ur ti car i a,   the  m echani sm   m ay   i nv ol v e  m ast  cel l s
or   m ay   be  the  unbr i dl ed  acti v ati on  of   the  com pl em ent  and  k i ni n
sy stem s  because  of   l ack   of   a  m ajor   com pl em ent  contr ol   pr otei n,   C1
i nhi bi tor (C1  IN H).

3.   How   i s  HAE  di agnosed?

The  cl i ni cal   cl ue  to  HAE  i s  a  hi stor y   of   r epeated  bouts  of   angi oedem a
w i thout  ur ti car i a  ar i si ng  any w her e  i n  the  body ,   such  as  the  f ace,
tongue,   and  ex tr em i ti es.   The  ai r w ay   can  be  com pr om i sed.   Som e
pati ents  ex per i ence  di f f use  abdom i nal   pai n  and  m ay   hav e  had
l apar otom i es  at  w hi ch  onl y   bow el   edem a  i s  f ound.   These  l esi ons  do  not
i tch  and  m ay   be  pai nf ul .   HAE  i s  tr ansm i tted  as  an  autosom al   dom i nant
tr ai t.   N ev er thel ess,   the  f am i l y   hi stor y   i s  negati v e
P. 7
i n  50%  of   the  pati ents  and  these  cases  ar e  ei ther   due  to  new   m utati ons
or   m i stak en  pater ni ty .   The  l abor ator y   cl ues  poi nt  to  the  com pl em ent
sy stem ,   w i th  a  def i ci ency   of   com pl em ent  contr ol   pr otei ns  at  f aul t.

Case
A  25­y ear ­ol d  w om an  pr esents  to  the  ED  w i th  com pl ai nts  of   sev er e  f aci al
sw el l i ng  r esul ti ng  i n  di f f i cul ty   sw al l ow i ng  begi nni ng  on  the  day   af ter   she  had
under gone  an  endoscopi c  pr ocedur e.   She  noted  m i l d  f aci al   sw el l i ng  on
aw ak eni ng  i n  the  m or ni ng.   Thr oughout  the  ensui ng  day ,   the  sw el l i ng  has
w or sened  to  i nv ol v e  her   l ef t  cheek ,   upper   and  l ow er   l i ps,   and  tongue.
Appr ox i m atel y   6  hour s  bef or e  com i ng  to  the  ED,   she  noted  she  w as
becom i ng  hoar se.   She  had  under gone  the  endoscopy   as  par t  of   an  ev al uati on
f or   i nter m i ttent  abdom i nal   pai n.   Pr ev i ous  i nv esti gati ons  i ncl ude  a  bar i um
sw al l ow   and  enem a,   and  the  r esul ts  of   both  w er e  negati v e.   Si nce  19  y ear s
of   age  she  has  had  abdom i nal   pai n,   w hi ch  she  descr i bes  as  cr am py   and
occasi onal l y   associ ated  w i th  nausea,   v om i ti ng,   or   di ar r hea.   These  sy m ptom s
usual l y   r esol v e  w i thi n  3  to  4  day s  w i th  no  speci f i c  m edi cal   i nter v enti on  and
ar e  not  associ ated  w i th  her   m enstr ual   per i ods.   The  sy m ptom s  began  w hen
she  star ted  usi ng  bi r th  contr ol   pi l l s.   She  has  had  one  other   epi sode  of   f aci al
sw el l i ng  3  y ear s  bef or e,   af ter   a  tooth  ex tr acti on  (al though  i t  w as  m uch  l ess
i ntense  and  not  associ ated  w i th  di f f i cul ty   sw al l ow i ng).   The  sw el l i ng  r esol v ed
spontaneousl y   af ter   appr ox i m atel y   3  day s.   Ther e  i s  no  f am i l y   hi stor y   of
si m i l ar   sy ndr om es.

1.   What  i s  the  m ost  l i k el y   di agnosi s  i n  thi s  w om an?


2.   What  l abor ator y   tests  f or   com pl em ent  ar e  usef ul   i n  m ak i ng  a  di agnosi s
of   HAE?
3.   How   shoul d  an  acute  attack   of   HAE  be  tr eated?
4.   What  pr ophy l acti c  m easur es  ar e  av ai l abl e  f or   HAE?

Case Discussion
1.   What  i s  the  m ost  l i k el y   di agnosi s  i n  thi s  w om an?

The  m ost  l i k el y   di agnosi s  i s  HAE.   Al though  ex posur e  to  l atex   gl ov es  can
cause  an  al l er gi c  r eacti on  that  i ncl udes  angi oedem a,   a  di agnosi s  of   HAE
i s  m uch  m or e  consi stent  w i th  thi s  pati ent's  case  hi stor y .   Local
anestheti cs,   l i k e  any   m edi cati on,   can  cause  angi oedem a  but  thi s  i s
r ar e.   HAE  char acter i sti cal l y   pr esents  as  a  sw el l i ng  of   the  subm ucosal
and  subcutaneous  ti ssues.   Al though  v i r tual l y   any   body   par t  can  be
i nv ol v ed,   usual l y   the  f ace  and  ex tr em i ti es  ar e  af f ected.   Mucosal   edem a
m ay   occur .   Thi s  can  cause  abdom i nal   pai n  w hen  the  sm al l   bow el   i s
af f ected,   or   a  change  i n  v oi ce  or   ev en  str i dor   w hen  the  l ar y nx   i s
af f ected.   Thi s  pati ent  had  both  of   these  sy m ptom s.   U r ti car i a  i s  not  a
par t  of   the  HAE  sy ndr om e.

The  angi oedem a  associ ated  w i th  HAE  f r equentl y   occur s  af ter   l ocal
tr aum a  (i ncl udi ng  dental   pr ocedur es  and  endoscopy ),   i l l ness,   or
em oti onal   str ess,   but  can  al so  ar i se  i n  the  absence  of   a  speci f i c
tr i gger .   HAE  epi sodes  usual l y   begi n  dur i ng  chi l dhood,   but  the  onset  can
occur   at  v i r tual l y   any   age.   Attack s  v ar y   gr eatl y   i n  i ntensi ty   and
f r equency .   Most  pati ents  ex per i ence  sel f ­l i m i ted  f aci al   or   ex tr em i ty
sw el l i ng,   but  other s  can  hav e  l i f e­thr eateni ng  l ar y ngeal   edem a.   Attack s
usual l y   l ast  f or   1  to  4  day s.   They   m ay   i ncr ease  i n  the  pr em enstr ual   or
postpar tum   per i ods.   Thi s
P. 8
di sease  ex hi bi ts  an  autosom al   dom i nant  her edi tar y   patter n,   but  the
f am i l y   hi stor y   i s  negati v e  i n  50%  of   pati ents.
2.   What  l abor ator y   tests  f or   com pl em ent  ar e  usef ul   i n  m ak i ng  a  di agnosi s
of   HAE?

A  ser um   C4  l ev el   r epr esents  the  best  scr een  f or   thi s  di sease  because  i t
i s  l ow   ev en  w hen  the  pati ent  i s  asy m ptom ati c  and  i s  v er y   l ow   w hen  the
pati ent  i s  ex per i enci ng  sw el l i ng.   Measur em ent  of   ser um   C1  IN H  i s  a
l abor ator y   test  that  can  be  per f or m ed  to  establ i sh  the  di agnosi s.   HAE  i s
caused  by   a  decr ease  i n  the  l ev el   and/or   the  f uncti on  of   C1  IN H,   w hi ch
i s  the  i nhi bi tor   of   the  acti v ated  f i r st  com ponent  of   com pl em ent  (C1
w hi ch  contai ns  a  cr i ti cal   ester ase  acti v i ty   cal l ed  C1s)  and  i s  al so  an
i nhi bi tor   of   k al l i k r ei n,   w hi ch  gener ates  br ady k i ni n  f r om   k i ni nogen.
When  the  l ev el   of   thi s  i nhi bi tor   i s  l ow   or   absent,   the  ear l y   cl assi c
com pl em ent  pathw ay   i s  acti v ated  and  v ar i ous  com pl em ent  com ponents
ar e  then  used  up  f aster   than  they   can  be  sy nthesi zed.   In  addi ti on,
ex cess  br ady k i ni n,   a  v er y   potent  v asoacti v e  pepti de,   i s  gener ated  i n
ex cess.   In  85%  of   pati ents,   the  l ev el   of   the  C1  IN H  pr otei n  i s
decr eased,   w her eas  i n  15%  of   the  pati ents  the  pr otei n  i s  pr esent  but
dy sf uncti onal .   Ther ef or e,   i n  a  subgr oup  of   these  pati ents  the  C1  IN H
l ev el   i s  nor m al   and  the  natur e  of   the  di sease  cannot  be  detected  unl ess
the  C1  IN H  f uncti on  i s  assessed.   Al though  w e  m ak e  the  di agnosi s  by
m oni tor i ng  the  com pl em ent  sy stem ,   the  actual   cause  of   the  sy m ptom s
appear s  to  be  edem a  gener ated  by   the  f or m ati on  of   br ady k i ni n.
Hi stam i ne  r el ease  i s  not  par t  of   thi s  condi ti on.

3.   How   shoul d  an  acute  attack   of   HAE  be  tr eated?

N ei ther   anti hi stam i nes  nor   cor ti coster oi ds  hav e  a  r ol e  i n  the  tr eatm ent
of   an  acute  attack   of   HAE,   w her eas  they   ar e  ef f ecti v e  i n  the  tr eatm ent
of   al l er gi c  ur ti car i a  that  i ncl udes  angi oedem a.   Because  the  m edi cal
tr eatm ent  of   HAE  i s  not  al w ay s  ef f ecti v e,   and  i f   upper   ai r w ay
com pr om i se  i s  pr esent  or   pendi ng,   stat  otol ar y ngol ogy   and
anesthesi ol ogy   consul tati ons  shoul d  be  obtai ned  because  a
tr acheostom y   m ay   be  r equi r ed  to  pr ev ent  ai r w ay   cl osur e.   The  onl y
al ter nati v e  to  tr acheostom y   i s  nasotr acheal   i ntubati on,   w hi ch  shoul d  be
per f or m ed  onl y   i n  an  oper ati ng  r oom   w i th  a  sur geon  pr esent  i n  the
ev ent  an  em er gent  tr acheostom y   i s  r equi r ed.   Tr eatm ent  w i th  f r esh
f r ozen  pl asm a  i s  som ew hat  contr ov er si al   because  thi s  substance
pr ov i des  f ur ther   pr otei ns  that,   w hen  acti v ated  secondar y   to  a  decr ease
i n  C1  IN H,   m i ght  w or sen  the  angi oedem a.   N ev er thel ess,   m any
phy si ci ans  r outi nel y   adm i ni ster   tw o  uni ts  of   f r esh  f r ozen  pl asm a.   As
m enti oned  ear l i er ,   these  di seases  can  pr esent  as  cr am py   abdom i nal
pai n,   som eti m es  m i si nter pr eted  as  an  acute  abdom i nal   condi ti on,   but
thi s  di sease  can  be  di f f er enti ated  f r om   an  acute  abdom i nal   condi ti on  by
the  l ack   of   abdom i nal   r i gi di ty   and  f ev er   and  absence  of   an  el ev ated
w hi te  bl ood  cel l   count  w i th  a  l ef tw ar d  shi f t.   Thi s  abdom i nal   pai n  can  be
r el i ev ed  by   nar coti cs  such  as  m eper i di ne.   Som e  ex per i enced  phy si ci ans
tr eat  the  abdom i nal   or   ex tr em i ty   pai n  associ ated  w i th  these
angi oedem a  attack s  (w hi ch,   i n  gener al ,   ar e  sel f ­l i m i ted)  usi ng
m eper i di ne.   They   r eser v e  f r esh  f r ozen  pl asm a  (w hi ch  suppl i es  C1  IN H)
and,   of   cour se,   tr acheostom y   f or   thr eatened  ai r w ay   cl osur e.   Pur i f i ed  C1
IN H  and  a  br ady k i ni n  r eceptor   antagoni st  m ay   soon  be  av ai l abl e  f or
tr eatm ent  of   acute  attack s.

4.   What  pr ophy l acti c  m easur es  ar e  av ai l abl e  f or   HAE?

Chr oni c  pr ophy l ax i s  f or   thi s  l i f e­thr eateni ng  di sease  i s  i m por tant.   We
use  an  attenuated  andr ogen  (such  as  danazol ).   These  “i m pededâ€​
andr ogeni c  ster oi ds  cause
P. 9
an  i ncr ease  i n  sy nthesi s  of   C1  IN H.   Hi gh  doses  br i ng  about  cor r ecti on
of   both  the  C1  IN H  and  C4  l ev el s.   U nf or tunatel y ,   unacceptabl e  si de
ef f ects  m ay   ar i se  at  these  doses,   i ncl udi ng  w ei ght  gai n,   headaches,
m uscl e  cr am pi ng,   m enom etr or r hagi a,   andr ogeni c  ef f ects,   and  m i l d
i ncr eases  i n  the  ser um   aspar tate  and  al ani ne  am i notr ansf er ase  l ev el s.
HAE  can  usual l y   be  contr ol l ed  w i th  l ow er   doses  of   these  andr ogens,
w hi ch  do  not  enti r el y   cor r ect  these  l abor ator y   abnor m al i ti es,   but
pr oduce  f ew er   si de  ef f ects.   Ther ef or e,   the  andr ogen  dose  i s  adjusted  to
achi ev e  sy m ptom ati c  r el i ef ,   not  to  cor r ect  the  l abor ator y
abnor m al i ti es.   Andr ogens  ar e  not  hel pf ul   i n  m anagi ng  acute
ex acer bati ons  of   thi s  di sease,   and  nei ther   cor ti coster oi ds  nor
anti hi stam i nes  hav e  a  pr ophy l acti c  ef f ect.

Wom en  w i th  HAE  f r equentl y   note  a  w or seni ng  of   thei r   di sease  w hen
they   star t  tak i ng  bi r th  contr ol   pi l l s.   Thi s  i s  possi bl y   due  to  the
anti andr ogeni c  ef f ect  of   the  pi l l s.   Pr egnant  w om en  w i th  HAE,   how ev er ,
do  w el l   i n  l ate  pr egnancy   and  del i v er y .

Suggested Readings
Br acho  FA.   Her edi tar y   angi oedem a.   Cur r   Opi n  Hem atol   2005;12:493.

Ci car di   M,   Zi ngal e  L,   Zani cher l l i   A,   etal .   C1  i nhi bi tor :  m ol ecul ar   and
cl i ni cal   aspects.   Spr i nger   Sem i n  Im m unopathol   2005;27:286.

Kapl an  AP,   Gr eav es  MW.   Angi oedem a.   J  Am   Acad  Der m atol   2005;53:373.

Chronic Urticaria
1.   What  i s  the  def i ni ti on  of   chr oni c  ur ti car i a?

2.   What  i s  the  pathogenesi s  of   chr oni c  ur ti car i a?

Discussion
1.   What  i s  the  def i ni ti on  of   chr oni c  ur ti car i a?

Chr oni c  ur ti car i a  i s  def i ned  as  ur ti car i a  that  per si sts  f or   6  w eek s  or
m or e.

2.   What  i s  the  pathogenesi s  of   chr oni c  ur ti car i a?

The  pathogenesi s  of   chr oni c  ur ti car i a  i ncl udes  a  spectr um   of   ev ents.


Com m onl y ,   ther e  i s  si m pl e  l ocal   edem a  and  i tchi ng  caused  by   the
r el ease  of   hi stam i ne  f r om   m ast  cel l s.   Other   m or e  sev er e  cases  hav e  an
i nf l am m ator y   com ponent,   such  as  v ascul i ti s  that  i s  r ev eal ed  by   bi opsy
speci m ens.   In  these  cases,   the  r esponsi bl e  m echani sm   m ay   be  ei ther
tum or   necr osi s  f actor   α ­l eased  f r om   acti v ated  m ast  cel l s,   or   anti genâ
€“anti body   com pl ex es,   w hi ch  i n  tur n  acti v ate  the  com pl em ent  and  l ead
to  the  pr oducti on  of   anaphy l atox i ns.   These  substances  tr i gger   l ocal
m ast  cel l   acti v ati on,   w i th  subsequent  i tchi ng,   er y them a,   and  w heal
f or m ati on.

Case
A  25­y ear ­ol d  w om an  i s  seen  because  of   a  pr ur i ti c  r ash  char acter i zed  by
m ul ti pl e,   ci r cum scr i bed,   r ai sed  ar eas  of   er y them a  v ar y i ng  i n  si ze  f r om   2
m m   to  3  cm   and  occur r i ng
P. 10
ov er   the  sk i n.   Each  l esi on  l asts  1  or   2  day s,   but  new   ones  ar i se  as  ol d  ones
f ade.   The  r ash  has  per si sted  f or   9  w eek s.   She  does  not  sm ok e  or   dr i nk
al cohol ,   nor   has  she  tak en  any   m edi cati ons  i n  the  l ast  10  w eek s,   i ncl udi ng
anti bi oti cs  or   aspi r i n,   al though  she  i s  sex ual l y   acti v e  and  on  bi r th  contr ol
pi l l s.   She  r etur ned  f r om   tr ek k i ng  i n  N epal   3  m onths  ago  but  has  been  w el l
si nce,   ex cept  f or   the  r ash.   Her   f am i l y   hi stor y   i s  negati v e  f or   atopi c  di seases
such  as  al l er gi c  r hi ni ti s,   asthm a,   or   eczem a.   Her   phy si cal   ex am i nati on
f i ndi ngs  ar e  nor m al   ex cept  f or   the  pr esence  of   er y them atous,   papul ar
w heal s  l ocated  ov er   her   tr unk ,   back ,   and  ar m s,   w hi ch  bl anch  w i th  pr essur e.
The  l esi ons  ar e  5  to  25  m m   i n  di am eter   and  of ten  ov er l ap.   She  ex hi bi ts
der m atogr aphi sm .   Her   com pl ete  bl ood  count(CBC)  i s  nor m al   and  the
er y thr ocy te  sedi m entati on  r ate  (ESR)  i s  11  m m   per   hour   (nor m al ).

1.   What  causes  of   ur ti car i a  shoul d  be  consi der ed  i n  thi s  w om an?
2.   What  di agnosti c  appr oach  shoul d  be  tak en  i n  thi s  pati ent?
3.   What  ther apeuti c  appr oach  i s  desi r abl e?
4.   What  i s  the  pr ognosi s  i n  thi s  pati ent  i f   no  speci f i c  al l er gen  i s
i denti f i ed?

Case Discussion
1.   What  causes  of   ur ti car i a  shoul d  be  consi der ed  i n  thi s  w om an?

The  m ost  com m on  cause  of   acute  ur ti car i a  i s  an  al l er gi c  r eacti on  to  a
f ood  or   dr ug.   By   contr ast,   usual l y   no  ex ter nal   cause  i s  f ound  i n  80%  to
90%  of   the  pati ents  w i th  chr oni c  ur ti car i a.   As  a  gr oup,   these  pati ents
ar e  not  atopi c;  that  i s,   the  pr ev al ence  of   eczem a,   al l er gi c  r hi ni ti s,   or
asthm a  i s  not  i ncr eased.   The  pr esence  of   der m atogr aphi sm   i ndi cates  a
gener al   i ncr ease  i n  the  sensi ti v i ty   of   the  m ast  cel l s  and  bl ood  v essel s
i n  the  sk i n,   but  the  cause  of   der m atogr aphi sm   i s  unk now n.
N ev er thel ess,   i t  i s  i m por tant  to  tak e  a  car ef ul   hi stor y   to  uncov er   any
under l y i ng  cause,   i f   pr esent.   Al m ost  any   m edi cati on  can  cause
ur ti car i a.   Bi r th  contr ol   pi l l s  as  w el l   as  ov er ­the­counter   pr epar ati ons
such  as  aspi r i n,   v i tam i ns,   and  col d  tabl ets  shoul d  be  consi der ed  as
possi bl e  cul pr i ts.   Foods  som eti m es  cause  chr oni c  ur ti car i a  and  shoul d
be  consi der ed  i f   i ndi cated  by   the  pati ent's  hi stor y .

U r ti car i a  can  al so  be  associ ated  w i th  under l y i ng  sy stem i c  i l l nesses  such
as  sy stem i c  l upus  er y them atosus,   Sjögr en's  sy ndr om e,   r heum atoi d
ar thr i ti s,   and  hy per thy r oi di sm   or   hy pothy r oi di sm .   These  pati ents  of ten
hav e  el ev ated  ESRs,   and  sk i n  bi opsy   speci m ens  m ay   r ev eal   the
pr esence  of   tr ue  v ascul i ti s  w i th  pol y m or phonucl ear   i nf i l tr ates  and  the
deposi ti on  of   i m m unogl obul i ns  and  com pl em ent.   Inf ecti ons  ar e  al so
r ar e  causes  of   ur ti car i a,   i ncl udi ng  v i r al   i nf ecti ons  such  as  pr odr om al
i nf ecti ous  hepati ti s  or   i nf ecti ous  m ononucl eosi s,   as  w el l   as  hel m i nthi c
i nf ecti ons.

Cer tai nl y ,   any   sy m ptom s  or   si gns  of   i nf ecti on  shoul d  be  pur sued  and
tr eated;  how ev er ,   i t  i s  not  w or thw hi l e  to  do  speci f i c  w or k ups  i n  pur sui t
of   cr y pti c  i nf ecti ons.

2.   What  di agnosti c  appr oach  shoul d  be  tak en  i n  thi s  pati ent?

In  di agnosi ng  the  cause  of   the  ur ti car i a  i n  thi s  pati ent,   a  good  m edi cal
hi stor y   and  com pl ete  phy si cal   ex am i nati on  ar e  i m por tant  to  ex cl ude
any   under l y i ng  sy stem i c  di sease.   Som e  speci al   tests  m ay   be  done  to
i nv esti gate  any   cl ues  r ev eal ed  by   the  hi stor y .   These  m i ght  i ncl ude  f ood
sk i n  tests  f or   a  suspected  f ood  sensi ti v i ty   or   an
P. 11
“i ce­cube  testâ€​   of   the  sk i n  i f   col d  ur ti car i a  i s  suspected.   If   no
cause  i s  appar ent,   any   under l y i ng  sy stem i c  di sease  can  be  r ul ed  out  by
a  CBC,   ur i nal y si s,   ESR,   and  bl ood  chem i str y   pr of i l e.   Anti bodi es  to
FcεRI  hav e  been  descr i bed  i n  up  to  40%,   def i ni ng  a  subgr oup  w i th
autoi m m une  ur ti car i a.

If   the  pr esence  of   an  el ev ated  ESR  suggests  the  possi bi l i ty   of


v ascul i ti s,   the  f ol l ow i ng  tests  shoul d  be  consi der ed:  CH 5 0   (total
hem ol y ti c  com pl em ent),   C 3   (thi r d  com ponent  of   com pl em ent),   C 4
(f our th  com ponent  of   com pl em ent),   sk i n  bi opsy   w i th  i m m unof l uor escent
stai ni ng,   hepati ti s  B  sur f ace  anti gen  and  anti body   (HBsAg  and  HBsAb),
cr y ogl obul i ns,   anti nucl ear   anti body ,   and  ci r cul ati ng  i m m une  com pl ex es.

In  thi s  pati ent,   i t  i s  not  unr easonabl e  to  obtai n  stool   sam pl e  to  test  f or
ov a  and  par asi tes  because  of   her   r ecent  tr i p  to  N epal .
If   none  of   these  appr oaches  i s  successf ul   i n  r ev eal i ng  a  cause,   hav e
the  pati ent  stop  tak i ng  bi r th  contr ol   pi l l s  f or   a  m onth  and  obser v e  the
ur ti car i a,   untr eated.

3.   What  ther apeuti c  appr oach  i s  desi r abl e?

Ther e  i s  no  k now n  “cur eâ€​   f or   ur ti car i a  unl ess  the  al l er gen  i s
i denti f i ed  and  el i m i nated.   Other w i se,   tr eatm ent  i s  ai m ed  at  pr ov i di ng
sy m ptom ati c  r el i ef .   Ty pe  H 1   anti hi stam i nes  such  as  di phenhy dr am i ne
[Benadr y l   (Par k e­Dav i s,   Mor r i s  Pl ai ns,   N J)]  or   hy dr ox y zi ne  [Atar ax
(Roer i g,   N ew   Yor k ,   N Y)]  ar e  com m onl y   used  f i r st.   For   l onger ­ter m
tr eatm ent,   f our   nonsedati ng  anti hi stam i nes  ar e  av ai l abl e:  f ex of enadi ne
[Al l egr a  (Hoechst  Mar i on  Roussel ,   Kansas  Ci ty ,   MO)  and  gener i c],
l or atadi ne  [Cl ar i ti n  (Scher i ng­Pl ough,   Madi son,   N J)  and  gener i c],
ceti r i zi ne  [Zy r tec  (Pf i zer ,   N ew   Yor k ,   N Y)],   and  desl or atadi ne/l or atadi ne
[Cl ar i nex /Cl ar i ti n  (Scher i ng­Pl ough,   Madi son,   N J)].   Thi s  f or m   of
tr eatm ent  i s  based  on  the  concept  that  m ast  cel l s  r el ease  hi stam i ne,
and  hi stam i ne  i s  the  pr i m ar y   of f ender   i n  ur ti car i a.

Com bi ni ng  H 1   and  H 2   anti hi stam i nes  has  al so  been  hel pf ul   i n  som e
pati ents.   Shor t  cour ses  of   cor ti coster oi ds  m ay   be  used  i n  sev er e,
poor l y   contr ol l ed  cases;  how ev er ,   the  l ong­ter m   use  of   ster oi ds  shoul d
be  av oi ded,   i f   possi bl e,   because  of   the  sev er e  si de  ef f ects  associ ated
w i th  these  agents.   Fi nal l y ,   som e  al l er gi sts  m ay   tr y   out  an  el i m i nati on
di et  or   a  f ast  i n  sev er el y   af f ected  pati ents  to  r ul e  out  a  f ood  or
pr eser v ati v e  al l er gy ,   ev en  w hen  no  speci f i c  agent  i s  suspected.
Pati ents  w i th  sev er e  di sease  m ay   be  tr eated  w i th  i m m unom odul ator y
dr ugs  such  as  hy dr ochl or oqui n,   sul phasal azi ne,   or   cy cl ospor i n.

4.   What  i s  the  pr ognosi s  i n  thi s  pati ent  i f   no  speci f i c  al l er gen  i s


i denti f i ed?

Assum i ng  that  no  cause  has  been  f ound,   and  ther e  i s  no  autoi m m une
di sease  pr esent,   the  pr ognosi s  i s  qui te  good.   The  si gns  and  sy m ptom s
al m ost  al w ay s­di sappear   w i thi n  2  y ear s,   but  the  r eason  f or   thi s  i s  not
k now n.

Suggested Readings
Bax i   S,   Di nak ar   C.   U r ti car i a  and  angi oedem a.   Im m unol   Al l er gy   Cl i n
N or th  Am   2005;25:353.

Di bber n  DA  Jr .   U r ti car i a:  sel ected  hi ghl i ghts  and  r ecent  adv ances.   Med
Cl i n  N or th  Am   2006;90:187.

Var adar ajul u  S.   U r ti car i a  and  angi oedem a.   Contr ol l i ng  acute  epi sodes,
copi ng  w i th  chr oni c  cases.   Postgr ad  Med  2005;117:25.
P. 12

Monoclonal Gammopathy
1.   What  i s  the  def i ni ti on  of   a  m onocl onal   gam m opathy ?

2.   What  cl i ni cal   pi ctur es  ar e  seen  i n  pati ents  w i th  m onocl onal
gam m opathi es?

Discussion
1.   What  i s  the  def i ni ti on  of   a  m onocl onal   gam m opathy ?

A  m onocl onal   gam m opathy   i s  def i ned  as  the  ov er pr oducti on  of   a
par ti cul ar   i m m unogl obul i n  pr otei n  by   a  si ngl e  cl one  of   ov er acti v e  or
m al i gnant  B  cel l s.   Thi s  cl one  can  pr oduce  a  w hol e  i m m unogl obul i n,
com posed  of   both  heav y   and  l i ght  chai ns,   or   i t  can  pr oduce  just  heav y
chai ns,   just  l i ght  chai ns,   or   a  com bi nati on  of   w hol e  i m m unogl obul i n
pl us  ex cess  l i ght  chai ns.   The  m onocl onal   l i ght  chai ns  ar e  cal l edBence
Jones  pr otei n.

2.   What  cl i ni cal   pi ctur es  ar e  seen  i n  pati ents  w i th  m onocl onal
gam m opathi es?

Som e  m onocl onal   ser um   i m m unogl obul i ns  ar e  di scov er ed  i nci dental l y .
These  ar e  usual l y   sm al l   (< 2  g/dL)  and  ther e  ar e  no  associ ated  si gns,
sy m ptom s,   or   l abor ator y   abnor m al i ti es.   Pati ents  w i th  pl asm a  cel l
(m ul ti pl e)  m y el om a  usual l y   pr esent  w i th  back   pai n  (v er tebr al   f r actur e
or   com pr essi on),   anem i a,   hy per cal cem i a,   and  of ten  r enal   di sease.   The
cl i ni cal   pi ctur e  of   Wal denstr om 's  m acr ogl obul i nem i a  r esem bl es  that  of
a  l y m phom a,   and  consi sts  of   f ev er ,   l y m phadenopathy ,   and  som eti m es
hepatospl enom egal y .   Hy per v i scosi ty   can  be  a  com ponent  of   thi s
sy ndr om e.   Li ght  chai n  di sease  can  pr esent  as  am y l oi dosi s.

Case
A  62­y ear ­ol d  m an  i s  seen  i n  the  ED  because  of   a  r i ght  upper   quadr ant
abdom i nal   pai n  of   5  day s'  dur ati on.   The  pai n  r adi ates  ar ound  to  hi s  back   and
i s  w or se  w i th  m ov em ent  and  coughi ng.   He  deni es  nausea,   v om i ti ng,   or   a
change  i n  hi s  bow el   habi ts  but  adm i ts  to  hav i ng  i nter m i ttent  epi gastr i c  pai n,
f r equent  ni ght  sw eats,   a  f eel i ng  of   “w eak ness, â€​   gener al   m al ai se,   and  a
15­pound  (6. 75­k g)  w ei ght  l oss  ov er   the  l ast  y ear .   Hi s  past  m edi cal   hi stor y
i s  r em ar k abl e  f or   a  back   i njur y   i ncur r ed  f r om   a  m otor   v ehi cl e  acci dent  10
y ear s  bef or e  and  the  pr esence  of   m i l d  hy per tensi on.   Hi s  phy si cal
ex am i nati on  f i ndi ngs  ar e  unr em ar k abl e,   ex cept  f or   the  f ol l ow i ng.   Hi s  bl ood
pr essur e  i s  150/110  m m   Hg.   He  has  a  gr ade  2/6  sy stol i c  ejecti on  m ur m ur
that  can  be  hear d  al ong  the  l ef t  ster nal   bor der .   Rectal   ex am i nati on  r ev eal s
a  2+   pr ostate.   Hi s  stool   i s  hem e  negati v e.   A  sl i ght  k y phosi s  i s  noted  and
ther e  i s  questi onabl e  decr eased  sensati on  to  pi npr i ck   al ong  the  r i ght  l ow er
r i b  cage  (T9  di str i buti on).   A  chest  r adi ogr aphi c  study ,   CBC,   and  chem i str y
panel   ar e  per f or m ed.   The  chest  r adi ogr aph  show s  no  i nf i l tr ates,   but  a
com pr essi on  f r actur e  of   undeter m i ned  age  i s  noted  at  T9.   Hi s  hem ogl obi n  i s
10  g/dL;  hem atocr i t,   31%;  and  pl atel et  count,   275, 000.   Hi s  chem i str y   panel
show s  ser um   cr eati ni ne,   2. 2  m g/dL;  bl ood  ur ea  ni tr ogen,   22  m g/dL;  total
pr otei n,   10. 2  m g/dL  (nor m al ,   6. 8  to  8. 4  m g/dL);  al bum i n,   3. 1  m g/dL
(nor m al ,   3. 7  to  4. 9  m g/dL);  and  cal ci um ,   11. 0  m g/dL  (nor m al ,   8. 5  to  10. 0
m g/dL).   You  concl ude  that  hi s  pai n  i s  m ost  l i k el y   due  to  the  T9  com pr essi on
f r actur e.   Because  of   concer n  about  hi s  r enal   i nsuf f i ci ency ,   y ou  av oi d
pr escr i bi ng  N SAIDs  but  i nstead  pr escr i be
P. 13
acetam i nophen  w i th  codei ne.   You  or der   som e  addi ti onal   l abor ator y   studi es
on  the  ex tr a  tubes  of   bl ood  sam pl es  bef or e  the  pati ent  i s  di schar ged.

1.   If   y ou  w er e  consi der i ng  a  di agnosi s  of   a  m onocl onal   gam m opathy


(w hi ch  y ou  shoul d  hav e),   w hat  scr eeni ng  test  w oul d  y ou  or der ?
2.   What  f ur ther   i m m unol ogi c  tests  shoul d  be  or der ed?
3.   What  f ur ther   tests  ar e  i m por tant  i n  thi s  case?
4.   What  i s  the  i m m unol ogi c  capabi l i ty   of   thi s  pati ent  w ho  has  ex cess
gam m a  gl obul i n?
5.   What  i s  the  cur r ent  tr eatm ent  f or   such  a  pati ent?
Whi l e  y ou  ar e  ev al uati ng  thi s  pati ent,   y our   col l eague  l ear ns  of   y our
ex per ti se  i n  thi s  f i el d  and  ask s  y ou  about  a  61­y ear ­ol d  m an  w i th
atopi c  der m ati ti s  w ho  had  the  i nci dental   f i ndi ng  of   a  hi gh  pr otei n­to­
al bum i n  r ati o  on  a  com pr ehensi v e  chem i str y   panel .   A  scr eeni ng  (SPEP),
has  show n  a  m onocl onal   band,   w hi ch  has  been  i denti f i ed  as  IgM  k   on
i m m unof i x ati on  el ectr ophor esi s  (IFE).   The  band  w as  quanti f i ed  at  2. 0
g/dL.
6.   What  cour se  of   acti on  do  y ou  r ecom m end  i n  the  61­y ear ­ol d  pati ent?
7.   What  i s  the  di agnosi s  i n  the  pati ent  descr i bed  i n  questi on  6,   and  w hat
i s  the  pr ognosi s?

Case Discussion
1.   If   y ou  w er e  consi der i ng  a  di agnosi s  of   a  m onocl onal   gam m opathy
(w hi ch  y ou  shoul d  hav e),   w hat  scr eeni ng  test  w oul d  y ou  or der ?

The  scr eeni ng  test  that  shoul d  be  or der ed  i n  thi s  pati ent  i s  an  SPEP,
w hi ch  show s  a  m onocl onal   spi k e  i n  m ost  cases.   The  cl i ni cal   suspi ci on
f or   m y el om a  shoul d  be  hi gh  because  he  ex hi bi ts  the  cl assi c  tr i ad  of
anem i a,   back   pai n,   and  r enal   i nsuf f i ci ency ,   w hi ch  i s  associ ated  w i th
m ul ti pl e  m y el om a.   The  m ost  com m on  pr esenti ng  com pl ai nt  i s  back
pai n.   Mul ti pl e  m y el om a  i s  the  m ost  com m on  l y m phor eti cul ar   neopl asm
i n  nonw hi te  m en  and  the  thi r d  m ost  com m on  i n  w hi tes.   Its  annual
i nci dence  i s  3  i n  100, 000,   and  m or e  than  90%  of   al l   af f ected  pati ents
ar e  ol der   than  40  y ear s.   Other   f actor s  that  i m pl i cate  m ul ti pl e  m y el om a
i n  thi s  case  ar e  the  el ev ated  total   ser um   pr otei n  content  and  the
r el ati v el y   decr eased  al bum i n  l ev el .   These  suggest  that  ther e  i s  an
i ncr ease  i n  the  gl obul i n  f r acti on.

2.   What  f ur ther   i m m unol ogi c  tests  shoul d  be  or der ed?

Fur ther   i m m unol ogi c  tests  that  shoul d  be  done  i ncl ude  an  IFE,   w hi ch
can  i denti f y   the  heav y   and  l i ght  chai ns  i n  the  m onocl onal   pr otei n.   A
ur i ne  el ectr ophor esi s  can  i denti f y   the  spi l l i ng  of   l i ght  chai ns  (Bence
Jones  pr otei n)  or ,   i f   the  gl om er ul us  i s  dam aged,   the  pr esence  of
com pl ete  m onocl onal   pr otei n.

3.   What  f ur ther   tests  ar e  i m por tant  i n  thi s  case?

A  sk el etal   sur v ey   i s  an  i m por tant  addi ti onal   test  to  docum ent  the
ex tent  of   bone  di sease.   In  thi s  si tuati on  i t  i s  better   than  a  bone  scan.
The  sk el etal   sur v ey   shoul d  i ncl ude  the  sk ul l ,   com pl ete  spi ne  (both
anter oposter i or   and  l ater al   v i ew s),   the  pel v i s,   and  the  chest.   A
com puted  tom ogr aphi c  scan  of   the  abdom en  w oul d  be  usef ul   onl y   i f   a
sol i tar y   ex tr am edul l ar y   pl asm acy tom a  i s  suspected.

P. 14
A  bone  m ar r ow   aspi r ati on  i s  essenti al   to  conf i r m   the  di agnosi s.   In  al l
but  the  r ar est  cases,   cl um ps  and  sheets  of   pl asm a  cel l s  ar e  seen.

A  ser um   cal ci um   deter m i nati on  i s  needed  because  hy per cal cem i a  can
pr oduce  sy m ptom s  such  as  l ethar gy ,   nausea,   and  v om i ti ng.   El ev ated
v al ues  f or   β­2  m acr o­gl obul i n,   C­r eacti v e  pr otei n,   and  l actate
dehy dr ogenase  suggest  a  m or e  di r e  pr ognosi s.

4.   What  i s  the  i m m unol ogi c  capabi l i ty   of   thi s  pati ent  w ho  has  ex cess
gam m a  gl obul i n?

The  i m m unol ogi c  capabi l i ty   i n  thi s  pati ent  i s  pr obabl y   com pr om i sed,
such  that  he  i s  suscepti bl e  to  hi gh­gr ade  bacter i al   pathogens.   The
ex cess  i m m unogl obul i n  r epr esented  by   the  spi k e  on  SPEP  i s  usel ess  i n
f i ghti ng  i nf ecti on,   and  i t  i s  l i k el y   that  he  i s  sev er el y   depl eted  of   nor m al
pol y cl onal   gam m a  gl obul i ns.   In  f act,   these  pati ents  ar e  f uncti onal l y
hy pogam m agl obul i nem i c.   They   ar e  pr one  to  i nf ecti ons  w i th  py ogeni c
or gani sm s,   and  they   do  not  pr oduce  adequate  anti bodi es  af ter
pr ophy l acti c  i m m uni zati ons.   They   shoul d  be  car ef ul l y   w atched  f or   ear l y
si gns  of   i nf ecti on.   Som e  phy si ci ans  pr escr i be  m onthl y   IV  gam m a
gl obul i n.

5.   What  i s  the  cur r ent  tr eatm ent  f or   such  a  pati ent?

Mel phal an  and  pr edni sone  consti tute  the  chem other apy   m ost  of ten  used
to  tr eat  m y el om a.   Bone  m ar r ow   tr anspl antati on  i s  bei ng  used  m or e
f r equentl y .   Thal i dom i de  has  al so  been  show n  to  be  ef f ecti v e.

6.   What  cour se  of   acti on  do  y ou  r ecom m end  i n  the  61­y ear ­ol d  pati ent?

For   the  second  pati ent,   y ou  r ecom m end  that  a  detai l ed  ex am i nati on  be
under tak en  f or   l y m phadenopathy ,   hepatospl enom egal y ,   anem i a,
hy per cal cem i a,   l y ti c  bone  l esi ons,   and  Bence  Jones  pr otei n.   Ev er y thi ng
r etur ns  nor m al .   Si nce  the  m onocl onal   spi k e  i s  IgM,   a  v er y   l ar ge  pr otei n
(850  k d)  that  pr edi sposes  to  hy per v i scosi ty ,   y ou  r ecom m end  that  the
ser um   v i scosi ty   be  deter m i ned.   Thi s  too  pr ov es  to  be  nor m al .

7.   What  i s  the  di agnosi s  i n  the  pati ent  descr i bed  i n  questi on  6,   and  w hat
i s  the  pr ognosi s?

Thi s  i s  the  cl i ni cal   pi ctur e  of   m onocl onal   gam m opathy   of   undeter m i ned
si gni f i cance,   w hi ch  consi sts  of   a  sm al l   m onocl onal   ser um   spi k e
(gener al l y   l ess  than  2. 0  g/dL)  w i thout  other   si gns,   sy m ptom s,   or
l abor ator y   i ndi cati ons  of   m y el om a  or   m acr ogl obul i nem i a.   The  pr ognosi s
i n  these  pati ents  i s  uncl ear .   Som e  pati ents  pr ogr ess  to  f r ank   di sease;
other s  do  not.   The  best  pl an  i s  to  obser v e  the  pati ent  by   per f or m i ng  a
phy si cal   ex am i nati on  and  scr eeni ng  SPEP  ev er y   6  to  12  m onths.

Suggested Readings
Hi deshi m a  T,   Ber gsagel   PL,   Kuehl   WM,   etal .   Adv ances  i n  bi ol ogy   of
m ul ti pl e  m y el om a:  cl i ni cal   appl i cati ons.   Bl ood  2004;104:607–618.

Ky l e  RA,   Rajk um ar   SV.   Mul ti pl e  m y el om a.   N   Engl   J  Med  2004;351


(18):1860;  [Er r atum   appear s  i n  N   Engl   J  Med  2005;352(11):1163].

Ky l e  RA,   Rajk um ar   SV.   Monocl onal   gam m opathi es  of   undeter m i ned
si gni f i cance.   Bai l l i er e's  Best  Pr act  Cl i n  Haem atol   2005;18:689.

Ter pos  E,   Di m opoul os  MA.   My el om a  bone  di sease:  pathophy si ol ogy   and
m anagem ent.   Ann  Oncol   2005;16:1223.

P. 15

Penicillin Allergy
1.   What  ar e  the  cl i ni cal   pi ctur es  that  can  r esul t  f r om   an  al l er gy   to
peni ci l l i n?

2.   What  ar e  the  i m m unol ogi c  m echani sm s  r esponsi bl e  f or   the  cl i ni cal


sy ndr om es  associ ated  w i th  peni ci l l i n  al l er gy ?

Discussion
1.   What  ar e  the  cl i ni cal   pi ctur es  that  can  r esul t  f r om   an  al l er gy   to
peni ci l l i n?
Ther e  ar e  tw o  categor i es  of   cl i ni cal   pi ctur es  that  can  r esul t  f r om
peni ci l l i n  al l er gy :  acute  and  subacute.   Peni ci l l i n  al l er gi es  can  be
m edi ated  by   IgE  or   IgG  anti bodi es.   The  acute  al l er gi c  r eacti on  can  ar i se
i m m edi atel y   or   r api dl y ,   w i thi n  a  m atter   of   m i nutes  to  an  hour   or   tw o.
It  can  i ncl ude  sudden  anaphy l ax i s  w i th  hy potensi on,   or   asthm a,   r hi ni ti s,
and  ur ti car i a  (see  questi on  1  under   the  secti on  on  Anaphy l ax i s).
Conti nued  peni ci l l i n  adm i ni str ati on  can  cause  conti nued  sy m ptom s.   A
l ess  dr am ati c  pi ctur e  m ay   occur   7  to  10  day s  af ter   peni ci l l i n  tr eatm ent
star ts,   or   1  to  2  day s  af ter   r epeat  ther apy .   In  thi s  setti ng,   the  pi ctur e
i s  subacute  and  can  i ncl ude  ur ti car i a,   f ev er ,   and  ar thr al gi as  or
ar thr i ti s,   and  r ar el y   nephr i ti s  or   neur i ti s.

2.   What  ar e  the  i m m unol ogi c  m echani sm s  r esponsi bl e  f or   the  cl i ni cal


sy ndr om es  associ ated  w i th  peni ci l l i n  al l er gy ?

Acute  r eacti ons  r esul t  f r om   peni ci l l i n  r eacti ng  w i th  pr ef or m ed  IgE  to
peni ci l l i n,   a  r esul t  of   pr ev i ous  peni ci l l i n  tr eatm ent  that  m ay   hav e
pr oduced  no  v i si bl e  al l er gi c  r eacti on.   The  IgE  i s  bound  to  FcεRI  on
m ast  cel l s  and  basophi l s.   When  the  peni ci l l i n  hapten  bi nds  to  the  IgE,
the  m ast  cel l s  and  basophi l s  degr anul ate,   r el easi ng  hi stam i ne  and  other
m edi ator s.   These  substances  ar e  r esponsi bl e  f or   pr oduci ng  the  si gns
and  sy m ptom s.   The  subacute  r eacti on  i s  caused  by   pr ef or m ed  IgG  to
peni ci l l i n,   al so  a  r esul t  of   pr ev i ous  peni ci l l i n  tr eatm ent.   The  IgG  f i x es
com pl em ent.   The  com bi nati on  of   peni ci l l i n  and  the  IgG  anti body   f or m
an  i m m une  com pl ex .   When  thi s  i s  deposi ted  i n  the  ti ssue,   com pl em ent
i s  acti v ated  and  the  com pl em ent  br eak dow n  pr oducts  pr oduce
i nf l am m ati on.   The  i nf l am m ati on  i s  r esponsi bl e  f or   the  si gns  and
sy m ptom s  i n  the  or gans  w her e  the  i m m une  com pl ex es  l odge,   such  as
the  sk i n,   joi nts,   and  k i dney s.

Case
A  26­y ear ­ol d  w om an  w ho  has  m i tr al   stenosi s  r equi r es  ex tensi v e  dental
sur ger y .   Peni ci l l i n  pr ophy l ax i s  agai nst  str eptococci   i s  i ndi cated,   but  the
pati ent  i s  al l er gi c  to  peni ci l l i n.   She  states  that  15  y ear s  ago  she  had  hi v es
and  w heezi ng  30  m i nutes  af ter   she  had  tak en  or al   peni ci l l i n.

1.   How   w oul d  y ou  deter m i ne  w hether   the  pati ent  i s  l i k el y   to  hav e  an
al l er gi c  r eacti on  i f   she  i s  tr eated  w i th  peni ci l l i n  now ?
2.   What  do  y ou  do  i f   the  sk i n  test  r esul t  to  peni ci l l i n  i s  posi ti v e?
3.   What  i s  the  pr ev al ence  of   al l er gi c  cr oss­sensi ti v i ty   betw een  peni ci l l i n
and  cephal ospor i ns?
4.   If   the  pati ent's  sk i n  test  r esul t  to  peni ci l l i n  pr ov es  to  be  negati v e,   how
cer tai n  i s  i t  that  she  i s  not  al l er gi c?
P. 16
5.   If   the  sk i n  test  r esul t  to  peni ci l l i n  i s  posi ti v e,   coul d  y ou  av oi d  a
r eacti on  by   gi v i ng  peni ci l l i n  or al l y   i nstead  of   by   i njecti on?
6.   If   peni ci l l i n  m ust  be  used  because  ther e  i s  no  acceptabl e  al ter nati v e,
can  thi s  pati ent  be  r api dl y   desensi ti zed?

Case Discussion
1.   How   w oul d  y ou  deter m i ne  w hether   the  pati ent  i s  l i k el y   to  hav e  an
al l er gi c  r eacti on  i f   she  i s  tr eated  w i th  peni ci l l i n  now ?

Sk i n  testi ng  f or   peni ci l l i n  can  be  an  ex tr em el y   usef ul   pr ocedur e  f or


deter m i ni ng  w hether   a  pati ent,   w ho  has  a  hi stor y   of   peni ci l l i n  al l er gy
and  i n  w hom   an  IgE­m edi ated  i m m unol ogi c  m echani sm   i s  suspected,   i s
l i k el y   to  hav e  an  al l er gi c  r eacti on  to  a  l ater   ex posur e  to  peni ci l l i n.   If
al l   of   the  r eagents  ar e  av ai l abl e,   the  r el i abi l i ty   of   these  tests  has  been
as  hi gh  as  96%  i n  studi es  of   pati ents  w ho  had  a  hi stor y   of   al l er gy ,
w hose  sk i n  test  r esul ts  w er e  negati v e,   and  w ho  w er e  subsequentl y
chal l enged  w i th  peni ci l l i n.   The  testi ng  shoul d  be  done  by   a  per son
f am i l i ar   w i th  the  pr ocedur e.   The  r eagents  used  i ncl ude  hi stam i ne  (the
posi ti v e  contr ol ),   sal i ne  (the  negati v e  contr ol ),   peni ci l l oy l   pol y l y si ne
(Pr e­Pen),   the  m i nor   deter m i nant  m i x   (MDM),   and  the  peni ci l l i n  that
w i l l   be  used  f or   tr eatm ent.   For   the  test  r esul t  to  be  posi ti v e,   the
pati ent  m ust  show   a  posi ti v e  r eacti on  to  hi stam i ne,   a  negati v e  r eacti on
to  sal i ne,   and  a  posi ti v e  r eacti on  to  Pr e­Pen,   MDM,   and/or   the  nati v e
peni ci l l i n.   The  posi ti v e  r eacti on  consi sts  of   a  w heal   and  f l ar e  that
appear s  i n  15  m i nutes.   U nf or tunatel y ,   because  of   the  r ar e  need  f or   thi s
test  and  ex tr em e  cauti on  by   the  U . S.   Food  and  Dr ug  Adm i ni str ati on
(FDA),   nei ther   Pr e­Pen  nor   the  MDM  i s  av ai l abl e.   For   thi s  r eason,   w e
al m ost  al w ay s  use  al ter nati v e  dr ugs.   If   the  hi stor y   i s  not  suggesti v e  of
a  ty pe  I  i m m edi ate  hy per sensi ti v i ty   r eacti on  and  no  al ter nati v e  dr ugs
ar e  sati sf actor y ,   a  test  dose  i s  gi v en  under   contr ol l ed  condi ti ons.   If   the
hi stor y   i s  suggesti v e  of   a  ty pe  I  r eacti on  and  no  al ter nati v e  dr ug  i s
av ai l abl e,   w e  can  desensi ti ze  the  pati ent  (see  questi on  6  i n  f ol l ow i ng
tex t).

2.   What  do  y ou  do  i f   the  sk i n  test  r esul t  to  peni ci l l i n  i s  posi ti v e?

Fi r st,   y ou  al w ay s  l ook   f or   an  ef f ecti v e  nonpeni ci l l i n  dr ug  substi tute  and
use  i t.   If   one  i s  not  f ound,   consi der   desensi ti zi ng  the  pati ent  to
peni ci l l i n.

3.   What  i s  the  pr ev al ence  of   al l er gi c  cr oss­sensi ti v i ty   betw een  peni ci l l i n


and  cephal ospor i ns?

The  cephal ospor i ns  r esem bl e  the  peni ci l l i ns  chem i cal l y ,   but  the  tr ue
pr ev al ence  of   cr oss­r eacti v i ty   betw een  sem i sy ntheti c  peni ci l l i ns  and
cephal ospor i ns  i s  not  k now n  because  i nv esti gator s  ci te  di scor dant
r esul ts.   A  r easonabl e  esti m ate  i s  that  5%  of   peni ci l l i n­al l er gi c  pati ents
ar e  sensi ti v e  to  thi r d­gener ati on  cephal ospor i ns.

4.   If   the  pati ent's  sk i n  test  r esul t  to  peni ci l l i n  pr ov es  to  be  negati v e,   how
cer tai n  i s  i t  that  she  i s  not  al l er gi c?
Assum i ng  that  the  enti r e  panel   of   sk i n  tests  (i ncl udi ng  the  contr ol s)
w er e  done  pr oper l y   and  the  r esul ts  i nter pr eted  cor r ectl y ,   a  negati v e
sk i n  test  r esul t  i s  a  r el i abl e
P. 17
i ndi cator   that  an  acute  IgE­m edi ated  r eacti on  w i l l   not  occur .   How ev er ,
the  sk i n  test  has  no  bear i ng  on  IgG­m edi ated  r eacti ons.

5.   If   the  sk i n  test  r esul t  to  peni ci l l i n  i s  posi ti v e,   coul d  y ou  av oi d  a


r eacti on  by   gi v i ng  peni ci l l i n  or al l y   i nstead  of   by   i njecti on?

N o.   Or al   peni ci l l i n  can  al so  sensi ti ze  and  el i ci t  an  acute  r eacti on  i n
al r eady   sensi ti zed  i ndi v i dual s.

6.   If   peni ci l l i n  m ust  be  used  because  ther e  i s  no  acceptabl e  al ter nati v e,
can  thi s  pati ent  be  r api dl y   desensi ti zed?

Yes.   How ev er ,   thi s  i s  a  potenti al l y   danger ous  and  al w ay s  ti m e­


consum i ng  pr ocedur e.   It  shoul d  be  done  i n  the  i ntensi v e  car e  uni t  by   an
ex per i enced  al l er gi st  usi ng  publ i shed  pr otocol s.

Suggested Readings
Gr uchal l a  RS,   Pi r m oham ed  M.   Cl i ni cal   pr acti ce.   Anti bi ot  al l er gy .   N   Engl   J
Med  2006;354:601.

Pi chi cher o  ME.   A  r ev i ew   of   ev i dence  suppor ti ng  the  Am er i can  Academ y


of   Pedi atr i cs  r ecom m endati on  f or   pr escr i bi ng  cephal ospor i n  anti bi oti cs
f or   peni ci l l i n­al l er gi c  pati ents.   Pedi atr i cs  2005;115:1048.
Editors :  Sc hrie r,  Robe rt W .
Title :  Inte rna l Me dic ine  Ca s e book ,  The : Re a l P a tie nts ,  Re a l Ans w e rs ,  3rd Edition

Copy r i ght  ©2007  Li ppi ncott  Wi l l i am s  &  Wi l k i ns

>  T a b le   o f   C o nte nts   >  C ha p te r   2  ­  C a r d io lo g y

Chapter 2
Cardiology

Simon Sha k a r
Rona ld Zolty
Joa nn Linde nfe ld

Acute Pericarditis and Cardiac Tamponade
1.   What  ar e  the  m ost  com m on  causes  of   acute  per i car di ti s?

2.   What  i s  car di ac  tam ponade?

3.   Does  acute  per i car di ti s  of ten  r esul t  i n  car di ac  tam ponade?

4.   What  ar e  the  si gns  and  sy m ptom s  of   per i car di ti s  and  tam ponade?

5.   How   i s  the  echocar di ogr am   hel pf ul   i n  the  di agnosi s  of   per i car di ti s  or   tam ponade?

6.   What  i s  the  tr eatm ent  f or   car di ac  tam ponade?

P. 19

Discussion
1.   What  ar e  the  m ost  com m on  causes  of   acute  per i car di ti s?

The  m ost  com m on  causes  of   acute  per i car di ti s  ar e  i di opathi c,   v i r al   i nf ecti on,   ur em i a,   m y ocar di al
i nf ar cti on  MI),   tr aum a,   car di ac  sur ger y ,   and  neopl asm .

2.   What  i s  car di ac  tam ponade?

Car di ac  tam ponade  r esul ts  f r om   accum ul ati on  of   f l ui d  w i thi n  the  per i car di um .   As  f l ui d  accum ul ates,
i ntr aper i car di al   pr essur e  i ncr eases,   l i m i ti ng  f i l l i ng  of   the  hear t  and  r educi ng  str ok e  v ol um e.   As
i ntr aper i car di al   pr essur e  r i ses,   car di ac  f i l l i ng  i s  i ncr easi ngl y   l i m i ted.   U l ti m atel y ,   pr essur es  equal i ze
i n  the  l ef t  atr i um ,   pul m onar y   v ascul atur e,   r i ght  atr i um ,   and  super i or   v ena  cav a  (SVC);  v entr i cul ar
f i l l i ng  i s  pr ogr essi v el y   i m pai r ed  and  ci r cul ator y   col l apse  ensues.

3.   Does  acute  per i car di ti s  of ten  r esul t  i n  car di ac  tam ponade?

Acute  per i car di ti s  r esul ts  i n  tam ponade  onl y   r ar el y .   Tam ponade  i s  m or e  com m on  i n  end­stage  r enal
di sease  and  neopl asti c  di sease  despi te  the  f r equent  absence  of   an  i denti f i abl e  epi sode  of   acute
per i car di ti s  i n  these  condi ti ons.

4.   What  ar e  the  si gns  and  sy m ptom s  of   per i car di ti s  and  tam ponade?

The  m ost  com m on  sy m ptom   of   acute  per i car di ti s  i s  chest  pai n.   The  pai n  i s  gener al l y   shar p  and  i s
w or se  w i th  cough,   deep  i nspi r ati on,   and  r ecum bency .   A  per i car di al   f r i cti on  r ub  i s  the  m ost  com m on
f i ndi ng  i n  acute  per i car di ti s.   It  of ten  has  thr ee  com ponents  that  occur   i n  sy stol e,   and  ear l y   and  l ate
di astol e  w hen  the  hear t  i s  m ov i ng  and  the  per i car di al   sur f aces  r ub  agai nst  one  another .   Sy m ptom s
of   tam ponade  depend  on  the  degr ee  of   hem ody nam i c  com pr om i se.   The  com m on  sy m ptom s  of
per i car di al   ef f usi on  w i th  tam ponade  i ncl ude  dy spnea  (80%),   cough  (30%),   or thopnea  (25%),   and
chest  pai n  (20%).   The  com m on  si gns  of   per i car di al   ef f usi on  w i th  tam ponade  ar e  jugul ar   v enous
di stensi on  and  tachy car di a  (both  near l y   100%),   pul sus  par adox us  (89%),   sy stol i c  bl ood  pr essur e  â
‰¤90  m m   Hg  (52%),   and  per i car di al   r ub  (22%).

5.   How   i s  the  echocar di ogr am   hel pf ul   i n  the  di agnosi s  of   per i car di ti s  or   tam ponade?

The  echocar di ogr am   i s  the  m ost  accur ate  and  easi l y   av ai l abl e  tool   to  detect  and  quanti f y
per i car di al   f l ui d.   How ev er ,   i t  i s  of ten  not  of   di agnosti c  v al ue  i n  acute  per i car di ti s  because  the
absence  of   per i car di al   f l ui d  does  not  ex cl ude  the  di agnosi s  of   acute  per i car di ti s,   especi al l y   i n
i di opathi c  or   v i r al   per i car di ti s.   In  pati ents  w i th  per i car di ti s  due  to  neopl asm ,   bacter i al   i nf ecti on,
tr aum a,   or   car di ac  sur ger y ,   the  echocar di ogr am   m ay   pr ov i de  hel pf ul   i nf or m ati on  about  the  eti ol ogy
of   the  ef f usi on.   For   ex am pl e,   m etastases  m ay   be  v i si bl e  on  the  per i car di al   sur f aces.

The  echocar di ogr am   i s  the  m ost  com m onl y   used  techni que  f or   the  di agnosi s  of   car di ac  tam ponade.
Ty pi cal   f i ndi ngs  i n  addi ti on  to  the  pr esence  of   per i car di al   f l ui d  i ncl ude  r i ght  atr i al   and  r i ght
v entr i cul ar   di astol i c  col l apse,   ex agger ated  r espi r ator y   changes  i n  tr i cuspi d  and  m i tr al   v al v e  f l ow ,
and  pl ethor a  of   the  i nf er i or   v ena  cav a.   Because  the  l i m i tati on  of   car di ac  f i l l i ng  i s  pr ogr essi v e  as
the  ef f usi on  i ncr eases,   f i ndi ngs  of   tam ponade  m ay   be  detected  by
P. 20
echocar di ogr am   bef or e  the  cl assi cal l y   descr i bed  cl i ni cal   tr i ad  of   hy potensi on,   par adox i cal   pul se,
and  i ncr eased  sy stem i c  v enous  pr essur e.

6.   What  i s  the  tr eatm ent  f or   car di ac  tam ponade?

Car di ac  tam ponade  r equi r es  i m m edi ate  tr eatm ent  to  r el i ev e  the  i ncr eased  end­di astol i c  pr essur e
and  i nadequate  v entr i cul ar   f i l l i ng.   The  tr eatm ent  of   car di ac  tam ponade  consi sts  of   w i thdr aw al   of
f l ui d  f r om   the  per i car di al   space,   gener al l y   thr ough  a  needl e  i nser ted  per cutaneousl y —a  pr ocedur e
cal l ed  per i car di ocentesi s.   Per i car di ocentesi s  m ay   be  per f or m ed  usi ng  echocar di ogr aphi c  gui dance  to
pl ace  a  needl e  or   a  catheter   i n  the  i ntr aper i car di al   space  or   i n  the  car di ac  catheter i zati on
l abor ator y   usi ng  f l uor oscopi c  gui dance.   Intr av enous  (IV)  f l ui ds  such  as  bl ood  or   sal i ne  m ay   be
used,   but  onl y   as  a  tem por i zi ng  m easur e.   Vol um e  adm i ni str ati on  i s  usef ul   onl y   i n  hy pov ol em i c
pati ents.   In  nor m ov ol em i c  pati ents,   the  adm i ni str ati on  of   f l ui d  m ay   ex acer bate  the  i ntr aper i car di al
pr essur e.

Case
A  78­y ear ­ol d  m an  w i th  a  past  hi stor y   r em ar k abl e  onl y   f or   gout  i s  seen  because  of   the  acute  onset  of
chest  pai n.   He  descr i bes  a  4­day   pr odr om e  of   r hi nor r hea,   nonpr oducti v e  cough,   m y al gi as,   and  anor ex i a.
Appr ox i m atel y   8  hour s  bef or e  he  i s  seen  i n  the  em er gency   r oom   (ER),   he  began  to  noti ce  the  gr adual
onset  of   shar p  subster nal   chest  pai n,   w or se  w i th  i nspi r ati on,   r el i ev ed  by   si tti ng  up,   and  associ ated  w i th
di aphor esi s.
The  pai n  i s  sl i ghtl y   w or se  w i th  ex er ti on  but  i s  not  r el i ev ed  by   subl i ngual   ni tr ogl y cer i n  (N TG)
adm i ni ster ed  i n  the  ER,   al though  m or phi ne  sul f ate  and  ox y gen  do  seem   to  al l ev i ate  hi s  di scom f or t.   Hi s
tem per atur e  i s  101°F  (38. 5°C),   hi s  hear t  r ate  i s  105  beats  per   m i nute  and  r egul ar ,   hi s  r espi r ator y
r ate  i s  17  per   m i nute,   and  hi s  bl ood  pr essur e  i s  105/65  m m   Hg.   The  r em ai nder   of   the  phy si cal
ex am i nati on  i s  nor m al .   The  el ectr ocar di ogr am   (ECG)  i s  i nter pr eted  by   the  ER  staf f   to  show   “si nus
tachy car di a  w i th  ST­segm ent  el ev ati ons  i nf er i or l y   and  nonspeci f i c  ST­  and  T­w av e  changes  el sew her e. â€​
An  ar ter i al   bl ood  gas  deter m i nati on  per f or m ed  on  r oom   ai r   show s  nor m al   ar ter i al   ox y genati on.   The
chest  r adi ogr aphi c  study   i s  nor m al .
The  ER  staf f   star ts  an  IV  hepar i n  dr i p  and  a  pl atel et  gl y copr otei n  IIb­IIIa  i nhi bi tor   i nf usi on  f or   the
tr eatm ent  of   a  pr esum ed  acute  cor onar y   sy ndr om e  (ACS).   An  IV  N TG  i nf usi on  and  ox y gen  ther apy   ar e
i nsti tuted  but,   despi te  these  m easur es,   the  pai n  conti nues.   The  car di ac  catheter i zati on  team   i s  cal l ed  to
consi der   cor onar y   angi ogr aphy .   Antaci d  ther apy   does  not  r el i ev e  the  pai n  and  onl y   m or phi ne  sul f ate
seem s  to  of f er   r el i ef .   Bl ood  tests  r ev eal   a  nor m al   tr oponi n,   nor m al   el ectr ol y tes,   nor m al   D­di m er ,   and
nor m al   r enal   f uncti on.   The  hem ogl obi n  i s  nor m al   but  the  w hi te  bl ood  cel l   count  i s  m i l dl y   el ev ated.
The  pati ent  i s  tak en  to  the  catheter i zati on  l abor ator y   and  hi s  cor onar y   angi ogr am   r ev eal s  di f f use,   m i l d,
nonobstr ucti v e  cor onar y   ar ter y   di sease  (CAD).   The  IIb­IIIa  i nhi bi tor   i s  di sconti nued.   When  the  pati ent  i s
tr ansf er r ed  to  the  cor onar y   car e  uni t,   the  ECG  show s  conti nued  “ev ol uti onâ€​   w i th  ST­segm ent
el ev ati ons  of   l ess  than  2  m m   i n  l eads  I,   II,   III,   aVL,   aVF,   and  V 2   to  V 6   that  do  not  r espond  to  IV  N TG.
The  pati ent's  chest  pai n  per si sts.
Fur ther   i ncr em ents  of   N TG  ar e  gi v en  i n  an  IV  i nf usi on  and  the  pati ent's  bl ood  pr essur e  begi ns  to
decr ease.   Af ter   2  hour s,   the  pati ent  conti nues  to  w r i the  i n  pai n,   com pl ai ns  of   f eel i ng  di zzy   and  hav i ng  a
sev er e  headache,   and  v om i ts  af ter   the  f i f th  dose  of   IV  m or phi ne
P. 21
sul f ate.   You  ar e  ask ed  to  see  the  pati ent  and  y our   ex am i nati on  r ev eal s  si nus  tachy car di a,   a  bl ood
pr essur e  of   82/50  m m   Hg  (no  pul sus  par adox us),   a  r espi r ator y   r ate  of   16  per   m i nute,   a  tem per atur e  of
101°F  (38. 5°C),   cl ear   l ung  f i el ds,   and  no  el ev ati on  i n  the  jugul ar   v enous  pr essur e,   but  a  thr ee­
com ponent  per i car di al   f r i cti on  r ub  i s  hear d  ov er   the  pr ecor di um .   The  hem ogl obi n  l ev el   i s  stabl e.

1.   What  i s  the  m ost  l i k el y   cl i ni cal   di agnosi s  of   thi s  pati ent's  chest  pai n?
2.   On  the  basi s  of   y our   cl i ni cal   i m pr essi on  of   thi s  pati ent's  pr esentati on,   w hat  f eatur es  w oul d  be
ex pected  on  the  ECG?
3.   Is  a  nor m al   tr oponi n  hel pf ul   i n  acute  MI?
4.   What  i s  the  m ost  ef f ecti v e  tr eatm ent  f or   acute  per i car di ti s?
5.   What  i s  the  m ost  l i k el y   cause  of   the  hy potensi on  i n  thi s  pati ent

Case Discussion
1.   What  i s  the  m ost  l i k el y   cl i ni cal   di agnosi s  of   thi s  pati ent's  chest  pai n?

The  m ost  l i k el y   cl i ni cal   di agnosi s  of   thi s  pati ent's  chest  pai n  i s  acute  i di opathi c  or   v i r al
per i car di ti s.   Rel ati v el y   com m on  causes  of   acute  chest  pai n  that  m ust  be  consi der ed  ar e  MI  or   ACS,
per i car di ti s,   aor ti c  di ssecti on,   pneum oni a,   pul m onar y   em bol us,   costochondr i ti s,   and  pneum othor ax .
The  per ti nent  f eatur es  of   the  hi stor y   and  phy si cal   ex am i nati on  that  l ead  to  thi s  di agnosi s  ar e  that
the  pai n  w as  pr eceded  by   a  v i r al   pr odr om e  and  w as  v er y   cl ear l y   posi ti onal   and  ex acer bated  by
i nspi r ati on,   w hi ch  str ongl y   suggests  per i car di al   pai n.   Per i car di al   pai n  does  not  i m pr ov e  w i th  N TG,
but  the  l ack   of   r esponse  to  N TG  does  not  ex cl ude  an  acute  MI.   The  pati ent's  v i tal   si gns  w er e  stabl e
ex cept  f or   a  sl i ght  f ev er   and  tachy car di a  that  ar e  al so  v er y   f r equent  i n  ei ther   acute  per i car di ti s  or
MI.   The  absence  of   tachy pnea,   together   w i th  the  nor m al   ex am i nati on  f i ndi ngs  and  nor m al   D­di m er ,
m ak e  acute  pul m onar y   em bol i zati on  unl i k el y .   Acute  costochondr i ti s  i s  of ten  posi ti onal   but
associ ated  w i th  ex qui si te  pai n  on  pal pati on  of   the  i nv ol v ed  costochondr al   juncti on,   and  i s  not
associ ated  w i th  ECG  changes.   If   the  ex am i nati on  and  chest  r adi ogr aphi c  f i ndi ngs  ar e  nor m al   and
ther e  i s  no  past  hi stor y   of   sm ok i ng,   f or cef ul   coughi ng,   or   tr aum a,   the  l i k el i hood  of   acute
pneum othor ax   i s  l ow .

The  r em ai ni ng  tw o  di agnoses,   acute  per i car di ti s  v er sus  MI,   can  of ten  be  di f f er enti ated  on  the  basi s
of   the  hi stor y   and  phy si cal   ex am i nati on  f i ndi ngs,   the  ECG,   and  tr oponi n.   The  shar p  qual i ty   of   the
subster nal   chest  pai n,   w hi ch  i s  associ ated  m or e  w i th  the  r ecum bent  posi ti on,   deep  br eathi ng,   and
coughi ng,   and  w hi ch  i s  i m pr ov ed  by   si tti ng  up,   i s  aty pi cal   f or   MI  but  a  cl assi c  sy m ptom   of
per i car di ti s.   The  ECG  w as  i ni ti al l y   m or e  consi stent  w i th  per i car di ti s  but  an  acute  MI  coul d  not  be
ex cl uded.   The  absence  of   si gni f i cant  cor onar y   obstr ucti on  str ongl y   ar gued  agai nst  an  acute  MI,   a
f i ndi ng  conf i r m ed  by   the  nor m al   tr oponi n.

2.   On  the  basi s  of   y our   cl i ni cal   i m pr essi on  of   thi s  pati ent's  pr esentati on,   w hat  f eatur es  w oul d  be
ex pected  on  the  ECG?

Si nus  tachy car di a  and  ST­segm ent  el ev ati on  ar e  of ten  the  ear l i est  ECG  f i ndi ngs,   al though  the
absence  of   ECG  changes  does  not  ex cl ude  the  di agnosi s  of   per i car di ti s.
P. 22
The  ty pi cal   changes  of   acute  per i car di ti s  of ten  ev ol v e  ov er   hour s  or   day s  and  ar e  thought  to  be
caused  by   a  m y ocar di al   cur r ent  of   i njur y   due  to  i nf l am m ati on.   The  ECG  i n  acute  per i car di ti s  ev ol v es
usual l y   thr ough  f our   stages  ov er   sev er al   day s.   Ther e  i s  ear l y   di f f use  ST­segm ent  el ev ati on  i n  stage
1.   Thi s  di f f er s  f r om   the  ST­segm ent  el ev ati on  of   acute  MI,   w hi ch  i s  usual l y   l ocal i zed  (anter i or ,
i nf er i or ,   or   l ater al ),   w i th  the  ST  segm ents  conv ex   upw ar d.   In  per i car di ti s,   the  ST­segm ent  el ev ati on
i s  concav e  upw ar d  and  usual l y   i nv ol v es  al l   the  l eads  ex cept  aVR  and  V 1 .   Stage  2  i s  def i ned  by
nor m al i zati on  of   the  ST  segm ents  and  stage  3  i s  char acter i zed  by   the  dev el opm ent  of   di f f use  T­
w av e  i nv er si ons.   In  stage  4,   the  T  w av es  r etur n  to  thei r   nor m al   conf i gur ati on.   PR  segm ent
depr essi on  i s  al so  com m on  i n  the  ear l y   phases  of   acute  per i car di ti s  ev en  i n  the  absence  of   ST­
segm ent  el ev ati on  and  i s  str ongl y   suggesti v e  of   acute  per i car di ti s.   An  i m por tant  ex cepti on  i s  i n
per i car di ti s  f ol l ow i ng  an  acute  MI,   i n  w hi ch  ty pi cal   ECG  changes  of   per i car di ti s  m ay   not  be  pr esent
or   m ay   be  aty pi cal .

3.   Is  a  nor m al   tr oponi n  hel pf ul   i n  ex cl udi ng  an  acute  MI?

A  nor m al   tr oponi n  8  or   m or e  hour s  af ter   the  onset  of   chest  pai n  gener al l y   ex cl udes  acute  MI  but
does  not  ex cl ude  ACS.   How ev er ,   a  m i l dl y   el ev ated  tr oponi n  m ay   be  pr esent  w i th  acute
m y oper i car di ti s.   My ocar di ti s  i s  an  i nf l am m ator y   di sease  of   the  car di ac  m uscl e,   w hi ch  can  be  caused
by   a  v ar i ety   of   di f f er ent  i l l nesses,   m any   of   w hi ch  ar e  i nf ecti ous.   Ty pi cal l y ,   m y ocar di ti s  i s
associ ated  w i th  car di ac  enzy m e  el ev ati on  that  r ef l ects  m y ocar di al   necr osi s.   When  chest  pai n  occur s
i n  the  setti ng  of   m y ocar di ti s  i t  m ay   be  associ ated  w i th  concom i tant  per i car di ti s  and  i s  cal l ed
m y oper i car di ti s.

4.   What  i s  the  m ost  ef f ecti v e  tr eatm ent  f or   acute  per i car di ti s?

In  the  tr eatm ent  of   i di opathi c  or   v i r al   per i car di ti s,   the  goal s  of   ther apy   ar e  r el i ef   of   pai n  and
r esol uti on  of   i nf l am m ati on.   Fi r st­choi ce  ther apy   i s  the  adm i ni str ati on  of   nonster oi dal
anti i nf l am m ator y   dr ugs  (N SAIDs)  or   aspi r i n.   The  adm i ni str ati on  of   col chi ci ne  al one  or   i n
com bi nati on  w i th  N SAIDs  m i ght  be  another   ther apeuti c  al ter nati v e.   The  use  of   cor ti coster oi ds  i s
usual l y   r eser v ed  f or   pati ents  w i th  per i car di ti s  secondar y   to  autoi m m une  di sease.

5.   What  i s  the  m ost  l i k el y   cause  of   the  hy potensi on  i n  thi s  pati ent?

The  hy potensi on  i n  thi s  pati ent  i s  m ost  l i k el y   due  to  the  cum ul ati v e  ef f ects  of   the  m edi cati ons  he
has  been  gi v en  (m or phi ne  and  N TG).   The  accum ul ati on  and  potenti ati on  of   m edi cati ons,   especi al l y
i n  the  el der l y ,   i s  a  com m on  cl i ni cal   pr obl em   i n  the  acute  car e  setti ng.   The  com bi nati on  of   m or phi ne
and  N TG  i n  thi s  pati ent  m ay   hav e  i nduced  suf f i ci ent  v asodi l ati on  to  cause  hy potensi on.

Bl eedi ng  i s  al so  a  possi bl e  cause  of   the  hy potensi on.   The  adm i ni str ati on  of   IV  hepar i n,   aspi r i n,   and
pl atel et  gl y copr otei n  IIb­IIIa  i nhi bi tor   agents  m ay   r esul t  i n  gastr oi ntesti nal   bl eedi ng  and  m el anoti c
stool s.   The  absence  of   jugul ar   v enous  di stenti on  and  a  par adox i cal   pul se  ar gues  agai nst  tam ponade,
but  these  f i ndi ngs  m ay   be  absent  w i th  v asodi l ati on  or   v ol um e  depl eti on.   A  m or e  w or r i som e
possi bi l i ty   i s  hem or r hagi c  per i car di ti s,   especi al l y   because  a  new   f r i cti on  r ub  i s  hear d.   If   the
hy potensi on  does  not  r esol v e  qui ck l y   w i th  di sconti nuati on  of   N TG  and  m or phi ne,   an  echocar di ogr am
i s  i ndi cated  to  ex cl ude  car di ac  tam ponade.

P. 23

Suggested Readings
Im azi o  M,   Bobbi o  M,   Cecchi   E,   etal .   Col chi ci ne  i n  addi ti on  to  conv enti onal   ther apy   f or   acute
per i car di ti s:  r esul ts  of   the  COl chi ci ne  f or   acute  PEr i car di ti s  (COPE)  tr i al .   Ci r cul ati on
2005;112(13):2012–2016.

LeWi nter   MM,   Kabbani   S.   Per i car di al   di seases.   In:  Br aunw al d  E,   ed.   Hear t  di sease:  a  tex tbook   of
car di ov ascul ar   m edi ci ne,   7th  ed,   Phi l adel phi a:  WB  Saunder s,   2005:1757.

Mer ce  J,   Sagr i sta­Saul eda  J,   Per m any er ­Mi r al da  G,   etal .   Cor r el ati on  betw een  cl i ni cal   and  Doppl er
echocar di ogr aphi c  f i ndi ngs  i n  pati ents  w i th  m oder ate  and  l ar ge  per i car di al   ef f usi on:  i m pl i cati ons  f or
the  di agnosi s  of   car di ac  tam ponade.   Am   Hear t  J  1999;138:759–764.

Spodi ck   DH.   Acute  car di ac  tam ponade.   N   Engl   J  Med  2003;349:684–690.

Tr oughton  RW,   Asher   CR,   Kl ei n  AL.   Per i car di ti s.   Lancet  2004;363:717–727.

Acute Pulmonary Edema
1.   What  ar e  the  tw o  m ost  com m on  under l y i ng  m echani sm s  of   pul m onar y   edem a?

2.   What  ar e  the  m ost  com m on  causes  of   acute  car di ogeni c  pul m onar y   edem a?

3.   What  i s  the  i m m edi ate  tr eatm ent  of   acute  car di ogeni c  pul m onar y   edem a?

Discussion
1.   What  ar e  the  tw o  m ost  com m on  under l y i ng  m echani sm s  of   pul m onar y   edem a?

Acute  pul m onar y   edem a  can  hav e  a  c a rdioge nic   or   nonc a rdioge nic   eti ol ogy .   In  c a rdioge nic
pulmona ry e de ma ,   a  hi gh  pul m onar y   capi l l ar y   pr essur e  i s  r esponsi bl e  f or   the  tr ansudati on  of
pr otei n­poor   f l ui d  i nto  the  l ungs  caused  by   an  i m bal ance  of   Star l i ng's  f or ces.   Wi th  acute  r i ses  i n
pul m onar y   capi l l ar y   pr essur e,   the  pul m onar y   l y m phati cs  cannot  r api dl y   i ncr ease  the  r ate  of   f l ui d
r em ov al ;  as  a  r esul t,   pul m onar y   edem a  occur s.

Nonc a rdioge nic  pulmona ry e de ma   i s  caused  by   al ter ed  al v eol ar   capi l l ar y   per m eabi l i ty   due  to
acute  l ung  i njur y .   Tr ansudati on  of   f l ui d  i nto  the  al v eol ar   space  i s  not  dependent  on  an  el ev ated
pul m onar y   capi l l ar y   w edge  pr essur e  but  i s  ex acer bated  by   an  el ev ated  pul m onar y   capi l l ar y
pr essur e.   The  di sor der s  m ost  f r equentl y   r esul ti ng  i n  i ncr eased  per m eabi l i ty   pul m onar y   edem a  ar e
the  acute  r espi r ator y   di str ess  sy ndr om e  (ARDS)  and,   l ess  com m onl y ,   hi gh  al ti tude  and  neur ogeni c
pul m onar y   edem a.
2.   What  ar e  the  m ost  com m on  causes  of   acute  car di ogeni c  pul m onar y   edem a?

The  m ost  com m on  causes  of   acute  car di ogeni c  pul m onar y   edem a  ar e  acute  i schem i a  and
accel er ated  hy per tensi on,   both  causi ng  a  sudden  i ncr ease  i n  l ef t  v entr i cul ar   end­di astol i c  pr essur e.
Both  eti ol ogi es  r esul t  i n  a  sti f f   l ef t  v entr i cl e  and  decr eased  di astol i c  v entr i cul ar   com pl i ance,
i m pai r i ng  v entr i cul ar   f i l l i ng  dur i ng  di astol e  (di astol i c  dy sf uncti on).   Sy stol i c  dy sf uncti on  m ay   al so
occur .   Other   causes  of   acute  car di ogeni c  pul m onar y   edem a  i ncl ude  acute  m i tr al   r egur gi tati on  such
as  m i ght  r esul t  f r om   acute  i schem i a  or   a  r uptur ed  chor dae
P. 24
tendi nea,   or   i nf ecti ous  endocar di ti s,   or   di sconti nuati on  of   anti hy per tensi v e  m edi cati ons.   Acute
pul m onar y   edem a  m ay   be  pr eci pi tated  by   r api d  atr i al   f i br i l l ati on  or   other   dy sr hy thm i as.   Inf ecti on,
phy si cal   or   env i r onm ental   str esses,   changes  or   noncom pl i ance  w i th  m edi cal   ther apy ,   di etar y
i ndi scr eti on,   or   i atr ogeni c  v ol um e  ov er l oad  ar e  l ess  com m on,   but  i m por tant,   causes.

3.   What  i s  the  i m m edi ate  tr eatm ent  of   acute  car di ogeni c  pul m onar y   edem a?

The  i m m edi ate  tr eatm ent  of   acute  car di ogeni c  pul m onar y   edem a  shoul d  consi st  of   ox y gen  ther apy
to  m ai ntai n  an  ox y gen  satur ati on  w i thi n  the  nor m al   r ange  (95%  to  98%),   noni nv asi v e  posi ti v e­
pr essur e  v enti l ati on  i f   ox y gen  satur ati on  r em ai ns  l ow   [i . e. ,   conti nuous  posi ti v e  ai r w ay   pr essur e
(CPAP)  or   bi ­l ev el   posi ti v e  ai r w ay   pr essur e  (Bi PAP)],   IV  di ur esi s  w i th  f ur osem i de  or   other   l oop
di ur eti cs,   IV  m or phi ne,   and  IV  v asodi l ator s  w i th  N TG,   ni tr opr ussi de,   or   angi otensi n­conv er ti ng
enzy m e  (ACE)  i nhi bi tor s.   The  pati ent  shoul d  be  si tti ng  upr i ght  unl ess  hy potensi on  i s  pr esent.   If   the
pati ent  has  an  ACS,   ther apy   shoul d  be  dom i nated  by   i nter v enti on  to  m i ni m i ze  i schem i c  i njur y .   If   the
acute  pul m onar y   edem a  i s  associ ated  w i th  shock ,   IV  i notr opi c  dr ugs  such  as  m i l r i none  or
dobutam i ne  m ay   be  necessar y .   If   sev er e  hy per tensi on  i s  pr esent,   IV  ni tr opr ussi de  or   other   r api dl y
acti ng  agents  such  as  l abetal ol   shoul d  be  gi v en  to  l ow er   sy stem i c  bl ood  pr essur e.   N oni nv asi v e
posi ti v e­pr essur e  v enti l ati on  w i th  CPAP  or   Bi PAP  has  been  show n  to  r educe  the  need  f or   i nv asi v e
m echani cal   v enti l ati on  i n  pati ents  w i th  acute  car di ogeni c  pul m onar y   edem a  and  ev en  to  r educe
m or tal i ty   com par ed  w i th  standar d  ther apy   (ox y gen  by   f ace  m ask ,   di ur eti cs,   and  ni tr ates);  the  sam e
has  been  show n  i n  a  r ecent  m eta­anal y si s  study .   (see  secti on  on  Essenti al   Hy per tensi on  and
Hy per tensi v e  Em er genci es).

Case
A  65­y ear ­ol d  m an  w i th  a  hi stor y   of   hy per tensi on,   di abetes  m el l i tus,   and  ex er ti onal   chest  pr essur e  i s
seen  i n  the  ER  com pl ai ni ng  of   sudden  onset  of   chest  pai n  and  sev er e  dy spnea  at  r est.   He  i s  cur r entl y
tak i ng  enal apr i l   (5  m g  tw i ce  a  day )  to  contr ol   hi s  bl ood  pr essur e.   Phy si cal   ex am i nati on  r ev eal s  a  pal e
w hi te  m al e  i n  acute  r espi r ator y   di str ess,   w ho  i s  anx i ous  and  di aphor eti c.   Hi s  bl ood  pr essur e  i s  180/100
m m   Hg,   hi s  api cal   pul se  i s  170  beats  per   m i nute  and  i r r egul ar l y   i r r egul ar ,   and  hi s  r espi r ator y   r ate  i s  40
per   m i nute.   Ex am i nati on  of   the  l ungs  r ev eal s  r al es  ex tendi ng  tw o  thi r ds  up  f r om   the  base  of   the  l ung
f i el ds  bi l ater al l y .   Ex am i nati on  of   the  hear t  r ev eal s  a  jugul ar   v enous  pr essur e  of   12  cm   of   w ater ,   a  thi r d
sound  (S 3 ),   and  a  gr ade  2/6  hol osy stol i c  m ur m ur   hear d  at  the  apex .   Ar ter i al   bl ood  gas  deter m i nati ons
per f or m ed  on  r oom   ai r   show   a  par ti al   pr essur e  of   ox y gen  of   50  m m   Hg,   a  par ti al   pr essur e  of   car bon
di ox i de  of   30  m m   Hg,   and  a  pH  of   7. 48.   A  chest  r adi ogr aph  show s  an  enl ar ged  hear t  and  pul m onar y
edem a.   The  ECG  r ev eal s  atr i al   f i br i l l ati on  w i th  a  v entr i cul ar   r esponse  of   170  beats  per   m i nute,   a  l oss  of
R  w av es,   and  4  m m   of   ST  el ev ati on  anter i or l y —  f i ndi ngs  that  ar e  consi stent  w i th  an  acute  anter i or   MI.
A  di agnosi s  of   acute  anter i or   w al l   MI  com pl i cated  by   atr i al   f i br i l l ati on  and  pul m onar y   edem a  i s  m ade.

1.   What  i s  causi ng  the  pul m onar y   edem a  i n  thi s  pati ent?
2.   What  m edi cal   ther apy   shoul d  be  used  to  tr eat  thi s  pati ent  acutel y ,   and  w hy ?

P. 25

Case Discussion
1.   What  i s  causi ng  the  pul m onar y   edem a  i n  thi s  pati ent?

Ther e  ar e  sev er al   causes  of   the  pul m onar y   edem a  i n  thi s  pati ent.

1.   MI  i m pai r s  both  the  sy stol i c  and  di astol i c  f uncti on  of   the  l ef t  v entr i cl e.   A  l oss  of   the
contr acti l e  f uncti on  of   the  l ar ge  anter i or   w al l   of   the  l ef t  v entr i cl e  (sy stol i c  dy sf uncti on)  and
acute  sti f f eni ng  of   the  dam aged  m y ocar di um   (di astol i c  dy sf uncti on)  l ead  to  el ev ated  f i l l i ng
pr essur es  of   the  l ef t  v entr i cl e  and  the  l ef t  atr i um .   El ev ated  pul m onar y   v enous  and  pul m onar y
capi l l ar y   pr essur es  pr oduce  an  i m bal ance  i n  the  Star l i ng's  f or ces,   r esul ti ng  i n  the  tr ansudati on
of   f l ui d  i nto  the  i nter sti ti um   and  then  i nto  the  al v eol ar   space.
2.   Atria l fibrilla tion  w i th  a  r api d  v entr i cul ar   r esponse  (170  beats  per   m i nute)  contr i butes  to  the
pul m onar y   edem a  because  (a)  the  l oss  of   atr i al   sy stol i c  contr acti on  i m pai r s  l ef t  v entr i cul ar
f i l l i ng,   w hi ch  f ur ther   el ev ates  the  l ef t  atr i al   pr essur e;  (b)  the  r api d  v entr i cul ar   r ate  r esul ts  i n
si gni f i cant  shor teni ng  of   di astol i c  f i l l i ng  ti m e  f ur ther   i m pai r i ng  f i l l i ng  of   the  l ef t  v entr i cl e;  and
(c)  the  r api d  v entr i cul ar   r ate  i ncr eases  m y ocar di al   ox y gen  dem ands,   w hi ch  m ay   i ncr ease
i schem i a,   w hi ch  i n  tur n  w or sens  the  pul m onar y   edem a.

3.   Hype rte ns ion,   especi al l y   w hen  chr oni c  and  poor l y   contr ol l ed,   pr oduces  a  sti f f ,   hy per tr ophi ed
m y ocar di um   causi ng  el ev ated  v entr i cul ar   f i l l i ng  pr essur es.   In  the  setti ng  of   acute  MI,   an
i ncr ease  i n  bl ood  pr essur e  caused  by   anx i ety ,   pai n,   a  catechol am i ne  sur ge,   and  per i pher al
v asoconstr i cti on  augm ents  the  af ter l oad  agai nst  w hi ch  the  al r eady   com pr om i sed  l ef t  v entr i cl e
has  to  w or k .   Thi s  l eads  to  a  f ur ther   el ev ati on  i n  v entr i cul ar   f i l l i ng  pr essur es,   and  w or sens  any
i schem i a  and  m i tr al   r egur gi tati on  al r eady   pr esent.

4.   Anx ie ty  secondar y   to  the  pai n  and  br eathl essness  i s  l i k el y   to  i ncr ease  the  hear t  r ate  and
bl ood  pr essur e,   ther eby   contr i buti ng  to  pul m onar y   edem a  by   i ncr easi ng  the  af ter l oad.

5.   A s ys tolic  murmur  i n  thi s  setti ng  m ost  l i k el y   r epr esents  m i tr al   r egur gi tati on  secondar y   to
i schem i a  and  papi l l ar y   m uscl e  dy sf uncti on  or ,   l ess  com m onl y ,   r uptur e  of   papi l l ar y   m uscl e,   or
an  acute  v entr i cul ar   septal   def ect  (VSD).   Both  acute  m i tr al   r egur gi tati on  and  a  VSD  r esul t  i n  a
sy stol i c  m ur m ur   at  the  l ow er   l ef t  ster nal   bor der .   When  m i tr al   r egur gi tati on  i s  acute  and
sev er e,   the  sy stol i c  m ur m ur   m ay   be  sof t  and  m ay   not  be  hol osy stol i c  because  the  l ef t  atr i al
pr essur e  i ncr eases  r api dl y   i n  sy stol e  decr easi ng  the  m i tr al   r egur gi tati on  jet  and  m ur m ur .   The
m ur m ur   of   VSD  i s  gener al l y   l oud,   har sh,   and  hol osy stol i c  due  to  the  v i br ati on  of   the  m uscul ar
v entr i cul ar   septum   and  a  hi gh  pr essur e  gr adi ent  betw een  the  l ef t  and  r i ght  v entr i cl es
thr oughout  sy stol e.

2.   What  m edi cal   ther apy   shoul d  be  used  to  tr eat  thi s  pati ent  acutel y ,   and  w hy ?

The  m ost  i m por tant  i nter v enti on  i n  thi s  pati ent  i s  acute  r eper f usi on.   An  acute  cor onar y   angi ogr aphy
w i l l   def i ne  the  cor onar y   anatom y .   Per cutaneous  cor onar y   i nter v enti on  (PCI)  w i th  angi opl asty   and/or
cor onar y   stenti ng  w i l l   be  i ndi cated  i f   ther e  i s  no  papi l l ar y   m uscl e  r uptur e  or   VSD.   Car di ac  sur ger y
i s  necessar y   i f   ei ther   r epai r   of   a  VSD  or   m i tr al   v al v e  r epl acem ent  f or   papi l l ar y   m uscl e  r uptur e  i s
necessar y .
P. 26
Ther e  ar e  sev er al   com ponents  to  the  acute  suppor ti v e  tr eatm ent  of   thi s  pati ent's  pul m onar y   edem a.
The  adm i ni str ati on  of   ox yge n to ma inta in a rte ria l ox yge n s a tura tion a bove  90%   i s  i m por tant
because  the  al v eol ar   edem a  i nter f er es  w i th  adequate  ox y gen  di f f usi on.   N oni nv asi v e  posi ti v e­
pr essur e  suppor t  v enti l ati on  i s  al so  benef i ci al   and  shoul d  be  used  i n  pati ents  w ho  ar e  sti l l   hy pox i c
despi te  m edi cal   tr eatm ent.   Morphine   (1  to  3  m g  at  a  ti m e  i n  an  IV  push)  di m i ni shes  anx i ety   and
decr eases  centr al   sy m patheti c  outf l ow ,   ther eby   r educi ng  both  v enous  and  ar ter i al   v asoconstr i cti on,
r esul ti ng  i n  decr eases  i n  v entr i cul ar   pr el oad  and  af ter l oad,   r especti v el y .   Mor phi ne  shoul d  not  be
gi v en  to  pati ents  w i th  di m i ni shed  sensor i um   or   r espi r ator y   dr i v e  or   hy per capni a  because  i t  m ay
pr eci pi tate  r espi r ator y   ar r est.   Furos e mide   (20  to  80  m g  i n  a  sl ow   IV  push)  or   other   l oop  di ur eti cs
cause  i m m edi ate  v enodi l ati on,   f ol l ow ed  by   di ur esi s  w i thi n  appr ox i m atel y   5  to  10  m i nutes.   IV
s odium nitroprus s ide   m ay   be  used  to  r educe  bl ood  pr essur e  i f   hy per tensi on  i s  pr esent.   NTG ,
adm i ni ster ed  as  subl i ngual   tabl ets  or   by   IV  dr i p,   r el i ev es  the  pul m onar y   edem a  by   pr oduci ng
v enodi l ati on  and  tr eati ng  acute  i schem i a.   Digox in  m ay   be  used  to  sl ow   the  v entr i cul ar   r esponse  to
atr i al   f i br i l l ati on.   IV  di l ti azem   or   a  β­bl ock er   m ay   be  used  to  r educe  the  v entr i cul ar   r esponse  i f   the
pati ent  can  tol er ate  a  negati v e  i notr opi c  agent.

Mul ti pl e  studi es  com par i ng  N TG  to  f ur osem i de  or   m or phi ne  sul f ate  hav e  dem onstr ated  gr eater
ef f i cacy   and  saf ety   and  a  f aster   onset  of   acti on  f or   N TG.   Al though  ACE  i nhi bi tor s  ar e  gener al l y
consi der ed  the  cor ner stone  f or   tr eati ng  chr oni c  hear t  f ai l ur e  (HF),   sev er al   v er y   sm al l   studi es  hav e
dem onstr ated  good  r esul ts  f or   tr eatm ent  of   acute  pul m onar y   edem a  w i th  thi s  cl ass  of   agent.
N ev er thel ess,   ACE  i nhi bi tor s  shoul d  be  used  w i th  ex tr em e  cauti on  i n  pati ents  w i th  hy potensi on  or
si gni f i cantl y   i m pai r ed  r enal   f uncti on.

It  has  been  dem onstr ated  that  sy stem i c  i nf usi on  of   nesi r i ti de  has  benef i ci al   hem ody nam i c  acti ons
but  m ay   cause  si gni f i cant  hy potensi on  and  no  si gni f i cant  benef i t  i n  cl i ni cal   outcom es  com par ed  w i th
IV  N TG.   Al so,   som e  concer ns  hav e  been  r ai sed  that  nesi r i ti de  m ay   be  associ ated  w i th  an  i ncr eased
r i sk   of   death  and  w or seni ng  r enal   f uncti on.

Suggested Readings
Annane  D,   Bel l i ssant  E,   Pussar d  E,   etal .   Pl acebo­contr ol l ed,   r andom i zed,   doubl e­bl i nd  study   of
i ntr av enous  enal apr i l at  ef f i cacy   and  saf ety   i n  acute  car di ogeni c  pul m onar y   edem a.   Ci r cul ati on
1996;94(6):1316–1324.

Bel tr am e  JF,   Zei tz  CJ,   U nger   SA,   etal .   N i tr ate  ther apy   i s  an  al ter nati v e  to  f ur osem i de/m or phi ne
ther apy   i n  the  m anagem ent  of   acute  car di ogeni c  pul m onar y   edem a.   J  Car d  Fai l   1998;4:271–279.

Cotter   G,   Metzk or   E,   Kal usk i   E,   etal .   Random i zed  tr i al   of   hi gh­dose  i sosor bi de  di ni tr ate  pl us  l ow ­dose
f ur osem i de  v er sus  hi gh­dose  f ur osem i de  pl us  l ow ­dose  i sosor bi de  di ni tr ate  i n  sev er e  pul m onar y
edem a.   Lancet  1998;351:389–393.

Pi er ar d  LA,   Lancel otti   P.   The  r ol e  of   i schem i c  m i tr al   r egur gi tati on  i n  the  pathogenesi s  of   acute
pul m onar y   edem a.   N   Engl   J  Med  2004;35:1681–1684.

Sack ner ­Ber nstei n  JD,   Kow al sk i   M,   Fox   M,   etal .   Shor t­ter m   r i sk   of   death  af ter   tr eatm ent  w i th
nesi r i ti de  f or   decom pensated  HF:  a  pool ed  anal y si s  of   r andom i zed  contr ol l ed  tr i al s.   JAMA
2005;293:1900–1905.

P. 27

War e  LB,   Matthay   MA.   Cl i ni cal   pr acti ce.   Acute  pul m onar y   edem a.   N   Engl   J  Med  2005;353:2788â
€“2796.

Aortic Dissection
1.   What  i s  acute  aor ti c  di ssecti on?

2.   What  i s  the  m ost  com m on  cause  of   aor ti c  di ssecti on  i n  the  gener al   popul ati on,   i n  m en  y ounger   than
40  y ear s,   and  i n  w om en  y ounger   than  40  y ear s?

3.   What  i s  the  m ost  sensi ti v e  i ni ti al   di agnosti c  test  f or   aor ti c  di ssecti on?

4.   Wher e  ar e  the  m ost  com m on  poi nts  of   or i gi n  f or   aor ti c  di ssecti ons?

Discussion
1.   What  i s  acute  aor ti c  di ssecti on?

Acute  aor ti c  di ssecti on  r esul ts  f r om   a  tear   i n  the  aor ti c  i nti m a.   Dr i v en  by   sy stem i c  pr essur e,
ar ter i al   bl ood  enter s  the  di seased  m edi a  of   the  v essel .   Wi thi n  thi s  l ay er ,   bl ood  cr eates  a  separ ati on
pl ane  as  i t  di ssects  the  aor ta  l ongi tudi nal l y .   The  ar ea  of   di ssecti on  f i l l ed  w i th  bl ood  i s  cal l ed  the
f al se  l um en.   The  shear   f or ces  of   the  di ssecti ng  bl ood  can  cause  addi ti onal   i nti m al   tear s.   As  the
f al se  l um en  f i l l s  w i th  bl ood,   i t  m ay   com pr ess  the  tr ue  l um en,   r esul ti ng  i n  obstr ucti on  of   m ajor
ar ter i es.   Inf r equentl y ,   di ssecti on  can  be  i ni ti ated  by   hem or r hage  i nto  the  m edi a  w i thout  an  i nti m al
tear .

2.   What  i s  the  m ost  com m on  cause  of   aor ti c  di ssecti on  i n  the  gener al   popul ati on,   i n  m en  y ounger   than
40  y ear s,   and  i n  w om en  y ounger   than  40  y ear s?

In  the  ascendi ng  aor ta,   the  m ost  com m on  cause  of   aor ti c  di ssecti on  i n  the  gener al   popul ati on  i s
m edi al   degener ati on  usual l y   associ ated  w i th  agi ng  and  hy per tensi on.   In  the  abdom i nal   aor ta,
ather oscl er osi s  pl ay s  a  m or e  i m por tant  r ol e.   In  m en  y ounger   than  40  y ear s,   the  m ost  com m on
cause  of   di ssecti on  i s  Mar f an's  sy ndr om e  associ ated  w i th  the  m or e  ty pi cal   cy sti c  m edi al
degener ati on  l esi ons.   In  w om en  y ounger   than  40  y ear s,   50%  of   al l   di ssecti ons  occur   dur i ng
pr egnancy .

3.   What  i s  the  m ost  sensi ti v e  i ni ti al   di agnosti c  test  f or   aor ti c  di ssecti on?

The  sensi ti v i ti es  of   tr ansesophageal   echocar di ogr aphy   (TEE),   m agneti c  r esonance  i m agi ng  (MRI),
and  com puted  tom ogr aphy   (CT)  scan  f or   detecti on  of   di ssecti on  ar e  si m i l ar ,   w i th  TEE  pr obabl y
hav i ng  a  sl i ght  adv antage.   In  m ost  cases,   the  pr ef er r ed  i ni ti al   m odal i ty   i s  CT  scanni ng  because  of
av ai l abi l i ty ,   saf ety ,   and  conv eni ence.   If   the  pati ent  i s  not  stabl e,   TEE  shoul d  be  consi der ed  f i r st  as
i t  can  be  per f or m ed  i n  a  m oni tor ed  setti ng  w her e  acute  m edi cal   ther apy   can  be  adm i ni ster ed.

4.   Wher e  ar e  the  m ost  com m on  poi nts  of   or i gi n  f or   aor ti c  di ssecti ons?

The  m ost  com m on  poi nt  of   or i gi n  f or   ascendi ng  aor ti c  di ssecti on  i s  w i thi n  2  i n.   of   the  aor ti c  v al v e.
For   descendi ng  aor ti c  di ssecti on,   thi s  poi nt  i s  at
P. 28
the  l i gam entum   ar ter i osum ,   just  bey ond  the  tak eof f   of   the  l ef t  subcl av i an  ar ter y .

Case
A  63­y ear ­ol d  m an  w i th  a  hi stor y   of   CAD  and  pr ev i ous  i nf er i or   MI  has  the  f ol l ow i ng  car di ac  r i sk   f actor s:
30  y ear s  of   m oder atel y   contr ol l ed  hy per tensi on,   75  pack ­y ear s  of   tobacco  use,   ty pe  2  non–i nsul i n­
dependent  di abetes  m el l i tus,   and  a  f am i l y   hi stor y   of   CAD.   Hi s  total   chol ester ol   l ev el   6  m onths  bef or e
thi s  adm i ssi on  w as  260  m g/dL.
The  pati ent  has  been  ex per i enci ng  hi s  usual   ex er ti onal   angi na,   w hi ch  i s  r el i ev ed  w i th  N TG  and  r est,
w i thout  a  change  i n  patter n  or   char acter   dur i ng  the  m onth  bef or e  pr esentati on.   At  11:00  a. m .   on  the  day
of   adm i ssi on,   he  w as  l i f ti ng  a  50­l b  bag  of   f er ti l i zer   w hen  he  ex per i enced  an  acute  sev er e  (10/10),
tear i ng  l ef t  pr ecor di al   chest  pai n  w i thout  r adi ati on,   but  w i th  di aphor esi s,   nausea,   and  l i ghtheadedness.
The  pai n  w as  si m i l ar   to  hi s  angi na,   but  he  obtai ned  no  r el i ef   w i th  N TG  (0. 4  m g  subl i ngual l y ).   He  com es
to  the  ER,   w her e  the  phy si cal   ex am i nati on  r ev eal s  a  r i ght  ar m   bl ood  pr essur e  of   80/40  m m   Hg,   a  pul se
r ate  of   110  per   m i nute,   and  a  r espi r ator y   r ate  of   24  per   m i nute.   He  i s  a  di aphor eti c  el der l y   m an  w ho  i s
w r i thi ng  i n  bed  and  com pl ai ni ng  of   l ef t  chest  pai n,   w hi ch  i s  now   r adi ati ng  to  the  thr oat  and  i nter scapul ar
ar ea.   The  car di ov ascul ar   ex am i nati on  r ev eal s  a  tachy car di a.   The  f i r st  (S 1 )  and  second  (S 2 )  sounds  ar e
nor m al   and  a  f our th  sound  (S 4 )  i s  pr esent.   Ther e  i s  a  gr ade  3/4  di astol i c  m ur m ur   consi stent  w i th  aor ti c
i nsuf f i ci ency   hear d  at  the  second  r i ght  and  l ef t  i nter costal   spaces.   Ex am i nati on  of   the  per i pher al   pul ses
r ev eal s  a  di m i ni shed  r i ght  r adi al   pul se,   a  nor m al   l ef t  r adi al   pul se,   and  nor m al   f em or al   pul ses.

1.   What  tests  w oul d  y ou  do  f i r st  to  establ i sh  a  w or k i ng  di agnosi s?
2.   How   ar e  aor ti c  di ssecti ons  cl assi f i ed,   w hat  ar e  the  causes,   and  w hat  ar e  the  com m on  si gns  and
sy m ptom s?
3.   What  i ni ti al   ther apy   i s  i ndi cated  to  stabi l i ze  thi s  pati ent's  condi ti on?
4.   Because  aor ti c  di ssecti on  i s  thought  to  be  pr esent,   w hat  i m agi ng  techni ques  shoul d  be  done  to
conf i r m   the  di agnosi s  and  assi st  i n  pl anni ng  f ur ther   ther apy ?
5.   What  def i ni ti v e  ther apy   shoul d  be  i nsti tuted?
6.   What  l ong­ter m   car e  i s  i ndi cated  f or   thi s  pati ent?

Case Discussion
1.   What  tests  w oul d  y ou  do  f i r st  to  establ i sh  a  w or k i ng  di agnosi s?

The  f i r st  pr ocedur e  to  per f or m   i s  a  car ef ul   phy si cal   ex am i nati on.   Your   ex am i nati on  i n  thi s  pati ent
conf i r m s  the  ER  f i ndi ngs,   but  the  bl ood  pr essur e  i n  the  l ef t  ar m   i s  190/110  m m   Hg,   and  the  r i ght
ar m   bl ood  pr essur e  i s  sti l l   80/40  m m   Hg.   The  di scr epancy   i n  pul se  and  bl ood  pr essur e  betw een  the
r i ght  and  l ef t  ar m s  i s  str ongl y   suggesti v e  of   aor ti c  di ssecti on  i nv ol v i ng  the  pr ox i m al   aor ti c  ar ch.
The  f i ndi ng  of   aor ti c  i nsuf f i ci ency   i s  consi stent  w i th  i nv ol v em ent  of   the  pr ox i m al   ascendi ng  aor ta.   A
chest  r adi ogr aph  shoul d  al so  be  obtai ned.   It  i s  l i k el y   to  show   a  w i dened  m edi asti num   w i th  aor ti c
k nob  i nti m al   cal ci um   separ ated  f r om   the  adv enti ti al   bor der   by   1. 2  cm .   Thi s  “cal ci um   si gnâ€​  is
def i ned  as  a  separ ati on  that  ex ceeds  1. 0  cm ,   and  i t  i s  pathognom oni c  f or   aor ti c  di ssecti on.   An  ECG
shoul d  al so  be  obtai ned  to  deter m i ne
P. 29
i f   ther e  i s  an  acute  MI,   w hi ch  m ay   r esul t  f r om   occl usi on  of   the  cor onar y   ar ter y   by   the  di ssecti on.   In
thi s  pati ent,   the  ECG  show s  di f f use,   nonspeci f i c  ST­segm ent  and  T­w av e  changes.   On  the  basi s  of
the  hi stor y   of   “tear i ngâ€​   pai n  and  these  f i ndi ngs,   the  l i k el i hood  of   aor ti c  di ssecti on  i s  deem ed
hi gh  i n  thi s  pati ent.

2.   How   ar e  aor ti c  di ssecti ons  cl assi f i ed,   w hat  ar e  the  causes,   and  w hat  ar e  the  com m on  si gns  and
sy m ptom s?

Sev er al   cl assi f i cati ons  f or   aor ti c  di ssecti on  hav e  been  pr oposed,   but  the  m ost  com m onl y   used  i s  the
f ol l ow i ng  DeBak ey   cl assi f i cati on:
Ty pe  I:  Di ssecti on  or i gi nati ng  i n  the  ascendi ng  aor ta,   ex tendi ng  to  or   bey ond  the  aor ti c  ar ch

Ty pe  II:  Di ssecti on  l i m i ted  to  the  ascendi ng  aor ta

Ty pe  III:  Di ssecti on  or i gi nati ng  i n  the  descendi ng  aor ta  and  ex tendi ng  di stal l y   dow n  the  aor ta
or ,   r ar el y ,   ex tendi ng  r etr ogr ade  i nto  the  aor ti c  ar ch  and  ascendi ng  aor ta

Another   cl assi f i cati on  i s  the  Dai l y   or   Stanf or d  schem e  that  i s  si m pl er ,   and  as  f ol l ow s:

Ty pe  A:  Al l   di ssecti ons  i nv ol v i ng  the  ascendi ng  aor ta,   r egar dl ess  of   the  si te  of   or i gi n

Ty pe  B:  Al l   di ssecti ons  not  i nv ol v i ng  the  ascendi ng  aor ta

DeBak ey   ty pes  I  and  II  and  Stanf or d  A  both  i nv ol v e  the  ascendi ng  aor ta  and  ar e  ter m ed  pr ox i m al
di ssecti ons,   and  DeBak ey   III  and  Stanf or d  B  i nv ol v e  the  descendi ng  aor ta  and  ar e  ter m ed  di stal
di ssecti ons.

The  tr eatm ent  of   aor ti c  di ssecti on  depends  on  w hether   the  di ssecti on  i nv ol v es  the  pr ox i m al   or
di stal   aor ta.   The  cl i ni cal   m ani f estati ons  ar e  deter m i ned  by   i nv ol v em ent  of   ar ter i al   br anches  of   the
aor ta  (the  r i ght  br achi ocephal i c  ar ter y   i n  thi s  pati ent),   the  aor ti c  v al v e  (aor ti c  i nsuf f i ci ency   i n  thi s
pati ent)  or   cor onar y   ar ter i es,   or   both.   A  di ssecti on  that  r eaches  pr ox i m al l y   i nto  the  per i car di al
space  can  cause  tam ponade.   Appr ox i m atel y   tw o  thi r ds  of   aor ti c  di ssecti ons  ar e  pr ox i m al ,   w her eas
one  thi r d  i s  di stal .

The  eti ol ogy   of   nontr aum ati c  aor ti c  di ssecti on  i nv ol v es  degener ati on  of   the  col l agen  and  el asti n
f i ber s  of   the  m edi a  of   the  aor ta,   w hi ch  usual l y   occur s  i n  pati ents  ex per i enci ng  a  chr oni c  ar ter i al
str ess,   such  as  hy per tensi on.   A  speci f i c  ty pe  of   m edi al   degener ati on  cal l ed  cy sti c  m edi al   necr osi s
occur s  i n  pati ents  w i th  Mar f an's  and  Ehl er s­Danl os  sy ndr om es.

Other   pr edi sposi ng  f actor s  f or   di ssecti on  i ncl ude  congeni tal   coar ctati on  of   the  aor ta,   bi cuspi d  aor ti c
v al v e,   ather oscl er osi s,   N oonan's  and  Tur ner 's  sy ndr om es,   and  gi ant  cel l   ar ter i ti s.   Di r ect  ex ter nal
tr aum a  as  w el l   as  i ntr av ascul ar   tr aum a  due  to  ar ter i al   catheter i zati on  and  i ntr aaor ti c  bal l oon
pum ps  m ay   r esul t  i n  aor ti c  di ssecti on.   Aor ti c  tr aum a  dur i ng  car di ac  sur ger y ,   especi al l y   aor ti c  v al v e
r epl acem ent,   m ay   r ar el y   r esul t  i n  di ssecti on.

The  i nci dence  of   aor ti c  di ssecti on  peak s  i n  the  si x th  and  sev enth  decades.   Ther e  i s  a  pr eponder ance
of   m al e  pati ents  w i th  a  m al e­to­f em al e  r ati o  of   2:1.

The  m ost  com m on  sy m ptom   at  pr esentati on,   seen  i n  m or e  than  90%  of   pati ents,   i s  sudden  onset  of
sev er e  chest  pai n  that  i s  i m m edi atel y   m ax i m al   i n  i ntensi ty .   The  pai n  i s  unbear abl e,   and  of ten
descr i bed  as  a  shar p,   tear i ng  or   r i ppi ng  sensati on.   Thi s  di f f er enti ates  i t  f r om   that  of   an  MI,   w hi ch
i s  f r equentl y   cr escendo  i n  natur e  and
P. 30
pr essur e­l i k e.   The  pai n  can  m i gr ate  usual l y   f ol l ow i ng  the  path  of   di ssecti on.   Anter i or   chest  pai n  i s
usual l y   associ ated  w i th  a  prox ima l dis s e c tion,   w her eas  an  i nter scapul ar   pai n  i ndi cates  a  dis ta l
dis s e c tion.   The  di f f er enti al   di agnosi s  of   aor ti c  di ssecti on  i ncl udes  MI  or   i schem i a,   a  thor aci c
nondi ssecti ng  aneur y sm ,   m uscul osk el etal   pai n,   m edi asti nal   tum or s,   and  per i car di ti s.

Other   si gns  and  sy m ptom s  of   acute  aor ti c  di ssecti on  depend  on  i nv ol v em ent  of   m ajor   ar ter i al
br anches  or   the  aor ti c  v al v e  and  ar e  m or e  com m on  w i th  pr ox i m al   di ssecti on.   Aor ti c  i nsuf f i ci ency
occur s  i n  up  to  tw o  thi r ds  of   al l   cases  of   pr ox i m al   di ssecti on  and  i s  due  to  di l ati on  of   the  aor ti c
r oot,   hem atom a  i nter f er i ng  w i th  l eaf l et  coaptati on,   tear i ng  of   the  annul us  or   l eaf l et,   or   a
com bi nati on  of   these.   Aor ti c  i nsuf f i ci ency   i s  the  m ost  com m on  cause  of   HF  i n  these  pati ents.
N eur ol ogi c  def i ci ts  can  i ncl ude  str ok e,   par apl egi a,   or   al ter ed  consci ousness.   Other   com pl i cati ons
i ncl ude  Hor ner   sy ndr om e  r esul ti ng  f r om   super i or   cer v i cal   gangl i on  com pr essi on  and  l ef t  r ecur r ent
l ar y ngeal   ner v e  par al y si s  causi ng  hoar seness.   The  i nv ol v em ent  of   m ajor   ar ter i al   br anches  can  l ead
to  m y ocar di al ,   m esenter i c,   or   r enal   i nf ar cti ons.

Ruptur e  of   an  aor ti c  di ssecti on  i s  m or e  com m on  w i th  the  pr ox i m al   ty pe  and  can  cause  acute
hem oper i car di um   w i th  car di ac  tam ponade  or   a  l ef t  pl eur al   ef f usi on.   Ruptur e  i nto  the  ai r w ay s  or
esophagus  can  r esul t  i n  hem opty si s  or   hem atem esi s.

3.   What  i ni ti al   ther apy   i s  i ndi cated  to  stabi l i ze  thi s  pati ent's  condi ti on?

Medi cal   ther apy   i s  i ndi cated  i ni ti al l y   to  stop  the  pr ogr essi on  of   the  di ssecti on.   The  pati ent  shoul d
be  adm i tted  to  an  i ntensi v e  car e  uni t  w i th  hem ody nam i c  m oni tor i ng.   Medi cal   ther apy   i s  ai m ed  at
r educi ng  the  m ean  ar ter i al   bl ood  pr essur e  and  the  v el oci ty   of   the  l ef t  v entr i cul ar   ejecti on  (ar ter i al
dP/dt)  to  m i ni m i ze  ar ter i al   shear   str ess.

Sodi um   ni tr opr ussi de  i s  a  di r ect  v asodi l ator   and  decr eases  ar ter i al   pr essur e  i n  a  dose­dependent
m anner .   The  ai m   i s  to  r educe  sy stol i c  bl ood  pr essur e  to  100  to  120  m m   Hg  as  l ong  as  ther e  i s
adequate  or gan  per f usi on.   N i tr opr ussi de  i ncr eases  dP/dt  i f   used  al one  and  the  adm i ni str ati on  of   β­
bl ock i ng  agents  bl unts  thi s  ef f ect.   If   ther e  ar e  no  contr ai ndi cati ons  to  β­bl ock er s,   they   shoul d  be
gi v en  i ntr av enousl y   to  r each  a  hear t  r ate  of   60  to  80  beats  per   m i nute.   Esm ol ol ,   a  shor t­acti ng  IV
β­bl ock er ,   m ay   be  par ti cul ar l y   usef ul   because  i t  can  be  ti tr ated  m i nute­to­m i nute  to  r educe  hear t
r ate.   Labetal ol   i s  al so  a  good  choi ce  f or   the  tr eatm ent  of   acute  aor ti c  di ssecti on  because  i t  i s  both
an  α ­  and  β­bl ock i ng  dr ug.   In  pati ents  w ho  hav e  a  contr ai ndi cati on  to  β­bl ock er s,   cal ci um   channel
bl ock er s  such  as  v er apam i l   or   di l ti azem   del i v er ed  by   IV  r oute  coul d  be  used  to  decr ease  hear t  r ate
and  bl ood  pr essur e.

4.   Because  aor ti c  di ssecti on  i s  thought  to  be  pr esent,   w hat  i m agi ng  techni ques  shoul d  be  done  to
conf i r m   the  di agnosi s  of   aor ti c  di ssecti on  and  assi st  i n  pl anni ng  f ur ther   ther apy ?

A  tra ns thora c ic  e c hoc a rdiogra m  i s  a  qui ck   and  noni nv asi v e  m odal i ty   to  conf i r m   aor ti c
i nsuf f i ci ency ,   assess  segm ental   l ef t  v entr i cul ar   sy stol i c  f uncti on,   and  assess  the  pr ox i m al   aor ti c
r oot  f or   the  pr esence  of   di l ati on.   How ev er ,   i t  has  poor   sensi ti v i ty   especi al l y   f or   di stal   di ssecti ons.
In  gener al ,   TEE,   CT,   and  MRI  ar e  the  i m agi ng  m odal i ti es  used  to  detect  di ssecti on.

TEE  i s  m uch  m or e  sensi ti v e  f or   the  detecti on  of   di ssecti on,   l i k el y   the  m ost  sensi ti v e  of   the  i m agi ng
m odal i ti es.   It  i s  l i m i ted,   how ev er ,   i n  i ts  capabi l i ty   to  assess  the  di stal   ascendi ng  aor ta  and  the
pr ox i m al   ar ch.   It  can  assess  the  pr ox i m al   aor ta,   the  degr ee
P. 31
of   aor ti c  i nsuf f i ci ency ,   l ef t  v entr i cul ar   f uncti on,   the  pr esence  of   per i car di al   ef f usi on,   and  of ten
per m i ts  v i sual i zati on  of   the  pr ox i m al   cor onar y   ar ter i es.   Ther ef or e,   i t  of f er s  a  m or e  com pl ete
assessm ent  of   the  di sease  and  i ts  com pl i cati ons.   In  hem ody nam i cal l y   unstabl e  pati ents,   thi s  test
can  be  qui ck l y   per f or m ed  at  the  bedsi de  w hi l e  tr eatm ent  i s  bei ng  pr ov i ded  concom i tantl y ,   m ak i ng  i t
the  pr ocedur e  of   choi ce  i n  thi s  i nstance.

MRI  i s  hi ghl y   sensi ti v e  and  speci f i c  i n  assessi ng  these  pati ents  and  can  v i sual i ze  the  enti r e
thor aci c  aor ta  i n  one  v i ew .   U si ng  gadol i ni um ,   the  pr esence  of   aor ti c  i nsuf f i ci ency   as  w el l   as
i nv ol v em ent  of   m ajor   br anch  v essel s  can  be  assessed  i n  a  l ar ge  num ber   of   pati ents  (i . e. ,   the
subcl av i an  or   car oti d  ar ter y ).   Thi s  techni que  cannot  be  used  i n  pati ents  w i th  pacem ak er s  and
def i br i l l ator s.   MRI  scanner s  l i m i t  access  to  the  pati ent  dur i ng  the  test  f or   up  to  30  to  40  m i nutes,
w hi ch  i s  di sadv antageous  i n  unstabl e  pati ents.

A  contr ast  CT s c a n  (especi al l y   hel i cal   CT)  i s  good  f or   def i ni ng  the  ex tent  of   an  aor ti c  di ssecti on,
that  i s,   pr ox i m al   v er sus  di stal .   CT  angi ogr aphy   can  al so  assess  i nv ol v em ent  of   m ajor   aor ti c
br anches.   Its  m ajor   adv antages  ar e  v er y   hi gh  sensi ti v i ty   and  av ai l abi l i ty .   A  di sadv antage  i s  that  i t
r ar el y   def i nes  the  si te  of   the  i nti m al   tear .

The  pr ev i ous  gol d  standar d  f or   the  di agnosi s  of   aor ti c  di ssecti on  w as  aor togr aphy .   Thi s  m odal i ty
can  def i ne  the  si te  of   the  i nti m al   tear ,   the  sev er i ty   of   aor ti c  i nsuf f i ci ency ,   cor onar y   ar ter y
i nv ol v em ent,   and  the  ex tent  of   the  di ssecti on—pr ox i m al   v er sus  di stal .   How ev er ,   aor togr aphy   has
been  show n  to  hav e  a  l ow er   sensi ti v i ty   com par ed  w i th  the  other   m odal i ti es  di scussed  abov e.
Ther ef or e,   the  cur r ent  gol d  standar d  v ar i es  dependi ng  on  the  av ai l abi l i ty   of   i m agi ng  m odal i ti es.   A
hel i cal   CT  i s  av ai l abl e  i n  m ost  i nsti tuti ons  and  i s  v er y   accur ate.   A  TEE,   especi al l y   i n  unstabl e
pati ents,   has  been  r ecom m ended  as  the  f i r st  test  by   the  Eur opean  Soci ety   of   Car di ol ogy .   MRI  i s
consi der ed  by   m any   as  the  f i r st  test  to  be  per f or m ed  but  i s  not  al w ay s  av ai l abl e.

5.   What  def i ni ti v e  ther apy   shoul d  be  i nsti tuted?

U ntr eated  acute  aor ti c  di ssecti on  i s  associ ated  w i th  25%  m or tal i ty   at  24  hour s  and  a  death  r ate  of
m or e  than  75%  at  1  m onth.   In  gener al ,   sur gi cal   r epai r   i s  pr ef er r ed  f or   acute  pr ox i m al   di ssecti on  or
i n  di stal   di ssecti ons  w hen  v i tal   or gan  or   l i m b  com pr om i se  i s  pr esent,   f or   r api d  ex pansi on  or
f or m ati on  of   a  saccul ar   aneur y sm ,   f or   r uptur e,   i n  the  pr esence  of   uncontr ol l ed  pai n,   or   i n  pati ents
w i th  Mar f an's  sy ndr om e.   Medi cal   ther apy   (r educi ng  the  bl ood  pr essur e  and  dP/dt)  i s  adequate  f or
uncom pl i cated  acute  di stal   di ssecti ons  as  ther e  i s  l ess  r i sk   of   com pl i cati ons.   It  i s  al so
r ecom m ended  i n  chr oni c  (pr esent  f or   > 2  w eek s)  pr ox i m al   or   di stal   di ssecti ons  as  these  pati ents
hav e  sur v i v ed  the  per i od  of   hi ghest  m or tal i ty   r i sk .

6.   What  l ong­ter m   car e  i s  i ndi cated  f or   thi s  pati ent?

It  i s  essenti al   to  r api dl y   contr ol   the  pati ent's  hy per tensi on  and  decr ease  the  r ate  of   pr essur e  r i se  i n
the  l ef t  v entr i cl e,   pr ef er abl y   w i th  βbl ock er s.   The  l ong­ter m   pr ognosi s  i n  hospi tal   sur v i v or s  i s  good,
w i th  an  actuar i al   sur v i v al   r ate  onl y   sl i ghtl y   w or se  than  that  f or   age­m atched  subjects.   The  ty pe  of
di ssecti on  or   ther apy   used  does  not  i nf l uence  the  outcom e  af ter   di schar ge  f r om   the  hospi tal .   The
hi ghest  r i sk   f or   r ecur r ent  di ssecti on  or   aneur y sm   ex pansi on  i s  i n  the  f i r st  2  y ear s.   Car ef ul   f ol l ow ­
up  dur i ng  thi s  i ni ti al   per i od  i s  i m por tant  to  ensur e  adequate  bl ood  pr essur e  contr ol   and  m oni tor   f or
r ecur r ence.   Thi s  w oul d  i ncl ude  phy si cal   ex am i nati on  and  chest  x ­r ay s.   Ser i al   i m agi ng  w i th  CT
scanni ng,   TEE,   or   MRI  shoul d  al so  be  par t  of   thi s  f ol l ow ­up.

P. 32

Suggested Readings
Hagan  PG,   N i enaber   CA,   Issel bacher   EM,   etal .   The  i nter nati onal   r egi str y   of   acute  aor ti c  di ssecti on
(IRAD):  new   i nsi ghts  i nto  an  ol d  di sease.   2000;283(7):897–903.

N i enaber   CA,   Eagl e  KA.   Aor ti c  di ssecti on:  new   f r onti er s  i n  di agnosi s  and  m anagem ent:  par t  I:  f r om
eti ol ogy   to  di agnosti c  str ategi es.   Ci r cul ati on  2003;108:628–635.

N i enaber   CA,   Eagl e  KA.   Aor ti c  di ssecti on:  new   f r onti er s  i n  di agnosi s  and  m anagem ent:  Par t  II:
ther apeuti c  m anagem ent  and  f ol l ow   up.   Ci r cul ati on  2003;108:772–778.

Sabi k   JF,   Ly tl e  BW,   Bl ack stone  EH,   etal .   Long­ter m   ef f ecti v eness  of   oper ati ons  f or   ascendi ng  aor ti c
di ssecti ons.   J  Thor ac  Car di ov asc  Sur g  2000;119(5):946–962.

Chronic Heart Failure
1.   What  ar e  the  m ost  com m on  under l y i ng  di seases  causi ng  chr oni c  HF  i n  the  U . S.   popul ati on?

2.   Is  HF  al w ay s  associ ated  w i th  a  decr eased  ejecti on  f r acti on  (EF)?

3.   What  i s  m y ocar di al   r em odel i ng  and  w hat  ar e  i ts  consequences?

4.   Whi ch  dr ug  cl asses  hav e  been  show n  to  pr ol ong  sur v i v al   i n  pati ents  w i th  HF?

5.   What  dev i ces  hav e  been  show n  to  pr ol ong  sur v i v al   i n  pati ents  w i th  HF?

Discussion
1.   What  ar e  the  m ost  com m on  under l y i ng  di seases  causi ng  chr oni c  HF  i n  the  U . S.   popul ati on?

In  the  U ni ted  States  and  m ost  dev el oped  countr i es,   hy per tensi on  and  i schem i c  hear t  di sease  ar e
the  m ost  com m on  causes  of   HF.   Val v ul ar   hear t  di sease  and  car di om y opathy   ar e  l ess  com m on
causes,   but  ar e  sti l l   f r equentl y   encounter ed.

2.   Is  HF  al w ay s  associ ated  w i th  a  decr eased  EF?

Sy stol i c  dy sf uncti on  i s  def i ned  as  a  decr ease  i n  contr acti l e  f uncti on  m ost  com m onl y   m easur ed  as  a
decr ease  i n  EF.   Many   pati ents  w i th  HF  hav e  a  decr eased  EF.   How ev er ,   al m ost  hal f   of   al l   pati ents
w i th  HF  hav e  a  nor m al   EF.   In  som e  of   these  pati ents,   di astol i c  dy sf uncti on  i s  the  cause.   Di astol i c
dy sf uncti on  r esul ts  w hen  the  hear t  i s  sti f f   and  v entr i cul ar   f i l l i ng  i s  i m pai r ed,   r esul ti ng  i n  i ncr eased
end­di astol i c  pr essur es.   Pati ents  m ay   hav e  di astol i c  dy sf uncti on  w i th  or   w i thout  sy stol i c
dy sf uncti on.   Ty pi cal   si gns  and  sy m ptom s  of   HF  occur   w i th  ei ther   nor m al   or   abnor m al   EF.   Ty pi cal l y ,
the  pr ev al ence  of   HF  w i th  a  nor m al   EF  i s  m ost  com m on  i n  el der l y   w om en.

3.   What  i s  m y ocar di al   r em odel i ng  and  w hat  ar e  i ts  consequences?

Af ter   m y ocar di al   i njur y   w i th  r esul ti ng  sy stol i c  dy sf uncti on,   ther e  i s  of ten  a  pr ogr essi v e
deter i or ati on  i n  the  str uctur e  and  f uncti on  of   the  v entr i cul ar   m y ocar di um —a  pr ocess  ter m ed
m y ocar di al   r em odel i ng.   My ocar di al   r em odel i ng  i s  char acter i zed  by   pr ogr essi v e  v entr i cul ar
enl ar gem ent  and  decr easi ng  EF.   Thi s  pr ogr essi v e  r em odel i ng  i s  at  l east  par ti al l y   r esponsi bl e  f or   the
hi gh
P. 33
m or tal i ty   r ates  i n  pati ents  w i th  HF.   Al though  the  speci f i c  m ol ecul ar   and  cel l ul ar   ev ents  that  l ead  to
r em odel i ng  ar e  not  enti r el y   under stood,   m any   f actor s  that  pr om ote  r em odel i ng  hav e  been
descr i bed.   These  m echani sm s  i ncl ude  i ncr eased  w al l   str ess  and  acti v ati on  of   the  r eni nâ
€“angi otensi n  and  β­adr ener gi c  sy stem s.   Bl ock ade  of   these  sy stem s  w oul d  be  ex pected  to  sl ow   or
pr ev ent  m y ocar di al   r em odel i ng  and  i m pr ov e  sur v i v al   i n  pati ents  w i th  HF  and  sy stol i c  dy sf uncti on.

4.   Whi ch  dr ug  cl asses  hav e  been  show n  to  pr ol ong  sur v i v al   i n  pati ents  w i th  HF?
In  pati ents  w i th  HF  due  to  a  decr eased  EF,   ACE  i nhi bi tor s  and  β­adr ener gi c  r eceptor   antagoni sts  β­
bl ock er s)  hav e  been  show n  to  i m pr ov e  sur v i v al .   Angi otensi n­r eceptor   bl ock er s  (ARBs)  ar e  pr obabl y
equi v al ent  to  ACE  i nhi bi tor s  and  m ay   be  substi tuted,   especi al l y   i f   ther e  i s  i ntol er ance  to  ACE
i nhi bi tor s  due  to  cough.   Ei ther   al doster one  antagoni sts  or   ARBs  al so  i m pr ov e  sur v i v al   w hen  added
to  ACE  i nhi bi tor s  and  β­bl ock er s,   but  car e  m ust  be  tak en  to  m oni tor   pati ents  car ef ul l y   to  av oi d
hy per k al em i a;  the  use  of   al l   f our   dr ug  cl asses  together   i s  not  adv i sed  f or   m ost  pati ents  because  of
the  r i sk   of   hy per k al em i a.   Di gox i n  m ay   be  hel pf ul   to  i m pr ov e  sy m ptom s  but  does  not  i m pr ov e
sur v i v al .   Loop  di ur eti cs  such  as  f ur osem i de,   bum etani de,   and  tor sem i de  cl ear l y   r el i ev e  congesti on
caused  by   sal t  and  w ater   r etenti on  but  hav e  not  been  show n  to  i m pr ov e  sur v i v al .   The  com bi nati on
of   hy dr al azi ne  and  i sosor bi de  di ni tr ate  i m pr ov es  sur v i v al   i n  Af r i can  Am er i cans  w i th  sy stol i c
dy sf uncti on  and  N ew   Yor k   Hear t  Associ ati on  (N YHA)'s  cl ass  III—IV  HF.

In  pati ents  w i th  HF  and  nor m al   EF  onl y   one  m ajor   tr i al   has  been  conducted.   Thi s  tr i al   usi ng  the  ARB
candesar tan  di d  not  show   a  si gni f i cant  benef i t  on  hospi tal i zati on  or   m or tal i ty   r ate.   Loop  di ur eti cs
ar e  v al uabl e  i n  r el i ev i ng  congesti v e  sy m ptom s  i n  these  pati ents  but  no  cl i ni cal   tr i al s  hav e  been
conducted.

5.   What  dev i ces  hav e  been  show n  to  pr ol ong  sur v i v al   i n  pati ents  w i th  HF?

Pati ents  w i th  si gni f i cant  sy stol i c  dy sf uncti on  (EF  ≤35%),   and  N YHA  cl ass  II­III  hear t  f ai l ur e  hav e
i m pr ov ed  sur v i v al   w hen  an  i nter nal   car di ac  def i br i l l ator   (ICD)  i s  i m pl anted.   The  ICD  detects  ser i ous
v entr i cul ar   ar r hy thm i as  and  cor r ects  them   ei ther   w i th  paci ng  or   a  shock .   Car di ac  r esy nchr oni zati on
ther apy   (CRT)  i s  based  on  the  concept  that  pati ents  w i th  l ef t  v entr i cul ar   sy stol i c  dy sf uncti on  of ten
hav e  v entr i cul ar   dy ssy nchr ony .   Dy ssy nchr ony   i s  m ost  of ten  seen  w hen  the  QRS  dur ati on  i s  120
m i l l i seconds  or   m or e  and  m ost  cl i ni cal   tr i al s  hav e  used  thi s  QRS  dur ati on  as  an  entr y   cr i ter i a.
Dy ssy nchr ony   m eans  that  the  l ef t  v entr i cul ar   contr acti on  i s  di scoor di nated,   r esul ti ng  i n  a  l ow er
str ok e  v ol um e  and  i ncr eased  w al l   str ess.   By   paci ng  both  the  v entr i cul ar   septum   (w i th  a  pacer   i n  the
r i ght  v entr i cul ar   apex )  and  the  l ater al   w al l   of   the  l ef t  v entr i cl e  (thr ough  a  pacer   adv anced  thr ough
the  cor onar y   si nus  i nto  a  l ater al   cor onar y   v ei n)  the  coor di nati on  of   v entr i cul ar   contr acti on  i s
i m pr ov ed,   i ncr easi ng  car di ac  output.   In  pati ents  w i th  an  EF  of   35%  or   l ess  and  HF,   CRT  i m pr ov es
sy m ptom s,   hospi tal i zati ons,   and  m or tal i ty .

P. 34
Case
A  42­y ear ­ol d  w hi te  m an  i s  seen  i n  the  ER  w i th  a  chi ef   com pl ai nt  of   shor tness  of   br eath  that  has  l asted
f or   1  w eek .   He  r epor ts  hav i ng  had  a  v i r al   sy ndr om e  appr ox i m atel y   3  w eek s  bef or e  adm i ssi on.
Subsequentl y ,   he  noted  the  dev el opm ent  of   l ow er   ex tr em i ty   edem a,   a  15­l b  w ei ght  gai n,   dy spnea  on
ex er ti on,   and  or thopnea.   Cur r entl y   he  com pl ai ns  of   dy spnea  at  r est.   Phy si cal   ex am i nati on  r ev eal s  an
i r r egul ar l y   i r r egul ar   hear t  r ate  of   130  per   m i nute.   Hi s  bl ood  pr essur e  i s  90/60  m m   Hg,   and  hi s
r espi r ator y   r ate  i s  22  per   m i nute.   Ex am i nati on  of   the  jugul ar   v enous  pr essur e  dem onstr ates  a  m ean
pr essur e  of   12  to  14  cm   of   w ater   w i th  a  pr om i nent  V  w av e.   Lung  ex am i nati on  r ev eal s  bi basi l ar   dul l ness
w i th  r al es  ex tendi ng  one  f our th  of   the  w ay   up  f r om   the  basal   l ung  f i el ds  bi l ater al l y .   Car di ac  ex am i nati on
f i ndi ngs  ar e  si gni f i cant  f or   a  di f f use  poi nt  of   m ax i m al   i m pul se,   w hi ch  i s  di spl aced  to  the  anter i or
ax i l l ar y   l i ne.   The  S 1   and  S 2   ar e  of   v ar i abl e  i ntensi ty ,   and  a  pr om i nent  S 3   gal l op  ov er   the  di spl aced
car di ac  apex   i s  appr eci ated.   Ther e  i s  a  gr ade  2/6  hol osy stol i c  m ur m ur   that  i s  hear d  best  at  the  car di ac
apex ,   w i th  pr om i nent  r adi ati on  to  the  ax i l l a  and  no  change  w i th  r espi r ati on.   On  ex am i nati on  of   the
abdom en,   an  enl ar ged,   tender   l i v er   i s  f ound.   The  ex tr em i ti es  ar e  cool   and  ex hi bi t  2+   pi tti ng  edem a.   The
ECG  show s  atr i al   f i br i l l ati on  w i th  nonspeci f i c  ST­T–w av e  changes,   a  l ef t  bundl e  br anch  bl ock   (LBBB)
and  occasi onal   v entr i cul ar   pr em atur e  beats.   Ar ter i al   bl ood  gas  m easur em ents  per f or m ed  w i th  the
pati ent  on  4  L  of   ox y gen  per   m i nute  by   nasal   cannul a  r ev eal   a  pH  of   7. 46,   a  PO 2   of   52  m m   Hg,   a  PCO 2

of   32  m m   Hg,   and  a  bi car bonate  (HCO 3 ­ )  concentr ati on  of   26  m m ol /L.

1.   Does  thi s  pati ent  hav e  l ef t,   r i ght,   or   bi v entr i cul ar   f ai l ur e?


2.   An  S 3   i s  hear d,   but  no  S 4 .   Why ?

3.   What  chest  r adi ogr aphi c  f i ndi ngs  w oul d  y ou  ex pect  to  see  i n  thi s  pati ent?
4.   What  neur ohor m onal   m echani sm s  ar e  l i k el y   to  be  acti v ated  i n  thi s  pati ent?
5.   What  di agnosti c  tests  shoul d  be  per f or m ed?
6.   What  tr eatm ent  opti ons  w oul d  l i k el y   be  benef i ci al   i n  thi s  pati ent?
7.   Is  i t  possi bl e  that  the  v entr i cul ar   f uncti on  w i l l   i m pr ov e  w i th  m edi cal   ther apy ?
Your   pati ent  i m pr ov ed  af ter   di ur esi s  and  adm i ni ster i ng  ACE  i nhi bi tor s  and  β­bl ock er s.   Si x   m onths
l ater   hi s  EF  has  i ncr eased  f r om   20%  to  29%.   He  i s  on  di gox i n  w i th  ther apeuti c  l ev el s  and  an
al doster one  antagoni st  w i th  nor m al   ser um   cr eati ni ne  and  potassi um .   He  has  no  r esti ng  dy spnea  or
edem a,   but  does  hav e  dy spnea  w i th  si m pl e  task s.
8.   In  w hi ch  N YHA  cl ass  and  Am er i can  Col l ege  of   Car di ol ogy /Am er i can  Hear t  Associ ati on  (ACC/AHA)
stage  w oul d  y ou  categor i ze  thi s  pati ent's  sy m ptom s?
9.   What  i s  thi s  pati ent's  ex pected  m or tal i ty   r ate  i n  hi s  cur r ent  condi ti on?

Case Discussion
1.   Does  thi s  pati ent  hav e  l ef t,   r i ght,   or   bi v entr i cul ar   f ai l ur e?

Thi s  pati ent  has  f i ndi ngs  i ndi cati ng  both  r i ght  and  l ef t  v entr i cul ar   f ai l ur e  (bi v entr i cul ar   f ai l ur e).   The
cool   ex tr em i ti es,   tachy car di a,   and  nar r ow   pul se  pr essur e  suggest  poor   f or w ar d  car di ac  output  and
coul d  r ef l ect  ei ther   r i ght  or   l ef t  v entr i cul ar   f ai l ur e.   A  l ef t  v entr i cul ar   S 3   gal l op  and  pul m onar y   r al es
ar e  si gns  of   l ef t  v entr i cul ar
P. 35
f ai l ur e.   The  bi basi l ar   dul l ness  suggests  the  pr esence  of   bi l ater al   pl eur al   ef f usi ons,   w hi ch  m ay   be
seen  i n  the  setti ng  of   ei ther   r i ght  or   l ef t  v entr i cul ar   dy sf uncti on.   The  api cal   m ur m ur   m ost  l i k el y
r epr esents  m i tr al   r egur gi tati on  because  i t  i s  l oudest  at  the  apex ,   i t  r adi ates  to  the  ax i l l a,   and  i t
does  not  change  w i th  r espi r ati on.   We  do  not  k now   f r om   the  hi stor y   w hether   the  pati ent  had  a
pr eex i sti ng  v al v ul ar   di sor der .   Secondar y   m i tr al   or   tr i cuspi d  r egur gi tati on  occur s  com m onl y   i n
pati ents  w i th  v entr i cul ar   enl ar gem ent  and  dy sf uncti on  due  to  di stor ti on  of   the  suppor ti ng  str uctur es
of   the  atr i ov entr i cul ar   v al v es.   Tr i cuspi d  r egur gi tati on  causes  a  l ar ge  V  w av e  i n  the  jugul ar   v enous
pul se.

Ther e  ar e  m any   si gns  of   r i ght  v entr i cul ar   f ai l ur e  i n  thi s  pati ent.   El ev ated  centr al   v enous  pr essur e
i s  appar ent  f r om   the  pati ent's  jugul ar   v enous  di stenti on.   Kussm aul 's  si gn  i s  the  l ack   of   a  f al l   i n  the
jugul ar   v enous  pr essur e  w i th  i nspi r ati on  and  i s  due  to  the  r i ght  v entr i cl e's  i nabi l i ty   to  handl e  the
augm ented  v enous  r etur n.   It  m ay   be  encounter ed  i n  pati ents  w i th  r i ght  v entr i cul ar   f ai l ur e  or
constr i cti v e  per i car di al   di sease.   The  pati ent's  enl ar ged  l i v er   i s  the  r esul t  of   hepati c  congesti on
stem m i ng  f r om   i ncr eased  back   pr essur e  on  the  hepati c  v ei n.   The  pi tti ng  edem a  i n  the  l ow er
ex tr em i ti es  i s  caused  by   el ev ated  hy dr ostati c  pr essur e  i n  the  v enous  sy stem ,   r esul ti ng  i n
ex tr av asati on  of   f l ui d  i nto  the  i nter sti ti al   space  of   the  ank l es,   w her e  the  f or ces  of   gr av i ty   ar e  the
gr eatest.

2.   An  S 3   i s  hear d,   but  no  S 4 .   Why ?

An  S 3   i s  a  l ow ­f r equency   sound  hear d  0. 13  to  0. 16  second  af ter   S 2 .   An  S 3   occur s  at  the  end  of   the
r api d  phase  of   v entr i cul ar   f i l l i ng  and  i s  m ost  l i k el y   due  to  the  v i br ati on  of   the  chor dae  tendi neae  or
the  l ef t  v entr i cul ar   w al l   w i th  r api d  f i l l i ng,   and  m ay   ar i se  f r om   the  r i ght  or   l ef t  v entr i cl e.   A  l ef t
v entr i cul ar   S 3   i s  best  hear d  w i th  the  bel l   of   the  stethoscope  at  the  car di ac  apex .   A  r i ght  v entr i cul ar
S 3   i s  al so  best  hear d  w i th  the  bel l ,   but  i s  m ost  audi bl e  at  the  l ow er   l ef t  ster nal   bor der   or   ov er   the
epi gastr i um .   An  S 3   i s  a  nor m al   f i ndi ng  i n  chi l dr en  or   y oung  adul ts,   but  i n  m i ddl e­aged  or   ol der
pati ents  i t  i s  usual l y   a  si gn  of   v ol um e  ov er l oad  m ost  of ten  due  to  HF,   as  i t  i s  i n  thi s  pati ent.

An  S 4   i s  a  pr esy stol i c  atr i al   sound  (gal l op)  that  i s  hear d  w hen  the  v entr i cl e  i s  poor l y   com pl i ant.
Gi v en  the  pati ent's  v ol um e  ov er l oad,   i t  i s  l i k el y   that  both  v entr i cl es  ar e  poor l y   com pl i ant.   How ev er ,
the  pati ent  i s  i n  atr i al   f i br i l l ati on,   and  ther ef or e  ther e  ar e  no  ef f ecti v e  atr i al   sy stol es  to  gi v e  r i se
to  an  S 4   (r ar el y ,   an  S 4   m ay   be  hear d  ev en  i n  atr i al   f i br i l l ati on  because  of   the  hi gh  l ef t  atr i al
pr essur e  and  i ncr eased  f l ow   i n  l ate  di astol e).

3.   What  chest  r adi ogr aphi c  f i ndi ngs  w oul d  y ou  ex pect  to  see  i n  thi s  pati ent?

The  l i k el y   f i ndi ngs  on  a  chest  r adi ogr aphy   stem   f r om   the  ef f ects  of   v ol um e  ov er l oad  and  el ev ated
pul m onar y   v enous  pr essur e.   Car di om egal y ,   w hi ch  i s  def i ned  as  a  car di ac­to­thor aci c  di am eter   r ati o
ex ceedi ng  0. 5,   i s  pr esent  i n  m ost  cases  i n  w hi ch  ther e  i s  depr essed  l ef t  v entr i cul ar   sy stol i c
f uncti on.   Cephal i zati on  of   the  pul m onar y   bl ood  f l ow   occur s  and  i s  ev i denced  by   the  enl ar ged
pul m onar y   v essel s  i n  the  super i or   por ti on  of   the  pul m onar y   tr ee.   The  hazi ness  of   the  centr al
v ascul atur e  i s  a  r esul t  of   the  i ncr eased  hy dr ostati c  pr essur e  and  subsequent  tr ansudati on  of   f l ui d
i nto  the  ti ssue  sur r oundi ng  the  v essel s.   Ker l ey   B  l i nes  ar e  hor i zontal ,   thi n,   shar p  l i nes  that  ex tend
i nw ar d  f r om   the  per i pher y   of   the  l ungs.   They   r epr esent  edem a  f or m ati on
P. 36
w i thi n  the  l ungs  and  hy per tr ophy   of   the  i nter l obul ar   septa.   Pl eur al   ef f usi ons  m ay   be  f ound  i n  the
setti ng  of   r i ght  or   l ef t  v entr i cul ar   f ai l ur e.   When  pul m onar y   congesti on  i s  sev er e  and  al v eol ar   edem a
i s  pr esent,   a  “butter f l y â€​
  or   “bat­w i ngâ€​
  i nf i l tr ate  m ay   be  seen  center ed  ov er   the  m ai n
pul m onar y   ar ter y .

4.   What  neur ohor m onal   m echani sm s  ar e  l i k el y   to  be  acti v ated  i n  thi s  pati ent?

The  tw o  neur ohor m onal   m echani sm s  m ost  l i k el y   to  be  acti v ated  i n  thi s  pati ent  ar e  the  r eni nâ
€“angi otensi n–al doster one  sy stem   and  the  adr ener gi c  ner v ous  sy stem .   The  ser um   nor epi nephr i ne
l ev el   has  been  show n  to  cor r el ate  i nv er sel y   w i th  the  EF  and  pati ent  sur v i v al   i n  those  w i th  chr oni c
HF.   Car di ac  adr ener gi c  acti v ati on  occur s  ev en  ear l i er   than  sy stem i c  adr ener gi c  acti v ati on.   Other
hor m ones  that  m ay   be  acti v ated  i ncl ude  v asopr essi n,   endothel i n,   and  m ul ti pl e  cy tok i nes  such  as
tum or   necr osi s  f actor   α   and  i nter l euk i n  1.

5.   What  di agnosti c  tests  shoul d  be  per f or m ed?

Ini ti al l y   a  com pl ete  bl ood  count,   and  thy r oi d  sti m ul ati ng  hor m one  (TSH),   el ectr ol y te,   r enal   and
hepati c  f uncti on  tests  shoul d  be  obtai ned  to  deter m i ne  i f   ther e  ar e  el ectr ol y te  abnor m al i ti es  that
need  to  be  cor r ected,   i f   ther e  i s  si gni f i cant  under l y i ng  r enal   or   hepati c  di sease,   and  to  deter m i ne  i f
anem i a  or   thy r oi d  abnor m al i ti es  m ay   hav e  ex acer bated  the  hear t  f ai l ur e.   An  ECG  shoul d  be
per f or m ed  to  deter m i ne  i f   ther e  has  been  an  MI  or   i f   ar r hy thm i as  ar e  pr esent.   A  chest  x ­r ay   w i l l
conf i r m   the  HF  and  detect  si gni f i cant  under l y i ng  pul m onar y   pr obl em s.   An  echocar di ogr am   w i l l
ev al uate  v entr i cul ar   si ze  and  f uncti on,   the  pr esence  of   v al v e  abnor m al i ti es,   and  m ay   of ten  suggest
the  under l y i ng  eti ol ogy   of   the  v entr i cul ar   dy sf uncti on.   For   ex am pl e,   i f   the  anter i or   w al l   i s  ak i neti c
and  scar r ed,   a  pr ev i ous  MI  can  be  i nf er r ed.   When  the  l ef t  v entr i cl e  i s  l ar ge  and  has  gl obal
dy sf uncti on,   i t  m ay   be  di f f i cul t  to  deter m i ne  i f   ther e  i s  under l y i ng  CAD.   When  the  pati ent  has
stabi l i zed  an  ex er ci se,   echocar di ogr am   or   nucl ear   study   m ay   r ev eal   r ev er si bl e  i schem i a.   Cor onar y
angi ogr aphy   m ay   be  necessar y   to  ex cl ude  si gni f i cant  CAD  i f   noni nv asi v e  studi es  do  not  cl ear l y
ex cl ude  i schem i a.   If   ther e  i s  no  si gni f i cant  cor onar y   di sease  and  no  si gni f i cant  v al v e  di sease,   the
di agnosi s  i s  l i k el y   i di opathi c  car di om y opathy .   The  ty pe  of   car di om y opathy   can  gener al l y   be
categor i zed  by   echocar di ogr am   as  di l ated,   hy per tr ophi c,   or   r estr i cti v e  w i th  di l ated  car di om y opathy
bei ng  the  m ost  com m on.   A  ser i es  of   tests,   dependi ng  on  the  ty pe  of   car di om y opathy ,   shoul d  be
car r i ed  out  to  ex cl ude  speci f i c  eti ol ogi es.

6.   What  tr eatm ent  opti ons  w oul d  l i k el y   be  benef i ci al   i n  thi s  pati ent?

The  gener al   goal s  f or   the  m edi cal   tr eatm ent  of   HF  ar e  as  f ol l ow s:

a.   Identi f y   and  tr eat  the  under l y i ng  condi ti on.

b.   El i m i nate  any   pr eci pi tati ng  f actor s.

c.   Tr eat  the  sy m ptom s.

d.   Im pr ov e  sur v i v al .

The  f i r st  step  i s  to  i denti f y   the  under l y i ng  cause  of   HF.   Thi s  m ay   be  hy per tensi on,   CAD,
car di om y opathy ,   v al v ul ar   hear t  di sease,   or   m any   other   causes.   Tr eatm ent  i ncl udes  m edi cal
tr eatm ent  f or   hy per tensi on,   cor onar y   angi ogr aphy   and  cor onar y   angi opl asty   or   cor onar y   by pass
sur ger y   f or   cor onar y   di sease,   and  v al v e  r epl acem ent  or   r epai r   f or   v al v e  di sease.

P. 37
In  pati ents  w i th  HF,   i t  i s  i m por tant  to  el i m i nate  pr eci pi tati ng  f actor s  (e. g. ,   di etar y   or   m edi cati on
noncom pl i ance,   ar r hy thm i as,   anem i a).   Ex cess  al cohol   use  m ay   cause  a  car di om y opathy ,   but  ex cess
al cohol   m ay   al so  ex acer bate  HF  of   any   cause.

Sy m ptom ati c  i m pr ov em ent  i s  usual l y   achi ev ed  by   r el i ev i ng  the  ex cess  sal t  and  w ater   r etenti on  w i th
di ur eti cs  and  by   i m pr ov i ng  pr el oad  and  af ter l oad  w i th  v asodi l ator s—par ti cul ar l y   the  ACE
i nhi bi tor s.   Loop  di ur eti cs  such  as  f ur osem i de,   tor sem i de,   or   bum etani de  ar e  m ost  of ten  used
because  they   ar e  m or e  ef f ecti v e  than  thi azi de  di ur eti cs  w hen  r enal   per f usi on  i s  decr eased.   Car e
m ust  be  tak en  to  av oi d  ov er di ur esi s  and  to  r epl ace  potassi um   and  m agnesi um   because  hy pok al em i a
and/or   hy pom agnesem i a  m ay   pr om ote  v entr i cul ar   ar r hy thm i as.   The  com bi nati on  of   ACE  i nhi bi tor s
and  β­adr ener gi c  antagoni sts  (β­bl ock er s)  i s  the  cor ner stone  of   ther apy   f or   pati ents  w i th  HF  due
to  sy stol i c  dy sf uncti on.   Many   ACE  i nhi bi tor s  ar e  now   av ai l abl e  (captopr i l ,   enal apr i l ,   l i si nopr i l ,
qui napr i l ,   r am i pr i l ,   benazepr i l ,   tr andol apr i l ,   f osi nopr i l ,   m oex i pr i l ),   and  ther e  does  not  appear   to  be
a  cl ear   ther apeuti c  adv antage  to  the  use  of   one  ov er   another .   How ev er ,   the  tar get  dose  of   an  ACE
i nhi bi tor   i s  best  deter m i ned  f r om   the  i ndi v i dual   agents  that  hav e  been  studi ed  i n  pati ents  w i th  HF.
By   decr easi ng  the  conv er si on  of   angi otensi n  I  to  angi otensi n  II,   these  dr ugs  r educe  pr el oad  and
af ter l oad,   i m pr ov e  sy m ptom s,   and  pr ol ong  sur v i v al   i n  pati ents  w i th  sy stol i c  dy sf uncti on.   Cough  i s
the  m ost  com m on  si de  ef f ect  of   ACE  i nhi bi tor s,   but  cough  i s  al so  a  com m on  sy m ptom   of   HF.   Car e
shoul d  be  tak en  to  ex cl ude  HF  as  a  cause  of   the  cough  bef or e  these  dr ugs  ar e  di sconti nued.
Hy potensi on,   r enal   i nsuf f i ci ency ,   and  hy per k al em i a  ar e  l ess  f r equent  but  ser i ous  si de  ef f ects  of   the
ACE  i nhi bi tor s.   In  gener al ,   these  occur   i n  pati ents  w i th  sev er e  HF  and/or   pr eex i sti ng  r enal
i nsuf f i ci ency .   In  pati ents  w i th  sev er e  HF  or   i ntr i nsi c  r enal   i nsuf f i ci ency ,   the  ACE  i nhi bi tor s  shoul d
be  star ted  i n  v er y   l ow   doses,   and  the  bl ood  pr essur e  and  ser um   potassi um   and  cr eati ni ne  l ev el s
m ust  be  m oni tor ed  car ef ul l y .

β­Adr ener gi c  bl ock er s  hav e  al so  been  show n  to  i m pr ov e  sur v i v al   i n  pati ents  w i th  sy stol i c
dy sf uncti on  and  HF.   Al though  the  benef i t  on  sy m ptom s  i s  l ess  cl ear   than  w i th  ACE  i nhi bi tor s,   β­
bl ock er s  pr oduce  a  l ar ger   i m pr ov em ent  i n  r em odel i ng,   EF,   and  sur v i v al .   Because  β­bl ock er s  r educe
hear t  r ate  and  i ni ti al l y   decr ease  contr acti l i ty ,   i ntr oducti on  of   tr eatm ent  or   up­ti tr ati on  m ay   r esul t
i n  w or seni ng  of   sy m ptom s.   These  dr ugs  m ust  ther ef or e  be  star ted  i n  l ow   doses  and  up­ti tr ated
sl ow l y ,   and  pati ents  m ust  be  m oni tor ed  car ef ul l y .   Pati ents  w i th  decom pensated  HF  usual l y   shoul d
not  be  gi v en  β­bl ock er s.   Sev er al   β­bl ock er s  (car v edi l ol ,   m etopr ol ol ,   and  bi sopr ol ol )  hav e  been
show n  to  r educe  m or tal i ty   i n  pati ents  w i th  HF.   It  i s  not  y et  cl ear   i f   ther e  ar e  adv antages  of   one
ov er   another .

Ther e  ar e  sev er al   possi bl e  addi ti ons  to  m edi cal   ther apy   i n  pati ents  w i th  chr oni c  HF  due  to  sy stol i c
dy sf uncti on,   w ho  r em ai n  sy m ptom ati c  af ter   m ax i m um   tol er abl e  doses  of   ACE  i nhi bi tor s  and  β­
bl ock er s.   The  addi ti on  of   al doster one  antagoni sts  r educes  m or tal i ty   i n  pati ents  w i th  sev er e  chr oni c
HF  and  i n  those  w ho  hav e  ex per i enced  HF  f ol l ow i ng  an  MI.   ARBs  such  as  l osar tan,   candesar tan,
i r besar tan,   and  v al sar tan  bl ock   the  angi otensi n  II  r eceptor   di r ectl y .   They   appear   to  hav e  benef i ci al
ef f ects  i n  r educi ng  car di ov ascul ar   m or tal i ty   and  hospi tal i zati on  f or   HF  w hen  added  to  ACE  i nhi bi tor s
and  β­bl ock er s.   Addi ng  both  al doster one  antagoni sts  and  ARBs  to  ACE  i nhi bi tor s  i s  pr obabl y   not
r easonabl e  f or   m ost  pati ents  because  of   the  i ncr eased  r i sk
P. 38
of   hy per k al em i a.   The  com bi nati on  of   hy dr al azi ne  and  i sosor bi de  di ni tr ate  r educes  m or tal i ty   and
hospi tal i zati ons  and  i m pr ov es  qual i ty   of   l i f e  i n  Af r i can  Am er i cans  w i th  sev er e  chr oni c  HF  due  to
sy stol i c  dy sf uncti on.   The  benef i ts  ar e  l ess  cl ear   i n  non–Af r i can  Am er i cans.   Another   opti on  i s  the
use  of   di gox i n,   w hi ch  r esul ts  i n  an  i m pr ov em ent  i n  sy m ptom s  and  a  r educti on  i n  hospi tal
adm i ssi ons  but  no  r educti on  i n  m or tal i ty .   Sodi um   r estr i cti on  i s  an  essenti al   par t  of   any   pr ogr am
desi gned  to  tr eat  pati ents  w i th  HF.   Pati ents  shoul d  av oi d  ex cess  sal t  and  w ater ,   w ei gh  them sel v es
dai l y ,   av oi d  N SAIDs,   and  r epor t  any   i ncr ease  i n  sy m ptom s  or   w ei ght  gai n  pr om ptl y   to  thei r
phy si ci ans.

7.   Is  i t  possi bl e  that  v entr i cul ar   f uncti on  w i l l   i m pr ov e  w i th  m edi cal   ther apy ?

Thi s  pati ent  has  an  EF  of   20%.   ACE  i nhi bi tor s  hel p  pr ev ent  f ur ther   deter i or ati on  i n  EF.   β­Bl ock er s,
i f   up­ti tr ated  to  r ecom m ended  doses,   ar e  l i k el y   to  i m pr ov e  thi s  i ndi v i dual 's  EF  by   7%  to  10%.   The
f ul l   i m pr ov em ent  m ay   not  be  seen  f or   up  to  6  m onths.   Your   pati ent  i m pr ov ed  af ter   di ur esi s  and
adm i ni ster i ng  ACE  i nhi bi tor s  and  β­bl ock er s.   Si x   m onths  l ater   hi s  EF  has  i ncr eased  f r om   20%  to
29%.   He  i s  on  di gox i n  w i th  ther apeuti c  l ev el s  and  an  al doster one  antagoni st  w i th  nor m al   ser um
cr eati ni ne  and  potassi um .   He  has  no  r esti ng  dy spnea  or   edem a,   but  does  hav e  dy spnea  w i th  si m pl e
task s.

8.   In  w hi ch  N YHA  cl ass  and  AHA/ACC  stage  w oul d  y ou  categor i ze  thi s  pati ent's  sy m ptom s?

Thi s  pati ent  conti nues  to  hav e  sy m ptom s  of   N YHA  cl ass  III  HF.

The  f our   categor i es  that  m ak e  up  the  N YHA  cl assi f i cati on,   and  thei r   def i ni ti ons,   ar e:

Cl ass  I:  N o  sy m ptom s  w i th  any   l ev el   of   ex er ci se

Cl ass  II:  Sy m ptom s  on  m or e  than  or di nar y   acti v i ty

Cl ass  III:  Sy m ptom s  on  acti v i ti es  of   dai l y   l i v i ng

Cl ass  IV:  Sy m ptom s  at  r est

The  AHA/ACC  stages  of   HF  ar e:

Stage  A:  Ri sk   f actor s  f or   HF

Stage  B:  Str uctur al   hear t  di sease,   but  no  HF

Stage  C:  Str uctur al   hear t  di sease  and  HF

Stage  D:  Str uctur al   hear t  di sease  and  r ef r actor y   HF.

Your   pati ent  w as  i ni ti al l y   i n  N YHA  cl ass  IV  and  AHA/ACC  stage  C–D.   He  has  i m pr ov ed  to  N YHA
cl ass  III,   stage  C.

9.   What  i s  thi s  pati ent's  ex pected  m or tal i ty   r ate  i n  hi s  cur r ent  condi ti on?
Hi s  ex pected  m or tal i ty   r ate  i n  N YHA  cl ass  IV  HF,   i f   untr eated,   w as  25%  to  50%  i n  1  y ear .   Wi th
good  m edi cal   ther apy   and  i nter nal   car di ac  def i br i l l ator   (ICD)­bi v entr i cul ar   paci ng  dev i ce,   hi s  y ear l y
m or tal i ty   m ay   i m pr ov e  to  as  l ow   as  8%  per   y ear .   An  ICD  can  pr ol ong  sur v i v al   i n  pati ents  w i th  HF
and  sy stol i c  dy sf uncti on  and  shoul d  be  i m pl anted  at  thi s  poi nt  as  m edi cal   ther apy   i s  unl i k el y   to
cause  si gni f i cant  addi ti onal   gai ns  i n  l ef t  v entr i cul ar   f uncti on.   Bi v entr i cul ar   paci ng  (i m pl anted  i n
conjuncti on  w i th  the  ICD)  i s  i ndi cated  i n  thi s  pati ent  w ho  has  sy stol i c  dy sf uncti on,   ongoi ng  HF
sy m ptom s,   and  a  QRS  dur ati on  gr eater   than  120  m i l l i seconds.   Bi v entr i cul ar   paci ng  i s  l i k el y   to
i m pr ov e  v entr i cul ar   f uncti on,   sy m ptom s,   and  sur v i v al .

P. 39

Suggested Readings
Adam s  KF,   Li ndenf el d  J,   Ar nol d  JMO,   etal .   Ex ecuti v e  sum m ar y :  HFSA  2006  com pr ehensi v e  HF  pr acti ce
gui del i nes.   J  Car d  Fai l   2006;12:10–38.

Br i stow   MR.   Beta­adr ener gi c  r eceptor   bl ock ade  i n  chr oni c  HF.   Ci r cul ati on  2000;101:558–569.

Cl el and  JG,   Dauber t  JC,   Er dm ann  E,   etal .   The  ef f ect  of   car di ac  r esy nchr oni zati on  on  m or bi di ty   and
m or tal i ty   i n  HF.   N   Engl   J  Med  2005;352:1539–1549.

Fr anci s  GS,   Tang  WH.   Pathol ogy   of   congesti v e  HF.   Rev   Cadi ov asc  Med  2003;4(Suppl   2):S14–S20.

Hunt  SA,   Abr aham   WT,   Chi n  MH,   etal .   ACC/AHA  2005  gui del i ne  update  f or   the  di agnosi s  and
m anagem ent  of   chr oni c  HF  i n  the  adul t.   Ci r cul ati on  2005;112:e154–e235.

Jessup  M,   Br ozena  S.   Hear t  f ai l ur e.   N   Engl   J  Med  2003;348:2007–2018.

McCl el l an  MB,   Loeb  JM,   Cl ancy   CM,   etal .   Angi otensi n­conv er ti ng  enzy m e  i nhi bi tor s  and  angi otensi n­
r eceptor   bl ock er s  i n  chr oni c  HF.   Ann  Inter n  Med  2005;142:386–387.

Essential Hypertension and Hypertensive Emergencies
1.   What  i s  the  esti m ated  pr ev al ence  of   sy stem i c  hy per tensi on  i n  the  U . S.   popul ati on?

2.   What  i s  the  m ost  com m on  cause  of   sy stem i c  hy per tensi on?

3.   How   i s  hy per tensi on  cl assi f i ed?

4.   What  i s  the  natur al   hi stor y   of   untr eated  hy per tensi on?

5.   Does  m edi cal   ther apy   i m pr ov e  outcom es  i n  hy per tensi on?

6.   What  i s  a  hy per tensi v e  cr i si s?

Discussion
1.   What  i s  the  esti m ated  pr ev al ence  of   sy stem i c  hy per tensi on  i n  the  U . S.   popul ati on?

Hy per tensi on  i n  the  U ni ted  States  af f ects  appr ox i m atel y   65  m i l l i on  Am er i cans.   How ev er ,   the
pr ev al ence  i ncr eases  w i th  age,   so  that  m or e  than  60%  of   the  popul ati on  ol der   than  70  y ear s  has
hy per tensi on.   The  Fr am i ngham   Hear t  Study   has  dem onstr ated  that  55­y ear ­ol d  nor m otensi v e
i ndi v i dual s  hav e  a  90%  l i f eti m e  r i sk   of   dev el opi ng  hy per tensi on.   The  i nci dence  of   hy per tensi on  and
i ts  sev er i ty   i s  gr eater   i n  bl ack s  than  w hi tes  i n  ev er y   age­gr oup  bey ond  adol escence.

2.   What  i s  the  m ost  com m on  cause  of   sy stem i c  hy per tensi on?

N o  cause  i s  f ound  f or   appr ox i m atel y   90%  of   pati ents  w i th  hy per tensi on.   These  pati ents  ar e  sai d  to
hav e  essenti al   hy per tensi on.   Al though  the  m echani sm   of   essenti al   hy per tensi on  i s  unk now n,   ther e
ar e  appar entl y   both  geneti c  and  env i r onm ental   f actor s.
3.   How   i s  hy per tensi on  cl assi f i ed?

N or m al   bl ood  pr essur e  i s  l ess  than  120/80  m m   Hg.   Bl ood  pr essur es  of   130  to  139  m m   Hg  sy stol i c
and  80  to  89  m m   Hg  di astol i c  ar e  consi der ed
P. 40
pr ehy per tensi on.   Indi v i dual s  w i th  pr ehy per tensi on  hav e  tw i ce  the  l i f eti m e  r i sk   of   dev el opi ng
hy per tensi on  as  thei r   nor m otensi v e  counter par ts.   Stage  1  hy per tensi on  i s  def i ned  as  a  sy stol i c
bl ood  pr essur e  of   140  to  159  m m   Hg  and  a  di astol i c  bl ood  pr essur e  of   90  to  99  m m   Hg,   w her eas
stage  2  hy per tensi on  i s  a  sy stol i c  bl ood  pr essur e  of   160  m m   Hg  and  gr eater   and  a  di astol i c  bl ood
pr essur e  of   100  m m   Hg  and  gr eater   (Tabl e  2­1).

4.   What  i s  the  natur al   hi stor y   of   untr eated  hy per tensi on?

U ncom pl i cated  hy per tensi on  of ten  r em ai ns  asy m ptom ati c  f or   10  to  20  y ear s  or   m or e.   How ev er ,
ther e  i s  a  di r ect  r el ati onshi p  betw een  the  l ev el s  of   both  sy stol i c  and  di astol i c  bl ood  pr essur es  and
the  i nci dence  of   str ok e,   CAD,   and  HF.   Indeed,   f or   ev er y   20  and  10  m m   Hg  i ncr em ent  i n  sy stol i c  and
di astol i c  pr essur e  r especti v el y ,   i ndi v i dual s  aged  40  to  70  y ear s  hav e  a  doubl i ng  of   car di ov ascul ar
r i sk   f r om   bl ood  pr essur es  of   115/75  to  185/115  m m   Hg.   The  ov er al l   r i sk   of   pr em atur e
car di ov ascul ar   di sease  i ncr eases  substanti al l y   w hen  addi ti onal   car di ov ascul ar   r i sk   f actor s  ar e
pr esent.   In  f act,   the  l i k el i hood  of   a  v ascul ar   ev ent  ov er   the  nex t  10  y ear s  can  be  esti m ated  f or   any
pati ent  on  the  basi s  of   thei r   age,   sex ,   and  other   r i sk s  (Am er i can  Hear t  Associ ati on's  Cor onar y   and
Str ok e  Ri sk   Handbook ).   If   pati ents  w i th  hy per tensi on  ar e  not  tr eated,   appr ox i m atel y   50%  di e  of
cor onar y   di sease,   33%  of   str ok e,   and  10%  to  15%  of   r enal   f ai l ur e.

5.   Does  m edi cal   ther apy   i m pr ov e  outcom es  i n  hy per tensi on?

Cl i ni cal   tr i al s  of   anti hy per tensi v e  ther apy   hav e  dem onstr ated  an  av er age  m ean  r educti on  of   40%
f or   str ok e,   50%  f or   HF,   and  20%  to  25%  f or   MI.

6.   What  i s  a  hy per tensi v e  cr i si s?

A  hy per tensi v e  cr i si s  i s  an  acute  l i f e­thr eateni ng  com pl i cati on  of   accel er ated  hy per tensi on.   In
pati ents  w i th  chr oni c  hy per tensi on  and  hy per tensi v e  cr i si s,   the  bl ood  pr essur e  i s  gener al l y   180/120
m m   Hg  or   gr eater ,   but  m ay   be  l ow er   i n  pati ents  w hose  bl ood  pr essur e  w as  pr ev i ousl y   nor m al   (e. g. ,
ecl am psi a).   Mal i gnant  hy per tensi on  i s  pr esent  w hen  ther e  ar e  r eti nal   hem or r hages,   ex udates  or
papi l l edem a,   and/or   m al i gnant  nephr oscl er osi s.   When  ther e  ar e  si gns  of   cer ebr al   edem a,
hy per tensi v e  encephal opathy   i s  sai d  to  be  pr esent.   Ex am pl es  of   hy per tensi v e  cr i ses  i ncl ude:

Accel er ated/m al i gnant  hy per tensi on

Hy per tensi v e  encephal opathy

Ather othr om boti c  cer ebr al   i nf ar cti on  w i th  sev er e  hy per tensi on

Aor ti c  di ssecti on

Acute  pul m onar y   edem a  or   l ef t  v entr i cul ar   f ai l ur e

Acute  MI

Ecl am psi a

Dr ug­i nduced  hy per tensi on  (cocai ne)

Case
A  45­y ear ­ol d  Af r i can­Am er i can  m an  i s  seen  i n  the  outpati ent  depar tm ent  com pl ai ni ng  of   i nter m i ttent
thr obbi ng  headaches  that  hav e  occur r ed  ev er y   m or ni ng  f or   2  w eek s.   He  has  a  hi stor y   of   untr eated,
asy m ptom ati c,   sustai ned  hi gh  bl ood  pr essur e  (160  to
P. 41
P. 42
170/100  m m   Hg)  of   4  y ear s'  dur ati on.   He  has  no  hi stor y   of   pal pi tati ons,   sw eati ng,   tr em or ,   or   per i odi c
par al y si s.   Hi s  f ather   w as  al so  hy per tensi v e  and  di ed  of   str ok e  at  67  y ear s.   The  pati ent  has  sm ok ed
ci gar ettes,   tw o  pack s  per   day ,   f or   30  y ear s.   He  i s  tak i ng  no  m edi cati ons.

Table 2­1 Classification and Management of Blood Pressure for Adults
Aged 18 Years or Older
        Ma na ge me nt a

          Initia l Drug The ra py

BP Sys tolic   Dia s tolic Life s tyle W ithout W ith Compe lling


Cla s s ific a tion BP BP  (mm Modific a tion Compe lling Indic a tions b
(mm Hg) a Indic a tions
Hg) a

N or m al < 120 and < 80 Encour age    

Pr ehy per tensi on 120­ or 80  ­89 Yes No Dr ug(s)  f or   the


139 anti hy per tensi v e com pel l i ng
dr ug  i ndi cated i ndi cati ons c

Stage  1 140­ or 90­99 Yes Thi azi de­ty pe Dr ug(s)  f or   the


hy per tensi on 159 di ur eti cs  f or com pel l i ng
m ost;  m ay i ndi cati ons
consi der   ACE
i nhi bi tor ,   ARB,
β­bl ock er ,   CCB,
or   com bi nati on

Other
anti hy per tensi v e
dr ugs  (di ur eti cs,
           
ACE  i nhi bi tor ,
ARB,   β­bl ock er ,
CCB)  as  needed

Stage  2 ≠or ≠Yes Tw o­dr ug Dr ug(s)  f or   the


hy per tensi on ¥160 ¥100 com bi nati on  f or com pel l i ng
m ost  (usual l y i ndi cati ons
thi azi de­ty pe
di ur eti c  and  ACE
i nhi bi tor   or   ARB
or   β­bl ock er   or
CCB) d

Other
anti hy per tensi v e
dr ugs  (di ur eti cs,
           
ACE  i nhi bi tor ,
ARB,   β­bl ock er ,
CCB)  as  needed

a Tr eatm ent  deter m i ned  by   hi ghest  BP  categor y .

b See  r ef er ence  bel ow .

c Tr eat  pati ents  w i th  chr oni c  k i dney   di sease  or   di abetes  to  BP  goal   of   < 130/80  m m   Hg.

d Ini ti al   com bi ned  ther apy   shoul d  be  used  cauti ousl y   i n  those  at  r i sk   f or   or thostati c  hy potensi on.
BP,   bl ood  pr essur e;  ACE,   angi otensi n­conv er ti ng  enzy m e;  ARB,   angi otensi n­r eceptor   bl ock er ;  CCB,
cal ci um   channel   bl ock er .

Fr om   Chobani an  AV,   Bak r i s  GL,   Bl ack   GL,   et  al .   JAMA  2003;289:2560­2572.

Hi s  phy si cal   ex am i nati on  r ev eal s  a  bl ood  pr essur e  of   170/110  m m   Hg  and  a  hear t  r ate  of   90  per   m i nute
and  r egul ar .   Hi s  w ei ght  i s  244  l b  and  he  i s  5  f t  10  i n.   tal l .   Fundus  ex am i nati on  r ev eal s  the  pr esence  of
ar ter i al   v asoconstr i cti on.   Car di ac  ex am i nati on  r ev eal s  a  l ater al l y   di spl aced  sustai ned  poi nt  of   m ax i m al
i m pul se,   S 4 ,   no  S 3 ,   and  no  m ur m ur .   Dur i ng  abdom i nal   ex am i nati on,   no  br ui t  or   m ass  i s  f ound  and  the
neur ol ogi c  and  other   sy stem s  ar e  unr em ar k abl e.

1.   How   shoul d  bl ood  pr essur e  be  m easur ed?


2.   What  i s  the  m ost  l i k el y   cause  of   thi s  pati ent's  hy per tensi on?
3.   What  l abor ator y   tests  ar e  i ndi cated?
4.   Wi l l   l i f esty l e  changes  i m pr ov e  hi s  bl ood  pr essur e?
5.   Is  dr ug  ther apy   i ndi cated  at  thi s  ti m e?
6.   What  i s  the  tar get  bl ood  pr essur e  w i th  tr eatm ent?

Case Discussion
1.   How   shoul d  bl ood  pr essur e  be  m easur ed?

The  pati ent  shoul d  be  seated  i n  a  chai r   w i th  hi s  f eet  on  the  f l oor   i n  a  qui et  pl ace  f or   at  l east  5
m i nutes.   At  l east  tw o  m easur em ents  shoul d  be  m ade  w i th  a  cal i br ated  i nstr um ent.   Sy stol i c  bl ood
pr essur e  i s  def i ned  as  the  bl ood  pr essur e  at  w hi ch  the  f i r st  sound  i s  hear d  and  di astol i c  pr essur e  i s
def i ned  as  the  pr essur e  at  the  di sappear ance  of   the  sounds.

2.   What  i s  the  m ost  l i k el y   cause  of   thi s  pati ent's  hy per tensi on?

The  m ost  l i k el y   cause  of   thi s  pati ent's  hy per tensi on  i s  essenti al   hy per tensi on.

3.   What  l abor ator y   tests  ar e  i ndi cated?

The  l abor ator y   i nv esti gati on  shoul d  i ncl ude  chest  r adi ogr aphy   (nor m al ),   ECG  (si nus  r hy thm   w i th
i ncr eased  v ol tage  but  no  ST­T–w av e  changes),   ur i nal y si s,   hem atocr i t,   cal ci um   (al l   nor m al ),   and
m easur em ent  of   the  f asti ng  bl ood  sugar   (nor m al ),   bl ood  ur ea  ni tr ogen,   ser um   cr eati ni ne,
el ectr ol y tes  (nor m al ),   chol ester ol   [total   and  l ow ­densi ty   l i popr otei n  (LDL)  and  hi gh­densi ty
l i popr otei n  (HDL)],   and  tr i gl y cer i de  (total   chol ester ol   i s  240  w i th  LDL  of   170,   HDL  of   40,   and  nor m al
tr i gl y cer i des).

4.   Wi l l   l i f esty l e  changes  i m pr ov e  hi s  bl ood  pr essur e?

Li f esty l e  m odi f i cati ons  ar e  an  i m por tant  par t  of   bl ood  pr essur e  m anagem ent.   Benef i ci al   l i f esty l e
m odi f i cati ons  i ncl ude  w ei ght  r educti on  i n  ov er w ei ght  or   obese  peopl e,   r egul ar   ex er ci se,   adopti on  of
theDASH  (Di etar y   Appr oaches  to  Stop  Hy per tensi on)  di et  w hi ch  i s  hi gh  i n  cal ci um   and  potassi um ,
l i m i tati on  of   sodi um   i ntak e,   and  m oder ati on  of   al cohol   consum pti on.

5.   Is  dr ug  ther apy   i ndi cated  at  thi s  ti m e?

Medi cal   ther apy   i s  i ndi cated  i n  thi s  pati ent  w ho  has  dem onstr ated  stage  2  hy per tensi on  (Tabl e  2­1
and  Fi g.   2­1).   The  pl an  of   m anagem ent  shoul d  com m ence
P. 43
w i th  i nstr ucti on  i n  l i f esty l e  changes  and  or al   anti hy per tensi v e  dr ugs  w i th  the  ai m   of   m ai ntai ni ng
bl ood  pr essur e  at  l ess  than  140/90  m m   Hg.   El i m i nati ng  coex i sti ng  car di ov ascul ar   r i sk   f actor s
(especi al l y   sm ok i ng)  and  tr eati ng  the  el ev ated  chol ester ol   w i l l   not  l ow er   the  bl ood  pr essur e  but  w i l l
l ow er   hi s  r i sk   of   subsequent  car di ov ascul ar   ev ents.
Figure  2­1  Al gor i thm   f or   tr eatm ent  of   hy per tensi on.   BP,   bl ood  pr essur e;  ACE,   angi otensi n­
conv er ti ng  enzy m e;  ARB,   angi otensi n­r eceptor   bl ock er ;  CCB,   cal ci um   channel   bl ock er .   (Fr om
Chobani an  AV,   Bak r i s  GL,   Bl ack   GL,   etal .   The  sev enth  r epor t  of   the  Joi nt  N ati onal   Com m i ttee
on  pr ev enti on,   detecti on,   ev al uati on,   and  tr eatm ent  of   hi gh  bl ood  pr essur e.   JAMA
2003;289:2560–2572. )

6.   What  i s  the  tar get  bl ood  pr essur e  w i th  tr eatm ent?

The  tar get  bl ood  pr essur e  w i th  tr eatm ent  i s  l ess  than  140/90  m m   Hg.   If   the  pati ent  had  di abetes  or
chr oni c  k i dney   di sease  the  r ecom m ended  tar get  bl ood  pr essur e  w oul d  be  l ess  than  130/80  m m   Hg.

You  i nstr uct  y our   pati ent  i n  l i f esty l e  changes  and  star t  hi m   on  l i si nopr i l   10  m g  once  dai l y .   In  2
w eek s  y ou  i ncr ease  the  l i si nopr i l   to  20  m g  dai l y   because  the  bl ood  pr essur e  i s  sti l l   160/98  m m   Hg.
El ectr ol y tes  and  cr eati ni ne  l ev el s  ar e  unchanged.   The  i ncr eased  l i si nopr i l   does  not  si gni f i cantl y
al ter   the  pr essur e  and  y ou  add  chl or thal i done  at  25  m g  dai l y .   In  4  w eek s  hi s  bl ood  pr essur e  i s
139/88  m m   Hg
P. 44
and  hi s  el ectr ol y tes  and  cr eati ni ne  l ev el s  ar e  nor m al .   The  bl ood  pr essur e  r em ai ns  w el l   contr ol l ed
f or   the  nex t  6  m onths,   but  the  pati ent  does  not  r etur n  f or   the  nex t  f ol l ow ­up  v i si t  and  does  not
r espond  to  phone  cal l s.   Tw o  y ear s  l ater ,   he  pr esents  to  the  ER  com pl ai ni ng  of   bl ur r ed  v i si on  and
sev er e  headaches.   Hi s  phy si cal   ex am i nati on  at  that  ti m e  r ev eal s  a  bl ood  pr essur e  of   240/140  m m
Hg  and  hear t  r ate  of   100  per   m i nute.   He  i s  m i l dl y   conf used  and  the  f undus  ex am i nati on  r ev eal s
r eti nal   hem or r hages,   ex udates,   and  papi l l edem a.   Hear t  ex am i nati on  show s  cl ear   l ungs  and  a
sustai ned  l ef t  v entr i cul ar   api cal   i m pul se  and  S 4 .   The  chest  r adi ogr aph  show s  m i l d  to  m oder ate
car di om egal y .   Hi s  ser um   cr eati ni ne  l ev el   i s  2. 4  m g/dL.   The  ECG  show s  nor m al   si nus  r hy thm   w i th
i ncr eased  v ol tage  and  ST­segm ent  depr essi on  and  T­w av e  i nv er si on.   Tr oponi n  i s  nor m al   and  he  has
no  chest  pai n.

a.   What  i s  the  di agnosi s?

b.   What  i s  the  m ost  l i k el y   r eason  f or   the  f undus  f i ndi ngs  and  the  ser um   cr eati ni ne  l ev el   of   2. 4
m g/dL?

c.   What  shoul d  be  the  pl an  of   tr eatm ent  now ?

a.   What  i s  the  di agnosi s?

The  di agnosi s  i s  hy per tensi v e  cr i si s  and  accel er ated  m al i gnant  hy per tensi on.   A  hy per tensi v e
cr i si s  i s  consi der ed  a  m edi cal   em er gency .   Such  hi gh  bl ood  pr essur e  can  cause  i m m edi ate
v ascul ar   dam age,   as  seen  i n  thi s  pati ent.   The  pr esence  of   sev er e  hy per tensi on  (di astol i c  bl ood
pr essur e  of   115  m m   Hg  or   gr eater )  i n  conjuncti on  w i th  gr ade  3  (r eti nal   hem or r hage  and
ex udate)  or   gr ade  4  (papi l l edem a)  f unduscopi c  changes  i s  def i ned  as  accel er ated  or   m al i gnant
hy per tensi on.

b.   What  i s  the  m ost  l i k el y   r eason  f or   the  f undus  f i ndi ngs  and  the  ser um   cr eati ni ne  l ev el   of   2. 4
m g/dL?

Modest  i ncr eases  i n  bl ood  pr essur e  r esul t  i n  ar ter i ol ar   v asoconstr i cti on.   The  v asoconstr i cti on
k eeps  ti ssue  per f usi on  constant.   How ev er ,   w i th  the  m ar k ed  i ncr ease  i n  bl ood  pr essur e  causi ng
a  sudden  i ncr ease  i n  ti ssue  per f usi on,   ther e  i s  dam age  to  the  v ascul ar   endothel i um   causi ng
f i br i noi d  necr osi s  i n  the  v essel s  of   the  ey e  and  i n  the  k i dney .   These  changes  ar e  ex acer bated
by   acti v ati on  of   the  r eni n–angi otensi n  sy stem .

c.   What  shoul d  be  the  pl an  of   tr eatm ent  now ?

The  pati ent  shoul d  be  adm i tted  to  a  m oni tor ed  uni t.   An  ECG,   chest  x ­r ay ,   el ectr ol y tes,
ur i nal y si s,   hem atocr i t,   and  tr oponi n  shoul d  be  obtai ned.   Because  the  papi l l edem a  i s  consi stent
w i th  the  pr esence  of   sev er e  hy per tensi on  and  r epr esents  ear l y   br ai n  edem a,   w hi ch  m ay
com pr om i se  the  autor egul ati on  of   cer ebr al   bl ood  f l ow ,   the  tr eatm ent  appr oach  shoul d  be  to
star t  hi m   on  par enter al   anti hy per tensi v e  dr ug  ther apy .   The  goal   of   tr eatm ent  shoul d  be  to
decr ease  the  bl ood  pr essur e  by   20%  to  25%  m ai ntai ni ng  the  di astol i c  bl ood  pr essur e  betw een
100  and  110  m m   Hg  or   the  m ean  ar ter i al   pr essur e  at  not  l ess  than  120  m m   Hg,   because  an
abr upt  decr ease  i n  the  bl ood  pr essur e  to  “nor m al â€​   l ev el s  m ay   pr oduce  hy poper f usi on  to
the  br ai n,   hear t,   and  k i dney   due  to  l ack   of   autor egul ati on.

P. 45
A  l i st  of   par enter al   agents  used  to  tr eat  hy per tensi v e  em er genci es  i s  gi v en  i n  Tabl e  2­2.

Table 2­2 Parenteral Agents Used to Treat Hypertensive
Emergencies

Drug Route Ons e t Dura tion Dos e  or Dos a ge

Va s odila tors        

Sodi um IV Seconds­ 3­5  m i n 0. 25­10  µ  g/k g/m i n


ni tr opr ussi de i nf usi on 1  m i n

N i car di pi ne IV 5­10  m i n 1­4  hr 2­10  m g/hr


hy dr ochl or i de

Fenol dopam IV < 5  m i n 30  m i n 0. 1­0  . 3  µ  g/k g/m i n


m esy l ate i nf usi on

N i tr ogl y cer i n IV 1­2  m i n 3­5  m i n 5­100  µ  g/m i n


i nf usi on

Di azox i de IV  bol us 1­5  m i n 6­12  hr 50  m g  IV  ev er y   5­10  m i n  ov er   30  s,


or or   15­30  m g/m i n  by   IV  i nf usi on
i nf usi on

Hy dr al azi ne IV 10­20 3­8  hr 10­20  m g  IV


min

IM  (al so 30  m i n 3­8  hr 10­50  m g  IM


 
or al )
Enal apr i l at IV  bol us 15­30 6  hr 1. 25­5  m g
min

Adre ne rgic
       
Inhibitors

Labetal ol IV 5  m i n 3­6  hr 0. 5­2  m g/m i n  IV  i nf usi on  or   20­80


m g  ev er y   10  m i n  to  a  m ax i m um
cum ul ati v e  dose  of   300  m g

Tr i m ethaphan IV 1­5  m i n 10  m i n 0. 5­5  m g/m i n


i nf usi on

Phentol am i ne IV  bol us 1­2  m i n 3­10 Load  5­15  m g  IV  ev er y   5  m i n


min

Esm ol ol IV  bol us 1­2  m i n 10­20 250­500  µ  g/k g  bol us,   then  50­300
then min µ  g/k g/m i n  i nf usi on
i nf usi on

IV,   i ntr av enous;  IM,   i ntr am uscul ar .

Fr om   Chobani an  AV,   Bak r i s  GL,   Bl ack   GL,   et  al .   The  sev enth  r epor t  of   the  Joi nt  N ati onal
Com m i ttee  on  pr ev enti on,   detecti on,   ev al uati on,

and  tr eatm ent  of   hi gh  bl ood  pr essur e.   Hy per tensi on  2003;42:1206­1252.

Ov er   the  subsequent  3  m onths,   bl ood  pr essur e  shoul d  be  l ow er ed  gr adual l y   to  l ess  than  140/90  m m
Hg  (or   to  130/80  m m   Hg  i f   ther e  i s  di abetes  or   chr oni c  k i dney   di sease,   as  i n  y our   pati ent)  w i th  or al
agents.

Suggested Readings
Appel   LJ,   Br ands  MW,   Dani el s  SR,   etal .   Di etar y   appr oaches  to  pr ev ent  and  tr eat  hy per tensi on.
Hy per tensi on  2006;47:296–308.

Bender   KR,   Fi l i ppone  JD,   Hei tz  S,   etal .   A  sy stem ati c  appr oach  to  hy per tensi v e  ur genci es  and
em er genci es.   Cur r   Hy per tens  Rev   2005;1:275–281.

Bol l i   P,   My er s  M,   McKay   D.   Canadi an  hy per tensi on  educati on  pr ogr am .   Appl y i ng  the  2005  Canadi an
hy per tensi on  educati on  pr ogr am   r ecom m endati ons:  1.   Di agnosi s  of   hy per tensi on.   CMAJ
2005;173:480–483.

Chobani an  AV,   Bak r i s  GL,   Bl ack   HR,   etal .   The  sev enth  r epor t  of   the  Joi nt  N ati onal   Com m i ttee  on
pr ev enti on,   detecti on,   ev al uati on,   and  tr eatm ent  of   hi gh  bl ood  pr essur e.   JAMA  2003;289:2560â
€“2572.

P. 46

Hem m el gar n  BR,   Gr ov er   S,   Fel dm an  RD.   Canadi an  hy per tensi on  educati on  pr ogr am .   Appl y i ng  the
2005  Canadi an  hy per tensi on  educati on  pr ogr am   r ecom m endati ons:  2.   assessi ng  and  r educi ng  gl obal
ather oscl er oti c  r i sk   am ong  hy per tensi v e  pati ents.   CMAJ  2005;173:593–595.
Khan  N A,   Ham et  P,   Lew anczuk   RZ.   Canadi an  hy per tensi on  educati on  pr ogr am .   Appl y i ng  the  2005
Canadi an  hy per tensi on  educati on  pr ogr am   r ecom m endati ons:  4.   Managi ng  uncom pl i cated
hy per tensi on.   CMAJ  2005;173:865–867.

MacMahon  S,   Peto  R,   Cutl er   J,   etal .   Bl ood  pr essur e,   str ok e,   and  cor onar y   hear t  di sease:  par t  1.
pr ol onged  di f f er ences  i n  bl ood  pr essur e:  pr ospecti v e  obser v ati onal   studi es  cor r ected  f or   the
r egr essi on  di l uti on  bi as.   Lancet  1990;335:827–838.

Padw al   R,   Cam pbel l   N ,   Touy z  RM.   Canadi an  hy per tensi on  educati on  pr ogr am .   Appl y i ng  the  2005
Canadi an  hy per tensi on  educati on  pr ogr am   r ecom m endati ons:  3.   Li f esty l e  m odi f i cati ons  to  pr ev ent
and  tr eat  hy per tensi on.   CMAJ  2005;173:749–751.

Tobe  S,   McAl i ster   FA,   Lei ter   L.   Appl y i ng  the  2005  Canadi an  hy per tensi on  educati on  pr ogr am
r ecom m endati ons:  5.   Ther apy   f or   pati ents  w i th  hy per tensi on  and  di abetes  m el l i tus.   CMAJ
2005;173:1154–1157.

Vasan  RS,   Bei ser   A,   Ser shadr i   S,   etal .   Resi dual   l i f eti m e  r i sk   f or   dev el opi ng  hy per tensi on  i n  m i ddl e­
aged  w om en  and  m en:  the  Fr am i ngham   hear t  study .   JAMA  2002;287:1003–1010.

Wi l son  PW.   Establ i shed  r i sk   f actor s  and  cor onar y   ar ter y   di sease:  The  Fr am i ngham   Study .   Am   J
Hy per tens  1994;7:7S–12S.

ST­Elevation Myocardial Infarction
1.   What  ar e  the  m ajor   k now n  r i sk   f actor s  f or   CAD?

2.   What  i s  the  l i f eti m e  r i sk   of   i schem i c  hear t  di sease  deaths  i n  m en  and  w om en  i n  the  U . S.
popul ati on?

3.   What  i s  the  m ost  com m on  cause  of   acute  MI?

4.   In  pl acebo­contr ol l ed  tr i al s,   w hat  ty pes  of   tr eatm ents  hav e  been  show n  to  i m pr ov e  outcom e  i n
pati ents  w i th  acute  MI?

Discussion
1.   What  ar e  the  m ajor   k now n  r i sk   f actor s  f or   CAD?

The  establ i shed  m ajor   r i sk   f actor s  f or   CAD  i ncl ude  sm ok i ng,   hy per tensi on,   dy sl i pi dem i aâ
€”speci f i cal l y   i ncr eased  LDL  chol ester ol   and  l ow   HDL  chol ester ol ,   di abetes  m el l i tus,   f am i l y   hi stor y
of   CAD  i n  a  f i r st­degr ee  r el ati v e,   m al e  gender ,   and  age.   The  f i r st  f our   f actor s  ar e  m odi f i abl e  w hi l e
the  l ast  thr ee  ar e  not.   Ther e  ar e  other   establ i shed  r i sk   f actor s  that  can  be  m odi f i ed  such  as
obesi ty ,   phy si cal   i nacti v i ty ,   an  ather ogeni c  di et,   m ental   str ess,   and  depr essi on.   The  m etabol i c
sy ndr om e  i s  al so  consi der ed  an  i m por tant  r i sk   f actor   by   the  N ati onal   Chol ester ol   Educati on  Pr ogr am
(N CEP)  gui del i nes.   Em er gi ng  r i sk   f actor s  i ncl ude  hi gh  sensi ti v i ty   C­r eacti v e  pr otei n  (hs­CRP),
hom ocy stei ne,   l i popr otei n  a,   sm al l   dense  LDL,   pr othr om boti c  f actor s  f i br i nogen,   i m bal ance  betw een
ti ssue  pl asm i nogen  acti v ator   and  pl asm i nogen  acti v ator   i nhi bi tor   1  (PAI­1),   pr oi nf l am m ator y   f actor s
other   than  hs­CRP,   and  i ncr eased  ox i dati v e  str ess.

P. 47
2.   What  i s  the  l i f eti m e  r i sk   of   i schem i c  hear t  di sease  deaths  i n  m en  and  w om en  i n  the  U . S.
popul ati on?

The  l i f eti m e  r i sk   of   dev el opi ng  CAD  af ter   age  40  i s  49%  i n  m en  and  32%  i n  w om en.   CAD  i s  the
l eadi ng  cause  of   death  i n  both  m en  and  w om en,   accounti ng  f or   appr ox i m atel y   20%  of   al l   deaths  i n
the  U ni ted  States.   How ev er ,   w om en  dev el op  sy m ptom ati c  CAD  10  to  15  y ear s  l ater   than  m en.

3.   What  i s  the  m ost  com m on  cause  of   acute  MI?

Pl aque  r uptur e,   i n  the  setti ng  of   cor onar y   ather oscl er osi s,   i s  the  under l y i ng  cause  of   MI  i n  m ost
pati ents.   At  the  si te  of   pl aque  r uptur e,   ther e  i s,   i n  m ost  cases,   f or m ati on  of   an  acute  thr om bus.
Inf r equent  causes  of   MI  i ncl ude  i nf l am m ati on  (ar ter i ti s),   tr aum a,   or   cor onar y   em bol i sm .

4.   In  pl acebo­contr ol l ed  tr i al s,   w hat  ty pes  of   tr eatm ents  hav e  been  show n  to  i m pr ov e  outcom e  i n
pati ents  w i th  acute  MI?

Aspi r i n,   cl opi dogr el ,   r ev ascul ar i zati on  w i th  thr om bol y ti c  agents  or   PCI,   β­adr ener gi c  bl ock er s,   ACE
i nhi bi tor s  or   ARBs,   and  al doster one  i nhi bi tor s  (epl er enone)  hav e  al l   been  show n  to  r educe  m or tal i ty
af ter   MI.   Ther e  ar e  l ess  com pel l i ng  data  r egar di ng  the  uti l i ty   of   GIK  (gl ucose,   i nsul i n,   and
potassi um )  sol uti on  i nf usi on,   and  hepar i n  and  ni tr ate  use.   In  the  l ong  ter m ,   l ow er i ng  the  LDL
chol ester ol   w i th  HMG­CoA  r eductase  i nhi bi tor s  (stati ns)  and  the  use  of   w ar f ar i n  hav e  been  show n  to
be  benef i ci al .   The  benef i t  of   epl er enone  has  been  show n  onl y   i n  hi gh­r i sk   pati ents.   The  use  of
w ar f ar i n  i s  cum ber som e  and  not  w i despr ead.   The  use  of   aspi r i n  i m par ts  near l y   the  sam e  sur v i v al
benef i t  as  the  use  of   thr om bol y ti cs  and  other   m or e  ex pensi v e  ther api es.   Fi nal l y ,   ear l y   tr eatm ent
al so  i m par ts  the  hi ghest  benef i t  (the  gol den  hour ).

Case 1
A  62­y ear ­ol d  m an  w i th  a  hi stor y   of   hy per tensi on  i s  m ow i ng  hi s  l aw n  at  9:00  a. m .   on  a  Satur day
m or ni ng  w hen  he  ex per i ences  a  heav y   sensati on  i n  hi s  chest.   He  stops  m ow i ng  the  l aw n  and  w i thi n  10
m i nutes  hi s  sy m ptom s  r esol v e,   and  he  r esum es  cutti ng  the  gr ass.   Appr ox i m atel y   10  m i nutes  l ater ,   he
ex per i ences  sev er e,   cr ushi ng  chest  pai n  associ ated  w i th  shor tness  of   br eath  and  pai n  r adi ati ng  dow n  hi s
l ef t  ar m .   As  he  w al k s  to  hi s  house,   he  becom es  di aphor eti c  and  nauseated,   and  v om i ts  tw i ce.   At  thi s
poi nt,   he  cal l s  an  am bul ance  and  i s  tak en  to  the  ER.   When  y ou  ar r i v e  to  ex am i ne  hi m ,   he  i s  sti l l
ex per i enci ng  sev er e  pai n.   A  12­l ead  ECG  r ev eal s  3­m m   ST­segm ent  el ev ati on  i n  l eads  V 2 ,   V 3 ,   V 4 ,   and  V 5
w i th  i nf er i or   ST­segm ent  depr essi on.   The  pai n  has  been  pr esent  f or   a  total   of   appr ox i m atel y   45  m i nutes.

1.   What  i ni ti al   acti ons  shoul d  be  tak en  i n  thi s  pati ent?
2.   Is  thi s  pati ent's  hy per tensi on  a  contr ai ndi cati on  to  thr om bol y ti c  ther apy ?
3.   What  ar e  the  r i sk s  associ ated  w i th  thr om bol y ti c  ther apy   and  how   l ong  af ter   the  onset  of   acute  MI  i s
ther apy   benef i ci al ?
4.   Whi ch  i s  the  better   r eper f usi on  ther apy   f or   acute  MI—thr om bol y ti c  ther apy   or   pr i m ar y
per cutaneous  tr ansl um i nal   cor onar y   angi opl asty   (PTCA)?
5.   What  ther api es  shoul d  be  adm i ni ster ed  acutel y   w i th  thr om bol y si s  or   pr i m ar y   PTCA?
P. 48
6.   What  m easur es  shoul d  be  car r i ed  out  bef or e  thi s  pati ent  i s  di schar ged?
7.   U nder   w hat  ci r cum stances  shoul d  the  pati ents  under go  cor onar y   angi ogr aphy   i f   they   di d  not
under go  acute  angi opl asty   and/or   stenti ng  on  adm i ssi on?

Case Discussion
1.   What  i ni ti al   acti ons  shoul d  be  tak en  i n  thi s  pati ent?

The  f i r st  acti ons  that  shoul d  be  tak en  i n  thi s  pati ent  ar e  to  adm i ni ster   subl i ngual   N TG,   adm i ni ster
ox y gen  i f   ox y gen  satur ati on  i s  bel ow   90%,   and  establ i sh  v enous  access.   IV  β­bl ock er s  and  aspi r i n
shoul d  be  gi v en.   Anal gesi cs  such  as  m or phi ne  shoul d  be  gi v en  i f   the  pai n  does  not  r esol v e  w i th
N TG.   Im m edi ate  tr ansf er   to  the  car di ac  catheter i zati on  l abor ator y   f or   cor onar y   angi opl asty   and
r eper f usi on  of   the  i nf ar ct­r el ated  ar ter y   i s  the  tr eatm ent  of   choi ce  i f   i t  can  be  accom pl i shed  w i thi n
3  hour s  of   the  onset  of   chest  pai n.   If   not,   thr om bol y ti c  agents  shoul d  be  adm i ni ster ed  i m m edi atel y
i f   ther e  ar e  no  contr ai ndi cati ons.   Pati ents  shoul d  be  questi oned  about  contr ai ndi cati ons  to
thr om bol y ti c  agents  bef or e  adm i ni str ati on.   Inv asi v e  pr ocedur es  such  as  ar ter i al   punctur e  shoul d  be
m i ni m i zed  i f   thr om bol y ti c  agents  ar e  to  be  adm i ni ster ed  to  av oi d  bl eedi ng.   If   the  pati ent  pr esents
m or e  than  3  hour s  f ol l ow i ng  the  onset  of   chest  pai n  PCI  i s  cl ear l y   pr ef er abl e,   because  of   the
di f f i cul ty   i n  l y si ng  the  cl ot  af ter   3  hour s.   How ev er ,   studi es  hav e  show n  that  ei ther   thr om bol y ti cs  or
PCI  i s  benef i ci al   f or   at  l east  12  hour s  af ter   the  onset  of   pai n.

2.   Is  thi s  pati ent's  hy per tensi on  a  contr ai ndi cati on  to  thr om bol y ti c  ther apy ?

Hy per tensi on  al one  i s  not  a  contr ai ndi cati on  to  thr om bol y ti c  ther apy .   If   the  hy per tensi on  i s
uncontr ol l ed  and  cannot  be  l ow er ed  qui ck l y   to  a  l ev el   bel ow   180/110  m m   Hg,   the  r i sk   of
i ntr acr ani al   bl eedi ng  i s  i ncr eased.   These  thr om bol y ti c  agents  can  sti l l   be  consi der ed  i n  i ndi v i dual
pati ents.   Absol ute  contr ai ndi cati ons  to  thr om bol y ti c  ther apy   i ncl ude  any   pr i or   i ntr acr ani al
hem or r hage  (ICH),   k now n  cer ebr ov ascul ar   or   i ntr acr ani al   neopl asti c  l esi on,   i schem i c  str ok e  w i thi n
3  m onths,   acti v e  bl eedi ng  ex cl udi ng  m enses,   suspected  aor ti c  di ssecti on,   and  si gni f i cant  cl osed
head  or   f aci al   tr aum a  w i thi n  3  m onths.   Rel ati v e  contr ai ndi cati ons  i n  addi ti on  to  uncontr ol l ed
hy per tensi on  ar e  an  ol d  hi stor y   of   str ok e,   pr ol onged  car di opul m onar y   r esusci tati on  (CPR)  (m or e
than  10  m i nutes)  or   m ajor   sur ger y   w i thi n  3  w eek s,   i nter nal   bl eedi ng  i n  the  l ast  4  w eek s,   acti v e
pepti c  ul cer ,   a  k now n  bl eedi ng  di athesi s  or   use  of   anti coagul ants,   and  pr egnancy .

3.   What  ar e  the  r i sk s  associ ated  w i th  thr om bol y ti c  ther apy   and  how   l ong  af ter   the  onset  of   acute  MI  i s
ther apy   benef i ci al ?

The  m ajor   r i sk   of   thr om bol y ti c  ther apy   i s  bl eedi ng.   Thi s  r i sk   i s  l ow est  w i th  str eptok i nase,   and
hi ghest  w i th  new er   agents  and  w hen  hepar i n  i s  added  to  ther apy .   Wi th  al tepl ase­l i k e  agents,   m ajor
bl eedi ng  occur s  i n  appr ox i m atel y   5%  of   pati ents  and  ICH  occur s  i n  0. 9%.   Factor s  that  i ncr ease  ICH
i ncl ude  age  (especi al l y   gr eater   than  75  y ear s),   w ei ght  l ess  than  70  k g,   and  hy per tensi on  (160/95  or
hi gher )  at  pr esentati on  and  the  use  of   al tepl ase.   Pati ents  w i th  m or e  than  thr ee  r i sk   f actor s  hav e
tw o  or   thr ee  ti m es  hi gher   r i sk   of   ICH.   Pati ents  w i th  acute  MI  benef i t  f r om
P. 49
thr om bol y ti cs  f or   up  to  12  hour s  af ter   the  onset  of   the  i nf ar cti on  w i th  ear l i er   tr eatm ent  l eadi ng  to
hi gher   sur v i v al .

4.   Whi ch  i s  the  better   r eper f usi on  ther apy   f or   acute  MI—thr om bol y ti c  ther apy   or   pr i m ar y   PTCA?

PTCA  i s  gener al l y   pr ef er r ed  ov er   thr om bol y ti c  agents  as  a  r eper f usi on  ther apy .   Thr om bol y ti cs  can
be  used  i n  pati ents  pr esenti ng  ear l y   (< 3  hour s)  especi al l y   w hen  a  catheter i zati on  l abor ator y   i s  not
r eadi l y   av ai l abl e.   Pr i m ar y   PTCA  i s  pr ef er r ed  i n  m ost  i nstances  w her e  ther e  i s  r api d  access  to  a
sk i l l ed  l abor ator y ,   especi al l y   i n  hi gher   r i sk   pati ents  ei ther   due  to  car di ogeni c  shock   or   si gni f i cant
HF.   It  i s  al so  pr ef er r ed  i n  pati ents  pr esenti ng  l ater   than  3  hour s  f r om   sy m ptom   onset,   w hen  ther e
ar e  si gni f i cant  contr ai ndi cati ons  to  thr om bol y ti cs  or   w hen  the  di agnosi s  i s  i n  doubt.

5.   What  ther api es  shoul d  be  adm i ni ster ed  acutel y   w i th  thr om bol y si s  or   pr i m ar y   PTCA?

Chew abl e  aspi r i n  (162. 5  m g)  shoul d  be  adm i ni ster ed  i m m edi atel y   once  the  di agnosi s  i s  m ade  i n  al l
pati ents  unl ess  ther e  i s  a  contr ai ndi cati on  to  aspi r i n  (i . e. ,   aspi r i n  al l er gy   or   acti v e  bl eedi ng).   IV  β­
bl ock ade  shoul d  be  i nsti tuted  unl ess  ther e  ar e  contr ai ndi cati ons  to  thei r   use  such  as  pul m onar y
edem a,   si gni f i cant  atr i ov entr i cul ar   bl ock ,   hear t  r ate  l ess  than  60  per   m i nute,   sy stol i c  bl ood
pr essur e  l ess  than  100  m m   Hg  or   si gni f i cant  br onchospasm   and  hi stor y   of   asthm a.   ACE  i nhi bi tor s
(or   ARBs  f or   al l er gi c  pati ents)  shoul d  be  begun  w i thi n  the  f i r st  24  hour s  i n  the  absence  of
contr ai ndi cati ons  such  as  sy stol i c  bl ood  pr essur e  l ess  than  100  m m   Hg,   r enal   i nsuf f i ci ency   (ser um
cr eati ni ne  gr eater   than  3. 0  m g/dL),   or   hy per k al em i a.   Cl opi dogr el   shoul d  al so  be  used  dur i ng  the
hospi tal   stay .

6.   What  m easur es  shoul d  be  car r i ed  out  bef or e  thi s  pati ent  i s  di schar ged?

Ther apy   w i th  a  stati n  shoul d  be  star ted.   LDL  chol ester ol   l ev el s  f al l   af ter   the  f i r st  24  hour s  af ter   an
acute  MI,   so  l i pi d  l ev el   m easur em ents  shoul d  be  done  w i thi n  24  hour s  of   adm i ssi on.   Ri sk
str ati f i cati on  w i th  subm ax i m al   ex er ci se  test  and  assessm ent  of   l ef t  v entr i cul ar   EF  shoul d  be
per f or m ed  i n  pati ents  w ho  w er e  not  str ati f i ed  by   angi ogr aphy .   If   a  stent  w as  pl aced  dur i ng  PTCA,
cl opi dogr el   i s  added  f or   3  to  6  m onths  and  per haps  l onger   i f   a  dr ug­el uti ng  stent  w as  pl aced.
Fi nal l y ,   an  al doster one  antagoni st  shoul d  be  added  f or   pati ents  w i th  abnor m al   car di ac  f uncti on  and
HF  or   di abetes.

Al l   pati ents  shoul d  be  counsel ed  on  sm ok i ng  cessati on  and  a  l ow ­f at  di et.   Each  pati ent  shoul d  be
taught  how   to  use  N TG  and  shoul d  be  i nstr ucted  w hen  to  cal l   f or   pr obl em s.

7.   U nder   w hat  ci r cum stances  shoul d  pati ents  under go  cor onar y   angi ogr aphy   i f   they   di d  not  under go
acute  angi opl asty   and/or   stenti ng  on  adm i ssi on?

Resi dual   i schem i c  m y ocar di um   and  l ow   EF  ar e  m ajor   r i sk   f actor s  f or   m or tal i ty .   Thi s  i s  w hy   pati ents
w i th  r ecur r ent  i schem i c  chest  pai n,   a  posi ti v e  subm ax i m al   ex er ci se  test,   or   an  EF  l ess  than  40%
usual l y   under go  cor onar y   angi ogr aphy   to  deter m i ne  i f   r esi dual   l esi ons  causi ng  i schem i a  can  be
cor r ected.

Case 2
A  67­y ear ­ol d  w om an  i s  i n  tow n  v i si ti ng  her   chi l dr en  w hen  she  pr esents  to  y our   of f i ce  com pl ai ni ng  of
sev er e  sy m ptom s  of   shor tness  of   br eath  that  has  w or sened  ov er   the  l ast  12  hour s.   She  tel l s  y ou  that
she  has  had  di abetes  m el l i tus  f or   the  l ast  20  y ear s  and
P. 50
hy per tensi on  that  has  been  f ai r l y   w el l   contr ol l ed  f or   15  y ear s.   Your   ex am i nati on  r ev eal s  an  S 3   gal l op
and  r al es  to  her   m i dscapul ar   ar ea.   She  al so  tel l s  y ou  that  she  has  ex per i enced  r ecur r ent  chest
heav i ness  ov er   the  l ast  2  day s.   When  the  ECG  i s  done,   ther e  ar e  Q  w av es  i n  l eads  V 2 ,   V 3 ,   V 4 ,   and  V 5 .   A
cal l   to  her   r egul ar   phy si ci an  r ev eal s  she  had  a  nor m al   ECG  w hen  he  saw   her   1  m onth  ago.

1.   At  thi s  poi nt,   w hat  shoul d  y ou  do?


2.   What  ther apeuti c  i nter v enti ons  shoul d  be  i nsti tuted  at  the  ti m e  of   adm i ssi on?
3.   Bef or e  di schar ge,   she  has  an  echocar di ogr am   per f or m ed.   What  f i ndi ngs  w oul d  f av or   l ong­ter m
anti coagul ant  ther apy   w i th  sodi um   w ar f ar i n?
4.   Shoul d  thi s  pati ent  under go  cor onar y   angi ogr aphy   or   shoul d  she  hav e  a  subm ax i m al   ex er ci se  test?
5.   Woul d  y ou  r ecom m end  PTCA,   sur ger y ,   or   m edi cal   ther apy

Case Discussion
1.   At  thi s  poi nt,   w hat  shoul d  y ou  do?

Your   pati ent  has  had  a  r ecent  anter i or   MI  w i th  l ef t  v entr i cul ar   f ai l ur e  causi ng  her   sy m ptom s.   She
needs  to  be  hospi tal i zed  i m m edi atel y ,   tr eated  f or   HF,   m oni tor ed  f or   ar r hy thm i as  and  r ecur r ent
i schem i a,   and  r i sk ­str ati f i ed.   Thr om bol y ti c  ther apy   or   PCI  i s  not  i ndi cated  because  thi s  i s  a
com pl eted  i nf ar cti on,   near l y   48  hour s  ol d.

2.   What  ther apeuti c  i nter v enti ons  shoul d  be  i nsti tuted  at  the  ti m e  of   adm i ssi on?

Ini ti al   tr eatm ent  consi sts  of   ox y gen  adm i ni str ati on  f or   hy pox em i a  and  di ur esi s  w hi l e  av oi di ng
hy pok al em i a.   The  goal   of   di ur esi s  i s  to  r esol v e  pul m onar y   congesti on.   Aspi r i n  shoul d  be  star ted.
Tel em etr y   m oni tor i ng  i s  necessar y   to  detect  ar r hy thm i as.   ACE  i nhi bi tor s  (or   an  ARB  i f   al l er gi c)
shoul d  be  star ted  i f   the  pati ent  i s  not  hy potensi v e  and  has  no  contr ai ndi cati ons  to  thei r   use.
Al doster one  bl ock er s  shoul d  be  i ntr oduced.   Hepar i n  shoul d  be  consi der ed  i n  thi s  pati ent  w i th  a
l ar ge  anter i or   MI  because  of   the  r i sk   of   l ef t  v entr i cul ar   api cal   thr om bus  f or m ati on  and  em bol i sm .   A
β­bl ock er   shoul d  be  consi der ed  onl y   af ter   r esol uti on  of   the  pati ent's  sy m ptom s  of   HF.

3.   Bef or e  di schar ge,   she  has  an  echocar di ogr am   per f or m ed.   What  f i ndi ngs  w oul d  f av or   l ong­ter m
anti coagul ant  ther apy   w i th  sodi um   w ar f ar i n?

An  api cal   thr om bus,   especi al l y   i f   m obi l e,   i ncr eases  the  r i sk   of   em bol i sm   and  i s  consi der ed  an
acceptabl e  i ndi cati on  f or   anti coagul ati on.   The  sam e  i s  tr ue  of   a  dy sk i neti c  or   ak i neti c  v entr i cul ar
segm ent.   In  these  cases,   w ar f ar i n  i s  conti nued  f or   3  to  6  m onths  or   unti l   a  thr om bus  i s  no  l onger
pr esent.   How ev er ,   these  r ecom m endati ons  ar e  not  based  on  pr ospecti v e  r andom i zed  tr i al s.

Tw o  cl ear   i ndi cati ons  f or   anti coagul ati on  i n  thi s  setti ng  ar e  the  pr esence  of   atr i al   f i br i l l ati on  or   a
hi stor y   of   a  pr ev i ous  em bol i c  epi sode.

4.   Shoul d  thi s  pati ent  under go  cor onar y   angi ogr aphy   or   shoul d  she  hav e  a  subm ax i m al   ex er ci se  test?

Thi s  w om an  pr esented  w i th  a  l ar ge  MI  and  HF  suggesti ng  sev er e  CAD.   Her   m or tal i ty   r i sk   i s  hi gh  and
ther ef or e  ex er ci se  testi ng  f or   r i sk   str ati f i cati on  i s  not  necessar y .   She  shoul d  ther ef or e  be
ev al uated  di r ectl y   w i th  cor onar y   angi ogr aphy .

P. 51
Cor onar y   angi ogr aphy   show s  a  90%  pr ox i m al   r i ght  cor onar y   ar ter y   obstr ucti on,   a  90%  pr ox i m al   l ef t
anter i or   descendi ng  (LAD)  obstr ucti on,   and  a  100%  pr ox i m al   ci r cum f l ex   obstr ucti on.   Her   EF  by   l ef t
v entr i cul ar   angi ogr aphy   i s  34%,   w i th  m oder ate  anter i or   hy pok i nesi s.

5.   Woul d  y ou  r ecom m end  PTCA,   sur ger y ,   or   m edi cal   ther apy ?

Wi th  sev er e  thr ee­v essel   di sease  and  l ef t  v entr i cul ar   dy sf uncti on,   cor onar y   ar ter y   by pass  sur ger y
i s  i ndi cated  i n  thi s  pati ent.   The  pr esence  of   di abetes  f av or s  sur ger y   ov er   PTCA  i n  thi s  par ti cul ar
case  ev en  i f   the  EF  i s  not  l ow .

Suggested Readings
Antm an  EM,   Anbe  DT,   Ar m str ong  PW,   etal .   ACC/AHA  gui del i nes  f or   the  m anagem ent  of   pati ents  w i th
ST­el ev ati on  m y ocar di al   i nf ar cti on—ex ecuti v e  sum m ar y :  a  r epor t  of   the  Am er i can  Col l ege  of
Car di ol ogy /Am er i can  Hear t  Associ ati on  Task   For ce  on  Pr acti ce  Gui del i nes  (Wr i ti ng  Com m i ttee  to
Rev i se  the  1999  gui del i nes  f or   the  m anagem ent  of   pati ents  w i th  acute  m y ocar di al   i nf ar cti on).
Ci r cul ati on  2004;110:588–636.
Keel ey   EC,   Bour a  JA,   Gr i nes  CL.   Pr i m ar y   angi opl asty   v er sus  i ntr av enous  thr om bol y ti c  ther apy   f or
acute  m y ocar di al   i nf ar cti on:  a  quanti tati v e  r ev i ew   of   23  r andom i zed  tr i al s.   Lancet  2003;361:13–20.

Li bby   P.   Cur r ent  concepts  of   the  pathogenesi s  of   the  acute  cor onar y   sy ndr om es.   Ci r cul ati on
2001;104:365–372.

Ver m a  VK,   Hol l enber g  SM.   U pdate  on  acute  cor onar y   sy ndr om es  and  ST­el ev ati on  m y ocar di al
i nf ar cti on.   Cur r   Opi n  Cr i t  Car e  2005;11:401–405.

Unstable Angina and Non–ST­Elevation Myocardial Infarction
1.   What  i s  an  ACS?

2.   What  i s  unstabl e  angi na?

3.   What  i s  a  non–ST­el ev ati on  m y ocar di al   i nf ar cti on  (N STEMI)?

4.   What  com m on  pathophy si ol ogi c  pr ocesses  under l i e  both  unstabl e  angi na  and  N STEMI?

5.   What  i s  the  esti m ated  i nci dence  of   si l ent  i schem i c  epi sodes  i n  the  setti ng  of   unstabl e  angi na?

6.   What  m easur es  hav e  been  show n  to  i m pr ov e  the  cl i ni cal   outcom e  i n  the  setti ng  of   ACSs?

Discussion
1.   What  i s  an  ACS?

The  ACS  spectr um   i ncl udes  unstabl e  angi na,   N STEMI,   and  ST­el ev ati on  m y ocar di al   i nf ar cti on
(STEMI).   Pati ents  hav i ng  ACS  pr esent  to  the  ER  w i th  chest  pai n.   The  pathophy si ol ogy   of   the
sy ndr om e  i s  si m i l ar   i n  these  pati ents.

P. 52
2.   What  i s  unstabl e  angi na?

Stabl e  angi na  i s  a  stabl e  patter n  of   chest  or   ar m   di scom f or t  caused  by   si m i l ar   degr ees  of   phy si cal
or   em oti onal   str ess.   U nstabl e  angi na  i s  angi na  that  occur s  at  r est  or   w i th  m i ni m al   ex er ti on,   or   i s  of
r ecent  onset  (l ess  than  1  m onth)  or   has  a  cr escendo  qual i ty   (i . e. ,   occur s  m or e  f r equentl y ),   and  i s
m or e  sev er e  or   of   l onger   dur ati on.

3.   What  i s  an  N STEMI?

Thi s  def i ni ti on  ev ol v ed  f r om   the  ol d  descr i pti on  of   non–Q­w av e  MI.   It  appl i es  to  pati ents  w ho
hav e  a  pr esentati on  si m i l ar   to  unstabl e  angi na,   especi al l y   w i th  pr ol onged  pai n  at  r est,   and  w ho
hav e  ev i dence  of   m y ocar di al   necr osi s.   They   ar e  di sti nct  f r om   those  w i th  STEMI  because  they   do  not
hav e  per si stent  ST  el ev ati on  on  pr esentati on.   Instead,   ei ther   ST  depr essi on  or   T­w av e  changes  i s
m or e  com m on.   Thi s  cl assi f i cati on  has  ther apeuti c  i m pl i cati ons  because  Q  w av es  do  not  al w ay s
i ndi cate  a  tr ansm ur al   MI  and  subendocar di al   necr osi s  occur s  onl y   i n  50%  of   the  cases  of   non–Q­
w av e  MI.   Ther ef or e,   the  new   cl assi f i cati on  of   STEMI  and  N STEMI  w as  i ntr oduced.   N STEMI  i s
f r equentl y   seen  i n  the  el der l y ,   those  w i th  a  pr ev i ous  MI,   and  com par ed  w i th  STEMI  i s  m or e
com m onl y   associ ated  w i th  i ncom pl ete  occl usi on  of   the  cor onar y   ar ter y .   Thi s  def i ni ti on  al l ow s
pati ents  w i th  STEMI  to  go  ur gentl y   f or   r ev ascul ar i zati on  w hi l e  N STEMI  pati ents  can  of ten  w ai t.   Ear l y
m or tal i ty   i s  hi gher   w i th  STEMI.   A  Q­w av e  or   a  non–Q­w av e  MI  can  be  caused  by   ei ther   STEMI  or
N STEMI.   Pati ents  w i th  STEMI  hav e  hi gher   i n­hospi tal   m or tal i ty   w her eas  those  w i th  N STEMI  hav e
hi gher   r ei nf ar cti on  and  m or tal i ty   r ates  i n  the  subsequent  6  m onths  to  1  y ear .

4.   What  com m on  pathophy si ol ogi c  pr ocesses  under l i e  both  unstabl e  angi na  and  N STEMI?

By   f ar   the  m ost  com m on  m echani sm   of   unstabl e  angi na/N STEMI  i s  ather oscl er osi s­r el ated  cor onar y
pl aque  r uptur e,   usual l y   w i th  super i m posed  thr om bus.   Other   possi bl e  m echani sm s  i ncl ude  cor onar y
spasm   or   i nf l am m ati on  as  w el l   as  i ncr eased  m y ocar di al   ox y gen  r equi r em ents.   These  v ul ner abl e
pl aques  ar e  l i pi d  r i ch  w i th  a  thi n  f i br ous  cap.   Inf i l tr ati on  of   thi s  cap  w i th  i nf l am m ator y   cel l s  l eads
to  i ts  di sr upti on,   f ol l ow ed  by   ex posur e  of   the  subendothel i al   m atr i x   to  the  bl ood  str eam   w i th
pl atel et  acti v ati on  and  aggr egati on.   Thi s  l eads  i ni ti al l y   to  the  f or m ati on  of   a  pl atel et­r i ch  (gr ay )
thr om bus.   Later ,   how ev er ,   ther e  i s  al so  acti v ati on  of   the  coagul ati on  cascade  w i th  f or m ati on  of   a
f i br i n  (r ed)  thr om bus.   In  N STEMI,   thi s  thr om bus  i s  nonoccl usi v e  i n  m ost  cases.

5.   What  i s  the  esti m ated  i nci dence  of   si l ent  i schem i c  epi sodes  i n  the  setti ng  of   unstabl e  angi na?

Si l ent  i schem i a  i s  the  pr esence  of   i schem i c  ECG  changes  w i thout  angi na.   The  r epor ted  i nci dence
af ter   unstabl e  angi na/N STEMI  v ar i es  f r om   one  f our th  to  tw o  thi r ds  of   the  pati ents.   It  i s  detected  by
conti nuous  ECG  m oni tor i ng  usi ng  a  hol ter   m oni tor .   Si l ent  i schem i a  i s  f r equentl y   pr eceded  by
i ncr eases  i n  bl ood  pr essur e  and  hear t  r ate,   causi ng  i ncr eased  m y ocar di al   ox y gen  consum pti on.   It  i s
associ ated  w i th  hi gher   death  and  MI  r ates  and  shoul d  be  tr eated  once  detected.

P. 53
6.   What  m easur es  hav e  been  show n  to  i m pr ov e  the  cl i ni cal   outcom e  i n  the  setti ng  of   ACSs?

Dur i ng  the  acute  phase  of   car e,   the  use  of   aspi r i n,   thi enopy r i di ne  der i v ati v es  (e. g. ,   cl opi dogr el ),
gl y copr otei n  IIb/IIIa  i nhi bi tor s,   unf r acti onated  and  l ow ­m ol ecul ar ­w ei ght  hepar i ns,   and  di r ect
thr om bi n  i nhi bi tor s  (e. g. ,   l epi r udi n)  as  w el l   as  r ev ascul ar i zati on  (especi al l y   i n  hi gh­r i sk   pati ents)
hav e  been  show n  to  r educe  the  r ate  of   MI  and  death.   The  use  of   β­bl ock er s  w as  show n  to  r educe
r ecur r ent  i nf ar cti ons.   The  use  of   cer tai n  cal ci um   channel   bl ock er s  i n  pati ents  i ntol er ant  to  β­
bl ock er s  has  al so  been  show n  to  r educe  r ecur r ent  MI.   In  the  i nter m edi ate  and  l ong  ter m s,   the  use
of   aspi r i n,   cl opi dogr el ,   stati ns,   ACE  i nhi bi tor s,   and  β­bl ock er s  i s  r ecom m ended  to  r educe  m or tal i ty
and  r ecur r ent  MI.   The  concom i tant  use  of   w ar f ar i n  has  been  show n  to  i m pr ov e  outcom es  but  i ts  use
i s  l i m i ted  to  pati ents  w ho  hav e  another   i ndi cati on  f or   w ar f ar i n  (such  as  atr i al   f i br i l l ati on).
Modi f i cati on  of   cor onar y   r i sk   f actor s  i s  al so  w ar r anted.

Case 1
A  42­y ear ­ol d  r egi ster ed  nur se  i s  seen  because  of   pai n  i n  the  chest.   She  descr i bes  a  “pai n  i n  m y
hear tâ€​  and  poi nts  to  a  1­cm 2   ar ea  abov e  the  l ef t  br east.   The  pai n  i s  i ntensi f i ed  by   deep  br eathi ng,
coughi ng,   r ecum bency ,   and  tw i sti ng  m oti ons.   It  has  l asted  conti nuousl y   f or   2  day s.   Thr ee  day s  ago,   she
noted  ex tr em e  f ati gue  and  shor tness  of   br eath  l asti ng  f or   24  hour s.   Fi ndi ngs  f r om   a  com pl ete  phy si cal
ex am i nati on  ar e  nor m al .

1.   What  i s  the  m ost  l i k el y   di agnosi s  i n  thi s  pati ent,   and  w hy ?


As  y ou  ar e  about  to  di schar ge  thi s  pati ent,   her   husband  tel l s  y ou  he  i s  concer ned  about  hi s  w i f e
because  her   si ster   under w ent  cor onar y   by pass  sur ger y   at  44  y ear s  and  her   br other   at  34  y ear s.
Because  the  pai n  has  som e  f eatur es  of   per i car di ti s,   y ou  deci de  to  do  an  ECG.   It  show s  nor m al   si nus
r hy thm   w i th  Q  w av es  i n  the  i nf er i or   l eads  and  di f f use  ST­segm ent  el ev ati on.
2.   What  i s  y our   di agnosi s,   and  w hat  w oul d  y ou  do?

Case Discussion
1.   What  i s  the  m ost  l i k el y   di agnosi s  i n  thi s  pati ent,   and  w hy ?

Chest  w al l   pai n  or   per i car di ti s  w oul d  be  the  m ost  l i k el y   i ni ti al   di agnosi s  i n  thi s  pati ent.   Angi na
pector i s  i s  uncom m on  i n  w om en  i n  thi s  age­gr oup,   and  thi s  pai n  i s  not  angi nal   i n  char acter .   Aor ti c
di ssecti on  pai n  i s  ty pi cal l y   v er y   sev er e  f r om   the  star t,   “shar p,   tear i ngâ€​   and  can  r adi ate  to  the
back .   Acute  chol ecy sti ti s  m ani f ests  cl i ni cal l y   w i th  r i ght  upper   quadr ant  tender ness  and  occasi onal l y
a  pal pabl e  gal l bl adder .   Pneum oni a  and  pl eur i sy   ar e  di f f er enti ated  because  of   the  associ ati on  w i th
f ev er   and  cough  w i th  abnor m al   chest  ex am i nati on.   A  pneum othor ax   i s  associ ated  w i th  acute
shor tness  of   br eath  f i ndi ngs  of   hy per r esonance  to  per cussi on  and  di m i ni shed  br eath  sounds  on  the
af f ected  si de.   Pai n  ar i si ng  f r om   the  chest  w al l   i s  the  m ost  com m on  cause  of   chest  pai n  i n  any   age­
gr oup,   and  of ten  has  no  di scer ni bl e  cause.   It  can  be  r epr oduced  by   pr essur e  ov er   the  pai nf ul   ar ea.
Per i car di ti s  i s  of ten  accom pani ed
P. 54
by   a  f r i cti on  r ub.   Thi s  r ub  has  a  coar se,   “l eather y , â€​
  or   “w al k i ng  on  cr unchy   snow â€​   sound,
w i th  accentuati on  dur i ng  sy stol e  as  w el l   as  ear l y   and  l ate  di astol e  (how ev er ,   som eti m es  onl y   one
or   tw o  com ponents  ar e  audi bl e).   Inspi r ati on  i ntensi f i es  the  r ub.   Its  f eatur es  of ten  i ncl ude  a  pr eci se
l ocal i zati on  and  tender ness  on  pal pati on  ov er   the  af f ected  ar ea.   Deep  br eathi ng,   posi ti on  changes,
and  speci f i c  body   m ov em ents  such  as  tw i sti ng  of ten  accentuate  the  pai n.   Its  dur ati on  v ar i es  f r om   a
f ew   seconds  to  day s.   Ther apy   i s  nonspeci f i c,   consi sti ng  of   r eassur ance  and  si m pl e  anal gesi cs  or
nonster oi dal   dr ugs.

2.   What  i s  y our   di agnosi s,   and  w hat  do  y ou  do?

Thi s  pati ent  pr obabl y   had  a  si l ent  i nf er i or   MI  a  f ew   day s  ago  and  now   pr esents  w i th  posti nf ar cti on
per i car di ti s.   The  Q  w av es  ar e  i nf er i or l y   r el ated  to  the  MI  w hi l e  the  di f f use  ST­segm ent  el ev ati on  i s
com pati bl e  w i th  per i car di ti s.   Addi ti onal   hi stor y   i s  that  she  has  had  ty pe  2  di abetes  m el l i tus  f or   20
y ear s  and  a  r ecent  chol ester ol   scr eeni ng  at  a  heal th  f ai r .   Her   LDL  chol ester ol   w as  242  m g/dL,
consi stent  w i th  f am i l i al   heter ozy gous  hy per chol ester em i a.

The  pati ent  shoul d  be  adm i tted  f or   tel em etr y   obser v ati on.   Tr oponi n  I  i s  l i k el y   to  be  el ev ated  and
shoul d  be  dr aw n.   When  y ou  ask   the  pati ent  to  si t  up,   l ean  f or w ar d,   and  ex hal e,   a  tw o­com ponent
per i car di al   f r i cti on  r ub  i s  noted.   Aspi r i n  shoul d  be  gi v en  f or   her   MI  and  can  be  used  at  m uch  hi gher
doses  to  tr eat  the  concom i tant  per i car di ti s.   Ibupr of en  i s  the  N SAID  of   choi ce  f or   per i car di ti s;
how ev er ,   i ts  use  shoul d  be  av oi ded  i n  the  setti ng  of   an  acute  MI  as  i t  coul d  i nter f er e  w i th  scar
f or m ati on.   The  r em ai nder   of   her   tr eatm ent  i s  as  di scussed  f or   pati ents  w i th  acute  MI,   ex cept  that
thi s  i nf ar cti on  i s  ol der   and  acute  r eper f usi on  i s  not  i ndi cated.   Si l ent  i schem i a  i s  m or e  com m on  i n
di abetes.   Al though  w om en  y ounger   than  50  y ear s  do  not  of ten  hav e  sy m ptom ati c  cor onar y   di sease,
thi s  adv antage  i s  neutr al i zed  by   the  pr esence  of   di abetes.   When  ev al uati ng  pati ents  w i th  chest
pai n,   attenti on  to  CAD  r i sk   f actor s  i s  par am ount.

Case 2
A  57­y ear ­ol d  autom obi l e  sal esm an  w ho  i s  hy per tensi v e  and  a  heav y   ci gar ette  sm ok er   descr i bes  a
pr essur e­l i k e  sensati on  that  dev el oped  f or   the  f i r st  ti m e  3  w eek s  bef or e.   The  di scom f or t,   w hi ch  begi ns
i n  the  r etr oster nal   ar ea,   r adi ates  to  the  l ef t  si de  of   hi s  l ow er   jaw ,   occur s  w hen  he  w al k s  r api dl y   i n  col d
ai r ,   and  m or e  r ecentl y   occur s  at  r est.   Car ef ul   hi stor y   r ev eal s  that  i t  l asts  f or   10  to  15  m i nutes,   but  an
especi al l y   sev er e  epi sode  aw ak ened  hi m   the  ni ght  bef or e  and  l asted  near l y   hal f   an  hour   bef or e  r esol v i ng
spontaneousl y .   Ex cept  f or   a  bl ood  pr essur e  of   150/100  m m   Hg,   the  phy si cal   ex am i nati on  f i ndi ngs  ar e
nor m al .   An  ECG  (obtai ned  af ter   the  pai n  has  di sappear ed)  r ev eal s  deep  and  sy m m etr i c  T­w av e  i nv er si on
i n  l eads  V 1   to  V 4 .   The  pati ent  i s  adm i tted  and  gi v en  IV  hepar i n  and  or al   aspi r i n.

1.   What  i s  y our   di agnosi s?


2.   What  ar e  som e  com m on  phy si cal   f i ndi ngs  dur i ng  an  i schem i c  epi sode?
Appr ox i m atel y   4  hour s  af ter   adm i ssi on,   the  pati ent  agai n  ex per i ences  tr ansi ent  chest  pr essur e.   You
or der   an  ECG.   The  T  w av es  ar e  now   upr i ght  i n  l eads  V 1   to  V 4 .

3.   What  ar e  these  ECG  changes  cal l ed,   and  w hat  do  they   r epr esent?
4.   How   shoul d  the  r ecur r ent  chest  pai n  be  tr eated?
5.   What  shoul d  be  done  nex t?

P. 55

Case Discussion
1.   What  i s  y our   di agnosi s?

Thi s  pati ent  has  ei ther   unstabl e  angi na  or   an  N STEMI.   The  pai n  i s  both  new   i n  onset  and  occur s  at
r est.   The  T­w av e  i nv er si ons  conf i r m   the  di agnosi s  of   i schem i a.   The  r esul ts  of   car di ac  enzy m e  tests
separ ate  unstabl e  angi na  (negati v e  enzy m es)  f r om   N STEMI  (posi ti v e  enzy m es).

2.   What  ar e  som e  com m on  phy si cal   f i ndi ngs  dur i ng  an  i schem i c  attack ?

Incr eases  i n  hear t  r ate  and  bl ood  pr essur e  ar e  the  m ost  com m on  f i ndi ngs  dur i ng  i schem i a.   Phy si cal
ex am i nati on  per f or m ed  dur i ng  an  i schem i c  attack   m ay   r ev eal   an  S 4   or   a  m ur m ur   of   m i tr al
r egur gi tati on.   If   the  i schem i c  ar ea  i s  l ar ge  then  an  S 3 ,   pul m onar y   r al es,   a  dy sk i neti c  api cal
i m pul se,   and  hy potensi on  coul d  be  noted.   Ischem i a  decr eases  l ef t  v entr i cul ar   com pl i ance  (i ncr eased
“sti f f nessâ€​ )   w i th  subsequent  i ncr ease  i n  l ef t  v entr i cul ar   f i l l i ng  pr essur e.   The  r esi stance  to
f i l l i ng  dur i ng  atr i al   contr acti on  i s  w hat  pr oduces  the  S 4   sound.   How ev er ,   an  S 4   i s  a  nonspeci f i c
f i ndi ng  and  i s  f r equentl y   hear d  i n  ol der   adul ts.   Local i zed  contr acti on  abnor m al i ti es  m ay   pr oduce
tr ansi ent  papi l l ar y   m uscl e  dy sf uncti on  and  f ai l ur e  of   com pl ete  apposi ti on  of   the  l eaf l ets,   r esul ti ng
i n  m i tr al   r egur gi tati on.   Si m i l ar   contr acti on  abnor m al i ti es  can  cause  an  outw ar d  bul ge  of   the  l ef t
v entr i cl e  w i th  dy sk i neti c  api cal   i m pul se.   Thi s  can  be  f el t  by   usi ng  the  pal m   of   the  hand  w hi l e  the
pati ent  i s  i n  the  l ef t  l ater al   decubi tus  posi ti on.

3.   What  ar e  these  ECG  changes  cal l ed,   and  w hat  do  they   r epr esent?

When  pr ev i ousl y   i nv er ted  T  w av es  becom e  upr i ght  i n  the  pr esence  of   chest  pai n,   i t  i s  cal l ed
pseudonor m al i zati on.   Thi s  i s  str ongl y   suggesti v e  of   i schem i a.
4.   How   shoul d  the  r ecur r ent  chest  pai n  be  tr eated?

The  pseudonor m al i zati on  of   the  T  w av es  cl ear l y   i ndi cates  m y ocar di al   i schem i a.   Thi s  pai n  shoul d  be
tr eated  w i th  subl i ngual   N TG  and  IV  m or phi ne,   f ol l ow ed  by   IV  N TG  and  β­bl ock ade  i f   ther e  ar e  no
contr ai ndi cati ons.   The  pati ent  i s  al r eady   r ecei v i ng  aspi r i n  and  hepar i n  (l ow ­m ol ecul ar ­w ei ght
hepar i n  can  al so  be  used  i n  thi s  case).   Pl atel et  gl y copr otei n  IIb/IIIa  i nhi bi tor s  ar e  of   v al ue  i n  the
tr eatm ent  of   hi gh­r i sk   pati ents  w i th  unstabl e  angi na/N STEMI.   Stati ns  hav e  been  show n  to  r educe
car di ac  ev ent  r ates  w hen  used  acutel y   i n  thi s  popul ati on.   If   the  pati ent  i s  not  a  candi date  f or
cor onar y   ar ter y   by pass  gr af t  (CABG),   then  cl opi dogr el   shoul d  be  consi der ed.

5.   What  shoul d  be  done  nex t?

Thi s  pati ent  had  si gns  of   m y ocar di al   i schem i a  on  adm i ssi on  w i th  r ecur r ent  pai n  on  IV  hepar i n.   Thi s
i s  suspi ci ous  f or   the  pr esence  of   substanti al   i schem i a,   and  the  deepl y   i nv er ted  T  w av es  i n  l eads  V 1
to  V 4   m ostl y   l i k el y   r epr esent  a  hi gh­gr ade  pr ox i m al   LAD  stenosi s.   Thi s  pati ent  shoul d  under go
cor onar y   angi ogr aphy .   PCI  shoul d  be  per f or m ed  i n  gener al   f or   one­  or   tw o­v essel   di sease  and
nor m al   or   near ­nor m al   l ef t  v entr i cul ar   f uncti on.   For   m ost  pati ents  w i th  thr ee­v essel   di sease  or   l ef t
v entr i cul ar   dy sf uncti on,   especi al l y   i n  the  pr esence  of   di abetes,   cor onar y   ar ter y   by pass  sur ger y
(usi ng  w henev er   possi bl e  an  i nter nal   m am m ar y   ar ter y   gr af t  to  the  LAD)  i s  i ndi cated.   The  use  of   PCI
v er sus  cor onar y   by pass  sur ger y   m ay   v ar y   dependi ng  on  pati ent  or   phy si ci an  pr ef er ence,   l esi on
anatom y ,   the  pr esence  of   pr ox i m al   LAD  di sease,   or   the  pati ent's  com or bi di ti es.

P. 56

Suggested Readings
Antm ann  EM,   Cohen  M,   Ber ni nk   PJLM,   etal .   The  TIMI  r i sk   scor e  f or   unstabl e  angi na/non­ST  el ev ati on
MI.   A  m ethod  f or   pr ognosti cati on  and  ther apeuti c  deci si on  m ak i ng.   JAMA  2000;284:835–842.

Boden  WE,   McKay   RG.   Opti m al   tr eatm ent  of   acute  cor onar y   sy ndr om es­  an  ev ol v i ng  str ategy .   N   Engl
J  Med  2001;344:1939–1942.

Br aunw al d  E,   Antm an  EM,   Beasl ey   JW,   etal .   Am er i can  Col l ege  of   Car di ol ogy /Am er i can  Hear t
Associ ati on  Task   For ce  on  Pr acti ce  Gui del i nes  (Com m i ttee  on  the  Managem ent  of   Pati ents  Wi th
U nstabl e  Angi na).   ACC/AHA  gui del i ne  update  f or   the  m anagem ent  of   pati ents  w i th  unstabl e  angi na
and  non­ST­segm ent  el ev ati on  m y ocar di al   i nf ar cti on—2002:  sum m ar y   ar ti cl e:  a  r epor t  of   the
Am er i can  Col l ege  of   Car di ol ogy /Am er i can  Hear t  Associ ati on  Task   For ce  on  Pr acti ce  Gui del i nes
(Com m i ttee  on  the  Managem ent  of   Pati ents  Wi th  U nstabl e  Angi na).   Ci r cul ati on  2002;106:1893â
€“1900.

Cannon  CP,   Wei ntr aub  WS,   Dem opoul os  LA,   etal .   Com par i son  of   ear l y   i nv asi v e  and  conser v ati v e
str ategi es  i n  pati ents  w i th  unstabl e  cor onar y   sy ndr om es  tr eated  w i th  the  gl y copr otei n  IIb/IIIa
i nhi bi tor   ti r of i ban.   For   the  TACTICS­thr om bol y si s  i n  m y ocar di al   i nf ar cti on  18  i nv esti gator s.   N   Engl   J
Med  2001;344:1879–1887.

Cohn  PF,   Fox   KM,   Dal y   C.   Si l ent  m y ocar di al   i schem i a.   Ci r cul ati on  2003;108:1263.

Fuster   V,   Mor eno  PR,   Fay ad  ZA,   etal .   Ather othr om bosi s  and  the  hi gh­r i sk   pl aque.   Par t  I:  ev ol v i ng
concepts.   J  Am   Col l   Car di ol   2005;46:937–954.

Sudden Cardiac Death
1.   What  k i nd  of   hear t  di sease  i s  seen  m ost  com m onl y   i n  adul ts  w ho  di e  suddenl y ?  In  y oung  athl etes?

2.   Whi ch  ty pes  of   ar r hy thm i as  ar e  associ ated  w i th  car di ac  ar r est  and  sudden  car di ac  death  (SCD)?

3.   Whi ch  pati ents  ar e  at  hi ghest  r i sk   f or   SCD?

4.   What  i s  the  cause  of   SCD  i n  the  l ong  QT  sy ndr om e?


Discussion
1.   What  k i nd  of   hear t  di sease  i s  seen  m ost  com m onl y   i n  adul ts  w ho  di e  suddenl y ?  In  y oung  athl etes?

Appr ox i m atel y   90%  of   cases  of   SCD  ar e  due  to  v entr i cul ar   f i br i l l ati on  i n  the  setti ng  of   pr eex i sti ng
str uctur al   hear t  di sease;  5%  to  10%  occur   i n  the  absence  of   or gani c  hear t  di sease.   In  i ndi v i dual s
y ounger   than  30  y ear s  and  y oung  athl etes,   SCD  i s  v er y   r ar e,   but  w hen  i t  does  occur   i t  i s  usual l y
due  to  hy per tr ophi c  car di om y opathy .   Ar r hy thm ogeni c  r i ght  v entr i cul ar   dy spl asi a  and  acute
m y ocar di ti s  ar e  other   i nf r equent  causes  of   SCD  i n  the  y oung  adul t.   Af ter   the  age  of   40,   65%  to  70%
of   al l   SCDs  ar e  attr i butabl e  to  CAD.

P. 57
2.   Whi ch  ty pes  of   ar r hy thm i as  ar e  associ ated  w i th  car di ac  ar r est  and  SCD?

In  the  f i el d,   par am edi cs  m ost  com m onl y   r ecor d  v entr i cul ar   f i br i l l ati on  or   v entr i cul ar   tachy car di a
dur i ng  car di ac  ar r est.   Less  f r equentl y   seen,   and  associ ated  w i th  a  poor er   pr ognosi s,   ar e
br ady ar r hy thm i as,   asy stol e,   and  pul sel ess  el ectr i cal   acti v i ty   (el ectr i cal –m echani cal   di ssoci ati on).

Car di ac  ar r est  due  to  any   ar r hy thm i a  r esul ts  i n  r api d  depl eti on  of   ox y gen  i n  v i tal   or gans.   Af ter   6
m i nutes,   br ai n  dam age  i s  ex pected  to  occur ,   ex cept  i n  cases  of   hy pother m i a.   Ther ef or e,   ear l y   CPR
and  r api d  adv anced  car di ac  l i f e  suppor t  (ACLS),   w i th  def i br i l l ati on,   ar e  essenti al   i n  i m pr ov i ng
sur v i v al   and  neur ol ogi c  r ecov er y .

3.   Whi ch  pati ents  ar e  at  hi ghest  r i sk   f or   SCD?

Pati ents  at  hi ghest  r i sk   f or   SCD  ar e  those  w ho  hav e  pr ev i ousl y   sur v i v ed  an  epi sode  of   SCD  or   hav e
a  hi stor y   of   r api d,   sustai ned  v entr i cul ar   tachy car di a  especi al l y   i n  the  setti ng  of   r educed  l ef t
v entr i cul ar   f uncti on.   One  of   the  best  m easur es  of   r i sk   of   SCD  i s  l ef t  v entr i cul ar   f uncti on  and  the
r i sk   of   SCD  i ncr eases  as  l ef t  v entr i cul ar   f uncti on  decr eases.

The  i nci dence  of   SCD  i s  gr eater   i n  m en  than  i n  w om en.   Thi s  i ncr eased  r i sk   r em ai ns  despi te
adjusti ng  f or   the  pr esence  of   com or bi di ti es  such  as  i schem i c  hear t  di sease  and  age.   The  r i sk   of   SCD
i ncr eases  w i th  age.

4.   What  i s  the  cause  of   SCD  i n  the  l ong  QT  sy ndr om e?

The  l ong  QT  sy ndr om e  i s  a  cause  of   sy ncope  and  SCD  i n  pati ents  w i th  str uctur al l y   nor m al   hear ts,
but  m ay   al so  r esul t  i n  SCD  i n  pati ents  w i th  str uctur al   hear t  di sease.   The  l ong  QT  sy ndr om e  can  be
ei ther   acqui r ed  or   i nher i ted.   Sev er al   geneti c  def ects  i nv ol v i ng  car di ac  i on  channel s  hav e  been
i denti f i ed  i n  f am i l i es  w i th  i nher i ted  l ong  QT  sy ndr om e.   In  acqui r ed  l ong  QT  sy ndr om e,   sev er al
cl asses  of   dr ugs  that  af f ect  car di ac  i on  channel s  and  sev er al   m edi cal   condi ti ons  associ ated  w i th
el ectr ol y te  abnor m al i ti es  hav e  been  i denti f i ed.   In  both  i nher i ted  and  acqui r ed  f or m s,   car di ac
r epol ar i zati on  i s  pr ol onged,   and  r ef l ected  i n  a  l ong  QT  i nter v al   on  the  ECG.   Sy ncope  and  SCD  i n
l ong  QT  sy ndr om e  ar e  caused  by   a  speci f i c,   pol y m or phi c  v entr i cul ar   tachy car di a  cal l ed  tor sade  de
poi ntes  (â€​ t w i sti ng  of   the  poi ntsâ€​ ) .   Dr ugs  that  pr ol ong  the  QT  i nter v al   (e. g. ,   phenothi azi nes,
tr i cy cl i c  anti depr essants,   and  cer tai n  anti ar r hy thm i cs)  ar e  par ti cul ar l y   l i k el y   to  cause  SCD  i n
pati ents  w i th  another   cause  of   pr ol onged  QT  i nter v al   or   i n  pati ents  w i th  str uctur al   hear t  di sease.

Case
A  65­y ear ­ol d  m an  com pl ai ns  of   chest  di scom f or t  on  the  gol f   cour se  and  w i thi n  seconds  col l apses  and  i s
unr esponsi v e.   Hi s  com pani ons  i ni ti ate  by stander   CPR  and  an  am bul ance  i s  cal l ed.   Par am edi cs  ar r i v e
w i thi n  10  m i nutes.   A  “qui ck   l ook â€​   at  the  r hy thm   usi ng  the  def i br i l l ator   paddl es  r ev eal s  v entr i cul ar
f i br i l l ati on.   Af ter   one  shock   at  200  J  usi ng  a  bi phasi c  f i br i l l ator ,   si nus  r hy thm   i s  r estor ed  and  a  pul se  i s
f el t.   The  pati ent  i s  tr anspor ted  to  the  hospi tal .   Ini ti al   ECG  show s  Q  w av es  i n  the  pr ecor di al   l eads,   and
di f f use,   nonspeci f i c  ST­segm ent  and  T­w av e  abnor m al i ti es,   w i th  a  nor m al   QT  i nter v al .   Ser um
el ectr ol y tes  ar e
P. 58
nor m al .   Ini ti al   and  subsequent  car di ac  enzy m e  deter m i nati ons  do  not  i ndi cate  ev i dence  of   an  acute  MI.
Fam i l y   m em ber s  state  that  the  pati ent  w as  not  on  car di ac  m edi cati ons  and  had  no  car di ac  hi stor y .   The
pati ent,   i ni ti al l y   unr esponsi v e  and  r equi r i ng  m echani cal   v enti l ati on,   r ecov er s  neur ol ogi cal l y   ov er   the
nex t  48  hour s  and  i s  ex tubated.   Apar t  f r om   a  m i l d  shor t­ter m   m em or y   def i ci t,   he  seem s  to  be  back   to
hi s  usual   sel f   and  none  the  w or se  f or   the  ex per i ence.   Tr oponi n  w as  not  el ev ated  dur i ng  the
hospi tal i zati on.

1.   What  tests  shoul d  be  per f or m ed  on  thi s  pati ent  now ?
2.   Wi l l   cor onar y   r ev ascul ar i zati on  be  of   benef i t  i n  pr ev enti ng  r ecur r ent  SCD  i n  thi s  pati ent?
3.   Is  ther e  a  r ol e  f or   el ectr ophy si ol ogi c  testi ng  i n  thi s  pati ent?
4.   What  i s  the  best  tr eatm ent  to  pr ev ent  r ecur r ent  SCD  i n  thi s  pati ent?

Case Discussion
1.   What  tests  shoul d  be  per f or m ed  on  thi s  pati ent  now ?

Despi te  the  near ­m i r acul ous  r ecov er y   of   thi s  pati ent,   hi s  r i sk   of   r ecur r ent  SCD  i s  hi gh  and
m easur es  shoul d  be  tak en  to  i denti f y   the  cause  of   SCD  i n  hi s  case  and  pr ev ent  r ecur r ence.   Al though
m ost  cases  of   SCD  i n  hi s  age­gr oup  ar e  r el ated  to  CAD,   i t  can  be  uncl ear   i n  a  par ti cul ar   pati ent
w hether   the  i ni ti ati ng  ev ent  w as  a  pr i m ar y   ar r hy thm i a  or   i schem i a.   Ischem i a  can  be  due  to
i ncr eased  m etabol i c  dem ands  (e. g. ,   ex er ci se)  i n  the  f ace  of   a  f i x ed  cor onar y   obstr ucti on,   or   due  to
tr ansi ent  decr eased  cor onar y   bl ood  f l ow   caused  by   ather oscl er oti c  pl aque  r uptur e  or   cor onar y
v asospasm .   It  i s  l i k el y   that  m any   epi sodes  of   SCD  ar e  m ul ti f actor i al ,   super i m posi ng  tr ansi ent
tr i gger i ng  ev ents  (e. g. ,   i schem i a,   changes  i n  autonom i c  tone,   el ectr ol y te  abnor m al i ti es,   or
pr em atur e  v entr i cul ar   com pl ex es)  on  an  ar r hy thm ogeni c  substr ate  such  as  the  cel l   dam age  cr eated
by   a  pr ev i ous  MI.   Thi s  pati ent's  com pl ai nt  of   chest  di scom f or t  bef or e  col l apse  m ay   on  f i r st
consi der ati on  suggest  i schem i a  as  the  i ni ti ati ng  ev ent,   but  pati ents  w i th  cor onar y   di sease  w ho  hav e
v entr i cul ar   tachy car di a  som eti m es  com pl ai n  of   chest  pai n  because  they   becom e  i schem i c  secondar y
to  the  r api d  hear t  r ate.   The  ECG  ev i dence  of   an  anter i or   i nf ar cti on  w i thout  the  enzy m e  changes
char acter i sti c  of   acute  i nf ar cti on  suggests  that,   despi te  hi s  negati v e  car di ac  hi stor y ,   he  m ay   hav e
had  a  pr ev i ous  “si l entâ€​   MI.   Thi s  ol d  v entr i cul ar   scar   m ay   be  a  substr ate  f or   a  pr i m ar y
r eentr ant  v entr i cul ar   tachy car di a.   To  def i ne  hi s  car di ac  di sease  better ,   i ncl udi ng  hi s  l ef t  v entr i cul ar
f uncti on,   and  to  deter m i ne  i f   he  has  a  substr ate  f or   r ecur r ent  i schem i a,   car di ac  catheter i zati on
shoul d  be  per f or m ed.   In  a  Fr ench  study   of   84  sur v i v or s  of   out­of ­hospi tal   car di ac  ar r est  w i thout
obv i ous  noncar di ac  cause  of   the  ar r est,   i m m edi ate  cor onar y   angi ogr aphy   w i th  angi opl asty   w as
show n  to  be  saf e  w i th  potenti al   l ong­ter m   benef i t  w hen  per f or m ed  by   an  ex per i enced  team .   Car di ac
catheter i zati on  i n  thi s  pati ent  show ed  a  100%  pr ox i m al   LAD  ar ter y   occl usi on  as  w el l   as  a  90%
occl usi on  of   the  f i r st  obtuse  m ar gi nal   br anch  of   the  l ef t  ci r cum f l ex   ar ter y .   The  r i ght  cor onar y
ar ter y   w as  nor m al .   The  l ef t  v entr i cul ar   EF  w as  r educed  at  30%  (nor m al ≥55%).   Ther e  w as  an
anter oapi cal   l ef t  v entr i cul ar   aneur y sm .   The  l ater al   w al l   of   the  l ef t  v entr i cl e  (suppl i ed  by   the  l ef t
ci r cum f l ex   ar ter y )  had  nor m al   m oti on.

P. 59
2.   Wi l l   cor onar y   r ev ascul ar i zati on  be  of   benef i t  i n  pr ev enti ng  r ecur r ent  SCD  i n  thi s  pati ent?

As  di scussed  pr ev i ousl y ,   SCD  m ay   be  m ul ti f actor i al   and  i t  i s  di f f i cul t  to  deter m i ne  the  pr eci se
tr i gger s  f or   an  SCD  epi sode.   Thi s  pati ent  has  ev i dence  on  catheter i zati on  of   a  pr ev i ous  anter i or
i nf ar cti on,   w i th  an  anter oapi cal   l ef t  v entr i cul ar   aneur y sm .   Thi s  aneur y sm   m ay   be  a  substr ate  f or
r eentr ant­sustai ned  m onom or phi c  v entr i cul ar   tachy car di a.   In  addi ti on,   he  has  a  si gni f i cant  stenosi s
i n  an  obtuse  m ar gi nal   ar ter y ,   w i th  nor m al   l ef t  v entr i cul ar   w al l   m oti on  i n  the  r egi on  ser v ed  by   thi s
ar ter y .   Thi s  i s  a  substr ate  f or   i schem i a.   In  an  attem pt  to  cor r ect  possi bl e  tr i gger i ng  f actor s  l i k e
i schem i a,   i t  w oul d  be  r easonabl e  to  di l ate  the  obtuse  m ar gi nal   stenosi s  w i th  bal l oon  angi opl asty ,
and  thi s  w as  per f or m ed  i n  thi s  pati ent.   U nf or tunatel y ,   i n  pati ents  w i th  r educed  l ef t  v entr i cul ar
f uncti on  w ho  hav e  an  abor ted  epi sode  of   SCD,   ther e  i s  no  ev i dence  that  anti i schem i a  m easur es
al one  pr ev ent  r ecur r ent  SCD.

3.   Is  ther e  a  r ol e  f or   el ectr ophy si ol ogi c  testi ng  i n  thi s  pati ent?

Most  pati ents  w i th  si m i l ar   pr esentati ons  hav e  i nduci bl e  sustai ned  m onom or phi c  v entr i cul ar
tachy car di a  dur i ng  el ectr ophy si ol ogi c  testi ng.   In  the  past,   m any   such  pati ents  under w ent
el ectr ophy si ol ogi c  testi ng  and  w er e  then  tr eated  w i th  anti ar r hy thm i c  dr ug  ther apy   gui ded  by   ser i al
el ectr ophy si ol ogi c  testi ng.   Pati ents  w ho  f ai l ed  dr ug  ther apy   or   w ho  di d  not  hav e  i nduci bl e  sustai ned
m onom or phi c  v entr i cul ar   tachy car di a  w er e  tr eated  w i th  em pi r i c  am i odar one  or   an  i m pl antabl e
def i br i l l ator .   A  num ber   of   r andom i zed  contr ol l ed  cl i ni cal   tr i al s  hav e  been  per f or m ed  that  com par e
the  ef f i cacy   of   the  i m pl antabl e  def i br i l l ator   w i th  anti ar r hy thm i c  dr ug  ther apy .   Al l   these  tr i al s
suggest  that  pr ophy l acti c  ther apy   w i th  i m pl antabl e  def i br i l l ator   i s  super i or   to  anti ar r hy thm i c  dr ug
ther apy   gui ded  by   el ectr ophy si ol ogi c  testi ng  or   em pi r i c  am i odar one  i n  pr ev enti ng  r ecur r ent  SCD.   In
the  AVID  (Anti ar r hy thm i cs  Ver sus  Im pl antabl e  Def i br i l l ator s)  tr i al ,   pati ents  w ho  w er e  enr ol l ed  had
hem ody nam i cal l y   si gni f i cant  sustai ned  v entr i cul ar   tachy car di a  and  a  l ef t  v entr i cul ar   EF  of   40%  or
l ess,   or   v entr i cul ar   f i br i l l ati on.   Ther e  w as  a  31%  r educti on  i n  the  total   m or tal i ty   r ate  af ter   3  y ear s
w i th  i m pl antabl e  def i br i l l ator s  com par ed  w i th  anti ar r hy thm i c  dr ug  ther apy .   The  tr i al   w as  stopped
ear l y   w hen  a  sur v i v al   benef i t  w as  noted  i n  pati ents  r ecei v i ng  the  ICD  com par ed  w i th  those  tr eated
w i th  am i odar one  or   sotal ol .   The  MADIT  II  (Mul ti center   Autom ati c  Def i br i l l ator   Im pl antati on  Tr i al )
dem onstr ated  that  ICDs  si gni f i cantl y   i m pr ov e  sur v i v al   i n  pati ents  w i th  CAD  and  l ef t  v entr i cul ar   EF
of   30%  or   l ess.   In  those  pati ents  w ho  hav e  sur v i v ed  SCD  and/or   hav e  r educed  LV  f uncti on,   an  ICD
i s  the  tr eatm ent  of   choi ce.   El ectr ophy si ol ogi c  testi ng  i s  usual l y   not  per f or m ed  i n  SCD  sur v i v or s,
and  w as  not  per f or m ed  i n  thi s  pati ent.

4.   What  i s  the  best  tr eatm ent  to  pr ev ent  r ecur r ent  SCD  i n  thi s  pati ent?

Thi s  pati ent  w as  a  good  candi date  f or   an  i m pl antabl e  def i br i l l ator ,   and  he  r ecei v ed  one.   The
def i br i l l ator   w as  i m pl anted  i n  the  l ef t  pector al   r egi on  i n  the  el ectr ophy si ol ogy   l abor ator y ,   usi ng
l ocal   anesthesi a  and  consci ous  sedati on.   The  pati ent  w as  di schar ged  hom e  the  day   af ter   the
i m pl ant.   In  addi ti on,   thi s  pati ent  r ecei v ed  other   m edi cal   ther apy   that  has  been  show n  to  r educe  the
r i sk   of   SCD.   β­Bl ock i ng  dr ugs  hav e  been  show n  to  r educe  total   m or tal i ty   af ter   MI  as  w el l   as
i m pr ov e  pum p  f uncti on  i n  som e  cases.   Aspi r i n  m ay   hel p  pr ev ent  r ei nf ar cti on.   ACE  i nhi bi tor s  hav e
P. 60
been  show n  to  i m pr ov e  sur v i v al   i n  pati ents  w i th  r educed  l ef t  v entr i cul ar   f uncti on.   Spi r onol actone,   a
m i ner al ocor ti coi d  r eceptor   antagoni st  has  been  si m i l ar l y   show n  to  r educe  al l   cause  m or tal i ty   and
SCD  i n  pati ents  w i th  l ef t  v entr i cul ar   sy stol i c  f uncti on  of   35%  or   l ess.   Li pi d­l ow er i ng  agents  m ay
pr ev ent  pr ogr essi on  of   ather oscl er osi s  and  shoul d  be  used  i n  pati ents  w i th  l i pi d  abnor m al i ti es.
Other   r i sk   f actor   m odi f i cati ons  such  as  sm ok i ng  cessati on  w oul d  be  r ecom m ended.   Am i odar one  m ay
r educe  SCD  af ter   MI,   but  i t  does  not  cl ear l y   r educe  total   m or tal i ty .   In  the  SCD­HeFT  study ,
am i odar one  w as  show n  to  be  not  benef i ci al   i n  pr ev enti ng  SCDs  i n  pati ents  w i th  a  l ef t  v entr i cul ar
sy stol i c  f uncti on  of   35%  or   l ess  com par ed  w i th  the  pl acebo  gr oup  i n  both  i schem i c  and  noni schem i c
pati ents.   In  thi s  pati ent,   ther e  w oul d  be  no  added  v al ue  i n  usi ng  am i odar one  because  he  al r eady
has  an  i m pl antabl e  def i br i l l ator .   In  f act,   am i odar one,   by   i ts  ef f ect  of   i ncr easi ng  the  el ectr i cal
def i br i l l ati on  thr eshol d,   m i ght  actual l y   i nter f er e  w i th  the  f uncti on  of   the  def i br i l l ator ,   and  shoul d
ther ef or e  be  av oi ded.   How ev er ,   because  ICDs  do  not  pr ev ent  ar r hy thm i as,   pati ents  w ho  hav e
f r equent  dev i ce  di schar ges  f r om   r ecur r ent  ar r hy thm i as  m ay   benef i t  f r om   adjuncti v e  anti ar r hy thm i c
dr ug  ther apy ,   l i k e  am i odar one.   Such  tr eatm ent,   by   r educi ng  the  f r equency   of   appr opr i ate  shock s,
i m pr ov es  the  pati ent's  qual i ty   of   l i f e.

Suggested Readings
The  Anti ar r hy thm i cs  Ver sus  Im pl antabl e  Def i br i l l ator s  (AVID)  Inv esti gator s.   A  com par i son  of
anti ar r hy thm i c  dr ug  ther apy   w i th  i m pl antabl e  def i br i l l ator s  i n  pati ents  r esusci tated  f r om   near   f atal
v entr i cul ar   ar r hy thm i as.   N   Engl   J  Med  1997;337:1576.

Bar dy   GH,   Lee  KL,   Mar k   DB,   etal .   Am i odar one  or   an  i m pl antabl e  car di ov er ter ­def i br i l l ator   f or
congesti v e  hear t  f ai l ur e.   N   Engl   J  Med  2005;352:225.

Hui k ur i   HV,   Castel l anos  A,   My er bur g  RJ.   Sudden  death  due  to  car di ac  ar r hy thm i as.   N   Engl   J  Med
2001;345:1473–1482.

Mar on  BJ,   Shi r ani   J,   Pol i ac  LC,   etal .   Sudden  death  i n  y oung  com peti ti v e  athl etes:  cl i ni cal ,
dem ogr aphi c,   and  pathol ogi cal   pr of i l es.   JAMA  1996;276(3):199–204.

Moss  AJ,   Long  QT.   Sy ndr om e.   JAMA  2003;289:2041–2044.

Moss  AJ,   Zar eba  W,   Hal l   WJ,   etal .   Pr ophy l acti c  i m pl antati on  of   a  def i br i l l ator   i n  pati ents  w i th
m y ocar di al   i nf ar cti on  and  r educed  ejecti on  f r acti on.   N   Engl   J  Med  2002;346:877.

Spaul di ng  CM,   Jol y   LM,   Rosenber g  A,   etal .   Im m edi ate  cor onar y   angi ogr aphy   i n  sur v i v or s  of   out­of ­
hospi tal   car di ac  ar r est.   N   Engl   J  Med  1997;336:1629.

Zheng  ZJ,   Cr of t  JB,   Gi l es  WH,   etal .   Sudden  car di ac  death  i n  the  U ni ted  States,   1989  to  1998.
Ci r cul ati on  2001;104:2158.

Valvular Heart Disease
1.   What  i s  the  di f f er ence  betw een  v al v ul ar   i nsuf f i ci ency   and  v al v ul ar   r egur gi tati on?

2.   What  ty pes  of   m y ocar di al   hy per tr ophy   can  r esul t  f r om   v al v ul ar   abnor m al i ti es?

3.   What  i s  the  m ost  ser i ous  l ong­ter m   consequence  of   ei ther   concentr i c  or   eccentr i c  hy per tr ophy ?

P. 61
4.   What  i s  the  r el ati onshi p  betw een  the  pr essur e  gr adi ent  acr oss  a  stenoti c  v al v e,   the  bl ood  f l ow
acr oss  the  v al v e,   and  the  v al v e  ar ea?

Discussion
1.   What  i s  the  di f f er ence  betw een  v al v ul ar   i nsuf f i ci ency   and  v al v ul ar   r egur gi tati on?

Regur gi tati on  and  i nsuf f i ci ency   ar e  i nter changeabl e  ter m s  to  descr i be  back w ar d  f l ow   of   bl ood
acr oss  a  v al v e  at  a  ti m e  i n  the  car di ac  cy cl e  w hen  ther e  w oul d  be  no  si gni f i cant  f l ow   acr oss  a
com petent  v al v e.

2.   What  ty pes  of   m y ocar di al   hy per tr ophy   can  r esul t  f r om   v al v ul ar   abnor m al i ti es?

When  ther e  i s  a  pr essur e  l oad  i m posed  on  the  v entr i cl e  (such  as  aor ti c  stenosi s  f or   the  l ef t
v entr i cl e  or   pul m oni c  stenosi s  f or   the  r i ght  v entr i cl e),   concentr i c  hy per tr ophy   dev el ops.   Concentr i c
hy per tr ophy   m eans  that  the  m y ocar di al   w al l   thi ck ness  i s  i ncr eased  w i th  a  nor m al   or   decr eased
i nter nal   v entr i cul ar   di am eter .   A  v ol um e  l oad  (such  as  aor ti c  i nsuf f i ci ency   or   m i tr al   r egur gi tati on  f or
the  l ef t  v entr i cl e  or   tr i cuspi d  r egur gi tati on  f or   the  r i ght  v entr i cl e)  r esul ts  i n  eccentr i c  hy per tr ophy ;
the  w al l   thi ck ness  i s  nor m al   but  the  i nter nal   di am eter   of   the  v entr i cl e  i s  i ncr eased.   Ov er al l ,   l ef t
v entr i cul ar   m ass  i s  i ncr eased  i n  both  ty pes  of   hy per tr ophy .

3.   What  i s  the  m ost  ser i ous  l ong­ter m   consequence  of   ei ther   concentr i c  or   eccentr i c  hy per tr ophy ?

Wi th  l ong­standi ng  hy per tr ophy   of   ei ther   ty pe,   m y ocar di al   dy sf uncti on  m ay   occur   r esul ti ng  i n  HF.

4.   What  i s  the  r el ati onshi p  betw een  the  pr essur e  gr adi ent  acr oss  a  stenoti c  v al v e,   the  bl ood  f l ow
acr oss  the  v al v e,   and  the  v al v e  ar ea?

The  pr essur e  gr adi ent  acr oss  a  v al v e  i s  pr opor ti onal   to  the  bl ood  f l ow   acr oss  the  v al v e  di v i ded  by
the  v al v e  ar ea.   The  pr essur e  gr adi ent  i s  a  r esul t  of   the  f l ow   acr oss  the  stenoti c  v al v e  and  the
degr ee  of   stenosi s.   For   ex am pl e,   i f   f l ow   r em ai ns  the  sam e  and  the  v al v e  becom es  m or e  stenoti c
ov er   ti m e,   the  pr essur e  gr adi ent  i ncr eases.   How ev er ,   an  i ncr eased  pr essur e  gr adi ent  m ay   not
al w ay s  m ean  that  stenosi s  has  pr ogr essed.   For   ex am pl e,   bl ood  f l ow   m ay   i ncr ease  w i th  f ev er   or
anem i a,   r esul ti ng  i n  an  i ncr eased  pr essur e  gr adi ent  w i thout  any   change  i n  v al v e  ar ea.   On  the  other
hand,   a  decr ease  i n  the  pr essur e  gr adi ent  does  not  m ean  an  i m pr ov em ent  i n  v al v e  stenosi s.   For
ex am pl e,   i f   m y ocar di al   f uncti on  deter i or ates  and  bl ood  f l ow   (car di ac  output)  decr eases,   the
pr essur e  gr adi ent  decr eases.   Thi s  decr ease,   how ev er ,   does  not  r ef l ect  a  l ess  stenoti c  v al v e,   but
i ndi cates  a  deter i or ati on  i n  m y ocar di al   f uncti on  because  the  hear t  can  no  l onger   pum p  the  sam e
bl ood  f l ow .

Case 1
A  pr ev i ousl y   heal thy   but  i nacti v e  42­y ear ­ol d  m an  i s  seen  i n  the  ER  af ter   a  f i r st  epi sode  of   sy ncope,
w hi ch  occur r ed  w hi l e  he  w as  pl ay i ng  bask etbal l .   On  questi oni ng,   he  descr i bes  a  2­m onth  hi stor y   of
ex er ti onal   chest  pai n.   He  has  not  seen  a  phy si ci an  dur i ng  hi s  adul t  l i f e.   Phy si cal   ex am i nati on  r ev eal s  the
f ol l ow i ng  f i ndi ngs.   Hi s  supi ne  bl ood  pr essur e  i s  116/80  m m   Hg  w i thout  any   si gni f i cant  or thostati c
change.   Ther e  i s  no  jugul ar   v enous
P. 62
di stenti on,   but  ther e  ar e  sl ow l y   r i si ng,   sm al l ­am pl i tude,   and  som ew hat  sustai ned  car oti d  pul ses.   Hi s
l ungs  ar e  cl ear .   A  sustai ned  and  sl i ghtl y   l ater al l y   di spl aced  apex   i m pul se  i s  noted,   as  w el l   as  a  sof t  f i r st
hear t  sound  and  a  si ngl e  second  hear t  sound,   a  pr om i nent  f our th  hear t  sound,   and  a  gr ade  3/6  har sh,
l ate­peak i ng,   cr escendo–decr escendo  sy stol i c  m ur m ur   hear d  best  at  the  car di ac  base  and  r adi ati ng  to
the  car oti ds  w i th  a  hi gh­f r equency   com ponent  at  the  car di ac  apex .   N o  cl ubbi ng,   cy anosi s,   or   edem a  i s
noted.

1.   What  i s  the  m ost  l i k el y   v al v ul ar   l esi on  i n  thi s  pati ent?


2.   What  i s  the  m ost  l i k el y   cause  of   aor ti c  stenosi s  i n  thi s  age­gr oup?
3.   What  i s  the  av er age  sur v i v al   of   pati ents  w i th  uncor r ected  aor ti c  stenosi s  af ter   the  onset  of
sy ncope?
4.   How   i s  the  sev er i ty   of   aor ti c  stenosi s  m ost  accur atel y   deter m i ned?
5.   What  i s  the  best  ther apy   f or   sy m ptom ati c  aor ti c  stenosi s?

Case Discussion
1.   What  i s  the  m ost  l i k el y   v al v ul ar   l esi on  i n  thi s  pati ent?

The  hi stor y   of   angi na  and  sy ncope  and  the  cl assi c  phy si cal   ex am i nati on  f i ndi ngs  m ak e  aor ti c
stenosi s  an  al m ost  cer tai n  di agnosi s  i n  thi s  pati ent.   The  char acter i sti c  ar ter i al   pul ses  descr i bed
hav e  been  r ef er r ed  to  as  pul sus  par v us  et  tar dus.   The  si ngl e  second  hear t  sound  i ndi cates  the
absence  of   the  aor ti c  com ponent,   suggesti ng  sev er e  i m m obi l i ty   of   the  aor ti c  v al v e.   The  m ur m ur   i s
al so  char acter i sti c  of   aor ti c  stenosi s  w i th  i ts  cr escendo–decr escendo  qual i ty   and  the  l ate  peak i ng.
Do  not  be  f ool ed  by   the  hi gh­f r equency   com ponent  at  the  car di ac  apex .   Al though  the  m ur m ur   of
aor ti c  stenosi s  i s  m ost  of ten  hear d  at  the  upper   car di ac  bor der   w i th  r adi ati on  to  the  car oti d
ar ter i es,   the  m ur m ur   m ay   al so  r adi ate  to  the  apex ,   w her e  i t  m ay   be  m i stak en  f or   the  m ur m ur   of
m i tr al   r egur gi tati on.

2.   What  i s  the  m ost  l i k el y   cause  of   aor ti c  stenosi s  i n  thi s  age­gr oup?

Betw een  35  and  65  y ear s  of   age,   degener ati v e  change  i n  a  congeni tal l y   bi cuspi d  aor ti c  v al v e  i s  the
pr edom i nant  cause  of   aor ti c  stenosi s.   Bey ond  65  y ear s  of   age,   aor ti c  stenosi s  usual l y   r esul ts  f r om
cal ci f i cati on  of   a  pr ev i ousl y   nor m al   tr i cuspi d  aor ti c  v al v e  (seni l e  cal ci f i c  aor ti c  stenosi s).   Al though
the  ex act  cause  of   seni l e  aor ti c  stenosi s  i s  unk now n,   i t  i s  associ ated  w i th  hy per tensi on  and
hy per l i pi dem i a.   Isol ated  aor ti c  stenosi s  i n  the  U ni ted  States  r ar el y   r esul ts  f r om   r heum ati c  di sease.

3.   What  i s  the  av er age  sur v i v al   of   pati ents  w i th  uncor r ected  aor ti c  stenosi s  af ter   the  onset  of
sy ncope?

Pati ents  w i th  aor ti c  stenosi s  m ay   r em ai n  asy m ptom ati c  f or   y ear s,   but  once  sy m ptom s  dev el op  the
cour se  of   the  di sease  m ay   be  qui te  f ul m i nant.   Accor di ng  to  studi es  conducted  bef or e  v al v e  sur ger y
w as  av ai l abl e,   such  pati ents  w i th  sy ncope  due  to  aor ti c  stenosi s  coul d  ex pect  to  sur v i v e  an  av er age
of   3  y ear s  af ter   the  onset  of   sy ncope.   The  av er age  sur v i v al   af ter   the  onset  of   angi na  pector i s  or   HF
i s  5  and  2  y ear s,   r especti v el y .   Ther ef or e,   the  onset  of   angi na,   sy ncope,   or   HF  due  to  aor ti c
stenosi s  si gnal s  the  need  f or   v al v e  r epl acem ent.   Pati ents  shoul d  al so  be  questi oned
P. 63
about  m or e  subtl e  sy m ptom s  such  as  ex er ti onal   dy spnea  or   a  decr ease  i n  ex er ci se  capaci ty ,   as
these  sy m ptom s  m ay   al so  i ndi cate  the  need  f or   sur ger y .

4.   How   i s  the  sev er i ty   of   aor ti c  stenosi s  m ost  accur atel y   deter m i ned?

The  sev er i ty   of   aor ti c  stenosi s  can  be  pr eci sel y   deter m i ned  by   ei ther   car di ac  catheter i zati on  or
Doppl er /echocar di ogr aphy .   Both  techni ques  can  pr ov i de  accur ate  esti m ati ons  of   the  pr essur e
gr adi ent  and  the  aor ti c  v al v e  ar ea.   Doppl er   echocar di ogr aphy   i s  gener al l y   used  f or   ev al uati on  and
f ol l ow ­up  because  i t  i s  noni nv asi v e  and  easi l y   r epeated.   The  nor m al   aor ti c  v al v e  ar ea  i s  3  cm 2 .
Mi l d,   m oder ate,   and  sev er e  aor ti c  stenosi s  ar e  pr esent  w hen  the  v al v e  ar ea  i s  > 1. 5  cm 2 ,   betw een
1. 5  and  1  cm 2 ,   and  < 1  cm 2 ,   r especti v el y .   Aor ti c  stenosi s  i s  sai d  to  be  cr i ti cal   w hen  the  v al v e  ar ea
i s  0. 7  cm 2   or   l ess.   Pr essur e  gr adi ent  m easur em ents  al one  ar e  not  adequate  to  deter m i ne  the
sev er i ty   of   aor ti c  stenosi s  because,   as  al r eady   di scussed,   pr essur e  gr adi ents  ar e  deter m i ned  by
both  the  ar ea  of   the  stenoti c  v al v e  and  the  bl ood  f l ow   acr oss  the  v al v e.   Phy si cal   ex am i nati on
f i ndi ngs  such  as  the  l ate­peak i ng  m ur m ur   and  the  absent  aor ti c  com ponent  of   the  second  hear t
sound  m ay   be  suggesti v e  of   sev er e  aor ti c  stenosi s  but  ar e  poor l y   sensi ti v e  and  speci f i c  com par ed
w i th  the  i nf or m ati on  y i el ded  by   the  aor ti c  v al v e  ar ea.   Echocar di ogr aphy   can  al so  ev al uate
v entr i cul ar   f uncti on  and  hy per tr ophy   as  w el l   as  pr ov i de  i nf or m ati on  about  the  eti ol ogy   of   the  aor ti c
stenosi s.

5.   What  i s  the  best  ther apy   f or   sy m ptom ati c  aor ti c  stenosi s?

Aor ti c  v al v e  r epl acem ent  i s  the  best  ther apy   f or   sy m ptom ati c  aor ti c  stenosi s.   In  sy m ptom ati c
pati ents  w i th  sev er e  aor ti c  stenosi s,   aor ti c  v al v e  r epl acem ent  r esul ts  i n  a  postoper ati v e  sur v i v al
that  i s  cl ose  to  that  of   the  gener al   popul ati on.   Ol der   pati ents  al so  gener al l y   hav e  a  good  sur v i v al
f ol l ow i ng  aor ti c  v al v e  r epl acem ent  f or   aor ti c  stenosi s.

Long­ter m   r esul ts  of   bal l oon  aor ti c  v al v ul opl asty   (a  catheter ­based  pr ocedur e)  hav e  been
di sappoi nti ng.   Ther ef or e,   i t  i s  used  pr i m ar i l y   f or   pal l i ati on  i n  pati ents  w ho  ar e  not  candi dates  f or
aor ti c  v al v e  r epl acem ent  because  of   other   m edi cal   pr obl em s  or   as  a  br i dge  to  aor ti c  v al v e
r epl acem ent  i n  pati ents  deem ed  too  i l l   f or   sur ger y .   How ev er ,   ser i ous  com pl i cati ons  and  m or tal i ty
ar e  hi gh  i n  these  pati ents  and  r estenosi s  gener al l y   r ecur s  w i thi n  6  to  12  m onths.

Case 2
A  50­y ear ­ol d  w om an  w ho  had  an  “i nnocentâ€​   m ur m ur   di agnosed  i n  chi l dhood  pr esents  w i th  dy spnea
on  ex er ti on,   or thopnea,   and  par ox y sm al   noctur nal   dy spnea  of   sev er al   m onths'  dur ati on.   On  questi oni ng,
she  descr i bes  a  1­y ear   hi stor y   of   f ati gue  and  ex hausti on  that  has  l i m i ted  her   dai l y   acti v i ti es.   She  has
not  seen  a  phy si ci an  i n  y ear s.
On  phy si cal   ex am i nati on,   her   bl ood  pr essur e  i s  110/70  m m   Hg.   Her   jugul ar   v enous  pr essur e  i s  nor m al
and  she  has  m i l dl y   di m i ni shed  ar ter i al   pul se  am pl i tude  w i th  a  nor m al   ar ter i al   upstr ok e.   Her   l ungs  ar e
cl ear   to  per cussi on  and  auscul tati on.   Ther e  i s  a  l ater al l y   di spl aced  apex   i m pul se  and  a  pal pabl e  thi r d
hear t  sound  that  i s  easi l y   hear d.   The  f i r st  hear t  sound  i s  sof t  and  ther e  i s  a  w i del y   spl i t  second  hear t
sound  w i th  nor m al   r espi r ator y   spl i tti ng.   A  gr ade  3/4  bl ow i ng,   hi gh­pi tched  sy stol i c  m ur m ur   i s  hear d  at
the  apex   and  r adi ates  to  the  ax i l l a  and  l ef t  i nf r ascapul ar   ar ea.   Ther e  i s  tr ace  edem a  but  no  cl ubbi ng  or
cy anosi s.
P. 64

1.   What  i s  the  v al v ul ar   l esi on  i n  thi s  pati ent?


2.   What  i s  the  m ost  com m on  under l y i ng  cause  of   sev er e  m i tr al   r egur gi tati on  i n  the  adul t  U . S.
popul ati on?
3.   Does  m edi cal   ther apy   pr ev ent  pr ogr essi on  of   m i tr al   r egur gi tati on?
4.   When  shoul d  sur ger y   be  consi der ed  f or   pati ents  w i th  sev er e  m i tr al   r egur gi tati on?
5.   What  ar e  the  choi ces  f or   m i tr al   v al v e  sur ger y ?

Case Discussion
1.   What  i s  the  v al v ul ar   l esi on  i n  thi s  pati ent?

Chr oni c  m i tr al   r egur gi tati on,   the  m ost  i nsi di ous  of   al l   l ef t­si ded  v al v ul ar   l esi ons,   i s  the  m ost  l i k el y
di agnosi s  i n  thi s  pati ent.   Sev er e  l ef t  v entr i cul ar   dy sf uncti on  i s  not  uncom m on  at  pr esentati on  i n
thi s  di sor der .   The  hol osy stol i c  api cal   m ur m ur   i s  char acter i sti c  of   chr oni c  m i tr al   r egur gi tati on.   The
thi r d  hear t  sound  suggests  that  the  m i tr al   r egur gi tati on  i s  sev er e,   and  i s  a  r ef l ecti on  of   the  l ar ge
v ol um e  of   bl ood  cr ossi ng  the  m i tr al   v al v e  i n  ear l y   di astol e.   How ev er ,   the  thi r d  hear t  sound  does
not  necessar i l y   i m pl y   HF.   In  chr oni c  m i tr al   r egur gi tati on,   the  l ar ge  r egur gi tant  v ol um e  enter i ng  the
l ef t  atr i um   r esul ts  i n  l ef t  atr i al   enl ar gem ent  w i th  the  l ef t  atr i um   of ten  touchi ng  the  spi ne.   The
m ur m ur   i s  tr ansm i tted  to  the  spi ne  thr ough  the  l ef t  atr i um ,   accounti ng  f or   i ts  r adi ati on  to  the
subscapul ar   ar ea.

The  m ur m ur   descr i bed  i s  ty pi cal   f or   chr oni c  m i tr al   r egur gi tati on,   but  m ust  be  di sti ngui shed  f r om
such  m ur m ur s  as  tr i cuspi d  r egur gi tati on,   aor ti c  stenosi s,   and  VSD.   The  m ur m ur   char acter i sti c  of
aor ti c  stenosi s  i s  di sti ngui shed  by   i ts  qual i ty   (cr escendo–decr escendo),   l ocati on,   and  r adi ati on,
as  descr i bed  i n  Case  1  under   secti on  on  Val v ul ar   Hear t  Di sease.   A  tr i cuspi d  i nsuf f i ci ency   m ur m ur   i s
usual l y   w el l   l ocal i zed  to  the  l ef t  ster nal   bor der   w i th  l i ttl e  r adi ati on,   and  has  the  char acter i sti c
f eatur e  of   an  i ncr ease  i n  i ntensi ty   w i th  i nspi r ati on.   The  m ur m ur   char acter i sti c  of   VSD  i s  ty pi cal l y
hear d  best  at  the  l ef t  ster nal   bor der ,   of ten  has  a  har sh  qual i ty ,   and  does  not  change  w i th
r espi r ati on.   The  m ur m ur   of   VSD  i s  r ar el y   hear d  i n  adul ts  because  m ost  congeni tal   VSDs  ar e
detected  i n  chi l dhood  and  r esol v e  spontaneousl y   or   ar e  sur gi cal l y   cor r ected.   A  VSD  i s  a  r ar e
com pl i cati on  of   acute  MI  i n  adul ts.

2.   What  i s  the  m ost  com m on  under l y i ng  cause  of   sev er e  m i tr al   r egur gi tati on  i n  the  adul t  U . S.
popul ati on?

My x om atous  m i tr al   v al v e  di sease,   usual l y   as  an  i sol ated  l esi on  or   som eti m es  associ ated  w i th  other
connecti v e  ti ssue  di sor der s  (e. g. ,   Mar f an's  and  Ehl er s­Danl os  sy ndr om es),   consti tutes  the  m ost
com m on  cause  of   sev er e  m i tr al   r egur gi tati on  necessi tati ng  m i tr al   v al v e  r epl acem ent  or   r epai r   i n
the  U ni ted  States,   especi al l y   i n  y ounger   peopl e.   A  sm al l er   num ber   of   cases  ar e  due  to  r heum ati c
hear t  di sease,   i nf ecti v e  endocar di ti s,   or   spontaneousl y   r uptur ed  chor dae  tendi neae.   In  ol der   peopl e,
sev er e  m i tr al   r egur gi tati on  of ten  accom pani es  l ef t  v entr i cul ar   enl ar gem ent  and  dy sf uncti on  due  to
CAD  and  MI.   The  m i tr al   r egur gi tati on  i s  not  due  to  an  abnor m al i ty
P. 65
of   the  v al v e  l eaf l ets  but  due  to  a  com bi nati on  of   abnor m al i ti es  of   the  suppor ti ng  str uctur es  of   the
v al v e  i ncl udi ng  str etchi ng  of   the  m i tr al   annul us  al ong  w i th  papi l l ar y   m uscl e  dy sf uncti on  and  w al l
m oti on  abnor m al i ti es.

3.   Does  m edi cal   ther apy   pr ev ent  pr ogr essi on  of   m i tr al   r egur gi tati on?

Ther e  ar e  no  cl i ni cal   tr i al s  dem onstr ati ng  benef i ts  of   any   m edi cal   ther apy   f or   chr oni c  m i tr al
r egur gi tati on  that  i s  due  to  a  pr i m ar y   abnor m al i ty   of   the  m i tr al   v al v e.   If   m i tr al   r egur gi tati on  i s  due
to  l ef t  v entr i cul ar   enl ar gem ent  w i th  dy sf uncti on  of   the  m i tr al   v al v e  appar atus,   ACE  i nhi bi tor s  and
β­bl ock er s  ar e  i ndi cated  to  i m pr ov e  v entr i cul ar   f uncti on.   If   v entr i cul ar   f uncti on  i m pr ov es  i n
pati ents  w i th  m i tr al   r egur gi tati on  due  to  v entr i cul ar   dy sf uncti on,   the  m i tr al   r egur gi tati on  w i l l
i m pr ov e  as  the  v al v e  appar atus  becom es  m or e  f uncti onal .

4.   When  shoul d  sur ger y   be  consi der ed  f or   pati ents  w i th  sev er e  m i tr al   r egur gi tati on?

Mi tr al   r egur gi tati on  r esul ts  i n  a  chr oni c  v ol um e  ov er l oad  on  the  l ef t  v entr i cl e  and  ul ti m atel y   r esul ts
i n  l ef t  v entr i cul ar   contr acti l e  dy sf uncti on.   Mi tr al   v al v e  sur ger y   shoul d  be  per f or m ed  bef or e
v entr i cul ar   dy sf uncti on  occur s,   but  ther e  i s  no  m ethod  to  pr eci sel y   pr edi ct  the  onset  of   v entr i cul ar
dy sf uncti on.   Ser i al   Doppl er   echocar di ogr aphy   studi es  shoul d  be  per f or m ed  to  ev al uate  l ef t
v entr i cul ar   si ze  and  f uncti on  and  assess  pul m onar y   pr essur es.   Sur ger y   i s  r ecom m ended  i n  the
asy m ptom ati c  pati ent  i f   the  EF  f al l s  bel ow   60%,   or   w hen  the  l ef t  v entr i cul ar   end­di astol i c
di m ensi on  ex ceeds  45  m m ,   or   i f   pul m onar y   hy per tensi on  or   atr i al   f i br i l l ati on  dev el ops.   Sur ger y   i s
r ecom m ended  f or   pati ents  w i th  sy m ptom s  of   HF  due  to  m i tr al   r egur gi tati on  unl ess  they   hav e  sev er e
l ef t  v entr i cul ar   dy sf uncti on  or   other   contr ai ndi cati ons  to  sur ger y .

5.   What  ar e  the  choi ces  f or   m i tr al   v al v e  sur ger y ?

Mi tr al   v al v e  r epl acem ent  w i th  m echani cal   or   bi opr ostheti c  v al v es  and,   m or e  r ecentl y ,   m i tr al   v al v e
r epai r   consti tute  the  sur gi cal   tr eatm ents  av ai l abl e  f or   sev er e  m i tr al   r egur gi tati on  due  to  a  pr i m ar y
v al v e  abnor m al i ty .   Mi tr al   v al v e  r epai r   i s  associ ated  w i th  a  l ow er   sur gi cal   m or tal i ty   r ate  and  a
better   l ong­ter m   outcom e  than  m i tr al   v al v e  r epl acem ent,   and  i s  the  pr ef er r ed  pr ocedur e  w hen
r epai r   i s  possi bl e.   Mi tr al   v al v e  sur ger y   i s  gener al l y   not  i ndi cated  i f   the  m i tr al   r egur gi tati on  i s  due
to  l ef t  v entr i cul ar   dy sf uncti on  w i thout  a  pr i m ar y   v al v e  abnor m al i ty .

Case 3
A  56­y ear ­ol d  m an  i s  seen  because  of   pr ogr essi v e  f ati gue,   dy spnea  on  ex er ti on,   or thopnea,   and
par ox y sm al   noctur nal   dy spnea.   On  phy si cal   ex am i nati on  hi s  bl ood  pr essur e  i s  160/60  m m   Hg.   Ther e  i s
no  jugul ar   v enous  di stenti on,   but  sy stol i c  pul sati ons  of   the  uv ul a  ar e  noted,   as  i s  qui ck   col l apse  of   the
ar ter i al   pul ses,   w hi ch  i s  seen  i n  the  nai l   beds  w i th  gentl e  pr essur e.   The  l ungs  ar e  cl ear   to  per cussi on
and  auscul tati on.   Ther e  i s  a  di f f use  and  hy per dy nam i c  apex   beat  that  i s  di spl aced  l ater al l y   and
i nf er i or l y ,   sof t  f i r st  and  second  hear t  sounds,   a  l oud  thi r d  hear t  sound,   and  a  gr ade  3/6,   hi gh­pi tched,
near l y   hol odi astol i c  m ur m ur   hear d  best  at  the  upper   l ef t  ster nal   bor der   al ong  w i th  a  gr ade  3/6  sy stol i c
ejecti on  ty pe  m ur m ur   at  the  upper   l ef t  ster nal   bor der   r adi ati ng  to  the  car oti ds.   A  l ate  di astol i c  r um bl e  i s
hear d  at  the  apex   as  w el l   as  a  thi r d  hear t  sound.
P. 66

1.   What  i s  the  m ost  l i k el y   v al v ul ar   l esi on  i n  thi s  pati ent?


2.   What  i s  the  l i k el y   under l y i ng  cause  of   aor ti c  r egur gi tati on  i n  thi s  pati ent?
3.   What  i s  the  appr opr i ate  m edi cal   ther apy   f or   the  pati ent  w i th  aor ti c  r egur gi tati on?
4.   When  shoul d  aor ti c  v al v e  r epl acem ent  be  consi der ed  i n  a  pati ent  w i th  chr oni c  aor ti c  r egur gi tati on?
5.   What  i s  the  sur gi cal   ther apy   f or   sev er e  aor ti c  r egur gi tati on?

Case Discussion
1.   What  i s  the  m ost  l i k el y   v al v ul ar   l esi on  i n  thi s  pati ent?

Thi s  pati ent  has  chr oni c  sev er e  aor ti c  r egur gi tati on.   Ther e  ar e  m any   phy si cal   si gns  to  l ook   f or   i n
the  setti ng  of   aor ti c  r egur gi tati on,   som e  of   w hi ch  ar e  seen  i n  thi s  pati ent.   These  i ncl ude  de
Musset's  si gn  (the  head  bobs  w i th  each  hear tbeat),   Cor r i gan's  si gn  or   w ater ham m er   pul ses,
Tr aube's  si gn  (boom i ng  sy stol i c  and  di astol i c  sounds  hear d  ov er   the  f em or al   ar ter i es),   Mul l er 's  si gn
(sy stol i c  pul sati on  of   the  uv ul a),   Qui nck e's  si gn  (capi l l ar y   pul sati ons  seen  i n  the  nai l   beds),   and
other s.   Al l   of   these  ar e  si gns  of   l ar ge  str ok e  v ol um e  and  w i de  pul se  pr essur e  char acter i sti c  of
chr oni c  aor ti c  r egur gi tati on.   The  l oud  3/6  di astol i c  m ur m ur   hear d  at  the  upper   l ef t  ster nal   bor der   i s
the  m ur m ur   of   aor ti c  i nsuf f i ci ency   and  the  sy stol i c  m ur m ur   i s  due  to  tur bul ent  f l ow   acr oss  the
aor ti c  v al v e  because  of   the  l ar ge  am ount  of   bl ood  cr ossi ng  the  aor ti c  v al v e.   The  m i d­to­l ate
di astol i c  r um bl e,   nam el y   the  Austi n  Fl i nt  m ur m ur ,   i s  cr eated  by   r api d  r etr ogr ade  f l ow   f r om   the
aor ta  str i k i ng  the  anter i or   m i tr al   l eaf l et.   Another   ex pl anati on  f or   thi s  m ur m ur   i s  that  the  l ar ge
v ol um e  of   r egur gi tant  f l ow   par ti al l y   cl oses  the  m i tr al   v al v e,   cr eati ng  a  l ate  di astol i c  m i tr al   v al v e
gr adi ent.

2.   What  i s  the  l i k el y   under l y i ng  cause  of   aor ti c  r egur gi tati on  i n  thi s  pati ent?

In  thi s  age­gr oup,   the  m ost  l i k el y   cause  of   aor ti c  r egur gi tati on  i s  a  bi cuspi d  aor ti c  v al v e.   Causes  of
aor ti c  r egur gi tati on  can  be  br ok en  dow n  i nto  tw o  gener al   categor i es—v al v ul ar   di sease  and  aor ti c
r oot  di sease.   Rheum ati c  hear t  di sease,   i nf ecti v e  endocar di ti s,   tr aum a,   bi cuspi d  v al v e,   other
congeni tal   v al v ul ar   def ects  (e. g. ,   a  f enestr ated  v al v e),   sy stem i c  l upus  er y them atosus,   r heum atoi d
ar thr i ti s,   ank y l osi ng  spondy l i ti s,   and  Whi ppl e's  di sease  m ay   cause  pr i m ar y   v al v ul ar   di sease.   Cy sti c
m edi al   necr osi s  of   the  aor ta  (i sol ated  or   associ ated  w i th  Mar f an's  sy ndr om e  or   Ehl er s­Danl os
sy ndr om e),   ather oscl er osi s,   hy per tensi on,   sy phi l i ti c  aor ti ti s,   and  other s  m ay   cause  aor ti c  r oot
di l atati on  and  def or m i ty   of   the  aor ti c  v al v e,   l eadi ng  to  i nabi l i ty   of   the  v al v e  to  coapt.   A  di ssecti on
of   the  aor ta  m ay   al so  cause  aor ti c  r egur gi tati on  by   di ssecti ng  i nto  the  v al v e  i tsel f .

3.   What  i s  the  appr opr i ate  m edi cal   ther apy   f or   the  pati ent  w i th  aor ti c  r egur gi tati on?

The  use  of   v asodi l ator s  to  del ay   the  pr ogr essi on  of   aor ti c  r egur gi tati on  and  l ef t  v entr i cul ar
dy sf uncti on  i n  asy m ptom ati c  pati ents  i s  contr ov er si al   and  def i ni te  ev i dence  of   a  benef i t  has  not
been  dem onstr ated.   In  pati ents  w i th  sy m ptom s  of   HF  due  to  aor ti c  r egur gi tati on,   v asodi l ator s  m ay
pr ov i de  sy m ptom ati c  benef i t  but  shoul d  not  del ay   r ef er r al   f or   aor ti c  v al v e  r epl acem ent.

P. 67
4.   When  shoul d  aor ti c  v al v e  r epl acem ent  be  consi der ed  i n  a  pati ent  w i th  chr oni c  aor ti c  r egur gi tati on?

Aor ti c  v al v e  sur ger y   shoul d  be  consi der ed  i f   the  pati ent  has  sy m ptom s  of   HF.   In  the  asy m ptom ati c
pati ent,   aor ti c  v al v e  r epl acem ent  i s  r ecom m ended  i f   the  EF  f al l s  bel ow   nor m al   or   i f   the  end­sy stol i c
di m ensi on  of   the  l ef t  v entr i cl e  i s  l ar ger   than  55  m m .

5.   What  i s  the  sur gi cal   ther apy   f or   sev er e  aor ti c  r egur gi tati on?

Aor ti c  v al v e  r epl acem ent  w i th  a  pr ostheti c  v al v e  i s  the  onl y   sur gi cal   opti on  i n  m ost  pati ents.

Case 4
A  32­y ear ­ol d  w om an  w ho  r ecentl y   m ov ed  to  the  U ni ted  States  f r om   Mex i co  i s  seen  because  of   the
r ecent  onset  of   pal pi tati ons  associ ated  w i th  dy spnea  on  ex er ti on,   or thopnea,   and  par ox y sm al   noctur nal
dy spnea  w i th  hem opty si s.
On  phy si cal   ex am i nati on,   her   bl ood  pr essur e  i s  112/90  m m   Hg  and  her   hear t  r ate  i s  120  per   m i nute  and
i r r egul ar l y   i r r egul ar .   Jugul ar   v enous  di stenti on  to  10  cm   H 2 O  w i th  a  pr om i nent  V  w av e  i s  noted,   as  ar e
di m i ni shed  ar ter i al   pul ses  and  bi basi l ar   r al es  (up  to  hal f   of   the  l ung  f i el ds  bi l ater al l y ).   Addi ti onal
f i ndi ngs  i ncl ude  a  nondi spl aced  apex   beat,   a  r i ght  v entr i cul ar   heav e  pal pabl e  i n  the  l ef t  par aster nal
r egi on,   a  pal pabl e  pul m oni c  cl osur e  sound  i n  the  second  l ef t  i nter costal   space,   an  accentuated  S 4 ,   a  l oud
pul m oni c  second  sound  (P 2 )  ov er   the  l ef t  v entr i cul ar   apex ,   a  snappi ng  sound  ov er   the  l ef t  v entr i cul ar
apex   i m pul se  just  af ter   the  second  hear t  sound,   and  a  gr ade  3/4,   l ow ­pi tched,   r um bl i ng,   near l y
hol odi astol i c  m ur m ur   hear d  best  at  the  car di ac  apex .   Ther e  i s  1  to  2+   pi tti ng  edem a  noted  i n  the  l ow er
ex tr em i ti es  and  pr esacr al   ar ea.

1.   What  i s  the  m ost  l i k el y   v al v ul ar   l esi on  i n  thi s  pati ent?


2.   What  i s  the  m ost  com m on  cause  of   m i tr al   stenosi s  i n  adul t  pati ents?
3.   What  i s  the  m or tal i ty   r ate  associ ated  w i th  m edi cal l y   tr eated  m i tr al   stenosi s?
4.   What  ar e  the  m ajor   com pl i cati ons  of   m i tr al   stenosi s?
5.   What  i s  the  best  tr eatm ent  f or   sy m ptom ati c  pati ents  w i th  m i tr al   stenosi s?

Case Discussion
1.   What  i s  the  m ost  l i k el y   v al v ul ar   l esi on  i n  thi s  pati ent?

The  cl i ni cal   pi ctur e  ex hi bi ted  by   thi s  pati ent  i s  char acter i sti c  of   sev er e  m i tr al   stenosi s  w i th
secondar y   pul m onar y   hy per tensi on  and  cor   pul m onal e.   The  sev er i ty   of   the  m i tr al   stenosi s  i s
i ndi cated  by   the  m i tr al   openi ng  snap,   w hi ch  cl osel y   f ol l ow s  the  second  hear t  sound  and  the
hol odi astol i c  r um bl e.   The  m i tr al   openi ng  snap  i s  a  char acter i sti c  si gn  of   m i tr al   stenosi s  and
appear s  to  be  due  to  a  sudden  tensi ng  of   the  v al v e  l eaf l ets  af ter   the  v al v e  cusps  hav e  com pl eted
thei r   openi ng  ex cur si ons,   and  occur s  shor tl y   af ter   (0. 08  to  0. 12  second)  the  aor ti c  com ponent  of
the  second  hear t  sound.   The  m i tr al   openi ng  snap  m ov es  cl oser   to  the  second  hear t  sound  as  the
pr essur e  betw een  the  l ef t  atr i um   and  l ef t  v entr i cl e  i ncr eases.   The  r um bl i ng,
P. 68
l ow ­pi tched  di astol i c  m ur m ur   hear d  at  the  apex   i s  char acter i sti c  of   m i tr al   stenosi s.   The  dur ati on  of
the  m ur m ur   thr oughout  di astol e  i ndi cates  that  ther e  i s  a  pr essur e  gr adi ent  acr oss  the  m i tr al   v al v e
thr oughout  di astol e.   Pul m onar y   hy per tensi on  i s  i ndi cated  by   the  l oud  pul m oni c  com ponent  of   the
second  hear t  sound  and  r i ght  v entr i cul ar   heav e.   A  P 2   that  can  be  hear d  at  the  l ef t  v entr i cul ar   apex
i ndi cates  pul m onar y   hy per tensi on.   Cor   pul m onal e  i s  r ef l ected  by   the  el ev ated  neck   v ei ns,   and
per i pher al   edem a.   The  l ar ge  V  w av e  i ndi cates  tr i cuspi d  r egur gi tati on—a  r esul t  of   the  pul m onar y
hy per tensi on  and  cor   pul m onal e.   Par ox y sm al   noctur nal   dy spnea  w i th  hem opty si s  i s  a  m ajor   cl ue  to
thi s  di agnosi s  and  r ef l ects  a  sudden  i ncr ease  i n  pul m onar y   capi l l ar y   pr essur e  w i th  i ntr aal v eol ar
edem a  and  hem or r hage  such  as  m i ght  occur   w i th  ex er ci se  or   new   onset  of   atr i al   f i br i l l ati on.

2.   What  i s  the  m ost  com m on  cause  of   m i tr al   stenosi s  i n  adul t  pati ents?

Mi tr al   stenosi s  i n  adul ts  i s  al m ost  ex cl usi v el y   due  to  r heum ati c  hear t  di sease.   Thi s  pati ent
descr i bed  a  pr ol onged  i l l ness  at  12  y ear s  of   age  consi stent  w i th  acute  r heum ati c  f ev er   but  hal f   of
al l   pati ents  w i th  r heum ati c  m i tr al   stenosi s  w i l l   not  hav e  a  cl ear   chi l dhood  hi stor y   of   r heum ati c
f ev er .

3.   What  i s  the  m or tal i ty   r ate  associ ated  w i th  m edi cal l y   tr eated  m i tr al   stenosi s?

Fr om   the  ti m e  of   the  i ni ti al   di agnosi s,   pati ents  w i th  m edi cal l y   tr eated  m i tr al   stenosi s  can  ex pect  a
m or tal i ty   r ate  of   20%  at  5  y ear s  and  40%  at  10  y ear s.   Thi s  pati ent  f aces  a  m uch  l ess  f av or abl e
pr ognosi s  because  of   her   pul m onar y   hy per tensi on  and  r i ght  v entr i cul ar   HF.   How ev er ,   the  r i sk   i s
si gni f i cantl y   r educed  i f   she  under goes  v al v e  r epl acem ent,   com m i ssur otom y ,   or   m i tr al   bal l oon
v al v otom y .

4.   What  ar e  the  m ajor   com pl i cati ons  of   m i tr al   stenosi s?

In  pati ents  w i th  uncor r ected  m i tr al   stenosi s,   ther e  i s  a  20%  l i f eti m e  r i sk   of   thr om boem bol i sm .   Thi s
i s  of ten  a  dev astati ng  com pl i cati on  because  the  em bol us  m ost  of ten  tr av el s  to  the  br ai n,   r esul ti ng
i n  a  str ok e.   Ei ghty   per cent  of   pati ents  w i th  sy stem i c  em bol i   ar e  i n  atr i al   f i br i l l ati on.   Thi s  r i sk   i s
decr eased  by   the  use  of   anti coagul ant  ther apy   w i th  sodi um   w ar f ar i n.   Inf ecti v e  endocar di ti s  occur s
l ess  f r equentl y   but  m ay   be  a  di sastr ous  com pl i cati on.   Atr i al   f i br i l l ati on  i s  a  com m on  com pl i cati on
of   m i tr al   stenosi s.   The  l ef t  atr i um   i s  of ten  v er y   l ar ge  due  to  a  com bi nati on  of   r heum ati c
i nv ol v em ent  of   the  atr i al   m uscl e  and  the  hi gh  l ef t  atr i al   pr essur es,   pr edi sposi ng  to  atr i al
f i br i l l ati on.

5.   What  i s  the  best  tr eatm ent  f or   sy m ptom ati c  pati ents  w i th  m i tr al   stenosi s?

Opti ons  f or   cor r ecti on  of   m i tr al   stenosi s  i ncl ude  per cutaneous  tr ansv enous  m i tr al   v al v ul opl asty ,
sur gi cal   m i tr al   com m i ssur otom y ,   or   m i tr al   v al v e  r epai r .   In  bal l oon  v al v ul opl asty ,   the  bal l oon  i s
passed  f r om   the  f em or al   v ei n  to  the  r i ght  atr i um ,   acr oss  the  atr i al   septum ,   and  acr oss  the  m i tr al
v al v e.   The  bal l oon  i s  i nf l ated,   cr ack i ng  open  the  v al v e.   Thi s  i s  the  pr ef er r ed  pr ocedur e  i n
ex per i enced  hands  i f   the  v al v e  anatom y   i s  f av or abl e  and  ther e  ar e  no  contr ai ndi cati ons.   Resul ts  ar e
al so  good  w i th  sur gi cal   com m i ssur otom y   or   m i tr al   v al v e  r epl acem ent.

Suggested Readings
Bonow   RO,   Lak atos  E,   Mar on  BJ,   etal .   Ser i al   l ong­ter m   assessm ent  of   the  natur al   hi stor y   of
asy m ptom ati c  pati ents  w i th  chr oni c  aor ti c  r egur gi tati on  and  nor m al   l ef t  v entr i cul ar   sy stol i c  f uncti on.
Ci r cul ati on  1991;84:1625–1635.

P. 69

Car abel l o  BA.   Concentr i c  v s  eccentr i c  r em odel i ng.   J  Car d  Fai l   2002;8:S258–S263.

Car abel l o  BA.   Moder n  m anagem ent  of   m i tr al   stenosi s.   Ci r cul ati on  2005;112:432–437.

Car abel l o  BA.   Vasodi l ator s  i n  aor ti c  r egur gi tati on­w her e  i s  the  ev i dence  of   thei r   ef f ecti v eness?  N
Engl   J  Med  2005;353:1400–1402.

Fr eem an  RV,   Otto  CM.   Spectr um   of   cal ci f i c  aor ti c  v al v e  di sease:  pathogenesi s,   di sease  pr ogr essi on,
and  tr eatm ent  str ategi es.   Ci r cul ati on  2005;111:3316–3326.

Gol dsm i th  I,   Tur pi e  AG,   Li p  GY.   Val v ul ar   hear t  di sease  and  pr ostheti c  hear t  v al v es.   Br   Med  J
2002;325:1228–1231.

Lam bo  N J,   Del l 'Ital i a  LJ,   Cr aw f or d  MH,   etal .   Bedsi de  di agnosi s  of   sy stol i c  m ur m ur s.   N   Engl   J  Med
1988;318:1572–1579.

Li eber m an  EG,   Bashor e  TM,   Her m i l l er   JB,   etal .   Bal l oon  aor ti c  v al v ul opl asty   i n  adul ts:  f ai l ur e  of
pr ocedur e  to  i m pr ov e  l ong­ter m   sur v i v al .   J  Am   Col l   Car di ol   1995;26:1522–1528.

Pel l i k k a  PA,   N i shi m ur a  RA,   Bai l ey   KR,   etal .   The  natur al   hi stor y   of   adul ts  w i th  asy m ptom ati c,
hem ody nam i cal l y   si gni f i cant  aor ti c  stenosi s.   J  Am   Col l   Car di ol   1990;15:1012–1017.

Rey es  VP,   Raju  BS,   Wy nne  J,   etal .   Per cutaneous  bal l oon  v al v ul opl asty   com par ed  w i th  open  sur gi cal
com m i ssur otom y   f or   m i tr al   stenosi s.   N   Engl   J  Med  1994;331:961–967.

Rozi ch  JD,   Car abel l o  BA,   U sher   BW,   etal .   Mi tr al   v al v e  r epl acem ent  w i th  and  w i thout  chor dal
pr eser v ati on  i n  pati ents  w i th  chr oni c  m i tr al   r egur gi tati on:  m echani sm s  f or   di f f er ences  i n
postoper ati v e  ejecti on  per f or m ance.   Ci r cul ati on  1992;86:1718–1726.

Vongpastanasi n  W,   Hi l l s  LD,   Lange  RA.   Medi cal   pr ogr ess:  pr ostheti c  hear t  v al v es.   N   Engl   J  Med
1996;335:407–416.
Editors :  Sc hrie r,  Robe rt W .
Title :  Inte rna l Me dic ine  Ca s e book ,  The : Re a l P a tie nts ,  Re a l Ans w e rs ,
3rd Edition

Copy r i ght  ©2007  Li ppi ncott  Wi l l i am s  &  Wi l k i ns

>  T a b le   o f   C o nte nts   >  C ha p te r   3  ­  End o c r ino lo g y ,  Me ta b o lis m ,  a nd   D ia b e te s

Chapter 3
Endocrinology, Metabolism, and Diabetes

Ire ne  E.  Sc ha ue r


Ja ne  E. B.  Re us c h
P. 71

Adrenal Insufficiency
1.   What  ar e  the  gener al   categor i es  of   adr enocor ti cal   i nsuf f i ci ency ?

2.   Can  y ou  ex pl ai n  w hy   thy r oi d  f uncti on  tests  shoul d  be  ev al uated  i n  a


pati ent  w i th  pr i m ar y   adr enal   f ai l ur e?

3.   What  ar e  the  char acter i sti c  si gns  and  sy m ptom s  of   acute  and  chr oni c
adr enal   i nsuf f i ci ency ?

4.   What  cr i ter i a  ar e  used  to  m ak e  the  di agnosi s  of   adr enal   i nsuf f i ci ency ?

5.   What  ar e  the  consi der ati ons  i n  deci di ng  on  l ong­ter m   r epl acem ent  ther apy
f or   Addi son's  di sease?

6.   What  other   m etabol i c  abnor m al i ti es  m ay   occur   i n  associ ati on  w i th  adr enal
i nsuf f i ci ency ?

7.   What  ar e  the  ev ents  that  tak e  pl ace  i n  the  r egul ati on  of   cor ti sol   secr eti on
by   the  hy pothal am i c–pi tui tar y –adr enal   ax i s?

8.   What  ar e  the  speci f i c  causes  of   pr i m ar y   and  secondar y   adr enal   f ai l ur e?

Discussion
1.   What  ar e  the  gener al   categor i es  of   adr enocor ti cal   i nsuf f i ci ency ?

Adr enocor ti cal   i nsuf f i ci ency   r esul ts  pr i m ar i l y   f r om   def i ci ent  cor ti sol
pr oducti on  and  i n  som e  cases  def i ci ent  al doster one  and  andr ogen
pr oducti on  by   the  adr enal   gl and.   Because  the  adr enal   cor tex   i s  nor m al l y
sti m ul ated  by   pi tui tar y   adr enocor ti cotr opi c  hor m one  (ACTH;
cor ti cotr opi n),   cor ti sol   def i ci ency   m ay   r esul t  f r om   adr enal   di sease
(prima ry a dre na l ins uffic ie nc y  or   Addi son's  di sease)  or   f r om   pi tui tar y   or
hy pothal am i c  di sease  w i th  ACTH  def i ci ency   (s e c onda ry a dre na l
ins uffic ie nc y).
2.   Can  y ou  ex pl ai n  w hy   thy r oi d  f uncti on  tests  shoul d  be  ev al uated  i n  a
pati ent  w i th  pr i m ar y   adr enal   f ai l ur e?

The  associ ati on  betw een  autoi m m une  thy r oi di ti s  and  autoi m m une  adr enal
di sease  i s  w el l   r ecogni zed.   In  gener al ,   pati ents  w i th  Addi son's  di sease  ar e
af f l i cted  m or e  f r equentl y   w i th  Hashi m oto's  thy r oi di ti s  than  w i th  Gr av es'
di sease.   Appr ox i m atel y   50%  or   m or e  of   af f ected  pati ents  hav e  hi gh  ti ter s
of   thy r oi d  anti m i cr osom al   anti bodi es,   al though  these  pati ents  of ten  hav e
no  thy r oi d­r el ated  sy m ptom s.   Gr av es'  hy per thy r oi di sm   can  occur   i n
associ ati on  w i th  pr i m ar y   adr enal   f ai l ur e.   The  associ ati on  betw een  thy r oi d
f ai l ur e  and  adr enal   f ai l ur e  can  al so  r ef l ect  hy popi tui tar i sm ,   w i th  a
consequent  def i ci ency   of   both  ACTH  and  thy r oi d­sti m ul ati ng  hor m one
(TSH).   Ther ef or e,   abnor m al   r esul ts  f r om   thy r oi d  f uncti on  tests  hav e  been
seen  i n  the  setti ngs  of   both  pr i m ar y   and  secondar y   hy poadr enal i sm ,
m ak i ng  thy r oi d  f uncti on  tests  an  i m por tant  com ponent  of   the  ev al uati on
of   a  pati ent  w i th  pr i m ar y   or   secondar y   adr enal   f ai l ur e.

3.   What  ar e  the  char acter i sti c  si gns  and  sy m ptom s  of   acute  and  chr oni c
adr enal   i nsuf f i ci ency ?

Acute  adr enal   i nsuf f i ci ency   i s  a  potenti al l y   f atal   m edi cal   em er gency ,   and
the  cl i ni cal   f eatur es  i ncl ude  nausea,   f ev er ,   and  shock ,   pr ogr essi ng  to
P. 72
di ar r hea,   m uscul ar   w eak ness,   i ncr eased  and  then  decr eased  body
tem per atur e,   hy pogl y cem i a,   hy ponatr em i a,   and  hy per k al em i a.

The  car di nal   si gns  of   chr oni c  adr enal   i nsuf f i ci ency   ar e  w eak ness,   f ati gue,
and  anor ex i a,   al ong  w i th  gastr oi ntesti nal   com pl ai nts  of   nausea,   v om i ti ng,
di ar r hea,   and  v ague  abdom i nal   pai n.   Other   sy m ptom s  i ncl ude  sal t  cr av i ng
(20%  of   the  pati ents)  and  m uscl e  cr am ps.   Phy si cal   f i ndi ngs  m ay   com pr i se
w ei ght  l oss,   hy per pi gm entati on,   hy potensi on,   and  v i ti l i go.   The  ear
car ti l age  m ay   cal ci f y   i n  pati ents  w i th  l ong­standi ng  adr enal   i nsuf f i ci ency .

4.   What  cr i ter i a  ar e  used  to  m ak e  the  di agnosi s  of   adr enal   i nsuf f i ci ency ?

The  di agnosi s  of   adr enocor ti cal   i nsuf f i ci ency   i s  based  pr i m ar i l y   on  the
pl asm a  cor ti sol   deter m i nati ons  m ade  dur i ng  the  r api d  ACTH  sti m ul ati on
test  (Cor tr osy n  test).   Any   scr eeni ng  tests  f or   adr enal   i nsuf f i ci ency   m ust
i ncl ude  deter m i nati on  of   a  basal   l ev el   of   cor ti sol   and  ACTH,   together   w i th
a  r api d  ACTH  sti m ul ati on  test.   Thi s  test  i s  per f or m ed  by   adm i ni ster i ng  25
uni ts  (0. 25  m g)  of   sy ntheti c  ACTH  i ntr av enousl y /i ntr am uscul ar l y   (IV  or
IM)  and  m easur i ng  the  r esponse  of   cor ti sol   and  al doster one.   It  i s
per f or m ed  to  assess  i ni ti al l y   w hether   the  adr enal s  can  r espond  to
ex ogenous  ACTH.   A  cl ear l y   nor m al   r esponse  ex cl udes  the  possi bi l i ty   of
pr i m ar y   and  chr oni c,   but  not  acute,   secondar y   adr enal   f ai l ur e.   For   the
cor ti sol   r esponse  to  be  nor m al ,   the  cor ti sol   l ev el   af ter   ACTH
adm i ni str ati on  shoul d  be  at  l east  18  ng/dL  and  i ncr eased  by   at  l east  9
ng/dL  abov e  the  basal   l ev el s.   N or m al l y ,   the  al doster one  l ev el s  par al l el
the  cor ti sol   l ev el s,   w i th  an  i ncr ease  of   at  l east  14  ng/dL  abov e  the  basal
l ev el s.   Pati ents  w i th  Addi son's  di sease  ex hi bi t  v er y   l ow   cor ti sol   l ev el s
and  a  cl ear l y   el ev ated  ACTH  l ev el ,   w her eas  the  l ev el s  of   both  tend  to  be
l ow   i n  pati ents  w i th  hy popi tui tar i sm .   In  the  cl assi c  si tuati on,   the  r esponse
of   al doster one  to  ACTH  i s  absent  i n  pati ents  w i th  pr i m ar y   adr enal   f ai l ur e,
w her eas  i t  i s  pr eser v ed  i n  pati ents  w i th  secondar y   adr enal   f ai l ur e.   The
m easur em ent  of   al doster one  i s  not  r outi ne  but  can  add  di agnosti c
i nf or m ati on  f or   pr i m ar y   adr enal   f ai l ur e.   A  l ow ­dose  (1  µg  cor tr osy n)
cor tr osy n  sti m ul ati on  test  i s  al so  av ai l abl e  and  m ay   be  m or e  sensi ti v e
w hen  appr opr i ate  cutof f   v al ues  ar e  used.   How ev er ,   addi ti onal   techni cal
di f f i cul ti es  i n  cor tr osy n  adm i ni str ati on  and  ti m i ng  of   bl ood  tests  hav e
pr ev ented  thi s  test  f r om   becom i ng  r outi nel y   accepted.

5.   What  ar e  the  consi der ati ons  i n  deci di ng  on  l ong­ter m   r epl acem ent  ther apy
f or   Addi son's  di sease?

Long­ter m   r epl acem ent  ther apy   i n  pati ents  w i th  Addi son's  di sease
i nv ol v es  the  or al   adm i ni str ati on  of   a  cor ti sone  pr epar ati on  i n  phy si ol ogi c
r epl acem ent  doses.   U sual l y ,   tw o  thi r ds  of   the  total   dose  i s  gi v en  i n  the
m or ni ng  and  the  r em ai nder   i s  gi v en  i n  the  ev eni ng  to  m i m i c  the  nor m al
ci r cadi an  secr eti on  of   cor ti sol .   Cor ti sone  acetate  can  be  tak en  as  a  dose
of   25  m g  i n  the  m or ni ng  and  12. 5  m g  i n  the  ev eni ng.   Al ter nati v el y ,
hy dr ocor ti sone  can  be  tak en  i n  a  dosage  of   30  to  40  m g  per   day .
How ev er ,   because  cor ti sone  m ust  be  conv er ted  to  hy dr ocor ti sone  i n  the
body ,   hy dr ocor ti sone  i s  consi der ed  the  m or e  phy si ol ogi c  agent.   Despi te
thi s,   pr edni sone  (5–7. 5  m g  per   day )  i s  f r equentl y   pr escr i bed  f or   l ong­
ter m   r epl acem ent  because  i t  costs  l ess  than  hy dr ocor ti sone.   The  si de
ef f ects  f r om   the  ex cessi v e  adm i ni str ati on  of   the  abov e
P. 73
gl ucocor ti coi ds  i ncl ude  i ncr eased  appeti te,   w ei ght  gai n,   i nsom ni a,   edem a,
and  hy per tensi on.

Mi ner al ocor ti coi d  r epl acem ent  (f l uor ohy dr ocor ti sone  ther apy )  i s
necessar y   i n  pati ents  w i th  pr i m ar y   Addi son's  di sease,   al though  the  ex act
r epl acem ent  dose  m ust  be  ti tr ated  to  the  pati ent's  r esponse.   Dr am ati c
f l ui d  r etenti on  m ay   occur   w i th  the  i ni ti al   tr eatm ent,   but  thi s  subsi des
once  the  dose  i s  adjusted.

6.   What  other   m etabol i c  abnor m al i ti es  m ay   occur   i n  associ ati on  w i th  adr enal
i nsuf f i ci ency ?

Hype rk a le mia   occur s  f r equentl y   i n  pati ents  w i th  pr i m ar y   adr enal   f ai l ur e


(appr ox i m atel y   64%).   Thi s  i s  l ar gel y   due  to  r enal   tubul ar   absor pti on  of
potassi um   at  the  ex pense  of   sodi um   stem m i ng  f r om   the  m i ner al ocor ti coi d
def i ci ency .   In  addi ti on,   gl ucocor ti coi ds  hel p  i n  m ai ntai ni ng  the  f uncti on  of
the  sodi um   pum p  and  the  nor m al   gr adi ent  betw een  the  i ntr acel l ul ar   and
ex tr acel l ul ar   concentr ati ons  of   sodi um   and  potassi um .   Wi thout  cor ti sol ,
thi s  gr adi ent  i s  not  m ai ntai ned,   so  that  potassi um   m ov es  out  of   the  cel l
and  sodi um   m ov es  i nto  the  cel l ,   ther eby   r esul ti ng  i n  hy per k al em i a.   Of
note,   i n  pati ents  w i th  secondar y   (pi tui tar y )  adr enal   i nsuf f i ci ency ,   the
m i ner al ocor ti coi d  ax i s  i s  i ntact  and  hy per k al em i a,   ar i si ng  f r om   the
second  m echani sm   onl y ,   i s  m i l d  or   absent.

Hypoglyc e mia   occur s  i nf r equentl y ,   and  pr i m ar i l y   i n  pati ents  w i th


Addi son's  di sease  w ho  hav e  f asted  f or   any   per i od.   It  i s  due  to  def ecti v e
gl uconeogenesi s.
A  m i l d  a c idos is   m ay   ev entuate  i n  pati ents  w i th  m i ner al ocor ti coi d
def i ci ency   because  of   the  decr eased  secr eti on  of   am m oni a  and  hy dr ogen
i ons.

Ci r cul ati ng  l ev el s  of   a ntidiure tic  hormone   (ADH)  m ay   i ncr ease  and
contr i bute  to  the  hy ponatr em i a.   The  ex cessi v e  l oss  of   sodi um   by   the  r enal
tubul es  l eads  to  an  i ncr eased  w ater   l oss.   Thi s  i s  counter bal anced  by   an
i ncr ease  i n  the  ADH  l ev el s,   w hi ch  tends  to  cause  w ater   r etenti on.   The  l ow
car di ac  output  and  hy pov ol em i a  al so  ser v e  as  sti m ul i   f or   ADH  r el ease.

The  ina bility to e x c re te  a  w a te r loa d  w as  once  used  as  a  di agnosti c  test
f or   Addi son's  di sease.   Thi s  phenom enon  i s  pr i m ar i l y   caused  by
gl ucocor ti coi d  def i ci ency ,   ev en  i n  the  pr esence  of   euv ol em i a.   A  bol us  of
cor ti sol   com pl etel y   r ev er ses  the  ef f ect  and  a  “w ater   di ur esi sâ€​
ensues,   but  thi s  al so  i nv ol v es  the  i nter pl ay   of   other   f actor s,   such  as  an
i m pr ov em ent  i n  car di ac  output,   an  i ncr ease  i n  the  ef f ecti v e  ci r cul ati ng
v ol um e,   an  i ncr ease  i n  the  gl om er ul ar   f i l tr ati on  r ate,   a  r educti on  i n  ADH
l ev el s,   and  di r ect  ef f ects  on  the  r enal   tubul e.

Per i pher al   e os inophilia   i s  a  com m on  f i ndi ng  i n  the  setti ng  of   pr i m ar y
adr enal   i nsuf f i ci ency .

7.   What  ar e  the  ev ents  that  tak e  pl ace  i n  the  r egul ati on  of   cor ti sol   secr eti on
by   the  hy pothal am i c–pi tui tar y –adr enal   ax i s?

Adr enocor ti cal   cel l   gr ow th  and  ster oi d  secr eti on  ar e  pr i m ar i l y   contr ol l ed
by   the  pi tui tar y   hor m one  ACTH.   The  secr etor y   r egul ati on  of   the
hy pothal am i c–pi tui tar y –adr enal   ax i s  i nv ol v es  the  r el ease  of
cor ti cotr opi n­r el easi ng  hor m one  (CRH)  by   the  hy pothal am us  i nto  the
hy pophy seal   por tal   sy stem .   Thi s  hor m one  causes  the  pi tui tar y   secr eti on
of   ACTH,   w hi ch  i s  tr anspor ted  by   the  per i pher al   ci r cul ati on  to  the  adr enal
gl ands,   w her e  i t  i s  bound  by
P. 74
speci f i c  r eceptor s  and  tr i gger s  ster oi d  sy nthesi s  and  secr eti on.   Cor ti sol
i nhi bi ts  both  CRH  and  ACTH  r el ease,   w her eas  ACTH  has  a  negati v e
f eedback   ef f ect  on  CRH  r el ease.   Hor m onal   and  neur al   i nput  f r om   hi gher
br ai n  center s  sti m ul ates  or   i nhi bi ts  CRH  sy nthesi s  and  secr eti on  i n  a  24­
hour   cy cl e,   w hi ch  causes  both  ACTH  and  cor ti sol   secr eti on  to  ex hi bi t  a
ci r cadi an  r hy thm .   The  ci r cadi an  r hy thm   can  be  ov er com e  by   str ess,
how ev er ,   l eadi ng  to  chr oni c  cor ti sol   sy nthesi s.   Cor ti sol   ci r cul ates  bound
to  cor ti sol ­bi ndi ng  gl obul i n  (tr anscor ti n)  and  the  f r ee  cor ti sol   enter s  a
cel l   and  i nter acts  w i th  a  speci f i c  r eceptor   to  ex er t  i ts  phy si ol ogi c  ef f ects.

8.   What  ar e  the  speci f i c  causes  of   pr i m ar y   and  secondar y   adr enal   f ai l ur e?

Pr i m ar y   adr enal   i nsuf f i ci ency   (Addi son's  di sease)  i s  m ost  com m onl y
caused  by   i di opathi c  adr enal   atr ophy   stem m i ng  f r om   autoi m m une
destr ucti on  (68%),   tuber cul osi s  (17%),   or   som e  other   eti ol ogy   (15%).
N i nety   per cent  of   the  gl and  m ust  hav e  been  destr oy ed  bef or e  Addi son's
di sease  becom es  appar ent.   Less  com m on  causes  of   adr enal   i nsuf f i ci ency
i ncl ude  other   gr anul om atous  di seases,   such  as  hi stopl asm osi s  and
sar coi dosi s,   or   i nf i l tr ati v e  di seases,   such  as  am y l oi dosi s,
hem ochr om atosi s,   m etastati c  tum or ,   and  adr enal   l euk ody str ophy ,   as  w el l
as  chr oni c  anti coagul ati on  and  bi l ater al   adr enal   hem or r hage.   Gr am ­
negati v e  septi cem i a,   bi l ater al   adr enal ectom y ,   abdom i nal   i r r adi ati on,
adr enal   v ei n  thr om bosi s,   adr enal   ar ter y   em bol us,   and  adr enol y ti c  dr ugs
ar e  al so  r ar e  causes  of   adr enal   f ai l ur e.

Adr enal   i nsuf f i ci ency   i s  f ound  i n  som e  pati ents  w i th  acqui r ed
i m m unodef i ci ency   sy ndr om e  (AIDS).   The  m ai n  pr esentati on  of   adr enal
i nsuf f i ci ency   i n  AIDS  i s  f ati gue;  el ectr ol y te  abnor m al i ti es  ar e  uncom m on.
Dev el opm ent  of   adr enal   i nsuf f i ci ency   i n  pati ents  w i th  at  l east  one  AIDS­
def i ni ng  di sease  i s  associ ated  w i th  poor   pr ognosi s.

Secondar y   adr enal   i nsuf f i ci ency   i s  com m onl y   caused  by   i atr ogeni c
cor ti coster oi d  ther apy ,   w hi ch  suppr esses  CRH  and  ACTH  secr eti on  and
r esul ts  i n  adr enal   atr ophy .   Other ,   l ess  com m on  causes  i ncl ude  pi tui tar y
and  hy pothal am i c  tum or s,   i r r adi ati on,   tr aum a,   pi tui tar y   necr osi s,   and
l y m phocy ti c  hy pophy si ti s  sur gi cal   pr ocedur es.

Case
A  60­y ear ­ol d  m an  i s  hospi tal i zed  because  of   sev er e  nausea,   v om i ti ng,   and
di ar r hea  of   4  day s'  dur ati on.   He  adm i ts  to  hav i ng  ex per i enced  m i l d  i ncr easi ng
f ati gue  and  m al ai se  f or   the  l ast  6  m onths  pl us  poor   appeti te,   f r equent
abdom i nal   cr am ps,   and  a  20­l b  (9­k g)  w ei ght  l oss  ov er   the  l ast  4  m onths.   He
f eel s  di zzy   i n  the  m or ni ng  and  l i ghtheaded  af ter   standi ng  f or   m or e  than  an
hour .   He  notes  that  he  tends  to  tak e  a  nap  i n  the  l ate  af ter noon.   Four   day s
bef or e  pr esentati on,   abdom i nal   cr am ps,   v om i ti ng,   and  di ar r hea  dev el oped.   He
deni es  any   sk i n  changes  and  pr ol onged  sun  ex posur e.   He  adm i ts  to  a  decl i ne
i n  sex ual   desi r e.   He  has  no  hi stor y   of   hy per tensi on,   di abetes,   asthm a,   or
tuber cul osi s,   and  tak es  no  m edi cati ons.
Phy si cal   ex am i nati on  r ev eal s  a  v er y   tanned  m an,   w ho  appear s  acutel y   i l l   and
som ew hat  dehy dr ated.   He  w ei ghs  63  k g.   Hi s  supi ne  bl ood  pr essur e  (BP)  i s
106/68  m m   Hg  and  hi s  supi ne  pul se  i s  90  beats  per   m i nute;  hi s  standi ng  BP  i s
80/50  m m   Hg  and  hi s  standi ng  pul se  i s  104  beats  per   m i nute.
P. 75
Hi s  sk i n  show s  decr eased  tur gor .   Hi s  f ace,   hands,   ex tensor   sur f aces,   chest,
and  back   ar e  notabl y   tanned.   The  f i ndi ngs  f r om   the  head,   ey es,   ear ,   nose,   and
thr oat  ex am i nati on  ar e  nor m al ,   ex cept  f or   the  pr esence  of   har dened  ear l obes.
N o  hear t  abnor m al i ti es  ar e  noted  and  hi s  l ungs  ar e  cl ear .   Abdom i nal
ex am i nati on  r ev eal s  the  pr esence  of   di f f use  tender ness,   but  no  r ebound  or
l ocal i zed  tender ness.   The  bow el   sounds  ar e  hy per acti v e.   Ther e  i s  decr eased
ax i l l ar y   hai r .   Hi s  testes  ar e  nor m al   and  centr al   ner v ous  sy stem   f i ndi ngs  ar e
unr em ar k abl e.
The  f ol l ow i ng  l abor ator y   data  ar e  obtai ned:  hem ogl obi n  (Hgb),   10. 6  g,
nor m ochr om i c  nor m ocy ti c  anem i a;  w hi te  bl ood  cel l   (WBC)  count,   6, 600
cel l s/m m 3 ;  sodi um ,   128  m Eq/L;  potassi um ,   5. 9  m Eq/L;  cr eati ni ne,   2. 0  m g/dL;
bi car bonate  (HCO 3 ­ ),   20  m Eq/L;  chl or i de,   96  m Eq/L;  bl ood  ur ea  ni tr ogen
(BU N ),   39  m g/dL;  and  cal ci um ,   11. 1  m g/dL.
The  chest  r adi ogr aphi c  study   f i ndi ngs  ar e  nor m al   and  the  abdom i nal
r adi ogr aphi c  study   show s  a  nor m al   gas  patter n,   but  bi l ater al   adr enal
cal ci f i cati on.   Hi s  el ectr ocar di ogr am   (ECG)  i s  nor m al .
Sev en  m onths  l ater ,   the  pati ent  becom es  sev er el y   f ati gued  and  w eak   and
com pl ai ns  of   col d  i ntol er ance,   dr y   sk i n,   som nol ence,   and  consti pati on.
Phy si cal   ex am i nati on  at  that  ti m e  r ev eal s  a  pal e  pati ent,   w i th  a  supi ne  BP  of
110/60  m m   Hg  and  supi ne  pul se  of   64  per   m i nute.   He  w ei ghs  72  k g.   Hi s  sk i n  i s
dr y   and  w ar m   and  ex hi bi ts  decr eased  tur gor .   Per i or bi tal   f r eck l i ng  and  v i ti l i go
ar e  pr esent,   as  w el l   as  m i l d,   di f f use  thy r om egal y .   N eur ol ogi c  ex am i nati on
r ev eal s  gener al i zed  m uscl e  w eak ness  and  decr eased  deep  tendon  r ef l ex es
sy m m etr i cal l y .

Labor ator y   data  ar e  as  f ol l ow s:  WBC,   6, 900  cel l s/m m 3   w i th  nor m al
di f f er enti al ;  ser um   sodi um ,   135  m Eq/L;  potassi um ,   4. 7  m Eq/L;  chl or i de,   99
m Eq/L;  HCO 3 ­ ,   24. 8  m Eq/L;  gl ucose,   78  m g/dL;  cr eati ni ne,   1. 0  m g/dL;  and
BU N ,   18  m g/dL.   Thy r oi d  f uncti on  tests  r ev eal   the  f ol l ow i ng  f i ndi ngs:  ser um
thy r ox i ne  (T 4 ),   3. 2  µg/dL  (nor m al ,   4  to  12  µg/dL);  tr i i odothy r oni ne  (T 3 )
r esi n  uptak e,   20%  (nor m al ,   25%  to  35%);  and  TSH,   16  µU /m L  (nor m al ,
0. 55. 0  µU /m L).   The  test  r esul t  f or   anti m i cr osom al   anti bodi es  i s  posi ti v e,   w i th
a  v al ue  of   1:50, 000.

1.   What  i s  the  m ost  l i k el y   di agnosi s  i n  thi s  pati ent?


2.   What  w oul d  be  the  f i r st  step  i n  the  di agnosti c  ev al uati on  of   thi s  pati ent?
3.   On  the  basi s  of   the  f i ndi ngs  f r om   the  i ni ti al   di agnosti c  ev al uati on,   w hat  i s
the  di agnosi s  i n  thi s  pati ent?
4.   What  w oul d  y ou  r ecom m end  as  an  i ni ti al   ther apy ?
5.   How   w oul d  y ou  tr eat  thi s  pati ent's  hy per cal cem i a?
6.   What  addi ti onal   abnor m al i ti es  m ay   be  seen  i n  associ ati on  w i th  Addi son's
di sease?
7.   On  the  basi s  of   the  f i ndi ngs  w hen  the  pati ent  i s  seen  7  m onths  l ater ,
w hat  k i nd  of   thy r oi d  di sease  does  he  hav e?
8.   What  i s  the  m ost  i m por tant  adv i ce  to  gi v e  thi s  pati ent?

Case Discussion
1.   What  i s  the  m ost  l i k el y   di agnosi s  i n  thi s  pati ent?

The  m ost  l i k el y   di agnosi s  i n  thi s  pati ent  i s  acute  adr enal   i nsuf f i ci ency
r esul ti ng  f r om   ei ther   pr i m ar y   or   secondar y   adr enal   f ai l ur e.   Thi s  pati ent
i l l ustr ates  the
P. 76
nonspeci f i c  natur e  of   sy m ptom s  i n  the  setti ng  of   chr oni c  adr enal
i nsuf f i ci ency ,   ev en  though  he  ex hi bi ts  the  cl assi c  hi stor y   and  f i ndi ngs.

Thi s  pati ent's  hy per pi gm entati on  and  el ectr ol y te  changes  suggest  pr i m ar y
adr enal   f ai l ur e.   The  hy per k al em i a  and  hy ponatr em i a  ar e  due  to
m i ner al ocor ti coi d  def i ci ency ,   of ten  seen  i n  the  setti ng  of   pr i m ar y   adr enal
f ai l ur e.   Because  ACTH  i s  not  the  pr edom i nant  r egul ator   of   al doster one
secr eti on,   el ectr ol y te  abnor m al i ti es  ar e  l ess  com m on  i n  pati ents  w i th
secondar y   adr enal   f ai l ur e.

Adr enal   cr i si s  occur s  w hen  a  str essf ul   si tuati on  br i ngs  about
decom pensati on.   The  natur e  of   the  str ess  m ay   r ange  f r om   m i l d  (e. g. ,   the
f l u)  to  sev er e  (e. g. ,   tr aum a  or   sur ger y ).   Adr enal   decom pensati on  i s
m ar k ed  by   dehy dr ati on,   hy pov ol em i a,   pr of ound  hy potensi on,
hy ponatr em i a,   hy per k al em i a,   hy pogl y cem i a,   and  hy pother m i a.   Cl assi c
r enal   f ai l ur e  can  m i m i c  sev er al   aspects  of   chr oni c  adr enal   f ai l ur e,
i ncl udi ng  f ati gue,   m al ai se,   anor ex i a,   hy ponatr em i a,   and  hy per k al em i a.   In
thi s  pati ent,   the  BU N   and  cr eati ni ne  abnor m al i ti es  ar e  m or e  i ndi cati v e  of
pr er enal   azotem i a  than  of   acute  r enal   f ai l ur e.

Decr eased  l i bi do,   w hi ch  i s  com m on  i n  pati ents  w i th  hy popi tui tar i sm ,   can
al so  be  seen  i n  pati ents  w i th  Addi son's  di sease  and  i s  due  to  the
debi l i tati ng  natur e  of   the  i l l ness,   the  associ ated  pr i m ar y   gonadal   f ai l ur e,
and  possi bl y   the  decr eased  adr enal   andr ogens.

Cal ci f i cati on  of   the  aur i cul ar   and  costal   car ti l age  i s  uncom m on  i n  pati ents
w i th  Addi son's  di sease,   but  can  occur   i nci dental l y .   A  l ack   of   ax i l l ar y   hai r
i s  actual l y   a  m or e  com m on  f i ndi ng  i n  f em al e  pati ents.   The  am ount  of
pubi c  hai r   m ay   al so  be  di m i ni shed.

2.   What  w oul d  be  the  f i r st  step  i n  the  di agnosti c  ev al uati on  of   thi s  pati ent?

The  ACTH  sti m ul ati on  test  shoul d  be  per f or m ed  i ni ti al l y   to  assess  w hether
the  adr enal   gl ands  can  r espond  to  ex ogenous  ACTH  by   i ncr easi ng  the
l ev el s  of   cor ti sol   and  al doster one.   Si m ul taneousl y ,   the  pl asm a  ACTH  l ev el
shoul d  be  m easur ed  because  pati ents  w i th  Addi son's  di sease  hav e  v er y
l ow   cor ti sol   l ev el s  but  el ev ated  ACTH  l ev el s.   It  i s  cr i ti cal   to  hav e  the
l abor ator y   pr ocess  the  sam pl es  cor r ectl y   (check   w i th  y our   l abor ator y   to
deter m i ne  the  appr opr i ate  pr ocess  f or   bl ood  col l ecti on).   Adr enal
autoanti body   testi ng  i s  now   av ai l abl e  and  has  a  70%  sensi ti v i ty .   In
addi ti on,   because  of   the  abdom i nal   r adi ogr aphi c  f i ndi ng  of   adr enal
cal ci f i cati on,   a  pur i f i ed  pr otei n  der i v ati v e  (PPD)  sk i n  test  shoul d  be
per f or m ed  to  assess  f or   tuber cul osi s.

An  ACTH  sti m ul ati on  test  r ev eal s  a  basal   cor ti sol   l ev el   of   2. 8  µg/dL,
w hi ch  i s  then  2. 8  µg/dL  at  30  m i nutes  and  3. 0  µg/dL  at  60  m i nutes.   The
al doster one  l ev el   i s  2. 5  ng/m L  at  0  m i nutes,   2. 5  ng/m L  at  30  m i nutes,
and  3. 1  ng/m L  at  60  m i nutes  (nor m al   v al ues—cor ti sol ,   9  to  25  µg/dL
a. m .   f asti ng,   and  2  to  16  µg/dL  p. m .   f asti ng;  al doster one,   nor m al   sal t
upr i ght:  m en,   6  to  22  ng/dL;  w om en,   4  to  31  ng/dL).   The  pl asm a  ACTH
l ev el   i s  f ound  to  be  779  pg/m L  (nor m al ,   < 580  pg/m L  at  8:00  a. m .
upr i ght;  526  pg/m L  at  8:00  a. m .   supi ne;  and  < 517  pg/m L  at  4:00  p. m .
supi ne).   The  PPD  test  r esul t  i s  negati v e.

3.   On  the  basi s  of   the  f i ndi ngs  f r om   the  i ni ti al   di agnosti c  ev al uati on,   w hat  i s
the  di agnosi s  i n  thi s  pati ent?

The  r esul ts  of   the  ACTH  sti m ul ati on  test  i n  thi s  pati ent  ar e  cl ear l y
abnor m al ,   show i ng  subnor m al   r esponses  to  ACTH—i ndi cati v e  of   adr enal
f ai l ur e.   Thi s  i s  the
P. 77
cl assi c  si tuati on  i n  pati ents  w i th  pr i m ar y   adr enal   f ai l ur e,   that  i s,   the
r esponse  of   both  cor ti sol   and  al doster one  to  ACTH  i s  absent;  i n  secondar y
adr enal   f ai l ur e,   the  al doster one  r esponse  i s  pr eser v ed.

The  pl asm a  ACTH  l ev el   i s  m ar k edl y   el ev ated  i n  thi s  pati ent,   and  such
ex tr em e  el ev ati ons  m ay   be  seen  i n  the  contex t  of   sev er e  str ess,   such  as
that  caused  by   sur ger y ,   anesthesi a,   and  hy pogl y cem i a.   Cal ci f i cati on  of
the  adr enal   gl ands  can  occur   i n  the  setti ng  of   tuber cul osi s,
hi stopl asm osi s,   and  occasi onal l y   i n  autoi m m une  adr enal   di sease.
Ther ef or e,   the  cause  of   thi s  pati ent's  adr enal   gl and  f ai l ur e  i s  pr i m ar y
adr enal   f ai l ur e,   m ost  l i k el y   secondar y   to  the  autoi m m une  destr ucti on  of
the  adr enal s.   The  negati v e  PPD  r esul t  suppor ts  a  nontuber cul ous  eti ol ogy
of   the  pr i m ar y   adr enal   f ai l ur e.

4.   What  w oul d  y ou  r ecom m end  as  an  i ni ti al   ther apy ?

Because  the  cl i ni cal   pr esentati on  suggests  adr enal   cr i si s,   ther apy   shoul d
be  i nsti tuted  i m m edi atel y   because  adr enal   cr i si s  i s  a  l i f e­thr eateni ng
em er gency   and  any   del ay   i n  tr eatm ent  coul d  pr ov e  f atal .   Such  ther apy
i ncl udes  the  i m m edi ate  IV  adm i ni str ati on  of   a  sol ubl e  cor ti coster oi d
pr epar ati on,   such  as  hy dr ocor ti sone  (100  m g),   f ol l ow ed  by   r api d  i nf usi ons
of   gl ucose  and  nor m al   sal i ne  at  a  r ate  of   2  to  4  L  per   day .   For   tr ue  cr i si s,
l ar ge  v ol um es  (2  to  3  L)  of   0. 9%  sal i ne  sol uti on  or   5%  dex tr ose  i n  0. 9%
sal i ne  shoul d  be  i nf used  i ntr av enousl y   as  qui ck l y   as  possi bl e.

The  gl ucocor ti coi ds  and  v ol um e  r epl eti on  cause  the  ser um   potassi um
l ev el s  to  decr ease.   Def i ni ti v e  di agnosti c  testi ng  shoul d  be  car r i ed  out
af ter   the  acute  ther apy   has  been  i nsti tuted.   Mi ner al ocor ti coi d  ther apy
shoul d  be  def er r ed  unti l   the  pati ent  can  tak e  m edi cati on  or al l y .

5.   How   w oul d  y ou  tr eat  thi s  pati ent's  hy per cal cem i a?

Because  thi s  pati ent's  hy per cal cem i a  i s  m i l d,   no  speci al   tr eatm ent  other
than  hy dr ati on  w i th  nor m al   sal i ne  i s  r equi r ed.   Both  hy per cal cem i a  and
hy pocal cem i a  hav e  been  r epor ted  to  occur   dur i ng  an  adr enal   cr i si s.   Thi s
m ay   stem   f r om   dehy dr ati on,   but  m ay   al so  be  a  consequence  of   the
i ncr eased  absor pti on  of   cal ci um   f r om   the  gut,   due  to  gl ucocor ti coi d
def i ci ency .   Occasi onal l y ,   m i l d  hy per cal cem i a  and  hy per par athy r oi di sm
m ay   coex i st  w i th  adr enal   f ai l ur e  caused  by   a  pi tui tar y   tum or   that
com pr om i ses  the  f uncti on  of   cor ti cotr ophs  [m ul ti pl e  endocr i ne  neopl asi a
ty pe  1  (MEN   1)].   Hy pocal cem i a  m ay   occur   i n  pati ents  w hose
hy poadr enal i sm   i s  a  par t  of   the  autoi m m une  pol y gl andul ar   sy ndr om e  ty pe
I  (pol y gl andul ar   f ai l ur e).

6.   What  addi ti onal   abnor m al i ti es  m ay   be  seen  i n  associ ati on  w i th  Addi son's
di sease?

Other   abnor m al i ti es  that  m ay   ar i se  i n  pati ents  w i th  Addi son's  di sease
i ncl ude  hy pogl y cem i a,   hy per k al em i a,   hi gh  ADH  l ev el s,   m etabol i c  aci dosi s,
v i ti l i go,   and  hi gh  l ev el s  of   anti thy r oi d  anti bodi es.   Al l   of   these  can  be  a
f r equent  com ponent  of   the  cl i ni cal   pi ctur e  i n  pati ents  w i th  adr enal
i nsuf f i ci ency .
7.   On  the  basi s  of   the  f i ndi ngs  w hen  the  pati ent  i s  seen  7  m onths  l ater ,   w hat
k i nd  of   thy r oi d  di sease  does  he  hav e?

The  f i ndi ngs  ar e  consi stent  w i th  those  of   Hashi m oto's  thy r oi di ti s.   Pati ents
w i th  i di opathi c  Addi son's  di sease  ar e  pr one  to  other   autoi m m une
di sor der s,   w hi ch  m ay   dev el op  bef or e  or   af ter   adr enal   f ai l ur e  i s
di agnosed.   These  di sor der s  i ncl ude  Gr av es'  hy per thy r oi di sm ,   Hashi m oto's
thy r oi di ti s,   per ni ci ous  anem i a,   di abetes,   hy popar athy r oi di sm ,   pr i m ar y
hy pogonadi sm ,   v i ti l i go,   and  m oni l i asi s.   Ar eas  of   v i ti l i go
P. 78
f or m   i n  4%  to  6%  of   the  pati ents  w i th  Addi son's  di sease,   especi al l y   i n
those  w hose  di sease  has  an  autoi m m une  cause.

In  thi s  m an  w ho  has  a  goi ter ,   l ow   T 4   and  hi gh  TSH  l ev el s,   and  str ongl y
posi ti v e  anti thy r oi d  anti body   ti ter s,   l ev othy r ox i ne  ther apy   shoul d  be
star ted,   but  onl y   w hen  adequate  ster oi d  r epl acem ent  has  been  achi ev ed
and  af ter   the  pati ent  has  been  on  ster oi d  r epl acem ent  ther apy   f or   at  l east
2  w eek s.   An  adr enal   cr i si s  coul d  be  pr eci pi tated  i f   l ev othy r ox i ne  i s  gi v en
to  a  pati ent  w ho  i s  i n  a  hy poadr enal   state  because  of   the  r esul ti ng
i ncr eased  m etabol i c  dem ands  that  l ev othy r ox i ne  i m poses  on  the  body .

8.   What  i s  the  m ost  i m por tant  adv i ce  to  gi v e  thi s  pati ent?

In  any   pati ent  w i th  adr enal   i nsuf f i ci ency ,   i t  i s  cr i ti cal   to  em phasi ze  the
need  f or   i ncr easi ng  the  dosage  of   gl ucocor ti coi ds  dur i ng  per i ods  of   str ess
or   i l l ness,   such  as  col ds,   f l u,   di ar r hea,   i nf ecti ons,   tr aum a,   or   sur ger y .
Fai l ur e  to  do  so  m i ght  pr eci pi tate  the  r api d  dev el opm ent  of   an  acute
adr enal   cr i si s.   In  addi ti on,   the  pati ent  m ust  be  i nstr ucted  to  w ear   an
i denti f i cati on  br acel et  or   car r y   a  car d  at  al l   ti m es  i ndi cati ng  that  he  has
the  di sease  and  needs  suppl em ental   ster oi ds  dur i ng  str ess.   Thi s  i s  a
cr uci al   l i f e­pr eser v i ng  m easur e  and  cannot  be  ov er em phasi zed.

Suggested Readings
Annane  D,   Sebi l l e  V,   Char penti er   C,   et  al .   Ef f ect  of   tr eatm ent  w i th  l ow
doses  of   hy dr ocor ti sone  and  f l udr ocor ti sone  on  m or tal i ty   i n  pati ents  w i th
septi c  shock .   JAMA  2002;288(7):862–871.

Ar af ah  BM.   Medi cal   m anagem ent  of   hy popi tui tar i sm   i n  pati ents  w i th
pi tui tar y   adenom as.   Pi tui tar y   2002;5:109–117.

Knapp  PE,   Ar um   SM,   Mel by   JC.   Rel ati v e  adr enal   i nsuf f i ci ency   i n  cr i ti cal
i l l ness:  a  r ev i ew   of   the  ev i dence.   Cur r   Opi n  Endocr i nol   Di abetes
2004;11:147–152.

Li ndsay   JR,   N i em an  LK.   The  hy pothal am i c­pi tui tar y ­adr enal   ax i s  i n
pr egnancy :  chal l enges  i n  di sease  detecti on  and  tr eatm ent.   Endocr   Rev
2005;26:775–799.
N i em an  LK.   Dy nam i c  ev al uati on  of   adr enal   hy pof uncti on.   J  Endocr i nol
Inv est  2003;26(7  Suppl ):74–82.

Schr i er   RW.   Cur r ent  m edi cal   ther apy ,   2nd  ed.   N ew   Yor k :  Rav en  Pr ess,
1989.

Wi l son  JD,   Foster   DW,   eds.   Wi l l i am s'  tex tbook   of   endocr i nol ogy ,   8th  ed.
Phi l adel phi a:  WB  Saunder s,   1992.

Cushing's Syndrome
1.   What  i s  the  di f f er ence  betw een  Cushi ng's  sy ndr om e  and  Cushi ng's
di sease?

2.   What  i s  the  m ost  com m on  cause  of   Cushi ng's  sy ndr om e?

3.   What  ar e  the  cl i ni cal   f eatur es  of   Cushi ng's  sy ndr om e?

4.   What  ar e  the  bi ol ogi c  ef f ects  of   gl ucocor ti coi ds?

5.   What  ar e  the  scr eeni ng  tests  used  to  di agnose  Cushi ng's  sy ndr om e?

P. 79

Discussion
1.   What  i s  the  di f f er ence  betw een  Cushi ng's  sy ndr om e  and  Cushi ng's
di sease?

Cushi ng's  sy ndr om e  r ef er s  to  the  phenoty pi c  and  cl i ni cal   sequel ae  due  to
hy per cor ti sol i sm   r esul ti ng  f r om   any   cause.   Cushi ng's  di sease  r ef er s
speci f i cal l y   to  the  hy per cor ti sol i sm   due  to  an  ACTH­secr eti ng  pi tui tar y
cor ti cotr oph  adenom a  or   pi tui tar y   cor ti cotr oph  hy per pl asi a.

2.   What  i s  the  m ost  com m on  cause  of   Cushi ng's  sy ndr om e?

The  w i despr ead  use  of   potent  cor ti coster oi ds  i n  the  pr acti ce  of   cl i ni cal
m edi ci ne,   par ti cul ar l y   i n  the  tr eatm ent  of   autoi m m une,   al l er gi c,   and
pul m onar y   di sor der s,   has  m ade  i atr ogeni c  hy per cor ti sol i sm   the  m ost
com m on  cause  of   Cushi ng's  sy ndr om e.   How ev er ,   once  the  i atr ogeni c
causes  ar e  el i m i nated,   pi tui tar y   adenom a  (68%)  becom es  the  m ost
com m on  cause.

3.   What  ar e  the  cl i ni cal   f eatur es  of   Cushi ng's  sy ndr om e?

Cushi ng's  sy ndr om e  i s  associ ated  w i th  m any   cl i ni cal   f eatur es.   Obe s ity,
f ound  i n  94%,   i s  the  m ost  com m on  m ani f estati on  and  w ei ght  gai n  i s
usual l y   the  ear l i est  sy m ptom   of   Cushi ng's  sy ndr om e.   The  obesi ty   tends  to
be  centr al ,   but  f at  can  al so  be  r edi str i buted  to  the  f ace  (m oon  f aci es;
75%),   as  w el l   as  supr acl av i cul ar   (80%)  and  dor socer v i cal   ar eas  (â
€œbuf f al o  hum pâ€​ ;   80%).   The  l atter   tw o  ar eas,   par ti cul ar l y   the
supr acl av i cul ar   f at  pad,   ar e  m or e  speci f i c  f i ndi ngs  f or   Cushi ng's
sy ndr om e.

Sk in c ha nge s   occur   i n  85%  of   the  pati ents  and  ar i se  because  cor ti sol ­
i nduced  atr ophy   of   the  epi der m i s  l eads  to  thi nni ng  and  a  tr anspar ent
appear ance  of   the  sk i n,   f aci al   pl ethor a,   easy   br ui sabi l i ty ,   and  the
f or m ati on  of   str i ae.   The  l atter   ar e  pur pl i sh  r ed  ar eas  that  ar e  depr essed
bel ow   the  sk i n  sur f ace,   but  ar e  w i der   than  the  pi nk i sh  w hi te  str i ae  that
appear   af ter   pr egnancy   or   w ei ght  l oss.   Wounds  heal   sl ow l y   i n  these
pati ents  and  m ay   dehi sce.   Hy per pi gm entati on  occur s  i n  the  setti ng  of   the
ectopi c  ACTH  sy ndr om e,   but  i s  r ar e  i n  pati ents  w i th  Cushi ng's  di sease  and
shoul d  not  be  f ound  i n  those  w i th  pr i m ar y   adr enal   Cushi ng's  sy ndr om e.
Acne  (40%)  i s  al so  a  sy m ptom   and  i s  due  to  andr ogen  ex cess;  i t  m ay   be
m or e  gener al i zed  than  w hat  the  pati ent  ex per i enced  bef or e.

Hirs utis m  af f ects  80%  of   the  pati ents  and  ty pi cal l y   consi sts  of   a
dar k eni ng  and  coar seni ng  of   the  hai r .   Fem al e  pati ents  com pl ai n  of
i ncr eased  gr ow th  of   hai r   ov er   the  f ace,   upper   thi ghs,   abdom en,   and
br easts.   Vi r i l i sm   occur s  i n  appr ox i m atel y   20%  of   the  cases  of   adr enal
car ci nom a.   Hype rte ns ion  i s  a  pr obl em   i n  75%  of   the  pati ents.   El ev ated
di astol i c  BP  i s  a  cl assi c  f eatur e  of   spontaneous  Cushi ng's  sy ndr om e,   and
i t  contr i butes  gr eatl y   to  the  m or bi di ty   and  m or tal i ty   associ ated  w i th  the
di sor der .   The  i ncr eased  sodi um   r etenti on  al so  l eads  to  edem a  (18%).
Congesti v e  hear t  f ai l ur e  (22%)  can  be  aggr av ated  because  of   the
i ncr eased  BP  and  f l ui d  l oad.

G ona da l dys func tion  occur s  i n  75%  of   the  pati ents.   El ev ated  andr ogen
l ev el s  can  r esul t  i n  am enor r hea  and  i nf er ti l i ty   i n  75%  of   af f ected
pr em enopausal   w om en.   In  m en,   the  el ev ated  cor ti sol   l ev el   m ay   cause  a
decr ease  i n  l i bi do.

P. 80
Mus c le  w e a k ne s s   ar i ses  i n  60%  of   pati ents,   i n  par ti cul ar   pr ox i m al
w eak ness  that  m ost  of ten  occur s  i n  the  l ow er   ex tr em i ti es.   Thi s  w eak ness
stem s  f r om   the  catabol i c  ef f ects  of   gl ucocor ti coi ds  on  m uscl e  ti ssues,
ster oi d­i nduced  m y opathy ,   and  possi bl y   el ectr ol y te  i m bal ances.   Weak ness
can  be  assessed  cl i ni cal l y   by   ask i ng  the  pati ent  to  stand  f r om   the  chai r
w i thout  assi stance  of   ar m s.   Radi ogr aphi cal l y   detectabl e  os te oporos is   i s
pr esent  i n  m ost  pati ents  w i th  Cushi ng's  sy ndr om e  (60%),   and  back   pai n  i s
the  i ni ti al   com pl ai nt  i n  58%  of   the  cases.   Pathol ogi c  f r actur es  ar e  f ound
i n  the  r i bs  and  v er tebr ae  i n  sev er e  cases.   It  tak es  som e  ti m e  f or   the
hy per cor ti sol i sm   to  decal ci f y   bone;  ther ef or e,   Cushi ng's  sy ndr om e  due  to
adr enal   car ci nom a  and  som e  ectopi c  ACTH  cases  i s  not  pr esent  l ong
enough  to  cause  osteopor osi s.

P s yc hologic a l dis turba nc e s   can  ar i se  i n  40%  of   pati ents.   These


com pl ai nts  r ange  f r om   m i l d  sy m ptom s,   such  as  em oti onal   l abi l i ty ,
i ncr eased  i r r i tabi l i ty ,   anx i ety ,   i nsom ni a,   euphor i a,   poor   concentr ati on,
poor   m em or y ,   and  m i l d  depr essi on,   to  sev er e  sy m ptom s,   w hi ch  i ncl ude
f r ank   psy chosi s  associ ated  w i th  del usi ons  or   hal l uci nati ons,   par anoi a,
sev er e  depr essi on,   and  ev en  sui ci dal   behav i or .

Re na l c a lc uli  f or m   i n  15%  of   pati ents  as  a  r esul t  of   gl ucocor ti coi d­


i nduced  hy per cal cem i a.   Renal   col i c  m ay   be  the  pr esenti ng  sy m ptom   of
Cushi ng's  sy ndr om e.   Thirs t  and  polyuria   ar e  seen  i n  10%  of   pati ents.
The  thi r st  i s  due  to  gl ucocor ti coi d­i nduced  hy per gl y cem i a  [or   w or seni ng
of   ex i sti ng  di abetes  m el l i tus  (DM)]  that  causes  an  osm oti c  (gl ucose)
di ur esi s.   Di abeti c  k etoaci dosi s  (DKA)  and  di abeti c  m i cr ov ascul ar
com pl i cati ons  ar e  r ar e  i n  the  di abetes  seen  w i th  Cushi ng's  sy ndr om e.

4.   What  ar e  the  bi ol ogi c  ef f ects  of   gl ucocor ti coi ds?

Fr om   a  mole c ula r  per specti v e,   gl ucocor ti coi d  hor m ones  enter   the  cel l   by
di f f usi on  and  acti v ate  speci f i c  gene  tr anscr i pti on  by   bi ndi ng  to  the
nucl ear   gl ucocor ti coi d  r eceptor .   The  gl ucocor ti coi d  r eceptor   i s  ther ef or e  a
condi ti onal   tr ansacti v ator   that  i nf l uences  the  r ate  of   RN A  pol y m er ase  II
tr anscr i pti on  i ni ti ati on  by   bi ndi ng  to  speci f i c  shor t  DN A  sequence
el em ents  (gl ucocor ti coi d  r esponse  el em ents)  i n  the  pr om oter   r egul ator y
r egi on  of   the  v ar i ous  tar get  genes.   Al though  thi s  i s  the  best­establ i shed
pathw ay   of   gl ucocor ti coi d  acti on,   other   m echani sm s  that  m edi ate  the
r api d  ef f ects  of   gl ucocor ti coi ds,   such  as  the  f ast­f eedback   i nhi bi ti on  of
ACTH  secr eti on  and  possi bl y   m odul ati on  of   the  γ­am i nobuty r i c  aci d
r eceptor ,   m ust  al so  ex i st.

In  ter m s  of   thei r   ef f ects  on  me ta bolis m,   gl ucocor ti coi ds  accel er ate
hepati c  gl uconeogenesi s  by   sti m ul ati ng  phosphoenol py r uv ate
car box y k i nase  and  gl ucose­6­phosphatase  acti v i ty ,   and  i nduce  a
per m i ssi v e  ef f ect  i n  other   gl uconeogeni c  hor m ones  (gl ucagon  and
catechol am i nes).   Gl ucocor ti coi ds  al so  enhance  hepati c  gl y cogen  sy nthesi s
and  stor age  and  i nhi bi t  gl y cogen  br eak dow n.   In  m uscl es,   gl ucocor ti coi ds
i nhi bi t  am i no  aci d  uptak e  and  pr otei n  sy nthesi s  and  sti m ul ate  pr otei n
br eak dow n  as  w el l   as  the  r el ease  of   am i no  aci ds,   l actate,   f r ee  f atty   aci ds
(FFAs),   and  gl y cer ol .   In  adi pose  ti ssue,   gl ucocor ti coi ds  pr i m ar i l y
accel er ate  l i pol y si s,   w i th  a  r esul tant  r el ease  i n  the  f or m ati on  of   gl y cer ol
and  FFAs.   Al though  gl ucocor ti coi ds  ar e  l i pol y ti c,   an  i ncr eased  centr al   f at
deposi ti on  i s  a
P. 81
cl assi c  f eatur e  of   Cushi ng's  sy ndr om e.   The  ster oi d­i nduced  i ncr ease  i n
appeti te  and  hy per i nsul i nem i a  m ay   account  f or   thi s,   but  the  basi s  f or   thi s
abnor m al   f at  deposi ti on  i n  the  setti ng  of   hy per cor ti sol i sm   r em ai ns
unk now n.

5.   What  ar e  the  scr eeni ng  tests  used  to  di agnose  Cushi ng's  sy ndr om e?

A  k ey   aspect  of   the  i ni ti al   w or k up  i n  a  pati ent  w i th  suspected  Cushi ng's


sy ndr om e  i s  to  di sti ngui sh  tr ue  hy per cor ti sol i sm   f r om   obesi ty ,
depr essi on,   or   al cohol i sm ,   or   a  com bi nati on  of   these,   because  m any
cl i ni cal   and  l abor ator y   f eatur es  of   these  di sor der s  di spl ay   si gni f i cant
ov er l ap.   A  k ey   ai d  i n  establ i shi ng  the  cl i ni cal   di agnosi s  of
hy per cor ti sol i sm   i s  ex am i ni ng  the  pati ent's  sequenti al   photogr aphs  that
span  sev er al   y ear s.   Once  Cushi ng's  sy ndr om e  i s  suspected  on  cl i ni cal
gr ounds,   the  ov er ni ght  1­m g  dex am ethasone  suppr essi on  test  (DST)  and
the  24­hour   ur i nar y   f r ee  cor ti sol   (U FC)  deter m i nati on  ar e  used  as
scr eeni ng  tests.   If   the  r esul ts  of   the  ov er ni ght  1­m g  DST  ar e  nor m al   (8
a. m .   pl asm a  cor ti sol   < 2  µg/dL  af ter   the  adm i ni str ati on  of   1  m g  of
dex am ethasone  at  11  p. m .   the  ni ght  bef or e),   the  di agnosi s  i s  v er y
unl i k el y .   If   the  r esul ts  of   the  U FC  test  ar e  al so  nor m al   (i . e. ,   < 90  to  100
µg  per   day ),   Cushi ng's  sy ndr om e  i s  ef f ecti v el y   ex cl uded.   A  thi r d,
r ecentl y   av ai l abl e,   but  not  y et  w i del y   accepted  scr eeni ng  test  i s  the  l ate
ni ght  sal i v ar y   cor ti sol .   Thi s  test  tak es  adv antage  of   the  l oss  of   nor m al
ci r cadi an  v ar i ati on  i n  cor ti sol   l ev el   i n  Cushi ng's  di sease  by   m easur i ng
cor ti sol   at  a  ti m e  w hen  i t  i s  nor m al l y   v i r tual l y   absent.   Sev er al   si tuati ons
can  cause  f al se­posi ti v e  r esul ts  f or   the  scr eeni ng  DST,   i ncl udi ng  acute
and  chr oni c  i l l ness,   obesi ty ,   hi gh­estr ogen  states,   cer tai n  dr ugs
(pheny toi n  and  phenobar bi tal ),   al cohol i sm ,   anor ex i a,   r enal   f ai l ur e,   and
depr essi on.   How ev er ,   i n  the  setti ng  of   obesi ty ,   hi gh­estr ogen  states,   and
cer tai n  dr ugs,   the  r esul ts  of   a  24­hour   U FC  ar e  nor m al .   In  the  other
si tuati ons,   r epeated  testi ng  i s  necessar y   to  ex cl ude  the  di agnosi s.   Rar el y
f al se­negati v e  r esul ts  can  occur ,   such  as  i n  the  ev ent  of   pr ol onged
dex am ethasone  cl ear ance  or   epi sodi c  hy per cor ti sol i sm .

Case
A  36­y ear ­ol d  w hi te  w om an  com es  to  y ou  com pl ai ni ng  of   f ati gue,   i r r i tabi l i ty ,
depr essi on,   and  a  30­l b  (13. 5­k g)  w ei ght  gai n  ov er   the  l ast  2  y ear s.   She
r ecounts  that  she  has  noti ced  a  si gni f i cant  change  i n  her   ener gy   l ev el   f or   at
l east  the  l ast  2  y ear s.   She  states  that  she  has  al w ay s  been  a  har d  w or k er   but
6  m onths  bef or e  she  had  to  qui t  her   job  as  a  w ai tr ess  because  of   ex tr em e
m uscl e  w eak ness  and  f ati gue.   She  has  al so  noted  i ncr eased  m ood  sw i ngs,
m ani f ested  by   i ncr eased  i r r i tabi l i ty ,   spontaneous  cr y i ng  epi sodes,   and
depr essi on.   She  r epor ts  that  her   f ace  seem s  r ounder   than  i t  w as  2  y ear s
bef or e.   On  f ur ther   questi oni ng,   she  adm i ts  that  her   m enstr ual   per i ods  hav e
been  i r r egul ar   f or   the  l ast  2  y ear s.   She  al so  adm i ts  to  dr i nk i ng  “a  si x ­pack
of   beer â€​  at  l east  once  a  w eek ,   but  deni es  sm ok i ng.   She  has  al so  noted  that
she  br ui ses  easi l y .   She  deni es  any   other   m edi cal   pr obl em s,   and  states  that
she  i s  not  tak i ng  any   m edi cati ons.   She  speci f i cal l y   deni es  any   gl ucocor ti coi d
ther apy .   On  ask i ng  about  her   f am i l y   hi stor y ,   y ou  f i nd  out  that  her   m other   has
adul t­onset  DM.
Phy si cal   ex am i nati on  r ev eal s  an  obese  w hi te  w om an  w ho  i s  cr y i ng  w hi l e  she
si ts  on  the  ex am i ni ng  tabl e,   but  other w i se  she  does  not  appear   to  be  v er y   i l l .
Her   w ei ght  i s  193  l b  (87  k g);  hei ght,   5  f t  7  i n.   (167. 5  cm );  BP,   165/100  m m
Hg;  and  hear t  r ate,   86  per   m i nute  and  r egul ar .
P. 82
Her   f ace  i s  v er y   r ound  and  pl ethor i c  com par ed  w i th  that  i n  ol d  photogr aphs.
Dor socer v i cal   (buf f al o  hum p)  and  supr acl av i cul ar   f at  pads  ar e  noted.   She  has
m i l d  f aci al   hi r suti sm ,   som e  acne  i s  noted  ov er   the  f ace  and  chest,   and  w i de
pur pl e  str i ae  ar e  pr esent  on  the  l ow er   abdom en.   Her   ex tr em i ti es  ar e  thi n  and
she  has  pr ox i m al   m uscl e  w eak ness.
The  f ol l ow i ng  ar e  the  l abor ator y   f i ndi ngs:  f asti ng  bl ood  gl ucose,   180  m g/dL;
potassi um ,   3  m Eq/L;  HCO 3 ­ ,   34  m Eq/L;  l i v er   f uncti on  tests,   al l   nor m al ;  8  a. m .
cor ti sol ,   38  µg/dL,   w hi ch  decr eases  to  32  µg/dL  af ter   the  adm i ni str ati on  of   1
m g  of   dex am ethasone.   The  24­hour   U FC  l ev el   i s  876  µg.

1.   What  i s  the  m ost  l i k el y   di agnosi s  i n  thi s  pati ent,   and  w hy ?


2.   What  studi es  w oul d  y ou  per f or m   to  establ i sh  the  anatom i c  cause  of   her
hy per cor ti sol i sm ?
3.   What  i s  the  r ol e  of   m agneti c  r esonance  i m agi ng  (MRI)  and  com puted
tom ogr aphi c  (CT)  scanni ng  of   the  pi tui tar y   and  adr enal   gl ands,   as  w el l   as
i nf er i or   petr osal   si nus  sam pl i ng,   i n  pati ents  w i th  Cushi ng's  sy ndr om e?
4.   What  i s  the  opti m al   ther apeuti c  appr oach  f or   thi s  pati ent?
5.   Why   i s  ther e  a  need  f or   ster oi d  ther apy   i n  the  postoper ati v e  per i od,   and
som eti m es  bey ond,   i n  pati ents  w i th  Cushi ng's  di sease?

Case Discussion
1.   What  i s  the  m ost  l i k el y   di agnosi s  i n  thi s  pati ent,   and  w hy ?

Hav i ng  ex cl uded  ex ogenous  gl ucocor ti coi d  m edi cati ons  i n  the  hi stor y ,   the
di f f er enti al   di agnosi s  l i st  w oul d  i ncl ude  (a)  pi tui tar y   cor ti cotr oph
adenom a  or   hy per pl asi a  (Cushi ng's  di sease),   (b)  ectopi c  ACTH  or
cor ti cotr opi n­r el easi ng  f actor   (CRF)  sy ndr om e,   (c)  adr enal   adenom a,   (d)
adr enal   cancer ,   (e)  obesi ty ,   (f )  depr essi on,   and  (g)  al cohol i sm .

The  m ost  f r equentl y   encounter ed  di l em m a  i n  the  di f f er enti al   di agnosi s  of


Cushi ng's  sy ndr om e  i s  the  cl i ni cal   pi ctur e  consi sti ng  of   an  obese,
depr essed  pati ent  w ho  consum es  ex cessi v e  am ounts  of   al cohol .   These
pati ents  can  di spl ay   m any   of   the  phenoty pi c  f eatur es  and  l abor ator y
f i ndi ngs  consi stent  w i th  hy per cor ti sol i sm ,   and  y et  not  hav e  Cushi ng's
sy ndr om e.   Ther ef or e,   the  pati ent's  hi stor y   of   consum i ng  a  si x ­pack   of
beer   per   w eek   i s  of   concer n  because  thi s  coul d  pr oduce  al cohol i c  pseudo­
Cushi ng's  sy ndr om e.   In  thi s  di sor der ,   the  ef f ects  of   chr oni c  al cohol i sm
r esul t  i n  centr al   obesi ty   (asci tes),   a  r ound  pl ethor i c  f ace,   easy   br ui si ng,
and  som e  abnor m al   r esul ts  f r om   the  scr eeni ng  tests  f or   Cushi ng's
sy ndr om e.   How ev er ,   thi s  pati ent  has  no  abnor m al   l i v er   f uncti on  f i ndi ngs
and  she  has  phy si cal   f i ndi ngs  (a  m ar k ed  change  i n  her   f aci al   appear ance
com par ed  w i th  that  i n  ol d  photogr aphs,   hy per tensi on,   dor socer v i cal   and
supr acl av i cul ar   f at  pads,   pur pl e  abdom i nal   str i ae,   acne,   and  hi r suti sm )
and  l abor ator y   data  (hy per gl y cem i a,   hy pok al em i a,   an  el ev ated  basal
cor ti sol   l ev el   that  does  not  suppr ess  i n  r esponse  to  the  1­m g  DST,   and  an
el ev ated  24­hour   U FC)  that  ar e  al l   hi ghl y   consi stent  w i th  the  cl i ni cal
suspi ci on  of   hy per cor ti sol i sm .

The  l ack   of   v i r i l i zati on  and  the  r el ati v el y   sl ow   (> 2  y ear s)  onset  of   the
cl i ni cal   sy m ptom s  ar gue  agai nst  adr enal   car ci nom a.   In  addi ti on,   the  l ack
of   a  sm ok i ng  hi stor y   and  any   hy per pi gm entati on,   together   w i th  the  sl ow
onset,   suggest  that
P. 83
ectopi c  ACTH  ar i si ng  f r om   sm al l   cel l   l ung  car ci nom a  i s  unl i k el y   to  be  the
cause.   Thi s  l eav es  pi tui tar y   adenom a  (or   hy per pl asi a),   ectopi c  ACTH  or
CRF  (f r om   a  car ci noi d,   pancr eati c  i sl et  cel l   tum or ,   m edul l ar y   thy r oi d
car ci nom a,   or   pheochr om ocy tom a),   and  adr enal   adenom a  i n  the
di f f er enti al   di agnosi s.   Gi v en  that  pi tui tar y   adenom as  consti tute  68%  of   al l
noni atr ogeni c  causes  of   hy per cor ti sol i sm ,   thi s  i s  the  m ost  l i k el y
di agnosi s.   How ev er ,   f ur ther   w or k up  i s  r equi r ed  to  docum ent  the  pr eci se
sour ce  of   the  el ev ated  cor ti sol   l ev el s  i n  thi s  pati ent.

2.   What  studi es  w oul d  y ou  per f or m   to  establ i sh  the  anatom i c  cause  of   her
hy per cor ti sol i sm ?

Once  the  di agnosi s  of   hy per cor ti sol i sm   (Cushi ng's  sy ndr om e)  has  been
conf i r m ed  by   the  f i ndi ngs  of   the  cl i ni cal   ev al uati on  and  scr eeni ng
l abor ator y   tests,   the  com bi ned  use  of   the  f ol l ow i ng  di agnosti c  techni ques
can  establ i sh  the  di agnosi s  i n  al m ost  al l   i nstances:  deter m i nati on  of   a
basal   pl asm a  ACTH  l ev el ,   a  hi gh­dose  (8  m g)  DST,   r adi ogr aphi c  i m agi ng,
and  i nf er i or   petr osal   sam pl i ng  (w i th  or   w i thout  CRF  sti m ul ati on).   By
si m ul taneousl y   m easur i ng  the  pl asm a  cor ti sol   and  ACTH  l ev el s  the
possi bi l i ty   of   an  adr enal   adenom a  can  be  assessed  because  the
autonom ous  pr oducti on  of   gl ucocor ti coi ds  by   the  adr enal   adenom a
suppr esses  ACTH  to  l ev el s  bel ow   20  pg/m L.   To  di f f er enti ate  betw een  a
pi tui tar y   adenom a  and  the  ectopi c  tum or   pr oducti on  of   ACTH,   sev er al
tests  need  to  be  per f or m ed  because  m any   of   the  l abor ator y   and
r adi ogr aphi c  r esul ts  can  ov er l ap  f or   these  tw o  di sti nct  causes  of   Cushi ng's
sy ndr om e.   For   ex am pl e,   the  ACTH  l ev el   can  r ange  betw een  40  and  200
pg/m L  i n  the  setti ng  of   Cushi ng's  di sease  and  betw een  100  and  10, 000
pg/m L  i n  the  setti ng  of   ectopi c  ACTH.   In  the  cl assi c  2­day   hi gh­dose  DST
(2  m g  of   dex am ethasone  i s  gi v en  ev er y   6  hour s  f or   2  day s,   and  24­hour
U FC  sam pl es  ar e  col l ected  the  day   bef or e  and  on  the  second  day   of
dex am ethasone  adm i ni str ati on),   pati ents  w i th  pi tui tar y   tum or s  (Cushi ng's
di sease)  ty pi cal l y   ex hi bi t  a  suppr essi on  to  l ess  than  50%  of   basel i ne
v al ues;  those  w i th  ectopi c  ACTH  or   pr i m ar y   adr enal   hy per cor ti sol i sm
di spl ay   l i ttl e  or   no  r educti on.   How ev er ,   som e  car ci noi d  tum or s  that
pr oduce  ACTH  ectopi cal l y   m ai ntai n  som e  degr ee  of   negati v e  f eedback
thr ough  the  i nf l uence  of   ex ogenous  ster oi ds,   and  the  suppr essi on
obser v ed  m ay   be  equi v al ent  to  that  seen  i n  pati ents  w i th  pi tui tar y
tum or s.

The  abbr ev i ated  hi gh­dose  DST  i nv ol v es  adm i ni ster i ng  8  m g  of


dex am ethasone  at  11  p. m .   the  ni ght  bef or e  and  m easur i ng  the  pl asm a
cor ti sol   l ev el   the  nex t  m or ni ng  at  8  a. m .   In  thi s  test,   a  suppr essi on  bel ow
50%  of   basal   pl asm a  cor ti sol   l ev el s  i s  seen  i n  pati ents  w i th  pi tui tar y
tum or ,   but  not  i n  those  w i th  ectopi c  ACTH  and  pr i m ar y   adr enal
cor ti sol i sm .   Thi s  v er si on  of   the  hi gh­dose  DST  i s  pr ef er r ed  because  i t
appear s  to  be  m or e  speci f i c  and  does  not  r equi r e  tw o  24­hour   ur i ne
col l ecti ons.   For   a  m or e  pr eci se  def i ni ti on  of   the  cause  of   the  di sor der ,
how ev er ,   speci f i c  r adi ogr aphi c  pr ocedur es  m ust  be  per f or m ed.

3.   What  i s  the  r ol e  of   MRI  and  CT  scanni ng  of   the  pi tui tar y   and  adr enal
gl ands,   as  w el l   as  i nf er i or   petr osal   si nus  sam pl i ng,   i n  pati ents  w i th
Cushi ng's  sy ndr om e?

The  m ajor   pr obl em   w i th  the  CT  and  MRI  ev al uati on  of   the  pi tui tar y   and
adr enal   gl ands  i s  that  they   can  detect  asy m ptom ati c  l esi ons  i n  up  to  15%
and  8%,   r especti v el y ,   of   the  nor m al   popul ati on.   Because  of   thi s  i nci dence
of   nonspeci f i c  r adi ogr aphi c  “l esi onsâ€​ ,   the  cl i ni ci an  m ust  be  cauti ous
about  basi ng  the  di agnosi s  of   pi tui tar y   or   adr enal   Cushi ng's  sy ndr om e  on
the  r esul ts  of   these  i m agi ng  studi es.

P. 84
P ituita ry a de noma s   causi ng  Cushi ng's  di sease  tend  to  be  sm al l   (1  to  5
m m ;  r ar el y   > 10  m m ),   and  ar e  ther ef or e  detectabl e  by   contr ast­enhanced
CT  scanni ng  i n  as  f ew   as  30%  to  35%  of   cases  and  by   gadol i ni um â
€“DTPA­enhanced  MRI  i n  55%  of   cases.   Ther ef or e,   because  of   i ts  better
sensi ti v i ty ,   MRI  has  r epl aced  CT  i n  the  assessm ent  of   these  tum or s.
Pati ents  w hose  i m agi ng  studi es  y i el d  negati v e  f i ndi ngs  need  to  under go
i nf er i or   petr osal   sam pl i ng  to  f ur ther   docum ent  the  pi tui tar y   anatom i c
l ocati on  of   the  tum or .   In  addi ti on,   as  al r eady   di scussed,   ev en  i f   an
abnor m al i ty   i s  detected  by   these  i m agi ng  m ethods,   thi s  does  not
consti tute  unequi v ocal   ev i dence  that  the  abnor m al i ty   i s  r esponsi bl e  f or
the  sy ndr om e.   As  the  r esol uti on  of   CT  and  MRI  i m pr ov es,   the  abi l i ty   to
detect  these  “i nci dental â€​   and  cl i ni cal l y   si l ent  m i cr oadenom as  w i l l
al so  i ncr ease  and  f ur ther   conf ound  the  di agnosti c  w or k up.   An  ectopi c  CRF
sy ndr om e  coul d  al so  r esul t  i n  an  enl ar ged  pi tui tar y   due  to  cor ti cotr oph
hy per pl asi a,   and  y et  the  pr i m ar y   di sor der   m ay   actual l y   be  a  car ci noi d  of
the  l ung.

CT,   MRI,   ul tr asonogr aphy ,   and  i sotope  scanni ng  w i th  i odochol ester ol   can
be  used  to  def i ne  the  natur e  of   a dre na l le s ions .   These  pr ocedur es  ar e
not  necessar y   i n  pati ents  w i th  ACTH  hy per secr eti on,   how ev er .
N ev er thel ess,   som e  phy si ci ans  use  these  tests  to  ex cl ude  the  pr esence  of
a  sol i tar y   adr enal   adenom a  or   car ci nom a,   and  ther eby   conf i r m   the
pr esence  of   bi l ater al   adr enal   hy per pl asi a  or   nodul ar   adr enal   hy per pl asi a
i n  the  setti ng  of   pi tui tar y ­based  di sease.   These  pr ocedur es  ar e  m ost
usef ul   f or   l ocal i zi ng  adr enal   tum or s  because  these  tum or s  m ust  usual l y
be  l ar ger   than  1. 5  cm   to  cause  si gni f i cant  cor ti sol   pr oducti on  and  r esul t
i n  Cushi ng's  sy ndr om e.   How ev er ,   as  noted  pr ev i ousl y ,   because  of   the  1%
to  8%  i nci dence  of   si l ent  adr enal   nodul es  bi ochem i cal   testi ng  m ust  be
per f or m ed  w i th  l ocal i zati on  studi es  to  ensur e  that  the  l esi on  i denti f i ed  i s
bi ol ogi cal l y   si gni f i cant.

To  di sti ngui sh  betw een  the  v ar i ous  causes  of   Cushi ng's  sy ndr om e  w hen
conf l i cti ng  or   ov er l appi ng  data  ar e  obtai ned,   bi l ater al   si m ul taneous
infe rior pe tros a l ve nous  s a mpling  (w i th  or   w i thout  CRF  sti m ul ati on)  can
successf ul l y   di sti ngui sh  Cushi ng's  di sease  f r om   ectopi c  ACTH  secr eti on
and  adr enal   di sease  w i th  gr eater   accur acy   than  any   other   test.   Because
ACTH  i s  r api dl y   m etabol i zed  (hal f ­l i f e,   7  to  12  m i nutes)  and  i s  secr eted
epi sodi cal l y ,   adv antage  can  be  tak en  of   the  concentr ati on  gr adi ent
betw een  the  pi tui tar y   v enous  dr ai nage  thr ough  the  i nf er i or   petr osal   si nus
(centr al )  and  the  per i pher al   v enous  v al ues  of   ACTH  to  f ur ther   deter m i ne
w hether   an  ACTH­pr oduci ng  cor ti cotr oph  adenom a  i s  pr esent  i n  the
pi tui tar y ;  the  i ncl usi on  of   CRF  sti m ul ati on  m ak es  the  test  m or e  sensi ti v e.
Bi l ater al   and  si m ul taneous  i nf er i or   petr osal   si nus  sam pl es  ar e  obtai ned  to
ci r cum v ent  the  pr obl em   of   i sol ated  secr etor y   bur sts  or   ti m i ng  i ssues  i f
catheter s  hav e  to  be  r eposi ti oned.   Ther ef or e,   ACTH  sam pl es  ar e  obtai ned
f r om   the  i nf er i or   petr osal   si nus,   f r om   the  jugul ar   bul b,   and  f r om   other
si tes  (e. g. ,   super i or   or   i nf er i or   v ena  cav a),   and  the  f i ndi ngs  ar e
com par ed  w i th  those  f r om   si m ul taneousl y   obtai ned  per i pher al   v ei n
sam pl es.   In  pati ents  w i th  Cushi ng's  di sease,   the  i nf er i or   petr osal
si nus/per i pher al   (IPS  :  P)  r ati o  of   ACTH  ex ceeds  2.   In  pati ents  w i th
ectopi c  ACTH,   the  r ati o  i s  l ess  than  2  and  sel ecti v e  v enous  sam pl i ng
(e. g. ,   of   the  pul m onar y ,   pancr eati c,   or   i ntesti nal   beds)  m ay   l ocal i ze  the
ectopi c  tum or .   The  adm i ni str ati on  of   CRF  dur i ng  bi l ater al   i nf er i or   petr osal
si nus  sam pl i ng  can  i ncr ease  the  di agnosti c  accur acy   of   the  test  by
el i ci ti ng
P. 85
an  ACTH  r esponse  i n  the  f ew   pati ents  w i th  pi tui tar y   tum or s  w ho  do  not
ex hi bi t  a  di agnosti c  IPS  :  P  gr adi ent  i n  the  basal   sam pl es.   Most  pati ents
w i th  Cushi ng's  di sease  hav e  an  IPS  :  P  r ati o  gr eater   than  3  af ter   CRF
sti m ul ati on,   w her eas  pati ents  w i th  ectopi c  ACTH  or   adr enal   di sease  hav e
an  IPS  :  P  r ati o  of   ACTH  l ess  than  3  af ter   CRF  sti m ul ati on.   Inf er i or
petr osal   si nus  sam pl i ng  (w i th  or   w i thout  CRF  sti m ul ati on)  has  not  been
ex tensi v el y   studi ed  i n  the  contex t  of   heal thy   peopl e,   how ev er ,   and
ther ef or e  the  cor r ect  i nter pr etati on  of   the  r esul ts  r equi r es  that  the
pati ent  m ust  be  hy per cor ti sol em i c  at  the  ti m e  of   the  study   so  that  the
r esponse  of   nor m al   cor ti cotr ophs  to  CRF  i s  suppr essed.

4.   What  i s  the  opti m al   ther apeuti c  appr oach  f or   thi s  pati ent,   and  w hy ?

Once  the  tum or   has  been  l ocal i zed  to  the  pi tui tar y ,   the  nex t  goal   i s  to
sur gi cal l y   r em ov e  the  cor ti cotr oph  adenom a  usi ng  the  techni que  of
sel ecti v e  tr anssphenoi dal   sur ger y .   Because  the  tum or s  ar e  sm al l ,   i t
r equi r es  an  ex per i enced  neur osur geon  to  successf ul l y   i denti f y   and  r esect
the  adenom a.   Meti cul ous  ex pl or ati on  of   the  i ntr asel l ar   contents  i s
m andator y ,   and  any   i denti f i ed  adenom a  i s  sel ecti v el y   r em ov ed,   l eav i ng
the  r em ai ni ng  nor m al   pi tui tar y   i ntact.   If   the  tum or   cannot  be  i denti f i ed,   i t
i s  necessar y   to  per f or m   l ar ger   pi tui tar y   r esecti ons  and,   i n  som e  cases,   a
total   hy pophy sectom y   m ay   be  necessar y .   Tr anssphenoi dal   sur ger y   i s
successf ul   i n  appr ox i m atel y   85%  of   pati ents  w i th  m i cr oadenom as  (tum or
< 10  m m ),   and  sur gi cal   dam age  to  the  nor m al   anter i or   pi tui tar y   i s  r ar e.
The  m ajor   si de  ef f ects  of   the  pr ocedur e  i ncl ude  tr ansi ent  di abetes
i nsi pi dus,   cer ebr ospi nal   f l ui d  l eak ,   si nusi ti s,   and,   r ar el y ,   postoper ati v e
bl eedi ng.   Al l   pati ents  w i th  Cushi ng's  di sease  w ho  ar e  successf ul l y   tr eated
w i th  tr anssphenoi dal   sur ger y   becom e  adr enal l y   i nsuf f i ci ent  f or   v ar i abl e
per i ods  of   ti m e  and  m ust  r ecei v e  r epl acem ent  doses  of   gl ucocor ti coi ds
(see  questi on  5  w hi ch  f ol l ow s).

The  success  r ates  f or   tr anssphenoi dal   sur ger y   dr op  dr asti cal l y   (15%  to
25%)  i n  the  setti ng  of   l ar ge  (> 10  m m )  tum or s,   l ocal l y   i nv asi v e  tum or s,
tum or s  not  i denti f i ed  at  sur ger y ,   and  cor ti cotr oph  hy per pl asi a.   In  these
i nstances,   adjuncti v e  r adi ati on  ther apy   i s  usual l y   adm i ni ster ed.   How ev er ,
the  m ajor   pr obl em   w i th  r adi ati on  ther apy   i s  the  l ag  ti m e  (6  to  12  m onths)
f or   i t  to  tak e  ef f ect  and  the  10%  to  20%  i nci dence  of   hy popi tui tar i sm   and
v i sual   f i el d  def i ci ts,   ev en  bl i ndness,   that  m ay   ev entuate.   A  new er   opti on
i s  the  m or e  pr eci se  ster eotacti c  r adi osur ger y   usi ng  the  gam m a  k ni f e  or
photon  k ni f e.   Ri sk   of   v i sual   com pl i cati ons  i s  l ar gel y   el i m i nated,   and  the
r i sk   of   pi tui tar y   def i ci ency   i s  r educed.

5.   Why   i s  ther e  a  need  f or   ster oi d  ther apy   i n  the  postoper ati v e  per i od  and
bey ond,   i n  pati ents  w i th  Cushi ng's  di sease?

The  successf ul   sur gi cal   r em ov al   of   the  ACTH­pr oduci ng  pi tui tar y
m i cr oadenom a  el i m i nates  the  dr i v e  f or   adr enal   gl ucocor ti coi d  pr oducti on
and  r ender s  the  pati ent  dependent  on  the  r em ai ni ng  nor m al   cor ti cotr ophs.
How ev er ,   because  these  cel l s  hav e  been  suppr essed  f or   y ear s  by   the
ex cess  cor ti sol   they   ar e  dor m ant.   Ther ef or e,   those  pati ents  w i th
Cushi ng's  di sease  w ho  hav e  been  successf ul l y   tr eated  ex per i ence
tr ansi ent  (1  to  18  m onths)  adr enocor ti cal   i nsuf f i ci ency   and  r equi r e
ex ogenous  gl ucocor ti coi d  suppor t;  i n  those  pati ents  not  cur ed  by   the
sur gi cal   pr ocedur e,   the  pr oducti on  of   ex cessi v e  am ounts  of
gl ucocor ti coi ds  conti nues  and  they   do  not  depend  on  an  ex ogenous  sour ce
of   ster oi ds.

P. 86

Suggested Readings
Fel i g  P,   Bax ter   JD,   Br oadus  AE,   et  al .   eds.   Di seases  of   the  anter i or
pi tui tar y .   In:  Endocr i nol ogy   and  m etabol i sm ,   2nd  ed.   N ew   Yor k :  McGr aw ­
Hi l l ,   1987.

Fi ndl i ng  JW,   Raf f   H.   Scr eeni ng  and  di agnosi s  of   Cushi ng's  sy ndr om e.
Endocr i nol   Metab  Cl i n  N or th  Am   2005;34:385–402.

Mansm ann  G,   Lau  J,   Bal k   E,   et  al .   The  cl i ni cal l y   i nappar ent  adr enal   m ass:
update  i n  di agnosi s  and  m anagem ent.   Endocr   Rev   2004;25(2):309–340.

N ew el l ­Pr i ce  J,   Tr ai ner   P,   Besser   M,   et  al .   The  di agnosi s  and  di f f er enti al


di agnosi s  of   Cushi ng's  sy ndr om e  and  pseudo­Cushi ng's  states.   Endocr   Rev
1998;19:657.

Schuf f   KG.   Issues  i n  the  di agnosi s  of   Cushi ng's  sy ndr om e  f or   the  pr i m ar y


car e  phy si ci an.   Pr i m   Car e  Of f i ce  Pr act  2003;30:791–799.

Ty r r el l   JB,   Ron  DC,   For sham   PH.   Gl ucocor ti coi ds  and  adr enal   andr ogens.
In:  Gr eenspan  FS,   ed.   Basi c  and  cl i ni cal   endocr i nol ogy ,   3r d  ed.   N or w al k ,
CT:  Appl eton  &  Lange,   1991.
Xi ao  XR,   Ye  LY,   Shi   LX,   et  al .   Di agnosi s  and  tr eatm ent  of   adr enal   tum our s:
a  r ev i ew   of   35  y ear s'  ex per i ence.   Br   J  U r ol   1998;82:199.

Diabetes Mellitus
1.   What  ar e  the  cl i ni cal   m ani f estati ons  of   DM?

2.   What  ar e  the  m ajor   ty pes  of   DM  and  w hat  ar e  thei r   di sti ngui shi ng
f eatur es?

3.   What  ar e  the  m ajor   acute  and  chr oni c  com pl i cati ons  of   the  di sease?

4.   What  aspects  of   the  m edi cal   hi stor y   r equi r e  speci al   em phasi s?

5.   What  aspects  of   the  phy si cal   ex am i nati on  r equi r e  speci al   attenti on?

6.   What  l abor ator y   tests  ar e  essenti al   i n  the  ev al uati on  of   the  pati ent  w i th
suspected  di abetes?

7.   What  ar e  the  goal s  of   di abetes  ther apy   and  w hat  tr eatm ent  m odal i ti es  ar e
av ai l abl e?  How   shoul d  these  be  i ndi v i dual i zed?

Discussion
1.   What  ar e  the  cl i ni cal   m ani f estati ons  of   DM?

DM  i s  a  com pl ex   m etabol i c  di sor der   char acter i zed  by   abnor m al i ti es  of
car bohy dr ate,   l i pi d,   and  pr otei n  m etabol i sm   r esul ti ng  ei ther   f r om   a
def i ci ency   of   i nsul i n  or   f r om   tar get  ti ssue  r esi stance  to  i ts  cel l ul ar
m etabol i c  ef f ects.   It  i s  the  m ost  com m on  endocr i ne­m etabol i c  di sor der
and  af f ects  an  esti m ated  22  m i l l i on  peopl e  i n  the  U ni ted  States,   w i th  the
i nci dence  of   new   cases  i ncr easi ng  by   m or e  than  700, 000  per   y ear .

Di abetes  i s  m ani f ested  by   the  f i ndi ng  of   hy per gl y cem i a  and  the  ti m e­
dependent  dev el opm ent  of   chr oni c  com pl i cati ons  (r eti nopathy ,
neur opathy ,   nephr opathy ,   and  accel er ated  ather oscl er osi s)  r esul ti ng  f r om
the  m ul ti pl e  m etabol i c  der angem ents.   Accor di ngl y ,   the  pr esenti ng  cl i ni cal
si gns  and  sy m ptom s  can  be  due  to  hy per gl y cem i a  or   the  com pl i cati ons  of
the  di sease,   or   both.   In  gener al ,   the  m ajor   cl assi c  sy m ptom s  of
pol y di psi a,   pol y ur i a,   w ei ght  l oss,
P. 87
and  f ati gue  ar e  f ound  i n  the  setti ng  of   new ­onset  di abetes  i n  y oung
pati ents  w hose  di sease  i s  due  to  i nsul i nopeni a.   On  the  other   hand,   ol der
pati ents  w i th  di abetes  m ay   be  r el ati v el y   f r ee  of   sy m ptom s  f or   a  l ong
ti m e.   In  such  pati ents,   the  di abetes  i s  f i r st  detected  ei ther   i nci dental l y   or
because  one  of   i ts  chr oni c  com pl i cati ons  i s  di scov er ed.   It  i s  esti m ated
that  appr ox i m atel y   one  thi r d  of   al l   the  adul t  cases  of   di abetes  i n  the
U ni ted  States  r em ai n  undi agnosed.

2.   What  ar e  the  m ajor   ty pes  of   DM  and  w hat  ar e  thei r   di sti ngui shi ng
f eatur es?

The  cur r ent  cl assi f i cati on  (accor di ng  to  the  N ati onal   Di abetes  Data  Gr oup)
of   DM  and  other   categor i es  of   gl ucose  i ntol er ance  consi sts  of   thr ee
cl i ni cal   cl asses:  (a)  DM  w hi ch  i ncl udes  ty pe  1  di abetes  m el l i tus  (T1DM),
[pr ev i ousl y   i nsul i n­dependent  di abetes  m el l i tus  (IDDM)  or   juv eni l e  onset
di abetes],   and  ty pe  2  di abetes  m el l i tus  (T2DM),   [pr ev i ousl y   nonâ
€“i nsul i n­dependent  di abetes  m el l i tus  (N IDDM)];  (b)  i m pai r ed  gl ucose
tol er ance/i m pai r ed  f asti ng  gl ucose;  and  (c)  gestati onal   DM.   Of   these,
T1DM  and  T2DM  r epr esent  the  l ar gest  categor y   and  ar e  di scussed  her e  i n
f ur ther   detai l .   Im pai r ed  gl ucose  tol er ance  and  i m pai r ed  f asti ng  gl ucose
ar e  def i ned  as  an  abnor m al i ty   i n  gl ucose  l ev el s  i nter m edi ate  betw een
nor m al   and  ov er t  di abetes,   w her eas  gestati onal   DM  i s  def i ned  as
car bohy dr ate  i ntol er ance  w i th  onset  or   f i r st  r ecogni ti on  dur i ng  pr egnancy .

T1DM  consti tutes  appr ox i m atel y   5%  to  10%  of   al l   cases  of   di abetes  and  i s
due  to  i nsul i n  def i ci ency   r esul ti ng  f r om   the  autoi m m une  destr ucti on  of
i nsul i n­pr oduci ng  pancr eati c  i sl et  cel l s.   Ther ef or e,   such  pati ents  ar e
pr one  to  k etoaci dosi s  and  ar e  absol utel y   dependent  on  ex ogenous  i nsul i n
to  sustai n  l i f e  (hence  the  ter m   i nsul i n­dependent  di abetes).   The  onset  i n
these  pati ents  i s  r el ati v el y   abr upt  and  occur s  usual l y   i n  y outh  (m ean  age,
12  y ear s),   al though  i t  m ay   ar i se  at  any   age  and  i s  of ten  m i sdi agnosed  i n
adul ts.

T2DM  accounts  f or   appr ox i m atel y   90%  to  95%  of   al l   cases  of   di abetes.


These  pati ents  hav e  a  dual   i m pai r m ent  of   i nsul i n  r esi stance  (decr eased
tar get  or gan  r esponse  to  i nsul i n,   i . e. ,   decr eased  gl ucose  tr anspor t  to
m uscl e  or   i nef f ecti v e  suppr essi on  of   hepati c  gl ucose  output)  and
i nadequate  i nsul i n  secr eti on  to  com pensate  f or   the  i nsul i n  r esi stance.   The
r ecent  obesi ty   ex pl osi on,   w hi ch  i s  r el ated  to  sedentar y   l i f esty l e  and
i ncr eased  f ood  i ntak e,   has  ex agger ated  i nsul i n  r esi stance  i n  suscepti bl e
peopl e  and  contr i buted  to  the  di abetes  epi dem i c.   Fi g.   3­1  i l l ustr ates  the
natur al   hi stor y   of   the  tr ansi ti on  f r om   i m pai r ed  gl ucose  tol er ance  to  ov er t
di abetes.   T2DM  i s  now   af f ecti ng  3%  to  6%  of   the  popul ati on  and  occur r i ng
i n  y ounger   peopl e  (ev en  i ncl udi ng  chi l dr en).   Ther e  i s  usual l y   a  str ong
f am i l y   hi stor y   of   DM  i n  pati ents  dev el opi ng  T2DM  i n  y outh.

3.   What  ar e  the  m ajor   acute  and  chr oni c  com pl i cati ons  of   the  di sease?

DKA,   hy per gl y cem i c,   hy per osm ol ar ,   nonk etoti c  com a  (HHN KC),   and
hy pogl y cem i a  ar e  the  m ajor   acute  com pl i cati ons  of   DM.   DKA  i s  m ost
com m onl y   a  com pl i cati on  of   T1DM  and  i s  i ni ti ated  by   an  absol ute  or
r el ati v e  i nsul i n  def i ci ency   and  an  i ncr ease  i n  counter r egul ator y   hor m ones
(gl ucagon,   epi nephr i ne),   l eadi ng  to  the  hepati c  ov er pr oducti on  of   gl ucose
and  k etone  bodi es.   HHN KC  i s  char acter i zed  by   the  i nsi di ous  dev el opm ent
of   m ar k ed  hy per gl y cem i a,   hy per osm ol ar i ty ,   dehy dr ati on,   and  pr er enal
azotem i a  i n  the
P. 88
absence  of   si gni f i cant  hy per k etonem i a  or   aci dosi s.   Fi nal l y ,   hy pogl y cem i a
can  occur   as  an  acute  com pl i cati on  of   the  ther apy   of   both  T1DM  and
T2DM,   and  i s  the  m ost  com m on  acute  l i f e­thr eateni ng  com pl i cati on  of
di abetes.   It  i s  m ost  com m on  w i th  i ntensi v e  i nsul i n  ther apy ,   and  r ecur r ent
hy pogl y cem i a  can  i nduce  a  condi ti on  k now n  as  hy pogl y cem i a
unaw ar eness,   a  bl unti ng  of   the  adr ener gi c  and  neur ogl y copeni c  si gns  and
sy m ptom s  of   hy pogl y cem i a.   The  r i sk   of   hy pogl y cem i a  unaw ar eness  can  be
m i ni m i zed  and  ex i sti ng  unaw ar eness  tr eated  by   str i ct  av oi dance  of
hy pogl y cem i a.

Figure  3­1  N atur al   hi stor y   of   ty pe  2  di abetes  m el l i tus.   Adapted  f r om


Ber genstal   RM,   Kendal l   DM,   Fr anz  MJ,   et  al .   Managem ent  of   ty pe  2  di abetes:
a  sy stem ati c  appr oach  to  m eeti ng  the  standar ds  of   car e.   II:  or al   agents,
i nsul i n,   and  m anagem ent  of   com pl i cati ons.   In:  Degr oot  LJ,   Jam eson  JL,   eds.
Endocr i nol ogy .   phi l adel phi a:  WB  Saunder s,   2001:821–835.   w i th  per m i ssi on
f r om   El sev i er .

The  m ost  com m on  chr oni c  com pl i cati on  of   di abetes  and  the  l eadi ng  cause
of   death  f or   peopl e  w i th  di abetes  i s  car di ov ascul ar   (CV)  di sease.
Sev enty ­sev en  per cent  of   al l   hospi tal i zati ons  and  80%  of   al l   m or tal i ty   i n
di abetes  i s  secondar y   to  CV  di sease.   Di abetes  i s  an  i ndependent  r i sk
f actor   f or   CV  di sease.   The  i nci dence  of   CV  ev ents  i s  so  hi gh  i n  subjects
w i th  di abetes  that  di abetes  i s  consi der ed  a  CV  r i sk   equi v al ent.   CV  di sease
i ncl udes  m y ocar di al   i schem i a,   str ok e,   and  per i pher al   v ascul ar   di sease.
Peopl e  w i th  di abetes  al so  hav e  an  i ncr eased  i nci dence  of   hear t  f ai l ur e,
w hi ch  w i l l   not  be  addr essed  i n  thi s  secti on,   as  the  pathophy si ol ogy   i s
poor l y   under stood.   Outcom es  af ter   acute  m y ocar di al   i nf ar cti on  (MI)  i n
peopl e  w i th  di abetes  ar e  w or se  than  contr ol s,   but  can  be  i m pr ov ed  w i th
i ntensi v e  gl y cem i c  contr ol   i n  the  hospi tal .   Inter v enti onal   studi es
dem onstr ate  that  l i pi d  l ow er i ng  si gni f i cantl y   decr eases  m or tal i ty   and  CV
ev ents  i n  peopl e  w i th  di abetes.   In  f act,   i t  appear s  that  peopl e  w i th  DM
m ay   benef i t  f r om   stati ns  r egar dl ess  of   i ni ti al   l ow ­densi ty   l i popr otei n
(LDL).   Addi ti onal   l ar ge  pr ospecti v e  tr i al s  dem onstr ate  decr eased  CV
m or tal i ty   w i th  i ntensi v e  BP  contr ol .   Ther e  i s  no  com pel l i ng  data  that
i m pr ov em ent  of   gl y cem i c  contr ol   af f ects  CV  m or bi di ty   or   m or tal i ty   ex cept
i n  subjects  w i th  T1DM.

Mi cr ov ascul ar   com pl i cati ons  (r eti nopathy ,   nephr opathy   and  neur opathy )
ar e  speci f i c  to  di abetes  and  ar e  r el ated  di r ectl y   to  poor   gl y cem i c  contr ol
w i th
P. 89
a  sm al l er   contr i buti on  f r om   hy per tensi on  and  dy sl i pi dem i a.   Di abetes  i s
the  l eadi ng  cause  of   bl i ndness  i n  the  U ni ted  States.   By   10  y ear s'  dur ati on
of   di abetes,   appr ox i m atel y   90%  of   i ndi v i dual s  w i l l   hav e  som e  degr ee  of
r eti nopathy .   Reti nopathy   i s  l ar gel y   a  pr ev entabl e  com pl i cati on  of
di abetes.   Annual   ophthal m ol ogi c  ex am i nati ons  per m i t  i denti f i cati on  of
i ndi v i dual s  w i th  pr ogr essi v e  r eti nopathy .   Tw o  l ar ge  m ul ti center   studi es
hav e  pr ov ed  that  ear l y   i nter v enti on  at  thi s  stage  w i th  panr eti nal
photocoagul ati on  can  pr ev ent  or   decr ease  v i sual   l oss.   The  Di abetes
Contr ol   and  Com pl i cati ons  Tr i al   (DCCT)  establ i shed  that  ti ght  gl y cem i c
contr ol   al so  pr ev ents  or   del ay s  r eti nopathy .   Macul ar   edem a,   cor neal
ul cer ati on,   gl aucom a,   and  catar acts  ar e  addi ti onal   ocul ar   com pl i cati ons  of
di abetes.

Di abetes  i s  the  l eadi ng  cause  of   r enal   f ai l ur e/di al y si s  and


tr anspl antati ons  nati onw i de.   For ty   per cent  to  60%  of   i ndi v i dual s  w i th
T1DM  and  10%  to  30%  of   i ndi v i dual s  w i th  T2DM  w i l l   dev el op
m i cr oal bum i nur i a,   pr otei nur i a,   and  end­stage  r enal   di sease  secondar y   to
di abetes.   Hy per tensi on  and  gl y cem i c  contr ol   ar e  the  pr i m ar y   f actor s  that
pr om ote  pr ogr essi on  of   nephr opathy   i n  peopl e  w i th  di abetes.
N or m al i zati on  of   BP  and  gl ucose  dr am ati cal l y   sl ow   the  pr ogr essi on  f r om
i nci pi ent  nephr opathy   (detectabl e  m i cr oal bum i n)  to  ov er t  nephr opathy .   Al l
peopl e  w i th  DM  shoul d  hav e  a  BP  l ow er   than  130/80  m m   Hg,   pr ef er abl y
m uch  l ow er .   Aggr essi v e  contr ol   of   hy per gl y cem i a  (w i th  i ntensi v e  ther apy )
and  BP  [w i th  angi otensi n­conv er ti ng  enzy m e  (ACE)  i nhi bi tor s  or
angi otensi n  r eceptor   bl ock er s]  has  been  show n  to  r etar d  the  pr ogr essi on
of   nephr opathy   i n  pati ents  w i th  DM.

N eur opathy   i s  a  com m on  com pl i cati on  of   di abetes  af f ecti ng  m or e  than
50%  of   pati ents  w i th  ti m e.   The  m ost  com m on  f or m   of   ner v e  i njur y   i n
di abetes  i s  di stal   sy m m etr i c  pol y neur opathy ,   w hi ch  occur s  i n  a  stock i ng­
gl ov e  di str i buti on;  i t  can  be  pai nl ess  or   pai nf ul .   Thi s  ty pe  of   neur opathy
i ncr eases  the  r i sk   f or   tr aum ati c  f oot  i njur y   and  am putati on.   Other   f or m s
of   neur opathy   i ncl ude:  autonom i c  neur opathy   (associ ated  w i th  an
i ncr eased  r i sk   of   CV  death  and  hy pogl y cem i a  unaw ar eness);  m ononeur i ti s
m ul ti pl ex   (a  v ascul ar   occl usi on  to  a  si ngl e  ner v e  di str i buti on  that  w i l l
ty pi cal l y   r ecov er   w i th  ti m e);  and  di abeti c  am y otr ophy   (a  pr of ound,
uncom m on  dem y el i nati ng  neur om uscul ar   w asti ng  sy ndr om e).

Di abetes  i s  al so  associ ated  w i th  i m pai r ed  bl ood  f l ow   and  sensati on  to  the
ex tr em i ti es.   Thi s  l eads  to  a  hi gh  i nci dence  of   m echani cal   tr aum a  and
i nf ecti ous  com pl i cati ons,   l eadi ng  to  am putati on  and  hospi tal i zati on.
Di abetes  i s  the  m ost  f r equent  cause  of   nontr aum ati c  l ow er   l i m b
am putati ons.   Each  y ear ,   m or e  than  56, 200  am putati ons  ar e  per f or m ed
am ong  peopl e  w i th  di abetes.   Thi s  com pl i cati on  i s  l ar gel y   pr ev entabl e  by
appr opr i ate  f ootw ear ,   r egul ar   f oot  ex am i nati on,   and  educati on.

4.   What  aspects  of   the  m edi cal   hi stor y   r equi r e  speci al   em phasi s?

A  com pr ehensi v e  m edi cal   hi stor y   i n  a  pati ent  w i th  suspected  di abetes
shoul d  be  di r ected  not  onl y   tow ar d  conf i r m i ng  the  di agnosi s  but  shoul d
al so  be  used  to  r ev i ew   the  natur e  of   pr ev i ous  tr eatm ent  pr ogr am s  and
di abetes  educati on,   f am i l y   hi stor y ,   the  degr ee  of   past  and  r ecent
gl y cem i c  contr ol ,   and  hi stor y   of   acute  and  chr oni c  com pl i cati ons.   Pati ents
shoul d  al so  be  quer i ed
P. 90
about  thei r   di etar y ,   w ei ght,   and  ex er ci se  hi stor y .   Cur r ent  m edi cati ons  f or
the  m anagem ent  of   di abetes,   as  w el l   as  other   m edi cati ons  that  m ay
af f ect  gl y cem i c  contr ol ,   shoul d  be  r ecor ded.   In  addi ti on,   the  pr esence,
sev er i ty ,   and  tr eatm ent  of   the  acute  and  chr oni c  com pl i cati ons  of
di abetes  shoul d  be  r ev i ew ed,   i ncl udi ng  sex ual   f uncti on  and  dental   car e.
Al l   pati ents  shoul d  hav e  a  car ef ul   hi stor y   f or   di abeti c  heal th  car e
m ai ntenance  docum ented  at  each  v i si t.   Thi s  i ncl udes  gl y cem i c  contr ol ,
l i pi d  m anagem ent  (LDL  < 100,   or   < 70  i n  hi gh  r i sk   or   k now n  CV  di sease),
BP  m anagem ent  (< 130/80  m m   Hg),   ey e  ex am i nati on  (annual ),   f oot
ex am i nati on  (each  v i si t),   di et,   ex er ci se,   and  sel f   m anagem ent  (Fi g.   3­2).
Figure  3­2  Di abetes  car e  f l ow   sheet.   BMI,   body   m ass  i ndex ;  BP,
bl ood  pr essur e;  LDL,   l ow ­densi ty   l i popr otei n;  CAD,   cor onar y   ar ter y
di sease;  HDL,   hi gh­l ev el   l i popr otei n.

P. 91
5.   What  aspects  of   the  phy si cal   ex am i nati on  r equi r e  speci al   attenti on?

The  v i tal   si gns  ar e  cr i ti cal   f or   pati ents  w i th  di abetes.   BP  gr eater   than
130/80  m m   Hg  i ncr eases  the  r i sk   f or   al l   com pl i cati ons,   r esti ng
tachy car di a  suggests  autonom i c  ner v ous  sy stem   dy sf uncti on,   and  w ei ght
gai n  or   l oss  pr ov i des  v al uabl e  i nf or m ati on  on  sev er i ty   of   i l l ness  and
adher ence  to  ther apy .   On  phy si cal   ex am i nati on,   denti ti on  i s  i m por tant  as
per i odontal   di sease  can  i m pact  gl y cem i c  contr ol   and  i s  a  r i sk   f actor   f or
ather oscl er osi s  (chr oni c  i nf l am m ati on).   Com pl ete  CV  ex am i nati on
[i ncl udi ng  br ui ts  and  ank l e  br achi al   i ndex   (ABI)]  and  ev al uati on  f or
edem a  can  detect  CV  di sease  and  hear t  f ai l ur e.   Loss  of   r espi r ator y
v ar i ati on  i n  hear t  r ate  i s  an  ear l y   w ar ni ng  of   autonom i c  neur opathy .   Foot
ex am i nati on,   i ncl udi ng  pul ses  and  m onof i l am ent  testi ng,   can  i denti f y
hi gh­r i sk   f eet  and  pr ev ent  am putati ons.   The  r eti nal   ex am i nati on
[undi l ated  by   a  pr i m ar y   car e  phy si ci an  (PCP)]  i s  not  sensi ti v e  f or
detecti on  of   r eti nopathy   and  needs  to  be  done  by   an
P. 92
ophthal m ol ogi st  or   by   usi ng  r eti nal   photogr aphs.   It  m ay   be  conducted  by
the  PCP,   but  the  needed  f or m al   annual   ev al uati on  shoul d  al so  be
ar r anged.

6.   What  l abor ator y   tests  ar e  essenti al   i n  the  ev al uati on  of   the  pati ent  w i th
suspected  di abetes?

The  Am er i can  Di abetes  Associ ati on  (ADA)  and  r egul ator y   agenci es  hav e
establ i shed  standar ds  f or   l abor ator y   ev al uati on  of   di abetes.   The  di agnosi s
of   di abetes  i s  f or m al l y   m ade  on  the  basi s  of   one  of   the  f ol l ow i ng  cr i ter i a:
(a)  f asti ng  gl ucose  126  m g/dL  or   m or e  on  tw o  occasi ons,   (b)  r andom
gl ucose  200  m g/dL  or   m or e  on  tw o  occasi ons,   or   (c)  one  abnor m al
r eadi ng  as  abov e  together   w i th  sy m ptom s  consi stent  w i th  di abetes
(pol y ur i a,   noctur i a,   pol y di psi a,   w ei ght  l oss,   bl ur r ed  v i si on).   Hem ogl obi n
A I c   (HbA I c )  i s  not  y et  r ecom m ended  as  a  di agnosti c  test  because  of   a  l ack

of   standar di zati on  of   ex i sti ng  assay s,   but  i s  i ncr easi ngl y   bei ng  consi der ed
by   the  ADA  and  r egul ator y   agenci es  as  a  potenti al   di agnosti c  tool .

7.   What  ar e  the  goal s  of   di abetes  ther apy   and  w hat  tr eatm ent  m odal i ti es  ar e
av ai l abl e?  How   shoul d  these  be  i ndi v i dual i zed?

In  gener al ,   the  goal s  of   di abetes  ther apy   ar e  (a)  to  al l ev i ate  the  si gns
and  sy m ptom s  of   the  di sease  (e. g. ,   pol y di psi a,   pol y ur i a,   and  noctur i a);
(b)  to  pr ev ent  the  acute  com pl i cati ons  (i . e. ,   hy pogl y cem i a,   DKA,   and
HHN KC);  and  (c)  to  pr ev ent  the  l ong­ter m   com pl i cati ons  of   the  di sease
(i . e. ,   r eti nopathy ,   nephr opathy ,   neur opathy ,   and  ather oscl er oti c  CV
di sease).   The  DCCT  f i r st  dem onstr ated  that  ti ght  m etabol i c  contr ol   of
T1DM  l eads  to  def i ni te  benef i ci al   ef f ects  on  the  r ate  of   com pl i cati ons.
Thi s  al so  hel d  tr ue  f or   pati ents  w i th  T2DM  i n  the  r ecentl y   com pl eted
U ni ted  Ki ngdom   Pr ospecti v e  Di abetes  Study   (U KPDS).   Ev i dence
docum enti ng  the  i m por tance  of   gl y cem i c  contr ol   f or   the  pr ev enti on  of
m i cr ov ascul ar   com pl i cati on  i s  unequi v ocal .   Ther ef or e,   i ntensi v e  gl y cem i c
contr ol   i s  now   r outi ne  w i th  HbA I c   goal s  of   6. 5%  to  7%.   The  l i m i ti ng  f actor

i n  such  attem pts  i s  an  i ncr eased  f r equency   of   hy pogl y cem i c  epi sodes.
Recent  data  f r om   the  DCCT  f ol l ow ­up  study   Epi dem i ol ogy   of   Di abetes
Inter v enti ons  and  Com pl i cati ons  (EDIC)  now   i ndi cate  that  i ntensi v e
contr ol   of   bl ood  gl ucose  i n  T1DM  al so  pr ev ents  m acr ov ascul ar   di sease.   In
pati ents  w i th  T2DM,   m ul ti tar geted  ther apy   (l i pi d,   BP,   and  gl ucose  contr ol )
i s  the  m ost  ef f ecti v e  str ategy   f or   CV  di sease  pr ev enti on.

Insul i n  i s  r equi r ed  f or   gl y cem i c  contr ol   i n  T1DM  w her eas  T2DM  r equi r es  a
m ul ti f aceted  appr oach.   Di et  and  ex er ci se  ar e  the  m ai nstay s  of   T2DM
ther apy .   They   shoul d  be  i nsti tuted  f i r st  and  pati ent  adher ence  encour aged
and  m ax i m i zed.   Regar dl ess  of   the  ul ti m ate  r egi m en,   di et  and  ex er ci se
r em ai n  i m por tant.   Or al   sul f ony l ur ea  agents  that  enhance  β­cel l   i nsul i n
secr eti on,   m etf or m i n  that  decr eases  hepati c  gl ucose  output,   or
thi azol i di nedi ones  that  enhance  i nsul i n  acti on  i n  the  per i pher y   ar e  added
to  thi s  tr eatm ent  i f   di et  and  ex er ci se  al one  f ai l   to  contr ol   hy per gl y cem i a
opti m al l y .   These  agents  can  al so  be  used  i n  com bi nati on  because  they
hav e  di f f er ent  m echani sm s  and  thei r   acti ons  ar e  addi ti v e.   Com bi nati on
ther apy   w i th  m ul ti pl e  cl asses  of   dr ugs  i s  ef f ecti v e,   but  cost  and
m oni tor i ng  f or   tox i ci ty   can  be  pr ohi bi ti v e.   Wi th  i ncr eased  dur ati on  of
T2DM,   β­cel l   m ass  and  f uncti on  ar e  di m i ni shed  and  l ead  to  r el ati v e
P. 93
i nsul i n  i nsuf f i ci ency .   At  som e  poi nt,   i nsul i n  ther apy   becom es  necessar y
f or   opti m al   gl y cem i c  contr ol .   In  f act,   i nsul i n  ther apy   i s  the  best  w ay   to
nor m al i ze  gl ucose  i n  pati ents  not  r espondi ng  w el l   to  or al   agents  and
shoul d  be  em pl oy ed  as  soon  as  gl ucose  r i ses  and  not  as  a  l ast  r esor t.
N ew   i njectabl e  agents  that  r egul ate  gl ucagon,   gastr i c  em pty i ng,   sati ety
and  i nsul i n  secr eti on:  am y l i n  and  gl ucagon­l i k e  pepti de­1  (GLP­1)
agoni sts  ar e  r ecent  addi ti ons  to  the  l i st  of   anti ­hy per gl y cem i c  agents.   The
m ost  r ecent  addi ti on(s)  ar e  or al   i nhi bi tor s  of   di pepti dy l   pepti dase  4
(DPP4),   the  enzy m e  that  i nacti v ates  GLP­1.   These  agents  ar e  cur r entl y
used  by   pr ov i der s  speci al i zi ng  i n  di abetes.

Case 1
A  14­y ear ­ol d  boy   w i th  an  8­y ear   hi stor y   of   DM  has  been  si ck   si nce  y ester day
w hen  he  began  v om i ti ng.   Hi s  di abetes  has  been  r easonabl y   w el l   contr ol l ed
w i th  a  dosage  of   20  uni ts  of   gl ar gi ne  i nsul i n  tak en  dai l y .   He  uses  a
car bohy dr ate  r ati o  of   1:20  and  cor r ecti on  f actor   of   1:50  f or   m eal ti m e  bol us
i nsul i n.   He  has  had  sev er al   epi sodes  of   DKA  i n  the  past,   but  not  f or
appr ox i m atel y   4  y ear s.   Yester day ,   w hen  he  began  v om i ti ng,   gl ucose
concentr ati on  w as  400  and  hi s  ur i ne  acetone  w as  negati v e,   so  he  took   hi s
usual   dose  of   i nsul i n.   He  has  had  i ntense  pol y ur i a  and  pol y di psi a  f or   the  l ast
24  hour s.   Thi s  m or ni ng,   appr ox i m atel y   6  hour s  ago,   hi s  m other   deci ded  to
w i thhol d  hi s  i nsul i n  because  of   conti nued  nausea  and  v om i ti ng.
Phy si cal   ex am i nati on  r ev eal s  a  dr ow sy   y oung  m an  w ho  can  r espond  to
questi oni ng.   Hi s  BP  i s  90/70  m m   Hg;  pul se,   124  per   m i nute;  r espi r ati ons,   30
per   m i nute;  and  tem per atur e,   38. 3°C  (100. 9°F).   Hi s  m ucous  m em br anes  ar e
dr y   and  the  ocul ar   gl obes  ar e  sof t  and  sunk en,   but  the  f unduscopi c  f i ndi ngs
ar e  nor m al .   Bow el   sounds  ar e  absent  and  he  has  gener al i zed  abdom i nal
tender ness  w i thout  r ebound.   The  deep  tendon  r ef l ex es  ar e  hy poacti v e,   but
ther e  ar e  no  l ocal i zi ng  neur ol ogi c  si gns.   The  r est  of   the  ex am i nati on  f i ndi ngs
ar e  nor m al .
Labor ator y   data  consi st  of   the  f ol l ow i ng:  Hgb,   16. 4  g/dL;  hem atocr i t  (Hct),
53%;  WBC,   16, 942/  m m 3   (93%  pol y m or phonucl ear   l euk ocy tes);  BU N ,   40
m g/dL;  cr eati ni ne,   1. 8  m g/dL;  gl ucose,   847  m g/dL;  ser um   k etones,   str ongl y
posi ti v e  at  1:4  di l uti on;  sodi um ,   126  m Eq/L;  potassi um ,   4. 3  m Eq/L;  chl or i de,
100  m Eq/L;  and  bi car bonate,   6  m Eq/L.   U r i nal y si s  r ev eal s  a  speci f i c  gr av i ty   of
1. 030;  gl ucose  of   4+ ;  acetone,   str ongl y   posi ti v e;  and  tr ace  am ounts  of
pr otei n.   Ar ter i al   bl ood  gas  anal y si s  r ev eal s  a  pH  of   7. 08,   par ti al   pr essur e  of
car bon  di ox i de  (PCO 2 )  of   12  m m   Hg,   and  par ti al   pr essur e  of   ox y gen  (PO 2 )  of
80  m m   Hg.   An  ECG  show s  si nus  tachy car di a  w i th  f l at  T  w av es.   Chest
r adi ogr aphi c  study   i s  nor m al .   Abdom i nal   r adi ogr aphs  show   gastr i c  di stenti on,
but  other w i se  the  f i ndi ngs  ar e  nor m al .

1.   What  i s  the  di agnosi s  and  pathophy si ol ogi c  pr ocess  of   thi s  pati ent's
di sease?
2.   How   i s  the  l i v er   i nv ol v ed  i n  the  genesi s  of   DKA?
3.   What  i s  the  status  of   the  pati ent's  f l ui d  and  el ectr ol y te  l ev el s?
4.   What  ar e  the  m ajor   goal s  of   ther apy ?
5.   What  pr eci pi tated  thi s  epi sode  of   DKA?

Case Discussion
1.   What  i s  the  di agnosi s  and  pathophy si ol ogi c  pr ocess  of   thi s  pati ent's
di sease?

Thi s  pati ent  has  T1DM  and  i s  pr esenti ng  w i th  an  epi sode  of   DKA.   DKA  i s
i ni ti ated  by   an  absol ute  or   r el ati v e  i nsul i n  def i ci ency   and  an  i ncr ease  i n
the  l ev el
P. 94
of   counter r egul ator y   catabol i c  hor m ones,   l eadi ng  to  the  hepati c
ov er pr oducti on  of   gl ucose  and  k etone  bodi es.   Consi stent  w i th  thi s,   the
pati ent's  l abor ator y   data  show   the  pr esence  of   m ar k ed  hy per gl y cem i a,
k etonem i a,   k etonur i a,   and  sev er e  m etabol i c  aci dosi s.   The  pati ent's
tachy pnea  i s  al so  consi stent  w i th  hi s  aci doti c  state.

The  destr ucti on  of   pancr eati c  β  cel l s  l eadi ng  to  T1DM  i s  thought  to  be
m edi ated  by   the  acti v ati on  of   autoi m m une  pr ocesses  i n  geneti cal l y
pr edi sposed  peopl e.   The  pr esence  of   anti i sl et  and  anti i nsul i n  anti bodi es,
the  ex i stence  of   i nf l am m ator y   cel l s  ar ound  the  i sl et  cel l s,   and  the
tem por ar y   am el i or ati on  of   new ­onset  T1DM  by   i m m unosuppr essi v e
ther apy   al l   pr ov i de  str ong  ev i dence  f or   an  autoi m m une  basi s  of
pancr eati c  β­cel l   destr ucti on.

2.   How   i s  the  l i v er   i nv ol v ed  i n  the  genesi s  of   DKA?

Hepati c  k etogenesi s  and  the  dev el opm ent  of   DKA  depend  on  both  the  r ate
of   substr ate  (FFA)  suppl y   to  the  l i v er   and  the  acti v ati on  of   the  hepati c
k etogeni c  pr ocess,   the  l atter   bei ng  m odul ated  by   the  r el ati v e  i ncr ease  i n
the  gl ucagon­to­i nsul i n  r ati o  that  pr ev ai l s  dur i ng  DKA.   The  i nsul i n
def i ci ency   l eads  to  the  acti v ati on  of   l i pol y si s  and  an  i ncr eased  suppl y   of
ci r cul ati ng  FFA.   In  the  l i v er   these  m ol ecul es  under go  successi v e  β­
ox i dati on  to  acety l   coenzy m e  A  (CoA).   Dur i ng  DKA  the  unr estr ai ned  FFA
m obi l i zati on  and  ox i dati on  tr i gger   the  pr oducti on  of   ex cess  am ounts  of
acety l   CoA,   w hi ch  under go  condensati on  to  acetoacety l   CoA,   a  pr ecur sor
of   the  k etone  bodi es  acetoacetate,   acetone,   and  β­hy dr ox y buty r ate.

3.   What  i s  the  status  of   the  pati ent's  f l ui d  and  el ectr ol y te  l ev el s?

The  pati ent's  phy si cal   ex am i nati on  r ev eal s  si gns  of   sev er e  dehy dr ati on
and  i ntr av ascul ar   hy pov ol em i a  (note  hi s  hy potensi on,   tachy car di a,   and
the  dr y   m ucous  m em br anes).   DKA,   i f   not  tr eated  ear l y ,   r esul ts  i n  a
sev er e  total ­body   depl eti on  of   f l ui d  (usual l y   sev er al   l i ter s)  and
el ectr ol y tes  due  to  the  f ol l ow i ng  f actor s:

1.   The  hy per gl y cem i a  and  hy per k etonem i a  l ead  to  osm oti c  di ur esi s  and
the  ur i nar y   l oss  of   f l ui d  and  el ectr ol y tes.

2.   Because  of   aci dosi s,   potassi um   i s  al so  shi f ted  f r om   the  i ntr acel l ul ar
to  ex tr acel l ul ar   f l ui d  space  and  then  l ost  dur i ng  osm oti c  di ur esi s.
Ther ef or e,   the  ser um   potassi um   l ev el s  m ay   not  accur atel y   r ef l ect
the  total ­body   def i ci ency .

3.   Vom i ti ng,   as  i n  thi s  pati ent,   causes  the  f ur ther   l oss  of   f l ui d  and
el ectr ol y tes.

4.   Muscl e  catabol i sm   (pr oteol y si s),   w hi ch  r esul ts  f r om   the  i nsul i n
def i ci ency ,   l eads  to  the  l oss  of   potassi um ,   phosphate,   m agnesi um ,
and  ni tr ogen.

4.   What  ar e  the  m ajor   goal s  of   ther apy ?

The  i m m edi ate  ther apeuti c  goal s  ar e  (a)  to  r epl eni sh  the  f l ui d  (star ti ng
w i th  i sotoni c  sal i ne)  and  el ectr ol y tes;  and  (b)  to  pr ov i de  adequate  i nsul i n
to  i nhi bi t  l i pol y si s  and  k etogenesi s  and  nor m al i ze  car bohy dr ate
m etabol i sm ,   both  i n  the  l i v er   (by   i nhi bi ti ng  gl ucose  pr oducti on)  and  i n  the
per i pher al   ti ssues  (by   enhanci ng  di sposal   of   gl ucose  and  k etone  bodi es).
Insul i n  ther apy   i s  best  adm i ni ster ed  i n  the  f or m   of   a  conti nuous  IV
i nf usi on.   Dur i ng  the  f l ui d,   el ectr ol y te,   and  i nsul i n  ther apy ,   the  pati ent's
bl ood  gl ucose  and  el ectr ol y te  l ev el s  (especi al l y   potassi um )  shoul d  be
m oni tor ed  f r equentl y   and  appr opr i ate  adjustm ents  m ade.   Addi ti onal
ther apeuti c  objecti v es  i ncl ude  the  i denti f i cati on  and  m anagem ent  of
possi bl e  pr eci pi tati ng  f actor s  (e. g. ,   i nf ecti on,   str ess,   and  m edi cati on
er r or s)  and  the  i m pl em entati on  of   m easur es  to  pr ev ent  the  r ecur r ence  of
DKA.

P. 95
5.   What  pr eci pi tated  thi s  epi sode  of   DKA?

The  i m m edi ate  pr eci pi tati ng  ev ent  of   thi s  pati ent's  DKA  i s  the  w i thhol di ng
of   i nsul i n.   An  under l y i ng  str ess  or   i nf ecti on  (e. g. ,   gastr oenter i ti s),   w hi ch
m ay   al so  be  pr esent  i n  thi s  pati ent,   shoul d  be  ev al uated  and  m anaged.

Case 2
A  63­y ear ­ol d  m an  i s  br ought  to  the  em er gency   r oom   i n  an  unconsci ous  state.
He  w as  appar entl y   i n  good  heal th  unti l   1  w eek   bef or e  adm i ssi on,   w hen  he
ex per i enced  an  i nsati abl e  thi r st  that  he  attem pted  to  sati sf y   by   dr i nk i ng  l ar ge
quanti ti es  of   beer   and  soda  dr i nk s.   He  had  com pl ai ned  of   hav i ng  noctur i a  f or
sev er al   day s,   and  had  sev er al   bouts  of   di ar r hea  y ester day .   He  took   to  hi s  bed
y ester day   and  w as  f ound  unconsci ous  thi s  m or ni ng.   He  tak es  no  dr ugs,   has  not
seen  a  phy si ci an  f or   sev er al   y ear s,   and  w or k s  r egul ar l y   as  a  house  pai nter .
Hi s  heal th  has  been  good  pr ev i ousl y .   Hi s  m other   had  di abetes  i n  her   ei ghti es
and  di ed  of   a  str ok e.
Phy si cal   ex am i nati on  r ev eal s  a  deepl y   unconsci ous,   acutel y   i l l   m an  w ho  has
sev er al   f ocal   r i ght­si ded  sei zur es  dur i ng  ex am i nati on.   Hi s  sk i n  and  m ucous
m em br anes  ar e  dr y   and  hi s  ocul ar   gl obes  ar e  qui te  sof t.   Hi s  BP  i s  98/60  m m
Hg,   pul se  i s  120  per   m i nute,   and  r ectal   tem per atur e  i s  38°C  (100. 9°F),   and
he  ex hi bi ts  unl abor ed  r espi r ati ons  at  a  r ate  of   13  per   m i nute.   Ex cept  f or   the
f i ndi ngs  of   m i ni m al   hepatom egal y ,   absent  k nee  jer k s,   and  bi l ater al   Babi nsk i 's
r ef l ex es,   the  ex am i nati on  f i ndi ngs  ar e  other w i se  nor m al .
Labor ator y   data  consi st  of   the  f ol l ow i ng:  Hgb,   16. 2  g/dL;  Hct,   51%;  and  WBC,
21, 340/  m m 3   (92%  pol y m or phonucl ear   l euk ocy tes).   U r i nal y si s  r ev eal s  a
speci f i c  gr av i ty   of   1. 030;  pH,   6. 0;  gl ucose,   4+ ;  acetone,   m oder ate  am ounts;
and  pr otei n,   tr ace  am ounts.   Ar ter i al   bl ood  gas  anal y si s  r ev eal s  a  pH  of   7. 41,
PCO 2   of   35  m m   Hg,   and  PO 2   of   68  m m   Hg.   Both  chest  r adi ogr aphi c  and  head
CT  scan  f i ndi ngs  ar e  nor m al .   Hi s  ECG  show s  si nus  tachy car di a  w i th  nonspeci f i c
ST­T–w av e  changes.   Ser um   f i ndi ngs  ar e  BU N ,   68  m g/dL;  cr eati ni ne,   2. 3
m g/dL;  gl ucose,   1, 420  m g/dL;  k etones,   tr ace  am ounts;  sodi um ,   153  m Eq/L;
potassi um ,   4. 6  m Eq/L;  chl or i de,   110  m Eq/L;  and  bi car bonate,   26  m Eq/L.

1.   What  i s  the  di agnosi s  i n  thi s  pati ent  and  how   w oul d  y ou  r el ate  i t  to  the
m ajor   phy si cal   and  l abor ator y   f i ndi ngs?
2.   What  i s  the  natur e  of   thi s  pati ent's  endogenous  i nsul i n  secr eti on,   and  i s
thi s  ty pe  of   di abetes  her edi tar y ?
3.   Why   di d  k etoaci dosi s  not  dev el op  i n  thi s  pati ent?
4.   How   i s  hi s  l i v er   i nv ol v ed  i n  the  pathogenesi s  of   hi s  hy per gl y cem i a?
5.   What  ar e  the  m ajor   hor m ones  that  ar e  counter r egul ator y   to  i nsul i n
acti on?  Ar e  they   pl ay i ng  any   r ol e  i n  thi s  m an's  i l l ness?
6.   What  w oul d  y ou  pr edi ct  about  the  state  of   hi s  i ntr av ascul ar   v ol um e?
7.   What  ar e  the  m ajor   ther apeuti c  goal s  i n  thi s  pati ent?

Case Discussion
1.   What  i s  the  di agnosi s  i n  thi s  pati ent  and  how   w oul d  y ou  r el ate  i t  to  the
m ajor   phy si cal   and  l abor ator y   f i ndi ngs?

Thi s  el der l y   pati ent  pr esents  i n  a  com atose  state  pr eceded  by   sev er al
day s  of   pr ogr essi v e  sy m ptom s  of   pol y ur i a,   pol y di psi a,   and  noctur i a.   Hi s
l abor ator y   data  show
P. 96
the  pr esence  of   m ar k ed  hy per gl y cem i a  but  no  aci dosi s.   In  thi s  setti ng,   hi s
m oder ate  k etonem i a  and  k etonur i a  ar e  m ost  l i k el y   secondar y   to
star v ati on.   Ther ef or e,   the  di agnosi s  i n  thi s  pati ent  i s  HHN KC.   Hi s  ser um
osm ol al i ty   can  be  cal cul ated  usi ng  the  f or m ul a:  esti m ated  osm ol al i ty   =
2([N a]  +   [K])  +   [gl ucose]/18  +   [BU N ]/2. 8.   For   thi s  pati ent,   the  esti m ated
osm ol al i ty   i s  cal cul ated  to  be  418,   w hi ch  i s  consi stent  w i th  a  sev er e
hy per osm ol ar   state.

2.   What  i s  the  natur e  of   thi s  pati ent's  endogenous  i nsul i n  secr eti on,   and  i s
thi s  ty pe  of   di abetes  her edi tar y ?
Thi s  pati ent  has  T2DM.   When  T2DM  i s  of   shor t  dur ati on,   such  as  i n  thi s
pati ent,   and  w hen  pati ents  ar e  obese,   the  endogenous  i nsul i n  l ev el s  ar e
ty pi cal l y   nor m al   or   el ev ated.   Such  pati ents  ar e  sti l l   abl e  to  m ai ntai n
suf f i ci ent  endogenous  i nsul i n  secr eti on  to  pr ev ent  k etoaci dosi s  f r om
dev el opi ng  under   basal   condi ti ons.   Onl y   sev er e  str ess  w i th  el ev ated
catechol am i nes  pl us  gl ucagon  and  decr eased  i nsul i n  secr eti on  w i l l
pr eci pi tate  DKA  i n  peopl e  w i th  T2DM.

Her edi ty   pl ay s  an  i m por tant  r ol e  i n  T2DM,   al though  the  m ode  of


i nher i tance  i s  l ar gel y   unk now n.   T2DM  i s  al so  a  heter ogeneous  di sor der ,
and  di f f er ent  f or m s  of   geneti c  i nf l uences  or   def ects  m ay   ex i st.   Ev i dence
f or   a  geneti c  i nf l uence  i n  the  acqui si ti on  of   T2DM  i ncl ude  (a)  a  str ong
f am i l y   hi stor y   of   the  di sease,   (b)  a  v er y   hi gh  pr ev al ence  of   the  di sease  i n
cer tai n  popul ati on  gr oups  (e. g. ,   the  Pi m a  Indi ans  and  Mi cr onesi ans  of
N aur u),   (c)  a  concor dance  r ate  of   90%  to  100%  i n  m onozy goti c  tw i ns,   and
(d)  an  appar ent  autosom al   dom i nant  m ode  of   tr ansm i ssi on  of   m atur i ty ­
onset  di abetes  of   the  y oung  (an  uncom m on  m onogeni c  f or m   of   T2DM).

3.   Why   di d  k etoaci dosi s  not  dev el op  i n  thi s  pati ent?

Thi s  pati ent  has  suf f i ci ent  endogenous  i nsul i n  to  pr ev ent  (a)  l i pol y si s  (FFA
l ev el s  ar e  l ow er   i n  the  setti ng  of   HHN KC  than  of   DKA),   and  (b)  f ul l
acti v ati on  of   the  hepati c  k etogeni c  sy stem .   In  the  pr esence  of   a
r easonabl e  l ev el   of   endogenous  i nsul i n,   the  gl ucagon­to­i nsul i n  r ati o  i s
not  hi gh  enough  to  l ead  to  si gni f i cant  k etogenesi s  and  k etoaci dosi s.

4.   How   i s  hi s  l i v er   i nv ol v ed  i n  the  pathogenesi s  of   hi s  hy per gl y cem i a?

The  hy per gl y cem i a  i n  thi s  pati ent  r esul ts  f r om   the  i ncr eased  hepati c
pr oducti on  of   gl ucose  due  to  i ncr eased  gl y cogenol y si s  and
gl uconeogenesi s,   and  f r om   the  decr eased  uptak e  and  uti l i zati on  of
gl ucose  by   the  l i v er ,   m uscl e,   and  adi pose  ti ssue.   Al l   of   these  changes  ar e
due  to  the  under l y i ng  i nsul i n  r esi stance  of   T2DM  and  the  r el ati v e,   but  not
absol ute,   i nsul i n  def i ci ency   i n  the  pr esence  of   acute  str essf ul   condi ti ons.
In  addi ti on,   peopl e  w i th  T1DM  and  under l y i ng  r enal   di sease  m ay   pr esent
w i th  HHN KC  due  to  decr eased  cl ear ance  of   i nsul i n.

5.   What  ar e  the  m ajor   hor m ones  that  ar e  counter r egul ator y   to  i nsul i n
acti on?  Ar e  they   pl ay i ng  any   r ol e  i n  thi s  m an's  i l l ness?

Gl ucagon,   cor ti sol ,   catechol am i nes,   and  gr ow th  hor m one  (GH)  ar e  the
chi ef   i nsul i n  counter r egul ator y   hor m ones  that  ar e  el ev ated  i n  m ajor
str essf ul   condi ti ons  l i k e  HHN KC.   Thr ough  the  oper ati on  of   sev er al   speci f i c
m echani sm s,   they   counter act  the  ef f ects  of   i nsul i n,   and  thi s  w or sens  the
hy per gl y cem i c  state.

6.   What  w oul d  y ou  pr edi ct  about  the  state  of   hi s  i ntr av ascul ar   v ol um e?

The  i ntr av ascul ar   v ol um e  i s  sev er el y   depl eted  i n  thi s  pati ent  (note  the
r el ated  f i ndi ngs  r ev eal ed  by   the  phy si cal   ex am i nati on).   The  f ol l ow i ng
sequence  of   ev ents
P. 97
m ay   tak e  pl ace  i n  pati ents  w i th  T2DM  i f   they   ar e  not  adequatel y   tr eated:
hy per gl y cem i a  →  osm oti c  di ur esi s  →  l oss  of   f l ui d  and  el ectr ol y tes  →
dehy dr ati on  →  w or seni ng  hy per osm ol ar i ty   and  osm oti c  di ur esi s  →
el ev ated  counter r egul ator y   hor m ones  →  hem oconcentr ati on  and
hy pov ol em i a  →  pr er enal   azotem i a  →  ci r cul ator y
i nsuf f i ci ency /shock /l acti c  aci dosi s  →  i r r ev er si bl e  com a  →  death.
Ther ef or e,   i f   thi s  pati ent's  condi ti on  i s  not  r api dl y   tr eated,   i r r ev er si bl e
com a  and  death  m ay   ensue.

7.   What  ar e  the  m ajor   ther apeuti c  goal s  i n  thi s  pati ent?

The  m ajor   i m m edi ate  ther apeuti c  goal s  ar e  (a)  r epl acem ent  of   f l ui d  and
el ectr ol y tes,   (b)  cor r ecti on  of   the  hy per gl y cem i a  (r el ati v el y   sm al l   doses
of   i nsul i n  ar e  suf f i ci ent  f or   pati ents  i n  HHN KC  com par ed  w i th  DKA),   and
(c)  i denti f i cati on  and  m anagem ent  of   the  pr eci pi tati ng  f actor s.   HHN KC  i s
a  v er y   ser i ous  m edi cal   em er gency   w i th  a  hi gh  r i sk   of   m or tal i ty   unl ess  an
i m m edi ate,   aggr essi v e,   and  com pr ehensi v e  m anagem ent  r egi m en  i s
i nsti tuted.   Once  the  acute  si tuati on  i s  r esol v ed,   the  di abetes  m ay   be
m anaged  i n  the  l ong  ter m   ei ther   w i th  di et  and  or al   agents  or   w i th  i nsul i n.

Suggested Readings
Am er i can  Col l ege  of   Endocr i nol ogy   Task   For ce  on  Inpati ent  Di abetes  and
Metabol i c  Contr ol .   Am er i can  Col l ege  of   Endocr i nol ogy   posi ti on  statem ent  on
i npati ent  di abetes  and  m etabol i c  contr ol .   Endocr   Pr act  2004;10(1):77–82.

Am er i can  Di abetes  Associ ati on.   Standar ds  of   m edi cal   car e  i n  di abetesâ
€”2006.   Di abetes  Car e  2006;29(Suppl   1):S4–S42.

Bode  BW,   Br ai thw ai te  SS,   Steed  RD,   et  al .   Intr av enous  i nsul i n  i nf usi on
ther apy :  i ndi cati ons,   m ethods,   and  tr ansi ti on  to  subcutaneous  i nsul i n
ther apy .   Endocr   Pr act  2004;10(Suppl   2):71–80.

Br etzel   RG,   Voi gt  K,   Schatz  H.   The  U ni ted  Ki ngdom   pr ospecti v e  di abetes
study   (U KPDS)  i m pl i cati ons  f or   the  phar m acother apy   of   ty pe  2  di abetes
m el l i tus.   Ex p  Cl i n  Endocr i nol   Di abetes  1998;106:369.

Cr y er   PE.   Di v er se  causes  of   hy pogl y cem i a­associ ated  autonom i c  f ai l ur e  i n


di abetes.   N   Engl   J  Med  2004;350(22):2272–2279.

Di abetes  Contr ol   and  Com pl i cati ons  Tr i al   Resear ch  Gr oup.   The  ef f ect  of
i ntensi v e  tr eatm ent  of   di abetes  on  the  dev el opm ent  and  pr ogr essi on  of
l ong­ter m   com pl i cati ons  i n  i nsul i n  dependent  di abetes  m el l i tus.   N   Engl   J
Med  1993;329:977.

Eck el   RH,   Bar ouch  WW,   Er show   AG.   Repor t  of   the  N ati onal   Hear t,   Lung,   and
Bl ood  Insti tute­N ati onal   Insti tute  of   Di abetes  and  Di gesti v e  and  Ki dney
Di seases  Wor k i ng  Gr oup  on  the  pathophy si ol ogy   of   obesi ty ­associ ated
car di ov ascul ar   di sease.   Ci r cul ati on  2002;105(24):2923–2928.

Eck el   RH,   Gr undy   SM,   Zi m m et  PZ.   The  m etabol i c  sy ndr om e.   Lancet
2005;365:1415–1428.

Eck el   RH,   Wassef   M,   Chai t  A,   et  al .   Pr ev enti on  conf er ence  VI:  di abetes  and
car di ov ascul ar   di sease:  w r i ti ng  gr oup  II:  pathogenesi s  of   ather oscl er osi s  i n
di abetes.   Ci r cul ati on  2002;105(18):e138–e143.

Ex per t  Panel   on  Detecti on,   Ev al uati on,   and  Tr eatm ent  of   Hi gh  Bl ood
Chol ester ol   i n  Adul ts.   Ex ecuti v e  sum m ar y   of   the  thi r d  r epor t  of   the
nati onal   chol ester ol   educati on  pr ogr am   (N CEP)  ex per t  panel   on  detecti on,
ev al uati on,   and  tr eatm ent  of   hi gh  bl ood  chol ester ol   i n  adul ts  (Adul t
Tr eatm ent  Panel   III).   JAMA  2001;285(19):2486–2497.

P. 98

Gr undy   SM,   Br ew er   HB,   Cl eem an  JI,   et  al .   For   the  conf er ence  par ti ci pants.
Def i ni ti on  of   m etabol i c  sy ndr om e:  r epor t  of   the  N ati onal   Hear t,   Lung,   and
Bl ood  Insti tute/Am er i can  Hear t  Associ ati on  Conf er ence  on  Sci enti f i c  Issues
Rel ated  to  Def i ni ti on.   Ci r cul ati on  2004;109:433–438.

Gr undy   SM,   Cl eem an  JI,   Mer z  CN ,   et  al .   N ati onal   Hear t,   Lung,   and  Bl ood
Insti tute;  Am er i can  Col l ege  of   Car di ol ogy   Foundati on;  Am er i can  Hear t
Associ ati on.   Im pl i cati ons  of   r ecent  cl i ni cal   tr i al s  f or   the  N ati onal
Chol ester ol   Educati on  Pr ogr am   Adul t  Tr eatm ent  Panel   III  Gui del i nes.
Ci r cul ati on  2004;110(2):227–239.

Hi l l   JO,   Catenacci   V,   Wy att  HR.   Obesi ty :  ov er v i ew   of   an  epi dem i c.


Psy chi atr   Cl i n  N or th  Am   2005;28:1–23.

Kushner   RF,   Roth  JL.   Assessm ent  of   the  obese  pati ent.   Endocr i nol   Metab
Cl i n  N or th  Am   2003;32:915–933.

Moghi ssi   ES,   Hi r sch  IB.   Hospi tal   m anagem ent  of   di abetes.   Endocr i nol   Metab
Cl i n  N or th  Am   2005;34:99–116.

N ati onal   Di abetes  Data  Gr oup.   Cl assi f i cati on  and  di agnosi s  of   di abetes
m el l i tus  and  other   categor i es  of   gl ucose  i ntol er ance.   Di abetes
1979;28:1039.

Ri ddl e  MC.   Gl y cem i c  m anagem ent  of   ty pe  2  di abetes:  an  em er gi ng  str ategy
w i th  or al   agents,   i nsul i ns,   and  com bi nati ons.   Endocr i nol   Metab  Cl i n  N or th
Am   2005;34:77–98.

Tur ner   RC.   The  U . K.   pr ospecti v e  di abetes  study :  a  r ev i ew .   Di abetes  Car e


1998;21:35.

U K  Pr ospecti v e  Di abetes  Study   (U KPDS)  Gr oup.   Ef f ect  of   i ntensi v e  bl ood­


gl ucose  contr ol   w i th  m etf or m i n  on  com pl i cati ons  i n  ov er w ei ght  pati ents
w i th  ty pe  2  di abetes  (U KPDS  34):  U K  Pr ospecti v e  Di abetes  Study   (U KPDS)
Gr oup.   Lancet  1998;352:854.

U K  Pr ospecti v e  Di abetes  Study   (U KPDS)  Gr oup.   Intensi v e  bl ood­gl ucose


contr ol   w i th  sul phoy l ur eas  or   i nsul i n  com par ed  w i th  conv enti onal   tr eatm ent
and  r i sk   of   com pl i cati ons  i n  pati ents  w i th  ty pe  2  di abetes  (U KPDS  33):  U K
Pr ospecti v e  Di abetes  Study   (U KPDS)  Gr oup.   Lancet  1998;352:837.

Wy att  HR,   Hi l l   JO.   What  r ol e  f or   w ei ght­l oss  m edi cati on?  Wei ghi ng  the  pr os
and  cons  f or   obese  pati ents.   Postgr ad  Med  2004;115(1):38–40, 43â
€“45, 58.

Disorders of the Thyroid
1.   What  ar e  the  k ey   f eatur es  i n  a  pati ent's  hi stor y   that  ar e  i m por tant  i n
assessi ng  f or   a  possi bl e  f uncti onal   thy r oi d  di sor der ?

2.   What  ar e  the  i m por tant  phy si cal   ex am i nati on  f i ndi ngs?

3.   What  l abor ator y   data  ar e  used  to  conf i r m   or   r ef ute  the  ex i stence  of   a
f uncti onal   thy r oi d  abnor m al i ty ?

Discussion
1.   What  ar e  the  k ey   f eatur es  i n  a  pati ent's  hi stor y   that  ar e  i m por tant  i n
assessi ng  f or   a  possi bl e  f uncti onal   thy r oi d  di sor der ?

When  assessi ng  a  pati ent's  hi stor y   f or   cl ues  to  a  f uncti onal   thy r oi d
di sease,   i t  i s  i m por tant  to  k eep  i n  m i nd  that  thy r oi d  hor m ones  i n  gener al
contr ol   m etabol i sm .   Ther ef or e,   w hen  questi oni ng  pati ents  about  thei r
m edi cal   hi stor y ,   i t  i s  i m por tant  to  ask   speci f i cal l y   about  el em ents  r el ated
to  m etabol i sm .   For   ex am pl e,   i n  the  setti ng  of   hy per thy r oi di sm ,   w ei ght
l oss,   anx i ety ,   tr em or ,   pal pi tati ons,   heat  i ntol er ance,   hy per def ecati on,
i nsom ni a,   r estl essness,   and  changes  i n  the  hai r   or   sk i n  ar e  i m por tant
f eatur es.   In  contr ast,   i n  pati ents  w i th
P. 99
suspected  hy pothy r oi di sm ,   l ook   f or   cl ues  that  i ndi cate  decr eased
m etabol i c  acti v i ty .   These  i ncl ude  w ei ght  gai n;  col d  i ntol er ance;
consti pati on;  dr y ,   scal y   sk i n;  thi ck   hai r ;  depr essi on;  i ncr eased  sl eepi ng
and  f ati gue;  and  gener al i zed  l ethar gy .

2.   What  ar e  the  i m por tant  phy si cal   ex am i nati on  f i ndi ngs?

Li k e  the  hi stor y ,   the  phy si cal   ex am i nati on  shoul d  be  per f or m ed  to  l ook
f or   si gns  of   hy per m etabol i sm   or   hy pom etabol i sm .   In  the  setti ng  of
hy per thy r oi di sm ,   a  f ast  pul se;  tr em or ;  sw eati ng;  thi n,   sof t,   and  v el v ety
hai r ;  v er y   br i sk   r ef l ex es;  and  a  hy per dy nam i c  pr ecor di um   ar e  al l   f eatur es
of   i ncr eased  m etabol i sm .   In  addi ti on,   a  v er y   cr i ti cal   f i ndi ng  i s  an
enl ar ged  thy r oi d  gl and.   If   thi s  i s  f ound  i n  conjuncti on  w i th  a  br ui t,   then
the  cl i ni ci an  can  assum e  that  the  thy r oi d  gl and  i tsel f   i s  ov er acti v e  and
ov er pr oduci ng  thy r oi d  hor m one.   In  contr ast,   the  f i ndi ngs  char acter i sti c  of
hy pothy r oi di sm   i ncl ude  pal e,   sal l ow   sk i n;  thi ck   hai r ;  puf f i ness  i n  the  f ace
and  ank l es;  cool   ex tr em i ti es;  v er y   del ay ed  deep  tendon  r el ax ati on;
br ady car di a;  and  a  v er y   qui et  pr ecor di um .   Agai n,   an  enl ar ged  thy r oi d  i s
an  i m por tant  phy si cal   ex am i nati on  f i ndi ng.   In  thi s  ev ent,   a  f i r m ,   w oody ,
or   pebbl y   tex tur e  w oul d  i ndi cate  the  pr esence  of   l y m phocy ti c  i nf i l tr ati on
or   Hashi m oto's  thy r oi di ti s.

3.   What  l abor ator y   data  ar e  used  to  conf i r m   or   r ef ute  the  ex i stence  of   a
f uncti onal   thy r oi d  abnor m al i ty ?

Ther e  i s  now   a  v er y   sensi ti v e  and  speci f i c  l abor ator y   pr otocol   to


deter m i ne  w hether   the  pati ent  has  a  f uncti onal   thy r oi d  di sor der .   The  f i r st
di agnosti c  test  shoul d  be  m easur em ent  of   the  ser um   TSH  l ev el   usi ng  the
sensi ti v e  TSH  assay s.   If   thi s  assay   r esul t  pr ov es  to  be  w i thi n  the  nor m al
r ange,   then  a  f uncti onal   abnor m al i ty   of   the  thy r oi d  has  v i r tual l y   been
ex cl uded.   In  contr ast,   an  el ev ated  TSH  l ev el   m eans  the  thy r oi d  gl and  i s
f ai l i ng  and  the  pati ent  has  pr i m ar y   thy r oi d  gl and  f ai l ur e,   m ost  com m onl y
due  to  autoi m m une  thy r oi d  di sease.   Conv er sel y ,   i f   the  ser um   TSH  l ev el   i s
l ow   and  undetectabl e,   thi s  i ndi cates  hy per thy r oi di sm   due  to  Gr av es'
di sease,   a  m ul ti nodul ar   goi ter ,   a  hot  nodul e,   ex cessi v e  thy r oi d  hor m one
i ngesti on,   subacute  thy r oi di ti s,   postpar tum   thy r oi di ti s,   or   si l ent
thy r oi di ti s.   If   the  TSH  v al ue  i s  abnor m al ,   then  thy r oi d  hor m one  status
shoul d  be  assessed.   Thi s  can  be  done  ei ther   by   obtai ni ng  a  total   T 4   w i th  a
T 3   r esi n  uptak e  (to  assess  T 4 ­bi ndi ng  gl obul i n),   or   by   si m pl y   or der i ng  a
f r ee  T 4 .   Onl y   i n  speci al   ci r cum stances  i s  i t  necessar y   to  test  f or   total   T 3
or   f r ee  T 3   l ev el s.   Fi nal l y ,   i n  ev al uati ng  a  pati ent  w i th  suspected
hy per thy r oi di sm ,   i f   both  the  TSH  l ev el   i s  l ow   and  the  f r ee  T 4   l ev el   i s
hi gh,   the  nex t  step  i s  to  per f or m   a  r adi oacti v e  i odi ne  uptak e  test  and
scan.   Thi s  test  i s  v er y   i m por tant  i n  di sti ngui shi ng  causes  of
hy per thy r oi di sm   r el ated  to  ov er pr oducti on  (i . e. ,   Gr av es'  di sease,   a
m ul ti nodul ar   goi ter ,   or   a  hot  nodul e)  f r om   those  r el ated  to  ex cessi v e
r el ease  but  not  pr oducti on  (i . e. ,   subacute  thy r oi di ti s,   postpar tum
thy r oi di ti s,   or   si l ent  thy r oi di ti s),   as  w el l   as  ex cessi v e  thy r oi d  hor m one
i ngesti on.   The  scan  al so  i nf er s  the  ther apeuti c  appr oach.   In  the  setti ng  of
str ong  cl i ni cal   ev i dence  f or   hy pothy r oi di sm   w i th  a  l ow   or   nor m al   TSH,   i t
i s  al so  i m por tant  to  consi der   the  r el ati v el y   r ar e  possi bi l i ty   of   centr al ,   or
secondar y ,   hy pothy r oi di sm   (def ecti v e  TSH  pr oducti on  by   the  pi tui tar y
gl and).

P. 100
Case
A  31­y ear ­ol d  m other   of   tw o  i s  seen  because  of   com pl ai nts  of   headaches  and
am enor r hea,   w hi ch  hav e  l asted  f or   3  m onths.   She  del i v er ed  her   second  chi l d
10  m onths  ago.   The  headaches  dev el oped  af ter   she  w as  i n  a  m otor   v ehi cl e
acci dent  4  m onths  bef or e.   At  that  ti m e,   the  pati ent  ex per i enced  a  tem por ar y
l oss  of   consci ousness  but  has  si nce  been  nor m al ;  an  ex tensi v e  neur ol ogi c
ex am i nati on  i n  the  em er gency   r oom   y i el ded  negati v e  f i ndi ngs.   On  f ur ther
questi oni ng,   the  pati ent  al so  adm i ts  to  a  15­l b  (6. 75­k g)  w ei ght  l oss  despi te  a
nor m al   appeti te,   as  w el l   as  m i l d  heat  i ntol er ance  and  ex cessi v e  sw eati ng
dur i ng  the  sum m er   m onths.   Recentl y ,   she  has  noted  that  her   hands  shak e  and
her   handw r i ti ng  has  becom e  unev en.   Of   si gni f i cance  i s  her   ex er ci se  hi stor y ;
she  had  been  r unni ng  5  to  6  m i   a  day ,   5  day s  a  w eek ,   and  has  par ti ci pated  i n
m ar athon  r unni ng  com peti ti ons.   How ev er ,   ov er   the  l ast  3  m onths,   her
tol er ance  f or   ex er ci se  has  decr eased,   and  her   r unni ng  ti m es  hav e
deter i or ated.   On  questi oni ng  her   about  her   f am i l y   hi stor y ,   i t  i s  f ound  that  her
m other   tak es  l ev othy r ox i ne  f or   hy pothy r oi di sm ,   a  m ater nal   gr andm other   has
T2DM,   and  her   f ather   has  hy per l i pi dem i a  and  cor onar y   ar ter y   di sease.
Phy si cal   ex am i nati on  r ev eal s  a  w el l ­dev el oped,   w el l ­nour i shed,   thi n  w om an
w ho  appear s  som ew hat  anx i ous.   Her   BP  i s  130/50  m m   Hg  and  her   pul se  i s  120
beats  per   m i nute.   Her   hai r   i s  f i ne  w i th  str eak s  of   gr ay .   Her   ey es  ex hi bi t  no
ex ophthal m os,   but  ther e  i s  a  star e  and  l i d  l ag.   The  ex tr aocul ar   m uscl es  ar e
nor m al .   The  thy r oi d  i s  di f f usel y   enl ar ged  at  appr ox i m atel y   40  g.   Ther e  i s  a
hi gh­i ntensi ty   br ui t  audi bl e  ov er   the  r i ght  l obe  of   the  thy r oi d.   The  car di ac
ex am i nati on  r ev eal s  a  nor m al   f i r st  and  second  hear t  sound  and  a  gr ade  1/6
sy stol i c  ejecti on  m ur m ur .   The  l ungs  ar e  cl ear   to  auscul tati on  and  per cussi on.
Abdom i nal   ex am i nati on  f i ndi ngs  ar e  negati v e.   Her   hands  ex hi bi t  an
outstr etched  tr em or   and  her   sk i n  i s  noted  to  be  w ar m ,   sm ooth,   and  sl i ghtl y
m oi st.   Her   r ef l ex es  ar e  sy m m etr i cal l y   br i sk .   Ther e  i s  sl i ght  pr ox i m al   m uscl e
w eak ness  detected  i n  the  thi ghs  and  shoul der   gi r dl e  m uscl es.

1.   What  i s  the  di f f er enti al   di agnosi s  i n  thi s  pati ent,   and  shoul d  i t  i ncl ude  a
nor m al   pr egnancy ?
2.   What  i s  the  m ost  ef f i ci ent  appr oach  to  the  l abor ator y   ev al uati on  i n  thi s
pati ent?
3.   To  di sti ngui sh  si l ent  thy r oi di ti s  f r om   Gr av es'  hy per thy r oi di sm ,   w hat  i s  the
m ost  i m por tant  di agnosti c  tool ?
4.   If   the  pati ent  has  si l ent  thy r oi di ti s,   w hat  i s  the  appr opr i ate  ther apy ?
5.   If   the  pati ent  has  Gr av es'  di sease  and  i s  tr eated  w i th  r adi oacti v e  i odi ne,
w hat  i s  l i k el y   to  occur ?
6.   If   the  pati ent  i s  tr eated  w i th  anti thy r oi d  dr ugs,   w hat  i s  the  l i k el y   shor t­
and  l ong­ter m   pr ognosi s?
Case Discussion
1.   What  i s  the  di f f er enti al   di agnosi s  i n  thi s  pati ent,   and  shoul d  i t  i ncl ude  a
nor m al   pr egnancy ?

A  nor m al   pr egnancy   can  m i m i c  m any   of   the  sy m ptom s  of


hy per thy r oi di sm ,   i ncl udi ng  i ncr eased  ener gy ,   anx i ety ,   heat  i ntol er ance,
sw eati ng,   and,   i n  ar eas  of   the  w or l d  w her e  i odi ne  def i ci ency   i s  com m on,
a  m i l d  i ncr ease  i n  the  thy r oi d  gl and  si ze.   In  addi ti on,   pr egnancy   can
cer tai nl y   decr ease  the  tol er ance  f or   m ax i m al   ex er ci se.   Featur es  that  ar e
not  char acter i sti c  of   pr egnancy   ar e  the  m oder ate  (40  g)  thy r oi d
P. 101
enl ar gem ent,   the  l i d  l ag  and  star e,   and,   m ost  i m por tant,   the  br ui t  ov er
the  r i ght  si de  of   the  thy r oi d  gl and.   A  br ui t  i n  the  thy r oi d  gl and  r ef l ects
the  pr esence  of   i ncr eased  bl ood  f l ow   due  to  hy per pl asi a  and  ex cessi v e
thy r oi d  gl and  f uncti on.   These  f eatur es  ar e  absent  i n  pr egnancy ,   and
ther ef or e  a  br ui t  w oul d  not  be  hear d.   How ev er ,   pr ev i ousl y   si l ent  thy r oi d
di sease  m ay   becom e  appar ent  dur i ng  pr egnancy .   In  addi ti on,   a  15­l b
(6. 75­k g)  w ei ght  l oss,   despi te  a  nor m al   appeti te,   w oul d  be  di sti nctl y
unusual   i n  the  pr egnant  state.   Ther ef or e,   a  nor m al   pr egnancy   i s  an
unl i k el y   cause  of   thi s  pati ent's  sy m ptom s.   The  di f f er enti al   di agnosi s
ther ef or e  consi sts  of   hy per thy r oi di sm   due  to  Gr av es'  di sease,   a
m ul ti nodul ar   goi ter ,   and  si l ent  thy r oi di ti s.   A  m ul ti nodul ar   goi ter   i s
unusual   i n  a  31­y ear ­ol d  pati ent,   and  i s  usual l y   seen  i n  the  ol der
popul ati on.   Fur ther m or e,   m ul ti nodul ar   goi ter s  ar e  not  associ ated  w i th  a
br ui t,   ev en  w hen  pr oduci ng  hy per thy r oi di sm .   The  di f f use  enl ar gem ent  of
the  thy r oi d  gl and  at  40  g  i s  al so  unusual   i n  the  setti ng  of   a  m ul ti nodul ar
goi ter   because,   i n  thi s  ev ent,   m ul ti pl e  nodul es  shoul d  be  appr eci ated  on
the  phy si cal   ex am i nati on.   A  star e  and  l i d  l ag  can  be  f ound  i n  pati ents
w i th  hy per thy r oi di sm   due  to  a  m ul ti nodul ar   goi ter   because  these  f i ndi ngs
r ef l ect  the  hy per thy r oi di sm ,   not  the  autoi m m une  pr ocess.

The  i m por tant  di f f er enti al   di agnosti c  ex er ci se  i n  thi s  case  shoul d  f ocus  on
w hether   the  hy per thy r oi di sm   i s  due  to  Gr av es'  di sease  or   si l ent
thy r oi di ti s.   Gr av es'  di sease  i s  the  m ost  com m on  cause  of   hy per thy r oi di sm
and  i s  f ound  m or e  f r equentl y   i n  w om en  than  i n  m en,   w i th  a  r ati o  of   4:1.
In  addi ti on,   hy per thy r oi di sm   due  to  Gr av es'  di sease  usual l y   af f l i cts
y ounger   peopl e  betw een  20  and  50  y ear s  of   age.   The  15­l b  (6. 75­k g)
w ei ght  l oss,   heat  i ntol er ance,   ex cessi v e  sw eati ng,   tr em or ,   decr eased
ex er ci se  tol er ance,   m oder ate  enl ar gem ent  of   the  thy r oi d  w i th  a  br ui t,   and
the  w ar m ,   sm ooth,   and  sl i ghtl y   m oi st  sk i n  ar e  al l   char acter i sti c  of
hy per thy r oi di sm   due  to  Gr av es'  di sease.   In  addi ti on,   the  pati ent  has  a
f am i l y   hi stor y   of   autoi m m une  di sease,   i n  that  the  m other   i s  bei ng  tr eated
f or   hy pothy r oi di sm   and  the  gr andm other   has  adul t­onset  DM.   In  addi ti on,
the  pati ent  appear s  to  hav e  pr em atur e  gr ay   hai r .   The  absence  of
ex ophthal m os  does  not  conf l i ct  w i th  thi s  di agnosi s  because  thi s  f i ndi ng
m ay   be  cl i ni cal l y   ev i dent  i n  onl y   10%  to  20%  of   pati ents  w i th  Gr av es'
hy per thy r oi di sm .   (How ev er ,   w hen  m or e  sophi sti cated  techni ques  f or
ev al uati ng  ey e  f uncti on  ar e  used,   as  m any   as  80%  to  90%  of   the  pati ents
w i th  Gr av es'  hy per thy r oi di sm   pr ov e  to  hav e  di scer ni bl e  ey e
abnor m al i ti es. )

Hy per thy r oi di sm   due  to  si l ent  thy r oi di ti s  i s  becom i ng  an  i ncr easi ngl y
w el l ­r ecogni zed  di agnosti c  enti ty .   The  ex act  eti ol ogy   of   thi s  di sor der   i s
obscur e,   but  appear s  to  be  an  autoi m m une  pr ocess.   Si l ent  thy r oi di ti s  i s
per haps  i denti cal   to  postpar tum   thy r oi di ti s,   w hi ch  ar i ses  i n  5%  to  8%  of
al l   pr egnanci es  i n  the  U ni ted  States.   Thi s  di sor der   appear s  usual l y
betw een  2  and  6  m onths  postpar tum   (10  m onths  i s  sl i ghtl y   ex cessi v e).
The  pati ent  ex hi bi ts  the  si gns  and  sy m ptom s  of   hy per thy r oi di sm   and  an
enl ar ged  thy r oi d  gl and,   but  has  no  ev i dence  of   ex ophthal m os  or   pr eti bi al
P. 102
of   the  tw o  di sor der s  ar e  qui te  di f f er ent.   Because  am enor r hea  can  occur   i n
al l   f or m s  of   hy per thy r oi di sm ,   i t  i s  not  a  hel pf ul   cl ue  f or   i denti f y i ng  the
ul ti m ate  cause  of   the  hy per thy r oi di sm .   Fi nal l y ,   the  br ui t  ov er   the  r i ght
l obe  of   the  thy r oi d  i s  an  i m por tant  cl ue  that  poi nts  tow ar d  a  di agnosi s  of
Gr av es'  di sease  i n  thi s  pati ent.   A  br ui t  ov er   the  thy r oi d  gl and  i s  usual l y
not  pr esent  i n  hy per thy r oi di sm   due  to  a  m ul ti nodul ar   goi ter ,   si l ent
thy r oi di ti s,   or   subacute  thy r oi di ti s.   Ther ef or e,   on  the  basi s  of   the
pati ent's  hi stor y ,   phy si cal   ex am i nati on  f i ndi ngs,   and  stati sti cal
consi der ati ons  the  m ost  l i k el y   di agnosi s  i s  hy per thy r oi di sm   due  to
Gr av es'  di sease.   How ev er ,   f or m al   l abor ator y   studi es  shoul d  be  done  f i r st
to  deter m i ne  w hether   hy per thy r oi di sm   i s  i ndeed  pr esent  and  then  to
i denti f y   the  cause  of   the  hy per thy r oi di sm .

2.   Whi ch  i s  the  m ost  ef f i ci ent  appr oach  to  the  l abor ator y   ev al uati on  i n  thi s
pati ent?

The  k ey   i ssue  i n  deci di ng  on  the  natur e  of   the  l abor ator y   ev al uati on  i s
w hi ch  test  best  deter m i nes  i f   hy per thy r oi di sm   i s  i ndeed  pr esent.   Total   T 4
and  T 3   r esi n  uptak e  w er e  tr adi ti onal l y   r egar ded  as  the  best  tests  f or
establ i shi ng  the  pr esence  of   hy per thy r oi di sm   and  f or   di sti ngui shi ng
hy per thy r oi di sm   f r om   a  nor m al   pr egnancy .   In  al l   f or m s  of
hy per thy r oi di sm ,   both  total   T 4   and  T 3   r esi n  uptak e  ar e  el ev ated,   w her eas
i n  pr egnancy ,   the  total   T 4   i s  el ev ated  because  of   an  i ncr ease  i n  the  T 4 ­
bi ndi ng  gl obul i n  l ev el ,   and  the  T 3   r esi n  uptak e  i s  r educed,   agai n  because
of   the  i ncr eased  T 4 ­bi ndi ng  gl obul i n  l ev el .   How ev er ,   these  sam e
abnor m al i ti es  i n  thy r oi d  f uncti on  can  occur   i n  the  absence  of   pr egnancy ,
such  as  i n  a  pati ent  tak i ng  bi r th  contr ol   pi l l s  or   r epl acem ent  estr ogen,   or
i n  a  w om an  w i th  congeni tal   X­l i nk ed  T 4 ­bi ndi ng  gl obul i n  ex cess.
Fur ther m or e,   conf usi on  can  ar i se  w hen  both  hy per thy r oi di sm   and
ex cessi v e  estr ogens  coex i st  because  the  T 4   concentr ati on  can  be  el ev ated
and  the  T 3   r esi n  uptak e  m ay   be  v ar i abl e  (l ow ,   nor m al ,   or   hi gh  v al ues)  i n
thi s  setti ng.   The  total   T 3   i s  a  good  test  f or   hy per thy r oi di sm   but  because
T 3   i s  al so  bound  to  T 4 ­bi ndi ng  gl obul i n  i ts  l ev el s  ar e  el ev ated  i n  the
contex t  of   a  nor m al   pr egnancy   or   ex ogenous  estr ogen.   The  m ost  ef f i ci ent
l abor ator y   tests  i n  thi s  case  ar e  deter m i nati ons  of   the  f r ee  T 4   and  TSH
l ev el s.   The  f r ee  T 4   l ev el   i s  el ev ated  i n  hy per thy r oi di sm   but  nor m al   i n
pr egnancy .   Li k ew i se,   the  TSH  l ev el   i s  suppr essed  and  undetectabl e  i n  the
setti ng  of   hy per thy r oi di sm   but  nor m al   i n  pr egnancy .   Ther ef or e,   thi s
l abor ator y   pr of i l e  i s  i deal   f or   di scr i m i nati ng  betw een  hy per thy r oi di sm   and
pr egnancy ­r el ated  changes  i n  thy r oi d  l ev el s.   How ev er ,   the  f r ee  T 4   and
TSH  l ev el s  cannot  di scr i m i nate  betw een  hy per thy r oi di sm   due  to  Gr av es'
di sease,   a  m ul ti nodul ar   goi ter ,   or   si l ent  thy r oi di ti s.

3.   To  di sti ngui sh  si l ent  thy r oi di ti s  f r om   Gr av es'  hy per thy r oi di sm ,   w hat  i s  the
m ost  i m por tant  di agnosti c  tool ?

The  m ost  i m por tant  di agnosti c  tool   f or   di sti ngui shi ng  si l ent  thy r oi di ti s
f r om   Gr av es'  di sease  i s  a  r adi oacti v e  i odi ne  uptak e  test.   Gr av es'  di sease,
w hi ch  i s  a  state  of   thy r oi d  hor m one  ov er pr oducti on,   i nv ol v es  an  ex cessi v e
uptak e  of   i odi ne  i nto  the  thy r oi d  gl and  and  ther ef or e  a  hi gh  r adi oacti v e
i odi ne  uptak e.   In  contr ast,   si l ent  thy r oi di ti s  i s  not  an  ov er pr oducti on
state  but  a  state  i n  w hi ch  ther e  i s  an  ex cessi v e  r el ease  of   thy r oi d
hor m one  i nto  the  ci r cul ati on  that  suppr esses  TSH,   stem m i ng  f r om
autoi m m une  dam age  to  thy r oi d  f ol l i cul ar   cel l s.   These  ev ents  r ender   the
thy r oi d  gl and  i ncapabl e  of   tak i ng  up  i odi ne.   Ther ef or e,   i n  the  setti ng  of
si l ent  thy r oi di ti s,   the  r adi oacti v e  i odi ne  uptak e  i s  v er y   l ow ,   i n  str i k i ng
contr ast  to  the  el ev ated  v al ues  f ound  i n  pati ents  w i th  Gr av es'
hy per thy r oi di sm .

P. 103
4.   If   the  pati ent  has  si l ent  thy r oi di ti s,   w hat  i s  the  appr opr i ate  ther apy ?

Di sti ngui shi ng  si l ent  thy r oi di ti s  f r om   Gr av es'  di sease  i s  i m por tant  because
the  ther api es  f or   the  tw o  condi ti ons  ar e  dr am ati cal l y   di f f er ent.   Because
si l ent  thy r oi di ti s  i s  a  destr ucti v e  pr ocess  w i thout  ov er pr oducti on,   i t  does
not  r espond  to  ei ther   anti thy r oi d  dr ugs  or   r adi oacti v e  i odi ne.   The  l ow
r adi oacti v e  i odi ne  uptak e  i n  si l ent  thy r oi di ti s  r ender s  r adi oacti v e  i odi ne
ther apy   i nef f ecti v e,   and  the  l ack   of   ov er pr oducti on  of   thy r oi d  hor m ones
negates  the  ef f ecti v eness  of   anti thy r oi d  dr ugs.   Because  si l ent  thy r oi di ti s
i s  a  sel f ­l i m i ted  pr ocess  w i th  a  tr i phasi c  cour se,   the  r ecom m ended
ther apy   i s  the  judi ci ous  use  of   β­bl ock er s  to  contr ol   sy m ptom s,
par ti cul ar l y   tr em or   and  tachy car di a,   and  obser v ati on  of   the  pati ent  dur i ng
the  spontaneous  r esol uti on  of   the  pr ocess.   Ty pi cal l y ,   the  hy per thy r oi d
phase  l asts  f or   1  to  3  m onths,   af ter   w hi ch  the  pr ocess  i s  dr am ati cal l y
r ev er sed;  i n  f act,   hy pothy r oi di sm   can  occur   tr ansi entl y   f or   another   1  to  3
m onths.   Dur i ng  the  hy pothy r oi d  phase,   m any   pati ents  benef i t  f r om   a
shor t  cour se  of   thy r oi d  hor m one  r epl acem ent.   How ev er ,   i n  m ost  cases,
both  the  hy per thy r oi di sm   and  hy pothy r oi di sm   r esol v e  spontaneousl y   and
nor m al   thy r oi d  f uncti on  i s  r estor ed.   In  onl y   10%  to  25%  of   cases  does
per m anent  hy pothy r oi di sm   ev entuate.   Per haps  the  m ost  i m por tant  cl i ni cal
cl ue  to  the  r esol uti on  of   si l ent  thy r oi di ti s  i s  nor m al i zati on  of   the  thy r oi d
gl and  si ze.   Because  si l ent  thy r oi di ti s  i s  a  sel f ­l i m i ted  pr ocess  that
r esol v es  spontaneousl y ,   thy r oi d  sur ger y   i s  usual l y   unnecessar y .   How ev er ,
thi s  opti on  can  be  r eser v ed  f or   par ti cul ar l y   sev er e  cases  that  hav e
pr otr acted  or   r ecur r ent  cour ses.

5.   If   the  pati ent  has  Gr av es'  di sease  and  i s  tr eated  w i th  r adi oacti v e  i odi ne,
w hat  i s  l i k el y   to  occur ?

The  m ost  com m on  ther apy   f or   Gr av es'  di sease  i n  the  U ni ted  States  i s
r adi oacti v e  i odi ne,   w hi ch  i s  gi v en  i n  the  f or m   of   a  sm al l   capsul e
contai ni ng  5  to  10  m Ci   of   i odi ne­131  ( 1 3 1 I).   The  r adi oacti v e  i odi ne  i s
qui ck l y   absor bed  f r om   the  gastr oi ntesti nal   tr act  i nto  the  bl oodstr eam   and
then  i ncor por ated  i nto  the  thy r oi d  gl and,   w her e  i t  i nduces  r adi oacti v e
dam age  and  k i l l s  thy r oi d  cel l s.   Radi oacti v e  i odi ne  that  does  not  enter   the
thy r oi d  gl and  i s  qui ck l y   ex cr eted  thr ough  the  k i dney s  i nto  the  ur i ne.
Because  r adi oacti v e  i odi ne  i nduces  dam age  and  ev entual   death  of   thy r oi d
f ol l i cul ar   cel l s,   the  chance  of   hy pothy r oi di sm   dev el opi ng  i s  v er y   hi gh.   In
gener al ,   hy pothy r oi di sm   dev el ops  i n  the  f i r st  y ear   i n  50%  to  60%  of   the
pati ents  tr eated;  ther eaf ter ,   the  r ate  of   dev el opm ent  of   hy pothy r oi di sm   i s
1%  to  3%  per   y ear .   Ther ef or e,   hy pothy r oi di sm   w i l l   dev el op  i n  m ost
pati ents  tr eated  w i th  r adi oacti v e  i odi ne  and  they   w i l l   r equi r e  l i f el ong
thy r oi d  hor m one  r epl acem ent  ther apy .

6.   If   the  pati ent  i s  tr eated  w i th  anti thy r oi d  dr ugs,   w hat  i s  the  l i k el y   shor t­
and  l ong­ter m   pr ognosi s?

Anti thy r oi d  dr ugs  (pr opy l thi our aci l   and  m ethi m azol e)  ar e  der i v ati v es  of
thi our ea  and  thei r   m echani sm   of   acti on  i s  to  i nhi bi t  both  thy r oi d  hor m one
sy nthesi s  and,   i n  the  case  of   pr opy l thi our aci l ,   the  per i pher al   conv er si on
of   T 4   to  T 3 .   These  dr ugs  ar e  an  i deal   choi ce  of   ther apy   f or   pati ents  w i th
hy per thy r oi di sm   due  to  Gr av es'  di sease.   They   ar e  m ost  com m onl y   used  i n
chi l dr en  and  pr egnant  pati ents,   and  i n  r el ati v el y   m i l d  cases  of
hy per thy r oi di sm   i n  w hi ch  the  thy r oi d  gl and  i s  onl y   m oder atel y   enl ar ged.
Pr opy l thi our aci l   i s  gi v en  i n  a  dose  of   appr ox i m atel y   100  m g  thr ee  or   f our
ti m es  a  day .   Methi m azol e  (Tapazol e;  El i   Li l l y ,   Indi anapol i s,   IN )  has  a
sl i ghtl y   l onger   hal f ­l i f e
P. 104
than  pr opy l thi our aci l   and  m ay   be  gi v en  i n  a  si ngl e  dai l y   dose  of   10  to  30
m g  each  day .   Both  agents  ex hi bi t  a  si m i l ar   pr of i l e  of   si de  ef f ects,   w hi ch
occur   i n  1%  to  3%  of   the  pati ents.   The  m ost  com m on  si de  ef f ects  ar e
sk i n  r ash,   ur ti car i a,   ar thr al gi as,   f ev er ,   and  tr ansi ent  l euk openi a.   Mi nor
gastr oi ntesti nal   si de  ef f ects  and  ar thr i ti s  occur   occasi onal l y .   The  m ajor
r ar e  si de  ef f ect  i s  agr anul ocy tosi s,   w hi ch  occur s  i n  0. 2%  to  0. 5%  of   the
pati ents.   Other   r ar e  si de  ef f ects  i ncl ude  apl asti c  anem i a,   hepati ti s,
thr om bocy topeni a,   v ascul i ti s,   and  chol estati c  jaundi ce.   The  si de  ef f ects
usual l y   ar i se  ear l y   i n  the  cour se  of   ther apy ,   and  i n  the  case  of
m ethi m azol e  the  r eacti ons  appear   to  be  dose  dependent.

The  anti thy r oi d  dr ugs  ar e  usual l y   gi v en  f or   a  1­  to  2­y ear   per i od  i n  the
hope  of   i nduci ng  per m anent  r em i ssi on.   The  f r equency   w i th  w hi ch
per m anent  r em i ssi on  tak es  pl ace  has  been  anal y zed  i n  m any   studi es  and
has  been  f ound  to  occur   i n  those  pati ents  w ho  hav e  m i l d  di sease  and
sm al l er   thy r oi d  gl ands.   How ev er ,   w hen  al l   pati ents  ar e  consi der ed,   the
chance  f or   per m anent  r em i ssi on  i s  onl y   20%  to  40%.   Ther ef or e,   60%  to
80%  of   pati ents  hav e  a  r el apse  of   thei r   hy per thy r oi di sm ,   usual l y   w i thi n  2
y ear s  of   di sconti nui ng  the  anti thy r oi d  dr ug.   In  cases  of   r el apse,   the
anti thy r oi d  dr ug  can  ei ther   be  r ei nsti tuted  or   a  m or e  def i ni ti v e  abl ati v e
f or m   of   ther apy ,   such  as  r adi oacti v e  i odi ne,   can  be  adm i ni ster ed.

Suggested Readings
Bar tal ena  L,   Mar cocci   C,   Bogazzi   F,   et  al .   Rel ati on  betw een  ther apy   f or
hy per thy r oi di sm   and  the  cour se  of   Gr av es'  ophthal m opathy .   N   Engl   J  Med
1998;338:73.

Bur guer a  B,   Ghar i b  H.   Thy r oi d  i nci dental om as:  pr ev al ence,   di agnosi s,


si gni f i cance,   and  m anagem ent.   Endocr i nol   Metab  Cl i n  N or th  Am
2000;29(1):187–203.

Castr o  MR,   Ghar i b  H.   Conti nui ng  contr ov er si es  i n  the  m anagem ent  of
thy r oi d  nodul es.   Ann  Inter n  Med  2005;142:926–931.

Cooper   DS,   Doher ty   GM,   Haugen  BR,   et  al .   Managem ent  gui del i nes  f or
pati ents  w i th  thy r oi d  nodul es  and  di f f er enti ated  thy r oi d  cancer .   Thy r oi d
2006;16(2):1–33.

Ghar i b  H,   Tuttl e  RM,   Bask i n  HJ,   et  al .   Subcl i ni cal   thy r oi d  dy sf uncti on:  a
joi nt  statem ent  on  m anagem ent  f r om   the  Am er i can  Associ ati on  of   Cl i ni cal
Endocr i nol ogi sts,   the  Am er i can  Thy r oi d  Associ ati on,   and  the  Endocr i ne
Soci ety .   J  Cl i n  Endocr i nol   Metab  2005;90(1):581–585;  di scussi on586â
€“587.

Haugen  BR.   Ini ti al   tr eatm ent  of   di f f er enti ated  thy r oi d  car ci nom a.   Rev
Endocr   Metab  Di sor d  2000;1(3):139–145.

Kahal y   JG,   Di l l m ann  HW.   Thy r oi d  hor m one  acti on  i n  the  hear t.   Endocr   Rev
2005;26:704–728.

Lev y   EG,   Ri dgw ay   EC,   War tof sk y   L.   Al gor i thm s  f or   di agnosi s  and
m anagem ent  of   thy r oi d  di sor der s,   ©  2003­2004.   Av ai l abl e  f r om :
http://w w w . Thy r oi dtoday . com /Ex per tOpi ni ons/Thy r oi dDi seaseAl gor i thm s. pdf .

Sar l i es  N J,   Gour gi oti s  L.   Thy r oi d  em er genci es.   Rev   Endocr   Metab  Di sor d
2003;4:129–136.

Stathatos  N ,   War tof sk y   L.   The  euthy r oi d  si ck   sy ndr om e:  i s  ther e  a


phy si ol ogi c  r ati onal e  f or   thy r oi d  hor m one  tr eatm ent?  J  Endocr i nol   Inv est
2003;26(12):1174–1179.
Sur k s  MI,   Or ti z  E,   Dani el s  GH,   et  al .   Subcl i ni cal   thy r oi d  di sease:  sci enti f i c
r ev i ew   and  gui del i nes  f or   di agnosi s  and  m anagem ent.   JAMA
2004;291(2):228–238.

P. 105

Growth Hormone­Secreting Pituitary Tumors
1.   What  ar e  the  cl i ni cal   sy m ptom s  of   GH  ex cess  (i . e. ,   acr om egal y )?

2.   What  ar e  the  phy si cal   si gns  of   acr om egal y ?

3.   What  i s  the  best  scr eeni ng  test  to  ex cl ude  the  di agnosi s  of   acr om egal y ?

4.   What  l abor ator y   tests  can  conf i r m   the  di agnosi s  of   acr om egal y   and  assess
other   pi tui tar y   hor m one  f uncti ons?

5.   What  i m agi ng  studi es  ar e  necessar y ?

6.   What  i s  the  tr eatm ent  of   choi ce  and  w hat  ar e  the  al ter nati v es?

Discussion
1.   What  ar e  the  cl i ni cal   sy m ptom s  of   GH  ex cess  (i . e. ,   acr om egal y )?

Acr om egal y   i s  the  cl i ni cal   sy ndr om e  r esul ti ng  f r om   ex cessi v e  GH


pr oducti on  and  i s  usual l y   due  to  a  pi tui tar y   tum or .   The  sy m ptom s  and
si gns  ar e  gr adual   i n  onset,   w hi ch  of ten  cause  di agnosi s  to  be  del ay ed  f or
6  to  8  y ear s.   The  cl assi c  sy m ptom s  consi st  of   headache,   v i sual
di stur bances  (due  to  an  enl ar gi ng  tum or   com pr essi ng  on  the  opti c
ner v es),   enl ar gem ent  i n  gl ov e  and  shoe  si ze,   ex cessi v e  sw eati ng,
ar thr al gi as,   l oss  of   l i bi do,   i m potence  i n  m en,   am enor r hea  i n  w om en,
m uscl e  w eak ness,   and  pr obl em s  w i th  an  under bi te  or   spaces  betw een  the
teeth.

2.   What  ar e  the  phy si cal   si gns  of   acr om egal y ?

The  phy si cal   si gns  of   acr om egal y   i ncl ude  coar seni ng  of   the  phy si cal
f eatur es  (w hi ch  can  be  deter m i ned  by   l ook i ng  at  ol d  photogr aphs);  f r ontal
bossi ng;  thi ck ,   coar se  sk i n;  doughy ,   sw eaty   pal m s;  pr ognathi sm   (an
enl ar ged  m andi bl e);  w i del y   spaced  teeth  and  under bi te;  sev er e
osteoar thr i ti s;  pr om i nent  l i ps,   tongue,   and  nose;  acanthosi s  ni gr i cans;
sk i n  tags;  enl ar gem ent  of   al l   or gans;  entr apm ent  of   per i pher al   ner v es
(i . e. ,   car pal   tunnel );  hy per tensi on  w i th  car di om y opathy ;  obstr ucti v e  sl eep
apnea  v i sual   f i el d  abnor m al i ti es;  gl ucose  i ntol er ance;  and  di abetes.

3.   What  i s  the  best  scr eeni ng  test  to  ex cl ude  the  di agnosi s  of   acr om egal y ?

The  best  scr eeni ng  test  f or   acr om egal y   i s  m easur em ent  of   the  i nsul i n­l i k e
gr ow th  f actor   I  (IGF­I;  som atom edi n  C)  l ev el .   Thi s  i s  a  l i v er   pr otei n
i nduced  by   GH  and  the  test  consti tutes  an  i ntegr ated  assessm ent  of   GH
acti on.   GH  l ev el s  ar e  pul sati l e  i n  natur e—f asti ng  l ev el s  ar e  usual l y   l ess
than  10  ng/m L  i n  adul ts  and  the  GH  concentr ati on  i ncr eases  at  ni ght
dur i ng  sl eep.   As  the  l ev el s  i n  pati ents  w i th  acr om egal y   can  be  as  l ow   as
5–10  ng/m L,   the  GH  l ev el   i s  hel pf ul   i f   v er y   hi gh,   but  i s  nei ther
sensi ti v e  nor   speci f i c  i f   nor m al   or   m i l dl y   el ev ated.

4.   What  l abor ator y   tests  can  conf i r m   the  di agnosi s  of   acr om egal y   and  assess
other   pi tui tar y   hor m one  f uncti ons?

To  conf i r m   the  di agnosi s  and  assess  the  pi tui tar y   f uncti on  i n  a  pati ent,
the  GH  r esponse  to  an  or al   gl ucose  l oad  i s  assessed.   To  per f or m   thi s,   100
g  or al   gl ucose  i s  gi v en  to  the  f asti ng  pati ent  and  bl ood  f or   GH
deter m i nati on  i s
P. 106
obtai ned  at  0,   30,   60,   90,   and  120  m i nutes.   In  a  nor m al   r esponse  to  a
gl ucose  l oad,   the  GH  l ev el s  ar e  suppr essed  to  l ess  than  2  ng/m L.   In
pati ents  w i th  acr om egal y ,   the  GH  l ev el s  ar e  ei ther   not  suppr essed  or   they
par adox i cal l y   i ncr ease  (appr ox i m atel y   30%  of   pati ents).   Thi s  test,   i f   done
i n  conjuncti on  w i th  an  IGF­1  test,   i s  the  best  w ay   of   assessi ng  a  cur e
af ter   sur ger y .   In  addi ti on,   appr ox i m atel y   30%  of   the  pati ents  show   an
i ncr ease  i n  thei r   GH  l ev el s  af ter   a  thy r otr opi n­r el easi ng  hor m one  (500
µg)  IV  push;  heal thy   i ndi v i dual s  do  not.   Thi s  test  i s  m or e  of ten  used  i n  a
r esear ch  setti ng  or   f or   the  pur pose  of   assessi ng  tum or   r ecur r ence.

Other   bl ood  tests  f or   deter m i ni ng  pi tui tar y   f uncti on  i ncl ude  m easur em ent
of   the  pr ol acti n  l ev el ,   w hi ch  can  be  el ev ated  because  of   the  i nter r upti on
of   dopam i ne  tone  secondar y   to  stal k   com pr essi on  or   the  cosecr eti on  of
pr ol acti n  by   the  tum or .   The  f ol l i cl e­sti m ul ati ng  hor m one  (FSH),   l utei ni zi ng
hor m one  (LH),   and  testoster one  or   estr ogen  l ev el s  ar e  used  to  assess  the
status  of   the  r epr oducti v e  ax i s.   The  thy r oi d  status  i s  check ed  by   a  f r ee
T 4 ;  m easur em ent  of   the  TSH  l ev el   i s  not  hel pf ul   because,   i f   ther e  i s
thy r oi d  def i ci ency ,   i t  i s  secondar y   to  TSH  def i ci ency .   The  adr enal   ax i s  i s
assessed  w i th  a  m or ni ng  cor ti sol   deter m i nati on;  i f   l ess  than  5  m g/dL,   a
f or m al   cosy ntr opi n  (Cor tr osy n;  Or ganon  Tek ni k a,   Dur ham ,   N C)
sti m ul ati on  test  of   the  ACTH  r eser v e  shoul d  be  per f or m ed.   Bl ood  i s  dr aw n
at  0,   30,   and  60  m i nutes  af ter   one  am pul e  (0. 25  m g)  of   sy ntheti c  ACTH  IV
push  or   IM.   In  heal thy   subjects,   the  cor ti sol   l ev el s  i ncr ease  to  18  or   m or e
and  ar e  of ten  doubl e  the  basel i ne  v al ue.   An  α ­subuni t  l ev el   m ay   be
hel pf ul   as  a  tum or   m ar k er   because  som e  GH  tum or s  cosecr ete  other
pi tui tar y   hor m ones.

5.   What  i m agi ng  studi es  ar e  necessar y ?

The  i m agi ng  pr ocedur e  of   choi ce  i s  an  MRI  scan  of   the  pi tui tar y .   It
pr ov i des  the  gr eatest  detai l   of   the  tum or 's  ex tent  and  l andm ar k s  f or   the
sur geon's  use  dur i ng  sur gi cal   r em ov al .   A  cor onal   CT  scan  w i th  f i ne  cuts
can  detect  m ost  l ar ge  tum or s,   but  a  l ater al   sk ul l   f i l m   i s  not  sensi ti v e.
For m al   v i sual   f i el d  testi ng  shoul d  be  done  i n  al l   pati ents  w i th
m acr oadenom as  to  ser v e  as  a  basel i ne  f or   assessi ng  postoper ati v e
i m pr ov em ent.

6.   What  i s  the  tr eatm ent  of   choi ce  and  w hat  ar e  the  al ter nati v es?
The  tr eatm ent  of   choi ce  f or   these  tum or s,   w hi ch  ar e  usual l y
m acr oadenom as  (> 1  cm ),   i s  tr anssphenoi dal   r esecti on  of   the  tum or ,
al though  sur gi cal   cur e  i s  of ten  di f f i cul t.   If   postoper ati v e  hor m onal   testi ng
r ev eal s  conti nued  abnor m al   GH  pr oducti on,   r adi ati on  ther apy   i s  of ten
necessar y .   Medi cal   ther apy   w i th  br om ocr i pti ne  (i f   the  tum or   costai ns  f or
pr ol acti n)  m ay   al so  r educe  the  GH  l ev el s  and  tum or   si ze,   w hi l e  the
r esul ts  of   i r r adi ati on  ar e  aw ai ted.   Som atostati n  anal ogs  can  i nhi bi t  GH
pr oducti on.   They   ar e  now   av ai l abl e  i n  shor t­acti ng  and  l ong­acti ng  f or m s.
Long­acti ng  octr eoti de  nor m al i zes  IGF­I  l ev el s  i n  41%  to  75%  and  r educes
tum or   si ze  i n  30%  of   subjects  i n  publ i shed  tr i al s.

Mor e  r ecentl y   a  GH  r eceptor   antagoni st,   pegv i som ant,   has  pr ov ed
ef f ecti v e  f or   r educti on  of   IGF­I  l ev el s  i n  acr om egal y   and  has  r ecei v ed
U . S.   Food  and  Dr ug  Adm i ni str ati on  (FDA)  appr ov al .   Studi es  dem onstr ate
that  pegv i som ant  nor m al i zes  IGF­I  l ev el s  i n  m or e  than  95%  of   pati ents
w ho  hav e
P. 107
f ai l ed  other   ther api es.   Ini ti al   concer ns  that  pi tui tar y   tum or   gr ow th  w oul d
pr oceed  uncheck ed  w i th  thi s  tr eatm ent  hav e  not  been  bor ne  out  by
ex per i ence  to  date.

Case
A  40­y ear ­ol d  m an  i s  seen  because  of   headaches,   m uscl e  aches,   and  chr oni c
l ow   back   and  joi nt  pai n.   As  he  enter s  the  of f i ce,   y ou  noti ce  hi s  coar se  f aci al
f eatur es,   f r ontal   bossi ng,   and  l ar ge  jaw .   When  y ou  shak e  hi s  hand,   y ou  f i nd  he
has  l ar ge,   doughy ,   sw eaty   pal m s  and,   w hen  he  sm i l es,   y ou  note  hi s  teeth  ar e
w i del y   spaced.
He  has  not  seen  a  phy si ci an  i n  10  y ear s  and  i s  tak i ng  no  m edi cati ons.   Hi s  back
and  joi nt  pai n  hav e  been  w or seni ng  f or   6  y ear s,   but  hi s  headaches  star ted  6
m onths  ago.
Hi s  phy si cal   ex am i nati on  f i ndi ngs  ar e  si gni f i cant  f or   a  BP  of   150/100  m m   Hg,
pul se  of   60  per   m i nute,   and  r espi r ator y   r ate  of   12  per   m i nute.
He  r etur ns  i n  2  w eek s  w i th  ol d  photogr aphs  that  conf i r m   a  change  i n  hi s
phy si cal   appear ance  ov er   ti m e,   and  the  l abor ator y   test  r esul ts  conf i r m   y our
cl i ni cal   i m pr essi on.

1.   What  i s  y our   i ni ti al   di agnosi s  i n  thi s  pati ent?


2.   Besi des  the  back   and  joi nt  pai n  and  the  headaches,   w hat  other   sy m ptom s
w oul d  y ou  l ook   f or   to  conf i r m   or   r ef ute  y our   di agnosi s?
3.   Besi des  the  phy si cal   f eatur es  y ou  obser v e  i ni ti al l y ,   w hat  other
abnor m al i ti es  w oul d  y ou  l ook   f or   on  phy si cal   ex am i nati on?
4.   What  l abor ator y   tests  shoul d  be  per f or m ed  i ni ti al l y ?
5.   What  addi ti onal   testi ng  shoul d  be  per f or m ed  once  the  i ni ti al   l abor ator y
r esul ts  ar e  k now n?
6.   What  i s  the  pr ef er r ed  tr eatm ent  i n  thi s  pati ent?
Case Discussion
1.   What  i s  y our   i ni ti al   di agnosi s  i n  thi s  pati ent?

Acr om egal y   shoul d  be  y our   i ni ti al   di agnosi s.

2.   Besi des  the  back   and  joi nt  pai n  and  the  headaches,   w hat  other   sy m ptom s
w oul d  y ou  l ook   f or   to  conf i r m   or   r ef ute  y our   di agnosi s?

Other   sy m ptom s  to  l ook   f or   i n  thi s  pati ent  i ncl ude  a  change  i n  gl ov e,
r i ng,   and  shoe  si zes,   spaces  betw een  the  teeth  and  an  under bi te,
decr eased  l i bi do  and  i m potence,   sw eati ng,   new   snor i ng,   pol y ur i a,
pol y di psi a,   and  a  change  i n  v i si on.

3.   Besi des  the  phy si cal   f eatur es  y ou  obser v e  i ni ti al l y ,   w hat  other
abnor m al i ti es  w oul d  y ou  l ook   f or   on  phy si cal   ex am i nati on?

Other   phy si cal   f eatur es  to  l ook   f or   i n  thi s  pati ent  i ncl ude  thi ck   coar se
sk i n,   sk i n  tags,   enl ar ged  ex tr em i ti es  and  or gans,   entr apm ent
neur opathi es,   v i sual   f i el d  abnor m al i ti es,   and  decr eased  body   hai r   and
testi cul ar   si ze.   Ol d  pi ctur es  w oul d  conf i r m   the  cl i ni cal   suspi ci on.

4.   What  l abor ator y   tests  shoul d  be  per f or m ed  i ni ti al l y ?

Ini ti al   l abor ator y   tests  i n  thi s  pati ent  w oul d  consi st  of   the  m easur em ent
of   IGF­1  (som atom edi n  C)  and  f asti ng  GH  l ev el s.   If   the  l ev el s  ar e
el ev ated  i t  w oul d  suggest  the  di agnosi s  of   acr om egal y ,   w hi ch  w oul d  be
conf i r m ed  i f   the  GH  l ev el s  di d  not
P. 108
suppr ess  to  l ess  than  2  ng/m L  i n  r esponse  to  a  gl ucose  l oad  (som e
studi es  suggest  1  ng/m L,   w hi ch  m ay   r ef l ect  the  i ncr eased  sensi ti v i ty   of
the  assay ).

5.   What  addi ti onal   testi ng  shoul d  be  per f or m ed  once  the  i ni ti al   l abor ator y
r esul ts  ar e  k now n?

If   the  i ni ti al   l abor ator y   r esul ts  i ndi cate  acr om egal y ,   a  f asti ng  bl ood  sugar
test  shoul d  be  per f or m ed  to  r ul e  out  di abetes.   Pi tui tar y   tests  shoul d
i ncl ude  m easur em ent  of   the  pr ol acti n,   FSH,   LH,   testoster one,   and  α ­
subuni t  l ev el s.   An  MRI  scan  can  show   the  ex tent  of   the  tum or ,   and  f or m al
v i sual   f i el d  testi ng  shoul d  be  per f or m ed.

6.   What  i s  the  pr ef er r ed  tr eatm ent  i n  thi s  pati ent?

The  pr ef er r ed  i ni ti al   tr eatm ent  i s  sur gi cal   r em ov al   of   the  tum or .


Br om ocr i pti ne  or   a  som atostati n  (octr eoti de)  anal og  m ay   be  usef ul   as
m edi cal   adjuncts.   Pegv i som ant  m ay   be  consi der ed  f or   r esi dual   tum or s
that  ar e  r ef r actor y   to  other   m edi cal   m anagem ent.   Radi ati on  ther apy   m ay
be  i ndi cated  f or   the  destr ucti on  of   r esi dual   tum or   i f   r eoper ati on  or
sur gi cal   cur e  i s  not  f easi bl e.   Postoper ati v e  hor m onal   testi ng  i s  i ndi cated
to  r eassess  pi tui tar y   f uncti on.   Echocar di ogr aphy   and  col onoscopy   shoul d
be  per f or m ed  to  ev al uate  f or   car di om egal y   and  col on  pol y ps.
Prolactin­Secreting Pituitary Tumors
1.   What  sy m ptom s  and  si gns  ar e  associ ated  w i th  an  el ev ated  pr ol acti n  l ev el
i n  w om en  and  i n  m en?

2.   What  i s  the  under l y i ng  pathophy si ol ogi c  pr ocess  r esponsi bl e  f or   the


ef f ects  of   el ev ated  pr ol acti n  l ev el s?

3.   What  ar e  the  causes  of   an  el ev ated  pr ol acti n  l ev el   other   than  a  pi tui tar y
tum or ?

4.   What  testi ng  i s  necessar y   to  conf i r m   or   r ef ute  a  di agnosi s  of   a


pr ol acti nom a?

5.   What  ar e  the  tr eatm ent  opti ons  f or   a  pr ol acti nom a?

Discussion
1.   What  sy m ptom s  and  si gns  ar e  associ ated  w i th  an  el ev ated  pr ol acti n  l ev el
i n  w om en  and  i n  m en?

In  w om en,   an  el ev ated  pr ol acti n  l ev el   i s  associ ated  w i th  di stur bance  of
the  m enstr ual   cy cl e—r angi ng  f r om   the  occur r ence  of   shor t  cy cl es  w i th
an  i nadequate  l uteal   phase,   ol i goov ul ati on,   and  i nf er ti l i ty ,   to  am enor r hea.
Gal actor r hea,   hi r suti sm ,   m ood  di stur bances,   and  headaches  ar e  al so
f r equent  com pl ai nts.   In  m en,   sy m ptom s  i ncl ude  decr eased  l i bi do,
i m potence,   and  i nf er ti l i ty .   Gal actor r hea  i s  a  r ar e  f i ndi ng.   Vi sual   f i el d
def ects  ar e  seen  i n  the  setti ng  of   l ar ge  tum or s.   Osteopeni a  and  f r actur es
can  occur   i n  both  sex es  and  ar e  due  to  the  secondar y   hy pogonadi sm .

2.   What  i s  the  under l y i ng  pathophy si ol ogi c  pr ocess  r esponsi bl e  f or   the


ef f ects  of   el ev ated  pr ol acti n  l ev el s?

Pr ol acti n  i s  under   toni c  i nhi bi tor y   contr ol   f r om   dopam i ne  i n  the


hy pothal am us.   Stal k   com pr essi on,   w hi ch  causes  dopam i ne  tone  to  be
i nhi bi ted,   or   pr ol acti n  secr eti on  f r om   a  tum or   i nhi bi ts  the  hy pothal am i câ
€“pi tui tar y –gonadal
P. 109
ax i s  at  al l   thr ee  l ev el s.   The  m ajor   ef f ect,   how ev er ,   i s  ter m i nati on  of   the
gonadotr opi n­r el easi ng  hor m one–i nduced  pul sati l e  r el ease  of   the
pi tui tar y   gonadotr opi ns,   LH  and  FSH.   Thi s  di sor der ed  gonadotr opi n
secr eti on  then  r esul ts  i n  i nadequate  gam etogenesi s  and  ster oi dogenesi s,
and  hence  hy pogonadi sm   w i th  or   w i thout  i nf er ti l i ty .   For   gal actor r hea  to
occur ,   ther e  m ust  be  estr ogen  pr i m i ng  of   the  br east  i n  addi ti on  to  an
el ev ated  pr ol acti n  l ev el ,   w hi ch  i s  w hy   m i l k   pr oducti on  does  not  dev el op  i n
m ost  m en  unl ess  thei r   pr ol acti n  l ev el   i s  chr oni cal l y   v er y   el ev ated  w i th
suppr essi on  of   testoster one  r el ease,   el ev ati on  of   the  estr adi ol   l ev el ,   and
gy necom asti a.

3.   What  ar e  the  causes  of   an  el ev ated  pr ol acti n  l ev el   other   than  a  pi tui tar y
tum or ?
El ev ated  pr ol acti n  l ev el s  m ay   be  due  to  phy si ol ogi c  causes  such  as
pr egnancy ,   str ess,   sl eep,   ex er ci se,   or   f r equent  br east  sti m ul ati on.
Sy stem i c  di sor der s  associ ated  w i th  el ev ated  pr ol acti n  l ev el s  i ncl ude
hy pothy r oi di sm ,   hy poadr enal i sm ,   chr oni c  r enal   f ai l ur e  (el ev ated
pr oducti on  and  decr eased  cl ear ance),   and  l i v er   f ai l ur e.   Dr ugs  that  el ev ate
the  pr ol acti n  l ev el   i ncl ude  phenothi azi des,   tr i cy cl i c  anti depr essants,
opi ates,   m etocl opr am i de,   ci m eti di ne,   m ethy l dopa,   r eser pi ne,   and
am phetam i nes,   al l   of   w hi ch  i nter f er e  w i th  dopam i ne  i nhi bi tor y   tone.

4.   What  testi ng  i s  necessar y   to  conf i r m   or   r ef ute  a  di agnosi s  of   a


pr ol acti nom a?

A  pr ol acti n  l ev el   of   m or e  than  100  ng/m L  suggests  the  pr esence  of   a


tum or ,   al though  tum or s  or   other   causes  can  be  associ ated  w i th  l ow er
el ev ati ons.   N o  sti m ul ati on  or   suppr essi on  test  i s  needed.   An  MRI  of   the
pi tui tar y   i s  necessar y   to  detect  a  m i cr oadenom a  and  ex cl ude  a  l ar ge
pi tui tar y   or   hy pothal am i c  m ass  that  i s  causi ng  stal k   com pr essi on.

5.   What  ar e  the  tr eatm ent  opti ons  f or   a  pr ol acti nom a?

The  tr eatm ent  of   choi ce  f or   pr ol acti nom as  i s  the  dopam i ne  agoni st,
br om ocr i pti ne.   It  ef f ecti v el y   l ow er s  pr ol acti n  l ev el s  and  r educes  tum or
si ze.   Other   dopam i ne  agoni sts  com m onl y   used  i ncl ude  caber gol i ne  and
per gol i de.   Sur gi cal   r em ov al   i s  r eser v ed  f or   noncom pl i ant  or
br om ocr i pti ne­i ntol er ant  pati ents  because  of   the  hi gh  r ecur r ence  r ate  of
20%  to  50%  at  5  y ear s.   Major   si de  ef f ects  of   br om ocr i pti ne  ther apy
i ncl ude  or thostati c  di zzi ness,   dr y   m outh,   nausea,   and  v om i ti ng,   al though
these  m ay   be  m i ni m i zed  by   sl ow   ti tr ati on  of   the  dr ug  al ong  w i th  f ood
i ntak e  at  ni ght.   Li f el ong  ther apy   i s  pr obabl y   necessar y .   Lack   of   tr eatm ent
l eads  to  pr ol onged  gonadal   ster oi d  def i ci ency   and  the  r i sk   of   osteopeni a
and  f r actur e.   The  pr em atur e  CV  r i sk   has  not  been  assessed.

Case
A  28­y ear ­ol d  w om an  i s  seen  because  of   i r r egul ar   per i ods  and  i nf er ti l i ty .   Her
m enar che  occur r ed  at  12  y ear s  of   age  and  she  had  r egul ar   per i ods  w i th
m ol i m i nal   sy m ptom s  (br east  tender ness,   bl oati ng,   and  cr am pi ng)  unti l
appr ox i m atel y   2  y ear s  ago.   Af ter   that,   her   per i ods  hav e  becom e  l i ghter   and
i r r egul ar   w i thout  m ol i m i nal   sy m ptom s.   She  has  decr eased  l i bi do,   occasi onal
headaches,   and  i s  m oody   and  i r r i tabl e.   She  has  noted  a  m i l k y   di schar ge  f r om
both  ni ppl es.   She  took   bi r th  contr ol   pi l l s  f or   2  y ear s,   5  y ear s  ago.
Her   ex am i nati on  i s  si gni f i cant  f or   the  f ol l ow i ng  f i ndi ngs:  nor m al   v i sual   f i el ds,
gal actor r hea,   and  a  decr eased  estr ogen  ef f ect  on  the  v agi nal   m ucosa.
P. 110

1.   What  i s  the  m ost  l i k el y   di agnosi s  i n  thi s  pati ent?


2.   What  other   hi stor i cal   f acts  ar e  i m por tant  to  el i ci t  i n  an  ef f or t  to
deter m i ne  the  cause  of   her   sy m ptom s?
3.   What  l abor ator y   tests  or   studi es  w oul d  y ou  hav e  done?
4.   What  i s  the  tr eatm ent  of   choi ce  i n  thi s  pati ent?
Case Discussion
1.   What  i s  the  m ost  l i k el y   di agnosi s  i n  thi s  pati ent?

The  m ost  l i k el y   di agnosi s  i n  thi s  pati ent  i s  hy per pr ol acti nem i a.

2.   What  other   hi stor i cal   f acts  ar e  i m por tant  to  el i ci t  i n  an  ef f or t  to


deter m i ne  the  cause  of   her   sy m ptom s?

It  i s  i m por tant  to  f i nd  out  w hether   she  m i ght  be  pr egnant  and  w hether
she  tak es  dr ugs  that  w oul d  i nhi bi t  dopam i ne  tone.   In  addi ti on,   a  hi stor y
of   hy pothy r oi di sm ,   hy poadr enal i sm ,   ex cessi v e  br east  sti m ul ati on,   and
r enal   or   l i v er   di sease  shoul d  be  sought.

3.   What  l abor ator y   tests  or   studi es  w oul d  y ou  hav e  done?

A  ser um   pr ol acti n  l ev el   shoul d  be  m easur ed  to  deter m i ne  the  ex tent  of
the  el ev ati on.   In  addi ti on,   l i v er   f uncti on  studi es  and  deter m i nati on  of   the
BU N   and  cr eati ni ne  l ev el s  shoul d  be  done  to  r ul e  out  l i v er   or   k i dney
di sease.   The  hum an  chor i oni c  gonadotr opi n  l ev el   shoul d  be  m easur ed  to
r ul e  out  pr egnancy ,   as  w el l   as  the  TSH  and  cor ti sol   l ev el s,   i f   ther e  ar e
sy m ptom s  or   si gns  of   hy pothy r oi di sm   or   hy poadr enal i sm .   An  MRI  scan
shoul d  be  obtai ned  to  di sti ngui sh  betw een  a  m i cr oadenom a  and  a
m acr oadenom a.

4.   What  i s  the  tr eatm ent  of   choi ce  i n  thi s  pati ent?

The  tr eatm ent  of   choi ce  i s  the  dopam i ne  agoni st  br om ocr i pti ne.
Tr eatm ent  i s  begun  at  ni ght  w i th  the  i ntak e  of   f ood  to  decr ease  the  si de
ef f ects  of   postur al   hy potensi on,   nausea,   and  dr y   m outh.   The  goal   i s  to
nor m al i ze  the  pr ol acti n  l ev el s.   Other   l ong­acti ng  dopam i ne  agoni sts  such
as  caber gol i ne,   per gol i de,   and  di hy dr oer gotox i ne  ar e  av ai l abl e  i f   pati ents
f ai l   to  tol er ate  br om ocr i pti ne.

Suggested Readings
Ar af ah  BM.   Medi cal   m anagem ent  of   hy popi tui tar i sm   i n  pati ents  w i th
pi tui tar y   adenom as.   Pi tui tar y   2002;5:109–117.

Ar on  DC,   How l ett  TA.   Pi tui tar y   i nci dental om as.   Endocr i nol   Metab  Cl i n  N or th
Am   2000;29(1):205–221.

Mel m ed  S,   Casanuev a  FF,   Cav agni ni   F,   et  al .   Gui del i nes  f or   acr om egal y
m anagem ent.   J  Cl i n  Endocr i nol   Metab  2002;87(9):4054–4058.

Mol i tch  ME.   Medi cal   m anagem ent  of   pr ol acti n­secr eti ng  pi tui tar y   adenom as.
Pi tui tar y   2002;5:55–65.
Pi ck ett  CA.   Di agnosi s  and  m anagem ent  of   pi tui tar y   tum or s:  r ecent
adv ances.   Pr i m   Car e  Of f i ce  Pr act  2003;30:765–789.

Shi m on  I,   Mel m ed  S.   Managem ent  of   pi tui tar y   tum or s.   Ann  Inter n  Med
1998;129:472.

P. 111

Sw ear i ngen  B,   Bar k er   FG  II,   Katznel son  L,   et  al .   Long­ter m   m or tal i ty   af ter
tr anssphenoi dal   sur ger y   and  adjuncti v e  ther apy   f or   acr om egal y .   J  Cl i n
Endocr i nol   Metab  1998;83:3419.

Hypercalcemia
1.   What  condi ti ons  can  cause  hy per cal cem i a?

2.   What  tw o  m edi cal   condi ti ons  account  f or   m ost  cases  of   hy per cal cem i a?

3.   In  the  hy per cal cem i c  pati ent,   w hat  ar e  the  l abor ator y   f i ndi ngs  seen  i n  the
setti ng  of   hy per par athy r oi di sm ?

4.   What  i s  the  tr eatm ent  f or   hy per cal cem i a?

5.   What  ar e  the  i ndi cati ons  f or   par athy r oi dectom y ?

Discussion
1.   What  condi ti ons  can  cause  hy per cal cem i a?

The  causes  of   hy per cal cem i a  that  need  to  be  consi der ed  i n  any   pati ent
w ho  ex hi bi ts  a  bona  f i de  el ev ati on  i n  the  ser um   cal ci um   l ev el   as
docum ented  i n  at  l east  thr ee  r epeat  deter m i nati ons  ar e  l i sted  i n  Tabl e  3­
1.

2.   What  tw o  m edi cal   condi ti ons  account  f or   m ost  cases  of   hy per cal cem i a?

Of   the  m any   causes  of   hy per cal cem i a  l i sted  i n  Tabl e  3­1,   the  m ost
com m on  ar e  m al i gnancy   (45%)  and  hy per par athy r oi di sm   (45%).   The
l engthy   di f f er enti al   di agnosi s  (see  Tabl e  3­1)  i ncl udes  the  other   10%  of
the  causes.   Hence,   f r om   a  pr acti cal   standpoi nt,   hy per cal cem i c  di sor der s
can  be  br ok en  dow n  i nto  tw o  categor i es:  par athy r oi d  hor m one  (PTH)­
m edi ated  hy per cal cem i a  and  non–PTH­m edi ated  hy per cal cem i a.

3.   In  the  hy per cal cem i c  pati ent,   w hat  ar e  the  l abor ator y   f i ndi ngs  seen  i n  the
setti ng  of   hy per par athy r oi di sm ?

For   the  sak e  of   si m pl i ci ty ,   the  m any   causes  of   hy per cal cem i a  can  be
separ ated  i nto  tw o  categor i es  accor di ng  to  the  PTH  l ev el   and  l abor ator y
f i ndi ngs  r esul t  f r om   the  pr esence  or   absence  of   the  acti on  of   PTH.   (Tabl es
3­2  and  3­3).
4.   What  i s  the  tr eatm ent  of   hy per cal cem i a?

A  hy per cal cem i c  em er gency   i s  di agnosed  w hen  the  cal ci um   l ev el   ex ceeds


14  m g/dL  or   the  pati ent  ex hi bi ts  sy m ptom s  of   hy per cal cem i a,   consi sti ng
of   pr of ound  w eak ness,   i m pai r ed  m ental   f uncti on,   nausea  and  v om i ti ng,
and  centr al   ner v ous  sy stem   depr essi on  l eadi ng  to  stupor ,   l ethar gy ,   or
com a.   U r gent  tr eatm ent  of   the  hy per cal cem i a  i s  m andator y   i n  these
si tuati ons  (Tabl e  3­4).

5.   What  ar e  the  i ndi cati ons  f or   par athy r oi dectom y ?

The  f ol l ow i ng  i ndi cati ons  f or   par athy r oi dectom y   i n  hy per par athy r oi d
pati ents  hav e  been  pr oposed  by   a  N ati onal   Insti tutes  of   Heal th  (N IH)
consensus  conf er ence:

1.   Pati ent  y ounger   than  50  y ear s

2.   El ev ated  ser um   cal ci um   to  a  concentr ati on  of   1. 0  to  1. 6  m g/dL  abov e
nor m al   l abor ator y   v al ues

P. 112
3.   Hi stor y   of   a  l i f e­thr eateni ng  hy per cal cem i c  epi sode

4.   Reduced  cr eati ni ne  cl ear ance

5.   Pr esence  of   k i dney   stones

6.   U r i ne  cal ci um   ex cr eti on  of   gr eater   than  400  m g  per   24  hour s

7.   Bone  m ass  r educed  by   m or e  than  2  standar d  dev i ati ons  bel ow


nor m al

Table 3­1 Causes of Hypercalcemia

Pr i m ar y   hy per par athy r oi di sm


Spor adi c  (90%­95%  of   al l   cases  of   hy per par athy r oi di sm )
Fam i l i al   sy ndr om es  (MEN   1  and  MEN   2)
MEN   1  (tum or s  of   pi tui tar y ,   pancr eas,   and  par athy r oi d)
MEN   2A  (m edul l ar y   thy r oi d  car ci nom a,
hy per par athy r oi di sm ,   pheochr om ocy tom a)
MEN   2B  (m edul l ar y   thy r oi d  car ci nom a,
pheochr om ocy tom a,   m ucosal   neur om as,   m ar f anoi d
habi tus,   and  par athy r oi d  hy per pl asi a)
N eopl asti c  di seases
Local   osteol y si s  (br east  and  l ung  car ci nom a  m etastati c  to
bone,   and  m y el om a)
Hum or al   hy per cal cem i a  of   m al i gnancy
Endocr i ne  di sor der s
Hy per thy r oi di sm
Adr enal   i nsuf f i ci ency
Beni gn  f am i l i al   hy pocal ci ur i c  hy per cal cem i a
Medi cati ons
Thi azi de  di ur eti cs
Vi tam i n  D  and  r ar el y   v i tam i n  A  i ntox i cati on
Mi l k ­al k al i   sy ndr om e
Li thi um
Gr anul om atous  di seases
Sar coi dosi s
Ber y l l i osi s,   tuber cul osi s,   cocci di oi dom y cosi s,   hi stopl asm osi s
Mi scel l aneous
Im m obi l i zati on  (associ ated  w i th  hi gh  bone­tur nov er   states
such  as  i n  chi l dr en  or   i n  pati ents  w i th  Paget's  di seases)
Recov er y   phase  of   acute  and  r enal   f ai l ur e  (r ar e)
Idi opathi c  hy per cal cem i a  of   i nf ancy   (r ar e)
Dehy dr ati on  (due  to  hem oconcentr ati on)
MEN ,   m ul ti pl e  endocr i ne  neopl asi a.

Case
A  47­y ear ­ol d  w hi te  m al e  com puter   consul tant  i s  seen  i n  the  w al k ­i n  cl i ni c
com pl ai ni ng  of   sev er e  r i ght  hi p  pai n  and  di f f i cul ty   i n  w al k i ng.   He  has  been
tak i ng  i bupr of en  f or   pai n  r el i ef .   The  pai n  i n  hi s  r i ght  hi p  and  r i ght  pr ox i m al
l ow er   ex tr em i ty   has  been  pr esent  f or   appr ox i m atel y   4  m onths,   and  has
pr ogr essed  to  becom e  a  shar p,   l ocal i zed  r i ght  hi p  joi nt  pai n  dur i ng  the  past
m onth.   The  pati ent  has  noted  a  10­l b  (4. 5­k g)  w ei ght  l oss  ov er   the  pr ecedi ng
2  m onths,   but  ascr i bes  thi s  to  a  sel f ­enf or ced  di et.   He  has  noctur i a  w i th  tw o
to
P. 113
f i v e  m i ctur i ti ons  per   ni ght,   and  com pl ai ns  of   ex cessi v e  thi r st.   In  addi ti on,   he
i s  aw ar e  of   a  decr ease  i n  hi s  abi l i ty   to  concentr ate  ov er   the  pr ecedi ng  sev er al
m onths.

Table 3­2 Parathyroid Hormone Action on Kidney

Ac tion Ma nife s ta tion

Incr eases  tubul ar   r esor pti on  of   cal ci um Hy per cal cem i a,   m i l d
hy per cal ci ur i a

Inhi bi ts  pr ox i m al   tubul e  bi car bonate Ty pe  II  r enal   tubul ar


r esor pti on aci dosi s,
hy per chl or em i c
m etabol i c  aci dosi s
Incr eases  phosphate  cl ear ance  (decr eases Phosphatur i a,
tubul ar   r esor pti on  of   PO 4 ) decr eased  [PO 4 ]
(i ncr eased  Cl /PO 4
> 33)

Sti m ul ates  r enal   cAMP Incr eased


nephr ogenous  cAMP

Incr eases  1α ­hy dr ox y l ase  acti v i ty   f or   1, 25­ Incr eased  l ev el s  of


di hy dr ox y v i tam i n  D  sy nthesi s 1, 25­
di hy dr ox y v i tam i n  D

Am i noaci dur i a Am i noaci dur i a

Acti v ates  r enal   tubul ar   enzy m es  (al k al i ne Incr eased  r enal
phosphatase,   gl ucose­6­phosphate  dehy dr o gl ucose  pr oducti on
genase),   pr om otes  r enal   gl uconeogenesi s

PO 4 ,   phosphate  r adi cal ;  [PO 4 ],   PO 4   concentr ati on;  cAMP,   cy cl i c


adenosi ne  m onophosphate.

Hi s  past  m edi cal   hi stor y   i s  si gni f i cant  f or   nephr ol i thi asi s  r equi r i ng
hospi tal i zati on  10  y ear s  ear l i er ,   and  an  upper   gastr oi ntesti nal   hem or r hage  5
y ear s  ago,   secondar y   to  pepti c  ul cer   di sease.   Hi s  f am i l y   hi stor y   i s
noncontr i butor y .
Radi ogr aphi c  studi es  of   the  pati ent's  pel v i s  and  r i ght  hi p  show   a  l y ti c  l esi on  i n
the  r i ght  sacr um   and  f em or al   acetabul um .   The  pati ent  i s  adm i tted  f or   f ur ther
ev al uati on.
Phy si cal   ex am i nati on  r ev eal s  a  pl easant,   m i ddl e­aged  m an  w ho  i s  ex per i enci ng
consi der abl e  pai n  i n  hi s  r i ght  hi p.   Hi s  BP  i s  160/98  m m   Hg;  pul se,   96  beats
per   m i nute;  r espi r ator y   r ate,   20  per   m i nute;  and  tem per atur e,   97. 8°F  (36. 8Â
°C).   N o  si gni f i cant  sk i n
P. 114
P. 115
l esi ons  ar e  f ound.   On  ex am i nati on  of   the  or al   phar y nx ,   a  w hi te  m ass  on  the
har d  pal ate  i s  noted.   The  pati ent  has  no  cer v i cal ,   ax i l l ar y ,   or   i ngui nal
adenopathy .   Thy r oi d  ex am i nati on  r ev eal s  a  f ul l ness  i n  the  l ef t  l ow er   l obe.
Range­of ­m oti on  ex er ci ses  of   the  r i ght  l ow er   ex tr em i ty   el i ci t  sev er e  r i ght  hi p
tender ness.   A  neur ol ogi c  ex am i nati on  r ev eal s  di m i ni shed  str ength  i n  the  r i ght
hi p  f l ex or s  and  ex tender s  w i th  nor m al   deep  tendon  r ef l ex es  thr oughout.   The
pati ent's  m ental   status  i s  appr opr i ate.
Table 3­3 Causes of Hypercalcemia

Va ria ble s P TH­Me dia te d Non­P TH­Me dia te d

Phosphate Low   (< 2. 2  m g/dL) Low ,   nor m al ,   or   hi gh

Chl or i de Hi gh  (> 104 U sual l y   < 100


m Eq/dL) m Eq/dL

Metabol i c  aci dosi s Mi l d N ot  pr esent

Cl /PO 4 > 33 < 33

PTH Hi gh Low

Hy per par athy r oi di sm a N eopl asi a  w i th  or   w i thout  hum or al


hy per cal cem i a  of   m al i gnancy

Other   non–PTH­m edi ated  causes  (see


 
Tabl e  3­1)

a Rem em ber   to  ex cl ude  beni gn  f am i l i al   hy pocal ci ur i c  hy per cal cem i a.

PTH,   par athy r oi d  hor m one;  PO 4 ,   phosphate  r adi cal .

Table 3­4 Therapy for Hypercalcemia

U r gent  ther apy

1.   Sal i ne
a.   U sual l y   saf e  w i th  200–300  m L/hr   but  m ay   need  > 10  L/d
w i th  car ef ul   m oni tor i ng.   U se  N S:  D 5 W  al ter nate  i n  4  :  1

r ati o  w i th  20  m Eq  KCl /bottl e  (can  f ol l ow   ur i nar y   K + ,   N a + ,


and  v ol um e  to  docum ent  l osses)
b.   May   need  15  m g  m agnesi um /hr
2.   Sal i ne  pl us  f ur osem i de
a.   Wi th  aggr essi v e  m anagem ent,   80­100  m g  f ur osem i de  IV
q1–2  h  and  r epl ace  ur i nar y   el ectr ol y tes  (Suk i   WN ,   Yi um
JJ,   Von  Mi nden  M,   et  al .   Acute  tr eatm ent  of   hy per cal cem i a
w i th  f ur osem i de.   N   Engl   J  Med  1970;283:836).
b.   Less  ur gent  m anagem ent—40  m g  f ur osem i de  q4­6  h
c.   Bef or e  usi ng  f ur osem i de,   be  sur e  pati ent  i s  adequatel y
hy dr ated
3.   Cal ci toni n:  4­8  IU /k g  subcutaneousl y   q6­12  h
4.   Cal ci toni n  pl us  gl ucocor ti coi ds
a.   4­8  MRC  uni ts/k g  q6­12  h
b.   Pr edni sone:  40­60  m g/d
5.   IV  bi sphosphonates
a.   IV  eti dr onate  (Di dr onel ):  7. 5  m g/k g,   w i th  3  L  of   sal i ne
gi v en  ov er   24  hr   and  r epeat  dai l y   f or   3  d
b.   IV  pam i dr onate  (Ar edi a):  60­90  m g  as  si ngl e  24­hr   i nf usi on
w i th  adequate  sal i ne  hy dr ati on;  al l ow   a  m i ni m um   of   7  d  to
el apse  bef or e  r etr eatm ent
c.   IV  zol edr onate  (Zom eta):  4  m g  IV  ov er   15  m i nutes.
d.   IV  i bandr onate  (Boni v a):  2­4  m g  IV
6.   Gal l i um   ni tr ate  (av oi d  use  i f   cr eati ni ne  > 2. 5  m g/dL):  100­200
m g/m 2   of   body   sur f ace  ar ea  i n  1, 000  m L  N S  ov er   24  hr   dai l y   f or
5  d
7.   IV  phosphate
a.   Gi v en  as  1, 000  m g  of   el em ental   phosphate  (0. 16  m g/k g)
ov er   8­12  hr   dur i ng  each  24­hr   per i od  (cauti on:  can  cause
hy potensi on)
b.   Av oi d  use  i f   ser um   phosphate  el ev ated
8.   Di al y si s
9.   IV  EDTA
a.   Av oi d  use  because  of   f or m ati on  of   i nsol ubl e  cal ci um
com pounds  that  dam age  k i dney

Long­ter m   ther apy   (adjunct  ther apy   i n  addi ti on  to  tr eatm ent  of
pr i m ar y   cause)

1.   Mobi l i zati on
2.   Or al   phosphates
a.   1, 000­2, 000  m g  of   el em ental   phosphate  (K­Phos;  thr ee
tabl ets  thr i ce  dai l y )
b.   Av oi d  use  i f   el ev ated  ser um   phosphate
3.   Mi thr am y ci n  (m ay   al so  be  used  i n  sem i acute  si tuati ons):  25
µg/k g  i n  50  m L  D5W  gi v en  as  i nf usi on  ov er   3  hr
4.   Gl ucocor ti coi ds—pr edni sone:  50­60  m g/d
5.   Di phosphonates—or al   eti dr onate:  5­20  m g/k g/d

N S,   nor m al   sal i ne;  D5W,   5%  dex tr ose  i n  w ater ;  MRC,   Medi cal
Resear ch  Counci l ;  IV,   i ntr av enous;  EDTA,   ethy l enedi am i netetr a
acetate.

Ini ti al   l abor ator y   data  r ev eal   the  f ol l ow i ng:  WBC,   5, 800;  Hgb,   13. 3  g/dL;  Hct,
39. 7%;  and  pl atel et  count,   274  ×  10 3 m m 3 .   The  f ol l ow i ng  el ectr ol y te  and
ser um   chem i str y   v al ues  ar e  r epor ted:  sodi um ,   138  m Eq/L;  potassi um ,   3. 9
m Eq/L;  chl or i de,   108  m Eq/L;  CO 2 ,   21. 5  m Eq/L;  BU N ,   18  m g/dL;  cr eati ni ne,   1. 0
m g/dL;  and  f asti ng  gl ucose,   94  m g/dL.   Other   si gni f i cant  l abor ator y   v al ues
i ncl ude  the  f ol l ow i ng:  cal ci um ,   11. 5  m g/dL;  phosphate,   2. 0  m g/dL;  total
pr otei n,   6. 8  g/dL;  al bum i n,   2. 8  g/dL;  and  m agnesi um ,   1. 7  m Eq/L.   U r i nal y si s
f i ndi ngs  ar e  nor m al .   The  er y thr ocy te  sedi m entati on  r ate  i s  9  m m   per   hour   and
the  al k al i ne  phosphatase  l ev el   i s  396  IU /L.
Chest  f i l m   f i ndi ngs  ar e  nor m al .   In  a  r ev i ew   of   the  pel v i s  and  hi p  r adi ogr aphi c
studi es,   l y ti c  changes  w i th  bony   destr ucti on  ar e  f ound  i n  both  hem i pel v es,   but
these  ar e  gr eater   on  the  r i ght.   Ri ght  f em or al   head  i nv ol v em ent  i s  al so  noted.
A  pel v i c  CT  scan  show s  the  ex i stence  of   m ul ti pl e  destr ucti v e  sof t  ti ssue
l esi ons  i n  the  bone  of   the  pel v i s;  the  l ar gest  of   the  l esi ons  m easur es  8  cm .   A
r adi onucl i de  bone  scan  r ev eal s  i ncr eased  uptak e  i n  the  pel v i c  l esi ons  and  i n
sev er al   r i bs,   as  w el l .   A  l ar ge­bor e  needl e  bi opsy   speci m en  f r om   the
gi ngi v opal atal   m ass  and  the  r i ght  i l i um   show s  the  appear ance  of   a  gi ant  cel l
tum or   m i x ed  w i th  f i br obl asts.
Speci al   endocr i ne  studi es  r ev eal   an  i oni zed  cal ci um   l ev el   of   2. 7  m m ol /L
(nor m al ,   1. 15  to  1. 35  m m ol /L).   The  24­hour   ur i ne  cal ci um   and  phosphate
ex cr eti ons  ar e  290  and  856  m g,   r especti v el y .

1.   Gi v en  the  pati ent's  hy poal bum i nem i a  of   2. 8  g/dL,   w hat  i s  the  cor r ected
cal ci um   l ev el ?
2.   What  i s  the  ex pl anati on  f or   the  pati ent's  pol y ur i a  and  pol y di psi a?
3.   Based  sol el y   on  the  pati ent's  adm i ssi on  el ectr ol y te  l ev el s,   w hat  i s  the
l i k el y   di agnosi s?
4.   What  i s  the  m ost  l i k el y   ex pl anati on  f or   the  m ul ti pl e  bone  l esi ons  i n  thi s
pati ent?
5.   What  i s  the  speci al   l abor ator y   test  that  needs  to  be  per f or m ed  i n  thi s
pati ent?
6.   What  i s  the  best  l ocal i zi ng  pr ocedur e  i n  pati ents  such  as  thi s  one?

Case Discussion
1.   Gi v en  the  pati ent's  hy poal bum i nem i a  of   2. 8  g/dL,   w hat  i s  the  cor r ected
cal ci um   l ev el ?

As  a  r ul e,   appr ox i m atel y   45%  of   the  m easur ed  ser um   cal ci um   i s  pr otei n
bound;  55%  i s  di f f usi bl e.   The  pr otei n­bound  f r acti on  i s  gr eater   f or
al bum i n  than  f or   gl obul i n.   For   a  ser um   cal ci um   l ev el   of   10  m g/dL,
appr ox i m atel y   0. 8  m g/dL  i s  bound  to  gl obul i n  and  3. 7  m g/dL  i s  bound  to
al bum i n.   In  the  setti ng  of   a  l ow   al bum i n  state,   appr ox i m atel y   1  g  of
al bum i n  bi nds  0. 8  m g  of   cal ci um .   For   ex am pl e,   thi s  pati ent  has
P. 116
a  ser um   cal ci um   l ev el   of   11. 5  m g/dL  and  a  ser um   al bum i n  l ev el   of   2. 8
g/dL.   The  cor r ected  cal ci um   l ev el   i s  cal cul ated  as  f ol l ow s:

2.   What  i s  the  ex pl anati on  f or   the  pati ent's  pol y ur i a  and  pol y di psi a?

Hy per cal cem i a  causes  a  v asopr essi n­r esi stant  nephr ogeni c  di abetes
i nsi pi dus.   Thi s  can  pr om ote  dehy dr ati on  i n  hy per cal cem i c  pati ents,
ther eby   aggr av ati ng  the  sy m ptom s  and  w or seni ng  the  hy per cal cem i a.

3.   Based  sol el y   on  the  pati ent's  adm i ssi on  el ectr ol y te  l ev el s,   w hat  i s  the
l i k el y   di agnosi s?

The  el ectr ol y te  l ev el s  i n  thi s  pati ent  str ongl y   suppor t  a  di agnosi s  of
pr i m ar y   hy per par athy r oi di sm .   Hy pophosphatem i a  i s  seen  i n  near l y   40%  to
60%  of   pati ents  w i th  hy per par athy r oi di sm ,   and  i ts  pr esence  depends  on
the  di etar y   phosphate  i ntak e.   In  addi ti on,   the  chl or i de  concentr ati on
gr eater   than  104  m m ol /L  and  the  ser um   bi car bonate  v al ue  i n  the  m i l dl y
aci doti c  r ange  suggest  hy per par athy r oi di sm .   A  chl or i de­to­phosphate  r ati o
of   gr eater   than  33  i s  seen  i n  the  setti ng  of   hy per par athy r oi di sm .   In  thi s
pati ent,   thi s  r ati o  i s  54,   w hi ch  i ndi cates  PTH­m edi ated  hy per cal cem i a.   An
el ev ated  1, 25­di hy dr ox y v i tam i n  D  l ev el   m ay   be  seen  i n  pati ents  w i th
pr i m ar y   hy per par athy r oi di sm ,   but,   i f   ther e  i s  m agnesi um   def i ci ency ,
these  l ev el s  m ay   be  nor m al   or   l ow .

4.   What  i s  the  m ost  l i k el y   ex pl anati on  f or   the  m ul ti pl e  bone  l esi ons  i n  thi s
pati ent?

The  tur nov er   state  of   bone  f or m ati on  and  r esor pti on  i s  hi gh  i n  pati ents
w i th  hy per par athy r oi di sm .   The  cl assi c  hi stol ogi c  pi ctur e  f ound  i n  bone
bi opsy   speci m ens  i s  an  i ncr eased  num ber   of   osteocl asts,   together   w i th
i ncr eased  tetr acy cl i ne  l abel i ng  and  i ncr eased  r ates  of   bone  f or m ati on.
The  m ar r ow   i n  these  pati ents  m ay   show   f ocal   ar eas  of   f i br osi s.   In
ex tr em el y   adv anced  cases  of   hy per par athy r oi di sm ,   osteocl astom as  or
gi ant  cel l   tum or s  of   bone  m ay   be  seen.   Thi s  pati ent  had  m ul ti pl e  such
tum or s.

5.   What  i s  the  speci al   l abor ator y   test  that  needs  to  be  per f or m ed  i n  thi s
pati ent?

The  speci al   l abor ator y   test  that  needs  to  be  done  i n  thi s  pati ent  i s
m easur em ent  of   hi s  PTH  l ev el ,   w hi ch  pr ov es  to  be  m ar k edl y   el ev ated  to  a
v al ue  of   811  pg/m L  (nor m al ,   10  to  65  pg/m L).

6.   What  i s  the  best  l ocal i zi ng  pr ocedur e  i n  pati ents  such  as  thi s  one?

Appr ox i m atel y   80%  to  90%  of   pati ents  w i th  pr i m ar y   hy per par athy r oi di sm
hav e  a  si ngl e  par athy r oi d  adenom a,   10%  to  15%  hav e  par athy r oi d
hy per pl asi a,   and  l ess  than  1%  hav e  par athy r oi d  car ci nom a.   The
pr eoper ati v e  l ocal i zi ng  pr ocedur es  such  as  a  Sestam i bi   scan  add  to  the
m anagem ent  of   thi s  di sease  as  they   per m i t  a  di r ected  m i ni m al l y   i nv asi v e
appr oach.   How ev er ,   the  m ost  r el i abl e  appr oach  to  l ocal i zati on  i s  an
ex per i enced  sur geon  w ho  can,   i n  al m ost  al l   cases,   r em ov e  the  adenom a
or   i denti f y   the  par athy r oi d  hy per pl asi a  and  r em ov e  thr ee  and  one­hal f
gl ands.   Thi s  pati ent  pr ov ed  to  hav e  a  l ar ge,   16­g  par athy r oi d  adenom a,
w hi ch  w as  i denti f i ed  easi l y   and  r em ov ed.

P. 117

Suggested Readings
Bi l ezi k i an  JB.   Managem ent  of   acute  hy per cal cem i a.   N   Engl   J  Med
1992;326:1196.

Bi l ezi k i an  JP,   Br andi   ML,   Rubi n  M,   et  al .   Pr i m ar y   hy per par athy r oi di sm :  new
concepts  i n  cl i ni cal ,   densi tom etr i c  and  bi ochem i cal   f eatur es.   J  Inter n  Med
2005;257(1):6–17.

Br oadus  AE,   Mangi n  M,   Ik eda  K,   et  al .   Hum or al   hy per cal cem i a  of   cancer :
i denti f i cati on  of   a  nov el   par athy r oi d  hor m one­l i k e  pepti de.   N   Engl   J  Med
1988;319:556.

Col ao  A,   Di   Sar no  A,   Landi   ML,   et  al .   Long­ter m   and  l ow ­dose  tr eatm ent
w i th  caber gol i ne  i nduces  m acr opr ol acti n  shr i nk age.   J  Cl i n  Endocr i nol   Metab
1997;82:3574.

Consensus  Dev el opm ent  Conf er ence.   Di agnosi s  and  m anagem ent  of
asy m ptom ati c  pr i m ar y   hy per par athy r oi di sm .   Ann  Inter n  Med  1991;114:593.

Dav i es  PH,   Stew ar t  SE,   Lancr anjan  L,   et  al .   Long­ter m   ther apy   w i th  l ong­
acti ng  octr eoti de  (Sandostati n­LAR)  f or   the  m anagem ent  of   acr om egal y .
Cl i n  Endocr i nol   1998;48:311.

De  Luca  F,   Bar on  J.   Mol ecul ar   bi ol ogy   and  cl i ni cal   i m por tance  of   the
Ca(2+ )­sensi ng  r eceptor .   Cur r   Opi n  Pedi atr   1998;10:435.

Hender son  JE,   Shusti k   C,   Kr em er   R,   et  al .   Ci r cul ati ng  concentr ati ons  of
par athy r oi d  hor m one­l i k e  pepti de  i n  m al i gnancy   and  i n
hy per par athy r oi di sm .   J  Bone  Mi ner   Res  1990;5:105.

Inzucchi   SE.   Managem ent  of   hy per cal cem i a.   Di agnosti c  w or k up,   ther apeuti c
opti ons  f or   hy per par athy r oi di sm   and  other   com m on  causes.   Postgr ad  Med
2004;115(5):27–36.

Inzucchi   SE.   U nder standi ng  hy per cal cem i a:  i ts  m etabol i c  basi s,   si gns  and
sy m ptom s.   Postgr ad  Med  2004;115(4):69–76.

Luf k i n  EG,   Kao  PC,   Heath  H.   Par athy r oi d  hor m one  r adi oi m m unoassay s  i n
the  di f f er enti al   di agnosi s  of   hy per cal cem i a  due  to  pr i m ar y
hy per par athy r oi di sm   or   m al i gnancy .   Ann  Inter n  Med  1987;160:559.

Mur ator i   M,   Ar osi o  M,   Gam bi no  G,   et  al .   U se  of   caber gol i ne  i n  the  l ong­
ter m   tr eatm ent  of   hy per pr ol acti nem i c  and  acr om egal i c  pati ents.   J
Endocr i nol   Inv est  1997;20:537.

Ral ston  SH,   Gal l acher   SJ,   Patel   U ,   et  al .   Cancer ­associ ated  hy per cal cem i a:
m or bi di ty   and  m or tal i ty .   Ann  Inter n  Med  1990;112:499.

Yeh  PJ,   Chen  JW.   Pi tui tar y   tum or s:  sur gi cal   and  m edi cal   m anagem ent.   Sur g
Oncol   1997;6:67.

Hypoglycemia
1.   What  consti tutes  m edi cal l y   si gni f i cant  hy pogl y cem i a?

2.   What  ar e  the  com m on  sy m ptom s  of   hy pogl y cem i a?

3.   What  i s  the  best  f i r st  step  i n  cl assi f y i ng  hy pogl y cem i a?

4.   What  ar e  the  causes  of   m edi cal l y   si gni f i cant  hy pogl y cem i a?

5.   In  peopl e  w i th  di abetes,   w hat  f actor s  ar e  associ ated  w i th  an  i ncr eased
r i sk   of   hy pogl y cem i a?

6.   What  i s  r eacti v e  hy pogl y cem i a  and  how   shoul d  i t  be  ev al uated?

Discussion
1.   What  consti tutes  m edi cal l y   si gni f i cant  hy pogl y cem i a?

Medi cal l y   si gni f i cant  hy pogl y cem i a  i s  di agnosed  on  the  basi s  of   onl y   thr ee
f i ndi ngs  (Whi ppl e's  tr i ad):  (a)  bl ood  gl ucose  l ev el   of   l ess  than  50  m g/dL;
(b)  the
P. 118
pr esence  of   sy m ptom s  consi stent  w i th  hy pogl y cem i a;  and  (c)  the
r esol uti on  of   sy m ptom s  af ter   the  i ngesti on  of   car bohy dr ates.   The  l ow er
l i m i t  of   nor m al   f or   gl ucose  i s  70  m g/dL,   but  thi s  i s  the  l ow er   l i m i t  f or   â
€œheal thy â€​   peopl e  af ter   a  12­hour   f ast.   Dur i ng  a  72­hour   f ast,   up  to
40%  of   “heal thy â€​   w om en  m ay   hav e  bl ood  gl ucose  v al ues  bel ow   45
m g/dL  and  som e  as  l ow   as  betw een  20  and  30  m g/dL.   These  l ow   v al ues
m ay   al so  be  seen  i n  appar entl y   heal thy   w om en  3  to  4  hour s  af ter   the
adm i ni str ati on  of   75  g  of   gl ucose  or al l y   (the  or al   gl ucose  tol er ance  test),
but  al m ost  none  hav e  sy m ptom s  of   hy pogl y cem i a  and,   ther ef or e,
m edi cal l y   si gni f i cant  hy pogl y cem i a.   Conv er sel y ,   m any   peopl e  w ho  ex hi bi t
sy m ptom s  consi stent  w i th  hy pogl y cem i a  3  to  4  hour s  af ter   eati ng,   w hi ch
r espond  to  the  i ngesti on  of   car bohy dr ate,   al so  do  not  hav e  tr ue
hy pogl y cem i a.   The  bl ood  gl ucose  l ev el s  i n  these  i ndi v i dual s  ar e  r ar el y
l ess  than  50  m g/dL  at  the  ti m e  they   ex per i ence  sy m ptom s.   These  peopl e
hav e  a  condi ti on  that  has  been  cal l ed  postpr andi al   sy ndr om e  or   f uncti onal
hy pogl y cem i a.

2.   What  ar e  the  com m on  sy m ptom s  of   hy pogl y cem i a?

The  sy m ptom s  of   hy pogl y cem i a  can  be  di v i ded  i nto  tw o  categor i es:
adr ener gi c  and  neur ogl y copeni c  (Tabl e  3­5).   A  substanti al   r educti on  i n  the
bl ood  gl ucose  l ev el   sti m ul ates  the  r el ease  of   cor ti sol ,   GH,   gl ucagon,   and
catechol am i nes.   The  attendant  r i se  i n  sy m patheti c  ner v ous  sy stem
acti v i ty   i s  ex per i enced  as  ner v ousness,   sw eati ng,   and  pal pi tati ons.
Because  the  br ai n  i s
P. 119
cr i ti cal l y   dependent  on  gl ucose  f or   nor m al   neur onal   f uncti oni ng,
i nadequate  del i v er y   of   gl ucose  to  the  br ai n  r api dl y   r esul ts  i n  al ter ati ons
i n  m entati on,   w hi ch  can  tak e  m any   f or m s.   The  si gns  and  sy m ptom s  of
neur ogl y copeni a  can  ev en  m i m i c  those  associ ated  w i th  str uctur al   br ai n
l esi ons  or   psy chi atr i c  condi ti ons.

Table 3­5 Symptoms of Hypoglycemia

Adr ener gi c N eur ogl y copeni c

Anx i ety Headache

N er v ousness Bl ur r ed  v i si on

Tr em ul ousness Par esthesi as

Sw eati ng Weak ness


Hunger Ti r edness

Pal pi tati ons Conf usi on

Ir r i tabi l i ty Di zzi ness

Pal l or Am nesi a

N ausea Incoor di nati on

Fl ushi ng Abnor m al   m entati on

Angi na Behav i or al   change

  Feel i ng  col d

  Di f f i cul ty   w ak i ng  i n  the  m or ni ng

  Seni l e  dem enti a

  Or gani c  per sonal i ty   sy ndr om e

  Tr ansi ent  hem i pl egi a

  Tr ansi ent  aphasi a

  Sei zur es

  Com a

3.   What  i s  the  best  f i r st  step  i n  cl assi f y i ng  hy pogl y cem i a?

Ther e  ar e  a  v ar i ety   of   m ethods  f or   categor i zi ng  the  condi ti ons  that  cause
hy pogl y cem i a,   but  none  of   these  schem es  i s  com pl etel y   sati sf actor y .   One
appr oach  i s  to  di v i de  the  causes  i nto  those  i nv ol v i ng  i ncr eased  i nsul i n
l ev el s,   those  i nv ol v i ng  i ncr eased  gl ucose  consum pti on,   or   those  i nv ol v i ng
decr eased  gl ucose  pr oducti on.   In  r eal i ty ,   how ev er ,   m ost  of   the  causes  of
hy pogl y cem i a  em br ace  a  com bi nati on  of   these  m echani sm s.   An
al ter nati v e  and  m or e  usef ul   schem e  i s  based  on  the  hi stor y   and  phy si cal
ex am i nati on  f i ndi ngs.   The  k ey   f eatur es  of   thi s  appr oach  ar e  to  assess
w hether   the  hy pogl y cem i a  occur s  w i th  f asti ng  or   postpr andi al l y ,   and
w hether   the  af f ected  per son  appear s  heal thy .   In  gener al ,   the
hy pogl y cem i a  that  occur s  w i th  f asti ng  or   that  i s  f ound  i n  peopl e  w ho
appear   gener al l y   i l l   i s  a  m or e  om i nous  f or m   of   the  di sor der .

4.   What  ar e  the  causes  of   m edi cal l y   si gni f i cant  hy pogl y cem i a?

The  speci f i c  causes  of   hy pogl y cem i a  ar e  num er ous  (Tabl e  3­6).   The
hi stor y ,   phy si cal   ex am i nati on,   and  i ni ti al   l abor ator y   tests  ar e  per f or m ed
i n  an  ef f or t  to  r ul e  out  the  com m on  causes.

The  m ost  com m on  cause  of   hy pogl y cem i a  ov er al l   i s  the  adm i ni str ati on  of
a  hy pogl y cem i c  agent,   ei ther   i nsul i n  or   an  or al   hy pogl y cem i c  agent.
These  m edi cati ons  m ay   hav e  been  pr escr i bed  f or   the  contr ol   of   di abetes
or   m ay   be  i ngested  i n  er r or .   If   thi s  cause  i s  not  obv i ous  f r om   the
pati ent's  hi stor y ,   the  di agnosi s  can  be  m ade  by   per f or m i ng  an  or al
hy pogl y cem i c  scr een  on  a  sam pl e  of   pl asm a,   or   by   m easur i ng  the  i nsul i n
and  C  pepti de  l ev el   at  the  ti m e  of   hy pogl y cem i a.   C  pepti de  i s  a  by ­
pr oduct  of   endogenous  i nsul i n  pr oducti on.   If   the  i nsul i n  pr oduci ng
hy pogl y cem i a  i s  ex ogenous,   the  i nsul i n  l ev el   i s  hi gh  and  the  C  pepti de
l ev el   i s  suppr essed.

In  one  ser i es  consi sti ng  of   hospi tal i zed  pati ents  w i th  hy pogl y cem i a,   the
second  m ost  com m on  cause  of   hy pogl y cem i a  w as  r enal   f ai l ur e.   Renal
f ai l ur e  causes  hy pogl y cem i a  f or   sev er al   r easons.   Fi r st,   because  the
k i dney s  pl ay   an  i m por tant  r ol e  i n  i nsul i n  cl ear ance,   i nsul i n  cl ear ance  m ay
be  decr eased  and  i nsul i n  l ev el s  i nappr opr i atel y   hi gh  i n  the  pr esence  of
r enal   f ai l ur e.   Second,   dur i ng  pr ol onged  f asti ng,   the  k i dney s  m ay   be
r esponsi bl e  f or   as  m uch  as  30%  of   the  net  gl uconeogenesi s  that  tak es
pl ace,   and  thi s  w oul d  be  com pr om i sed  i n  the  setti ng  of   r enal   f ai l ur e.
Fi nal l y ,   i t  appear s  that  ur em i c  tox i ns  m ay   suppr ess  hepati c  gl ucose
output.   As  w i th  other   f or m s  of   hy pogl y cem i a,   i nadequate  cal or i c  i ntak e
dur i ng  a  m edi cal   i l l ness  of ten  contr i butes  to  the  dev el opm ent  of
hy pogl y cem i a.

Hy pogl y cem i a  m ay   occur   i n  associ ati on  w i th  a  num ber   of   tum or s  i ncl udi ng
i sl et  cel l   tum or s  and  non–i sl et  cel l   tum or s  (Tabl e  3­7).   The  l atter   ar e
usual l y   l ar ge  tum or s  l ocated  i n  the  m edi asti num   or   r etr oper i toneum .   The
m echani sm   by   w hi ch  these  tum or s  cause  hy pogl y cem i a  r em ai ns  som ew hat
obscur e.   One
P. 120
ex pl anati on  m ay   be  hi gh  l ev el s  of   gl ucose  ex tr acti on  and  uti l i zati on  by
the  tum or   m ass.   A  second  contr i buti ng  f eatur e  i s  poor   nutr i ti on  i n  these
pati ents.   An  i ncr eased  acti v i ty   of   IGF­II  has  been  show n  i n  som e  pati ents
w i th  non–i sl et  cel l   tum or s.   IGF­II  can  i nter act  w i th  the  i nsul i n  r eceptor ,
al though  w i th  l ess  af f i ni ty   than  i nsul i n  i tsel f .   N or m al l y ,   IGF­II  cl eav es  to
a  sm al l er   pr otei n  w i th  m i ni m al   i nsul i n­l i k e  acti v i ty .   It  has  been  show n
that  al though  the  IGF­II  l ev el s  ar e  not  i ncr eased  i n  these  pati ents  w i th
hy pogl y cem i a  associ ated  w i th  cancer ,   ther e  ar e  i ncr eased  l ev el s  of   â
€œbi g  IGF­II. â€​   Thi s  i s  the  uncl eav ed  f or m   of   the  hor m one  that  has  m or e
i nsul i n­l i k e  acti v i ty .

Table 3­6 Etiologic Classification of
Hypoglycemia

Hy pogl y cem i a  associ ated  pr edom i nantl y   w i th  f asti ng


Hy per secr eti on  of   i nsul i n  due  to  i sl et  cel l   adenom a,
car ci nom a,   hy per pl asi a,   or   nesi di obl astosi s
Hepati c  di sease
Gener al i zed  hy pof uncti on
Ethanol   hy pogl y cem i a  associ ated  w i th  pr i or   poor
nutr i ti on  and  decr eased  gl y cogen  stor es
Sepsi s
Endocr i ne  def i ci enci es
Anter i or   pi tui tar y   i nsuf f i ci ency —gr ow th
hor m one,   adr enocor ti cotr opi c  hor m one
Adr enocor ti cal   i nsuf f i ci ency
Hy pothy r oi di sm
Lar ge  noni sl et  cel l   tum or s
Renal   di sease
Def i ci ent  car bohy dr ate  stor es  or   i ntak e
Sev er e  i nani ti on
Sev er e  ex er ci se
Autoi m m une  w i th  i nsul i n  anti bodi es  or   anti bodi es  to
the  i nsul i n  r eceptor
Dr ug  i nduced
Reacti v e  or   sti m ul ati v e  hy pogl y cem i a
Idi opathi c  f uncti onal   hy pogl y cem i a
Al i m entar y   hy per i nsul i ni sm
Pr edi abeti c  f uncti onal   hy pogl y cem i a
Endocr i ne  def i ci enci es
Facti ti ous  and  ar ti f actual   hy pogl y cem i a
Sur r epti ti ous  i nsul i n  adm i ni str ati on
Sur r epti ti ous  sul f ony l ur ea  i ngesti on
El ev ated  l euk ocy te  count—l euk em i a  or   pol y cy them i a
Hy pogl y cem i a  of   i nf ancy
Abnor m al i ti es  i n  hor m one  secr eti on
Abnor m al i ti es  of   pr oducti on  and  uti l i zati on  of
m etabol i c  f uel s
Abnor m al i ti es  i n  substr ate  av ai l abi l i ty

Another   com m on  cause  of   hy pogl y cem i a  i s  the  i ngesti on  of   a  dr ug  that
sti m ul ates  per i pher al   gl ucose  uti l i zati on,   i nhi bi ts  hepati c  gl ucose
pr oducti on,
P. 121
or   sti m ul ates  i nsul i n  r el ease,   and  ther e  ar e  a  l ar ge  num ber   of   such  dr ugs.
The  dr ugs  m ost  of ten  i m pl i cated  ar e  i n  par t  a  f uncti on  of   the  age  of   the
pati ent  (Tabl e  3­8).   Al cohol   m ay   actual l y   be  the  m ost  com m on  dr ug
associ ated  w i th  hy pogl y cem i a  because  i t  causes  an  i ncr ease  i n  the  r ati o
of   ni coti nam i de  adeni ne  di nucl eoti de  hy dr ogenase  (N ADH)  to  N AD + ,   w hi ch
decr eases  the  gl uconeogeni c  capaci ty   of   the  l i v er .   The  anti par asi ti c  dr ug
pentam i di ne  i s  now   w i del y   used  i n  the  tr eatm ent  of   Pneum ocy sti s  car i ni i
pneum oni a  i n  pati ents  w i th  AIDS.   It  can  pr oduce  hy pogl y cem i a  by   i njur i ng
the  pancr eati c  i sl et  cel l s,   ther eby   causi ng  i nsul i n  r el ease  and
i nappr opr i ate  hy per i nsul i nem i a.   As  w i th  al l   f or m s  of   hy pogl y cem i a,
i nadequate  cal or i c  i ntak e  of ten  contr i butes  to  the  dev el opm ent  of
sy m ptom ati c  hy pogl y cem i a.

Table 3­7 Non–Islet Cell Tumors Associated
with Hypoglycemia

Mesenchy m al
Mesothel i om a
Fi br osar com a
Rhabdom y osar com a
Lei om y osar com a
Li posar com a
Hem angi oper i cy tom a
Car ci nom as
Hepati c:  hepatom a,   bi l i ar y   car ci nom a
Adr enocor ti cal   car ci nom a
Geni tour i nar y :  hy per nephr om a,   Wi l m s'  tum or ,   pr ostate
car ci nom a
Repr oducti v e:  cer v i cal   car ci nom a,   br east  car ci nom a
N eur ol ogi c  and  neur oendocr i ne
Pheochr om ocy tom a
Car ci noi d  tum or
N eur of i br om a
Hem atol ogi c
Leuk em i as
Ly m phom a
My el om a

Leuk em i a  and  pol y cy them i a  v er a  can  cause  pseudohy pogl y cem i a  because
of   the  hi gh  WBC  or   Hct  v al ue  i n  these  setti ngs,   w hi ch  can  r esul t  i n
conti nued  gl ucose  consum pti on  i n  the  test  tube  af ter   the  bl ood  sam pl e  has
been  obtai ned.   In  thi s  si tuati on,   the  bl ood  gl ucose  l ev el   i s  ex tr em el y   l ow
but  the  pati ent  i s  w i thout  sy m ptom s.   To  deter m i ne  the  actual   bl ood
gl ucose  l ev el   i n  such  pati ents,   bl ood  shoul d  be  dr aw n  i nto  a  tube  that
contai ns  a  substance  that  poi sons  the  bl ood  el em ents  and  pr ev ents
gl y col y si s  f r om   occur r i ng  af ter   col l ecti on.

Postpr andi al   (r eacti v e)  hy pogl y cem i a  can  occur   i n  as  m any   as  20%  of
pati ents  af ter   gastr i c  sur gi cal   pr ocedur es.   Thi s  condi ti on  i s  al so  cal l ed
P. 122
al i m entar y   hy pogl y cem i a  and  can  occur   af ter   a  v ar i ety   of   pr ocedur es,
i ncl udi ng  gastr i c  by pass,   gastr ectom y ,   gastr oenter ostom y ,   py l or opl asty ,
and  v agotom y .   Al though  bi ochem i cal   hy pogl y cem i a  i s  not  r ar e  i n  these
pati ents  dur i ng  a  l ong  or al   gl ucose  tol er ance  test,   sy m ptom ati c
hy pogl y cem i a  i s  uncom m on.

Table 3­8 Drugs Associated with
Hypoglycemia in a Variety of Age­groups

Age No.  of Drugs  Mos t Fre que ntly U s e d (No.  of
Ra nge P a tie nts Ca s e s )
(yr)

N ew bor n 47 Sul f ony l ur ea  (m other )  (14);  pr opr anol ol


(19);  r i todr i ne,   etc.   (14)

0­2 26 Sal i cy l ate  (17);  pr opr anol ol   (9)

2­10 48 Al cohol   (28);  qui ni ne  (15);  pr opr anol ol


(3);  sul f ony l ur ea  (2)

11­30 79 Sul f ony l ur ea  (34);  i nsul i n  (f acti ti ous)


(20);  qui ni ne  (10);  al cohol
    (8);  i nsul i n  +   dr ug a (3);  i nsul i n  +
al cohol   (2);  pr opr anol ol   (2)

31­40 78 Al cohol   (50);  sul f ony l ur ea  (14);  qui ni ne


(4);  i nsul i n  +   al cohol
    (3)  or   dr ug  (3);  i nsul i n  (f acti ti ous)  (2);
pr opr anol ol   (2)

41­50 71 Al cohol   (33);  sul f ony l ur ea  (19);  i nsul i n


+   al cohol   (5);  pr o  pr anol ol
    (3);  al cohol   +   dr ug  (2);  qui ni ne  (2);
di sopy r am i de  (1)
51­60 177 Sul f ony l ur ea  (86);  al cohol   (72);
pr opr anol ol   (4);
    sul f ony l ur ea  +   i nsul i n  (3)  or   al cohol
(3)  or   dr ug  (3);
    di sopy r am i de  (3);  qui ni ne  (1)

61­70 242 Sul f ony l ur ea  (173);  al cohol   (35);


sul f ony l ur ea  +   dr ug  (10)  or
    pheny l butazone  (8)  or   i nsul i n  (4);
di sopy r am i de  (5);  pr o
    pr anol ol   (3)

Ol der 273 Sul f ony l ur ea  (219);  al cohol   (23);


than  71 sul f ony l ur ea  +   dr ug  (12)  or
    phenf or m i n  (6);  di sopy r am i de  (5);
pr opr anol ol   (3)

Total 1, 041
 
(69%) b

a An  agent  w i thout  i ntr i nsi c  hy pogl y cem i c  acti v i ty .

b Per centage  of   1, 418  pati ents  f or   w hom   data  w er e  av ai l abl e.

Other   causes  of   hy pogl y cem i a  ar e  m uch  l ess  com m on.   Fasti ng  by   i tsel f   i s
a  r ar e  cause.   How ev er ,   ex tr em el y   l ong  per i ods  of   i nadequate  nutr i ti on
ar e  r equi r ed  f or   hy pogl y cem i a  to  occur   i n  the  absence  of   other   m etabol i c
def ects.   Thi s  i s  seen  i n  the  setti ng  of   anor ex i a  ner v osa  and  star v ati on.
Li k ew i se,   l i v er   di sease  pr oduces  hy pogl y cem i a  onl y   i n  i ts  m ost  sev er e
f or m s  or   i n  conjuncti on  w i th  i nadequate  cal or i c  i ntak e.   Hy pogl y cem i a  i s
occasi onal l y   pr oduced  by   the  pr esence  of   autoanti bodi es  ei ther   to  i nsul i n
i tsel f   or   to  the  i nsul i n  r eceptor ,   but  these  condi ti ons  usual l y   occur   i n  the
pr esence  of   a  k now n  autoi m m une  sy ndr om e.   Fi nal l y ,   age  pl ay s  an
i m por tant  r ol e  i n  the  suscepti bi l i ty   to  hy pogl y cem i a.   El der l y   peopl e  l ose
counter r egul ator y   hor m one  r esponses  to  hy pogl y cem i a,   ar e  f r equentl y   on
m ul ti pl e  m edi cati ons,   and  m ay   hav e  m i l d  or gan  dy sf uncti on  (r enal   f ai l ur e,
l i v er   dy sf uncti on,   or   congesti v e  hear t  f ai l ur e),   al l   of   w hi ch  can  i ncr ease
the  l i k el i hood  of   m ul ti f actor i al   hy pogl y cem i a.   In  addi ti on,   el der l y   pati ents
m ay   hav e  dem enti a  that  can  i nter f er e  w i th  i nsi ght  and  nor m al   f ood­
seek i ng  behav i or .

P. 123
5.   In  peopl e  w i th  di abetes,   w hat  f actor s  ar e  associ ated  w i th  an  i ncr eased
r i sk   of   hy pogl y cem i a?
Hy pogl y cem i a  occur s  al l   too  f r equentl y   i n  tr eated  di abeti c  pati ents,   and  i s
ei ther   di r ectl y   or   i ndi r ectl y   the  cause  of   death  i n  3%  to  5%  of   al l   pati ents
w i th  T1DM.   It  r esul ts  f r om   ex cessi v e  i nsul i n  adm i ni str ati on,   i nadequate
cal or i c  i ntak e,   or   ex cessi v e  ex er ci se.   In  nondi abeti c  peopl e,   i f
hy pogl y cem i a  dev el ops,   a  num ber   of   hor m ones  r espond  to  i ncr eased
gl ucose  pr oducti on  and  m ai ntai n  a  nor m al   bl ood  sugar   l ev el .   In  addi ti on,
the  per son  noti ces  sy m ptom s  of   hy pogl y cem i a  and  i ngests  car bohy dr ate
to  counter act  these.   In  peopl e  w i th  l ong­standi ng  di abetes,   how ev er ,
ther e  m ay   be  hy pogl y cem i c  unaw ar eness  and  autonom i c  neur opathy ,   both
of   w hi ch  bl unt  the  nor m al   r esponse  to  hy pogl y cem i a.   Another   f actor   that
pl ay s  a  r ol e  i n  the  hy pogl y cem i a  that  occur s  i n  di abeti c  pati ents  has  to  do
w i th  the  i ntr oducti on  of   r ecom bi nant  hum an  i nsul i n,   such  that  v er y   f ew
pati ents  now   r em ai n  on  pur i f i ed  por k   i nsul i n.

Dur i ng  the  dev el opm ent  of   T1DM,   ther e  m ay   be  a  per i od  w hen  i sl et  cel l s
ar e  dam aged  but  sti l l   r etai n  thei r   capaci ty   to  sy nthesi ze  and  stor e
i nsul i n.   Dur i ng  thi s  per i od,   i nsul i n  m ay   be  r el eased  i n  a  dy sf uncti onal
m anner   i n  r esponse  to  nonphy si ol ogi c  sti m ul i ,   or   i n  i nappr opr i ate
quanti ti es.   Thi s  m ay   r esul t  i n  epi sodes  of   sy m ptom ati c  hy pogl y cem i a,
but,   l ater ,   as  the  i sl et  cel l s  ar e  com pl etel y   destr oy ed,   i nsul i nopeni a  and
hy per gl y cem i a  pr edom i nate.

Hi stor i cal l y ,   ther e  has  been  a  str ong  desi r e  to  nor m al i ze  the  bl ood
gl ucose  l ev el s  i n  pati ents  w i th  di abetes  i n  an  ef f or t  to  pr ev ent  l ong­ter m
com pl i cati ons.   Wi th  the  adv ent  of   hom e  gl ucose  m oni tor i ng,   m ul ti pl e  dai l y
i njecti ons  of   shor t­  and  l ong­acti ng  i nsul i ns,   i nsul i n  pum ps,   and
gl y cosy l ated  Hgb  deter m i nati ons,   ti ght  contr ol   i s  attai nabl e.   What  has
been  l ear ned,   how ev er ,   i s  that  ther e  i s  a  tr ade­of f ,   i n  that  ti ght  contr ol
can  be  achi ev ed  onl y   by   accepti ng  a  substanti al   i ncr ease  i n  the  r i sk   of
sy m ptom ati c  and  l i f e­thr eateni ng  hy pogl y cem i a.   The  DCCT  has
dem onstr ated  that  ti ght  contr ol   of   bl ood  gl ucose  i n  pati ents  w i th  T1DM
pr ev ents  or   del ay s  the  dev el opm ent  of   di abeti c  com pl i cati ons.   It  i s
pr udent  to  str i v e  tow ar d  ti ght  contr ol   w hi l e  av oi di ng  f r equent
hy pogl y cem i c  epi sodes.

6.   What  i s  r eacti v e  hy pogl y cem i a  and  how   shoul d  i t  be  ev al uated?

The  ter m   hy pogl y cem i a  i s  r ecogni zed  by   m uch  of   the  l ay   publ i c  as  a
com m on  pr obl em   that  occur s  at  10:30  a. m .   i n  w om en  w hose  br eak f ast
consi sted  of   a  cup  of   cof f ee  and  a  str aw ber r y   Dani sh.   Som e  phy si ci ans
hav e  ev al uated  these  r eacti v e  hy pogl y cem i c  sy m ptom s  w i th  or al   gl ucose
tol er ance  tests.   How ev er ,   thi s  appr oach  i s  pr obl em ati c  because  m ost  of
the  w om en  w i th  these  sy m ptom s  do  not  hav e  bl ood  gl ucose  l ev el s  that
ar e  l ess  than  50  m g/dL  at  the  ti m e  of   thei r   sy m ptom s  and,   i n  f act,   m ost
of   these  sy m ptom s  r esol v e  spontaneousl y   w i thout  the  i ngesti on  of
car bohy dr ate.   In  addi ti on,   som e  “heal thy â€​   w om en  can  hav e  bl ood
gl ucose  v al ues  that  ar e  l ess  than  50  m g/dL  3  to  4  hour s  af ter   a  100­g  or al
gl ucose  l oad,   and  y et  not  hav e  sy m ptom s.   In  gener al ,   these  peopl e  do  not
hav e  a  ser i ous  i l l ness  and  v i r tual l y   nev er   hav e  an  i nsul i nom a  i n  the
absence  of   m or e  ty pi cal   epi sodes  that  occur   w i th  f asti ng.   Instead,   they
need  r eassur ance  and  a  pr acti cal   appr oach  to  thei r   sy m ptom s.   Di ets  that
ar e  l ow   i n  car bohy dr ate,   hi gh
P. 124
i n  pr otei n,   and  hi gh  i n  f i ber   hav e  not  been  concl usi v el y   show n  to  be  of
benef i t,   and  ex tr em e  di ets  shoul d  be  av oi ded.   The  r egul ar   i ngesti on  of   a
bal anced  di et  i n  per haps  f our   to  f i v e  m eal s  ov er   the  cour se  of   the  day
i nstead  of   the  tr adi ti onal   thr ee  m ay   be  of   benef i t  i n  these  peopl e.   In
som e  cases,   r eacti v e  hy pogl y cem i a  m ay   be  a  r esul t  of   i nsul i n  r esi stance,
hy per i nsul i nem i a  i n  r esponse  to  a  hi gh  car bohy dr ate  m eal ,   and  m i sm atch
of   i nsul i n  and  gl ucose  cl ear ance  af ter   the  m eal .   In  these  cases,
hy pogl y cem i a  m ay   r espond  to  i nsul i n  sensi ti zer s  as  w el l   as  the  abov e
m easur es.

Case
A  52­y ear ­ol d  w hi te  w om an  has  an  18­m onth  hi stor y   of   epi sodi c  conf usi on  and
poor   w or k   per f or m ance  but  neur ol ogi c  ev al uati on,   i ncl udi ng  CT  scan  of   the
head  and  an  el ectr oencephal ogr am ,   i s  unr ev eal i ng.   Di l anti n  and  then
phenobar bi tal   ar e  pr escr i bed  but  do  not  al ter   the  f r equency   of   the  attack s,   and
ar e  ev entual l y   di sconti nued.   On  the  day   of   adm i ssi on,   she  has  a  gener al i zed
sei zur e  at  w or k .   The  par am edi cs  ar e  cal l ed,   f i nd  her   unconsci ous,   and
adm i ni ster   nal ox one  hy dr ochl or i de  [N ar can  (Du  Pont  Mer ck   Phar m aceuti cal ,
Wi l m i ngton,   DE)]  and  1  am pul e  of   50%  dex tr ose  IV.   She  then  r egai ns
consci ousness.   Her   bl ood  gl ucose  l ev el   bef or e  r ecei v i ng  the  50%  dex tr ose  i s
28  m g/dL.   She  deni es  consum i ng  al cohol   or   tak i ng  any   pr escr i pti on
m edi cati ons.   Her   f am i l y   hi stor y   i s  unr em ar k abl e  and  she  has  no  hi stor y   of
gastr i c  sur ger y .   On  phy si cal   ex am i nati on,   she  i s  f ound  to  be  a  thi n  w om an  w ho
appear s  to  be  i n  good  heal th.   Her   ex am i nati on  f i ndi ngs  ar e  nor m al ,   as  ar e  her
i ni ti al   l abor ator y   r esul ts.

1.   What  i s  the  l i k el y   di agnosi s  i n  thi s  pati ent?


2.   If   she  had  a  f am i l y   hi stor y   of   thi s  pr obl em ,   w hat  other   endocr i ne  tum or s
w oul d  y ou  l ook   f or ?
3.   What  di agnosti c  test,   or   tests,   ar e  usef ul   i n  m ak i ng  thi s  di agnosi s?
4.   If   the  r esul ts  of   the  bi ochem i cal   studi es  i ndi cate  she  has  an  i nsul i nom a,
w hat  shoul d  the  nex t  test  be?
5.   What  i s  the  pr oper   ther apy   f or   an  i nsul i nom a?

Case Discussion
1.   What  i s  the  l i k el y   di agnosi s  i n  thi s  pati ent?

The  pati ent's  hi stor y   suggests  the  pr esence  of   an  i nsul i nom a  because  the
hy pogl y cem i a  i s  sev er e,   r ecur r ent,   pr ogr essi v e,   sy m ptom ati c,   and
r ev er sed  by   the  adm i ni str ati on  of   IV  gl ucose.   The  sy m ptom ati c  epi sodes
of   hy pogl y cem i a  associ ated  w i th  an  i nsul i nom a  m ay   occur   i n  the
postpr andi al   state,   but  al m ost  nev er   ex cl usi v el y   i n  thi s  state  (Tabl e  3­9).
Most  peopl e  w i th  adr enal   i nsuf f i ci ency ,   tum or ­associ ated  hy pogl y cem i a,
or   al i m entar y   hy pogl y cem i a  hav e  other   si gns  or   sy m ptom s,   appear   i l l ,   or
hav e  a  k now n  sur gi cal   hi stor y .

2.   If   she  had  a  f am i l y   hi stor y   of   thi s  pr obl em ,   w hat  other   endocr i ne  tum or s
w oul d  y ou  l ook   f or ?

Ther e  ar e  thr ee  gener al l y   r ecogni zed  sy ndr om es  of   MEN   [or   m ul ti pl e
endocr i ne  adenom atosi s  (MEA)].   Peopl e  w i th  MEN   1  can  hav e  tum or s  of
the  pi tui tar y   (e. g. ,
P. 125
pr ol acti nom as  or   Cushi ng's  di sease),   the  pancr eas  (i nsul i nom a  and
gastr i nom a  m ost  com m onl y ),   or   the  par athy r oi d  gl ands.   U sual l y ,
hy per cal cem i a  due  to  hy per par athy r oi di sm   dev el ops  f i r st.   Those  af f ected
w i th  MEN   2A  ar e  at  r i sk   f or   m edul l ar y   car ci nom a  of   the  thy r oi d,
pheochr om ocy tom a,   and,   l ess  com m onl y ,   hy per par athy r oi di sm .   Al l   these
f eatur es  can  be  f ound  i n  peopl e  w i th  MEN   2B,   together   w i th  m ucosal
neur om as.   These  sy ndr om es  can  occur   ei ther   i n  f am i l i es  or   spor adi cal l y .
In  al l   pati ents  w i th  i nsul i nom as,   the  ser um   cal ci um   and  pr ol acti n  l ev el s
shoul d  be  check ed  and  a  com pl ete  hi stor y   and  phy si cal   ex am i nati on
per f or m ed  to  l ook   f or   ev i dence  of   the  other   potenti al l y   associ ated
condi ti ons.   Am ong  the  cases  of   spor adi c  nonf am i l i al   i nsul i nom as,   80%  ar e
sol i tar y   and  beni gn,   11%  ar e  m ul ti pl e  and  beni gn,   and  6%  ar e  si ngl e  and
m al i gnant.   The  r em ai ni ng  3%  of   the  pati ents  hav e  m ul ti pl e  m al i gnant
tum or s  or   i sl et  hy per pl asi a.   Ten  per cent  of   the  i nsul i nom as  occur   i n
associ ati on  w i th  MEN   1,   and  ar e  m ul ti f ocal   80%  of   the  ti m e.

Table 3­9 Association of Hypoglycemia
Symptoms with Eating in People with
Insulinoma

Timing of Symptoms No.  of P e rc e nta ge  of


P a tie nts Tota l

1.   Sy m ptom s  dur i ng  or   af ter 20 26


ov er ni ght  f ast
onl y   (bef or e  br eak f ast)

2.   Fasti ng  and  day ti m e 21 27


postpr andi al   (bef or e
l unch  or   di nner )  sy m ptom s

3.   Sy m ptom s  af ter   m i ssed  m eal 6 8


onl y

4.   Postpr andi al   (bef or e  l unch 23 29


and  di nner )
sy m ptom s  onl y

5.   U ncer tai n  about  ti m i ng  of 7 9


sy m ptom s

6.   N o  sy m ptom s  ex per i enced 1/78 1/100

Sy m ptom s  ex acer bated  by 24 31


ex er ci se

3.   What  di agnosti c  test,   or   tests,   ar e  usef ul   i n  m ak i ng  thi s  di agnosi s?

The  tr adi ti onal   di agnosti c  appr oach  i n  pati ents  w i th  a  suspected
i nsul i nom a  i s  a  super v i sed  72­hour   f ast.   If   sy m ptom ati c  hy pogl y cem i a
dev el ops  and  the  bl ood  gl ucose  l ev el   i s  l ess  than  50  m g/dL,   then  i nsul i n
and  C  pepti de  l ev el s  shoul d  be  deter m i ned.   In  one  ser i es  of   pati ents  w i th
i nsul i nom as,   hy pogl y cem i a  occur r ed  i n  the  f i r st  12  hour s  of   f asti ng  i n
29%,   w i thi n  24  hour s  i n  71%,   w i thi n  48  hour s  i n  92%,   w i thi n  60  hour s  i n
92%,   and  w i thi n  72  hour s  i n  98%.   In  thi s  ser i es,   the  bl ood  gl ucose  l ev el
at  the  ti m e  sy m ptom s  appear ed  w as  l ess  than  46  m g/dL  i n  100%,   l ess
than  39  m g/dL  i n  70%,   l ess  than  35  m g/dL  i n  50%,   and  l ess  than  28
m g/dL  i n  25%.   Because  the  i nsul i n  secr eti on  f r om   an  i nsul i nom a  i s  of ten
spor adi c,   not  al l   i nsul i nom as  can  be  di agnosed  on  the  basi s  of   a  si ngl e
f ast.   It  i s  i m por tant  to  deter m i ne  the  C  pepti de  l ev el   to  dem onstr ate  that
the  i nsul i n  i s  pr oduced  endogenousl y .   A  pr oi nsul i n  l ev el   can  al so  be
hel pf ul   i n  di agnosi ng  i nsul i nom as.   Pr oi nsul i n  i s  the  pr ohor m one  f r om
w hi ch  i nsul i n  and  C  pepti de  ar e  cl eav ed,   and  accounts  f or   onl y   15%  to
20%  of   the  ci r cul ati ng  i m m unor eacti v e  i nsul i n  i n  heal thy   peopl e.   In  those
P. 126
w i th  an  i nsul i nom a,   how ev er ,   i t  consti tutes  30%  to  90%  of   the  i nsul i n
m ass.   Ther e  ar e  other   speci al i zed  tests  f or   ev al uati ng  a  pati ent  w i th  a
suspected  i nsul i nom a  but,   al though  adv ocated  by   som e,   they   ar e  usual l y
not  necessar y .   A  ser um   dr ug  scr een  to  r ul e  out  a  dr ug­i nduced
hy pogl y cem i a  and  an  ACTH  sti m ul ati on  test  to  r ul e  out  adr enal
i nsuf f i ci ency   ar e  usef ul   i n  the  ev al uati on  of   hy pogl y cem i a  of   unk now n
cause,   but  ar e  not  hel pf ul   i n  establ i shi ng  the  di agnosi s  of
hy per i nsul i ni sm .

4.   If   the  r esul ts  of   the  bi ochem i cal   studi es  i ndi cate  she  has  an  i nsul i nom a,
w hat  shoul d  the  nex t  test  be?

Once  the  bi ochem i cal   di agnosi s  of   i nsul i nom a  has  been  establ i shed,   an
anatom i c  study   i s  usual l y   done.   Al though  no  si ngl e  study   i s  com pl etel y
sati sf actor y ,   abdom i nal   CT  scanni ng  and  ul tr asonogr aphy   possess  a
r el ati v el y   hi gh  sensi ti v i ty ,   pose  no  r i sk   to  the  pati ent,   and  ar e  r el ati v el y
i nex pensi v e,   m ak i ng  them   a  good  f i r st  step.   Abdom i nal   ul tr asonogr aphy   i s
adv ocated  by   som e  as  the  super i or   study ,   but  i ts  uti l i ty   v ar i es  f r om
i nsti tuti on  to  i nsti tuti on.   Angi ogr aphy   i s  m or e  sensi ti v e  but  car r i es  som e
r i sk   and  i s  qui te  ex pensi v e.   Som e  gr oups  hav e  adv ocated  tr anshepati c
v enous  sam pl i ng.   In  thi s  m ethod,   by   m easur i ng  i nsul i n  l ev el s  i n  the
v enous  bl ood  dr ai ni ng  a  par ti cul ar   r egi on  of   the  pancr eas,   the  tum or   can
be  l ocal i zed,   al though  not  v i sual i zed.   The  new est  pr eoper ati v e  l ocal i zi ng
techni que  i s  endoscopi c  ul tr asonogr aphy .   In  thi s  techni que,   the  ul tr asound
tr ansducer   i s  endoscopi cal l y   pl aced  i n  the  duodenum .   Fr om   thi s  si te,   the
head  of   the  pancr eas  can  be  w el l   v i sual i zed,   y i el di ng  a  better   sensi ti v i ty
than  that  of   tr adi ti onal   abdom i nal   ul tr asonogr aphy .   How ev er ,   thi s
technol ogy   i s  not  y et  w i del y   av ai l abl e.   The  m ai n  pr obl em   w i th  al l   these
appr oaches  i s  that  m ost  i nsul i nom as  ar e  sm al l   (av er age,   1. 5  cm ,   2  g),
and  the  di agnosi s  hi nges  on  the  cl i ni cal   pr esentati on  and  the  r esul ts  of
bi ochem i cal   studi es.   If   the  anatom i c  studi es  ar e  unr ev eal i ng  and  the
bi ochem i cal   r esul ts  ar e  conv i nci ng,   the  pati ent  shoul d  under go
ex pl or ator y   sur ger y   per f or m ed  by   an  ex per i enced  pancr eati c  sur geon.   For
thi s  r eason,   ex tensi v e  pr eoper ati v e  anatom i c  studi es  ar e  not  adv ocated.

5.   What  i s  the  pr oper   ther apy   f or   an  i nsul i nom a?

Sur gi cal   r em ov al   per f or m ed  by   an  ex per i enced  sur geon  i s  the  pr i m ar y
f or m   of   ther apy   f or   i nsul i nom as.   Intr aoper ati v e  di r ect  ul tr asonogr aphi c
ex am i nati on  of   the  pancr eas  com bi ned  w i th  m anual   pal pati on  by   an
ex per i enced  sur geon  successf ul l y   l ocal i zes  the  tum or   80%  to  90%  of   the
ti m e.   Once  the  tum or s  ar e  r esected,   m ost  pati ents  ar e  cur ed.   For   those
w ho  ar e  not  cur ed  by   sur gi cal   m eans,   l ong­acti ng  som atostati n  anal ogs
can  be  used  to  decr ease  the  f r equency   and  sev er i ty   of   the  hy pogl y cem i c
epi sodes.   Di azox i de,   v er apam i l ,   pheny toi n,   and  pr opr anol ol   hav e  been
used  successf ul l y   i n  a  f ew   cases.   In  these  pati ents,   f r equent  schedul ed
m eal s  ar e  an  i m por tant  com ponent  of   ther apy .

Suggested Readings
Adr ogue  HJ.   Gl ucose  hom eostasi s  and  the  k i dney .   Ki dney   Int  1992;42:1266.

Fi el d  JB.   Hy pogl y cem i a:  def i ni ti on,   cl i ni cal   pr esentati ons,   cl assi f i cati on,
and  l abor ator y   tests.   Endocr i nol   Metab  Cl i n  N or th  Am   1989;18:27.

Fi scher   KF,   Lees  JA,   N ew m an  JH.   Hy pogl y cem i a  i n  hospi tal i zed  pati ents:
causes  and  outcom es.   N   Engl   J  Med  1986;315:1245.

P. 127

Kur l an  R.   Postpr andi al   (r eacti v e)  hy pogl y cem i a  and  r estl ess  l eg  sy ndr om e:
r el ated  neur ol ogi c  di sor der s?  Mov   Di sor d  1998;13:619.

Leonetti   F,   Foni ci el l o  M,   Iozzo  P,   et  al .   Incr eased  nonox i dati v e  gl ucose


m etabol i sm   i n  i di opathi c  r eacti v e  hy pogl y cem i a.   Metabol i sm   1996;45:606.

Pi edr ol a  G,   Cassado  JL,   Lopez  E,   et  al .   Cl i ni cal   f eatur es  of   adr enal
i nsuf f i ci ency   i n  pati ents  w i th  acqui r ed  i m m unodef i ci ency   sy ndr om e.   Cl i n
Endocr i nol   1996;45:97.

Ross  RJ,   Tr ai ner   PJ.   Endocr i ne  i nv esti gati on:  Cushi ng's  sy ndr om e.   Cl i n
Endocr i nol   1998;49:153.

Sel tzer   HS.   Dr ug­i nduced  hy pogl y cem i a:  a  r ev i ew   of   1418  cases.
Endocr i nol   Metab  Cl i n  N or th  Am   1989;18:163.

Ser v i ce  FJ.   Hy pogl y cem i as.   West  J  Med  1991;154:442.

Ser v i ce  FJ.   Hy pogl y cem i a.   Endocr i nol   Metab  Cl i n  N or th  Am   1997;26:937.

Soder ber gh  A,   Wi nqv i st  O,   N or hei m   I,   et  al .   Adr enal   autoanti bodi es  and
or gan­speci f i c  autoi m m uni ty   i n  pati ents  w i th  Addi son's  di sease.   Cl i n
Endocr i nol   1996;45:453.

Metabolic Bone Disease
1.   Whi ch  di seases  of   bone  ar e  consi der ed  to  be  m etabol i c  i n  or i gi n?

2.   What  i s  osteopeni a?

3.   What  condi ti ons  m ay   cause  osteopeni a?

4.   What  ar e  the  r i sk   f actor s  f or   osteopor osi s?

5.   What  si m pl e  l abor ator y   tests  can  hel p  assess  the  pati ent  w i th  osteopeni a?

6.   When  ar e  bone  densi ty   m easur em ents  i ndi cated?

Discussion
1.   Whi ch  di seases  of   bone  ar e  consi der ed  to  be  m etabol i c  i n  or i gi n?

Metabol i c  bone  di seases  ar e  those  condi ti ons  i n  w hi ch  al l   the  m etabol i c


bone  uni ts  thr oughout  the  sk el eton  ar e  equal l y   af f ected  by   the  di sease
pr ocess.   These  di seases  i ncl ude  osteopor osi s,   osteom al aci a,   ostei ti s
f i br osa  cy sti ca,   and  osteogenesi s  i m per f ecta.   Di seases  that  af f ect  ei ther
a  si ngl e  ar ea  or   m ul ti pl e  ar eas  i n  bone  ar e  consi der ed  l ocal i zed  bone
di seases,   and  i ncl ude  Paget's  di sease,   f i br ous  dy spl asi a,   bone  cy sts,
heal i ng  f r actur es,   Sudeck 's  atr ophy ,   and  i njur y   di suse  osteopor osi s.
2.   What  i s  osteopeni a?

Osteopeni a  consti tutes  a  di agnosi s  based  on  r adi ogr aphi c  f i ndi ngs,   i n  that
the  m i ner al   content  of   the  bones  i s  seen  to  be  r educed  on  r adi ogr aphy .
U sual l y ,   bef or e  these  studi es  can  show   bone  l oss,   how ev er ,
appr ox i m atel y   30%  to  40%  of   the  sk el eton  m ust  hav e  dem i ner al i zed.   In
addi ti on,   osteopeni a  i s  now   def i ned  as  a  bone  densi ty   that  i s  1  to  2. 5
standar d  dev i ati ons  bel ow   the  m ean  f or   y oung  w om en,   on  dual   ener gy   x ­
r ay   absor pti om etr y   (DEXA)  i m agi ng.   Any   of   the  m etabol i c  bone  condi ti ons
l i sted  can  cause  osteopeni a.

3.   What  condi ti ons  m ay   cause  osteopeni a?

Ther e  ar e  m any   di sease  pr ocesses  to  be  consi der ed  i n  the  osteopeni c
pati ent.   The  condi ti on  m ost  of ten  encounter ed  i n  such  pati ents  i s  age­
r el ated,
P. 128
i di opathi c  osteopor osi s.   Ty pe  I  osteopor osi s  i s  postm enopausal
osteopor osi s  and  i s  usual l y   m ani f ested  cl i ni cal l y   by   v er tebr al   f r actur es;
ty pe  II  osteopor osi s  has  been  ter m ed  seni l e  osteopor osi s  and  i s
char acter i zed  by   hi p  f r actur e.

Ther e  ar e  m any   secondar y   causes  of   osteopeni a  seen  i n  the  setti ng  of


nutr i ti onal   def i ci ency ;  r enal ,   l i v er ,   gastr oi ntesti nal ,   and  endocr i ne  and
m etabol i c  di sease;  dr ug  usage;  and  cer tai n  l i f esty l es  (Tabl e  3­10).   In
m any   of   these  condi ti ons,   al ter ati ons  i n  the  cal ci um   l ev el   or   v i tam i n  D
m etabol i sm ,   secondar y   hy per par athy r oi di sm ,   osteom al aci a,   aci dosi s,   or   a
com bi nati on  of   these  condi ti ons  i s  the  under l y i ng  m echani sm   r esponsi bl e
f or   the  osteopeni a.

4.   What  ar e  the  r i sk   f actor s  f or   osteopor osi s?

Sm ok i ng,   poor   cal ci um   i ntak e,   i m m obi l i zati on,   m al nutr i ti on,   a


hy pogonadal   state,   and  a  f am i l y   hi stor y   ar e  al l   r i sk   f actor s  f or
osteopor osi s.   Sm ok i ng  i s  a  r i sk   f actor   because  i t  i nduces  hepati c  enzy m es
to  i nacti v ate  ci r cul ati ng  sex   hor m ones,   such  as  estr ogen.   A  hy pogonadal
state  can  occur   i n  ei ther   m en  or   w om en,   but  i n  w om en  i t  m ay   r esul t  f r om
a  total   hy ster ectom y   and  oophor ectom y   or   f r om   the  spontaneous
m enopausal   state,   both  of   w hi ch  l ead  to  l ow er ed  estr ogen  l ev el s.   Other
f actor s  i ncl ude  the  i ngesti on  of   sof t  dr i nk s,   m ost  of   w hi ch  contai n
phosphor i c  aci d.   Thi s  substance  i ncr eases  the  i ngested  phosphate  l oad,
w hi ch  i n  tur n  depr esses  the  ser um   cal ci um   l ev el   and  sti m ul ates  PTH
r el ease.   Cof f ee  i s  a  cal ci ur eti c  substance,   and,   as  such,   ex cessi v e
consum pti on  contr i butes  to  osteopor osi s.   The  f at  cel l   can  act  as  an
endocr i ne  or gan;  ther ef or e,   i n  l ean  peopl e  w hose  f at  cel l   m ass  i s
decr eased,   the  conv er si on  of   adr enal   andr ogens  to  estr ogens  i s
decr eased,   and  thi s  can  l ead  to  osteopor osi s.   Som e  of   the  l i f esty l e  r i sk
f actor s  can  be  m odi f i ed  to  pr ev ent  osteopor osi s.

5.   What  si m pl e  l abor ator y   tests  can  hel p  assess  the  pati ent  w i th  osteopeni a?

A  com pl ete  bl ood  count  w i th  er y thr ocy te  sedi m entati on  r ate  and  a
standar d  ser um   chem i str y   pr of i l e  that  i ncl udes  el ectr ol y te,   cal ci um ,
phosphate,   al k al i ne  phosphatase,   cr eati ni ne,   BU N ,   cal ci um ,   and  phosphate
m easur em ents  pl us  l i v er   f uncti on  tests  ar e  the  si m pl e  bl ood  tests  needed.
A  24­hour   ur i ne  speci m en  i s  obtai ned  f or   deter m i nati on  of   the  cal ci um ,
phosphate,   and  cr eati ni ne  content.   Bone  densi tom etr y   establ i shes  the
sev er i ty   of   bone  l oss.   Al l   these  l abor ator y   tests  can  be  used  to  qui ck l y
assess  the  pati ent  w i th  osteopeni a.   If   the  pati ent  has  anem i a  and  an
el ev ated  sedi m entati on  r ate,   the  cl i ni ci an  shoul d  consi der   the  possi bi l i ty
of   a  m ul ti pl e  m y el om a  and  hav e  ei ther   a  ser um   pr otei n  or   ur i ne  pr otei n
el ectr ophor esi s,   or   both,   per f or m ed.   Abnor m al i ti es  i n  cal ci um   bal ance  can
be  assessed  by   i denti f y i ng  hy pocal cem i c  or   hy per cal cem i c  di sor der s.
Abnor m al i ti es  of   l i v er   and  k i dney   f uncti on  r epr esent  secondar y   causes  of
osteopor osi s.   The  el ectr ol y te  l ev el s  hel p  suggest  the  pr esence  of   r enal
tubul ar   aci dosi s  sy ndr om es.   Al k al i ne  phosphatase  i s  a  m ar k er   of   bone
osteobl ast  f uncti on  and  i ts  m easur em ent  hel ps  i denti f y   those  pati ents
w i th  hi gh­tur nov er   osteopor osi s  or   osteom al aci a.   A  24­hour   ur i ne  cal ci um
deter m i nati on  can  i denti f y   pati ents  w ho  hav e  i di opathi c  hy per cal ci ur i a  or
l ow   ur i ne  cal ci um   l osses,   suggesti ng  a  cal ci um ­def i ci ent  state.   An
ex tr em el y   l ow   ur i ne  phosphate  v al ue  m ay   r ef l ect  the  consum pti on  of   a
v egetar i an  di et.   A  25­hy dr ox y   v i tam i n
P. 129
P. 130
D  l ev el   assesses  f or   v i tam i n  D  def i ci ency ,   a  condi ti on  now   r ecogni zed  to
be  m or e  com m on  than  pr ev i ousl y   appr eci ated.   Di agnosi s  i s  i m por tant,   as
r epl eti on  i s  essenti al   bef or e  bi sphosphonate  ther apy .   Other   l abor ator y
tests,   i ncl udi ng  m easur em ent  of   the  PTH  l ev el ,   ser um   osteocal ci n  l ev el ,
and  ur i ne  hy dr ox y pr ol i ne  or   hy dr ox y py r i di ni um ,   ar e  r eser v ed  f or   those
pati ents  i n  w hom   these  ar e  speci f i cal l y   i ndi cated.   A  bone  densi ty
m easur em ent  establ i shes  the  pr esence  or   absence  of   si gni f i cant
osteopor osi s.

Table 3­10 Causes of Osteopenia

Idi opathi c  age­r el ated


Juv eni l e
Young  adul ts
Postm enopausal   (ty pe  I)
Seni l e  (ty pe  II)
Secondar y   to  di sease  states
Metabol i c  condi ti ons
Cal ci um   def i ci ency
Vi tam i n  D  def i ci ency   states
Mal nutr i ti on
Idi opathi c  hy per cal ci ur i a
Renal   tubul ar   aci dosi s  and  other   sy stem i c
aci dosi s
Di abetes  m el l i tus
Scur v y
Endocr i ne  condi ti ons
Thy r otox i cosi s
Cushi ng's  sy ndr om e
Mal e  and  f em al e  hy pogonadal   state
Hy poam enor r hei c  f em al e  r unner s
Pr ol acti nom a
Hy per par athy r oi di sm
Renal   di sease
Gastr oi ntesti nal ­l i v er   di sease
Inher i Tabl e  connecti v e  ti ssue  di sease
Osteogenesi s  i m per f ecta
Hom ocy sti nur i a
Ehl er s­Danl os  sy ndr om e
Mar f an's  sy ndr om e
Bone  m ar r ow   i nf i l tr ati on
Mul ti pl e  m y el om a
Ly m phom a
Leuk em i a
Dr ugs
Di l anti n
Phenobar bi tal
Thy r oi d  hor m one
Cor ti coster oi ds
Pr ol onged  hepar i n  ther apy
Li f esty l e
N utr i ti on
Al cohol
Sm ok i ng
Inacti v i ty
Im m obi l i zati on
Ex cessi v e  cof f ee  and  sof t  dr i nk s
Mi scel l aneous
Rheum atoi d  ar thr i ti s
Sy stem i c  m astocy tosi s

6.   When  ar e  bone  densi ty   m easur em ents  i ndi cated?

The  N ati onal   Osteopor osi s  Foundati on  has  r ecom m ended  that  bone
m i ner al   densi ty   be  m easur ed  i n  al l   postm enopausal   w om en  y ounger   than
65  y ear s  w i th  one  or   m or e  r i sk   f actor s  f or   osteopor osi s  other   than
m enopause,   and  i n  al l   w om en  ol der   than  65.   For m al   r ecom m endati ons  f or
scr eeni ng  i n  m en  do  not  ex i st  at  thi s  ti m e,   but  scr eeni ng  shoul d  be
consi der ed  i n  m en  w i th  r i sk   f actor s  (especi al l y   hy pogonadi sm ,   ster oi d
use,   or   untr eated  hy per par athy r oi di sm )  or   af ter   a  f r actur e.

Case
A  thi n,   55­y ear ­ol d,   w hi te,   postm enopausal   w om an  i s  seen  i n  her   pr i m ar y   car e
cl i ni c  because  of   m uscl e  aches  and  w eak ness.   She  has  been  seen  by   num er ous
phy si ci ans  f or   ev al uati on  of   thi s  condi ti on,   and  has  been  r ef er r ed  to  the
psy chi atr y   depar tm ent  f or   tr eatm ent  of   a  “str ess  r eacti on. â€​   The  pati ent's
past  m edi cal   hi stor y   i s  si gni f i cant  f or   a  gastr ectom y   appr ox i m atel y   15  y ear s
ear l i er   f or   the  tr eatm ent  of   pepti c  ul cer   di sease.   She  has  noti ced  l oose  stool s
si nce  that  ti m e.   The  pati ent  adm i ts  to  a  poor   cal ci um   i ntak e,   but  other w i se
consum es  a  nonv egetar i an  di et.   She  suf f er s  hot  f l ashes  and  i nsom ni a,   but  has
nev er   been  ev al uated  f or   estr ogen  ther apy .   Dur i ng  her   ev al uati on,   osteopeni c
changes  ar e  noted  on  the  chest  f i l m .   The  pati ent's  l abor ator y   ev al uati on
r ev eal s  the  f ol l ow i ng  f i ndi ngs:  cal ci um ,   8. 4  m g/dL  (nor m al ,   8. 7  to  10. 3
m g/dL);  phosphate,   2. 0  m g/dL  (nor m al ,   2. 7  to  4. 5  m g/dL);  chl or i de,   108
m Eq/L;  sodi um ,   145  m Eq/L;  potassi um ,   4. 5  m Eq/L;  CO 2 ,   23  m Eq/L;  and
al bum i n,   4. 1  g/dL.   Her   k i dney   and  l i v er   f uncti on  test  r esul ts  ar e  nor m al .   The
al k al i ne  phosphatase  l ev el   i s  el ev ated  to  380  IU /L  (nor m al ,   39  to  117  IU /L).
Her   24­hour   ur i ne  ex cr eti on  of   cal ci um   i s  40  m g  (nor m al ,   100  to  300  m g);
cr eati ni ne,   1. 1  g;  total   hy dr ox y pr ol i ne,   86  m g  (nor m al ,   25  to  77  m g);  and
phosphate,   780  m g  (nor m al ,   400  to  800  m g).   The  osteocal ci n  l ev el   i s  7. 1
ng/m L  (nor m al ,   1. 8  to  6. 6  ng/m L).

1.   What  ar e  the  r i sk   f actor s  f or   osteopor osi s  i n  thi s  pati ent?


2.   On  the  basi s  of   the  pati ent's  hi stor y   and  l abor ator y   f i ndi ngs,   w hat  i s  the
di f f er enti al   di agnosi s?
3.   What  addi ti onal   l abor ator y   tests  shoul d  be  obtai ned  i n  thi s  pati ent?
4.   On  the  basi s  of   the  l abor ator y   f i ndi ngs,   w hat  w oul d  y ou  anti ci pate  the
bone  bi opsy   speci m en  to  show ?
5.   What  shoul d  the  tr eatm ent  be  i n  thi s  pati ent?
6.   What  w oul d  y ou  adv i se  thi s  pati ent  r egar di ng  the  adv antages  and
di sadv antages  of   estr ogen  r epl acem ent  ther apy ?

P. 131

Case Discussion
1.   What  ar e  the  r i sk   f actor s  f or   osteopor osi s  i n  thi s  pati ent?

Thi s  thi n,   w hi te,   postm enopausal   w om an  w i th  poor   cal ci um   i ntak e  i s  at
r i sk   f or   osteopor osi s.

2.   On  the  basi s  of   the  pati ent's  hi stor y   and  l abor ator y   f i ndi ngs,   w hat  i s  the
di f f er enti al   di agnosi s?

Thi s  pati ent's  hi stor y   suggests  that,   at  her   age  of   55  y ear s,   she  i s
enter i ng  a  postm enopausal   state,   as  i ndi cated  by   the  hot  f l ashes  and
i nsom ni a.   In  addi ti on,   poor   cal ci um   bal ance  m ay   be  l i k el y   because  of   her
l i f el ong  hi stor y   of   poor   cal ci um   i ntak e  and  the  gastr ectom y   f or   pepti c
ul cer   di sease,   w hi ch  coul d  l ead  to  poor   v i tam i n  D  absor pti on.   Conf i r m i ng
a  state  of   negati v e  cal ci um   bal ance  i s  the  pati ent's  hy pocal cem i a,   l ow
ur i ne  cal ci um   ex cr eti on,   and  el ectr ol y te  l ev el s,   al l   of   w hi ch  suggest  the
pr esence  of   secondar y   hy per par athy r oi di sm   w i th  hy per chl or em i a  and  l ow
ser um   phosphate  l ev el s.

3.   What  addi ti onal   l abor ator y   tests  shoul d  be  obtai ned  i n  thi s  pati ent?

The  pati ent  m ay   be  def i ci ent  i n  v i tam i n  D.   Measur i ng  the  25­
hy dr ox y v i tam i n  D  l ev el ,   w hi ch  i s  the  m ajor   ci r cul ati ng  f or m   of   v i tam i n  D,
can  establ i sh  the  di agnosi s  of   si m pl e  v i tam i n  D  def i ci ency .   Som e  pati ents
m ay   al so  hav e  a  def i ci ency   of   1, 25­di hy dr ox y v i tam i n  D,   par ti cul ar l y   ol der
pati ents  and  those  w i th  r enal   di sease.   A  PTH  v al ue  can  establ i sh  the
di agnosi s  of   secondar y   hy per par athy r oi di sm   due  to  a  cal ci um ­def i ci ent
state  stem m i ng  f r om   the  v i tam i n  D  def i ci ency .   Once  tr eatm ent  i s
i ni ti ated,   a  PTH  v al ue  that  r etur ns  to  nor m al   conf i r m s  a  state  of   nor m al
cal ci um   bal ance.

In  thi s  pati ent,   the  25­v i tam i n  D  l ev el   i s  10  ng/m L  (nor m al ,   16  to  74
ng/m L)  and  the  PTH  v al ue  i s  120  pg/m L  (nor m al ,   10  to  65  pg/m L).

4.   On  the  basi s  of   the  l abor ator y   f i ndi ngs,   w hat  w oul d  y ou  anti ci pate  the
bone  bi opsy   speci m en  to  show ?

A  tetr acy cl i ne­l abel ed  bone  bi opsy   i s  per f or m ed  by   hav i ng  the  pati ent
i ngest  250  m g  of   tetr acy cl i ne  f our   ti m es  a  day   f or   3  day s,   then  w i thhol d
the  tetr acy cl i ne  f or   10  day s,   and  then  tak e  tetr acy cl i ne  f or   another   3
day s.   These  tw o  tetr acy cl i ne  l abel s  deter m i ne  the  r ate  of   bone  f or m ati on.
Osteocl ast  counts  can  be  deter m i ned  f r om   bone  hi stom or phol ogi c
anal y si s,   and  the  am ount  of   tetr acy cl i ne  that  has  sur f aced  can  be
m easur ed  as  an  i ndi cati on  of   acti v e  bone  f or m ati on.   Thi s  pati ent  pr ov ed
to  hav e  a  hi gh­tur nov er   osteopor osi s  w i th  an  i ncr eased  tetr acy cl i ne
sur f ace  and  an  i ncr eased  osteocl ast  count,   as  bor ne  out  by   the  hi gh  PTH
l ev el .   In  addi ti on,   the  hi gh  osteocal ci n,   al k al i ne  phosphatase,   and  ur i nar y
hy dr ox y pr ol i ne  or   py r i di ni um   l ev el s  i ndi cate  a  state  of   hi gh  bone
tur nov er .   Ear l y   i n  v i tam i n  D  def i ci ency   (hy pov i tam i nosi s­D  I),   secondar y
hy per par athy r oi di sm   pr edom i nates,   l eadi ng  to  a  hi gh­tur nov er
osteopor osi s.   In  the  setti ng  of   sev er e  v i tam i n  D  def i ci ency ,   especi al l y
chi l dhood  r i ck ets  (hy pov i tam i nosi s­D  II  and  III),   a  l ow –bone­tur nov er
state  ex i sts  i n  w hi ch  ther e  i s  l i ttl e  tetr acy cl i ne  uptak e.

5.   What  shoul d  the  tr eatm ent  be  i n  thi s  pati ent?

Thi s  pati ent  has  a  com bi ned  di sor der   of   both  estr ogen  and  v i tam i n  D
def i ci ency   contr i buti ng  to  her   pr esum ed  osteopeni a.   Ther e  i s  no  doubt
that  she  w i l l   benef i t
P. 132
f r om   v i tam i n  D  r epl eti on  and  thi s  shoul d  be  i ni ti ated  i m m edi atel y .   She
w i l l   m ost  l i k el y   not  r espond  to  sm al l   doses  of   v i tam i n  D,   but  m ay   r equi r e
50, 000  uni ts  of   v i tam i n  D  (er gocal ci f er ol ),   gi v en  once  or   tw i ce  w eek l y ,   or
cal ci f edi ol   (Cal der ol ),   20  to  50  µg  dai l y ,   because  of   the  poor
gastr oi ntesti nal   absor pti on  of   v i tam i n  D  stem m i ng  f r om   her   gastr ectom y .
Bef or e  the  r el ease  of   the  r esul ts  of   the  Wom en's  Heal th  Ini ti ati v e  (WHI),
thi s  pati ent  w oul d  al so  hav e  been  tr eated  w i th  hor m one  r epl acem ent
ther apy   of   dai l y   estr ogen  w i th  ei ther   dai l y   or   cy cl i c  pr ogester one.   Si nce
the  WHI,   thi s  has  becom e  a  deci si on  that  r equi r es  car ef ul   consi der ati on  of
the  r i sk s  and  benef i ts  of   hor m one  ther apy   descr i bed  f ur ther   bel ow .
Another   ef f ecti v e  m eans  of   tr eati ng  osteopor osi s  i s  w i th  bi sphosphonates.
The  i ni ti al   ex per i ence  w i th  i ncr eased  bone  densi ty   usi ng  bi sphosphonates
w as  w i th  eti dr onate  (Di dr onel ).   How ev er ,   eti dr onate  w as  nev er   f or m al l y
appr ov ed  f or   osteopor osi s  and  has  been  super ceded  by   the  new er   or al
bi sphosphonates,   al endr onate  (Fosam ax ),   r i sedr onate  (Actonel ),   and
r ecentl y   i bandr onate  (Boni v a).   Al l   thr ee  ar e  av ai l abl e  as  or al
f or m ul ati ons  and  hav e  been  show n  to  i ncr ease  bone  m i ner al   densi ty   and
decr ease  v er tebr al   f r actur e  r i sk .   Al endr onate  and  r i sedr onate  can  be
adm i ni ster ed  dai l y   or   w eek l y ,   i bandr onate  dai l y   or   m onthl y .   Other
bi sphosphonates  (pam i dr onate,   eti dr onate,   and  zol edr onate)  ar e  appr ov ed
f or   bone  pr eser v ati on  and  ser um   cal ci um   r educti on  i n  m al i gnancy ,   but  not
f or   osteopor osi s.   Bi sphosphonates  shoul d  not  be  adm i ni ster ed  to  pati ents
w i th  v i tam i n  D  or   cal ci um   def i ci ency   bef or e  at  l east  par ti al   r epl eti on.

6.   What  w oul d  y ou  adv i se  thi s  pati ent  r egar di ng  the  adv antages  and
di sadv antages  of   estr ogen  r epl acem ent  ther apy ?

Estr ogens  ar e  ef f ecti v e  agents  f or   tr eati ng  osteopor osi s  by   stabi l i zi ng


bone  densi ty   and  pr ev enti ng  f r actur es.   How ev er ,   estr ogen  ther apy   al one
i n  a  pati ent  w i th  an  i ntact  uter us  i s  associ ated  i n  a  dose­dependent
m anner   w i th  an  i ncr eased  i nci dence  of   endom etr i al   cancer ;  that  can  be
abol i shed  by   the  addi ti on  of   10  to  14  day s  of   a  pr ogesti n  at  l east  thr ee  to
f our   ti m es  annual l y .   On  the  basi s  of   obser v ati onal   studi es,   hor m one
r epl acem ent  ther apy   w as  pr ev i ousl y   thought  to  hav e  addi ti onal
car di opr otecti v e  benef i ts.   How ev er ,   such  benef i ts  w er e  not  f ound  i n  the
l ar ge  r andom i zed  por ti on  of   the  WHI.   Thi s  study ,   i n  f act,   dem onstr ated
that  hor m one  r epl acem ent  ther apy   i s  not  w i thout  r i sk   and  m ay   i ncr ease
the  r i sk   f or   CV  di sease  and  str ok e,   as  w el l   as  f or   br east  cancer .   A  cav eat
to  thi s  concl usi on  i s  that  a  l ar ge  por ti on  of   the  w om en  i n  the  study   w er e
m any   y ear s  postm enopausal   and  m ay   hav e  r esponded  v er y   di f f er entl y   to
hor m one  ther apy   than  w om en  w ho  w er e  r ecentl y   estr ogen  suf f i ci ent.
How ev er ,   as  the  onl y   l ar ge  r andom i zed  contr ol l ed  study   to  date,   the  WHI
m ust  be  car ef ul l y   consi der ed  w hen  the  deci si on  to  star t  estr ogen  f or
osteopor osi s  i s  m ade.

As  a  r esul t  of   the  WHI,   estr ogen  ther apy   shoul d  now   be  consi der ed  as
tr eatm ent  f or   osteopor osi s  onl y   i n  w om en  w ho  ar e  sy m ptom ati cal l y
postm enopausal .   Ev en  i n  these  w om en  i t  i s  contr ai ndi cated  i n  pati ents
w ho  hav e:  (a)  a  per sonal   hi stor y   of   estr ogen­r el ated  neopl asi a  of   the
br east,   (b)  a  per sonal   or   str ong  f am i l y   hi stor y   of   br east  car ci nom a,   (c)  a
per sonal   hi stor y   of   thr om boem bol i c  di sease  or   k now n  v ascul ar   di sease,   or
(d)  si gni f i cant  CV  r i sk   f actor s,   especi al l y   tobacco  use,   obesi ty ,   or
hy per tensi on.   Estr ogen  ther apy   i s  r el ati v el y   contr ai ndi cated  i n  pati ents
P. 133
w i th  estr ogen­r el ated  headaches.   The  use  of   estr ogen  ther apy   m ay   al so
be  associ ated  w i th  an  i ncr eased  i nci dence  of   gal l stones.   A  m ar k ed
tr i gl y cer i de  el ev ati on  m ay   dev el op  i n  som e  pati ents  w hen  estr ogen
ther apy   i s  i ni ti ated;  hence,   l i pi d  l ev el s  need  to  be  check ed  w i thi n  4  to  8
w eek s  of   star ti ng  ther apy .   Pati ents  w ho  dev el op  adv er se  l i pi d
abnor m al i ti es  to  or al   estr ogens  m ay   do  better   w i th  tr ansder m al   estr adi ol
ther apy .

Suggested Readings
Ar m anento­Vi l l er eal   R,   Vi l l er eal   DT,   Av i ol i   LV,   et  al .   Estr ogen  status  and
her edi ty   ar e  m ajor   deter m i nants  of   pr em enopausal   bone  m ass.   J  Cl i n
Inv est  1992;90:2464.

Ber enson  JR,   Li pton  A.   Phar m acol ogy   and  cl i ni cal   ef f i cacy   of
bi sphosphonates.   Cur r   Opi n  Oncol   1998;10:566.

Ful f ar o  F,   Casucci o  A,   Ti cozzi   C,   et  al .   The  r ol e  of   bi sphosphonates  i n  the


tr eatm ent  of   pai nf ul   m etastati c  bone  di sease:  a  r ev i ew   of   phase  III  tr i al s.
Pai n  1998;78:157.

Hodsm an  BA,   Bauer   DC,   Dem pster   DW,   et  al .   Par athy r oi d  hor m one  and
ter i par ati de  f or   the  tr eatm ent  of   osteopor osi s:  a  r ev i ew   of   the  ev i dence
and  suggested  gui del i nes  f or   i ts  use.   Endocr   Rev   2005;26:688–703.

Hof el dt  FD.   Pr ox i m al   f em or al   f r actur es.   Cl i n  Or thop  1987;218:12.

Hol i ck   MF.   Vi tam i n  D:  the  under appr eci ated  D­l i ghtf ul   hor m one  that  i s
i m por tant  f or   sk el etal   and  cel l ul ar   heal th.   Cur r   Opi n  Endocr i nol   Di abetes
2002;9:87–98.

Jack son  RD,   Wactaw sk i ­Wende  J,   Lacr oi x   AZ,   et  al .   For   the  Wom en's  Heal th
Ini ti ati v e  Inv esti gator s.   Ef f ects  of   conjugated  equi ne  estr ogen  on  r i sk   of
f r actur es  and  BMD  i n  postm enopausal   w om en  w i th  hy ster ectom y :  r esul ts
f r om   the  w om en's  heal th  i ni ti ati v e  r andom i zed  tr i al .   J  Bone  Mi ner   Res
2006;21(6):817–828.

McCl ung  M,   Cl em m esen  B,   Dai f oti s  A,   et  al .   Al endr onate  pr ev ents


postm enopausal   bone  l oss  i n  w om en  w i thout  osteopor osi s:  a  doubl e­bl i nd,
r andom i zed,   contr ol l ed  tr i al :  Al endr onate  Osteopor osi s  Pr ev enti on  Study
Gr oup.   Ann  Inter n  Med  1998;128:253.

Ol szy nsk i   WP,   Shaw n  DK,   Adachi   JD,   et  al .   Osteopor osi s  i n  m en:
epi dem i ol ogy ,   di agnosi s,   pr ev enti on,   and  tr eatm ent.   Cl i n  Ther
2004;26(1):15–28.

Par f i tt  AM,   Rao  DS,   Stanci u  AR,   et  al .   Ir r ev er si bl e  bone  l oss  i n
osteom al aci a:  com par i son  of   r adi al   photon  absor pti om etr y   w i th  i l i ac  bone
hi stom or phom etr y   dur i ng  tr eatm ent.   J  Cl i n  Inv est  1985;76:2403.

Ri ggs  BL,   Mel ton  U .   The  pr ev enti on  and  tr eatm ent  of   osteopor osi s.   N   Engl   J
Med  1992;327:620.

Rosen  CJ.   Postm enopausal   osteopor osi s.   N   Engl   J  Med  2005;353(6):595â


€“603.

Rubi n  MR,   Bi l ezi k i an  JP.   N ew   anabol i c  ther api es  i n  osteopor osi s.   Endocr i nol
Metab  Cl i n  N   Am   2003;32(1):285–307.

Sr i v astav a  AK,   Vl i et  EL,   Lew i eck i   EM,   et  al .   Cl i ni cal   use  of   ser um   and  ur i ne
bone  m ar k er s  i n  the  m anagem ent  of   osteopor osi s.   Cur r   Med  Res  Opi n
2005;21(7):1015–1026.

Stei n  E,   Shane  E.   Secondar y   osteopor osi s.   Endocr i nol   Metab  Cl i n  N   Am


2003;32:115–134.

U del l   JA,   Fi scher   MA,   Br ook har t  MA,   et  al .   Ef f ect  of   the  w om en's  heal th
i ni ti ati v e  on  osteopor osi s  ther apy   and  ex pendi tur e  i n  m edi cai d.   J  Bone
Mi ner   Res  2006;21(5):765–771.

Wi m al aw ansa  SJ.   A  f our ­y ear   r andom i zed  contr ol l ed  tr i al   of   hor m one
r epl acem ent  and  bi sphosphonate,   al one  or   i n  com bi nati on,   i n  w om en  w i th
postm enopausal   osteopor osi s.   Am   J  Med  1998;104:219.

P. 134

Erectile Dysfunction
Case 1
A  65­y ear ­ol d  m an  pr esented  w i th  er ecti l e  dy sf uncti on—he  had  noted  gr adual
onset  of   di f f i cul ty   i n  achi ev i ng  and  m ai ntai ni ng  an  er ecti on  dur i ng  the  l ast  4
y ear s.   He  had  had  hy per tensi on  f or   10  y ear s  and  has  r ecentl y   been  tol d  that
hi s  bl ood  chol ester ol   l ev el   w as  hi gh.   Hi s  f am i l y   hi stor y   w as  posi ti v e  f or
cor onar y   ar ter y   di sease,   hy per tensi on,   and  hy per chol ester ol em i a.
The  pati ent's  m edi cati ons  i ncl uded  atenol ol ,   50  m g  tw i ce  a  day ;
hy dr ochl or othi azi de,   50  m g  per   day ;  and  aspi r i n,   325  m g  per   day .   He  had
sm ok ed  a  pack   of   ci gar ettes  a  day   f or   30  y ear s,   but  qui t  2  y ear s  ear l i er .   He
dr ank   thr ee  beer s  each  ni ght.
Phy si cal   ex am i nati on  show ed  a  BP  of   160/90  m m   Hg,   the  pr esence  of   ar cus
cor neae,   and  an  S 4   hear t  sound.   The  l i v er   and  testi cul ar   ex am i nati ons  w er e
nor m al ,   as  w er e  r ef l ex es.   The  pedal   pul ses  w er e  di m i ni shed.
Labor ator y   test  r esul ts  w er e  testoster one,   450  ng/dL  (nor m al ,   300  to  1, 000
ng/dL);  l i v er   enzy m es,   nor m al ;  total   chol ester ol ,   350  m g/dL;  tr i gl y cer i des,
300  m g/dL;  and  hi gh­densi ty   l i popr otei n  (HDL),   25  m g/dL.

Case Discussion
Fr om   the  hi stor y   al one,   i t  w oul d  be  ex pected  that  thi s  pati ent's  er ecti l e
dy sf uncti on  had  a  v ascul ar   cause  and  per haps  i atr ogeni c  ex acer bati on.
Cor onar y   ar ter y   di sease  i s  a  r i sk   f actor   f or   er ecti l e  dy sf uncti on,   and  r ecent
studi es  hav e  suggested  that  m er el y   hav i ng  a  hi stor y   of   hy per chol ester ol em i a
poi nts  to  an  under l y i ng  v ascul ar   eti ol ogy .   Hi s  l ong­standi ng  hy per tensi on  al so
suggests  v ascul ar   di sease.

Thi s  pati ent  i s  tak i ng  tw o  m edi cati ons  that  hav e  been  associ ated  w i th  er ecti l e
dy sf uncti on.   Am ong  the  cl asses  of   cur r entl y   used  anti hy per tensi v e  agents,   β­
bl ock er s  and  di ur eti cs  ar e  m ost  of ten  at  f aul t.   Of   the  di ur eti cs,
hy dr ochl or othi azi de  i s  m or e  of   a  pr obl em   than  f ur osem i de.

Sm ok i ng,   of   cour se,   i ncr eases  the  r i sk   of   v ascul ar   di sease.   Ex cessi v e  al cohol
i ntak e  i s  di r ectl y   tox i c  to  the  testi cl es  and  can  r esul t  i n  decr eased
testoster one  pr oducti on.   Al cohol   i s  al so  di r ectl y   tox i c  to  the  l i v er ,   and  the
r esul ti ng  l i v er   dy sf uncti on  can  cause  i m bal ance  i n  testoster one  and  estr adi ol
m etabol i sm ,   w hi ch  i s  of ten  associ ated  w i th  gy necom asti a.

The  pati ent's  BP  r eadi ng  i ndi cates  that  hi s  hy per tensi on  i s  i nadequatel y
contr ol l ed,   and  the  S 4   hear t  sound  i ndi cates  that  the  hy per tensi on  i s  l ong
standi ng  and  has  af f ected  hi s  hear t.   The  pr esence  of   ar cus  cor neae  si gni f i es
pr ol onged  hy per chol ester ol em i a.   Di m i ni shed  pedal   pul ses  of f er   f ur ther
ev i dence  f or   v ascul ar   di sease.

Hy pogonadi sm   cannot  be  r el i abl y   detected  by   cl i ni cal   assessm ent  al one;


hence,   m easur em ent  of   the  testoster one  l ev el   w as  i ndi cated.   Li v er   f uncti on
testi ng  w as  per f or m ed  i n  l i ght  of   the  hi stor y   of   si gni f i cant  al cohol   i ntak e.   The
l i pi d  panel   conf i r m ed  hy per chol ester ol em i a.

Ther e  hav e  been  m any   studi es  on  how   to  di sti ngui sh  betw een  psy chogeni c  and
v ascul ar   er ecti l e  dy sf uncti on—f or   ex am pl e,   by   m oni tor i ng  f or   noctur nal
er ecti ons.   N o  contr ol l ed  study   has  show n  that  the  m ethods  change  the
m anagem ent  str ategy ;  how ev er ,
P. 135
the  w or k up  can  be  l i m i ted  to  the  hi stor y ,   phy si cal   ex am i nati on,   and  som e
l abor ator y   testi ng  to  ex cl ude  other   tr eatabl e  causes  of   er ecti l e  dy sf uncti on.

In  thi s  pati ent,   atenol ol   and  hy dr ochl or othi azi de  w er e  r epl aced  w i th  enal apr i l .
The  pati ent  w as  counsel ed  on  di etar y   changes  that  w oul d  hel p  l ow er   hi s
chol ester ol   l ev el .   A  v acuum   pum p  dev i ce  w as  pr escr i bed  f or   the  er ecti l e
dy sf uncti on.

Tw o  m onths  l ater ,   the  pati ent's  BP  w as  nor m al .   He  r epor ted  successf ul
r esum pti on  of   sex ual   i nter cour se  usi ng  the  v acuum   pum p.

Ev al uati on  of   thi s  pati ent's  er ecti l e  dy sf uncti on  pr ov i ded  the  oppor tuni ty   to
addr ess  the  under l y i ng  hy per tensi on  and  hy per chol ester ol em i a.   Other w i se,   he
m i ght  not  hav e  pr esented  unti l   a  str ok e  or   hear t  attack   occur r ed.

Changi ng  anti hy per tensi v e  m edi cati on  i s  especi al l y   i m por tant  i f   the  i ni ti ati on
of   tr eatm ent  and  onset  of   er ecti l e  dy sf uncti on  coi nci de.   In  thi s  case,   a
m edi cati on  change  w as  f ur ther   justi f i ed  because  of   i nadequate  BP  contr ol .   ACE
i nhi bi tor s  do  not  appear   to  cause  er ecti l e  dy sf uncti on  and  cal ci um   channel
bl ock er s  r ar el y   do,   so  these  ar e  the  dr ugs  that  m ay   be  pr escr i bed  i f
m edi cati on  i s  i nter f er i ng  w i th  sex ual   f uncti oni ng.   U nf or tunatel y ,   a  change  i n
anti hy per tensi v e  m edi cati on  al one  i s  unl i k el y   to  r estor e  er ecti l e  f uncti on.

Cor r ecti on  of   thi s  pati ent's  bl ood  l i pi ds  i s  l ong  ov er due.   If   di etar y   changes  do
not  suf f i ci entl y   i m pr ov e  hi s  l i pi d  pr of i l e  w i thi n  a  f ew   m onths,   he  w i l l   be  a
candi date  f or   ther apy   w i th  an  HMG­CoA  r eductase  i nhi bi tor .

The  v acuum   pum p  dev i ce  can  tr eat  er ecti l e  dy sf uncti on  i n  a  case  l i k e  thi s.   The
v acuum   pum p  dev i ce  consi sts  of   a  Luci te  tube  and  pum p;  the  sucti on  pul l s
bl ood  i nto  the  peni s.   Once  an  er ecti on  has  been  pr oduced,   a  r ubber   r i ng  i s
pl aced  at  the  base  of   the  peni s  to  m ai ntai n  the  er ecti on.   The  v acuum   pum p  has
no  m ajor   si de  ef f ects,   i t  can  be  used  as  of ten  as  the  pati ent  w i shes,   i t  can  be
used  i n  al l   ty pes  of   er ecti l e  dy sf uncti on,   and  i t  has  the  hi ghest  success  r ateâ
€”i t  i s  ef f ecti v e  i n  90%  to  95%  of   cases.   Obv i ousl y ,   i t  i s  not  m eant  f or   a  m an
w ho  i s  not  i n  a  stabl e  r el ati onshi p,   l ar gel y   because  of   poor   pati ent  acceptance.
Ther e  has  been  som e  specul ati on  that  v acuum   pum p  dev i ces  m i ght  be
contr ai ndi cated  i n  pati ents  tak i ng  w ar f ar i n  because  of   the  potenti al   f or
ecchy m osi s  f r om   the  r i ng,   but  studi es  hav e  el i m i nated  that  concer n.

Case 2
A  52­y ear ­ol d  m an  w i th  di abetes  pr esented  w i th  er ecti l e  dy sf uncti on.   Hi s
puber tal   dev el opm ent  had  been  nor m al .   The  di abetes  had  been  di agnosed  15
y ear s  ear l i er .   At  the  ti m e  of   di agnosi s,   he  had  had  pr obl em s  w i th  i m potence
that  r esol v ed  as  the  hy per gl y cem i a  w as  br ought  under   contr ol .   Er ecti l e
dy sf uncti on  had  r etur ned  gr adual l y   dur i ng  the  l ast  2  y ear s.   He  r ar el y   had
m or ni ng  er ecti ons.   The  er ecti l e  dy sf uncti on  has  cr eated  str ess  i n  hi s
r el ati onshi p  w i th  hi s  w i f e.
The  pati ent  had  tak en  an  or al   hy pogl y cem i c  agent  f or   5  y ear s  af ter   di agnosi s
of   di abetes  and  had  been  on  i nsul i n  f or   the  l ast  10  y ear s.   He  had  di abeti c
com pl i cati ons,   i ncl udi ng  m i l d  r eti nopathy ,   pr otei nur i a,   and  m i l d  per i pher al
neur opathy .   Sy m ptom s  of   gastr opar esi s  had  dev el oped  dur i ng  the  l ast  6
m onths.
Hi s  cur r ent  i nsul i n  r egi m en  consi sted  of   30  uni ts  of   N PH  (neutr al   pr otam i ne
Hagedor n)  and  15  uni ts  of   r egul ar   i nsul i n  i n  the  m or ni ng,   and  10  uni ts  of   N PH
and  8  uni ts  of   r egul ar   i nsul i n  i n  the  ev eni ng.
Other   m edi cati ons  i ncl uded  l i si nopr i l   (15  m g  per   day )  and  si m v astati n  (10  m g
per   day ).   He  di d  not  sm ok e  or   dr i nk   ex cessi v e  am ounts  of   al cohol .
P. 136
N otew or thy   f i ndi ngs  on  the  phy si cal   ex am i nati on  i ncl uded  a  BP  r eadi ng  of
120/80  m m   Hg  w i thout  si gni f i cant  or thostasi s,   r eti nopathy ,   absence  of   an  S 4
hear t  sound,   and  sl i ghtl y   sof t  testes.   Sensati on  to  pi npr i ck   on  the  cal f   w as
decr eased.
Labor ator y   test  r esul ts  w er e  ser um   testoster one,   200  ng/dL;  total   chol ester ol ,
150  m g/dL;  tr i gl y cer i des,   250  m g/dL;  and  HDL,   35  m g/dL.   Gl y cosy l ated  Hgb
w as  10%  (nor m al ,   < 6. 5%).

Case Discussion
Di abetes  i s  one  of   the  m ost  com m on  causes  of   er ecti l e  dy sf uncti on.   A
com bi nati on  of   v ascul ar   and  neur ol ogi c  di sease  i s  usual l y   at  f aul t,   al though
hor m one  def i ci ency ,   m edi cati ons,   and  psy chogeni c  aspects  al so  m ay   be
i nv ol v ed.   Al l   f i v e  com ponents  m ay   be  pr esent  i n  a  si ngl e  pati ent.

Men  w i th  T2DM  of ten  hav e  acute  er ecti l e  dy sf uncti on  at  the  onset  of   the
di sease,   si m pl y   as  a  r esul t  of   sev er e  hy per gl y cem i a.   The  m echani sm   of
er ecti l e  dy sf uncti on  m ay   i ncl ude  hy pogonadotr opi c  hy pogonadi sm   as  w el l   as
m etabol i c  and  neur ol ogi c  dy sf uncti on  (caused  by   gl ucose  tox i ci ty )  i n  the
testes.   Vascul ar   f actor s  m ay   al so  be  i nv ol v ed  because  the  hy per gl y cem i a  i s
usual l y   associ ated  w i th  sev er e  hy per l i pi dem i a.   The  er ecti l e  dy sf uncti on
associ ated  w i th  new ­onset  di abetes  m ay   i m pr ov e  w hen  hy per gl y cem i a  i s
br ought  under   contr ol .

In  a  pati ent  w i th  l ong­standi ng  di abetes,   the  pr esence  of   other   end­or gan
com pl i cati ons  m ak es  i t  m or e  l i k el y   that  er ecti l e  dy sf uncti on  i s  due  to  di abetes.
In  thi s  pati ent,   cl i ni cal   assessm ent  suggests  a  str ong  neur ogeni c  com ponent;
the  di m i ni shed  sensati on  denotes  per i pher al   neur opathy   and  the  gastr opar esi s
i ndi cates  autonom i c  neur opathy   (al though  the  l ack   of   or thostasi s  suggests  that
the  neur opathy   i s  not  sev er e).   The  pr otei nur i a  suggests  a  v ascul ar   com ponent
and,   ev en  though  the  absence  of   an  S 4   ar gues  agai nst  that,   i t  shoul d  be
r em em ber ed  that  an  S 4   i s  not  al w ay s  pr esent  i n  di abeti c  pati ents  w i th
cor onar y   ar ter y   di sease.

Dr ug­i nduced  er ecti l e  dy sf uncti on  does  not  appear   to  be  an  i ssue  i n  thi s
pati ent  because  nei ther   the  ACE  i nhi bi tor   nor   the  HMG­CoA  r eductase  i nhi bi tor
causes  er ecti l e  dy sf uncti on.

The  testi cul ar   sof tness  suggests  a  m i nor   hor m onal   com ponent,   and  i ndeed  the
testoster one  l ev el   i s  sl i ghtl y   decr eased.   A  l ow ­nor m al   or   sl i ghtl y   l ow
testoster one  l ev el   i s  a  ty pi cal   f i ndi ng  i n  di abeti c  pati ents  w i th  er ecti l e
dy sf uncti on.   Al though  the  r eadi ng  conf i r m s  that  hor m one  def i ci ency   i s  one  of
hi s  pr obl em s,   a  testoster one  l ev el   of   200  ng/dL  w oul d  not  by   i tsel f   cause
si gni f i cant  hy pogonadi sm   and  sy m ptom s.

In  addi ti on,   thi s  pati ent's  BP  needs  to  be  m oni tor ed;  i f   i t  i ncr eases,   he  w i l l
need  an  addi ti onal   anti hy per tensi v e  m edi cati on  because  he  i s  al r eady   tak i ng  a
m ax i m al   dose  of   l i si nopr i l .   Incr easi ng  data  suggest  that  ti ght  contr ol   of   BP
w i th  ACE  i nhi bi tor s  hel ps  pr ev ent  both  the  r enal   and  the  v ascul ar   com pl i cati ons
of   di abetes.   Consequentl y ,   aggr essi v e  anti hy per tensi v e  ther apy   to  l ow er   BP  to
l ess  than  130/85  m m   Hg  i s  i ndi cated.

The  pati ent's  LDL  l ev el   i s  l ow ,   but  the  tr i gl y cer i de  l ev el   i s  not  opti m al .   If
changes  i n  hi s  di et  do  not  r educe  the  tr i gl y cer i de  l ev el ,   he  w i l l   be  a  candi date
f or   tr eatm ent  w i th  gem f i br ozi l   or   a  stati n.
P. 137
The  pati ent  w as  m anaged  w i th  i ntr acav er nosal   i njecti on  of   al pr ostadi l   and
andr ogen  r epl acem ent  w i th  a  l ow ­dose  testoster one  patch.   He  r epor ted
i m pr ov ed  er ecti l e  f uncti on,   i ncr eased  ener gy ,   and  a  sense  of   w el l ­bei ng.   In
addi ti on,   the  pati ent  r ecei v ed  di etar y   counsel i ng  and  the  i nsul i n  r egi m en  w as
adjusted.   The  gl y cosy l ated  Hgb  decr eased  to  8%.

In  di abeti c  pati ents  w i th  er ecti l e  dy sf uncti on,   i njecti on  of   al pr ostadi l   i nto  the
cor por a  cav er nosa  of   the  peni s  can  be  ef f ecti v e.   The  tr eatm ent  i s  par ti cul ar l y
sui ted  to  di abeti c  pati ents  because  they   of ten  hav e  neur ol ogi c  com pl i cati ons,
m ak i ng  the  i njecti ons  l ess  pai nf ul   than  i n  other   pati ents.   In  addi ti on,   those
w ho  ar e  tak i ng  i nsul i n  ar e  al r eady   f am i l i ar   w i th  needl es  and  sy r i nges  and  ar e
l ess  l i k el y   to  be  squeam i sh  about  i njecti ng  the  peni s.   Intr acav er nosal   i njecti on
i s  ef f ecti v e  i n  appr ox i m atel y   65%  of   cases.   The  v acuum   pum p  i s  hel d  i n
r eser v e  as  second­l i ne  tr eatm ent.

Im pl antabl e  peni l e  pr ostheses  w er e  com m onl y   used  to  tr eat  er ecti l e


dy sf uncti on  i n  the  1970s  and  1980s.   They   ar e  used  m uch  l ess  f r equentl y   today
because  they   ar e  ex pensi v e  and  m ay   hav e  m any   com pl i cati ons.   Inf ecti on  and
poor   w ound  heal i ng  ar e  par ti cul ar   pr obl em s  i n  di abeti c  pati ents,   of ten
necessi tati ng  r em ov al   of   the  i m pl ant—at  w hi ch  poi nt  the  opti on  f or   i njecti on
ther apy   has  been  el i m i nated.   How ev er ,   som e  of   the  new er   i m pl ants  m ay   be
appr opr i ate  f or   y oung  m en  w i th  sev er e  er ecti l e  dy sf uncti on  that  does  not
r espond  to  other   ther apy .

Intr aur ethr al   pl acem ent  of   v asoacti v e  m edi cati on  w as  i ntr oduced  as  an
al ter nati v e  to  i ntr acav er nosal   i njecti ons.   How ev er ,   sev er al   studi es  hav e  show n
i t  to  be  l ess  ef f ecti v e,   w i th  a  success  r ate  as  l ow   as  30%  i n  di abeti c  pati ents.

Si de  ef f ects  of   i ntr acav er nosal   i njecti on  i ncl ude  pr i api sm   and  peni l e  f i br osi s.
Pati ents  w i th  neur ogeni c  or   psy chogeni c  er ecti l e  dy sf uncti on  shoul d  use  a  l ow
dose  of   al pr ostadi l .   If   the  dose  i s  too  hi gh,   the  r i sk   of   pr i api sm   i s  si gni f i cant.
When  pr i api sm   occur s,   the  pati ent  has  to  go  to  an  em er gency   r oom ,   w her e  he
i s  tr eated  w i th  IV  epi nephr i ne  or   an  18­gauge  needl e  that  i s  i nser ted  i nto  the
cor por a  cav er nosa  to  w i thdr aw   bl ood.   Ther e  hav e  been  onl y   r ar e  r epor ts  of
m or e  sev er e  consequences,   such  as  l oss  of   the  peni s  due  to  i nf ar cti on.

The  r ate  of   pr i api sm   as  a  com pl i cati on  v ar i es  accor di ng  to  the  agent  used.
Al pr ostadi l   has  a  m uch  l ow er   r i sk   of   pr i api sm   and  f i br osi s  than  do  papav er i ne
and  phentol am i ne.   How ev er ,   al pr ostadi l   i s  m or e  l i k el y   to  cause  a  bur ni ng
sensati on.   For   that  r eason,   i t  used  to  be  m i x ed  w i th  papav er i ne,   but
papav er i ne  has  been  w i thdr aw n  as  a  tr eatm ent  f or   outpati ents.   Because  of
neur opathy ,   di abeti c  pati ents  m ay   not  ex per i ence  a  bur ni ng  sensati on  w i th
al pr ostadi l .

It  i s  not  cl ear   w hether   better   contr ol   of   thi s  pati ent's  di abetes  dur i ng  the
pr ev i ous  10  y ear s  w oul d  hav e  pr ev ented  er ecti l e  dy sf uncti on.   It  seem s  l ogi cal
that  ti ght  contr ol   of   bl ood  gl ucose  l ev el s  w i l l   f or estal l   er ecti l e  dy sf uncti on,   just
as  i t  can  pr ev ent  r eti nopathy ,   r enal   f ai l ur e,   and  m acr ov ascul ar   di sease.
N ev er thel ess,   ther e  ar e  no  pr ospecti v e,   doubl e­bl i nded,   pl acebo­contr ol l ed
studi es  to  conf i r m   that  l ong­ter m   ti ght  bl ood  gl ucose  contr ol   r educes  the
i nci dence  of   er ecti l e  dy sf uncti on.
Case 3
A  48­y ear ­ol d  m an  had  ex per i enced  acute  onset  of   er ecti l e  dy sf uncti on  6
m onths  ear l i er .   He  had  no  other   m edi cal   pr obl em s.   Puber tal   dev el opm ent  had
been  nor m al .   He  w as  the  f ather   of   thr ee  chi l dr en.
P. 138
On  f ur ther   questi oni ng,   the  pati ent  sai d  that  he  had  l ost  hi s  job  4  m onths  ago.
He  w as  hav i ng  pr obl em s  i n  hi s  r el ati onshi p  w i th  hi s  w i f e,   and  had  i ncr eased
hi s  al cohol   consum pti on  f r om   tw o  beer s  a  w eek   to  f our   beer s  a  day .
The  phy si cal   ex am i nati on  w as  nor m al .   The  ser um   testoster one  l ev el   w as  450
ng/dL.
The  pati ent  w as  adv i sed  that  hi s  dr i nk i ng  w as  pr obabl y   contr i buti ng  to  hi s
er ecti l e  dy sf uncti on  and  that  he  shoul d  r educe  hi s  i ntak e.   Ref er r al   f or
psy chol ogi cal   counsel i ng  w as  of f er ed,   but  he  r ef used  because  of   the  cost.
Instead,   the  phy si ci an  di scussed  the  pati ent's  ci r cum stances  w i th  hi m .   A  6­
w eek   tr i al   of   y ohi m bi ne,   5. 4  m g  thr i ce  a  day ,   w as  pr escr i bed.
The  pati ent  r etur ned  8  w eek s  l ater   and  r epor ted  som e  i m pr ov em ent  i n  er ecti l e
f uncti on.
He  f el t  that  y ohi m bi ne  had  been  hel pf ul ;  how ev er ,   he  had  al so  f ound  a  job,
w as  ex per i enci ng  l ess  psy chol ogi cal   str ess,   and  had  r educed  hi s  al cohol
consum pti on.

1.   What  w as  the  m ajor   f actor   i n  thi s  pati ent's  er ecti l e  dy sf uncti on?
2.   How   do  y ou  appr oach  psy chol ogi cal   er ecti l e  dy sf uncti on?
3.   What  ar e  the  phar m acol ogi c  opti ons  f or   tr eatm ent?

Case Discussion
1.   What  w as  the  m ajor   f actor   i n  thi s  pati ent's  er ecti l e  dy sf uncti on?

Al though  the  hi stor y   i n  thi s  case  i ndi cated  that  psy chol ogi cal   str ess  w as
the  m ajor   tr i gger   f or   the  er ecti l e  dy sf uncti on,   i t  w as  i m por tant  to
consi der   the  possi bi l i ty   of   other   com ponents.   As  noted,   er ecti l e
dy sf uncti on  r ar el y   r esul ts  f r om   an  i sol ated  cause.   In  thi s  case,   f ur ther
questi oni ng  w as  needed  to  r ev eal   that  al cohol   w as  al m ost  cer tai nl y   a
m ajor   contr i butor .

Obtai ni ng  an  accur ate  hi stor y   of   al cohol   i ntak e  i s  notor i ousl y   di f f i cul t.
Instead  of   ask i ng  the  pati ent,   “Do  y ou  dr i nk ?â€​   ask ,   “When  y ou
dr i nk ,   do  y ou  dr i nk   beer ,   w hi sk ey ,   or   w i ne?â€​   Af ter   i denti f y i ng  the  dr i nk
of   choi ce,   pi ck   a  l ar ge  am ount  and  l et  pati ents  com e  dow n  f r om   ther e;
w i th  beer ,   f or   ex am pl e,   ask   i f   they   dr i nk   a  si x ­pack   at  a  ti m e.
Deter m i ni ng  the  tr ue  am ount  of   al cohol   i ntak e  of ten  r equi r es  sev er al
di scussi ons.

Al so  ask   pati ents  w hen  they   dr i nk ,   because  they   m ay   not  under stand  that
i nter m i ttent  dr i nk i ng  can  hav e  per si stent  ef f ects.   Som e  pati ents  w ho
dr ank   heav i l y   on  the  w eek end  and  nothi ng  at  al l   dur i ng  the  w eek   m ay
pr esent  w i th  er ecti l e  dy sf uncti on  and  pai nf ul   r i ght­si ded  gy necom asti a
(w hi ch  w as  w or se  on  Monday s).   Thei r   l i v er   enzy m es  w er e  not  sev er el y
el ev ated,   but  the  dr i nk i ng  had  nev er thel ess  caused  a  sy m ptom ati c
i m bal ance  of   testoster one  and  estr adi ol .

In  pati ents  w i th  a  hi stor y   of   chr oni c  al cohol   abuse,   l i v er   f uncti on  tests
shoul d  be  or der ed.   Thei r   er ecti l e  f uncti on  m ay   not  r etur n  ev en  i f   they
r educe  thei r   al cohol   i ntak e.   Because  thi s  pati ent's  i ncr ease  i n  al cohol
i ntak e  w as  f ai r l y   acute,   hi s  er ecti l e  f uncti on  i m pr ov ed  as  soon  as  he
began  to  dr i nk   l ess.

2.   How   do  y ou  appr oach  psy chol ogi cal   er ecti l e  dy sf uncti on?

Despi te  the  ubi qui ty   of   the  psy chol ogi cal   com ponent  i n  er ecti l e
dy sf uncti on,   ther e  hav e  been  no  contr ol l ed  studi es  to  show   w hether
psy chother apy   or   counsel i ng  actual l y   hel ps.   Ev en  assum i ng  that  such
i nter v enti on  w oul d  be  hel pf ul ,   ther e  ar e  no
P. 139
data  on  the  best  appr oach.   Shoul d  pati ents  r ecei v e  behav i or al   ther apy ?
Counsel i ng?  Is  si m pl y   tal k i ng  w i th  the  pr i m ar y   car e  phy si ci an  suf f i ci ent?â
€”consi der ati ons  such  as  these  can  hel p  i n  appr oachi ng  psy chol ogi cal
er ecti l e  dy sf uncti on,   but  ther e  ar e  no  cl ear   answ er s.

The  pr i m ar y   car e  phy si ci an  shoul d  at  l east  ack now l edge  psy chol ogi cal
str ess  as  a  com ponent  of   er ecti l e  dy sf uncti on.   Som eti m es  ack now l edgi ng
the  pr obl em   i s  enough;  the  pati ent  just  needs  to  tal k   about  i t.   Som eti m es
f ur ther   i nter v enti on  i s  r equi r ed.   Whether   thi s  i s  pr ov i ded  by   the  pr i m ar y
car e  phy si ci an  depends  on  hi s  or   her   l ev el   of   com f or t  w i th  that  aspect  of
tr eatm ent.   Insur ance  cov er age  i s  of ten  an  i m por tant  f actor   as  w el l .

3.   What  ar e  the  phar m acol ogi c  opti ons  f or   tr eatm ent?

The  opti ons  f or   tr eatm ent  of   er ecti l e  dy sf uncti on  hav e  r adi cal l y   changed
w i th  the  i ntr oducti on  of   si l denaf i l   (Vi agr a),   the  f i r st  tr ul y   ef f ecti v e  or al
m edi cati on  f or   thi s  condi ti on,   and  m or e  r ecentl y   appr ov ed  r el ated
m edi cati ons,   v ar denaf i l   (Lev i tr a)  and  tadal af i l   (Ci al i s).

Adv ances  i n  our   k now l edge  of   the  phy si ol ogy   of   er ecti on  hav e  f aci l i tated
under standi ng  of   the  phar m acody nam i cs  of   si l denaf i l .   Er ecti on  i s  i ni ti ated
by   di l ati on  of   the  ar ter i al   bed,   w hi ch  i ncr eases  bl ood  f l ow   and  pr essur e;
i t  i s  m ai ntai ned  by   r estr i cti on  of   v enous  outf l ow .   Pr ev i ousl y   i t  w as
bel i ev ed  that  the  par asy m patheti c  sy stem   w as  cr i ti cal   i n  m ai ntai ni ng
er ecti on.   N ow ,   w e  k now   that  the  m ajor   pl ay er   i s  the  nonadr ener gi c,
nonchol i ner gi c  (N A­N C)  sy stem ,   w hi ch  w as  i denti f i ed  50  y ear s  ago  but
nev er   studi ed  i n  detai l   unti l   r el ati v el y   r ecentl y .   The  N A­N C  sy stem   uses
ni tr i c  ox i de  as  a  neur otr ansm i tter .   Thr ough  i ts  second  m essenger ,   cy cl i c
guani ne  m onophosphate  (cGMP),   ni tr i c  ox i de  tr i gger s  r el ax ati on  of   peni l e
endothel i um   and  sm ooth  m uscl e,   al l ow i ng  ex pansi on  of   the  l acunar   spaces
w i thi n  the  cor por a  and  the  tr appi ng  of   bl ood  by   com pr essi on  of   per i pher al
dr ai ni ng  v enul es.

Si l denaf i l ,   a  ty pe  5  phosphodi ester ase  i nhi bi tor ,   pr ev ents  the  br eak dow n
of   cGMP,   ther eby   pr ol ongi ng  er ecti on.   It  has  no  ef f ect  on  l i bi do  and  does
not  cause  er ecti on  w i thout  sti m ul ati on,   but  i t  m ai ntai ns  an  er ecti on  once
i t  has  been  achi ev ed.   Al though  the  N A­N C  sy stem   i s  par ti cul ar l y
pr om i nent  i n  the  peni s,   i t  i s  al so  f ound  i n  the  hear t,   the  br ai n,   and  other
or gans.   Its  pr esence  i n  the  ey e  ex pl ai ns  the  bl ue  v i sual   hue  that  som e
pati ents  ex per i ence  af ter   tak i ng  si l denaf i l .

The  m ost  com m on  si de  ef f ects  of   si l denaf i l   ar e  headache,   f l ushi ng,   and
dy spepsi a.   It  can  al so  decr ease  BP.   Because  the  decr ease  i n  BP  m ay   be
sy ner gi sti c  w i th  the  hy potensi v e  acti on  of   ni tr ates,   si l denaf i l   i s
contr ai ndi cated  i n  pati ents  tak i ng  a  m edi cati on  that  contai ns  ni tr ates,
such  as  ni tr ogl y cer i n.

In  addi ti on,   si l denaf i l   al ter s  the  hal f ­l i f e  of   m any   other   m edi cati ons,   and
m any   m edi cati ons  change  the  hal f ­l i f e  of   si l denaf i l .   The  l i st  of   agents
that  can  i nter act  w i th  si l denaf i l   i ncl udes  such  com m on  m edi cati ons  as
nonsel ecti v e  β­bl ock er s,   er y thr om y ci n,   i tr aconazol e,   potassi um ­spar i ng
di ur eti cs,   and  ci m eti di ne.   It  i s  not  k now n  w hether   those  i nter acti ons
af f ect  the  si de  ef f ects  of   si l denaf i l ,   par ti cul ar l y   the  i nci dence  or   sev er i ty
of   hy potensi on.   In  i ni ti al   cl i ni cal   tr i al s,   hy potensi on  w as  r epor ted  i n
appr ox i m atel y   3%  of   pati ents,   but  those  tr i al s  i ncl uded  a  l ar ge
per centage  of   y oung  m en  w i th  psy chogeni c  i m potence.   Obv i ousl y ,   pati ents
w i th  v ascul ar   di sease  or   di abetes  hav e  m or e  pr obl em s  w i th  BP  r egul ati on
and  theor eti cal l y   w i th  or thostati c
P. 140
hy potensi on.   Deaths  hav e  been  r epor ted  am ong  pati ents  tak i ng  si l denaf i l
si nce  i t  becam e  av ai l abl e.   The  FDA  i s  i nv esti gati ng  those  deaths.

The  f i r st  study   of   data  on  di f f er ent  pati ent  popul ati ons  tak i ng  si l denaf i l
w as  publ i shed  sev er al   m onths  af ter   the  dr ug  becam e  av ai l abl e  f or   cl i ni cal
use.   Al though  the  pack age  i nser t  i ndi cated  an  ov er al l   ef f i cacy   of   82%  (v s.
24%  f or   pl acebo),   anal y si s  si nce  has  f ound  m or e  m odest  ef f i cacy   of   68%
i n  pati ents  w i th  hy per tensi on,   57%  i n  di abetes,   and  61%  af ter
tr ansur ethr al   pr ostatectom y .   Mor eov er ,   publ i shed  r esul ts  ar e  f r equentl y
obtai ned  i n  a  sel ected  pati ent  popul ati on,   not  f r om   gener al   cl i ni cal   use.
Editors :  Sc hrie r,  Robe rt W .
Title :  Inte rna l Me dic ine  Ca s e book ,  The : Re a l P a tie nts ,  Re a l Ans w e rs ,
3rd Edition
Copy r i ght  ©2007  Li ppi ncott  Wi l l i am s  &  Wi l k i ns

>  T a b le   o f   C o nte nts   >  C ha p te r   4  ­  G a s tr o e nte r o lo g y

Chapter 4
Gastroenterology

W illia m R.  Brow n

Chronic Inflammatory Bowel Disease
1.   What  i s  the  pathogenesi s  r esponsi bl e  f or   chr oni c  ul cer ati v e  col i ti s
(CU C)  and  Cr ohn's  di sease?

2.   Com par e  and  contr ast  the  pr i nci pal   cl i ni cal   f eatur es  of   CU C  and  Cr ohn's
di sease.

3.   What  ar e  the  r especti v e  r i sk s  of   i ntesti nal   m al i gnancy   i n  CU C  and


Cr ohn's  di sease?

4.   What  ar e  the  pr i nci pal   m edi cal   ther apeuti c  m easur es  used  f or   pati ents
w i th  CU C  and  Cr ohn's  di sease?

P. 142

Discussion
1.   What  i s  the  pathogenesi s  r esponsi bl e  f or   CU C  and  Cr ohn's  di sease?

The  cause  and  pathogenesi s  of   both  these  chr oni c  i nf l am m ator y   bow el
di seases  (CIBDs)  ar e  unk now n.   Both  ar e  char acter i zed  by   a  chr oni c
i nf l am m ator y   cel l   i nf i l tr ate  of   the  bow el .   How ev er ,   w her eas  CU C  i s
r estr i cted  to  the  col on,   Cr ohn's  di sease  can  i nv ol v e  the  enti r e
al i m entar y   tr act  f r om   the  m outh  to  the  anus,   al though  the  di stal   i l eum
and  col on  ar e  the  por ti ons  m ost  f r equentl y   af f ected.   Another
di sti ngui shi ng  f eatur e  of   Cr ohn's  di sease  i s  the  i nv ol v em ent  of   al l
l ay er s  of   the  bow el ,   w her eas  the  i nf l am m ati on  seen  i n  CU C  i s  m ostl y
l i m i ted  to  the  m ucosa.   In  addi ti on,   f ocal   gr anul om as  ar e  com m on  i n
Cr ohn's  di sease  but  r ar e  i n  CU C.   How ev er ,   nei ther   di sease  has
pathognom oni c  f eatur es,   and  Cr ohn's  di sease  of   the  col on  cannot  be
hi stol ogi cal l y   di sti ngui shed  f r om   CU C  i n  15%  to  25%  of   cases  of
chr oni c  col i ti s.

2.   Com par e  and  contr ast  the  pr i nci pal   cl i ni cal   f eatur es  of   CU C  and  Cr ohn's
di sease.
The  sev er i ty ,   cl i ni cal   cour se,   and  pr ognosi s  of   CU C  and  Cr ohn's  di sease
ar e  w i del y   v ar i abl e.   Onset  i n  both  di seases  occur s  m ost  of ten  i n  ear l y
adul thood.   The  sy m ptom s  of   CU C  m ay   r ange  f r om   sl i ght  r ectal   bl eedi ng
to  f ul m i nant  di ar r hea  w i th  col oni c  hem or r hage  and  hy potensi on.   Most
pati ents  hav e  i nter m i ttent  attack s,   al though  som e  can  hav e  conti nuous
sy m ptom s  w i thout  r em i ssi on.   The  cl i ni cal   f eatur es  of   Cr ohn's  di sease
depend  on  the  sev er i ty   and  l ocati on  of   the  bow el   i nv ol v em ent;  the
pr i nci pal   f eatur es  ar e  di ar r hea,   abdom i nal   pai n,   hem atochezi a,
i ntesti nal   obstr ucti on,   f i ssur es,   and  f i stul as.

Ex tr ai ntesti nal   m ani f estati ons  ar e  com m on  i n  both  Cr ohn's  di sease  and
CU C,   but  m or e  com m on  i n  CU C.   The  m ani f estati ons  i ncl ude  ar thr i ti s,
ar thr al gi a,   i r i ti s,   uv ei ti s,   l i v er   di sease,   and  sk i n  l esi ons.   The  ar thr i ti s
m ay   pr esent  as  a  m i gr ator y   ar thr i ti s,   i nv ol v i ng  l ar ge  joi nts,   sacr oi l i i ti s,
or   ank y l osi ng  spondy l i ti s.   Pr i m ar y   scl er osi ng  chol angi ti s,   w hi ch  i s
associ ated  w i th  an  i ncr eased  f r equency   of   chol angi ocar ci nom a,   and
chr oni c  hepati ti s  ar e  com m on  hepatobi l i ar y   abnor m al i ti es.

The  pr i nci pal   f eatur es  that  di f f er enti ate  Cr ohn's  di sease  f r om   CU C  ar e
l i sted  i n  Tabl e  4. 1.

3.   What  ar e  the  r especti v e  r i sk s  of   i ntesti nal   m al i gnancy   i n  CU C  and


Cr ohn's  di sease?

The  f r equency   of   i ntesti nal   cancer   i s  i ncr eased  i n  Cr ohn's  di sease,   but
not  to  the  ex tent  i n  CU C.   Accor di ng  to  som e  r epor ts,   the  f r equency   of
col on  cancer   i n  adul ts  w ho  hav e  CU C  i nv ol v i ng  the  enti r e  col on  i s
appr ox i m atel y   25  ti m es  gr eater   than  that  i n  the  gener al   popul ati on.   The
r i sk   of   col on  cancer   dev el opi ng  i n  pati ents  w i th  CU C  i s  posi ti v el y
cor r el ated  w i th  the  ex tent  and  dur ati on  of   the  di sease.

4.   What  ar e  the  pr i nci pal   m edi cal   ther apeuti c  m easur es  used  f or   pati ents
w i th  CU C  and  Cr ohn's  di sease?

The  ge ne ra l me a s ure s   to  contr ol   the  sy m ptom s  of   both  di seases


i ncl ude  cor r ecti on  of   f l ui d–el ectr ol y te  i m bal ances;  i r on,   f ol ate,   or
v i tam i n  B 1 2   suppl em entati on  as  needed  f or   the  tr eatm ent  of   anem i a;
and  di etar y   adjustm ents  ai m ed  at  m ai ntai ni ng  adequate  nutr i ti on.   Tota l
pa re nte ra l nutrition  m ay   be
P. 143
r equi r ed  f or   the  shor t­ter m   tr eatm ent  of   sev er e  acute  di sease,   but  â
€œbow el   r estâ€​   and  hy per al i m entati on  ar e  of   dubi ous  v al ue  i n  the  l ong
ter m .   Antidia rrhe a l a ge nts   such  as  l oper am i de  ar e  usual l y
contr ai ndi cated  i n  pati ents  w i th  CU C  because  they   m ay   contr i bute  to
the  dev el opm ent  of   tox i c  m egacol on,   but  they   m ay   hel p  al l ev i ate  the
di ar r hea  and  abdom i nal   cr am ps  i n  the  setti ng  of   stabl e  Cr ohn's  di sease.

Table 4­1 Features that Distinguish between
Crohn's Disease and Ulcerative Colitis
Crohn's Disease and Ulcerative Colitis

Fa c tors Crohn's  Dis e a s e U lc e ra tive  Colitis

Pathol ogi c Tr ansm ur al Mucosal


f eatur es i nf l am m ati on i nf l am m ati on

  Deep  ul cer s Super f i ci al   ul cer s

Gr anul om as Gr anul om as  absent


 
com m on

Di str i buti on Mouth  to  anus Col on


(i l eum   and
pr ox i m al   col on
m ost  com m on)

Cl i ni cal
   
f eatur es

Rectal   bl eedi ng 20%–40% 98%

Ful m i nati ng U ncom m on Com m on


epi sodes

Obstr ucti on Com m on Rar e

Fi stul as Com m on Rar e

Per i anal Com m on Less  com m on


di sease

Si gm oi doscopi c
   
and

r adi ogr aphi c


   
f i ndi ngs
Rectal 50% 95%–100%
i nv ol v em ent

Ex tent Patchy Conti nuous

U l cer s Longi tudi nal ,   deep Shal l ow ,   col l ar


button

Pseudopol y ps U ncom m on Com m on

Str i ctur es Com m on U ncom m on

Il eal N ar r ow ed  l um en Di l ated  l um en  w i th


i nv ol v em ent w i th  thi ck ened di m i ni shed  f ol ds  but
w al l hi stol ogi cal l y   nor m al

Fr om   Schaef er   J,   Mal l or y   A.   Gastr oi ntesti nal   di sease.   In:


Schr i er   RW,   ed.   Medi ci ne:  di agnosi s  and  tr eatm ent.   Boston:
Li ttl e,   Br ow n  and  Com pany ,   1988.

In  CU C,   c ortic os te roids   ar e  usef ul   f or   i nduci ng  r em i ssi ons  or


i m pr ov em ent  i n  an  acute  attack ,   and  they   m ay   be  r equi r ed  f or   l ong­
ter m   m anagem ent.   How ev er ,   the  possi bl e  benef i ts  of   cor ti coster oi ds  i n
the  l ong  ter m   ar e  of f set  by   thei r   m any   adv er se  si de  ef f ects.   The  r ectal
adm i ni str ati on  of   ster oi ds  or   m esal am i ne  can  be  benef i ci al ,   especi al l y
w hen  r ectal   i nv ol v em ent  (pr octi ti s)  i s  sev er e.   How ev er ,   si gni f i cant
absor pti on  of   r ectal   ster oi ds  can  occur ,   so  sy stem i c  ef f ects  of   the
agents  (both  benef i ci al   and  undesi r abl e)  m ay   ar i se  w hen  they   ar e  gi v en
by   thi s  r oute.   Sulfa s a la zine   i s  m etabol i zed  by   col oni c  bacter i a,
r el easi ng  sul f apy r i di ne  and  5­am i nosal i cy l ate  (5­ASA);  the  l atter   i s
bel i ev ed  to  be  the
P. 144
acti v e  com pound.   Sul f apy r i di ne  i s  absor bed  sy stem i cal l y ,   w hi ch
accounts  f or   the  si de  ef f ects  of   sul f asal azi ne  (e. g. ,   headache,
occasi onal   m egal obl asti c  anem i a,   sk i n  r ash).   The  gr eatest  uti l i ty   of
sul f asal azi ne  i n  pati ents  w i th  CU C  i s  i n  l ong­ter m   m anagem ent,   w her e
i t  has  been  pr ov ed  to  r educe  the  f r equency   of   r el apses.   5­ASA,   gi v en
r ectal l y   by   enem a  or   supposi tor y ,   i s  w el l   tol er ated  and  ef f ecti v e.   Gi v en
or al l y ,   5­ASA  i s  r api dl y   denatur ed  by   gastr i c  aci d,   so  al ter nati v es  to
pl ai n  5­ASA,   such  as  m i cr oencapsul ated  (Pentasa;  Hoechst  Mar i on
Roussel ,   Kansas  Ci ty ,   MO)  or   acr y l i c­based  r esi n­coated  (Asacol ;
Pr octer   &  Gam bl e  Phar m aceuti cal ,   N or w i ch,   N Y)  f or m s  of   5­ASA,   m ay
be  used.   Because  the  r el ati v e  r i sk   f or   dev el opm ent  of   CU C  i s  gr eater   i n
nonsm ok er s  than  i n  sm ok er s  (the  opposi te  i s  tr ue  i n  Cr ohn's  di sease),
ni coti ne  i s  bei ng  tr i ed  i n  the  tr eatm ent  of   CU C;  som e  benef i t  has  been
r epor ted,   but  addi ti onal   r esear ch  i s  needed.

Ther e  i s  no  uni f or m l y   ef f ecti v e  tr eatm ent  av ai l abl e  f or   Cr ohn's  di sease.


How ev er ,   cor ti coster oi ds  hav e  docum ented  ef f i cacy   i n  di m i ni shi ng  the
acti v i ty   of   the  di sease  pr ocess.   Long­ter m   use  of   cor ti coster oi ds  i s  not
r ecom m ended  because  of   thei r   m any   si de  ef f ects,   such  as  osteopor osi s,
di abetes,   and  catar acts.   Sul f asal azi ne  has  som e  ef f ecti v eness,
especi al l y   i n  col oni c  Cr ohn's  di sease,   but  i s  l ess  ef f ecti v e  than
cor ti coster oi ds.   Pentasa,   i n  doses  of   m or e  than  3  m g  per   day   m ay   be
ef f i caci ous  i n  m i l d  to  m oder ate  Cr ohn's  di sease,   par ti cul ar l y   i n  i l eal
di sease.   Metr oni dazol e  m ay   be  at  l east  as  ef f ecti v e  as  sul f asal azi ne.
When  Cr ohn's  di sease  cannot  be  contr ol l ed  by   these  m edi cati ons,   the
i m m unosuppr essi v e  agent  azathi opr i ne  and  i ts  m etabol i te  6­
m er captopur i ne  ar e  of ten  used.   These  dr ugs  ar e  ef f ecti v e  i n  both
i nduci ng  and  m ai ntai ni ng  r em i ssi on  i n  i nf l am m ator y ­ty pe  and
f i stul i zi ng­ty pe  Cr ohn's  di sease.   Thei r   use  can  r esul t  i n  a  r educti on  i n
the  cor ti coster oi d  dose  needed,   but  thi s  adv antage  m ay   be  of f set  by
thei r   tox i c  ef f ects  (e. g. ,   pancr eati ti s,   al l er gi c  r eacti ons,   and
l eucopeni a).   Mor e  r ecentl y ,   i nf l i x i m ab,   a  chi m er i c  m onocl onal
anti tum or   necr osi s  f actor   anti body ,   has  been  show n  to  be  ef f ecti v e  i n
Cr ohn's  di sease,   both  i n  the  i nf l am m ator y   and  the  f i stul i zi ng  ty pes.   The
r ol e  of   i m m unodul ator   dr ugs  i n  CU C  i s  l ess  cl ear   than  i n  Cr ohn's
di sease.

Case
A  37­y ear ­ol d  m an  w i th  docum ented  CU C  w as  f i r st  seen  at  19  y ear s  because
of   sev er e  bl oody   di ar r hea  and  l ef t  l ow er   quadr ant  abdom i nal   pai n  that
necessi tated  hospi tal i zati on.   Af ter   10  day s  of   tr eatm ent  w i th  hi gh­dose
pr edni sone  and  sul f asal azi ne  hi s  sy m ptom s  w er e  contr ol l ed,   and  he  has
si nce  been  m anaged  w i th  these  m edi ci nes,   w i th  the  dosages  adjusted
dependi ng  on  hi s  di sease  acti v i ty .   He  has  not  r equi r ed  cor ti coster oi ds
ex cept  f or   f l ar e­ups  of   di sease.   Subsequent  to  hi s  i ni ti al   pr esentati on,   af ter
hi s  di sease  acti v i ty   had  subsi ded,   he  under w ent  col onoscopy   f or   hi stol ogi c
conf i r m ati on  of   the  di sease  and  to  deter m i ne  the  ex tent  of   i ntesti nal
i nv ol v em ent;  thi s  ex am i nati on  r ev eal ed  di f f use  m ucosal   i nf l am m ati on
i nv ol v i ng  the  enti r e  col on  (pancol i ti s).   The  ter m i nal   i l eum   appear ed  nor m al .
Col oni c  bi opsy   speci m ens  r ev eal ed  a  di f f use  m ucosal   i nf l am m ator y   i nf i l tr ate
w i th  l i ttl e  i nv ol v em ent  of   the  subm ucosa,   acute  and  chr oni c  i nf l am m ator y
cel l s,   and  f r equent  cr y pt  abscesses  but  no  gr anul om as.
P. 145
The  pati ent  w ent  on  to  gr aduate  f r om   col l ege  and  w as  then  hi r ed  as  a  sal es
r epr esentati v e  f or   a  phar m aceuti cal   com pany .   Because  hi s  di sease  has  been
qui escent  and  hi s  schedul e  v er y   busy   he  has  not  tak en  hi s  m edi cati ons
r egul ar l y   and  has  r ar el y   seen  hi s  phy si ci an.
Appr ox i m atel y   2  m onths  ago,   he  began  to  f eel   ti r ed,   and  i nter m i ttent  r ectal
bl eedi ng  dev el oped.   Hi s  phy si cal   ex am i nati on  f i ndi ngs  ar e  unr em ar k abl e,
but  the  f ecal   occul t  bl ood  test  r esul t  i s  posi ti v e.   The  hem ogl obi n  i s  11  g/dL;
hem atocr i t,   33%;  and  l euk ocy te  count,   7, 700  cel l s/m m 3 ,   w i th  a  nor m al
di f f er enti al   count.

1.   What  i s  y our   di f f er enti al   di agnosi s  of   hi s  r ecent  sy m ptom s?


2.   What  tests  ar e  necessar y   to  m ak e  the  cor r ect  di agnosi s?
3.   How   shoul d  thi s  pati ent's  CU C  hav e  been  m anaged  ov er   the  pr ev i ous  18
y ear s?

Case Discussion
1.   What  i s  y our   di f f er enti al   di agnosi s  of   hi s  r ecent  sy m ptom s?

The  di f f er enti al   di agnosi s  i n  thi s  pati ent  i ncl udes  thr ee  possi bi l i ti es.
Fi r st,   thi s  epi sode  coul d  be  an  acute  f l ar e­up  or   ex acer bati on  of   hi s
ul cer ati v e  col i ti s.   Second,   he  coul d  hav e  an  acute,   sel f ­l i m i ted  col i ti s
super i m posed  on  hi s  ul cer ati v e  col i ti s;  i nf ecti on  w i th  Cam py l obacter ,
Sal m onel l a,   or   Shi gel l a  speci es,   or   w i th  par asi tes  can  cause  such  a
col i ti s.   Thi r d,   the  r ectal   bl eedi ng  and  anem i a  coul d  be  the  r esul t  of
adenocar ci nom a.

2.   What  tests  ar e  necessar y   to  m ak e  the  cor r ect  di agnosi s?

Stool   cul tur es  and  the  ex am i nati on  of   stool   f or   ov a  and  par asi tes  w oul d
be  an  i m por tant  i ni ti al   l abor ator y   test  i n  thi s  pati ent.   These  pr ov ed  to
be  negati v e.

Fl ex i bl e  si gm oi doscopy   or   col onoscopy   w i th  the  acqui si ti on  of   bi opsy


speci m ens  i s  al so  an  i m por tant  di agnosti c  pr ocedur e.   In  contr ast  to
CIBD,   the  hi stol ogi c  f eatur es  of   acute  sel f ­l i m i ted  col i ti s  consi st  of
nor m al   cr y pt  ar chi tectur e  and  an  acute  but  not  chr oni c  i nf l am m ator y
i nf i l tr ate  i n  the  l am i na  pr opr i a.   Inf l am m ati on  i s  m or e  l i k el y   to  be  f ound
i n  the  upper   m ucosa  i n  acute  col i ti s,   and  i n  the  cr y pt  bases  i n  CIBD.
When  an  acute  sel f ­l i m i ted  col i ti s,   such  as  i nf ecti on  w i th
Cam py l obacter   jejuni ,   Sal m onel l a,   or   Shi gel l a,   r esol v es,   the  m ucosa  i s
nor m al ,   w her eas  cr y pt  di stor ti on  and  atr ophy   ar e  of ten  seen  i n  the
setti ng  of   heal ed  CIBD.   In  other   acute  col i ti des,   the  hi stol ogi c  f eatur es
f ound  i n  m ucosal   bi opsy   speci m ens  m ay   suggest  a  speci f i c  i nf ecti on;
these  i ncl ude  v i r al   i ncl usi ons,   par asi tes,   or   pseudom em br anes.

In  thi s  pati ent,   f l ex i bl e  si gm oi doscopy   w as  per f or m ed  to  a  depth  of   30
cm   and  r ev eal ed  m i l d  gr anul ar i ty   of   the  m ucosa  w i thout  bl eedi ng,
al though  som e  bl ood  w as  seen  com i ng  f r om   abov e  30  cm .   Acti v e  CU C
al m ost  al w ay s  i nv ol v es  the  r ectum ,   so  the  f i ndi ng  of   onl y   m i l d  changes
i n  thi s  pati ent's  r ectum   suggests  that  the  si gni f i cant  pathol ogi c  pr ocess
w as  hi gher   i n  the  col on.   A  col onoscopi c  ex am i nati on  show ed  a  sessi l e,
f ungati ng  m ass  i n  the  descendi ng  col on,   w hi ch  pr ov ed  to  be  an
adenocar ci nom a.

3.   How   shoul d  thi s  pati ent's  CU C  hav e  been  m anaged  ov er   the  pr ev i ous  18
y ear s?

Ther e  i s  not  y et  agr eem ent  on  the  m ost  cost­ef f ecti v e  appr oach  f or   the
sur v ei l l ance  f or   col oni c  cancer   i n  pati ents  w i th  CU C.   How ev er ,   af ter   a
pati ent  has
P. 146
had  ex tensi v e  di sease  f or   8  to  10  y ear s,   i t  i s  pr obabl y   w i se  to  per f or m
com pl ete  col onoscopy   ev er y   1  to  2  y ear s,   w i th  m ul ti pl e  bi opsy
speci m ens  obtai ned  ev er y   10  to  12  cm   f r om   nor m al ­appear i ng  m ucosa
and  tar geted  speci m ens  obtai ned  f r om   v i l l ous  ar eas  of   m ucosa,   ar eas
of   ul cer ati on  w i th  a  r ai sed  edge,   and  str i ctur es.   Col ectom y   i s
r ecom m ended  i f   m ul ti f ocal   or   hi gh­gr ade  dy spl asi a  i s  seen  i n  the
bi opsy   speci m ens  and  conf i r m ed  by   an  ex per i enced  pathol ogi st.   If   a
m ass  l esi on  associ ated  w i th  any   degr ee  of   dy spl asi a  i s  i denti f i ed,   thi s
i s  al so  a  gener al l y   accepted  i ndi cati on  f or   col ectom y .   The  m anagem ent
of   per si stent  l ow ­gr ade  dy spl asi a  w i thout  a  m ass  i s  m or e  contr ov er si al ,
but,   i ncr easi ngl y ,   col ectom y   i s  bei ng  r ecom m ended  f or   l ow ­gr ade
dy spl asi a  (Fi g.   4. 1).

Figure  4­1  A  pr oposed  sy stem   of   sur v ei l l ance  f or   cancer   i n


ul cer ati v e  col i ti s  usi ng  col onoscopy   and  bi opsy .   (Fr om   Stenson  WF
and  Kor zeni k   J.   Inf l am m ator y   bow el   di sease.   In:  Yam ada  T,
Al per s,   DH,   Kapl ow i tz  N ,   etal .   eds.   Tex tbook   of   gastr oenter ol ogy ,
4th  ed.   Phi l adel phi a:  Li ppi ncott  Wi l l i am s  &  Wi l k i ns,   2003:1748,   f i g
83­29. )

Cancer   pr ev enti on  i s  an  i m por tant  topi c  to  consi der   w hen  adv i si ng
y oung  pati ents  w i th  ex tensi v e  col i ti s  about  the  possi bl e  need  f or
sur gi cal   tr eatm ent.   The  deci si on  to  r ecom m end  pr ophy l acti c
pr octocol ectom y   af ter   m any   y ear s  of   col i ti s  m ust  be  based  on  sev er al
consi der ati ons  i n  the  i ndi v i dual   pati ent.   These  i ncl ude  the  i ntr actabi l i ty
of   sy m ptom s,   age,   psy chol ogi cal   m ak eup,   m edi cal   com pl i ance,   and  the
av ai l abi l i ty   of   new er   sur gi cal   pr ocedur es.   A  pr ophy l acti c  col ectom y
shoul d  be  r ecom m ended  to  a  noncom pl i ant  pati ent  w ho  acqui r es
ex tensi v e  ul cer ati v e  col i ti s  at  a  y oung  age.   Pati ents  w ho  hav e  CU C
shoul d  be  f ul l y   i nf or m ed  of   thei r   r i sk   f or   dev el opm ent  of   cancer ,   as
w el l   as  the  l i m i tati ons  of   endoscopi c  sur v ei l l ance  and  the  av ai l abi l i ty   of
sur gi cal   al ter nati v es.   If   a  pati ent  i s  unw i l l i ng  to  assent  to  the  sur gi cal
pr ocedur e,   then  he  or   she  m ust  be  com m i tted  to  under goi ng  r egul ar
sur v ei l l ance.

P. 147

Suggested Readings
Jew el l   DP.   U l cer ati v e  col i ti s.   In:  Fel dm an  M,   Fr i edm an  LS,   Sl ei senger
MH,   eds.   Sl ei senger   and  For dtr an's  gastr oi ntesti nal   and  l i v er   di sease.
Pathophy si ol ogy ,   di agnosi s,   m anagem ent,   7th  ed.   Phi l adel phi a:  WB
Saunder s,   2002:  2039–2067.

Sands  BE.   Cr ohn's  di sease.   In:  Fel dm an  M,   Fr i edm an  LS,   Sl ei senger   MH,
eds.   Sl ei senger   and  For dtr an's  gastr oi ntesti nal   and  l i v er   di sease.
Pathophy si ol ogy ,   di agnosi s,   m anagem ent,   7th  ed.   Phi l adel phi a:  WB
Saunder s,   2002:  2005–2038.

Stenson  WF,   Kor zeni k   J.   Inf l am m ator y   bow el   di sease.   In:  Yam ada  T,
Al per s  DH,   Kapl ow i tz  N ,   etal .   eds.   Tex tbook   of   gastr oenter ol ogy ,   4th  ed.
Phi l adel phi a:  Li ppi ncott  Wi l l i am s  &  Wi l k i ns,   2003:  1699.

Chronic Liver Disease
1.   What  ar e  som e  speci f i c  causes  of   chr oni c  l i v er   di sease?

2.   What  ar e  the  pr i nci pal   l abor ator y   abnor m al i ti es  i n  the  setti ng  of
chr oni c  l i v er   di sease?
3.   What  ar e  the  tw o  m ajor   hi stol ogi c  categor i es  of   chr oni c  hepati ti s  due  to
v i r al   i nf ecti on?

Discussion
1.   What  ar e  som e  speci f i c  causes  of   chr oni c  l i v er   di sease?

Chr oni c  l i v er   di sease  m ay   be  the  sequel a  of   sev er al   k i nds  of   tox i c,


m etabol i c,   i nf ecti ous,   i m m unol ogi c,   or   her edi tar y   condi ti ons.   Tabl e  4­2
contai ns  a  par ti al   l i st.

2.   What  ar e  the  pr i nci pal   l abor ator y   abnor m al i ti es  i n  the  setti ng  of
chr oni c  l i v er   di sease?

The  cl i ni cal l y   av ai l abl e  l i v er   f uncti on  tests  i ncl ude  those  that  assess,   at
l east  i n  par t,   l i v er   sy ntheti c  f uncti on  (ser um   al bum i n  and  bi l i r ubi n
concentr ati ons,   and  pr othr om bi n  ti m e)  and  those  that  m ostl y   ev al uate
the  hepatocel l ul ar   r el ease  of   enzy m es  (am i notr ansf er ases  and  al k al i ne
phosphatase).   Of ten,   the  l ev el s  of   am i notr ansf er ase  [al ani ne
am i notr ansf er ase  (ALT),   aspar tate  am i notr ansf er ase  (AST),   and
al k al i ne  phosphatase]  ar e  not  m ar k edl y   el ev ated  i n  pati ents  w i th
chr oni c  l i v er   di sease,   and  consequentl y   do  not  accur atel y   pr edi ct
pr ognosi s.   The  ser um   al bum i n  and  bi l i r ubi n  concentr ati ons  and  the
pr othr om bi n  ti m e  ar e  m or e  l i k el y   to  be  di sti nctl y   abnor m al ,   and  m or e
accur atel y   r ef l ect  the  tr ue  status  of   the  l i v er 's  f uncti onal   capaci ty .

3.   What  ar e  the  tw o  m ajor   hi stol ogi c  categor i es  of   chr oni c  hepati ti s  due  to
v i r al   i nf ecti on?

Categor i es  of   these  di seases,   constr ucted  by   i nter nati onal   com m i ttees,
consi st  of   thr ee  com ponents:  the  eti ol ogy   of   the  di seases,   gr adi ng  of
di sease  acti v i ty   (i . e. ,   the  sev er i ty   of   the  necr oi nf l am m ator y   pr ocess),
and  stagi ng  of   the  di sease  (i . e. ,   the  degr ee  of   f i br osi s  subsequent  to
necr oi nf l am m ator y   i nsul ts).   The  gr adi ng  and  stagi ng  ar e  usual l y   gi v en
a  sem i quanti tati v e  scor e  (0  to  4)  or   a  descr i pti v e  char acter i zati on
(e. g. ,   m i ni m al   to  sev er e  i nf l am m ati on,   or   no  f i br osi s  to  ci r r hosi s).

P. 148

Table 4­2 Specific Causes of Chronic Liver
Disease

Dr ugs  and  chem i cal s


Acetam i nophen
Al cohol a
Am i odar one
Ar seni c  and  i nor gani c  sal ts
Isoni azi d
N i tr of ur antoi n
Pr opy l thi our aci l
Vi ny l   chl or i de
Vi r al   hepati ti s
Hepati ti s  B  and  C a
Cy tom egal ov i r us  hepati ti s
Gr anul om atous  i nf ecti ons
Bacter i al   (tuber cul osi s),   spi r ochetal   (secondar y   sy phi l i s),
m y coti c  a
Dr ugs  and  f or ei gn  substances
Other   sour ces
Sar coi dosi s
Pr i m ar y   bi l i ar y   ci r r hosi s
Im m uni ty   di sor der s
Com pl i cati ons  of   ul cer ati v e  col i ti s  and  Cr ohn's  di sease
[pr i m ar y   bi l i ar y   ci r r hosi s  and
sm al l ­duct  pr i m ar y   scl er osi ng  chol angi ti s
(per i choangi ti s)] a
Pr i m ar y   bi l i ar y   ci r r hosi s a
Autoi m m une  chr oni c  hepati ti s a
Inher i ted  di seases
Wi l son's  di sease a
Hem ochr om atosi s a
Inbor n  er r or s  of   m etabol i sm   (gl y cogen  stor age  di sease
and  Gaucher 's  di sease)
α 1­Anti tr y psi n  def i ci ency

a Most  f r equentl y   encounter ed.

Case
A  60­y ear ­ol d  m an  i s  br ought  to  the  hospi tal   by   hi s  w i f e  because  he  has  not
been  acti ng  hi s  usual   sel f .   For   the  l ast  3  day s,   he  has  not  been  sl eepi ng  at
ni ght  and  has  been  nappi ng  dur i ng  the  day .   Ther e  i s  no  hi stor y   of   r ecent
tr aum a,   tak i ng  new   m edi cati ons,   or   sui ci dal   i deati on.   He  has  been  tak i ng
di azepam ,   5  m g  ni ghtl y ,   f or   i nsom ni a.   Ri sk   f actor s  f or   chr oni c  l i v er   di sease,
accor di ng  to  hi s  w i f e,   i ncl ude  the  consum pti on  of   tw o  beer s  ni ghtl y   f or   35
y ear s  and  a  bl ood  tr ansf usi on  f or   the  tr eatm ent  of   a  bl eedi ng  pepti c  ul cer
25  y ear s  ago,   at  w hi ch  ti m e  he  under w ent  an  “ul cer   sur ger y . â€​
On  phy si cal   ex am i nati on,   he  appear s  sl eepy   but  ar ousabl e.   The  v i tal   si gns
ar e  nor m al .   Sev er al   l ar ge  spi der   angi om as  ar e  pr esent  on  the  tor so.   Ther e
i s  no  scl er al   i cter us.   The  par oti d  gl ands  ar e  enl ar ged  bi l ater al l y ,   and
w asti ng  of   the  tem por al   m uscl es  i s  noted.   The  hear t  and  l ung  ex am i nati on
f i ndi ngs  ar e  nor m al .   Hi s  abdom en  i s  sl i ghtl y   di stended,   and
P. 149
shi f ti ng  dul l ness  and  a  m i dl i ne  scar   ar e  pr esent.   The  l i v er   i s  not  pal pabl e
bel ow   the  r i ght  costal   m ar gi n  but  i s  pal pabl e  10  cm   bel ow   the  x i phoi d
pr ocess;  i t  i s  f i r m   and  per cussed  to  a  span  of   8  cm   i n  the  r i ght
m i dcl av i cul ar   l i ne.   The  spl een  i s  pal pabl e.   The  abdom en  i s  not  tender   to
pal pati on  or   per cussi on.   The  testes  ar e  sm al l .   The  r ectum   i s  f ound  to
contai n  har d,   br ow n  stool ,   w hi ch  i s  posi ti v e  f or   occul t  bl ood.   Ther e  i s  m i l d
edem a  of   the  l egs  and  m oder ate  m uscl e  w asti ng.   Aster i x i s  i s  pr esent.   The
cr ani al   ner v es  and  deep  tendon  r ef l ex es  ar e  i ntact.   The  pati ent  i s  som ew hat
uncooper ati v e  but  hi s  m uscul ar   str ength  i s  not  f ocal l y   di m i ni shed;  hi s
pl antar   r ef l ex es  (Babi nsk i 's  si gn)  ar e  nor m al .
Labor ator y   data  ar e  as  f ol l ow s:  per i pher al   bl ood  w hi te  cel l   count,   2, 500
cel l s/m m 3 ;  hem ogl obi n,   10  g/dL;  hem atocr i t,   33%;  pl atel et  count,   125, 000/
m m 3 ;  ser um   AST,   100  IU /L  (nor m al ,   < 30  IU /L);  ALT,   80  IU /L  (nor m al ,   < 45
IU /L);  total   bi l i r ubi n,   1. 2  m g/dL;  al k al i ne  phosphatase,   150  IU /L  (nor m al ,
< 130  IU /L);  total   pr otei n,   8. 0  g/dL;  al bum i n,   3. 1  g/dL;  and  pr othr om bi n
ti m e,   13  seconds  (contr ol ,   11  seconds).

1.   What  f eatur es  hel p  y ou  to  di agnose  chr oni c  v er sus  acute  l i v er   di sease
i n  thi s  pati ent?
2.   Does  any   par ti cul ar   f actor   hel p  y ou  deter m i ne  the  cause  of   thi s  m an's
l i v er   di sease?
3.   What  r ev er si bl e  f actor s  coul d  be  contr i buti ng  to  thi s  m an's  pr esum ed
por tosy stem i c  encephal opathy   (PSE)?
4.   When,   i f   ev er ,   shoul d  thi s  m an's  asci tes  be  sam pl ed?  If   i t  shoul d,   how
and  w her e  shoul d  i t  be  sam pl ed?
5.   What  ar e  thr ee  possi bl e  ex pl anati ons  f or   the  occul t  bl ood  i n  hi s  stool ?
6.   What  i s  the  ser um –asci tes  al bum i n  gr adi ent,   and  of   w hat  v al ue  i s  i t?
7.   Woul d  y ou  star t  di ur eti c  ther apy   now ?  Why   or   w hy   not?
8.   Why   ar e  hi s  testes  sm al l ?
9.   Why   ar e  hi s  par oti d  gl ands  enl ar ged?
10.   Is  thi s  m an  at  i ncr eased  r i sk   f or   hepatocel l ul ar   car ci nom a?
11.   How   w oul d  y ou  ex cl ude  hepatocel l ul ar   car ci nom a?
12.   What  i s  i ncl uded  i n  y our   di f f er enti al   di agnosi s  of   thi s  m an's  chr oni c
l i v er   di sease?
13.   Why   i s  hepati ti s  A  not  i n  y our   di f f er enti al   di agnosi s?
The  r esul ts  of   addi ti onal   tests  ar e  av ai l abl e  w i thi n  4  hour s  of
adm i ssi on.   The  asci tes  i s  sam pl ed  f r om   a  l ef t  l ow er   quadr ant
par acentesi s,   y i el di ng  a  cl ear   y el l ow   f l ui d  w i th  a  w hi te  bl ood  cel l   count
of   380  cel l s/m m 3 ,   2%  pol y m or phonucl ear   l euk ocy tes,   an  al bum i n
concentr ati on  of   0. 5  g/dL,   and  a  total   pr otei n  l ev el   of   1  g/dL.   N o
or gani sm s  ar e  seen  on  Gr am 's­stai ned  speci m ens.
14.   Do  the  f i ndi ngs  f r om   the  addi ti onal   tests  on  the  asci ti c  f l ui d  suppor t
the  di agnosi s  of   por tal   hy per tensi on­associ ated  asci tes?  Why   or   w hy
not?
15.   Wi th  these  data  i n  m i nd,   w hat  tr eatm ent  w oul d  y ou  of f er   thi s  pati ent
now ,   and  w hy ?
16.   What  ar eas  of   the  pati ent's  hi stor y   shoul d  y ou  ex am i ne  at  gr eater
l ength,   and  w hy ?
17.   Woul d  y ou  of f er   thi s  pati ent  a  l i v er   bi opsy   and,   i f   so,   w hen?

Case Discussion
1.   What  f eatur es  hel p  y ou  to  di agnose  chr oni c  v er sus  acute  l i v er   di sease
i n  thi s  pati ent?

In  thi s  pati ent,   ther e  ar e  no  pathognom oni c  f eatur es  of   chr oni c  l i v er
di sease,   but  sev er al   that  suggest  thi s  condi ti on.   La rge  s pide r
a ngioma s   ar e  com m on  i n
P. 150
the  setti ng  of   chr oni c  l i v er   di sease,   but  not  acute  l i v er   di sease,
al though  sm al l ,   nonpal pabl e  spi der   angi om as  m ay   be  pr esent.   Mus c le
w a s ting  i s  com m on  i n  m oder atel y   adv anced  chr oni c  l i v er   di sease,   but
i s  not  due  to  poor   eati ng  habi ts.   Muscl e  w asti ng  i s  not  a  f eatur e  of
acute  l i v er   di sease  unl ess  i t  i s  the  r esul t  of   a  concom i tant,   unr el ated
pr obl em .   A  pa lpa ble ,  firm le ft lobe  of the  live r  (that  por ti on  pal pabl e
caudad  to  the  x i phoi d  pr ocess)  i s  usual l y   a  m ani f estati on  of   chr oni c
l i v er   di sease.   It  i s  al w ay s  i m por tant  to  pal pate  and  per cuss  f or   the
l i v er   i n  the  m i dl i ne,   as  w el l   as  i n  the  m i dcl av i cul ar   l i ne.   As c ite s ,   due
to  por tal   hy per tensi on,   i s  m uch  m or e  a  f eatur e  of   chr oni c  l i v er   di sease
than  of   any   other   di sor der .   Asci tes  m ay   occur   i n  the  setti ng  of   sev er e
acute  l i v er   di sease,   but  i t  i s  usual l y   not  of   si gni f i cant  quanti ty   to
w ar r ant  tr eatm ent.   One  notabl e  ex cepti on  i s  the  Budd­Chi ar i   sy ndr om e,
i n  w hi ch  ther e  m ay   be  asci tes,   al though  the  abdom i nal   di stenti on  i n
thi s  sy ndr om e  i s  par ti al l y   due  to  a  congested  and  enl ar ged  l i v er
stem m i ng  f r om   the  hepati c  v ei n  occl usi on.   Shifting dullne s s   i s
i ndi cati v e  of   a  l ar ge  am ount  (> 1. 0  to  1. 5  L)  of   asci tes.

P a nc ytope nia   i s  r el ated  to  the  spl eni c  sequestr ati on  of   bl ood  cel l s  and
i s  not  a  pr om i nent  f eatur e  of   l i v er   di sease  unl ess  the  spl een  i s
af f ected;  w hen  i t  i s,   i t  i s  usual l y   enl ar ged.   The  degr ee  of   pancy topeni a
(or   of   i ndi v i dual   cy topeni as)  m ay   not  cor r el ate  w i th  spl een  si ze.
Hepati ti s  C  m ay   be  associ ated  w i th  the  dev el opm ent  of   apl asti c
anem i a,   but  thi s  i s  r ar e.   Tr ansi ent  cy topeni as  m ay   be  seen  i n  hepati ti s,
as  i n  other   v i r al   i nf ecti ons.   A  low  s e rum a lbumin  l ev el   m ay   be  seen  i n
any   f or m   of   l i v er   di sease  that  has  l asted  f or   m or e  than  sev er al   w eek s.
A  high s e rum globulin  (total   pr otei n­al bum i n)  l ev el   i s  a  f eatur e  of
chr oni c  l i v er   di sease  r egar dl ess  of   the  cause.   Ex tr em el y   hi gh  ser um
gl obul i n  l ev el s  (i . e. ,   ≥10  g/dL)  shoul d  suggest  the  possi bi l i ty   of
autoi m m une  or   “l upoi dâ€​   hepati ti s;  thi s  di sor der   i s  usual l y   seen  i n
w om en  and  i s  f r equentl y   accom pani ed  by   other   autoi m m une  f eatur es,
such  as  thy r oi di ti s.   Autoi m m une  hepati ti s  i s  i m por tant  to  r ecogni ze
because  i t  can  usual l y   be  tr eated  w i th  cor ti coster oi ds.

2.   Does  any   par ti cul ar   f actor   hel p  y ou  deter m i ne  the  cause  of   thi s  m an's
l i v er   di sease?

Ther e  ar e  no  par ti cul ar   f actor s  that  poi nt  to  the  cause  of   thi s  pati ent's
l i v er   di sease.   The  m ajor   di f f er enti al   di agnoses  her e  ar e  al cohol i c  l i v er
di sease  and  chr oni c  acti v e  hepati ti s  (hepati ti s  C  f r om   hi s  bl ood
tr ansf usi on),   pr obabl y   i n  the  ci r r hoti c  stage.   N o  f eatur e  of   hi s  hi stor y ,
phy si cal   ex am i nati on,   or   r outi ne  l abor ator y   tests  hel ps  di sti ngui sh
betw een  these  tw o  causes.

3.   What  r ev er si bl e  f actor s  coul d  be  contr i buti ng  to  thi s  m an's  pr esum ed
PSE?

Be nzodia ze pine s ,   other   sedati v e  or   hy pnoti c  dr ugs,   and  opi ates  m ay


pr eci pi tate  PSE  i n  a  pati ent  w i th  sev er el y   i m pai r ed  hepati c  f uncti on.
Cons tipa tion  m ay   al so  pr eci pi tate  PSE  i n  suscepti bl e  pati ents  because
of   the  col oni c  absor pti on  of   ni tr ogenous  pr oducts.   Both  these  r ev er si bl e
r i sk   f actor s  ar e  pr esent  i n  thi s  pati ent.   Other   r ev er si bl e  f actor s
contr i buti ng  to  an  epi sode  of   PSE  i ncl ude  e le c trolyte  dis turba nc e s ,
notabl y   hy pok al em i a  and  m etabol i c  al k al osi s;  inc re a s e d inte s tina l
a bs orption of nitroge nous  produc ts ,   r esul ti ng  f r om   r el ati v el y
ex cessi v e  di etar y   pr otei n  i ntak e  or   an  upper   gastr oi ntesti nal   (GI)
hem or r hage;  and  a  s e rious  infe c tion  of   any   natur e.   In  pati ents  w i th
chr oni c  l i v er   di sease  w ho  hav e  acute  PSE,   cul tur e  of   the  body   f l ui dsâ
€”asci ti c  f l ui d,   bl ood,   ur i ne,   and  sputum —shoul d  be  done.   Thi s
pati ent's  PSE  i ndi cates  that  he  has  sev er e  l i v er   di sease.

P. 151
4.   When,   i f   ev er ,   shoul d  thi s  m an's  asci tes  be  sam pl ed?  If   i t  shoul d,   how
and  w her e  shoul d  i t  be  sam pl ed?

Di agnosti c  par acentesi s  shoul d  be  per f or m ed  as  soon  as  possi bl e  to
deter m i ne  w hether   the  pati ent  has  subacute  bacter i al   per i toni ti s.   Thi s
f or m   of   i nf ecti ous  per i toni ti s  i s  a  f r equent  cause  of   cl i ni cal
deter i or ati on  i n  pati ents  w i th  chr oni c  l i v er   di sease,   and  m ay   be  f atal   i f
not  r ecogni zed  and  tr eated  ear l y .

The  thr ee  saf est  l ocati ons  f or   par acentesi s  ar e  the  l ef t  l ow er   quadr ant,
r i ght  l ow er   quadr ant,   and  the  i nf r aum bi l i cal   m i dl i ne  ar ea.   A
supr aum bi l i cal   appr oach  shoul d  nev er   be  used  because  the  um bi l i cal   or
par aum bi l i cal   v essel s,   w hi ch  cour se  just  under   the  par i etal   per i toneum ,
ar e  f r equentl y   r ecanal i zed  i n  pati ents  w i th  por tal   hy per tensi on  w hose
por tal   v ei n  i s  patent.   It  i s  al so  i m por tant  to  al w ay s  stay   cl ear   of
(m edi al   or   l ater al   to)  the  r ectus  m uscl es  because  the  super f i ci al
epi gastr i c  v essel s  cour se  under   them   and  m ay   be  punctur ed.   Sk i n
punctur e  thr ough  or   near   an  abdom i nal   scar   i n  a  pati ent  w i th  suspected
or   k now n  por tal   hy per tensi on  shoul d  al w ay s  be  av oi ded.

5.   What  ar e  thr ee  possi bl e  ex pl anati ons  f or   the  occul t  bl ood  i n  hi s  stool ?

Thr ee  possi bl e  ex pl anati ons  ar e  (a)  por tal   hy per tensi v e  gastr opathy   or
enter opathy ,   (b)  r ectal   v ar i ces,   and  (c)  esophageal   v ar i ceal
hem or r hage  due  to  por tal   hy per tensi on.   Var i ceal   bl eedi ng  i s  usual l y   a
sudden  ev ent  of   l ar ge  v ol um e,   al though  uncom m onl y   v ar i ces  “ooze. â
€​

6.   What  i s  the  ser um –asci tes  al bum i n  gr adi ent,   and  of   w hat  v al ue  i s  i t?

The  ser um –asci tes  al bum i n  gr adi ent  i s  the  num er i c  di f f er ence  (not
r ati o)  betw een  the  ser um   al bum i n  concentr ati on  and  the  asci tes
al bum i n  concentr ati on.   When  the  gr adi ent  i s  1. 1  or   gr eater ,   por tal
hy per tensi on  i s  contr i buti ng  to  or   enti r el y   causi ng  the  asci tes.   When
the  gr adi ent  i s  l ess  than  1. 1,   per i toneal   car ci nom atosi s  or
i nf l am m ator y   di seases  ar e  l i k el y   causes  of   the  asci tes.   On  the  basi s  of
thi s  m an's  hi stor y ,   the  tw o  m ai n  causes  to  be  consi der ed  ar e  por tal
hy per tensi on  and  per i toneal   m al i gnancy .   Deter m i nati on  of   the  ser um â
€“asci tes  al bum i n  di f f er ence  i s  a  si m pl e,   m i ni m al l y   i nv asi v e,   and  f ai r l y
accur ate  w ay   to  di agnose  por tal   hy per tensi on.

7.   Woul d  y ou  star t  di ur eti c  ther apy   now ?  Why   or   w hy   not?

N o.   Di ur eti cs  ar e  not  essenti al   now ,   and  they   m ay   onl y   w or sen  the  PSE
and  i ncr ease  the  r i sk   of   hepator enal   sy ndr om e.

8.   Why   ar e  hi s  testes  sm al l ?

In  the  setti ng  of   hepati c  di sease,   the  pr oducti on  of   estr one  f r om
ci r cul ati ng  andr ostenedi one  m ay   be  i ncr eased.   The  ex act  cause  of   thi s
conv er si on  i s  unk now n  but  m ay   be  r el ated  to  the  decr eased  cl ear ance
of   andr ostenedi one  by   the  l i v er .   The  consum pti on  of   ex cessi v e  am ounts
of   ethanol   m ay   al so  hav e  contr i buted  to  the  testi cul ar   atr ophy   i n  thi s
pati ent.

9.   Why   ar e  hi s  par oti d  gl ands  enl ar ged?

Par oti d  enl ar gem ent  i s  seen  i n  peopl e  w ho  i ngest  ex cessi v e  am ounts  of
ethanol ,   and  i s  associ ated  w i th  f atty   i nf i l tr ati on  of   the  gl ands.   A  si m i l ar
si tuati on  m ay   be  seen  i n  di abeti c  pati ents.

10.   Is  thi s  m an  at  i ncr eased  r i sk   f or   hepatocel l ul ar   car ci nom a?

Yes.   Ther e  i s  a  r i sk   f or   the  dev el opm ent  of   hepatocel l ul ar   car ci nom a  i n


the  setti ng  of   any   f or m   of   ci r r hoti c  l i v er ,   w hi ch  thi s  m an  m ost  l i k el y
has.   Cer tai n  condi ti ons
P. 152
ar e  associ ated  w i th  hi gher   r i sk s  than  other s.   Those  associ ated  w i th
hi ghest  r i sk   ar e  geneti c  hem ochr om atosi s,   chr oni c  hepati ti s  B,   chr oni c
hepati ti s  C,   and  al cohol i c  l i v er   di sease.

11.   How   w oul d  y ou  ex cl ude  hepatocel l ul ar   car ci nom a?

U sef ul   tests  f or   i denti f y i ng  hepatocel l ul ar   car ci nom a  ar e  an  i m agi ng


test  [ul tr asonogr aphy   or   com puted  tom ogr aphy   (CT)  or   m agneti c
r esonance  i m agi ng]  and  a  ser um   α ­f etopr otei n  l ev el .   The  pr ef er r ed
i m agi ng  test  (to  ex cl ude  a  f ocal   l esi on)  depends  on  the  ex per ti se  of   the
i nsti tuti on.   Ar ter i al ­phase  CT  i s  r egar ded  as  m ost  r el i abl e.
Hepatocel l ul ar   car ci nom as  ar e  especi al l y   di f f i cul t  to  detect  i n  ci r r hoti c
l i v er s;  ther ef or e  i t  i s  i m por tant  that  ar ter i al ­phase  CT  be  used  i n  thi s
setti ng.   The  ser um   α ­f etopr otei n  l ev el   i s  v er y   hi gh  i n  60%  of   pati ents
w i th  al cohol i c  l i v er   di sease  w ho  hav e  a  super i m posed  hepatocel l ul ar
car ci nom a  and  i n  appr ox i m atel y   80%  to  90%  of   pati ents  w i th  chr oni c
hepati ti s  B  w ho  hav e  thi s  com pl i cati on.

12.   What  i s  i ncl uded  i n  y our   di f f er enti al   di agnosi s  of   thi s  m an's  chr oni c
l i v er   di sease?

The  di f f er enti al   di agnosi s  i n  thi s  pati ent  i ncl udes  al cohol i c  l i v er   di sease
and  chr oni c  acti v e  hepati ti s  w i th  ci r r hosi s,   due  to  ei ther   hepati ti s  B  or
C,   al though  hepati ti s  shoul d  be  r egar ded  as  the  m or e  l i k el y   di agnosi s.
The  hepati ti s  v i r uses  m ay   hav e  been  tr ansm i tted  to  hi m   by   the  bl ood
he  r ecei v ed  m any   y ear s  ago,   or   they   m ay   hav e  been  “spor adi cal l y â€​
acqui r ed.

13.   Why   i s  hepati ti s  A  not  i n  y our   di f f er enti al   di agnosi s?

Hepati ti s  A  has  nev er   been  r epor ted  to  cause  chr oni c  l i v er   di sease.

14.   Do  the  f i ndi ngs  f r om   the  addi ti onal   tests  on  the  asci ti c  f l ui d  suppor t  the
di agnosi s  of   por tal   hy per tensi on­associ ated  asci tes?  Why   or   w hy   not?

Yes,   the  f i ndi ngs  f r om   the  tests  on  the  asci ti c  f l ui d  do  suppor t  the
di agnosi s  of   por tal   hy per tensi on­associ ated  asci tes  because  the  ser um â
€“asci tes  al bum i n  gr adi ent  (2. 6)  ex ceeds  1. 1.   Ther e  ar e  tw o  cav eats  to
r em em ber   w hen  usi ng  the  ser um –asci tes  al bum i n  gr adi ent  i n  the
di agnosi s  of   asci tes.   Fi r st,   i f   m assi v e  hepati c  m etastases  cause  enough
l i v er   di sease  to  r esul t  i n  por tal   hy per tensi on  and  asci tes,   the  gr adi ent
r esem bl es  that  seen  i n  por tal   hy per tensi on.   Second,   i n  asci tes  of   m i x ed
eti ol ogy   (e. g. ,   por tal   hy per tensi on  pl us  tuber cul ous  per i toni ti s),   the
gr adi ent  usual l y   r esem bl es  that  seen  i n  the  setti ng  of   por tal
hy per tensi on.

15.   Wi th  these  data  i n  m i nd,   w hat  tr eatm ent  w oul d  y ou  of f er   thi s  pati ent
now ,   and  w hy ?

Hospi tal   adm i ssi on  i s  r equi r ed.   Str i ct  bed  r est  (f or   f ear   of   sel f ­har m )
seem s  pr udent.   N o  benzodi azepi nes  shoul d  be  adm i ni ster ed,   al though
the  pati ent  shoul d  be  m oni tor ed  f or   the  si gns  of   ethanol   w i thdr aw al â
€”agi tati on,   tachy car di a,   f ev er ,   and  hal l uci nosi s.   The  pati ent  shoul d
r ecei v e  an  enem a  i f   he  i s  consti pated.   Lactul ose  shoul d  al so  be
adm i ni ster ed  (by   m outh  or   nasogastr i c  tube)  i f   the  pati ent  becom es  too
di sor i ented  and  uncooper ati v e.   The  or al   or   nasogastr i c  l actul ose  dose  i s
v ar i abl e;  the  goal   of   ther apy   i s  to  pr oduce  tw o  to  thr ee  sof t  stool s  per
day .   Al ter nati v el y ,   a  nonabsor babl e  anti bi oti c  coul d  be  used,   such  as
neom y ci n  at  a  dosage  of   500  to  1, 000  m g  gi v en  or al l y   or   by   nasogastr i c
tube  ev er y   6  hour s,   or   r i f ax i m i n.   Ther e  i s  no  ev i dence  that  gi v i ng
l actul ose  and  an  anti bi oti c  together   i s  m or e  ef f ecti v e  than
adm i ni ster i ng  ei ther   al one.   Lactul ose  i s  pr obabl y   benef i ci al   i n  the
tr eatm ent  of   PSE  by   v i r tue  of   i ts  abi l i ty   to  decr ease  the  am ount  of
ni tr ogen  av ai l abl e  f or   absor pti on  (as  ur ea)  f r om   the  col on.   Lactul ose
m ay   accom pl i sh  thi s
P. 153
by   al ter i ng  the  col oni c  f l or a  to  m or e  ur ease­negati v e  f or m s  and  by
i nduci ng  an  osm oti c  di ar r hea.

16.   What  ar eas  of   the  pati ent's  hi stor y   shoul d  y ou  ex am i ne  at  gr eater
l ength,   and  w hy ?

One  ar ea  of   the  pati ent's  hi stor y   that  shoul d  be  ex am i ned  at  gr eater
l ength  i s  hi s  e tha nol c ons umption his tory.   Thi s  i nv ol v es  m or e
i nter v i ew i ng  of   hi s  f am i l y   and  f r i ends.   The  al l eged  am ount  of   ethanol
i ngested  (per   the  pati ent's  w i f e)  i s  too  l ow   to  cause  l i v er   di sease  i n
m en  because  the  al cohol   content  of   tw o  cans  of   beer   i s  appr ox i m atel y
12  g.   How ev er ,   the  par oti d  gl and  enl ar gem ent  and  testi cul ar   atr ophy
ar e  f i ndi ngs  that  suggest  hi s  ethanol   i ngesti on  has  been  m or e  than  he
has  adm i tted.   The  am ount  and  dur ati on  of   al cohol   i ngesti on  necessar y
to  cause  chr oni c  l i v er   di sease  i s  hi ghl y   v ar i abl e  am ong  i ndi v i dual s,
al though  the  i nci dence  of   bi opsy ­pr ov ed  ci r r hosi s,   al cohol i c  hepati ti s,
or   both,   i ncr eases  as  consum pti on  i s  i ncr eased.   It  i s  usual l y   bel i ev ed
that  the  thr eshol d  am ount  of   al cohol   consum pti on  that  l eads  to  these
ser i ous  f or m s  of   chr oni c  l i v er   di sease  i s  i n  the  or der   of   100  to  150  g
per   day   f or   sev er al   y ear s  i n  m en,   but  l ess  i n  w om en.   How ev er ,   a  l ar ge
pr opor ti on  of   heav y   dr i nk er s  do  not  contr act  ser i ous  l i v er   di seases.   It
i s  adv i sabl e  to  r ecor d  al cohol   consum pti on  i n  ter m s  of   gr am s  per   day
ti m es  the  num ber   of   y ear s  of   consum pti on.   A  quar t  of   80  pr oof   w hi sk ey
contai ns  appr ox i m atel y   300  g  of   ethanol ,   a  si x ­pack   of   4%  beer
appr ox i m atel y   75  g,   and  750  m L  of   w i ne  appr ox i m atel y   90  g  (150  g  f or
“f or ti f i edâ€​   w i ne).

A  second  ar ea  of   i nqui r y   shoul d  be  the  pati ent's  fa mily his tory.   In  thi s
pati ent,   y ou  shoul d  al so  ask   w hether   any one  i n  the  f am i l y   has  had  l i v er
di sease,   i ncl udi ng  geneti c  hem ochr om atosi s.   You  m i ght  phr ase  the
questi on  i n  thi s  w ay :  “Do  y ou  hav e  any   f am i l y   m em ber s  w ho  hav e
condi ti ons  that  r equi r e  bl ood  to  be  r em ov ed  as  tr eatm ent?â€​   The
m ani f estati ons  of   hem ochr om atosi s  m ay   di f f er   i n  v ar i ous  f am i l y
m em ber s,   and  m ay   consi st  of   car di om y opathy ,   di abetes,   ar thr i ti s,   or
pi tui tar y   i nsuf f i ci ency .   In  thi s  pati ent,   the  sm al l   l i v er   i s  i nconsi stent
w i th  a  di agnosi s  of   hem ochr om atosi s,   al though  al l   el se  i s.   Mor eov er ,   he
i s  an  ol der   m an—the  ty pi cal   age  and  sex   of   pati ents  w ho  hav e  sev er e
chr oni c  l i v er   di sease  caused  by   hem ochr om atosi s.

17.   Woul d  y ou  of f er   thi s  pati ent  a  l i v er   bi opsy   and,   i f   so,   w hen?

A  l i v er   bi opsy   w oul d  be  of   no  hel p  i n  the  i ni ti al   m anagem ent  of   hi s


decom pensated  l i v er   di sease.   How ev er ,   w hen  concl usi v e  docum entati on
of   the  di agnosi s  w oul d  hel p  deter m i ne  m anagem ent,   l i v er   bi opsy   m i ght
be  i m por tant.   Thi s  m i ght  be  the  case  i n  a  pati ent  w i th  suspected  Budd­
Chi ar i   sy ndr om e  because  i t  i s  of ten  tr eatabl e  by   hepati c  decom pr essi on
(as,   e. g. ,   w i th  a  si de­to­si de  por tacav al   anastom osi s),   or   i t  m i ght  be
the  case  i n  a  pati ent  w i th  hem ochr om atosi s.   Once  the  pati ent's
condi ti on  has  stabi l i zed,   a  l i v er   bi opsy   m i ght  be  of f er ed,   f or   thr ee
r easons.   Fi r st,   he  m ay   be  a  candi date  f or   speci f i c  ther apy .   How ev er ,   i t
i s  unl i k el y   that  ther e  i s  any   ther apy   f or   thi s  pati ent.   If ,   as  seem s
l i k el y ,   he  has  al cohol i c  ci r r hosi s  ther e  i s  no  ef f ecti v e  tr eatm ent  other
than  absti nence;  i f   he  has  hepati ti s  C–r el ated  ci r r hosi s,   i nter f er on
tr eatm ent  m ay   be  danger ous  because  of   the  hepatocy tol y si s  br ought
about  by   ther apy .   Second,   som e  author i ti es  bel i ev e  that  l i v er   bi opsy   i s
i ndi cated  i n  pati ents  w i th  suspected  al cohol i c  l i v er   di sease  because
conf i r m ati on  of   that  di agnosi s  m i ght  hel p  per suade  the  pati ent  to
abstai n  f r om   f ur ther   ethanol   i ngesti on.   Thi r d,   i f   the  pati ent  becom es  a
candi date  f or   hepati c  tr anspl antati on,   m ost  center s  r equi r e  a  def i ni ti v e
pr eoper ati v e  di agnosi s  bef or e  goi ng  ahead  w i th  the  pr ocedur e.

P. 154

Suggested Readings
Batts  KP,   Ludw i g  J.   Chr oni c  hepati ti s:  an  update  on  ter m i nol ogy   and
r epor ti ng.   Am   J  Pathol   1995;19:  1409.

Dasar athy   S,   McCul l ough  AJ.   Al cohol i c  l i v er   di sease.   In:  Schi f f   ER,
Sor r el l   MF,   Maddr ey   WC,   eds.   Schi f f 's  di seases  of   the  l i v er ,   9th  ed.
Phi l adel phi a:  Li ppi ncott  Wi l l i am s  &  Wi l k i ns,   2003:  1019–1057.

Dav i s  GL.   Hepati ti s  C.   In:  Schi f f   ER,   Sor r el l   MF,   Maddr ey   WC,   eds.
Schi f f 's  di seases  of   the  l i v er ,   9th  ed.   Phi l adel phi a:  Li ppi ncott  Wi l l i am s  &
Wi l k i ns,   2003:  807–861.

Diarrhea
1.   What  i s  the  di agnosti c  i m por tance  of   noctur nal   di ar r hea  i n  a  pati ent
w i th  chr oni c  di ar r hea?

2.   What  i s  the  di f f er ence  betw een  a  secr etor y   and  an  osm oti c  di ar r hea?

3.   What  happens  to  di ar r heal   stool   v ol um e  af ter   f asti ng  i n  the  f ol l ow i ng


setti ngs:  A  v asoacti v e  i ntesti nal   pepti de  (VIP)  tum or ,   the  abr upt  onset
of   w ater y   di ar r hea  af ter   tr av el i ng  outsi de  of   the  U ni ted  States,   or
di ar r hea  onl y   w hen  dr i nk i ng  l ar ge  am ounts  of   car bonated  bev er ages?

4.   What  i s  the  m ost  l i k el y   cause  of   di ar r hea  i n  a  pati ent  w ho  has  r ecentl y
tak en  am pi ci l l i n  and  then  has  l ow ­gr ade  f ev er   and  w ater y   di ar r hea?
What  i s  the  m ost  cost­ef f ecti v e  w ay   to  di agnose  thi s  di sease,   and  how
w oul d  y ou  tr eat  thi s  pati ent?

5.   Why   do  pati ents  w i th  gi ar di asi s  of ten  com pl ai n  of   i ncr eased  stool
v ol um e  and  abdom i nal   cr am pi ng  w hen  they   consum e  m i l k   pr oducts?

6.   Whi ch  or gani sm s  ar e  m ost  com m onl y   associ ated  w i th  di ar r hea  of   l ess
than  2  to  3  w eek s'  dur ati on,   and  w hat  ar e  thei r   cl i ni cal   char acter i sti cs?
How   ar e  such  cases  ev al uated,   and  w hat  ar e  the  v ar i ous  appr oaches  to
tr eatm ent?

7.   What  i s  the  uti l i ty   of   stai ni ng  stool   speci m ens  f or   l euk ocy tes?

8.   What  w oul d  the  cl i ni ci an  l ook   f or   i f   sur r epti ti ous  l ax ati v e  abuse  i s
suspected  as  a  cause  of   chr oni c  di ar r hea?

9.   A  24­y ear ­ol d  w om an  w ho  has  had  a  r ecur r i ng  r ectov agi nal   f i stul a  f or   2
y ear s  com pl ai ns  of   f r equent  sm al l ­v ol um e  stool s,   w hi ch  occasi onal l y
contai n  bl ood  and  m ucus.   Stool   cul tur es  y i el d  negati v e  f i ndi ngs.   What
i s  the  l i k el y   di sease  i n  thi s  w om an  w ho  has  a  r ectov agi nal   f i stul a,   and
w hat  w oul d  be  the  nex t  step  i n  ev al uati ng  her ?

Discussion
1.   What  i s  the  di agnosti c  i m por tance  of   noctur nal   di ar r hea  i n  a  pati ent
w i th  chr oni c  di ar r hea?

N octur nal   di ar r hea  suggests  an  or gani c  cause  of   the  di ar r hea.   Pati ents
w i th  i r r i tabl e  bow el   sy ndr om e  or   other   “f uncti onal â€​
  di ar r heas
r ar el y   hav e  di ar r hea  that  aw ak ens  them   f r om   sl eep.

P. 155
2.   What  i s  the  di f f er ence  betw een  a  secr etor y   and  an  osm oti c  di ar r hea?

Se c re tory dia rrhe a   i s  due  to  the  acti v e  secr eti on  of   w ater   and
el ectr ol y tes  i nto  the  i ntesti nal   l um en.   The  m echani sm   of   acti on
r esponsi bl e  f or   the  r el ease  of   the  secr etagogues  i s  v ar i abl e.   For
i nstance,   the  di ar r hea  of   chol er a,   the  cl assi c  ex am pl e  of   a  secr etor y
di ar r hea,   i s  caused  by   the  sti m ul ati on  of   adeny l ate  cy cl ase  acti v i ty   by
chol er a  tox i n;  thi s,   i n  tur n,   causes  an  i ncr ease  i n  the  i ntr acel l ul ar
concentr ati on  of   cy cl i c  adenosi ne  m onophosphate,   w hi ch  sti m ul ates
el ectr ogeni c  chl or i de  secr eti on  and  i nhi bi ts  el ectr oneutr al   sodi um
chl or i de  absor pti on.   Incr eases  i n  i ntr acel l ul ar   concentr ati ons  of   Ca 2 +   as
w el l   as  cy cl i c  guanosi ne  m onophosphate  hav e  been  pr oposed  as  the
abnor m al i ti es  at  w or k   i n  v ar i ous  other   f or m s  of   secr etor y   di ar r hea.
In  os motic  dia rrhe a ,   an  unabsor babl e  sol ute  (of ten  a  car bohy dr ate  or
di v al ent  m i ner al )  i ncr eases  the  osm ol al i ty   of   the  i ntesti nal   contents.
Thi s  i ncr eased  osm ol al i ty   passi v el y   “dr agsâ€​   w ater   i nto  the
i ntesti nal   l um en.   Pati ents  w i th  osm oti c  di ar r hea  usual l y   hav e  a  stool
osm ol al i ty   m easur e  that  i s  m uch  gr eater   than  that  y i el ded  by   the
f or m ul a:  2  ×  ser um   N a +   +   ser um   K + ;  thi s  condi ti on  consti tutes  an
osm oti c  gap.   A  com m on  osm oti c  di ar r hea  i s  that  w hi ch  occur s  af ter   the
i ngesti on  of   m i l k   or   m i l k   pr oducts  i n  peopl e  w ho  ar e  def i ci ent  i n  the
i ntesti nal   enzy m e  l actase,   or   those  w ho  i ngest  m agnesi um ­contai ni ng
antaci ds  or   l ax ati v es.

3.   What  happens  to  di ar r heal   stool   v ol um e  af ter   f asti ng  i n  the  f ol l ow i ng


setti ngs:  a  VIP  tum or ,   the  abr upt  onset  of   w ater y   di ar r hea  af ter
tr av el i ng  outsi de  of   the  U ni ted  States,   or   di ar r hea  onl y   w hen  dr i nk i ng
l ar ge  am ounts  of   car bonated  bev er ages?

VIP  i s  pr oduced  by   the  i ntesti nal   m ucosa  i n  i ncr eased  am ounts  i n  the
WDHA  (w ater y   di ar r hea,   hy pok al em i a,   and  achl or hy dr i a)  sy ndr om e.   VIP
causes  di ar r hea  by   sti m ul ati ng  m ucosal   adeny l ate  cy cl ase  acti v i ty ,   and
ther ef or e  w oul d  be  ex pected  to  cause  a  secr etor y   di ar r hea.   In  such  a
condi ti on,   f asti ng  w oul d  not  decr ease  the  stool   v ol um e  unti l   the  pati ent
becom es  sev er el y   dehy dr ated.

Ty pi cal l y ,   tr av el er s’   di ar r hea  i s  w ater y   and  occur s  w i thi n  3  to  6


day s  of   ar r i v i ng  i n  another   countr y ,   or   on  r etur n.   Sy m ptom s  usual l y
l ast  f or   2  to  3  day s  and  r esol v e  spontaneousl y .   The  m ost  com m on
pathogens  r esponsi bl e  ar e  the  enter otox i geni c  str ai ns  of   Escher i chi a
col i ,   w hi ch  can  el abor ate  heat­l abi l e  and  heat­stabl e  enter otox i ns.   The
heat­l abi l e  tox i n  acts  si m i l ar l y   to  chol er a  tox i n,   w her eas  the  heat­
stabl e  tox i n  sti m ul ates  m ucosal   guany l ate  cy cl ase  acti v i ty .   Other   ty pes
of   di ar r hea­pr oduci ng  E.   col i   and  thei r   associ ated  sy m ptom s  ar e  the
enter opathogeni c  ty pe,   w hi ch  causes  w ater y   di ar r hea,   pr edom i nantl y   i n
chi l dr en  and  new bor ns;  the  enter oi nv asi v e  ty pe,   w hi ch  causes  bl oody
di ar r hea  (dy senter y )  i n  chi l dr en  and  adul ts,   usual l y   af ter   the  i ngesti on
of   contam i nated  f ood  and  w ater ;  and  the  enter ohem or r hagi c  ty pe,
w hi ch  causes  bl oody   di ar r hea  i n  peopl e  of   al l   ages  and  i s  tr ansm i tted
thr ough  contam i nated  f ood  (of ten  poor l y   cook ed  ham bur ger ).   Ser oty pe
0157:H7  of   the  enter ohem or r hagi c  ty pe  has  been  i denti f i ed  i n  sev er al
outbr eak s  of   i nf ecti on  char acter i zed  by   par ti cul ar l y   sev er e  di sease
(hem ol y ti c  ur em i c  sy ndr om e).

P. 156
Other   pathogens  associ ated  w i th  tr av el er s’   di ar r hea  i ncl ude  Shi gel l a
and  Sal m onel l a  speci es,   C.   jejuni ,   and  Vi br i o  par ahaem ol y ti cus.
Because  of   the  num er ous  causes  of   tr av el er 's  di ar r hea,   the  ef f ect  of
f asti ng  i s  usual l y   unpr edi ctabl e.

The  osm oti c  di ar r hea  that  occur s  onl y   af ter   dr i nk i ng  l ar ge  am ounts  of
car bonated  bev er ages  i s  due  to  the  i ngesti on  of   l ar ge  am ounts  of
f r uctose,   w hi ch  i s  the  sugar   used  to  sw eeten  these  bev er ages  (al though
not  di et  dr i nk s)  and  com es  i n  the  f or m   of   cor n  sy r up.   Fr uctose  i s  poor l y
absor bed  by   the  pr ox i m al   sm al l   i ntesti nal   m ucosa.   Cessati on  of
f r uctose  i ntak e  shoul d  stop  the  di ar r hea.

4.   What  i s  the  m ost  l i k el y   cause  of   di ar r hea  i n  a  pati ent  w ho  has  r ecentl y
tak en  am pi ci l l i n  and  then  has  l ow ­gr ade  f ev er   and  w ater y   di ar r hea?
What  i s  the  m ost  cost­ef f ecti v e  w ay   to  di agnose  thi s  di sease,   and  how
w oul d  y ou  tr eat  thi s  pati ent?

Pseudom em br anous  col i ti s  (PMC)  caused  by   Cl ostr i di um   di f f i ci l e  i s  a


l i k el y   di agnosi s  i n  thi s  i nstance,   gi v en  the  pati ent's  r ecent  anti bi oti c
use.   The  di sease  i s  usual l y   sel f ­l i m i ted,   w i th  the  di ar r hea  di ssi pati ng  5
to  10  day s  af ter   di sconti nuati on  of   the  of f endi ng  anti bi oti c.   Cl i ndam y ci n
w as  the  f i r st  dr ug  pr ov ed  to  cause  PMC;  l ater ,   am pi ci l l i n,   because  of
i ts  w i despr ead  use,   w as  the  dr ug  m ost  com m onl y   i m pl i cated,   but
v i r tual l y   any   anti bi oti c  can  be  r esponsi bl e.   In  heal thy   adul ts,   C.
di f f i ci l e  col oni zati on  r ates  of   2%  to  3%  hav e  been  r epor ted,   w her eas
the  r ates  i n  adul ts  r ecei v i ng  anti m i cr obi al s  but  w i thout  di ar r heal
sy m ptom s  ar e  as  hi gh  as  10%  to  15%.

The  m ost  com m onl y   used  m ethod  f or   di agnosi ng  PMC  i s  the  cy totox i ci ty
assay ,   w hi ch  i nv ol v es  obser v ati on  of   the  cy topathi c  ef f ect  pr oduced  by
the  tox i n  on  a  cel l   cul tur e;  the  assay   has  a  sensi ti v i ty   of   95%  to  97%.
Al though  the  l atex   aggl uti nati on  test  f or   the  pr esence  of   tox i n  i s  both
cheaper   and  f aster   to  per f or m ,   i t  has  a  sensi ti v i ty   of   onl y
appr ox i m atel y   85%.   Gr oss  col oni c  abnor m al i ti es  i n  pati ents  w i th  PMC,
w hi ch  can  be  seen  endoscopi cal l y ,   ty pi cal l y   occur   i n  the  descendi ng  and
si gm oi d  col on,   m ak i ng  f l ex i bl e  si gm oi doscopy   an  adequate  ex am i nati on
i n  m ost  cases;  how ev er ,   cases  w i th  onl y   r i ght­si ded  i nv ol v em ent  hav e
been  r epor ted.   The  endoscopi c  f i ndi ngs  i n  pati ents  w i th  PMC  i ncl ude
er y them atous,   f r i abl e  m ucosa  w i th  char acter i sti c  pseudom em br anes.
Car e  m ust  be  tak en  to  r ul e  out  bacter i al   or   par asi ti c  i nf ecti ons
(especi al l y   C.   jejuni   and  Entam oeba  hi stol y ti ca)  and  i nf l am m ator y
bow el   di sease.

The  r ecom m ended  tr eatm ent  f or   l ess  sev er e  cases  of   PMC  consi sts  of
ei ther   or al   m etr oni dazol e  (250  m g  f our   ti m es  a  day )  or   v ancom y ci n
(125  to  500  m g  f our   ti m es  a  day ).   Par enter al   doses  of   m etr oni dazol e
[500  m g  i ntr av enousl y   (IV)  ev er y   6  hour s]  shoul d  be  gi v en  onl y   w hen
or al   m edi cati on  cannot  be  tol er ated.   The  IV  adm i ni str ati on  of
v ancom y ci n  i s  not  ef f ecti v e.   The  r ates  of   r el apse  ar e  si m i l ar   f or   both
m etr oni dazol e  and  v ancom y ci n  and  r ange  f r om   10%  to  15%.
Chol esty r am i ne  has  been  r epor ted  to  be  ef f ecti v e  i n  the  tr eatm ent  of
m i l d  PMC  or   as  an  adjuncti v e  m easur e,   pr esum abl y   by   bi ndi ng  the  tox i n
i ntr al um i nal l y .   Chol esty r am i ne  m ay   be  used  i n  conjuncti on  w i th
m etr oni dazol e  but  not  w i th  v ancom y ci n,   because  i t  can  bi nd  and
i nacti v ate  v ancom y ci n.   Recentl y ,   the  or al   adm i ni str ati on  of   the
nonabsor bed  anti bi oti c  r i f ax i m i n  and  pr obi oti cs  (pr epar ati ons  of   v i abl e
bacter i a  w i th  ther apeuti c  phy si ol ogi c  ef f ects)  has  been  r epor ted
ef f ecti v e  i n  the  tr eatm ent  of   r ecur r ent  PMC.
P. 157
5.   Why   do  pati ents  w i th  gi ar di asi s  of ten  com pl ai n  of   i ncr eased  stool
v ol um e  and  abdom i nal   cr am pi ng  w hen  they   consum e  m i l k   pr oducts?

Gi ar di a  l am bl i a  i nf ecti on  causes  a  def i ci ency   i n  the  i ntesti nal


di sacchar i dases,   i ncl udi ng  l actase.   The  di sacchar i dase  def i ci ency   can
cause  cr am pi ng  and  f l atul ence  af ter   the  i ngesti on  of   car bohy dr ates,
especi al l y   m i l k   pr oducts.

6.   Whi ch  or gani sm s  ar e  m ost  com m onl y   associ ated  w i th  di ar r hea  of   l ess
than  2  to  3  w eek s'  dur ati on,   and  w hat  ar e  thei r   cl i ni cal   char acter i sti cs?
How   ar e  such  cases  ev al uated,   and  w hat  ar e  the  v ar i ous  appr oaches  to
tr eatm ent?

The  ev al uati on  of   a  case  of   acute  di ar r hea  i nv ol v es  r outi ne  cul tur e  of
the  stool s,   ex am i nati on  of   the  stool s  f or   the  pr esence  of   ov a  and
par asi tes,   and,   i n  som e  i nstances,   f l ex i bl e  si gm oi doscopy .

One  of   the  vira l c a us e s   of   acute  di ar r hea  i s  the  N or w al k   agent  that  i s


seen  i n  f am i l y   and  com m uni ty   epi dem i cs,   usual l y   i n  ol der   chi l dr en  and
adul ts.   It  has  an  i ncubati on  per i od  of   1  to  2  day s.   Vom i ti ng  and  l ow ­
gr ade  f ev er   ar e  com m on.   Rotav i r us  i nf ecti on  i s  seen  i n  i nf ants  and
y oung  chi l dr en,   pr i m ar i l y   i n  w i nter ;  the  i ncubati on  per i od  i s  1  to  3
day s.   Vom i ti ng  (occur r i ng  i n  80%),   upper   r espi r ator y   sy m ptom s,   and
f ev er   (f ound  i n  30%)  ar e  com m on.   Enter i c  adenov i r us  i s  a  spor adi c
di sease  of   i nf ants  and  y oung  chi l dr en,   and  i s  of ten  associ ated  w i th
f ev er   and  upper   r espi r ator y   sy m ptom s.

Ther e  ar e  m any   ba c te ria l c a us e s   of   acute  di ar r hea.   In  Shi gel l a


i nf ecti on,   the  m ajor   si te  of   m ucosal   i nv asi on  i s  the  col on.   Penetr ati on
of   the  m ucosa  and  i nv asi on  of   the  bl oodstr eam   ar e  r ar e.   Cr am py
abdom i nal   pai n  and  tenesm us  ar e  hal l m ar k s  of   the  di sease.   The
or gani sm   pr oduces  an  enter otox i n  (Shi ga  tox i n)  that  acti v ates
adeny l ate  cy cl ase  and  causes  a  w ater y   di ar r hea  i n  the  ear l y   stages  of
the  di sease.   Bl oody   di ar r hea  soon  f ol l ow s.   The  m ai nstay   of   ther apy   i s
suppor ti v e,   w i th  r ehy dr ati on  m ost  i m por tant.   N ar coti cs  and
anti chol i ner gi c  m edi cati ons  shoul d  be  av oi ded.   Anti bi oti c  tr eatm ent  i s
r eser v ed  f or   those  cases  that  do  not  r esol v e  spontaneousl y   i n  sev er al
day s;  am pi ci l l i n  (500  m g  f our   ti m es  a  day ,   or al l y ,   f or   5  day s)  i s
usual l y   ef f ecti v e,   but  tr i m ethopr i m /sul f am ethox azol e  (one  doubl e­
str ength  tabl et  tw i ce  dai l y )  can  be  used  f or   r esi stant  str ai ns.   Chr oni c
car r i er s,   al though  uncom m on,   ar e  pr one  to  i nter m i ttent  attack s  of   the
di sease.

The  m ajor   si te  of   Sal m onel l a  i nv asi on  i s  the  i l eal   and,   som eti m es,   the
col oni c  m ucosa.   Bacter em i a,   w i th  or   w i thout  associ ated  GI  sy m ptom s,
occur s  i n  appr ox i m atel y   10%  of   the  cases.   Car r i er s  ar e  usual l y
asy m ptom ati c,   w i th  the  or gani sm   har bor ed  i n  the  gal l bl adder .
Per i um bi l i cal   pai n  and  bl oody   di ar r hea  l ast  appr ox i m atel y   5  day s.
Because  anti m i cr obi al   tr eatm ent  si gni f i cantl y   i ncr eases  the  car r i er
r ate,   i t  i s  r eser v ed  f or   those  cases  that  do  not  r esol v e  spontaneousl y
or   f or   those  pati ents  w ho  hav e  an  under l y i ng  pr edi sposi ng  condi ti on.

C.   jejuni   i s  a  com m on  bacter i al   pathogen  i sol ated  f r om   pati ents  w i th


acute  baci l l ar y   di ar r hea.   Inv asi on  of   the  m ucosa  occur s  pr edom i nantl y
i n  the  col on.   Tw o  f eatur es  that  m ay   di sti ngui sh  C.   jejuni   i nf ecti on  f r om
other   causes  of   bacter i al   di ar r hea  ar e  (a)  a  pr odr om e  of   consti tuti onal
sy m ptom s,   and  (b)  a  bi phasi c  cour se,   w i th  i ni ti al   i m pr ov em ent  f ol l ow ed
by   w or seni ng.   N o  anti bi oti c  r egi m en  has  been  show n  to  l essen  the
sy m ptom s  or   the  ti m e  cour se  of   the  di sease.

P. 158
Yer si ni a  enter ocol i ti ca  can  cause  enter ocol i ti s,   w i th  a  cl i ni cal   pi ctur e
consi sti ng  of   f ev er ,   abdom i nal   cr am pi ng,   and  bl oody   di ar r hea  l asti ng  1
to  3  w eek s.   Water y   di ar r hea  i s  seen,   possi bl y   due  to  enter otox i n
pr oducti on.   Inv asi v e  i l ei ti s  i s  al so  a  f eatur e  of   these  i nf ecti ons.

Other   di ar r hea­pr oduci ng  enter i c  pathogens  i ncl ude  E.   hi stol y ti ca,   G.
l am bl i a,   and  Str ongy l oi des  ster cor al i s.

7.   What  i s  the  uti l i ty   of   stai ni ng  stool   speci m ens  f or   l euk ocy tes?

The  pr esence  of   num er ous  l euk ocy tes  i n  stool   speci m ens  i m pl i es  the
ex i stence  of   acti v e  i nf l am m ati on  of   the  i ntesti nal   m ucosa.   In  cases  of
acute  di ar r hea,   the  pr esence  of   pus  i m pl i es  i nv asi on  (Shi gel l a,
Sal m onel l a,   C.   jejuni ,   and  E.   hi stol y ti ca).   Al though  Shi gel l a,   E.
hi stol y ti ca,   and  C.   jejuni   i nf ecti ons  ar e  usual l y   associ ated  w i th  m ost
pus,   pati ents  w i th  PMC  al so  of ten  hav e  l ar ge  num ber s  of   f ecal
l euk ocy tes.   In  cases  of   chr oni c  di ar r hea,   the  pr esence  of   pus  m ost
of ten  i m pl i es  tuber cul osi s,   am ebi c  col i ti s,   i schem i c  col i ti s,   or
i nf l am m ator y   bow el   di sease  (ul cer ati v e  col i ti s  m or e  so  than  Cr ohn's
di sease,   unl ess  the  l atter   i nv ol v es  the  col on).

8.   What  w oul d  the  cl i ni ci an  l ook   f or   i f   sur r epti ti ous  l ax ati v e  abuse  i s
suspected  as  a  cause  of   chr oni c  di ar r hea?

Ther e  ar e  sev er al   tel l tal e  cl ues  to  sur r epti ti ous  l ax ati v e  abuse.
Mel anosi s  col i ,   a  dar k   pi gm entati on  of   the  col or ectal   m ucosa,   m ay   ex i st
i f   the  di ar r hea  i s  due  to  l ong­standi ng  use  of   anthr acene  l ax ati v es
(al oe  and  cascar a).   The  pi gm entati on  usual l y   di sappear s  w i thi n  12
m onths  of   di sconti nuati on  of   the  l ax ati v e.   If   the  i ngesti on  of
phenol phthal ei n­contai ni ng  l ax ati v es  i s  the  cause  of   the  di ar r hea,
al k al i zati on  of   a  stool   speci m en  by   addi ng  sodi um   hy dr ox i de  tur ns  i t
pi nk .   (How ev er ,   r ai si ng  the  pH  too  hi gh  r esul ts  i n  l oss  of   the  col or   and
hence  a  f al se­negati v e  r esul t. )  An  osm oti c  gap  of   the  stool   m ay   be
pr esent  i f   the  i ngesti on  of   m agnesi um   sul f ate  i s  the  cause  of   the
di ar r hea.   Sodi um   sul f ate  and  phosphate,   how ev er ,   w hi ch  cause  an
osm oti c  di ar r hea  due  to  the  f or m ati on  of   the  ani ons  sul f ate  and
phosphate,   do  not  cause  an  osm oti c  gap,   and  shoul d  be  suspected  i n
those  thought  to  abuse  l ax ati v es  but  hav e  an  appar ent  secr etor y
di ar r hea.
9.   What  i s  the  l i k el y   di sease  i n  thi s  w om an  w ho  has  a  r ectov agi nal   f i stul a,
and  w hat  w oul d  be  the  nex t  step  i n  ev al uati ng  her ?

Cr ohn's  di sease  i s  the  m ost  com m on  cause  of   r ectov agi nal   f i stul as  i n
y oung  w om en  and  m ust  be  consi der ed  i n  the  ev al uati on  of   such
f i stul as.   Sm al l ­v ol um e,   bl oody   di ar r hea  i s  suggesti v e  of   anor ectal
i nv ol v em ent.   Af ter   r outi ne  stool   cul tur e  and  ex am i nati on  f or   ov a  and
par asi tes,   f l ex i bl e  si gm oi doscopy   or   col onoscopy   shoul d  be  per f or m ed.
In  cases  of   Cr ohn's  di sease  i n  w hi ch  sm al l   bow el   i nv ol v em ent  i s
consi der ed  l i k el y ,   a  sm al l   bow el   r adi ogr aphi c  study   m i ght  be  nex t  i n
or der .   In  any   ev ent,   cul tur es  and  ti ssue  f or   hi stol ogi c  anal y si s  shoul d
be  obtai ned  bef or e  em pi r i c  tr eatm ent  w i th  cor ti coster oi ds  i s  i nsti tuted,
to  ensur e  that  i nf ecti ous  col i ti s  i s  not  the  cause.

Case
A  32­y ear ­ol d  w om an  w i th  a  hi stor y   of   Cr ohn's  di sease  si nce  chi l dhood
com pl ai ns  of   hav i ng  10  to  12  l oose,   f r othy ,   bur ni ng  bow el   m ov em ents.   Bl ood
i s  occasi onal l y   i nter m i x ed
P. 159
i n  the  stool .   The  i ncr eased  stool   f r equency   has  been  a  pr obl em   ev er   si nce
her   m ost  r ecent  hospi tal i zati on  f or   the  tr eatm ent  of   sm al l   bow el
obstr ucti on,   5  w eek s  ear l i er .   At  that  ti m e,   the  r em ai ni ng  120  cm   of   her
i l eum   and  100  cm   of   her   jejunum   w er e  r esected  because  of   f i stul i zati on  and
the  f or m ati on  of   adhesi ons.   She  deni es  hav i ng  f ev er ,   chi l l s,   and  ni ght
sw eats.   Her   appeti te  has  been  good,   and  she  deni es  hav i ng  abdom i nal   pai n
associ ated  w i th  f ood  i ngesti on;  how ev er ,   she  has  l ost  15  l b  (6. 75  k g)  si nce
her   l ast  sur ger y .   Her   m edi cati ons  hav e  not  been  changed  si nce  her
di schar ge  f r om   the  hospi tal ,   and  consi st  of   m etr oni dazol e  (250  m g  f our
ti m es  dai l y ),   pr edni sone  (20  m g  dai l y ),   cal ci um ,   and  m onthl y   v i tam i n  B 1 2
i njecti ons.   She  has  had  a  l ong­standi ng  hi stor y   of   w ater y   di ar r hea,   w hi ch
w as  contr ol l ed  w i th  the  use  of   chol esty r am i ne  bef or e  the  m ost  r ecent
sur ger y .   Her   w ei ght  had  been  stabl e  f or   m any   y ear s.
The  pati ent  i s  pal e  but  i n  no  acute  di str ess  and  w i thout  f ev er .   N o  or thostati c
changes  i n  her   v i tal   si gns  ar e  noted.   Her   abdom en  i s  sof t  w i th  acti v e  bow el
sounds.   A  heal i ng  m i dl i ne  i nci si on  and  m ul ti pl e  scar s  f r om   pr ev i ous
sur ger i es  ar e  pr esent.   The  l i v er   span  i s  7  cm   i n  the  m i dcl av i cul ar   l i ne.   N o
abdom i nal   or   r ectal   m asses  ar e  f ound.   Her   l egs  ar e  sl i ghtl y   edem atous.   Her
stool   i s  dar k   br ow n  and  posi ti v e  f or   occul t  bl ood.
The  f ol l ow i ng  l abor ator y   r esul ts  ar e  obtai ned:  hem atocr i t,   32%;  m ean
cor puscul ar   v ol um e,   98  µ  m 3 ;  ser um   sodi um ,   136  m Eq/L;  potassi um ,   3. 0
m Eq/L;  chl or i de,   91  m Eq/L;  bi car bonate,   19  m Eq/L;  and  cr eati ni ne,   0. 6
m g/dL.   The  w hi te  bl ood  cel l   count,   pl atel et  count,   pr othr om bi n  ti m e,   and
l i v er   test  r esul ts  ar e  al l   nor m al .

1.   What  i s  the  f i r st  set  of   tests  y ou  w oul d  or der   i n  thi s  pati ent  to  hel p
ex pl ai n  the  di ar r hea?
2.   Is  thi s  l i k el y   to  be  a  f l ar e­up  of   the  pati ent's  Cr ohn's  di sease?
3.   If   the  chol esty r am i ne  tr eatm ent  i s  r esum ed,   how   w i l l   thi s  af f ect  the
v ol um e  of   her   di ar r hea,   and  w hy ?
4.   What  other   tr eatm ent  m i ght  be  pr escr i bed  i n  an  attem pt  to  contr ol   her
di ar r hea  and  ai d  her   nutr i ti on?

Case Discussion
1.   What  i s  the  f i r st  set  of   tests  y ou  w oul d  or der   i n  thi s  pati ent  to  hel p
ex pl ai n  the  di ar r hea?

The  f i r st  step  i n  ev al uati ng  the  pati ent  w i th  postoper ati v e  di ar r hea,
w hose  condi ti on  i s  stabl e,   i s  to  r ul e  out  enter i c  i nf ecti on,   w i th  cul tur es
and  ex am i nati on  of   the  stool s  f or   ov a  and  par asi tes.   Because  thi s
pati ent  i s  tak i ng  m etr oni dazol e,   the  possi bi l i ty   of   PMC  shoul d  al so  be
consi der ed,   and  a  stool   cy totox i ci ty   assay   car r i ed  out.   (Al though
m etr oni dazol e  i s  com m onl y   used  to  tr eat  PMC,   the  dr ug  has  al so  been
i m pl i cated  i n  sev er al   cases  as  the  causati v e  anti bi oti c. )  Fl ex i bl e
si gm oi doscopy   coul d  pr obabl y   be  saf el y   per f or m ed  soon  af ter   the
r esecti on,   but  as  l ong  as  the  pati ent's  condi ti on  i s  stabl e,   the  pr ocedur e
can  be  postponed  pendi ng  the  cul tur e  r esul ts.

2.   Is  thi s  l i k el y   to  be  a  f l ar e­up  of   the  pati ent's  Cr ohn's  di sease?

N o,   because  consti tuti onal   sy m ptom s  hav e  not  appear ed  or   changed,
and  al so  because  the  char acter   of   the  stool   i s  m or e  suggesti v e  of   a
m al absor pti v e  di sor der   than  of   an  acti v e  f l ar e­up  of   Cr ohn's  di sease.

P. 160
3.   If   the  chol esty r am i ne  tr eatm ent  i s  r esum ed,   how   w i l l   thi s  af f ect  the
v ol um e  of   her   di ar r hea,   and  w hy ?

Most  l i k el y   the  pati ent's  di ar r hea  w i l l   w or sen  i f   she  tak es


chol esty r am i ne,   w her eas  i t  w as  ef f ecti v e  bef or e  her   r ecent  sur ger y .
The  ex pl anati on  f or   thi s  di f f er ence  i n  ef f ect  i s  as  f ol l ow s:  Because  bi l e
aci ds  ar e  nor m al l y   absor bed  acti v el y   i n  the  di stal   i l eum   and  r esecr eted
by   the  l i v er   i nto  bi l e,   w hen  the  di stal   i l eum   i s  ei ther   sev er el y   di seased
or   r esected,   unabsor bed  bi l e  aci ds  enter   the  col on  and  sti m ul ate  w ater
and  el ectr ol y te  secr eti on,   r esul ti ng  i n  di ar r hea.   If   the  am ount  of   i l eum
i nv ol v ed  or   r esected  i s  l ess  than  appr ox i m atel y   100  cm ,   the  l i v er   can
com pensate  f or   the  l oss  of   bi l e  aci ds  by   i ncr easi ng  bi l e  aci d  sy nthesi s,
and  f at  m al absor pti on  and  w ei ght  l oss  ar e  ther eby   l ar gel y   pr ev ented.
Chol esty r am i ne  i s  ef f ecti v e  i n  contr ol l i ng  di ar r hea  i n  thi s  si tuati on  by
bi ndi ng  bi l e  aci ds  i n  the  sm al l   bow el   and  pr ev enti ng  thei r   secr etor y
ef f ects  i n  the  col on.   Such  a  set  of   ci r cum stances  appar entl y   ex i sted  i n
thi s  pati ent  bef or e  her   r ecent  sur ger y .   How ev er ,   her   l ast  sur ger y
r esul ted  i n  the  l oss  of   her   r em ai ni ng  i l eum   and  par t  of   the  jejunum .
Such  a  l ar ge  l oss  of   bow el   w oul d  m ost  l i k el y   r esul t  i n  depl eti on  of   her
bi l e  aci d  pool   bey ond  the  l i v er 's  abi l i ty   to  com pensate  f or   i t  by
i ncr easi ng  bi l e  aci d  sy nthesi s,   w i th  consequent  m al absor pti on  of   f at.
The  adm i ni str ati on  of   chol esty r am i ne  w oul d  f ur ther   depl ete  the  bi l e
aci d  pool   and  aggr av ate  the  f at  m al absor pti on,   w hi ch  i n  tur n  w oul d
w or sen  the  di ar r hea.   The  m echani sm   r esponsi bl e  f or   thi s  l atter   ev ent  i s
the  sti m ul ator y   ef f ect  of   unabsor bed  f atty   aci ds  or   thei r   hy dr ox y
der i v ati v es  on  col oni c  w ater   and  el ectr ol y te  secr eti on.

4.   What  other   tr eatm ent  m i ght  be  pr escr i bed  i n  an  attem pt  to  contr ol   her
di ar r hea  and  ai d  her   nutr i ti on?

Medi um ­chai n  tr i gl y cer i des  gi v en  or al l y   shoul d  be  tr i ed  to  contr ol   her
di ar r hea  and  enhance  nutr i ti on.   They   do  not  r equi r e  sol ubi l i zati on  by
bi l e  aci ds  f or   ef f i ci ent  absor pti on.   At  the  sam e  ti m e,   the  pati ent  shoul d
obser v e  a  l ow ­f at  di et.

Suggested Readings
Bar tl ett  JG.   Anti bi oti c­associ ated  di ar r hea.   In:  Bl aser   MJ,   Sm i th  PD,
Rav di n  JI,   etal .   eds.   Inf ecti ons  of   the  gastr oi ntesti nal   tr act.   N ew   Yor k :
Rav en  Pr ess,   1995:893.

DuPont  HL.   Tr av el er 's  di ar r hea.   In:  Bl aser   MJ,   Sm i th  PD,   Rav di n  JI,   etal .
eds.   Inf ecti ons  of   the  gastr oi ntesti nal   tr act.   N ew   Yor k :  Rav en  Pr ess,
1995:299.

Pow el l   DW.   Appr oach  to  the  pati ent  w i th  di ar r hea.   In:  Yam ada  T,   Al per s
DH,   Kapl ow i tz  N ,   etal .   eds.   Tex tbook   of   gastr oenter ol ogy ,   4th  ed.
Phi l adel phi a:  Li ppi ncott  Wi l l i am s  &  Wi l k i ns,   2003:844.

Schi l l er   LR,   Sel l i n  JH.   Di ar r hea.   In:  Fel dm an  M,   Fr i edm an  LS,   Sl ei senger
MH,   eds.   Sl ei senger   and  For dtr an's  gastr oi ntesti nal   and  l i v er   di sease.
Pathophy si ol ogy ,   di agnosi s,   m anagem ent,   7th  ed.   Phi l adel phi a:  WB
Saunder s,   2002:131.

Malabsorption
1.   What  ar e  the  m ajor   steps  i n  the  di gesti on  and  absor pti on  of   di etar y
l i pi ds,   car bohy dr ates,   and  pr otei ns?

P. 161
2.   What  ar e  the  pr i nci pal   si tes  of   i ntesti nal   absor pti on  of   v ar i ous
nutr i ents?

3.   Of   w hat  does  the  enter ohepati c  ci r cul ati on  of   bi l e  aci ds  consi st?

4.   What  ar e  som e  of   the  m ajor   di sor der s  of   m al di gesti on  or


m al absor pti on?

Discussion
1.   What  ar e  the  m ajor   steps  i n  the  di gesti on  and  absor pti on  of   di etar y
l i pi ds,   car bohy dr ates,   and  pr otei ns?

The  pr ocess  of   di gesti on  can  be  di v i ded  i nto  thr ee  m ajor   steps:  (a)
i ntr al um i nal   di gesti on,   i ncl udi ng  the  acti on  of   bi l e  aci ds  and  pancr eati c
enzy m es;  (b)  di gesti on  by   the  i ntesti nal   epi thel i um ;  and  (c)  the
tr anspor t  of   nutr i ents  acr oss  the  epi thel i um   to  the  ci r cul ati on.

The  m ajor   ev ents  i n  the  dige s tion a nd a bs orption of die ta ry lipid


i ncl ude  (a)  the  l i pol y si s  of   di etar y   tr i gl y cer i des  by   pancr eati c  l i pase;
(b)  m i cel l ar   sol ubi l i zati on  of   the  r esul ti ng  l ong­chai n  f atty   aci ds  and  β­
m onogl y cer i des  by   bi l e  aci ds;  (c)  the  absor pti on  of   f atty   aci ds  and  β­
m onogl y cer i des  i nto  enter ocy tes;  (d)  the  r eester i f i cati on  and
i ncor por ati on  (al ong  w i th  chol ester ol ,   chol ester ol   ester s,   phosphol i pi d,
and  β­l i popr otei ns)  i nto  chy l om i cr ons  and  v er y   l ow –densi ty
l i popr otei ns;  and  (e)  the  tr anspor t  of   chy l om i cr ons  f r om   the  m ucosal
cel l   i nto  the  i ntesti nal   l y m phati cs.

In  the  dige s tion a nd a bs orption of die ta ry c a rbohydra te s ,   star ch,
w hi ch  accounts  f or   m ost  of   the  car bohy dr ate  i ntak e,   i s  i ni ti al l y
hy dr ol y zed  m ostl y   by   pancr eati c  am y l ase,   y i el di ng  sm al l er   sugar s
(m al tose,   m al totr i ose,   and  dex tr i ns).   These  pr oducts,   as  w el l   as
i ngested  di sacchar i des  such  as  l actose  (m i l k   sugar )  and  sucr ose,   ar e
hy dr ol y zed  f ur ther   i nto  thei r   com ponent  m onosacchar i des  by
gl ucosi dases  (m al tase,   sucr ase  α ­dex tr i nase,   and  l actase),   w hi ch  ar e
pr esent  i n  the  br ush  bor der   of   epi thel i al   cel l s  i n  the  pr ox i m al   i ntesti ne.
The  m onosacchar i des  ar e  then  absor bed  by   the  epi thel i al   cel l s  and
enter   the  por tal   ci r cul ati on.

For   the  dige s tion a nd a bs orption of die ta ry prote in  to  tak e  pl ace,
pr otei ns  ar e  f i r st  hy dr ol y zed  by   pancr eati c  enzy m es  i n  the  i ntesti nal
l um en.   These  enzy m es  i ncl ude  endopepti dases  (tr y psi n,   chy m otr y psi n,
and  el astase)  and  ex opepti dases  (car box y pepti dases  A  and  B).
Ol i gopepti des  pr oduced  by   the  pancr eati c  enzy m es  ar e  f ur ther
hy dr ol y zed  by   am i nopepti dases  l ocated  on  the  br ush  bor der   as  w el l   as
i n  the  cy topl asm   of   i ntesti nal   epi thel i al   cel l s.   The  r esul tant  am i no
aci ds,   and  cer tai n  di pepti des  and  tr i pepti des,   then  enter   the  por tal
ci r cul ati on.

2.   What  ar e  the  pr i nci pal   si tes  of   i ntesti nal   absor pti on  of   v ar i ous
nutr i ents?

Al l   di etar y   nutr i ents,   w i th  the  ex cepti on  of   v i tam i n  B 1 2   (cobal am i n),
ar e  absor bed  pr ef er enti al l y   i n  the  pr ox i m al   sm al l   i ntesti ne;  m ost
absor pti on  of   the  com ponents  of   a  m eal   occur s  w i thi n  the  f i r st  150  cm ,
al though  absor pti on  (especi al l y   of   sugar s  and  am i no  aci ds)  can  occur
m or e  di stal l y   (as  i n  the  ev ent  of   di sease  or   sur gi cal   by pass  of   the
pr ox i m al   i ntesti ne).   Vi tam i n  B 1 2   i s  absor bed  by   the  di stal   i l eum ,   w her e
ther e  i s  a  speci f i c  r eceptor   f or   the  cobal am i n  –  i ntr i nsi c  f actor
com pl ex .

P. 162
3.   Of   w hat  does  the  enter ohepati c  ci r cul ati on  of   bi l e  aci ds  consi st?

Bi l e  aci ds  ar e  sy nthesi zed  f r om   chol ester ol   by   the  l i v er   and  ar e


conjugated  to  ei ther   taur i ne  or   gl y ci ne  bef or e  secr eti on  i nto  bi l e.
Dur i ng  f asti ng,   the  bi l e  aci ds  ar e  stor ed  i n  the  gal l bl adder .   Af ter   a
m eal ,   they   ar e  secr eted  i nto  the  duodenum .   The  bi l e  aci ds  ar e  v er y
ef f i ci entl y   absor bed  f r om   the  di stal   i l eum ,   car r i ed  back   to  the  l i v er   by
the  por tal   v ei n,   ef f i ci entl y   ex tr acted  and  r econjugated  by   the  l i v er ,   and
then  secr eted  agai n  i nto  bi l e.   Dur i ng  each  cy cl e,   m or e  than  95%  of   the
bi l e  aci ds  ar e  absor bed,   but  onl y   sm al l   am ounts  ar e  absor bed  i n  the
pr ox i m al   sm al l   i ntesti ne.

4.   What  ar e  som e  of   the  m ajor   di sor der s  of   m al di gesti on  or


m al absor pti on?

Tabl e  4. 3  l i sts  the  r epr esentati v e  di sor der s.

Case
A  27­y ear ­ol d  w om an  com pl ai ns  of   11  m onths  of   di ar r hea,   gas,   and
abdom i nal   cr am ps.   She  has  f i v e  or   si x   l oose  bow el   m ov em ents  a  day ,   and
di ar r hea  of ten  aw ak ens  her   f r om   sl eep.   She  al so  com pl ai ns  of   abdom i nal
cr am ps  that  ar e  m ost  sev er e  just  bef or e  a  bow el   m ov em ent,   and  ar e  then
tem por ar i l y   r el i ev ed  w i th  the  bow el   m ov em ent.   In  addi ti on,
P. 163
she  f eel s  ti r ed  and  has  l ost  appr ox i m atel y   8  l b  (3. 6  k g)  w i thout  di eti ng.   She
has  noted  a  tendency   to  br ui se  easi l y .   She  dr i nk s  f our   gl asses  of   m i l k   a
day .

Table 4­3 Major Disorders of Maldigestion or
Malabsorption

Intr al um i nal   di sor der s


Pancr eati c  ex ocr i ne  (enzy m e)  i nsuf f i ci ency
Chr oni c  pancr eati ti s
Pancr eati c  r esecti on
Cy sti c  f i br osi s
Bi l e  aci d  def i ci ency
Pancr eati c  or   bi l e  duct  car ci nom a
Ex tensi v e  di stal   i l eal   r esecti on  or   di sease
Bacter i al   ov er gr ow th  i n  the  pr ox i m al   i ntesti ne
Sur gi cal   di sr upti on  of   the  conti nui ty   of   the  upper   bow el
(a  Bi l l r oth  II  gastr ojejunostom y )
Di sor der s  of   enter ocy tes
Pr i m ar y   def ects  (epi thel i um   hi stol ogi cal l y   nor m al )
Pr i m ar y   l actase  def i ci ency
Sucr ase–i som al tase  def i ci ency
Secondar y   def ects  (epi thel i um   hi stol ogi cal l y   abnor m al )
N ontr opi cal   spr ue  (cel i ac  di sease  and  gl uten­
sensi ti v e  enter opathy )
Tr opi cal   spr ue
Acqui r ed  i m m unodef i ci ency   sy ndr om e  enter opathy
Whi ppl e's  di sease
Di stur bed  tr ansf er   of   m etabol i tes  f r om   enter ocy tes  i nto  l y m ph
or   por tal   bl ood  Inf i l tr ati v e  pr ocesses  of   the  m ucosa
(am y l oi dosi s  and  l y m phom a)  Intesti nal   l y m phangi ectasi a

Her   past  m edi cal   hi stor y   i s  posi ti v e  onl y   f or   f ati gue,   f or   w hi ch  she  saw
another   phy si ci an  11  m onths  ago,   bef or e  the  di ar r hea  dev el oped.   The
phy si ci an  tol d  her   that  she  had  an  i r on­def i ci ency   anem i a.   Si nce  then,   she
has  tak en  f er r ous  sul f ate  (300  m g  f our   ti m es  dai l y ),   but  sti l l   f eel s  f ati gued.
She  tak es  no  other   m edi cati on.
Phy si cal   ex am i nati on  r ev eal s  a  y oung  w om an  w ho  appear s  m i l dl y
under w ei ght  but  i s  other w i se  nor m al .
Labor ator y   test  r esul ts  ar e  as  f ol l ow s:  w hi te  bl ood  cel l   count,   nor m al ;
hem atocr i t,   34%;  m ean  cor puscul ar   v ol um e,   74  µm 3 ;  ser um   i r on,   50
m g/dL;  total   i r on­bi ndi ng  capaci ty ,   435  m g/dL;  stool   l euk ocy te  test,
negati v e;  stool   ex am i nati on  f or   ov a  and  par asi tes,   negati v e;  ser um   al bum i n,
3. 2  m g/dL;  ser um   el ectr ol y tes,   nor m al ;  and  pr othr om bi n  ti m e,   2  seconds
gr eater   than  contr ol .
Whi l e  aw ai ti ng  these  l abor ator y   r esul ts,   y ou  adv i se  the  pati ent  to  stop
i ngesti ng  al l   m i l k   pr oducts.   The  pati ent  r epor ts  that  thi s  r educes  but  does
not  el i m i nate  the  di ar r hea  or   gas.

1.   What  addi ti onal   hi stor y   shoul d  y ou  obtai n  f r om   the  pati ent?
2.   What  m i ght  l ead  y ou  to  suspect  that  m al absor pti on  i s  the  cause  of   thi s
pati ent's  di ar r hea,   and  w hy ?  What  test  shoul d  be  per f or m ed  to  conf i r m
thi s,   and  w hy ?
Thi s  pati ent's  f ecal   f at  ex cr eti on  i s  m easur ed  and  f ound  el ev ated,
w hi ch  pr ov es  she  has  m al di gesti on  or   m al absor pti on.
3.   Consi der i ng  that  the  pati ent  has  ei ther   m al di gesti on  or   m al absor pti on,
w hat  ar e  the  tw o  di sor der s  that  m ay   decr ease  the  bi l e  aci d  pool ,   tw o
di sor der s  that  decr ease  pancr eati c  l i pase  acti v i ty ,   and  tw o  di sor der s
that  m ay   decr ease  absor pti on  by   sm al l   bow el   enter ocy tes?
4.   How   does  the  D­x y l ose  test  di f f er enti ate  pr obl em s  w i th  di gesti on  (e. g. ,
bi l e  sal t  depl eti on  and  pancr eati c  l i pase  def i ci ency )  f r om   pr obl em s  w i th
absor pti on?  N am e  one  di sor der   that  m ay   pr oduce  a  f al se­posi ti v e
r esul t.
The  D­x y l ose  test  i n  thi s  pati ent  r ev eal s  poor   absor pti on  of   thi s  sugar ,
w hi ch  i ndi cates  that  the  sm al l   bow el   absor pti on  pr obabl y   i s  abnor m al .
5.   On  the  basi s  of   the  r esul ts  of   the  D­x y l ose  test,   w hat  test  shoul d  be
per f or m ed  now ?
A  sm al l   bow el   bi opsy   speci m en  i n  thi s  pati ent  r ev eal s  m ucosal   v i l l ous
atr ophy   and  cr y pt  hy per pl asi a,   accom pani ed  by   an  i ncr eased  num ber   of
pl asm a  cel l s  and  l y m phocy tes  i n  the  l am i na  pr opr i a  and  an  i ncr eased
num ber   of   l y m phocy tes  i n  the  epi thel i um .
6.   Al though  the  bi opsy   f i ndi ngs  i ndi cate  cel i ac  spr ue,   w hat  other   di sor der s
coul d  pr oduce  such  a  “f l atâ€​   m ucosa?
7.   How   can  the  di agnosi s  of   cel i ac  spr ue  be  conf i r m ed?
8.   If   the  D­x y l ose  test  r esul t  w as  abnor m al ,   but  the  sm al l   bow el   bi opsy
f i ndi ngs  w er e  nor m al ,   a  bacter i al   ov er gr ow th  i n  the  pr ox i m al   sm al l
i ntesti ne  m i ght  be  suspected.   How   shoul d  thi s  possi bi l i ty   be  ev al uated?
9.   If   thi s  pati ent's  D­x y l ose  absor pti on  test  r esul t  had  been  nor m al ,   w hat
di sor der   m i ght  y ou  suspect  and  how   shoul d  y ou  ev al uate  thi s
possi bi l i ty ?
10.   Why   di d  the  sy m ptom s  i n  thi s  pati ent,   w ho  had  cel i ac  spr ue,   abate
w hen  she  stopped  dr i nk i ng  m i l k ?

P. 164

Case Discussion
1.   What  addi ti onal   hi stor y   shoul d  y ou  obtai n  f r om   the  pati ent?

Thi s  pati ent  has  chr oni c  di ar r hea,   w hi ch  i s  ar bi tr ar i l y   def i ned  as
di ar r hea  that  l asts  l onger   than  3  w eek s.   Chr oni c  di ar r hea  i s  a  f ai r l y
com m on  com pl ai nt,   w i th  a  l engthy   di f f er enti al   di agnosi s.   The  cl i ni cal
hi stor y   r em ai ns  the  m ai nstay   of   the  i ni ti al   appr oach  to  di agnosi s,   and
the  hi stor y   tak i ng  m ust  i ncl ude  questi ons  concer ni ng  the  f ol l ow i ng
f actor s:

Food:  Mi l k   consum pti on,   sor bi tol   (added  to  di et  f oods  and  f r ui t),
f r uctose  (f ound  i n  nondi et  sof t  dr i nk s,   candy ,   and  f r ui t),   and
unpasteur i zed  m i l k   (Yer si ni a  i nf ecti on).

Tr av el :  To  ar eas  w her e  gi ar di asi s,   am ebi asi s,   or   schi stosom i asi s


m i ght  be  contr acted.

Iatr ogeni c  f actor s:  Sur ger i es  i n  the  GI  tr act.   A  par ti al   gastr ectom y
can  r esul t  i n  dum pi ng  (r api d  em pty i ng  of   the  gastr i c  contents  i nto
the  sm al l   i ntesti ne)  and,   i f   a  stagnant  ar ea  of   bow el   i s  cr eated,
bacter i al   ov er gr ow th  can  r esul t  (the  bl i nd  l oop  sy ndr om e).
Medi cati ons  ar e  al so  a  com m on  cause  of   di ar r hea.   The
adm i ni str ati on  of   anti bi oti cs  can  r esul t  i n  C.   di f f i ci l e  col i ti s,   and
antaci d  use  can  pr oduce  an  osm oti c  di ar r hea.   β­Adr ener gi c
antagoni sts,   col chi ci ne,   l ax ati v es,   and  i nnum er abl e  other   dr ugs
can  al so  cause  di ar r hea.

Ri sk   f actor s  f or   acqui r ed  i m m unodef i ci ency   sy ndr om e  (AIDS).

Rev i ew   of   sy stem s:  Thi s  m ay   r ev eal   ar thr i ti s,   w hi ch  can


accom pany   i nf l am m ator y   bow el   di sease  or   Whi ppl e's  di sease;
pepti c  ul cer   di sease,   w hi ch  can  be  associ ated  w i th  the  Zol l i nger ­
El l i son  sy ndr om e;  sy m ptom s  or   a  hi stor y   of   di abetes;  or
hy per thy r oi di sm .

Past  m edi cal   pr obl em s,   w i th  an  em phasi s  on  chi l dhood  di ar r hea  or
m al nutr i ti on  and  sur ger i es.

Fur ther   char acter i zati on  of   the  di ar r hea:  Does  i t  aw ak en  the
pati ent  at  ni ght?  Is  i t  constant  or   does  i t  al ter nate  w i th
consti pati on?  The  m ost  com m on  cause  of   chr oni c  di ar r hea  i n  the
U . S.   popul ati on  i s  the  i r r i tabl e  bow el   sy ndr om e,   w hi ch  i s  a  poor l y
under stood  m oti l i ty   di sor der .   It  r ar el y   r esul ts  i n  di ar r hea  that
aw ak ens  the  pati ent  at  ni ght,   r ar el y   pr oduces  w ei ght  l oss,   and
m ay   hav e  di ar r hea  al ter nati ng  w i th  consti pati on.

2.   What  m i ght  l ead  y ou  to  suspect  that  m al absor pti on  i s  the  cause  of   thi s
pati ent's  di ar r hea,   and  w hy ?  What  test  shoul d  be  per f or m ed  to  conf i r m
thi s,   and  w hy ?

Mal absor pti on  i s  suspected  as  the  cause  of   the  di ar r hea  because  of   the
i r on  def i ci ency   that  does  not  r espond  to  or al   i r on  tr eatm ent  and
because  the  pr othr om bi n  ti m e  i s  el ev ated  w i thout  si gns  of   l i v er
di sease.   A  2­  or   3­day   stool   col l ecti on  f or   quanti tati v e  f at  anal y si s  i s
the  si ngl e  m ost  usef ul   test  to  docum ent  m al absor pti on.   Because  f at
absor pti on  i s  a  com pl ex   pr ocess  (r equi r i ng  the  di gesti on  of
tr i gl y cer i des  by   pancr eati c  l i pase,   sol ubi l i zati on  of   these  pr oducts  by
bi l e  sal ts,   and  absor pti on  of   the  subsequent  pr oducts  by   enter ocy tes  of
the  sm al l   i ntesti ne),   abnor m al i ti es  i n  any   of   these  steps  r esul t  i n  f at
m al absor pti on  and  an  i ncr ease  i n  f ecal   f at  ex cr eti on.   Ther ef or e,
m easur em ent  of   the  f ecal   f at  content  i s  a  test  f or   m any   steps  i n  the
P. 165
di gesti on  and  absor pti on  pathw ay s.   One  of   the  f ew   k i nds  of
m al absor pti on  that  does  not  cause  i ncr eased  f ecal   f at  l oss  i s  that  due
to  the  l ack   of   an  i ntesti nal   enzy m e  needed  i n  the  di gesti on  of   a
par ti cul ar   car bohy dr ate,   despi te  a  hi stol ogi cal l y   nor m al   i ntesti ne.   The
m ost  com m on  ex am pl e  of   thi s  i s  pr i m ar y   l actase  def i ci ency ,   i n  w hi ch
l actose  i s  not  absor bed  nor m al l y   but  f at  i s.

3.   Consi der i ng  that  the  pati ent  has  ei ther   m al di gesti on  or   m al absor pti on,
w hat  ar e  the  tw o  di sor der s  that  m ay   decr ease  the  bi l e  aci d  pool ,   tw o
di sor der s  that  decr ease  pancr eati c  l i pase  acti v i ty ,   and  tw o  di sor der s
that  m ay   decr ease  absor pti on  by   sm al l   bow el   enter ocy tes?

Resecti on  or   di sease  of   the  di stal   sm al l   bow el   can  cause  a  decr eased
r eabsor pti on  of   bi l e  aci ds,   r esul ti ng  i n  i nsuf f i ci ent  bi l e  sal t
concentr ati ons  i n  the  pr ox i m al   i ntesti ne  to  al l ow   the  nor m al
sol ubi l i zati on  and  absor pti on  of   f at.   Com pl ete  bl ock age  of   the  com m on
bi l e  duct,   as  by   pancr eati c  cancer   or   cancer   of   the  duct,   pr ev ents  bi l e
aci ds  f r om   enter i ng  the  duodenum .

Chr oni c  pancr eati ti s  or   pancr eati c  cancer   can  bl ock   the  pancr eati c  duct,
r esul ti ng  i n  decr eased  secr eti on  of   l i pase.   Incr eased  aci d  content  i n  the
duodenum ,   such  as  occur s  i n  the  Zol l i nger ­El l i son  sy ndr om e,   can
i nacti v ate  pancr eati c  l i pase  i n  the  i ntesti nal   l um en.

Decr eased  absor pti on  by   sm al l   bow el   enter ocy tes  m ay   be  caused  by
cel i ac  spr ue,   tr opi cal   spr ue,   Whi ppl e's  di sease,   sm al l   i ntesti nal
l y m phom a,   AIDS  enter opathy ,   and  sev er al   other   di seases.

4.   How   does  the  D­x y l ose  test  di f f er enti ate  pr obl em s  w i th  di gesti on  (e. g. ,
bi l e  sal t  depl eti on  and  pancr eati c  l i pase  def i ci ency )  f r om   pr obl em s  w i th
absor pti on?  N am e  one  di sor der   that  m ay   pr oduce  a  f al se­posi ti v e
r esul t.

D­x y l ose  i s  a  f i v e­car bon  sugar   that  can  be  absor bed  w i thout  the  ai d  of
bi l e  sal ts,   pancr eati c  enzy m es,   or   i ntesti nal   enzy m es.   It  shoul d  be
absor bed  nor m al l y   i f   the  sm al l   bow el   i s  i ntact.   Ther ef or e,   the  test  i s
usef ul   i n  di sti ngui shi ng  pancr eati c  enzy m e  i nsuf f i ci ency   f r om
enter ocy te  abnor m al i ti es.   How ev er ,   bacter i al   ov er gr ow th  i n  the
pr ox i m al   i ntesti ne  i s  a  condi ti on  that  can  cause  m al absor pti on  of   D­
x y l ose  w i thout  af f ecti ng  the  enter ocy te  (the  bacter i a  w i l l   consum e  the
D­x y l ose  bef or e  i t  can  be  absor bed),   ther eby   pr oduci ng  a  f al se­posi ti v e
r esul t.

5.   On  the  basi s  of   the  r esul ts  of   the  D­x y l ose  test,   w hat  test  shoul d  be
per f or m ed  now ?

The  sm al l   bow el   shoul d  be  ex am i ned,   and  ther e  ar e  tw o  appr opr i ate
w ay s  to  do  thi s:  sm al l   bow el   bi opsy   and  a  sm al l   bow el   bar i um
r adi ogr aph.   A  bi opsy   speci m en  gi v es  m or e  i nf or m ati on  about  the
m ucosa,   w her eas  the  r adi ogr aph  m ay   per m i t  better   ev al uati on  of
di v er ti cul a,   r egi onal   i l ei ti s,   or   bl i nd  l oops.

6.   Al though  the  bi opsy   f i ndi ngs  i ndi cate  cel i ac  spr ue,   w hat  other   di sor der s
coul d  pr oduce  such  a  “f l atâ€​   m ucosa?

Tr opi cal   spr ue,   soy   and  m i l k   pr otei n  al l er gy   (pr i m ar i l y   i n  chi l dr en),
di f f use  i ntesti nal   l y m phom a,   hy pogam m agl obul i nem i a,   and  the
Zol l i nger ­El l i son  sy ndr om e  can  pr oduce  a  f l at  m ucosal   l esi on  that
r esem bl es  that  of   cel i ac  spr ue.

7.   How   can  the  di agnosi s  of   cel i ac  spr ue  be  conf i r m ed?
The  di agnosi s  of   cel i ac  spr ue  can  be  conf i r m ed  by   obser v i ng  the
pati ent's  r esponse  to  a  gl uten­f r ee  di et.   Adher ence  to  a  gl uten­f r ee  di et
shoul d  br i ng  about  a
P. 166
cessati on  or   m ar k ed  r educti on  i n  the  di ar r hea  and  other   i ntesti nal
sy m ptom s,   w ei ght  gai n,   and  hi stol ogi c  i m pr ov em ent  i n  the  i ntesti nal
m ucosa.   Gl uten  i s  f ound  i n  w heat,   r y e,   bar l ey ,   and  oats,   but  not  i n  r i ce
and  cor n.

8.   If   the  D­x y l ose  test  r esul t  w as  abnor m al ,   but  the  sm al l   bow el   bi opsy
f i ndi ngs  w er e  nor m al ,   a  bacter i al   ov er gr ow th  i n  the  pr ox i m al   sm al l
i ntesti ne  m i ght  be  suspected.   How   shoul d  thi s  possi bi l i ty   be  ev al uated?

Thi s  w oul d  be  m or e  l i k el y   to  occur   i n  pati ents  w ho  hav e  had  a  sur ger y
that  r esul ted  i n  a  bl i nd  l oop  of   sm al l   i ntesti ne,   or   i n  el der l y   pati ents
w ho  ar e  m or e  l i k el y   to  hav e  m ul ti pl e  sm al l   bow el   di v er ti cul a.   A  sm al l
bow el   bar i um   r adi ogr aphi c  ex am i nati on  shoul d  r ev eal   these
abnor m al i ti es.   The  bi l e  aci d  br eath  test  coul d  be  used  to  docum ent
bacter i al   deconjugati on  of   bi l e  aci ds.   In  thi s  test,   a  r adi ol abel ed
conjugated  bi l e  aci d,   such  as  [ 1 4 C]­gl y cochol i c  aci d,   i s  gi v en  or al l y ,
and  the  am ount  and  the  ti m e  cour se  of   the  [ 1 4 C]­O 2   ex hal ed  i s
m easur ed.   N or m al l y ,   m ost  of   the  l abel ed  bi l e  aci d  i s  absor bed  i ntact  i n
the  di stal   i l eum ;  a  m i nor   am ount  r eaches  the  col on,   w her e  anaer obi c
bacter i a  cl eav e  the  gl y ci ne  m oi ety   f r om   the  chol i c  aci d  m oi ety .   The
[ 1 4 C]­O 2   r el eased  i n  the  col on  i s  absor bed  and  ex hal ed.   If   the  upper
i ntesti ne  i s  popul ated  by   ex cessi v e  num ber s  of   anaer obi c  bacter i a,   the
deconjugati on  of   [ 1 4 C]­gl y cochol i c  aci d  occur s  ear l i er   and  to  a  gr eater
degr ee  than  nor m al ,   r esul ti ng  i n  an  ear l y   and  hi gh  r i se  i n  the  ex hal ed
[ 1 4 C]­O 2   l ev el .

9.   If   thi s  pati ent's  D­x y l ose  absor pti on  test  r esul t  had  been  nor m al ,   w hat
di sor der   m i ght  y ou  suspect  and  how   shoul d  y ou  ev al uate  thi s
possi bi l i ty ?

Pancr eati c  i nsuf f i ci ency   shoul d  be  suspected  i n  pati ents  w ho  hav e  a
hi stor y   of   chr oni c  pancr eati ti s  or ,   l ess  com m onl y ,   i n  m i ddl e­aged  or
el der l y   peopl e  w ho  m ay   pr esent  w i th  a  pancr eati c  cancer   obstr ucti ng
the  pancr eati c  duct.   A  pati ent  w ho  has  m al absor pti on  and  a  hi stor y   of
pancr eati ti s  shoul d  under go  a  tr i al   of   pancr eati c  enzy m e  tr eatm ent.   If
thi s  al l ev i ates  the  di ar r hea,   the  tr i al   can  be  both  di agnosti c  and
ther apeuti c.   The  secr eti n  test  can  be  used  to  ev al uate  pancr eati c
f uncti on,   but  i t  i s  ex pensi v e  and  di f f i cul t  to  per f or m ,   so  i t  i s  r ar el y
used.   If   a  pancr eati c  cancer   i s  suspected,   an  i m agi ng  study   such  as  CT
scanni ng  or   endoscopi c  r etr ogr ade  chol angi opancr eatogr aphy   (ERCP)
shoul d  be  per f or m ed.

10.   Why   di d  the  sy m ptom s  i n  thi s  pati ent,   w ho  had  cel i ac  spr ue,   abate
w hen  she  stopped  dr i nk i ng  m i l k ?
Cel i ac  spr ue  dam ages  the  i ntesti nal   epi thel i um ,   ther eby   decr easi ng  the
am ounts  of   di gesti v e  enzy m es,   such  as  l actase,   that  ar e  nor m al l y
pr esent  i n  the  v i l l us  cel l s.

Suggested Readings
Far r el   RJ,   Kel l y   CP.   Cel i ac  spr ue  and  r ef r actor y   spr ue.   In:  Fel dm an  M,
Fr i edm an  LS,   Sl ei senger   MH,   eds.   Sl ei senger   and  For dtr an's
gastr oi ntesti nal   and  l i v er   di sease:  pathophy si ol ogy ,   di agnosi s,
m anagem ent,   7th  ed.   Phi l adel phi a:  WB  Saunder s,   2002:1817.

Hogenauer   C,   Ham m er   HF.   Mal di gesti on  and  m al absor pti on.   In:  Fel dm an
M,   Fr i edm an  LS,   Sl ei senger   MH,   eds.   Sl ei senger   and  For dtr an's
gastr oi ntesti nal   and  l i v er   di sease:  pathophy si ol ogy ,   di agnosi s,
m anagem ent,   7th  ed.   Phi l adel phi a:  WB  Saunder s,   2002:1751.

P. 167

Pancreatitis
1.   What  ar e  the  com m on  and  uncom m on  causes  of   acute  pancr eati ti s?

2.   What  pathogeneti c  m echani sm   i s  hy pothesi zed  to  be  com m on  to  these
causes  of   acute  pancr eati ti s,   and  how   does  i t  ex pl ai n  the  cl i ni cal
f eatur es  of   the  di sease?

3.   What  sy m ptom s  and  si gns  ty pi f y   acute  pancr eati ti s?

4.   What  di f f i cul ti es  m ay   be  encounter ed  i n  conf i r m i ng  the  di agnosi s  of
acute  pancr eati ti s  thr ough  the  m easur em ent  of   am y l ase  l ev el s,   and
how   m i ght  the  di agnosti c  accur acy   be  i m pr ov ed?

5.   What  cl i ni cal   and  l abor ator y   i ndi ces  can  be  used  to  assess  the
pr ognosi s  i n  a  case  of   acute  pancr eati ti s?

6.   What  ev ents  si gnal   the  dev el opm ent  of   l ocal   com pl i cati ons  of   acute
pancr eati ti s,   and  how   ar e  they   best  ev al uated?

7.   What  ar e  the  m ai nstay s  of   tr eatm ent  of   acute  pancr eati ti s,   and  w hat  i s
the  r ati onal e  f or   thei r   use?

8.   What  car di nal   f eatur e  di sti ngui shes  chr oni c  pancr eati ti s  f r om   acute
pancr eati ti s?

9.   How   does  the  eti ol ogy   of   chr oni c  pancr eati ti s  di f f er   f r om   that  of   acute
pancr eati ti s?

10.   What  ar e  the  m ai nstay s  of   tr eatm ent  of   chr oni c  pancr eati ti s?
Discussion
1.   What  ar e  the  com m on  and  uncom m on  causes  of   acute  pancr eati ti s?

The  com m on  causes  of   acute  pancr eati ti s  ar e  al cohol   (60%),   gal l stones
(25%  to  30%),   and  i di opathi c  causes.   Tabl e  4. 4  l i sts  uncom m on  causes.

2.   What  pathogeneti c  m echani sm   i s  hy pothesi zed  to  be  com m on  to  these
causes  of   acute  pancr eati ti s,   and  how   does  i t  ex pl ai n  the  cl i ni cal
f eatur es  of   the  di sease?

Autodi gesti on  i s  the  pathogeneti c  m echani sm   com m on  to  al l   the  causes
of   acute  pancr eati ti s.   Eti ol ogi c  f actor s  ar e  bel i ev ed  to  l ead  to  the
pr em atur e  acti v ati on  of   pancr eati c  pr oenzy m es  w i thi n  the  gl and.
Destr ucti on  of   the  pancr eas  by   the  acti v ated  enzy m es  l eads  to  l ocal
i njur y   (edem a,   necr osi s,   and  hem or r hage).   In  addi ti on,   the  acti v ati on
and  r el ease  of   pr oi nf l am m ator y   cy tok i nes,   v asoacti v e  pepti des,   and
enzy m es  l eads  to  the  sy stem i c  ef f ects  that  of ten  accom pany   pancr eati c
i njur y   (shock ,   di ssem i nated  i ntr av ascul ar   coagul ati on,   adul t  r espi r ator y
di str ess  sy ndr om e,   r enal   f ai l ur e,   hy per gl y cem i a,   and  hy pocal cem i a).

3.   What  sy m ptom s  and  si gns  ty pi f y   acute  pancr eati ti s?

Pai n  i s  a  char acter i sti c  sy m ptom   of   acute  pancr eati ti s  and  i s  l ocated  i n
the  m i depi gastr i c  and  per i um bi l i cal   r egi ons.   Com m onl y ,   i t  r adi ates  to
the  back   and  i s  m or e  constant  and  sustai ned  than  the  pai n  associ ated
w i th  other   abdom i nal   pr ocesses.   It  i s  of ten  m or e  i ntense  i n  the  supi ne
posi ti on  and  am el i or ated  by   si tti ng  f or w ar d.   Pati ents  m ay   ex hi bi t
m ar k ed  abdom i nal   tender ness  and  guar di ng.

N ausea  and  v om i ti ng  ar e  other   sy m ptom s.   In  thi s  setti ng,   the  abdom en
m ay   be  di stended  f r om   the  accum ul ati on  of   i ntr aabdom i nal   and
P. 168
f l ui d,   par al y ti c  i l eus,   and  chem i cal   per i toni ti s.   The  bow el   sounds  m ay
be  di m i ni shed.

Table 4­4 Common and Uncommon Causes
of Acute Pancreatitis

Postoper ati v e  causes


Af ter   endoscopi c  r etr ogr ade  chol angi opancr eatogr aphy
Tr aum a
Metabol i c  causes  (hy per tr i gl y cer i dem i a,
hy per par athy r oi di sm ,   r enal   f ai l ur e,   and  acute  f atty   l i v er
of   pr egnancy )
Her edi tar y   causes
Inf ecti ons  (m um ps,   My copl asm a,   cox sack i e  v i r us,   and
echov i r us)
Vascul i ti des  (sy stem i c  l upus  er y them atosus,   thr om boti c
thr om bocy topeni c  pur pur a,   Henoch­Schonl ei n  pur pur a,
necr oti zi ng  angi i ti s)
Am pul l a  of   Vater   obstr ucti on  (Cr ohn'sease,   duodenal
di v er ti cul a,   penetr ati ng  duodenal   ul cer ,   pancr eas
di v i sum ,   and  scor pi on  v enom )
Dr ugs
Azathi opr i ne/6­m er captopur i ne
Thi azi de  di ur eti cs
Estr ogens
Fur osem i de
Sul f onam i des  (sul f asal azi ne,   tr i m ethopr i m â
€“sul f am ethox azol e)
Tetr acy cl i ne
Methy l dopa
Sul i ndac
Val pr oate
Pentam i di ne
Di danosi ne
Or al   5­am i nosal i cy l ate  (ol sal zi ne  and  m esal am i ne)
Octr eoti de

Hy potensi on  m ay   be  pr esent  i n  as  m any   as  hal f   of   the  pati ents;  i t
r esul ts  f r om   v asodi l ati on,   m y ocar di al   depr essant  f actor ,   and  the  l oss  of
pl asm a  and  bl ood  i nto  the  r etr oper i toneum .

Less  com m on,   but  i m por tant,   f i ndi ngs  i ncl ude  per i um bi l i cal   (Cul l en's
si gn)  or   f l ank   ecchy m oses  (Gr ey   Tur ner 's  si gn).

4.   What  di f f i cul ti es  m ay   be  encounter ed  i n  conf i r m i ng  the  di agnosi s  of
acute  pancr eati ti s  thr ough  the  m easur em ent  of   am y l ase  l ev el s,   and
how   m i ght  the  di agnosti c  accur acy   be  i m pr ov ed?

Al though  the  ser um   am y l ase  l ev el   usual l y   r i ses  w i thi n  12  hour s  of   the
onset  of   pai n  and  r em ai ns  el ev ated  f or   3  to  5  day s,   a  nor m al   ser um
am y l ase  v al ue  does  not  ex cl ude  pancr eati ti s.   Spur i ousl y   nor m al   ser um
am y l ase  l ev el s  m ay   r esul t  f r om   the  r api d  cl ear ance  of   am y l ase  i nto  the
ur i ne,   and  m ay   be  seen  w i th  hy per tr i gl y cer i dem i a  and  i n  l ate­stage  (â
€œbur ned  outâ€​ )   chr oni c  pancr eati ti s.   The  m agni tude  of   the  am y l ase
el ev ati on  i n  ser um   or   ur i ne  does  not  cor r el ate
P. 169
w i th  the  sev er i ty   of   pancr eati ti s.   In  addi ti on,   hy per am y l asem i a  i s  not  a
speci f i c  f i ndi ng  f or   pancr eati ti s  because  i t  m ay   occur   i n  a  v ar i ety   of
pancr eati c  and  nonpancr eati c  di seases.   Ther e  ar e  sal i v ar y   as  w el l   as
pancr eati c­ty pe  i soam y l ases,   and  sal i v ar y   am y l ase  accounts  f or   60%  to
65%  of   the  total   am y l ase  content.   Sal i v ar y   hy per am y l asem i a  can  occur
i n  the  setti ngs  of   di abeti c  k etoaci dosi s,   al cohol i sm ,   and  m al i gnancy
(especi al l y   w i th  hepati c  m etastasi s).   Macr oam y l asem i a  occur s  w i thout
any   r el ati onshi p  to  pancr eati ti s  and  r esul ts  i n  el ev ated  ser um   (but  not
ur i ne)  am y l ase  l ev el s.

Attem pts  at  i m pr ov i ng  the  sensi ti v i ty ,   and  especi al l y   the  speci f i ci ty ,   of
the  l abor ator y ­based  di agnosi s  of   pancr eati ti s  hav e  i ncl uded
m easur em ent  of   the  r enal   am y l ase  cl ear ance  and  the  r ati o  of   r enal
am y l ase  cl ear ance  to  cr eati ni ne  cl ear ance  (C am /C cr ).   How ev er ,   the
speci f i ci ty   of   the  C am /C cr   i s  questi onabl e  because  i t  m ay   be  el ev ated  i n
the  setti ngs  of   di abeti c  k etoaci dosi s,   bur ns,   r enal   f ai l ur e,   chr oni c
hem odi al y si s,   pancr eati c  neopl asm s,   and  al cohol i c  l i v er   di sease.
Measur em ent  of   the  pancr eati c  i soam y l ase  l ev el s  has  al so  been  tr i ed.
Thi s  m ay   pr ov i de  i nf or m ati on  that  changes  the  cl i ni cal   di agnosi s  i n  20%
to  40%  of   pati ents  w i th  hy per am y l asem i a.   Measur em ent  of   the  ser um
l i pase  l ev el   i s  sl i ghtl y   l ess  sensi ti v e  f or   the  di agnosi s  of   pancr eati ti s
than  that  of   the  ser um   am y l ase  l ev el ,   but  the  l i pase  concentr ati on
r em ai ns  el ev ated  l onger   and  i s  m or e  speci f i c  than  the  am y l ase  v al ue.

5.   What  cl i ni cal   and  l abor ator y   i ndi ces  can  be  used  to  assess  the
pr ognosi s  i n  a  case  of   acute  pancr eati ti s?

A  set  of   the  ear l y   r i sk   f actor s,   k now n  as  Ranson's  cr i ter i a,   has  been
used  to  pr edi ct  the  potenti al   com pl i cati ons  and  m or tal i ty   i n  a  pati ent
w i th  acute  pancr eati ti s  (see  Tabl es  4. 4–4. 5).

The  m or tal i ty   r ate  associ ated  w i th  these  si gns  has  been  deter m i ned  as
f ol l ow s:  tw o  or   f ew er   si gns,   1%;  thr ee  or   f our   si gns,   16%;  f i v e  or   si x
si gns,   40%;  and  m or e  than  si x   si gns,   100%.

Table 4­5 Ranson's

At  adm i ssi on


Age,   ol der   than  55  y
Whi te  bl ood  cel l   count,   > 16, 000/  m m   3
Bl ood  gl ucose,   >   200  m g/dL
Ser um   l actate  dehy dr ogenase,   < 350  IU /L
Aspar tate  am i notr ansf er ase,   >   250  IU /L
Dur i ng  i ni ti al   48  hr
Hem atocr i t  decr ease,   >   10%
Bl ood  ur ea  ni tr ogen  r i se,   >   5  m g/dL
Ser um   cal ci um ,   < 8  m g/dL
Ar ter i al   par ti al   pr essur e  of   ox y gen  (Po2), < 60  m m
Hg
Base  def i ci t,   > 4  m Eq/L
Esti m ated  f l ui d  sequestr ati on,   >   6  L
P. 170
Measur em ent  of   tryps inoge n a c tiva tion pe ptide   i n  ur i ne  m ay
di sti ngui sh  m i l d  f r om   sev er e  pancr eati ti s,   but  the  test  i s  not  gener al l y
av ai l abl e.

6.   What  ev ents  si gnal   the  dev el opm ent  of   l ocal   com pl i cati ons  of   acute
pancr eati ti s,   and  how   ar e  they   best  ev al uated?

Loc a l a nd infe c tious  c omplic a tions   of   acute  pancr eati ti s  account  f or
80%  of   the  m or tal i ty   associ ated  w i th  the  di sease;  ther ef or e,   detecti on
of   these  com pl i cati ons  i s  cr uci al   i n  m i ni m i zi ng  the  l i k el i hood  of   a  f atal
outcom e.   A  pa nc re a tic  ps e udoc ys t  shoul d  be  suspected  i n  the  setti ng
of   per si stent  pai n  and  hy per am y l asem i a,   and  m ay   be  m ani f ested  as  a
pal pabl e  m ass  i n  the  upper   abdom en.   P a nc re a tic  ne c ros is   or
phl egm on,   and  pa nc re a tic  a bs c e s s   ar e  of ten  di f f i cul t  to  di sti ngui sh
because  they   both  com m onl y   cause  pr ol onged  abdom i nal   pai n  and
tender ness,   f ev er ,   l euk ocy tosi s,   and  a  pal pabl e  m ass.

A  CT  scan  w i th  or al   and  IV  contr ast  enhancem ent  i s  the  best  m ethod  f or
i m agi ng  these  com pl i cati ons.   Ex tr al um i nal   gas  m ay   be  seen  on  the
studi es  and  can  be  used  to  di sti ngui sh  pancr eati c  necr osi s  f r om
pancr eati c  abscess.   How ev er ,   i t  i s  CT­gui ded  per cutaneous  needl e
aspi r ati on  that  usual l y   al l ow s  f or   the  ear l y   di agnosi s  of   pancr eati c
i nf ecti on  and  abscess,   w hi ch  r equi r e  ei ther   per cutaneous  or   sur gi cal
dr ai nage.

7.   What  ar e  the  m ai nstay s  of   tr eatm ent  of   acute  pancr eati ti s,   and  w hat  i s
the  r ati onal e  f or   thei r   use?

By   el i m i nati ng  or al   i ntak e  (N PO),   the  neur al   and  hor m onal   sti m ul i   to
pancr eati c  ex ocr i ne  secr eti on  m ay   be  m i ni m i zed,   ther eby   l i m i ti ng  the
cy cl e  of   pancr eati c  autodi gesti on  and  i nf l am m ati on.   El i m i nati ng  f ood
i ntak e  r educes  the  v agal   sti m ul ati on  of   pancr eati c  secr eti on  and
r educes  the  del i v er y   of   aci d,   f atty   aci ds,   and  am i no  aci ds  to  the
duodenum ,   w hi ch  w oul d  el i ci t  r el ease  of   secr eti n  and  chol ecy stok i ni n.
Par enter al   nutr i ti on  i s  of ten  adm i ni ster ed,   but  enter al   nutr i ti on  thr ough
a  tube  pl aced  i n  the  jejunum   i s  pr ef er r ed  because  of   i ts  l ow er   cost  and
f ew er   com pl i cati ons.   N asogastr i c  sucti on  i s  usual l y   not  adv ocated,   but
i t  m ay   be  usef ul   i n  those  pati ents  ex per i enci ng  nausea  and  v om i ti ng
r esul ti ng  f r om   par al y ti c  i l eus.   Adequate  r epl acem ent  of   f l ui d  and
el ectr ol y te  l osses  (especi al l y   cal ci um )  stem m i ng  f r om   the
r etr oper i toneal   i nf l am m ati on  and  ex udati on  i s  essenti al .   Hy pocal cem i a
i s  bel i ev ed  to  r esul t  f r om   a  com bi nati on  of   f actor s:  hy poal bum i nem i a,
the  sequestr ati on  of   cal ci um   i n  ar eas  of   f at  necr osi s,   and  an  i nadequate
par athor m one  r esponse.   Anal gesi c  adm i ni str ati on  i s  usual l y   r equi r ed  to
contr ol   the  pai n,   w hi ch  i s  of ten  i ntense  and  pr ol onged.

8.   What  car di nal   f eatur e  di sti ngui shes  chr oni c  pancr eati ti s  f r om   acute
pancr eati ti s?

The  per m anent  destr ucti on  of   the  pancr eati c  gl and  i s  a  car di nal   f eatur e
of   chr oni c  pancr eati ti s.   Pathol ogi cal l y ,   ther e  i s  atr ophy   of   the  aci ni ,   a
l oss  of   i sl et  cel l s,   f i br osi s,   and  pl uggi ng  of   i r r egul ar   pancr eati c  ducts
by   pr otei n.   The  pr otei n  pl ugs  m ay   be  cal ci f i ed  and,   on  r adi ogr aphi c
studi es,   30%  of   pati ents  ex hi bi t  pancr eati c  cal ci f i cati on.   The  cl i ni cal
sequel ae  of   gl andul ar   destr ucti on  i ncl ude  ex ocr i ne  and  endocr i ne
i nsuf f i ci ency ,   m ani f ested  by   steator r hea  and  di abetes  m el l i tus,
r especti v el y   (the  f or m er   occur r i ng  onl y   w hen  ther e  i s  a  m or e  than  90%
r educti on  i n  ex ocr i ne  f uncti on).   Abdom i nal   pai n  i s  not
P. 171
uni f or m l y   seen  and  m ay   be  i nter m i ttent,   constant,   or   absent.   Because
of   the  aci nar   destr ucti on,   the  ser um   am y l ase  l ev el s  m ay   be  onl y   m i l dl y
el ev ated  or   nor m al .

9.   How   does  the  eti ol ogy   of   chr oni c  pancr eati ti s  di f f er   f r om   that  of   acute
pancr eati ti s?

In  w ester n  countr i es,   m ost  (appr ox i m atel y   90%)  cases  of   chr oni c
pancr eati ti s  ar e  attr i butabl e  to  al cohol i sm .   Other   possi bl e  causes
i ncl ude  m etabol i c  di sor der s  such  as  hy per cal cem i a  of   any   cause
(per haps  hy per par athy r oi di sm ),   hy per l i pi dem i a,   and  congeni tal   or
her edi tar y   condi ti ons  (pancr eas  di v i sum ,   cy sti c  f i br osi s,   and  her edi tar y
pancr eati ti s).

10.   What  ar e  the  m ai nstay s  of   tr eatm ent  of   chr oni c  pancr eati ti s?

Acute  r el apses  of   chr oni c  pancr eati ti s  m ay   r equi r e  m anagem ent


i denti cal   to  that  f or   acute  pancr eati ti s,   and  m ay   be  accom pani ed  by
pseudocy st  f or m ati on  and  pancr eati c  asci tes.   Ex ocr i ne  i nsuf f i ci ency
r esul ti ng  i n  steator r hea  and  w ei ght  l oss  i s  tr eated  w i th  or al   pancr eati c
enzy m e  r epl acem ent,   w her eas  endocr i ne  i nsuf f i ci ency   (di abetes
m el l i tus)  r equi r es  i nsul i n  ther apy .   Managem ent  of   the  chr oni c  pai n  has
been  pr obl em ati c,   and  pati ents  f r equentl y   becom e  addi cted  to  nar coti c
anal gesi cs.   The  or al   adm i ni str ati on  of   hi gh  doses  of   pancr eati c
enzy m es  m ay   r educe  the  pai n.   Sur gi cal   i nter v enti on  (gangl i onectom y ,
par ti al   and  total   pancr eatectom y ,   and  pancr eati c  duct  dr ai nage
oper ati ons)  conf er s  i nconsi stent  benef i ts  and  i s  f r aught  w i th  l ong­ter m
m or bi di ty .

Case 1
A  66­y ear ­ol d  m an  i s  adm i tted  w i th  com pl ai nts  of   pr ogr essi v el y   sev er e,
constant  upper   abdom i nal   pai n,   nausea,   and  v om i ti ng  of   48  hour s’
dur ati on.   Recentl y ,   he  has  consum ed  l ar ge  quanti ti es  of   v odk a,   but  has  no
hi stor y   of   bi l i ar y   tr act  di sease  and  i s  tak i ng  no  m edi cati ons.
He  i s  a  thi n  m an,   w i nci ng  and  cl utchi ng  hi s  abdom en.   Hi s  tem per atur e  i s
38°C  (100. 4°F);  bl ood  pr essur e,   100/60  m m   Hg;  pul se,   90  beats  per
m i nute;  and  r espi r ati ons,   18  per   m i nute.   Hi s  abdom en  i s  f l at  and  the  bow el
sounds  ar e  hy poacti v e.   Ther e  i s  m ar k ed  di r ect  tender ness  w i th  guar di ng  i n
the  m i depi gastr i um ,   but  no  per i toneal   si gns.
The  f ol l ow i ng  l abor ator y   data  ar e  gather ed:  w hi te  bl ood  cel l   count,   10, 000
cel l s/m m 3 ;  hem atocr i t,   50%;  ser um   cr eati ni ne,   1. 3  m g/dL;  total   ser um
bi l i r ubi n,   3. 4  m g/dL;  al k al i ne  phosphatase,   246  IU /L;  AST,   209  IU /L;  and
ser um   am y l ase,   741  U /L.
Pl ai n  abdom i nal   r adi ogr aphs  r ev eal   the  pr esence  of   scatter ed  ai r ­f l ui d
l ev el s,   pr edom i nantl y   i n  the  sm al l   bow el ,   but  no  cal ci f i cati on  or
subdi aphr agm ati c  f r ee  ai r .   An  abdom i nal   ul tr asound  ex am i nati on  r ev eal s  a
di l ated,   f l ui d­f i l l ed  gal l bl adder ,   a  di l ated  com m on  bi l e  duct  w i thout  def i ni te
cal cul i ,   and  a  poor l y   v i sual i zed  pancr eas  because  of   ov er l y i ng  bow el   gas.
A  nasogastr i c  tube  i s  i nser ted  and  pl aced  at  l ow   sucti on,   and  the  pati ent
r em ai ns  N PO,   r ecei v i ng  onl y   IV  f l ui ds.   Ov er   the  ensui ng  48  hour s,   he
r equi r es  r egul ar   doses  of   m eper i di ne  f or   the  contr ol   of   per si stent,   sev er e
pai n  and  i s  noted  to  hav e  a  r i se  i n  hi s  bi l i r ubi n  (8. 0  m g/dL),   al k al i ne
phosphatase  (450  IU /L),   and  AST  (375  IU /L)  l ev el s.   ERCP,   per f or m ed  on  the
thi r d  hospi tal   day ,   dem onstr ates  a  di l ated  com m on  bi l e  duct  that  taper s
sm oothl y   i n  i ts  i ntr apancr eati c  por ti on  and  contai ns  no  stones.   The
gal l bl adder   i s  di l ated  and  al so  contai ns  no  stones.   N o  pancr eatogr am   i s
obtai ned.
P. 172
The  af or em enti oned  m anagem ent  i s  conti nued,   and  total   par enter al   nutr i ti on
i s  star ted.   The  pati ent's  pai n,   abdom i nal   tender ness,   and  l i v er   test
abnor m al i ti es  gr adual l y   abate  ov er   the  subsequent  10  day s.

1.   Why   w as  an  ERCP  obtai ned?


2.   What  w as  the  cause  of   the  pati ent's  bi l i ar y   obstr ucti on?

Case Discussion
1.   Why   w as  an  ERCP  obtai ned?

The  pati ent's  l abor ator y   data  i ncl uded  abnor m al   l i v er   test  r esul ts
consi stent  w i th  chol estasi s,   and  com m on  bi l e  duct  di l ati on  w as  seen  on
the  ul tr asound  ex am i nati on.   These  f i ndi ngs  and  the  f ai l ur e  of   hi s
sy m ptom s  to  subsi de  dur i ng  the  ear l y   hospi tal   cour se  r ai sed  concer n
about  a  gal l stone  at  the  am pul l a  of   Vater   and  “gal l stone
pancr eati ti s. â€​   Per f or m i ng  an  em er gency   ERCP,   w i th  papi l l otom y   w hen
am pul l ar y   or   com m on  bi l e  duct  stones  ar e  f ound,   has  been  adv ocated
w i thi n  24  hour s  i n  pati ents  w ho  hav e  acute  bi l i ar y   pancr eati ti s.

2.   What  w as  the  cause  of   the  pati ent's  bi l i ar y   obstr ucti on?

Com pr essi on  of   the  i ntr apancr eati c  com m on  bi l e  duct  by   an  i nf l am ed
pancr eas  i s  the  cause  of   the  bi l i ar y   obstr ucti on  i n  thi s  pati ent.   Thi s  i s
show n  by   the  absence  of   gal l stones  on  the  ERCP  study   and  the  pati ent's
gr adual   i m pr ov em ent  w i th  conser v ati v e  m anagem ent  of   acute
pancr eati ti s.
Case 2
A  40­y ear ­ol d,   al cohol i c  m an  com pl ai ns  of   chr oni c  abdom i nal   pai n  and
w ei ght  l oss.   He  had  consum ed  tw o  pi nts  of   bour bon  dai l y   f or   the  l ast  10
y ear s,   unti l   4  y ear s  ago,   w hen  he  had  hi s  f i r st  epi sode  of   abdom i nal   pai n,
w hi ch  w as  char acter i zed  as  a  shar p,   conti nuous  epi gastr i c  pai n  r adi ati ng  to
the  back ,   and  associ ated  w i th  nausea  and  v om i ti ng.   He  w as  adm i tted  to  the
hospi tal ,   w her e  hi s  sy m ptom s  gr adual l y   abated  w i th  tr eatm ent,   consi sti ng  of
bow el   r est  and  IV  f l ui ds  f or   1  w eek .   Hi s  abdom i nal   r adi ogr aphs  at  that  ti m e
r ev eal ed  cal ci f i cati on  i n  the  ar ea  of   the  pancr eas.   He  subsequentl y   r educed
hi s  al cohol   i ntak e,   but  r equi r ed  r eadm i ssi on  to  the  hospi tal   on  sev er al
occasi ons  af ter   the  consum pti on  of   r el ati v el y   sm al l   quanti ti es  of   al cohol .
In  r ecent  m onths,   the  pati ent  has  l ost  25  l b  (11. 25  k g),   coi nci dent  w i th  the
passi ng  of   per si stentl y   l oose  and  occasi onal l y   gr easy   stool s.   Hi s  abdom i nal
pai n  has  becom e  constant,   and  a  m acr ocy ti c  anem i a  has  dev el oped.
The  pati ent  i s  cachecti c,   w ei ghi ng  125  l b  (56. 25  k g).   He  has  a  scaphoi d
abdom en  w i th  nor m al   bow el   sounds  and  m i l d  di r ect  tender ness  i n  the
m i depi gastr i um   i n  r esponse  to  pal pati on.   Ther e  i s  m oder ate  pedal   edem a.

Rel ev ant  l abor ator y   data  ar e:  w hi te  bl ood  cel l   count,   4, 900  cel l s/m m 3 ,   w i th
65%  segm ented  cel l s,   20%  l y m phocy tes,   and  10%  m onocy tes;  hem atocr i t,
37%;  m ean  cor puscul ar   v ol um e,   106  µm 3 ;  pr othr om bi n  ti m e,   14  seconds
(contr ol ,   12  seconds);  ser um   al bum i n,   2. 7  g/dL;  ser um   gl ucose,   nor m al ;
ser um   and  el ectr ol y tes  and  l i v er   f uncti on  tests  ar e  other w i se  nor m al ;  ser um
v i tam i n  B 1 2 ,   96  pg/m L  (nor m al ,   > 200  pg/m L);  ser um   f ol ate,   nor m al ;  and
72­hour   f ecal   f at  ex cr eti on,   42  g  (nor m al ,   < 15  g).
P. 173
The  pati ent  i s  star ted  on  a  r egi m en  of   m onthl y   v i tam i n  B 1 2   i njecti ons  and
or al   pancr eati c  enzy m es,   thr ee  capsul es  w i th  each  m eal   and  one  capsul e
w i th  snack s.   At  f i r st,   he  f ai l s  to  gai n  w ei ght  and  obser v es  no  r educti on  i n
the  f r equency   of   hi s  bow el   m ov em ents;  hi s  abdom i nal   pai n  per si sts  at  a
m oder ate  sev er i ty .   The  dose  of   enzy m es  i s  i ncr eased  to  si x   capsul es  w i th
each  m eal ,   and  he  al so  begi ns  tak i ng  ci m eti di ne  (300  m g  or al l y   f our   ti m es  a
day ).   Ov er   a  per i od  of   1  m onth,   hi s  pai n  subsi des  consi der abl y   and  he  gai ns
15  l b  (6. 75  k g).

1.   Is  i t  unusual   that  the  pati ent  had  hi s  f i r st  attack   of   pancr eati ti s  pai n
af ter   10  y ear s  of   heav y   al cohol   consum pti on,   and  at  that  ti m e  he
al r eady   had  si gns  of   chr oni c  pancr eati ti s  (pancr eati c  cal ci f i cati on)?
2.   What  i s  the  pathophy si ol ogi c  basi s  f or   v i tam i n  B 1 2   def i ci ency   i n  the
setti ng  of   chr oni c  pancr eati ti s?
3.   Why   di d  the  pati ent  begi n  to  gai n  w ei ght  onl y   af ter   hi s  pancr eati c
enzy m e  dose  w as  i ncr eased  and  ci m eti di ne  added?
4.   Why   m i ght  the  pati ent's  pai n  hav e  subsi ded  tow ar d  the  end  of   the
descr i bed  cour se?
Case Discussion
1.   Is  i t  unusual   that  the  pati ent  had  hi s  f i r st  attack   of   pancr eati ti s  pai n
af ter   10  y ear s  of   heav y   al cohol   consum pti on,   and  at  that  ti m e  he
al r eady   had  si gns  of   chr oni c  pancr eati ti s  (pancr eati c  cal ci f i cati on)?

N o.   It  i s  bel i ev ed  that  m ost  peopl e  m ust  consum e  at  l east  50  g  of
al cohol   dai l y   on  a  pr ol onged  basi s  bef or e  chr oni c  pancr eati ti s  dev el ops,
and  m ost  hav e  been  dr i nk i ng  ex cessi v el y   f or   5  to  20  y ear s  bef or e  thei r
f i r st  attack .   Al cohol ­i nduced  pancr eati ti s  i s  pr obabl y   chr oni c,   ev en  at
the  ti m e  of   the  f i r st  attack .   Pancr eati c  cal ci f i cati ons  ar e  seen  i n  25%  to
50%  of   the  pati ents  and  ar e  par ti cul ar l y   com m on  i n  al cohol i cs  w ho
hav e  chr oni c  pancr eati ti s.

2.   What  i s  the  pathophy si ol ogi c  basi s  f or   v i tam i n  B 1 2   def i ci ency   i n  the


setti ng  of   chr oni c  pancr eati ti s?

The  v i tam i n  B 1 2   def i ci ency   stem s  f r om   the  ex ocr i ne  i nsuf f i ci ency .
Pancr eati c  pr oteases  ar e  necessar y   to  cl eav e  R  pr otei n  f r om   v i tam i n
B 1 2   i n  the  pr ox i m al   i ntesti ne,   so  that  the  l atter   m ay   be  absor bed  as  a
com pl ex   w i th  i ntr i nsi c  f actor   (i n  the  ter m i nal   i l eum ).   Appr ox i m atel y
50%  of   pati ents  w i th  adv anced  pancr eati ti s  hav e  v i tam i n  B 1 2   def i ci ency
due  to  ex ocr i ne  i nsuf f i ci ency .

3.   Why   di d  the  pati ent  begi n  to  gai n  w ei ght  onl y   af ter   hi s  pancr eati c
enzy m e  dose  w as  i ncr eased  and  ci m eti di ne  added?

Pancr eati c  enzy m es  can  be  i nacti v ated  by   gastr i c  aci d,   and  thi s
i nacti v ati on  can  be  r educed  by   the  adm i ni str ati on  of   antaci ds  or
hi stam i ne  2  (H 2 )  r eceptor   antagoni sts.   In  addi ti on,   ev i dence  suggests
that  cer tai n  enzy m e  capsul es  ar e  m or e  ef f ecti v e  at  del i v er i ng  acti v e
enzy m e  to  the  sm al l   i ntesti ne  than  other s.

4.   Why   m i ght  the  pati ent's  pai n  hav e  subsi ded  tow ar d  the  end  of   the
descr i bed  cour se?

Sustai ned  pai n  r el i ef   i n  pati ents  w i th  chr oni c  pancr eati ti s  of ten  occur s
af ter   sev er al   y ear s  and  onl y   w i th  m ar k ed  pr ogr essi on  of   the  pancr eati c
ex ocr i ne  i nsuf f i ci ency ,   r ather   than  bei ng  a  r esul t  of   ther apeuti c
i nter v enti on.   How ev er ,   thi s
P. 174
pati ent's  pai n  seem ed  to  subsi de  r ather   qui ck l y   w i th  the  i nsti tuti on  of
hi gh  doses  of   pancr eati c  enzy m e  ther apy .   The  suppr essi on  of
pancr eati c  ex ocr i ne  secr eti on  has  been  accom pl i shed  i n  pati ents  w ho
r ecei v ed  i ntr aduodenal   per f usi ons  of   pancr eati c  ex tr act,   and  the  pai n  of
chr oni c  pancr eati ti s  has  been  show n  to  r espond  to  tr eatm ent  w i th  or al l y
adm i ni ster ed  pancr eati c  enzy m es  i n  som e  pati ents.

Suggested Readings
Ow y ang  C.   Chr oni c  pancr eati ti s.   In:  Yam ada  T,   Al per s  DH,   Kapl ow i tz  N ,
etal .   eds.   Tex tbook   of   gastr oenter ol ogy ,   4th  ed.   Phi l adel phi a:  Li ppi ncott
Wi l l i am s  &  Wi l k i ns,   2003:  2061.

Topazi an  M,   Gor el i ck   FS.   Acute  pancr eati ti s.   In:  Yam ada  T,   Al per s  DH,
Kapl ow i tz  N ,   etal .   eds.   Tex tbook   of   gastr oenter ol ogy ,   4th  ed.
Phi l adel phi a:  Li ppi ncott  Wi l l i am s  &  Wi l k i ns,   2003:2026.

Acute Lower Gastrointestinal Hemorrhage
1.   What  i s  one  of   the  m ost  i m por tant  di agnosi s  to  r ul e  out  i n  a  pati ent
w i th  l ar ge­v ol um e  hem atochezi a?

2.   Whi ch  m ethod  of   i m agi ng  the  GI  tr act  has  no  r ol e  i n  the  ev al uati on  of   a
pati ent  w i th  acute  l ow er   GI  bl eedi ng?

3.   What  ar e  the  tw o  m ost  com m on  causes  of   acute  m ajor   l ow er   GI


bl eedi ng?

Discussion
1.   What  i s  one  of   the  m ost  i m por tant  di agnosi s  to  r ul e  out  i n  a  pati ent
w i th  l ar ge­v ol um e  hem atochezi a?

An  upper   GI  bl eedi ng  sour ce  m ust  be  r ul ed  out  i n  ev er y   pati ent  w i th
l ar ge­v ol um e  hem atochezi a.   Low er   GI  bl eedi ng  i s  def i ned  as  bl eedi ng
f r om   a  sour ce  di stal   to  the  l i gam ent  of   Tr ei tz,   the  str uctur e  that
di v i des  the  duodenum   f r om   the  jejunum .   Appr ox i m atel y   10%  of   pati ents
w i th  upper   GI  bl eedi ng  hav e  hem atochezi a  because  of   a  r api d  r ate  of
bl ood  l oss  and  the  subsequent  r api d  tr ansi t  of   bl ood  thr ough  the  GI
tr act.   Because  the  str ategy   f or   tr eatm ent  of   an  upper   GI  hem or r hage
m ay   di f f er   dr asti cal l y   f r om   that  f or   a  l ow er   GI  hem or r hage,   f i r st  r ul i ng
out  an  upper   GI  bl eedi ng  sour ce  i s  m andator y   i n  pati ents  pr esenti ng
w i th  hem atochezi a.

The  easi est  w ay   to  ex cl ude  a  si gni f i cant  upper   GI  hem or r hage  w i th
substanti al   r el i abi l i ty   i s  thr ough  nasogastr i c  l av age  and  aspi r ati on.   The
aspi r ati on  of   bi l i ous  contents  f r om   a  nasogastr i c  tube  m ak es  i t  v er y
l i k el y   that  the  bl eedi ng  or i gi nates  f r om   a  l ow er   sour ce,   but  thi s  i s  not
an  i nf al l i bl e  f i ndi ng.   If   the  sour ce  of   bl eedi ng  r em ai ns  i n  doubt  af ter
nasogastr i c  l av age,   upper   GI  endoscopy   shoul d  be  per f or m ed.

P. 175
2.   Whi ch  m ethod  of   i m agi ng  the  GI  tr act  has  no  r ol e  i n  the  ev al uati on  of   a
pati ent  w i th  acute  l ow er   GI  bl eedi ng?

A  bar i um   enem a  ex am i nati on  shoul d  not  be  per f or m ed  i n  the  setti ng  of
acute  l ow er   GI  bl eedi ng  because  i t  has  no  ther apeuti c  potenti al ,   and
the  bar i um   i nter f er es  w i th  the  per f or m ance  of   m or e  appr opr i ate  tests,
nam el y   techneti um   Tc  99m   ( 9 9 m Tc)­l abel ed  er y thr ocy te  scanni ng,
angi ogr aphy ,   and  col onoscopy .

At  m ost  hospi tal s,   the  9 9 m Tc­l abel ed  er y thr ocy te  scan  i s  the  pr ef er r ed
nucl ear   m edi ci ne  test  f or   l ocal i zi ng  the  sour ce  of   acute  l ow er   GI
bl eedi ng.   To  per f or m   thi s,   the  pati ent's  r ed  bl ood  cel l s  ar e  l abel ed  w i th
r adi oacti v e  techneti um .   A  sci nti l l ati on  cam er a  then  tr ack s  w her e  the
l abel ed  r ed  bl ood  cel l s  col l ect  i n  the  pati ent.   The  9 9 m Tc­l abel ed
er y thr ocy te  scan  m ay   hel p  l ocal i ze  a  bl eedi ng  sour ce  to  the  gener al
r egi on  of   the  sm al l   bow el ,   r i ght  col on,   or   l ef t  col on,   ther eby   di r ecti ng
the  cour se  of   ther apy .   U nder   the  cor r ect  ci r cum stances,   thi s  scan  m ay
l ocal i ze  a  sour ce  of   bl eedi ng  at  r ates  as  l ow   as  0. 5  m L  per   m i nute.

If   ar ter i al   bl eedi ng  i s  occur r i ng  at  a  r ate  of   appr ox i m atel y   1  m L  per
m i nute,   angi ogr aphy   i s  usef ul   f or   both  di agnosi s  and  ther apy .   Once
catheter i zed,   the  bl eedi ng  ar ter y   m ay   then  be  sel ecti v el y   i nf used  w i th
v asopr essi n,   or   em bol i zed  w i th  m etal   coi l s.   Angi ogr aphy   i s  usual l y   not
usef ul   i f   the  bl eedi ng  has  stopped.

Col onoscopy ,   because  i t  m ay   y i el d  a  di agnosi s  and  pr ov i de  a  m eans  f or


del i v er i ng  ther apy ,   r egar dl ess  of   w hether   the  pati ent  i s  acti v el y
bl eedi ng,   shoul d  be  per f or m ed  bef or e  nucl ear   m edi ci ne  scans  or
angi ogr aphy   i n  m ost  pati ents  w i th  acute  l ow er   GI  bl eedi ng.   If   possi bl e,
the  l ow er   bow el   shoul d  be  r api dl y   f l ushed  w i th  a  pol y ethy l ene  gl y col
el ectr ol y te  sol uti on  bef or e  col onoscopy   i s  per f or m ed.   Wi th  m oder n
techni ques,   the  di agnosti c  accur acy   of   col onoscopy   i s  at  l east  as  good
as  that  of   angi ogr aphy ,   unl ess  the  r ate  of   bl eedi ng  i s  so  br i sk   as  to
obscur e  col onoscopi c  v i sual i zati on  com pl etel y .

3.   What  ar e  the  tw o  m ost  com m on  causes  of   acute  m ajor   l ow er   GI


bl eedi ng?

The  m ost  com m on  cause  of   m ajor   l ow er   GI  bl eedi ng  i s  di v er ti cul osi s,
accounti ng  f or   appr ox i m atel y   40%  of   al l   cases.   Di v er ti cul a  ar e
her ni ati ons  i n  the  col on  w al l   that  ar e  bel i ev ed  to  be  acqui r ed  w i th  age.
Causal   associ ati ons  betw een  l ow   di etar y   f i ber   i ntak e  and  di v er ti cul osi s
hav e  not  been  uni v er sal l y   accepted,   and  the  tr ue  eti ol ogy   i s  pr obabl y
m ul ti f actor i al .   In  di v er ti cul osi s,   as  the  col on  w al l   her ni ates,   the
i ntr am ur al   ar ter i es  (v asa  r ecta)  m ay   r uptur e,   ther eby   pr oduci ng  a  br i sk
but  pai nl ess  hem or r hage.   Al though  the  hem or r hage  stops  spontaneousl y
i n  appr ox i m atel y   80%  of   pati ents,   di v er ti cul ar   bl eedi ng  m ay   l ead  to
l i f e­thr eateni ng  bl ood  l oss,   par ti cul ar l y   i n  the  el der l y .   Di v er ti cul a  ar e
m or e  com m on  i n  the  l ef t  col on,   y et  di v er ti cul ar   bl eedi ng  m ost  of ten
or i gi nates  f r om   the  r i ght  col on.   The  di agnosi s  i s  usual l y   m ade  on  the
basi s  of   f i ndi ngs  r ev eal ed  by   ur gent  col onoscopy   or   by   angi ogr aphy .
Ther apy   w i th  sel ecti v e  angi ogr aphi c  catheter i zati on  i s  successf ul   i n
m any   cases.
Ar ter i ov enous  m al f or m ati ons  (AVMs)  or   angi ody spl asi as  i n  the  col on
ar e  the  second  m ost  com m on  cause  of   m ajor   l ow er   GI  bl eedi ng,
accounti ng  f or   appr ox i m atel y   20%  of   al l   cases.   These  v ascul ar   ectasi as
ar e  l ocated  just  beneath  the  col um nar   epi thel i um ,   and  m ost  ar e  due  to
the  degener ati v e  changes  of   agi ng.   A
P. 176
causal   associ ati on  betw een  aor ti c  stenosi s  and  col oni c  AVMs  has  been
pr oposed  but  not  def i ni tel y   establ i shed.   AVMs  ar e  usual l y   l ocated  i n  the
r i ght  col on  and  m ay   pr esent  as  ei ther   an  acute  l ow er   GI  hem or r hage  or
as  chr oni c  l ow ­v ol um e  bl eedi ng  m ani f ested  by   i r on­def i ci ency   anem i a.
If   the  bl eedi ng  i s  br i sk   and  per si stent,   angi ogr aphy   i s  usual l y   the
pr ef er r ed  m ethod  f or   m ak i ng  the  di agnosi s  and  car r y i ng  out  ther apy .   If
the  bl eedi ng  has  sl ow ed  or   stopped,   ur gent  col onoscopi c  ther apy   w i th
ther m al   cauter i zati on  or   i njecti on  i s  of ten  usef ul .

Less  com m on  causes  of   acute  l ow er   GI  bl eedi ng  ar e  col oni c  neopl asm s,
i nf l am m ator y   bow el   di sease,   i nf ecti ous  col i ti s,   and  i schem i c  col i ti s.
Ischem i c  col i ti s  usual l y   pr esents  w i th  acute,   cr am py   l ow er   abdom i nal
pai n,   the  ur ge  to  def ecate,   and  passage  of   bl oody   di ar r hea.   â
€œWater shedâ€​   ar eas  of   the  col on,   such  as  the  spl eni c  f l ex ur e  and
si gm oi d  col on,   ar e  m ost  com m onl y   i nv ol v ed  because  of   thei r   poor   bl ood
f l ow .

Case
A  70­y ear ­ol d  w om an  i s  seen  i n  the  em er gency   r oom   com pl ai ni ng  of   r ectal
bl eedi ng.   Her   f i r st  epi sode  occur r ed  appr ox i m atel y   6  hour s  ago,   w hen  she
passed  r ed  bl ood  and  cl ots.   At  f i r st  she  attr i buted  the  bl eedi ng  to  her
hem or r hoi ds,   but  she  has  had  f i v e  m or e  epi sodes  si nce,   the  l ast  of   w hi ch
w as  accom pani ed  by   a  sensati on  of   di zzi ness.   She  does  not  sm ok e  or   dr i nk
al cohol i c  bev er ages.   She  tak es  sev er al   aspi r i n  a  day   f or   the  tr eatm ent  of
ar thr i ti s.   Phy si cal   ex am i nati on  r ev eal s  a  w om an  w ho  i s  pal e  and  anx i ous.
Her   bl ood  pr essur e  and  pul se  i n  the  supi ne  posi ti on  ar e  110/70  m m   Hg  and
100  beats  per   m i nute,   r especti v el y .   When  she  stands,   her   bl ood  pr essur e
and  pul se  ar e  85/50  m m   Hg  and  130  beats  per   m i nute,   r especti v el y .   The
abdom i nal   ex am i nati on  r ev eal s  no  abnor m al   f i ndi ngs.   Rectal   ex am i nati on
r ev eal s  r ed  bl ood  i n  the  v aul t  and  no  m asses.

1.   What  ar e  the  thr ee  m ost  l i k el y   causes  of   thi s  w om an's  hem atochezi a?
2.   What  di agnosti c  and  ther apeuti c  m aneuv er s  m ust  y ou  do  w i thi n  the
f i r st  hour ?
3.   What  di agnosti c  and  ther apeuti c  tests  shoul d  y ou  consi der   doi ng  ov er
the  nex t  24  to  48  hour s?

Case Discussion
1.   What  ar e  the  thr ee  m ost  l i k el y   causes  of   thi s  w om an's  hem atochezi a?
Di v er ti cul osi s,   col oni c  AVMs,   and  upper   GI  bl eedi ng  ar e  the  m ost  l i k el y
causes  of   thi s  w om an's  bl eedi ng,   w hi ch  i s  associ ated  w i th  hem ody nam i c
i nstabi l i ty .   Hem or r hoi ds,   i nf l am m ator y   bow el   di sease,   and  col oni c
neopl asm s  r ar el y   cause  bl eedi ng  of   thi s  degr ee.

2.   What  di agnosti c  and  ther apeuti c  m aneuv er s  m ust  y ou  do  w i thi n  the  f i r st
hour ?

Thi s  w om an  ex hi bi ts  si gni f i cant  hem ody nam i c  i nstabi l i ty ,   as


dem onstr ated  by   the  or thostati c  changes  i n  her   bl ood  pr essur e  and
pul se.   You  m ust  pl ace  at  l east  tw o  l ar ge­bor e  (18­gauge)  IV  catheter s
and  star t  IV  v ol um e  ex pansi on  usi ng  cr y stal l oi d  f l ui d  (usual l y ,   0. 9%
sodi um   chl or i de  or   an  equi v al ent  of   l actated  Ri nger 's  sol uti on).   At  the
sam e  ti m e,   y ou  shoul d  dr aw   bl ood  f or   ty pi ng  and  cr oss­m atch  studi es,
hem ogr am ,   coagul ati on  studi es,   and  ser um   chem i str y   pr of i l e.   N ex t,   y ou
shoul d  pl ace  a  nasogastr i c  tube  to  obtai n  gastr i c  contents  and
deter m i ne  w hether
P. 177
ther e  i s  an  upper   GI  bl eedi ng  sour ce.   The  aspi r ati on  of   bl ood  shoul d
pr om pt  str ong  consi der ati on  f or   per f or m i ng  em er gency   upper   GI
endoscopy .

3.   What  di agnosti c  and  ther apeuti c  tests  shoul d  y ou  consi der   doi ng  ov er
the  nex t  24  to  48  hour s?

If   the  bl eedi ng  sl ow s  or   stops,   r api d  GI  l av age  w i th  pol y ethy l ene  gl y col
el ectr ol y te  sol uti on  f ol l ow ed  by   col onoscopy   i s  usual l y   the  best  test  f or
di agnosi s,   w i th  a  y i el d  of   40%  to  50%  i n  thi s  setti ng.   Col onoscopy   can
sti l l   be  usef ul   i f   the  bl eedi ng  i s  per si stentl y   br i sk ;  how ev er ,   i n  thi s
si tuati on,   angi ogr aphy   i s  the  pr ef er r ed  test  i n  m any   hospi tal s.   If
col onoscopy   and  angi ogr aphy   f ai l   to  i denti f y   the  sour ce  of   the  bl eedi ng,
a  9 9 m Tc­l abel ed  er y thr ocy te  scan  m ay   hel p  l ocal i ze  the  sour ce.

Suggested Readings
El ta  GH.   Appr oach  to  the  pati ent  w i th  gr oss  gastr oi ntesti nal   bl eedi ng.   In:
Yam ada  T,   Al per s  DH,   Kapl ow i tz  N ,   etal .   eds.   Tex tbook   of
gastr oenter ol ogy ,   4th  ed.   Phi l adel phi a:  Li ppi ncott  Wi l l i am s  &  Wi l k i ns,
2003:698.

Zuccar o  G  Jr .   Am er i can  Col l ege  of   Gastr oenter ol ogy .   Pr acti ce


Par am eter s  Com m i ttee.   Managem ent  of   the  adul t  pati ent  w i th  acute
l ow er   gastr oi ntesti nal   bl eedi ng.   Am   J  Gastr oenter ol   1998;93:1202.

Acute Upper Gastrointestinal Hemorrhage
1.   Is  m easur em ent  of   the  hem ogl obi n  concentr ati on  and  hem atocr i t  the
best  w ay   to  deter m i ne  the  sev er i ty   of   GI  bl eedi ng?  Why   or   w hy   not?

2.   Is  esophagi ti s  a  com m on  cause  of   sev er e  upper   GI  bl eedi ng?

3.   Is  β­adr enor eceptor   bl ock ade  a  tr eatm ent  opti on  f or   acute  v ar i ceal
bl eedi ng?  If   so,   w hy ?  If   not,   w hat  ar e  som e  tr eatm ent  opti ons?

Discussion
1.   Is  m easur em ent  of   the  hem ogl obi n  concentr ati on  and  hem atocr i t  the
best  w ay   to  deter m i ne  the  sev er i ty   of   GI  bl eedi ng?  Why   or   w hy   not?

N o.   The  best  w ay   to  deter m i ne  the  sev er i ty   of   GI  bl eedi ng  i s  to
m easur e  the  v i tal   si gns  w i th  the  pati ent  i n  the  supi ne  and  standi ng
posi ti ons.   If   or thostati c  changes  i n  the  v i tal   si gns  (postur al
hy potensi on)  occur   i n  the  setti ng  of   GI  bl eedi ng,   they   usual l y   i ndi cate
at  l east  a  20%  l oss  i n  the  total   bl ood  v ol um e.   Thi s  f i ndi ng  m andates
i m m edi ate  IV  v ol um e  ex pansi on  and  pr epar ati on  f or   bl ood  tr ansf usi on.
Other   phy si cal   f i ndi ngs  associ ated  w i th  sev er e  bl ood  l oss  ar e  r esti ng
tachy car di a  and  hy potensi on,   pal l or ,   and  agi tati on.

The  i ni ti al   bl ood  count  (hem ogr am )  obtai ned  f r om   an  acutel y   bl eedi ng
pati ent  i s  a  v er y   poor   r ef l ecti on  of   the  am ount  of   bl ood  l ost.   To  be
accur ate,   the  bl ood  count  m ust  be  obtai ned  w hen  the  pati ent's
i ntr av ascul ar   v ol um e  i s  nor m al .   Af ter   an  acute  l oss  of   bl ood,
r eequi l i br ati on  m ay   tak e  up  to  72  hour s.   The  IV  adm i ni str ati on  of
cr y stal l oi d  hastens  thi s  pr ocess  (of ten  k now n  as  hem odi l uti on).

P. 178
Pl acem ent  of   a  nasogastr i c  tube  i s  v er y   hel pf ul   i n  the  i ni ti al
assessm ent  of   a  pati ent  w i th  GI  bl eedi ng,   but  the  f i ndi ngs  y i el ded  can
be  m i sl eadi ng.   The  absence  of   bl oody   aspi r ate  m ay   suggest  that
bl eedi ng  has  stopped  f or   the  m om ent,   y et  the  am ount  of   bl ood  al r eady
l ost  m ay   be  l i f e  thr eateni ng.   Conv er sel y ,   up  to  15%  of   pati ents  w i th  an
acti v el y   bl eedi ng  upper   GI  sour ce  m ay   hav e  a  nonbl oody   gastr i c
aspi r ate.   The  pr esence  of   bi l e  i n  the  gastr i c  aspi r ate  of f er s  som e
r eassur ance  that  an  upper   GI  bl eed  has  stopped;  how ev er ,   up  to  50%
of   phy si ci ans  m i sjudge  the  pr esence  or   absence  of   a  bi l i ous  aspi r ate.
The  per si stence  of   bl oody   gastr i c  aspi r ate  i ndi cates  si gni f i cant  ongoi ng
bl eedi ng.

Mel ena  i s  pr oduced  w hen  hem ogl obi n  i s  degr aded  by   bacter i a  i n  the  GI
tr act,   and  m ay   or i gi nate  f r om   ei ther   an  upper   or   l ow er   GI  sour ce.   The
i ngesti on  of   appr ox i m atel y   100  to  200  m L  of   bl ood  i s  enough  to  cause
m el ena;  hence,   the  pr esence  of   m el ena  al one  does  not  necessar i l y
m ean  the  pati ent  has  had  a  substanti al   l oss  of   bl ood.   On  the  other
hand,   f r equent  epi sodes  of   m el ena  i ndi cate  si gni f i cant  bl eedi ng.
Hem atochezi a  r esul ti ng  f r om   an  upper   GI  bl eedi ng  sour ce  i ndi cates
m assi v e  bl eedi ng.
2.   Is  esophagi ti s  a  com m on  cause  of   sev er e  upper   GI  bl eedi ng?

N o.   Esophagi ti s  accounts  onl y   f or   appr ox i m atel y   8%  of   al l   cases  of


upper   GI  bl eedi ng.   It  i s  usual l y   caused  by   gastr oesophageal   r ef l ux   of
aci d,   but  m ay   al so  be  caused  by   i nf ecti ous  agents  such  as  Candi da
al bi cans,   her pes  si m pl ex   v i r us,   and  cy tom egal ov i r us.   Sev er e  bl eedi ng
r esul ti ng  f r om   esophagi ti s  i s  r ar e  and  usual l y   occur s  i n  the  setti ng  of
an  al r eady   hospi tal i zed  and  cr i ti cal l y   i l l   pati ent,   especi al l y   dur i ng
m echani cal   r espi r ati on.

P e ptic  ulc e r dis e a s e ,   w hi ch  accounts  f or   40%  to  50%  of   al l   the  cases
of   upper   GI  bl eedi ng,   i s  di scussed  i n  the  nex t  secti on.

Va ric e a l ble e ding  accounts  f or   10%  to  30%  of   al l   cases  of   upper   GI


bl eedi ng.   Var i ces  ar e  a  com pl i cati on  of   por tal   hy per tensi on,   the  cause
of   w hi ch  m ay   be  cl assi f i ed  as  pr ehepati c  (por tal   v ei n  obstr ucti on),
hepati c  (ci r r hosi s),   or   posthepati c  (thr om bosi s  of   the  hepati c  v ei ns  or
i nf er i or   v ena  cav a).   The  m ost  com m on  cause  of   por tal   hy per tensi on  i n
the  U . S.   popul ati on  i s  ethanol ­i nduced  ci r r hosi s,   al though  hepati ti s  Câ
€“associ ated  ci r r hosi s  i s  an  i ncr easi ngl y   com m on  cause.   The  6­w eek
m or tal i ty   r ate  associ ated  w i th  bl eedi ng  v ar i ces  i s  appr ox i m atel y   40%,
and  pati ents  w i th  v ar i ceal   bl eedi ng  shoul d  be  m anaged  i n  an  i ntensi v e
car e  uni t.   These  pati ents  of ten  hav e  concur r ent  m edi cal   pr obl em s,   such
as  a  coagul opathy   and  hepati c  encephal opathy .

Ma llory­W e is s  te a rs   ar e  par ti al ­thi ck ness  m ucosal   l acer ati ons  near
the  gastr oesophageal   juncti on,   usual l y   caused  by   f or cef ul   r etchi ng,
of ten  i n  the  setti ng  of   ethanol   i ngesti on.   These  tear s  account  f or
appr ox i m atel y   10%  of   upper   GI  hem or r hages.   Most  Mal l or y ­Wei ss  tear s
stop  bl eedi ng  spontaneousl y .   Per si stent  bl eedi ng  can  be  tr eated  w i th
ei ther   endoscopi c  hem ostasi s  or   angi ogr aphi c  em bol i zati on.

Si gni f i cant  acute  and  chr oni c  bl ood  l oss,   som eti m es  l eadi ng  to  i r on­
def i ci ency   anem i a,   can  occur   f r om   gastr i c  er osi ons  or   ul cer s  associ ated
w i th  l ar ge  sl i di ng  hi atal   her ni as.   These  l esi ons,   som eti m es  cal l ed
Cam er on  l esi ons,   w hi ch  ar e  usual l y   l ocated  on  the  cr ests  of   m ucosal
f ol ds  at  or   near   the  l ev el   of
P. 179
the  di aphr agm ,   seem   to  r esul t  f r om   the  r i di ng  m oti on  of   the  her ni ated
stom ach  i n  and  out  of   the  chest  dur i ng  r espi r ati on.

3.   Is  β­adr enor eceptor   bl ock ade  a  tr eatm ent  opti on  f or   acute  v ar i ceal
bl eedi ng?  If   so,   w hy ?  If   not,   w hat  ar e  som e  tr eatm ent  opti ons?

N o.   β­Adr enor eceptor   bl ock er s,   such  as  pr opr anol ol ,   pr oduce  a


negati v e  chr onotr opi c  and  i notr opi c  ef f ect.   The  use  of   such  dr ugs  i n  an
acutel y   bl eedi ng  pati ent,   w ho  i s  dependi ng  on  an  adr ener gi c  r esponse
to  m ai ntai n  an  adequate  bl ood  pr essur e,   i s  ther ef or e  contr ai ndi cated.
Som e  ev i dence  suppor ts  the  use  of   β­adr enor eceptor   bl ock er s  i n
sel ected  pati ents  to  r educe  the  r i sk   of   r ecur r ent  v ar i ceal   bl eedi ng  af ter
the  acute  hem or r hage  has  been  w el l   contr ol l ed.   Al though  the  com pl ete
m echani sm   of   acti on  of   these  dr ugs  i s  y et  to  be  del i neated,   they   ar e
thought  to  ex er t  thei r   benef i ci al   ef f ect  i n  par t  by   l ow er i ng  the  por tal
pr essur e.

Other   phar m acol ogi c  appr oaches  to  the  contr ol   of   acute  v ar i ceal
bl eedi ng  ar e  the  use  of   par enter al   v asopr essi n  or   som atostati n.
Al though  v asopr essi n  has  been  com m onl y   used,   i ts  ef f i cacy   has  not
been  f i r m l y   establ i shed  and  i t  has  associ ated  car di ov ascul ar   si de
ef f ects.   The  dr ug's  m echani sm   of   acti on  i s  thought  to  be  spl anchni c
ar ter i ol ar   v asoconstr i cti on,   r esul ti ng  i n  decr eased  por tal   pr essur e.
Incr easi ngl y ,   som atostati n  or   i ts  l onger   acti ng  anal og,   octr eoti de,   has
becom e  popul ar   f or   the  contr ol   of   v ar i ceal   bl eedi ng,   and  al so  because
of   i ts  pur por ted  l ow er i ng  of   por tal   pr essur e.   Som atostati n  has  been
show n  to  be  as  ef f ecti v e  as,   or   m or e  ef f ecti v e  than,   v asopr essi n  i n
contr ol l i ng  acute  v ar i ceal   hem or r hage.

Endoscopi c  hem ostasi s  can  be  achi ev ed  by   e ndos c opic  inje c tion
s c le rothe ra py  or   e ndos c opic  va ric e a l liga tion.   The  l atter   i s  now   the
m or e  popul ar   techni que  because  i t  i s  as  ef f ecti v e  as  scl er other apy   and
consi der abl y   saf er .

If   endoscopi c  techni ques  ar e  unav ai l abl e  or   unsuccessf ul ,   acute


hem ostasi s  m ay   be  achi ev ed  w i th  bal l oon  tam ponade  dev i ces.   Al though
the  tam ponade  accom pl i shed  w i th  dev i ces  such  as  the  Sengstak en­
Bl ak em or e  tube  or   the  Mi nnesota  tube  i s  ef f ecti v e  i n  appr ox i m atel y
40%  to  90%  of   pati ents,   the  r ebl eedi ng  r ate  associ ated  w i th  thei r   use
i s  appr ox i m atel y   50%.   Ther e  i s  al so  an  appr ox i m atel y   30%  r ate  of
ser i ous  com pl i cati ons,   such  as  aspi r ati on  pneum oni a  or   esophageal
r uptur e,   r esul ti ng  f r om   the  use  of   these  tubes;  ther ef or e,   thei r   use  i s
consi der ed  a  tem por ar y   m easur e  onl y .

Inter v enti onal   r adi ol ogi c  techni ques,   such  as  the  angi ogr aphi c
em bol i zati on  of   v ar i ces  or   the  pl acem ent  of   a  tr ansjugul ar   i ntr ahepati c
por tosy stem i c  stent  shunt  (TIPS),   m ay   al so  be  usef ul   i f   endoscopi c
hem ostasi s  f ai l s  or   i s  unav ai l abl e.   The  sur gi cal   cr eati on  of   a
por tosy stem i c  shunt  i n  an  acutel y   bl eedi ng  pati ent  i s  associ ated  w i th
m or tal i ty   r ates  of   50%  to  80%,   and  has  f al l en  out  of   f av or .
Fur ther m or e,   such  shunts  m ak e  l i v er   tr anspl antati on  techni cal l y   m or e
di f f i cul t.

Case
A  42­y ear ­ol d  m an  i s  br ought  to  the  em er gency   r oom   by   am bul ance  af ter   an
epi sode  of   hem atem esi s  and  sy ncope  at  a  l ocal   bar .   He  has  not  had  pr ev i ous
GI  bl eedi ng.   He  r egul ar l y   tak es  aspi r i n  f or   the  r el i ef   of   chr oni c  back   pai n.
Dur i ng  y our   i nter v i ew ,   he  passes  sev er al   l i qui d,   m ar oon  stool s.   Phy si cal
ex am i nati on  r ev eal s  a  supi ne  bl ood  pr essur e  and  pul se  of   120/75  m m   Hg
and  110  beats  per   m i nute,   r especti v el y .   When  y ou  m ak e  hi m
P. 180
si t  upr i ght  he  com pl ai ns  of   f eel i ng  f ai nt  and  hi s  sy stol i c  pr essur e  dr ops  to
90  m m   Hg  by   pal pati on.   Hi s  abdom en  i s  nontender   and  di stended.   Shi f ti ng
dul l ness  i s  el i ci ted  and  the  spl een  ti p  i s  pal pabl e.   The  i ni ti al   hem ogl obi n  i s
15  g/dL  and  the  hem atocr i t  i s  45%.

1.   How   do  y ou  k now   thi s  m an  has  l ost  a  si gni f i cant  am ount  of   bl ood?
2.   What  ar e  the  m ost  l i k el y   causes  of   thi s  m an's  upper   GI  bl eedi ng,   and
w hat  shoul d  the  nex t  di agnosti c  step  be?

Case Discussion
1.   How   do  y ou  k now   thi s  m an  has  l ost  a  si gni f i cant  am ount  of   bl ood?

He  has  si gni f i cant  or thostati c  changes  i n  hi s  v i tal   si gns,   i ndi cati ng  at
l east  a  20%  l oss  of   total   bl ood  v ol um e.   The  hem ogl obi n  concentr ati on
or   the  hem atocr i t  m ay   be  f al sel y   r eassur i ng  i n  the  setti ng  of   acute  GI
bl eedi ng.

2.   What  ar e  the  m ost  l i k el y   causes  of   thi s  m an's  upper   GI  bl eedi ng,   and
w hat  shoul d  the  nex t  di agnosti c  step  be?

The  m ost  l i k el y   causes  of   such  sev er e  upper   GI  bl eedi ng  ar e  pepti c
ul cer   di sease  and  esophageal   or   gastr i c  v ar i ces,   the  l atter   r esul ti ng
f r om   por tal   hy per tensi on.   Thi s  m an  has  r i sk   f actor s  f or   both  condi ti ons.
Ther e  i s  no  w ay   to  di sti ngui sh  one  f r om   the  other   based  on  the  hi stor y
and  ex am i nati on  f i ndi ngs.   Af ter   hem ody nam i c  stabi l i zati on,   em er gency
esophagogastr oduodenoscopy   (EGD)  shoul d  be  per f or m ed  f or   di agnosi s
and,   i f   i ndi cated,   to  car r y   out  acute  hem ostasi s.

Suggested Readings
Bar k un  A,   Bar dou  M,   Mar shal l   JK.   N onv ar i ceal   U pper   GI  Bl eedi ng
Consensus  Conf er ence  Gr oup.   Consensus  r ecom m endati ons  f or   m anagi ng
pati ents  w i th  nonv ar i ceal   upper   gastr oi ntesti nal   bl eedi ng.   Ann  Inter n  Med
2003;139:843.

El ta  GH.   Appr oach  to  the  pati ent  w i th  gr oss  gastr oi ntesti nal   bl eedi ng.   In:
Yam ada  T,   Al per s  DH,   Kapl ow i tz  N ,   etal .   eds.   Tex tbook   of
gastr oenter ol ogy ,   4th  ed.   Phi l adel phi a:  Li ppi ncott  Wi l l i am s  &  Wi l k i ns,
2003:698.

Peptic Ulcer Disease
1.   What  ar e  the  m ajor   r i sk   f actor s  f or   the  dev el opm ent  of   pepti c  ul cer s?

2.   Is  di etar y   adher ence  to  bl and  m eal s  and  m i l k   an  accepted  tr eatm ent  of
pepti c  ul cer   di sease?  If   not,   w hat  shoul d  the  tr eatm ent  be?

Discussion
1.   What  ar e  the  m ajor   r i sk   f actor s  f or   the  dev el opm ent  of   pepti c  ul cer s?

The  m ajor   r i sk   f actor s  f or   the  dev el opm ent  of   pepti c  ul cer   di sease  ar e
ci gar ette  sm ok i ng,   the  i ngesti on  of   nonster oi dal   anti i nf l am m ator y   dr ugs
(N SAIDs),   and  a  f am i l y   hi stor y   of   pepti c  ul cer .   Pepti c  ul cer s  ar e
thought  to  f or m
P. 181
w hen  the  ef f ects  of   gastr i c  aci d  and  pepsi n  ov er w hel m   the  pr otecti v e
m ucosal   bar r i er .   Di seases  such  as  the  Zol l i nger ­El l i son  sy ndr om e
i ncr ease  the  secr eti on  of   gastr i c  aci d.   Other   f actor s  pr om ote  the
br eak dow n  of   the  m ucosal   bar r i er .

U l cer s  ar e  tw i ce  as  l i k el y   to  dev el op  i n  ci gar ette  sm ok er s  than  i n


nonsm ok er s.   In  addi ti on,   ul cer s  heal   m or e  sl ow l y   and  ar e  m or e  l i k el y
to  r ecur   i n  sm ok er s.   The  m echani sm   r esponsi bl e  f or   ci gar ette  sm ok e's
ul cer ogeni c  ef f ect  i s  not  com pl etel y   under stood.   N SAIDs  di sr upt  the
m ucus–bi car bonate  bar r i er ,   al l ow i ng  aci d  to  dam age  the  under l y i ng
m ucosa.   The  GI  com pl i cati ons  of   N SAIDs  ar e  a  m ajor   cause  of   upper   GI
bl eedi ng  and  per f or ati on,   par ti cul ar l y   i n  el der l y   w om en,   and  ar e
r esponsi bl e  f or   a  tw o­  to  thr eef ol d  i ncr eased  m or tal i ty   r i sk   i n  l ong­
ter m   user s  of   N SAIDs.   The  com bi ned  use  of   N SAIDs  and  cor ti coster oi ds
appear s  to  i ncr ease  the  r i sk   ev en  f ur ther .

Peopl e  w ho  hav e  f i r st­degr ee  r el ati v es  w i th  pepti c  ul cer s  hav e  thr ee
ti m es  the  r i sk   of   acqui r i ng  ul cer s  com par ed  w i th  the  gener al
popul ati on.   The  r i sk   i s  ev en  hi gher   f or   the  i denti cal   tw i n  of   a  pati ent
w i th  ul cer   di sease.

Inf ecti on  of   the  gastr i c  m ucosa  by   Hel i cobacter   py l or i   i s  str ongl y
associ ated  w i th  l ow er   r ates  of   duodenal   ul cer   heal i ng  and  w i th  hi gher
r ates  of   ul cer   r ecur r ence.   The  ex act  m anner   i n  w hi ch  H.   py l or i   i nf ecti on
pr om otes  ul cer s  i s  not  k now n.

N o  concl usi v e  ev i dence  l i nk s  di etar y   substances,   i ncl udi ng  ethanol ,


caf f ei ne,   and  spi cy   f oods,   w i th  the  dev el opm ent  of   pepti c  ul cer s.
Si m i l ar l y ,   al though  a  cr i ti cal l y   i l l   hospi tal i zed  pati ent  m ay   hav e  str ess
ul cer s,   env i r onm ental   str essor s  at  hom e  or   at  w or k   hav e  not  been
concl usi v el y   l i nk ed  w i th  the  dev el opm ent  of   pepti c  ul cer s.

2.   Is  di etar y   adher ence  to  bl and  m eal s  and  m i l k   an  accepted  tr eatm ent  of
pepti c  ul cer   di sease?  If   not,   w hat  shoul d  the  tr eatm ent  be?

N o.   Bef or e  the  adv ent  of   m oder n  phar m acol ogi c  ther apy ,   the  tr eatm ent
of   ul cer   di sease  w i th  f r equent  bl and  m eal s  and  m i l k   w as  w i del y
accepted.   U nf or tunatel y ,   such  tr eatm ent  actual l y   i ncr eases  the
pr oducti on  of   gastr i c  aci d  and  does  not  accel er ate  ul cer   heal i ng.
H 2   r eceptor   antagoni sts,   of   w hi ch  ci m eti di ne  w as  the  f i r st  agent
r el eased  f or   use,   ar e  w i del y   accepted  as  saf e  and  ef f ecti v e  f or   the
tr eatm ent  of   pepti c  ul cer s.   These  agents  di r ectl y   i nhi bi t  hi stam i ne­
sti m ul ated  gastr i c  aci d  secr eti on  and  i ndi r ectl y   i nhi bi t  the  hi stam i ne­
potenti ated,   gastr i n­sti m ul ated  aci d  secr eti on.   When  gi v en  i n  suf f i ci ent
doses,   the  v ar i ous  H 2   r eceptor   antagoni sts  act  equal l y   w el l ,   w i th
duodenal   ul cer   heal i ng  r ates  of   75%  af ter   4  w eek s,   and  85%  to  95%
af ter   8  w eek s  of   ther apy .   The  sel ecti on  of   a  par ti cul ar   agent  shoul d  be
deter m i ned  by   the  pati ent's  abi l i ty   to  com pl y   w i th  the  dosi ng  r egi m en,
as  w el l   as  the  cost  per   dose.

Pr oton  pum p  i nhi bi tor s,   such  as  om epr azol e,   and  esom epr azol e,   ar e
concentr ated  i n  the  hi ghl y   aci di c  env i r onm ent  of   the  par i etal   cel l
secr etor y   canal i cul i .   When  acti v ated  by   pr otonati on,   these  agents
cov al entl y   bi nd  to  H + /K +   AT­Pase,   ther eby   causi ng  i r r ev er si bl e
i nhi bi ti on  of   the  enzy m e  and  a  90%  to  99%  suppr essi on  of   gastr i c  aci d
pr oducti on  w i thi n  24  hour s.   At  doses  of   20  to  40  m g  per   day ,
om epr azol e  achi ev es  m or e  r api d  pai n  r el i ef
P. 182
and  f aster   heal i ng  of   pepti c  ul cer s  than  do  standar d  doses  of   H 2
r eceptor   antagoni sts.   Pr oton  pum p  i nhi bi tor s  ar e  the  tr eatm ent  of
choi ce  f or   pati ents  w i th  nonsur gi cal l y   cor r ectabl e  Zol l i nger ­El l i son
sy ndr om e.   These  agents  hav e  di spl ay ed  an  ex cel l ent  shor t­ter m   saf ety
pr of i l e,   and,   w i th  i ncr easi ng  use,   thei r   l ong­ter m   r i sk   seem s  l ess  than
i ni ti al l y   f ear ed.

Sucr al f ate  i s  an  al um i num   sal t  of   sul f ated  sucr ose.   When  pl aced  i n  an
aci di c  env i r onm ent,   i t  bi nds  tenaci ousl y   to  ul cer s  and  pr om otes
heal i ng.   It  has  no  ef f ect  on  aci d  secr eti on  and  has  m i ni m al   aci d­
neutr al i zi ng  ef f ects.   The  enti r e  m echani sm   of   sucr al f ate's  benef i ci al
acti ons  has  not  been  deter m i ned.   Sucr al f ate  appear s  to  be  as  ef f ecti v e
as  H 2   r eceptor   antagoni sts  i n  pr om oti ng  the  heal i ng  of   acute  pepti c
ul cer s.   Its  sy stem i c  absor pti on  i s  m i ni m al ,   al though  i ts  l ong­ter m
ef f ects  on  al um i num   deposi ti on  ar e  unk now n.   Its  pr i m ar y   si de  ef f ect  i s
dose­r el ated  consti pati on.

Antaci ds  ar e  al so  ef f ecti v e  i n  pr om oti ng  the  heal i ng  of   gastr i c  and
duodenal   ul cer s.   Fr equent  dosi ng  i s  usual l y   r equi r ed  to  achi ev e
ef f ecti v eness  equal   to  that  of   H 2   r eceptor   antagoni sts.   Such  a  dosi ng
schedul e  of ten  r esul ts  i n  poor   pati ent  com pl i ance,   not  to  m enti on  the
si de  ef f ect  of   di ar r hea  associ ated  w i th  the  use  of   m agnesi um ­
contai ni ng  antaci ds.

Ther e  i s  no  ev i dence  to  suppor t  the  use  of   these  agents  i n  v ar i ous
com bi nati ons  f or   the  pr i m ar y   tr eatm ent  of   pepti c  ul cer s.   Com bi nati on
ther apy   w i th  anti bi oti cs,   aci d­suppr essi v e  m edi cati ons,   and  bi sm uth
com pounds  i s  ef f ecti v e  i n  heal i ng  duodenal   ul cer s  associ ated  w i th  H.
py l or i   i nf ecti on,   and  i n  pr ev enti ng  the  r ecur r ence  of   such  ul cer s.
Case
A  50­y ear ­ol d  m an  has  had  r ecur r ent  and  at  ti m es  sev er e  epi gastr i c
abdom i nal   pai n  f or   the  l ast  sev er al   y ear s.   Antaci ds  hav e  gi v en  hi m
sy m ptom ati c  r el i ef .   The  m ost  r ecent  epi sode  began  1  w eek   ago  and  has  not
r esponded  com pl etel y   to  antaci ds.   The  pai n  now   w ak es  hi m   up  at  ni ght.   He
sm ok es  one  pack   of   ci gar ettes  per   day ,   and  he  tak es  aspi r i n  sev er al   ti m es  a
w eek .   Hi s  f am i l y   hi stor y   i s  unr em ar k abl e.   Phy si cal   ex am i nati on  r ev eal s
m oder ate  epi gastr i c  tender ness  w i thout  ev i dence  of   a  m ass.   The  stool   i s
br ow n  and  posi ti v e  f or   occul t  bl ood.

1.   What  ar e  thi s  m an's  r i sk   f actor s  f or   pepti c  ul cer   di sease?


2.   What  di agnosti c  tests  shoul d  y ou  consi der ?
3.   When  w oul d  y ou  consi der   tr eatm ent  f or   H.   py l or i ?

Case Discussion
1.   What  ar e  thi s  m an's  r i sk   f actor s  f or   pepti c  ul cer   di sease?

Hi s  sm ok i ng  and  N SAID  i ngesti on  ar e  both  r i sk   f actor s  f or   pepti c  ul cer


di sease.

2.   What  di agnosti c  tests  shoul d  y ou  consi der ?

If   the  pati ent  w er e  y ounger   than  40  y ear s,   had  onl y   m i l d  and
i nter m i ttent  sy m ptom s,   and  had  no  ev i dence  of   sy stem i c  di sease  or
r i sk   f actor s  f or   m al i gnancy ,   a  tr i al   of   em pi r i c  anti ­ul cer   ther apy
w i thout  pr i or   di agnosti c  tests  w oul d  be  acceptabl e.   Other w i se,   ei ther
EGD  or   a  doubl e­contr ast  upper   GI  r adi ogr aphi c  ser i es  i s  r ecom m ended.
When  ther e  i s  a  possi bi l i ty   of   m al i gnancy   and  i f   bi opsy   speci m ens
P. 183
ar e  needed,   EGD  i s  consi der ed  super i or   to  r adi ogr aphy   f or   the  pur pose
of   di agnosi s.   Because  the  m an  descr i bed  i s  ol der   than  40  y ear s,
sm ok es  ci gar ettes,   has  occul t  bl ood  i n  the  stool ,   and  i s  hav i ng
i ncr easi ngl y   sev er e  pai n,   a  di agnosti c  w or k up  (pr ef er abl y   EGD)  r ather
than  em pi r i c  ther apy   i s  r ecom m ended.

3.   When  w oul d  y ou  consi der   tr eatm ent  f or   H.   py l or i ?

Er adi cati on  of   H.   py l or i   i s  usual l y   adv ocated  w hen  associ ated  w i th


duodenal   ul cer ,   and  r esul ts  i n  a  dr am ati c  r educti on  i n  ul cer   r ecur r ence.
Inf ecti on  can  be  dem onstr ated  by   endoscopi c  bi opsy ,   ser ol ogy ,   or
r adi oi sotope  br eath  test  f i ndi ngs.   A  m ul ti pl e­dr ug  r egi m en  i s  r equi r ed
f or   r el i abl e  er adi cati on  of   the  or gani sm s.   A  com m onl y   used
com bi nati on  has  been  that  of   a  bi sm uth­contai ni ng  com pound,
tetr acy cl i ne,   m etr oni dazol e,   and  ei ther   a  pr oton  pum p  i nhi bi tor   or   H 2
r eceptor   antagoni st.   Better   pati ent  com pl i ance  and  equal   ef f i cacy   hav e
been  r epor ted  w i th  com bi nati ons  of   cl ar i thr om y ci n,   am ox i ci l l i n,
bi sm uth,   and  a  pr oton  pum p  i nhi bi tor   or   H 2   r eceptor   antagoni st.
Suggested Readings
Del   Val l e  J,   Chey   WD,   Schei m an  JM.   Aci d  pepti c  di sor der s.   In:  Yam ada  T,
Al per s  DH,   Kapl ow i tz  N ,   etal .   eds.   Tex tbook   of   gastr oenter ol ogy ,   4th  ed.
Phi l adel phi a:  Li ppi ncott  Wi l l i am s  &  Wi l k i ns,   2003:  1321.

Spechl er   SJ.   Pepti c  ul cer   and  i ts  com pl i cati ons.   In:  Fel dm an  M,   Fr i edm an
LS,   Sl ei senger   MH,   eds.   Sl ei senger   and  For dtr an's  gastr oi ntesti nal   and
l i v er   di sease:  pathophy si ol ogy ,   di agnosi s,   m anagem ent,   7th  ed.
Phi l adel phi a:  WB  Saunder s,   2002:747.

Gallstone Disease
1.   Whi ch  gr oup  of   peopl e  has  the  hi ghest  k now n  pr ev al ence  of   gal l stones?

2.   What  ar e  the  di f f er ent  ty pes  of   gal l stones,   and  how   do  they   f or m ?

3.   Shoul d  al l   pati ents  w i th  gal l stones  under go  chol ecy stectom y ?

4.   What  ar e  the  com m on  sy m ptom s  of   gal l stone  di sease,   and  w hat
per centage  of   pati ents  w i th  asy m ptom ati c  gal l stones  ev entual l y
ex hi bi ts  sy m ptom s?

5.   What  i s  the  best  i m agi ng  techni que  to  dem onstr ate  chol el i thi asi s?

6.   What  tr eatm ent  of   sy m ptom ati c  chol el i thi asi s  i s  the  standar d  agai nst
w hi ch  other   tr eatm ents  ar e  com par ed?

Discussion
1.   Whi ch  gr oup  of   peopl e  has  the  hi ghest  k now n  pr ev al ence  of   gal l stones?

Ex am i nati on  of   autopsy   f i ndi ngs  hav e  r ev eal ed  that  the  hi ghest  k now n
pr ev al ence  of   gal l stones  i s  i n  the  N or th  Am er i can  Pi m a  Indi ans:
appr ox i m atel y   60%  f or   w om en  and  25%  f or   m en.   The  popul ati on  of
Thai l and  has  one  of   the  l ow est  k now n  pr ev al ences:  appr ox i m atel y   5%
f or   w om en  and  3%  f or   m en.   The  pr ev al ence  r ate  f or   w hi tes  i n  the
U ni ted  States  and  i n  nor th­centr al   Eur ope  i s  appr ox i m atel y   30%  f or
w om en  and  15%  f or   m en.

P. 184
The  com posi ti on  of   gal l stones  v ar i es  w i del y   f r om   popul ati on  to
popul ati on.   The  w hi te  popul ati on  of   the  U ni ted  States  tends  to  hav e
gal l stones  consi sti ng  l ar gel y   of   chol ester ol ,   w her eas  the  Asi an
popul ati on  tends  to  hav e  br ow n,   cal ci um   bi l i r ubi nate  stones.
The  w i despr ead  v ar i ati on  i n  gal l stone  pr ev al ence  r ates,   the  v ar i ati on  i n
gal l stone  com posi ti on  am ong  ethni c  popul ati ons,   and  the  gener al
f em al e­to­m al e  r ati o  of   appr ox i m atel y   2:1  al l   str ongl y   i m pl i cate  both
her edi tar y   and  env i r onm ental   f actor s  i n  the  eti ol ogy   of   gal l stone
di sease.

2.   What  ar e  the  di f f er ent  ty pes  of   gal l stones,   and  how   do  they   f or m ?

Ther e  ar e  thr ee  ty pes  of   gal l stones:  chol ester ol ,   br ow n  pi gm ent,   and
bl ack   pi gm ent.   Chol ester ol   gal l stones  ar e  com posed  pr i m ar i l y   of
chol ester ol   m onohy dr ate  cr y stal s  m i x ed  w i th  m uci n  gl y copr otei n.   Br ow n
pi gm ent  gal l stones,   w hi ch  ar e  associ ated  w i th  bacter i al   i nf ecti on  of   the
bi l i ar y   tr ee  and  usual l y   f or m   i n  the  bi l e  ducts,   ar e  com posed  pr i m ar i l y
of   cal ci um   bi l i r ubi nate.   Bl ack   pi gm ent  gal l stones  f or m   i n  the  gal l
bl adder   and  ar e  associ ated  w i th  chr oni c  hem ol y si s,   adv anci ng  age,
l ong­ter m   par enter al   nutr i ti on  and  ci r r hosi s;  these  stones  ar e
com posed  pr i m ar i l y   of   an  i nsol ubl e  bi l i r ubi n  pi gm ent  pol y m er .

The  f or m ati on  of   gal l stones  depends  on  the  i nter pl ay   of   thr ee  f actor s:
the  pr oducti on  of   l i thogeni c  bi l e,   gal l bl adder   m oti l i ty ,   and  the
nucl eati on  of   gal l stones.   Condi ti ons  that  f oster   i ncr eased  bi l i ar y
chol ester ol   secr eti on,   such  as  obesi ty ,   r educed  bi l e  aci d  secr eti on  (as
i n  ter m i nal   i l eal   Cr ohn's  di sease),   and  i ncr eased  bi l i r ubi n  pr oducti on
(as  i n  si ck l e  cel l   hem ogl obi nopathy ),   m ay   al l   cause  the  pr oducti on  of
l i thogeni c  bi l e.   Bi l i ar y   stasi s,   such  as  that  associ ated  w i th  pr ol onged
total   par enter al   nutr i ti on,   al so  pr om otes  gal l stone  f or m ati on.
Decr eased  bi l i ar y   i m m unogl obul i n  A  (IgA)  secr eti on,   such  as  that  f ound
i n  m any   Asi ans,   m ay   al l ow   the  gr ow th  of   bacter i a  that  pr oduce  β­
gl ucur oni dase;  the  r esul ti ng  hy dr ol y si s  of   conjugated  bi l i r ubi n  pr om otes
the  pr eci pi tati on  of   cal ci um   bi l i r ubi nate.   These  cal ci um   sal ts  m ay   then
f or m   the  nucl ei   f or   gal l stones.

3.   Shoul d  al l   pati ents  w i th  gal l stones  under go  chol ecy stectom y ?

N o.   Most  pati ents  w i th  gal l stones  r em ai n  asy m ptom ati c,   and  those  w ho
do  becom e  sy m ptom ati c  ar e  not  at  i ncr eased  r i sk   of   death  f r om   ei ther
the  di sease  or   the  sur ger y .   Thi s  i s  al so  tr ue  f or   pati ents  w i th
concur r ent  di abetes  m el l i tus.   Ther ef or e,   pr ophy l acti c  chol ecy stectom y
i s  not  r ecom m ended  f or   m ost  asy m ptom ati c  pati ents,   i ncl udi ng  those
w i th  di abetes  m el l i tus.   How ev er ,   pr ophy l acti c  chol ecy stectom y   f or
asy m ptom ati c  gal l stones  i s  r ecom m ended  f or   cer tai n  gr oups  w ho  f ace  a
hi gh  r i sk   of   m or bi di ty .   The  r i sk   of   gal l bl adder   cancer   i s  hi gh  i n  N ati v e
Am er i cans  w i th  chol el i thi asi s.   Sy m ptom s  dev el op  i n  near l y   al l   chi l dr en
w ho  hav e  chol el i thi asi s.   The  l i k el i hood  of   com pl i cati ons  af ter
em er gency   chol ecy stectom y   i s  i ncr eased  i n  pati ents  w i th  si ck l e  cel l
hem ogl obi nopathy .

Pati ents  w i th  asy m ptom ati c  stones  i n  the  com m on  bi l e  duct
(chol edochol i thi asi s)  ex per i ence  a  m or e  m or bi d  cour se;  i n  50%  of
pati ents  w i th  chol edochol i thi asi s  f ound  postm or tem ,   these  ductal   stones
contr i buted  to  thei r   death.   Ther ef or e,   such  stones  shoul d  be  r em ov ed
ei ther   sur gi cal l y   or   by   ERCP.

P. 185
4.   What  ar e  the  com m on  sy m ptom s  of   gal l stone  di sease,   and  w hat
per centage  of   pati ents  w i th  asy m ptom ati c  gal l stones  ev entual l y   ex hi bi t
sy m ptom s?

Onl y   10%  to  20%  of   the  peopl e  w i th  asy m ptom ati c  gal l stones
ev entual l y   ex hi bi t  sy m ptom s.   The  onset  of   sy m ptom s  m ost  com m onl y
consi sts  of   r ecur r ent  bi l i ar y   pai n  due  to  a  stone  i n  the  cy sti c  duct.   Thi s
pai n  star ts  usual l y   i n  the  r i ght  upper   quadr ant  or   epi gastr i um   and  m ay
r adi ate  to  the  back   or   r i ght  shoul der .   Bi l i ar y   pai n  ty pi cal l y   i s  gr adual   i n
onset  and  l asts  sev er al   hour s.   Contr ar y   to  com m on  bel i ef ,   ther e  i s  no
par ti cul ar   tem por al   r el ati onshi p  to  f ood  i ntak e  or   di et.

Per si stent  bl ock age  of   the  cy sti c  duct  r esul ts  i n  acute  i nf l am m ati on  of
the  gal l bl adder ,   or   acute  chol ecy sti ti s.   Pati ents  w i th  acute  chol ecy sti ti s
usual l y   ex per i ence  nausea,   v om i ti ng,   and  f ev er ,   and  they   com pl ai n  of
sev er e  r i ght  upper   quadr ant  pai n.   El i ci tati on  of   r i ght  upper   quadr ant
abdom i nal   tender ness  i n  com bi nati on  w i th  l euk ocy tosi s  i s  al so  hi ghl y
suggesti v e  of   acute  chol ecy sti ti s.   The  def i ni ti v e  tr eatm ent  i s
chol ecy stectom y .

Obstr ucti on  of   the  com m on  bi l e  duct  by   a  gal l stone  m ay   r esul t  i n
chol angi ti s.   Char cot's  tr i ad  of   sy m ptom s  (f ev er ,   chi l l s,   and  jaundi ce)  i s
ex hi bi ted  by   onl y   50%  to  75%  of   pati ents  w i th  acute  chol angi ti s.   Most
pati ents  r espond  r api dl y   to  appr opr i ate  anti bi oti c  ther apy ;  how ev er ,
def i ni ti v e  tr eatm ent  consi sts  of   decom pr essi on  of   the  bi l e  duct  by
ERCP,   per cutaneous  dr ai nage,   or   bi l i ar y   sur ger y .

Acute  bi l i ar y   pancr eati ti s  m ay   r esul t  f r om   a  com m on  bi l e  duct  stone


that  i s  tr ansi entl y   bl ock i ng  the  pancr eati c  duct  w i thi n  the  am pul l a  of
Vater .   U r gent  ERCP  w i th  endoscopi c  sphi ncter otom y   shoul d  be
consi der ed  f or   these  pati ents.

5.   What  i s  the  best  i m agi ng  techni que  to  dem onstr ate  chol el i thi asi s?

U l tr asonogr aphy   i s  the  best  i ni ti al   m ethod  f or   dem onstr ati ng


chol el i thi asi s  because  i t  has  90%  to  95%  sensi ti v i ty   and  95%
speci f i ci ty .   Al so,   i t  i s  noni nv asi v e  and  unencum ber ed  by   m any   techni cal
l i m i tati ons.   Its  m ai n  uti l i ty   i s  i n  the  dem onstr ati on  of   gal l stones,
al though  i t  i s  capabl e  of   detecti ng  som e  addi ti onal   f i ndi ngs,   such  as
per i chol ecy sti c  f l ui d,   thi ck eni ng  of   the  gal l bl adder   w al l ,   and  di stenti on
of   the  gal l bl adder ,   w hi ch  ar e  ev i dence  of   acti v e  i nf l am m ati on.
U l tr asonogr aphy   can  of ten  r ev eal   ductal   di l atati on  or   com m on  bi l e  duct
stones,   but  f ai l ur e  to  do  so  does  not  r ul e  out  chol edochol i thi asi s.

Bef or e  ul tr asonogr aphy   becam e  av ai l abl e,   or al   chol ecy stogr aphy   w as


the  test  of   choi ce  f or   i denti f y i ng  gal l stones.   How ev er ,   thi s  pr ocedur e
tak es  sev er al   hour s  to  per f or m ,   i s  not  ef f ecti v e  w hen  the  bi l i r ubi n  l ev el
ex ceeds  2. 0  m g/dL,   and  can  be  m ade  unr el i abl e  by   v om i ti ng  and
di ar r hea.   Thi s  techni que  m ay   be  usef ul   i f   ther e  i s  a  str ong  cl i ni cal
suspi ci on  of   chol el i thi asi s  but  the  ul tr asonogr aphi c  f i ndi ngs  ar e
equi v ocal .

Hepatobi l i ar y   sci nti gr aphy   i s  pr i m ar i l y   used  to  assi st  i n  establ i shi ng  the
di agnosi s  of   acute  chol ecy sti ti s.   If   the  r adi oacti v e  tr acer   does  not
i m age  the  gal l bl adder ,   obstr ucti on  of   the  cy sti c  duct  i s  hi ghl y   l i k el y ;
how ev er ,   thi s  techni que  cannot  i denti f y   stones  w i thi n  the  gal l bl adder .

P. 186
ERCP  i s  the  best  techni que  f or   di agnosi ng  com m on  bi l e  duct  stones,   but
i t  i s  f ar   l ess  sensi ti v e  i n  detecti ng  stones  i n  the  gal l bl adder   than
ul tr asonogr aphy   or   or al   chol ecy stogr aphy .   Magneti c  r esonance
chol angi opancr eatogr aphy   (MRCP)  i s  i ncr easi ngl y   r ecogni zed  as  an
accur ate,   noni nv asi v e  m eans  of   v i sual i zi ng  the  bi l e  ducts  and
pancr eati c  ducts.

In  gener al ,   CT  scanni ng  v i sual i zes  gal l stones  poor l y   and  adds  l i ttl e
i nf or m ati on  i n  the  ov er al l   ef f or t  to  di agnose  gal l stone  di sease.

6.   What  tr eatm ent  f or   sy m ptom ati c  chol el i thi asi s  i s  the  standar d  agai nst
w hi ch  other   tr eatm ents  ar e  com par ed?

Open  chol ecy stectom y   has  been  the  standar d  tr eatm ent  of   sy m ptom ati c
gal l stone  di sease.   In  a  pati ent  y ounger   than  50  y ear s  w ho  i s  f r ee  of
com pl i cati ng  f actor s,   the  m or tal i ty   r ate  associ ated  w i th  el ecti v e  open
chol ecy stectom y   i s  l ess  than  1%.   The  pati ent  i s  usual l y   hospi tal i zed  f or
appr ox i m atel y   5  day s  and  r em ai ns  on  m edi cal   di sabi l i ty   l eav e  f or   an
addi ti onal   4  to  6  w eek s.

Incr easi ngl y ,   l apar oscopi c  chol ecy stectom y   has  becom e  the  pr ef er r ed
tr eatm ent  of   sy m ptom ati c  gal l stone  di sease  i n  m ost  pati ents.   Al though
thi s  m ethod  has  a  sl i ghtl y   hi gher   r ate  of   com m on  bi l e  duct  i njur y ,
pati ents  w ho  under go  thi s  usual l y   r equi r e  a  shor ter   hospi tal   stay   and
l ess  ti m e  of f   f r om   w or k   than  those  w ho  under go  open  chol ecy stectom y .

The  di ssol uti on  of   gal l stones  thr ough  the  or al   adm i ni str ati on  of   bi l e
aci ds,   such  as  ur sodeox y chol i c  aci d  (ur sodi ol ),   i s  r eser v ed  f or   those
pati ents  w ho  ar e  ei ther   unabl e  or   unw i l l i ng  to  under go  sur ger y .   Thi s
ther apy   i s  m ost  ef f ecti v e  f or   those  15%  of   pati ents  w i th  chol el i thi asi s
w ho  hav e  sm al l   chol ester ol   gal l stones  f l oati ng  i n  a  f uncti onal
gal l bl adder .   Af ter   the  6  to  12  m onths  of   ther apy   i s  com pl eted,   the
r ecur r ence  r ate  of   gal l stones  i s  appr ox i m atel y   50%  at  5  y ear s.

Case
A  54­y ear ­ol d  Hi spani c  w om an  pr esents  to  the  em er gency   r oom   com pl ai ni ng
of   constant,   sev er e  r i ght  upper   quadr ant  pai n  r adi ati ng  to  her   r i ght  scapul a
that  has  l asted  f or   appr ox i m atel y   6  hour s.   She  has  v om i ted  tw i ce  w i thout
r el i ef   of   the  pai n.   She  ex per i enced  tw o  si m i l ar ,   but  l ess  sev er e,   epi sodes  of
such  pai n  sev er al   w eek s  ago,   f or   w hi ch  she  di d  not  seek   m edi cal   car e.   She
does  not  hav e  any   chr oni c  i l l ness.   Ex am i nati on  r ev eal s  a  m oder atel y   obese
w om an  w i th  a  tem per atur e  of   38°C  (100. 4°F).   Her   scl er ae  ar e  sl i ghtl y
i cter i c.   She  ex hi bi ts  abdom i nal   guar di ng,   w i th  m oder ate  r i ght  upper
quadr ant  tender ness  on  pal pati on,   hal ti ng  of   i nspi r ati on  dur i ng  pal pati on,
and  nor m al   bow el   sounds.   The  w hi te  bl ood  cel l   count  i s  14, 000  cel l s/m m 3 ,
and  the  al k al i ne  phosphatase  l ev el   i s  el ev ated  at  200  IU /L.   The  total
bi l i r ubi n  l ev el   i s  4  m g/dL.   The  ser um   am i notr ansf er ase  v al ues  ar e  nor m al .

1.   What  i s  the  m ost  l i k el y   di agnosi s  i n  thi s  pati ent?


2.   What  i m agi ng  study   shoul d  be  per f or m ed?
3.   How   shoul d  y ou  m anage  thi s  pati ent's  condi ti on?

P. 187

Case Discussion
1.   What  i s  the  m ost  l i k el y   di agnosi s  i n  thi s  pati ent?

The  m ost  l i k el y   di agnosi s  i s  acute  chol ecy sti ti s  associ ated  w i th


chol edochol i thi asi s  and  obstr ucti on  of   the  com m on  bi l e  duct  by   a
gal l stone.   Thi s  w om an's  pr ev i ous  sy m ptom s,   w hi ch  ar e  consi stent  w i th
r ecur r ent  bi l i ar y   pai n,   suggest  gal l stone  di sease.   The  m or e  sev er e
sy m ptom s  she  now   has,   w hi ch  ar e  associ ated  w i th  a  l euk ocy tosi s,   r i ght
upper   quadr ant  abdom i nal   tender ness  and  i nspi r ator y   ar r est  w i th
pal pati on  i n  the  r i ght  upper   quadr ant  (Mur phy 's  si gn),   suggest  acute
chol ecy sti ti s.   The  el ev ated  al k al i ne  phosphatase  and  total   bi l i r ubi n
l ev el s  ar e  ev i dence  that  the  com m on  bi l e  duct  i s  obstr ucted.   (The  total
bi l i r ubi n  l ev el   r ar el y   r i ses  abov e  3  m g/dL  i n  chol ecy sti ti s  al one. )

2.   What  i m agi ng  study   shoul d  be  per f or m ed?

Abdom i nal   ul tr asonogr aphy   shoul d  be  per f or m ed  r outi nel y   i n  pati ents
suspected  of   hav i ng  gal l stone  di sease.   In  thi s  pati ent,   the  ty pi cal
pr esentati on,   w hi ch  poi nts  tow ar d  acute  chol ecy sti ti s  and  chol el i thi asi s,
m ak es  addi ti onal   i m agi ng  studi es  unnecessar y .   If   the  ul tr asonogr am
f ai l s  to  dem onstr ate  stones,   a  hepatobi l i ar y   sci nti gr am   coul d  assi st  i n
m ak i ng  the  di agnosi s.

3.   How   shoul d  y ou  m anage  thi s  pati ent's  condi ti on?

Ini ti al   m anagem ent  shoul d  consi st  of   the  IV  adm i ni str ati on  of   f l ui ds  and
anti bi oti c  cov er age  f or   gr am ­negati v e  or gani sm s,   together   w i th
nasogastr i c  sucti on.   Chol ecy stectom y   shoul d  be  per f or m ed  soon  af ter
the  pati ent's  condi ti on  has  stabi l i zed;  a  del ay   i n  sur ger y   i s  associ ated
w i th  hi gher   m or bi di ty   r ates.   If   open  chol ecy stectom y   i s  per f or m ed,   an
ex pl or ati on  of   the  com m on  bi l e  duct  shoul d  be  str ongl y   consi der ed.   If
the  l apar oscopi c  m ethod  i s  chosen,   pr eoper ati v e  ERCP  shoul d  be
per f or m ed  to  r em ov e  the  stone  i n  the  com m on  bi l e  duct.   If   the
pati ent's  condi ti on  does  not  i m pr ov e  r api dl y   and  she  sti l l   has
obstr ucti v e  jaundi ce,   an  ur gent  ERCP  shoul d  be  per f or m ed  to
decom pr ess  the  bi l i ar y   sy stem .
Suggested Readings
Lee  SP,   Ko  CW.   Gal l stones.   In:  Yam ada  T,   Al per s  DH,   Kapl ow i tz  N ,   etal .
eds.   Tex tbook   of   gastr oenter ol ogy ,   4th  ed.   Phi l adel phi a:  Li ppi ncott
Wi l l i am s  &  Wi l k i ns,   2003:  2177.

Pom posel l i   J,   Jenk i ns  RL.   Sur gi cal   appr oaches  to  di seases  of   the  bi l i ar y
sy stem .   In:  Schi f f   ER,   Sor r el l   MF,   Maddr ey   WC,   eds.   Schi f f 's  di seases  of
the  l i v er ,   9th  ed.   Phi l adel phi a:  Li ppi ncott  Wi l l i am s  &  Wi l k i ns,   2003:713.

Acute Hepatocellular Disease
1.   What  ar e  the  m ajor   si gns  and  sy m ptom s  of   acute  hepatocel l ul ar   i njur y ,
and  w hi ch  ar e  speci f i c  to  a  par ti cul ar   pr ocess?

P. 188
2.   At  w hat  ser um   bi l i r ubi n  l ev el   i s  jaundi ce  detectabl e,   and  w hat  ar e  the
m ai n  deter m i nants  of   the  ser um   bi l i r ubi n  concentr ati on?

3.   What  ar e  the  f our   gener al   causes  of   acute  l i v er   i njur y ?

4.   What  ar e  the  f eatur es  of   v i r al   hepati ti s  A,   B,   C,   D,   and  E?

5.   Match  the  f ol l ow i ng  ser ol ogi c  r esul ts  w i th  the  m ost  l i k el y   cl i ni cal   state.

a .   HBsAg­,   anti ­HBs+ ,   anti ­ 1.   Acute  hepati ti s  B


HBc+

b.   HBsAg­,   anti ­HBs+ ,   anti ­HBc­ 2.   Acute  hepati ti s  A

c .   HBsAg+ ,   IgM  anti ­HBc+ 3.   Pr i or   HBV  i nf ecti on,   now


i m m une

d.   HBsAg+ ,   IgM  anti ­HBc­ 4.   Pr i or   HAV  i nf ecti on,   now


i m m une

e .   Anti ­HAV(total )+ ,   IgM  anti ­ 5.   Hepati ti s  B  chr oni c  car r i er


HAV­

f.   Anti ­HAV(total )+ ,   IgM  anti ­ 6.   Recei v ed  hepati ti s  B  v acci ne


HAV+

HBsAg,   hepati ti s  B  sur f ace  anti gen;  anti ­HBs,   anti body   to  hepati ti s  B
sur f ace  anti gen;  anti ­HBc,   anti body   to  hepati ti s  B  cor e  anti gen;  anti ­
HAV,   anti bodi es  to  hepati ti s  A  anti gens  [total   and  i m m unogl obul i n  M
(IgM)  cl ass];­,   negati v e;  + ,   posi ti v e;  HBV,   hepati ti s  B  v i r us;  HAV,
hepati ti s  A  v i r us.
6.   Whi ch  hepati c  enzy m e  patter n  suggests  the  pr esence  of   al cohol i c  l i v er
di sease?

7.   What  ar e  the  cl i ni cal   and  l abor ator y   f i ndi ngs  char acter i sti c  of   i schem i c
l i v er   i njur y ?

8.   What  i s  f ul m i nant  hepati c  f ai l ur e?

Discussion
1.   What  ar e  the  m ajor   si gns  and  sy m ptom s  of   acute  hepatocel l ul ar   i njur y ,
and  w hi ch  ar e  speci f i c  to  a  par ti cul ar   pr ocess?

The  ty pi cal   sy m ptom s  of   acute  hepati ti s  i ncl ude  m al ai se,   f ati gue,
anor ex i a,   nausea,   dar k   ur i ne,   abdom i nal   pai n,   headache,   f ev er ,
m y al gi a,   and  ar thr al gi a.   Si gns  i ncl ude  jaundi ce,   scl er al   i cter us,
hepatom egal y ,   tender   l i v er ,   spl enom egal y ,   and  r ash.   In  gener al ,   these
f eatur es  ar e  nonspeci f i c  and  do  not  hel p  i n  i denti f y i ng  the  cause  of
l i v er   i njur y .

2.   At  w hat  ser um   bi l i r ubi n  l ev el   i s  jaundi ce  detectabl e,   and  w hat  ar e  the
m ai n  deter m i nants  of   the  ser um   bi l i r ubi n  concentr ati on?

A  ser um   bi l i r ubi n  l ev el   i n  the  r ange  of   2. 5  to  3. 0  m g/dL  usual l y


pr oduces  detectabl e  scl er al   i cter us.   The  ser um   bi l i r ubi n  concentr ati on
i s  deter m i ned  by   the  r ates  of   bi l i r ubi n  pr oducti on  (r esul ti ng  f r om   the
catabol i sm   of   hem ogl obi n  and  other   hem e­contai ni ng  enzy m es)  and
el i m i nati on  (i ncl udi ng  ex cr eti on  i nto  bi l e  and  the  r enal   ex cr eti on  of
conjugated  bi l i r ubi n).   As  a  r esul t,   hem ol y si s  and  changes  i n  r enal
f uncti on  can  consi der abl y   al ter   the  ser um   bi l i r ubi n  concentr ati on.

3.   What  ar e  the  f our   gener al   causes  of   acute  l i v er   i njur y ?

Ex pos ure  to tox ins   i s  a  com m on  cause  of   acute  l i v er   i njur y .   Such
tox i ns  i ncl ude  ethanol ,   acetam i nophen,   hal ogenated  hy dr ocar bons,   and
the  tox i n  f r om
P. 189
the  m ushr oom   Am ani ta  phal l oi des.   Infe c tions   can  al so  cause  acute
l i v er   i njur y .   The  m ost  com m on  i nf ecti ons  ar e  those  caused  by   hepati ti s
v i r uses  A,   B,   C,   D,   and  E,   but  par asi tes,   bacter i a,   and  f ungi   al so  can
cause  i nf ecti ous  hepati ti s.   Hepati c  i njur y   can  al so  stem   f r om   is c he mia ;
thi s  i s  usual l y   a  r esul t  of   sev er e  sy stem i c  hy potensi on  or   congesti v e
hear t  f ai l ur e.   Other   sour ces  of   acute  l i v er   i njur y   ar e  the  v ar i ous
me ta bolic  dis orde rs   such  as  Wi l son's  di sease  and  Rey e's  sy ndr om e.

4.   What  ar e  the  f eatur es  of   v i r al   hepati ti s  A,   B,   C,   D,   and  E?

He pa titis  A  i s  caused  by   an  RN A  enter ov i r us  that  i s  usual l y   tr ansm i tted
by   f ecal –or al   contam i nati on.   The  hepati ti s  A  v i r us  i s  pr esent  i n  the
stool   f or   appr ox i m atel y   2  w eek s  af ter   i nf ecti on,   but  sy m ptom s  do  not
appear   unti l   appr ox i m atel y   4  w eek s  af ter   i nf ecti on.   Thi s  per i od  of
asy m ptom ati c  i nf ecti v i ty   i s  par ti al l y   r esponsi bl e  f or   the  occasi onal
outbr eak s  of   hepati ti s  A  spr ead  by   an  unsuspecti ng  f ood  handl er   at  a
r estaur ant,   or   by   chi l dr en  at  a  day ­car e  center .   Sy m ptom s  usual l y
consi st  of   nausea,   v om i ti ng,   jaundi ce,   and  m al ai se,   al though  the  enti r e
cour se  of   the  di sease  m ay   be  subcl i ni cal ,   especi al l y   i n  chi l dr en.
Pr ogr essi on  to  f ul m i nant  hepati c  f ai l ur e  i s  v er y   r ar e,   and  f ul l   r ecov er y
i s  ex pected  af ter   3  w eek s  of   sy m ptom s.   The  best  ser ol ogi c  test  f or
conf i r m i ng  acute  v i r al   hepati ti s  A  i s  the  IgM  anti ­HAV  deter m i nati on,
w hi ch  shoul d  be  posi ti v e  at  the  onset  of   sy m ptom s.   The  pr esence  of
IgG  anti ­HAV  i m pl i es  that  the  per son  had  hepati ti s  A  i n  the  past  and  i s
i m m une.   Suscepti bl e  peopl e  shoul d  be  passi v el y   i m m uni zed  w i th  hum an
i m m une  ser um   gl obul i n  w i thi n  2  w eek s  of   ex posur e  to  the  hepati ti s  A
v i r us.   Acti v e  pr ophy l ax i s  agai nst  hepati ti s  A  f or   cer tai n  hi gh­r i sk
popul ati ons  and  pati ents  w i th  chr oni c  l i v er   di sease  i s  av ai l abl e  i n  the
f or m   of   hepati ti s  A  v acci ne.

He pa titis  B  i s  caused  by   a  DN A  v i r us  that  i s  tr ansm i tted  by   par enter al


ex posur e  to  i nf ected  bl ood,   usual l y   thr ough  sk i n  punctur es  by
contam i nated  needl es.   Because  thi s  v i r us  can  al so  be  tr ansm i tted
thr ough  m i nute  br eak s  i n  m ucous  m em br anes,   r i sk   f actor s  f or   hepati ti s
B  i nf ecti on  i ncl ude  sex ual   contact  and  the  shar i ng  of   r azor s  and
toothbr ushes  w i th  an  i nf ected  per son;  tr ansm i ssi on  at  bi r th  f r om
m other   to  chi l d  i s  al so  com m on.   The  hepati ti s  B  v i r us  i s  pr esent  i n  the
bl ood  appr ox i m atel y   2  m onths  af ter   i nf ecti on,   w i th  sy m ptom s  appear i ng
at  appr ox i m atel y   3  m onths.   IgM  anti ­HBc  appear s  ear l y   i n  the  di sease
and  i ts  m easur em ent  i s  the  best  si ngl e  ser ol ogi c  test  to  conf i r m   acute
v i r al   hepati ti s  B.   The  cl i ni cal   cour se  m ay   pr ogr ess  to  f ul m i nant  hepati c
f ai l ur e  and  death  i n  up  to  2%  of   the  pati ents,   or   the  i nf ecti on  m ay
sm ol der   i n  a  chr oni c  car r i er   state  i n  up  to  10%  of   the  pati ents.   Chr oni c
car r i er s  ar e  at  hi gh  r i sk   f or   ear l y   death  f r om   ci r r hosi s  or
hepatocel l ul ar   car ci nom a  and  shoul d  be  consi der ed  candi dates  f or
tr eatm ent  w i th  agents  ai m ed  at  suppr essi ng  hepati ti s  B  r epl i cati on,   f or
ex am pl e,   α ­i nter f er on,   l am i v udi ne,   or   adef ov i r .   Im m uni zati on  w i th
v acci ne  m ade  f r om   r ecom bi nant  HBsAg  i s  hi ghl y   ef f ecti v e  and  conf er s
l ong­l asti ng  pr otecti on  agai nst  i nf ecti on.

He pa titis  C  i s  caused  by   an  RN A  v i r us  that,   l i k e  the  hepati ti s  B  v i r us,


i s  bel i ev ed  to  be  tr ansm i tted  pr i m ar i l y   by   par enter al   ex posur e  to
i nf ected  bl ood,   al though  a  substanti al   per centage  of   pati ents  hav e  no
i denti f i abl e  r i sk   f actor s.   Sy m ptom s  of   acute  i nf ecti on  ar e  of ten  m i l d.
Mor e  than  50%  of   i nf ected  peopl e
P. 190
becom e  chr oni c  car r i er s.   Such  per sons  ar e  at  hi gh  r i sk   f or   chr oni c
hepati ti s,   ci r r hosi s,   and  hepatocel l ul ar   car ci nom a.   Pati ents  w i th  chr oni c
hepati ti s  C  shoul d  be  consi der ed  candi dates  f or   tr eatm ent  w i th  α ­
i nter f er on  or   the  com bi nati on  of   i nter f er on  and  r i bav i r i n.   Ser ol ogi c
tests  f or   hepati ti s  C  conti nue  to  be  i m pr ov ed  as  our   k now l edge  of   the
v i r us  i ncr eases.   Ther e  i s  no  k now n  pr otecti v e  anti body   and  no  k now n
v acci ne.
He pa titis  D  i s  caused  by   a  def ecti v e  RN A  v i r us  that  r equi r es  the
pr esence  of   HBsAg  f or   ex pr essi on.   Hence,   i nf ecti on  w i th  the  hepati ti s  D
v i r us  occur s  onl y   as  a  coi nf ecti on  w i th  hepati ti s  B  v i r us,   or   as  a
super i nf ecti on  i n  those  w ho  ar e  chr oni c  hepati ti s  B  v i r us  car r i er s.   The
sy m ptom s  of   hepati ti s  D  ar e  usual l y   m or e  sev er e  than  those  seen  w i th
acute  hepati ti s  B,   w i th  pr ogr essi on  to  f ul m i nant  hepati c  f ai l ur e  and
death  i n  up  to  20%  of   the  pati ents.   The  speci f i c  ser ol ogi c  test  f or
hepati ti s  D,   anti ­HD,   shoul d  be  car r i ed  out  onl y   i f   the  HBsAg  ser ol ogy
i s  posi ti v e.   Ther e  i s  no  v acci ne  speci f i c  to  the  hepati ti s  D  v i r us,
al though  i m m uni zati on  agai nst  hepati ti s  B  conf er s  pr otecti on  agai nst
hepati ti s  D.

He pa titis  E  i s  caused  by   an  RN A  v i r us  that,   l i k e  the  hepati ti s  A  v i r us,


i s  tr ansm i tted  pr i m ar i l y   by   f ecal –or al   contam i nati on.   Outbr eak s  of
the  di sease  can  r each  epi dem i c  pr opor ti ons  i n  ar eas  of   the  w or l d  w her e
f l oodi ng  and  poor   sani tati on  ar e  pr ev al ent.   Al though  sy m ptom s  ar e  m i l d
i n  m ost  pati ents,   hepati ti s  E  has  a  20%  m or tal i ty   r ate  i f   i t  i s  acqui r ed
dur i ng  pr egnancy .   Speci f i c  ser ol ogi c  tests  ar e  under   dev el opm ent.
Ther e  i s  no  k now n  v acci ne  f or   i t.

5.   Match  the  f ol l ow i ng  ser ol ogi c  r esul ts  w i th  the  m ost  l i k el y   cl i ni cal   state.

The  cor r ect  pai r i ngs  of   the  ser ol ogi c  r esul ts  w i th  the  m ost  l i k el y
cl i ni cal   state  ar e:  a  w i th  3,   b  w i th  6,   c  w i th  1,   d  w i th  5,   e  w i th  4,   and  f
w i th  2.   HBsAg  i s  pr esent  i n  the  setti ngs  of   acute  i nf ecti on,   chr oni c
i nf ecti on,   and  the  car r i er   state.   Anti ­HBs  and  anti ­HBc  appear   and  the
HBsAg  l ev el   decl i nes  as  the  acute  i nf ecti on  r esol v es.   IgM  anti ­HBc  or
IgM  anti ­HAV  i s  usual l y   pr esent  onl y   dur i ng  acute  i nf ecti on,   w her eas
the  IgG  cl asses  of   anti ­HBc  and  anti ­HAV  per si st,   i ndi cati ng  a  state  of
i m m uni ty   af ter   r esol uti on  of   the  acute  i nf ecti on.   Anti ­HBs  appear s
al one,   w i thout  anti ­HBc,   i n  r esponse  to  hepati ti s  B  v acci ne.

6.   Whi ch  hepati c  enzy m e  patter n  suggests  the  pr esence  of   al cohol i c  l i v er


di sease?

In  the  setti ng  of   al cohol i c  hepati ti s,   the  ser um   AST  l ev el   i s  usual l y
hi gher   than  the  ser um   ALT  l ev el .   In  addi ti on,   the  ser um   l ev el   of   γ­
gl utam y l tr anspepti dase  i s  of ten  el ev ated  because  of   i nducti on  of   thi s
enzy m e  by   chr oni c  ethanol   i ngesti on.

7.   What  ar e  the  cl i ni cal   and  l abor ator y   f i ndi ngs  char acter i sti c  of   i schem i c
l i v er   i njur y ?

Ischem i c  l i v er   i njur y ,   or   “shock   l i v er , â€​  usual l y   occur s  i n  the  setti ng


of   a  r ecogni zed  ci r cul ator y   di stur bance,   such  as  hy potensi on  or   acute
m y ocar di al   i nf ar cti on.   A  r api d  and  dr am ati c  r i se  i n  the  AST  and  ALT
l ev el s  i s  seen,   w i th  an  equal l y   r api d  decl i ne.   The  am i notr ansf er ase
l ev el s  can  r i se  i nto  the  thousands,   appr oachi ng  l ev el s  seen  w i th  acute
v i r al   hepati ti s.   A  sl ow ,   steady   i ncr ease  i n  the  ser um   bi l i r ubi n
concentr ati on  subsequentl y   occur s  and  peak s  sev er al   day s
P. 191
l ater .   A  l i v er   bi opsy   i s  not  needed  f or   di agnosi s,   but,   w hen  speci m ens
ar e  obtai ned,   they   show   centr i l obul ar   necr osi s.

8.   What  i s  f ul m i nant  hepati c  f ai l ur e?

Ful m i nant  hepati c  f ai l ur e  i s  def i ned  as  pr ogr essi on  to  si gns  of   l i v er
f ai l ur e,   i ncl udi ng  hepati c  encephal opathy ,   w i thi n  8  w eek s  of   the  onset
of   sy m ptom s.   Such  a  pi ctur e  occur r i ng  8  to  24  w eek s  f r om   the  onset  of
sy m ptom s  i s  consi der ed  subf ul m i nant  hepati c  f ai l ur e.   Ful m i nant  hepati c
f ai l ur e  m ay   be  caused  by   v i r al ,   tox i c,   i schem i c,   or   other   causes  of
hepatocel l ul ar   i njur y .   The  m or tal i ty   r ate  f or   these  enti ti es  i s  ex tr em el y
hi gh.   Intensi v e  suppor t  i s  i ndi cated  i n  af f ected  pati ents,   and  l i v er
tr anspl antati on  shoul d  be  consi der ed  i f   spontaneous  r ecov er y   does  not
occur .

Case
A  37­y ear ­ol d  housew i f e  r epor ts  3  w eek s  of   gener al   f ati gue,   sev er al   day s  of
dar k   ur i ne,   and  1  day   of   scl er al   i cter us.   She  deni es  v om i ti ng,   but  com pl ai ns
of   m i l d,   conti nuous  pai n  i n  the  r i ght  upper   quadr ant,   and  i nter m i ttent
nausea.
Phy si cal   ex am i nati on  r ev eal s  the  pati ent  to  be  jaundi ced  but  com f or tabl e.
She  show s  no  si gns  of   m al nutr i ti on  and  has  no  spi der   angi om as  or   pal m ar
er y them a.   The  l i v er   i s  tender   and  m easur es  15  cm   by   per cussi on  i n  the
m i dcl av i cul ar   l i ne;  i t  i s  pal pabl e  4  cm   bel ow   the  costal   m ar gi n  on
i nspi r ati on.   The  spl een  i s  not  pal pabl e,   and  the  ex am i nati on  f i ndi ngs  ar e
other w i se  unr em ar k abl e.

1.   What  i s  y our   f i r st  di agnosti c  i m pr essi on,   and  w hy ?  Match  the


l abor ator y   f i ndi ngs  w i th  the  v ar i ous  di agnosti c  possi bi l i ti es.

Tota l Alk a line


AST ALT
Bilirubin P hos pha ta s e Dia gnos is
(IU /L) (IU /L)
(mg/dL) (IU /L)

a .   235 90 5. 5 190 1.   Acute  v i r al


hepati ti s

b. 1, 320 5. 5 190 2.   Chr oni c


1, 100 v i r al
hepati ti s

c .   235 325 5. 5 190 3.   Al cohol i c


hepati ti s

d.   235 325 10. 5 990 4.   Bi l e  duct


obstr ucti on

AST,   aspar tate  am i notr ansf er ase;  ALT,   al ani ne  am i notr ansf er ase.
2.   What  other   hi stor i cal   i nf or m ati on  i s  needed  per tai ni ng  to  r i sk   f actor s?
3.   What  tests  w oul d  y ou  or der   i f   y ou  suspected  acute  v i r al   hepati ti s?
4.   If   the  pati ent  has  acute  hepati ti s  A  or   hepati ti s  B,   w hat  shoul d  y ou  tel l
her   about  the  r i sk   to  her   f am i l y ,   and  w hat  i s  the  appr opr i ate  f ol l ow ­up
af ter   she  r ecov er s?
5.   What  i f   al l   i ni ti al   v i r al   hepati ti s  ser ol ogy   r esul ts  ar e  nonr eacti v e?
6.   If   the  pati ent  has  a  str ong  f am i l y   hi stor y   of   l i v er   di sease,   w hat  tests
ar e  av ai l abl e  to  scr een  f or   i nher i ted  di sor der s?
7.   Is  a  l i v er   bi opsy   i ndi cated  i n  thi s  pati ent?
8.   Shoul d  thi s  pati ent  be  adm i tted  to  the  hospi tal ?

P. 192

Case Discussion
1.   What  i s  y our   f i r st  di agnosti c  i m pr essi on,   and  w hy ?  Match  the  l abor ator y
f i ndi ngs  w i th  the  v ar i ous  di agnosti c  possi bi l i ti es.

The  sy m ptom s  and  ex am i nati on  f i ndi ngs  ar e  nonspeci f i c,   and  the
l abor ator y   f i ndi ngs  and  the  v ar i ous  di agnosti c  possi bi l i ti es  ar e  pai r ed,
as  f ol l ow s:  a  w i th  3,   b  w i th  1,   c  w i th  2,   and  d  w i th  4.   Ver y   hi gh
am i notr ansf er ase  l ev el s  (> 1, 000  IU /L)  usual l y   i ndi cate  an  acute
hepatocel l ul ar   i njur y .   Moder atel y   hi gh  l ev el s  (tw o  to  f i v e  ti m es  nor m al )
can  be  seen  i n  m any   si tuati ons,   such  as  ear l y   or   l ate  i n  the  cour se  of
an  acute  i njur y ,   or   i n  chr oni c  di seases  such  as  chr oni c  v i r al   hepati ti s  or
al cohol i c  l i v er   di sease.   An  AST/ALT  r ati o  that  ex ceeds  1  suggests  the
pr esence  of   al cohol i c  l i v er   di sease.   Al k al i ne  phosphatase  and  bi l i r ubi n
l ev el s  that  ar e  el ev ated  out  of   pr opor ti on  to  the  am i notr ansf er ase
concentr ati ons  suggest  a  bi l i ar y   obstr ucti v e  pr ocess,   but  these  ar e  not
speci f i c  w i th  r egar d  to  the  l ev el   of   obstr ucti on  (ex tr ahepati c
obstr ucti on  v er sus  i ntr ahepati c  chol estasi s).

2.   What  other   hi stor i cal   i nf or m ati on  i s  needed  per tai ni ng  to  r i sk   f actor s?

A  pati ent  pr esenti ng  w i th  l i v er   di sease  shoul d  be  ask ed  about  tr av el
and  hepati ti s  ex posur e  (hepati ti s  A);  par enter al   r i sk   f actor s,   i ncl udi ng
tr ansf usi ons,   IV  dr ug  use,   sex ual   contacts,   and  pr of essi onal   ex posur e
(heal th  car e  w or k er s—hepati ti s  B  and  C);  m edi cati ons;  env i r onm ental
ex posur e;  al cohol   i ntak e;  chi l dhood  l i v er   di sease;  and  f am i l y   hi stor y .
Al though  pr ol onged  ex cessi v e  al cohol   i ntak e  i s  of ten  easi l y   r ecogni zed,
som eti m es  i t  i s  cov er t.

3.   What  tests  w oul d  y ou  or der   i f   y ou  suspected  acute  v i r al   hepati ti s?

The  sel ecti on  of   tests  shoul d  be  gui ded  by   the  natur e  of   the  cl i ni cal
hi stor y .   The  f ol l ow i ng  tests  shoul d  be  done  i n  a  per son  suspected  of
hav i ng  acute  v i r al   hepati ti s:  (a)  IgM  anti ­HAV  to  check   f or   acute
hepati ti s  A;  (b)  IgM  anti ­HBc  to  check   f or   acute  hepati ti s  B;  and  (c)
anti bodi es  to  hepati ti s  C  anti gens  (anti ­HCV)  to  check   f or   acute
hepati ti s  C.

4.   If   the  pati ent  has  acute  hepati ti s  A  or   hepati ti s  B,   w hat  shoul d  y ou  tel l
her   about  the  r i sk   to  her   f am i l y ,   and  w hat  i s  the  appr opr i ate  f ol l ow ­up
af ter   she  r ecov er s?

The  househol d  and  sex ual   contacts  of   peopl e  w i th  acute  hepati ti s  A
shoul d  be  passi v el y   i m m uni zed  w i th  i m m une  gl obul i n,   and  they   shoul d
ex er ci se  car ef ul   standar d  pr ecauti ons  to  av oi d  f ecal   –  or al
tr ansm i ssi on.   The  househol d  contacts  of   peopl e  w i th  acute  hepati ti s  B
shoul d  av oi d  par enter al   contact  (the  shar i ng  of   r azor s,   toothbr ushes,
and  the  l i k e).   Sex ual   contact  shoul d  be  m i ni m i zed  dur i ng  the  acute
stage  of   the  i l l ness.   Af ter   cl i ni cal   r ecov er y ,   i t  i s  i m por tant  to
deter m i ne  w hether   the  HBsAg  has  di sappear ed  and  anti ­HBs  has
appear ed.   Fai l ur e  to  cl ear   HBsAg  suggests  dev el opm ent  of   a  chr oni c
hepati ti s  B  car r i er   state.   Sex ual   or   househol d  contacts  of   hepati ti s  B
car r i er s  shoul d  be  i m m uni zed  w i th  hepati ti s  B  v acci ne.

5.   What  i f   al l   i ni ti al   v i r al   hepati ti s  ser ol ogy   r esul ts  ar e  nonr eacti v e?

Repeat  testi ng  f or   anti ­HCV  i n  6  m onths  i s  appr opr i ate  because  thi s
test  m ay   not  be  posi ti v e  i n  the  acute  setti ng.

P. 193
6.   If   the  pati ent  has  a  str ong  f am i l y   hi stor y   of   l i v er   di sease,   w hat  tests
ar e  av ai l abl e  to  scr een  f or   i nher i ted  di sor der s?

A  l ow   ser um   cer ul opl asm i n  l ev el   and  a  hi gh  ur i nar y   copper   ex cr eti on


ar e  hi ghl y   suggesti v e  of   Wi l son's  di sease.   A  hi gh  ser um   f er r i ti n  l ev el
and  hi gh  tr ansf er r i n  satur ati on  ar e  hi ghl y   suggesti v e  of
hem ochr om atosi s.   The  def i ni ti v e  test  f or   both  of   these  di sor der s  i s  a
l i v er   bi opsy .   Geneti c  testi ng  f or   f am i l i al   hem ochr om atosi s  i s  now
av ai l abl e.

7.   Is  a  l i v er   bi opsy   i ndi cated  i n  thi s  pati ent?

Li v er   bi opsy   i s  not  usual l y   needed  f or   di agnosi s  or   pr ognosi s  i n  pati ents


w i th  acute  l i v er   di seases.   Ex cepti ons  m i ght  i ncl ude  establ i shi ng  the
di agnosi s  of   dr ug­i nduced  or   tox i c  hepati ti s,   i schem i c  l i v er   i njur y ,
gr anul om atous  di sease,   and,   r ar el y ,   al cohol i c  hepati ti s.

8.   Shoul d  thi s  pati ent  be  adm i tted  to  the  hospi tal ?

Most  pati ents  w i th  acute  hepati ti s  do  not  r equi r e  hospi tal   adm i ssi on.
How ev er ,   those  w ho  ex hi bi t  ev i dence  of   sev er e  l i v er   i njur y ,   such  as
hepati c  encephal opathy ,   a  bi l i r ubi n  l ev el   abov e  15  m g/dL,   or   an
i ncr easi ng  pr othr om bi n  ti m e,   and  those  w i th  sev er e  anor ex i a  or
nausea,   shoul d  be  hospi tal i zed.

Suggested Readings
Ber enguer   M,   Wr i ght  TL.   Vi r al   hepati ti s.   In:  Fel dm an  M,   Fr i edm an  LS,
Sl ei senger   MH,   eds.   Sl ei senger   and  For dtr an's  gastr oi ntesti nal   and  l i v er
di sease:  pathophy si ol ogy ,   di agnosi s,   m anagem ent,   7th  ed.   Phi l adel phi a:
WB  Saunder s,   2002:1278.

Schi f f   ER.   Vi r al   hepati ti s.   In:  Schi f f   ER,   Sor r el l   MF,   Maddr ey   WC,   eds.
Schi f f 's  di seases  of   the  l i v er ,   9th  ed.   Phi l adel phi a:  Li ppi ncott  Wi l l i am s  &
Wi l k i ns,   2003:741.
Editors :  Sc hrie r,  Robe rt W .
Title :  Inte rna l Me dic ine  Ca s e book ,  The : Re a l P a tie nts ,  Re a l Ans w e rs ,
3rd Edition
Copy r i ght  ©2007  Li ppi ncott  Wi l l i am s  &  Wi l k i ns

>  T a b le   o f   C o nte nts   >  C ha p te r   5  ­  G e r ia tr ic s

Chapter 5
Geriatrics

La ure nc e  Robbins

Dementia
1.   What  i s  the  m ost  com m on  cause  of   pr i m ar y   dem enti a  i n  the  U . S.
popul ati on?

2.   What  ar e  the  pathognom oni c  postm or tem   f i ndi ngs  of   Al zhei m er 's
di sease  (AD)?

3.   Can  the  chi l dr en  of   pati ents  w i th  AD  be  geneti cal l y   tested  and  tol d  w i th
assur ance  w hether   they   w i l l   i nher i t  the  di sease?

4.   How   can  cogni ti v e  f uncti on  be  tested  qui ck l y   and  r el i abl y ?

5.   Can  the  i ntel l ectual   decl i ne  seen  i n  pati ents  w i th  AD  be  hal ted  or
r ev er sed  w i th  m edi cati ons?

Discussion
1.   What  i s  the  m ost  com m on  cause  of   pr i m ar y   dem enti a  i n  the  U . S.
popul ati on?

AD  i s  the  m ost  com m on  cause  of   dem enti a  i n  the  U . S.   popul ati on.
Dem enti a  cur r entl y   af f ects  appr ox i m atel y   4. 5  m i l l i on  peopl e  i n  the
U ni ted  States  and  thi s  num ber   w i l l   gr ow   to  an  esti m ated  10  m i l l i on  by
2050.   As  the  m ost  com m on  eti ol ogy   of   dem enti a,   i t  accounts  f or   70%
or   m or e  of   al l
P. 195
dem enti a  di agnoses.   Adv anci ng  age  r em ai ns  the  si ngl e  gr eatest  r i sk
f actor   f or   AD.   Cur r entl y ,   i t  af f l i cts  appr ox i m atel y   2%  of   the  popul ati on
betw een  65  and  70  y ear s  of   age,   and  appr ox i m atel y   30%  of   the
popul ati on  ol der   than  80  y ear s.   The  i nci dence  of   new   di sease  i s
appr ox i m atel y   3%  per   y ear   i n  com m uni ty ­dw el l i ng  el der l y   w i th  an
av er age  age  of   75.   Acqui r ed  i m pai r m ent  of   shor t­ter m   m em or y   i s  i ts
hal l m ar k   w i th  at  l east  one  of   the  f ol l ow i ng  f our   sy m ptom s  as  w el l :
aphasi a,   apr ax i a,   agnosi a,   and  ex ecuti v e  dy sf uncti on.   Aphasi a  m ay   be
f l uent  or   nonf l uent.   Pati ents  m ay   hav e  di f f i cul ty   com i ng  up  w i th  the
cor r ect  w or d  w hen  tr y i ng  to  nam e  objects,   of ten  substi tuti ng  w or ds  that
descr i be  an  object  (e. g. ,   w hen  ask ed  to  nam e  a  w atch,   the  pati ent  w i th
AD  m i ght  say   “i t's  a  thi ng  y ou  use  to  tel l   ti m eâ€​ ) .   Apr ax i a  i s  the
i nabi l i ty   to  car r y   out  m otor   task s  i n  the  absence  of   m otor   w eak ness
(e. g. ,   a  pati ent  i s  no  l onger   abl e  to  k ni t  al though  ther e  i s  no  w eak ness
of   hands  or   ar m s  because  they   cannot  r epr oduce  the  necessar y   m oti on
to  cr eate  a  sti tch).   Agnosi a  i s  the  i nabi l i ty   to  r ecogni ze  sensor y
i nf or m ati on  (v i sual ,   audi tor y ,   etc. );  i t  m ay   i ncl ude  getti ng  l ost  i n
f am i l i ar   sur r oundi ngs  or   f ai l i ng  to  r ecogni ze  f am i l i ar   peopl e.   Ex ecuti v e
dy sf uncti on  i s  the  i nabi l i ty   to  com pl ete  a  sequence  of   task s  i n  pr oper
or der .   An  ex am pl e  of   ex ecuti v e  dy sf uncti on  m i ght  i ncl ude  l osi ng  the
abi l i ty   to  bal ance  a  check book .   On  a  m or e  basi c  l ev el ,   i t  m i ght  af f ect
the  abi l i ty   to  get  dr essed  (i . e. ,   i nabi l i ty   to  put  cl othes  on  i n  the  pr oper
sequence).

Other   causes  of   dem enti a  ar e  l ess  com m on.   The  absence  of   cl i ni cal
di agnosti c  cr i ter i a  that  unequi v ocal l y   separ ate  one  cause  of   dem enti a
f r om   another   obf uscates  ef f or ts  to  pi npoi nt  the  pr ev al ence  of   any
speci f i c  cause  of   dem enti a.   Vascul ar   dem enti a  (VD)  i s  ar guabl y   the
second  m ost  com m on  cause  of   dem enti a.   Tw o  f eatur es  hel p  di sti ngui sh
VD  f r om   AD,   cl i ni cal l y .   Al though  AD  pr i m ar i l y   af f ects  the  gr ay   m atter
of   the  tem por al   l obes,   VD  tends  to  i ncl ude  m ul ti pl e  sm al l   i nf ar cts  i n  the
deep  w hi te  m atter   of   the  br ai n.   In  VD,   thi s  di str i buti on  of   i schem i a
l eads  to  m ar k ed  sl ow i ng  i n  pati ent  r esponse  ti m e  to  questi ons  and  i s
m or e  l i k el y   to  pr oduce  f ocal   neur ol ogi c  m otor   and  sensor y   f i ndi ngs,
i ncl udi ng  gai t  di sor der s.   Spar i ng  of   the  m otor   cor tex   m ak es  m otor
f i ndi ngs,   i ncl udi ng  gai t  di sor der s,   m uch  l ess  com m on  i n  ear l y   AD  than
i n  VD.

Si nce  f i r st  descr i bed  i n  1961,   di f f use  Lew y   body   dem enti a  (DLBD)  has
r ecei v ed  i ncr easi ng  attenti on  as  a  m or e  com m on  cause  of   degener ati v e
dem enti a  than  pr ev i ousl y   r ecogni zed.   Al though  Lew y   bodi es  ar e  the
hal l m ar k   of   Par k i nson's  di sease  w hen  f ound  i n  the  basal   gangl i a,
par ti cul ar l y   the  substanti a  ni gr a,   they   m ay   appear   i n  cor ti cal   and
subcor ti cal   ar eas  as  w el l .   The  astute  cl i ni ci an  w i l l   suspect  a  di agnosi s
of   DLBD  w hen  pati ents  pr esent  w i th  a  tr i ad  of   pr ogr essi v e  but
f l uctuati ng  cogni ti v e  decl i ne,   par k i nsoni sm ,   and  v i sual   hal l uci nati ons
(hal l uci nati ons  ar e  not  ty pi cal   ear l y   i n  the  cour se  of   AD).   Si m i l ar l y ,
pati ents  w i th  a  di agnosi s  of   Par k i nson's  di sease  m ay   i ni ti al l y   appear
cogni ti v el y   i ntact  but  ov er   ti m e,   usual l y   w el l   af ter   the  m otor   si gns  of
Par k i nson's  di sease  hav e  pr ogr essed,   they   dev el op  pr ogr essi v e
dem enti a.   These  l atter   pati ents  hav e  som ew hat  ar bi tr ar i l y   been
di agnosed  as  hav i ng  the  “dem enti a  of
P. 196
Par k i nson's  di seaseâ€​
  to  di sti ngui sh  them   f r om   pati ents  w i th  DLBD  w ho
hav e  the  si m ul taneous  onset  of   m otor   and  cogni ti v e  dy sf uncti on.

Fr ontotem por al   dem enti as  ar e  a  heter ogeneous  gr oup  of   di sor der s  that
pr i m ar i l y   af f ect  the  f r ontal   and  tem por al   ar eas  of   the  br ai n.   Most  hav e
nonspeci f i c  degener ati v e  changes  and  not  the  Pi ck   bodi es  that
char acter i ze  Pi ck 's  di sease,   the  f i r st  of   these  di sor der s  to  be
speci f i cal l y   r ecogni zed.   These  pati ents  m ost  of ten  com e  to  m edi cal
attenti on  f or   behav i or al   and  speech  pr obl em s  (both  f l uent  and
nonf l uent)  r ather   than  pr i m ar i l y   f or   m em or y   l oss.

Besi des  VD,   the  non­AD  causes  of   dem enti a  ar e  r el ati v el y   uncom m on,
each  accounti ng  f or   l ess  than  5%  of   al l   dem enti a.   Other   eti ol ogi es  ar e
ev en  m or e  r ar e,   such  as  Cr eutzf el dt­Jak ob  di sease,   a  pr i on­r el ated
di sease  that  m ay   af f ect  as  f ew   as  one  i n  a  m i l l i on  peopl e  i n  the  U ni ted
States.   Other   r ar e  pr i m ar y   degener ati v e  neur ol ogi c  di seases  causi ng
dem enti a  w oul d  i ncl ude  Hunti ngton's  chor ea  or   pr ogr essi v e
supr anucl ear   pal sy ,   each  w i th  i ts  ow n  r el ati v el y   di sti nct  set  of   cl i ni cal
f eatur es.

2.   What  ar e  the  pathognom oni c  postm or tem   f i ndi ngs  of   AD?

N eur of i br i l l ar y   tangl es  and  neur i ti c  pl aques  ar e  postm or tem   f i ndi ngs
pathognom oni c  f or   AD,   and  the  di agnosi s  i s  cer tai n  onl y   i f   the
pathol ogi st  i denti f i es  a  si gni f i cant  num ber   of   these  l esi ons  i n  the
ty pi cal   di str i buti on  (i . e. ,   heav y   concentr ati ons  i n  the  hi ppocam pus  and
sur r oundi ng  ar eas  of   the  tem por al   l obes).   N i nety   per cent  or   m or e  of
pati ents  w i th  cl i ni cal l y   di agnosed  dem enti a  of   the  Al zhei m er 's  ty pe
hav e  the  di agnosi s  conf i r m ed  at  postm or tem   ex am i nati on.   Pl aques  and
neur of i br i l l ar y   tangl es  ar e  al so  f ound  i n  the  br ai ns  of   heal thy   el der l y
subjects,   but  i n  m uch  sm al l er   num ber s  than  i n  the  el der l y   pati ents  w i th
AD.   Depl eti on  of   chol i ner gi c  neur ons  i s  another   pathol ogi c  hal l m ar k ,
and  m ai ntenance  or   suppl em entati on  of   chol i ner gi c  f uncti on  has  been
the  f ocus  of   sev er al   tr eatm ents  of   AD.

Other   condi ti ons  that  m ay   be  cl i ni cal l y   conf used  w i th  AD  ar e  associ ated
w i th  di f f er ent  pathol ogi c  f i ndi ngs.   A  m ul ti f ocal   l oss  of   br ai n  ti ssue
secondar y   to  i schem i a  i s  seen  i n  the  setti ng  of   m ul ti i nf ar ct  dem enti a.
Degener ati on  of   the  dopam i ner gi c  cel l s  i n  the  substanti a  ni gr a  and
Lew y   bodi es  ar e  f ound  i n  pati ents  w i th  Par k i nson's  di sease.   Som eti m es
pathol ogi sts  f i nd  cor ti cal   and  subcor ti cal   neur onal   l oss  associ ated  w i th
Lew y   bodi es  outsi de  the  tr adi ti onal   di str i buti on  of   these  l esi ons  i n
Par k i nson's  di sease.   Thi s  enti ty   i s  now   i denti f i ed  as  DLBD  and  m ay
r epr esent  the  second  m ost  com m on  cause  of   neur odegener ati v e
dem enti a  af ter   AD.

3.   Can  the  chi l dr en  of   pati ents  w i th  AD  be  geneti cal l y   tested  and  tol d  w i th
assur ance  w hether   they   w i l l   i nher i t  the  di sease?

The  ev i dence  f or   a  her edi tar y   pr edi sposi ti on  of   AD  has  l ed  to  geneti c
r esear ch  that  has  i denti f i ed  sev er al   chr om osom al   abnor m al i ti es  that
i ncr ease  the  r i sk   f or   dev el opi ng  AD.   Resear cher s  hav e  i denti f i ed
def ecti v e  genes  i n  chr om osom es  1,   14,   and  21  that  ar e  l i nk ed  to
autosom al   dom i nant  i nher i tance  patter ns  of   AD  i n  a  sm al l   num ber   of
f am i l i es.   Af f l i cted  pati ents  i n  these  f am i l i es  of ten  hav e  ear l i er   onset  of
dem enti a,   betw een  35  and  65  y ear s  of   age,   w hi ch  i s  consi der abl y
ear l i er   than  the  usual   onset  i n  pati ents  w i th  l ate­l i f e  AD  ty pi cal l y
P. 197
begi nni ng  i n  the  ei ghth  decade  or   l ater .   Late­l i f e  onset  of   AD  occur s
m or e  of ten  i n  pati ents  w ho  hav e  the  Apol i popr otei n  (Apo)  E4  al l el e  on
chr om osom e  19.   Thr ee  ApoE  al l el es  hav e  been  descr i bed,   nam el y
ApoE2,   ApoE3,   and  ApoE4.   Al though  ApoE4  appear s  to  i ncr ease  the  r i sk
f or   dev el opm ent  of   l ate­onset  AD,   ApoE3  i s  the  m ost  com m onl y
i nher i ted  al l el e  and  appear s  to  conf er   nei ther   a  gr eater   nor   l esser   r i sk
of   dev el opi ng  AD.   ApoE2  i s  v er y   r ar e  (appr ox i m atel y   1%  of   the
popul ati on)  and  m ay   conf er   a  sl i ghtl y   l ow er   r i sk   of   AD.   N ot  al l
i ndi v i dual s  w i th  an  ApoE4  al l el e  w i l l   dev el op  AD  and,   conv er sel y ,   AD
occur s  am ong  m any   peopl e  w ho  ar e  hom ozy gous  f or   ApoE3.   Ther ef or e,
geneti c  testi ng,   w i th  the  ex cepti on  of   an  autosom al   dom i nant  patter n
i nher i tance  of   the  di sease,   does  not  r el i abl y   pr edi ct  an  i ndi v i dual 's  r i sk
of   dev el opi ng  AD.   Adv anci ng  age  r em ai ns  the  si ngl e  gr eatest  r i sk   f or
dev el opi ng  AD.   The  absence  of   consi stent  cor r el ati on  betw een  the
pr esence  or   absence  of   cur r entl y   k now n  geneti c  m ar k er s  and  the  r i sk
of   AD,   and  the  absence  of   i nter v enti ons  that  cl ear l y   del ay   or   pr ev ent
the  dev el opm ent  of   AD  (see  f ol l ow i ng  tex t)  suggest  that  geneti c  testi ng
cur r entl y   has  l i ttl e  cl i ni cal   uti l i ty .

4.   How   can  cogni ti v e  f uncti on  be  tested  qui ck l y   and  r el i abl y ?

N um er ous  studi es  hav e  show n  that  phy si ci ans  ov er l ook   m or e  than  50%
of   pati ents  w ho  hav e  cogni ti v e  i m pai r m ent.   Thi s  i s  m ost  of ten  due  to
the  cl i ni ci an's  f ai l ur e  to  do  f or m al   m ental   status  testi ng  that  w oul d
objecti v el y   i denti f y   these  def i ci ts.   The  Fol stei n  Mi ni –Mental   Status
Ex am i nati on  (MMSE)  and  si m i l ar   br i ef   m ental   status  tests  (e. g. ,   the
Pf ei f f er   and  the  Bl essed  Dem enti a  Scal es)  ar e  qui ck ,   r el i abl e  scr eeni ng
tool s  to  assess  cogni ti v e  f uncti on  and  m ay   esti m ate  the  sev er i ty   of
m ental   status  i m pai r m ent.   The  MMSE  m easur es  or i entati on,   m em or y ,
and  attenti on  as  w el l   as  the  status  of   w r i tten  and  spok en  l anguage  and
v i suospati al   sk i l l s.   Wi th  a  sensi ti v i ty   of   87%  and  speci f i ci ty   of   82%,
the  MMSE  r esul ts  ar e  r epr oduci bl e  w hen  the  test  i s  adm i ni ster ed  ei ther
by   a  heal th  car e  pr of essi onal   or   by   som eone  tr ai ned  to  adm i ni ster   the
test.   One  of   the  best  si ngl e­i tem   scr eeni ng  tests  i s  cl ock   dr aw i ng.   The
i nabi l i ty   to  dr aw   f am i l i ar ,   r el ati v el y   si m pl e  objects  m ay   r ef l ect
apr ax i a,   of ten  an  ear l y   si gn  of   dem enti a.   The  ex am i ner   ask s  the
pati ent  to  dr aw   a  cl ock   f ace,   f i l l   i n  the  num ber s,   and  then  dr aw   the
hour   and  m i nute  hands  i ndi cati ng  a  ti m e,   such  as  “10  m i nutes  past
2. â€​   Studi es  suggest  that  thi s  si m pl e  test  has  a  sensi ti v i ty   and
speci f i ci ty   si m i l ar   to  m or e  el abor ate  scr eeni ng  tool s  l i k e  the  MMSE.

5.   Can  the  i ntel l ectual   decl i ne  seen  i n  pati ents  w i th  AD  be  hal ted  or
r ev er sed  w i th  m edi cati ons?

Ef f or ts  to  hal t  or   at  l east  del ay   the  pr ogr essi on  of   cogni ti v e  decl i ne  i n
pati ents  suspected  of   hav i ng  AD  i s  ex tr em el y   chal l engi ng.   Fi r st,   the
cl i ni ci an  m ust  r ul e  out  potenti al   r ev er si bl e  f actor s  that  m ay   hasten  a
pati ent's  deter i or ati on.   Depr essi on  i s  a  com m on  com pl i cati on  of   AD.
Lef t  unr ecogni zed,   depr essi on  m ay   l ead  to  a  l oss  of   i nter est,   and
decr ease  i n  abi l i ty   to  concentr ate  and  f uncti on  i n  pati ents  w i th  AD.
Tr eatm ent  of   depr essi on  can  “r ev er seâ€​   som e  of   the  addi ti onal
decl i ne  i n  i ntel l ectual   f uncti on  that  occur s  w hen  depr essi on  i s  l ef t
untr eated.   Second,   m edi cati on  si de  ef f ects  can  gi v e  the  appear ance  of
pr ogr essi on  of   AD.   A  l ar ge  num ber   of   m edi cati ons,   i ncl udi ng
anti conv ul sants,   m uscl e
P. 198
r el ax er s,   anal gesi cs,   and  other s,   m ay   be  i m pl i cated.   Psy choacti v e
m edi cati ons,   par ti cul ar l y   those  w i th  anti chol i ner gi c  si de  ef f ects  such  as
tr i cy cl i c  anti depr essants,   ar e  notor i ous  f or   causi ng  r ev er si bl e  i ncr eased
conf usi on  and  cogni ti v e  decl i ne  i n  pati ents  w i th  under l y i ng  dem enti a.
The  ex per i enced  phy si ci an  w i l l   w or k   m ethodi cal l y   to  r educe  or
el i m i nate  m edi cati ons  that  m ay   ex acer bate  cogni ti v e  l osses  of   pati ents
w i th  AD,   r ecogni zi ng  that  m edi cati ons  ar e  the  si ngl e  m ost  com m on
cause  of   r ev er si bl e  cogni ti v e  i m pai r m ent.   In  addi ti on  to  r educi ng
m edi cati ons  that  m ay   ex acer bate  cogni ti v e  decl i ne,   cl i ni ci ans  shoul d
r ul e  out  abnor m al i ti es  such  as  B 1 2   def i ci ency ,   hy pothy r oi di sm ,   hy po­  or
hy per gl y cem i a,   hy ponatr em i a,   or   other   m etabol i c  pr obl em s  that  m ay
al so  hasten  cogni ti v e  i m pai r m ent.   Fi nal l y ,   str uctur al   abnor m al i ti es  such
as  subdur al   hem atom as,   nor m al   pr essur e  hy dr ocephal us,   or   br ai n
tum or s  occasi onal l y   l ead  to  r ev er si bl e  deter i or ati on  i n  m em or y   and
r el ated  i ntel l ectual   f uncti on.   The  pr esence  of   f ocal   neur ol ogi c  si gns
and/or   the  pr esence  of   a  gai t  di sor der   ar e  not  consi stent  w i th  a
di agnosi s  of   AD  and  m ay   tr i gger   a  r equest  f or   a  br ai n  i m agi ng  study   to
r ul e  out  one  of   the  thr ee  str uctur al   centr al   ner v ous  sy stem   pr obl em s
noted  i n  the  pr ecedi ng  tex t  that  m ay   pr esent  oppor tuni ti es  f or
i nter v enti on  to  r ev er se  cogni ti v e  l osses.

When  pati ents  w i th  AD  hav e  no  ev i dence  of   r ev er si bl e  contr i butor s  to
thei r   cogni ti v e  decl i ne,   ther api es  ai m ed  at  hal ti ng  or   r ev er si ng  di sease
pr ogr essi on  hav e  been  onl y   m odestl y   successf ul   to  date.   Recogni zi ng
that  chol i ner gi c  neur onal   l oss  i s  a  pr edom i nant  pathol ogi c  f i ndi ng  i n
AD,   i nv esti gator s  hav e  f ocused  on  f i ndi ng  w ay s  to  enhance  cer ebr al
chol i ner gi c  acti v i ty .   Thi s  ef f or t  l ed  to  the  dev el opm ent  of
chol i nester ase  i nhi bi tor s  that  bl ock   the  br eak dow n  of   acety l chol i ne  i n
the  br ai ns  of   pati ents  w i th  AD.   The  U . S.   Food  and  Dr ug  Adm i ni str ati on
(FDA)  has  appr ov ed  a  total   of   f i v e  m edi cati ons  f or   the  tr eatm ent  of   AD,
f our   of   w hi ch  ar e  chol i nester ase  i nhi bi tor s.   The  f i r st  of   the
chol i nester ase  i nhi bi tor s,   nam el y   tacr i ne  (Cognex ),   i s  no  l onger   used
because  i t  m ust  be  tak en  on  an  em pty   stom ach  f our   ti m es  a  day   and
has  been  associ ated  w i th  gastr oi ntesti nal   and  hepati c  tox i ci ty .
Donepezi l   (Ar i cept)  w as  the  second  agent  appr ov ed  and  can  be  tak en
once  a  day ,   usual l y   at  bedti m e  and  has  m i ni m al   gastr oi ntesti nal
tox i ci ty   and  no  r epor ted  hepatotox i ci ty .   The  FDA  al so  appr ov ed
gal antam i ne  (Razady ne)  and  r i v asti gm i ne  (Ex el on)  w hi ch  ar e  pr escr i bed
tw i ce  a  day   and  m ay   cause  sl i ghtl y   m or e  gastr oi ntesti nal   upset  but  al so
hav e  no  appar ent  hepatotox i ci ty .   Mor e  than  9, 000  pati ents  hav e  now
par ti ci pated  i n  r andom i zed  contr ol l ed  tr i al s  of   chol i nester ase  i nhi bi tor s
l asti ng  up  to  1  y ear .   Al l   w er e  phar m aceuti cal   com pany   sponsor ed  and
had  str i ct  cr i ter i a  f or   par ti ci pati on  that  som e  ex per ts  suggest  w oul d
hav e  ex cl uded  90%  of   pati ents  w i th  dem enti a.   Al l   of   these  studi es
show ed  m odest  sl ow i ng  of   pr ogr essi on  on  scal es  that  m easur ed
cogni ti v e  f uncti on,   behav i or ,   and  gl obal   f uncti on.   Thi s  ef f ect  i s
equi v al ent  to  pr ev enti ng  pr ogr essi on  of   AD  f or   a  f ew   m onths.   In  a
m odestl y   successf ul   attem pt  to  study   l onger ­ter m   ef f ects  of
chol i nester ase  i nhi bi tor s  i n  a  m or e  i ncl usi v e  gr oup  of   dem ented
pati ents  ov er   sev er al   y ear s,   a  gr oup  of   Br i ti sh  i nv esti gator s  r epor ted
thei r   r esul ts  i n  2004  f or   a  study   enti tl ed  “AD  2000. â€​   At  the  end  of
thi s  3­y ear   study ,   they   f ound  no  di f f er ences  i n  cl i ni cal l y   si gni f i cant
outcom es  such  as
P. 199
car egi v er   r epor t  of   the  pati ents'  f uncti on,   car egi v er   bur nout,   nur si ng
hom e  pl acem ent,   or   hospi tal i zati on.   The  study   di d  dem onstr ate  a
per si stent  sl ow i ng  of   decl i ne  i n  cogni ti v e  tests  that  w as  equi v al ent  to
del ay i ng  di sease  pr ogr essi on  f or   3  m onths.   The  absence  of   a  cl i ni cal l y
si gni f i cant  benef i t  w as  di sappoi nti ng.   Mem anti ne  (N am enda)  i s  the  onl y
other   m edi cati on  cur r entl y   FDA  appr ov ed  f or   tr eatm ent  of   AD.   Thi s  dr ug
i s  a  par ti al   antagoni st  of   the  N ­m ethy l   D­aspar tate  (N MDA)  r eceptor   i n
the  br ai n,   an  i m por tant  m edi ator   of   gl utam ate  acti v i ty .   Ex per i m ental
ev i dence  suggested  that  ex cessi v e  acti v i ty   of   the  N MDA  r eceptor   m ay
be  associ ated  w i th  pr ogr essi on  of   AD  and  suppr essi on  of   N MDA  acti v i ty
m i ght  sl ow   the  pr ogr essi on  of   the  di sease.   In  studi es  of   appr ox i m atel y
1, 000  pati ents,   m em anti ne  has  a  si m i l ar   ef f ect  as  the  chol i nester ase
i nhi bi tor s  i n  sl ow i ng  the  deter i or ati on  of   pati ent  per f or m ance  on
sev er al   scal es  i n  studi es  l asti ng  up  to  1  y ear .   Li k e  the  chol i nester ase
i nhi bi tor s,   m em anti ne  has  m i ni m al   tox i ci ty   but  has  not  been  subjected
to  l ong­ter m   r andom i zed  tr i al s  to  deter m i ne  i ts  ef f ecti v eness  i n  sl ow i ng
the  cl i ni cal   deter i or ati on  of   AD.

Epi dem i ol ogi c  and  sm al l   i nter v enti on  studi es  hav e  suggested  that
m edi cati ons  i ncl udi ng  estr ogen,   nonster oi dal   anti i nf l am m ator y   agents
such  as  i bupr of en,   v i tam i n  E,   sel egi l i ne  (a  m onoam i ne  ox i dase
i nhi bi tor ),   gi nk go  bi l oba,   and  other s  m ay   sl ow   the  pr ogr essi on  of   AD.
U nf or tunatel y ,   none  of   these  agents  hav e  pr ov ed  ef f ecti v e  to  date  i n
l ong­ter m ,   r andom i zed  studi es.   Ther ef or e,   none  of   these  m edi cati ons
has  r ecei v ed  FDA  appr ov al   i n  the  pr ev enti on  or   tr eatm ent  of   AD.

In  sum m ar y ,   no  cur r entl y   av ai l abl e  m edi cati ons  f or   the  tr eatm ent  of   AD
hav e  si gni f i cant  cl i ni cal   i m pact  on  the  pr ev enti on  or   pr ogr essi on  of   thi s
di sease.   Im pr ov em ent  i n  cogni ti on  and  f uncti on  i s  m ost  l i k el y   to  occur
w hen  the  cl i ni ci an  r educes  or   di sconti nues  m edi cati on  that  can  i nter f er e
w i th  cogni ti v e  f uncti on,   r ecogni zes  and  tr eats  depr essi on,   and  cor r ects
ov er l ook ed  m edi cal   condi ti ons  (e. g. ,   congesti v e  hear t  f ai l ur e,
em phy sem a)  or   m etabol i c  abnor m al i ti es  (e. g. ,   hy ponatr em i a,
hy pogl y cem i a).
Case
An  80­y ear ­ol d  w hi te  m an  i s  br ought  to  y ou  by   hi s  77­y ear ­ol d  w i f e  because
she  i s  concer ned  about  hi s  m em or y .   The  pati ent's  onl y   m edi cal   pr obl em   i s
m i l d  hy per tensi on,   tr eated  w i th  hy dr ochl or othi azi de  (12. 5  m g  dai l y ).   Dur i ng
the  i ni ti al   outpati ent  i nter v i ew ,   hi s  w i f e  conf i des  that  appr ox i m atel y   2  y ear s
ago  she  began  to  noti ce  he  w as  becom i ng  m or e  f or getf ul   and  i r r i tabl e.   A
r eti r ed  school teacher ,   he  had  al w ay s  been  a  l i ttl e  stubbor n  but  i ncr easi ng
stubbor nness  i s  tax i ng  hi s  w i f e's  pati ence.   One  y ear   ago,   the  w i f e  took   ov er
r esponsi bi l i ti es  f or   w r i ti ng  check s  and  pay i ng  bi l l s  w hen  her   husband  f el l
behi nd  i n  thi s  r esponsi bi l i ty   and  they   began  to  r ecei v e  ov er due  noti ces.
Gr adual l y ,   hi s  i nter ests  and  i nv ol v em ent  i n  acti v i ti es  that  he  pr ev i ousl y
enjoy ed  hav e  decl i ned.   He  has  begun  to  nap  dur i ng  the  day   and  then  stay   up
at  ni ght.   Som eti m es  she  has  f ound  hi m   i n  the  k i tchen  “pr epar i ng  di nner â
€​  at  3:00  a. m .   She  has  becom e  af r ai d  to  l eav e  hi m   al one  at  hom e.   Si x
m onths  ago,   he  w as  i nv ol v ed  i n  a  m i nor   m otor   v ehi cl e  acci dent  and  w as
char ged  w i th  f ai l ur e  to  y i el d  the  r i ght­of ­w ay ,   but  has  r ef used  to  stop
dr i v i ng  despi te  sev er al   near ­col l i si ons  si nce  then.
P. 200
You  f i nd  the  pati ent  to  be  a  tal l ,   w el l ­dr essed  m an  w i th  a  f r i endl y   m anner
but  l i ttl e  spontanei ty .   Hi s  bl ood  pr essur e  i s  165/80  m m   Hg;  pul se,   75  beats
per   m i nute  and  r egul ar ;  and  r espi r ati ons,   18  per   m i nute.   Hi s  tem per atur e  i s
37°C  (98. 6°F).   Fi ndi ngs  dur i ng  the  phy si cal   ex am i nati on,   i ncl udi ng  a
thor ough  neur ol ogi c  ex am i nati on,   ar e  nor m al   ex cept  f or   bi l ater al   gr asp
r ef l ex es  (i nv ol untar y   gr aspi ng  of   the  ex am i ner 's  hand  w hen  the  pati ent's
pal m s  ar e  str ok ed  by   the  ex am i ner 's  f i nger s).   He  ex hi bi ts  di f f i cul ty
f ol l ow i ng  si m pl e  com m ands.   Hi s  Fol stei n  MMSE  scor e  i s  20/30  (nor m al ,   > 23)
and  he  i s  unaw ar e  of   hi s  er r or s.   He  scor es  3/30  on  the  Ger i atr i c  Depr essi on
Scal e  (nor m al   < 15/30),   suggesti ng  that  he  i s  not  depr essed.   When  ask ed
how   thi ngs  ar e  at  hom e,   he  hesi tates  and  say s,   “f i ne. â€​   On  f ur ther
questi oni ng  about  hi s  r el ati onshi p  w i th  hi s  w i f e,   al l   he  say s  i s  that  hi s  w i f e
i s  a  “good  w om an. â€​   Hi s  sel f ­assessm ent  i s  that  he  i s  doi ng  w el l   “f or
an  ol d  m an. â€​   When  ask ed  about  hi s  m em or y ,   he  say s  that  “i t's  goodâ€​
and  he  has  no  pr obl em s  r em em ber i ng  “i m por tant  thi ngs. â€​   Labor ator y
ev al uati on  r ev eal s  nor m al   hem atocr i t  and  ser um   cr eati ni ne  v al ues.   Li v er
f uncti on  test  r esul ts  ar e  nor m al .   Hi s  v i tam i n  B 1 2   l ev el   i s  480  pg/m L
(nor m al ,   225  to  800  pg/m L);  f ol ate,   at  10  ng/m L,   and  thy r oi d­sti m ul ati ng
hor m one,   at  3  IU /m L,   w er e  al so  nor m al .   A  r api d  pl asm a  r eagi n  test  (f or
sy phi l i s)  i s  nonr eacti v e.   A  head  com puted  tom ogr aphy   (CT)  scan  obtai ned  at
the  ti m e  of   hi s  autom obi l e  acci dent  6  m onths  ago  r epor tedl y   show ed  â
€œcer ebr al   atr ophy ,   consi stent  w i th  age. â€​

1.   Whi ch  aspect  of   thi s  pati ent's  pr esentati on  i s  m ost  v al uabl e  i n
f or m ul ati ng  a  di f f er enti al   di agnosi s?
2.   On  a  CT  scan  or   m agneti c  r esonance  i m agi ng  (MRI),   w hat  f i ndi ngs  ar e
m ost  char acter i sti c  of   AD  or   other   causes  of   dem enti a?
3.   For   w hat  potenti al l y   tr eatabl e  cause  of   m em or y   l oss  shoul d  thi s  pati ent
be  scr eened?
4.   Can  any thi ng  be  done  to  hel p  hi s  w i f e  m anage  the  behav i or   of   her
husband?

Case Discussion
1.   Whi ch  aspect  of   thi s  pati ent's  pr esentati on  i s  m ost  v al uabl e  i n
f or m ul ati ng  a  di f f er enti al   di agnosi s?

An  i m m edi ate  cl ue  to  the  pati ent's  di agnosi s  i s  hi s  pr esentati on.   Hi s
w i f e  m ade  the  appoi ntm ent  because  she  i s  concer ned  about  hi s
m em or y ,   al though  the  pati ent  seem s  l ess  aw ar e  of   hi s  def i ci ts.   Thi s
patter n  i s  char acter i sti c  of   dem enti a.   If   the  pati ent  had  m ade  the
appoi ntm ent  hi m sel f   and  had  com e  al one  com pl ai ni ng  about  hi s
m em or y   or   di f f i cul ty   i n  concentr ati on,   thi s  patter n  w oul d  be  m or e
consi stent  w i th  depr essi on.   When  dem enti a  i s  adv anced,   i ts  di agnosi s  i s
obv i ous.   Ear l y   on,   how ev er ,   the  pati ent  m ay   hi de  or   r ati onal i ze  hi s
def i ci ts  and  hi s  cogni ti v e  changes  m ay   be  so  subtl e  that  they   ar e  m or e
appar ent  at  hom e  than  i n  the  cl i ni ci an's  of f i ce.   Thi s  i s  w her e  the
f am i l y 's  obser v ati ons  becom e  ex tr em el y   hel pf ul .   In  thi s  case,   the
pati ent's  w i f e  suppl i ed  m any   cl ues  to  her   husband's  dem enti a.

A  nor m al   phy si cal   ex am i nati on  i s  com m on  i n  a  pati ent  w i th  ear l y   AD.
The  f i r st  pathol ogi c  changes  i n  AD  occur   m ostl y   i n  the  tem por al   and
par i etal   l obes  of   the  br ai n  and  spar e  the  m otor   str i p.   Ther ef or e,   the
f i r st  si gns  of   di sease  ar e  f r equentl y   l i m i ted  to  m em or y   i m pai r m ent,
subtl e  per sonal i ty   changes  (e. g. ,   i ncr eased  i r r i tabi l i ty   or   f l atteni ng  of
af f ect),   aphasi a,   and  apr ax i a.   Gai t  di sor der   and  m otor   f i ndi ngs  ar e
unusual .

P. 201
The  onl y   si gni f i cant  f i ndi ng  dur i ng  thi s  pati ent's  neur ol ogi c
ex am i nati on,   besi des  hi s  abnor m al   m ental   status  ex am i nati on,   w as
bi l ater al   gr asp  r ef l ex es.   Thi s  r esponse,   the  i nv ol untar y   gr aspi ng  of   the
ex am i ner 's  f i nger s  w hen  the  ex am i ner   str ok es  the  pati ent's  pal m ,   i s  a
pr i m i ti v e  r ef l ex   that  m ay   appear   w i th  bi l ater al   f r ontal   l obe  di sease,
w hi ch  m ay   occur   i n  AD  as  w el l   as  other   dem enti as.

2.   On  a  CT  scan  or   MRI,   w hat  f i ndi ngs  ar e  m ost  char acter i sti c  of   AD  or
other   causes  of   dem enti a?

CT  scanni ng  or   MRI  m ay   show   ev i dence  of   tem por al   l obe  atr ophy   i n
ear l y   AD.   How ev er ,   neur oi m agi ng  ev i dence  of   cer ebr al   atr ophy
cor r el ates  m or e  w i th  adv anci ng  age  than  i t  does  w i th  m ental   status
decl i ne.   CT  scan  or   MRI  f i ndi ngs  of   w hi te  m atter   di sease  consi stent
w i th  m ul ti i nf ar ct  dem enti a  hav e  been  r epor ted  i n  pati ents  w i th  nor m al
cogni ti on.   Conv er sel y ,   MRI  and  CT  scan  f ai l   to  show   abnor m al i ti es  i n
20%  of   pati ents  w ho  hav e  cl i ni cal l y   di agnosed  AD.   Ther ef or e,   i t  i s  not
sur pr i si ng  that  the  pati ent's  CT  scan  f i ndi ngs  w er e  nor m al   f or   hi s  age.
If   the  dem enti a  has  gr adual l y   pr ogr essed  f or   2  or   m or e  y ear s,   i f   the
m ental   status  ex am i nati on  show s  sev er e  i m pai r m ent,   and  i f   the  pati ent
has  no  f ocal   neur ol ogi c  f i ndi ngs  or   gai t  di sor der ,   neur oi m agi ng  i s
ex tr em el y   unl i k el y   to  r ev eal   f i ndi ngs  that  w i l l   al ter   m anagem ent.

3.   For   w hat  potenti al l y   tr eatabl e  cause  of   m em or y   l oss  shoul d  thi s  pati ent
be  scr eened?

The  goal   of   the  ev al uati on  i s  to  i denti f y   di seases  that  can  be  di agnosed
conf i dentl y ,   or   f or   w hi ch  ther e  i s  tr eatm ent  that  m i ght  r ev er se  the
cogni ti v e  def i ci ts.   Ther ef or e,   the  phy si ci an  shoul d  r outi nel y   tak e  a
car ef ul   hi stor y ,   com pl ete  a  car ef ul   phy si cal   ex am i nati on,   and  or der   a
basi c  l abor ator y   ev al uati on  i ncl udi ng  a  com pl ete  bl ood  count,   ser um
el ectr ol y tes,   cal ci um ,   cr eati ni ne,   thy r oi d­sti m ul ati ng  hor m one,   and
v i tam i n  B 1 2   l ev el .   The  phy si ci an  shoul d  or der   other   tests,   such  as  CT
scan  or   MRI,   based  on  the  r esul ts  of   the  hi stor y   and  phy si cal
ex am i nati on.   For   ex am pl e,   i f   the  pati ent  has  had  a  hi stor y   of   r ecent  or
sudden  onset  of   cogni ti v e  i m pai r m ent  af ter   head  tr aum a,   the  possi bi l i ty
of   a  subdur al   hem atom a  w oul d  i ndi cate  the  need  f or   br ai n  i m agi ng.
Thi s  i s  par ti cul ar l y   tr ue  i f   the  phy si cal   ex am i nati on  r ev eal s  a  gai t
di sor der   or   f ocal   neur ol ogi c  si gns.   The  tr i ad  of   dem enti a  of   r ecent
onset,   gai t  di sor der ,   and  ur i nar y   i nconti nence  m ay   suggest  the
di agnosi s  of   nor m al   pr essur e  hy dr ocephal us,   another   potenti al l y
r ev er si bl e  cause  of   cogni ti v e  decl i ne.   Thi s  di sor der   i s  ex tr em el y   r ar e,
and,   al though  som e  pati ents  m ay   ex per i ence  i m pr ov em ent  w i th
v entr i cul ar   shunti ng,   postoper ati v e  com pl i cati ons  (e. g. ,   subdur al
hem atom a,   i nf ecti on,   and  shunt  obstr ucti on)  ar e  v er y   com m on.   For   the
pati ent  descr i bed  i n  the  pr ecedi ng  tex t,   these  di agnosti c  possi bi l i ti es
w oul d  not  be  l i k el y .

Hy pothy r oi di sm   and  v i tam i n  B 1 2   def i ci ency   suf f i ci ent  to  af f ect  neur onal
f uncti on  usual l y   cause  di stur bances  i n  attenti on  and  consci ousness,   and
ar e  di agnosed  and  tr eated  l ong  bef or e  dem enti a  appear s.   Occasi onal l y ,
how ev er ,   a  pati ent  del ay s  getti ng  m edi cal   car e  unti l   dem enti a  i s
pr esent,   so  al l   pati ents  shoul d  be  ev al uated  f or   these  condi ti ons.

N eur osy phi l i s  i s  no  l onger   a  com m on  cause  of   cogni ti v e  i m pai r m ent.
These  pati ents  usual l y   hav e  other   neur ol ogi c  f i ndi ngs,   such  as  dor sal
col um n  di sease  m ani f est  by   l oss  of   posi ti on  and  v i br ator y   sensati on,   i n
addi ti on  to  m ental   status  decl i ne.

A  sev er el y   depr essed  pati ent  m ay   seem   di sor i ented  and  per f or m   poor l y
on  tests  of   cogni ti v e  f uncti on.   These  def i ci ts  m ay   be  due  to  r ev er si bl e
changes  that  m i m i c
P. 202
the  i r r ev er si bl e  changes  of   dem enti a.   Because  the  di agnosi s  of
depr essi on  can  be  di f f i cul t  and  i t  i s  based  on  subtl e  f i ndi ngs  i n  an
el der l y   pati ent,   m any   tool s,   such  as  the  Ger i atr i c  Depr essi on  Scal e,
hav e  been  dev el oped  to  ai d  i n  i ts  di agnosi s.
U nf or tunatel y ,   the  pati ent  descr i bed  her e  di d  not  ex hi bi t  any   of   these
potenti al l y   tr eatabl e  abnor m al i ti es.

4.   Can  any thi ng  be  done  to  hel p  hi s  w i f e  m anage  the  behav i or   of   her
husband?

Yes.   Ther e  ar e  w ay s  to  hel p  the  pati ent's  w i f e  m anage  her   husband's
behav i or .   Car i ng  f or   a  dem ented  pati ent  i s  a  phy si cal l y   and  em oti onal l y
ex hausti ng  job.   As  r ecom m endati ons  ar e  m ade,   the  phy si ci an  m ust
consi der   not  onl y   the  pati ent  but  al so  the  car egi v er .   Al l ow i ng
car egi v er s  to  v ent  em oti ons,   ack now l edgi ng  the  di f f i cul ty   of   thei r   task ,
tel l i ng  them   w hat  to  ex pect  as  the  di sease  pr ogr esses,   of f er i ng  r espi te
car e,   and  r ef er r i ng  them   to  suppor t  gr oups  ar e  sm al l   thi ngs  that  m ay
hel p  them   cope  better   w i th  the  pati ent  and  hi s  or   her   needs.

The  tr eatm ent  of   behav i or al   pr obl em s  i s  di f f i cul t,   but  can  be  ef f ecti v e.
Regul ar   ex er ci se  and  l i m i ti ng  the  num ber   and  dur ati on  of   l ate  af ter noon
or   ev eni ng  naps  m ay   hel p  r educe  the  noctur nal   i nsom ni a  that  of ten
com pl i cates  the  m anagem ent  of   dem ented  el der l y   pati ents.   Most
sedati v es  and  hy pnoti cs,   par ti cul ar l y   the  l ong­acti ng  ones,   shoul d  not
be  used  because  they   m ay   cause  ov er sedati on  or   a  par adox i cal
i ncr ease  i n  agi tati on,   and  m ay   onl y   w or sen  cogni ti v e  and  behav i or al
def i ci ts.

Del usi ons  ar e  com m on  i n  dem enti a  sy ndr om es.   In  f act,   appr ox i m atel y
50%  of   the  pati ents  w i th  AD  or   m ul ti i nf ar ct  dem enti a  ex per i ence
del usi ons.   Agi tati on  and  com bati v e  behav i or s  can  accom pany   these
sy m ptom s.   The  cauti ous  use  of   l ow   doses  of   hal oper i dol ,   or   other
anti psy choti cs,   m ay   be  hel pf ul   i n  am el i or ati ng  these  behav i or s.

Suggested Readings
AD  2000  Col l abor ati v e  Gr oup.   Long­ter m   donepezi l   tr eatm ent  i n  565
pati ents  w i th  Al zhei m er 's  di sease  (AD2000):  r andom i zed  doubl e­bl i nd
tr i al .   Lancet  2004;363:2105.

Boustani   M,   Peter son  B,   Hanson  L,   etal .   Scr eeni ng  f or   dem enti a  i n
pr i m ar y   car e:  a  sum m ar y   of   the  ev i dence  f or   the  U . S.   Pr ev enti v e
Ser v i ces  Task   For ce.   Ann  Inter n  Med  2003;138:927.

Inouy e  SK.   Del i r i um   i n  ol der   per sons.   N   Engl   J  Med  2006;354:11.

Kaw as  C.   Ear l y   Al zhei m er 's  di sease.   N   Eng  J  Med  2003;349:1056.

Falls in the Elderly
1.   How   com m onl y   do  f al l s  occur   i n  the  el der l y ?

2.   How   of ten  does  i njur y   or   death  r esul t  f r om   a  f al l ?

3.   What  f actor s  m ak e  the  el der l y   m or e  l i k el y   to  f al l ?

4.   What  shoul d  the  hi stor y   and  phy si cal   ex am i nati on  f ocus  on  i n  a  pati ent
w ho  i s  hav i ng  pr obl em s  w i th  f al l i ng?

P. 203

Discussion
1.   How   com m onl y   do  f al l s  occur   i n  the  el der l y ?

Thi r ty   per cent  of   the  el der l y   ol der   than  65  y ear s  w ho  l i v e  out  i n  the
com m uni ty   ex per i ence  f al l s  annual l y .   Most  pati ents  ar e  r el uctant  to  tel l
thei r   phy si ci an  or   f am i l y   m em ber s  of   these  f al l s,   so  thi s  f i gur e  i s
pr obabl y   an  under esti m ate.   N ear l y   8%  of   i ndi v i dual s  ol der   than  70
y ear s  w i l l   v i si t  an  em er gency   r oom   annual l y   and  one  thi r d  of   these  w i l l
be  hospi tal i zed  f or   an  av er age  of   m or e  than  1  w eek .   Because  a  hi stor y
of   f al l s  i ncr eases  the  r i sk   of   f utur e  f al l s  (i . e. ,   60%  chance  w i th  the
f i r st  y ear   af ter   an  i ndex   f al l )  and  ser i ous  i njur y ,   phy si ci ans  shoul d
r outi nel y   ask   el der l y   pati ents  i f   they   hav e  f al l en  and  then  i nter v ene  to
r educe  the  r i sk   of   f utur e  f al l s.

2.   How   of ten  does  i njur y   or   death  r esul t  f r om   a  f al l ?

Acci dental   i njur y   i s  the  si x th  l eadi ng  cause  of   death  i n  peopl e  ol der
than  65  y ear s,   and  tw o  thi r ds  of   these  deaths  ar e  r el ated  to  f al l s.
Fr actur es  occur   i n  appr ox i m atel y   5%  of   f al l s,   and  the  m ost  com m on
f r actur e  si tes  ar e  the  spi ne,   hi p,   hum er us,   w r i st,   and  pel v i s.   Another
5%  of   f al l s  cause  sof t  ti ssue  i njur i es,   such  as  spr ai ns,   joi nt
di sl ocati ons,   and  hem atom as.   Ev en  i n  those  cases  i n  w hi ch  no  i njur y   i s
ev i dent,   ther e  ar e  sti l l   consequences;  a  per son  w ho  has  f al l en  m ay
becom e  em oti onal l y   par al y zed  by   a  “f ear   of   f al l i ngâ€​   and  begi n  to
l i m i t  acti v i ti es.   Soon  they   becom e  soci al l y   i sol ated  and  becom e  ev en
w eak er   as  they   ar e  l ess  acti v e.   Ther ef or e,   ev en  w hen  a  f al l   does  not
cause  si gni f i cant  str uctur al   dam age,   i t  m ay   hav e  a  negati v e  i m pact  on
a  per son's  qual i ty   of   l i f e  and  i ndependence.   In  nur si ng  hom es,   50%  or
m or e  of   the  am bul ator y   r esi dents  f al l   each  y ear ,   despi te  the  pr esence
of   tr ai ned  staf f   and  car ef ul   obser v ati on  of   saf ety   m easur es.
Appr ox i m atel y   4%  of   al l   pati ents  i n  nur si ng  hom es  hav e  tr aum ati c  bone
f r actur es  annual l y ,   i ncl udi ng  a  1%  r i sk   of   hi p  f r actur e  each  y ear .

3.   What  f actor s  m ak e  the  el der l y   m or e  l i k el y   to  f al l ?

Fr equentl y ,   m ul ti pl e  f actor s,   r ather   than  a  si ngl e  pr obl em ,   contr i bute  to


an  el der l y   pati ent's  r i sk   of   f al l i ng.   It  i s  of ten  best  to  di v i de  these
f actor s  i nto  tw o  categor i es:  i ntr i nsi c  and  ex tr i nsi c.   Intrins ic  fa c tors
ar e  those  r el ated  to  agi ng  and  di sease  pr ocesses.   These  i ncl ude
changes  i n  bal ance  and  gai t,   pai n  and  sti f f ness  due  to  ar thr i ti s,
decr eased  m uscl e  str ength,   di zzi ness,   postur al   hy potensi on,   sensor y
l osses  (hear i ng,   v i si on,   and  pr opr i ocepti on),   cogni ti v e  i m pai r m ent,   and
sy ncope.   Other   i ntr i nsi c  causes  to  consi der   ar e  v er tebr obasi l ar
i nsuf f i ci ency ,   depr essi on,   hy pothy r oi di sm ,   m echani cal   f oot  pr obl em s,
or   car di ac  ar r hy thm i as.

Pati ents  tend  to  attr i bute  thei r   f al l s  to  e x trins ic  fa c tors ,   such  as
tr i ppi ng  ov er   obstacl es,   but,   w i th  adv anci ng  age,   i t  becom es  l ess  l i k el y
that  ex tr i nsi c  f actor s  al one  ar e  at  f aul t.   Indeed,   m ost  f al l s  i n  f r ai l
el der l y   occur   dur i ng  r outi ne  acti v i ti es  of   dai l y   l i v i ng.   For   ex am pl e,   a
one  out  of   f our   f al l s  occur s  w hen  the  pati ent  i s  cl i m bi ng  or   descendi ng
stai r s.   How ev er ,   cer tai n  ex tr i nsi c  f actor s  such  as  m edi cati on  si de
ef f ects  (e. g. ,   or thostasi s,   di zzi ness,   i m bal ance)  ar e  com m on.   Si m i l ar l y ,
f al l i ng  m ay   be  the  f i r st  cl ue  to  suggest  a  di agnosi s  of   occul t
al cohol i sm ,   l eadi ng  to  poor   bal ance  and  subsequent  f al l s.   Other
ex tr i nsi c  f actor s  that  m ay   contr i bute  to  f al l s  i ncl ude  i nadequate
l i ghti ng,   sl i pper y   f l oor s,   l oose
P. 204
thr ow   r ugs,   ex posed  el ectr i cal   cor ds,   i tem s  out  of   r each  (so  that
pati ents  stand  on  unstabl e  chai r s  or   other   suppor ts  and  l ose  thei r
bal ance  m or e  easi l y ),   l ack   of   assi sti v e  dev i ces  such  as  bathr oom   r ai l s
to  steady   them sel v es  w hen  they   ar e  usi ng  the  tub  or   show er ,   too  hi gh  a
bed  (so  that  f al l s  f r om   the  bed  m or e  l i k el y   r esul t  i n  si gni f i cant  i njur y ),
unsaf e  stai r s,   and  poor l y   f i tti ng  shoes.   Identi f y i ng  and  el i m i nati ng  or
r educi ng  ex tr i nsi c  f actor s  r equi r e  a  com pr ehensi v e  r ev i ew   of   the
pati ent's  l i v i ng  si tuati on,   as  w el l   as  f i ndi ngs  on  phy si cal   ex am i nati on.
An  occupati onal   ther api st  or   other   m em ber s  of   a  hom e  car e  team   m ay
com pl ete  a  hom e  saf ety   ev al uati on,   i denti f y   hazar ds,   and  cor r ect  them
and  ther eby   r educe  the  r i sk   of   subsequent  f al l s.

4.   What  shoul d  the  hi stor y   and  phy si cal   ex am i nati on  f ocus  on  i n  a  pati ent
w ho  i s  hav i ng  pr obl em s  w i th  f al l i ng?

A  car ef ul   hi stor y   of   the  f al l i ng  epi sodes  shoul d  be  obtai ned.   Thi s
i ncl udes  the  f r equency   of   f al l s,   the  pati ent's  acti v i ty   at  the  ti m e  of   the
f al l ,   w her e  they   occur ,   and  associ ated  sy m ptom s  such  as  l oss  of
consci ousness.   It  i s  i m por tant  to  get  i nf or m ati on  f r om   any one  w ho  m ay
hav e  w i tnessed  the  f al l   and  can  pr ov i de  a  m or e  detai l ed  descr i pti on  of
the  ci r cum stances.   For   ex am pl e,   i f   the  f al l s  ar e  associ ated  w i th
di zzi ness  and  consi stentl y   occur   30  to  60  m i nutes  af ter   a  m eal ,
postpr andi al   or thostati c  hy potensi on  m ay   be  suspected.   Ask   car ef ul l y
about  dr ug  usage,   i ncl udi ng  ov er ­the­counter   m edi cati ons  as  ev en
dr ugs  such  as  di phenhy dr am i ne  (Benadr y l ),   a  com m on  i ngr edi ent  i n
ov er ­the­counter   sl eep  ai ds,   hav e  anti chol i ner gi c  pr oper ti es  that  m ay
contr i bute  to  poor   bal ance  and  subsequent  f al l s.   Phy si cal   ex am i nati on
m ust  i ncl ude  com pr ehensi v e  v i tal   si gns,   i ncl udi ng  pul se  and  bl ood
pr essur e,   tak en  l y i ng  and  standi ng,   to  i denti f y   or thostati c  hy potensi on
that  of ten  contr i butes  to  f al l   r i sk .   On  neur ol ogi c  ex am i nati on,   v i sual
acui ty   and  per i pher al   v i si on,   str ength,   and  cer ebel l ar ,   sensor y ,   and
m ental   status  m ust  be  assessed,   l ook i ng  f or   i m pai r ed  v i si on,   w eak ness,
atax i a,   neur opathy ,   or   dem enti a.   A  usef ul   scr eeni ng  test  f or   bal ance,
str ength,   m obi l i ty ,   and  endur ance  i s  the  “get­up­and­goâ€​   test.   The
ex am i ner   ask s  the  pati ent  to  get  up  f r om   a  chai r   (w i thout  usi ng  hi s  or
her   hands  to  push  up  f r om   the  chai r ),   w al k   appr ox i m atel y   15  to  20
f eet,   tur n  ar ound,   w al k   back   to  the  chai r ,   and  si t  dow n,   agai n  w i thout
usi ng  thei r   ar m s  to  l ow er   them sel v es  i nto  the  chai r .   The  “get­up­
and­goâ€​   test  tak es  v er y   l i ttl e  ti m e  and  r ev eal s  m uch  about  the
pati ent's  gai t  and  saf ety .   Bey ond  scr eeni ng  l abor ator y   tests  such  as  a
com pl ete  bl ood  count  or   chem i str y   panel ,   v i tam i n  B 1 2   and  thy r oi d­
sti m ul ati ng  hor m one  l ev el s  shoul d  be  m easur ed  i f   ther e  i s  ev i dence  of
a  per i pher al   neur opathy   or   of   di f f use  m uscul ar   w eak ness.   Other   tests
such  as  v i sual   acui ty ,   assessm ent  of   v esti bul ar   f uncti on  (e. g. ,
el ectr ony stagm ogr aphy ),   am bul ator y   car di ac  m oni tor i ng,   or   CT
scanni ng  shoul d  be  done  onl y   i f   ther e  ar e  cl i ni cal   cl ues  to  speci f i c
di sor der s  that  m ay   cause  f al l s  (i . e. ,   v er ti go,   sy ncope,   or   f ocal
neur ol ogi c  f i ndi ngs).

Case
An  86­y ear ­ol d  m an  i s  seen  because  of   a  hi stor y   of   f r equent  f al l s,   r epor ted
by   hi s  w i f e.   She  r epor ts  that  he  f al l s  at  l east  thr ee  ti m es  per   w eek ,   usual l y
w i thout  i njur y .   How ev er ,   he  has  r equi r ed  tw o  tr i ps  to  the  em er gency   r oom
i n  the  l ast  3  m onths  w her e  he  r equi r ed
P. 205
sutur i ng  of   l acer ati ons  r ecei v ed  i n  the  f al l s.   These  f al l s  ar e  not
accom pani ed  by   l oss  of   consci ousness,   pal pi tati ons,   or   sei zur e  acti v i ty .   Hi s
m edi cal   hi stor y   i s  r em ar k abl e  f or   m i l d  dem enti a,   sev er e  degener ati v e  joi nt
di sease  w i th  chr oni c  l ow   back   pai n,   decr eased  hear i ng  and  v i si on,   beni gn
f am i l i al   tr em or ,   and  ur i nar y   i nconti nence  (r el ated  to  hi s  dem enti a).   Hi s
cur r ent  m edi cati ons  i ncl ude  cal ci um   suppl em entati on,   pr opr anol ol   (40  m g
thr ee  ti m es  a  day ),   an  ov er ­the­counter   sl eep  m edi cati on  that  contai ns  50
m g  of   di phenhy dr am i ne  w hi ch  he  tak es  ni ghtl y   f or   chr oni c  i nsom ni a,   and
acetam i nophen  as  needed.   Hi s  w i f e  r epor ts  that  he  has  a  cane  and  a  w al k er
but  r ar el y   uses  them .
Phy si cal   ex am i nati on  r ev eal s  a  pl easant,   thi n,   dem ented  m an.   Hi s
tem per atur e  i s  37°C  (98. 6°F);  r espi r ati ons  20  per   m i nute;  pul se,   55  beats
per   m i nute;  and  supi ne  bl ood  pr essur e  105/70  m m   Hg.   On  standi ng,   hi s
pul se  r ate  r em ai ns  at  55  beats  per   m i nute  but  hi s  bl ood  pr essur e  dr ops  to
85/65  m m   Hg  and,   w hen  ask ed,   he  say s  he  f eel s  “w oozy . â€​   Car di ac
ex am i nati on  f i ndi ngs  ar e  unr em ar k abl e;  the  r hy thm   i s  r egul ar   and  ther e  ar e
no  m ur m ur s  or   gal l ops.   Hi s  gai t  i s  som ew hat  atax i c.   Hi s  cr ani al   ner v es  ar e
i ntact  and  ther e  i s  no  ny stagm us.   He  has  a  f i ne  tr em or   i n  both  hands  w hen
they   ar e  hel d  i n  ex tensi on,   but  he  has  nor m al   tone  and  str ength  i n  al l
ex tr em i ti es.   Sensor y   ex am i nati on  i s  i ntact.   Fi nger ­to­nose  and  heel ­to­shi n
testi ng  dem onstr ates  no  dy sm etr i a.   Hi s  postur e  i s  stooped  and  hi s  w i de­
based  gai t  i s  unsteady .   He  w al k s  by   hol di ng  on  to  the  of f i ce  f ur ni tur e.
Labor ator y   tests  consi sti ng  of   com pl ete  bl ood  count  and  el ectr ol y te  and
cr eati ni ne  m easur em ents  y i el d  unr em ar k abl e  f i ndi ngs.

1.   What  pr obl em s  ar e  contr i buti ng  to  thi s  pati ent's  f al l s?


2.   What  di agnosti c  tests  m ay   be  the  m ost  hel pf ul   i n  thi s  pati ent?
3.   What  i nter v enti on,   or   i nter v enti ons,   w oul d  y ou  i nsti tute  to  decr ease
thi s  pati ent's  r i sk   of   f al l i ng?

Case Discussion
1.   What  pr obl em s  ar e  contr i buti ng  to  thi s  pati ent's  f al l s?

The  hi stor y   and  phy si cal   ex am i nati on  f i ndi ngs  suggest  a  num ber   of
f actor s  contr i buti ng  to  thi s  pati ent's  f al l s.   Degener ati v e  joi nt  di sease
i ncr eases  the  r i sk   of   f al l s  i n  a  num ber   of   w ay s.   Fi r st,   sti f f ness  and
change  i n  postur e  af f ect  bal ance  and,   second,   joi nt  pai n  ex per i enced
w hi l e  w al k i ng  m ay   di scour age  acti v i ty   and  ex er ci se,   w hi ch,   i n  tur n,
contr i butes  to  decr eased  m uscl e  tone  and  bal ance  and  an  i ncr eased  r i sk
of   f al l i ng.   Thi s  v i ci ous  cy cl e  m ay   f oster   a  f ear   of   f al l i ng  and  the
ev entual   cessati on  of   w al k i ng.   Dem enti a  m ay   be  associ ated  w i th  poor
judgm ent,   w hi ch  al so  adds  to  the  r i sk   of   f al l i ng.   For   ex am pl e,   w i thout
super v i si on,   a  dem ented  pati ent  m ay   tr y   to  m ai ntai n  hi s  bal ance  by
gr aspi ng  an  unstabl e  chai r   or   other   object  that  cannot  suppor t  hi s  or
her   w ei ght.   Ev en  i f   a  dem ented  pati ent  has  a  cane  or   a  w al k er ,   he  or
she  m ay   not  r em em ber   to  use  i t.   The  pati ent's  car egi v er   needs  to  be
educated  about  the  need  f or   the  f r equent  v er bal   “cuei ngâ€​   of
dem ented  pati ents  (e. g. ,   r em i ndi ng  them   to  use  assi sti v e  dev i ces  or
not  to  gr asp  unstabl e  objects  f or   bal ance).   The  phy si ci an  m ust  al w ay s
r ev i ew   the  natur e  of   the  pati ent's  m edi cati on  to  deter m i ne  i f   si de
ef f ects  m ay   be  contr i buti ng  to  f al l   r i sk .   In  thi s  case,   the  pati ent  i s
tak i ng  a  β­adr ener gi c  bl ock er   (pr opr anol ol )  f or   the  tr eatm ent  of
tr em or ,   w hi ch  m ay   be
P. 206
causi ng  br ady car di a,   and  thi s,   i n  tur n,   m ay   contr i bute  to  the  pati ent's
w eak ness  and  r i sk   of   f al l i ng.   Or thostati c  hy potensi on  caused  by   v ar i ous
m edi cati ons  i s  a  f r equent  sour ce  of   f al l   r i sk .   Anti hy per tensi v e
m edi cati ons  and  anti chol i ner gi c  m edi cati ons  (e. g. ,   hi s  di phenhy dr am i ne
or   tr i cy cl i c  anti depr essants,   such  as  am i tr i pty l i ne)  ar e  tw o  cl asses  of
m edi cati ons  that  f r equentl y   cause  or thostati c  hy potensi on.   Sedati v e
m edi cati ons  not  onl y   al ter   the  l ev el   of   consci ousness  but  m ay   al so
bl unt  postur al   r ef l ex es.   Car di ac  dy sr hy thm i as  ar e  r esponsi bl e  f or   25%
to  35%  of   al l   sy ncopal   epi sodes,   and  they   account  f or   2%  to  10%  of
f al l s.   How ev er ,   i n  thi s  pati ent,   w ho  has  no  hi stor y   of   car di ac  di sease  or
l oss  of   consci ousness,   a  dy sr hy thm i a  i s  an  unl i k el y   contr i butor   to  hi s
f al l s.

2.   What  di agnosti c  tests  m ay   be  the  m ost  hel pf ul   i n  thi s  pati ent?
Because  sensor y   def i ci ts  and  untr eated  m edi cal   pr obl em s  m ay
contr i bute  to  the  r i sk   of   f al l s  and  m ay   hav e  cor r ectabl e  eti ol ogi es,
ev al uati on  shoul d  i ncl ude  a  com pr ehensi v e  assessm ent  f or   these
pr obl em s.   Ophthal m ol ogi c  ev al uati on  of   hi s  poor   v i si on  m ay   i denti f y   a
r ev er si bl e  pr obl em   such  as  catar acts.   Wi th  better   v i si on,   the  pati ent
m ay   be  abl e  to  nav i gate  m or e  saf el y   and  ther eby   r educe  hi s  f al l   r i sk .
Cor r ecti on  of   hear i ng  l oss,   al though  not  di r ectl y   r el ated  to  v esti bul ar
f uncti on,   m ay   hel p  i n  i m pr ov i ng  gai t  stabi l i ty   and  r educi ng  f al l s.
Im pai r m ent  of   sensati on  i n  the  di stal   ex tr em i ti es,   par ti cul ar l y   l oss  of
posi ti on  sensati on,   suggesti ng  dor sal   spi nal   tr ack   di sease  shoul d
tr i gger   a  sear ch  f or   tr eatabl e  causes  of   per i pher al   neur opathy   such  as
di abetes  or   B 1 2   def i ci ency .

Thi s  pati ent's  ur i nar y   ur gency   m ay   contr i bute  to  hi s  f al l s  w hen  he  tr i es
to  r ace  to  the  bathr oom .   Check i ng  f or   a  ur i nar y   tr act  i nf ecti on  or
tr eati ng  sy m ptom ati c  pr ostati c  hy per tr ophy   m ay   r educe  ur ge  sy m ptom s
and  r educe  the  r i sk   of   f al l i ng.

Mor e  sophi sti cated  and  ex pensi v e  tests,   such  as


el ectr oencephal ogr aphy ,   Hol ter   m oni tor i ng,   and  CT  scanni ng  of   the
head,   m ay   be  per f or m ed  i n  pati ents  w ho  hav e  f al l s  but  shoul d  be  done
sel ecti v el y ,   based  on  the  pati ent's  hi stor y   and  phy si cal   ex am i nati on
f i ndi ngs.   Because  thi s  pati ent  has  no  ev i dence  of   sei zur e  acti v i ty ,   an
el ectr oencephal ogr am   i s  unl i k el y   to  be  r ev eal i ng.   Li k ew i se,   the
di agnosti c  y i el d  of   24­hour   am bul ator y   car di ac  m oni tor i ng  w oul d
pr obabl y   be  v er y   l ow ,   gi v en  the  l ack   of   car di ac  sy m ptom s,   and  i s
unnecessar y   f or   thi s  pati ent.   If   he  had  any   f ocal   neur ol ogi c  def i ci ts,   a
gai t  di sor der ,   or   changes  i n  cogni ti on,   an  i m agi ng  study   of   the  br ai n
(i . e. ,   CT  scan  or   MRI)  m i ght  be  hel pf ul   i n  r ul i ng  out  a  subdur al
hem atom a,   a  str ok e,   a  br ai n  tum or ,   or   nor m al   pr essur e  hy dr ocephal us
(char acter i zed  by   a  tr i ad  of   cogni ti v e  i m pai r m ent,   gai t  di sor der ,   and
ur i nar y   i nconti nence).   If   the  pati ent  has  ev i dence  of   upper   m otor
neur on  di sease  (e. g. ,   hy per r ef l ex i a,   pl antar   f l ex or   or   Babi nsk i 's
r esponse,   i ncr eased  m uscl e  tone)  and  no  ev i dence  of   cr ani al   ner v e  or
cor ti cal   si gns  (e. g. ,   m em or y   i m pai r m ent,   aphasi a,   etc. ),   then  cer v i cal
m y el opathy   m ay   be  the  eti ol ogy   of   the  pati ent's  poor   bal ance  and
i m agi ng  of   the  neck   shoul d  be  consi der ed,   par ti cul ar l y   i f   the  pati ent  i s
consi der ed  to  be  a  candi date  f or   sur gi cal   cor r ecti on  of   spi nal   cor d
i m pi ngem ent.   Because  thi s  pati ent  i s  atax i c  and  has  m i l d  dem enti a  and
ur i nar y   i nconti nence,   an  i m agi ng  study   of   hi s  br ai n  w oul d  hel p  r ul e  out
a  potenti al l y   tr eatabl e  cause  of   hi s  gai t  di sor der   and  f al l s,   such  as
nor m al   pr essur e  hy dr ocephal us.

P. 207
3.   What  i nter v enti on,   or   i nter v enti ons,   w oul d  y ou  i nsti tute  to  decr ease
thi s  pati ent's  r i sk   of   f al l i ng?

Car ef ul   scr uti ny   of   thi s  pati ent's  m edi cati on  l i st  m ay   r ev eal
oppor tuni ti es  to  r educe  or   el i m i nate  m edi cati ons  that  contr i bute  to  hi s
r i sk   of   f al l i ng.   Pr opr anol ol   m ay   r educe  hi s  tr em or   m odestl y   but  the
l ow er i ng  of   hi s  bl ood  pr essur e  and  pul se  m ay   be  contr i buti ng  to  hi s  f al l
r i sk .   If   the  di phenhy dr am i ne  i s  causi ng  hi m   to  be  m or e  cogni ti v el y
i m pai r ed,   then  he  i s  al so  m or e  l i k el y   to  hav e  an  i ncr eased  r i sk   of   f al l s
as  w el l .   Er i c  Lar son  etal .   noted  that  pati ents  w hose  cogni ti v e
i m pai r m ent  w as  i n  par t  due  to  m edi cati on  si de  ef f ects  w er e  al so  thr ee
ti m es  m or e  l i k el y   to  f al l   than  those  w hose  cogni ti v e  i m pai r m ent  w as
due  to  other   causes.   A  tr i al   per i od  of   r educi ng  or   stoppi ng  the
pr opr anol ol   and  di phenhy dr am i ne  shoul d  be  consi der ed  to  see  i f   the
pati ent's  conf usi on,   di zzi ness,   and  or thostasi s  r esol v e  and  hi s  bal ance
i m pr ov es.

Ev en  w hen  a  phy si ci an  f ai l s  to  i denti f y   speci f i c,   r ev er si bl e  eti ol ogi es  of
a  pati ent's  r i sk   f or   f al l i ng,   sev er al   i nter v enti ons  m ay   hel p  i n  r educi ng
the  r i sk   of   subsequent  f al l s  and  i njur y .   A  pati ent  w ho  i s  el der l y   and  has
m any   m edi cal   pr obl em s  i s  of ten  ex pected  to  be  f r ai l   and  w eak .
How ev er ,   ev en  the  f r ai l   el der l y   m ay   i m pr ov e  thei r   str ength  and  bal ance
by   par ti ci pati ng  i n  a  r egul ar   ex er ci se  pr ogr am .   Ev i dence  has  em er ged
that  m odest  r esi stance  tr ai ni ng,   i n  addi ti on  to  aer obi c  condi ti oni ng,
m ay   f ur ther   hel p  i n  i ncr easi ng  m uscl e  m ass  and  bal ance,   and  r educe
the  r i sk   of   f al l s.   Ref er r al   to  a  phy si cal   ther api st  m ay   l ead  to  a  bal ance
and  str engtheni ng  pr ogr am   that  m ay   r educe  the  r i sk   of   f al l s.   How ev er ,
i f   a  phy si cal   ther api st  i s  not  av ai l abl e,   studi es  hav e  show n  that
enr ol l m ent  i n  a  gr oup  f i tness  pr ogr am   m ay   hav e  the  sam e  benef i ts  as
i ndi v i dual   ther apy .   In  one  study ,   el der l y   pati ents  r andom i zed  to
par ti ci pate  i n  the  anci ent  m ar ti al   ar t  of   Tai   Chi   had  i m pr essi v e
r educti on  i n  f al l   r i sk   com par ed  w i th  those  w ho  di d  not  par ti ci pate.
Ther ef or e,   r egul ar   ex er ci se  thr ough  a  v ar i ety   of   opti ons  m ay   hel p  i n
r educi ng  the  r i sk   of   f al l i ng.

Most  f al l s  am ong  the  el der l y   occur   i n  the  hom e  setti ng.   Appr ox i m atel y
one  thi r d  of   f al l s  ar e  r el ated  to  acci dents  or   env i r onm ental   f actor s.   A
hom e  saf ety   assessm ent,   usual l y   com pl eted  by   a  phy si cal   or
occupati onal   ther api st,   m ay   i denti f y   pr ev entabl e  ex tr i nsi c  causes  of
f al l s  such  as  thr ow   r ugs  that  sl i p  w hen  stepped  on,   poor   l i ghti ng,
w ear i ng  of   unsuppor ti v e  or   sl i pper y   f ootw ear ,   stor age  of   com m onl y
used  i tem s  i n  out  of   r each  pl aces,   etc.   Because  a  si ngl e  v i si t  m ay
i denti f y   and  el i m i nate  m any   of   these  r i sk s,   a  hom e  saf ety   assessm ent
i s  l i k el y   to  be  a  cost­ef f ecti v e  i nter v enti on  to  r educe  the  r i sk   of   f al l i ng.

Recent  data  al so  suggests  that  i ncr easi ng  the  i ntak e  of   v i tam i n  D  m ay
not  onl y   i ncr ease  bone  densi ty   (and  ther eby   r educe  r i sk   of   f r actur e)
but  al so  r educe  the  l i k el i hood  that  f al l s  w i l l   occur .   The  m echani sm   i s
not  w el l   def i ned,   but  m ay ,   i n  par t,   be  r el ated  to  the  posi ti v e  ef f ects  of
v i tam i n  D  on  m uscl e  str ength.   The  opti m al   dose  of   suppl em ental
v i tam i n  D  that  shoul d  be  pr escr i bed  i s  not  cl ear ,   but  800  IU   dai l y
appear s  to  be  saf e  and  adequate.

Fi nal l y ,   pati ents,   thei r   f am i l i es,   and  thei r   phy si ci ans  m ay   be  f aced  w i th
the  conti nued  occur r ence  of   f al l s  despi te  com pr ehensi v e,   m ul ti f actor i al
i nter v enti ons  to  pr ev ent  them .   The  choi ces  ar e  not  easy   ones.   Al l ow i ng
the  pati ent  to  conti nue  to
P. 208
am bul ate  w i l l   i ncur   a  r i sk   of   f utur e  f al l s.   Restr i cti ng  a  pati ent's
m obi l i ty   m ay   r educe  the  r i sk   of   f al l s  but  at  the  cost  of   i ncr easi ng
w eak ness,   di suse  atr ophy ,   l oss  of   i ndependence,   and  a  sense  of
despai r .   Al though  any   i njur y   that  occur s  dur i ng  a  f al l   i s  si gni f i cant,   hi p
f r actur es  cl ear l y   car r y   the  gr eatest  m or bi di ty   and  m or tal i ty .   Studi es  of
speci al l y   desi gned  hi p  pads  suggest  that  i f   el der l y   i ndi v i dual s  can  be
per suaded  to  w ear   them ,   the  r i sk   of   hi p  f r actur e,   i f   they   do  f al l ,   i s
r educed.

Suggested Readings
Bi schof f ­Fer r ar i   HA,   Daw son­Hughes  B,   Wi l l ett  C,   etal .   Ef f ect  of   Vi tam i n
D  on  f al l s:  a  m etanal y si s.   JAMA  2004;291:1999.

Kannus  P,   Par k k ar i   J,   N i em i   S,   etal .   Pr ev enti on  of   hi p  f r actur e  i n  el der l y


peopl e  w i th  use  of   a  hi p  pr otector .   N   Eng  J  Med  2000;343:1506.

Ki ng  MD,   Ti netti   ME.   Fal l s  i n  com m uni ty ­dw el l i ng  ol der   per sons.   J  Am
Ger i atr   Soc  1995;43:1146.

Ti netti   ME.   Pr ev enti ng  f al l s  i n  el der l y   per sons.   N   Eng  J  Med  2003;348:42.

Ti netti   ME,   Wi l l i am s  CS.   Fal l s,   i njur i es  due  to  f al l s,   and  the  r i sk   of
adm i ssi on  to  a  nur si ng  hom e.   N   Eng  J  Med  1997;337:1279.

Urinary Incontinence
1.   How   com m on  i s  ur i nar y   i nconti nence  i n  the  el der l y ?

2.   What  ar e  the  nor m al   changes  i n  bl adder   phy si ol ogy   that  occur   w i th


agi ng?

3.   How   i s  i nconti nence  cl assi f i ed,   and  w hat  ar e  the  char acter i sti cs  of   the
di f f er ent  ty pes?

4.   Of   w hat  does  the  di f f er enti al   di agnosi s  of   tr ansi ent  ur i nar y   i nconti nence
consi st?

Discussion
1.   How   com m on  i s  ur i nar y   i nconti nence  i n  the  el der l y ?

U r i nar y   i nconti nence,   the  i nv ol untar y   l oss  of   ur i ne,   af f ects  10  m i l l i on


Am er i cans.   Of   peopl e  ol der   than  65  y ear s,   5%  of   m en  and  25%  of
w om en  hav e  pr obl em s  w i th  i nconti nence.   In  1987  al one,   the  di r ect  cost
of   the  pr obl em   w as  m or e  than  $10  bi l l i on.   Inconti nence  adds  $3  to  $12
per   day   to  the  cost  of   nur si ng  hom e  car e,   and  50%  to  90%  of   al l
nur si ng  hom e  r esi dents  ex per i ence  som e  i nconti nence.

Besi des  the  si gni f i cant  ex pense  caused,   i nconti nence  i s  a  sour ce  of
m any   m edi cal   com pl i cati ons,   such  as  r ashes,   pr essur e  ul cer s,
catheter i zati on,   ur i nar y   tr act  i nf ecti ons,   f al l s,   and  f r actur es.   Ther e  ar e
al so  soci al   consequences,   such  as  em bar r assm ent,   i sol ati on,   and
depr essi on.   It  al so  adds  to  car egi v er   str ess.

2.   What  ar e  the  nor m al   changes  i n  bl adder   phy si ol ogy   that  occur   w i th


agi ng?

Bl adder   capaci ty   and  com pl i ance  decl i ne  w i th  agi ng,   as  does  the  abi l i ty
to  postpone  v oi di ng.   Ther e  ar e  al so  m or e  f r equent  uni nhi bi ted  bl adder
P. 209
contr acti ons  and  an  i ncr ease  i n  the  r esi dual   v ol um e  of   ur i ne.   Because
of   an  age­r el ated  decr ease  i n  the  gl om er ul ar   f i l tr ati on  r ate  and  a  del ay
i n  the  ex cr eti on  of   a  w ater   l oad,   appr ox i m atel y   tw o  thi r ds  of   f l ui d  i s
ex cr eted  i n  the  ev eni ng  r ather   than  dur i ng  the  day .   Thi s  l eads  to
noctur nal   ur i nar y   f r equency   and  the  r i sk   of   noctur nal   ur i nar y
i nconti nence.

Ther e  ar e  al so  sex ­speci f i c  changes.   Estr ogen  def i ci ency   i n  w om en


l eads  to  w eak ened  sphi ncter   tone  and  changes  i n  the  posi ti on  of   the
bl adder   neck .   The  ur ethr al   l ength  shor tens  and  the  m ax i m al   ur ethr al
cl osur e  pr essur es  decl i ne.   In  m en,   pr ostati c  enl ar gem ent  can
potenti al l y   bl ock   ur i ne  outf l ow .   Al l   of   these  changes  pr edi spose  el der l y
pati ents  to  i nconti nence.   The  encour agi ng  new s  i s  that  50%  of   the
cases  ar e  tr ansi ent  and  tw o  thi r ds  of   the  r em ai ni ng  cases  can  be  ei ther
cur ed  or   m ar k edl y   al l ev i ated  w i th  ther apy .

3.   How   i s  i nconti nence  cl assi f i ed,   and  w hat  ar e  the  char acter i sti cs  of   the
di f f er ent  ty pes?

U rge  inc ontine nc e   i s  the  m ost  com m on  ty pe  of   i nconti nence  i n  the
el der l y ,   accounti ng  f or   appr ox i m atel y   80%  of   cases.   Af f l i cted  pati ents
of ten  descr i be  a  sudden  uncontr ol l abl e  ur ge  to  v oi d  that  m ay   not  al l ow
them   ti m e  to  r each  the  bathr oom .   The  ur ge  i s  caused  by   contr acti on  of
the  bl adder 's  detr usor   m uscl e,   w hi ch  f or ces  m oder ate  to  l ar ge  v ol um es
of   ur i ne  out  thr ough  the  ur ethr a.   Centr al   ner v ous  sy stem   di seases
(e. g. ,   str ok e,   AD,   Par k i nson's  di sease,   a  pr i m ar y   br ai n  tum or ,   or
m etastati c  di sease)  and  pr i m ar y   di sease  of   the  bl adder   (e. g. ,
car ci nom a,   the  ef f ects  of   r adi ati on  tr eatm ent,   or   bl adder   outl et
obstr ucti on)  m ay   be  associ ated  w i th  ur ge  i nconti nence.

Stre s s  inc ontine nc e   i s  par ti cul ar l y   com m on  i n  el der l y   w om en.   In  pur e
str ess  i nconti nence,   l eak age  occur s  w i th  i ncr eases  i n  pr essur e  caused
by   coughi ng,   sneezi ng,   l aughi ng,   or   l i f ti ng;  onl y   a  sm al l   am ount  of
ur i ne  l eak s  out  af ter   a  del ay   of   5  to  15  seconds.   The  sour ce  of   the
pr obl em   i n  w om en  i s  usual l y   ur ethr al   hy per m obi l i ty   due  to  l ax i ty   of   the
pel v i c  f l oor   m uscul atur e  caused  by   chi l dbear i ng.   In  m en,   str ess
i nconti nence  i s  l ess  com m on  but  m ay   occur   i f   the  ur ethr al   sphi ncter   i s
dam aged  dur i ng  tr ansur ethr al   or   r adi cal   pr ostatectom y .

Ove rflow  inc ontine nc e   i s  caused  ei ther   by   outl et  obstr ucti on  or   by   an
atoni c  bl adder   (i . e. ,   i nef f ecti v e  detr usor   contr acti on  due  to  m y ogeni c
or   neur ol ogi c  causes).   Leak age  of   sm al l   am ounts  of   ur i ne  m ay   occur
thr oughout  the  day   and  ni ght.   Pati ents  m ay   al so  descr i be  ur i nar y
hesi tancy   and  a  f eel i ng  of   i ncom pl ete  em pty i ng.   On  abdom i nal
ex am i nati on,   a  di stended  bl adder   m ay   be  pal pated  ev en  af ter   the
pati ent  has  attem pted  to  v oi d.

Re fle x  inc ontine nc e   i s  usual l y   due  to  a  supr asacr al   spi nal   cor d  l esi on.
As  the  bl adder   di stends,   contr acti on  occur s.   Leak age  i s  not  associ ated
w i th  str ess  and  ther e  i s  no  w ar ni ng  bef or e  the  onset  of   ur i nati on.
Inconti nence  epi sodes  ar e  of   m oder ate  v ol um e  and  occur   f r equentl y .

Func tiona l inc ontine nc e   i s  due  to  a  pr obl em   unr el ated  to  the  ur i nar y
tr act.   Ex am pl es  ar e  i m pai r ed  m obi l i ty   or   m etabol i c  pr obl em s  such  as
hy per gl y cem i a  or   m i l d  r enal   i nsuf f i ci ency .   Thi s  di agnosi s  can  be  m ade
onl y   by   tak i ng  a  v er y   car ef ul   hi stor y   and  af ter   ex cl udi ng  the  pr ev i ousl y
l i sted  causes.   Functi onal   i nconti nence  m ay   r esul t  f r om   the  use  of
i atr ogeni c  dr ugs  that  i m pai r   cogni ti on  or   f r om   i m posed  l i m i tati ons  on
m obi l i ty ,   such  as  r estr ai nts.

P. 210
Because  ur i nar y   i nconti nence  i n  the  el der l y   i s  of ten  m ul ti f actor i al ,   al l
potenti al l y   contr i buti ng  r i sk   f actor s  shoul d  be  car ef ul l y   r ev i ew ed  to
deter m i ne  i ts  cause.

4.   Of   w hat  does  the  di f f er enti al   di agnosi s  of   tr ansi ent  ur i nar y   i nconti nence
consi st?

An  acute  change  i n  a  pati ent's  m ental   status  (del i r i um )  or   a  m ood


di sor der   (depr essi on)  m ay   contr i bute  to  f uncti onal   i nconti nence,   such
as  the  pati ent  w ho  i s  too  conf used  to  f i nd  a  bathr oom   or   too
despondent  to  car e  about  per sonal   hy gi ene.   End­stage  dem enti a  m ay
r ender   a  pati ent  i ncapabl e  of   r ecogni zi ng  the  ur ge  to  v oi d.

U r i nar y   tr act  i nf ecti on  m ay   l ead  to  i r r i tati on  of   the  detr usor   m uscl e  and
ther ef or e  cause  ur i nar y   f r equency   and  ur gency .   Si m i l ar l y ,   i nf l am m ati on
caused  by   atr ophi c  ur ethr i ti s  or   v agi ni ti s  m ay   contr i bute  to  i ncr eased
ur ge.

Medi cati ons  m ay   f oster   ur i nar y   i nconti nence  i n  a  v ar i ety   of   w ay s.


Sedati v es  depr ess  the  l ev el   of   consci ousness,   l eadi ng  to  a  f uncti onal
i nconti nence.   Di ur eti cs  i ncr ease  the  ur i ne  v ol um e,   w hi ch  then  i ncr eases
ur i nar y   f r equency .   A  l ar ger   ur i ne  v ol um e  m ay   then  tr i gger   m or e
f r equent  epi sodes  of   bl adder   spasm   and  augm ent  the  r i sk   of   ur ge
i nconti nence.   Anti chol i ner gi c  m edi cati ons,   such  as  anti hi stam i nes,
tr i cy cl i c  anti depr essants,   and  anti psy choti cs,   i nhi bi t  detr usor
contr acti ons  and  l ead  to  ov er f l ow   i nconti nence.   Cal ci um   channel
bl ock er s  (e. g. ,   ni f edi pi ne,   v er apam i l ,   and  di l ti azem )  m ay   si m i l ar l y
decr ease  detr usor   contr acti l i ty   and  w or sen  ov er f l ow   i nconti nence.

Endocr i ne  condi ti ons  such  as  hy per gl y cem i a  and  hy per cal cem i a  cause
ur i nar y   f r equency   and,   l i k e  di ur eti cs,   i ncr ease  ur i ne  v ol um es,   and
hence  the  r i sk   of   i nconti nence.   Restr i cted  m obi l i ty   due  to  sev er e
ar thr i ti s,   str ok e,   car di ac  di sease,   or   any   other   debi l i tati ng  condi ti on
m ay   si m pl y   pr ev ent  a  pati ent  f r om   r eachi ng  the  bathr oom   i n  ti m e  (i . e. ,
f uncti onal   i nconti nence).   Fi nal l y ,   stool   i m pacti on  m ay   contr i bute  to
pel v i c  ner v e  com pr essi on,   l eadi ng  to  an  atoni c  bl adder   and  ov er f l ow
i nconti nence.

Case
A  73­y ear ­ol d  m other   of   si x   i s  br ought  to  y our   of f i ce  by   her   daughter   to
establ i sh  her   m other 's  pr i m ar y   car e  i n  tow n.   The  pati ent  has  com e  to  l i v e
w i th  her   daughter   af ter   her   husband  di ed  6  m onths  ago.   The  pati ent's
chi l dr en  ar e  concer ned  that  she  i s  depr essed  and  r epor t  she  i s  not  getti ng
out  of   her   house  ex cept  w hen  she  has  a  doctor 's  appoi ntm ent.   Her   past
m edi cal   hi stor y   i s  r em ar k abl e  f or   hy per tensi on  (f or   w hi ch  she  tak es
hy dr ochl or othi azi de  dai l y )  and  som e  ar thr i ti s  i n  her   k nees.   She  has
under gone  no  sur gi cal   pr ocedur es.   She  deni es  any   other   pr obl em s,   but,
w hen  speci f i cal l y   ask ed,   she  adm i ts  to  hav i ng  ur i nar y   i nconti nence  f or
sev er al   y ear s,   w hi ch  has  been  w or se  dur i ng  the  past  f ew   w eek s.   She
descr i bes  getti ng  the  ur ge  to  v oi d  al m ost  ev er y   hour   and,   i f   she  does  not
get  to  the  bathr oom   i n  a  m atter   of   m i nutes,   she  has  star ted  to  l ose  enough
ur i ne  such  that  she  now   needs  to  w ear   adul t  pads.   When  ask ed  i f   she  l oses
ur i ne  w hen  she  coughs  or   l aughs,   she  conf i r m s  that  thi s  has  occur r ed  f or
m any   y ear s.   She  say s  that  she  has  not  r epor ted  thi s  em bar r assi ng  pr obl em
to  her   pr ev i ous  phy si ci ans  because  they   nev er   ask ed,   and  has  attr i buted  i t
to  “just  getti ng  ol d. â€​   She  and  her   husband  had  stopped  hav i ng  sex
because  she  w as  af r ai d  that  i t  w oul d  m ak e  her   i nconti nence  w or se.
P. 211
Phy si cal   ex am i nati on  r ev eal s  a  heal thy ­appear i ng  el der l y   w om an  w hose  v i tal
si gns  ar e  nor m al ,   i ncl udi ng  her   bl ood  pr essur e,   w hi ch  i s  130/76  m m   Hg.   Her
ex am i nati on  f i ndi ngs  ar e  unr em ar k abl e  ex cept  f or   her   pel v i s,   w hi ch  ex hi bi ts
atr ophi c  m ucosa  and  a  gr ade  III  cy stocel e  (bl adder   and  ur ethr a  pr otr udi ng).
She  i s  ask ed  to  cough  and  a  sm al l   am ount  of   ur i ne  l eak s  f r om   the  ur ethr a.
The  r ectal   f i ndi ngs  ar e  nor m al   and  ther e  i s  good  r ectal   tone.   N eur ol ogi c
f i ndi ngs  ar e  nor m al ,   i ncl udi ng  a  nor m al   anal   w i nk   (suggesti ng  i ntact
sphi ncter ),   and  ther e  ar e  no  l um bosacr al   neur ol ogi c  f i ndi ngs.   Labor ator y
ev al uati on  r ev eal s  nor m al   el ectr ol y te  and  cr eati ni ne  v al ues  and  a  r andom
bl ood  gl ucose  l ev el   of   240  m g/dL.   U r i nal y si s,   w i th  ur i ne  obtai ned  by
catheter i zati on,   r ev eal s  5  to  10  w hi te  bl ood  cel l s  per   hi gh­pow er   f i el d,   no
epi thel i al   cel l s,   2+   bacter i a,   and  3+   gl ucose.
1.   Of   w hat  does  the  di f f er enti al   di agnosi s  of   thi s  pati ent's  i nconti nence
consi st?
2.   What  conser v ati v e  tr eatm ents  coul d  y ou  tr y   i n  thi s  pati ent?
3.   If   these  m easur es  hel p  but  do  not  el i m i nate  her   i nconti nence
com pl etel y ,   w hat  w oul d  be  the  nex t  step  i n  tr eatm ent?
4.   How   w oul d  the  em phasi s  of   y our   ev al uati on  di f f er   f or   a  m al e  pati ent?

Case Discussion
1.   Of   w hat  does  the  di f f er enti al   di agnosi s  of   thi s  pati ent's  i nconti nence
consi st?

Thi s  pati ent  descr i bes  sy m ptom s  of   the  m ost  com m on  ty pe  of
i nconti nence  i n  the  el der l y ,   nam el y ,   ur ge  i nconti nence.   She  has  a
hi stor y   of   si x   v agi nal   del i v er i es  and  al so  has  cor r espondi ng  sy m ptom s
of   str ess  i nconti nence,   w i th  suggesti v e  f i ndi ngs  di scov er ed  on
ex am i nati on  (i . e. ,   ur i ne  l eak s  w hen  she  l aughs).   The  ur i nal y si s  f i ndi ngs
ar e  abnor m al ,   i ndi cati ng  a  possi bl e  ur i nar y   tr act  i nf ecti on  that  coul d  be
ex acer bati ng  her   sy m ptom s  of   ur gency   and  f r equency .   It  m ay   ex pl ai n
the  w or seni ng  of   sy m ptom s  i n  the  past  f ew   w eek s.   An  el ev ated  bl ood
gl ucose  l ev el   m ay   f oster   an  osm oti c  di ur esi s  that  i ncr eases  ur i ne
v ol um es  and  ther eby   adds  to  the  r i sk   of   i nconti nence.

Fear   of   l eav i ng  her   hom e  and  the  consequent  soci al   i sol ati on  m ay   hav e
been  pr eci pi tated  by   her   i nconti nence  because  pati ents  pr one  to  ur ge
i nconti nence  of ten  l i m i t  thei r   acti v i ti es  to  av oi d  em bar r assi ng
acci dents.   On  v i sual   i nspecti on,   she  w as  f ound  to  hav e  an  atr ophi c
m ucosa,   w hi ch  m ay   suggest  estr ogen  def i ci ency .   Si m i l ar   atr ophy   m ay
occur   i n  the  ur ethr al   m ucosa,   w hi ch  i n  tur n  r educes  ur ethr al   sphi ncter
com petence.

She  i s  on  a  di ur eti c,   w hi ch  m ay   al so  ex acer bate  her   i nconti nence.   Her
bl ood  pr essur e  m ay   r espond  ei ther   to  another   agent  or   to  di et  al one,
w i th  w ei ght  r educti on  and  sodi um   r estr i cti on.   She  al so  has  ar thr i ti s  i n
her   k nees,   w hi ch  l i m i ts  her   abi l i ty   to  get  to  the  bathr oom   i n  ti m e.   It
m ay   be  that  the  bathr oom   i s  f ar ther   f r om   the  bedr oom   i n  her
daughter 's  hom e,   and  thi s  coul d  contr i bute  to  the  r ecent  w or seni ng  of
sy m ptom s.   A  si m pl e  r ear r angem ent  of   her   bedr oom   f ur ni tur e  m ay   put
her   bed  cl oser   to  the  toi l et  and  r educe  the  r i sk   of   noctur nal
i nconti nence  by   shor teni ng  the  di stance  she  needs  to  tr av el   to  the
bathr oom .

2.   What  conser v ati v e  tr eatm ents  coul d  y ou  tr y   i n  thi s  pati ent?

Fi r st,   the  easi l y   r ev er si bl e  causes  need  to  be  el i m i nated.   A  car ef ul


ev al uati on,   i ncl udi ng  a  pel v i c  ex am i nati on,   m ust  pr ecede  any
tr eatm ent.   It  w oul d  be  r easonabl e
P. 212
to  hav e  her   ur i ne  cul tur ed  and  to  tr eat  her   f or   a  ur i nar y   tr act  i nf ecti on,
to  see  i f   the  ur gency   di ssi pates.   If   she  has  no  contr ai ndi cati ons  to
estr ogen  r epl acem ent,   such  tr eatm ent  coul d  be  gi v en  or al l y   or   topi cal l y
to  al l ev i ate  the  atr ophi c  ur ethr i ti s.   Her   di ur eti c  m edi cati on  coul d  be
di sconti nued  and  r epl aced  w i th  a  di f f er ent  agent,   i f   w ei ght  l oss  and
sodi um   r educti on  f ai l   to  contr ol   her   bl ood  pr essur e.   If   the  di stance
f r om   the  bedr oom   to  the  bathr oom   i s  a  sour ce  of   noctur nal
i nconti nence,   getti ng  the  pati ent  a  bedsi de  com m ode  can  al l ev i ate  the
pr obl em .   Tr eatm ent  of   her   di abetes,   ei ther   w i th  di et,   or al   agents,   or
i nsul i n,   w i l l   hel p  decr ease  the  ur i ne  v ol um e.   In  gener al ,   i t  i s
r easonabl e  to  counsel   al l   pati ents  com pl ai ni ng  of   i nconti nence  to
r ef r ai n  f r om   dr i nk i ng  too  m uch  f l ui d  bef or e  goi ng  out  or   near   bedti m e.

3.   If   these  m easur es  hel p  but  do  not  el i m i nate  her   i nconti nence
com pl etel y ,   w hat  w oul d  be  the  nex t  step  i n  tr eatm ent?

The  pati ent  has  sy m ptom s  of   both  ur ge  and  str ess  i nconti nence.
How ev er ,   the  possi bi l i ty   of   ov er f l ow   i nconti nence  shoul d  al so  be
assessed.   Thi s  i s  done  by   catheter i zi ng  the  pati ent  af ter   she  has  v oi ded
to  see  i f   ther e  i s  ur i nar y   r etenti on  (> 100  m L),   w hi ch  w oul d  i ndi cate
possi bl e  ov er f l ow   i nconti nence.   N ei ther   str ess  nor   ur ge  i nconti nence
al one  shoul d  cause  a  hi gh  postv oi d  r esi dual   v ol um e.   Behav i or al
techni ques  ar e  v er y   ef f ecti v e  i n  al l ev i ati ng  both  ur ge  and  str ess
i nconti nence.   Because  the  uni nhi bi ted  bl adder   spasm s  associ ated  w i th
ur ge  i nconti nence  ar e  br i ef ,   the  pati ent  shoul d  be  i nstr ucted  to  si t
cal m l y   and  al l ow   the  ur ge  to  pass.   Jum pi ng  up  to  go  to  the  bathr oom
onl y   accentuates  abdom i nal   pr essur e  dur i ng  the  contr acti on  and  m ak es
the  l eak age  of   ur i ne  m or e  l i k el y .   Behav i or   m odi f i cati on  al one  can
consi der abl y   ease  the  pati ent's  ur ge  i nconti nence.   Wom en  w i th  str ess
i nconti nence  m ay   r educe  l oss  of   ur i ne  by   per f or m i ng  ex er ci ses  that
str engthen  the  pel v i c  f l oor   m uscl es.   To  teach  pati ents  these  ex er ci ses
(Kegel   ex er ci ses),   ask   the  pati ent  to  f eel   the  m uscl es  she  uses  to  stop
her   str eam   of   ur i ne  or   a  bow el   m ov em ent.   She  m ust  contr act  these
m uscl es  w i thout  al so  contr acti ng  the  abdom i nal   m uscl es  10  to  15  ti m es,
thr ee  ti m es  a  day .   Thi s  pr acti ce  m ust  be  conti nued  to  r em ai n  ef f ecti v e.
If   the  i nconti nence  per si sts  ev en  af ter   di l i gent  ex er ci si ng  f or   sev er al
w eek s  or   m onths,   the  pati ent  shoul d  be  r ef er r ed  to  a  gy necol ogi st  f or
consi der ati on  of   sur gi cal   cor r ecti on  of   pel v i c  f l oor   l ax i ty .

4.   How   w oul d  the  em phasi s  of   y our   ev al uati on  di f f er   f or   a  m al e  pati ent?

Ov er f l ow   i nconti nence  associ ated  w i th  bl adder   outl et  obstr ucti on
r esul ti ng  f r om   beni gn  pr ostati c  hy per tr ophy   i s  an  i m por tant  cause  of
i nconti nence,   uni que  to  m en.   Ther ef or e,   m en  shoul d  be  ask ed  car ef ul l y
about  ur i nar y   f r equency   and  hesi tancy ,   a  decr ease  i n  the  f or ce  of   the
ur i ne  str eam ,   and  i f   they   ex per i ence  a  sensati on  of   i ncom pl ete
em pty i ng.   It  i s  m or e  i m por tant  to  ev al uate  the  postv oi d  r esi dual
v ol um e  ear l y   i n  the  w or k up  of   a  m an.   A  l ow   r esi dual   v ol um e  does  not
absol utel y   r ul e  out  obstr ucti on  because  of   the  i nter m i ttent  natur e  of
such  an  obstr ucti on.   How ev er ,   i f   the  v ol um e  i s  gr eater   than
appr ox i m atel y   250  m L,   the  di agnosi s  of   bl adder   outl et  obstr ucti on  i s
v er y   l i k el y .   Most  ur ol ogi sts  per f or m   cy stoscopy   bef or e  pr ostati c
sur ger y   to  conf i r m   the  di agnosi s  and  r ul e  out  detr usor   f l acci di ty   i n  m en
w i th  l ar ge  postv oi d  r esi dual   ur i ne  v ol um es.   If   the  pati ent  has  a  f l acci d
bl adder ,   the  ur ol ogi st  m ay   be  r el uctant  to  per f or m   pr ostate  sur ger y
because  such  pati ents  ar e  l i k el y   to  conti nue  to  r equi r e  ei ther
per m anent  or   i nter m i ttent  postoper ati v e  catheter i zati on.

P. 213
U r ge  i nconti nence  i s  al so  an  i m por tant  di agnosi s  i n  m al e  pati ents.   It
of ten  coex i sts  w i th  obstr ucti on  because  a  di stended  bl adder   i s  m or e
pr one  to  contr acti ons.   Ther ef or e,   m en  tr eated  f or   sy m ptom s  of   outl et
obstr ucti on  m ust  al so  be  ask ed  about  sy m ptom s  of   ur gency   that  m ay
r equi r e  addi ti onal   tr eatm ent  to  pr ev ent  conti nued  ur i nar y   i nconti nence
af ter   sur gi cal   cor r ecti on  of   ur i nar y   tr act  obstr ucti on.

Suggested Readings
Consensus  Conf er ence.   U r i nar y   i nconti nence  i n  adul ts.   JAMA
1989;261:2685.

Fantl   JA,   N ew m an  DK,   Col l i ng  J,   etal .   U r i nar y   i nconti nence  i n  adul ts:
acute  and  chr oni c  m anagem ent.   U . S.   Depar tm ent  of   Heal th  and  Hum an
Ser v i ces,   Agency   f or   Heal th  Car e  Pol i cy   and  Resear ch.   Rock v i l l e,   MD,
U . S.   Gov er nm ent  Pr i nti ng  Of f i ce,   1996.

McDow el l   BJ,   Bur gi o  KL,   Dom br ow sk i   M,   etal .   An  i nter di sci pl i nar y
appr oach  to  the  assessm ent  and  behav i or al   tr eatm ent  of   ur i nar y
i nconti nence  i n  ger i atr i c  outpati ents.   J  Am   Ger i atr   Soc  1991;40:370.

Medication Use in the Elderly
1.   What  i s  pol y phar m acy ,   and  i s  i t  a  si gni f i cant  pr obl em   i n  the  el der l y ?  If
so,   w hy ?

2.   Why   do  el der l y   pati ents  ex per i ence  an  i ncr eased  i nci dence  of   adv er se
dr ug  r eacti ons  (ADRs)?

3.   N am e  sev er al   w ay s  i n  w hi ch  ADRs  m i ght  be  associ ated  w i th  each  of   the
f ol l ow i ng  i n  el der l y   pati ents:  new   m edi cati ons,   the  l ong­ter m   use  of
dr ugs,   and  the  sudden  cessati on  of   m edi cati ons.

Discussion
1.   What  i s  pol y phar m acy ,   and  i s  i t  a  si gni f i cant  pr obl em   i n  the  el der l y ?  If
so,   w hy ?

Pol y phar m acy   i s  the  concur r ent  use  of   m any   m edi cati ons.   Al though  thi s
ter m   i s  m ost  of ten  used  to  r ef er   to  the  use  of   “too  m any
m edi cati ons, â€​   som e  pati ents  w i th  m ul ti pl e  m edi cal   pr obl em s  m ay   be
appr opr i atel y   r ecei v i ng  sev er al   pr escr i pti on  m edi cati ons.   Because
m edi cati ons  ar e  a  com m on  cause  of   r ev er si bl e  pr obl em s  i n  the  el der l y
(e. g. ,   dr ug­i nduced  conf usi on  and  or thostati c  hy potensi on),   each
m edi cati on  pr escr i bed  f or   an  el der l y   per son  m ust  be  car ef ul l y
scr uti ni zed  to  deter m i ne  w hether   the  benef i ts  outw ei gh  the  adv er se
ef f ects  of   the  dr ug.   Because  the  adv er se  ef f ects  f r equentl y   outw ei gh
the  benef i ts,   i t  has  been  sai d  of   good  ger i atr i ci ans  that  they   “stop
m or e  m edi cati ons  than  they   star t. â€​

Pol y phar m acy   i s  a  ser i ous  pr obl em   i n  the  el der l y .   An  av er age  el der l y
per son  tak es  tw o  to  f i v e  pr escr i pti on  m edi cati ons  as  w el l   as  thr ee  to
f our   ov er ­the­counter   dr ugs.   Al though  el der l y   Am er i cans  (ol der   than  65
y ear s)  consti tute  12%  of   the  U . S.   popul ati on,   they   consum e
appr ox i m atel y   25%  of
P. 214
al l   pr escr i pti on  m edi cati ons.   They   ar e  ther ef or e  not  onl y   ex posed  to
m or e  dr ugs  but  al so  to  m or e  potenti al   adv er se  dr ug  ef f ects.   Indeed,
ol der   peopl e  hav e  thr ee  to  sev en  ti m es  m or e  ADRs  than  y ounger
pati ents,   and  the  f r equency   of   ADRs  cor r el ates  w i th  the  num ber   of
m edi cati ons  used.

2.   Why   do  el der l y   pati ents  ex per i ence  an  i ncr eased  i nci dence  of   ADRs?

The  m anagem ent  of   dr ug  ther apy   i n  the  el der l y   di f f er s  f r om   that  i n


y ounger   pati ents,   and  the  r esul ti ng  hi gher   i nci dence  of   pol y phar m acy   i n
the  el der l y   popul ati on  i ncr eases  the  r i sk   of   dr ug–dr ug  i nter acti ons.
These  i nter acti ons  m ay   r esul t  f r om   the  al ter ed  absor pti on,   ex cr eti on,
or   pr otei n  bi ndi ng  of   the  dr ugs  i nv ol v ed.   In  addi ti on,   unanti ci pated  dr ug
ef f ects  m ay   occur   i f   one  dr ug  enhances  or   i nter f er es  w i th  the  hepati c
m etabol i sm   of   another .   Such  i nter acti ons  m ay   r esul t  i n  ei ther   tox i c  or
subther apeuti c  dr ug  l ev el s.

Com or bi di ty   adds  to  the  i nci dence  of   ADRs  because  the  si gns  and
sy m ptom s  of   a  pr eex i sti ng  di sease  m ay   be  w or sened  by   the  ef f ects  of
m edi cati ons  gi v en  to  tr eat  another   di sor der .   Thi s  can  r esul t  ei ther   f r om
the  w or seni ng  of   an  under l y i ng  di sease  pr ocess  by   the  of f endi ng  dr ug
(e. g. ,   the  use  of   β 2 ­bl ock er s  i n  pati ents  w i th  chr oni c  obstr ucti v e
pul m onar y   di sease  or   congesti v e  hear t  f ai l ur e)  or   because  the  si gns
and  sy m ptom s  of   the  dr ug's  si de  ef f ects  m i r r or   and,   ther ef or e,
i ntensi f y   those  of   the  under l y i ng  di sease  pr ocess.   An  ex am pl e  of   thi s  i s
the  ur i nar y   r etenti on  caused  by   anti chol i ner gi c  m edi cati ons  (e. g. ,
tr i cy cl i c  anti depr essants  and  di phenhy dr am i ne)  i n  a  pati ent  w i th  an
enl ar ged  pr ostate.   The  r etenti on  occur s  because  the  enl ar ged  pr ostate
obstr ucts  the  ur i ne  f l ow   and  the  anti chol i ner gi c  m edi cati on  w eak ens
detr usor   contr acti on.

El der l y   pati ents  of ten  hav e  l ess  phy si ol ogi c  r eser v e  and  ther ef or e
handl e  phy si ol ogi c  str ess  l ess  successf ul l y   than  y ounger   pati ents.   The
am ount  of   phy si ol ogi c  r eser v e  v ar i es  am ong  el der l y   pati ents  and  ev en
am ong  di f f er ent  or gan  sy stem s  i n  the  sam e  i ndi v i dual .   Som eti m es
phy si ci ans  obtai n  basel i ne  m easur em ents  to  assess  a  pati ent's  r eser v e.
For   ex am pl e,   the  cr eati ni ne  cl ear ance  m ay   suggest  how   m uch  k i dney
r eser v e  i s  l ef t.   Ther ef or e,   the  r i sk   of   ADRs  m ay   be  m i ni m i zed  by   the
car ef ul   ev al uati on  of   an  i ndi v i dual   el der l y   pati ent's  r enal   f uncti on
bef or e  pr escr i bi ng  potenti al l y   tox i c  m edi cati ons.   Som eti m es,   si m pl y
r educi ng  the  dosage  m ay   conf er   an  adequate  ther apeuti c  ef f ect  w i thout
pr oduci ng  tox i ci ty .   Those  el der l y   pati ents  w i th  better   r eser v e  w ho  ar e
capabl e  of   m or e  nor m al   m etabol i sm   of   m edi cati ons  m ay   need  the  sam e
dosage  as  y ounger   pati ents  to  obtai n  a  ther apeuti c  ef f ect.   In  sum m ar y ,
ther apy   m ust  be  i ndi v i dual i zed  to  obtai n  the  opti m um   ef f ect  f r om
m edi cati on  w hi l e  av oi di ng  tox i ci ty .

Age­r el ated  phy si ol ogi c  changes  i n  the  el der l y   i ncl ude  a  decl i ne  i n  l ean
m uscl e  m ass  and  total   body   w ater   content,   w i th  an  i ncr eased
pr opor ti on  of   total   body   f at.   These  changes  af f ect  dr ug  di sposi ti on  i n
the  f ol l ow i ng  m anner :  l ess  total   body   w ater   tr ansl ates  i nto  a  sm al l er
v ol um e  of   di str i buti on  f or   w ater ­sol ubl e  m edi cati ons,   r esul ti ng  i n
hi gher ­than­anti ci pated  ser um   concentr ati ons.   Because  adi pose  ti ssue
i s  of ten  pr opor ti onatel y   gr eater   i n  ol der   pati ents,   the  v ol um e  of
di str i buti on  f or   f at­sol ubl e  m edi cati ons  i ncr eases,
P. 215
pr ol ongi ng  the  el i m i nati on  per i od.   Another   phy si ol ogi c  change  of   gr eat
i m por tance  i s  a  decl i ne  i n  r enal   f uncti on  w i th  age,   occur r i ng  i n
appr ox i m atel y   65%  of   el der l y   peopl e.   For   the  el der l y ,   the  ser um
cr eati ni ne  concentr ati on  al one  i s  an  unr el i abl e  i ndi cator   of   k i dney
f uncti on  because  i t  depends  on  the  am ount  of   m uscl e  m ass,   w hi ch
decr eases  w i th  adv anci ng  age.   Instead,   cr eati ni ne  cl ear ance  i s  a  m or e
accur ate  esti m ate  of   r enal   f uncti on  i n  the  el der l y .   Age­r el ated
phy si ol ogi c  changes  i n  hepati c  m etabol i sm   and  pr otei n  bi ndi ng  usual l y
hav e  l ess  i m pact  on  dr ug  m etabol i sm   than  the  decl i ne  i n  r enal   f uncti on.

3.   N am e  sev er al   w ay s  i n  w hi ch  ADRs  m i ght  be  associ ated  w i th  each  of   the
f ol l ow i ng  i n  el der l y   pati ents:  new   m edi cati ons,   the  l ong­ter m   use  of
dr ugs,   and  the  sudden  cessati on  of   m edi cati ons.

Ne w  me dic a tions   m ay   el i ci t  ADRs  by   pr oduci ng  pr edi ctabl e  si de


ef f ects,   especi al l y   i f   the  si de  ef f ects  ex acer bate  pr eex i sti ng  di sease­
r el ated  sy m ptom s.   For   ex am pl e,   pr eex i sti ng  postur al   hy potensi on  can
be  w or sened  by   tr i cy cl i c  anti depr essants.   N ew   m edi cati ons  can  al so
cause  adv er se  ef f ects  i f   the  dosages  pr escr i bed  ar e  not  appr opr i ate  f or
the  el der l y ,   l eadi ng  to  dr ug  i ntox i cati on.   For   ex am pl e,   di gox i n  tox i ci ty
m ay   occur   i f   the  phy si ci an  f ai l s  to  adjust  the  dosage  to  accom m odate
r enal   i m pai r m ent.   When  new   m edi cati ons  ar e  added  to  an  al r eady
com pl i cated  m edi cal   r egi m en,   thi s  m ay   al so  f oster   noncom pl i ance,
ei ther   because  of   pati ent  f r ustr ati on  about  hav i ng  to  tak e  so  m any   pi l l s
or   because  of   conf usi on  ov er   com pl i cated  dosi ng  schedul es.   N ew
m edi cati ons  can  pr eci pi tate  ADRs  w hen  they   becom e  i nv ol v ed  i n  dr ugâ
€“dr ug  i nter acti ons,   as  pr ev i ousl y   di scussed.   Fi nal l y ,   pati ents  m ay   not
tol er ate  new   m edi cati ons  f or   i di osy ncr ati c  r easons,   ther eby
em phasi zi ng  the  need  f or   phy si ci ans  to  m ai ntai n  v i gi l ance  i n  detecti ng
an  ADR.   Contr i buti ng  to  thi s  i s  the  f act  that  dr ug  si de  ef f ects  m ay   not
be  r ecogni zed  as  such  by   pati ents  because  they   ascr i be  thei r   sy m ptom s
to  ol d  age.

The  long­te rm us e  of me dic a tions   m ay   be  associ ated  w i th  an  ADR
w hen  a  pati ent's  r enal ,   hepati c,   or   nutr i ti onal   status  changes  w i thout  a
concom i tant  dose  adjustm ent.   For   i nstance,   a  dr ug  dose  tol er ated  f or
m any   y ear s  m ay   becom e  tox i c  as  r enal   cl ear ance  decl i nes.   In  addi ti on,
ADRs  r esul t  w hen  new   m edi cati ons  adv er sel y   af f ect  the
phar m acok i neti cs  of   m edi cati ons  that  el der l y   pati ents  hav e  other w i se
tol er ated  f or   y ear s.   An  ex am pl e  of   such  i nter acti ons  i s  the  di gox i n
tox i ci ty   that  occur s  secondar y   to  decr eased  cl ear ance  af ter   the  addi ti on
of   v er apam i l   to  a  m edi cal   r egi m en.

Changes  i n  com pl i ance  can  r esul t  i n  ADRs.   N oncom pl i ance  i s  a  com m on


ger i atr i c  pr obl em ,   w i th  esti m ates  r angi ng  f r om   26%  to  59%  f or   the
ger i atr i c  popul ati on,   and  pol y phar m acy   i ncr eases  the  i nci dence  of
noncom pl i ance.   Com pl i ance  can  be  i m pr ov ed  w hen  phy si ci ans  r egul ar l y
ask   thei r   pati ents  i n  a  nonjudgm ental   m anner   about  thei r   m edi cati on
use,   si m pl i f y   the  m edi cal   r egi m en,   and  r em i nd  el der l y   pati ents  about
the  need  f or   each  m edi cati on.   Other   f actor s  that  i nf l uence  com pl i ance
i ncl ude  cogni ti v e,   f i nanci al ,   and  f uncti onal   changes.

P. 216
ADRs  can  al so  occur   w hen  a  pati ent  i s  hospi tal i zed  and  star ted  on  a
m edi cal   r egi m en  that  the  phy si ci an  i ncor r ectl y   assum ed  w as  bei ng
f ol l ow ed  at  hom e.   If   the  pati ent  has  been  tak i ng  f ew er   pi l l s  than
actual l y   pr escr i bed,   thi s  “enf or cedâ€​   com pl i ance  m ay   pr eci pi tate
tox i ci ty   al though  the  dosi ng  schedul e  m ay   seem   cor r ect.

Fi nal l y ,   m any   dr ugs  com m onl y   used  i n  the  el der l y   ar e  associ ated  w i th
w ithdra w a l s yndrome s .   Of   par ti cul ar   i m por tance  ar e  the  psy chotr opi c
dr ugs,   such  as  the  benzodi azepi nes,   anti psy choti cs,   and
anti depr essants.   Dr ug  w i thdr aw al   sy ndr om es  m ay   occur   i f   these
m edi cati ons  ar e  di sconti nued  abr uptl y   or   taper ed  too  qui ck l y ,   and
shoul d  be  consi der ed  as  a  potenti al   sour ce  of   a  m ar k ed  change  i n  an
el der l y   pati ent's  behav i or .   Dr ug  w i thdr aw al   m ay   occur   ev en  w hen
ther apeuti c  and  not  necessar i l y   hi gh  doses  ar e  abr uptl y   di sconti nued.
U nf or tunatel y ,   dr ug  w i thdr aw al   f r equentl y   goes  unr ecogni zed,   l eadi ng
to  potenti al l y   pr ev entabl e  adv er se  com pl i cati ons.   Agi tati on  and
del i r i um   ar e  am ong  the  m or e  com m on  sy m ptom s  associ ated  w i th
w i thdr aw al   f r om   som e  psy chotr opi c  dr ugs.   For   thi s  r eason,   i t  i s
i m por tant  to  consi der   dr ug  w i thdr aw al   as  a  possi bl e  cause  of   any
unex pl ai ned  del i r i um .
Case
An  81­y ear ­ol d  m an  w ho  w as  adm i tted  to  the  hospi tal   2  day s  ago  f or   the
ev al uati on  of   epi gastr i c  bur ni ng  i n  conjuncti on  w i th  hem occul t­posi ti v e
stool s  and  anem i a  suddenl y   ex hi bi ts  conf usi on.   He  has  under gone  endoscopy
and  w as  f ound  to  hav e  gastr i ti s.   Hi s  hem atocr i t  r eadi ng  has  r em ai ned  stabl e
and  di schar ge  pl anni ng  i s  i n  pr ogr ess.   Hi s  abdom i nal   sy m ptom s  hav e  been
al l ev i ated  w i th  the  addi ti on  of   the  H 2   bl ock er ,   ci m eti di ne.

Hi s  past  m edi cal   hi stor y   i s  l i m i ted,   and  he  i s  v ague  w hen  answ er i ng
questi ons  about  i t.   Hi s  daughter   has  r epor ted  that  he  has  dem enti a.   N o
other   m edi cal   pr obl em s  hav e  been  i denti f i ed.   The  pati ent  deni ed  al cohol   or
tobacco  use  on  adm i ssi on.   The  m edi cati ons  he  w as  tak i ng  bef or e  adm i ssi on
ar e  unk now n,   but  he  i s  cur r entl y   bei ng  gi v en  ci m eti di ne  (400  m g  or al l y
tw i ce  dai l y )  and  di phenhy dr am i ne  (25  m g  or al l y   at  ni ght,   as  needed)  f or
i nsom ni a.   He  has  no  k now n  dr ug  al l er gi es.
The  pati ent  i s  a  w i dow ed,   r eti r ed  pl um ber   w ho  l i v es  al one.   Hi s  f am i l y
hi stor y   i s  noncontr i butor y   and  a  r ev i ew   of   sy stem s  i s  si gni f i cant  f or
i nsom ni a.
The  nur ses  r el ate  that  the  pati ent  w as  w el l   dur i ng  the  day ,   but  becam e
pr ogr essi v el y   conf used  dur i ng  the  ev eni ng.   He  i s  f ound  to  be  di sor i ented
and  i r r i tabl e,   w i th  hi s  m ental   status  f l uctuati ng  betw een  agi tati on,   w i th
per ceptual   di stor ti ons  and  v i sual   hal l uci nati ons,   and  hy per som nol ence.
Phy si cal   ex am i nati on  r ev eal s  the  f ol l ow i ng  f i ndi ngs:  bl ood  pr essur e,   140/80
m m   Hg;  tem per atur e,   98. 6°F  (37. 0°C);  pul se,   80  beats  per   m i nute;  and
r espi r ati ons,   16  per   m i nute.   Ther e  ar e  no  or thostati c  changes.
The  pati ent  i s  unabl e  to  cooper ate  f ul l y   w i th  m ental   status  testi ng  but  i s
noted  to  be  di sor i ented  to  ti m e  and  pl ace  and  appear s  anx i ous.   He  i s
f l ushed.   Head,   ey e,   ear ,   nose,   and  thr oat  f i ndi ngs,   as  w el l   as  the  car di ac,
pul m onar y ,   and  abdom i nal   f i ndi ngs  ar e  unr em ar k abl e.   N eur ol ogi c
ex am i nati on  r ev eal s  nonf ocal   f i ndi ngs.   Hi s  cr ani al   ner v es  ar e  i ntact  and
ther e
P. 217
i s  no  aster i x i s.   Hi s  r ef l ex es  ar e  2+   and  sy m m etr i c.   Hi s  m otor   abi l i ty   i s
scor ed  as  5/5  and  sy m m etr i c.   Hi s  sensati on  i s  i ntact  to  l i ght  touch,   al though
other   sensor y   m odal i ti es  cannot  be  tested.   Hi s  toes  ar e  dow ngoi ng
bi l ater al l y   and  he  has  no  cer ebel l ar   abnor m al i ti es.
You  cor r ectl y   ascer tai n  that  the  pati ent's  cur r ent  behav i or   cannot  si m pl y   be
due  to  w or seni ng  of   hi s  under l y i ng  dem enti a  but  i s  consi stent  w i th  del i r i um .
To  ex cl ude  m etabol i c,   i nf ecti ous,   tr aum ati c,   or   neur ol ogi c  causes,   the
f ol l ow i ng  data  ar e  obtai ned:  w hi te  bl ood  cel l   count,   6, 000  cel l s/m m 3   w i th  a
nor m al   di f f er enti al ;  hem atocr i t,   35%  and  stabl e  com par ed  w i th  adm i ssi on;
pl atel ets,   350  ×  10 3 /m m 3 ;  sodi um ,   140  m Eq/L;  chl or i de,   105  m Eq/L;
cr eati ni ne,   0. 9  m g/dL;  potassi um ,   4. 0  m Eq/L;  CO 2 ,   27  m Eq/L;  bl ood  ur ea
ni tr ogen,   12  m g/dL;  gl ucose,   125  m g/dL;  cal ci um ,   9. 0  m g/dL;  ar ter i al   bl ood
gases,   nor m al ;  aspar tate  am i notr ansf er ase,   20  U /L  (nor m al   r ange,   14  to  30
IU /L);  al k al i ne  phosphatase,   175  IU /L  (nor m al   r ange,   30  to  110  IU /L);  total
bi l i r ubi n,   0. 4  m g/dL;  thy r oi d­sti m ul ati ng  hor m one,   4  U /m L  (nor m al   r ange,
0. 5  to  5  µ  U /m L);  v i tam i n  B 1 2 ,   600  pg/m L  (nor m al   r ange,   225  to  800
pg/m L);  and  r api d  pl asm a  r eagi n,   nonr eacti v e.
N o  py ur i a  i s  f ound  on  ur i nal y si s  and  bl ood  speci m ens  ar e  sent  f or   cul tur e.   A
chest  r adi ogr aph  and  el ectr ocar di ogr am   ar e  nor m al ,   as  i s  a  CT  scan  of   the
head,   w hi ch  show s  no  ev i dence  of   hem or r hage.
The  di phenhy dr am i ne  i s  di sconti nued  and  the  pati ent's  daughter   i s  cal l ed
and  tol d  of   her   f ather 's  condi ti on.   She  r epor ts  that  on  enter i ng  her   f ather 's
apar tm ent  that  ev eni ng,   she  di scov er ed  a  hal f ­em pty   bottl e  of   l or azepam   (a
benzodi azepi ne)  that  she  had  not  k now n  he  w as  tak i ng.

1.   Whi ch  of   the  pati ent's  sy m ptom s  ar e  consi stent  w i th  del i r i um ?
2.   Is  the  pr esentati on  of   benzodi azepi ne  w i thdr aw al   i n  el der l y   pati ents
di f f er ent  f r om   that  i n  y ounger   pati ents?
3.   How   coul d  thi s  w i thdr aw al   sy ndr om e  hav e  been  pr ev ented?
4.   Ar e  ther e  other   dr ugs  ci ted  i n  the  case  that  can  cause  conf usi on?  If   so,
how ?

Case Discussion
1.   Whi ch  of   the  pati ent's  sy m ptom s  ar e  consi stent  w i th  del i r i um ?

The  sy m ptom s  that  ar e  consi stent  w i th  del i r i um   i n  thi s  pati ent,   w hi ch
m ay   r ef l ect  benzodi azepi ne  w i thdr aw al ,   i ncl ude  f l uctuati ons  i n
consci ousness,   anx i ety ,   conf usi on,   i r r i tabi l i ty ,   per ceptual   di stur bances,
and  hal l uci nati ons.   Because  dr ug  w i thdr aw al   i s  f r equentl y
unr ecogni zed,   phy si ci ans  shoul d  consi der   the  possi bi l i ty   of   w i thdr aw al
i n  any   ger i atr i c  pati ent  w ho  ex hi bi ts  an  abr upt  al ter ati on  i n  behav i or
and  cogni ti on,   par ti cul ar l y   w hen  other   sy stem i c  causes  hav e  been
ex cl uded.   Phy si ci ans  m ust  al so  m ai ntai n  a  hi gh  i ndex   of   suspi ci on  f or
al cohol   w i thdr aw al   i n  both  el der l y   m en  and  w om en.   Fi nal l y ,   as
ex em pl i f i ed  i n  thi s  pati ent,   com m uni cati on  w i th  f am i l y ,   f r i ends,   and
car egi v er s  m ay   pr ov i de  v al uabl e  i nf or m ati on  about  other w i se
unr epor ted  psy choacti v e  dr ug  use  or   abuse.

Lor azepam   i s  an  i nter m edi ate­acti ng  benzodi azepi ne,   w i th  an  onset  of
w i thdr aw al   sy m ptom s  ty pi cal l y   occur r i ng  i n  the  f i r st  24  to  72  hour s
af ter   di sconti nuati on,
P. 218
w hi ch  i s  consi stent  w i th  thi s  pati ent's  cl i ni cal   pi ctur e.   Rar el y ,
w i thdr aw al   sy m ptom s  m ay   be  del ay ed  f or   up  to  2  w eek s  i n  pati ents
tak i ng  l onger ­acti ng  benzodi azepi nes,   such  as  di azepam   and
f l ur azepam .   The  age­r el ated  i ncr ease  i n  the  pr opor ti on  of   total   body   f at
of   ger i atr i c  pati ents  m ay   pr ov i de  a  l ar ger   v ol um e  of   di str i buti on  f or
f at­sol ubl e  benzodi azepi nes,   ther eby   l engtheni ng  the  el i m i nati on  per i od
and  postponi ng  the  onset  of   w i thdr aw al   sy m ptom s.

2.   Is  the  pr esentati on  of   benzodi azepi ne  w i thdr aw al   i n  el der l y   pati ents
di f f er ent  f r om   that  i n  y ounger   pati ents?

The  cl i ni cal   m ani f estati ons  of   benzodi azepi ne  w i thdr aw al   i n  the  el der l y
f r equentl y   di f f er   f r om   those  seen  i n  y ounger   pati ents.   The  di f f er ence  i n
the  cl i ni cal   m ani f estati ons  of   w i thdr aw al   i n  el der l y   pati ents  i s  i n
gener al   due  to  com or bi di ty   stem m i ng  f r om   other   di seases  and  i m pai r ed
hom eostati c  r eser v e.   In  som e  cases,   these  f actor s  r esul t  i n  m or e
sev er e  and  ev en  l i f e­thr eateni ng  w i thdr aw al   sy m ptom s.   Benzodi azepi ne
w i thdr aw al   i s  associ ated  w i th  i ncr eased  autonom i c  ner v ous  sy stem
acti v i ty .   In  y ounger   pati ents,   thi s  m ani f ests  as  tachy car di a,   m i l d
hy per tensi on,   and  di aphor esi s.   In  el der l y   pati ents  w i th  l i m i ted
phy si ol ogi c  r eser v e,   the  i ncr eased  autonom i c  ner v ous  sy stem   acti v i ty
m ay   pr eci pi tate  sev er e  car di ov ascul ar   com pl i cati ons.   In  other
si tuati ons,   com or bi di ty   or   i m pai r ed  hom eostati c  r eser v e,   or   both,   m ay
r esul t  i n  m or e  subtl e  w i thdr aw al   sy m ptom s  i n  el der l y   pati ents.   Heal th
car e  pr of essi onal s  m ay   m i stak enl y   attr i bute  changes  i n  m ental   status
to  w or seni ng  dem enti a.   In  ger i atr i c  pati ents,   abr upt  and  i sol ated
conf usi on  i s  som eti m es  the  onl y   cl ue  to  benzodi azepi ne  w i thdr aw al .

3.   How   coul d  thi s  w i thdr aw al   sy ndr om e  hav e  been  pr ev ented?

Thi s  pati ent's  benzodi azepi ne  w i thdr aw al   m i ght  hav e  been  pr ev ented,
f i r st,   by   f i ndi ng  out  w hether   he  i s  tak i ng  a  m edi cati on  associ ated  w i th
a  w i thdr aw al   sy ndr om e.   When  adm i tti ng  el der l y   pati ents  w i th  cogni ti v e
i m pai r m ent,   i t  i s  i m por tant  to  com m uni cate  w i th  the  pr i m ar y   car egi v er
because  thi s  f r equentl y   pr ov i des  v i tal l y   i m por tant  i nf or m ati on.   Second,
i f   benzodi azepi nes  ar e  to  be  di sconti nued,   the  dose  shoul d  be  gr adual l y
taper ed  by   10%  to  20%  per   w eek .

4.   Ar e  ther e  other   dr ugs  ci ted  i n  the  case  that  can  cause  conf usi on?  If   so,
how ?

The  pati ent  w as  star ted  on  tw o  new   m edi cati ons  dur i ng  hi s
hospi tal i zati on—ci m eti di ne  and  di phenhy dr am i ne.   Both  of   these  dr ugs
can  cause  conf usi on  i n  el der l y   pati ents,   and  dr ug­i nduced  del i r i um   i s
m or e  com m on  i n  pati ents  w i th  pr eex i sti ng  dem enti a  (an  ex am pl e  of   a
dr ug–di sease  i nter acti on).

Ci m eti di ne,   an  H 2   bl ock er ,   can  pr oduce  a  host  of   sy stem i c  ef f ects  and
m ay   par ti ci pate  i n  dr ug–dr ug  i nter acti ons  because  of   i ts  abi l i ty   to
decr ease  the  m etabol i sm   of   m edi cati ons  that  ar e  el i m i nated  by   the
l i v er .   Ci m eti di ne  al so  r ar el y   causes  centr al   ner v ous  sy stem   sy m ptom s
such  as  conf usi on  and  hal l uci nati ons.   The  m echani sm   r esponsi bl e  f or
ci m eti di ne­m edi ated  m ental   status  changes  i s  unk now n.

Di phenhy dr am i ne  i s  f r equentl y   used  to  pr om ote  sl eep,   but  i s  actual l y   a


poor   choi ce  f or   f r ai l ,   el der l y   pati ents.   Of   par ti cul ar   concer n  ar e  i ts
potenti al   anti chol i ner gi c  si de  ef f ects,   w hi ch  i ncl ude  dr y   m outh,   ur i nar y
r etenti on,   consti pati on,   bl ur r ed  v i si on,   and  conf usi on.   Ger i atr i c  pati ents
m ay   be  m or e  sensi ti v e  to  anti chol i ner gi c  si de  ef f ects  than  y ounger
peopl e  because  of   age­r el ated  changes  i n  acety l chol i ne
neur otr ansm i ssi on.

P. 219

Suggested Readings
Ahr onhei m   JC.   Handbook   of   pr escr i bi ng  m edi cati ons  f or   ger i atr i c
pati ents.   Boston:  Li ttl e,   Br ow n  and  Com pany ,   1992:1–12, 96â
€“100, 347–348.

Beer s  MH.   Pol y phar m acy   and  appr opr i ate  pr escr i bi ng.   In:  Beck   JC,   ed.
Ger i atr i c  r ev i ew   sy l l abus,   1991–1992  ed.   N ew   Yor k :  Am er i can
Ger i atr i c  Soci ety ,   1991:218.

Ger ber   JG,   Br ass  EP.   Dr ug  use  i n  the  el der l y .   In:  Jahni gen  DW,   Schr i er
RW,   eds.   Ger i atr i c  m edi ci ne,   2nd  ed.   Cam br i dge,   MA:  Bl ack w el l   Sci ence,
1996.
Editors :  Sc hrie r,  Robe rt W .
Title :  Inte rna l Me dic ine  Ca s e book ,  The : Re a l P a tie nts ,  Re a l Ans w e rs ,
3rd Edition

Copy r i ght  ©2007  Li ppi ncott  Wi l l i am s  &  Wi l k i ns

>  T a b le   o f   C o nte nts   >  C ha p te r   6  ­  I nf e c tio us   D is e a s e s

Chapter 6
Infectious Diseases

Robe rt T.  Sc hoole y

Urinary Tract Infection
1.   What  host  f actor s  l ead  to  the  dev el opm ent  of   ur i nar y   tr act  i nf ecti ons
(U TIs),   and  how   ar e  these  f actor s  di f f er ent  f or   m en  and  w om en?

2.   What  or gani sm s  com m onl y   cause  l ow er   U TIs?

3.   What  ar e  the  si gns  and  sy m ptom s  of   l ow er   U TI,   and  how   do  these  di f f er
f r om   those  of   py el onephr i ti s?

Discussion
1.   What  host  f actor s  l ead  to  the  dev el opm ent  of   U TIs,   and  how   ar e  these
f actor s  di f f er ent  f or   m en  and  w om en?

Im pr oper   hy gi ene,   sex ual   acti v i ty ,   i nconti nence,   ur i nar y   tr act


i nstr um entati on,   contr acepti v e  di aphr agm s  w i th  or   w i thout  sper m i ci des,
di abetes  m el l i tus,   a  geneti c  pr edi sposi ti on,   and  dehy dr ati on  ar e  al l
f actor s  that  can  i ncr ease  the
P. 221
l i k el i hood  of   a  U TI.   Most  U TIs  ar e  caused  by   endogenous  f l or a  or i gi nati ng
f r om   the  gastr oi ntesti nal   tr act.   These  or gani sm s  hav e  been  show n  to
col oni ze  the  v agi nal   i ntr oi tus  and  per i ur ethr al   ar ea  bef or e  U TI  occur s.
Wom en  w ho  w i pe  thei r   per i neal   ar ea  f r om   the  poster i or   to  anter i or
di r ecti on  af ter   def ecati on,   r ather   than  v i ce  v er sa,   or   those  w ho  ar e
i nconti nent  of   stool ,   m ay   be  subject  to  m or e  f r equent  col oni zati on  of   the
shor t  f em al e  ur ethr a  w i th  Enter obacter i aceae.   The  l onger   ur ethr a  i n  m en
m ak es  access  to  the  bl adder   m or e  di f f i cul t  f or   enter i c  f l or a;  how ev er ,
thi s  f l or a  m ay   be  i ntr oduced  to  the  nor m al l y   ster i l e  bl adder   ar ea  as  the
r esul t  of   Fol ey   catheter i zati on  or   cy stoscopy .   One  of   the  natur al   def enses
agai nst  cy sti ti s  af ter   ur ethr al   col oni zati on  i s  the  m echani cal   f l ushi ng  of
the  ur i nar y   bl adder ,   w hi ch  tak es  pl ace  dur i ng  ur i nati on.   Obv i ousl y ,
any one  w ho  i s  dehy dr ated  cannot  benef i t  f r equentl y   f r om   thi s  natur al
def ense  m echani sm .   Sex ual   acti v i ty   can  pr edi spose  w om en  to  acqui r i ng
U TI.   In  addi ti on,   as  the  r esul t  of   a  poor l y   under stood  m echani sm ,   w om en
w ho  use  a  di aphr agm   f or   contr acepti on,   especi al l y   w i th  sper m i ci des,
seem   to  be  m or e  suscepti bl e  to  ur ethr al   col oni zati on  and  U TI.   It  has  been
pr oposed  that  thi s  pr edi sposi ti on  m i ght  be  due,   at  l east  i n  par t,   to  a  shi f t
i n  v agi nal   m i cr obi al   f l or a  caused  by   the  acti v i ty   of   sper m i ci des.   Di abetes
m el l i tus  m ay   pr edi spose  to  U TI  thr ough  a  v ar i ety   of   m echani sm s,
i ncl udi ng  the  def ecti v e  chem otax i s  of   l euk ocy tes,   phagocy ti c  def ects,   and
enhanced  gr ow th  condi ti ons  f or   bacter i a.   Geneti cal l y   deter m i ned  f actor s,
such  as  the  ty pe  and  num ber   of   r eceptor s  on  ur oepi thel i al   cel l s  to  w hi ch
bacter i a  m ay   attach,   al so  appear   to  hei ghten  suscepti bi l i ty   to  U TIs.

2.   What  or gani sm s  com m onl y   cause  l ow er   U TIs?

Escher i chi a  col i   causes  m ost  (up  to  80%)  of   the  com m uni ty ­acqui r ed
uncom pl i cated  U TIs,   w i th  Kl ebsi el l a,   Enter obacter ,   and  Pr oteus  or gani sm s
m or e  l i k el y   to  cause  com pl i cated  or   hospi tal ­acqui r ed  U TIs.   These  ar e  al l
gr am ­negati v e  or gani sm s  that  usual l y   or i gi nate  f r om   the  pati ent's  ow n
gastr oi ntesti nal   f l or a.   Ther e  ar e,   how ev er ,   sev er al   gr am ­posi ti v e
or gani sm s  that  occur   as  ur i nar y   pathogens.   Staphy l ococcus  sapr ophy ti cus,
a  coagul ase­negati v e  Staphy l ococcus  or gani sm ,   causes  20%  or   m or e  of
the  U TIs  i n  w om en  16  to  35  y ear s  of   age.   Str eptococcus  f aecal i s  causes
2%  to  3%  of   the  U TIs  i n  other w i se  heal thy   y oung  w om en.   When
Staphy l ococcus  aur eus  i s  f ound  i n  the  ur i ne,   a  bacter em i c  i nf ecti on  of   the
k i dney   shoul d  be  suspected.

Chl am y di a,   U r eapl asm a,   My copl asm a,   and  N ei sser i a  gonor r hoeae  ar e


sex ual l y   tr ansm i tted  pathogens  that  usual l y   cause  v agi nal   or   cer v i cal
i nf ecti ons;  how ev er ,   they   m ay   be  i m pl i cated  i n  cases  of   acute  ur ethr al
sy ndr om e  i n  w hi ch  Gr am 's­stai ned  ur i ne  sam pl es  ex hi bi t  py ur i a  w i thout
bacter i ur i a.

Pseudom onas  and  Ser r ati a  ar e  m or e  com m onl y   nosocom i al   gr am ­negati v e


pathogens  that  ar e  not  usual l y   seen  i n  com m uni ty ­acqui r ed,
uncom pl i cated  U TIs.

3.   What  ar e  the  si gns  and  sy m ptom s  of   l ow er   U TI,   and  how   do  these  di f f er
f r om   those  of   py el onephr i ti s?

The  ter m   l ow er   U TI  actual l y   encom passes  cy sti ti s  and  ur ethr i ti s,   as  w el l
as  pr ostati ti s.   Sy m ptom s  cl assi cal l y   i ncl ude  ur i nar y   f r equency ,   ur gency ,
dy sur i a,
P. 222
and  supr apubi c  di scom f or t.   Si gns  m ay   i ncl ude  f ev er ,   cl oudy   or   f oul ­
sm el l i ng  ur i ne,   and  hem atur i a.   Because  upper   U TIs  (i . e. ,   py el onephr i ti s,
acute  l obar   nephr i ti s,   and  a  per i nephr i c  abscess)  of ten  star t  as  cy sti ti s,
the  sam e  si gns  and  sy m ptom s  m ay   ex i st;  how ev er ,   the  f ev er   i s  usual l y
m or e  sev er e,   and  m ay   be  accom pani ed  by   shak i ng  chi l l s.   An  upper   U TI  i s
of ten  accom pani ed  by   costov er tebr al   angl e  tender ness  on  the  i nv ol v ed
si de.   El der l y   peopl e  and  those  w i th  di abetes  m ay   ex hi bi t  f ew er   si gns  and
sy m ptom s  than  other w i se  nor m al   hosts.

Case
A  19­y ear ­ol d,   sex ual l y   acti v e  w om an  pr esents  to  the  em er gency   r oom
com pl ai ni ng  of   a  2­day   hi stor y   of   ur i nar y   f r equency ,   bur ni ng,   and  ur gency .
She  deni es  v agi nal   di schar ge  or   i tchi ng,   f ev er ,   chi l l s,   nausea,   v om i ti ng,   back
pai n,   abdom i nal   pai n,   or   hem atur i a.   She  has  no  hi stor y   of   U TI  or   a  sex ual l y
tr ansm i tted  di sease.   She  r ecentl y   began  usi ng  a  di aphr agm   f or   bi r th  contr ol ,
and  r epor ts  that  her   l ast  m enstr ual   per i od  occur r ed  3  w eek s  ago.   She  has  onl y
one  sex ual   par tner ,   w ho  deni es  peni l e  di schar ge  or   bur ni ng  on  ur i nati on.   On
phy si cal   ex am i nati on,   she  i s  noted  to  be  af ebr i l e  w i th  a  nor m al   bl ood  pr essur e
and  pul se.   Ther e  i s  no  costov er tebr al   angl e  tender ness.   Her   abdom en  i s  sof t
and  ther e  i s  m i l d  supr apubi c  tender ness  i n  r esponse  to  pal pati on.   A  ur i nal y si s
r ev eal s  1+   pr otei n,   2+   l euk ocy tes,   and  1+   bl ood.   The  ur i ne  pH  i s  5. 6.   Gr am 's
stai ni ng  of   an  unspun  ur i ne  speci m en  r ev eal s  abundant  pol y m or phonucl ear
l euk ocy tes  and  m oder ate  gr am ­negati v e  r ods.   A  cl ean­catch  ur i ne  speci m en  i s
sent  to  the  m i cr obi ol ogy   l abor ator y   f or   cul tur e.
The  em er gency   r oom   phy si ci an  di agnoses  an  uncom pl i cated  U TI  and  pr escr i bes
tr i m ethopr i m ­sul f am ethox azol e  (TMP­SMX),   one  doubl e­str ength  tabl et  tw i ce  a
day   f or   3  day s.

1.   What  other   ther apeuti c  opti ons  w oul d  hav e  been  appr opr i ate  i n  thi s
pati ent?
2.   What  can  thi s  w om an  do  to  hel p  pr ev ent  r ecur r ent  U TIs?
3.   Shoul d  thi s  w om an's  sex ual   par tner   be  ev al uated  f or   U TI?
4.   Was  the  Gr am 's  stai ni ng  an  i m por tant  di agnosti c  test,   and  i n  w hat  w ay
di d  the  f i ndi ngs  al ter   the  m anagem ent  of   thi s  case?
5.   What  i s  the  v al ue  of   k now i ng  the  ur i ne  pH  i n  thi s  setti ng?
6.   What  other   di agnosti c  or   l abor ator y   tests  shoul d  hav e  been  per f or m ed?
7.   What  w oul d  be  an  appr opr i ate  anal gesi c  f or   a  pati ent  w i th  U TI  w ho  i s
ex per i enci ng  sev er e  ur ethr al   di scom f or t?
8.   What  si de  ef f ects  of   ther apy   shoul d  thi s  w om an  k now   about?
9.   What  possi bl e  consequences  coul d  ar i se  i f   thi s  w om an  does  not  com pl y
w i th  ther apy ?

Case Discussion
1.   What  other   ther apeuti c  opti ons  w oul d  hav e  been  appr opr i ate  i n  thi s
pati ent?

TMP­SMX  r em ai ns  the  dr ug  of   choi ce  f or   the  em pi r i cal l y   based  tr eatm ent
of   uncom pl i cated  U TIs.   For   sul f a­al l er gi c  pati ents,   am pi ci l l i n,   am ox i ci l l i n,
a  f i r st­gener ati on  cephal ospor i n,   or   a  qui nol one  i s  the  appr opr i ate
al ter nati v e.   Ther apy   m ay   then  be  m odi f i ed  on  the  basi s  of   the  ur i ne
cul tur e  r esul ts  and  the  sensi ti v i ti es  of   the  i nf ecti ng  or gani sm .   Enter ococci
ar e  not  suscepti bl e  to  ei ther   TMP­SMX  or
P. 223
cephal ospor i ns,   w hi ch  poi nts  out  the  uti l i ty   of   per f or m i ng  ur i ne  Gr am 's
stai ni ng  w hen  deci di ng  on  anti bi oti c  ther apy .   The  pr ev al ence  of
am pi ci l l i n­r esi stant  E.   col i   m ay   be  as  hi gh  as  30%  i n  som e  com m uni ti es,
and  thi s  needs  to  be  consi der ed  w hen  sel ecti ng  an  appr opr i ate  anti bi oti c.
S.   sapr ophy ti cus  r esponds  to  am pi ci l l i n,   TMP­SMX,   and  the  qui nol ones.   In
the  past,   tr eatm ent  of   l ow er   U TIs  f or   5  to  7  day s  w as  r ecom m ended.
Shor t  cour se  ther apy   w i th  agents  that  achi ev e  hi gh  and  sustai ned  ur i nar y
concentr ati ons  (si ngl e  dose  w i th  one  or   tw o  doubl e­str ength  TMP/SMX  or
3  g  of   am ox i ci l l i n)  w i l l   usual l y   suf f i ce  f or   uncom pl i cated  i nf ecti ons.
Fai l ur es  of   shor t  cour se  ther apy   ar e  i ndi cati ons  that  com pl i cati ng  f actor s
r equi r i ng  m or e  ex tensi v e  ev al uati on  m i ght  be  pr esent.   In  gener al ,   si ngl e­
dose  ther apy   i s  contr ai ndi cated  i n  pati ents  w i th  k now n  anatom i c  or
f uncti onal   abnor m al i ti es,   or   w i th  i m m unocom pr om i si ng  di seases  such  as
di abetes  m el l i tus.   Af ter   si ngl e­dose  ther apy   ur i ne  cul tur es  shoul d  be
per f or m ed  1  to  2  w eek s  l ater ,   to  docum ent  the  cur e.   In  the  ev ent  of
tr eatm ent  f ai l ur e,   a  l onger   cour se  of   the  appr opr i ate  anti bi oti c  shoul d  be
adm i ni ster ed  and  an  ev al uati on  of   potenti al l y   com pl i cati ng  f actor s  shoul d
be  under tak en.

Regar dl ess  of   the  pathogen  and  the  choi ce  of   anti bi oti cs,   aggr essi v e  or al
hy dr ati on  i s  a  r easonabl e  r ecom m endati on  i n  the  m anagem ent  of   an
uncom pl i cated  U TI.   Al though  ther e  i s  no  ev i dence  that  hy dr ati on  i m pr ov es
the  r esul ts  of   appr opr i ate  anti m i cr obi al   ther apy ,   i t  does  di l ute  the
bacter i a  and  r em ov es  i nf ected  ur i ne  by   f r equent  bl adder   em pty i ng.

2.   What  can  thi s  w om an  do  to  hel p  pr ev ent  r ecur r ent  U TIs?

Som e  w om en  f i nd  that  sw i tchi ng  to  another   m ethod  of   bi r th  contr ol
consi der abl y   r educes  the  f r equency   of   r ecur r ent  bacter i al   U TIs.   Thor ough
cl eansi ng  of   the  per i neal   ar ea  bef or e  sex ual   r el ati ons  m ay   decr ease  the
i nci dence  of   postcoi tal   U TI  i n  those  pr one  to  U TI;  how ev er ,   m ost  pati ents
f i nd  thi s  to  be  an  i m pr acti cal   and  not  com pl etel y   ef f ecti v e  pr ev enti v e
m easur e.

Choosi ng  another   m ethod  of   bi r th  contr ol   m ay   not  be  necessar y   f or   m ost
w om en  i f   they   r em em ber   to  dr i nk   a  l ar ge  gl ass  of   w ater   bef or e
i nter cour se  and  v oi d  af ter   i nter cour se;  how ev er ,   studi es  hav e  show n  that
di aphr agm   usage  i s  an  i ndependent  r i sk   f actor   f or   U TI.   Regul ar   anti bi oti c
pr ophy l ax i s  shoul d  be  r eser v ed  f or   those  pati ents  w i th  a  hi stor y   of
m ul ti pl e  r ecur r ent  U TIs,   or   com pl i cated  U TI  or   upper   tr act  i nf ecti ons,   or
f or   i m m unocom pr om i sed  hosts.   The  di sadv antages  of   ongoi ng  pr ophy l ax i s
i ncl ude  the  dev el opm ent  of   dr ug­r el ated  si de  ef f ects  and  col oni zati on  w i th
m ul ti dr ug­r esi stant  or gani sm s.

3.   Shoul d  thi s  w om an's  sex ual   par tner   be  ev al uated  f or   U TI?

N o.   Al though  l ow er   U TIs  i n  w om en  ar e  associ ated  w i th  sex ual   acti v i ty ,


thi s  i s  not  a  sex ual l y   tr ansm i tted  di sease.   The  i nf ecti ng  or gani sm s  ar e
usual l y   endogenous  f l or a.   Heal thy   m en  w i thout  pr edi sposi ng  f actor s  such
as  ur i nar y   tr act  i nstr um entati on  or   di abetes  m el l i tus  r ar el y   get  l ow er
U TIs.   Bacter i al   pr ostati ti s  does  not  put  hi s  sex ual   par tner   at  r i sk   f or
cy sti ti s.

4.   Was  the  Gr am 's  stai ni ng  an  i m por tant  di agnosti c  test,   and  i n  w hat  w ay
di d  the  f i ndi ngs  al ter   the  m anagem ent  of   thi s  case?

When  bacter i ur i a  i s  f ound  i n  Gr am 's­stai ned,   uncentr i f uged  ur i ne,   thi s  i s  a


v er y   speci f i c  f i ndi ng  f or   the  di agnosi s  of   U TI.   The  f i ndi ng  of   m i cr oscopi c
bacter i ur i a  cor r esponds  to  ur i ne  cul tur e  col ony   counts  of   10 5 /m L  i n  m or e
than  90%  of   such
P. 224
speci m ens.   Di sti ngui shi ng  betw een  gr am ­posi ti v e  and  gr am ­negati v e
i nf ecti ons  can  be  qui te  usef ul   i n  m ak i ng  ther apeuti c  deci si ons.

5.   What  i s  the  v al ue  of   k now i ng  the  ur i ne  pH  i n  thi s  setti ng?

Al k al i ne  ur i ne  m ay   be  caused  by   i nf ecti on  w i th  Pr oteus  speci es,   w hi ch
pr oduce  ur ease.   The  pr esence  of   nonal k al i ne  ur i ne  i n  thi s  pati ent  m ak es
i nf ecti on  w i th  a  ur ea­spl i tti ng  or gani sm   unl i k el y .

6.   What  other   di agnosti c  or   l abor ator y   tests  shoul d  hav e  been  per f or m ed?

The  phy si cal   ex am i nati on  and  di agnosti c  studi es  per f or m ed  i n  an
em er gency   r oom   setti ng  shoul d  be  di r ected  tow ar d  el uci dati ng  the  natur e
of   the  pati ent's  chi ef   com pl ai nt  and  hi stor y .   A  pel v i c  ex am i nati on  w oul d
be  appr opr i ate  i f   the  pati ent  had  r epor ted  sy m ptom s  of   i ncr eased  v agi nal
di schar ge,   dy spar euni a,   or   ex posur e  to  a  k now n  sex ual l y   tr ansm i tted
di sease  i n  the  par tner .   The  i ndi cati ons  f or   per f or m i ng  cul tur es  f or
sex ual l y   tr ansm i tted  pathogens  ar e  si m i l ar   to  those  f or   a  pel v i c
ex am i nati on.   Chl am y di a,   U r eapl asm a,   N .   gonor r hoeae,   or   My copl asm a
i nf ecti on  shoul d  hav e  been  consi der ed  i n  thi s  pati ent  i f   no  or gani sm s
w er e  seen  on  the  Gr am 's­stai ned  ur i ne  speci m ens,   or   i f   subsequent
r outi ne  bacter i al   cul tur es  gr ew   no  or gani sm s.

Intr av enous  py el ogr aphy   and  a  r enal   ul tr asound  ex am i nati on  shoul d  be
r eser v ed  f or   w hen  a  com pl i cated  U TI  or   upper   U TI  such  as  py el onephr i ti s
i s  suspected.   A  pr egnancy   test  shoul d  be  per f or m ed  i n  any   w om an  of
chi l dbear i ng  age  bef or e  pr escr i bi ng  an  anti bi oti c  that  m ay   be
contr ai ndi cated  i n  pr egnancy .

7.   What  w oul d  be  an  appr opr i ate  anal gesi c  f or   a  pati ent  w i th  U TI  w ho  i s
ex per i enci ng  sev er e  ur ethr al   di scom f or t?

Phenazopy r i di ne  hy dr ochl or i de  i s  a  ur i nar y   tr act  anal gesi c  agent  that
ex er ts  a  topi cal   anal gesi c  ef f ect  on  the  m ucosa  of   the  ur i nar y   tr act
thr ough  an  unk now n  m echani sm   of   acti on.   The  si de  ef f ects  ar e  m i ni m al ,
and  i ncl ude  the  ur i ne  acqui r i ng  a  r ed  or   or ange  col or   that  m ay   stai n
f abr i c.   It  i s  usual l y   not  necessar y   to  pr escr i be  m or e  than  a  2­day   suppl y
to  pati ents  w i th  uncom pl i cated  U TIs  w ho  ar e  r ecei v i ng  appr opr i ate
anti bi oti c  ther apy .   Opi oi d  anal gesi cs  ar e  r el ati v el y   contr ai ndi cated  i n  U TI
because  they   m ay   cause  acute  ur i nar y   r etenti on.

8.   What  si de  ef f ects  of   ther apy   shoul d  thi s  w om an  k now   about?

Vagi nal   candi di asi s  com m onl y   dev el ops  af ter   anti m i cr obi al   ther apy
because  anti bi oti cs  el i m i nate  m uch  of   the  nor m al   v agi nal   f l or a  and  cr eate
an  i deal   env i r onm ent  f or   the  ov er gr ow th  of   Candi da  al bi cans.
Hy per sensi ti v i ty   r eacti ons  m ay   occur   w i th  any   anti bi oti c;  how ev er ,   TMP­
SMX  m ay   r ar el y   al so  be  associ ated  w i th  i nter sti ti al   nephr i ti s,   asepti c
m eni ngi ti s,   Stev ens­Johnson  sy ndr om e,   or   er y them a  m ul ti f or m e.   A
car ef ul   hi stor y   to  r ul e  out  k now n  dr ug  al l er gy   i s  i m por tant.

9.   What  possi bl e  consequences  coul d  ar i se  i f   thi s  w om an  does  not  com pl y
w i th  ther apy ?

The  consequences  of   noncom pl i ance  w i th  ther apy   i ncl ude  conti nui ng
sy m ptom s,   the  i nducti on  of   anti bi oti c­r esi stant  str ai ns  of   m i cr oor gani sm s,
and,   m ost  i m por tant,   ascendi ng  i nf ecti on  l eadi ng  to  acute  py el onephr i ti s
or   ev en  a  per i nephr i c  abscess.

Suggested Readings
Bent  S,   Sai nt  S.   The  opti m al   use  of   di agnosti c  testi ng  i n  w om en  w i th  acute
uncom pl i cated  cy sti ti s.   Am   J  Med  2002;113(Suppl   1A):20S.

P. 225

Dunagan  WC,   Ri dner   ML.   Manual   of   m edi cal   ther apeuti cs,   26th  ed.   Boston:
Li ttl e,   Br ow n  and  Com pany ,   1995:257.

Fi hn  SD,   Latham   RH,   Rober ts  P,   et  al .   Associ ati on  betw een  di aphr agm   use
and  ur i nar y   tr act  i nf ecti on.   JAMA  1985;254:240.

Hooten  TM,   Schol es  D,   Hughes  JP,   et  al .   A  pr ospecti v e  study   of   r i sk   f actor s
f or   sy m ptom ati c  ur i nar y   tr act  i nf ecti ons  i n  y oung  w om en.   N   Engl   J  Med
1996;335:468.

Latham   RH,   Runni ng  K,   Stam m   WE.   U r i nar y   tr act  i nf ecti on  i n  y oung  adul t
w om en  caused  by   Staphy l ococcus  sapr ophy ti cus.   JAMA  1983;250:3063.

Lei bov i ci   L,   Al per t  G,   Laor   L,   et  al .   U r i nar y   tr act  i nf ecti ons  and  sex ual
acti v i ty   i n  y oung  w om en.   Ar ch  Inter n  Med  1987;147:345.

N or r by   SR.   Shor t  ter m   tr eatm ent  of   uncom pl i cated  l ow er   ur i nar y   tr act


i nf ecti ons  i n  w om en.   Rev   Inf ect  Di s  1990;12:458.

Rubi n  RH,   Fang  LST,   Jones  SR,   et  al .   Si ngl e­dose  am ox i ci l l i n  ther apy   f or
ur i nar y   tr act  i nf ecti on.   JAMA  1980;244:561.

Sobel   JD,   Kay e  D.   U r i nar y   tr act  i nf ecti ons.   In:  Mandel l   GL,   Bennett  JE,
Dol i n  R,   eds.   Pr i nci pl es  and  pr acti ce  of   i nf ecti ous  di seases,   6th  ed.   N ew
Yor k :  El sev i er   Sci ence,   2005:875.
Stam ey   TA.   Pathogenesi s  and  tr eatm ent  of   ur i nar y   tr act  i nf ecti ons.
Bal ti m or e:  Wi l l i am s  &  Wi l k i ns,   1980.

The Acquired Immunodeficiency Syndrome
1.   What  ar e  the  pr i nci pl es  of   anti r etr ov i r al   chem other apy ?

2.   When  shoul d  anti r etr ov i r al   chem other apy   be  star ted?

3.   What  ar e  the  m ost  i m por tant  hum an  i m m unodef i ci ency   v i r us  HIV­1â
€“associ ated  oppor tuni sti c  i nf ecti ons,   and  the  tr eatm ents  used  i n  the  HIV­
1–i nf ected  i ndi v i dual s  w ho  l i v e  i n  dev el oped  countr i es?

Discussion
1.   What  ar e  the  pr i nci pl es  of   anti r etr ov i r al   chem other apy ?

HIV­1–associ ated  m or bi di ty   and  m or tal i ty   i s  the  di r ect  r esul t  of


i m m unosuppr essi on  m edi ated  by   v i r al   r epl i cati on.   The  goal   of
anti r etr ov i r al   chem other apy   i s  to  dr i v e  pl asm a  HIV­1  l ev el s  to  bel ow   the
l i m i ts  of   detecti on  w i th  the  m ost  sensi ti v e  av ai l abl e  assay .   Thi s  appr oach
af f or ds  tw o  m ajor   benef i ts:  (a)  Successf ul   suppr essi on  of   v i r al   r epl i cati on
ar r ests  destr ucti on  of   the  i m m une  r esponse  and  al l ow s  f or   i m m une
r econsti tuti on.   Thi s,   i n  tur n,   r esul ts  i n  a  dr am ati c  decl i ne  i n  HIV­1â
€“associ ated  m or bi di ty   and  m or tal i ty .   (b)  The  em er gence  of   dr ug
r esi stance  can  be  el i m i nated  or   gr eatl y   r educed  by   dr i v i ng  v i r al
r epl i cati on  r ates  to  ex tr em el y   l ow   l ev el s.

Suppr essi on  of   pl asm a  HIV­1  RN A  to  l ev el s  of   20  copi es/m L  i s  cur r entl y
best  achi ev ed  thr ough  the  use  of   a  com bi nati on  r egi m en  contai ni ng  at
l east  thr ee  agents.   The  i ncl usi on  of   m ul ti pl e  agents  i s  r equi r ed  both  f or
potency   and  f or   i nter posi ng  a  si gni f i cant  geneti c  bar r i er   to  the  v i r us  w i th
r espect  to  the  em er gence  of   r esi stance.   HIV­1  r epl i cati on  occur s  at  the
r ate  of   appr ox i m atel y   10  bi l l i on  v i r al   par ti cl es  per   day   i n  each  i nf ected
per son.   Wi th  the
P. 226
r epl i cati v e  i nf i del i ty   of   HIV­1's  r ev er se  tr anscr i pti on  m echani sm ,   thi s
hi gh  l ev el   of   r epl i cati on  r api dl y   r esul ts  i n  the  cr eati on  of   a  di v er se
quasi speci es  of   v i r us.   Ther ef or e,   i t  i s  l i k el y   that  at  the  i nsti tuti on  of
ther apy ,   v i r al   v ar i ants  ex i st  that  ar e  r esi stant  to  each  cur r entl y   av ai l abl e
agent.   The  use  of   m ul ti pl e  agents  w i th  nonov er l appi ng  r esi stance
m echani sm s  r equi r es  the  v i r us  to  m ak e  m ul ti pl e  geneti c  changes  i n  each
v i r i on  i n  or der   to  per si st  i n  the  pr esence  of   al l   agents  i n  the  r egi m en.

At  pr esent,   r educti on  of   HIV­1  RN A  to  20  copi es/m L  i s  best  achi ev ed  w i th
the  sel ecti on  of   tw o  nucl eosi de  anal ogs  usual l y   adm i ni ster ed  as  f i x ed
dose  com bi nati ons  (and  an  “anchor â€​   dr ug—ef av i r enz  or   a  r i tonav i r ­
boosted  pr otease  i nhi bi tor ).   Al though  ther e  i s  no  si ngl e  r egi m en  that  i s
appr opr i ate  f or   al l   pati ents,   the  nucl eosi de  com bi nati on  of   tenof ov i r   and
em tr i ci tabi ne  w i th  ef av i r enz  has  becom e  the  m ost  f r equent  i ni ti al
com bi nati on  r egi m en.   The  r ecent  i ntr oducti on  of   a  si ngl e  tabl et  contai ni ng
these  tw o  nucl eosi des  and  ef av i r enz  has  pr ov i ded  the  f i r st  once­dai l y
si ngl e  pi l l   anti r etr ov i r al   r egi m en.   Other   f i x ed­dose  nucl eosi de
com bi nati ons,   i ncl udi ng  ei ther   zi dov udi ne  and  l am i v udi ne  or   abacav i r   and
l am i v udi ne,   m ay   al so  be  used  i n  com bi nati on  w i th  ef av i r enz.

Al though  ef av i r enz  i s  potent  and  w el l   tol er ated  i n  m ost  pati ents,   i t  cannot
be  used  i n  15%  to  20%  of   pati ents.   Ef av i r enz  i s  associ ated  w i th  centr al
ner v ous  sy stem   (CN S)  si de  ef f ects  that  r equi r e  up  to  10%  of   pati ents  to
seek   another   dr ug.   Ef av i r enz  i s  al so  ter atogeni c  and  shoul d  not  be  used  i n
sex ual l y   acti v e  w om en  of   chi l dbear i ng  age  w ho  ar e  not  usi ng  ef f ecti v e
bi r th  contr ol   m ethods.   Because  tr ansm i ssi on  of   dr ug­r esi stant  v i r uses  i s
i ncr easi ngl y   f r equent,   i t  i s  best  to  check   dr ug  suscepti bi l i ty   bef or e
i ni ti ati ng  anti r etr ov i r al   dr ugs.   Tr ansm i tted  dr ug  r esi stance  to  ef av i r enz  i s
f ound  i n  appr ox i m atel y   10%  of   pati ents  i ni ti ati ng  ther apy   f or   the  f i r st
ti m e  i n  cer tai n  l ocati ons  i n  the  U ni ted  States  and  Eur ope.   In  pati ents  f or
w hom   ef av i r enz  i s  not  an  opti m al   choi ce,   a  r i tonav i r ­boosted  pr otease
i nhi bi tor   (usual l y   r /l opi nav i r   or   r /atazanav i r )  i s  gener al l y   the  best  i ni ti al
choi ce.   Appr opr i ate  m anagem ent  of   anti r etr ov i r al   chem other apy   i s  both
an  ar t  and  a  sci ence  that  i s  best  accom pl i shed  by   phy si ci ans  w i th
substanti al   ex per i ence  i n  m anagem ent  of   pati ents  w i th  HIV­1  i nf ecti on.

2.   When  shoul d  anti r etr ov i r al   chem other apy   be  star ted?

Ther e  i s  no  si ngl e  answ er   that  i s  appr opr i ate  f or   ev er y   pati ent.   Ongoi ng
v i r al   r epl i cati on  i s  al w ay s  dam agi ng  to  the  i m m une  r esponse  of   the  host.
On  the  other   hand,   cur r ent  anti r etr ov i r al   r egi m ens  m ay   be  associ ated
w i th  si de  ef f ects,   and  r equi r e  si gni f i cant  di sci pl i ne  to  achi ev e  the  l ev el   of
v i r al   suppr essi on  associ ated  w i th  dur abl e  success.   As  CD4  cel l   counts
decl i ne,   pati ents  ar e  at  gr eater   r i sk   f or   HIV­1–associ ated  oppor tuni sti c
i nf ecti ons.   Ri si ng  pl asm a  HIV­1  RN A  l ev el s  ar e  associ ated  w i th  m or e
r api d  i m m unol ogi c  and  cl i ni cal   di sease  pr ogr essi on.   Adequate  suppr essi on
of   HIV­1  i s  best  achi ev ed  i n  pati ents  w i th  hi gh  CD4  cel l   counts  and  l ow
pl asm a  HIV­1  RN A  l ev el s.   Ther ef or e,   al l   thi ngs  bei ng  equal ,   i t  coul d  be
ar gued  that  ear l y   i nsti tuti on  of   ther apy   i s  associ ated  w i th  the  best  chance
of   l ong­ter m   success.   The  desi r e  to  star t  ther apy   ear l y   m ust  be  bal anced
by   a  consi der ati on  of   l ong­ter m   tox i ci ti es  and  the  com m i tm ent  of   the
pati ent  to  str i ct  adher ence  of   the  r egi m en  chosen.   In  gener al ,   the  ur gency
to
P. 227
star t  ther apy   i ncr eases  as  CD4  cel l   counts  f al l   and  the  pl asm a  HIV­1  RN A
l ev el s  r i se.   Most  ex per ts  r ecom m end  tr eatm ent  f or   any   pati ent  w i th  HIV­
1–r el ated  sy m ptom s  and  that  the  ther apy   be  star ted  f or   asy m ptom ati c
HIV­1  i nf ected  per sons  as  thei r   CD4  cel l   count  passes  i nto  the  250  to  350
cel l /m L  r ange.

3.   What  ar e  the  m ost  i m por tant  HIV­1–associ ated  oppor tuni sti c  i nf ecti ons,
and  the  tr eatm ents  used  i n  the  HIV­1–i nf ected  i ndi v i dual s  w ho  l i v e  i n
dev el oped  countr i es?
In  gener al ,   the  r i sk   of   v ar i ous  HIV­1–r el ated  i nf ecti ons  i ncr eases  w i th
di sease  pr ogr essi on  and  decl i ni ng  CD4  cel l   counts.   Tw o  notabl e
ex cepti ons  to  thi s  r ul e,   how ev er ,   ar e  pneum ococcal   pneum oni a  and
tuber cul osi s.   Al l   HIV­1–i nf ected  pati ents  ar e  at  i ncr eased  r i sk   f or
acqui r i ng  pneum ococcal   pneum oni a  and  sepsi s.   Whether   the
adm i ni str ati on  of   pneum ococcal   v acci ne  can  pr ev ent  or   l essen  the
sev er i ty   of   pneum ococcal   di sease  i n  these  pati ents  has  not  been  pr ov ed,
but  the  cur r ent  pr acti ce  i s  to  adm i ni ster   pneum ococcal   v acci ne  to  al l   HIV­
1–i nf ected  pati ents  w hose  CD4  l y m phocy te  counts  ex ceed  500/m m 3 .   The
r esponse  to  v acci nati on  i n  pati ents  w i th  counts  of   l ess  than  500/m m 3   i s
l i k el y   to  be  enhanced  i f   they   ar e  on  ef f ecti v e  anti r etr ov i r al   ther apy   at  the
ti m e  of   v acci nati on.

Tuber cul osi s  i s  one  of   the  f ew   HIV­1–r el ated  i nf ecti ons  that  i s
tr ansm i ssi bl e  to  i m m unocom petent  per sons.   HIV­i nf ected  per sons  ar e  at
i ncr eased  r i sk   f or   acqui r i ng  tuber cul osi s  r egar dl ess  of   the  stage  of   thei r
HIV­1  i nf ecti on.   Because  pati ents  w i th  HIV­1  i nf ecti on  hav e  r educed
cel l ul ar   i m m uni ty ,   a  thr eshol d  of   5  cm   i s  consi der ed  to  be  a  posi ti v e
tuber cul i n  sk i n  test.   As  the  CD4  cel l   counts  f al l ,   pati ents  m ay   becom e
aner gi c  and  the  tuber cul i n  sk i n  test  f ur ther   l oses  i t  sensi ti v i ty .

Or al   candi di asi s  (thr ush)  m ost  f r equentl y   occur s  w hen  the  CD4
l y m phocy te  count  f al l s  bel ow   300/m m 3 .   Thr ush  can  usual l y   be  tr eated
w i th  topi cal   anti f ungal   agents  (ny stati n  sw i sh  and  sw al l ow ,   or
cl otr i m azol e  tr oches),   but  m or e  sev er e  cases,   especi al l y   w hen  esophageal
l esi ons  ar e  pr esent,   m ay   r equi r e  sy stem i c  anti f ungal   agents  such  as
f l uconazol e.

Ear l y   i n  the  acqui r ed  i m m unodef i ci ency   sy ndr om e  (AIDS)  epi dem i c,
Pneum ocy sti c  ji r ov eci   (f or m er l y   car i ni i )  pneum oni a  w as  the  m ost  com m on
AIDS­def i ni ng  i l l ness.   Wi th  the  adv ent  of   ef f ecti v e  pr ophy l acti c  r egi m ens,
thi s  i l l ness  has  becom e  m uch  l ess  f r equent.   Pneum ocy sti s  pneum oni a  i s
usual l y   tr eated  w i th  TMP­SMX.   Al ter nati v el y ,   i ntr av enous  pentam i di ne,
or al   tr i m ethopr i m /  dapsone,   or   or al   atov aquone  can  be  used  i n  sul f a­
al l er gi c  pati ents.

HIV­1–i nf ected  per sons  w i th  l ess  than  200  CD4  l y m phocy tes/m m 3   ar e  at
r i sk   f or   sev er al   ty pes  of   CN S  i nf ecti ons.   One  of   the  m ost  com m on  causes
of   i ntr acr ani al   m asses  i n  HIV­1–i nf ected  pati ents,   Tox opl asm a  gondi i ,   i s
tr eated  w i th  py r i m etham i ne  and  sul f adi azi ne.   Cr y ptococcus  neof or m ans,
Hi stopl asm a  capsul atum ,   and  Cocci di oi des  i m m i ti s  can  cause  CN S  di sease
or   di ssem i nated  di sease  i n  HIV­1–i nf ected  pati ents;  i nf ecti on  w i th  these
pathogens  i s  usual l y   tr eated  w i th  am photer i ci n  B.

Pati ents  w i th  l ess  than  50  CD4  l y m phocy tes/m m 3   ar e  at  r i sk   f or   suf f er i ng
di ssem i nated  i nf ecti on  w i th  My cobacter i um   av i um   [m y cobacter i um   av i um
com pl ex   (MAC)]  or   ocul ar   or   sy stem i c  cy tom egal ov i r us  i nf ecti ons.
Di ssem i nated
P. 228
MAC  i s  com m onl y   m ani f ested  cl i ni cal l y   by   the  appear ance  of   sy stem i c
sy m ptom s  (f ev er ,   w ei ght  l oss,   ni ght  sw eats,   and  anem i a).   Tr eatm ent  w i th
a  com bi nati on  of   tw o  or   thr ee  acti v e  agents  i s  r equi r ed  f or   MAC  i nf ecti on.
Cy tom egal ov i r us  r eti ni ti s  pr esents  w i th  pai nl ess  l oss  of   v i si on  and  m ay   be
accom pani ed  by   sy stem i c  ev i dence  of   i nf ecti on,   m ani f est  by   f ev er ,   w ei ght
l oss,   or   gastr oi ntesti nal   sy m ptom s.   Tr eatm ent  w i th  ganci cl ov i r   (or
v al ganci cl ov i r ),   f oscar net,   or   ci dof ov i r   i s  usual l y   ef f ecti v e.

Case
A  32­y ear ­ol d  w om an  i s  f ound  to  be  HIV­1  ser oposi ti v e  at  the  ti m e  of   a  l i f e
i nsur ance  phy si cal   ex am i nati on.   The  pati ent  has  had  no  pr i or   ser i ous  m edi cal
i l l nesses  al though  she  has  ex per i enced  i ncr eased  v agi nal   i tchi ng  dur i ng  the
l ast  y ear .   Her   phy si cal   ex am i nati on  i s  nor m al   ex cept  f or   v agi nal   thr ush.   Her
soci al   hi stor y   r ev eal s  that  she  has  been  m ar r i ed  to  the  sam e  m an  f or   the  l ast
3  y ear s.   He  i s  al so  heal thy .   U pon  detai l ed  questi oni ng,   he  adm i tted  to  hav i ng
ex per i m ented  w i th  sex   w i th  m en  on  sev er al   occasi ons  w hi l e  tr av el i ng  to  San
Fr anci sco  8  y ear s  ear l i er .   He  i s  subsequentl y   f ound  to  be  HIV­1  ser oposi ti v e
w i th  a  CD4  cel l   count  of   860  cel l s/m m 3   and  a  pl asm a  HIV­1  RN A  l ev el   of
13, 000  copi es/m L.
The  l abor ator y   ev al uati on  r ev eal s  that  she  has  a  posi ti v e  enzy m e­l i nk ed
i m m unosor bent  assay   (ELISA)  f or   anti bodi es  to  HIV­1.   HIV­1  ser oposi ti v i ty
w as  conf i r m ed  by   a  Wester n  bl ot  assay .   Her   pur i f i ed  pr otei n  der i v ati v e  (PPD)
i s  negati v e,   as  i s  her   ser ol ogy   f or   T.   gondi i .   Her   r api d  pl asm a  r eagi n  (RPR)  i s
negati v e.   Her   hem atocr i t  i s  43.   Her   w hi te  bl ood  cel l   count  i s  5, 200/m m 3 .   Her
CD4  cel l   count  i s  340  cel l s/m m 3 .   Her   pl asm a  HIV­1  RN A  l ev el   i s  143, 000
copi es/m L.

1.   What  w oul d  y ou  r ecom m end  to  her   w i th  r espect  to  anti r etr ov i r al
chem other apy ?
2.   How   w oul d  y ou  al ter   y our   r ecom m endati ons  i f   y ou  l ear ned  she  i s
pr egnant  at  the  ti m e  of   pr esentati on?  If   she  w er e  pr egnant,   i s  i t  l i k el y
that  her   chi l d  w oul d  be  i nf ected?
3.   What  w oul d  y ou  r ecom m end  to  her   husband  w i th  r espect  to  anti r etr ov i r al
chem other apy ?

Case Discussion
1.   What  w oul d  y ou  r ecom m end  to  her   w i th  r espect  to  anti r etr ov i r al
chem other apy ?

You  shoul d  r ecom m end  to  her   that  she  i ni ti ate  anti r etr ov i r al   ther apy .
Al though  her   CD4  cel l   count  i s  i n  a  r ange  that  w oul d  pr om pt  som e
pr acti ti oner s  to  r ecom m end  def er r i ng  ther apy ,   the  pr esence  of   v agi nal
thr ush  i s  a  cl i ni cal   i ndi cator   of   HIV­1  di sease  and  pl aces  her   at  gr eater
r i sk   f or   an  oppor tuni sti c  i nf ecti on  than  a  w om an  w i th  the  sam e  CD4  cel l
count  and  no  sy m ptom s.   Ther apy   shoul d  not  be  i ni ti ated  unti l   a  v i r al
suscepti bi l i ty   test  r esul t  has  been  obtai ned.   In  her   case,   i t  r ev eal s  that
her   v i r us  i s  r esi stant  to  ef av i r enz,   l i k el y   r ef l ecti ng  the  acqui si ti on  of   the
dr ug­r esi stant  v i r us  f r om   her   husband.   Because  of   thi s,   y ou  shoul d
r ecom m end  a  r egi m en  that  uses  a  boosted  pr otease  i nhi bi tor   as  the
anchor   dr ug  such  as  tenof ov i r ,   em tr i ci tabi ne,   and  r /l opi nav i r .

2.   How   w oul d  y ou  al ter   y our   r ecom m endati ons  i f   y ou  l ear ned  she  i s
pr egnant  at  the  ti m e  of   pr esentati on?  If   she  w er e  pr egnant,   i s  i t  l i k el y
that  her   chi l d  w oul d  be  i nf ected?

The  gener al   appr oach  to  anti r etr ov i r al   chem other apy   i n  pr egnancy   shoul d
be  the  sam e  as  i t  i s  i n  a  nonpr egnant  w om an.   Ef f ecti v e  m anagem ent  of
an  i nf ected
P. 229
w om an  r equi r es  cl ose  col l abor ati on  am ong  an  i nter ni st  or   i nf ecti ous
di sease  speci al i st  w i th  ex per i ence  usi ng  anti r etr ov i r al   ther apy ,   an
obstetr i ci an  w i th  ex per i ence  deal i ng  w i th  HIV­1–i nf ected  m other s,   and  a
pedi atr i ci an  w i th  HIV­1  ex per ti se.   The  dual   goal s  of   ther apy   i n  thi s
setti ng  ar e  to  suppr ess  v i r al   r epl i cati on  to  benef i t  the  m other   and  to
decr ease  the  r i sk   of   tr ansm i ssi on  of   HIV­1  to  her   baby .   Wi th  her   CD4  cel l
count  and  pl asm a  HIV­1  RN A  l ev el ,   she  i s  at  r i sk   f or   di sease  pr ogr essi on
ov er   the  nex t  sev er al   y ear s  and,   as  m enti oned  i n  the  pr ecedi ng  tex t,
m ost  ex per ts  w oul d  r ecom m end  anti r etr ov i r al   chem other apy   to  her   once
she  i s  thr ough  her   f i r st  tr i m ester   of   pr egnancy .   Al though  ther e  i s  no
ev i dence  that  tenof ov i r   pl aces  f etuses  at  r i sk ,   ther e  i s  m or e  ex per i ence
w i th  zi dov udi ne  and  l am i v udi ne  i n  pr egnancy ;  so  i t  w oul d  be  pr ef er abl e  to
use  these  agents  i nstead  of   tenof ov i r   and  em tr i ci tabi ne.   Because  her
v i r us  i s  not  suscepti bl e  to  ef av i r enz,   i t  w oul d  not  be  used.   N onethel ess,
ev en  i f   her   v i r us  w er e  suscepti bl e  to  the  dr ug,   i t  shoul d  not  be  used  i n
pr egnancy   because  of   concer ns  about  ter atogeni ci ty .   Al though  d4T  and  ddI
ar e  used  l ess  f r equentl y   these  day s,   they   shoul d  be  av oi ded  w henev er
possi bl e  i n  pr egnant  w om en  because  of   l acti c  aci dosi s,   hepati c  steatosi s,
and  pancr eati ti s.   N ev i r api ne  has  been  associ ated  w i th  sev er e  hepati ti s,
especi al l y   i n  w om en  w i th  CD4  cel l   count  m or e  than  250/m m 3   and  shoul d
be  av oi ded  i n  thi s  pati ent  f or   that  r eason.   Ther ef or e,   she  w oul d  best  be
tr eated  w i th  a  pr otease  i nhi bi tor   as  the  anchor   dr ug  i n  her   r egi m en.
Al though  nel f i nav i r   i s  of ten  used  i n  pr egnancy ,   concer ns  about  i ts  potency
dam pen  the  enthusi asm   f or   i t—ev en  i n  thi s  setti ng.   R/l opi nav i r   i s  a  v er y
r easonabl e  choi ce,   al though  ther e  ar e  data  that  suggest  i ncr eased
m etabol i sm   of   l opi nav i r   dur i ng  pr egnancy .   Ther ef or e,   dr ug  l ev el s  shoul d
be  f ol l ow ed­up.

Bef or e  the  adv ent  of   anti r etr ov i r al   chem other apy ,   the  l i k el i hood  of
tr ansm i tti ng  HIV­1  f r om   m other   to  chi l d  w as  i n  the  r ange  of   25%.
Zi dov udi ne  m onother apy   adm i ni ster ed  dur i ng  the  thi r d  tr i m ester   of
pr egnancy ,   coupl ed  w i th  i ntr av enous  zi dov udi ne  dur i ng  del i v er y   and  6
w eek s  of   zi dov udi ne  f or   the  i nf ant,   r educed  the  r i sk   of   per i natal
tr ansm i ssi on  to  8%.   Mor e  potent  contem por ar y   anti r etr ov i r al
chem other apeuti c  r egi m ens  hav e  r educed  thi s  r i sk   to  bel ow   1%.   The  goal
of   ther apy   i n  the  m other   shoul d  be  to  r educe  pl asm a  HIV­1  RN A  l ev el s  to
l ess  than  20  copi es/m L  by   del i v er y .   The  baby   shoul d  al so  r ecei v e
anti r etr ov i r al   chem other apy   as  par t  of   the  per i natal   tr ansm i ssi on
pr ev enti on  str ategy .   The  neonate  shoul d  not  be  br eastf ed,   r egar dl ess  of
the  m other 's  pl asm a  HIV­1  l ev el ,   i n  v i ew   of   the  r i sk   of   tr ansm i ssi on  of
HIV­1  by   thi s  r oute.

3.   What  w oul d  y ou  r ecom m end  to  her   husband  w i th  r espect  to  anti r etr ov i r al
chem other apy ?

Her   husband  has  been  i nf ected  f or   m or e  than  5  y ear s  and  has  m ai ntai ned
a  l ow   pl asm a  HIV­1  RN A  l ev el   and  a  near   nor m al   CD4  cel l   count.   Al though
he  i s  techni cal l y   not  a  l ong­ter m   nonpr ogr essor   because  he  has  not  been
docum ented  to  be  i nf ected  f or   m or e  than  10  y ear s,   hi s  pl asm a  HIV­1  RN A
l ev el   and  CD4  cel l   count  pr edi ct  that  hi s  di sease  pr ogr essi on  r i sk   i s  v er y
l ow .   Most  ex per ts  w oul d  not  r ecom m end  anti r etr ov i r al   chem other apy   to
hi m   at  thi s  poi nt.   Al though  anti r etr ov i r al   chem other apy   i s  not  i ndi cated,
he  shoul d  under go  a  f ul l   i ni ti al   ev al uati on  f or   HIV­1  i ncl udi ng  a  PPD,   an
RPR,   and  T.   gondi i   and  cy tom egal ov i r us  ser ol ogy   and  shoul d  be  f ol l ow ed­
up  at  3­  to  6­m onth  i nter v al s  f or   ev i dence  of   a  r i si ng  pl asm a  HIV­1
P. 230
RN A  l ev el   and/or   a  f al l i ng  CD4  cel l   count.   He  shoul d  al so  be  v acci nated
agai nst  pneum ococcal   di sease.   It  w oul d  al so  be  pr udent  to  test  hi s  v i r us
f or   r esi stance  to  anti r etr ov i r al   dr ugs  to  gui de  sel ecti on  of   hi s  r egi m en
w hen  he  r equi r es  ther apy .   Ev en  i f   hi s  v i r us  i s  f ound  to  be  suscepti bl e  to
ef av i r enz,   i t  shoul d  not  be  r el i ed  upon  i n  hi s  case  because  i t  i s  pr esum ed
that  hi s  w i f e  acqui r ed  her   r esi stant  v i r us  f r om   hi m   and  i t  i s  k now n  that
dr ug­r esi stant  v i r us  m ay   be  ov er gr ow n  by   w i l d  ty pe  v i r us  i n  the  pl asm a.
In  these  si tuati ons,   the  l i f el ong  per si stence  of   dr ug­r esi stant  m i nor
speci es  v ar i ants  l eads  to  tr eatm ent  f ai l ur e  i f   these  agents  ar e  used.

Suggested Readings
Bar nes  PF,   Bl och  AP,   Dav i dson  PT,   et  al .   Tuber cul osi s  i n  pati ents  w i th
hum an  i m m unodef i ci ency   v i r us  i nf ecti on.   N   Engl   J  Med  1991;324:1644.

Car penter   CJ,   Fi schl   MA,   Ham m er   SM,   et  al .   Anti r etr ov i r al   ther apy   f or   HIV
Inf ecti on  i n  1998.   JAMA  1998;280:78–86.

Connor   EM,   Sper l i ng  RS,   Gel ber   R,   et  al .   Pedi atr i c  ACTG  Pr otocol   076  Study
Gr oup.   Reducti on  of   m ater nal ­i nf ant  tr ansm i ssi on  of   hum an
i m m unodef i ci ency   v i r us  ty pe  1  w i th  zi dov udi ne  tr eatm ent.   N   Engl   J  Med
1994;331:1173–1180.

Gal l ant  JE,   Staszew sk i   S,   Pozni ak   AL,   et  al .   Ef f i cacy   and  saf ety   of
tenof ov i r   DF  v s  stav udi ne  i n  com bi nati on  ther apy   i n  anti r etr ov i r al ­naïv e
pati ents:  a  3­y ear   r andom i zed  tr i al .   JAMA  2004;292:191.
Depar tm ent  of   Heal th  and  Hum an  Ser v i ces  (DHHS)  Panel   on  Anti r etr ov i r al
Gui del i nes  f or   Adul ts  and  Adol escents.   Gui del i nes  f or   the  use  of
anti r etr ov i r al   agents  i n  HIV­1  i nf ected  adul ts  and  adol escents.   AIDS
Tr eatm ent  Gui del i nes  Panel   of   the  Depar tm ent  of   Heal th  and  Hum an
Ser v i ces,   Web  si te  (http://AIDSinfo. nih. gov),   2006.

Ham m er   SM,   Saag  MS,   Schechter   M,   et  al .   Tr eatm ent  f or   adul t  HIV
i nf ecti on:  2006  r ecom m endati ons  of   the  Inter nati onal   AIDS  Soci ety –U SA
Panel .   JAMA,   2006;296:827–843.

Ho  DD,   N eum ann  AU ,   Per el son  AS.   Rapi d  tur nov er   of   pl asm a  v i r i ons  and
CD4  l y m phocy tes  i n  HIV­1  i nf ecti on.   N atur e  1995;373:123–126.

Ki tahata  MM,   Koepsel l   TD,   Dey o  RA,   et  al .   Phy si ci ans'  ex per i ence  w i th  the
acqui r ed  i m m unodef i ci ency   sy ndr om e  as  a  f actor   i n  pati ents'  sur v i v al .   N
Engl   J  Med  1996;334(11):701–706.

Masur   H,   Ogni bene  FP,   Yar choan  R,   et  al .   CD4  counts  as  pr edi ctor s  of
oppor tuni sti c  pneum oni as  i n  hum an  i m m unodef i ci ency   v i r us  (HIV)  i nf ecti on.
Ann  Inter n  Med  1989;111:223.

Pal el l a  FJ,   Del aney   KM,   Moor m an  AC,   et  al .   Decl i ni ng  m or bi di ty   and
m or tal i ty   am ong  pati ents  w i th  adv anced  hum an  i m m unodef i ci ency   v i r us
i nf ecti on.   HIV  outpati ent  study   i nv esti gator s.   N   Engl   J  Med  1998;338:853â
€“860.

Cellulitis
1.   What  f actor s  pr edi spose  to  the  dev el opm ent  of   cel l ul i ti s?

2.   What  ar e  the  si gns  and  sy m ptom s  of   cel l ul i ti s?

3.   What  or gani sm s  m ost  f r equentl y   cause  cel l ul i ti s?

P. 231

Discussion
1.   What  f actor s  pr edi spose  to  the  dev el opm ent  of   cel l ul i ti s?

Al though  any   per son  can  acqui r e  cel l ul i ti s,   ther e  ar e  sev er al   f actor s  that
hei ghten  the  r i sk   of   thi s  i nf ecti on.   Any   com pr om i se  of   sk i n  i ntegr i ty   can
i ntr oduce  or gani sm s  i nto  the  sk i n  and  subcutaneous  ti ssues.   Ther ef or e,
sur gi cal   pr ocedur es,   tr aum a,   the  pl acem ent  of   i ntr av enous  catheter s,
bur ns,   and  bi te  w ounds  ar e  al l   f actor s  that  pr edi spose  to  the  dev el opm ent
of   cel l ul i ti s.   The  r i sk   f or   dev el opm ent  of   cel l ul i ti s  i s  al so  i ncr eased  i n
hosts  w hose  sensati on  i s  i m pai r ed,   such  as  di abeti c  pati ents  w i th
per i pher al   neur opathy   w hose  abi l i ty   to  per cei v e  and  r eact  appr opr i atel y
to  tr aum a  i s  di m i ni shed.

Im pai r ed  ar ter i al   ci r cul ati on  al so  pr edi sposes  to  the  dev el opm ent  of
cel l ul i ti s.   Host  i m m une  m echani sm s,   such  as  pol y m or phonucl ear
l euk ocy tes  and  com pl em ent,   ar e  del i v er ed  thr ough  the  ci r cul ati on.
Ther ef or e,   i f   the  host  ci r cul ati on  i s  i m pai r ed,   nor m al   i m m une
m echani sm s,   w hi ch  m i ght  easi l y   er adi cate  an  or gani sm ,   cannot  be
m ounted.   Thi s  i s  w hy   cel l ul i ti s  i s  m or e  f r equent  i n  pati ents  w i th  i m pai r ed
ar ter i al   ci r cul ati on,   such  as  those  w i th  di abetes  and  sm ok er s  w i th
per i pher al   v ascul ar   di sease.

Pati ents  w hose  v enous  and  l y m phati c  dr ai nage  i s  com pr om i sed  ar e  al so


l ess  abl e  to  cl ear   bacter i a  f r om   thei r   bodi es,   and  ar e  consequentl y
pr edi sposed  to  cel l ul i ti s.   Pati ents  w i th  chr oni c  edem a  of   the  l ow er
ex tr em i ti es  ar e  par ti cul ar l y   v ul ner abl e  to  cel l ul i ti s,   w hi ch  m ay   spr ead
v er y   r api dl y .   A  di sti ncti v e  f or m   of   cel l ul i ti s  has  been  f ound  i n  pati ents
w hose  saphenous  v ei ns  hav e  been  r em ov ed  f or   cor onar y   ar ter y   by pass
gr af ti ng.   These  pati ents,   w ho  m ost  l i k el y   hav e  both  v enous  i nsuf f i ci ency
and  i m pai r ed  l y m phati c  dr ai nage,   hav e  been  f ound  to  acqui r e  cel l ul i ti s  at
the  si te  of   the  saphenous  v enectom y .   Fr equentl y ,   the  por tal   of   entr y   f or
the  i nf ecti on  i s  associ ated  w i th  ti nea  pedi s.   Besi des  the  tr eatm ent  of
cel l ul i ti s,   the  ti nea  pedi s  shoul d  be  tr eated  w i th  a  topi cal   anti f ungal
agent.

Im m unocom pr om i sed  pati ents,   such  as  those  under goi ng  chem other apy   or
tr anspl antati on  pr ocedur es,   ar e  al so  v ul ner abl e  to  cel l ul i ti s.   The  i nf ecti on
i n  these  pati ents  m ay   be  m or e  di f f i cul t  to  di agnose  because  the
char acter i sti c  sy m ptom s  and  si gns  m ay   be  m or e  subtl e  ow i ng  to  the
anti i nf l am m ator y   pr oper ti es  of   the  i m m unosuppr essi on.

2.   What  ar e  the  si gns  and  sy m ptom s  of   cel l ul i ti s?

The  cl assi c  appear ance  ex hi bi ted  by   cel l ul i ti s  i s  a  hot,   sw ol l en,   r ed,   and
tender   sk i n  l esi on.   The  pati ent  m ay   be  f ebr i l e,   and  r egi onal
l y m phadenopathy   i s  com m on.   Acute  l y m phangi ti s,   i ndi cated  by   r ed
str eak s  cour si ng  up  the  pati ent's  l i m b  f r om   the  si te  of   the  cel l ul i ti s,
si gni f i es  the  spr ead  of   i nf ecti on  al ong  subcutaneous  l y m phati c  channel s.
N ot  al l   cases  of   cel l ul i ti s  ar e  associ ated  w i th  l y m phangi ti s,   but  i t  m ay   be
the  har bi nger   of   ser i ous  sy stem i c  i l l ness  w i th  bacter em i a.

3.   What  or gani sm s  m ost  f r equentl y   cause  cel l ul i ti s?

The  m ost  com m on  causes  of   cel l ul i ti s  i n  gener al   ar e  gr oup  A  str eptococci
and  S.   aur eus.   These  gr am ­posi ti v e  cocci   ar e  nor m al   consti tuents  of   the
P. 232
hum an  sk i n  f l or a  and  ar e  easi l y   i ntr oduced  i nto  w ounds  by   tr aum a.   Other
str eptococci   m ay   al so  occasi onal l y   cause  cel l ul i ti s.   Al though  one  coul d
r el y   on  β­l actam ase  r esi stant  peni ci l l i ns  or   cephal ospor i ns  i n  the  past  to
tr eat  m ost  cases  of   com m uni ty ­acqui r ed  S.   aur eus  i nf ecti on,   ther e  has
been  a  dr am ati c  i ncr ease  i n  m ethi ci l l i n­r esi stant  S.   aur eus  i nf ecti on
am ong  pati ents  w i th  com m uni ty ­acqui r ed  i nf ecti on  i n  m any   par ts  of   the
U ni ted  States.   In  these  ar eas,   pr esum pti v e  ther apy   w i th  v ancom y ci n  or
l i nezol i d,   pendi ng  i denti f i cati on  and  suscepti bi l i ty   testi ng,   i s  r equi r ed.
Pr acti ti oner s  m ust  be  aw ar e  of   l ocal   condi ti ons  i n  m ak i ng  em pi r i c
anti m i cr obi al   choi ces.

Less  com m on  pathogens  m ay   al so  be  i ntr oduced  i nto  a  w ound  by   tr aum a.
For   ex am pl e,   soi l ­contam i nated  w ounds  m ay   becom e  i nf ected  w i th  f ungi
or   Cl ostr i di um   speci es.   Ani m al   bi te  w ounds  m ay   becom e  i nf ected  w i th
bacter i a  f r om   the  ani m al 's  m outh.   Er y si pel oi d,   caused  by   Er y si pel othr i x
r husi opathi ae,   i s  a  cel l ul i ti s  that  af f ects  peopl e  w ho  handl e  sal t­w ater
f i sh,   shel l f i sh,   poul tr y ,   m eat,   or   ani m al   hi des.   Var i ous  Vi br i o  speci es  m ay
cause  cel l ul i ti s  i n  peopl e  w i th  w ounds  ex posed  to  sal t  w ater   or   r aw
seaf ood.

Less  f r equentl y ,   cel l ul i ti s  m ay   be  acqui r ed  thr ough  bacter em i a.   Rar e
cases  of   pneum ococcal   cel l ul i ti s  hav e  been  r epor ted.
Im m unocom pr om i sed  pati ents  m ay   al so  acqui r e  cel l ul i ti s  by   m eans  of   a
bacter em i a  caused  by   or gani sm s,   such  as  C.   neof or m ans  or   E.   col i ,   that
ar e  not  usual   causes  of   cel l ul i ti s  i n  heal thy   hosts.

Case
A  27­y ear ­ol d  m an  pr esents  to  the  em er gency   r oom   com pl ai ni ng  of   pai n  i n  hi s
r i ght  hand.   He  w as  w el l   unti l   the  pr ev i ous  day ,   w hen  he  sustai ned  a  deep
scr atch  at  the  base  of   hi s  r i ght  thum b  w hi l e  pl ay i ng  w i th  hi s  cat.   He  w ashed
the  w ound  and  bandaged  i t  ti ghtl y   to  stop  the  bl eedi ng.   Ov er ni ght,   how ev er ,
hi s  pal m   began  to  sw el l ,   tur ned  r ed,   and  becam e  i ncr easi ngl y   pai nf ul .
Hi s  bl ood  pr essur e  i s  120/70  m m   Hg,   hear t  r ate  i s  90  beats  per   m i nute,
r espi r ator y   r ate  i s  12  per   m i nute,   and  tem per atur e  i s  38. 5°C  (101. 3°F).
Phy si cal   ex am i nati on  f i ndi ngs  ar e  notabl e  f or   a  l acer ati on  on  the  r i ght  thenar
em i nence  that  i s  2  cm   l ong  and  0. 5  cm   deep.   The  w ound  i s  par ti al l y   cr usted
ov er   w i th  bl ood,   w i th  a  sm al l   am ount  of   ser osangui neous  di schar ge.   The
sur r oundi ng  ti ssue  i s  er y them atous,   hot,   and  ex qui si tel y   tender .   Ther e  ar e  tw o
r ed  str eak s  ascendi ng  the  l ow er   hal f   of   hi s  anter i or   f or ear m .   He  has  a  tender ,
m obi l e,   1­cm   l y m ph  node  i n  the  r i ght  ax i l l a.   Ther e  i s  f ul l   r ange  of   m oti on
w i thout  di scom f or t  i n  any   of   the  di gi ts  or   the  w r i st  of   hi s  r i ght  upper
ex tr em i ty .   N eur ol ogi c  ex am i nati on  of   the  hand  r ev eal s  nor m al   f i ndi ngs,   and
Al l en's  test  r esul t  i s  nor m al .

The  f ol l ow i ng  l abor ator y   data  ar e  f ound:  w hi te  bl ood  cel l   count,   15, 000/m m 3 ,
w i th  a  di f f er enti al   count  of   75%  pol y m or phonucl ear   l euk ocy tes,   5%  band
f or m s,   17%  l y m phocy tes,   2%  m onocy tes,   and  1%  eosi nophi l s.   Hi s  ser um
chem i str y   v al ues  ar e  nor m al .   A  r adi ogr aphi c  study   of   the  hand  r ev eal s  no
ev i dence  of   a  f or ei gn  body   or   subcutaneous  em phy sem a.   Gr am 's  stai ni ng  of
the  ser osangui neous  di schar ge  f r om   the  w ound  r ev eal s  l ar ge  num ber s  of   sm al l
gr am ­negati v e  r ods  and  a  f ew   gr am ­posi ti v e  cocci   i n  chai ns.   Sam pl es  of   the
di schar ge  and  bl ood  ar e  sent  f or   cul tur e.
The  pati ent  w as  bor n  and  r ai sed  i n  the  U ni ted  States.   He  has  been  i n  good
heal th  bef or e  thi s  i l l ness  and  has  no  hi stor y   of   hospi tal i zati ons.   He  r ecal l s
hav i ng  had  a  tetanus
P. 233
booster   shot  7  y ear s  ago.   He  has  no  hi stor y   of   al l er gi c  r eacti ons  to
m edi cati ons.   Hi s  7­y ear ­ol d  cat  w as  al so  bor n  and  r ai sed  i n  the  U ni ted  States,
has  r ecei v ed  al l   appr opr i ate  v acci nati ons,   and  i s  appar entl y   heal thy .

1.   What  i nf ecti ous  agents  shoul d  be  consi der ed  as  possi bl e  causes  of   thi s
pati ent's  cel l ul i ti s?
2.   What  w oul d  be  the  m ost  appr opr i ate  anti bi oti c  tr eatm ent  f or   thi s  pati ent?
3.   In  addi ti on  to  anti bi oti cs,   w hat  other   m easur es  shoul d  be  tak en  to  tr eat
thi s  cel l ul i ti s?

Case Discussion
1.   What  i nf ecti ous  agents  shoul d  be  consi der ed  as  possi bl e  causes  of   thi s
pati ent's  cel l ul i ti s?

Gr oup  A  str eptococci   and  S.   aur eus  m ust  al w ay s  be  consi der ed  as
potenti al   causes  of   cel l ul i ti s  because  they   ar e  the  m ost  com m on  eti ol ogi c
agents.   In  the  ev ent  of   ani m al   bi tes  or   scr atches,   the  or al   f l or a  of   the
ani m al   m ay   be  an  i m por tant  sour ce  of   i nf ecti on  as  w el l .   Pasteur el l a
m ul toci da  i s  f ound  i n  the  or ophar y nx   of   50%  to  70%  of   heal thy   cats  and
12%  to  60%  of   heal thy   dogs.   Thi s  gr am ­negati v e  r od  i s  f r equentl y
i m pl i cated  i n  i nf ecti ons  r esul ti ng  f r om   cat  bi tes  or   scr atches,   and  i s  f ound
l ess  of ten  i n  w ounds  i nf l i cted  by   dogs.   Other   i m por tant  ani m al   or al   f l or a
to  consi der   i n  pati ents  w i th  bi tes  and  scr atch  w ounds  i ncl ude  aer obi c  and
anaer obi c  str eptococcal   or gani sm s,   as  w el l   as  gr am ­negati v e  anaer obes
such  as  Bacter oi des  speci es  and  Fusobacter i um .  Or gani sm s  f ound  i n  soi l ,
such  as  Cl ostr i di a  speci es,   m ay   al so  be  tr ansm i tted  by   scr atches  or   bi tes.

The  r api d  tem po  of   thi s  pati ent's  i l l ness,   w i th  the  dev el opm ent  of   an
ex qui si tel y   pai nf ul   cel l ul i ti s  w i thi n  24  hour s  of   a  cat  scr atch,   i s
char acter i sti c  of   P.   m ul toci da  i nf ecti on,   al though  such  a  r api d  cour se  m ay
al so  be  seen  i n  the  setti ng  of   str eptococcal   i nf ecti ons.   It  w oul d  be
unusual ,   how ev er ,   f or   a  staphy l ococcal   i nf ecti on  to  pr ogr ess  thi s  r api dl y .
Mor eov er ,   the  di schar ge  f r om   a  staphy l ococcal   i nf ecti on  w oul d  m or e
l i k el y   be  pur ul ent  than  ser osangui neous.   The  f i ndi ng  of   m any   gr am ­
negati v e  r ods  on  the  Gr am 's­stai ned  speci m en  of   the  w ound  di schar ge
al so  suggests  a  P.   m ul toci da  i nf ecti on,   or   a  gr am ­negati v e  anaer obi c
i nf ecti on.   How ev er ,   a  f ew   gr am ­posi ti v e  cocci   i n  chai ns  w er e  al so  f ound,
m ak i ng  str eptococcal   i nf ecti on  a  par t  of   the  di f f er enti al   di agnosi s.

2.   What  w oul d  be  the  m ost  appr opr i ate  anti bi oti c  tr eatm ent  f or   thi s  pati ent?

Thi s  pati ent  has  a  ser i ous  hand  i nf ecti on,   al ong  w i th  an  i m pendi ng
sy stem i c  i l l ness.   Any one  w i th  such  a  ser i ous  hand  i nf ecti on  shoul d  be
hospi tal i zed  and  r ecei v e  i ntr av enous  anti bi oti cs  to  pr ev ent  adv anci ng
i nf ecti on,   as  w el l   as  to  av er t  the  potenti al l y   dev astati ng  consequences  of
subopti m al   ther apy .   Peni ci l l i n  i s  the  dr ug  of   choi ce  f or   P.   m ul toci da
i nf ecti ons,   and  w oul d  al so  be  ef f ecti v e  f or   the  m anagem ent  of   both
str eptococcal   and  anaer obi c  i nf ecti ons.   Ther ef or e,   i ntr av enous  peni ci l l i n
w oul d  be  the  best  anti bi oti c  i n  thi s  case.   For   pati ents  w ho  ar e  al l er gi c  to
peni ci l l i n,   tetr acy cl i ne  i s  the  best  al ter nati v e  dr ug  f or   the  tr eatm ent  of   P.
m ul toci da  i nf ecti ons.   The  pati ent  shoul d  al so  be  seen  i n  consul tati on  w i th
a  hand  sur geon  to  be  cer tai n  that  sur gi cal   i nter v enti on  f or   dr ai nage  or
decom pr essi on  i s  not  r equi r ed.

P. 234
3.   In  addi ti on  to  anti bi oti cs,   w hat  other   m easur es  shoul d  be  tak en  to  tr eat
thi s  cel l ul i ti s?

Ov er esti m ati ng  the  ef f i cacy   of   anti bi oti cs,   and  under esti m ati ng  the
cr i ti cal   r ol es  pl ay ed  by   debr i dem ent,   dr ai nage,   w ound  el ev ati on,   and
i m m obi l i zati on,   ar e  pr obabl y   the  m ost  f r equent  m i stak es  m ade  i n  the
tr eatm ent  of   cel l ul i ti s.   Dr ai nage  of   a  cl osed­space  i nf ecti on  and  r em ov al
of   necr oti c  ti ssue  ar e  essenti al   f or   cur i ng  any   i nf ecti on.   Ev en  w hen  the
appr opr i ate  anti bi oti cs  ar e  adm i ni ster ed,   an  i nf ecti on  can  w or sen  i f
abscesses  or   necr oti c  ti ssue  ar e  not  dr ai ned  or   r em ov ed.   The  r eason  f or
thi s  i s  that  abscesses  and  necr oti c  ti ssue  ar e  not  w el l   v ascul ar i zed,
m ak i ng  them   i naccessi bl e  to  both  the  anti bi oti cs  and  the  host  i m m une
m echani sm s,   such  as  pol y m or phonucl ear   l euk ocy tes  and  com pl em ent,
w hi ch  ar e  nor m al l y   conv ey ed  thr ough  the  bl oodstr eam .   Ther ef or e,   i n
these  i naccessi bl e  r egi ons  bacter i a  can  f r eel y   m ul ti pl y   and,   i n  som e
i nstances,   such  i nf ecti on  can  r esul t  i n  sepsi s  and  death  despi te  an
appr opr i ate  anti bi oti c  r egi m en.

Abscesses  tend  to  dev el op  i n  the  setti ng  of   P.   m ul toci da  i nf ecti on.   In
addi ti on,   the  hand  contai ns  sev er al   phy si ol ogi c  spaces,   such  as  the  thenar
em i nence,   that  can  ser v e  as  pock ets  of   i nf ecti on.   Ther ef or e,   a  P.
m ul toci da  cel l ul i ti s  of   the  hand  m ay   r equi r e  sur gi cal   debr i dem ent  and
dr ai nage.   Inci si on  of   a  hand  w ound  shoul d  not  be  per f or m ed  by   a  nov i ce,
because  ther e  i s  a  gr eat  potenti al   f or   dam agi ng  i nter nal   str uctur es  or
cr eati ng  w ounds  that  w oul d  r esul t  i n  ser i ous  contr actur es.   A  hand  sur geon
shoul d  be  consul ted  f or   thi s  pur pose.

The  objecti v e  of   el ev ati on  and  i m m obi l i zati on  i n  the  tr eatm ent  of
cel l ul i ti s  i s  to  di m i ni sh  the  edem a,   w hi ch  i m pedes  the  bl ood  f l ow   to  an
i nf ected  r egi on.   El ev ati on  of   the  af f ected  l i m b  abov e  the  l ev el   of   the
hear t  i s  necessar y   to  achi ev e  opti m al   r esul ts.   In  the  ev ent  of   a  l ow er
ex tr em i ty   cel l ul i ti s,   m er el y   pl aci ng  the  af f ected  l i m b  on  a  chai r   w hi l e
seated  i s  not  adequate  because  the  abdom i nal   contents  sti l l   ex er t
pr essur e  on  the  l y m phati c  v essel s  i n  thi s  posi ti on,   ther eby   per petuati ng
the  edem a.   In  addi ti on  to  the  m easur es  just  descr i bed,   thi s  pati ent  shoul d
r ecei v e  a  tetanus  booster   shot.   Any   pati ent  w i th  a  bi te  or   deep  scr atch
w ound  w ho  has  not  had  a  tetanus  booster   shot  w i thi n  the  pr ecedi ng  5
y ear s  shoul d  r ecei v e  one.

Suggested Readings
Center s  f or   Di sease  Contr ol .   Di phther i a,   tetanus,   and  per tussi s:  gui del i nes
f or   v acci ne  pr ophy l ax i s  and  other   pr ev enti v e  m easur es.   Ann  Inter n  Med
1985;103:896.

El l i ot  DL,   Tol l e  SW,   Gol dber g  L,   et  al .   Pet­associ ated  i l l ness.   N   Engl   J  Med
1985;313:985.

Fr anci s  DP,   Hol m es  MA,   Br andon  G.   Pasteur el l a  m ul toci da:  i nf ecti ons  af ter
dom esti c  ani m al   bi tes  and  scr atches.   JAMA  1975;233:42.

Fr i dk i n  SK,   Hagem an  JC,   Mor r i son  M,   et  al .   Methi ci l l i n­r esi stant
Staphy l ococcus  aur eus  di sease  i n  thr ee  com m uni ti es.   N   Engl   J  Med
2005;352:1436–1444.

Gol dstei n  EJC.   Bi tes.   In:  Mandel l   GL,   Bennett  JE,   Dol i n  R,   eds.   Pr i nci pl es
and  pr acti ce  of   i nf ecti ous  di seases,   6th  ed.   N ew   Yor k :  El sev i er   Sci ence,
2005:3552.

Mi l l er   LG,   Per dr eau­Rem i ngton  F,   Rei g  G,   et  al .   Four teen  pati ents  w i th
necr oti zi ng  f asci i ti s  caused  by   com m uni ty ­associ ated  m ethi ci l l i n­r esi stant
Staphy l ococcus  aur eus  i n  Los  Angel es.   N   Engl   J  Med  2005;352:1445.

Stev ens  DL,   Her r   D,   Lam per i s  H,   et  al .   Li nezol i d  v er sus  v ancom y ci n  f or   the
tr eatm ent  of   m ethi ci l l i n­r esi stant  Staphy l ococcus  aur eus  i nf ecti ons.   Cl i n
Inf ect  Di s  2002;34:1481.

P. 235

Sw ar tz  MN ,   Paster nak   MS.   Cel l ul i ti s  and  subcutaneous  ti ssue  i nf ecti ons.   In:
Mandel l   GL,   Bennett  JE,   Dol i n  R,   eds.   Pr i nci pl es  and  pr acti ces  of   i nf ecti ous
di seases,   6th  ed.   N ew   Yor k :  El sev i er   Sci ence,   2005:1172.

A Late Complication of Tuberculosis
1.   What  ar e  the  goal s  of   the  m oder n  dr ug  tr eatm ent  of   acti v e  pul m onar y
tuber cul osi s?

2.   What  f actor s  ar e  l i k el y   to  pr om ote  r el apse?

3.   What  f actor s  ar e  l i k el y   to  f oster   the  acqui si ti on  of   dr ug­r esi stant  di sease?

Discussion
1.   What  ar e  the  goal s  of   the  m oder n  dr ug  tr eatm ent  of   acti v e  pul m onar y
tuber cul osi s?

Fundam ental   to  the  m oder n  dr ug  tr eatm ent  of   tuber cul osi s  i s  the  use  of
m ul ti pl e­dr ug  r egi m ens.   Ther e  ar e  tw o  goal s  to  thi s  appr oach.

The  f i r st  object  of   m ul ti pl e­dr ug  tr eatm ent  i s  to  pr ev ent  the  em er gence  of
r esi stant  or gani sm s.   The  f i ndi ngs  f r om   ear l y   studi es  on  the  use  of
str eptom y ci n  dr am ati cal l y   dem onstr ated  the  f uti l i ty   of   m onother apy ,   i n
that  pati ents  w i th  sev er e  di sease  show ed  an  i ni ti al   gr ati f y i ng  r esponse  to
tr eatm ent,   but  af ter   som e  w eek s  thei r   condi ti on  began  to  deter i or ate.
Thei r   sputum   sm ear s  becam e  posi ti v e  f or   or gani sm s  once  agai n,   and
dr ug­r esi stant  di sease  dev el oped.   It  i s  bel i ev ed  that  m onother apy   sel ects
f or ,   r ather   than  i nduces  the  m utati on  of ,   r esi stant  or gani sm s.   Ther ef or e,
the  l ar ger   the  popul ati on  of   or gani sm s,   the  hi gher   the  l i k el i hood  that
r esi stant  or gani sm s  ar e  pr esent.   Ther ef or e,   i n  the  setti ng  of   an
asy m ptom ati c  pr i m ar y   i nf ecti on  that  i nv ol v es  f ew   or gani sm s,
m onother apy   (usual l y   consi sti ng  of   i soni azi d)  can  be  used  saf el y   as
pr ophy l ax i s.   In  pati ents  w i th  acti v e  di sease  (especi al l y   cav i tati ng
pul m onar y   di sease  i n  w hi ch  the  bur den  of   i nf ecti on  i s  i m m ense),   the
pr obabi l i ty   of   r esi stant  or gani sm s  i s  hi gh.   Mutati ons  l eadi ng  to  dr ug
r esi stance  ar e  unl i nk ed,   how ev er ,   so  the  use  of   tw o  dr ugs  (e. g. ,   i soni azi d
pl us  r i f am pi n)  ef f ecti v el y   pr ev ents  the  em er gence  of   secondar y   dr ug
r esi stance  (i . e. ,   dr ug  r esi stance  acqui r ed  dur i ng  tr eatm ent).

The  second  goal   of   ther apy   i s  to  shor ten  the  dur ati on  of   tr eatm ent.   To
achi ev e  a  l asti ng  cur e  i n  a  hi gh  pr opor ti on  of   cases,   r egi m ens  that
com pr i se  onl y   r i f am pi n  pl us  i soni azi d  m ust  be  conti nued  f or   9  m onths.
How ev er ,   thi s  can  be  r educed  to  6  m onths  by   the  addi ti on  of
py r azi nam i de  f or   the  f i r st  2  m onths.   Py r azi nam i de  i s  a  pow er f ul
ster i l i zi ng  dr ug  that  m ay   ex er t  i ts  ef f ect  by   acti ng  on  speci al
subpopul ati ons  of   or gani sm s,   such  as  those  i n  a  m or e  aci d  env i r onm ent.
It  has  been  show n  that  ther e  i s  no  addi ti onal   benef i t  i n  conti nui ng  thi s
ex pensi v e  dr ug  bey ond  the  f i r st  2  m onths.

A  f actor   to  be  consi der ed  w hen  pl anni ng  m ul ti dr ug  tr eatm ent  i s  that
i ni ti al   dr ug  r esi stance  ex i sts  w hen  the  di sease  i s  caused  by   or gani sm s
that  ar e  r esi stant  to  at  l east  one  dr ug  bef or e  any   tr eatm ent  i s  gi v en.
When  thi s  i s  suspected  on  epi dem i ol ogi c  gr ounds,   an  addi ti onal   dr ug
(usual l y   str eptom y ci n
P. 236
or   etham butol )  i s  added  to  the  r egi m en  dur i ng  the  f i r st  2  m onths,   w hi l e
the  r esul ts  of   dr ug  suscepti bi l i ty   studi es  ar e  aw ai ted.   Thi s  appr oach
r educes  the  r i sk   of   onl y   one  ef f ecti v e  dr ug  bei ng  gi v en.

2.   What  f actor s  ar e  l i k el y   to  pr om ote  r el apse?

Rel apse  (i . e. ,   the  endogenous  r eacti v ati on  of   pr ev i ousl y   tr eated


tuber cul osi s)  i s  m ost  l i k el y   to  occur   dur i ng  the  f i r st  y ear   af ter   the  end  of
tr eatm ent  and  i n  those  pati ents  w ho  i ni ti al l y   had  m or e  ex tensi v e  di sease.
Pati ents  w ho  di sconti nue  thei r   tr eatm ent  ear l y   ar e  m ost  l i k el y   to  hav e  a
r el apse.   Ther ef or e,   ensur i ng  com pl i ance  w i th  tr eatm ent  i s  centr al   to
pr ev enti ng  r el apse.

3.   What  f actor s  ar e  l i k el y   to  f oster   the  acqui si ti on  of   dr ug­r esi stant  di sease?

For   the  r easons  al r eady   outl i ned,   dr ug­r esi stant  or gani sm s  em er ge  w hen
a  pati ent  ef f ecti v el y   r ecei v es  onl y   m onother apy .   Thi s  m ay   occur   f or   a
v ar i ety   of   r easons,   and  the  f ol l ow i ng  i l l ustr ates  how   i t  can  happen.   A
pati ent  on  r i f am pi n  pl us  i soni azi d  m ay   sel l   hi s  pow er f ul   r ed  r i f am pi n
capsul es  to  hi s  f r i ends  (or   the  w i tch  doctor )  f or   use  i n  the  tr eatm ent  of
gonor r hea  and  then  tak e  onl y   the  i soni azi d  hi m sel f ,   r esul ti ng  i n  i soni azi d
m onother apy !  Thi s  m an  acqui r es  i soni azi d  r esi stance,   cough  r ecur s,   and
etham butol   i s  added  by   a  k i ndl y   phy si ci an,   w hi ch  ef f ecti v el y   now
consti tutes  etham butol   m onother apy .   Soon,   r esi stance  to  etham butol
em er ges  and  the  m an's  heal th  conti nues  to  decl i ne.   Per haps  he  w i l l   star t
tak i ng  hi s  r i f am pi n,   w hi ch  m eans  he  i s  r ecei v i ng  r i f am pi n  m onother apy .

Such  pati ents  f i r st  need  to  k now   that  they   m ust  ei ther   tak e  both
m edi cati ons  and  get  better ,   or   tak e  nei ther   and  get  w or se,   but  at  l east,   i n
thi s  l atter   i nstance,   the  di sease  r em ai ns  dr ug  suscepti bl e.   It  i s  al w ay s  a
m i stak e  to  add  one  dr ug  at  a  ti m e  to  a  f ai l i ng  r egi m en.   Instead,   at  l east
tw o  new   dr ugs  shoul d  be  added  to  pr otect  agai nst  the  em er gence  of
r esi stance  to  each  other .   Ful l y   super v i sed  ther apy   pr ev ents  scenar i os
such  as  these  f r om   happeni ng,   but,   unf or tunatel y ,   at  pr esent  i t  i s  not
f easi bl e  on  a  gl obal   scal e.

Case
A  73­y ear ­ol d  m an  i s  adm i tted  because  of   a  3­m onth  hi stor y   of   i nter m i ttent
hem opty si s.   Appr ox i m atel y   once  a  w eek   he  has  been  coughi ng  up  sm al l
am ounts  of   bl ood­str eak ed  sputum ,   but,   on  the  day   bef or e  adm i ssi on,   he
star ted  to  cough  f r equentl y   and  pr oduced  appr ox i m atel y   hal f   a  cupf ul   of   r ed
and  cl otted  bl ood  ov er   a  24­hour   per i od.   The  pati ent  had  em i gr ated  to  Am er i ca
i n  the  1940s  af ter   hav i ng  been  i nter ned  i n  a  l abor   cam p  i n  Eur ope  dur i ng
Wor l d  War   II.   A  m edi cal   ex am i nati on  at  the  ti m e  of   hi s  l i ber ati on  r ev eal ed  he
had  tuber cul osi s.   He  w as  then  adm i tted  to  a  sanator i um ,   w her e  he  stay ed  f or
18  m onths,   w i th  tr eatm ent  consi sti ng  of   ar ti f i ci al   pneum othor ax .   In  the  1950s,
he  had  a  r el apse  and  w as  tr eated  f or   18  m onths  w i th  i soni azi d,
par aam i nosal i cy l i c  aci d,   and  str eptom y ci n.   He  conti nued  to  sm ok e  a  pack   of
ci gar ettes  per   day   unti l   an  attack   of   pneum oni a  5  y ear s  bef or e,   w hi ch  caused
hi m   to  stop  sm ok i ng.   For   the  past  y ear ,   he  has  been  i ncr easi ngl y   di sabl ed  by
ex er ti onal   dy spnea,   such  that  he  i s  now   unabl e  to  cl i m b  a  f l i ght  of   stai r s
w i thout  stoppi ng.   He  has  al so  had  r ecur r ent  ex acer bati ons  of   br eathl essness
w i th  pr oducti v e  cough,   but  no  pr ev i ous  hem opty si s.   He  has  r ecentl y   noted
i ncr easi ng  ank l e  edem a.   He  has  no  hi stor y   of   w ei ght  l oss  or   f ev er .
P. 237
On  ex am i nati on,   he  i s  f ound  to  be  thi n  and  anx i ous,   af ebr i l e,   and
nor m otensi v e,   w i th  a  r egul ar   pul se  of   110  beats  per   m i nute.   He  i s  sl i ghtl y
tachy pnei c  and  has  both  centr al   and  per i pher al   cy anosi s.   The  jugul ar   v enous
pul se  i s  v i si bl e  appr ox i m atel y   3  cm   abov e  the  cl av i cl e  w hen  he  i s  at  an  angl e
of   45  degr ees.   He  has  a  di scr ete,   f i r m ,   nontender   l y m ph  node  that  i s  enl ar ged
to  appr ox i m atel y   2  cm   i n  the  r i ght  supr acl av i cul ar   f ossa.   The  tr achea  i s
dev i ated  to  the  r i ght,   and  the  r i ght  upper   chest  i s  noted  to  be  i ndr aw n  bel ow
the  cl av i cl e;  i t  i s  dul l   to  per cussi on  and  br onchi al   br eath  sounds  ar e  hear d.
The  apex   of   the  hear t  i s  not  pal pabl e  and  auscul tati on  of   the  hear t  r ev eal s  a
l oud  pul m onar y   second  sound.   He  has  bi l ater al   ank l e  edem a.
The  chest  r adi ogr aphi c  study   on  adm i ssi on  depi cts  bi l ater al ,   sev er e  f i br oti c
l ung  di sease,   w hi ch  i s  m ost  m ar k ed  i n  the  upper   l obes,   w i th  el ev ati on  of   the
hi l a;  the  abnor m al i ti es  ar e  m or e  pr onounced  on  the  r i ght.   The  hor i zontal
f i ssur e  i s  el ev ated  on  the  r i ght  and  pr ojects  upw ar d.   Dev i ati on  of   the  tr achea
to  the  r i ght  i s  conf i r m ed.   Ther e  ar e  thi n­w al l ed  cav i ti es  bi l ater al l y ,   and  a
cav i ty   on  the  r i ght  i s  f ound  to  contai n  an  opaci ty   that  i s  outl i ned  by   a
cr escent­shaped  r i m   of   ai r .   A  r ev i ew   of   hi s  l abor ator y   r ecor ds  r ev eal s  that
ser um   pr eci pi ti ns  f or   asper gi l l us  w er e  f ound  i n  hi s  bl ood  on  a  test  sent  f r om
the  outpati ent  depar tm ent  ear l i er   i n  the  m onth.

1.   What  i s  the  m ost  l i k el y   di agnosi s  i n  thi s  pati ent?


2.   If   m assi v e  hem opty si s  super v enes,   how   shoul d  thi s  be  m anaged?
3.   What  i s  the  m ost  usef ul   i nv esti gati on  to  conf i r m   the  di agnosi s?
4.   Shoul d  the  pati ent  r ecei v e  i ntr av enous  am photer i ci n  B?
5.   What  addi ti onal   l ate  com pl i cati on  of   tuber cul osi s  does  thi s  pati ent
ex hi bi t,   and  how   can  thi s  be  r el i ev ed?

Case Discussion
1.   What  i s  the  m ost  l i k el y   di agnosi s  i n  thi s  pati ent?

Di agnoses  that  shoul d  be  consi der ed  i n  thi s  pati ent  i ncl ude  asper gi l l om a,
r eacti v ati on  of   the  tuber cul osi s,   car ci nom a  of   the  br onchus,   and
br onchi ectasi s.   How ev er ,   thi s  pati ent  ex hi bi ts  the  cl assi c  cl i ni cal   pi ctur e
of   asper gi l l om a.   Asper gi l l us  col oni zes  and  gr ow s  sapr ophy ti cal l y   i n
cav i ti es  cr eated  by   pr eex i sti ng  l ung  di sease  (ty pi cal l y   those  caused  by
tuber cul osi s,   al though  occasi onal l y   other   di seases  such  as  sar coi dosi s,
br onchi ectasi s,   or   pul m onar y   f i br osi s  can  cause  the  f or m ati on  of   cav i ti es
hospi tabl e  to  such  i nf ecti on).   A  f ungus  bal l   dev el ops  i n  the  pr eex i sti ng
cav i ty ,   w hi ch  i s  l i ned  w i th  br onchi al   epi thel i um   or   gr anul ati on  ti ssue.
Chest  r adi ogr aphi c  studi es,   tom ogr am s,   or   com puted  tom ogr aphi c  (CT)
scans  can  show   the  r ounded  opaci ty   w i thi n  the  cav i ty ,   together   w i th  a
cr escent­shaped  r i m   of   ai r   betw een  the  cav i ty   and  i ts  w al l .   The  bal l   m ay
l i e  f r ee  w i thi n  the  cav i ty   (i n  w hi ch  case  i t  can  be  seen  to  change  posi ti on
on  decubi tus  chest  r adi ogr aphs)  or   i t  m ay   be  attached  by   gr anul ati on
ti ssue.   Of ten  the  pati ent  i s  asy m ptom ati c,   but  hem opty si s  i s  the  m ost
i m por tant  com pl i cati on  of   asper gi l l om a.   Ser um   pr eci pi ti ns  to  asper gi l l us
ar e  f ur ther   suppor ti v e  ev i dence  f or   thi s  di agnosi s  because  these  ar e
f ound  i n  90%  to  95%  of   cases  of   asper gi l l om a.

In  thi s  pati ent,   r eacti v ati on  of   the  tuber cul osi s  i s  l ess  l i k el y   than
asper gi l l om a  because,   despi te  a  3­m onth  hi stor y ,   the  pati ent  has  not  l ost
w ei ght  or   had  a  f ev er .   In
P. 238
addi ti on,   the  cav i ti es  seen  on  the  chest  r adi ogr aphs  hav e  thi n  w al l s,
suggesti ng  the  pr esence  of   i nacti v e  di sease.   How ev er ,   r adi ol ogi cal l y ,   i t  i s
i m possi bl e  to  di sti ngui sh  w i th  cer tai nty   betw een  acti v e  and  i nacti v e
tuber cul osi s.

Car ci nom a  of   the  br onchus  m i ght  al so  be  ex pected  to  cause  w ei ght  l oss,
and  i t  i s  an  i m por tant  consi der ati on  i n  pati ents  w i th  a  hi stor y   of
tuber cul osi s  because  they   ar e  at  hi gher   r i sk   than  the  gener al   popul ati on
because  of   the  car ci nom a  (usual l y   adenocar ci nom a  or   al v eol ar   cel l
car ci nom a)  that  can  f or m   i n  scar r ed  ti ssue,   as  an  af ter m ath  of   i nf ecti on.
Thi s  pati ent  i s  al so  at  r i sk   f or   br onchi al   car ci nom a  because  of   hi s  l ong
hi stor y   of   sm ok i ng.

Tuber cul osi s  com m onl y   l eads  to  br onchi ectasi s,   w hi ch  w oul d  undoubtedl y
coex i st  i n  a  pati ent  l i k e  thi s  w ho  has  sev er e  destr ucti v e  l ung  di sease.
How ev er ,   thi s  pati ent's  epi sode  i s  dom i nated  by   hem opty si s,   r ather   than
by   the  ex pector ati on  of   copi ous,   pur ul ent  sputum ,   w hi ch  w oul d  be  m or e
suggesti v e  of   an  ex acer bati on  of   br onchi ectasi s.

2.   If   m assi v e  hem opty si s  super v enes,   how   shoul d  thi s  be  m anaged?

The  m ajor   r i sk   f r om   asper gi l l om a  i s  l i f e­thr eateni ng  hem opty si s.   The


pr ognosi s  f or   pul m onar y   asper gi l l om a  i s  negati v el y   i nf l uenced  by   a
num ber   of   f actor s  i ncl udi ng  the  docum entati on  of   i ncr easi ng  si ze  or
num ber s  of   the  asper gi l l om as  on  chest  r adi ogr aphs,   sev er e  under l y i ng
l ung  di sease,   i ncr easi ng  Asper gi l l us­speci f i c  i m m unogl obul i n  G  (IgG)
anti bodi es,   i m m unosuppr essi on  such  as  cor ti coster oi ds  or   HIV  i nf ecti on,
and  the  pr esence  of   sar coi dosi s.   Sur gi cal   r em ov al   i s  the  pr ef er r ed
tr eatm ent  i n  the  setti ng  of   l i f e­thr eateni ng  hem opty si s  due  to
asper gi l l om a,   al though  i n  thi s  pati ent  (as  i n  m any   w i th  asper gi l l om a),
sev er e  under l y i ng  l ung  di sease  i ndi cates  the  l i k el i hood  of   a  poor   outcom e
af ter   pul m onar y   sur ger y .   Em bol i zati on  of   the  br onchi al   ar ter y   has  been
used  successf ul l y   to  contr ol   sev er e  hem opty si s  due  to  tuber cul osi s,
am ong  other   causes,   but  has  not  been  successf ul   i n  the  m anagem ent  of
sev er e  hem opty si s  caused  by   asper gi l l om a,   pr obabl y   because  of   the  l ar ge
col l ater al   ci r cul ati on  i nv ol v ed.

Whi l e  ar r angem ents  ar e  m ade  f or   def i ni ti v e  m anagem ent,   a  pati ent  w i th


sev er e  hem opty si s  shoul d  be  posi ti oned  on  the  si de  of   the  suspected
sour ce  (i n  thi s  case,   the  r i ght  si de)  to  m i ni m i ze  f l oodi ng  of   the
unaf f ected  l ung  w i th  bl ood.   Sedati on  of   the  pati ent  i s  l i k el y   to  be
r equi r ed.   An  i ntr av enous  l i ne  shoul d  be  establ i shed  and  bl ood
cr ossm atched  and  adm i ni ster ed  w hen  needed.

3.   What  i s  the  m ost  usef ul   i nv esti gati on  to  conf i r m   the  di agnosi s?

N o  i nv esti gati on  (other   than  pathol ogi c  anal y si s  of   the  sur gi cal   speci m en)
i s  speci f i c  i n  conf i r m i ng  the  di agnosi s  of   asper gi l l om a,   but  Asper gi l l us
pr eci pi ti ns  ar e  pr esent  i n  a  hi gh  pr opor ti on  of   cases  of   asper gi l l om a  and
can  ser v e  to  conf i r m   the  di agnosi s  i n  a  pati ent  w i th  the  char acter i sti c
cl i ni cal   and  r adi ol ogi c  pr esentati on,   such  as  that  descr i bed  her e.   Sputum
cul tur e  f or   Asper gi l l us  or gani sm s  i s  l ess  hel pf ul   because  i t  m ay   y i el d  no
or gani sm s  i f   the  cav i ty   does  not  com m uni cate  w i th  the  br onchus.   Sk i n
tests  w i th  Asper gi l l us  anti gens  ar e  al so  l ess  r el i abl e.

Mi cr oscopi c  ex am i nati on  of   the  sputum   f or   aci d­al cohol –f ast  baci l l i
shoul d  be  done  i n  a  case  such  as  thi s  to  ex cl ude  coex i sti ng  acti v e
tuber cul osi s,   al though,   both  cl i ni cal l y   and  r adi ol ogi cal l y ,   thi s  i s  a  l ess
l i k el y   di agnosi s.   A  r easonabl e  pr ecauti on
P. 239
w oul d  be  to  pl ace  the  pati ent  i n  r espi r ator y   i sol ati on  unti l   negati v e  sm ear
r esul ts  hav e  been  obtai ned.

4.   Shoul d  the  pati ent  r ecei v e  i ntr av enous  am photer i ci n  B?

Ther e  hav e  been  no  pr ospecti v e  studi es  com par i ng  the  outcom e  i n
pati ents  w i th  asper gi l l om a  tr eated  w i th  i ntr av enous  am photer i ci n  B
v er sus  the  outcom e  i n  untr eated  pati ents.   The  f i ndi ngs  f r om   r etr ospecti v e
studi es  suggest,   how ev er ,   that  thi s  tr eatm ent  conf er s  no  benef i ci al   ef f ect,
and  thi s  i s  not  sur pr i si ng,   gi v en  that  the  f ungal   bal l   i s  i sol ated  f r om   the
bl oodstr eam .   Asy m ptom ati c  pati ents  m ay   si m pl y   be  m oni tor ed  and
r esol uti on  m ay   occur   spontaneousl y .   Pr ophy l acti c  sur gi cal   r em ov al   m ay
be  consi der ed  i n  pati ents  w ho  ar e  f i t  f or   the  pr ocedur e  because  of   the
potenti al   of   asper gi l l om a  to  cause  f atal   hem opty si s,   and  because  sur ger y
m ay   ef f ect  a  l asti ng  cur e.   How ev er ,   the  poor   ex er ci se  tol er ance  and  the
cl i ni cal   f i ndi ngs  i ndi cati ng  r espi r ator y   f ai l ur e  i n  thi s  case  suggest  that  the
pati ent  w oul d  be  unl i k el y   to  tol er ate  the  pr ocedur e.   Thi s  coul d  be
conf i r m ed  by   f or m al   pul m onar y   f uncti on  tests.   Poor   pr ognosti c  i ndi cator s
w oul d  be  an  ar ter i al   bl ood  gas  anal y si s  show i ng  an  el ev ated  par ti al
pr essur e  of   car bon  di ox i de  (PCO 2 )  and  a  f or ced  ex pi r ator y   v ol um e  of   l ess
than  1  L  per   second.

5.   What  addi ti onal   l ate  com pl i cati on  of   tuber cul osi s  does  thi s  pati ent
ex hi bi t,   and  how   can  thi s  be  r el i ev ed?

The  pati ent  has  centr al   cy anosi s,   i ndi cati ng  that  he  has  hy pox i a  at  r est.
Thi s  i s  an  addi ti onal   l ate  com pl i cati on  of   pul m onar y   tuber cul osi s,
pr obabl y   ex acer bated  i n  hi s  case  by   chr oni c  obstr ucti v e  pul m onar y
di sease  due  to  sm ok i ng.   The  hy pox i a  has  r esul ted  i n  pul m onar y
v asoconstr i cti on  and,   hence,   i n  pul m onar y   hy per tensi on  (i ndi cated  by   the
l oud  pul m onar y   second  sound  hear d  on  auscul tati on  of   the  hear t).   Thi s,   i n
tur n,   has  r esul ted  i n  r i ght  v entr i cul ar   f ai l ur e  (i ndi cated  by   the  r ai sed
jugul ar   v enous  pr essur e  and  edem a),   or   cor   pul m onal e.   Conti nuous
adm i ni str ati on  of   ox y gen  i s  i ndi cated  f or   r el i ef   of   thi s  sy ndr om e.

Suggested Readings
Ak bar i   JG,   Var m a  PK,   N eem a  PK,   et  al .   Cl i ni cal   pr of i l e  and  sur gi cal
outcom e  f or   pul m onar y   asper gi l l om a:  a  si ngl e  center   ex per i ence.   Ann
Thor aci c  Sur g  2005;80:1067.
Gl i m p  RA,   Bay er   AS.   Pul m onar y   asper gi l l om a:  di agnosti c  and  ther apeuti c
consi der ati ons.   Ar ch  Inter n  Med  1983;143:303.

Gr eene  R.   The  r adi ol ogi cal   spectr um   of   pul m onar y   asper gi l l osi s.   Med  My col
2005;43(Suppl   1):S147.

Kauf f m an  C.   Quandar y   about  tr eatm ent  of   asper gi l l om as  per si sts.   Lancet
1996;347:1640.

Ki m   YT,   Kang  MC,   Sung  SW,   et  al .   Good  l ong­ter m   outcom es  af ter   sur gi cal
tr eatm ent  of   si m pl e  and  com pl ex   pul m onar y   asper gi l l om a.   Ann  Thor ac  Sur g
2005;79:294.

Mi tchi son  DA.   Basi c  m echani sm s  of   chem other apy .   Chest  1979;76(6
Suppl ):771.

Shapi r o  MJ,   Al bel da  SM,   May ock   RL,   et  al .   Sev er e  hem opty si s  associ ated
w i th  pul m onar y   asper gi l l om a:  per cutaneous  i ntr acav i tar y   tr eatm ent.   Chest
1988;94:1225.

Stev ens  DA,   Vi r gi ni a  L,   Kan  VL,   et  al .   Pr acti ce  gui del i nes  f or   di seases
caused  by   asper gi l l us.   Cl i n  Inf ect  Di s  2000;30:696.

P. 240

Sepsis
1.   Is  ther e  a  cl i ni cal   di sti ncti on  betw een  bacter em i a  and  sepsi s?

2.   What  i s  the  di sti ncti on  betw een  chi l l s  and  r i gor s?

3.   What  f actor s  ar e  associ ated  w i th  a  poor   pr ognosi s  i n  the  setti ng  of   gr am ­
negati v e  sepsi s?

Discussion
1.   Is  ther e  a  cl i ni cal   di sti ncti on  betw een  bacter em i a  and  sepsi s?

It  i s  i m por tant  to  di f f er enti ate  am ong  bacter em i a,   sepsi s,   and  septi c
shock .   Bacter em i a  i s  def i ned  as  the  pr esence  of   v i abl e  bacter i a  i n  the
bl ood,   as  dem onstr ated  by   a  posi ti v e  bl ood  cul tur e.   Bacter em i as  m ay   be
f ur ther   cl assi f i ed  as  tr ansi ent,   sustai ned,   or   i nter m i ttent,   dependi ng  on
the  l ength  of   ti m e  bl ood  cul tur es  ar e  posi ti v e.   Tr ansi ent  bacter em i as  ar e
com m on  and  l ast  f or   onl y   sev er al   m i nutes.   When  m ul ti pl e  bl ood  cul tur es
ar e  posi ti v e  ov er   the  cour se  of   sev er al   hour s  to  sev er al   day s,   thi s
i ndi cates  a  sustai ned  bacter em i a.   Inter m i ttent  bacter em i as  ar e  those  i n
w hi ch  the  bl ood  cul tur es  ar e  i nter m i ttentl y   posi ti v e.   Sepsi s  i s  a  cl i ni cal
ter m   that  r ef er s  to  a  phy si ol ogi c  state  that  i s  associ ated  w i th  sev er e
i nf ecti on.   In  septi c  shock ,   ther e  i s  hy potensi on  (sy stol i c  bl ood  pr essur e
< 90  m m   Hg  or   a  one­thi r d  r educti on  f r om   the  pr i or   sy stol i c  bl ood
pr essur e)  and  ev i dence  of   end­or gan  dam age  secondar y   to  r educed  bl ood
f l ow .

2.   What  i s  the  di sti ncti on  betw een  chi l l s  and  r i gor s?

It  i s  v er y   i m por tant  to  k now   the  di f f er ence  betw een  chi l l s  and  r i gor s.
Ri gor   (a  tr ue  shak i ng  chi l l )  i s  v er y   of ten  associ ated  w i th  bacter em i a.   The
pati ent  m ay   ex per i ence  teeth  chatter i ng  and  body   tr em or s  that  usual l y
l ast  f or   15  to  30  m i nutes.   A  chi l l   i s  m or e  appr opr i atel y   descr i bed  as  a
chi l l y   sensati on,   not  a  cl i ni cal   pr esentati on.   Ri gor s  m ay   be  seen  i n  the
setti ng  of   v i r al   i nf ecti ons  as  w el l   as  bacter em i as.

3.   What  f actor s  ar e  associ ated  w i th  a  poor   pr ognosi s  i n  the  setti ng  of   gr am ­
negati v e  sepsi s?

Despi te  adv ances  i n  suppor ti v e  ther apy ,   the  m or tal i ty   r ate  associ ated
w i th  gr am ­negati v e  septi c  shock   appr oaches  40%.   Factor s  that  contr i bute
to  thi s  poor   pr ognosi s  ar e  i ncr eased  age,   poor   nutr i ti onal   status,   ster oi d
use,   ci r r hosi s,   di abetes,   congesti v e  hear t  f ai l ur e,   and  gr anul ocy topeni a.
Outcom e  i s  al so  adv er sel y   af f ected  by   v ol um e  depl eti on,   i nappr opr i ate
anti bi oti c  use,   and  del ay   i n  ther apy .

Case
A  74­y ear ­ol d  w hi te  m an  w i th  Al zhei m er 's  di sease  i s  br ought  to  the  em er gency
r oom   by   am bul ance  af ter   a  1­day   hi stor y   of   f ev er   and  m ental   status  changes.
On  ar r i v al   i n  the  em er gency   r oom ,   hi s  bl ood  pr essur e  i s  f ound  to  be  100/60
m m   Hg,   hear t  r ate  i s  100  beats  per   m i nute,   tem per atur e  i s  38. 5°C  (101. 3Â
°F),   and  r espi r ator y   r ate  i s  24  per   m i nute.   The  pati ent  i s  unabl e  to  gi v e  any
hi stor y ;  how ev er ,   hi s  w i f e  states  that  he  had  been  i n  hi s  usual   heal th  unti l   the
ev eni ng  bef or e  adm i ssi on,   w hen  he  began  to  com pl ai n  of   gener al i zed
abdom i nal   pai n  and  had  becom e  m or e  conf used  than  usual .
P. 241
Phy si cal   ex am i nati on  r ev eal s  an  agi tated  el der l y   m an  w ho  i s  i n  no  acute
di str ess.   Hi s  or al   m ucosa  i s  dr y   and  the  l ung  ex am i nati on  r ev eal s  decr eased
br eath  sounds  at  the  bases  bi l ater al l y .   Car di ac  ex am i nati on  r ev eal s  si nus
tachy car di a.   Abdom i nal   ex am i nati on  r ev eal s  nor m al   bow el   sounds  and  a
pal pabl e  m ass  i n  the  l ow er   abdom en  ex tendi ng  f r om   2  cm   bel ow   the  um bi l i cus
dow n  to  the  pel v i s.   Rectal   ex am i nati on  r ev eal s  an  enl ar ged,   f i r m   pr ostate,   and
the  stool   i s  hem e  negati v e.   Hi s  ex tr em i ti es  ar e  cool   and  cl am m y   and  ther e  i s
decr eased  sk i n  tur gor .
Adm i ssi on  l abor ator y   r esul ts  ar e  as  f ol l ow s:  w hi te  bl ood  cel l   count,
16, 000/m m 3   w i th  a  di f f er enti al   count  of   85%  pol y m or phonucl ear   l euk ocy tes,
10%  band  f or m s,   and  5%  l y m phocy tes;  hem atocr i t,   47%;  cr eati ni ne,   2. 3
m g/dL;  bl ood  ur ea  ni tr ogen,   40  m g/dL;  sodi um ,   141  m Eq/L;  potassi um ,   4. 5
m Eq/L;  chl or i de,   107  m Eq/L;  and  car bon  di ox i de,   17  m Eq/L.   Ar ter i al   bl ood  gas
m easur em ent  per f or m ed  on  r oom   ai r   r ev eal s  a  pH  of   7. 29,   a  PO 2   of   68  m m
Hg,   and  a  PCO 2   of   30  m m   Hg.   Hi s  chest  r adi ogr aphi c  f i ndi ngs  ar e
unr em ar k abl e.   The  pati ent  i s  ask ed  f or   a  ur i ne  speci m en  but  i s  abl e  to  v oi d
onl y   5  m L  of   cl oudy ,   dar k   y el l ow   ur i ne,   w hi ch  i s  sent  to  the  l abor ator y   f or
ur i nal y si s  and  cul tur e.
A  Fol ey   catheter   w as  subsequentl y   pl aced  and  500  m L  of   f oul ­sm el l i ng  ur i ne
w as  obtai ned.   A  Gr am 's  stai n  r ev eal ed  num er ous  pol y m or phonucl ear   cel l s  and
gr am ­negati v e  r ods.   On  r epeat  ex am i nati on,   hi s  abdom en  w as  f ound  to  be  sof t
and  the  m ass  had  di sappear ed.

1.   What  sy stem   i s  the  l i k el y   sour ce  of   i nf ecti on  i n  thi s  pati ent,   and  how
coul d  i nf ecti on  at  thi s  si te  ex pl ai n  hi s  other   si gns  and  sy m ptom s?
2.   What  gr oup  of   or gani sm s  i s  m ost  l i k el y   associ ated  w i th  the  sepsi s
sy ndr om e  i n  thi s  pati ent,   and  how   does  thi s  gr oup  di f f er   f r om   the  other
m ajor   gr oups  of   bacter i a  and  f ungi ?
3.   How   does  endotox i n  af f ect  m acr ophages,   and  w hat  chem i cal   si gnal s  ar e
pr oduced  by   m acr ophages  to  contr i bute  to  the  sepsi s  sy ndr om e?
4.   Of   w hat  shoul d  the  i ni ti al   m anagem ent  of   a  pati ent  w i th  the  sepsi s
sy ndr om e  consi st?

Case Discussion
1.   What  sy stem   i s  the  l i k el y   sour ce  of   i nf ecti on  i n  thi s  pati ent,   and  how
coul d  i nf ecti on  at  thi s  si te  ex pl ai n  hi s  other   si gns  and  sy m ptom s?

The  m ost  l i k el y   di agnosi s  that  f i ts  w i th  thi s  pati ent's  constel l ati on  of
sy m ptom s  i s  ur osepsi s.   As  br ought  to  l i ght  by   the  phy si cal   ex am i nati on,
the  pati ent  has  the  si gns  of   septi c  shock —i m pai r ed  ti ssue  per f usi on,
hy potensi on,   and  l acti c  aci dosi s  i n  associ ati on  w i th  posi ti v e  bl ood
cul tur es.   Tachy car di a,   tachy pnea,   and  ol i gur i a  ar e  al so  usual l y   seen  i n
the  setti ng  of   geni tour i nar y ,   gastr oi ntesti nal ,   bi l i ar y ,   and  gy necol ogi c
i nf ecti ons,   and  ther ef or e  ar e  not  speci f i c  to  ur osepsi s.   Abnor m al i ti es  i n
m ental   status  m ay   al so  be  a  f eatur e  of   the  i ni ti al   pr esentati on,   ev en
w i thout  i nf ecti on  i n  the  CN S.   In  el der l y   pati ents,   the  sy m ptom s  of   m ental
obtundati on  m ay   be  subtl e  and  consi st  onl y   of   w i thdr aw al   or   agi tati on,
and  they   m ay   consti tute  the  sol e  i ndi cati on  of   sev er e  i nf ecti on.   The  chest
r adi ogr aphi c  study   i n  thi s  pati ent  w as  negati v e  w i th  no  ev i dence  of   an
i nf i l tr ate,   m ak i ng  pneum oni a  unl i k el y .   How ev er ,   the  cl i ni ci an  m ust  al w ay s
k eep  i n  m i nd  that  w i th  hy dr ati on  an  i nf i l tr ate  m ay   bl ossom ,   so  the
pati ent's
P. 242
r espi r ator y   status  shoul d  be  m oni tor ed  cl osel y .   Because  the  m ental   status
changes  m ay   be  the  onl y   ear l y   m ani f estati on  of   sepsi s  i n  the  el der l y ,
m or e  of ten  than  not  a  l um bar   punctur e  y i el ds  nor m al   f l ui d.   Thi s  pati ent's
phy si cal   ex am i nati on  f i ndi ngs  w er e  al so  r em ar k abl e  f or   an  abdom i nal
m ass,   and  i ndeed  he  had  com pl ai ned  of   di f f use  abdom i nal   pai n  f or   at
l east  1  day   bef or e  adm i ssi on.   Cer tai nl y ,   el der l y   pati ents  m ay   hav e
appendi ci ti s,   di v er ti cul i ti s  w i th  an  abscess,   or   a  col on  car ci nom a  w i th
subsequent  bacter em i a,   and  al l   these  condi ti ons  m ust  be  i ncl uded  i n  the
di f f er enti al   di agnosi s.   He  w as  noted  to  hav e  an  enl ar ged  pr ostate  and
al so  had  di f f i cul ty   v oi di ng.

2.   What  gr oup  of   or gani sm s  i s  m ost  l i k el y   associ ated  w i th  the  sepsi s


sy ndr om e  i n  thi s  pati ent,   and  how   does  thi s  gr oup  di f f er   f r om   the  other
m ajor   gr oups  of   bacter i a  and  f ungi ?

The  or gani sm s  m ost  f r equentl y   i sol ated  i n  the  bl ood  of   pati ents  w i th  the
sepsi s  sy ndr om e  ar e  gr am ­negati v e  baci l l i .   Shock   occur s  l ess  f r equentl y
i n  the  setti ng  of   bacter em i a  due  to  gr am ­posi ti v e  or gani sm s.   Thi s
di f f er ence  m ay   stem   f r om   v ar i ati ons  i n  the  host  r esponse  to  di f f er ent
bacter i al   cel l   w al l   consti tuents.   The  l i popol y sacchar i de  por ti on  of   the  cel l
w al l   of   gr am ­negati v e  baci l l i   (cal l ed  endotox i n)  el i ci ts  a  v i gor ous
i nf l am m ator y   r esponse  w hen  i njected  i ntr av enousl y   i nto  ani m al s.   The
i nf l am m ator y   r esponse  to  l i potei choi c  aci d,   a  cel l   w al l   consti tuent  of
gr am ­posi ti v e  or gani sm s,   i s  m uch  l ess  pr onounced.   Thi s  pati ent  had  not
been  hospi tal i zed,   nor   had  he  under gone  any   k i nd  of   i nstr um entati on.
Ther ef or e,   the  m ost  l i k el y   or gani sm   to  cause  a  U TI  w i th  subsequent
bacter em i a  and  sepsi s  sy ndr om e  i n  thi s  pati ent  i s  E.   col i .   Other   potenti al
gr am ­negati v e  or gani sm s  that  m ay   pr eci pi tate  septi c  shock   i ncl ude
Kl ebsi el l a  pneum oni ae,   Pseudom onas  aer ugi nosa,   Enter obacter   aer ogenes,
and  Ser r ati a  m ar cescens.   The  pr i m ar y   por tal   of   entr y   i s  the  geni tour i nar y
tr act,   but  the  gastr oi ntesti nal   tr act,   r espi r ator y   tr act,   and  sk i n  ar e  al so
i m por tant  sour ces  of   bacter em i a.   Enter ococcus  m ust  al so  be  consi der ed
as  a  potenti al   cause  of   thi s  pati ent's  i l l ness  because  i t  can  f r equentl y
cause  pr ostati ti s.   Other   gr am ­posi ti v e  or gani sm s,   such  as  coagul ase­
posi ti v e  and  coagul ase­negati v e  Staphy l ococcus  speci es,   can  cer tai nl y
cause  bacter em i a  and  sepsi s  sy ndr om e;  how ev er ,   thi s  occur s  m ost
com m onl y   i n  hospi tal i zed  pati ents  w ho  hav e  had  som e  ty pe  of
i ntr av ascul ar   dev i ce  i nstal l ed.   Gr am 's  stai ns  ar e  par ti cul ar l y   usef ul   i n
establ i shi ng  pr esum pti v e  di agnoses  i n  U TIs,   as  i t  w as  i n  thi s  case.

3.   How   does  endotox i n  af f ect  m acr ophages,   and  w hat  chem i cal   si gnal s  ar e
pr oduced  by   m acr ophages  to  contr i bute  to  the  sepsi s  sy ndr om e?

Sev er al   bacter i al   f actor s  ar e  pow er f ul   m edi ator s  of   sepsi s,   and  one  of   the
m ost  potent  i s  endotox i n.   As  stated,   endotox i n  i s  the  l i popol y sacchar i de
com ponent  of   the  cel l   w al l   i n  gr am ­negati v e  bacter i a.   It  appear s  that
w hen  cel l   i njur y   occur s  w i th  the  acti v ati on  of   i m m une  def enses  or   the
i ni ti ati on  of   anti m i cr obi al   ther apy ,   bacter i al   cel l   l y si s  tak es  pl ace  and  the
ti ter   of   detectabl e  endotox i n  i n  the  pati ent's  bl ood  r i ses  dr am ati cal l y .
Endotox i n  bi nds  to  the  CD14  m ol ecul e  on  the  sur f ace  of   m acr ophages  that
acti v ate  one  of   the  m em ber s  of   the  Tol l ­l i k e  r eceptor   f am i l y   (TLR­4).
TLR­4  tr i gger i ng  acti v ates  a  cascade  of   i nf l am m ator y   cy tok i nes  i ncl udi ng
tum or   necr osi s  f actor   α   (TN F­Î± )  and  a  num ber   of   addi ti onal   dow nstr eam
m edi ator s  i ncl udi ng  i nter l euk i n  (IL)­1,   IL­2,   IL­6,   and  pl atel et­acti v ati ng
f actor .   Af ter   the  r el ease  of   TN F­Î± ,   IL­1,   and  pl atel et­acti v ati ng  f actor ,
ar achi doni c  aci d  i s  m etabol i zed  to  f or m   l euk otr i enes,
P. 243
thr om box ane  A 2 ,   and  pr ostagl andi n  E 2 .   IL­1  and  IL­6  acti v ate  T  cel l s  to
pr oduce  i nter f er on­Î³,   IL­2,   IL­4,   and  gr anul ocy te–m acr ophage  col ony ­
sti m ul ati ng  f actor .   The  coagul ati on  cascade  and  com pl em ent  sy stem   ar e
al so  acti v ated  (Fi g.   6­1).   Cl i ni cal l y ,   thi s  phenom enon  r esul ts  i n  a  l ow
centr al   v enous  or   pul m onar y   capi l l ar y   w edge  pr essur e,   as  w el l   as  a
m ar k ed  decr ease  i n  total   sy stem i c  v ascul ar   r esi stance.   In  addi ti on,   ther e
i s  a  com pensator y   i ncr ease  i n  car di ac  output  i n  an  attem pt  to  m ai ntai n
ar ter i al   per f usi on.   The  end  r esul t  of   thi s  pr ocess  i s  an  i ncr ease  i n  car di ac
output,   a  m ar k ed  f al l   i n  per i pher al   v ascul ar   r esi stance,   and  hy potensi on.
If   uncontr ol l ed,   pr ogr essi v e  l acti c  aci dosi s  ensues,   ul ti m atel y   l eadi ng  to
death.
Figure  6­1  Pathogenesi s  of   the  m i cr ov ascul ar   i njur y   and  death  due  to
endotox i n  shock .   (Fr om   Kar chm er   AW,   Bar za  M,   Dr ew   WL,   et  al .   Inf ecti ous
di sease  m edi ci ne  (MKSAP  IX).   Phi l adel phi a:  Am er i can  Col l ege  of   Phy si ci ans,
1991. )
4.   Of   w hat  shoul d  the  i ni ti al   m anagem ent  of   a  pati ent  w i th  the  sepsi s
sy ndr om e  consi st?

Al though  anti bi oti c  ther apy   i s  the  m ai nstay   of   tr eatm ent  of   sepsi s  caused
by   gr am ­negati v e  or gani sm s,   the  am el i or ati on  of   under l y i ng  condi ti ons,
el i m i nati on  of   pr edi sposi ng  f actor s,   dr ai nage  of   abscesses,   r em ov al   of
i nf ected  f or ei gn  bodi es,   and  adequate  suppor ti v e  car e  ar e  al so  of
par am ount  i m por tance  f or   cur i ng  the  i nf ecti on.   It  i s  cr i ti cal   to  r em em ber
that  the  i m m edi ate  ther apeuti c  i nter v enti on  shoul d  be  di r ected  at
i ncr easi ng  car di ac  output  and  ox y gen  del i v er y   to  pr ev ent  or   m i ni m i ze
hy poper f usi on  and  r educe  ti ssue  hy pox i a.   An  opti m al   i ntr av ascul ar
v ol um e  m ust  be  r estor ed  and  m ai ntai ned.   Fl ui d  r equi r em ents  ar e  v er y
unpr edi ctabl e  because  of   capi l l ar y   l eak ,   and,   at  the  v er y   l east,   centr al
v enous  pr essur es  shoul d  be  m oni tor ed  so  that  these  r equi r em ents  can  be
appr opr i atel y   m et.   Respi r ator y   status  and  aci d–base  di stur bances  can
be  obser v ed  w i th  ser i al   ar ter i al   bl ood  gas  m easur em ents,   w hi ch  ar e  al so
hel pf ul   i n  deter m i ni ng  a  pati ent's  pr ognosi s.   The  chance  f or   sur v i v al   i s
r educed  i n  pati ents  w ho  ar e  aci dem i c.   The  nex t  step  i s  to  obtai n
appr opr i ate  cul tur es  and  adm i ni ster   appr opr i ate  bacter i ci dal   agents.   The
anti bi oti c  r egi m en  shoul d  be  chosen  on  the  basi s  of   the  pr esum ed  sour ce
of   the  bacter i a  and  the  suscepti bi l i ty   patter n  of   or gani sm s  f r om   that
sour ce.   Anti m i cr obi al   ther apy   shoul d  be  adjusted  on  the  basi s  of
m i cr obi ol ogi c  data  as  they   becom e  av ai l abl e.   Addi ti onal   ther api es  that
ar e  under   i nv esti gati on  ar e  based  on  the  em er gi ng  k now l edge  of   the
pathophy si ol ogi c  sequence  of   bacter em i c  shock .   The  conduct  and
i nter pr etati on  of   m any   of   these  studi es  hav e  been  com pl i cated  by   a  l ack
of   pr eci si on  of   entr y   cr i ter i a  that  m ak e  the  ex tr apol ati on  of   study   r esul ts
to  cl i ni cal   pr acti ce,   di f f i cul t.

Suggested Readings
Abr aham   E,   Later r e  PF,   Gar g  R,   et  al .   Dr otr ecogi n  al f a  (acti v ated)  f or
adul ts  w i th  sev er e  sepsi s  and  a  l ow   r i sk   of   death.   N   Engl   J  Med
2005;353:1332.

Abr aham   E,   Rei nhar t  K,   Opal   S,   et  al .   Ef f i cacy   and  saf ety   of   ti f acogi n
(r ecom bi nant  ti ssue  f actor   pathw ay   i nhi bi tor )  i n  sev er e  sepsi s:  a
r andom i zed  contr ol l ed  tr i al .   JAMA  2003;290:238.

Cr oss  AS,   Opal   SM.   A  new   par adi gm   f or   the  tr eatm ent  of   sepsi s:  i s  i t  ti m e
to  consi der   com bi nati on  ther apy ?  Ann  Inter n  Med  2003;138:502.

Munf or d  RE.   Sepsi s,   sev er e  sepsi s  and  septi c  shock .   In:  Mandel l   GL,
Bennett  JE,   Dol i n  R,   eds.   Pr i nci pl es  and  pr acti ces  of   i nf ecti ous  di seases,
6th  ed.   N ew   Yor k :  El sev i er   Sci ence,   2005:906.
Ri edem ann  N C,   Guo  RF,   War d  PA.   N ov el   str ategi es  f or   the  tr eatm ent  of
sepsi s.   N at  Med  2003;9:517.

P. 244
P. 245

Sands  KE,   Bates  DW,   Lank en  PN ,   et  al .   Epi dem i ol ogy   of   sepsi s  sy ndr om e  i n
8  academ i c  m edi cal   center s.   JAMA  1997;278:234.

Tak eda  K,   Kai sho  T,   Ak i r a  S.   Tol l   l i k e  r eceptor .   Annu  Rev   Im m unol


2003;21:335.

Endocarditis
1.   What  ty pes  of   or gani sm s  can  cause  i nf ecti v e  endocar di ti s?

2.   What  ar e  som e  i m por tant  f actor s  that  i ncr ease  the  r i sk   f or   the
dev el opm ent  of   endocar di ti s?

3.   What  ar e  the  tw o  cl i ni cal   ty pes  of   bacter i al   endocar di ti s  and  thei r   cl i ni cal


char acter i sti cs?

4.   What  condi ti ons  cal l   f or   sur gi cal   i nter v enti on?

5.   Ar e  ther e  any   di f f er ences  i n  the  char acter i sti cs  of   endocar di ti s  betw een
the  i ntr av enous  dr ug  abuse  popul ati on  and  non–dr ug  abuser s?

Discussion
1.   What  ty pes  of   or gani sm s  can  cause  i nf ecti v e  endocar di ti s?

Ther e  ar e  m any   or gani sm s  that  can  i nf ect  the  hear t  and  cause
endocar di ti s.   The  m ost  com m on  ones  ar e  “tr adi ti onal â€​   bacter i a,   but
other   or gani sm s  i ncl udi ng  f ungi ,   Ri ck ettsi a,   and  Chl am y di a  m ay   i nv ade
m y ocar di al   ti ssues  and  pr oduce  di sease.

2.   What  ar e  som e  i m por tant  f actor s  that  i ncr ease  the  r i sk   f or   the
dev el opm ent  of   endocar di ti s?

The  tw o  m ajor   cl asses  of   r i sk   f actor s  contr i buti ng  to  the  dev el opm ent  of
endocar di ti s  i ncl ude  str uctur al   dam age  to  car di ac  ti ssue  i n  contact  w i th
bl ood  and  condi ti ons  associ ated  w i th  bacter em i a.   U nder l y i ng  hear t
di seases  such  as  r heum ati c  v al v ul ar   dam age,   a  bi cuspi d  aor ti c  v al v e,
patent  ductus  ar ter i osus,   and  sm al l   v entr i cul ar   septal   def ects  cause
dam aged  ti ssue  or   abnor m al   bl ood  f l ow ,   condi ti ons  under   w hi ch  bacter i a
can  adher e  to  the  sur f ace  and  cause  i nf ecti on.   Al so  i m pl i cated  f or   the
sam e  r easons  ar e  pr ostheti c  v al v es.   Intr av enous  dr ug  abuser s  ar e  at  r i sk
f or   endocar di ti s  because  thei r   v al v es  ar e  bei ng  constantl y   bom bar ded
w i th  i m pur i ti es  such  as  tal c,   w hi ch  causes  scar r i ng  of   the  v al v es,   and
al so  because  they   m i x   thei r   dr ug  of   choi ce  w i th  contam i nated  w ater .
N osocom i al   i nf ecti ons  m ay   r esul t  f r om   the  pl acem ent  of   i ntr av enous
catheter s  or   pacem ak er   w i r es,   or   f r om   w ound  i nf ecti ons  or   geni tour i nar y
m ani pul ati on.   El der l y   pati ents  ar e  al so  at  i ncr eased  r i sk   f or   endocar di ti s.

3.   What  ar e  the  tw o  cl i ni cal   ty pes  of   bacter i al   endocar di ti s  and  thei r   cl i ni cal


char acter i sti cs?

Al though  ther e  i s  ov er l ap,   the  tw o  cl i ni cal   ty pes  of   bacter i al   endocar di ti s


ar e  acute  and  subacute.   Acute  bacter i al   endocar di ti s  i s  m ost  com m onl y
associ ated  w i th  i ntr av enous  dr ug  abuse,   i ntr av enous  catheter   i nf ecti on,
and  pr ostheti c  v al v e  i nf ecti ons.   These  i nf ecti ons  m ay   be  r api dl y   f atal   i f
l ef t  untr eated,   and  sur gi cal   r epai r   or   r epl acem ent  of   the  dam aged  v al v e
m ay   be  necessar y .   Subacute
P. 246
endocar di ti s  dev el ops  m ost  of ten  i n  the  setti ng  of   str uctur al   hear t  di sease
(e. g. ,   m i tr al   v al v e  pr ol apse),   a  hi stor y   of   r heum ati c  hear t  di sease,   or
pr ostheti c  v al v es.   It  al so  af f ects  el der l y   pati ents,   or   i t  m ay   occur   i n  the
setti ng  of   no  k now n  v al v ul ar   di sease.   Its  onset  tends  to  be  m or e  i ndol ent.
Sy m ptom s  such  as  w eak ness,   f ati gue,   ni ght  sw eats,   and  w ei ght  l oss  m ay
hav e  ex i sted  f or   w eek s  to  m onths  bef or e  di agnosi s.   Its  onset  m ay   be
r el ated  to  antecedent  ev ents  such  as  dental   w or k ,   al though  no  def i ni te
pr edi sposi ng  ev ent  i s  appar ent  i n  m ost  cases.   Because  som e  pati ents  m ay
hav e  m ul ti pl e  r i sk   f actor s  and  ex hi bi t  a  v ar i abl e  cl i ni cal   pi ctur e,   thei r
di sease  cannot  be  easi l y   cl assi f i ed.   It  i s  al w ay s  i m por tant  to  k eep  i n  m i nd
the  m ax i m   that  “i f   y ou  don't  thi nk   about  endocar di ti s,   y ou  w on't
di agnose  i t!â€​

4.   What  condi ti ons  cal l   f or   sur gi cal   i nter v enti on?

Sur gi cal   i nter v enti on  shoul d  be  consi der ed  i f   (a)  ther e  i s  m or e  than  one
em bol i c  ev ent;  (b)  bacter em i a  per si sts  despi te  2  to  3  w eek s  of   adequate
anti bi oti c  ther apy ;  (c)  ther e  i s  pr ogr essi v e  or   r ef r actor y   congesti v e  hear t
f ai l ur e;  (d)  ther e  i s  si gni f i cant  v al v ul ar   dy sf uncti on  r esul ti ng  i n  m oder ate
to  sev er e  congesti v e  hear t  f ai l ur e  as  dem onstr ated  by   echocar di ogr aphy
or   other   l abor ator y   techni ques;  (e)  l ocal   suppur ati v e  com pl i cati ons  ar i se,
as  r ef l ected  by   the  appear ance  of   new ,   per si stent  el ectr ocar di ogr aphi c
conducti on  di stur bances,   echocar di ogr aphi c  ev i dence  of   a  par av al v ul ar
abscess  or   f i stul a,   pur ul ent  per i car di ti s,   or   per si stent  unex pl ai ned  f ev er
despi te  appr opr i ate  anti bi oti c  ther apy ;  (f )  ther e  i s  f ungal   endocar di ti s;  or
(g)  ther e  i s  appr opr i atel y   tr eated  pr ostheti c  v al v e  endocar di ti s  due  to
dr ug­r esi stant  or gani sm s  that  r ecur   despi te  appr opr i ate  anti m i cr obi al
ther apy .

5.   Ar e  ther e  any   di f f er ences  i n  the  char acter i sti cs  of   endocar di ti s  betw een
the  i ntr av enous  dr ug  abuse  popul ati on  and  nonaddi ct  popul ati on?

Ther e  ar e  sev er al   char acter i sti cs  of   endocar di ti s  r el ati v el y   uni que  to
i ntr av enous  dr ug  abuser s,   al though  these  ar e  onl y   gener al i zati ons.   A
hi stor y   of   docum ented  pr i or   hear t  di sease  i s  unusual ,   and  the  i nci dence  of
tr i cuspi d  v al v e  i nv ol v em ent  i s  appr ox i m atel y   50%  i n  thi s  popul ati on,
w hi ch  i s  m uch  hi gher   than  that  i n  the  nonaddi ct  popul ati on.   In  addi ti on,   a
m ur m ur   i s  f r equentl y   undetectabl e  and  ther e  i s  i sol ated  tr i cuspi d  v al v e
i nv ol v em ent,   unl i k e  the  m ur m ur s  of   aor ti c  or   m i tr al   v al v e  i nsuf f i ci ency
seen  m ost  com m onl y   i n  the  nonaddi ct  popul ati on.

Case
A  27­y ear ­ol d  w hi te  m an  pr esents  to  the  em er gency   r oom   w i th  a  chi ef
com pl ai nt  of   f ev er s,   shak i ng  chi l l s,   cough,   and  headache  of   2  day s'  dur ati on.
He  deni es  nausea,   v om i ti ng,   di ar r hea,   or   dy sur i a.   Hi stor y   r ev eal s  that  the
pati ent  sm ok es  one  pack   of   ci gar ettes  per   day ,   dr i nk s  a  si x ­pack   of   beer   per
day ,   and  has  r ecentl y   star ted  “sk i n­poppi ngâ€​   cocai ne.   He  has  had  no
pr ev i ous  hospi tal i zati ons  nor   has  he  under gone  any   sur gi cal   pr ocedur es.
Phy si cal   ex am i nati on  r ev eal s  a  tem per atur e  of   39. 0°C  (102. 2°F),   bl ood
pr essur e  of   120/80  m m   Hg,   pul se  of   114  beats  per   m i nute,   and  r espi r ator y
r ate  of   18  per   m i nute.   Hi s  conjuncti v ae  ar e  nor m al .   Hi s  or al   m ucosa  i s  m oi st
and  hi s  denti ti on  i s  good.   Lung  ex am i nati on  r ev eal s  som e  coar se  r honchi
bi l ater al l y .   Car di ac  ex am i nati on  r ev eal s  a  gr ade  2/6
P. 247
sy stol i c  m ur m ur   that  i s  hear d  best  at  the  l ef t  ster nal   bor der   but  does  not
r adi ate.   Abdom i nal   and  ex tr em i ty   f i ndi ngs  ar e  unr em ar k abl e.   N eur ol ogi c
ex am i nati on  r ev eal s  nonf ocal   f i ndi ngs,   al though  the  pati ent  does  com pl ai n  of   a
gl obal   headache.   Ther e  i s  no  m eni ngi sm us.

Labor ator y   v al ues  ar e  as  f ol l ow s:  w hi te  bl ood  cel l   count,   18, 000/m m 3   (85%
pol y m or phonucl ear   cel l s,   10%  bands,   and  5%  l y m phocy tes);  hem atocr i t,   38%;
and  pl atel ets,   170, 000/m m 3 .   A  chest  r adi ogr aphi c  study   r ev eal s  bi l ater al
nodul ar   i nf i l tr ates.   The  pati ent  i s  adm i tted  to  the  m edi cal   ser v i ce  f or   f ur ther
ev al uati on  and  tr eatm ent.

1.   What  ty pe  of   endocar di ti s  does  thi s  pati ent  l i k el y   hav e?


2.   What  i s  the  l i k el y   cause  of   hi s  pul m onar y   i nf i l tr ates?
3.   What  ar e  the  m ost  com m on  of f endi ng  pathogens  i n  thi s  setti ng?
4.   What  w oul d  y ou  pr escr i be  as  an  i ni ti al   anti bi oti c  r egi m en?

Case Discussion
1.   What  ty pe  of   endocar di ti s  does  thi s  pati ent  l i k el y   hav e?

Thi s  pati ent's  cl i ni cal   pr esentati on  i l l ustr ates  a  case  of   acute
endocar di ti s.   Tr i cuspi d  v al v e  (r i ght­si ded)  endocar di ti s  i s  m ost  l i k el y
because  i t  i s  com m onl y   associ ated  w i th  i ntr av enous  dr ug  abuse,   al though
the  m i tr al   and  aor ti c  v al v es  coul d  al so  be  i nv ol v ed.

2.   What  i s  the  l i k el y   cause  of   hi s  pul m onar y   i nf i l tr ates?

The  cause  of   thi s  pati ent's  pul m onar y   i nf i l tr ates  i s  septi c  em bol i   that
hav e  tr av el ed  to  the  l ung.   In  both  acute  and  subacute  bacter i al
endocar di ti s,   si gns  and  sy m ptom s  of   em bol i c  phenom ena  m ay   appear .
These  epi sodes  of   v ascul ar   occl usi on  cause  pai n  i n  the  chest  (pul m onar y
or   cor onar y ),   abdom en  (m esenter i c  or   spl eni c),   or   the  ex tr em i ti es.   Bone
pai n  (par ti cul ar l y   v er tebr al   and  sacr oi l i ac)  i s  al so  com m on  because  of   the
hem atogenous  spr ead  of   i nf ecti on  to  these  si tes.   Other   em bol i c
phenom ena  that  m ay   occur   i ncl ude  hem atur i a  (em bol i   to  the  k i dney s),
bl i ndness  r esul ti ng  f r om   r eti nal   ar ter y   occl usi on,   and  acute  neur ol ogi c
sy m ptom s  (str ok e,   m eni ngi ti s,   sei zur es,   and  headache).   Cer tai nl y ,
car di ac  i nv ol v em ent  such  as  congesti v e  hear t  f ai l ur e  m ay   occur   i n  thi s
setti ng  as  the  r esul t  of   pr ogr essi v e  v al v ul ar   i nsuf f i ci ency   or   m y ocar di ti s;
how ev er ,   thi s  w oul d  be  ev i denced  by   the  f i ndi ng  of   Ker l ey 's  B  l i nes  or
f l uf f y   pul m onar y   i nf i l tr ates  on  chest  r adi ogr aphi c  studi es.

3.   What  ar e  the  m ost  com m on  of f endi ng  pathogens  i n  thi s  setti ng?

The  or gani sm   that  w oul d  m ost  l i k el y   be  the  sour ce  of   thi s  pati ent's
i nf ecti on  i s  S.   aur eus.   Thi s  or gani sm   accounts  f or   appr ox i m atel y   20%  of
the  cases  of   endocar di ti s  i n  the  gener al   popul ati on,   and  f or   55%  of   the
cases  associ ated  w i th  i ntr av enous  dr ug  abuse.   It  shoul d  ther ef or e  be
suspected  as  the  eti ol ogi c  agent  i n  i nf ecti ons  associ ated  w i th  a  hi stor y   of
i ntr av enous  dr ug  abuse,   as  w el l   as  i n  the  contex t  of   acute  em bol i c
phenom ena  and  acute  bacter i al   endocar di ti s.   Coagul ase­negati v e
staphy l ococci   ar e  com m on  i n  the  setti ng  of   pr ostheti c  v al v e  endocar di ti s,
but  not  i n  the  setti ng  of   nonpr ostheti c  v al v e–associ ated  i nf ecti on.
Str eptococci   account  f or   appr ox i m atel y   70%  of   al l   cases  of   nati v e
v al v ul ar   bacter i al   endocar di ti s  i n  the  nonaddi ct  popul ati on,   and  i nf ecti on
due  to  the  v ar i ous  speci es  i s  br ok en  dow n  as  f ol l ow s:  40%  due  to
v i r i dans  str eptococci ;  10%  due  to  enter ococci   (gr oup  E
P. 248
str eptococci );  and  20%  due  to  other   nonhem ol y ti c,   m i cr oaer ophi l i c,
anaer obi c,   or   nonenter ococcal   gr oup  D  str eptococci .   Appr ox i m atel y   10%
of   the  cases  ar e  caused  by   other   f asti di ous  or gani sm s,   such  as  f ungi   and
gr am ­negati v e  baci l l i .

4.   What  w oul d  y ou  pr escr i be  as  an  i ni ti al   anti bi oti c  r egi m en?

The  i ni ti al   tr eatm ent  of   suspected  acute  bacter i al   endocar di ti s  shoul d  be


di r ected  tow ar d  S.   aur eus  because  i t  i s  the  m ost  com m on  or gani sm   i n
pati ents  w i th  acute  bacter i al   endocar di ti s.   The  cl i ni ci an  shoul d  al w ay s
dr aw   thr ee  to  f our   bl ood  speci m ens  f or   cul tur e  bef or e  i ni ti ati ng  anti bi oti c
ther apy .   Af ter   thi s  i s  done,   v ancom y ci n  (1  g  i ntr av enousl y   ev er y   12
hour s)  pl us  gentam i ci n  (1  m g/k g  i ntr av enousl y   ev er y   8  hour s)  ar e
appr opr i ate  as  an  i ni ti al   com bi nati on  unti l   the  cul tur e  r esul ts  ar e  k now n.
Thi s  com bi nati on  cov er s  both  S.   aur eus  (m ethi ci l l i n  sensi ti v e  and
r esi stant)  and  enter ococci   i nf ecti ons,   and,   w i th  f ew   ex cepti ons,   any   other
l i k el y   bacter i a.   In  the  past,   i ni ti al   ther apy   m i ght  hav e  consi sted  of
naf ci l l i n  and  gentam i ci n  but  the  i ncr easi ng  i nci dence  of   m ethi ci l l i n­
r esi stant  Staphy l ococcus  aur eus  (MRSA)  i nf ecti ons  m ak es  v ancom y ci n  a
m or e  pr udent  em pi r i c  choi ce,   especi al l y   i n  com m uni ti es  i n  w hi ch  MRSA  i s
f r equent.   The  anti m i cr obi al   r egi m en  shoul d  be  adjusted  on  the  basi s  of
r esul ts  f r om   the  cl i ni cal   m i cr obi ol ogy   l abor ator y ,   i ncl udi ng  bl ood  cul tur es
and  suscepti bi l i ty   tests.   Al though  ther e  ar e  a  v ar i ety   of   r ecom m endati ons
i n  the  l i ter atur e,   i t  i s  gener al l y   agr eed  that  a  pr ol onged  adm i ni str ati on  of
r el ati v el y   hi gh  doses  of   bacter i ci dal   agents  i s  i ndi cated.   Wi th  the
ex cepti on  of   i nf ecti on  caused  by   hi ghl y   r esi stant  or gani sm s,   i t  i s  usual l y
f ai r l y   easy   to  obtai n  a  good  sy m ptom ati c  r esponse  (e. g. ,   decl i ne  i n  f ev er
and  decr eased  m y al gi as)  and  ster i l i zati on  of   bl ood  cul tur es  w i thi n  a  f ew
day s  of   the  star t  of   ther apy .   A  bacter i ol ogi c  cur e  w i th  ster i l i zati on  of   the
l esi ons  i s  m uch  m or e  di f f i cul t,   how ev er ,   because  al though  v al v ul ar
l esi ons  ar e  bathed  i n  bl ood,   the  v al v es  them sel v es  ar e  r el ati v el y
av ascul ar .   Bacter i a  i n  v egetati ons  ar e  sur r ounded  by   f i br i n.   Thi s,   i n
com bi nati on  w i th  the  hi gh  f l ow   r ates  i n  the  car di ac  cham ber s,   m ak es  i t
di f f i cul t  f or   phagocy ti c  cel l s  to  adher e  to  the  si te  of   i nf ecti on.   Ther ef or e,
pr ol onged  tr eatm ent  w i th  hi gh  doses  of   bacter i ci dal   anti bi oti cs  i s
essenti al   f or   cur e.   Ther e  i s  i nv i tr o  and  i nv i v o  ev i dence  that  l ow ­dose
gentam i ci n  i n  com bi nati on  w i th  sem i sy ntheti c  peni ci l l i n  ef f ects  m or e  r api d
k i l l i ng  of   staphy l ococci   and  ster i l i zati on  of   v al v es  than  does  peni ci l l i n
al one.   Thi s  suggests  that  the  addi ti on  of   gentam i ci n  (1  m g/k g  ev er y   8
hour s  f or   3  to  5  day s)  i s  a  r easonabl e  r egi m en  (i f   the  pati ent  has  no
contr ai ndi cati ons  to  am i nogl y cosi de  use,   such  as  r enal   f ai l ur e)  i n  an
attem pt  to  cl ear   the  bacter em i a  r api dl y   and  m i ni m i ze  dam age  to  the
hear t  v al v es.   Ther e  ar e,   how ev er ,   no  data  f r om   r andom i zed,   bl i nded
studi es  show i ng  that  thi s  appr oach  has  an  i m pact  on  the  ul ti m ate  cl i ni cal
outcom e.   The  use  of   com bi nati on  ther apy   has  al so  per m i tted  shor ter
cour se  ther apy   of   r i ght­si ded  S.   aur eus  i nf ecti v e  endocar di ti s  i n
i ntr av enous  dr ug  user s.

Suggested Readings
Br andr i ss  MW,   Lam ber t  JS.   Car di ac  i nf ecti ons.   In:  Reese  RE,   Betts  RF,   eds.
A  pr acti cal   appr oach  to  i nf ecti ous  di seases,   3r d  ed.   Boston:  Li ttl e,   Br ow n
and  Com pany ,   1991:278.

Cham ber s  HF,   Kor zeni ow sk i   OM,   Sande  MA,   et  al .   Staphy l ococcus  aur eus
endocar di ti s:  cl i ni cal   m ani f estati ons  i n  addi cts  and  nonaddi cts.   Medi ci ne
(Bal ti m or e)  1983;62:170.

P. 249

Cham ber s  HF,   Mi l l er   RT,   N ew m an  MD.   Ri ght­si ded  Staphy l ococcus  aur eus
endocar di ti s  i n  i ntr av enous  dr ug  abuser s:  tw o­w eek   com bi nati on  ther apy .
Ann  Inter n  Med  1988;109:619.

Di N ubl e  M.   Abbr ev i ated  ther apy   f or   r i ght  si ded  Staphy l ococcus  aur eus
endocar di ti s  i n  i njecti ng  dr ug  user s.   Ann  Inter n  Med  1994;121:873.

Di Sal v o  G,   Habi b  G,   Per gol a  V,   et  al .   Echocar di ogr aphy   pr edi cts  em bol i c
ev ents  i n  i nf ecti v e  endocar di ti s.   J  Am   Col l   Car di ol   1991;37:1069.
Fr i dk i n  SK,   Hagem an  JC,   Mor r i son  M,   et  al .   Methi ci l l i n­r esi stant
Staphy l ococcus  aur eus  di sease  i n  thr ee  com m uni ti es.   N   Engl   J  Med
2005;352:1436–1444.

Mi l l er   LG,   Per dr eau­Rem i ngton  F,   Rei g  G,   et  al .   Four teen  pati ents  w i th
necr oti zi ng  f asci i ti s  caused  by   com m uni ty ­associ ated  m ethi ci l l i n­r esi stant
Staphy l ococcus  aur eus  i n  Los  Angel es.   N   Engl   J  Med  2005;352:1445.

Moon  MR,   Sti nson  EB,   Mi l l er   DC.   Sur gi cal   tr eatm ent  of   i nf ecti v e
endocar di ti s.   Pr og  Car di ov asc  Di s  1997;40:239.

Sul l am   PM,   Dr ak e  TA,   Sande  MA.   Pathogenesi s  of   endocar di ti s.   Am   J  Med


1985;78:110.

Fever and Abdominal Pain
1.   What  i s  the  si ngl e  best  test  to  ev al uate  the  f ebr i l e  pati ent  w i th  abdom i nal
pai n?

2.   What  ar e  the  m ost  i m por tant  pathogens  i n  the  bow el   f l or a?

3.   Besi des  obstr ucti on,   i schem i a,   and  i njur y   i nv ol v i ng  the  gut  (and  i ts
outpouchi ngs),   w hat  ar e  the  other   causes  of   per i toni ti s?

4.   What  ar e  sev er al   ex am pl es  of   ex tr aper i toneal   di seases  that  can  pr esent


w i th  abdom i nal   pai n  as  a  pr om i nent  sy m ptom ?

Discussion
1.   What  i s  the  si ngl e  best  test  to  ev al uate  the  f ebr i l e  pati ent  w i th  abdom i nal
pai n?

The  f ebr i l e  pati ent  w i th  abdom i nal   pai n  can  be  a  daunti ng  pr ospect.   The
di f f er enti al   di agnosi s  i n  thi s  setti ng  r anges  f r om   beni gn,   sel f ­l i m i ted
i nf ecti ons  such  as  v i r al   enter i ti s  to  sev er e,   l i f e­thr eateni ng  i nf ecti ons
such  as  per i toni ti s  r esul ti ng  f r om   an  i schem i c  bow el .   How ev er ,   despi te
the  av ai l abi l i ty   of   a  tr em endous  v ar i ety   of   i m agi ng  pr ocedur es  and  tests
of   bodi l y   f l ui ds,   the  si ngl e  best  appr oach  to  di agnosi s  i n  a  pati ent  w i th
f ev er   and  abdom i nal   pai n  r em ai ns  a  car ef ul   hi stor y   and  phy si cal
ex am i nati on.   Som eti m es  the  i nf or m ati on  y i el ded  i s  suf f i ci ent  to  m ak e  a
di agnosi s.   Mor e  of ten  tests  ar e  necessar y ,   but  a  car ef ul   cl i ni cal
ev al uati on  nar r ow s  the  l i st  of   questi ons  that  need  to  be  answ er ed  by
tests.   Fi shi ng  w i th  a  l ong  ser i es  of   tests  w i thout  w el l ­consi der ed  cl i ni cal
questi ons  occasi onal l y   hook s  the  tr ue  cul pr i t,   but  m or e  of ten  nets  a  catch
of   r ed  her r i ngs.
2.   What  ar e  the  m ost  i m por tant  pathogens  i n  the  bow el   f l or a?

A  com m on  concer n  i n  the  f ebr i l e  pati ent  w i th  abdom i nal   pai n  i s  the
possi bl e  contam i nati on  of   the  per i toneal   space  w i th  pathogens  f r om   the
bow el .   Al though  a  gr eat  v ar i ety   of   or gani sm s  l i v e  i n  the  gut,   the  num ber
of   i m por tant  pathogens  i s,   f or tunatel y ,   sm al l .   The  Enter obacter i aceae  ar e
per haps  the  best
P. 250
k now n  such  pathogens.   Anaer obes  ar e  the  dom i nant  or gani sm s  i n  the
col on  and,   of   thi s  cl ass  Bacter oi des  speci es  ar e  the  i m por tant  pathogens.
Fi nal l y ,   str eptococci ,   especi al l y   enter ococci ,   can  be  pr om i nent  pathogens
(they   ar e  al so  i m por tant  because  of   thei r   r esi stance  to  a  num ber   of
com m onl y   used  anti bi oti cs,   such  as  cephal ospor i ns).

3.   Besi des  obstr ucti on,   i schem i a,   and  i njur y   i nv ol v i ng  the  gut  (and  i ts
outpouchi ngs),   w hat  ar e  the  other   causes  of   per i toni ti s?

Pr eex i sti ng  asci ti c  f l ui d,   especi al l y   that  due  to  hepati c  ci r r hosi s,   can
becom e  i nf ected  and  cause  per i toni ti s.   Sal pi ngi ti s  and  endom etr i ti s  can
l ead  to  per i toni ti s  thr ough  di r ect  ex tensi on  of   the  i nf ecti on  out  of   the
open  abdom i nal   osti um   of   the  tube.   Pr i m ar y   per i toni ti s  i s  an  unusual   f or m
of   bacter i al   per i toni ti s  that  has  no  cl ear   pr edi sposi ng  f actor s;  i t  m ost
of ten  af f ects  chi l dr en.   Fi nal l y ,   ther e  ar e  noni nf ecti ous  causes  of
per i toni ti s,   i ncl udi ng  bl eedi ng  i nto  the  per i toneum ,   w hi ch  can  cause  pai n
and  l ow ­gr ade  f ev er s,   pl us  the  r ar e  f am i l i al   Medi ter r anean  f ev er .

4.   What  ar e  sev er al   ex am pl es  of   ex tr aper i toneal   di seases  that  can  pr esent


w i th  abdom i nal   pai n  as  a  pr om i nent  sy m ptom ?

Low er   l obe  pneum oni a  can  be  a  sour ce  of   consi der abl e  abdom i nal   pai n
and  tender ness.   N eur i ti c  pai n  r esul ti ng  f r om   a  v ar i ety   of   causes
(i nf ecti ous  causes  i ncl ude  her pes  zoster ,   Ly m e  di sease,   and  tabes
dor sal i s)  can  pr oduce  sev er e  abdom i nal   pai n,   w hi ch  i s  conv i nci ng  enough
at  ti m es  to  pr om pt  per f or m ance  of   an  ex pl or ator y   l apar otom y .

Case
A  24­y ear ­ol d  w om an  com es  to  the  em er gency   r oom   because  of   a  4­day
hi stor y   of   abdom i nal   pai n,   w hi ch  she  descr i bes  as  a  shar p,   pr ogr essi v el y
sev er e  pai n  i n  the  r i ght  l ow er   chest  and  upper   abdom en  that  i s  ex acer bated  by
tak i ng  a  deep  br eath,   w al k i ng,   or   si tti ng  er ect.   She  f eel s  nauseated,   but  has
not  v om i ted.   At  hom e  she  has  had  f ev er s  as  hi gh  as  38°C  (100. 4°F),   but  no
r i gor s.   She  has  had  no  pr ev i ous  si m i l ar   epi sodes  and  has  nev er   under gone
abdom i nal   sur ger y .   She  deni es  cough  or   dy spnea,   f atty   f ood  i ntol er ance,
jaundi ce  or   dar k   ur i ne,   dy sur i a,   or   ur i nar y   f r equency .   She  has  nev er   been
pr egnant;  her   l ast  m enstr ual   per i od  began  1  w eek   ago  and  i s  now   endi ng.   Her
past  m edi cal   hi stor y   i s  unr em ar k abl e;  her   onl y   m edi cati on  i s  an  or al
contr acepti v e.   She  dr i nk s  soci al l y   on  w eek ends,   but  does  not  use  tobacco.   Her
f am i l y   hi stor y   i s  notabl e  i n  that  her   m other   had  a  chol ecy stectom y   at  34
y ear s.
On  phy si cal   ex am i nati on,   she  i s  f ound  to  be  a  m i l dl y   obese  y oung  w om an  w ho
i s  i n  m oder ate  di str ess  and  l y i ng  cur l ed  up  on  her   r i ght  si de.   Her   tem per atur e
i s  37. 8°C  (100. 04°F),   bl ood  pr essur e  i s  96/60  m m   Hg,   and  pul se  i s  110
beats  per   m i nute.   Ex am i nati on  of   her   head  and  neck   y i el d  unr em ar k abl e
f i ndi ngs;  speci f i cal l y ,   ther e  i s  no  scl er al   i cter us  or   cer v i cal   adenopathy .   Her
chest  i s  cl ear   to  auscul tati on  and  per cussi on,   al though  she  i s  unabl e  to  tak e  a
deep  br eath  because  of   the  pai n  i n  her   r i ght  l ow er   chest.   She  has  hy poacti v e
bow el   sounds  and  ex hi bi ts  substanti al   tender ness  i n  the  r i ght  upper   quadr ant
associ ated  w i th  a  posi ti v e  Mur phy 's  si gn  (an  i nabi l i ty   to  tak e  a  deep  br eath
dur i ng  deep  pal pati on  of   the  r i ght  upper   quadr ant).   The  edge  of   her   l i v er   i s
not  pal pabl e
P. 251
and  the  span,   by   per cussi on,   i s  nor m al .   N o  m asses  or   tender ness  ar e  f ound
el sew her e  i n  the  abdom en,   and  the  spl een  i s  not  pal pabl e.   Rectal   ex am i nati on
r ev eal s  no  tender ness  and  the  stool   i s  guai ac  negati v e.   Her   sk i n  and
ex tr em i ti es  appear   nor m al .

She  has  a  w hi te  bl ood  cel l   count  of   10, 500/m m 3   w i th  85%  segm ented
neutr ophi l s  and  7%  band  f or m s,   a  hem atocr i t  of   39%,   and  a  pl atel et  count  of
216, 000/m m 3 .   Ser um   el ectr ol y te  and  cr eati ni ne  v al ues  ar e  nor m al .   The
aspar tate  am i notr ansf er ase  (AST)  l ev el   i s  el ev ated  at  56  U /m L  (nor m al ,   < 30
IU /m L),   but  her   ser um   bi l i r ubi n,   al k al i ne  phosphatase,   and  am y l ase  l ev el s  ar e
nor m al .   U r i nal y si s  i s  notabl e  f or   20  to  50  w hi te  bl ood  cel l s,   20  to  50  r ed  bl ood
cel l s,   m any   bacter i a,   and  m any   epi thel i al   cel l s  per   hi gh­pow er   f i el d.   She  i s
thought  to  hav e  acute  chol ecy sti ti s,   but  an  ul tr asound  scan  of   the  l i v er ,   bi l i ar y
ducts,   and  pancr eas  i s  negati v e.

1.   What  ar e  the  v ar i ous  w ay s  f or   y ou  to  pr oceed  at  thi s  poi nt?
2.   What  i s  the  m ost  l i k el y   di agnosi s  based  on  the  f i ndi ngs  f r om   y our   f ur ther
i nv esti gati ons?
3.   What  pathogens  can  cause  sal pi ngi ti s  w i th  per i hepati ti s?
4.   What  noni nv asi v e  tests  ar e  hel pf ul   f or   conf i r m i ng  a  speci f i c  cause  of
sal pi ngi ti s  w i th  per i hepati ti s?

Case Discussion
1.   What  ar e  the  v ar i ous  w ay s  f or   y ou  to  pr oceed  at  thi s  poi nt?

To  el uci date  the  natur e  of   thi s  pati ent's  di sor der ,   y ou  deci de  to  pr oceed
w i th  the  f ol l ow i ng:  r adi onucl i de  bi l i ar y   (l i dof eni n)  scanni ng,   a  chest
r adi ogr aphi c  study ,   r epeat  ur i nal y si s  and  cul tur e,   ser ol ogi c  tests  f or
hepati ti s  A,   B,   and  C,   ser um   β­hum an  chor i oni c  gonadotr opi n  (hCG)
m easur em ent,   and  sex ual   hi stor y   and  pel v i c  ex am i nati on.

Addi ti onal   case  detai l s:  Because  i t  i s  6:00  p. m . ,   the  nucl ear   m edi ci ne
f aci l i ti es  ar e  not  av ai l abl e,   and  ther ef or e  i t  i s  not  possi bl e  to  hav e  a
r adi onucl i de  bi l i ar y   scan  per f or m ed.   The  chest  r adi ogr aphi c  study   i s
nor m al .   Repeat  ur i nal y si s  on  a  catheter i zed  speci m en  y i el ds  nor m al
f i ndi ngs,   but  the  cul tur e  r esul ts  ar e  pendi ng,   as  ar e  the  r esul ts  of
ser ol ogi c  tests  f or   hepati ti s  A,   B,   and  C.   The  ser um   β­hCG  l ev el   show s
that  she  i s  not  pr egnant.   Sex ual   hi stor y   and  pel v i c  ex am i nati on  r ev eal
she  i s  sex ual l y   acti v e  w i th  a  new   par tner   i n  the  l ast  m onth.   Because  she
uses  an  or al   contr acepti v e,   her   par tner   does  not  use  condom s.   She  has
had  geni tal   w ar ts  and  y east  i nf ecti ons  i n  the  past,   but  has  no  k now n
hi stor y   of   other   sex ual l y   tr ansm i tted  di seases.   She  does  not  use
i ntr av enous  dr ugs,   has  nev er   r ecei v ed  a  bl ood  tr ansf usi on,   and  has  had
no  occupati onal   ex posur e  to  bl ood.   On  pel v i c  ex am i nati on,   her   ex ter nal
geni tal i a  ar e  f ound  to  be  nor m al .   Ther e  i s  a  sm al l   am ount  of   dar k   bl ood
f r om   the  cer v i cal   os,   and  m i l d  tender ness  w i th  cer v i cal   m oti on  and
pal pati on  of   the  r i ght  adnex a.   The  si ze  of   the  uter us  i s  nor m al   and  ther e
ar e  no  adnex al   m asses.

2.   What  i s  the  m ost  l i k el y   di agnosi s  based  on  the  f i ndi ngs  f r om   y our   f ur ther
i nv esti gati ons?

Al l   these  tests  i nv esti gate  i m por tant  causes  of   acute  abdom i nal   pai n  and
f ev er .   Inter pr etati on  of   the  r esul ts  al l ow s  a  f ai r l y   conf i dent  di agnosi s  to
be  m ade.   The
P. 252
i ni ti al   concer n  w as  acute  chol ecy sti ti s,   but  the  nor m al   ul tr asound  f i ndi ngs
m ak e  thi s  di agnosi s  unl i k el y ,   al though  they   do  not  com pl etel y   r ul e  i t  out
because  a  si ngl e  stone  m ay   be  l odged  i n  the  di stal   com m on  duct  and  be
m i ssed  on  ul tr asound  scanni ng.   Ar gui ng  agai nst  chol ecy sti ti s  ar e  her   age
and  l ack   of   pr ev i ous  pr egnanci es.   Another   possi bi l i ty   i s  r i ght  l ow er   l obe
pneum oni a.   In  m ost  cases  of   pneum oni a,   r espi r ator y   sy m ptom s  ar e  the
chi ef   com pl ai nt,   but  l ow er   l obe  pneum oni a,   by   i r r i tati ng  the  par i etal
pl eur a  ov er l y i ng  the  di aphr agm ,   can  assum e  the  char acter i sti cs  of   an
abdom i nal   pr esentati on.   In  thi s  case,   the  nor m al   chest  r adi ogr aphi c
f i ndi ngs  and  l ack   of   pul m onar y   sy m ptom s  m ak e  thi s  di agnosi s  v er y
unl i k el y .

Pati ents  w i th  py el onephr i ti s  can  ex per i ence  pai n  anter i or l y   (i n  the  upper
and  m i d­abdom en)  as  w el l   as  the  cl assi c  costov er tebr al   angl e  tender ness
i n  the  back .   How ev er ,   the  l ack   of   l ow er   ur i nar y   tr act  sy m ptom s  (dy sur i a,
ur i nar y   f r equency ,   and  supr apubi c  pai n)  i n  thi s  pati ent  i s  not  concl usi v e
ev i dence  agai nst  thi s  di agnosi s  because  these  sy m ptom s  ar e  f r equentl y
m i l d  or   ev en  absent  i n  pati ents  w i th  upper   U TIs.   The  i ni ti al   ur i nal y si s
r ev eal ed  m any   w hi te  bl ood  cel l s,   a  f i ndi ng  that  at  f i r st  bl ush  seem s  to
conf i r m   the  di agnosi s  of   py el onephr i ti s.   How ev er ,   ther e  ar e  m any
epi thel i al   cel l s  as  w el l ,   w hi ch  m ak es  i t  i m possi bl e  to  tel l   w hether   the
w hi te  bl ood  cel l s  cam e  f r om   the  ur i nar y   or   the  r epr oducti v e  tr act.   A
catheter i zed  ur i ne  speci m en  answ er s  thi s  questi on,   and  the  subsequent
nor m al   ur i nal y si s  f i ndi ngs  al m ost  r ul e  out  the  possi bi l i ty   of
py el onephr i ti s.   Rar el y ,   i f   ther e  i s  i nf ecti on  causi ng  an  obstr ucti on  of   the
ur eter ,   the  ur i nal y si s  r esul ts  can  be  nor m al .   Such  pati ents  ar e  usual l y
sev er el y   i l l ,   how ev er ,   r ender i ng  thi s  di agnosi s  v er y   unl i k el y   i n  thi s  case.

Al though  acute  v i r al   hepati ti s  can  cause  pr onounced  r i ght  upper   quadr ant
pai n  and  tender ness,   i t  i s  a  v er y   unl i k el y   di agnosi s  i n  thi s  pati ent.   At  the
onset  of   sy m ptom s,   the  tr ansam i nase  l ev el s  i n  the  setti ng  of   v i r al
hepati ti s  ar e  m ar k edl y   el ev ated—usual l y   ex ceedi ng  10  ti m es  nor m al .
The  m i nor   el ev ati on  i n  the  AST  l ev el   i n  thi s  pati ent  w oul d  be  v er y
aty pi cal   of   acute  v i r al   hepati ti s.

The  possi bi l i ty   of   a  r uptur ed  ectopi c  pr egnancy   shoul d  al w ay s  be


consi der ed  i n  a  y oung  w om an  w i th  abdom i nal   pai n  and  v agi nal   bl eedi ng,
but  the  ser um   β­hCG  m easur em ent  r ul es  out  thi s  possi bi l i ty .

Acute  sal pi ngi ti s  (i nf ecti on  of   the  f al l opi an  tubes  or   pel v i c  i nf l am m ator y
di sease)  can  be  m ani f ested  by   r i ght  upper   quadr ant  pai n.   Thi s  sy m ptom   i s
thought  to  ar i se  as  a  r esul t  of   secr eti ons  f r om   the  i nf ected  tube  l eak i ng
i nto  the  per i toneum   and  tr av el i ng  up  the  r i ght  per i col i c  gutter   to  the  r i ght
upper   quadr ant.   Thi s  can  pr oduce  i nf ecti on  of   the  hepati c  capsul e,   ter m ed
per i hepati ti s  (or   Fi tz­Hugh–Cur ti s  sy ndr om e).   Sur pr i si ngl y ,   the
sy m ptom s  of   per i hepati ti s  ar e  f r equentl y   m uch  m or e  pr om i nent  than
those  stem m i ng  f r om   the  or i gi nal   f ocus  of   i nf ecti on  i n  the  tube.
Ther ef or e,   these  pati ents  ar e  adm i tted  f r equentl y   and  som eti m es  tak en  to
sur ger y   f or   tr eatm ent  of   a  pr esum ed  chol ecy sti ti s.   Ther e  ar e  no
pathognom oni c  l abor ator y   or   i m agi ng  f i ndi ngs  that  can  conf i r m   thi s
di agnosi s;  thi s  r equi r es  l apar oscopy .   How ev er ,   acute  sal pi ngi ti s  shoul d  be
ser i ousl y   consi der ed  i n  thi s  pati ent—a  sex ual l y   acti v e  w om an  w i th  r i ght
upper   quadr ant  pai n,   f ev er s,   and  no  si gns  of   chol ecy sti ti s.   Wi th  the
addi ti onal   f actor   that  she  has  a  new   sex ual   par tner   coupl ed  w i th  the
f i ndi ng  of   r i ght  adnex al   tender ness,   sal pi ngi ti s  w i th  per i hepati ti s  becom es
the  m ost  l i k el y   di agnosi s.

P. 253
3.   What  pathogens  can  cause  sal pi ngi ti s  w i th  per i hepati ti s?

N ei sser i a  gonor r hea  i s  the  cl assi c  cause  of   thi s  sy ndr om e.   In  Fi tz­Hugh's
or i gi nal   descr i pti on,   Gr am 's  stai ni ng  of   the  f l ui d  f r om   the  hepati c  capsul e
show ed  gr am ­negati v e  di pl ococci .   Si nce  then,   as  i n  the  case  of   acute
ur ethr i ti s,   i t  has  becom e  cl ear   that  Chl am y di a  tr achom ati s  i s  a  com m on
cause  of   acute  sal pi ngi ti s  w i th  per i hepati ti s.

4.   What  noni nv asi v e  tests  ar e  hel pf ul   f or   conf i r m i ng  a  speci f i c  cause  of


sal pi ngi ti s  w i th  per i hepati ti s?

Gr am 's  stai ni ng  of   a  cer v i cal   sm ear ,   al though  r el ati v el y   i nsensi ti v e  (50%)
f or   detecti ng  gonococci ,   i s  speci f i c  enough  (95%)  to  be  used  as  the  basi s
f or   pr esum pti v e  ther apy   i f   r esul ts  ar e  posi ti v e.   A  cer v i cal   cul tur e  f or
gonor r hea  al l ow s  the  detecti on  of   sm ear ­negati v e  cases.   It  i s  possi bl e  to
cul tur e  Chl am y di a,   but  thi s  i s  a  r el ati v el y   ex pensi v e  pr ocedur e  and  the
m eans  of   doi ng  so  ar e  not  av ai l abl e  i n  m any   cl i ni cs  and  sm al l   hospi tal s.
How ev er ,   a  num ber   of   anti gen  detecti on  sy stem s  (usi ng,   f or   ex am pl e,   an
ELISA)  hav e  been  dev el oped  and  m ar k eted.   These  hav e  an  acceptabl e
sensi ti v i ty   and  speci f i ci ty ,   w i th  r esul ts  av ai l abl e  i n  24  hour s  or   l ess.

Suggested Readings
Fi tz­Hugh  T.   Acute  gonococci c  per i toni ti s  of   the  r i ght  upper   quadr ant  i n
w om en.   JAMA  1934;102:2094.
Katzm an  DK,   Fr i edm an  IM,   McDonal d  CA,   et  al .   Chl am y di a  tr achom ati s  Fi tz­
Hugh–Cur ti s  sy ndr om e  w i thout  sal pi ngi ti s  i n  f em al e  adol escents.   Am   J
Di s  Chi l d  1988;142:996.

Mul l er ­Schoop  JW,   Wang  SP,   Munzi nger   J,   et  al .   Chl am y di a  tr achom ati s  as
possi bl e  cause  of   per i toni ti s  and  per i hepati ti s  i n  y oung  w om en.   BMJ
1978;1:1022.

Shol es  D,   Ster gachi s  A,   Hei dr i ch  FE,   et  al .   Pr ev enti on  of   pel v i c
i nf l am m ator y   di sease  by   scr eeni ng  f or   cer v i cal   chl am y di al   i nf ecti on.   N   Engl
J  Med  1996;334:1362.

Soper   DE,   Br ock w el l   N J,   Dal ton  HP,   et  al .   Obser v ati ons  concer ni ng  the
m i cr obi al   eti ol ogy   of   acute  sal pi ngi ti s.   Am   J  Obstet  Gy necol
1994;170:1008.

Wood  JJ,   Bol ton  JP,   Cannon  SR,   et  al .   Bi l i ar y ­ty pe  pai n  as  a  m ani f estati on
of   geni tal   tr act  i nf ecti on:  the  Cur ti s–Fi tz­Hugh  sy ndr om e.   Br   J  Sur g
1982;69:251.

Central Nervous System Infection
1.   What  pr i nci pl es  ar e  i m por tant  i n  sel ecti ng  an  anti m i cr obi al   r egi m en  to
tr eat  a  CN S  i nf ecti on?

2.   How   do  cer ebr ospi nal   f l ui d  (CSF)  f i ndi ngs  such  as  the  pr otei n  and  gl ucose
l ev el s,   the  w hi te  bl ood  cel l   count,   and  di f f er enti al   hel p  deter m i ne  the
di f f er enti al   di agnosi s  of   a  CN S  i nf ecti on?

3.   What  ar e  the  m ost  com m on  pathogens  causi ng  bacter i al   m eni ngi ti s,   and
how   does  the  pr ev al ence  of   the  bacter i al   pathogens  that  cause  m eni ngi ti s
v ar y ,   dependi ng  on  the  age  of   the  host?

P. 254

Discussion
1.   What  pr i nci pl es  ar e  i m por tant  i n  sel ecti ng  an  anti m i cr obi al   r egi m en  to
tr eat  a  CN S  i nf ecti on?

The  bl ood–br ai n  bar r i er   f uncti ons  to  hel p  pr ev ent  the  entr y   of
ci r cul ati ng  pathogens  i nto  the  CN S.   U nf or tunatel y ,   how ev er ,   i t  al so  has
the  ef f ect  of   decr easi ng  anti bi oti c  penetr ati on  i nto  the  CSF.
Cephal ospor i ns,   peni ci l l i ns,   chl or am pheni col ,   and  TMP­SMX  ar e  the
com m onl y   used  anti bi oti cs  that  dem onstr ate  good  CSF  penetr ati on.   In
contr ast,   the  am i nogl y cosi des  hav e  ex tr em el y   poor   CSF  penetr ati on,   and
an  i nf ecti on  r equi r i ng  am i nogl y cosi de  ther apy   m ust  usual l y   be  m anaged
w i th  the  i ntr athecal   adm i ni str ati on  of   these  anti bi oti cs.   Other   dr ugs,   such
as  v ancom y ci n,   ex hi bi t  i nter m edi ate  CSF  penetr ati on,   and  thei r   ef f i cacy
depends  on  the  pr esence  of   i nf l am ed  m eni nges  to  per m i t  a  ther apeuti c
l ev el   of   anti bi oti c  to  be  r eached.

Another   i m por tant  pr i nci pl e  i s  to  choose  an  em pi r i c  anti bi oti c  r egi m en
that  cov er s  the  m ost  l i k el y   pathogens.   Thi s  choi ce  ther ef or e  depends  on
the  epi dem i ol ogi c  back gr ound  of   the  pati ent  and  on  hi s  or   her   r ecent
ex posur e  hi stor y .   Af ter   the  pathogen  has  been  i denti f i ed  and  the  dr ug
suscepti bi l i ty   deter m i ned,   ther apy   can  be  m or e  speci f i cal l y   tai l or ed.
Fi nal l y ,   as  w i th  any   new   dr ug  r egi m en,   the  pati ent's  hi stor y   of   dr ug
al l er gy   shoul d  be  car ef ul l y   r ev i ew ed.

2.   How   do  CSF  f i ndi ngs  such  as  the  pr otei n  and  gl ucose  l ev el s,   the  w hi te
bl ood  cel l   count,   and  di f f er enti al   hel p  deter m i ne  the  di f f er enti al   di agnosi s
of   a  CN S  i nf ecti on?

In  adul ts,   the  nor m al   r ange  of   the  CSF  gl ucose  l ev el   i s  45  to  80  m g/dL.   A
gl ucose  l ev el   of   l ess  than  30  m g/dL  suggests  bacter i al ,   f ungal ,   or
tuber cul ous  m eni ngi ti s.   An  el ev ated  l ev el   m ay   be  seen  i n  the  setti ng  of
di abetes  m el l i tus.   A  CSF  pr otei n  l ev el   gr eater   than  150  m g/dL  suggests
bacter i al   m eni ngi ti s,   and  an  ex tr em el y   hi gh  pr otei n  l ev el   (> 350  m g/dL)
suggests  a  com pl ete  bl ock   of   the  spi nal   canal ,   as  seen  i n  cer tai n  cases  of
epi dur al   abscesses  or   tum or s.   The  nor m al   r ange  f or   the  l um bar   CSF
pr otei n  l ev el   i n  adul ts  i s  9  to  58  m g/dL.   A  w hi te  bl ood  cel l   count  gr eater
than  1, 200/m m 3   suggests  bacter i al   m eni ngi ti s.   How ev er ,   a  l esser   count
does  not  necessar i l y   i m pl y   v i r al   i nf ecti on  because  bacter i al   m eni ngi ti s  i s
al so  f r equentl y   associ ated  w i th  thi s  f i ndi ng.   N eutr ophi l   pr edom i nance
(> 50%)  al so  suggests  bacter i al   m eni ngi ti s,   al though  ther e  i s  consi der abl e
ov er l ap  w i th  other   ty pes  of   m eni ngi ti s  i n  thi s  r egar d.   Ly m phocy te
pr edom i nance  m ay   be  seen  i n  the  contex t  of   tuber cul ous,   v i r al ,   f ungal ,
par ti al l y   tr eated  bacter i al ,   or   asepti c  m eni ngi ti s.

3.   What  ar e  the  m ost  com m on  pathogens  causi ng  bacter i al   m eni ngi ti s,   and
how   does  the  pr ev al ence  of   the  bacter i al   pathogens  that  cause  m eni ngi ti s
v ar y ,   dependi ng  on  the  age  of   the  host?

The  f our   m ost  com m on  pathogens  causi ng  m eni ngi ti s  f or   al l   age­gr oups
ar e  Str eptococcus  pneum oni ae,   Haem ophi l us  i nf l uenzae,   N ei sser i a
m eni ngi ti di s,   and  E.   col i .   How ev er ,   ther e  i s  consi der abl e  v ar i ati on  i n  the
pr ev al ence  of   these  v ar i ous  pathogens  am ong  the  di f f er ent  age­gr oups.
The  hi ghest  attack   r ate  of   bacter i al   m eni ngi ti s  occur s  i n  the  v er y   y oung
and  v er y   ol d.   Low er   attack   r ates  ar e  seen  i n  y oung  to  m i ddl e­aged  adul ts.
In  the  U ni ted  States,   i nf ants  up  to
P. 255
1  m onth  of   age  m ost  com m onl y   acqui r e  gr oup  B  str eptococcus,   E.   col i ,
and  Li ster i a  m eni ngi ti s;  chi l dr en  1  m onth  to  5  y ear s  of   age  pr edom i nantl y
acqui r e  H.   i nf l uenzae  m eni ngi ti s  (up  to  70%  of   the  cases).   For ty   per cent
of   the  pati ents  5  to  29  y ear s  of   age  acqui r e  N .   m eni ngi ti di s  i nf ecti on,   and
S.   pneum oni ae  i s  the  m ost  com m on  m eni ngi ti s  pathogen  i n  pati ents  29
y ear s  of   age  and  ol der .   El der l y   pati ents  ar e  m or e  v ul ner abl e  to  Li ster i a
m onocy togenes,   gr am ­negati v e  baci l l i ,   and  pneum ococcus.

Case
A  79­y ear ­ol d  m an  w ho  i s  a  r esi dent  of   a  nur si ng  hom e  i s  br ought  to  the
em er gency   r oom   by   the  nur si ng  hom e  staf f .   He  had  been  i n  hi s  usual   state  of
heal th  unti l   that  m or ni ng,   w hen  headache,   f ev er ,   and  chi l l s  dev el oped.   He
sl ept  thr ough  br eak f ast,   af ter   w hi ch  hi s  car etak er s  f ound  hi m   to  be  l ethar gi c,
and  thi s  pr om pted  them   to  br i ng  hi m   to  the  hospi tal .   On  i ni ti al   ex am i nati on  he
i s  f ound  to  be  stupor ous.   Hi s  tem per atur e  i s  102. 6°F  (39°C),   and  he  has
pr om i nent  nuchal   r i gi di ty .   Funduscopi c  ex am i nati on  r ev eal s  no  ev i dence  of
papi l l edem a.   A  gr ade  2/6  sy stol i c  ejecti on  m ur m ur   i s  f ound  on  car di ac
ex am i nati on.   Pul m onar y   auscul tati on  r ev eal s  the  pr esence  of   bi basi l ar   f i ne
cr ack l es.   An  i ndw el l i ng  Fol ey   catheter   i s  i n  pl ace,   w hi ch  the  nur si ng  hom e
staf f   ex pl ai ns  he  has  r equi r ed  f or   the  past  18  m onths  because  of   ur i nar y
i nconti nence.   A  past  m edi cal   hi stor y   i s  notabl e  f or   tw o  epi sodes  of   U TIs,   both
occur r i ng  af ter   the  i nser ti on  of   the  Fol ey   catheter ,   and  m i l d  chr oni c  i nter sti ti al
l ung  di sease.
A  CT  scan  of   the  head  r ev eal s  no  ev i dence  of   i ncr eased  i ntr acr ani al   pr essur e,
no  shi f t  or   m ass  ef f ect,   and  no  i ntr acr ani al   bl eedi ng,   but  atr ophi c  changes
consi stent  w i th  age.   A  l um bar   punctur e  i s  per f or m ed,   bl ood  and  ur i ne  cul tur es
ar e  obtai ned,   and  appr opr i ate  ther apy   i s  begun.

1.   What  i s  the  m ost  l i k el y   pathogenesi s  of   thi s  m an's  m eni ngi ti s?


2.   What  aspects  of   the  em er gency   r oom   m anagem ent  shoul d  hav e  been
di f f er ent  i n  thi s  case?
3.   What  em pi r i c  i ntr av enous  anti bi oti c  ther apy   w oul d  be  m ost  appr opr i ate  to
tr eat  the  bacter i al   m eni ngi ti s  i n  a  pati ent  of   thi s  age,   and  how   l ong
shoul d  he  be  tr eated?
4.   What  phy si cal   f i ndi ngs  coul d  poi nt  to  an  anatom i c  sour ce  of   bacter i al
m eni ngi ti s?
5.   What  CSF  f i ndi ngs  w oul d  be  ex pected  i f   thi s  pati ent  has  bacter i al
m eni ngi ti s?
6.   If   the  Gr am 's  stai ni ng  of   the  CSF  and  the  cul tur es  had  y i el ded  no
or gani sm s  i n  thi s  pati ent,   w hat  shoul d  y ou  suspect?
7.   If   thi s  pati ent  had  ex per i enced  the  gr adual   onset  of   f ev er s,   headache,
and  nuchal   r i gi di ty ,   w hat  other   possi bl e  di agnoses  m i ght  y ou  hav e
enter tai ned?
8.   Shoul d  the  pati ent  be  tr eated  w i th  dex am ethasone?

Case Discussion
1.   What  i s  the  m ost  l i k el y   pathogenesi s  of   thi s  m an's  m eni ngi ti s?

Sev er al   possi bl e  scenar i os  coul d  ex pl ai n  the  pr esence  of   bacter i a  i n


nor m al l y   ster i l e  CSF,   despi te  an  i ntact  bl ood–br ai n  bar r i er .   These
i ncl ude  any   of   a  num ber   of
P. 256
pr ocesses  l eadi ng  to  the  dev el opm ent  of   bacter em i a  and  m eni ngeal
seedi ng.   One  of   the  m or e  com m on  sour ces  of   i nf ecti on  i s  nasophar y ngeal
col oni zati on  by   bacter i a,   w hi ch  i s  then  f ol l ow ed  by   si nusi ti s,   l ocal
i nv asi on,   bacter em i a,   and  m eni ngeal   seedi ng.   Another   pathogeni c
m echani sm   i s  tr aum a  (such  as  an  occul t  sk ul l   f r actur e),   l eadi ng  to  a
br each  i n  the  bl ood–br ai n  bar r i er   and  the  entr y   of   sk i n  f l or a.

The  m ost  l i k el y   scenar i o  i n  thi s  pati ent  i s  that  a  pl ugged  Fol ey   catheter
l ed  to  the  r ef l ux   of   bl adder   contents  i nto  the  ur eter s  up  to  the  k i dney s,
w i th  consequent  seedi ng  of   the  bl oodstr eam   by   ur i nar y   pathogens.
Bacter em i a  can  then  l ead  to  m eni ngi ti s,   especi al l y   i n  the
i m m unocom pr om i sed  or   el der l y   host.   Al though  the  m echani sm   of   bacter i al
tr anspor t  acr oss  a  pr esum abl y   i ntact  bl ood–br ai n  bar r i er   i s  l ar gel y
unk now n,   the  f i ndi ngs  f r om   som e  studi es  hav e  suggested  that  a  hi gh
concentr ati on  of   bacter i a  i n  the  bl oodstr eam ,   and  the  pr esence  of
bacter i al   v i r ul ence  f actor s  such  as  anti phagocy ti c  pol y sacchar i de
capsul es,   the  S­f i m br i ae  of   E.   col i ,   or   other   com ponents  of   bacter i al   cel l
w al l s,   m ay   f aci l i tate  thi s  pr ocess.

2.   What  aspects  of   the  em er gency   r oom   m anagem ent  shoul d  hav e  been
di f f er ent  i n  thi s  case?

The  head  CT  scan  w as  unnecessar y   because  the  f unduscopi c  and  nonf ocal
neur ol ogi c  f i ndi ngs  w er e  suf f i ci ent  to  r ul e  out  a  si gni f i cant  i ntr acr ani al
m ass  ef f ect.   Other w i se,   the  m anagem ent  of   thi s  pati ent  w as  cor r ect,   and
i t  i l l ustr ated  a  num ber   of   i m por tant  concepts.   It  i s  i m por tant  to  attem pt
to  i denti f y   the  pathogen  i n  a  pati ent  w i th  m eni ngi ti s  bef or e  the  i ni ti ati on
of   anti bi oti c  ther apy .   Lum bar   punctur e  shoul d  be  del ay ed  or   def er r ed  onl y
i n  the  f ol l ow i ng  tw o  i nstances.   Fi r st,   l um bar   punctur e  shoul d  not  be
per f or m ed  i f   a  m i nor   del ay   i n  ther apy   coul d  be  hazar dous,   as  i n  pati ents
i n  bacter i al   shock   or   those  w ho  f ace  a  hi gh  r i sk   of   bacter i al   shock
because  of   the  r api d  onset  of   pur pur a  or   because  of   l ow   bl ood  pr essur e.
It  al so  shoul d  not  be  per f or m ed  w hen  ther e  i s  a  possi bl e  danger   of   uncal
her ni ati on,   as  i n  the  ev ent  of   r api dl y   dev el opi ng  com a,   f ocal   neur ol ogi c
si gns,   conv ul si ons,   or   papi l l edem a.   Other w i se,   appr opr i ate  m anagem ent
consi sts  of   qui ck l y   ex cl udi ng  papi l l edem a,   f ocal   neur ol ogi c  si gns,   shock ,
and  pur pur a,   f ol l ow ed  by   pr om pt  l um bar   punctur e  and  the  subsequent
adm i ni str ati on  of   appr opr i ate  anti bi oti cs.

The  r espi r ator y   i sol ati on  of   pati ents  w i th  suspected  m eni ngi ti s  i s
appr opr i ate  onl y   w hen  ther e  i s  a  str ong  suspi ci on  of   N .   m eni ngi ti di s,
My cobacter i um   tuber cul osi s,   or   H.   i nf l uenzae  ty pe  b  (i n  pedi atr i c
pati ents).   The  cl ose  contacts  of   pati ents  w i th  N .   m eni ngi ti di s  (such  as  the
per son  per f or m i ng  an  i ntubati on)  shoul d  r ecei v e  pr ophy l acti c  r i f am pi n
tr eatm ent  (10  m g/k g  or al l y   tw i ce  a  day   f or   2  day s,   to  a  m ax i m um   of   600
m g  tw i ce  a  day ).

3.   What  em pi r i c  i ntr av enous  anti bi oti c  ther apy   w oul d  be  m ost  appr opr i ate  to
tr eat  the  bacter i al   m eni ngi ti s  i n  a  pati ent  of   thi s  age,   and  how   l ong
shoul d  he  be  tr eated?

The  appr oaches  to  age­speci f i c  em pi r i c  anti bi oti c  ther apy   f or   bacter i al
m eni ngi ti s  ar e  based  on  k now l edge  of   the  m ost  com m on  pathogens  that
af f ect  each  gr oup,   as  al r eady   outl i ned.   For   i nf ants  y ounger   than  1  m onth,
a  com bi nati on  of   am pi ci l l i n  and  gentam i ci n  or   am pi ci l l i n  and  cef otax i m e
i s  a  r easonabl e  em pi r i c  anti bi oti c  choi ce.   The  com bi nati on  of   v ancom y ci n
pl us  a  thi r d­gener ati on  cephal ospor i n  i s  appr opr i ate  f or   i nf ants  1  to  24
m onths  of   age.   For   chi l dr en  aged  2  y ear s  or   ol der   and  adul ts  y ounger
than  50,   v ancom y ci n  and  a  thi r d­gener ati on  cephal ospor i n  ar e
P. 257
appr opr i ate.   Am pi ci l l i n  shoul d  be  added  to  thi s  r egi m en  f or   those  ol der
than  50  y ear s  to  ensur e  cov er age  f or   L.   m onocy togenes.   Once  the
pathogen  and  i ts  suscepti bi l i ty   patter n  ar e  k now n,   ther apy   can  be
i ndi v i dual i zed.   The  dur ati on  of   anti bi oti c  ther apy   f or   bacter i al   m eni ngi ti s
i s  sti l l   l ar gel y   based  on  tr adi ti on:  10  to  14  day s  f or   acute  bacter i al
m eni ngi ti s  caused  by   any   of   the  thr ee  m ajor   m eni ngeal   pathogens,   and
appr ox i m atel y   3  w eek s  f or   gr am ­negati v e  baci l l ar y   m eni ngi ti s.   The
dosages  of   the  anti bi oti cs  f or   the  tr eatm ent  of   m eni ngi ti s  ar e  usual l y
hi gher   than  those  used  f or   other   i nf ecti ons  to  ensur e  adequate
bacter i ci dal   acti v i ty   i n  the  CSF.   For   ex am pl e,   i ntr av enous  peni ci l l i n  G
shoul d  be  gi v en  ev er y   4  hour s  to  a  total   dai l y   dose  of   20  to  24  m i l l i on
uni ts,   and  2  g  of   cef tr i ax one  shoul d  be  gi v en  ev er y   12  hour s.   The
am i nogl y cosi des,   w hi ch  do  not  adequatel y   tr av er se  the  bl ood–br ai n
bar r i er ,   m ust  be  adm i ni ster ed  i ntr athecal l y   as  w el l   as  i ntr av enousl y   w hen
i ndi cated,   as  i n  the  ev ent  of   m eni ngi ti s  due  to  a  hi ghl y   r esi stant  gr am ­
negati v e  or gani sm .

4.   What  phy si cal   f i ndi ngs  coul d  poi nt  to  an  anatom i c  sour ce  of   bacter i al
m eni ngi ti s?

Im por tant  phy si cal   f i ndi ngs  that  ar e  cl ues  to  an  anatom i c  sour ce  of
bacter i al   m eni ngi ti s  i ncl ude  oti ti s  m edi a,   si nusi ti s,   sk ul l   f r actur e,   or
other   ev i dence  of   cr ani al   tr aum a  such  as  CSF  l eak i ng  f r om   the  ex ter nal
audi tor y   m eatus,   neur osur gi cal   scar s,   or   the  pr esence  of   a
v entr i cul ostom y   shunt.

Most  cases  of   m eni ngi ti s  r esul t  f r om   the  attachm ent  of   bacter i a  to


epi thel i al   cel l s  of   the  nasophar y ngeal   and  or ophar y ngeal   m ucosa,
f ol l ow ed  by   tr ansgr essi on  of   the  m ucosal   bar r i er .   These  ev ents  ar e  al so
associ ated  w i th  the  dev el opm ent  of   oti ti s  m edi a  and  si nusi ti s.   Any
anatom i c  br each  of   the  bl ood–br ai n  bar r i er ,   ei ther   thr ough  tr aum a  or
neur osur ger y ,   can  i ntr oduce  bacter i a  i nto  the  CN S.   Meni ngi ti s  dev el ops  i n
up  to  30%  of   pati ents  w ho  hav e  a  v entr i cul oatr i al   or   v entr i cul oper i toneal
shunt.   Other   per ti nent  phy si cal   f i ndi ngs  i n  a  pati ent  w i th  m eni ngi ti s
i ncl ude  a  der m al   si nus  or   m astoi di ti s.

5.   What  CSF  f i ndi ngs  w oul d  be  ex pected  i f   thi s  pati ent  has  bacter i al
m eni ngi ti s?

The  cl i ni ci an  w oul d  ex pect  to  see  the  f ol l ow i ng  constel l ati on  of   f i ndi ngs:  a
l ow   gl ucose  l ev el ,   a  hi gh  pr otei n  content,   and  an  el ev ated  w hi te  bl ood
cel l   count,   w i th  a  neutr ophi l   pr edom i nance  (see  ear l i er   di scussi on).

6.   If   the  Gr am 's  stai ni ng  of   the  CSF  and  the  cul tur es  had  y i el ded  no
or gani sm s  i n  thi s  pati ent,   w hat  shoul d  y ou  suspect?

The  l ack   of   or gani sm s  on  Gr am 's­stai ned  CSF  speci m ens  does  not  r ul e  out
bacter i al   m eni ngi ti s;  how ev er ,   thi s  test  shoul d  be  per f or m ed  on  a
centr i f uged  sedi m ent  of   CSF.   N egati v e  f i ndi ngs  ar e  encounter ed  i n  10%  to
20%  of   pati ents  w i th  bacter i al   m eni ngi ti s  w hose  CSF  cul tur es  ar e  posi ti v e
f or   or gani sm s.   In  cases  of   par ti al l y   tr eated  bacter i al   m eni ngi ti s,   Gr am 's
stai ni ng  of   the  CSF  m or e  of ten  y i el ds  negati v e  f i ndi ngs.   An  aci d­f ast
sm ear   of   spun  CSF  i s  onl y   r ar el y   posi ti v e  i n  cases  of   tuber cul ous
m eni ngi ti s.

7.   If   thi s  pati ent  had  ex per i enced  the  gr adual   onset  of   f ev er s,   headache,   and
nuchal   r i gi di ty ,   w hat  other   possi bl e  di agnoses  m i ght  y ou  hav e
enter tai ned?

Fungal   m eni ngi ti s,   a  br ai n  abscess,   tuber cul ous  m eni ngi ti s,   and
car ci nom atous  m eni ngi ti s  al l   tend  to  be  r ather   i nsi di ous  i n  onset  and
assum e  a  m or e  chr oni c  cour se  than  that  seen  w i th  acute  bacter i al
m eni ngi ti s.   The  onset  of   tuber cul ous  m eni ngi ti s  m ay   be  occasi onal l y   r api d
i n  an  i m m unocom pr om i sed  host,   but  thi s  w oul d  usual l y
P. 258
occur   onl y   i n  the  cour se  of   m i l i ar y   tuber cul osi s  or   w i th  the  r uptur e  of   a
subependy m al   tuber cl e.   Mor e  com m onl y ,   how ev er ,   pati ents  w i th
tuber cul ous  m eni ngi ti s  hav e  sy m ptom s  f or   m or e  than  2  w eek s.   One  of   the
hal l m ar k s  of   tuber cul ous  m eni ngi ti s  i s  the  dev el opm ent  of   ocul ar   pal si es,
seen  i n  30%  to  70%  of   the  cases.

An  epi dem i ol ogi c  hi stor y   i s  usef ul   i n  di agnosi ng  chr oni c  m eni ngi ti s.   Pr i or
ex posur e  to  tuber cul osi s,   a  hi stor y   of   sk i n  test  posi ti v i ty ,   or   r ecent  tr av el
to  or   r esi dence  i n  ar eas  endem i c  to  Hi stopl asm a  or   Cocci di oi des  i s
i m por tant  i nf or m ati on  to  el i ci t.   Car ci nom atous  m eni ngi ti s  occur s  usual l y
i n  the  setti ng  of   a  k now n  under l y i ng  m al i gnancy .

8.   Shoul d  the  pati ent  be  tr eated  w i th  dex am ethasone?

The  adjuncti v e  use  of   dex am ethasone  i n  the  tr eatm ent  of   bacter i al


m eni ngi ti s  has  been  the  subject  of   cl i ni cal   i nv esti gati on  f or   m any   y ear s.
As  i n  the  case  of   anti bi oti c  sel ecti on,   cur r ent  r ecom m endati ons  about  the
use  of   dex am ethasone  ar e  age  dependent.   Cl i ni cal   tr i al s  best  suppor t  the
use  of   dex am ethasone  i n  chi l dr en  w i th  H.   i nf l uenzae  ty pe  b  m eni ngi ti s
and  i n  adul ts  w i th  k now n  or   suspected  pneum ococcal   m eni ngi ti s.   The  use
of   dex am ethasone  i n  i nf ants  and  chi l dr en  w i th  pneum ococcal   m eni ngi ti s
r em ai ns  contr ov er si al .   Concer ns  hav e  been  r ai sed  that  the  use  of
dex am ethasone  i n  chi l dr en  and  adul ts  w i th  pneum ococcal   m eni ngi ti s  due
to  r esi stant  or gani sm s  m i ght  be  detr i m ental   because  a  r educti on  i n
m eni ngeal   i nf l am m ati on  caused  by   the  dex am ethasone  ther apy   m i ght
com pr om i se  penetr ati on  of   v ancom y ci n  acr oss  the  m eni nges.   Al though
these  concer ns  hav e  been  r ai sed,   cl i ni cal   data  ar e  i nsuf f i ci ent  i n  ei ther
di r ecti on  to  m ak e  a  cl i ni cal l y   v al i dated  r ecom m endati on.

Suggested Readings
de  Gans  J,   v an  de  Beek   D.   Dex am ethasone  i n  adul ts  w i th  bacter i al
m eni ngi ti s.   N   Engl   J  Med  2002;347:1549–1556.

Quagl i ar el l o  V,   Schel d  WM.   Bacter i al   m eni ngi ti s:  pathogenesi s,


pathophy si ol ogy   and  pr ogr ess.   N   Engl   J  Med  1992;327:864.

Schel d  WM,   Whi tl ey   RJ,   Dur ak   DT.   Cer ebr ospi nal   f l ui d  i n  centr al   ner v ous
sy stem   i nf ecti ons.   In:  Gi l l i n  BG,   Wei ngar ten  K,   Gam ache  PW,   et  al .   eds.
Inf ecti ons  of   the  centr al   ner v ous  sy stem .   N ew   Yor k :  Rav en  Pr ess,
1991:861.

Schoenbaum   SC,   Gar dner   P,   Shi l l i to  J.   Inf ecti ons  of   cer ebr ospi nal   f l ui d
shunts:  epi dem i ol ogy ,   cl i ni cal   m ani f estati ons  and  ther apy .   J  Inf ect  Di s
1975;131:543.

Tunk el   AR.   Bacter i al   m eni ngi ti s.   Phi l adel phi a:  Li ppi ncott  Wi l l i am s  &
Wi l k i ns,   2001.

Tunk el   AR,   Har tm an  BJ,   Kapl an  SL,   et  al .   Pr acti ce  gui del i nes  f or   the
m anagem ent  of   bacter i al   m eni ngi ti s.   Cl i n  Inf ect  Di s  2004;39:1267.

Tunk el   AR,   Wi spel w ay   B,   Schei d  WM.   Bacter i al   m eni ngi ti s:  r ecent  adv ances
i n  pathophy si ol ogy   and  tr eatm ent.   Ann  Inter n  Med  1990;112:610.

Fever of Unknown Origin
1.   What  ar e  the  def i ni ti ons  of   f ev er   and  f ev er   of   unk now n  or i gi n?

2.   What  i s  the  pathogenesi s  of   f ev er ?

P. 259
3.   What  ar e  the  gener al   categor i es  of   di sease  that  can  cause  f ev er ,   and
w hi ch  gener al   categor i es  ar e  the  m ost  com m onl y   encounter ed?

4.   Whi ch  l abor ator y   tests  shoul d  be  r outi nel y   per f or m ed  i n  a  pati ent  w i th
f ev er   of   unk now n  or i gi n?

5.   In  m ost  l ar ge  ser i es,   w hat  per centage  of   pati ents  w i th  f ev er   of   unk now n
or i gi n  hav e  been  f ound  to  ev ade  di agnosi s?

Discussion
Discussion
1.   What  ar e  the  def i ni ti ons  of   f ev er   and  f ev er   of   unk now n  or i gi n?

Fev er   i s  def i ned  as  an  el ev ati on  of   the  body   tem per atur e.   The  nor m al
tem per atur e  m ay   v ar y   f r om   per son  to  per son,   and  r anges  f r om   97. 0°F  to
99. 2°F  (36. 1°C  to  37. 5°C)  i n  heal thy   peopl e.   The  tem per atur e  can
al so  dem onstr ate  a  di ur nal   v ar i ati on,   i n  that  i t  tends  to  be  som ew hat
l ow er   ear l y   i n  the  m or ni ng.   It  i s  i m por tant  to  docum ent  a  f ev er   ov er   the
cour se  of   an  enti r e  day ,   and,   to  do  thi s,   pati ents  shoul d  be  i nstr ucted  to
k eep  a  l og  of   thei r   tem per atur e  at  hom e.

An  el ev ated  tem per atur e  can  be  the  hal l m ar k   of   i nf ecti on;  how ev er ,   a
pati ent  w i th  a  ser i ous  i nf ecti on  can  be  hy pother m i c  or   ev en  hav e  a
nor m al   tem per atur e,   especi al l y   i f   he  or   she  i s  el der l y   or
i m m unosuppr essed.   N ot  al l   f ev er s  ar e  caused  by   i nf ecti ons.

Peter sdor f   and  Beeson  or i gi nal l y   def i ned  f ev er   of   unk now n  or i gi n  as  f ev er
that  ex ceeds  38. 3°C  (100. 94°F)  on  sev er al   occasi ons,   l asts  at  l east  3
w eek s,   and  def i es  di agnosi s  af ter   at  l east  1  w eek   of   r outi ne  study   i n  the
hospi tal .   The  1­w eek   r outi ne  study   i s  thought  to  el i m i nate  m ost  shor t­
l i v ed  f ev er s  (e. g. ,   v i r al   i l l ness,   postoper ati v e  f ev er ,   and  f acti ti ous
f ev er ).   It  has  been  suggested  that  thi s  l ast  cr i ter i on  (hospi tal   adm i ssi on)
shoul d  be  m odi f i ed  to  “1  w eek   of   i ntel l i gent  and  i ntensi v e
i nv esti gati on, â€​   w hi ch  f or   m ost  pati ents  coul d  be  done  on  an  outpati ent
basi s.   Thi s  def i ni ti on  does  not  appl y   to  i m m unocom pr om i sed  pati ents,
how ev er .

2.   What  i s  the  pathogenesi s  of   f ev er ?

The  body   tem per atur e  i s  cl osel y   r egul ated  w i thi n  a  cer tai n  nor m al   r ange,
and  f ev er   occur s  w hen  the  cor e  body   tem per atur e  ex ceeds  thi s  r ange.
Ther e  ex i sts  a  bal ance  betw een  net  heat  pr oducti on  and  heat  l oss.   Heat  i s
pr oduced  thr ough  body   m etabol i sm   and  m uscl e  acti v i ty ;  heat  i s  l ost  by
m eans  of   di ssi pati on  thr ough  the  sk i n  and  the  l ungs.

A  centr al   r egul ator   of   body   tem per atur e  i s  the  pr eopti c  nucl eus  of   the
anter i or   hy pothal am us.   The  hy pothal am us  contr ol s  body   tem per atur e  by
sti m ul ati ng  the  autonom i c  ner v ous  sy stem   to  pr oduce  per i pher al
v asodi l ati on  and  sw eati ng.   The  hy pothal am us  can  al so  cause  heat  to  be
conser v ed  by   br i ngi ng  about  cutaneous  v asoconstr i cti on.   Shi v er i ng  can
al so  i ncr ease  heat  pr oducti on.

In  the  setti ng  of   i nf ecti on  or   other   i nf l am m ator y   states,   m ononucl ear
phagocy tes  pr oduce  cy tok i nes  such  as  IL­1  and  TN F  that  ar e  capabl e  of
r ai si ng  the  set  poi nt  of   the  hy pothal am us.   Thi s  i ni ti ates  the  com pl ex
m echani sm s  that  pr oduce  py r ex i a.   IL­1  appear s  to  sti m ul ate  the
hy pothal am us  thr ough  a  pr ostagl andi n  m echani sm ,   w hi ch  ex pl ai ns  w hy
pr ostagl andi n  i nhi bi tor s  such  as  aspi r i n  ar e  ef f ecti v e  anti py r eti c  agents.

P. 260
3.   What  ar e  the  gener al   categor i es  of   di sease  that  can  cause  f ev er ,   and
w hi ch  gener al   categor i es  ar e  the  m ost  com m onl y   encounter ed?
Fev er s  that  def y   al l   attem pts  at  di agnosi s  pose  a  chal l enge  to  the
cl i ni ci an.   Because  m any   causes  of   f ev er   of   unk now n  eti ol ogy   ar e  obscur e
on  the  i ni ti al   ev al uati on  of   a  pati ent,   i t  i s  hel pf ul   to  categor i ze  the
di agnosti c  possi bi l i ti es  i nto  gr oups  accor di ng  to  the  l i k el i hood  of   causi ng
the  f ev er .

Ther e  ar e  num er ous  di sease  states  associ ated  w i th  f ev er ,   but  i nf ecti ons
w ar r ant  speci al   attenti on.   Most  i nf ecti ous  causes  ar e  obv i ous  to  the
ev al uati ng  cl i ni ci an  once  a  car ef ul   hi stor y   and  phy si cal   ex am i nati on
coupl ed  w i th  r outi ne  di agnosti c  tests  ar e  com pl eted.   Cer tai n  sy stem i c
i nf ecti ous  di seases  that  ar e  par ti cul ar l y   associ ated  w i th  f ev er   of   unk now n
or i gi n  i ncl ude  tuber cul osi s  (par ti cul ar l y   the  ex tr apul m onar y   f or m )  and
bacter i al   endocar di ti s.   A  com pl ete  l i st  of   i nf ecti ous  causes  of   f ev er   of
unk now n  or i gi n  i s  bey ond  the  scope  of   thi s  tex t;  how ev er ,   py ogeni c
bacter i al ,   f ungal ,   m y cobacter i al ,   v i r al ,   r i ck ettsi al ,   par asi ti c,   and
spi r ochetal   i nf ecti ons  hav e  al l   been  associ ated  w i th  pr ol onged  f ev er .
Local i zed  causes  of   f ev er   of   unk now n  eti ol ogy   i ncl ude  i ntr aabdom i nal ,
per i nephr i c,   pr ostati c,   and  tooth  abscesses,   hepatobi l i ar y   i nf ecti ons,   and
pel v i c  i nf ecti ons.   These  sour ces  of   i nf ecti on  can  be  occul t  and  need  to  be
consi der ed  i n  a  pati ent  w i th  a  per pl ex i ng  f ev er .

Other   gener al   categor i es  of   f ev er   i ncl ude  m al i gnancy   and  col l agen­
v ascul ar   di sor der s.   Less  com m on  m i scel l aneous  di sor der s  i ncl ude
sar coi dosi s,   i nf l am m ator y   bow el   di sor der s,   pul m onar y   em bol l i ,
thy r oi di ti s,   a  r etr oper i toneal   hem atom a,   gr anul om atous  hepati ti s,   al l er gi c
r eacti ons  (dr ug  f ev er s),   and  i nher i ted  di seases  (f am i l i al   Medi ter r anean
f ev er ).   Facti ti ous  and  f abr i cated  f ev er s  hav e  al so  been  descr i bed,   but
these  consti tute  a  di agnosi s  of   ex cl usi on.   Fi nal l y ,   a  si gni f i cant  m i nor i ty   of
f ev er s  w i th  an  undeter m i ned  cause  ar e  i di opathi c.

4.   Whi ch  l abor ator y   tests  shoul d  be  r outi nel y   per f or m ed  i n  a  pati ent  w i th
f ev er   of   unk now n  or i gi n?

Al m ost  now her e  i n  the  pr acti ce  of   m edi ci ne  ar e  an  i n­depth  hi stor y   and
com pl ete  phy si cal   ex am i nati on  as  essenti al   as  i n  the  ev al uati on  of   a
pati ent  w i th  f ev er   of   unk now n  or i gi n,   and,   as  Peter sdor f   obser v ed  i n
1969,   i t  i s  i m por tant  to  r em em ber   that  “at  the  end  of   the  needl e,   the
x ­r ay   tube,   and  ev en  the  scal pel ,   i s  a  si ck   pati ent  w ho  deser v es  the  m ost
thoughtf ul   di agnosti c  appr oach  of   w hi ch  w e  ar e  capabl e. â€​

The  appr opr i ate  ev al uati on  of   each  pati ent  w i th  f ev er   of   unk now n  or i gi n
needs  to  be  i ndi v i dual i zed.   Attenti on  shoul d  be  pai d  to  the  pati ent's
ex posur e  hi stor y ,   tr av el   hi stor y ,   occupati on,   ani m al   ex posur e,   hobbi es,
and  m edi cati ons.   The  ex am i nati on  shoul d  be  thor ough  and  par ti cul ar
attenti on  shoul d  f ocus  on  the  l y m phoi d  or gans,   sk i n,   hear t,   ey e  gr ounds,
and  conjuncti v ae  i n  a  sear ch  f or   ev i dence  of   occul t  di sease,   such  as
bacter i al   endocar di ti s,   m al i gnancy ,   and  v ascul i ti s.

Bl ood  cul tur es  shoul d  be  done  r outi nel y ,   as  w el l   as  a  com pl ete  bl ood
count  w i th  a  di f f er enti al .   Chest  r adi ogr aphi c  studi es  shoul d  al so  be
obtai ned  to  r ul e  out  i nf ecti on  or   m al i gnancy .   Var i ous  other   di agnosti c
studi es,   i ncl udi ng  r adi ol ogi c  ex am i nati ons  and  bl ood  tests,   shoul d  al so  be
per f or m ed  as  di ctated
P. 261
by   the  natur e  of   the  cl i ni cal   pr esentati on.   Cer tai n  ser ol ogi c  tests  (such  as
a  Ly m e  anti body   test)  m ay   be  i ndi cated  i n  the  appr opr i ate  epi dem i ol ogi c
setti ng.   Cl ear l y ,   a  r andom   sear ch  f or   answ er s  i s  not  appr opr i ate.

5.   In  m ost  l ar ge  ser i es,   w hat  per centage  of   pati ents  w i th  f ev er   of   unk now n
or i gi n  hav e  been  f ound  to  ev ade  di agnosi s?

In  the  f ew   l ar ge  tr i al s  that  hav e  ex am i ned  thi s  questi on,   5%  to  25%  of
pati ents  w i th  f ev er   of   unk now n  or i gi n  hav e  been  f ound  to  el ude  a  speci f i c
di agnosi s.   Tabl e  6­1  sum m ar i zes  the  obser v ati ons  f r om   thr ee  of   the
m ajor   ser i es  of   pati ents  w i th  f ev er   of   unk now n  or i gi n.

Case
A  61­y ear ­ol d  w hi te  m an  i s  seen  because  of   a  f ev er .   He  w as  w el l   unti l   2
m onths  bef or e,   w hen  he  noted  the  onset  of   f ati gue,   f ev er ,   chi l l s,   and  w ei ght
l oss.   Tem per atur es  as  hi gh  as  40°C  (104°F)  hav e  occur r ed  i n  a  cy cl i c
m anner   (ev er y   2  to  3  day s),   but  r esol v e  w i th  acetam i nophen.   He  deni es
headaches,   ar thr al gi as,   v i sual   di stur bances,   abdom i nal   pai n,   and  di ar r hea.   Hi s
m edi cal   hi stor y   i s  r em ar k abl e  f or   asthm a,   env i r onm ental   al l er gi es  f or   w hi ch
he  i s  under goi ng  i m m unother apy ,   and  a  hi atal   her ni a.   Hi s  f am i l y   hi stor y   i s
unr em ar k abl e.   The  pati ent  does  not  consum e  al cohol   or   sm ok e  ci gar ettes.   He
i s  a  r eti r ed  f i r em an  and  has  not  tr av el ed  or   had  ex posur e  to  i l l   contacts.   He
has  no  pets  or   other   ani m al   ex posur es.   Ther e  ar e  none  of   the  usual l y
r ecogni zed  r i sk   f actor s  f or   HIV  i nf ecti on.   He  i s  tak i ng  no  m edi cati ons.
On  phy si cal   ex am i nati on,   the  pati ent  i s  f ound  to  be  a  ti r ed­appear i ng,   el der l y
m an.   Hi s  bl ood  pr essur e  i s  146/85  m m   Hg;  pul se,   106  beats  per   m i nute;
r espi r ati ons,   20  per   m i nute;  and  tem per atur e,   38. 3°C  (100. 94°F).   The  head,
ey es,   ear s,   nose,   and  thr oat  ex am i nati on  i s  r em ar k abl e  f or   the  f i ndi ng  of   dr y
m ucous  m em br anes;  hi s  or ophar y nx   i s  cl ear   and  the  ty m pani c  m em br anes  ar e
nor m al .   Ther e  i s  no  l y m phadenopathy   ex cept  f or   a  sm al l ,   1. 5  ×  2­cm ,
nontender   l y m ph  node  i n  the  r i ght  i ngui nal   ar ea.   The  hear t  sounds  ar e
unr em ar k abl e  ex cept  f or   a  r egul ar   tachy car di a.   The  l ungs  ar e  cl ear   to
auscul tati on  and  per cussi on.   Abdom i nal   ex am i nati on  r ev eal s  nor m al   bow el
sounds,   and  no  hepatospl enom egal y   or   m asses  ar e  pal pated.   Pr ostate  and
r ectal   f i ndi ngs  ar e  nor m al   and  a  test  f or   occul t  bl ood  i s  negati v e.   Hi s  sk i n
appear s  jaundi ced.   The  neur ol ogi c  f i ndi ngs  ar e  nor m al .

Table 6­1 Summary of Study Findings in Patients
with Fever of Unknown Origin a

Ca us e Ja c oby a nd La rs on e t a l. , Knoc k a e rt e t


Sw a rtz,  1973 (n 1982 (n = a l. ,  1992 (n =
= 128) 105) 199)
Inf ecti on 40 30 23

N eopl asm s 20 31 7

Col l agen­ 15 9 19
v ascul ar
di sease

Mi scel l aneous b 17­20 17 25

U ndi agnosed 5­8 12 26

a N um ber s  ar e  per centages.

b Incl udes  al l   di agnoses  not  f i tti ng  i nto  other   categor i es  (e. g. ,

sar coi d).

P. 262
A  chest  r adi ogr aph  i s  nor m al .   A  CT  scan  of   the  abdom en  r ev eal s  enl ar ged
por tacav al   l y m ph  nodes.   The  ser um   el ectr ol y te  v al ues  ar e  nor m al ,   and  the
f ol l ow i ng  l abor ator y   data  ar e  r epor ted:  w hi te  bl ood  cel l   count,   4, 000/µ  L;
hem ogl obi n,   11. 4  g/dL;  and  pl atel et  count,   134, 000/m m 3 .   The  di f f er enti al
count  r ev eal s  33%  segm ented  neutr ophi l s,   6%  band  f or m s,   18%  l y m phocy tes,
4%  r eacti v e  l y m phocy tes,   20%  m ononucl ear   cel l s,   and  15%  eosi nophi l s.   The
al bum i n  content  i s  3. 1  m g/dL;  total   bi l i r ubi n,   2. 8  m g/dL;  al ani ne
am i notr ansf er ase,   31  IU /L;  AST,   35  IU /L;  al k al i ne  phosphatase,   242  IU /L;  and
l actate  dehy dr ogenase,   567  IU /L.   Al l   bl ood  cul tur es  ar e  negati v e.   The
er y thr ocy te  sedi m entati on  r ate  i s  50  m m   per   hour .   A  PPD  of   My cobacter i um
tuber cul osi s  sk i n  test  i s  negati v e,   as  i s  the  ser um   anti nucl ear   anti body   test.
A  bone  m ar r ow   bi opsy   speci m en  show s  m i l d  chr oni c  i nf l am m ati on  and
ex tensi v e  gr anul om atosi s.   The  gr anul om atous  f oci   ar e  com posed  of
eosi nophi l s,   sm al l   l y m phocy tes  w i th  i r r egul ar   nucl ei ,   and  hi sti ocy tes.   Routi ne
bacter i al ,   aci d­f ast  baci l l i ,   and  f ungal   cul tur es  and  stai ns  ar e  negati v e.   A  test
f or   ur i nar y   Hi stopl asm a  anti gen  i s  negati v e.   A  needl e  bi opsy   speci m en  of   the
l i v er   r ev eal s  si nusoi dal   di l atati on,   tr i adi ti s,   bi l e  stasi s,   and  f ocal   per i por tal
f i br osi s  w i th  gr anul om as  and  di l atati on  of   the  por tal   v enous  channel s.
The  pati ent  i s  begun  em pi r i cal l y   on  a  r egi m en  of   i soni azi d,   etham butol ,   and
r i f am pi n  f or   a  pr esum pti v e  di agnosi s  of   ex tr apul m onar y   tuber cul osi s,   but
ther e  i s  l i ttl e  attendant  i m pr ov em ent  i n  hi s  cl i ni cal   status.

1.   What  i s  the  m ost  l i k el y   di agnosi s  i n  thi s  pati ent?


2.   What  di agnosti c  test  shoul d  be  per f or m ed  nex t?
3.   What  di sor der s  coul d  be  causi ng  the  gr anul om as  and  f ev er   i n  thi s
pati ent?

Case Discussion
1.   What  i s  the  m ost  l i k el y   di agnosi s  i n  thi s  pati ent?

The  m ost  l i k el y   cause  of   thi s  pati ent's  i l l ness  i s  l y m phom a.   M.


tuber cul osi s  i s  one  of   the  m ost  com m on  or gani sm s  to  be  cul tur ed  f r om
pati ents  w i th  f ev er   of   unk now n  or i gi n  and,   ther ef or e,   i t  i s  i m por tant  to
r ul e  i t  out,   especi al l y   consi der i ng  that  the  num ber   of   cases  of   M.
tuber cul osi s  i nf ecti on  hav e  been  i ncr easi ng  i n  the  U ni ted  States  si nce  the
m i d­1980s.   How ev er ,   the  di agnosi s  m ay   be  del ay ed  because  i t  can  tak e
cul tur es  4  to  6  w eek s  to  becom e  posi ti v e,   al though  sm ear s  of   sputum   or
other   appr opr i ate  cl i ni cal   speci m ens  m ay   be  posi ti v e  w hen  stai ned  w i th
aci d­f ast  stai n.   Occasi onal l y ,   the  PPD  sk i n  test  i s  negati v e,   especi al l y   i n
pati ents  w i th  di ssem i nated  di sease.   Thi s  em phasi zes  the  i m por tance  of
usi ng  contr ol   sk i n  tests  i n  addi ti on  to  the  PPD  test.   Thi s  pati ent  r epor ted
no  ex posur e  to  tuber cul osi s,   and  hi s  condi ti on  di d  not  i m pr ov e  on
anti tuber cul ous  m edi cati ons,   m ak i ng  thi s  di sease  l ess  l i k el y .

Cer tai n  bacter i al   i nf ecti ons  ar e  pr one  to  di ssem i nate  and  i nf ect  the
r eti cul oendothel i al   sy stem ,   i ncl udi ng  Br ucel l a  and  Li ster i a  speci es.
Al though  Br ucel l a  i nf ecti ons  can  be  associ ated  w i th  l y m phadenopathy   and
f ev er ,   thi s  pati ent  had  had  no  contact  w i th  l ar ge  ani m al s  or   occupati onal
ex posur es  that  w oul d  pl ace  hi m   at  r i sk   f or   br ucel l osi s.   In  addi ti on,
assum i ng  the  l abor ator y   i s  al er ted  to  thi s  possi bi l i ty ,   bone  m ar r ow
cul tur es  can  be  posi ti v e  i n  a  l ar ge  per centage  of   pati ents  w i th  Br ucel l a.

P. 263
Col l agen­v ascul ar   di sor der s  and  v ascul i ti s  ar e  other   causes  of   f ev er   of
unk now n  or i gi n.   Am ong  them   ar e  di seases  such  as  pol y m y al gi a
r heum ati ca,   sy stem i c  l upus  er y them atosus,   m i x ed  connecti v e  ti ssue
di sor der s,   and  juv eni l e  r heum atoi d  ar thr i ti s.   The  l ack   of   appr opr i ate
sy m ptom s  i n  thi s  pati ent,   and  a  negati v e  anti nucl ear   anti body   test,   m ak e
thi s  categor y   of   di sease  l ess  l i k el y .

Gi ant  cel l   ar ter i ti s  deser v es  speci al   m enti on  because  15%  of   pati ents  w i th
thi s  di sease  can  pr esent  w i th  f ev er .   Of ten,   the  sedi m entati on  r ate  i n
these  pati ents  ex ceeds  100  m m   per   hour .   The  l ack   of   v i sual   di stur bances,
tem por al   ar ter y   tender ness,   or   jaw   cl audi cati on  does  not  com pl etel y   r ul e
out  thi s  di agnosi s,   and  occasi onal l y   a  tem por al   ar ter y   bi opsy   i s  i ndi cated
to  el uci date  the  si tuati on.   In  one  l ar ge  ser i es,   gi ant  cel l   ar ter i ti s  w as
f ound  to  be  the  m ost  com m on  cause  of   f ev er   of   unk now n  or i gi n  i n
pati ents  ol der   than  50  y ear s.

N um er ous  m al i gnanci es  hav e  been  associ ated  w i th  f ev er .   N eopl asm s  of
the  r eti cul oendothel i al   sy stem   ar e  the  m ost  com m on  cl ass  of   tum or s
causi ng  f ev er .   Fev er   i n  a  pati ent  of   thi s  age  w ho  ex hi bi ts  both  w ei ght  l oss
and  adenopathy   suggests  a  m al i gnancy .   A  cy cl i c  patter n  of   f ev er s,   such
as  that  dem onstr ated  by   thi s  pati ent,   suggests—but  does  not  cl i nch—a
di agnosi s  of   Hodgk i n's  di sease.

Pati ents  w i th  l y m phom a  can  pr esent  w i th  r ecur r ent  f ev er   that  r em ai ns
obscur e.   Other   m al i gnanci es  associ ated  w i th  f ev er   i ncl ude  nonâ
€“Hodgk i n's  l y m phom a,   r enal   cel l   car ci nom a,   and  atr i al   m y x om as.

2.   What  di agnosti c  test  shoul d  be  per f or m ed  nex t?

The  phy si ci an  shoul d  al w ay s  pr oceed  i n  a  l ogi cal   and  stepw i se  m anner   i n
the  ev al uati on  of   a  pati ent  such  as  thi s  one.   The  w or k up  shoul d  star t  w i th
a  detai l ed  hi stor y   and  phy si cal   ex am i nati on,   f ol l ow ed  by   di r ected
l abor ator y   ev al uati ons  and  not  a  r andom   sear chi ng  f or   an  answ er .   Thi s
pati ent  under w ent  a  v er y   ex tensi v e  w or k up,   i ncl udi ng  r outi ne  bl ood  tests,
r adi ol ogi c  ev al uati ons,   and  cul tur es  that  di d  not  y i el d  a  di agnosi s.   The
nex t  m ost  l ogi cal   step  w oul d  be  to  per f or m   an  ex ci si onal   l y m ph  node
bi opsy .   It  i s  i m por tant  to  tr y   to  obtai n  the  enti r e  l y m ph  node,   f or   the
pur poses  of   both  hi stol ogi c  ex am i nati on  and  the  per f or m ance  of   speci al
stai ns  and  cul tur es.   Occasi onal l y ,   f i ne­needl e  aspi r ati on  of   a  l y m ph  node
can  be  a  r api d  and  r el i abl e  m ethod  f or   di agnosi s,   but  the  am ount  of
m ater i al   obtai ned  m ay   not  be  adequate  f or   com pl ete  hi stol ogi c
conf i r m ati on  of   l y m phom a.   If   no  per i pher al   l y m ph  nodes  ar e  am enabl e  to
bi opsy ,   a  l apar otom y   w i th  sam pl i ng  of   i ntr aabdom i nal   nodes  m ay   be
needed  but  thi s  shoul d  not  be  under tak en  unti l   noni nv asi v e  r adi ogr aphi c
studi es  hav e  been  uti l i zed  to  ev al uate  the  i ntr athor aci c  and
i ntr aabdom i nal   cav i ti es  f or   abnor m al i ti es  that  coul d  f ocus  the  sur gi cal
di agnosti c  i nter v enti on.

How ev er ,   i f   a  CT  scan  or   ul tr asound  study   detects  an  i ntr aabdom i nal
abnor m al i ty   that  cannot  be  cul tur ed  or   sam pl ed  f or   bi opsy
per cutaneousl y ,   l apar otom y   m ay   be  essenti al   to  obtai n  adequate  m ater i al
f or   hi stol ogi c  studi es  and  cul tur e.

In  ter m s  of   i nf ecti ous  di seases,   cer tai n  ser ol ogi c  tests  can  be  i nv al uabl e
i n  the  ev al uati on  of   a  pati ent  w i th  a  f ev er   of   undeter m i ned  eti ol ogy .
Ri si ng  anti body   ti ter s  can  be  di agnosti c  f or   cer tai n  i nf ecti ous  di seases,
but  of ten  acute  and  conv al escent  ti ter s  need  to  be  deter m i ned  as  a  pai r
to  conf i r m   the  ex i stence  of   an  acute  i nf ecti on.   Ther e  ar e  speci f i c
ser ol ogi c  tests  f or   m any   i nf ecti ous  di seases,   i ncl udi ng  those  caused
P. 264
by   Br ucel l a,   Fr anci sel l a  tul ar ensi s,   and  Cox i el l a  bur neti i ,   but  r esul ts
w oul d  unl i k el y   be  posi ti v e  i n  thi s  setti ng  unl ess  ther e  i s  an  ex posur e
hi stor y   f or   these  or gani sm s.

Inf ecti on  w i th  HIV  m ust  be  sought,   especi al l y   i f   the  accepted
epi dem i ol ogi c  r i sk   f actor s  ex i st  (e. g. ,   hom osex ual   ex posur es,   i ntr av enous
dr ug  abuse,   and  bl ood  pr oduct  tr ansf usi on  bef or e  the  w i despr ead
scr eeni ng  f or   HIV).

3.   What  di sor der s  coul d  be  causi ng  the  gr anul om as  and  f ev er   i n  thi s  pati ent?

Gr anul om as  i n  the  l i v er   and  bone  m ar r ow   ar e  nonspeci f i c  f i ndi ngs.


Because  these  or gans  ar e  r i ch  i n  r eti cul oendothel i al   cel l s,   they   can
r espond  to  anti gens  and  f or m   gr anul om as.   Gr anul om as  ar e  k now n  to  be
associ ated  w i th  a  num ber   of   f ebr i l e  di seases  such  as  i nf ecti ons.   Am ong
the  i nf ecti ous  causes  of   gr anul om a  ar e  tuber cul osi s,   f ungal   i nf ecti ons
(e. g. ,   hi stopl asm osi s),   br ucel l osi s,   Q  f ev er ,   tul ar em i a,   schi stosom i asi s,
sy phi l i s,   and  Whi ppl e's  di sease.

Am ong  the  noni nf ecti ous  causes  of   gr anul om a,   sar coi dosi s  i s  the  m ost
com m on.   Hepati c  gr anul om as  can  al so  be  f ound  i n  the  setti ng  of
connecti v e  ti ssue  di seases,   hy per sensi ti v i ty   r eacti ons,   pr i m ar y   l i v er
di seases,   and  m al i gnancy .   Of   the  m al i gnanci es,   hepati c  gr anul om as  can
be  seen  w i th  l y m phom as.   Fi nal l y ,   i n  near l y   a  thi r d  of   the  pati ents  w i th
hepati c  gr anul om as,   the  cause  cannot  be  ascer tai ned,   and  these  cases  ar e
deem ed  i di opathi c.

Suggested Readings
Ar now   JP,   Fl aher ty   JP.   Fev er   of   unk now n  or i gi n.   Lancet  1997;350:575.

Cor ey   L,   Boeck h  M.   Per si stent  f ev er   i n  pati ents  w i th  neutr openi a.   N   Engl   J
Med  2002;346:222.

Jacoby   GA,   Sw ar tz  MN .   Fev er   of   undeter m i ned  or i gi n.   N   Engl   J  Med


1973;289:1407.

Lar son  EB,   Feather sone  HJ,   Peter sdor f   RG.   Fev er   of   undeter m i ned  or i gi n:
di agnosi s  and  f ol l ow ­up  of   105  cases,   1970–1980.   Medi ci ne  (Bal ti m or e)
1982;61:269.

Peter sdor f   RG.   Fev er   of   unex pl ai ned  or i gi n:  r epor t  on  100  cases.   Medi ci ne
(Bal ti m or e)  1961;40:1.

Peter sdor f   RG.   Fev er   of   unk now n  or i gi n:  an  ol d  f r i end  r ev i si ted.   Ar ch
Inter n  Med  1992;152:21.

Pneumonia
1.   What  sy m ptom s  and  phy si cal ,   l abor ator y ,   and  r adi ogr aphi c  f i ndi ngs  ar e
com m onl y   obser v ed  i n  pati ents  w i th  com m uni ty ­acqui r ed  pneum oni a?

2.   What  ar e  the  com m on  causes  of   com m uni ty ­acqui r ed  pneum oni a?

3.   What  i s  the  r ol e  of   the  spl een  i n  com bati ng  bacter i al   i nf ecti ons?

Discussion
1.   What  sy m ptom s  and  phy si cal ,   l abor ator y ,   and  r adi ogr aphi c  f i ndi ngs  ar e
com m onl y   obser v ed  i n  pati ents  w i th  com m uni ty ­acqui r ed  pneum oni a?

The  cl i ni cal   f i ndi ngs  i n  pati ents  w i th  com m uni ty ­acqui r ed  pneum oni a  ar e
di v er se,   but  can  of ten  be  hel pf ul   i n  f or m ul ati ng  a  di f f er enti al   di agnosi s.
In
P. 265
the  setti ng  of   bacter i al   pneum oni a,   the  sy m ptom s  ar e  of ten  acute  at
onset.   Fr equentl y ,   ther e  ar e  shak i ng  r i gor s,   hi gh  f ev er ,   and  cough
pr oducti v e  of   pur ul ent  sputum .   On  phy si cal   ex am i nati on,   the  pati ent  m ay
appear   i l l ,   and  si gns  of   l obar   consol i dati on  ar e  of ten  f ound  on  chest
ex am i nati on.   A  com pl ete  bl ood  count  m ay   r ev eal   a  br i sk   l euk ocy tosi s  w i th
a  l ef t  shi f t,   and  a  chest  r adi ogr aphi c  study   m ay   show   segm ental   or   l obar
i nf i l tr ates.

Aty pi cal   pneum oni a  (e. g. ,   due  to  v i r uses,   or   Ri ck ettsi a,   Chl am y di a,   or
My copl asm a  or gani sm s)  m ay   al so  be  acute  at  onset,   but  the  cough  i s
usual l y   dr y   and  nonpr oducti v e  and  r i gor s  ar e  absent.   Chest  ex am i nati on
m ay   r ev eal   f i ne  di f f use  r al es,   or   f i ndi ngs  m ay   be  nor m al .   Sk i n
ex am i nati on  m ay   r ev eal   a  r ash.   A  com pl ete  bl ood  count  m ay   show   a  m i l d
l euk ocy tosi s,   or   r esul ts  m ay   be  nor m al .   A  chest  r adi ogr aphi c  study
ty pi cal l y   show s  the  pr esence  of   di f f use  i nf i l tr ates  thr oughout  both  l ungs.

Pneum oni a  due  to  anaer obi c  or gani sm s  (e. g. ,   aspi r ati on  pneum oni a)  i s
usual l y   i nsi di ous  at  onset  and  the  f ev er   m ay   be  l ow   gr ade.   The  cough
m ay   be  pr oducti v e  of   f oul ­sm el l i ng  sputum .   The  pati ent's  denti ti on  m ay
be  poor   and  he  or   she  m ay   hav e  f oul ­sm el l i ng  br eath.   Chest  ex am i nati on
m ay   r ev eal   consol i dati on  i n  the  l ow er   l ung  f i el ds.   A  m i l d  l euk ocy tosi s  and
l ow er   l obe  i nf i l tr ates  (par ti cul ar l y   i n  the  r i ght  l ow er   l obe)  m ay   be  seen
on  chest  r adi ogr aphi c  f i l m s.

Pul m onar y   tuber cul osi s  i s  al so  i nsi di ous  at  onset.   The  f ev er   m ay   be
accom pani ed  by   dr enchi ng  ni ght  sw eats,   and  cough  i s  usual l y   pr oducti v e.
Chest  auscul tati on  m ay   r ev eal   si gns  of   upper   l obe  or   api cal   consol i dati on.
The  com pl ete  bl ood  count  i s  of ten  nor m al   and  chest  r adi ogr aphi c  studi es
m ay   show   upper   l obe  i nf i l tr ates,   of ten  w i th  cav i tati on.   Cal ci f i ed  hi l ar
l y m ph  nodes,   w hi ch  ar e  a  r esi dual   ef f ect  f r om   the  pr i m ar y   tuber cul ous
i nf ecti on,   ar e  of ten  obser v ed.

2.   What  ar e  the  com m on  causes  of   com m uni ty ­acqui r ed  pneum oni a?

The  di f f er enti al   di agnosi s  of   com m uni ty ­acqui r ed  pneum oni a  i s  br oad,   but
can  be  nar r ow ed  consi der abl y   by   the  f i ndi ngs  obtai ned  f r om   a  car ef ul
hi stor y   and  phy si cal   ex am i nati on,   sputum   Gr am 's  stai ni ng,   and  chest
r adi ogr aphi c  ev al uati on.   Bacter i al   pneum oni a  can  be  caused  by   S.
pneum oni ae,   H.   i nf l uenzae,   S.   aur eus,   Br anham el l a  catar r hal i s,   and
Legi onel l a  pneum ophi l a.  U ncom m on  causes  of   bacter i al   pneum oni a  i ncl ude
Yer si ni a  pesti s  (pl ague),   F.   tul ar ensi s  (tul ar em i a),   and  Baci l l us  anthr aci s
(anthr ax ).   Aty pi cal   pneum oni as  ar e  com m onl y   due  to  My copl asm a  speci es
or   r espi r ator y   v i r uses.   Less  com m on  causes  of   aty pi cal   pneum oni a
i ncl ude  Chl am y di a  speci es  (psi ttacosi s),   C.   bur neti i   (Q  f ev er ),   H.
capsul atum ,   C.   i m m i ti s,   and  M.   tuber cul osi s.  Anaer obi c  or   cav i tar y
pneum oni a  i s  m ost  com m onl y   caused  by   or al   anaer obes  or   by   M.
tuber cul osi s.   Less  com m on  causes  i ncl ude  My cobacter i um   k ansasi i ,   H.
capsul atum ,   C.   i m m i ti s,   and  Bl astom y ces  der m ati ti di s.

The  i ni ti al   assessm ent  of   pati ents  w i th  pneum oni a  shoul d  i ncl ude  a
car ef ul   occupati onal   and  soci al   hi stor y   to  deter m i ne  w hether   ther e  has
been  ex posur e  to  w ater ­cool i ng  f aci l i ti es  (L.   pneum ophi l a),   w i l d  ani m al s
(tul ar em i a  or   pl ague),   bi r ds  (psi ttacosi s),   or   f ar m   ani m al s  (anthr ax   or   Q
f ev er ),   and  w hether   ther e  has  been  a  r ecent  l oss  of   consci ousness
(aspi r ati on  pneum oni a)  or   ex posur e  to  peopl e  w i th  tuber cul osi s.   Li k ew i se,
the  tr av el   hi stor y   i s  al so  i m por tant
P. 266
i n  nar r ow i ng  the  di f f er enti al   di agnosi s.   Recent  tr av el   to  the  southw ester n
deser ts  of   the  U ni ted  States  w oul d  suggest  cocci di oi dom y cosi s;  ex posur e
to  bi r d  dr oppi ngs  or   bat  guano  i n  the  Mi dw est  w oul d  suggest
hi stopl asm osi s.

3.   What  i s  the  r ol e  of   the  spl een  i n  com bati ng  bacter i al   i nf ecti ons?

The  spl een  i s  par t  of   the  r eti cul oendothel i al   sy stem   and  i s  i m por tant  i n
cl ear i ng  cer tai n  bacter i al   pathogens  f r om   the  bl oodstr eam .   Aspl eni c
peopl e  ar e  suscepti bl e  to  sepsi s  stem m i ng  f r om   encapsul ated  bacter i a
(pneum ococci ,   H.   i nf l uenzae,   and  N .   m eni ngi ti di s).   They   shoul d  ther ef or e
be  v acci nated  agai nst  these  i nf ecti ons,   pr ef er abl y   bef or e  spl enectom y   i f
done  el ecti v el y ,   because  the  spl een  i s  al so  i m por tant  i n  the  dev el opm ent
of   an  anti body   r esponse  to  these  v acci nes.

Case
A  64­y ear ­ol d  w om an  f r om   Topek a,   Kansas,   pr esents  w i th  an  8­hour   hi stor y   of
f ev er ,   r i gor s,   and  a  cough  pr oducti v e  of   bl ood­ti nged  sputum .   She  has  been  i n
good  heal th  al l   of   her   l i f e  ex cept  f or   abdom i nal   tr aum a  that  necessi tated  a
spl enectom y   30  y ear s  ago.   As  y ou  ar e  ex am i ni ng  her ,   she  ex per i ences  shak i ng
r i gor s  and  her   f ev er   i s  f ound  to  be  39°C  (102. 6°F);  she  al so  com pl ai ns  of   a
pl eur i ti c  pai n  ov er   the  r i ght  poster i or   chest.   Phy si cal   ex am i nati on  r ev eal s  an
i l l ­appear i ng  w om an  w i th  a  per si stent  cough  pr oducti v e  of   pur ul ent  sputum .
Ther e  i s  dul l ness  to  per cussi on,   egophony ,   and  m oi st  r al es  i n  the  r i ght
poster i or   chest.   Her   w hi te  bl ood  cel l   count  i s  15, 000/m m 3   and  a  chest
r adi ogr aphi c  study   show s  a  dense  consol i dati on  i n  the  r i ght  l ow er   l obe  w i th  ai r
br onchogr am s.   Gr am 's  stai ni ng  of   a  sputum   sam pl e  r ev eal s  num er ous
neutr ophi l s  and  abundant  i ntr acel l ul ar   gr am ­posi ti v e  di pl ococci .

1.   What  i s  the  m ost  l i k el y   di agnosi s  i n  thi s  pati ent?


2.   What  does  the  di f f er enti al   di agnosi s  of   pneum oni a  consi st  of   i n  thi s
pati ent?
3.   On  the  basi s  of   the  sputum   f i ndi ngs,   w hat  i s  the  m ost  l i k el y   cause  of   thi s
pati ent's  condi ti on?
4.   What  w oul d  be  the  m ost  appr opr i ate  tr eatm ent  f or   thi s  pati ent?
Case Discussion
1.   What  i s  the  m ost  l i k el y   di agnosi s  i n  thi s  pati ent?

The  r api d  onset  of   sy m ptom s  and  pur ul ent  sputum   ar e  f i ndi ngs  m ost
suggesti v e  of   acute  bacter i al   pneum oni a.   Thi s  di agnosi s  i s  f ur ther
i ndi cated  by   the  l obar   consol i dati on  depi cted  on  the  chest  r adi ogr aphi c
study .   Al though  a  pul m onar y   em bol i sm   can  cause  the  sudden  onset  of
pl eur i ti c  chest  pai n,   hem opty si s,   and  f ev er ,   i t  w oul d  be  unusual   f or   r i gor s
and  pur ul ent  sputum   to  occur   i n  thi s  setti ng.   Tuber cul osi s  w oul d  usual l y
assum e  a  m or e  subacute  pr esentati on.   Br onchogeni c  car ci nom a  can
pr esent  w i th  br onchi al   obstr ucti on  and  a  postobstr ucti v e  pneum oni a,
al though  a  hi l ar   or   per i hi l ar   m ass  w oul d  l i k el y   be  f ound  on  chest
r adi ogr aphi c  studi es.   Thi s  pati ent's  pr esentati on  w oul d  be  unusual   f or
aty pi cal   pneum oni as,   such  as  those  caused  by   v i r uses  or   My copl asm a  or
Chl am y di a  speci es.   The  pr esence  of   l ancet
P. 267
shaped  di pl ococci   i n  the  sputum   Gr am 's  stai n  and  the  posi ti v e  ur i nar y   test
r esul t  f or   pneum ococcal   anti gen  f ur ther   suppor t  the  di agnosi s.

2.   What  does  the  di f f er enti al   di agnosi s  of   pneum oni a  consi st  of   i n  thi s
pati ent?

Thi s  pati ent's  si gns  and  sy m ptom s  ar e  m ost  consi stent  w i th  those  of   acute
com m uni ty ­acqui r ed  bacter i al   pneum oni a.   The  m ost  com m on  cause  of
com m uni ty ­acqui r ed  bacter i al   pneum oni a  i s  S.   pneum oni ae
(pneum ococcus).   Other   potenti al   causes  i ncl ude  H.   i nf l uenzae,   anaer obes
(aspi r ati on  pneum oni a),   and  L.   pneum ophi l a  (usual l y   spr ead  by
contam i nated  aer osol s  gener ated  by   ai r ­condi ti oni ng  sy stem s,
hum i di f i er s,   and  bath  show er s).   The  di agnosi s  of   bacter i al   pneum oni a  can
usual l y   be  easi l y   and  r api dl y   m ade  thr ough  the  ex am i nati on  of   a  Gr am 's­
stai ned  speci m en  of   ex pector ated  sputum .   Thi s  si m pl e  and  i nex pensi v e
test  w oul d  al so  be  a  k ey   to  deter m i ni ng  the  m ost  appr opr i ate  ther apy   f or
thi s  pati ent.   The  l ack   of   dom i nant  bacter i a  on  the  Gr am 's­stai ned  sputum
sam pl e  suggests  the  possi bi l i ty   of   l ess  com m on  causes  of   acute  l obar
pneum oni a  such  as  L.   pneum ophi l a,   tuber cul osi s,   or   f ungi
(cocci di oi dom y cosi s  or   hi stopl asm osi s).   The  di agnosi s  of   pneum oni a  due
to  L.   pneum ophi l a  i s  m ade  usual l y   on  the  basi s  of   sputum   cul tur e  f i ndi ngs
or   on  those  y i el ded  by   a  di r ect  f l uor escent  anti body   stai n  of   the  sputum .
Li k ew i se,   i f   pul m onar y   tuber cul osi s  i s  suspected,   sputum   aci d­f ast
stai ni ng  shoul d  be  per f or m ed.   A  tuber cul i n  sk i n  test  can  scr een  f or
pr ev i ous  ex posur e  to  tuber cul osi s,   but  i s  of   l i ttl e  v al ue  i n  the  ev al uati on
of   acti v e  pul m onar y   i nf ecti on.   Fungal   pneum oni as  shoul d  be  consi der ed  i f
the  pati ent  has  been  ex posed  to  bi r d  or   bat  f eces,   has  been  i nv ol v ed  i n
spel unk i ng  (hi stopl asm osi s),   or   has  tr av el ed  to  Sonor an  deser t  ar eas  i n
the  southw ester n  U ni ted  States  (cocci di oi dom y cosi s).

3.   On  the  basi s  of   the  sputum   f i ndi ngs,   w hat  i s  the  m ost  l i k el y   cause  of   thi s
pati ent's  condi ti on?

The  pr esentati on  and  Gr am 's  stai n  f i ndi ngs  ar e  i ndi cati v e  of   pneum oni a
due  to  S.   pneum oni ae  (pneum ococcus),   the  m ost  com m on  cause  of
bacter i al   pneum oni a  i n  adul ts.   The  el der l y ,   debi l i tated,   and
i m m unosuppr essed  ar e  especi al l y   pr one  to  pneum ococcal   pneum oni a.   The
spl enectom y   i n  thi s  pati ent  al so  pr edi sposes  her   to  sepsi s  caused  by
encapsul ated  bacter i a  such  as  S.   pneum oni ae,   H.   i nf l uenzae,   and  N .
m eni ngi ti di s.

4.   What  w oul d  be  the  m ost  appr opr i ate  tr eatm ent  f or   thi s  pati ent?

Ther e  ar e  tw o  aspects  to  deci si on  m ak i ng  i n  thi s  pati ent:  w hether   she  can
be  tr eated  as  an  outpati ent  and  w hat  anti bi oti cs  she  shoul d  r ecei v e.   A
v er y   w el l   v al i dated  tool   ter m ed  the  Pneum oni a  Sev er i ty   Index   (PSI)  has
been  dev el oped  that  pr ov i des  ex cel l ent  gui dance  about  w hether   pati ents
shoul d  best  be  tr eated  i n  an  i npati ent  or   an  outpati ent  setti ng.   The  PSI
com bi nes  dem ogr aphi c  f eatur es  of   the  pati ent  such  as  age  and  sex ,
cl i ni cal   f eatur es  of   the  pati ent,   and  under l y i ng  condi ti ons  to  pr ov i de  a
scor e  that  pr ov i des  ex cel l ent  gui dance  about  w hether   a  pati ent  can  saf el y
be  m anaged  as  an  outpati ent  (Fi g.   6­2).   Once  the  m ost  appr opr i ate
setti ng  f or   i ni ti al   tr eatm ent  has  been  deter m i ned,   a  deci si on  about  i ni ti al
anti bi oti cs  m ust  be  m ade.   In  the  past,   f or   a  pati ent  such  as  thi s  one,   i t
w oul d  hav e  been  acceptabl e  to  i ni ti ate  ther apy   w i th  peni ci l l i n.   Ov er   the
l ast  sev er al   y ear s,   the  pr ev al ence  of   str ai ns  of   S.   pneum oni ae  that  ar e
r esi stant  to  peni ci l l i n  has  r i sen  shar pl y   i n  the  U ni ted  States.
P. 268
Al though  ther e  i s  contr ov er sy   about  w hether   m ost  pati ents  w i th
pneum ococcal   pneum oni a  w i th  m oder atel y   r esi stant  or gani sm s  can  be
tr eated  w i th  peni ci l l i n  or   am ox i ci l l i n,   thi s  pati ent  i s  acutel y   i l l   and  i s
f ur ther   com pr om i sed  by   hav i ng  had  a  spl enectom y .   Ther ef or e,   i t  w oul d  be
i m pr udent  to  em pi r i cal l y   use  peni ci l l i n  or   am ox i ci l l i n  i n  the  absence  of
suscepti bi l i ty   testi ng  data.   Thi s  pati ent  w oul d  best  be  tr eated  w i th  a
com bi nati on  of   azi thr om y ci n  or   cl ar i thr om y ci n  pl us  a  β­l actam
(cef otax i m e,   cef tr i ax one,   am pi ci l l i n­sul bactam ,   or   er tapenem ).
Figure  6­2  Pneum oni a  Sev er i ty   Index .   (Fr om   Hal m   EA,   Tei r stei n  AS.
Managem ent  of   com m uni ty ­acqui r ed  pneum oni a.   N   Engl   J  Med
2002;347:2039. )

Cor ti coster oi ds  hav e  no  r ol e  i n  the  tr eatm ent  of   uncom pl i cated
pneum ococcal   pneum oni a.   A  pneum ococcal   v acci ne  i s  adm i ni ster ed  to
pr ev ent  pneum ococcal   i nf ecti on  i n  hi gh­r i sk   pati ents  (e. g. ,   aspl eni c
peopl e  or   those  w i th  a  chr oni c  pul m onar y
P. 269
di sease  or   under l y i ng  i m m unodef i ci ency ),   but  has  no  r ol e  i n  the
m anagem ent  of   acute  pneum ococcal   pneum oni a.   Pneum ococcal
v acci nati on  i s  al so  r ecom m ended  f or   al l   peopl e  ol der   than  65  y ear s,   and
som e  phy si ci ans  adv ocate  v acci nati on  f or   al l   peopl e  ol der   than  55  y ear s.
Because  of   the  spl enectom y ,   thi s  pati ent  shoul d  r ecei v e  pneum ococcal
v acci nati on  as  soon  as  she  has  r ecov er ed  f r om   the  acute  i l l ness.
Suggested Readings
Bi sno  AL,   Fr eem an  JC.   The  sy ndr om e  of   aspl eni a,   pneum ococcal   sepsi s,
and  di ssem i nated  i ntr av ascul ar   coagul ati on.   Ann  Inter n  Med  1970;72:389.

Br oom e  CV,   Br ei m an  RF.   Pneum ococcal   v acci ne:  past,   pr esent  and  f utur e.
N   Engl   J  Med  1991;325:1506.

Fi ne  MJ,   Aubl e  TE,   Yeal y   DM,   et  al .   A  pr edi cti on  r ul e  to  i denti f y   l ow ­r i sk
pati ents  w i th  com m uni ty ­acqui r ed  pneum oni a.   N   Engl   J  Med  1997;336:243.

Guti er r ez  F,   Rodr i guez  JC,   Ay el o  A,   et  al .   Ev al uati on  of   the


i m m unochr om atogr aphi c  Bi nax   N OW  assay   f or   detecti on  of   Str eptococcus
pneum oni ae  ur i nar y   anti gen  i n  a  pr ospecti v e  study   of   com m uni ty ­acqui r ed
pneum oni a  i n  Spai n.   Cl i n  Inf ect  Di s  2003;36:286.

Hal m   EA,   Tei r stei n  AS.   Managem ent  of   com m uni ty ­acqui r ed  pneum oni a.   N
Engl   J  Med  2002;347:2039.

Jacoby   GA,   Ar cher   GL.   N ew   m echani sm s  of   bacter i al   r esi stance  to


anti m i cr obi al   agents.   N   Engl   J  Med  1991;324:601.

Kar l ow sk i   JA,   Thor nsber r y   C,   Jones  ME,   et  al .   Factor s  associ ated  w i th
r el ati v e  r ates  of   anti m i cr obi al   r esi stance  am ong  Str eptococcus  pneum oni ae
i n  the  U ni ted  States:  r esul ts  of   the  TRU ST  sur v ei l l ance  pr ogr am   (1998â
€“2002).   Cl i n  Inf ect  Di s  2003;36:963.

Mandel l   LA.   Rel ati onshi p  of   peni ci l l i n  r esi stance  to  m or tal i ty   i n
pneum ococcal   pneum oni a.   Cur r   Inf ect  Di s  Rep  2001;3:9.

Mandel l   LA,   Bar tl ett  JG,   Dow el l   SF,   et  al .   U pdate  of   pr acti ce  gui del i nes  f or
the  m anagem ent  of   com m uni ty ­acqui r ed  pneum oni a  i n  i m m unocom petent
adul ts.   Cl i n  Inf ect  Di s  2003;37:1405.

War d  J.   Anti bi oti c­r esi stant  Str eptococcus  pneum oni ae:  cl i ni cal   and
epi dem i ol ogi c  aspects.   Rev   Inf ect  Di s  1981;3:254.
Editors :  Sc hrie r,  Robe rt W .
Title :  Inte rna l Me dic ine  Ca s e book ,  The : Re a l P a tie nts ,  Re a l Ans w e rs ,  3rd
Edition

Copy r i ght  ©2007  Li ppi ncott  Wi l l i am s  &  Wi l k i ns

>  T a b le   o f   C o nte nts   >  C ha p te r   7  ­  He m a to lo g y   a nd   O nc o lo g y

Chapter 7
Hematology and Oncology

P a ul A.  Se ligma n

Acute Leukemia
1.   What  i s  the  pathol ogy   of   acute  l euk em i a?

2.   What  ar e  the  pr i m ar y   cl assi f i cati ons  of   acute  l euk em i a,   and  w hy   i s  thi s


di f f er enti ati on  i m por tant?

P. 271
3.   What  i s  the  Fr ench,   Am er i can,   and  Br i ti sh  (FAB)  cl assi f i cati on  of   acute
l euk em i a?

4.   Ar e  ther e  any   pr edi sposi ng  f actor s  associ ated  w i th  acute  l euk em i a?

5.   What  w or k up  and  other   pr epar ati ons  shoul d  be  done  bef or e  i ni ti ati ng
anti l euk em i c  ther apy ?

6.   What  ar e  i nducti on,   consol i dati on,   m ai ntenance  chem other apy ,   and  m eni ngeal
pr ophy l acti c  ther apy ,   and  how   do  they   di f f er   i n  the  tr eatm ent  of   acute
l y m phocy ti c  l euk em i a  (ALL)  and  acute  nonl y m phocy ti c  l euk em i a  (AN LL)?

7.   What  ar e  the  r i sk s  associ ated  w i th  anti l euk em i c  ther apy ,   and  w hat  r esul ts  can
be  ex pected?

Discussion
1.   What  i s  the  pathol ogy   of   acute  l euk em i a?

Acute  l euk em i a  i s  the  abnor m al   cl onal   ex pansi on  of   bl ood  cel l   pr ecur sor s.   The
abnor m al i ty   m ay   occur   at  di f f er ent  stages  of   m atur ati on  of   the  cel l ,   and  thi s
ex pl ai ns  the  di f f er ent  ty pes  of   l euk em i a.   Acute  l euk em i a  i s  usual l y   a  r api dl y
pr ogr essi v e  di sease,   al though  ther e  ar e  occasi onal   pati ents  w hose  di sease
r em ai ns  stabl e  f or   w eek s  or   ev en  m onths.   In  gener al ,   how ev er ,   i t  i s  not  the
l euk em i c  cel l s  per   se  that  cause  the  m or bi di ty   and  m or tal i ty   i n  thi s  di sor der ,
but  a  l ack   of   nor m al   bl ood  cel l s,   r esul ti ng  i n  anem i a,   thr om bocy topeni a,   and
l euk openi a.   Thi s  i s  br ought  about  by   the  l euk em i c  cel l s  “cr ow di ng  outâ€​   the
nor m al   cel l s  i n  the  bone  m ar r ow .   Other   data  suggest  that  especi al l y   m y el oi d
l euk em i a  cel l s  hav e  an  i nhi bi tor y   ef f ect  on  nor m al   m ar r ow   cel l s.   Thi s  l ack   of
nor m al   cel l s  m ay   ther ef or e  l ead  to  l i f e­thr eateni ng  hem or r hage  and  i nf ecti on.

2.   What  ar e  the  pr i m ar y   cl assi f i cati ons  of   acute  l euk em i a,   and  w hy   i s  thi s


di f f er enti ati on  i m por tant?

The  pr i m ar y   cl assi f i cati ons  of   acute  l euk em i a  ar e  ALL  and  acute


nonl y m phocy ti c  l euk em i a  (AN LL,   m y el oi d  l euk em i a).   The  di sti ncti on  i s
i m por tant  because  the  ther apy   di f f er s  f or   each  ty pe  (see  answ er   to  questi on
6).   The  ov er al l   r ati o  of   ALL  to  AN LL  i s  1  :  6.   ALL  occur s  m ost  com m onl y   i n
chi l dr en,   w her eas  AN LL  m or e  com m onl y   af f ects  adul ts.

3.   What  i s  the  FAB  cl assi f i cati on  of   acute  l euk em i a?

The  FAB  cl assi f i cati on  (Tabl e  7­1)  i s  based  l ar gel y   on  the  m or phol ogi c  and
hi stochem i cal   char acter i sti cs  di spl ay ed  by   the  l euk em i c  cel l s,   as  w el l   as  on  the
natur e  of   the  cel l   sur f ace  anti gens  and  cy togeneti c  f eatur es.   Thi s  i nf or m ati on
m ay   l ead  to  changes  i n  pati ent  m anagem ent,   ei ther   by   di r ecti ng  the  cour se  of
ther apy   or   by   def i ni ng  the  pr ognosi s  better .   Tabl e  7­1  al so  i ncl udes  m ol ecul ar
changes  that  m ay   af f ect  ther apy ,   but  m or e  of ten  af f ect  r esponse  to  ther apy .

4.   Ar e  ther e  any   pr edi sposi ng  f actor s  associ ated  w i th  acute  l euk em i a?

Cer tai n  geneti c  and  env i r onm ental   f actor s  m ay   pr edi spose  a  per son  to  acute
l euk em i a.   Many   chr om osom al   al ter ati ons  ex i st  i n  the  setti ng  of   the  l euk em i as.
The  i nci dence  of   l euk em i a  i s  i ncr eased  i n  pati ents  w i th  congeni tal   di sor der s
associ ated  w i th  aneupl oi dy ,   such  as  Dow n  sy ndr om e,   congeni tal
P. 272
P. 273
agr anul ocy tosi s,   cel i ac  di sease,   Fanconi   sy ndr om e,   and  v on  Reck l i nghausen's
neur of i br om atosi s.

Table 7­1 The French, American and British
(FAB) Classification of Acute Leukemia

      As s oc ia te d

FAB     Chromos ome

c la s s ific a tion De s c ription Comme nt Abnorma litie s

ALL  

L1 Sm al l   bl asts  w i th Most  com m on 12  :  21


l i ttl e  cy topl asm , m or phol ogy   i n
l i ttl e  cel l ­to­cel l chi l dhood  ALL
v ar i ati on

L2 Lar ger   cel l s  w i th Most  com m on —


gr eater   am ount m or phol ogy   i n
of   cy topl asm , adul t  ALL
gr eater   cel l ­to­
cel l   v ar i ati on;
i r r egul ar   nucl ei
w i th  m ul ti pl e
nucl eol i

L3 Lar ge  cel l s, Com m on  i n 8  :  14  (i . e. ,


str ongl y l euk em i a Bur k i tt's
basophi l i c associ ated  w i th l y m phom a)
cy topl asm ;  of ten Bur k i tt's
w i th  v acuol es; l y m phom a
nucl eol i   of ten
m ul ti pl e

ANLL

M1 Acute  m y el ocy ti c — —


l euk em i a:  cel l s
v er y
undi f f er enti ated
w i th  onl y
occasi onal
gr anul es

M2 b Acute  m y el ocy ti c — 8:21 a


l euk em i a:  cel l s
m or e
di f f er enti ated
w i th  gr anul es,
and  of ten  w i th
Auer   r ods

M3 Acute Of ten 15:17 a


pr om y el ocy ti c associ ated  w i th
l euk em i a: di ssem i nated
hy per gr anul ar i ntr av ascul ar
pr om y el ocy tes coagul ati on,
r esponds  to
di f f er enti ati on
agents

M4 b Acute Of ten  occur s Inv er si on a   16


m y el om onocy ti c w i th
l euk em i a:  both ex tr am edul l ar y
m onocy tes  and i nf i l tr ati on
m y el ocy tes (gi ngi v al
pr edom i nate hy per tr ophy ,
l euk em i a  cuti s,
and  m eni ngeal
l euk em i a)
M5 b Acute  m onocy ti c U sual l y   af f ects —
l euk em i a: chi l dr en  or
m onobl asts  w i th y oung  adul ts
r el ati v el y
agr anul ar
cy topl asm

M6 Er y thr ol euk em i a: Al so  cal l ed  Di —


r ed  bl ood  cel l Gugl i el m o's
pr ecur sor s sy ndr om e
pr edom i nate,   but
m y el oi d  bl asts
m ay   al so  be
seen

M7 Megak ar y ocy ti c Rar e  f or m   of —


l euk em i a: l euk em i a;  v er y
ex tr em el y poor   pr ognosi s
v ar i abl e
m or phol ogy ;
m ay   be
di agnosed  w i th
m onocl onal
anti bodi es  to
pl atel ets

a These  k ar y oty pes  ar e  gener al l y   consi der ed  to  be  m or e  l i k el y   to

r espond  to  chem other apy .

b When  M2,   M4,   and  M5  l euk em i a  occur   af ter   l ong­ter m   m y el ody spl asi a

11q  2;  3,   m onosom y   7  and  other   abnor m al

k ar y oty pes  suggest  decr eased  r esponse  to  chem other apy .

ALL,   acute  l y m phocy ti c  l euk em i a;  AN LL,   acute  nonl y m phocy ti c


l euk em i a.

Env i r onm ental   f actor s  i m pl i cated  i n  the  dev el opm ent  of   acute  l euk em i a,
par ti cul ar l y   AN LL,   i ncl ude  ex posur e  to  i oni zi ng  r adi ati on  and  chem i cal s.
Occupati ons  and  ther apy   that  i nv ol v e  r adi ati on  ex posur e  ar e  k now n  to  i ncr ease
the  r i sk   f or   acqui r i ng  acute  l euk em i a.   Chem i cal s,   par ti cul ar l y   the  i ndustr i al   use
of   benzene,   and  sev er al   ther apeuti c  dr ugs  (chl or am pheni col ,   pheny l butazone,
m el phal an,   chl or am buci l ,   and  other s)  ar e  causal   f actor s  i n  acute  l euk em i a.   The
f i ndi ngs  f r om   ani m al   studi es  l i nk   cer tai n  v i r uses  w i th  acute  l euk em i a;
how ev er ,   i t  i s  uncer tai n  w hi ch  v i r uses  ar e  actual l y   an  eti ol ogi c  f actor   i n  hum an
f or m s  of   l euk em i a,   ex cept  f or   l y m phom as  caused  by   v i r uses  that  dev el op  i nto
a  f or m   of   ALL.

5.   What  w or k up  and  other   pr epar ati ons  shoul d  be  done  bef or e  i ni ti ati ng
anti l euk em i c  ther apy ?

The  pr etr eatm ent  ev al uati on  shoul d  i ncl ude  the  pati ent's  m edi cal   and  w or k
hi stor y ,   especi al l y   the  natur e  of   any   r adi ati on  or   chem i cal   ex posur e.   A  phy si cal
ex am i nati on  shoul d  i ncl ude  the  pati ent's  tem per atur e,   pl us  ex am i nati on  of   the
opti c  f undi ,   l y m ph  node  ar eas,   or ophar y nx   and  gi ngi v ae,   per i anal   ar ea,   and
cr ani al   ner v es.   Labor ator y   studi es  shoul d  consi st  of   a  com pl ete  bl ood  count
w i th  di f f er enti al   (the  phy si ci an  shoul d  ex am i ne  the  sm ear ),   as  w el l   as  a  bl ood
chem i str y   pr of i l e  that  i ncl udes  the  m easur em ents  of   ur i c  aci d  and  l actate
dehy dr ogenase  (LDH).   Bone  m ar r ow   aspi r ates  and  bi opsy   speci m ens  shoul d  be
obtai ned,   and  i nv esti gati ons  shoul d  i ncl ude  cy togeneti c  studi es.   A  tr ansf usi on
w or k up  shoul d  i ncl ude  hum an  l y m phocy te  anti gen  (HLA)  ty pi ng.   Lum bar
punctur e  shoul d  be  per f or m ed  i n  al l   pati ents  suspected  of   hav i ng  ALL  or   AN LL­
M4,   and  the  cer ebr ospi nal   f l ui d  speci m en  shoul d  be  subjected  to  the  usual
studi es,   pl us  cy tol ogi c  anal y si s.   A  dental   ex am i nati on  shoul d  be  per f or m ed.

In  addi ti on,   the  pati ent's  condi ti on  shoul d  be  stabi l i zed  bef or e  anti l euk em i c
ther apy   i s  i ni ti ated.   Hem or r hage  and  i nf ecti on  shoul d  be  br ought  under   contr ol .
Gr eatl y   el ev ated  m y el obl ast  counts  (e. g. ,   > 50, 000/m m 3 )  that  occur   i n  the
setti ng  of   AN LL  can  l ead  to  pul m onar y   com pl i cati ons  as  w el l   as  f atal
i ntr acer ebr al   l euk ostasi s  and  hem or r hage.   Cr ani al   i r r adi ati on,   hy dr ox y ur ea,
and  l euk apher esi s  hav e  al l   been  used  to  decr ease  the  num ber s  of   ci r cul ati ng
l euk em i c  cel l s  r api dl y ,   and  hence  r educe  the  r i sk   of   com pl i cati ons.   (Because  of
the  phy si cal   pr oper ti es  of   the  l y m phocy ti c  l euk em i c  cel l ,   thi s  i s  r ar el y   a
pr obl em   i n  pati ents  w i th  ALL. )

Renal   dam age  stem m i ng  f r om   ur ate  nephr opathy   m ay   ex i st  at  the  ti m e  of
pr esentati on  or   m ay   occur   w i th  ther apy ,   ther ef or e  ur i ne  al k al i ni zati on  m ay
pr ev ent  the  need  f or   di al y si s.   Pati ents  shoul d  r ecei v e  al l opur i nol   (300  to  600
m g)  f or   at  l east  24  hour s  bef or e  ther apy   to  r educe  the  ur i c  aci d  l oad,   and  thi s
tr eatm ent  shoul d  be  conti nued  unti l   l euk openi a  and  bone  m ar r ow
hy pocel l ul ar i ty   hav e  been  achi ev ed.

P. 274
6.   What  ar e  i nducti on,   consol i dati on,   m ai ntenance  chem other apy ,   and  m eni ngeal
pr ophy l acti c  ther apy ,   and  how   do  they   di f f er   i n  the  tr eatm ent  of   ALL  and  AN LL?

These  ar e  the  phases  of   ther apy   used  f or   acute  l euk em i a.   Inducti on  ther apy   i s
usual l y   the  i ni ti al   ther apy   and  i s  i ntended  to  accom pl i sh  com pl ete  r em i ssi on
(that  i s,   no  si gns  or   sy m ptom s  of   di sease,   nor m al   bl ood  counts,   and  no
ev i dence  of   l euk em i a,   i . e. ,   < 5%  bl asts  i n  the  bone  m ar r ow ).   Thi s  ther apy   i s
usual l y   adm i ni ster ed  on  an  i npati ent  basi s,   and  i s  v er y   tox i c.   Consol i dati on
ther apy   i s  gi v en  af ter   com pl ete  r em i ssi on  i s  achi ev ed.   It  i s  si m i l ar l y   tox i c,   and
consi sts  of   ei ther   the  sam e  dr ugs  as  those  used  i n  i nducti on  ther apy   or
di f f er ent  ones.   Its  object  i s  to  r educe  the  now   cl i ni cal l y   undetectabl e  l euk em i c
cel l   m ass  as  m uch  as  possi bl e.   Mai ntenance  ther apy   i s  usual l y   gi v en  on  an
outpati ent  basi s  and  i s  l ess  tox i c,   al though  com pl i cati ons  of   ther apy   can  and  do
ar i se.   Thi s  phase  usual l y   l asts  f or   2  to  3  y ear s.   Meni ngeal   pr ophy l acti c  ther apy
i s  gi v en  by   m eans  of   l um bar   punctur e  or   thr ough  a  r eser v oi r   pl aced  under   the
scal p  that  cannul ates  the  thi r d  v entr i cl e.   Its  goal   i s  to  r educe  the  r ecur r ence
r ate  of   l euk em i a  i n  the  centr al   ner v ous  sy stem   (CN S),   w hi ch  i s  consi der ed  a
sanctuar y   si te.
Al l   f our   ther apy   phases  ar e  used  i n  ALL.   In  the  tr eatm ent  of   AN LL,   ther e  i s
contr ov er sy   ov er   the  use  of   m ai ntenance  ther apy ,   al though  a  second
consol i dati on  phase  m ay   be  used.   Meni ngeal   pr ophy l ax i s  i s  not  used  i n  the
tr eatm ent  of   adul t  AN LL.   How ev er ,   CN S  l euk em i a  i s  m or e  com m on  i n  chi l dhood
AN LL,   and  pr ophy l ax i s  i s  som eti m es  used  i n  thi s  setti ng.   In  gener al ,   the
r esponse  to  tr eatm ent  and  the  pr ognosi s  ar e  better   i n  pati ents  w i th  ALL  than  i n
those  w i th  AN LL.

7.   What  ar e  the  r i sk s  associ ated  w i th  anti l euk em i c  ther apy ,   and  w hat  r esul ts  can
be  ex pected?

As  al r eady   noted,   acute  l euk em i a  i s  usual l y   a  r api dl y   pr ogr essi v e  di sease  that
i s  f atal   w i thout  ther apy .   Because  the  ther apy   i tsel f   i s  tox i c,   the  m or tal i ty   r ate
dur i ng  i nducti on  ther apy   f or   AN LL  m ay   r each  as  hi gh  as  20%.   Som e  tox i ci ti es
ar e  speci f i c  to  the  dr ug  used,   and  these  ar e  not  di scussed  her e.   N ear l y   al l
ther api es  pr ov ok e  nausea  and  v om i ti ng,   w hi ch  can  be  contr ol l ed  w i th
m edi cati ons.   Mor e  si gni f i cantl y ,   anti l euk em i c  ther apy   i s  i ntended  to  depl ete
the  bone  m ar r ow ,   w i th  subsequent  r epopul ati on  by   nor m al   cel l s.   Dur i ng  thi s
per i od  of   depl eti on,   the  pati ent  becom es  sev er el y   thr om bocy topeni c  and  m ust
be  suppor ted  by   pl atel et  tr ansf usi ons  (gi v en  pr ophy l acti cal l y   at  v ar i ous
i nter v al s  to  k eep  the  pl atel et  count  abov e  10, 000)  and,   usual l y ,   al so  by   r ed
bl ood  cel l   tr ansf usi ons.

Pati ents  al so  becom e  sev er el y   l euk openi c,   and  thi s  m ak es  them   v er y
suscepti bl e  to  i nf ecti on.   The  ty pi cal   si gns  and  sy m ptom s  of   i nf ecti on  (pus  and
pur ul ent  sputum )  ar e  of ten  due  to  the  acti ons  of   gr anul ocy tes,   so  i nf ecti on  i s
of ten  subtl e.   The  or al   m ucosa  and  per i r ectal   ar eas  ar e  com m onl y   ov er l ook ed
si tes  of   i nf ecti on.   Fev er   i n  a  neutr openi c  pati ent  m ust  be  consi der ed  i nf ecti ous
by   or i gi n,   unti l   pr ov ed  other w i se.   When  thi s  happens,   ex am i nati on  and  cul tur es
shoul d  be  car r i ed  out  and  br oad­spectr um   anti bi oti c  ther apy   star ted  qui ck l y .
Anti f ungal   agents  ar e  usual l y   added  i f   no  i m pr ov em ent  i s  seen  af ter   4  to  7
day s  of   f ev er .   The  pati ent  m ust  be  m oni tor ed  car ef ul l y   and  tr eated  f or   her pes
v i r us  i nf ecti on  because  di ssem i nated  i nf ecti on  can  be  r api dl y   f atal .

P. 275
If   the  l euk em i c  cel l   bur den  i s  gr eat,   anti l euk em i c  ther apy   m ay   pr eci pi tate  the
tum or   l y si s  sy ndr om e,   caused  by   the  r api d  r el ease  of   cel l   degr adati on
pr oducts.   It  i s  char acter i zed  by   hy per ur i cem i a  (causi ng  ur ate  nephr opathy ),
hy per k al em i a,   hy per phosphatem i a,   and  hy pocal cem i a.   Adv ance  r ecogni ti on  of
pati ents  at  r i sk   and  subsequent  tr eatm ent  w i th  v i gor ous  hy dr ati on,   al l opur i nol ,
and  ur i ne  al k al i ni zati on  24  to  48  hour s  bef or e  the  star t  of   chem other apy   can
usual l y   pr ev ent  the  sy ndr om e.   These  pati ents  m ust  hav e  thei r   el ectr ol y te,   ur i c
aci d,   phosphor us,   cal ci um ,   and  cr eati ni ne  status  r epeatedl y   check ed.   Any
m etabol i c  abnor m al i ti es  shoul d  be  cor r ected  and,   i f   necessar y ,   r enal   di al y si s
i nsti tuted  ear l y .   Once  the  l euk em i c  cel l   bur den  i s  decr eased  and  degr adati on
pr oducts  cl ear ed,   the  sy ndr om e  r esol v es.

Most  chi l dr en  w i th  ALL  r espond  to  ther apy   and  achi ev e  l ong­ter m   sur v i v al .
Al though  90%  of   adul ts  w i th  ALL  ex per i ence  com pl ete  r em i ssi on  w i th  i ni ti al
ther apy ,   the  m edi an  r em i ssi on  dur ati on  r anges  f r om   48  to  60  m onths,
dependi ng  on  the  study .   Medi an  sur v i v al   i s  3  to  5  y ear s.   How ev er ,
appr ox i m atel y   one  thi r d  of   al l   pati ents  achi ev e  l ong­ter m   di sease­f r ee
sur v i v al .   Late  r ecur r ences  ar e  r ar e.
Pati ents  w i th  AN LL  f ace  a  w or se  pr ognosi s.   Appr ox i m atel y   75%  ex per i ence
com pl ete  r em i ssi on,   but  m ost  cases  r ecur   w i thi n  36  m onths.   Of   those  w ho
achi ev e  com pl ete  r em i ssi on,   20%  to  25%  show   l ong­ter m   di sease­f r ee
sur v i v al .   Bone  m ar r ow   or   stem   cel l   tr anspl antati on  w i th  hi gh­dose
chem other apy   i s  of ten  used,   but  i s  sti l l   under   i nv esti gati on  as  a  ther apy   af ter
the  i ni ti al   chem other apy   i n  ALL.   The  ti m i ng  of   tr anspl antati on  (f i r st  r em i ssi on,
f i r st  r el apse,   or   second  r em i ssi on),   especi al l y   i n  ALL,   i s  contr ov er si al .   In  AN LL,
bone  m ar r ow   tr anspl antati on  (bone  m ar r ow   r escue)  w i th  hi gh­dose
chem other apy   af ter   a  f i r st  r em i ssi on  has  been  associ ated  w i th  hi gher   l ong­
ter m   sur v i v al   r ates.   Ol der   age  (> 40  y ear s),   use  of   unr el ated  donor s,   and
ev i dence  f or   r esi dual   di sease  at  the  ti m e  of   tr anspl antati on  r educe  the  ef f i cacy
of   thi s  tr eatm ent  appr oach.

Case
A  63­y ear ­ol d  w hi te  m an  i s  seen  i n  the  em er gency   r oom   w i th  com pl ai nts  of   f ev er ,
f ati gue,   and  m al ai se.   He  r epor ts  hav i ng  i nter m i ttent  epi stax i s  dur i ng  the  l ast  w eek ,
m outh  sor es  f or   the  l ast  3  day s,   and  a  nonpr ur i ti c  r ash  ov er   hi s  l ow er   ex tr em i ti es,
w hi ch  w as  noted  24  hour s  bef or e.   He  has  ex per i enced  m i dchest  pai n  f or   the  l ast
day ,   onl y   on  sw al l ow i ng.   He  deni es  chem i cal ,   dr ug,   or   r adi ati on  ex posur e.
Phy si cal   ex am i nati on  r ev eal s  a  tem per atur e  of   38. 6°C  (101. 48°F).   He  has  m i l d
tachy car di a,   at  108  beats  per   m i nute.   Head,   ey es,   ear s,   nose,   and  thr oat  f i ndi ngs
consi st  of   a  f ew   petechi ae  ov er   the  sof t  pal ate.   Mul ti pl e  w hi te  pl aques  ar e  seen  on
the  or al   m ucosa,   and  ther e  i s  hy per tr ophy   of   the  gi ngi v ae.   Dur i ng  ex am i nati on  of
the  sk i n,   petechi ae  ar e  f ound  ov er   the  di stal   l ow er   ex tr em i ti es.   Other   ex am i nati on
f i ndi ngs  ar e  nor m al .   Speci f i cal l y ,   no  l y m phadenopathy   or   hepatospl enom egal y   ar e
f ound.   Other   si tes  of   possi bl e  i nf ecti on,   i ncl udi ng  the  chest  and  per i r ectal   ar ea,   ar e
cl ear .   The  chest  r adi ogr aphi c  study   i s  l i k ew i se  nor m al .

Labor ator y   f i ndi ngs  ar e  as  f ol l ow s:  w hi te  bl ood  cel l   count,   17, 200/m m 3   w i th  2%
pol y m or phonucl ear   l euk ocy tes,   1%  band  f or m s,   16%  l y m phocy tes,   4%  m onocy tes,
5%
P. 276
m etam y el ocy tes,   4%  basophi l s,   and  68%  bl astocy tes;  hem ogl obi n,   11. 1  g/dL;
hem atocr i t,   32. 6%;  and  pl atel ets,   14, 000/m m 3 .   Hi s  el ectr ol y te,   bl ood  ur ea  ni tr ogen
(BU N ),   cr eati ni ne,   and  am i notr ansf er ase  l ev el s  ar e  nor m al .   Hi s  ur i c  aci d  l ev el   i s
m i l dl y   i ncr eased  at  9. 2  m g/dL  (nor m al ,   3. 5  to  8. 0  m g/dL),   as  ar e  hi s  LDH  l ev el   at
373  IU /L  (nor m al ,   30  to  220  IU /L).   Ex am i nati on  of   a  per i pher al   bl ood  sm ear   r ev eal s
occasi onal   nucl eated  r ed  bl ood  cel l s,   f ew   pl atel ets,   and  m any   l ar ge  cel l s  contai ni ng
f i nel y   r eti cul ated  nucl ei ,   sev er al   nucl eol i ,   cy topl asm i c  gr anul es,   and  occasi onal
Auer   r ods.   Lar ge  cel l s  w i th  f ol ded  nucl ei   and  l ar ge,   pr om i nent  nucl eol i   ar e  al so
seen.

1.   What  i s  the  m ost  l i k el y   di agnosi s  i n  thi s  pati ent?


2.   How   i s  the  absol ute  neutr ophi l   count  (AN C)  cal cul ated,   and  w hat  i s  i t  i n  thi s
pati ent?
3.   Of   w hat  i m por tance  i s  the  AN C?
4.   Do  the  ev al uati on  f i ndi ngs  poi nt  to  any   speci f i c  i nf ecti ons?
5.   What  w oul d  y ou  ex pect  thi s  pati ent's  bone  m ar r ow   to  show ?
6.   Shoul d  a  l um bar   punctur e  be  per f or m ed  i n  thi s  pati ent?
Case Discussion
1.   What  i s  the  m ost  l i k el y   di agnosi s  i n  thi s  pati ent?

Consi der i ng  the  r esul ts  of   thi s  pati ent's  com pl ete  bl ood  count  and  per i pher al
bl ood  sm ear ,   he  has  AN LL.   The  gr anul ar   m y el ocy tes  and  m onocy tes  i n  the
sm ear   and  the  cl i ni cal   ev i dence  of   ex tr am edul l ar y   l euk em i c  i nf i l tr ati on
(gi ngi v al   hy per tr ophy )  poi nt  to  a  di agnosi s  of   M4,   or   acute  m y el om onocy ti c
l euk em i a.   Ex am i nati on  of   bone  m ar r ow   speci m ens  usi ng  speci al   stai ns  and
chr om osom al   anal y si s  can  hel p  conf i r m   thi s  di agnosi s.

2.   How   i s  the  AN C  cal cul ated,   and  w hat  i s  i t  i n  thi s  pati ent?

To  cal cul ate  the  AN C,   m ul ti pl y   the  total   w hi te  bl ood  cel l   count  by   the
per centage  of   pol y m or phonucl ear   l euk ocy tes  pl us  the  per centage  of   band
f or m s.   In  thi s  case,   the  pati ent  has  17, 200  w hi te  bl ood  cel l s,   w i th  2%
pol y m or phonucl ear   l euk ocy tes  and  1%  band  f or m s,   or :  17, 200  (0. 02  +   0. 01)  =
516  absol ute  neutr ophi l s.

3.   Of   w hat  i m por tance  i s  the  AN C?

The  AN C  f ur ni shes  a  r ough  esti m ate  of   the  pati ent's  abi l i ty   to  f i ght  i nf ecti on.   A
pati ent  w i th  an  AN C  of   l ess  than  500  i s  consi der ed  neutr openi c  and  v er y
suscepti bl e  to  ov er w hel m i ng  i nf ecti on.   Thi s  pati ent,   w i th  an  AN C  of
appr ox i m atel y   500,   f ev er ,   and  a  pr esum ed  di agnosi s  of   acute  l euk em i a,   f al l s
i nto  thi s  categor y .   Car ef ul   ex am i nati on,   together   w i th  cul tur es  of   bl ood,
sputum ,   or al   l esi ons,   and  other   possi bl e  si tes  of   i nf ecti on,   shoul d  be  done
qui ck l y ,   and  the  pati ent  star ted  on  br oad­spectr um   anti bi oti cs  i m m edi atel y .
Any   del ay   i n  the  w or k up  or   i nsti tuti on  of   anti bi oti cs  m ay   r esul t  i n
ov er w hel m i ng  and  possi bl y   f atal   i nf ecti on.   Cul tur es  ar e  of ten  negati v e  i n
neutr openi c  pati ents,   al though  cl i ni cal l y   they   appear   to  be  septi c  and  r espond
to  anti bi oti cs.

4.   Do  the  ev al uati on  f i ndi ngs  poi nt  to  any   speci f i c  i nf ecti ons?

Thi s  pati ent  com pl ai ns  of   m i dchest  pai n  on  sw al l ow i ng  and  phy si cal
ex am i nati on  r ev eal s  w hi te  or al   pl aques.   A  pr esum pti v e  di agnosi s  of   Candi da
esophagi ti s  can  be
P. 277
m ade  on  the  basi s  of   these  f i ndi ngs,   and  the  pati ent  shoul d  be  star ted  on
anti f ungal   agents  as  w el l   as  br oad­spectr um   anti bacter i al   anti bi oti cs.
N eutr openi c  pati ents  ar e  suscepti bl e  to  oppor tuni sti c  i nf ecti ons,   and
candi di asi s  i s  v er y   com m on  i n  them .

5.   What  w oul d  y ou  ex pect  thi s  pati ent's  bone  m ar r ow   to  show ?

The  bone  m ar r ow   i n  thi s  pati ent  w i th  AN LL  w oul d  l i k el y   ex hi bi t


hy per cel l ul ar i ty ,   w i th  cel l ul ar   el em ents  of ten  consti tuti ng  90%  or   m or e  of   the
m ar r ow .   The  num ber s  of   r ed  bl ood  cel l   pr ecur sor s  and  m egak ar y ocy tes  w i l l   be
decr eased.   The  m or phol ogy   m ay   be  nor m al ,   or   ther e  m ay   be  dy ser y thr opoi esi s
(asy nchr onous  m atur i ng  of   the  nucl ear   and  cy topl asm i c  el em ents).   The  m ar r ow
w i l l   pr i m ar i l y   show   a  m onotonous  patter n  of   cel l s  si m i l ar   to  those  seen  i n  the
per i pher al   sm ear .   Fl ow   cy tom etr y   shoul d  show   cel l   sur f ace  m ar k er s  i ndi cati v e
of   i m m atur e  m y el oi d  cel l s  w i th  m onocy toi d  char acter i sti cs.   The  chr om osom e
anal y si s  m ay   show   an  abnor m al i ty   such  as  m onosom y   7  (especi al l y   i f   the
pati ent  had  m y el ody spl asi a),   but  w i l l   not  show   the  abnor m al i ti es  associ ated
w i th,   f or   ex am pl e,   M3  l euk em i a  (Tabl e  7­1).   Recent  studi es  suggest  com pl ex
k ar y oty pes  i n  pati ents  ol der   than  60  y ear s,   that  i s,   thr ee  or   m or e  aber r ati ons
hav e  decr eased  r esponse  to  ther apy   and  based  on  com or bi d  f actor s  these
pati ents  shoul d  be  consi der ed  f or   i nv esti gati onal   ther apy   or   suppor ti v e  car e.

6.   Shoul d  a  l um bar   punctur e  be  per f or m ed  i n  thi s  pati ent?

Thi s  pati ent  has  a  pr esum pti v e  di agnosi s  of   acute  m y el om onocy ti c  l euk em i a.
Lum bar   punctur es  ar e  r outi nel y   done  i n  cases  of   ALL  and  AN LL­M4  because
these  l euk em i as  ar e  associ ated  w i th  m eni ngi ti s.   N ev er thel ess,   any   pati ent  w i th
acute  l euk em i a  and  sy m ptom s  of   m eni ngi ti s  or   cr ani al   ner v e  pal si es  shoul d
under go  a  di agnosti c  l um bar   punctur e,   r egar dl ess  of   the  l euk em i c  ty pe.

How ev er ,   the  pl atel et  count  i n  thi s  pati ent  i s  onl y   14, 000/m m 3 ,   and  l um bar
punctur es  shoul d  not  be  per f or m ed  w hen  the  pl atel et  count  i s  l ess  than
50, 000/m m 3   because  of   the  r i sk   of   hem or r hage.   Ther ef or e,   pl atel et
tr ansf usi ons  m ust  be  gi v en  bef or e  attem pti ng  l um bar   punctur e  to  br i ng  the
count  to  50, 000/m m 3   or   m or e.

Suggested Readings
Baccar ani   M,   Car bel l i   G,   Am ador i   S,   et  al .   Adol escent  and  adul t  acute
l y m phobl asti c  l euk em i a:  pr ognosti c  f eatur es  and  outcom e  of   ther apy —  a  study
of   293  pati ents.   Bl ood  1982;60:677.

Bennett  JM,   Young  ML,   Ander son  JW,   et  al .   Long­ter m   sur v i v al   i n  acute  m y el oi d
l euk em i a.   Cancer   1997;8:2205.

Bur nett  A,   Gol dstone  AH,   Stev ens  RMF,   et  al .   Random i zed  com par i son  of
addi ti on  of   autol ogous  bone­m ar r ow   tr anspl antati on  to  i ntensi v e  r em i ssi on:
r esul ts  of   MRC  AML  10  tr i al .   Lancet  1998;351:700.

Far ag  SS,   Ar cher   KJ,   Mr ozek   K,   et  al .   Pr etr eatm ent  cy togeneti cs  add  to  other
pr ognosti c  f actor s  pr edi cti ng  com pl ete  r em i ssi on  and  l ong­ter m   outcom e  i n
pati ents  60  y ear s  of   age  or   ol der   w i th  acute  m y el oi d  l euk em i a:  r esul ts  f r om
Cancer   and  Leuk em i a  Gr oup  B  8461.   Bl ood  2006;108:63.

Gal e  RP,   Hoel zer   D.   Acute  l y m phobl asti c  l euk em i a.   N ew   Yor k :  Wi l ey ­Li ss,   1990.

Koef f l er   HP.   Sy ndr om es  of   acute  nonl y m phocy ti c  l euk em i a.   Ann  Inter n  Med
1987;107:748.

P. 278

Anemia
1.   What  i s  the  def i ni ti on  of   anem i a,   and  w hat  i s  the  di f f er enti al   di agnosi s  based
on  the  m ean  cor puscul ar   v ol um e  (MCV)?
2.   Why   i s  i t  i m por tant  to  ex am i ne  the  per i pher al   bl ood  sm ear ,   and  w hat  ar e  the
m any   di agnosti c  er y thr ocy te  abnor m al i ti es  and  cor r espondi ng  cl i ni cal
condi ti ons?

3.   What  i s  a  r eti cul ocy te,   and  how   i s  the  r eti cul ocy te  count  used  to  char acter i ze
an  anem i a?  What  i s  the  r eti cul ocy te  i ndex ,   how   i s  i t  cal cul ated,   and  how   i s  i t
used  i n  the  di f f er enti al   di agnosi s  of   anem i a?

4.   What  i s  the  di f f er ence  betw een  α ­  and  β­thal assem i a,   how   ar e  they
di sti ngui shed  cl i ni cal l y ,   and  how   i s  el ectr ophor esi s  usef ul ?

5.   What  i s  si ck l e  cel l   anem i a,   and  how   i s  i t  m ani f ested  cl i ni cal l y ?  What  i s  the
si ck l e  cel l   tr ai t,   and  how   i s  i t  m ani f ested  cl i ni cal l y ?

Discussion
1.   What  i s  the  def i ni ti on  of   anem i a,   and  w hat  i s  the  di f f er enti al   di agnosi s  based
on  the  MCV?

Anem i a  i s  usual l y   def i ned  as  an  abnor m al l y   l ow   hem atocr i t  or   hem ogl obi n
concentr ati on,   and  occur s  w hen  the  r ate  of   er y thr ocy te  l oss  ex ceeds  the  r ate
of   er y thr ocy te  pr oducti on.   The  di f f er enti al   di agnosi s  of   anem i a  depends  on
w hether   the  MCV  i s  l ow ,   hi gh,   or   nor m al .   Tabl e  7­2  l i sts  the  v ar i ous  possi bl e
di agnoses  f or   each  of   these  categor i es.   Som eti m es  w i th  m i l d  anem i a,   a
di agnosi s  m ay   be  enter tai ned  i f   the  MCV  i s  i n  the  hi gh  or   l ow   r ange  of   nor m al .

2.   Why   i s  i t  i m por tant  to  ex am i ne  the  per i pher al   bl ood  sm ear ,   and  w hat  ar e  the
m any   di agnosti c  er y thr ocy te  abnor m al i ti es  and  cor r espondi ng  cl i ni cal
condi ti ons?

Per i pher al   bl ood  sm ear   ex am i nati on  can  r ev eal   er y thr ocy te  abnor m al i ti es  that
poi nt  to  the  cor r ect  di agnosi s  of   the  anem i a.   Echi nocy tes,   or   bur r   cel l s,   f or
ex am pl e  ar e  seen  i n  ur em i a  and  py r uv ate  k i nase  def i ci ency .   El l i ptocy tes  ar e
the  abnor m al   er y thr ocy tes  seen  i n  pati ents  w i th  her edi tar y   el l i ptocy tosi s.
P. 279
N ucl eated  r ed  cel l s  ar e  f ound  i n  the  setti ng  of   str ess  or   hem atol ogi c  di sease
w i th  bone  m ar r ow   i nv ol v em ent.   Schi stocy tes  or   f r agm ents  occur   i n  pati ents
w i th  m i cr oangi opathi c  hem ol y ti c  anem i a.   Si ck l e  cel l s  ar e  f ound  i n  the  setti ng
of   si ck l e  cel l   anem i a.   Spher ocy tes  occur   i n  i m m une­m edi ated  hem ol y ti c
anem i a  and  her edi tar y   spher ocy tosi s.   Tar get  cel l s  f or m   i n  the  pr esence  of   l i v er
di sease  and  i r on  def i ci ency ;  they   al so  occur   af ter   spl enectom y .

Table 7­2 Differential Diagnosis of Anemia Based
on Mean Corpuscular Volume (MCV)

Low  MCV Norma l MCV High MCV

α ­Thal assem i a Acute  bl ood  l oss Al cohol   abuse

β­Thal assem i a Apl asti c  anem i a Apl asti c  anem i a


Ir on  def i ci ency Chr oni c  di sease Cobal am i n  def i ci ency

Lead  poi soni ng Com bi nati on  of Fol ate  def i ci ency
m acr ocy ti c  and

Si der obl asti c m i cr ocy ti c  causes Hem ol y si s


anem i a

  Hem ogl obi nopathy Hy pothy r oi di sm

  Hem ol y si s Li v er   di sease

Ir on  def i ci ency My el ody spl asti c


 
sy ndr om es

3.   What  i s  a  r eti cul ocy te,   and  how   i s  the  r eti cul ocy te  count  used  to  char acter i ze
an  anem i a?  What  i s  the  r eti cul ocy te  i ndex ,   how   i s  i t  cal cul ated,   and  how   i s  i t
used  i n  the  di f f er enti al   di agnosi s  of   anem i a?

A  r eti cul ocy te  i s  a  y oung  ci r cul ati ng  r ed  bl ood  cel l   that  ex hi bi ts  basophi l i a
under   v i tal   stai ni ng.   The  r eti cul ocy te  count  i s  used  to  char acter i ze  the  bone
m ar r ow 's  attem pt  to  com pensate,   i f   at  al l ,   f or   the  anem i a  pr esent.   The
r eti cul ocy te  i ndex   (Tabl e  7­3)  i s  a  m or e  usef ul   m eans  of   char acter i zi ng  anem i a
because  i t  i s  deter m i ned  by   cor r ecti ng  the  r eti cul ocy te  count  f or   the
hem atocr i t,   assum i ng  a  nor m al   hem atocr i t  i s  45%.   Thi s  cor r ecti on  i s  necessar y
because  r eti cul ocy tes  ar e  counted  per   1, 000  r ed  bl ood  cel l s.

An  i ndex   of   l ess  than  2  i s  f ound  i n  the  setti ng  of   the  hypoprolife ra tive
a ne mia s .   These  consi st  of   di sor der s  of   hem e  or   gl obi n  sy nthesi s,   such  as  i r on
def i ci ency ,   anem i a  stem m i ng  f r om   chr oni c  di sease,   l ead  poi soni ng,
si der obl asti c  anem i as,   and  α ,   β,   and  other   thal assem i as;  m egal obl asti c
anem i as  r esul ti ng  f r om   cobal am i n  or   f ol ate  def i ci ency ;  m y el ody spl asti c
sy ndr om es;  apl asti c  anem i as;  and  other   m etabol i c  causes,   such  as  r enal
i nsuf f i ci ency   and  hy pothy r oi di sm .

Hype rprolife ra tive  a ne mia s   ar e  associ ated  w i th  a  r eti cul ocy te  i ndex   gr eater
than  2.   These  anem i as  ar i se  as  the  r esul t  of   acute  bl ood  l oss;  nutr i ent
r epl acem ent,   such  as  cobal am i n,   f ol ate,   or   i r on  r epl acem ent,   but  bef or e  the
r esol uti on  of   anem i a;  both  her edi tar y   and  acqui r ed  hem ol y si s;  and  pr i m ar y   or
secondar y   pol y cy them i a.

Autom ated  r eti cul ocy te  counts  i ntr oduced  f or   gener al   cl i ni cal   pr acti ce  ar e  m or e
accur ate  than  “hand  countsâ€​   and  autom ati cal l y   cal cul ate  the  r eti cul ocy te
i ndex .   These  autom ated  v al ues  al so  i ncl ude  the  total   num ber   of   r eti cul ocy tes,
a  m easur em ent  that  m i ght  be  hel pf ul   w hen  obtai ni ng  ser i al   v al ues.
Table 7­3 The Reticulocyte Index

He ma toc rit (% ) Corre c tion Fa c tor

45 1. 0

35 1. 5

25 2. 0

Cor r ecti on  of   the  r eti cul ocy te  i ndex   f or   shi f t  cel l s:  shi f t  cel l s­new l y
r el eased  er y thr ocy tes.
N ew er   autom ated  sy stem s  that  w i l l   becom e  av ai l abl e  w i l l   gi v e
cor r ected  r eti cul ocy te  count  and  a  r eti cul ocy te  m atur ati on  i ndex   to
account  f or   “shi f t  cel l s. â€​

P. 280
4.   What  i s  the  di f f er ence  betw een  α ­  and  β­thal assem i a,   how   ar e  they
di sti ngui shed  cl i ni cal l y ,   and  how   i s  el ectr ophor esi s  usef ul ?

The  α ­tha la s s e mia s   consti tute  abnor m al i ti es  of   the  gene,   or   genes,
r esponsi bl e  f or   the  sy nthesi s  of   the  α   chai n  of   hem ogl obi n.   Hum ans  contai n
f our   genes  f or   thi s  pur pose  and  each  i s  r esponsi bl e  f or   appr ox i m atel y   a  f our th
of   the  α   chai ns  sy nthesi zed.   Any   com bi nati on  of   f r om   one  to  f our   of   these  α
genes  m ay   be  m i ssi ng.   Thal assem i a  i s  unappar ent  cl i ni cal l y   w hen  onl y   one
gene  i s  m i ssi ng,   and  thi s  i s  cal l ed  α 1 ­thal assem i a.   Thi s  def ect  ex i sts  i n  up  to
30%  of   the  Am er i can  bl ack   popul ati on.   If   tw o  of   the  α   genes  ar e  m i ssi ng,   the
enti ty   i s  r ef er r ed  to  as  α 2 ­thal assem i a.   These  pati ents  ar e  usual l y
asy m ptom ati c,   al though  thei r   hem atocr i t  and  MCV  m ay   be  sl i ghtl y   l ow .   Thi s
def ect  af f ects  appr ox i m atel y   2%  of   Af r i can  Am er i cans.   When  thr ee  of   the  α
genes  ar e  l ack i ng,   the  pati ent  ex hi bi ts  the  phenoty pe  of   α ­thal assem i a
(Hem ogl obi n  H  di sease)  w i th  a  l ow   hem atocr i t  and  MCV,   and  β­chai n  tetr am er s
or   hem ogl obi n  H  i s  f ound  i n  the  r ed  bl ood  cel l s.   When  al l   f our   α   genes  ar e
m i ssi ng,   the  r esul t  i s  usual l y   a  sti l l bor n  i nf ant  w i th  hy dr ops  f etal i s.

α ­Thal assem i a  i s  the  m ost  com m on  f or m   of   thal assem i a  i n  the  Southeast
Asi an  popul ati on.

The  β­tha la s s e mia s   consi st  of   abnor m al i ti es  of   the  gene,   or   genes,
r esponsi bl e  f or   the  β  chai n  of   hem ogl obi n,   and  they   cause  i nsuf f i ci ent  β­chai n
sy nthesi s.   Thi s  l eads  to  the  f or m ati on  of   α ­chai n  tetr am er s  and  i ncl usi ons  of
thi s  hem ogl obi n  attached  to  the  pl asm a  m em br anes  of   er y thr ocy tes,   r esul ti ng
i n  hem ol y si s.   Pati ents  w i th  heter ozy gous  β­thal assem i a  ex hi bi t  a  m odest
decr ease  i n  thei r   hem atocr i t  v al ues  and  a  m ar k ed  decr ease  i n  thei r   MCVs.
Pati ents  w i th  hom ozy gous  β­thal assem i a  hav e  sev er e  anem i a  and  l ow   MCVs.
They   r equi r e  tr ansf usi on,   and  com pl i cati ons  m ay   ar i se  stem m i ng  f r om   the
ex cess  accum ul ati on  of   i r on.

El ectr ophor esi s  m ay   be  used  to  suggest  the  di agnosi s  of   α ­thal assem i a  i n
pati ents  m i ssi ng  thr ee  genes,   and  ther eby   hav i ng  suf f i ci ent  f ast­m i gr ati ng
hem ogl obi n  H  (α l ­  and  α 2 ­thal assem i a  tr ai ts  m ay   not  be  detected).   The
pr eci se  num ber   of   m i ssi ng  α   genes  can  be  deter m i ned  i n  hy br i di zati on  studi es
thr ough  the  use  of   a  com pl em entar y   DN A  pr obe.

The  f i ndi ngs  y i el ded  by   hem ogl obi n  el ectr ophor esi s  ar e  usual l y   di agnosti c  i n
the  setti ng  of   β­thal assem i a.   Because  α ­chai n  sy nthesi s  i s  nor m al   i n  these
pati ents,   the  other   hem ogl obi ns  seen  i n  adul ts,   i ncl udi ng  hem ogl obi n  A 2   and  F,
ar e  i ncr eased  i n  a  com pensator y   m anner .   Ther ef or e,   pati ents  w i th
heter ozy gous  β­thal assem i a  w oul d  hav e  el ev ated  hem ogl obi n  A 2   and  F  l ev el s
w i th  hem ogl obi n  A  pr esent.   Pati ents  w i th  hom ozy gous  β­thal assem i a  w oul d
hav e  no  hem ogl obi n  A  and  m ar k edl y   el ev ated  hem ogl obi n  F  and  A 2   l ev el s.

5.   What  i s  si ck l e  cel l   anem i a,   and  how   i s  i t  m ani f ested  cl i ni cal l y ?  What  i s  the
si ck l e  cel l   tr ai t,   and  how   i s  i t  m ani f ested  cl i ni cal l y ?

Si ck l e  cel l   di sease  i s  the  m ost  com m onl y   r ecogni zed  cl i ni cal l y   si gni f i cant
hem ogl obi nopathy .   It  stem s  f r om   a  substi tuti on  of   v al i ne  f or   gl utam i c  aci d  i n
the  β  chai n  of   hem ogl obi n  and  can  be  di agnosed  by   el ectr ophor esi s.   Si ck l e  cel l
anem i a  r esul ts  w hen  both  β  chai ns  ar e  abnor m al .   Si ck l ed  cel l s  shoul d  be
ev i dent  on
P. 281
a  per i pher al   bl ood  sm ear .   Hem ogl obi n  S  i s  l ess  sol ubl e  than  nor m al
hem ogl obi n  at  a  l ow   ox y gen  tensi on,   causi ng  the  hem ogl obi n  m ol ecul es  to
cr y stal l i ze,   w hi ch  def or m s  the  r ed  bl ood  cel l s.   These  m i sshapen  cel l s  gr eatl y
i ncr ease  the  bl ood  v i scosi ty ,   w hi ch  l eads  to  sm al l ­v essel   occl usi on  and  hence
pai n  and  or gan  i nf ar cti ons,   speci f i cal l y   str ok e  as  w el l   as  pul m onar y ,   r enal ,   and
bone  i nf ar cti on.

Si ck l e  cel l   di sease  m ay   be  m ani f ested  by   a  v ar i ety   of   cr i ses:  P a in  i s  the  m ost
com m on  sy m ptom ,   and  i s  thought  to  be  secondar y   to  r ed  bl ood  cel l   sl udgi ng
and  i nf ar cti on.   Sple nic  s e que s tra tion a nd da c tylitis   ar e  com m on  i n  chi l dr en,
but  r ar e  i n  adul ts.   Apla s tic  a ne mia   i s  uncom m on,   but  i s  ty pi cal l y   associ ated
w i th  i nf ecti ons,   and  i s  anti ci pated  w hen  r eti cul ocy te  counts  decr ease  i n  the
f ace  of   w or seni ng  of   anem i a.   Me ga lobla s tic  a ne mia   i s  usual l y   secondar y   to
f ol ate  def i ci ency ,   and  ar i ses  because  abnor m al   cel l s  hav e  a  shor tened  l i f e
span.   Thi s  i ncr eases  the  tur nov er   of   r ed  bl ood  cel l s  and  pl aces  an  i ncr eased
dem and  on  f ol ate  stor es.   Si ck l e  cel l   pati ents  w ho  enter   w i th  pai n  cr i si s  or   â
€œchest  sy ndr om eâ€​   ar e  at  r i sk   f or   m ul ti or gan  f ai l ur e.   When  i t  becom es
appar ent  that  l i v er ,   k i dney ,   and/or   pul m onar y   f uncti on  ar e  decl i ni ng,   these
pati ents  shoul d  be  consi der ed  f or   ex change  tr ansf usi on.

The  si ck l e  cel l   tr ai t  i s  al m ost  al w ay s  asy m ptom ati c  because  onl y   one  of   the
tw o  β  chai ns  i s  abnor m al .   It  can  al so  be  di agnosed  by   el ectr ophor esi s,   and
thi s  i s  m ost  i m por tant  f or   the  pur poses  of   geneti c  counsel i ng.

Case 1
A  42­y ear ­ol d  m an  i s  seen  by   hi s  pr i m ar y   car e  phy si ci an  because  of   a  r ectal
ur gency .   On  si gm oi doscopy ,   a  m ass  i s  l ocated  at  8  cm .   He  under goes  r esecti on  to
r em ov e  the  m ass  and  af ter   sur ger y   he  r ecei v es  adjuv ant  chem other apy   and
under goes  pel v i c  r adi ati on  ther apy .   Af ter   he  com pl etes  ther apy ,   he  r etur ns  to  hi s
pr i m ar y   car e  phy si ci an  6  m onths  l ater   w i th  com pl ai nts  of   f ati gue  and  dy spnea  on
ex er ti on.   As  par t  of   the  ev al uati on,   a  com pl ete  bl ood  count  i s  obtai ned  and  r ev eal s
the  f ol l ow i ng  f i ndi ngs:  w hi te  bl ood  cel l   count,   3. 9  ×  10 9 /L;  hem ogl obi n,   8. 2  g/dL;
hem atocr i t,   24. 4%;  MCV,   86  f L;  r eti cul ocy tes,   1%;  and  pl atel ets,   450, 000/m m 3 .   The
pati ent  has  a  ser um   i r on  content  of   23  µg/dL,   a  total   i r on­bi ndi ng  capaci ty   of   256
µg/dL,   and  a  f er r i ti n  l ev el   of   10  ng/m L.

1.   What  i s  the  l i k el y   cause  of   thi s  pati ent's  anem i a,   and  how   w oul d  y ou  ev al uate
hi m   f ur ther ?
2.   If   the  pati ent  i s  i r on  def i ci ent,   w hy   i s  hi s  MCV  86  f L?
3.   On  the  basi s  of   the  pati ent's  i r on  status,   w hat  tr eatm ent  shoul d  be  pr escr i bed,
and  how   shoul d  ther apy   be  m oni tor ed?

Case Discussion
1.   What  i s  the  l i k el y   cause  of   thi s  pati ent's  anem i a,   and  how   w oul d  y ou  ev al uate
hi m   f ur ther ?

The  cause  of   thi s  pati ent's  anem i a  i s  l i k el y   m ul ti f actor i al .   How ev er ,   the  f er r i ti n


l ev el   bel ow   12  ng/m L  and  the  per centage  tr ansf er r i n  satur ati on  (total   i r on­
bi ndi ng  Fe/capaci ty )  bel ow   10%  ar e  both  di agnosti c  f or   i r on  def i ci ency .   He
shoul d  r ecei v e  or al   i r on  suppl em entati on,   but  he  shoul d  be  ev al uated  f or   a
gastr oi ntesti nal   sour ce  of
P. 282
bl ood  l oss  (e. g. ,   r ecur r ent  tum or ,   second  pr i m ar y   cancer ,   or   som e  other
nonm al i gnant  sour ce).

The  pati ent  m ay   al so  be  anem i c  and  l euk openi c  due  to  the  ex tensi v e  ex posur e
of   the  bone  m ar r ow   to  r adi ati on  dur i ng  pel v i c  r adi ati on  ther apy .   Thi s  bone
m ar r ow   dam age  m ay   be  com pounded  by   the  concom i tant  chem other apy
tr eatm ent.

Fi nal l y ,   the  possi bi l i ty   of   other   contr i butor y   f actor s,   such  as  f ol ate  and
cobal am i n  def i ci ency ,   shoul d  al so  be  i nv esti gated.

2.   If   the  pati ent  i s  i r on  def i ci ent,   w hy   i s  hi s  MCV  86  f L?

The  MCV  m ay   be  nor m al   i n  the  setti ngs  of   ear l y   i r on  def i ci ency ,   al though  the
r ed  bl ood  cel l   di str i buti on  w i dth  i s  hi gh  under   these  ci r cum stances.   The  MCV
m ay   al so  be  nor m al   i n  i r on­def i ci ency   anem i a  com pl i cated  by   another
nutr i ti onal   def i ci ency ,   such  as  f ol ate  or   cobal am i n  def i ci ency .   In  thi s  pati ent,
the  MCV  i s  l i k el y   hi gher   than  ex pected  as  a  r esul t  of   hi s  r ecent  chem other apy
tr eatm ent  that  i s  associ ated  w i th  i nhi bi ti on  of   DN A  sy nthesi s.

3.   On  the  basi s  of   the  pati ent's  i r on  status,   w hat  tr eatm ent  shoul d  be  pr escr i bed,
and  how   shoul d  ther apy   be  m oni tor ed?

The  pati ent  has  i r on  def i ci ency .   Fer r ous  sul f ate  (300  m g  thr ee  ti m es  a  day )
pr ov i des  180  m g  of   el em ental   i r on  per   day ,   w hi ch  shoul d  nor m al i ze  the
hem atocr i t  ov er   the  cour se  of   sev er al   m onths.   The  hem atocr i t  shoul d  i ncr ease
by   1%  to  3%  each  w eek   and  hi s  r eti cul ocy te  count  shoul d  al so  i ncr ease
si gni f i cantl y   w i th  thi s  tr eatm ent.

The  status  of   the  absor pti on  of   or al   i r on  can  be  easi l y   dem onstr ated  by
deter m i ni ng  the  f asti ng  ser um   i r on  l ev el   bef or e  and  3  to  4  hour s  af ter   the
i ngesti on  of   a  si ngl e  300­m g  tabl et  of   f er r ous  sul f ate.   If   nor m al ,   the  l ev el
shoul d  r i se  by   a  m i ni m um   of   tw o  ti m es  the  basel i ne  (f asti ng)  v al ue.   If   the
pati ent  has  decr eased  i r on  absor pti on,   that  i s,   due  to  “i nf l am m ati on  bl ock â€​
that  decr eases  absor pti on,   he  shoul d  be  tr eated  w i th  i ntr av enous  i r on.

Case 2
A  67­y ear ­ol d  w om an  i s  seen  f or   com pl ai nts  of   m i l d  m em or y   l oss  and  f ati gue.   On
ev al uati on,   she  i s  f ound  to  hav e  an  anem i a,   w hi ch  i s  char acter i zed  by   the  f ol l ow i ng
l abor ator y   v al ues:  w hi te  bl ood  cel l   count,   5, 200/m m 3 ;  hem ogl obi n,   9. 1  g/dL;
hem atocr i t,   26. 9%;  MCV,   101  f L;  r eti cul ocy tes,   l ess  than  1%;  and  pl atel ets,
154/m m 3 .   Her   ser um   cobal am i n  l ev el   i s  260  pg/m L  and  her   f ol ate,   thy r oi d­
sti m ul ati ng  hor m one,   and  l i v er   f uncti on  tests  ar e  nor m al .   The  pati ent  does  not
abuse  al cohol ,   and  her   per i pher al   bl ood  sm ear   i s  unr ev eal i ng.

1.   How   w oul d  y ou  f ur ther   ev al uate  thi s  pati ent's  anem i a?


2.   On  the  basi s  of   the  l abor ator y   r esul ts  so  f ar ,   w hat  test,   or   tests,   m i ght  be
hel pf ul   i n  di agnosi ng  the  cause  of   thi s  pati ent's  anem i a?
3.   Why   m i ght  such  a  pati ent  be  def i ci ent  i n  cobal am i n?

Case Discussion
1.   How   w oul d  y ou  f ur ther   ev al uate  thi s  pati ent's  anem i a?

Ser um   cobal am i n  and  f ol ate  l ev el s  shoul d  be  deter m i ned.   In  addi ti on,   a  sear ch
f or   both  ethanol   abuse  and  l i v er   di sease  shoul d  be  under tak en  and
hy pothy r oi di sm
P. 283
r ul ed  out.   If   none  of   these  i s  f ound  to  be  a  l i k el y   cause,   other   r easons  f or   the
anem i a  (r ef r actor y   or   apl asti c  anem i a)  shoul d  be  ex pl or ed.   A  per i pher al   bl ood
sm ear   shoul d  be  ex am i ned  f or   possi bl e  cl ues  such  as  hy per segm ented
pol y m or phonucl ear   l euk ocy tes  (seen  i n  cobal am i n  def i ci ency )  or   tar get  cel l s
(seen  i n  l i v er   di sease).

2.   On  the  basi s  of   the  l abor ator y   r esul ts  so  f ar ,   w hat  test,   or   tests,   m i ght  be
hel pf ul   i n  di agnosi ng  the  cause  of   thi s  pati ent's  anem i a?

Thi s  pati ent  l i k el y   has  cobal am i n  def i ci ency ,   al though  her   ser um   cobal am i n
l ev el   of   260  pg/m L  i s  w i thi n  the  nor m al   r ange.   Because  studi es  hav e  show n
that  such  def i ci ency   r esul ts  i n  m ethy l m al oni c  aci dur i a  and  hom ocy sti nem i a,
these  m etabol i c  substr ates  shoul d  be  m easur ed  i n  thi s  pati ent.   Other   testi ng
that  m i ght  be  consi der ed  i ncl udes  a  Schi l l i ng  test  or   m easur em ent  of   anti â
€“i ntr i nsi c  f actor ­bl ock i ng  anti bodi es.

3.   Why   m i ght  such  a  pati ent  be  def i ci ent  i n  cobal am i n?

Ther e  ar e  v ar i ous  causes  of   cobal am i n  def i ci ency .   It  can  stem   f r om   the
i ngesti on  of   i nsuf f i ci ent  ani m al   pr otei n,   as  seen  i n  tr ue  v egetar i ans.   Fai l ur e  to
r el ease  cobal am i n  f r om   f ood  bi nder s  or   f ai l ur e  to  secr ete  i ntr i nsi c  f actor
r esul ts  i n  per ni ci ous  anem i a.   Fai l ur e  to  absor b  the  i ntr i nsi c  f actor –cobal am i n
com pl ex   i n  the  di stal   i l eum ,   as  occur s  i n  pati ents  w ho  hav e  under gone  an  i l eal
r esecti on  or   w ho  hav e  r egi onal   enter i ti s,   can  al so  l ead  to  cobal am i n  def i ci ency .
Rar e  causes  ar e  abnor m al   or   absent  enzy m es  or   tr anspor t  pr otei ns,   and  ni tr ous
ox i de  abuse.

Suggested Readings
Ak ar su  S,   Task i n  E,   Yi l m az  E,   et  al .   Tr eatm ent  of   i r on  def i ci ency   anem i a  w i th
i ntr av enous  i r on  pr epar ati ons.   Acta  Haem atol   2006;116:51.

Beutl er   E,   Li chtm an  MA,   Col ter   BS,   et  al . ,   eds.   Hem atol ogy ,   6th  ed.   N ew   Yor k :
McGr aw ­Hi l l ,   2000.

Wi ntr obe  MM,   ed.   Cl i ni cal   hem atol ogy ,   10th  ed.   Phi l adel phi a:  Lea  &  Febi ger ,
1998.

Bleeding Disorders
1.   What  ar e  the  m ajor   di v i si ons  of   the  coagul ati on  sy stem ?

2.   What  ar e  the  gener al   scr eeni ng  tests  f or   ev al uati ng  each  of   the  m ajor   di v i si ons
of   the  coagul ati on  sy stem ?

3.   What  com m on  di sor der s  ar e  associ ated  w i th  each  of   the  m ajor   di v i si ons  of   the
coagul ati on  sy stem ?

4.   What  ar e  the  cl i ni cal   m ani f estati ons  of   v ar i ous  bl eedi ng  di sor der s?

5.   What  w or k up  i s  i ndi cated  f or   a  bl eedi ng  pati ent?

6.   What  ther api es  ar e  av ai l abl e  f or   the  m anagem ent  of   bl eedi ng  di sor der s?

Discussion
1.   What  ar e  the  m ajor   di v i si ons  of   the  coagul ati on  sy stem ?

The  coagul ati on  sy stem   i s  qui te  com pl ex ,   but  can  be  v i ew ed  as  consi sti ng  of   at
l east  thr ee  m ajor   com ponents:  the  v ascul ar   endothel i um ,   the  bl ood
P. 284
coagul ati on  pr otei ns  (both  those  that  pr om ote  cl otti ng  and  those  that  l y se  cl ots
by   m eans  of   the  f i br i nol y ti c  sy stem ),   and  the  pl atel ets.   The  coagul ati on
cascade  r epr esents  a  ser i es  of   pr otei ns  that,   w hen  i ni ti ated,   f or m s  a  f i br i n
cl ot.   A  si m pl e  outl i ne  of   the  cascade  i s  show n  i n  Tabl e  7­4.   Com pl ex   i ssues
such  as  the  ex act  m echani sm s  by   w hi ch  anti coagul ants,   such  as  pr otei n  C  and
pr otei n  S,   f uncti on  and  how   f actor   VII  m ay   acti v ate  f actor   IX  ar e  not
com pl etel y   under stood.
Table 7­4 The Intrinsic and Extrinsic Pathways of
the Coagulation System

2.   What  ar e  the  gener al   scr eeni ng  tests  f or   ev al uati ng  each  of   the  m ajor   di v i si ons
of   the  coagul ati on  sy stem ?

Vascul ar   endothel i al   i ntegr i ty   can  be  assessed  usi ng  the  bl eedi ng  ti m e.   In  thi s
test,   a  ni ck   i s  m ade  i n  the  sk i n  under   standar di zed  condi ti ons,   and  the  ti m e  to
cessati on  of   bl eedi ng  i s  m easur ed.

The  bl ood  coagul ati on  pr otei ns  ar e  usual l y   ev al uated  by   i n  v i tr o  studi es  usi ng
the  pati ent's  ci tr ate­anti coagul ated  pl asm a.   Thi s  i s  done  by   addi ng  back
v ar i ous  com ponents  of   the  coagul ati on  cascade  to  the  pati ent's  pl asm a  to
i nduce  cl ot,   and  the  pr ocedur e  i s  standar di zed  agai nst  pl asm a  f r om   an
i ndi v i dual   w i th  nor m al   pl asm a  coagul ati on  com ponents.   The  tw o  m ost  com m on
tests  f or   doi ng
P. 285
thi s  ar e  the  pr othr om bi n  ti m e  (PT)  and  the  par ti al   ti ssue  thr om bopl asti n  ti m e
(PTT).   The  PT  m easur es  the  ex tr i nsi c  pathw ay   of   the  coagul ati on  cascade,   and
thi s  i s  done  by   addi ng  ti ssue  thr om bopl asti n  to  the  pati ent's  pl asm a.   If   ther e  i s
a  def i ci t  i n  any   of   the  com m on  pathw ay   com ponents  or   f actor   VII,   the  cl otti ng
ti m e  i s  pr ol onged  abnor m al l y .   The  PTT  m easur es  the  i ntr i nsi c  and  com m on
pathw ay s;  a  def i ci t  i n  the  com m on  or   i ntr i nsi c  pathw ay   pr otei ns  r esul ts  i n  a
pr ol onged  PTT.   A  thi r d,   l ess  com m onl y   used,   scr eeni ng  test  i s  the  thr om bi n
ti m e,   w hi ch  m easur es  onl y   the  l ast  step  i n  the  cascade—  the  conv er si on  of
f i br i nogen  to  f i br i n—  and  i s  done  by   addi ng  thr om bi n  to  the  pati ent's  pl asm a.
Ther ef or e,   i f   the  pati ent  has  too  l i ttl e  f i br i nogen  or   a  dy sf uncti onal   f i br i nogen
pr otei n,   the  ti m e  i s  pr ol onged.   Fi nal l y ,   each  of   the  com ponents  of   the  cascade,
i ncl udi ng  f actor s  I  to  XIII,   can  be  assay ed  di r ectl y   to  ev al uate  f or   def i ci ts.

Pl atel ets  can  be  ev al uated  both  quanti tati v el y   (by   the  pl atel et  count)  and
f uncti onal l y .   Pl atel et  f uncti on  can  be  assessed  by   the  bl eedi ng  ti m e;
qual i tati v el y   def ecti v e  pl atel ets  do  not  f or m   an  adequate  pl atel et  pl ug  and  the
bl eedi ng  ti m e  i s  pr ol onged.   In  addi ti on,   pl atel ets  can  be  anal y zed  i n  v i tr o  f or
thei r   aggr egabi l i ty   usi ng  pl atel et  sti m ul ants  (e. g. ,   r i stoceti n).

3.   What  com m on  di sor der s  ar e  associ ated  w i th  each  of   the  m ajor   di v i si ons  of   the
coagul ati on  sy stem ?

The  v ascul ar   endothel i um   m ay   be  f r agi l e  i n  the  setti ng  of   sev er al   acqui r ed
condi ti ons,   i ncl udi ng  v ascul i ti s  and  l ong­ter m   ster oi d  use.   Thi s  i s  i m por tant  to
r eal i ze  because  i t  m ay   cause  the  bl eedi ng  ti m e  to  be  pr ol onged  despi te  nor m al
pl atel et  num ber   and  f uncti on.

Def i ci ts  i n  the  bl ood  coagul ati on  pr otei ns  m ay   be  congeni tal   or   acqui r ed.   The
m ost  com m on  congeni tal   di sor der s  consi st  of   def i ci enci es  i n  f actor   VIII
(hem ophi l i a  A)  or   f actor   IX  (hem ophi l i a  B,   or   Chr i stm as  di sease),   w hi ch  ar e
i nher i ted  i n  an  X­l i nk ed  m anner .   Another   com m on  congeni tal   di sor der   i s  v on
Wi l l ebr and's  di sease,   i n  w hi ch  ther e  i s  a  def i ci t  i n  v on  Wi l l ebr and's  f actor .   Thi s
f actor   i s  bound  to  f actor   VIII  and  i s  necessar y   f or   both  pl atel et  f uncti on  and  f or
cl otti ng  to  tak e  pl ace  by   the  i ntr i nsi c  pathw ay .

Def i ci enci es  i n  v ar i ous  f actor s  can  be  acqui r ed  w hen  thei r   pr oducti on  i s
antagoni zed,   as  occur s  w i th  sodi um   w ar f ar i n  (Coum adi n;  DuPont  Phar m a,
Wi l m i ngton,   DE)  ther apy ,   a  substance  that  i nhi bi ts  the  pr oducti on  of   acti v ated
v i tam i n  K–dependent  f actor s  (f actor s  II,   VII,   IX,   X,   and  pr otei n  C  and  S).
Another   com m on  si tuati on  that  causes  def i ci enci es  i n  v ar i ous  f actor s  i s  l i v er
di sease;  because  the  l i v er   i s  the  si te  f or   the  sy nthesi s  of   near l y   al l   the
coagul ati on  f actor s,   sev er e  l i v er   di sease  r esul ts  i n  def i ci ent  pr oducti on  of
f actor s.   Mal nutr i ti on,   m al absor pti on,   and  l i v er   di sease  can  al l   l ead  to  a  def i ci t
i n  v i tam i n  K,   w i th  a  subsequent  def i ci t  i n  the  v i tam i n  K–dependent  f actor s.
Fi nal l y ,   the  ov er w hel m i ng  consum pti on  of   al l   f actor s  can  r esul t  i n  a
coagul opathy ,   as  occur s  i n  di ssem i nated  i ntr av ascul ar   coagul ati on  (DIC).

The  pl atel et  popul ati on  can  be  depr essed  because  of   ei ther   under pr oducti on  or
ex cessi v e  destr ucti on.   U nder pr oducti on  occur s  as  a  consequence  of   bone
m ar r ow   suppr essi on  (br ought  about  by   chem other apy ,   i nf ecti ons,   dr ugs,   or
i nf i l tr ati on  w i th  other   cel l s,   such  as  occur s  i n  the  setti ng  of   l euk em i a  or
P. 286
cancer ).   Ex cessi v e  destr ucti on  can  occur   i n  the  setti ng  of   an  enl ar ged  spl een
(sequestr ati on),   bl eedi ng  (consum pti on)  or   consum pti v e  di sor der s  (DIC  or
thr om boti c  thr om bocy topeni c  pur pur a/hem ol y ti c  ur em i c  sy ndr om e),   and  on  an
autoi m m une  basi s  [i di opathi c  thr om bocy topeni c  pur pur a  (ITP)].

Qual i tati v e  def ects  can  be  congeni tal ,   but  ar e  m or e  of ten  acqui r ed  and  due  to
dr ug  ex posur e  (aspi r i n,   nonster oi dal   anti i nf l am m ator y   dr ugs,   and  som e
anti bi oti cs)  or   ur em i a.
4.   What  ar e  the  cl i ni cal   m ani f estati ons  of   v ar i ous  bl eedi ng  di sor der s?

Al though  any   of   the  bl eedi ng  di sor der s  m ay   r esul t  i n  ex cessi v e  hem or r hage
associ ated  w i th  such  ev ents  as  sur gi cal   pr ocedur es,   tr aum a,   or   gastr oi ntesti nal
bl eedi ng,   each  di spl ay s  som e  char acter i sti c  f eatur es.   Vascul ar   f r agi l i ty   i s
ty pi cal l y   associ ated  w i th  subcutaneous  ecchy m oses.   Pl asm a  coagul ati on  pr otei n
def i ci enci es  i n  pati ents  w i th  hem ophi l i a  ar e  associ ated  w i th  spontaneous  sof t
ti ssue  and  joi nt  bl eeds.   Other   pl asm a  f actor   def i ci enci es,   as  w el l   as  pl atel et
def i ci ts,   ar e  associ ated  w i th  di f f use  ecchy m oses  (cutaneous  and  sof t  ti ssue).
Pl atel et  def i ci ts  ar e  al so  m ani f ested  by   petechi ae  (sm al l   capi l l ar y   hem or r hages
i n  m ucosal   sur f aces  and  ar eas  of   i ncr eased  hy dr ostati c  pr essur e,   such  as  the
ank l es  and  f eet)  and  pur pur a  (l ar ger   ar eas  of   hem or r hage).   Von  Wi l l ebr and's
di sease  i s  uni que  i n  that  i t  m ay   pr esent  w i th  both  sof t  ti ssue  bl eedi ng  (f actor
VII  def i ci ency )  and  m ucosal   bl eedi ng  (pl atel et  dy sf uncti on).

5.   What  w or k up  i s  i ndi cated  f or   a  bl eedi ng  pati ent?

Ev al uati on  of   the  bl eedi ng  pati ent  begi ns  w i th  a  good  hi stor y   tak i ng.   It  needs
to  be  deter m i ned  i f   the  condi ti on  i s  of   l ong  standi ng  or   i s  new .   Questi ons  about
pr ev i ous  bl eedi ng  epi sodes  (nosebl eeds,   br ui si ng,   m enstr ual   f l ow ,   bl eedi ng
w i th  tr aum a,   sur ger y ,   and  del i v er y )  as  w el l   as  f am i l y   hi stor y   ar e  v i tal   f or
deter m i ni ng  the  natur e  of   the  di sor der .   A  car ef ul   dr ug  hi stor y ,   i ncl udi ng  ov er ­
the­counter   dr ug  use,   m ust  be  tak en.   The  pati ent's  m edi cal   hi stor y   and  a
r ev i ew   of   sy m ptom s  m ay   r ev eal   ev i dence  of   autoi m m une  di sor der s  or
i nter cur r ent  i l l ness.

Phy si cal   ex am i nati on  i s  i m por tant  i n  ev al uati ng  the  si tes  of   bl eedi ng
(cutaneous,   m ucosal ,   sof t  ti ssue,   or   joi nt  bl eedi ng  si tes,   as  w el l   as  petechi ae).
An  enl ar ged  spl een  and  ev i dence  of   l i v er   di sease  (e. g. ,   spi der s  or
hem angi om ata)  or   m al nutr i ti on  shoul d  be  sought,   and  the  pati ent's  ov er al l
m edi cal   condi ti on  shoul d  be  assessed.

A  scr eeni ng  f or   bl eedi ng  di sor der s  shoul d  i ncl ude  a  pl atel et  count,   PT,   and  PTT;
i f   any   of   these  r esul ts  ar e  abnor m al   or   i f   ther e  i s  ev i dence  of   m ucosal
bl eedi ng,   deter m i nati on  of   a  bl eedi ng  ti m e  m ay   al so  be  i ndi cated.

If   the  PT  or   PTT  i s  pr ol onged,   the  nex t  step  i n  the  ev al uati on  shoul d  be  a  1:1
m i x   i n  w hi ch  the  pati ent's  pl asm a  i s  m i x ed  w i th  nor m al   pl asm a  and  the  PT  and
PTT  ar e  deter m i ned  agai n.   If   the  pati ent  i s  def i ci ent  i n  som e  f actor ,   the  nor m al
pl asm a  par ti al l y   cor r ects  thi s  def i ci ency   and  the  PT  or   PTT  ar e  cor r ected  to  a
nor m al   v al ue.   If   an  i nhi bi tor   to  a  par ti cul ar   f actor   i s  pr esent,   thi s  i nhi bi tor   al so
bl ock s  the  acti on  of   the  nor m al   pl asm a,   and  the  PT  or   PTT  ar e  not  cor r ected.
The  m ost  com m on  i nhi bi tor   i s  the  l upus  anti coagul ant,   w hi ch  i s  seen  i n  the
pr esence  and  absence  of   autoi m m une  di sease;  i t  i s  usual l y
P. 287
associ ated  w i th  an  el ev ated  PTT  that  i s  not  cor r ected  w i th  a  1:1  m i x .   It  i s
associ ated  w i th  an  i ncr eased  r i sk   of   cl otti ng,   not  bl eedi ng.

If   the  pl atel et  count  i s  v er y   l ow   (20, 000/m m 3 )  and  the  PT  and  PTT  ar e  nor m al ,
a  bone  m ar r ow   bi opsy   m ay   be  i ndi cated  to  deter m i ne  w hether   ther e  ar e
adequate  pl atel et  pr ecur sor s  i n  the  bone  m ar r ow .   If   pl atel et  pr ecur sor s  ar e
absent,   an  under pr oducti on  state  ex i sts;  i f   pr ecur sor s  ar e  pr esent,   thi s  i m pl i es
that  the  l ow   pl atel et  count  stem s  f r om   per i pher al   destr ucti on.   U si ng  the
detecti on  of   anti pl atel et  anti bodi es  as  ev i dence  f or   the  autoi m m une  destr ucti on
of   pl atel ets  i s  not  r el i abl e  because  som e  nor m al   peopl e  hav e  anti pl atel et
anti bodi es  w i thout  per i pher al   destr ucti on,   w her eas  the  ti ter s  i n  peopl e  w i th  ITP
m ay   be  l ow .

6.   What  ther api es  ar e  av ai l abl e  f or   the  m anagem ent  of   bl eedi ng  di sor der s?

Bl ood  com ponents  can  be  used  to  cor r ect  def i ci enci es  i n  the  di v i si ons  of   the
coagul ati on  sy stem .   Fr esh  f r ozen  pl asm a  contai ns  v ar i ous  per centages  of   each
of   the  coagul ati on  pr otei ns  and  can  be  used  w hen  m or e  than  one  f actor   i s
def i ci ent  (e. g. ,   v i tam i n  K–dependent  f actor s).   Cr y opr eci pi tate  contai ns  v on
Wi l l ebr and's  f actor ,   f i br i nogen,   and  f actor   VIII,   but  i s  m ost  com m onl y   used  i n
peopl e  w i th  an  acqui r ed  f i br i nogen  def i ci ency   (e. g. ,   DIC  and  l i v er   di sease).
Because  of   the  r i sk   of   v i r al   i nf ecti on  (i t  i s  pool ed  f r om   m ul ti pl e  donor s),
cr y opr eci pi tate  i s  no  l onger   used  as  f r equentl y   f or   pati ents  w i th  m i l d
hem ophi l i a  and  v on  Wi l l ebr and's  di sease.   Instead,   desm opr essi n  (DDAVP)  i s
now   used  i n  the  tr eatm ent  of   these  di seases,   as  w el l   as  i n  the  pl atel et
dy sf uncti on  associ ated  w i th  ur em i a  and  other   qual i tati v e  def ects.   Thi s  dr ug
w or k s  by   sti m ul ati ng  the  r el ease  of   v on  Wi l l ebr and's  f actor   (f actor   VIII)  f r om
the  endothel i um .   Ther e  ar e  al so  speci f i c  heat­tr eated  f actor   concentr ates  f or
f actor s  VIII  and  IX,   w hi ch  can  be  used  i n  the  m anagem ent  of   hem ophi l i a.

Quanti tati v e  pl atel et  pr obl em s  caused  by   under pr oducti on,   as  w el l   as  som e
consum pti v e  states  such  as  uncontr ol l ed  bl eedi ng,   can  be  tr eated  w i th  pl atel et
tr ansf usi ons.   Thi s  i s  of ten  f uti l e  i n  the  setti ng  of   autoi m m une  destr ucti on  unti l
the  autoi m m une  pr ocess  i s  ar r ested;  i n  f act,   pl atel et  tr ansf usi on  m ay
accel er ate  destr ucti on  by   sti m ul ati ng  the  i m m une  sy stem .   The  usual   i ni ti al
tr eatm ent  f or   ITP  i s  w i th  hi gh­dose  pr edni sone,   f ol l ow ed  by   spl enectom y   i f   the
pr edni sone  f ai l s  to  bl ock   the  i m m une  destr ucti on.   Tr ansf usi ng  pl atel ets  i nto  a
pati ent  w ho  has  ur em i a  or   w ho  i s  tak i ng  a  dr ug  that  r ender s  hi s  or   her   ow n
pl atel ets  dy sf uncti onal   i s  al so  f uti l e  because  the  tr ansf used  pl atel ets  qui ck l y
becom e  af f ected  as  w el l .

Case 1
A  47­y ear ­ol d  w hi te  m an  com es  to  the  em er gency   r oom   com pl ai ni ng  of
hem atem esi s  and  a  4­day   hi stor y   of   abdom i nal   pai n  and  passi ng  bl ack ,   tar r y   stool s.
He  gi v es  a  hi stor y   of   pepti c  ul cer   di sease  that  i s  l i nk ed  to  heav y   al cohol   use,   and
thi s  w as  associ ated  w i th  one  pr ev i ous  epi sode  of   bl eedi ng.   He  deni es  the  use  of   any
m edi cati ons,   i ncl udi ng  ov er ­the­counter   m edi ci nes,   and  deni es  a  f am i l y   hi stor y   of
bl eedi ng.   On  r ev i ew   of   the  sy stem s,   he  descr i bes  som e  i ncr eased  br ui si ng  dur i ng
the  l ast  2  to  3  m onths.   On  phy si cal   ex am i nati on  he  i s  f ound  to  be  jaundi ced  and  i n
m oder ate  di str ess;  al cohol   i s  sm el l ed  on  hi s  br eath.   Hi s  sk i n  i s  r em ar k abl e  f or
scatter ed  ecchy m oses  and  spi der   angi om as.   Hi s  l i v er
P. 288
span  i s  15  cm   and  ther e  i s  som e  tender ness  pl us  a  pal pabl e  spl een  ti p.   The  pati ent
i s  conti nui ng  to  pass  m el ena  and  v om i t  br i ght  r ed  bl ood.

The  f ol l ow i ng  i ni ti al   l abor ator y   v al ues  ar e  f ound:  w hi te  bl ood  cel l   count,   4, 500/m m 3
w i th  a  nor m al   di f f er enti al ;  hem ogl obi n,   6. 0  g/dL;  hem atocr i t,   18%;  pl atel ets,
87, 000/m m 3 ;  aspar tate  am i notr ansf er ase  (AST),   95  m U /m L  (nor m al ,   0  to  35
m U /m L);  al ani ne  am i notr ansf er ase  (ALT),   40  m U /m L  (nor m al ,   0  to  38  m U /m L);  total
bi l i r ubi n,   3. 5  m g/dL  (nor m al ,   < 1. 0  m g/dL);  and  al k al i ne  phosphatase,   450  m U /m L
(nor m al ,   0  to  125  m U /m L).

1.   How   w oul d  y ou  pr oceed  w i th  the  ev al uati on  of   thi s  pati ent's  bl eedi ng  pr obl em ?
2.   What  bl ood  pr oducts,   i f   any ,   w oul d  y ou  gi v e  thi s  pati ent?
3.   What  other   m edi ci nes,   i f   any ,   w oul d  y ou  gi v e  thi s  pati ent  to  m anage  hi s
bl eedi ng?
4.   What  f actor s  m ay   be  contr i buti ng  to  thi s  pati ent's  l ow   pl atel et  count?

Case Discussion
1.   How   w oul d  y ou  pr oceed  w i th  the  ev al uati on  of   thi s  pati ent's  bl eedi ng  pr obl em ?

Whi l e  em er gency   m edi cal   m anagem ent  of   hi s  bl eedi ng  i s  bei ng  pr ov i ded
thr ough  the  pl acem ent  of   a  nasogastr i c  tube,   together   w i th  the  i ntr av enous
adm i ni str ati on  of   f l ui ds  f or   bl ood  pr essur e  suppor t  as  needed  and  ty pi ng  and
cr ossm atchi ng  i n  pr epar ati on  f or   the  adm i ni str ati on  of   pack ed  r ed  bl ood  cel l s,
thi s  pati ent  w i th  appar ent  chr oni c  l i v er   di sease  needs  to  hav e  hi s  coagul ati on
status  ev al uated.   Both  the  PT  and  PTT  shoul d  be  deter m i ned  pr om ptl y   and
m easur em ent  of   the  f i br i nogen  l ev el   shoul d  be  consi der ed  because  i t  can  be
decr eased  i n  the  setti ng  of   chr oni c  l i v er   f ai l ur e.   In  thi s  case,   i f   the  PT  and  PTT
pr ov e  to  be  el ev ated,   as  ex pected,   ther e  i s  pr obabl y   l i ttl e  r eason  f or   a  1:1  m i x
i n  thi s  acutel y   i l l   pati ent  because  a  def i ci ency   state  i s  v er y   l i k el y .

2.   What  bl ood  pr oducts  w oul d  y ou  gi v e  thi s  pati ent,   i f   any ?

If   hi s  PT  or   PTT  pr ov es  to  be  el ev ated,   the  best  bl ood  pr oduct  f or   r epl aci ng  the
def i ci ent  f actor s  i s  f r esh  f r ozen  pl asm a.   In  addi ti on,   i f   hi s  f i br i nogen  l ev el   i s
m easur ed  and  f ound  to  be  l ess  than  100  m g/dL,   cr y opr eci pi tate  m ay   al so  be
i ndi cated.   Fi nal l y ,   i t  m ay   becom e  necessar y   to  adm i ni ster   pl atel ets  i f   hi s  count
f al l s  bel ow   20, 000/m m 3   i n  the  f ace  of   acti v e  bl eedi ng.

3.   What  other   m edi ci nes,   i f   any ,   w oul d  y ou  gi v e  thi s  pati ent  to  m anage  hi s
bl eedi ng?

If   hi stor y   and  phy si cal   ex am i nati on  f i ndi ngs  ar e  consi stent  w i th  al cohol i sm   and
l i v er   di sease,   v i tam i n  K  shoul d  al so  be  gi v en.

4.   What  f actor s  m ay   be  contr i buti ng  to  thi s  pati ent's  l ow   pl atel et  count?

Hi s  l ow   pl atel et  count  m ay   stem   f r om   m ul ti pl e  causes.   Fi r st,   the  pl atel et  count
can  f al l   i n  the  f ace  of   m assi v e  bl eedi ng  (consum pti on).   Second,   he  m ay   be
chr oni cal l y   under pr oduci ng  pl atel ets  ow i ng  to  ei ther   chr oni c  al cohol
suppr essi on  of   the  bone  m ar r ow   or   f ol i c  aci d  def i ci ency .   Fi nal l y ,   he  has  an
enl ar ged  spl een,   w hi ch  m ay   be  sequester i ng  hi s  pl atel ets.

Case 2
A  35­y ear ­ol d  Hi spani c  w om an  pr esents  to  the  em er gency   r oom   com pl ai ni ng  of   a
nosebl eed  that  has  per si sted  f or   sev er al   hour s.   She  deni es  a  hi stor y   of   pr ev i ous
bl eedi ng,
P. 289
al though  she  has  noti ced  som e  i ncr eased  br ui si ng  dur i ng  the  l ast  w eek   and  the
appear ance  of   a  sm al l ,   pur pl i sh  r ash  on  her   f eet  and  ank l es.   She  deni es  any
ex cessi v e  bl eedi ng  w i th  the  del i v er y   of   her   thr ee  chi l dr en  and  has  not  under gone
any   sur gi cal   pr ocedur es.   She  deni es  tak i ng  aspi r i n,   al though  she  has  tak en
acetam i nophen  f or   r el i ef   of   a  m i l d  back ache,   and  i s  on  no  other   m edi cati ons.   On
r ev i ew   of   her   sy m ptom s,   she  deni es  ar thr al gi as,   ar thr i ti s,   f ev er s,   col d  sy m ptom s,
or   other   i nf ecti ous  sy m ptom s;  she  has  been  i n  good  heal th  unti l   now .   On
ex am i nati on,   she  i s  f ound  to  be  w el l   dev el oped  and  i n  no  di str ess.   Ther e  i s  som e
f r esh  as  w el l   as  dr i ed  bl ood  obscur i ng  the  nasal   m ucosa;  she  has  no  conjuncti v al
hem or r hages  but  does  hav e  pal atal   petechi ae.   Her   spl een  i s  not  pal pabl e  but  ther e
i s  a  petechi al   r ash  ar ound  both  ank l es.   Her   nosebl eed  r equi r es  nasal   pack i ng  f or
contr ol .

The  f ol l ow i ng  i ni ti al   l abor ator y   v al ues  ar e  f ound:  w hi te  bl ood  cel l   count,   6, 700/m m 3
w i th  a  nor m al   di f f er enti al ;  hem ogl obi n,   14. 2  g/dL;  hem atocr i t,   42. 2%;  MCV,   85  µ 3 ;
pl atel ets,   5, 000/m m 3 ;  PT,   11. 5  seconds  (contr ol ,   12  seconds);  and  PTT,   28  seconds
(contr ol ,   28. 5  seconds).

1.   What  w oul d  y ou  do  nex t  to  ev al uate  thi s  pati ent's  bl eedi ng?
2.   What  r esul ts  w oul d  y ou  ex pect  f r om   the  f ur ther   ev al uati on  of   thi s  pati ent's
bl eedi ng?
3.   What  ther apy   w oul d  y ou  i nsti tute  i n  thi s  pati ent?

Case Discussion
1.   What  w oul d  y ou  do  nex t  to  ev al uate  thi s  pati ent's  bl eedi ng?

Wi th  the  nor m al   coagul ati on  f i ndi ngs  and  com pl ete  bl ood  count,   ex cept  f or   the
pl atel et  count,   and  the  absence  of   other   phy si cal   f i ndi ngs  such  as  an  enl ar ged
spl een,   a  bone  m ar r ow   bi opsy   i s  not  essenti al   to  ev al uate  f or   m egak ar y ocy tes.
Som e  cl i ni ci ans  m ay   choose  to  tr eat  f or   pr esum pti v e  ITP  and  ev al uate  the
pati ent  i n  24  hour s.

2.   What  r esul ts  w oul d  y ou  ex pect  f r om   the  f ur ther   ev al uati on  of   thi s  pati ent's
bl eedi ng?

Her   cl i ni cal   pi ctur e  i s  consi stent  w i th  that  of   ITP,   and,   i n  thi s  setti ng,   an
adequate  bone  m ar r ow   speci m en  w oul d  show   an  i ncr eased  or   nor m al   num ber   of
m egak ar y ocy tes.   If   the  phy si ci an  chooses  to  tr eat  the  pati ent  em pi r i cal l y   f or
ITP  (see  the  f ol l ow i ng  tex t),   the  pati ent  shoul d  hav e  si gni f i cant  i m pr ov em ent
(i . e. ,   pl atel et  count  ≥20, 000  w i th  l ess  i nci dence  of   bl eedi ng)  i n  24  hour s.

3.   What  ther apy   w oul d  y ou  i nsti tute  i n  thi s  pati ent?

Pl atel et  tr ansf usi ons  w oul d  not  be  hel pf ul   i n  thi s  pati ent  and  m i ght  ev en
accel er ate  the  destr ucti v e  pr ocess.   Pr edni sone  tr eatm ent  (60  to  100  m g  per
day )  shoul d  be  i ni ti ated  once  bone  m ar r ow   f i ndi ngs  conf i r m   the  di agnosi s  or   i f
the  pati ent  i s  tr eated  em pi r i cal l y .

Case 3
You  ar e  ask ed  to  consul t  on  the  case  of   a  65­y ear ­ol d  w hi te  m an  w i th  a  hi stor y   of
sev er e  r heum atoi d  ar thr i ti s,   w ho  has  cer v i cal   spi ne  i nstabi l i ty   that  now   r equi r es
or thopaedi c  stabi l i zati on.   The  pr eoper ati v e  l abor ator y   r esul ts  ar e  as  f ol l ow s:  w hi te
bl ood  cel l   count,   10, 000/m m 3   w i th  a  nor m al   di f f er enti al ;  hem ogl obi n,   12  g/dL;
hem atocr i t,   36%;  MCV,
P. 290
86  f L;  pl atel ets,   190, 000/m m 3 ;  PT,   12  seconds  (contr ol ,   11. 5  seconds);  PTT,   52. 2
seconds  (contr ol ,   32. 5  seconds);  and  bl eedi ng  ti m e,   10. 5  m i nutes  (nor m al ,   0  to  9. 5
m i nutes).
The  pati ent  deni es  any   bl eedi ng  hi stor y ,   and  had  under gone  a  r i ght  k nee
r epl acem ent  i n  the  past  w i thout  di f f i cul ty .   He  has  tak en  l ar ge  doses  of   aspi r i n  i n
the  past,   but  i s  cur r entl y   on  a  nonster oi dal   agent  and  tak es  no  other   m edi ci nes.
Ther e  i s  no  f am i l y   hi stor y   of   bl eedi ng  di sor der s.   On  ex am i nati on,   he  ex hi bi ts  the
sequel ae  of   sev er e  chr oni c  r heum atoi d  ar thr i ti s,   w i th  def or m ed  joi nts  of   the  hands.
He  has  no  si gni f i cant  sk i n  l esi ons.   Hi s  spl een  i s  not  pal pabl e  and  hi s  l i v er   i s  not
enl ar ged.

1.   What  f ur ther   pr eoper ati v e  ev al uati on  w oul d  y ou  do  to  r eassur e  the  sur geon
that  i ntr aoper ati v e  hem ostasi s  i s  adequate?
2.   What  bl ood  pr oducts,   i f   any ,   w oul d  y ou  use  i n  thi s  pati ent?
3.   What  changes,   i f   any ,   w oul d  y ou  m ak e  i n  thi s  pati ent's  m edi cati ons?

Case Discussion
1.   What  f ur ther   pr eoper ati v e  ev al uati on  w oul d  y ou  do  to  r eassur e  the  sur geon
that  i ntr aoper ati v e  hem ostasi s  i s  adequate?

The  pati ent's  m ai n  coagul ati on  abnor m al i ti es  i ncl ude  a  sl i ghtl y   pr ol onged
bl eedi ng  ti m e  and  an  el ev ated  PTT.   Hi s  m edi cati ons  i ncl ude  a  nonster oi dal
anti i nf l am m ator y   agent,   w hi ch  can  r ev er si bl y   af f ect  pl atel et  f uncti on;  thi s  i s
the  m ost  l i k el y   sour ce  of   hi s  m i l dl y   i ncr eased  bl eedi ng  ti m e.   In  the  absence  of
a  bl eedi ng  hi stor y   and  i f   no  em er gency   ci r cum stances  pr ev ai l ,   a  1:1  m i x   of   hi s
el ev ated  PTT  i s  i ndi cated.   Hi s  hi stor y   of   a  chr oni c  i nf l am m ator y   condi ti on  i s  a
str ong  i ndi cator   to  hav e  hi s  l upus  anti coagul ant  l ev el   deter m i ned.

2.   What  bl ood  pr oducts,   i f   any ,   w oul d  y ou  use  i n  thi s  pati ent?

If   a  1:1  m i x   does  not  cor r ect  i n  r esponse  to  nor m al   pl asm a,   thi s  i ndi cates  the
pr esence  of   an  i nhi bi tor .   Hi s  cl i ni cal   pi ctur e  i s  consi stent  w i th  a  l upus
anti coagul ant,   w hi ch  i s  actual l y   associ ated  w i th  a  r i sk   of   cl otti ng,   not  bl eedi ng,
so  no  bl ood  pr oducts  ar e  i ndi cated.   If   hi s  1:1  m i x   does  cor r ect,   i m pl y i ng  a
def i ci ency   state,   then  speci f i c  assay s  of   f actor   l ev el s,   i ncl udi ng  f actor s  VIII
and  IX,   m ay   be  necessar y   to  i denti f y   the  speci f i c  def i ci ency .   Thi s  i s  hi ghl y
unl i k el y   i n  the  absence  of   cl i ni cal   bl eedi ng.

Thi s  pati ent's  sl i ghtl y   pr ol onged  bl eedi ng  ti m e  does  not  r equi r e  any
i nter v enti on.

3.   What  changes,   i f   any ,   w oul d  y ou  m ak e  i n  thi s  pati ent's  m edi cati ons?

Hi s  nonster oi dal   m edi cati on  shoul d  be  stopped  f or   at  l east  5  to  7  day s  bef or e
the  spi ne  stabi l i zati on  pr ocedur e  to  al l ow   nor m al   pl atel et  f uncti on  to  r etur n.
Im m edi atel y   bef or e  sur ger y ,   hi s  bl eedi ng  ti m e  shoul d  be  check ed  agai n  to
conf i r m   thi s  r etur n  to  nor m al .

Suggested Readings
Beutl er   E,   Li chtm an  MA,   Col ter   BS,   et  al . ,   eds.   Hem atol ogy ,   5th  ed.   N ew   Yor k :
McGr aw ­Hi l l ,   1995.
Wi ntr obe  MW.   Cl i ni cal   hem atol ogy ,   9th  ed.   Phi l adel phi a:  Lea  &  Febi ger ,   1993.

P. 291

Breast Cancer
1.   What  i s  the  i nci dence  of   br east  cancer ?

2.   What  i s  the  natur al   hi stor y   of   br east  cancer ?

3.   What  ar e  the  r i sk   f actor s  f or   br east  cancer ?

4.   Of   w hat  does  the  scr eeni ng  f or   br east  cancer   consi st?

5.   What  i s  the  TN M  cl assi f i cati on,   and  w hat  ar e  the  stages  of   br east  cancer ?

6.   What  ar e  the  pr ognosti c  i ndi cator s  associ ated  w i th  br east  cancer ?

7.   What  i s  the  di f f er ence  betw een  m odi f i ed  r adi cal   m astectom y   and  l um pectom y
pl us  r adi ati on  ther apy   i n  the  tr eatm ent  of   stage  I  and  II  br east  cancer ,   and
w hat  ar e  the  i ndi cati ons  f or   each?

8.   What  i s  the  r ol e  f or   adjuv ant  chem other apy   i n  the  tr eatm ent  of   br east  cancer ?

9.   Of   w hat  does  the  tr eatm ent  of   node­negati v e  br east  cancer   consi st?

10.   What  i s  the  pur pose  and  the  under l y i ng  pr i nci pl es  of   endocr i ne  m ani pul ati on  i n
the  tr eatm ent  of   m etastati c  br east  cancer ?

11.   What  i s  the  r ol e  of   sy stem i c  chem other apy   i n  the  tr eatm ent  of   m etastati c
br east  cancer ?

Discussion
1.   What  i s  the  i nci dence  of   br east  cancer ?

Br east  cancer   i s  the  m ost  com m on  neopl asm   i n  w om en,   w i th  an  i nci dence  that
conti nues  to  r i se  and  cur r entl y   stands  at  1  i n  10  w om en.   The  i nci dence  r i ses
dr am ati cal l y   w i th  age.

2.   What  i s  the  natur al   hi stor y   of   br east  cancer ?

Br east  cancer   i s  consi der ed  to  be  a  sy stem i c  di sease  f r om   the  ti m e  of
di agnosi s,   r egar dl ess  of   the  stage.   The  av er age  doubl i ng  ti m e  v ar i es  f r om   23
to  500  day s.   Ther ef or e,   a  1­cm   tum or   m ay   hav e  ex i sted  f or   2  to  17  y ear s
bef or e  di agnosi s.

Despi te  l ocal   contr ol ,   af f ected  pati ents  conti nue  to  di e  at  a  r ate  f aster   than
that  seen  i n  age­m atched  contr ol   subjects  f or   the  f i r st  30  y ear s  af ter
tr eatm ent.   In  addi ti on,   pati ents  dy i ng  f r om   any   cause  ar e  f ound  to  hav e
ev i dence  of   tum or   at  autopsy .   The  m ost  com m on  si tes  of   di stant  m etastases
ar e  the  bone,   l i v er ,   and  l ung.

Par aneopl asti c  condi ti ons  that  m ay   be  associ ated  w i th  br east  cancer   i ncl ude
hy per cal cem i a,   neur om uscul ar   di sor der s,   der m atom y osi ti s,   acanthosi s
ni gr i cans,   and  hem ostati c  abnor m al i ti es.

Com m on  secondar y   m al i gnanci es  i n  pati ents  w i th  br east  cancer   consi st  of
cancer   i n  the  opposi te  br east,   ov ar i an  cancer ,   and  col or ectal   car ci nom a.

3.   What  ar e  the  r i sk   f actor s  f or   br east  cancer ?

High­ris k  fa c tors   (thr eef ol d  or   gr eater   i ncr ease)  f or   the  dev el opm ent  of
br east  cancer   ar e:

Age  gr eater   than  50  y ear s

Pr ev i ous  cancer   i n  one  br east,   especi al l y   that  occur r i ng  pr em enopausal l y

P. 292
Br east  cancer   i n  the  f am i l y ,   al though  the  r i sk   v ar i es  dependi ng  on
w hether   the  di sease  w as  i n  a  f i r st­degr ee  f am i l y   m em ber ,   w as  uni l ater al
or   bi l ater al ,   and  occur r ed  pr em enopausal l y   or   postm enopausal l y :  bi l ater al
and  pr em enopausal   di sease  car r i es  an  8. 8  ti m es  gr eater   r i sk ;  bi l ater al
and  postm enopausal   di sease  car r i es  a  5. 4  ti m es  gr eater   r i sk ;  uni l ater al
and  pr em enopausal   di sease  car r i es  a  3  ti m es  gr eater   r i sk ;  and  uni l ater al
and  postm enopausal   di sease  car r i es  a  1. 5  ti m es  gr eater   r i sk

Par i ty .   Wom en  w ho  ar e  nul l i par ous  or   w ho  w er e  f i r st  pr egnant  af ter   31
y ear s  of   age  hav e  a  thr ee  to  f our   ti m es  i ncr eased  r i sk

Ductal   car ci nom a  i n  si tu  car r i es  a  30%  r i sk   of   becom i ng  i nv asi v e

Cer tai n  f or m s  of   beni gn  br east  di sease  ar e  associ ated  w i th  an  i ncr eased
r i sk   of   cancer   Gr oss  cy sti c  di sease  w i th  l esi ons  ex ceedi ng  3  m m ,   m ul ti pl e
i ntr aductal   papi l l om as,   and  aty pi cal   ductal   hy per pl asi a  ar e  consi der ed
pr em al i gnant

Inte rme dia te ­ris k  fa c tors   (1. 2­  to  1. 5­f ol d  i ncr ease)  f or   the  dev el opm ent  of
br east  cancer   consi st  of   m enstr uati on  (ei ther   ear l y   m enar che  or   l ate
m enopause);  or al   estr ogen  w i th  pr ogester one  ther apy ;  al cohol   consum pti on;
di abetes  m el l i tus;  hi stor y   of   cancer   of   the  uter us,   ov ar y ,   or   col on;  and
obesi ty .

4.   Of   w hat  does  the  scr eeni ng  f or   br east  cancer   consi st?

Wom en  ol der   than  20  y ear s  shoul d  per f or m   br east  sel f ­ex am i nati on  ev er y
m onth.   Pr em enopausal   w om en  shoul d  ex am i ne  thei r   br easts  5  to  7  day s  af ter
the  end  of   thei r   m enstr ual   cy cl e,   and  postm enopausal   w om en  shoul d  do  thi s  on
the  sam e  day   of   ev er y   m onth.

Wom en  shoul d  hav e  thei r   br easts  ex am i ned  by   a  phy si ci an  ev er y   2  to  3  y ear s
betw een  the  ages  of   20  and  40  y ear s  and  annual l y   ther eaf ter .   The  Am er i can
Cancer   Soci ety   and  other   agenci es  hav e  r ecom m ended  that  a  basel i ne
m am m ogr am   shoul d  be  obtai ned  betw een  the  ages  of   35  and  39  y ear s,   w i th
m am m ogr am s  obtai ned  ev er y   1  to  2  y ear s  i n  w om en  aged  40  to  49  y ear s  and
then  y ear l y   af ter   the  age  of   50  y ear s.

5.   What  i s  the  TN M  cl assi f i cati on,   and  w hat  ar e  the  stages  of   br east  cancer ?

The  TN M  (pr i m ar y   tum or ,   r egi onal   nodes,   m etastases)  cl assi f i cati on  and  the
v ar i ous  stages  of   br east  cancer   ar e  outl i ned  i n  Tabl e  7­5.

6.   What  ar e  the  pr ognosti c  i ndi cator s  associ ated  w i th  br east  cancer ?

Tabl e  7­6  sum m ar i zes  the  5­  and  10­y ear   sur v i v al   stati sti cs  associ ated  w i th
the  v ar i ous  TN M  stages.   These  stati sti cs  do  not  tak e  i nto  account  r esul ts  of
adjuv ant  chem other apy ,   but  ar e  usef ul   i n  desi gni ng  tr i al s  usi ng  adjuv ant
chem other apy .

The  pati ent's  hor m onal   status  al so  has  a  bear i ng  on  her   pr ognosi s,   i n  that
estr ogen­  and  pr ogester one  r eceptor ­posi ti v e  tum or s  possess  a  70%  to  85%
chance  of   r espondi ng  to  hor m onal   ther apy ;  those  w om en  w i th  onl y   one­
r eceptor   posi ti v i ty   hav e  ex hi bi ted  a  sl i ghtl y   l ow er   r esponse  r ate  to  hor m one
m ani pul ati on.   Wom en  w i th  r eceptor ­negati v e  tum or s  do  not  r espond  to
hor m one  m ani pul ati on.

P. 293

Table 7­5 The TNM Classification of Breast Cancer

  Dis e a s e  Ex te nt  

Sta ge P rima ry Lymph Node s Dis ta nt TNM


G rouping a Tumor (T) (N) b Me ta s ta s e s Cla s s ific a tion
(M)

0 N oni nv asi v e Hom ol ater al N one Ti s  N 0  M1


car ci nom a ax i l l ar y   nodes
i n  si tu; negati v e  (N 0)
Paget's
di sease  of
the  ni ppl e
(Ti s)

I Gr eatest Hom ol ater al N one T1  N 0  M0


di m ensi on  â ax i l l ar y   nodes
‰¤2 negati v e  (N 0)
cm (T1) c

II Gr eatest Hom ol ater al T1  N 1  M0


di m ensi on ax i l l ar y   nodes T2  N 0  or
> 2  cm and  â posi ti v e  but N 1  M0
‰¤5  cm not  . x ed  (N 1)
(T2) c

IIIA Gr eatest Hom ol ater al N one T1  N 2  M0


di m ensi on ax i l l ar y   nodes T2  N 2  M0
> 5  cm (T3) c posi ti v e  and T3  N 0–2
. x ed  to  one M0
another ,   sk i n,
or   chest  w al l
(N 2)

IIIB Any   si ze Supr acl av i cul ar N one T4  any   N   M0


w i th  (T4) c or Any   T  N 3  M0
satel l i te i nf r acl av i cul ar
sk i n nodal
nodul es, i nv ol v em ent;
sk i n edem a  of   the
ul cer ati on, ar m   w i th  or
f i x ati on  to w i thout
sk i n  or pal pabl e
chest  w al l , ax i l l ar y   l y m ph
or   edem a nodes  (N 3)
of   br east,
i ncl udi ng
peau
d'or ange d

IV Any   si ze Any   status Pr esent Any   T  any   N


M1

a The  Am er i can  Joi nt  Com m i ttee  r ecogni zes  tw o  stage  gr oupi ngs:

postoper ati v e­pathol ogi c  (pr esented  i n  thi s  tabl e)  and  cl i ni cal ­di agnosti c.

b The  cl i ni cal ­di agnosti c  stage  gr oupi ng  subdi v i des  m ov abl e  hom ol ater al

ax i l l ar y   l y m ph  nodes  i nto  N 1a—nodes  not  consi der ed  to  contai n  tum or
(appr ox i m atel y   33%  ar e  hi stol ogi cal l y   posi ti v e);  and  N 1b—nodes
consi der ed  to  contai n  tum or   (appr ox i m atel y   25%  ar e  hi stol ogi cal l y
negati v e).

c T0  i ndi cates  no  tum or   dem onstr abl e  i n  br easts;  T1,   T2,   and  T3  i ncl ude

tum or   f i x ati on  to  under l y i ng  pector al   f asci a  or   m uscl e,   w hi ch  does  not
change  the  cl assi f i cati on  of   l esi ons.   (In  f l am m ator y   br east  cancer   i s
cl assi f i ed  as  a  separ ate  enti ty   and  i s  not  i ncl uded  i n  T4. )

d Sk i n  di m pl i ng  and  ni ppl e  r etr acti on  do  not  af f ect  stagi ng  cl assi f i cati on.

Adapted  f r om   the  Am er i can  Joi nt  Com m i ttee  f or   Cancer   Stagi ng  and  End­
Resul ts  Repor ti ng,   1983

7.   What  i s  the  di f f er ence  betw een  m odi f i ed  r adi cal   m astectom y   and  l um pectom y
pl us  r adi ati on  ther apy   i n  the  tr eatm ent  of   stage  I  and  II  br east  cancer ,   and
w hat  ar e  the  i ndi cati ons  f or   each?

Al though  m astectom y   contr ol s  l ocal   di sease,   i t  m ay   hav e  a  dev astati ng


psy chol ogi cal   i m pact  on  both  the  pati ents  and  thei r   f am i l i es;  ther ef or e,
sur gi cal   techni ques  desi gned  to  pr eser v e  the  br east  ar e  w ar r anted.   In  1976,
the  N ati onal
P. 294
Sur gi cal   Adjuv ant  Br east  Pr oject  (N SABP)  began  a  r andom i zed  tr i al   com par i ng
total   m astectom y ,   segm ental   m astectom y ,   and  segm ental   m astectom y   pl us
r adi ati on.   Al l   1, 843  pati ents  under w ent  ax i l l ar y   node  di ssecti on.   The  16­y ear
f ol l ow ­up  r ev eal ed  that  l um pectom y   i n  pati ents  w i th  tum or s  l ess  than  4  cm   i n
di am eter   and  w i th  f r ee  sur gi cal   m ar gi ns  i s  an  appr opr i ate  f or m   of   ther apy   i n
stage  I  and  II  br east  cancer .   In  addi ti on,   i r r adi ati on  pl us  l um pectom y   m ar k edl y
decr eases  the  l i k el i hood  of   l ocal   r ecur r ence.   Local   r ecur r ence,   ev en  10  y ear s
postl um pectom y ,   does  not  af f ect  ov er al l   sur v i v al .

Table 7­6 Prognostic Indicators for Breast
Cancer

P rognos tic  Indic a tors 5 y (% ) 10 y (% )

Cl i ni cal   stage    

  0     > 90     90

    I     80     65

        II     60     45

        IIIA     50     40

        IIIB     35     20

        IV  and  i nf l am m ator y   br east  cancer     10   5

Tum or   si ze  (cm )        

    <1     —     80

        3–4     —     55

        5–7. 5     —     15

Ax i l l ar y   nodes        

        N one  posi ti v e     80     65
        1–3  posi ti v e     65     40

        > 3  posi ti v e     30     15

8.   What  i s  the  r ol e  f or   adjuv ant  chem other apy   i n  the  tr eatm ent  of   br east  cancer ?

The  l y m ph  node  status  i s  the  m ost  i m por tant  pr ognosti c  i ndi cator   i n  thi s
di sease.   Pati ents  w i th  posi ti v e  nodes  ar e  at  a  hi gh  r i sk   f or   l ocal   r ecur r ences  as
w el l   as  m etastati c  di sease.

A  pr ospecti v e,   r andom i zed  tr i al   show ed  that  the  addi ti on  of   AC  [dox or ubi ci n
(Adr i am y ci n)  and  cy cl ophospham i de  (Cy tox an)]  to  the  tr eatm ent  pr otocol
i m pr ov es  the  10­y ear   ov er al l   sur v i v al   i n  both  pr em enopausal   and
postm enopausal   w om en.   The  addi ti on  of   a  tax ane  to  thi s  r egi m en  pr ov i des  a
sm al l   but  stati sti cal l y   si gni f i cant  i m pr ov em ent  i n  di sease­f r ee  sur v i v al .   If   the
tum or   i s  hor m one  r eceptor   posi ti v e,   addi ng  anti estr ogen  tr eatm ent  to  the
chem other apeuti c  r egi m en  has  been  show n  i n  m ul ti pl e  tr i al s  to  i ncr ease
di sease­f r ee  sur v i v al ,   and  ov er al l   sur v i v al   benef i t  has  been  show n.

9.   Of   w hat  does  the  tr eatm ent  of   node­negati v e  br east  cancer   consi st?

Som e  pati ents  thought  to  be  node  negati v e  i n  the  past  hav e  been  f ound  to  be
node  posi ti v e  by   car ef ul   anal y si s  usi ng  techni ques  such  as  senti nel   node
bi opsy .   The  tr eatm ent  of   node­negati v e  br east  cancer   i s  sti l l   contr ov er si al .
Or i gi nal l y ,
P. 295
w om en  w i th  negati v e  nodes  w er e  thought  to  hav e  a  v er y   good  pr ognosi s,   but
30%  w er e  sti l l   f ound  to  be  dy i ng  of   the  di sease.   Gi v i ng  ther apy   to  al l   such
pati ents  i s  not  w i thout  hazar d;  ther ef or e,   i t  w oul d  be  of   gr eat  v al ue  to  hav e
i ndi cator s  that  coul d  pr edi ct  w ho  w oul d  be  good  candi dates  f or   tr eatm ent.
Tests  can  pr edi ct  aggr essi v e  tum or s  such  as  i ncr eased  cel l s  i n  the  S  phase  of
the  cel l   cy cl e  and  the  pr esence  of   gr ow th  f actor   m ar k er s  such  as  Her 2  neu.
Other   f actor s,   such  as  the  si ze  of   the  pr i m ar y   tum or ,   the  hi stol ogi c  gr ade,   and
hor m one  r eceptor   status,   m ay   al so  be  i nf l uenti al .   Most  studi es  now   suggest
that  pati ents  w i th  tum or s  l ar ger   than  1  cm   shoul d  be  of f er ed  adjuv ant
chem other apy .   Pati ents  w i th  a  l ow ­gr ade  1. 3  cm   tum or   that  i s  estr ogen
r eceptor   (ER)  and  pr ogester one  r eceptor   (PR)  posi ti v e  and  Her 2  neu  negati v e
w oul d  show   f ar   l ess  benef i t  f r om   chem other apy   than  the  sam e  si ze  tum or   that
i s  hi gh  gr ade  and  both  ER  and  PR  negati v e.   Her 2  neu­posi ti v e  tum or s  benef i t
m ost  f r om   chem other apy   and  the  addi ti on  of   m onocl onal   anti body   agai nst  the
pr otei n  (tr azati nm i b),   par ti cul ar l y   i f   the  tum or s  ar e  l ar ger   than  1  cm   or   show
ev i dence  f or   aggr essi v e  di sease.   Tam ox i f en  i n  pr em enopausal   pati ents  and
ar om atase  i nhi bi tor s  i n  postm enopausal   pati ents  hav e  been  show n  to  pr otect
pati ents  f r om   the  dev el opm ent  of   br east  cancer   and  ar e  ef f ecti v e  i n  pr ev enti ng
r ecur r ence  i n  pati ents  w i th  sm al l   tum or s  that  ar e  node  negati v e  but  posi ti v e
f or   estr ogen  and/or   pr ogester one  r eceptor s.

10.   What  i s  the  pur pose  and  the  under l y i ng  pr i nci pl es  of   endocr i ne  m ani pul ati on  i n
the  tr eatm ent  of   m etastati c  br east  cancer ?

It  has  been  k now n  f or   m any   y ear s  that  ther e  i s  an  i nter r el ati onshi p  betw een
the  ov ar i es  and  the  br easts.   Pati ents  w i th  l ocal l y   r ecur r ent  br east  cancer   hav e
ex hi bi ted  a  dr am ati c  r esponse  to  bi l ater al   oophor ectom y .   Mor e  r ecentl y ,   w i th
the  abi l i ty   to  i denti f y   estr ogen  r eceptor s  i n  br east  ti ssue,   i t  w as  natur al   f or
anti estr ogen  ther apy   to  be  used  f or   the  tr eatm ent  of   br east  cancer .   The  f i r st
tr i al s  of   hor m onal   agents  w er e  conducted  i n  pati ents  w i th  m etastati c  di sease,
and  they   pr ov ed  that  these  agents  w er e  not  onl y   ef f i caci ous  but  al so  w el l
tol er ated,   w i th  w ei ght  gai n  bei ng  the  onl y   m ajor   si de  ef f ect.   Tam ox i f en  can
i ncr ease  r i sk   of   uter i ne  cancer   and  bl ood  cl ots;  these  si de  ef f ects  ar e  not
i ncr eased  w i th  ar om atase  i nhi bi tor s.

Because  of   thei r   success  i n  the  m anagem ent  of   adv anced  di sease,   hor m onal
agents  hav e  been  i nsti tuted  as  adjuv ant  ther apy   and  chem opr ev enti on  agents.
Al though  the  f i ndi ng  of   estr ogen  r eceptor   posi ti v i ty   consti tutes  the  gr eatest
adv antage,   m any   w om en  ar e  negati v e  f or   r eceptor s,   or   thei r   status  i s
unk now n.

11.   What  i s  the  r ol e  of   sy stem i c  chem other apy   i n  the  tr eatm ent  of   m etastati c
br east  cancer ?

Pati ents  ar e  candi dates  f or   chem other apy   i f   thei r   di ssem i nated  di sease  i s
hi ghl y   aggr essi v e,   they   ar e  hor m one  r eceptor   negati v e,   or   they   f ai l   to  r espond
to  endocr i ne  m ani pul ati on.   Ther e  ar e  sev er al   v ar i ati ons  of   com bi nati on
chem other apy   r egi m ens  contai ni ng  cy cl ophospham i de,   m ethotr ex ate,   5­
f l uor our aci l ,   dox or ubi ci n,   pacl i tax el ,   car bopl ati n,   and  other   agents.   The  use  of
si ngl e­agent  chem other apy   w i th  a  new   dr ug  added  af ter   one  f ai l s  i s  just  as
ef f i caci ous  as  the  use  of   com bi nati on  chem other apy ,   unl ess  the  ex tent  of
di sease  r equi r es  a  m or e  r api d  r esponse  f or   qual i ty   of   l i f e  i ssues.

P. 296
Case 1
A  35­y ear ­ol d  w hi te  w om an  w i th  a  f am i l y   hi stor y   of   br east  cancer   di scov er s  a  l um p
i n  her   r i ght  br east.   The  l um p  i s  conf i r m ed  on  phy si cal   ex am i nati on  and  a
m am m ogr am   i s  then  obtai ned.   A  1. 7­cm   l esi on  i s  i denti f i ed  and  sam pl ed  f or   bi opsy .
Pathol ogi c  anal y si s  of   the  bi opsy   ti ssue  r ev eal s  an  i nf i l tr ati ng  ductal   car ci nom a.   The
pati ent  el ects  to  under go  l um pectom y   w i th  senti nel   l y m ph  node  sam pl i ng  f ol l ow ed
by   ax i l l ar y   node  di ssecti on.   Tw o  of   ni ne  l y m ph  nodes  ar e  posi ti v e,   and  estr ogen  and
pr ogester one  r eceptor   studi es  ar e  negati v e.   The  hi stol ogi c  gr ade  of   the  tum or   i s
3/3,   and  the  per centage  S  phase  m easur ed  Ki ­67  m onocl onal   anti body ,   i s  18. 5%.
The  pati ent  r ecei v es  f our   cy cl es  of   AC,   tax ol ,   then  l ocal   i r r adi ati on.   She  has  no
ev i dence  of   di sease  and  i s  seen  ev er y   3  m onths  f or   f ol l ow ­up.

1.   What  i s  thi s  pati ent's  TN M  cl assi f i cati on  and  stage?


2.   Was  l um pectom y   an  appr opr i ate  tr eatm ent?
3.   Does  thi s  pati ent  hav e  poor   pr ognosti c  i ndi cator s?

Case Discussion
1.   What  i s  thi s  pati ent's  TN M  cl assi f i cati on  and  stage?

Thi s  pati ent  has  a  T1  l esi on  because  her   pr i m ar y   tum or   w as  l ess  than  2  cm .
Her   nodal   status  i s  N 1  because  tw o  of   the  nodes  show ed  tum or   i nf i l tr ati on  but
w er e  not  pal pabl e  at  pr esentati on,   and  her   m etastasi s  status  i s  gr aded  as  M0
because  no  m etastases  w er e  f ound.   Ther ef or e,   she  has  stage  II  di sease.
2.   Was  l um pectom y   an  appr opr i ate  tr eatm ent?

Ev i dence  suggests  that  l um pectom y   i s  an  al ter nati v e  to  m astectom y   i n  the
m anagem ent  of   stage  II  di sease.   Because  of   her   tw o  posi ti v e  nodes,   r adi ati on
ther apy   to  the  ax i l l a  i s  al so  r ecom m ended  to  l essen  her   i ncr eased  potenti al   f or
l ocal   r ecur r ence.   The  f i ndi ng  of   tw o  posi ti v e  nodes  al so  m ak es  her   a  candi date
f or   m or e  aggr essi v e  sy stem i c  chem other apy   to  l i m i t  the  chance  f or
dev el opm ent  of   di stant  m etastases.

3.   Does  thi s  pati ent  hav e  poor   pr ognosti c  i ndi cator s?

Thi s  pati ent  has  sev er al   poor   pr ognosti c  f eatur es:  her   age  of   35  y ear s;  a  hi gh­
gr ade  tum or   m or phol ogy   together   w i th  a  hi gh­per centage  S  phase;  negati v i ty
f or   both  r eceptor s;  and  the  posi ti v e  nodes.   Her   chance  of   sur v i v i ng  10  y ear s
w i th  no  sy stem ati c  tr eatm ent  i s  appr ox i m atel y   35%,   w i th  sy stem ati c  tr eatm ent
she  has  a  gr eater   than  50%  10­y ear   sur v i v al .   N ew   studi es  suggest  that  i f   she
w er e  Her 2  neu  posi ti v e  her   sur v i v al   w i thout  sy stem ati c  tr eatm ent  i s  l ess  than
30%,   but  the  addi ti on  of   tr astum abi b  to  her   chem other apy   m ay   actual l y
i m pr ov e  sur v i v al   com par ed  w i th  Her 2  neu  negati v e  pati ents.

Case 2
A  62­y ear ­ol d  w om an  w as  f i r st  seen  12  y ear s  ago  because  of   a  4­cm   l ef t  br east
m ass.   Bi opsy   r esul ts  r ev eal ed  adenocar ci nom a,   and  the  pati ent  under w ent  a
m odi f i ed  r adi cal   m astectom y   and  ax i l l ar y   node  di ssecti on.   Tw o  of   22  nodes  w er e
posi ti v e,   and  the  tum or   w as  posi ti v e  f or   estr ogen  and  pr ogester one  r eceptor s.   The
pati ent  w as  pl aced  on  chem other apy   f ol l ow ed  by   tam ox i f en  ther apy .
P. 297
She  di d  w el l   unti l   6  y ear s  ago,   w hen  r i ght  hi p  pai n  dev el oped.   A  bone  scan  r ev eal ed
the  pr esence  of   m etastati c  di sease  i n  her   spi ne,   r i bs,   and  r i ght  hi p.   She  w as  gi v en
anastr ozol e  (Ar i m i dex ;  Zeneca  Phar m aceuti cal s,   Wi l m i ngton,   DE),   an  ar om atase
i nhi bi tor   used  i n  postm enopausal   pati ents  as  hor m onal   agent.   Four teen  m onths
l ater ,   pai n  occur r ed  i n  her   l ef t  shoul der   and  she  becam e  i ncr easi ngl y   l ethar gi c.   A
r estagi ng  ev al uati on  show ed  pr ogr essi v e  bone  scan  f i ndi ngs  and  hy per cal cem i a.   She
w as  star ted  on  chem other apy   and  bi sphosphonates  w er e  i nsti tuted  to  tr eat  her
hy per cal cem i a  acutel y .   The  pati ent  r ecei v ed  si x   cy cl es  of   chem other apy ,   w i th
subsequent  stabi l i zati on  of   her   di sease;  how ev er ,   22  m onths  l ater ,   her   di sease
pr ogr essed  r api dl y .   She  w as  tr eated  w i th  tw o  other   chem other apeuti c  r egi m ens,   but
af ter   i ni ti al   stabi l i zati on  of   her   di sease  w i th  each  tr eatm ent,   she  di ed  36  m onths
l ater .

1.   What  w as  thi s  pati ent's  or i gi nal   TN M  cl assi f i cati on  and  stage?
2.   Is  her   cl i ni cal   cour se  ty pi cal   of   br east  cancer ?
3.   Was  a  second  hor m onal   agent  w ar r anted?
4.   What  w as  the  cause  of   her   hy per cal cem i a,   and  how   shoul d  i t  be  tr eated?

Case Discussion
1.   What  w as  thi s  pati ent's  or i gi nal   TN M  cl assi f i cati on  and  stage?

The  pati ent  or i gi nal l y   had  a  T2  l esi on  because  her   tum or   w as  4  cm ,   her   nodal
status  w as  N 1  because  ax i l l ar y   di ssecti on  r ev eal ed  sev en  posi ti v e  nodes,   and
her   m etastasi s  status  w as  M0  because  no  obv i ous  m etastati c  l esi ons  w er e
di scov er ed.   Tak en  together ,   she  or i gi nal l y   had  stage  III  di sease.

2.   Is  her   cl i ni cal   cour se  ty pi cal   of   br east  cancer ?

Br east  cancer   i s  consi der ed  a  chr oni c  di sease  based  on  the  hy pothesi s  that
m i cr om etastases  ex i st  at  the  ti m e  of   di agnosi s.   Thi s  theor y   i s  suppor ted  by   the
obser v ati on  that  w om en  w i th  ear l y ­stage  br east  cancer   sti l l   ex hi bi t  an
i ncr eased  r i sk   of   dy i ng  of   thei r   di sease  despi te  cur ati v e  i ntent,   f or   20  y ear s.

3.   Was  a  second  hor m onal   agent  w ar r anted?

The  best  pr edi ctor   of   hor m onal   r esponse  i s  a  r esponse  to  a  pr ev i ous  hor m onal
agent.   In  thi s  case,   the  pati ent  theor eti cal l y   r esponded  to  tam ox i f en  (based  on
del ay   i n  r ecur r ence);  ther ef or e,   another   hor m onal   agent  w as  appr opr i ate,   and
thi s  pr oduced  14  m onths  of   f ur ther   r esponse.

4.   What  w as  the  cause  of   her   hy per cal cem i a,   and  how   shoul d  i t  be  tr eated?

Tw o  gener al   m echani sm s  can  br i ng  about  hy per cal cem i a  i n  a  pati ent  w i th
cancer :  (a)  tum or   cel l s  i n  di r ect  contact  w i th  bone  can  i nduce  an  osteol y ti c
m echani sm ;  and  (b)  tum or   cel l s  can  secr ete  hum or al   substances  that  acti v ate
osteocl asts.

The  f i r st  m echani sm   pr i m ar i l y   oper ates  i n  br east  cancer .   Acute  i nter v enti on
r equi r es  bi sphosphonates  w i th  f l ui d  di ur esi s.   How ev er ,   the  pati ent  m ust  f i r st
be  hy dr ated  adequatel y   bef or e  di ur esi s  i s  star ted,   because  dehy dr ati on  onl y
w or sens  the  hy per cal cem i a.   Tr eatm ent  of   the  under l y i ng  cause  shoul d  then  be
i nsti tuted,   as  w as  done  i n  thi s  pati ent  w i th  chem other apy .   Bi sphosphonates  not
onl y   hel p  tr eat  hy per cal cem i a  but,   w i th  conti nual   use,   al so  i m pr ov e  sy m ptom s
caused  by   br east  m etastases  to  bone  and  m ay   i m pr ov e  sur v i v al .

P. 298

Suggested Readings
Cl ar k   GM,   Dr essl er   LG,   Ow ens  MA,   et  al .   Pr edi cti on  of   r el apse  or   sur v i v al   i n
pati ents  w i th  node­negati v e  br east  cancer   by   DN A  f l ow   cy tom etr y .   N   Engl   J  Med
1989;320:627.

Ear l y   Br east  Cancer   Tr i al i sts'  Col l abor ati v e  Gr oup.   Pol y chem other apy   f or   ear l y
br east  cancer :  an  ov er v i ew   of   the  r andom i zed  tr i al s.   Lancet  1998;352:930.

Fi sher   B,   Costanti no  JP,   Wi ck er ham   DL,   et  al .   Tam ox i f en  f or   pr ev enti on  of
br east  cancer :  r epor t  of   the  N ati onal   Sur gi cal   Adjuv ant  Br east  and  Bow el   Pr oject
P{­}1  Study .   J  N atl   Cancer   Inst  1998;90:1371.

Fi sher   B,   Redm ond  C,   Poi sson  R,   et  al .   Ei ght­y ear   r esul ts  of   r andom i zed  cl i ni cal
tr i al   com par i ng  total   m astectom y   and  l um pectom y   w i th  or   w i thout  i r r adi ati on  i n
the  tr eatm ent  of   br east  cancer .   N   Engl   J  Med  1989;320:822.

McGui r e  WL.   Adjuv ant  ther apy   f or   node­negati v e  br east  cancer .   N   Engl   J  Med
1989;320:525.
Ol i v otto  IA,   Bajdi k   CD,   Rav di n  PM,   et  al .   Popul ati on­based  v al i dati on  of   the
pr ognosti c  m odel   ADJU VAN T!  f or   ear l y   br east  cancer .   J  Cl i n  Oncol   2005;23:2716.

Chronic Myelogenous Leukemia
1.   What  i s  the  def i ni ti on  of   chr oni c  m y el ogenous  l euk em i a  (CML)?

2.   What  i s  the  eti ol ogy   of   CML?

3.   What  i s  the  pathogeni c  m echani sm   r esponsi bl e  f or   CML?

4.   What  i s  the  epi dem i ol ogy   of   CML?

5.   What  ar e  the  cl i ni cal   char acter i sti cs  of   CML?

6.   What  ar e  the  l abor ator y   f i ndi ngs  encounter ed  i n  the  setti ng  of   CML?

7.   What  ar e  the  cy togeneti c  and  bi ochem i cal   abnor m al i ti es  f ound  ty pi cal l y   i n
pati ents  w i th  CML?

8.   What  i s  the  tr eatm ent  f or   CML?

9.   What  i s  the  pr ognosi s  i n  pati ents  w i th  CML?

Discussion
1.   What  i s  the  def i ni ti on  of   CML?

CML  i s  a  hem atopoi eti c  stem   cel l   di sease  char acter i zed  by   anem i a,   ex tr em e
bl ood  gr anul ocy tosi s,   gr anul ocy ti c  i m m atur i ty ,   basophi l i a,   of ten
thr om bocy tosi s,   and  spl enom egal y .

2.   What  i s  the  eti ol ogy   of   CML?

The  eti ol ogy   of   CML  i s  unk now n,   but  ex posur e  to  i oni zi ng  r adi ati on  has  been
f ound  to  i ncr ease  the  r i sk   of   CML  abov e  the  ex pected  f r equency   i n  cer tai n
popul ati ons.   Som e  of   these  m ajor   popul ati ons  ar e  (a)  the  Japanese  ex posed  to
r adi ati on  f r om   the  N agasak i   and  Hi r oshi m a  atom i c  bom b  ex pl osi ons;  (b)  the
Br i ti sh  w i th  ank y l osi ng  spondy l i ti s  tr eated  w i th  spi nal   i r r adi ati on;  and  (c)
w om en  w i th  uter i ne  cer v i cal   car ci nom a  w ho  r equi r e  r adi ati on  ther apy .   The
f r equency   of   CML  (as  w el l   as  acute  l euk em i a)  i n  these  popul ati ons  i s
si gni f i cantl y   gr eater   than  that  ex pected  f or   com par abl e  unex posed  gr oups.
Chem i cal   l euk em ogens  hav e  not  been  i denti f i ed  as  causati v e  agents  of   CML.

P. 299
3.   What  i s  the  pathogeni c  m echani sm   r esponsi bl e  f or   CML?

CML  r esul ts  f r om   the  acqui r ed  (som ati c  m utati on)  m al i gnant  tr ansf or m ati on  of
a  si ngl e  stem   cel l   w hose  potency   dom i nates  hem atopoi esi s  i n  the  af f ected
per son,   w i th  the  i nv ol v em ent  of   er y thr opoi esi s,   neutr ophi l opoi esi s,
eosi nophi l opoi esi s,   basophi l opoi esi s,   m onocy topoi esi s,   and  thr om bopoi esi s.
Sev er al   obser v ati ons  suggest  that  som e  l y m phocy tes  m ay   be  der i v ed  f r om   the
pr i m or di al   m al i gnant  cel l   as  w el l ,   ther eby   pl aci ng  the  cul pr i t  l esi on  cl oser ,   i f
not  i n  the  pl ur i potenti al   stem   cel l .   The  ex act  m echani sm   that  causes  the
tr ansf or m ati on  to  tak e  pl ace  has  not  been  f ul l y   el uci dated,   but  the  Ph 1
(Phi l adel phi a)  chr om osom e  has  been  i m pl i cated.   The  hem atopoi eti c  cel l s
contai n  a  r eci pr ocal   tr ansl ocati on  betw een  chr om osom es  9  and  22  i n  m or e  than
90%  of   pati ents.   Thi s  l eads  to  an  ov er tl y   f or eshor tened  l ong  ar m   of   one  of   the
chr om osom e  22  pai r s.   Chr om osom e  9  contai ns  the  c­abl   gene  at  band  34;
chr om osom e  22  has  the  br eak   poi nt  cl uster   r egi on  (bcr )  and  c­si s  genes  at
band  11.   The  c­abl   gene  f r om   chr om osom e  9  i s  tr anspor ted  to  the  chr om osom e
22  bcr ,   w hi ch  i s  the  Ph 1   chr om osom e.   As  a  consequence  of   these  ev ents,   a  new
gene  i s  f or m ed,   the  bcr ­abl   gene,   w hi ch  codes  f or   a  new   pr otei n  thr ough  the
f or m ati on  of   a  new   m essenger   RN A.   In  som e  uses  the  chr om osom al
abnor m al i ty   i s  not  ev i dent  but  the  bcr ­abl   gene  i s  i denti f i ed  by   i n  si tu  by
hy br i di zati on.   Thi s  new   pr otei n  i s  a  phosphopr otei n  w i th  a  m ol ecul ar   w ei ght  of
210, 000  (DaP210  bcr ­abl )  and  possessi ng  ty r osi ne  k i nase  acti v i ty .   Its  abnor m al
acti v i ty   pr esum abl y   al ter s  the  r esponse  of   the  hem atopoi eti c  stem   cel l   so  that
i t  conti nues  to  pr ol i f er ate  r ather   than  bei ng  under   the  contr ol   of   hem atopoi eti c
gr ow th  f actor s.

4.   What  i s  the  epi dem i ol ogy   of   CML?

CML  accounts  f or   appr ox i m atel y   2%  of   al l   cases  of   l euk em i a  and  the  associ ated
m or tal i ty   r ate  i s  appr ox i m atel y   1. 5  per   100, 000  popul ati on  per   y ear .   The
di sease  occur s  sl i ghtl y   m or e  of ten  i n  m en,   but  i ts  m ani f estati ons  and  cour se
ar e  si m i l ar   f or   both  sex es.   Appr ox i m atel y   10%  of   the  cases  occur   i n  peopl e
betw een  5  and  20  y ear s  of   age,   and  CML  accounts  f or   appr ox i m atel y   3%  of   al l
the  chi l dhood  l euk em i as.

5.   What  ar e  the  cl i ni cal   char acter i sti cs  of   CML?

The  di sease  i s  char acter i zed  by   thr ee  phases:  (a)  a  chr oni c  phase,   (b)  an
accel er ated  phase,   and  (c)  a  bl ast  cr i si s.

The  m ost  f r equent  com pl ai nts  seen  dur i ng  the  c hronic  pha s e   i ncl ude  easy
f ati gabi l i ty ,   l oss  of   a  sense  of   w el l   bei ng,   decr eased  tol er ance  to  ex er ti on,
anor ex i a,   abdom i nal   di scom f or t,   ear l y   sati ety ,   w ei ght  l oss,   and  ex cessi v e
sw eati ng.   The  sy m ptom s  ar e  v ague,   nonspeci f i c,   and  gr adual   i n  onset.   Phy si cal
ex am i nati on  m ay   detect  pal l or   and  spl enom egal y .

U ncom m on  pr esenti ng  si gns  and  sy m ptom s  of   CML  i ncl ude  hy per m etabol i sm
that  si m ul ates  thy r otox i cosi s,   acute  gouty   ar thr i ti s,   pr i api sm ,   ti nni tus,   stupor ,
l ef t  upper   quadr ant  and  l ef t  shoul der   pai n  as  a  consequence  of   spl eni c
i nf ar cti on  and  per i spl eni ti s,   di abetes  i nsi pi dus,   and  acute  ur ti car i a,   w hi ch  i s
associ ated  w i th  hy per hi stam i nem i a.

In  som e  pati ents  i n  thi s  phase,   the  di sease  i s  di scov er ed  w hen  bl ood  cel l
counts  ar e  deter m i ned  dur i ng  a  r outi ne  m edi cal   ex am i nati on.   The  sy m ptom s
P. 300
and  si gns  of   the  di sease  and  the  l abor ator y   f i ndi ngs  ty pi cal l y   r em ai n  stabl e,
and  the  dur ati on  of   thi s  phase  i s  v ar i abl e.   U sual l y   i t  l asts  appr ox i m atel y   4
y ear s,   but  i t  can  l ast  f r om   w eek s  to  m any   y ear s  bef or e  tr ansf or m i ng  to  the
accel er ated  phase.

In  m ost  cases  of   CML,   the  pati ent's  di sease  ev entual l y   changes  to  a  m or e
aggr essi v e,   sy m ptom ati c,   and  tr oubl esom e  f or m   (the  a c c e le ra te d pha s e )  that
r esponds  poor l y   to  ther apy   that  f or m er l y   contr ol l ed  the  chr oni c  phase.   Thi s
m etam or phosi s  i s  of ten  gr adual   and  m ani f ested  by   r ef r actor y   spl enom egal y ;
ex tr am edul l ar y   tum or   m asses;  changes  i n  the  bl ood,   bone  m ar r ow ,   and
di f f er enti al   cel l   counts;  and  new   cy togeneti c  abnor m al i ti es.   The  onset  of   f ev er
w i thout  i nf ecti on,   w eak ness,   ni ght  sw eats,   w ei ght  l oss,   ar thr al gi as,   and  bone
or   l ef t  upper   quadr ant  pai n  m ay   occur   bef or e  ther e  i s  l abor ator y   ev i dence  of
the  accel er ated  phase.   These  l abor ator y   abnor m al i ti es  i ncl ude  a  decr ease  i n
the  hem ogl obi n  content  w i th  i ncr easi ng  r ed  bl ood  cel l   abnor m al i ti es,   an  abr upt
i ncr ease  or   f al l   i n  the  w hi te  bl ood  cel l   count  w i thout  tr eatm ent,   and  an
i ncr ease  i n  the  num ber   of   bl ast  or   i m m atur e  cel l s.   Thr om bocy tosi s  or
thr om bocy topeni a  and  an  i ncr ease  i n  the  num ber   of   basophi l s  or   eosi nophi l s
ar e  al so  seen.

The  bla s tic  pha s e   can  be  m ani f ested  by   an  ex tr am edul l ar y   bl ast  i nf i l tr ati on  or
by   a  bone  m ar r ow   bl ast  cr i si s.

An  ex tr am edul l ar y   bl ast  cr i si s  i s  the  f i r st  m ani f estati on  of   the  accel er ated
phase  i n  appr ox i m atel y   10%  of   pati ents,   and  thi s  pr i nci pal l y   i nv ol v es  the
l y m ph  nodes,   ser osal   sur f aces,   sk i n  and  sof t  ti ssue,   br easts,   and  the  CN S.
Bone  i nv ol v em ent  m ay   l ead  to  sev er e  pai n,   tender ness,   and  r adi ogr aphi c
changes.   The  CN S  i nv ol v em ent  i s  usual l y   m eni ngeal   and  m ay   be  pr eceded  by
headache,   v om i ti ng,   stupor ,   cr ani al   ner v e  pal si es,   and  papi l l edem a;  i t  i s
associ ated  w i th  an  i ncr ease  i n  the  num ber   of   cel l s  and  the  pr otei n  l ev el ,   as
w el l   as  the  pr esence  of   bl ast  cel l s  i n  the  spi nal   f l ui d.

Acute  l euk em i a,   the  bl ast  phase,   dev el ops  i n  m ost  pati ents  w i th  CML,   and  thi s
can  tak e  f r om   day s  to  y ear s  to  occur   af ter   the  di agnosi s  of   CML  dependi ng  on
the  ef f ecti v eness  of   i ni ti al   tr eatm ent.   The  si gns  and  sy m ptom s  ar e  f ev er ,
hem or r hage,   bone  pai n,   and  l y m phadenopathy ,   as  w el l   as  the  other
m ani f estati ons  al r eady   ci ted.   The  bl asti c  tr ansf or m ati on  i s  usual l y   m y el obl asti c
or   m y el om onocy ti c,   but  can  be  er y thr ocy ti c  or   l y m phoi d  i n  natur e.   Speci al
stai ni ng  techni ques,   bi ochem i cal   assay s,   or   m onocl onal   anti body
deter m i nati ons  ar e  needed  to  i denti f y   the  ty pe  of   tr ansf or m ati on  once  the
pati ent  i s  i n  the  bl asti c  phase.   Pati ents  usual l y   di e  w i thi n  w eek s  to  m onths.
The  m edi an  sur v i v al   i n  pati ents  i n  the  m y el oi d  bl ast  cr i si s  i s  appr ox i m atel y   6
to  12  m onths,   and  that  i n  pati ents  i n  the  l y m phoi d  bl ast  cr i si s  i s  12  m onths,
w i th  sur v i v al   bey ond  2  y ear s  unusual .   Sev er e  i nf ecti ons,   hem or r hage,   and
or gan  dy sf uncti on,   especi al l y   of   the  l i v er   and  k i dney ,   ar e  am ong  the  l eadi ng
causes  of   death.

6.   What  ar e  the  l abor ator y   f i ndi ngs  encounter ed  i n  the  setti ng  of   CML?

The  di agnosi s  of   CML  can  be  m ade  on  the  basi s  of   the  he ma tologic  findings ,
speci f i cal l y   those  y i el ded  by   the  bl ood  count  and  the  bl ood  sm ear .   Com m on
f i ndi ngs  ar e  a  decr ease  i n  the  hem atocr i t;  the  pr esence  of   nucl eated  r ed  bl ood
cel l s  i n  the  ci r cul ati on;  a  l euk ocy te  count  that  i s  al w ay s  el ev ated,
P. 301
of ten  ex ceedi ng  1, 000  ×  10 9 /L;  the  pr esence  of   al l   stages  of   gr anul ocy te
dev el opm ent  i n  the  bl ood  w i th  a  gener al l y   nor m al   appear ance;  and  a  bl ast  cel l
pr ev al ence  r angi ng  f r om   0. 5%  to  5%.   My el ocy tes,   m etam y el ocy tes,   and  band
f or m s  account  f or   appr ox i m atel y   40%.   The  num ber   of   basophi l s  i s  i ncr eased,
as  i s  the  total   absol ute  l y m phocy te  count  (m ean,   appr ox i m atel y   15  ×  10 9 /L).
In  addi ti on,   the  pl atel et  count  i s  el ev ated  i n  appr ox i m atel y   50%  of   pati ents  at
the  ti m e  of   di agnosi s;  pl atel et  counts  m or e  than  1, 000  ×  10 9 /L  ar e  not
unusual ;  and  neutr ophi l   al k al i ne  phosphatase  acti v i ty   i s  l ow   or   absent  i n  m or e
than  90%  of   pati ents.   The  def ects  i n  w hi te  cel l   adhesi on,   em i gr ati on,   and
phagocy tosi s  ar e  m i l d  and  com pensated  f or   by   hi gh  neutr ophi l   concentr ati ons,
and  ther ef or e  do  not  pr edi spose  pati ents  i n  the  chr oni c  phase  to  i nf ecti ons.
Pl atel et  dy sf uncti on  can  occur   but  i s  not  associ ated  w i th  spontaneous  or
ex agger ated  bl eedi ng,   as  w i th  other   m y el opr ol i f er ati v e  di sor der s.

In  ter m s  of   the  morphologic  findings ,   the  bone  m ar r ow   i s  m ar k edl y


hy per cel l ul ar   and  hem atopoi eti c  ti ssue  tak es  up  75%  to  90%  of   the  m ar r ow
v ol um e.   Gr anul opoi esi s  i s  dom i nant,   w i th  a  gr anul ocy ti c–er y thr oi d  r ati o  of
betw een  10  and  30:1  (nor m al ,   2  to  4:1).   Er y thr opoi esi s  i s  usual l y   decr eased,
the  m egak ar y ocy tes  ar e  nor m al   or   i ncr eased  i n  num ber ,   and  the  popul ati on  of
eosi nophi l s  and  basophi l s  m ay   be  i ncr eased.

7.   What  ar e  the  cy togeneti c  and  bi ochem i cal   abnor m al i ti es  ty pi cal l y   f ound  i n
pati ents  w i th  CML?

The  Ph 1   chr om osom e,   desi gnated  t(9;22)(q34;q11),   i s  pr esent  i n  m or e  than


90%  of   pati ents  w i th  CML.   Dur i ng  the  bl ast  phase,   m ost  pati ents  ex hi bi t
addi ti onal   chr om osom e  abnor m al i ti es,   usual l y   a  + 8,   the  gai n  of   a  second  Ph 1
chr om osom e,   or   r ar el y   a  chr om osom e  l oss  (­7).

Var i ant  Ph 1   chr om osom e  tr ansl ocati ons  occur   i n  appr ox i m atel y   5%  of   pati ents
and  usual l y   consi st  of   com pl ex   r ear r angem ents.   Ev er y   chr om osom e  i s  i nv ol v ed
ex cept  the  Y  chr om osom e.   Ther e  i s  a  sm al l   gr oup  of   pati ents  w i th  CML  w ho  do
not  hav e  the  Ph 1   chr om osom e,   but  v i r tual l y   al l   pati ents  hav e  an  abnor m al
chr om osom e  22  w i th  bcr   r ear r angem ents.   The  char acter i sti c  bi ochem i cal
abnor m al i ti es  consi st  of   an  i ncr ease  i n  the  ur i c  aci d  l ev el ,   an  i ncr ease  i n  the
ser um   l ev el   of   cobal am i n­bi ndi ng  capaci ty ,   a  r ai sed  cobal am i n  concentr ati on,
an  i ncr ease  i n  the  LDH  l ev el ,   pseudohy per k al em i a  (an  i n  v i tr o  hy per k al em i a
secondar y   to  K +   r el ease  f r om   pl atel ets),   pseudohy pogl y cem i a  (secondar y   to
l euk ocy te  uti l i zati on  i n  v i tr o),   hy per cal cem i a,   hy per gam m agl obul i nem i a,   and
l ow   l euk ocy te  al k al i ne  phosphatase  acti v i ty .

8.   What  i s  the  tr eatm ent  f or   CML?

Al l   the  bi ochem i cal   al ter ati ons  m ust  be  cor r ected.   The  hy per ur i cem i a  m ust  be
tr eated  w i th  adequate  hy dr ati on  and  al l opur i nol .   How ev er ,   the  speci f i c
tr eatm ent  f or   the  di sease  depends  on  the  stage  and  goal   of   ther apy .

For   c he mothe ra py,   hy dr ox y ur ea  i s  used  m ost  of ten  because  i t  has  f ew er   si de


ef f ects  than  al k y l ati ng  agents,   w hi ch  can  i nduce  apl asti c  anem i a  and  acute
l euk em i a  i n  pati ents  w i th  CML.   Hy dr ox y ur ea  tr eatm ent  has  a  m i ni m al   ef f ect  on
sur v i v al ,   contr ol s  the  hem atol ogi c  al ter ati ons  (w i thout  suppr essi ng  the  Ph 1
chr om osom e),   and  i m pr ov es  the  pati ent's  qual i ty   of   l i f e.

P. 302
Both  α ­  and  γ­inte rfe rons   hav e  show n  anti l euk em i c  acti v i ty   i n  the  setti ng  of
CML;  α ­i nter f er on  pr oduces  a  nor m al i zati on  of   bl ood  counts  i n  appr ox i m atel y
75%  of   pati ents  and  suppr esses  the  Ph 1   chr om osom e  i n  appr ox i m atel y   15%  of
tr eated  pati ents.   The  Ph 1 ­negati v e  cel l   al so  l ack s  the  bcr   r ear r angem ents.

Dr aw back s  to  i nter f er on  tr eatm ent  ar e  that  m ai ntenance  ther apy   i s  r equi r ed
and  i t  i s  not  f r ee  of   si de  ef f ects.   Som e  studi es  suggest  that  the  pr ol onged  use
of   i nter f er on  (i . e. ,   > 1  y ear )  i n  r esponder s  m ay   m ak e  pati ents  l ess  r esponsi v e
to  bone  m ar r ow   tr anspl antati on.
Most  r ecentl y ,   ty r osi ne  k i nase  i nhi bi tor s,   especi al l y   i m ati ni b  can  l ead  to  a
bi ol ogi c  r esponse  (nor m al   m ol ecul ar   f i ndi ngs)  i n  m or e  than  50%  of   pati ents.
These  pati ents  m ay   r em ai n  i n  r em i ssi on  f or   5  y ear s  or   m or e  al though  som e
pati ents  ar e  star ti ng  to  show   r ecur r ence.   Spl eni c  irra dia tion  m ay be  usef ul   to
contr ol   spl enom egal y   and  to  pal l i ate  the  sy m ptom s  r esul ti ng  f r om   i t.
Sple ne c tomy  m ay   be  usef ul   i n  car ef ul l y   sel ected  pati ents  w i th  sy m ptom ati c
thr om bocy topeni a,   w ho  do  not  r espond  to  chem other apy   and  hav e  a  gr eatl y
enl ar ged  spl een;  how ev er ,   i t  i s  onl y   a  pal l i ati v e  m easur e.

Al l ogenei c  bone  m ar r ow   tr anspl antati ons  can  be  usef ul   i n  the  tr eatm ent  of
som e  pati ents  w i th  CML.   Thi s  tr eatm ent  can  er adi cate  the  Ph 1 ­car r y i ng  cl one
and  has  l ed  to  an  appar ent  cur e  of   som e  pati ents  w i th  CML.   How ev er ,   success
w i th  agents  such  as  i m ati ni b  and  the  hi gh  tox i ci ty   r esul ti ng  f r om   the
pr ocedur e,   par ti cul ar l y   i n  those  w ho  l ack   sui tabl e  donor s  or   ar e  of   adv anced
age,   l i m i t  i ts  use.

Le uk a phe re s is   can  be  usef ul   i n  tw o  ty pes  of   pati ents:  pr egnant  w om en  w i th  a
v er y   hi gh  w hi te  bl ood  cel l   count  and  hy per l euk ocy ti c  pati ents  w ho  need  r api d
cy tor educti on  to  al l ev i ate  the  si gns  and  sy m ptom s  of   l euk ostasi s.

9.   What  i s  the  pr ognosi s  i n  pati ents  w i th  CML?

In  pati ents  w i th  CML  w ho  do  not  attai n  a  cy togeneti c  r esponse,   the  m edi an
sur v i v al   r anges  f r om   45  to  60  m onths.   Wi th  i m pr ov ed  i ni ti al   ther apy
appr ox i m atel y   60%  to  80%  of   pati ents  sur v i v e  5  y ear s,   and  40%  sur v i v e  8
y ear s.

Case
A  37­y ear ­ol d  w hi te  m an  i s  seen  because  of   l ack   of   ener gy ,   ni ght  sw eats,   and  poor
appeti te  w i th  a  sensati on  of   f ul l ness  af ter   eati ng  ev en  v er y   sm al l   am ounts  of   f ood.
Phy si cal   ex am i nati on  r ev eal s  si gns  of   anem i a,   spl enom egal y ,   and  the  ex i stence  of
petechi ae.   A  com pl ete  bl ood  count  i s  per f or m ed  and  y i el ds  the  f ol l ow i ng  f i ndi ngs:
hem atocr i t,   25%;  pl atel ets,   300, 000/m m 3 ,   and  w hi te  bl ood  cel l s,   72, 000/m m 3 .   A
bone  m ar r ow   bi opsy   i s  per f or m ed  and  the  speci m en  i s  f ound  to  ex hi bi t  a
gr anul ocy ti c–er y thr oi d  r ati o  of   10:1  w i th  100%  cel l ul ar i ty   and  1%  bl astocy tes.

1.   What  i s  the  di f f er enti al   di agnosi s  i n  thi s  pati ent,   based  on  the  phy si cal
ex am i nati on  f i ndi ngs?
2.   On  the  basi s  of   the  hem atol ogi c  f i ndi ngs,   w hat  hem atopoi eti c  abnor m al i ti es
w oul d  y ou  ex pect  i n  thi s  pati ent  w i th  suspected  CML?
3.   What  do  the  bone  m ar r ow   f i ndi ngs  i ndi cate  i n  thi s  pati ent?
P. 303
4.   What  w oul d  be  the  m ost  speci f i c  test  f or   establ i shi ng  the  di agnosi s  of   CML  i n
thi s  pati ent?
5.   If   the  pati ent  i s  star ted  on  si ngl e­agent  chem other apy ,   w hat  w oul d  be  the
l i k el y   ef f ect?

Case Discussion
1.   What  i s  the  di f f er enti al   di agnosi s  i n  thi s  pati ent,   based  on  the  phy si cal
ex am i nati on  f i ndi ngs?

When  the  di agnosi s  of   CML  i s  consi der ed,   other   possi bi l i ti es,   such  as  a  sol i d
cancer ,   l y m phom as,   and  chr oni c  i nf ecti ons  m ust  be  ex cl uded.   These  other
di seases  m ay   cause  a  l euk em oi d  r eacti on  by   i ncr eased  sti m ul ati on  of   nor m al
m y el opoi esi s.   U sual l y   a  l euk em oi d  r eacti on  r esul ts  i n  a  w hi te  bl ood  cel l   count
of   l ess  than  100, 000/m m 3 ,   and  l ess  than  10%  of   cel l s  ar e  m y el ocy tes  or   m or e
i m m atur e  f or m s.

Because  nor m al   hem atopoi esi s  i s  suppr essed,   the  pati ent  coul d  ex hi bi t  the
si gns  and  sy m ptom s  of   anem i a,   such  as  headache,   pal pi tati ons,   pal l or ,   and
car di ac  f ai l ur e.   Ver y   r ar el y ,   l y m ph  node  enl ar gem ent  i s  f ound  i n  pati ents  w i th
CML.   Spl enom egal y   i s  al m ost  the  r ul e  i n  pati ents  w i th  CML,   and  i t  i s  the  sour ce
of   poor   appeti te  and  upper   abdom i nal   pai n,   such  as  that  seen  i n  thi s  pati ent.
Fi nal l y ,   petechi ae,   al though  possi bl e,   ar e  not  v er y   f r equent  f i ndi ngs  i n  pati ents
w i th  CML.

2.   On  the  basi s  of   the  hem atol ogi c  f i ndi ngs,   w hat  hem atopoi eti c  abnor m al i ti es
w oul d  y ou  ex pect  i n  thi s  pati ent  w i th  suspected  CML?

N or m al   hem atopoi esi s  i s  suppr essed  by   the  l euk em i c  acti v i ty   i n  the  bone
m ar r ow ,   l eadi ng  to  a  decr eased  num ber   of   r ed  bl ood  cel l s,   as  w el l   as
decr eased  hem ogl obi n  l ev el   and  hem atocr i t.   Ty pi cal l y ,   the  anem i a  of   CML  i s
nor m ochr om i c  nor m ocy ti c.   Hy pochr om i c  m i cr ocy ti c  anem i a  i s  ty pi cal   of   i r on
def i ci ency .

Al though  i m m atur e,   m ost  of   the  w hi te  bl ood  cel l s  l ook   m or phol ogi cal l y   nor m al ,
and  m atur e  neutr ophi l s,   band  f or m s,   m etam y el ocy tes,   and  m y el ocy tes
consti tute  m ost  of   the  w hi te  bl ood  cel l s  i n  thi s  pati ent.   Another   char acter i sti c
f i ndi ng  i s  an  i ncr eased  num ber   of   basophi l s.   If   m ost  of   the  cel l s  ar e  bl asts,
thi s  i ndi cates  acute  l euk em i a  i n  m ost  cases,   al though  i t  can  al so  i ndi cate  that
the  pati ent  i s  i n  the  bl asti c  phase  of   CML.

3.   What  do  the  bone  m ar r ow   f i ndi ngs  i ndi cate  i n  thi s  pati ent?

The  bone  m ar r ow   f i ndi ngs  ar e  consi stent  w i th  a  di agnosi s  of   CML,   and  bone
m ar r ow   bi opsy   consti tutes  an  i m por tant  par t  of   the  di agnosti c  ev al uati on  i n
pati ents  w i th  any   k i nd  of   l euk em i a  (acute  and  chr oni c).   N or m al l y ,   the
gr anul ocy ti c–er y thr oi d  r ati o  r anges  f r om   2  to  4:1,   but,   i n  the  setti ng  of   CML,
cel l s  of   w hi te  l i neage  pr edom i nate  and  i ncr em ents  of   any   f or m   of   w hi te  bl ood
cel l s,   f r om   m y el obl asts  to  m atur e  neutr ophi l s,   can  be  f ound.   An  i ncr em ent  i n
l y m phocy tes  and  r ed  bl ood  cel l   pr ecur sor s  i s  not  char acter i sti c  of   CML.   The
nor m al   bone  m ar r ow   cel l ul ar i ty   i s  50%  f at  and  50%  or   l ess  cel l s,   but,   i n  the
l euk em i as,   the  accel er ated  pr oducti on  of   abnor m al   cel l s  causes  the  f at  to  be
r epl aced,   and  the  cel l ul ar i ty   i ncr eases  to  100%.   Fi nal l y ,   ev en  i n  nor m al   bone
m ar r ow ,   a  v er y   sm al l   num ber   of   bl ast  cel l s  can  be  f ound;  i n  CML,   a  sm al l
per centage  of   bl ast  cel l s  can  be  f ound,   but  thi s  does  not
P. 304
necessar i l y   si gni f y   acute  l euk em i a.   In  bl ast  cr i si s  or   acute  l euk em i a,   at  l east
20%  of   the  cel l s  i n  the  bone  m ar r ow   ar e  bl ast  cel l s.

4.   What  w oul d  be  the  m ost  speci f i c  test  f or   establ i shi ng  the  di agnosi s  of   CML  i n
thi s  pati ent?

The  m ost  speci f i c  test  f or   establ i shi ng  the  di agnosi s  of   CML  i s  a  cy togeneti c
i nv esti gati on  f or   the  Ph 1   chr om osom e,   or   t(9;22),   w hi ch  i s  f ound  i n  90%  of
cases  of   CML.   Of   the  r em ai ni ng  10%  at  l east  hal f   w i l l   hav e  bcr   r ear r angem ents
m easur ed  by   i n  si tu  by   hy br i di zati on.

5.   If   the  pati ent  i s  star ted  on  si ngl e­agent  chem other apy ,   w hat  w oul d  be  the
l i k el y   ef f ect?

The  chem other apeuti c  agent  m ost  com m onl y   used  i n  the  tr eatm ent  of   CML  i s
hy dr ox y ur ea.   Thi s  ther apy   can  i m pr ov e  the  pati ent's  qual i ty   of   l i f e  by   r api dl y
decr easi ng  the  num ber   of   w hi te  bl ood  cel l s  and  pl atel ets.   It  does  not  pr ol ong
sur v i v al   v er y   m uch,   i f   at  al l ,   i n  pati ents  w i th  CML.   The  i nter f er ons  can  i nduce
com pl ete  hem atol ogi c  and  cy togeneti c  r em i ssi ons,   w i th  suppr essi on  of   the  Ph 1
chr om osom e  i n  pati ents  w i th  CML.   Most  i m por tantl y   ty r osi ne  k i nase  i nhi bi tor s
hav e  hi gh  i nci dence  of   bi ol ogi c  r esponses  and  l ess  tox i ci ty .

Al l ogenei c  bone  m ar r ow   tr anspl antati on  has  been  the  onl y   cur ati v e  tr eatm ent
f or   CML  but  has  a  hi gh  r ate  of   com pl i cati ons.   Adv anced  age  and  the  l ack   of
sui tabl e  donor s  pr ecl ude  i ts  use  i n  m any   pati ents,   but  i t  m ay   be  the  ther apy   of
choi ce  i n  thi s  37­y ear ­ol d  m an  i f   he  does  not  attai n  a  bi ol ogi c  r em i ssi on  or
r el apses  af ter   thi s  r em i ssi on  i s  attai ned.

Suggested Readings
Canel l os  G.   Cl i ni cal   char acter i sti cs  of   the  bl ast  phase  of   chr oni c  m y el ogenous
l euk em i a.   Hem atol   Oncol   Cl i n  N or th  Am   1990;4:359.

Kur zr ock   R,   Gutter m an  JU ,   Tal paz  M.   The  m ol ecul ar   geneti cs  of   Phi l adel phi a
chr om osom e  posi ti v e  l euk em i as.   N   Engl   J  Med  1988;319:990.

Qui ntas­Car dam a  A,   Cor tes  JE.   Chr oni c  m y el oi d  l euk em i a:  di agnosi s  and
tr eatm ent.   May o  Cl i n  Pr oc  2006;81:973.

Rei ter   E,   Gr ei ni x   HT,   Br ugger   S,   et  al .   Long  ter m   f ol l ow   up  af ter   al l ogenei c  stem
cel l   tr anspl antati on  f or   chr oni c  m y el ogenous  l euk em i a.   Bone  Mar r ow   Tr anspl ant
1998;4:S86.

Rodr i guez  J,   Cor tes  J,   Sm i th  T,   et  al .   Deter m i nati ons  of   pr ognosi s  i n  l ate
chr oni c­phase  chr oni c  m y el ogenous  l euk em i a.   J  Cl i n  Oncol   1998;16:3782.

Colon Cancer
1.   What  i s  the  i nci dence  of   col on  cancer ?

2.   What  ar e  som e  of   the  k now n  r i sk   f actor s  f or   col on  cancer ?

3.   Shoul d  pati ents  be  scr eened  f or   col on  cancer ?

4.   What  i s  the  cur r ent  tr eatm ent  f or   pr i m ar y   col on  cancer ?

5.   What  stagi ng  pr ocedur es  need  to  be  done  to  adequatel y   stage  a  pati ent  w i th
col on  cancer ?
6.   What  i s  the  stagi ng  sy stem   f or   col on  cancer ?

P. 305
7.   What  i s  the  pr ognosi s  f or   pati ents  w i th  col on  cancer ,   based  on  thei r   stage?

8.   What  shoul d  the  f ol l ow ­up  consi st  of   i n  pati ents  w i th  col on  cancer   af ter   they
hav e  under gone  pr i m ar y   sur gi cal   r esecti on  f or   cur ati v e  i ntent?

9.   Ar e  ther e  any   ef f ecti v e  adjuv ant  tr eatm ents  to  decr ease  the  r i sk   of   r ecur r ence
i n  pati ents  w i th  col on  cancer   w ho  hav e  under gone  r esecti on?

10.   Is  ther e  any   ef f ecti v e  chem other apy   f or   pati ents  w i th  m etastati c  di sease?

Discussion
1.   What  i s  the  i nci dence  of   col on  cancer ?

Ther e  ar e  m or e  than  140, 000  new   cases  of   col on  cancer   each  y ear   i n  the
U ni ted  States.   It  af f ects  appr ox i m atel y   1  of   ev er y   20  peopl e  i n  Wester n
cul tur es  and  accounts  f or   15%  of   al l   cancer s.   In  the  U ni ted  States,   the  actual
i nci dence  r ate  i s  appr ox i m atel y   35  cases  per   100, 000  popul ati on  per   y ear .

2.   What  ar e  som e  of   the  k now n  r i sk   f actor s  f or   col on  cancer ?

Ther e  ar e  sev er al   i nher i ted  col oni c  pol y posi s  sy ndr om es  associ ated  w i th  an
i ncr eased  r i sk   of   cancer   of   the  l ar ge  bow el .   The  m ost  i m por tant  one  i s  the
f am i l i al   adenom atosi s  sy ndr om e,   w hi ch  i s  i nher i ted  as  an  autosom al   dom i nant
tr ai t.   In  af f ected  peopl e,   pol y ps  dev el op  ov er   the  enti r e  l ength  of   the  col on  by
30  y ear s  of   age.   If   a  total   col ectom y   i s  not  per f or m ed,   the  cancer   r ate
escal ates  to  as  hi gh  as  80%  to  90%  by   45  y ear s  of   age.   Ther e  ar e  al so  other ,
l ess­f r equent  pol y posi s  sy ndr om es  pr edi sposi ng  to  col on  cancer .

Ther e  appear s  to  be  a  cer tai n  geneti c  tendency   tow ar d  col on  car ci nom a  that  i s
i ndependent  of   the  i nher i ted  pol y posi s  sy ndr om es.   Fi r st­degr ee  r el ati v es  of
peopl e  w i th  col on  cancer   di agnosed  bef or e  the  age  of   60  hav e  a  tw o­  to
thr eef ol d  gr eater   chance  of   acqui r i ng  col on  cancer   than  the  gener al   popul ati on.

Pati ents  w i th  i nf l am m ator y   bow el   di sease  ar e  al so  at  i ncr eased  r i sk   f or   col on
cancer .   Those  w i th  ul cer ati v e  col i ti s  hav e  appr ox i m atel y   a  50%  to  60%  chance
f or   dev el opm ent  of   l ar ge  bow el   car ci nom a  i f   a  col ectom y   i s  not  per f or m ed.
Cr ohn's  di sease  i s  al so  associ ated  w i th  an  i ncr eased  r i sk   of   col on  cancer ,   but
to  a  m uch  l esser   degr ee  than  ul cer ati v e  col i ti s.

The  f i ndi ngs  f r om   sev er al   popul ati on  studi es  hav e  suggested  that  di et  pl ay s  a
l ar ge  r ol e  i n  the  dev el opm ent  of   col on  cancer .   Cul tur es  i n  w hi ch  the  popul ace
consum es  a  hi gh­f at,   l ow ­f i ber   di et  ex hi bi t  an  i ncr eased  i nci dence  of   col on
cancer ,   com par ed  w i th  cul tur es  i n  w hi ch  a  l ow ­f at,   hi gh­f i ber   di et  i s  consum ed.
Dai l y   aspi r i n  m ay   hel p  pr ev ent  col on  cancer .

3.   Shoul d  pati ents  be  scr eened  f or   col on  cancer ?

The  pr ognosi s  f or   col on  cancer   i s  dr am ati cal l y   i m pr ov ed  the  ear l i er   i t  i s
detected  and  tr eated.   Scr eeni ng  pr ogr am s  ar e  ai m ed  at  detecti ng  col on  cancer s
at  an  ear l y   stage  and  hav e  l ed  to  an  i m pr ov em ent  i n  sur v i v al   and  i n  the  r i sk   of
r el apse.   Most  scr eeni ng  pr ogr am s  ar e  usual l y   di r ected  at  popul ati ons  w i th  a
hi gh  r i sk   f or   col on  cancer ,   i ncl udi ng  the  gr oups  al r eady   m enti oned.
Scr eeni ng  techni ques  f or   col on  cancer   com pr i se  di gi tal   r ectal   ex am i nati on,   the
testi ng  of   stool   f or   occul t  bl ood,   si gm oi doscopy   w i th  an  ai r ­contr ast  bar i um
enem a,   and  col onoscopy .   Recom m endati ons  ar e  that  peopl e  shoul d  be  check ed
P. 306
f or   occul t  bl ood  at  50  y ear s  of   age  and  y ear l y   ther eaf ter ,   w i th  col onoscopy
al so  per f or m ed  at  50  y ear s  and  ev er y   5  to  10  y ear s,   i f   negati v e  ther eaf ter .

4.   What  i s  the  cur r ent  tr eatm ent  f or   pr i m ar y   col on  cancer ?

The  pr i m ar y   tr eatm ent  f or   col on  cancer   i s  sur gi cal .   Once  the  cancer   has  been
di agnosed  and  pr eoper ati v e  stagi ng  per f or m ed,   the  pati ent  shoul d  be  r ef er r ed
to  an  oncol ogi c  sur geon  f or   def i ni ti v e  tr eatm ent.   The  ex act  sur gi cal   appr oach
used  i s  di ctated  by   the  tum or 's  l ocati on  i n  the  col on.   For   tr ue  col on  cancer s
(i . e. ,   cancer s  abov e  the  per i toneal   r ef l ecti on),   a  hem i col ectom y   i s  usual l y
per f or m ed.   For   r ectal   car ci nom as  (i . e. ,   tum or s  bel ow   the  per i toneal
r ef l ecti on),   a  l ow   anter i or   r esecti on  or   an  abdom i noper i neal   r esecti on  i s
per f or m ed.   Regar dl ess  of   the  sur gi cal   pr ocedur e,   a  thor ough  ex pl or ati on  of   the
enti r e  abdom en,   i ncl udi ng  the  l i v er ,   shoul d  be  car r i ed  out  and  any   suspect
l esi ons  sam pl ed  f or   bi opsy .

5.   What  stagi ng  pr ocedur es  need  to  be  done  to  adequatel y   stage  a  pati ent  w i th
col on  cancer ?

The  pr eoper ati v e  stagi ng  ev al uati on  of   pati ents  w i th  col on  cancer   i ncl udes
hi stor y   tak i ng,   phy si cal   ex am i nati on,   com pl ete  bl ood  count,   l i v er   f uncti on
tests,   the  car ci noem br y oni c  anti gen  (CEA)  l ev el ,   and  a  chest  r adi ogr aph.
Bef or e  sur ger y   f ur ther   i nv esti gati on  by   com puted  tom ogr aphi c  (CT)  scanni ng  or
posi tr on­em i ssi on  tom ogr aphy   (PET)  shoul d  be  per f or m ed.   Sur gi cal   and
pathol ogi c  stagi ng  shoul d  then  be  done  to  deter m i ne  the  ex act  stage  of   the
di sease.   If   the  pr eoper ati v e  CEA  l ev el   i s  el ev ated,   r epeat  m easur em ent  shoul d
be  per f or m ed  appr ox i m atel y   1  m onth  af ter   sur ger y   to  see  i f   i t  r etur ns  to
nor m al .

6.   What  i s  the  stagi ng  sy stem   f or   col on  cancer ?

Ther e  ar e  m any   stagi ng  sy stem s  f or   col on  cancer .   The  m ost  w i del y   used  i s  the
Aster ­Col l er   m odi f i cati on  of   the  Duk es'  stagi ng  sy stem   (Tabl e  7­7).   It  i s  based
on  the  depth  of   tum or   i nv asi on,   r egi onal   l y m ph  node  i nv ol v em ent,   and  di stant
m etastasi s.

Table 7­7 The TNM Staging Modi.cation of the
Dukes' Staging System

Sta ge De pth of Inva s ion Lymph Me ta s ta s e s


Node Dis ta nt
Sta tus

Stage Inv ades  subm ucosa N egati v e Absent


I

Stage Inv ades  m uscul ar i s  pr opr i a N egati v e Absent


I

Stage Inv ades  ser osa N egati v e Absent


II

Stage Inv ades  thr ough  bow el   w al l   i nto N egati v e Absent


IIB adjacent  or gans

Stage Inv ades  posi ti v e  m uscul ar i s Posi ti v e Absent


IIIA pr opr i a

Stage Inv ades  posi ti v e  ser osa Posi ti v e Absent


IIIB

Stage Inv ades  thr ough  bow el   w al l   i nto Posi ti v e Absent


IIIC adjacent  or gans

Stage Any   depth  of   i nv asi on — Pr esent


IV

P. 307

Table 7­8 Five­Year Survival Rates for Aster­
Coller Stages

Sta ge a 5­y Surviva l (% )

Stage  I 90–95

Stage  IIA 78

Stage  IIB 63

Stage  IIIA 74

Stage  IIIB 48

Stage  IIIC 38

Stage  IV <5
a See  Tabl e  7­7  f or   def i ni ti on  of   stages.

7.   What  i s  the  pr ognosi s  f or   pati ents  w i th  col on  cancer ,   based  on  thei r   stage?

Tabl e  7­8  l i sts  the  5­y ear   sur v i v al   r ates  f or   the  v ar i ous  stages  of   the  TN M
stagi ng  IV  and  Aster ­Col l er   m odi f i cati on  of   the  Duk es'  stagi ng  sy stem ;  thi s
tak es  i nto  account  i m pr ov em ent  i n  sur v i v al   w i th  adjuv ant  chem other apeuti c
r egi m ens.

8.   What  shoul d  the  f ol l ow ­up  consi st  of   i n  pati ents  w i th  col on  cancer   af ter   they
hav e  under gone  pr i m ar y   sur gi cal   r esecti on  f or   cur ati v e  i ntent?

Routi ne  schedul ed  f ol l ow ­up  i s  v er y   i m por tant  f or   the  ear l y   detecti on  of   l ocal
and  di stant  r ecur r ences,   as  w el l   as  new   pr i m ar y   col on  cancer .   Pati ents  shoul d
be  seen  ev er y   3  to  4  m onths  f or   the  f i r st  3  y ear s.   Fol l ow ­up  ev al uati on  shoul d
i ncl ude  hi stor y ,   phy si cal   ex am i nati on,   l i v er   f uncti on  tests,   m easur em ent  of   the
CEA  l ev el ,   and  com pl ete  bl ood  counts.   Col onoscopy   and  a  chest  r adi ogr aph
shoul d  be  obtai ned  y ear l y .   CT  scanni ng  shoul d  be  per f or m ed  f or   f ur ther
ev al uati on  of   r i si ng  l i v er   f uncti on  tests  or   CEA  l ev el s.   Af ter   3  y ear s,   the
i nter v al   betw een  these  ev al uati ons  can  be  i ncr eased.

9.   Ar e  ther e  any   ef f ecti v e  adjuv ant  tr eatm ents  to  decr ease  the  r i sk   of   r ecur r ence
i n  pati ents  w i th  col on  cancer   w ho  hav e  under gone  r esecti on?

Sev er al   studi es  hav e  show n  that  6  m onths  of   postoper ati v e  tr eatm ent  w i th  5­
f l uor our aci l ,   l eucov or i n,   and  ox al i pl ati n  can  decr ease  by   50%  the  l i k el i hood  of
stage  III  col on  cancer   r ecur r ence.   The  death  r ate  i n  thi s  setti ng  w as  r educed
by   33%.   Mor e  r ecentl y ,   sel ected  studi es  of   5­f l uor our aci l ,   l eucov or i n,   and
ox al i pl ati n  ther apy   hav e  show n  a  decr ease  i n  both  r ecur r ence  and  death  r ates
f or   pati ents  w i th  l ate  stage  IIB  col on  cancer .

10.   Is  ther e  any   ef f ecti v e  chem other apy   f or   pati ents  w i th  m etastati c  di sease?

5­Fl uor our aci l ,   l eucov or i n,   and  ox al i pl ati n  ther apy   (w hi ch  m odul ates  ox al i pl ati n
and  bev aci zum ab  angi ogenesi s)  has  l ed  to  an  i ncr ease  i n  the  r esponse  r ates  i n
m etastati c  di sease  f r om   60%  to  80%.   The  use  of   bi ol ogi c  agents  such  as
anti angi ogenesi s  f actor s  al one  and  ar bi tux   (anti ­EGFR)  i ncr eased  sur v i v al   i n
stage  IV  pati ents.

Case
A  72­y ear ­ol d  w hi te  m an  i s  seen  i n  the  em er gency   r oom   because  of   sev er e  f ati gue
and  v ague  abdom i nal   di scom f or t.   He  has  no  si gni f i cant  past  m edi cal   hi stor y   other
than  sl i ght
P. 308
anem i a,   w hi ch  w as  noted  dur i ng  a  phy si cal   ex am i nati on  3  y ear s  ago.   Hi s  hem atocr i t
at  that  ti m e  w as  37%,   and  w hi te  bl ood  cel l   and  pl atel et  counts  w er e  nor m al .
Phy si cal   ex am i nati on  w as  r em ar k abl e  onl y   f or   cachex i a  and  a  pal e  appear ance.   Hi s
i ni ti al   l abor ator y   v al ues  at  the  cur r ent  ti m e  ar e  as  f ol l ow s:  w hi te  bl ood  cel l   count,
7, 800/m m 3 ;  hem ogl obi n,   7  g/dL;  hem atocr i t,   21%;  pl atel ets,   600, 000/m m 3 ;  MCV,
62  f L;  AST,   89  m U /m L  (nor m al ,   0  to  35  m U /m L);  ALT,   129  m U /m L  (nor m al ,   0  to  38
m U /m L);  al k al i ne  phosphatase,   360  m U /m L  (nor m al ,   0  to  125  m U /m L);  and  total
bi l i r ubi n,   0. 7  m g/dL  (nor m al ,   < 1. 0  m g/dL).
The  pati ent  i s  adm i tted  to  the  hospi tal   f or   bl ood  tr ansf usi on  and  ev al uati on  of   hi s
anem i a.   The  adm i ssi on  chest  r adi ogr aph  show s  num er ous  pul m onar y   nodul es,   and  a
bar i um   enem a  ex am i nati on  r ev eal s  a  near ­obstr ucti ng  l esi on  at  the  hepati c  f l ex ur e.
A  CT  scan  of   the  l i v er   depi cts  num er ous  l ow ­densi ty   l esi ons  i n  both  l obes  of   the
l i v er .   Col onoscopy   i s  per f or m ed,   and  thi s  r ev eal s  a  m ucosal   l esi on  at  the  hepati c
f l ex ur e.   Bi opsy   of   thi s  l esi on  r ev eal s  adenocar ci nom a.

1.   What  i s  the  cause  of   thi s  pati ent's  anem i a?


2.   Woul d  ear l i er   di agnosi s  of   the  cause  of   thi s  pati ent's  anem i a  hav e  m ade  any
di f f er ence?
3.   What  ty pe  of   tr eatm ent  w oul d  y ou  now   adv i se  f or   the  l esi on  i n  the  hepati c
f l ex ur e  of   the  col on?
4.   Woul d  y ou  r ecom m end  any   other   tr eatm ents  f or   the  l esi ons  i n  the  l ung  or
l i v er ?
5.   What  stage  i s  thi s  pati ent's  cancer ?
6.   What  i s  the  pr ognosi s  i n  thi s  pati ent?

Case Discussion
1.   What  i s  the  cause  of   thi s  pati ent's  anem i a?

The  pati ent  al m ost  cer tai nl y   has  i r on­def i ci ency   anem i a  secondar y   to  the
chr oni c  bl ood  l oss  i n  the  stool   stem m i ng  f r om   a  bl eedi ng  col on  cancer .

2.   Woul d  ear l i er   di agnosi s  of   the  cause  of   thi s  pati ent's  anem i a  hav e  m ade  any
di f f er ence?

It  i s  di f f i cul t  to  say   w hether   an  ear l i er   di agnosi s  w oul d  hav e  def i ni tel y   m ade  a
di f f er ence.   It  shoul d  cer tai nl y   hav e  been  possi bl e  to  di agnose  the  col on  cancer ,
and  at  an  ear l i er   stage  the  pr ognosi s  w oul d  l i k el y   hav e  been  better .
Recom m ended  scr eeni ng  pr ocedur es,   par ti cul ar l y   col onoscopy ,   w oul d  pr obabl y
hav e  i m pr ov ed  thi s  pati ent's  outcom e.

3.   What  ty pe  of   tr eatm ent  w oul d  y ou  now   adv i se  f or   the  l esi on  i n  the  hepati c
f l ex ur e  of   the  col on?

The  pati ent  needs  to  under go  a  hem i col ectom y   to  pr ev ent  obstr ucti on.   Thi s  i s
onl y   f or   pal l i ati on  and  w i l l   not  af f ect  the  ov er al l   pr ognosi s.

4.   Woul d  y ou  r ecom m end  any   other   tr eatm ents  f or   the  l esi ons  i n  the  l ung  or
l i v er ?

Thi s  pati ent  has  m etastati c  di sease  and  shoul d  be  of f er ed  sy stem i c
chem other apy   consi sti ng  of   5­f l uor our aci l ,   ox al i pl ati n,   and  l eucov or i n  w i th  an
anti angi ogenesi s  f actor :  the  standar d  appr oach  f or   m etastati c  col on  cancer .
The  best  doses  and  schedul es  of   adm i ni str ati on  ar e  y et  to  be  deter m i ned.   If
the  pati ent  does  not  r espond  to  thi s  standar d  ther apy   he  shoul d  be  enr ol l ed  i n
a  cl i ni cal   tr i al .

P. 309
5.   What  stage  i s  thi s  pati ent's  cancer ?

Thi s  pati ent  cl ear l y   has  m etastati c  di sease,   and  ther ef or e  i s  i n  stage  D
accor di ng  to  the  Aster ­Col l er   m odi f i cati on  of   the  Duk es'  stagi ng  sy stem .

6.   What  i s  the  pr ognosi s  i n  thi s  pati ent?

Thi s  pati ent  has  i ncur abl e  cancer .   He  has  appr ox i m atel y   a  30%  to  45%  chance
of   r espondi ng  to  standar d  ther apy .   If   he  r esponds,   he  w i l l   l i k el y   l i v e  l onger .   He
has  appr ox i m atel y   a  35%  to  40%  chance  of   5­y ear   sur v i v al .

Suggested Readings
Mi dgl ey   R,   Ker r   D.   Col or ectal   cancer .   Lancet  1999;353:391.

Sm i th  RE,   Col angel o  L,   Wi eand  HS,   et  al .   Random i zed  tr i al   of   adjuv ant  ther apy
i n  col on  car ci nom a:  10­y ear   r esul ts  of   N SABP  pr otocol   C­01.   J  N atl   Cancer   Inst
2004;96:1128.

Steel e  G  Jr .   Com bi ned­m odal i ty   ther apy   f or   r ectal   car ci nom a:  the  ti m e  has
com e.   N   Engl   J  Med  1991;324:764.

Steel e  G  Jr ,   Bl eday   R,   May er   RJ,   et  al .   A  pr ospecti v e  ev al uati on  of   hepati c


r esecti on  f or   col or ectal   car ci nom a  m etastases  to  the  l i v er :  Gastr oi ntesti nal
Tum or   Study   Gr oup  Pr otocol   6584.   J  Cl i n  Oncol   1991;9:1105.

Steel e  G  Jr ,   Bur t  R,   Wi naw er   SJ,   eds.   Basi c  and  cl i ni cal   per specti v es  of
col or ectal   pol y ps  and  cancer .   N ew   Yor k :  Al an  R.   Li ss,   1988.

Erythrocytosis
1.   What  i s  er y thr ocy tosi s?

2.   What  ar e  the  tw o  m ajor   ty pes  of   er y thr ocy tosi s?

3.   What  ar e  som e  causes  of   secondar y   er y thr ocy tosi s  due  to  appr opr i ate
er y thr opoi eti n  secr eti on?

4.   What  ar e  som e  causes  of   secondar y   er y thr ocy tosi s  due  to  i nappr opr i ate
er y thr opoi eti n  secr eti on?

5.   What  i s  pol y cy them i a  v er a?

6.   What  ar e  the  sy m ptom s  of   pol y cy them i a  v er a?

7.   How   i s  pol y cy them i a  v er a  di agnosed?

8.   What  i s  the  l i k el y   l ength  of   sur v i v al   i n  a  pati ent  w i th  pol y cy them i a  v er a?

9.   What  i s  the  r ar e  hepati c  com pl i cati on  that  can  ar i se  i n  pati ents  w i th
pol y cy them i a  v er a?

Discussion
1.   What  i s  er y thr ocy tosi s?

A  pati ent  w i th  a  hem atocr i t  gr eater   than  55%  that  i s  not  due  to  dehy dr ati on  i s
consi der ed  to  hav e  an  er y thr ocy tosi s.   The  chr om i um   151–l abel ed  r ed  bl ood
cel l   m easur em ent  of   the  total   r ed  bl ood  cel l   m ass  i s,   how ev er ,   the  gol d
standar d  f or   establ i shi ng  the  di agnosi s,   but  i f   the  pati ent  has  a  hem atocr i t
gr eater   than  60%,   no  f ur ther   studi es  ar e  necessar y .

P. 310
2.   What  ar e  the  tw o  m ajor   ty pes  of   er y thr ocy tosi s?

When  the  el ev ated  hem atocr i t  i s  due  to  i ncr eased  er y thr opoi eti n  secr eti on,   thi s
consti tutes  secondar y   er y thr ocy tosi s.   Pr i m ar y   er y thr ocy tosi s  i s  caused  by
i ncr eased  r ed  bl ood  cel l   pr oducti on  that  does  not  stem   f r om   i ncr eased
er y thr opoi eti n  secr eti on.

3.   What  ar e  som e  causes  of   secondar y   er y thr ocy tosi s  due  to  appr opr i ate
er y thr opoi eti n  secr eti on?

Any   di sor der   that  causes  ti ssue  hy pox i a  sti m ul ates  the  r enal   pr oducti on  of
er y thr opoi eti n.   These  di sor der s  i ncl ude  chr oni c  obstr ucti v e  l ung  di sease,   l i v i ng
at  hi gh  al ti tudes,   hem ogl obi n  (Hb  Chesapeak e  and  m ethem ogl obi n)  that  does
not  r el ease  ox y gen  cor r ectl y ,   or   car di ac  di sease  that  causes  “r i ght­to­l ef tâ€​
shunti ng.   In  r el ati v e  er y thr ocy tosi s,   the  r ed  bl ood  cel l   m ass  i s  nor m al ,   and  thi s
occur s  i n  the  setti ngs  of   dehy dr ati on  or   decr eased  pl asm a  v ol um e.

4.   What  ar e  som e  causes  of   secondar y   er y thr ocy tosi s  due  to  i nappr opr i ate
er y thr opoi eti n  secr eti on?

Many   di sease  states  can  be  associ ated  w i th  i ncr eased  er y thr opoi eti n
pr oducti on.   Di seased  k i dney s  m ay   secr ete  er y thr opoi eti n  i nappr opr i atel y   or
tum or s  m ay   secr ete  hor m ones  that  f uncti on  l i k e  er y thr opoi eti n.   Renal ,   adr enal ,
or   hepati c  tum or s,   ov ar i an  car ci nom a,   or   beni gn  uter i ne  m y om as  al l   secr ete
er y thr opoi eti n­l i k e  substances.   Other   causes  of   i ncr eased  er y thr opoi eti n
secr eti on  ar e  r enal   ar ter y   stenosi s,   hy dr onephr osi s,   r enal   cy sts,   or   r enal
tr anspl antati on.

5.   What  i s  pol y cy them i a  v er a?

Pol y cy them i a  v er a  i s  an  absol ute  er y thr ocy tosi s  secondar y   to  the  cl onal
ex pansi on  of   r ed  bl ood  cel l s,   m ak i ng  i t  a  m y el opr ol i f er ati v e  di sor der .

6.   What  ar e  the  sy m ptom s  of   pol y cy them i a  v er a?

Sy m ptom s  stem   f r om   v ascul ar   congesti on  or   obstr ucti on  due  to  i ncr eased  bl ood
v i scosi ty .   Pati ents  com pl ai n  of   headaches,   i tchi ng  and  bur ni ng  f eet,   or   m al ai se.
The  r eti nal   v ei ns  becom e  engor ged  and  hepatospl enom egal y   m ay   be  pr esent.
The  i nci dence  of   car di ov ascul ar   and  cer ebr ov ascul ar   di sease  i s  i ncr eased  i n
these  pati ents  because  of   the  el ev ated  bl ood  v i scosi ty .

7.   How   i s  pol y cy them i a  v er a  di agnosed?

An  i ncr eased  r ed  bl ood  cel l   m ass,   an  ox y gen  satur ati on  of   92%  or   m or e,   and
spl enom egal y   ar e  the  car di nal   si gns  of   pol y cy them i a  v er a.   If   spl enom egal y   i s
not  pr esent,   pol y cy them i a  v er a  i s  ev i dent  i f   the  pati ent  has  a  pl atel et  count
that  ex ceeds  400, 000/m m 3 ,   a  w hi te  bl ood  cel l   count  gr eater   than  12, 000/m m 3 ,
a  cobal am i n  l ev el   of   900  pg/m L  or   gr eater ,   or   an  el ev ated  neutr ophi l   al k al i ne
phosphatase  scor e.
8.   What  i s  the  l i k el y   l ength  of   sur v i v al   i n  a  pati ent  w i th  pol y cy them i a  v er a?

Wi thout  tr eatm ent,   hal f   the  pati ents  di e  w i thi n  24  m onths,   usual l y   due  to
v ascul ar   di sease.   Phl ebotom y   to  m ai ntai n  a  hem atocr i t  of   45%  can  pr ol ong  the
l i f e  span  to  m or e  than  6  y ear s,   and  m edi an  sur v i v al   i n  pati ents  w ho  r ecei v e
ef f ecti v e  chem other apy   i s  12. 5  y ear s.

P. 311
9.   What  i s  the  r ar e  hepati c  com pl i cati on  that  can  ar i se  i n  pati ents  w i th
pol y cy them i a  v er a?

Budd­Chi ar i   sy ndr om e  i s  an  occl usi on  of   the  hepati c  v ei ns  som eti m es  seen  i n
pati ents  w i th  pol y cy them i a  v er a  and  other   di seases  that  i ncr ease  bl ood
v i scosi ty .   Thi s  causes  r i ght  upper   quadr ant  pai n  and  el ev ati ons  i n  the  l i v er
enzy m e  l ev el s.   It  i s  v er y   di f f i cul t  to  tr eat,   and  i s  best  pr ev ented  by   tr eati ng
the  er y thr ocy tosi s  aggr essi v el y .

Case
A  55­y ear ­ol d  m an  w ho  i s  a  sm ok er   and  has  hy per tensi on  sees  hi s  i nter ni st  because
of   m al ai se  and  nasal   stuf f i ness  w i th  f ul l   sensati on  i n  hi s  f r ontal   si nuses.   On  f ur ther
questi oni ng,   the  pati ent  al so  descr i bes  hav i ng  i tchy ,   r ed  f eet  that  w or sen  i n  the
show er .   The  pati ent  has  no  shor tness  of   br eath  w i th  acti v i ty   and  does  not  snor e  or
ex per i ence  day ti m e  dr ow si ness.
Phy si cal   ex am i nati on  r ev eal s  a  pl ethor i c  pati ent  w ho  i s  i n  no  acute  di str ess.   Hi s
l ungs  ar e  cl ear   to  auscul tati on.   Hi s  l i v er   span  i s  18  cm   and  hi s  spl een  ti p  i s
pal pabl e.
The  f ol l ow i ng  l abor ator y   v al ues  ar e  r epor ted:  hem atocr i t,   65%;  w hi te  bl ood  cel l
count,   8, 500/m m 3 ;  pl atel ets,   210, 000/m m 3 ;  and  di f f er enti al :  50%  segm ented
neutr ophi l s,   30%  l y m phocy tes,   3%  basophi l s,   and  10%  m onocy tes.
Ar ter i al   bl ood  gas  deter m i nati ons  per f or m ed  on  r oom   ai r   r ev eal   a  par ti al   pr essur e
of   ox y gen  of   65  m m   Hg,   a  par ti al   pr essur e  of   car bon  di ox i de  of   38  m m   Hg,   and  an
ox y gen  satur ati on  of   93%.

1.   What  i s  the  di agnosi s  i n  thi s  pati ent?


2.   Why   i s  i t  i m por tant  to  k now   w hether   the  pati ent  snor es  or   ex per i ences  day ti m e
dr ow si ness?
3.   What  i s  the  cause  of   thi s  pati ent's  nasal   stuf f i ness?
4.   What  shoul d  be  the  i ni ti al   tr eatm ent  i n  thi s  pati ent?
5.   What  i s  thi s  pati ent's  pr ognosi s?

Case Discussion
1.   What  i s  the  di agnosi s  i n  thi s  pati ent?

Thi s  pati ent  m ost  l i k el y   has  pol y cy them i a  v er a.   The  ox y gen  satur ati on  gr eater
than  90%  and  the  pr esence  of   spl enom egal y   suppor t  the  di agnosi s.   The
pr esence  of   m ononucl ear   and  basophi l i c  cel l s  al so  suppor ts  the  di agnosi s  of   a
m y el opr ol i f er ati v e  di sor der ,   w hi ch  w oul d  be  f ur ther   suppor ted  by   a  bone
m ar r ow   bi opsy   that  show s  tr i l i near   hy per pl asi a.
2.   Why   i s  i t  i m por tant  to  k now   w hether   the  pati ent  snor es  or   ex per i ences  day ti m e
dr ow si ness?

Snor i ng  and  day ti m e  dr ow si ness  ar e  sy m ptom s  of   sl eep  apnea,   a  cause  of


secondar y   er y thr ocy tosi s.   Al though  phl ebotom y   can  cur e  the  pati ent's
er y thr ocy tosi s,
P. 312
i t  cannot  tr eat  the  ni ghtti m e  hy pox i a  or   sl eep  apnea,   and  the  pati ent  coul d  go
on  to  hav e  r i ght­si ded  hear t  f ai l ur e.

3.   What  i s  the  cause  of   thi s  pati ent's  nasal   stuf f i ness?

Al though  he  m ay   hav e  a  si nus  i nf ecti on,   the  nasal   stuf f i ness  i s  m ost  l i k el y   due
to  i ncr eased  bl ood  v i scosi ty .

4.   What  shoul d  be  the  i ni ti al   tr eatm ent  i n  thi s  pati ent?

Phl ebotom y   shoul d  be  per f or m ed  as  soon  as  possi bl e  to  decr ease  the
hem atocr i t  to  45%  to  50%.   The  i ncr eased  bl ood  v i scosi ty   pl aces  thi s  pati ent
w ho  has  tw o  other   r i sk   f actor s  f or   ather oscl er oti c  di sease,   nam el y   sm ok i ng
and  hy per tensi on,   at  r i sk   f or   a  str ok e  or   car di ov ascul ar   acci dent.

5.   What  i s  thi s  pati ent's  pr ognosi s?

Ev en  w i th  car ef ul   tr eatm ent  of   hi s  er y thr ocy tosi s  w i th  phl ebotom y   and
chem other apy ,   hi s  l i f e  ex pectancy   w i l l   pr obabl y   be  m or e  l i m i ted  because  of   hi s
sm ok i ng  and  hy per tensi on.

Suggested Readings
Conl ey   CL.   Pol y cy them i a  v er a,   di agnosi s  and  tr eatm ent.   Hosp  Pr act
1987;22:107.

El l i s  JT,   Peter son  P,   Gel l er   SA,   et  al .   Studi es  of   the  bone  m ar r ow   i n
pol y cy them i a  v er a  and  the  ev ol uti on  of   m y el of i br osi s  and  second  hem atol ogi c
m al i gnanci es.   Sem i n  Hem atol   1986;23:144.

Mur phy   S.   Di agnosti c  cr i ter i a  and  pr ognosi s  i n  pol y cy them i a  v er a  and  essenti al
thr om bocy topeni a.   Sem i n  Hem atol   1999;36:9.

Schw ar ts  RS.   Pol y cy them i a  v er a:  chance,   death,   and  m utabi l i ty   [Edi tor i al ].   N
Engl   J  Med  1998;338:613.

Lymphomas
1.   How   and  w hen  does  Hodgk i n's  di sease  ty pi cal l y   pr esent?

2.   What  i s  the  r el ati onshi p  betw een  the  hi stol ogi c  patter ns  and  the  stage  i n
Hodgk i n's  di sease?

3.   Of   w hat  shoul d  the  stagi ng  ev al uati on  i n  pati ents  w i th  Hodgk i n's  di sease
consi st,   and  how   do  the  f i ndi ngs  hav e  an  i m pact  on  ther apy ?
4.   What  ar e  the  cur e  r ates  and  the  l ong­ter m   sequel ae  of   the  tr eatm ent  f or
Hodgk i n's  di sease?

5.   What  ar e  the  k now n  causes  or   di seases  associ ated  w i th  the  dev el opm ent  of
non–Hodgk i n's  l y m phom a?

6.   How   does  the  Wor l d  Heal th  Or gani zati on's  w or k i ng  f or m ul ati on  of   nonâ
€“Hodgk i n's  l y m phom a  di f f er   f r om   the  ol der   cl assi f i cati ons?

7.   How   does  the  bi ol ogy   of   hi gh­gr ade  l y m phom a  di f f er   f r om   that  of   l ow ­gr ade
l y m phom a,   and  how   does  thi s  af f ect  tr eatm ent  and  sur v i v al ?

8.   What  ty pe  of   l y m phom a  ty pi cal l y   i nv ol v es  the  sk i n,   and  how   does  thi s  i nf l uence
stagi ng  and  tr eatm ent?

P. 313
9.   What  popul ati on  of   pati ents  i s  pr one  to  acqui r i ng  secondar y   CN S  l y m phom a,
and  can  thi s  be  pr ev ented?

10.   What  com pl i cati on  of   ther apy   can  occur   i n  the  setti ng  of   r api dl y   gr ow i ng
tum or s,   and  how   can  thi s  be  pr ev ented?

Discussion
1.   How   and  w hen  does  Hodgk i n's  di sease  ty pi cal l y   pr esent?

Hodgk i n's  di sease  ty pi cal l y   pr esents  i n  adol escence  or   y oung  adul thood.
How ev er ,   a  bi m odal   age  di str i buti on  has  been  obser v ed,   especi al l y   i n
dev el oped  countr i es.   The  f i r st  peak   i s  i n  adol escence  or   y oung  adul thood,
w her eas  the  second  peak   occur s  at  55  y ear s  of   age.   Hodgk i n's  di sease  ty pi cal l y
pr esents  as  a  w ax i ng  and  w ani ng  adenopathy ,   m ost  com m onl y   i n  the  neck   or
supr acl av i cul ar   ar ea.   Fi f ty   per cent  of   pati ents  pr esent  w i th  a  m edi asti nal   m ass
v i si bl e  on  chest  r adi ogr aphy ,   and  40%  pr esent  w i th  B  sy m ptom s  (f ev er ,   ni ght
sw eats,   and  10%  w ei ght  l oss  i n  the  pr ecedi ng  6  m onths).

2.   What  i s  the  r el ati onshi p  betw een  the  hi stol ogi c  patter ns  and  the  stage  i n
Hodgk i n's  di sease?

Ther e  ar e  f our   di sti nct  hi stol ogi c  patter ns  seen  i n  Hodgk i n's  di sease,   al l   of
w hi ch  possess  the  Ster nber g­Reed  cel l .   The  f our   hi stol ogi c  patter ns  and  thei r
pr ev al ences  ar e  nodul ar   scl er osi s  (70%),   l y m phocy te  pr edom i nance  (15%),
m i x ed  cel l ul ar i ty   (10%),   and  l y m phocy te  depl eti on  (5%).

Hodgk i n's  di sease  i s  staged  accor di ng  to  the  Ann  Ar bor   cl assi f i cati on:

Stage  I.   Inv ol v em ent  of   a  si ngl e  l y m ph  node  r egi on  (I)  or   a  si ngl e
ex tr al y m phati c  or gan  or   si te  (I E ).

Stage  II.   Inv ol v em ent  of   tw o  or   m or e  l y m ph  node  r egi ons  on  the  sam e
si de  of   the  di aphr agm   (II),   or   l ocal i zed  i nv ol v em ent  of   tw o  or   m or e
ex tr al y m phati c  or gans  or   si tes  (II E ).

Stage  III.   Inv ol v em ent  of   l y m ph  node  r egi ons  on  both  si des  of   the
di aphr agm   (III),   or   l ocal i zed  i nv ol v em ent  of   an  ex tr al y m phati c  or gan  or
si te  (III E )  or   spl een  (III S ),   or   both  (III SE ).   III 1   r ef er s  to  i nv ol v em ent  of
l y m ph  nodes  i n  the  upper   abdom en;  III 2   r ef er s  to  i nv ol v em ent  of   l ow er
abdom i nal   nodes.

Stage  IV.   Di f f use  or   di ssem i nated  i nv ol v em ent  of   one  or   m or e


ex tr al y m phati c  or gans,   w i th  or   w i thout  associ ated  l y m ph  node
i nv ol v em ent.   The  or gan,   or   or gans,   i nv ol v ed  m ay   be  i denti f i ed  by   a
sy m bol .

Al so:  A  =   asy m ptom ati c;  B  =   f ev er ,   ni ght  sw eats,   and  w ei ght  l oss
ex ceedi ng  10%  of   the  total   body   w ei ght.

Hodgk i n's  di sease  spr eads  thr ough  conti guous  l y m ph  nodes;  how ev er ,   the
spl een  i s  com m onl y   the  onl y   si te  of   i nv ol v em ent  i n  the  abdom en,   and  i ts
i nv ol v em ent  i s  thought  to  be  due  to  l y m phati c  spr ead.

Hi stol ogi c  pr ogr essi on  i n  Hodgk i n's  di sease  i nv ol v es  the  pr ogr essi v e  l oss  of
l y m phocy tes.   For   ex am pl e,   l y m phocy te  pr edom i nance  can  pr ogr ess  to  m i x ed
cel l ul ar i ty   and  ev entual l y   l y m phocy te  depl eti on.   Those  pati ents  w ho  pr esent
w i th  nodul ar   scl er osi s  m ay   al so  ex per i ence  som e  changes  i n  hi stol ogi c  ty pe,
al though  m ost  do  not  show   obv i ous  hi stol ogi c  pr ogr essi on.   The  hi stol ogi c
P. 314
patter n  al so  cor r el ates  w i th  the  stage  of   the  di sease,   and  ther eby   the
pr ognosi s.   N odul ar   scl er osi s  and  l y m phocy te  pr edom i nance  ar e  m or e  com m onl y
seen  i n  ear l y   di sease  (stages  I  and  II).   Ther ef or e,   these  hi stol ogi c  patter ns  ar e
associ ated  w i th  a  better   outcom e.   Mi x ed  cel l ul ar i ty   and  l y m phocy te  depl eti on
ar e  associ ated  w i th  a  poor er   pr ognosi s  and  ar e  of ten  seen  i n  pati ents  w i th
adv anced  di sease.

3.   Of   w hat  shoul d  the  stagi ng  ev al uati on  i n  pati ents  w i th  Hodgk i n's  di sease
consi st,   and  how   do  the  f i ndi ngs  hav e  an  i m pact  on  ther apy ?

The  stagi ng  ev al uati on  i n  pati ents  w i th  Hodgk i n's  di sease  i ncl udes  a  com pl ete
hi stor y   and  phy si cal   ex am i nati on.   Labor ator y   i nv esti gati ons  shoul d  i ncl ude  a
com pl ete  bl ood  count  and  ev al uati on  of   the  sm ear   f or   changes  i ndi cati ng
anem i a,   hem ol y si s,   or   abnor m al   w hi te  bl ood  cel l s  as  w el l   as  a  di f f er enti al ,
deter m i nati ons  of   the  sedi m entati on  r ate  and  al k al i ne  phosphatase  l ev el ,   and
ev al uati on  of   l i v er   and  r enal   f uncti on.   The  r adi ol ogi c  ev al uati on  shoul d  al w ay s
i ncl ude  CT  studi es  of   the  chest,   abdom en,   and  pel v i s  or   CT/PET  scans.

A  di agnosi s  based  on  ti ssue  f i ndi ngs  i s  a  m ust.   N eedl e  aspi r ati on  or   cy tol ogi c
f i ndi ngs  i s  not  adequate  because  the  ti ssue  obtai ned  by   these  m ethods  y i el ds
no  i nf or m ati on  about  the  nodal   ar chi tectur e.   It  i s  pr ef er abl e  to  obtai n  a  l y m ph
node  or   w edge  of   a  l ar ge  m ass,   but  ev en  then  i t  m ay   tak e  m or e  than  one
l y m ph  node  bi opsy   to  docum ent  the  pr esence  of   the  di sease  i f   onl y   r eacti v e
hy per pl asi a  i s  seen.   Bone  m ar r ow   bi opsy   i s  a  r equi r ed  par t  of   the  stagi ng
w or k up,   par ti cul ar l y   i n  sy m ptom ati c  pati ents,   but  shoul d  not  be  substi tuted  f or
the  ti ssue  ex am i nati on  because,   agai n,   the  nodal   ar chi tectur e  cannot  be
obser v ed.   Gener al   gui del i nes  no  l onger   suggest  per f or m i ng  a  stagi ng
l apar otom y   unl ess  i f   the  r esul ts  w oul d  af f ect  the  natur e  of   ther apy .   Thi s  m ay
happen  i n  ear l y   stage  di sease  (i . e. ,   stages  IB,   IIB,   and  IIIA),   w hen  the  f i ndi ngs
f r om   l apar otom y   coul d  al ter   a  deci si on  to  use  r adi ati on  ther apy   al one.

The  choi ce  of   ther apy   i n  pati ents  w i th  Hodgk i n's  di sease  i s  gov er ned  by   stage.
Pati ents  w i th  stage  I  and  II  di sease  can  be  tr eated  w i th  r adi ati on  ther apy
al one.   If   ther e  i s  bul k y   di sease,   com bi ned  chem other apy   and  i r r adi ati on  shoul d
be  used.   For   pati ents  w i th  stage  III  and  IV  di sease,   chem other apy   shoul d  be
used  w i th  r adi ati on  del i v er ed  to  si tes  of   bul k y   di sease.   Ther e  i s  sti l l   som e
contr ov er sy   about  w hat  i s  the  best  tr eatm ent  f or   stage  IIB  and  IIIA  di sease.
Most  ther apeuti c  opti ons  hav e  hi gh  r em i ssi on  r ates,   and  f r equentl y   l ong­ter m
si de  ef f ects  di ctate  the  choi ce.

4.   What  ar e  the  cur e  r ates  and  the  l ong­ter m   sequel ae  of   the  tr eatm ent  f or
Hodgk i n's  di sease?

Com bi nati on  chem other apy   and  adv ances  i n  r adi ati on  ther apy   hav e  achi ev ed
ov er al l   cur e  r ates  of   appr ox i m atel y   70%  i n  pati ents  w i th  Hodgk i n's  di sease.
The  cur e  r ates  seen  f or   ear l y ­stage  di sease  ar e  gr eater ,   w i th  a  90%  to  95%
l ong­ter m   sur v i v al   r ate  obser v ed  f or   pati ents  w i th  stage  I  and  II  di sease.   The
gr eat  num ber   of   sur v i v or s  has  al l ow ed  l ong­ter m   f ol l ow ­up  and  a  study   of   the
ef f ects  of   com bi nati on  chem other apy   and  r adi ati on  ther apy .

Hodgk i n's  di sease  i s  associ ated  w i th  i m m unol ogi c  abnor m al i ti es  i nv ol v i ng
changes  i n  both  l y m phocy te  f uncti on  and  hum or al   i m m uni ty .   These  def ects
P. 315
ar e  aggr av ated  by   tr eatm ent  and  thi s  i ncr eases  the  r i sk   of   such  i nf ecti ons  as
di ssem i nated  her pes  zoster .   Thi s  i m m une  dy sf uncti on  can  l ast  f or   y ear s  af ter
tr eatm ent.   Tr eatm ent  w i th  chem other apy   i s  associ ated  w i th  an  i ncr eased  r i sk
of   secondar y   l euk em i a,   w hi ch  i s  f ur ther   i ncr eased  i n  pati ents  ol der   than  40
y ear s,   heav i l y   tr eated  pati ents  (both  chem other apy   and  r adi ati on  ther apy ),   and
those  w ho  under go  pr ol onged  ther apy   w i th  al k y l ati ng  agents.

The  sequel ae  of   the  ther apy   f or   Hodgk i n's  di sease  ar e  v ar i ous.   It  i s  i m por tant
to  be  aw ar e  of   them ,   but  i t  i s  al so  i m por tant  that  ther apy   not  be  sev er el y
m odi f i ed  (i . e. ,   a  l ow er   dosage  of   ei ther   chem other apy   or   r adi ati on  ther apy )  to
m i ni m i ze  r i sk ,   because  attem pti ng  to  m i ni m i ze  the  r i sk   i n  thi s  m anner   m ay
com pr om i se  cur e.

5.   What  ar e  the  k now n  causes  or   di seases  associ ated  w i th  the  dev el opm ent  of
non–Hodgk i n's  l y m phom a?

The  r i sk   of   l y m phom a  i s  i ncr eased  i n  pati ents  w i th  cer tai n  connecti v e  ti ssue
and  i m m unol ogi c  di sor der s.   These  i ncl ude  hum an  i m m unodef i ci ency   v i r us  (HIV)
i nf ecti on,   Kl i nef el ter 's  sy ndr om e,   acqui r ed  hy pogam m agl obul i nem i a,   i atr ogeni c
i m m unosuppr essi on  (especi al l y   af ter   or gan  tr anspl antati on),   atax i aâ
€“tel angi ectasi a  sy ndr om e,   Sjögr en's  sy ndr om e,   r heum atoi d  ar thr i ti s  and
sy stem i c  l upus  er y them atosus,   Sw i ss­ty pe  agam m agl obul i nem i a,   com m on
v ar i abl e  i m m unodef i ci ency   di sease,   acqui r ed  i m m unodef i ci ency   sy ndr om e,   and
the  X­l i nk ed  l y m phopr ol i f er ati v e  sy ndr om e.

A  v i r al   eti ol ogy   of   l y m phom a  has  been  pr oposed,   but  no  cl ear   pr oof   of   thi s
v i r us  ex i sts  ex cept  f or   hum an  T­cel l   l euk em i a  v i r us  ty pe  1  (HTLV­1)  i nf ecti on.
Cer tai n  ty pes  of   m or e  com m on  l y m phom as  hav e  been  associ ated  w i th  a  v i r al
eti ol ogy   (e. g. ,   Bur k i tt's  l y m phom a  and  Epstei n­Bar r   v i r us  and  HIV  or   post
or gan  tr anspl antati on  l y m phom as).   The  sear ch  to  establ i sh  a  v i r al   cause  has
i m pl i cated  oncogenes,   l eadi ng  to  the  i denti f i cati on  of   v ar i ous  cy togeneti c
abnor m al i ti es  i n  l y m phom a.   The  com m on  patter n  i s  f or   a  k now n  oncogene  to  be
tr ansl ocated  i nto  an  i m m unogl obul i n  gene  l ocus.   The  com m on  tr ansl ocati ons
ar e  l i sted  i n  Tabl e  7­9.

6.   How   does  the  Wor l d  Heal th  Or gani zati on's  w or k i ng  f or m ul ati on  of   nonâ
€“Hodgk i n's  l y m phom a  di f f er   f r om   the  ol der   cl assi f i cati ons?

The  non–Hodgk i n's  l y m phom as  ar e  cl assi f i ed  accor di ng  to  hi stol ogi c  ty pe.
The  new er   Wor l d  Heal th  Or gani zati on  w or k i ng  f or m ul ati on  i s  based  on  the
m or phol ogi c  f eatur es  of   each  ty pe  of   l y m phom a.   Thi s  cl assi f i cati on  di v i des
P. 316
the  l y m phom as  i nto  thr ee  m ajor   subgr oups:  l ow   gr ade,   i nter m edi ate  gr ade,
and  hi gh  gr ade.   Ther ef or e,   the  l y m phom as  ar e  cl assi f i ed  accor di ng  to  both
thei r   m or phol ogi c  f eatur es  and  thei r   behav i or .   N ew er   cl assi f i cati ons  ex am i ne
m ol ecul ar   char acter i sti cs,   as  show n  i n  Tabl e  7­10.

Table 7­9 Common Translocations in Patients
with Lymphoma

Tra ns loc a tion His tologic  Type  of Lymphoma

t(8;14)  chr om osom e Bur k i tt's  and  non­Bur k i tt's

t(2;8)  chr om osom e Bur k i tt's  and  non­Bur k i tt's

t(8;22)  chr om osom e Bur k i tt's  and  non­Bur k i tt's

t(14;18)  chr om osom e Fol l i cul ar

t(2;5)  chr om osom e Anapl asti c  l ar ge  cel l

7.   How   does  the  bi ol ogy   of   hi gh­gr ade  l y m phom a  di f f er   f r om   that  of   l ow ­gr ade
l y m phom a,   and  how   does  thi s  af f ect  tr eatm ent  and  sur v i v al ?

Hi gh­gr ade  non–Hodgk i n's  l y m phom a  i s  a  gr oup  of   di seases  that  behav e


aggr essi v el y ,   especi al l y   com par ed  w i th  the  behav i or   that  i s  ty pi cal   of   l ow ­
gr ade
P. 317
non–Hodgk i n's  l y m phom a  or   Hodgk i n's  di sease.   The  m ean  sur v i v al   i n  pati ents
w i th  hi gh­gr ade  di sease  w ho  do  not  r espond  to  ther apy   i s  2  y ear s,   w her eas
pati ents  w i th  l ow ­gr ade  di sease  can  l i v e  f or   up  to  20  y ear s.

Table 7­10 WHO Classi.cation of the Nonâ
€“Hodgkin's Lymphomas

The  indole nt lymphoma s


B­cel l   neopl asm s
Sm al l   l y m phocy ti c  l y m phom a/B­cel l   chr oni c  l y m phocy ti c
l euk em i a
Ly m phopl asm acy ti c  l y m phom a  (Wal denstr om 's
m acr ogl obul i nem i a)
Pl asm a  cel l   l euk em i a
Hai r y   cel l   l euk em i a
Fol l i cul ar   l y m phom a  (gr ade  1  and  2)
Mar gi nal   cel l   l y m phom a a
T­cel l   neopl asm s
T­cel l   l ar ge  gr anul ar   l y m phocy te  l euk em i a
My cosi s  f ungoi des
T­cel l   pr ol y m phocy ti c  l euk em i a
N atur al   k i l l er   cel l   neopl asm s
N atur al   k i l l er   cel l   l ar ge  gr anul ar   l y m phocy te  l euk em i a
The  a ggre s s ive  lymphoma s
B­cel l   neopl asm s
Fol l i cul ar   l y m phom a  (gr ade  3)
Di f f use  l ar ge  B­cel l   l y m phom a
Mantl e  cel l   l y m phom a a
T­cel l   neopl asm
Per i pher al   T­cel l   l y m phom a
Anapl asti c  l ar ge  cel l   l y m phom a,   T/nul l   cel l
The  highly a ggre s s ive  lymphoma s
B­cel l   neopl asm s
Bur k i tt's  l y m phom a
Pr ecur sor   B­l y m phobl asti c  l euk em i a/l y m phom a
T­cel l   neopl asm s
Adul t  T­cel l   l y m phom a/l euk em i a
Pr ecur sor   T­l y m phobl asti c  l euk em i a/l y m phom a

a Mar gi nal   or   m antl e  cel l   l y m phom a  can  behav e  cl i ni cal l y   as  ei ther

i ndol ent  or   an  aggr essi v e  di sor der .

WHO,   Wor l d  Heal th  Or gani zati on.

Adapted  f r om   Har r i s  N L,   Jaf f e  ES,   Di ebol d  J,   et  al .   Wor l d  Heal th


Or gani zati on  cl assi f i cati on  of   neopl asti c  di seases  of   the  hem atopoi eti c
and  l y m phoi d  ti ssues:  r epor t  of   the  Cl i ni cal   Adv i sor y   Com m i ttee.   Ai r l i e
House,   Vi r gi ni a:  1997;  J  Cl i n  Oncol   1999;17:3835.

Pati ents  w i th  hi gh­gr ade  l y m phom a  can  pr esent  w i th  l ocal i zed  di sease  (< 20%),
but  m or e  com m onl y   ar e  i n  an  adv anced  stage.   Ther e  can  be  i nv ol v em ent  of
ei ther   ex tr anodal   (35%)  or   pr i v i l eged  si tes  (the  CN S  or   testes).   Most  di sease
i s  of   B­cel l   or i gi n  (85%),   w i th  the  r em ai nder   of   T­cel l   or i gi n.

Poor   pr ognosti c  f actor s  i ncl ude  poor   per f or m ance  status,   bul k y   di sease  (> 10
cm ),   hi gh  LDH  l ev el   (> 500  IU /dL),   bone  m ar r ow   i nv ol v em ent,   and  B  sy m ptom s.

Hi gh­gr ade  non–Hodgk i n's  l y m phom as  ar e  v er y   sensi ti v e  to  chem other apy ,
and  aggr essi v e  tr eatm ent  i s  the  onl y   chance  f or   cur e.   U p  to  60%  of   pati ents
can  be  cur ed  w i th  the  new er   chem other apeuti c  r egi m ens.   Thi s  i s  i n  shar p
contr ast  to  the  ex per i ence  w i th  l ow ­gr ade  l y m phom as,   i n  w hi ch  cur e  r ates  of
l ess  than  20%  ar e  seen  and  sur v i v al   does  not  seem   to  be  af f ected  by   the  ty pe
of   r esponse  to  chem other apy .

8.   What  ty pe  of   l y m phom a  ty pi cal l y   i nv ol v es  the  sk i n,   and  how   does  thi s  i nf l uence
stagi ng  and  tr eatm ent?

Ly m phom atous  i nv ol v em ent  of   the  sk i n  i s  com m onl y   seen  i n  the  setti ng  of   T­
cel l   l y m phom a.   It  occur s  i n  appr ox i m atel y   10%  of   al l   cases  of   non–Hodgk i n's
l y m phom a,   and  thi s  gr oup  of   di seases  i s  cal l ed  cutaneous  T­cel l   l y m phom a
(CTCL).   The  l ow ­gr ade  f or m   of   CTCL  i s  m y cosi s  f ungoi des  or   Sezar y   sy ndr om e.
Sezar y   sy ndr om e  i s  di agnosed  i n  the  setti ng  of   m y cosi s  f ungoi des  w hen  the
m al i gnant  cel l s  (Sezar y   cel l s)  ar e  f ound  i n  the  per i pher al   bl ood.

The  stagi ng  cl assi f i cati on  f or   CTCL  di f f er s  f r om   that  f or   other   f or m s  of   nonâ
€“Hodgk i n's  l y m phom a,   and  i s  based  on  the  TN M  sy stem ,   as  show n  i n  Tabl e  7­
11.

The  m ai nstay   of   m anagem ent  of   ear l y   m y cosi s  f ungoi des  and  Sezar y   sy ndr om e
has  been  topi cal   tr eatm ent,   but  ther e  i s  l i ttl e  ev i dence  that  thi s  pr ol ongs
sur v i v al .

9.   What  popul ati on  of   pati ents  i s  pr one  to  acqui r i ng  secondar y   CN S  l y m phom a,
and  can  thi s  be  pr ev ented?

CN S  i nv ol v em ent  i s  r ar el y   seen  i n  pati ents  w i th  l ow ­gr ade  l y m phom a.   When


seen,   a  hi stol ogi c  tr ansf or m ati on  to  hi gh­gr ade  l y m phom a  shoul d  be  suspected.
Wi thi n  thi s  gr oup,   CN S  i nv ol v em ent  i s  m or e  com m on  w hen  Wal dey er 's  tonsi l l ar
r i ng,   the  bone  m ar r ow ,   or   the  testes  ar e  af f ected.   Am ong  the  hi gh­gr ade
l y m phom as,   ther e  i s  a  gr oup  of   especi al l y   aggr essi v e  l y m phom as,   and  these
consi st  of   undi f f er enti ated  l y m phom as  (Bur k i tt's  and  non–Bur k i tt's  ty pe),
l y m phobl asti c  l y m phom a,   and  acute  T­cel l   l y m phom a.   The  i nci dence  of   CN S
i nv ol v em ent  i s  hi gh  i n  these  pati ents  and,   ther ef or e  the  CN S  shoul d  be  tr eated
pr ophy l acti cal l y   w i th  i ntr athecal   chem other apy .   Pati ents  on  i m m unosuppr essi on
af ter   or gan  tr anspl antati on  [posttr anspl ant  l y m phopr ol i f er ati v e  di sease  (PTLD)]
of ten  pr esent  w i th  CN S  l y m phom a.

P. 318

Table 7­11 National Cutaneous T­Cell Lymphoma
Workshop Staging Classification a

Tumor Sk in Node Lymph Me ta s ta s is Vis c e ra l


(T) (N) Node s (M) Orga ns

T1 Li m i ted N0 No M0 No
pl aques adenopathy , i nv ol v em ent
(< 10%  body hi stol ogy
sur f ace  ar ea) negati v e

T2 Gener al i zed N1 Adenopathy ; M1


pl aques hi stol ogy  
negati v e
T3 Cutaneous N2 No
tum or s adenopathy ;
   
hi stol ogy
posi ti v e

T4 Gener al i zed N3 Adenopathy ;


er y thr oder m a hi stol ogy    
posi ti v e

Stage  I:  Li m i ted  (IA)  or   gener al i zed  (IB)  pl aques  w i thout  adenopathy   or
hi stol ogi c  i nv ol v em ent  of   l y m ph  nodes  or   v i scer a  (T1  N 0  M0  or   T2  N 0  M0)

Stage  II:  Li m i ted  or   gener al i zed  pl aques  w i th  adenopathy   (IIA)  or   cutaneous
tum or s  w i th  or   w i thout  adenopathy   (IIB);  w i thout  hi stol ogi c  i nv ol v em ent  of
l y m ph  nodes  or   v i scer a  (T1–2  N 1  M0  or   T2  N 0–1  M0)

Stage  III:  Gener al i zed  er y thr oder m a  w i th  or   w i thout  adenopathy ;  w i thout
hi stol ogi c  i nv ol v em ent  of   l y m ph  nodes  or   v i scer a  (T4  N 0–2  M0)

Stage  IV:  Hi stol ogi c  i nv ol v em ent  of   l y m ph  nodes  (IVA)  or   v i scer a  (IVB)  w i th
any   sk i n  l esi on  and  w i th  or   w i thout  adenopathy   (T1–4  N 2–3  M0  f or   IVA;
T1–4  N 0–3  M1  f or   IVB)

a Bl ood  i nv ol v em ent  shoul d  be  r ecor ded  as  absent  (B0)  or   pr esent  (B1)  but  i s

not  cur r entl y   used  to  deter m i ne  f i nal   stage.

10.   What  com pl i cati on  of   ther apy   can  occur   i n  the  setti ng  of   r api dl y   gr ow i ng
tum or s,   and  how   can  thi s  be  pr ev ented?

The  tum or   l y si s  sy ndr om e  can  occur   i n  the  setti ng  of   tum or s  that  ar e
ex qui si tel y   sensi ti v e  to  chem other apy   and  i s  seen  w hen  ther e  i s  a  l ar ge  tum or
bur den.   The  sy ndr om e  i s  char acter i zed  by   hy per ur i caci dem i a,
hy per phosphatem i a,   hy per k al em i a,   and  hy pocal cem i a,   and  can  r esul t  i n  acute
r enal   f ai l ur e  and  sudden  death,   i f   not  tr eated.   For tunatel y ,   i f   the  si gns  ar e
car ef ul l y   w atched  f or ,   the  pati ent  can  be  spar ed  i ts  ef f ects.   The  m anagem ent
of   thi s  sy ndr om e  i ncl udes  aggr essi v e  hy dr ati on,   the  al k al i ni zati on  of   ur i ne,   and
al l opur i nol   ther apy   bef or e  and  dur i ng  chem other apy .

Case
A  42­y ear ­ol d  w om an  i s  r ef er r ed  to  y ou  by   her   f am i l y   phy si ci an  f or   the  ev al uati on
of   bi l ater al   neck   adenopathy .   She  has  noti ced  thi s  sw el l i ng  i nter m i ttentl y   f or
appr ox i m atel y   6  m onths.   She  has  occasi onal l y   noti ced  ax i l l ar y   node  sw el l i ng  but
deni es  any   other   adenopathy .   She  has  noti ced  that  she  ti r es  m or e  easi l y   and  seem s
to  “pi ck   up  ev er y   l i ttl e  v i r us. â€​
  She  adm i ts  to  ex per i enci ng  occasi onal   ear l y
sati ety ,   but  deni es  any   i ncr ease  i n  abdom i nal   gi r th  or   changes  i n  bow el   habi ts.   She
deni es  any   f ev er ,   chi l l s,   ni ght  sw eats,   w ei ght  l oss,   or   change  i n  appeti te.
Her   f am i l y   hi stor y   i s  r em ar k abl e  f or   a  m other   w i th  br east  cancer   (the  pati ent's  l ast
m am m ogr am   1  y ear   ago  w as  nor m al ).   She  does  not  sm ok e  or   dr i nk .
P. 319
Phy si cal   ex am i nati on  f i ndi ngs  ar e  r em ar k abl e  f or   bi l ater al   neck   and  ax i l l ar y
adenopathy .   She  has  no  or al   or   phar y ngeal   l esi ons  and  no  br east  m asses.   Her
spl een  i s  m i l dl y   enl ar ged  but  her   l i v er   si ze  i s  nor m al .   She  has  no  other   phy si cal
abnor m al i ti es.
Labor ator y   f i ndi ngs  ar e  r em ar k abl e  f or   a  m i l d  nor m ochr om i c,   nor m ocy ti c  anem i a
(hem ogl obi n,   13. 0  g/dL;  hem atocr i t,   39%);  the  pl atel et  count  i s  250, 000/m m 3   and
the  w hi te  bl ood  cel l   count  i s  5, 200/m m 3   w i th  a  nor m al   di f f er enti al .   A  chem i str y
panel   i s  r em ar k abl e  f or   a  sl i ghtl y   el ev ated  LDH  l ev el ,   but  the  AST,   ALT,   bi l i r ubi n,
and  al k al i ne  phosphatase  v al ues  ar e  nor m al .   Her   chest  r adi ogr aphi c  study   i s
nor m al .
A  stagi ng  ev al uati on  i s  done  and  r ev eal s  the  f ol l ow i ng  f i ndi ngs.   Ti ssue  anal y si s
r ev eal s  m al i gnant  l y m phom a  consi sti ng  of   f ol l i cul ar   sm al l   cl eav ed  (nodul ar   poor l y
di f f er enti ated)  cel l s  that  ar e  CD20  posi ti v e.   Bone  m ar r ow   bi opsy   r ev eal s  nor m al
cel l ul ar i ty   w i th  l y m phoi d  f ol l i cl es  (nor m al   f or   age),   a  sl i ght  i ncr ease  i n  the  num ber
of   er y thr oi d  pr ecur sor s,   nor m al   m egak ar y ocy tes,   and  a  decr ease  i n  the  i r on
content.   Cy togeneti c  ex am i nati on  i denti f i es  a  bal anced  tr ansl ocati on,   t(14;18).   CT
scan  of   the  abdom en  depi cts  m oder ate  spl enom egal y   and  m i l d  r etr oper i toneal
adenopathy .   Ser um   i m m unoel ectr ophor esi s  r ev eal s  m i l d  hy pogam m agl obul i nem i a
w i th  a  m onocl onal   i m m unogl obul i n  M  (IgM)  spi k e.

1.   On  the  basi s  of   the  phy si cal   ex am i nati on  and  l abor ator y   f i ndi ngs,   w hat  i s  the
di f f er enti al   di agnosi s  i n  thi s  pati ent?
2.   On  the  basi s  of   the  f i ndi ngs  f r om   the  stagi ng  ev al uati on,   w hat  stage  of   nonâ
€“Hodgk i n's  l y m phom a  i s  thi s  pati ent  i n,   and  w hat  ar e  her   tr eatm ent  opti ons
and  pr ognosi s?
3.   What  ar e  the  i m pl i cati ons  of   her   cy togeneti c  abnor m al i ti es?
4.   Is  i t  f ur ther   necessar y   to  ev al uate  or   tr eat  her   hy pogam m agl obul i nem i a?
5.   What  i s  the  si gni f i cance  of   the  m onocl onal   IgM  spi k e  i n  thi s  pati ent?
The  pati ent  i s  obser v ed  to  do  f i ne  at  her   6­m onth  v i si ts,   unti l   4  y ear s  l ater ,
w hen  pai nf ul   and  enl ar gi ng  nodes  dev el op.
6.   What  f or m   of   ther apy   w oul d  y ou  of f er   her   w hen  the  pai nf ul   and  enl ar gi ng
nodes  ar e  detected,   and  w hat  outcom e  can  she  ex pect?
Wi th  the  onset  of   ther apy   consi sti ng  of   or al   al k y l ati ng  agents,   a  sev er e  anem i a
dev el ops  i n  thi s  pati ent,   r equi r i ng  tr ansf usi on.
7.   How   w oul d  y ou  ev al uate  the  anem i a  that  dev el ops  w i th  the  al k y l ati ng  agent
ther apy ?
The  r esul ts  of   i nv esti gati ons  per f or m ed  to  deter m i ne  the  sour ce  of   her   anem i a
ar e  as  f ol l ow s:  Coom bs'  di r ect  and  i ndi r ect  test,   posi ti v e;  r eti cul ocy te  count,
9%;  bl ood  sm ear ,   spher ocy tes  and  i ncr eased  r eti cul ocy tes;  and  bone  m ar r ow
bi opsy ,   i ncr eased  cel l ul ar i ty   w i th  er y thr oi d  hy per pl asi a  pl us  the  pr esence  of
sm al l   l y m phocy tes,   suggesti ng  l y m phom atous  i nv ol v em ent.
8.   On  the  basi s  of   the  f i ndi ngs  y i el ded  by   the  i nv esti gati ons  f or   her   anem i a,   w hat
ar e  the  tr eatm ent  opti ons  at  thi s  poi nt?
Thi s  pati ent  does  w el l   f or   6  m onths  w i th  m onthl y   i ntr av enous  chem other apy
and  i m m unogl obul i n  ther apy ,   as  needed.   Tw o  y ear s  af ter   the  star t  of   ther apy ,
i ncr eased  spl enom egal y   dev el ops  that  appear s  r ef r actor y   to  the  pr ev i ous
chem other apy .
9.   Wi th  the  appear ance  of   i ncr eased  spl enom egal y ,   w hat  i s  the  di f f er enti al
di agnosi s,   and  how   shoul d  y ou  conf i r m   i t?
10.   What  ar e  the  tr eatm ent  opti ons  i n  thi s  pati ent  w hose  di sease  i s  now   i n  an
adv anced  stage?

P. 320

Case Discussion
1.   On  the  basi s  of   the  phy si cal   ex am i nati on  and  l abor ator y   f i ndi ngs,   w hat  i s  the
di f f er enti al   di agnosi s  i n  thi s  pati ent?

The  neck   adenopathy   i n  thi s  pati ent  can  r epr esent  a  nor m al   f i ndi ng;  50%  of
pati ents  can  hav e  l y m ph  nodes  that  ar e  l ess  than  0. 5  cm   i n  di am eter .   It  can
al so  si gni f y   acute  i nf ecti on  stem m i ng  f r om   acute  v i r al   i nf ecti ons,
m ononucl eosi s,   tox opl asm osi s,   or   pul m onar y   i nf ecti ons,   but  i n  thi s  setti ng  the
nodes  ar e  usual l y   f i r m   and  tender   and  r ecede  w i thi n  2  to  4  w eek s.   Sol i d
tum or s  ar e  al so  a  consi der ati on  i n  the  di f f er enti al   di agnosi s,   and  i ncl ude  head
and  neck   cancer ,   as  w el l   as  thy m i c,   l ung,   and  br east  cancer ;  l ung  and  br east
cancer s  ar e  m or e  com m onl y   associ ated  w i th  supr acl av i cul ar   and  ax i l l ar y
adenopathy .   A  f our th  possi bi l i ty   i s  Hodgk i n's  di sease  or   non–Hodgk i n's
l y m phom a.   Pati ents  w i th  l y m phom a  can  hav e  l y m ph  nodes  that  “com e  and
go. â€​

2.   On  the  basi s  of   the  f i ndi ngs  f r om   the  stagi ng  ev al uati on,   w hat  stage  of   nonâ
€“Hodgk i n's  l y m phom a  i s  thi s  pati ent  i n,   and  w hat  ar e  her   tr eatm ent  opti ons
and  pr ognosi s?

N on­Hodgk i n's  l y m phom a  i s  usual l y   staged  accor di ng  to  the  Ann  Ar bor
cl assi f i cati on  used  f or   Hodgk i n's  di sease.   Accor di ng  to  thi s  sy stem 's  cr i ter i a,
thi s  pati ent  has  stage  III  di sease.   In  the  setti ng  of   l ow ­gr ade  l y m phom a,   i t  i s
i m por tant  to  i denti f y   l ocal i zed  v er sus  di ssem i nated  di sease.   How ev er ,
pr ognosti c  f actor s  ar e  of ten  m ost  i m por tant.   Thi s  pati ent  has  di sease  abov e
and  bel ow   the  di aphr agm   as  w el l   as  pr obabl e  spl eni c  i nv ol v em ent.   She  does
not  cl ear l y   hav e  bone  m ar r ow   i nv ol v em ent,   because  l y m phoi d  f ol l i cl es  can  be  a
beni gn  f i ndi ng.

In  l i ght  of   these  f i ndi ngs,   her   pr ognosi s  i s  f ai r l y   good.   The  m edi an  sur v i v al   f or
tr eated  f ol l i cul ar   sm al l   cl eav ed  l y m phom a  (f ol l i cul ar   gr ade  1)  can  be  up  to  15
y ear s.   The  i ni ti al   tr eatm ent  f or   adv anced  l ow ­gr ade  l y m phom a  i s  v er y
contr ov er si al .   It  r esponds  to  both  si ngl e­  and  m ul ti pl e­agent  chem other apy   as
w el l   as  r adi ati on  ther apy .   How ev er ,   r egi m ens  that  i ncl ude  anti ­CD20  anti body
do  seem   to  i ncr ease  sur v i v al .   Attem pts  to  er adi cate  di sease  w i th  hi gh­dose
chem other apy   (w i th  or   w i thout  bone  m ar r ow   r escue)  hav e  not  been  show n  to
pr ol ong  ov er al l   sur v i v al .   It  w oul d  sti l l   not  be  unr easonabl e  to  w ai t  unti l   thi s
pati ent  becom es  sy m ptom ati c  bef or e  star ti ng  tr eatm ent.

3.   What  ar e  the  i m pl i cati ons  of   her   cy togeneti c  abnor m al i ti es?

The  t(14;18)  abnor m al i ty   i s  a  com m on  f i ndi ng  i n  the  setti ng  of   f ol l i cul ar   sm al l
cl eav ed  l y m phom a.   In  thi s  abnor m al i ty ,   the  bc12  oncogene  on  chr om osom e  18
has  been  tr ansl ocated  to  the  i m m unogl obul i n  heav y   chai n  l ocus  on  chr om osom e
14.   It  i s  now   w i del y   accepted  that  thi s  abnor m al i ty   i s  f ound  i n  v i r tual l y   al l
pati ents  w i th  thi s  hi stol ogi c  patter n.   It  i s  al so  f ound  i n  appr ox i m atel y   30%  of
pati ents  w i th  di f f use  l y m phom a,   and,   i n  these  cases,   i t  pr obabl y   r epr esents  a
hi stol ogi c  tr ansf or m ati on  f r om   f ol l i cul ar   sm al l   cl eav ed  l y m phom a,   and  can
ther ef or e  be  consi der ed  a  poor   pr ognosti c  i ndi cator .   It  i s  not  a  pr ognosti c
f actor   i n  the  setti ng  of   f ol l i cul ar   sm al l   cl eav ed  l y m phom a.   The  pr esence  of
CD20  sur f ace  anti gen  docum ents  that  thi s  i s  a  B­cel l   l y m phom a  and  pati ents
w i l l   pr obabl y   r espond  to  anti ­CD20  anti body .

4.   Is  i t  necessar y   to  f ur ther   ev al uate  or   tr eat  her   hy pogam m agl obul i nem i a?

Hy pogam m agl obul i nem i a  i s  occasi onal l y   f ound  i n  associ ati on  w i th  l ow ­gr ade
l y m phom a  and  m or e  of ten  chr oni c  l y m phocy ti c  l euk em i a.   It  i s  not  of ten  a
pr obl em ,
P. 321
as  i t  i s  i n  m ul ti pl e  m y el om a,   but  thi s  i m m unol ogi c  def ect  shoul d  be  consi der ed
w hen  i nf ecti ons  occur   i n  these  pati ents  or   w hen  neutr openi a  i s  pr eci pi tated  by
tr eatm ent  or   bone  m ar r ow   i nv ol v em ent.   Pati ents  w i th  l i f e­thr eateni ng
i nf ecti ons  m ay   benef i t  f r om   gam m a  gl obul i n  ther apy .

5.   What  i s  the  si gni f i cance  of   the  m onocl onal   IgM  spi k e  i n  thi s  pati ent?

Al though  the  si ze  of   the  spi k e  i s  not  quanti f i ed,   i t  i s  sti l l   i m por tant  i nf or m ati on
because  of   the  possi bi l i ty   of   hy per v i scosi ty   associ ated  w i th  IgM,   unl i k e  IgG  f or
w hi ch  hy per v i scosi ty   i s  f ar   l ess  l i k el y .   Wal denstr om 's  m acr ogl obul i nem i a  i s
associ ated  w i th  l y m phom a,   and  i s  usual l y   seen  i n  pati ents  w i th  di f f use  sm al l
l y m phocy ti c  l y m phom a.   A  m onocl onal   gam m opathy   can  af f ect  up  to  15%  of   the
pati ents  w i th  l ow ­gr ade  l y m phom a  and  i s  m ost  com m onl y   seen  w hen  the  cel l s
hav e  pl asm acy toi d  f eatur es.

6.   What  f or m   of   ther apy   w oul d  y ou  of f er   her   w hen  the  pai nf ul   and  enl ar gi ng
nodes  ar e  detected,   and  w hat  outcom e  can  she  ex pect?

The  deci si on  of   w hen  to  tr eat  l ow ­gr ade  non–Hodgk i n's  l y m phom a  i s,   as
al r eady   m enti oned,   a  contr ov er si al   i ssue.   Most  phy si ci ans  r ecom m end  w ai ti ng
unti l   sy m ptom s  appear ,   consi sti ng  of   r api dl y   enl ar gi ng  nodes,   B  sy m ptom s,
cy topeni as  due  to  bone  m ar r ow   i nv ol v em ent,   or   an  i ncr eased  adenopathy   that
thr eatens  or gan  f uncti on.

If   the  sy m ptom ati c  di sease  i s  l ocal i zed,   r adi ati on  ther apy   that  f ocuses  on  the
si te  i nv ol v ed  i s  a  v i abl e  opti on.   For   the  m anagem ent  of   m or e  gener al i zed
di sease,   a  si ngl e  al k y l ati ng  agent  w i th  or   w i thout  ster oi ds  can  be  v er y
ef f ecti v e.   Mor e  aggr essi v e  com bi nati on  chem other apy   can  al so  be  consi der ed,
especi al l y   w i th  anti ­CD20  m onocl onal   anti body   ther apy .

In  deter m i ni ng  the  l i k el y   outcom e  of   tr eatm ent  i n  thi s  pati ent,   com pl ete
r em i ssi ons  can  be  achi ev ed  i n  the  setti ng  of   l ow ­gr ade  l y m phom a,   m ostl y   f or
l ocal i zed  (stage  I  and  II)  di sease.   “Spontaneous  r em i ssi onâ€​   can  al so  occur
(appr ox i m atel y   5%  to  10%).   The  cur e  r ate  f or   adv anced  l ow ­gr ade  l y m phom a
i s  v er y   l ow .   Ev en  w i th  the  i nsti tuti on  of   aggr essi v e  chem other apy   (see
pr ecedi ng  tex t),   l ess  than  10%  of   af f ected  pati ents  r em ai n  di sease  f r ee  af ter   5
y ear s.

7.   How   w oul d  y ou  ev al uate  the  anem i a  that  dev el ops  w i th  the  al k y l ati ng  agent
ther apy ?
Dur i ng  thi s  pati ent's  i ni ti al   ev al uati on,   she  w as  noted  to  hav e  a  m i l d  anem i a
w i th  a  sl i ght  i ncr ease  i n  er y thr oi d  pr ecur sor s.   Her   LDH  l ev el   w as  el ev ated,   but
her   l i v er   enzy m e  v al ues  w er e  nor m al .   In  thi s  case,   the  r ai sed  LDH  l ev el   coul d
r epr esent  ei ther   a  hi gh  tur nov er   of   tum or   cel l s  or   the  destr ucti on  of   r ed  bl ood
cel l s  (hem ol y si s),   or   both.   The  sl i ght  el ev ati on  i n  the  num ber   of   r ed  bl ood  cel l s
coul d  al so  be  due  to  per i pher al   destr ucti on.   Thi s  pati ent  shoul d  under go  a
com pl ete  assessm ent  of   her   anem i a,   i ncl udi ng  ev al uati on  f or   hem ol y si s;  i r on,
cobal am i n,   and  f ol ate  def i ci ency ;  and  bone  m ar r ow   i nv ol v em ent  by   l y m phom a
(an  unl i k el y   cause  i n  thi s  setti ng).

8.   On  the  basi s  of   the  f i ndi ngs  y i el ded  by   the  i nv esti gati ons  f or   her   anem i a,   w hat
ar e  the  tr eatm ent  opti ons  at  thi s  poi nt?

Thi s  pati ent  has  hem ol y ti c  anem i a.   It  i s  m or e  com m onl y   seen  w i th  chr oni c
l y m phocy ti c  l euk em i a  (the  l euk em i a  phase  of   f ol l i cul ar   sm al l   cl eav ed
l y m phom a).   Of ten  pati ents  hav e  an  under l y i ng  com pensated  hem ol y si s,   as  thi s
pati ent  di d,   w i th  anem i a,   i ncr eased  er y thr oi ds,   and  an  i ncr eased  LDH  l ev el .
Begi nni ng  the  tr eatm ent
P. 322
w i th  chem other apy   can  unm ask   the  hem ol y si s  because  the  bone  m ar r ow
r esponse  i s  r etar ded  by   m ar r ow   suppr essi v e  agents.

The  tr eatm ent  f or   hem ol y ti c  anem i a  i n  thi s  setti ng  has  tw o  goal s  (a)  to
ter m i nate  the  hem ol y ti c  pr ocess,   and  (b)  to  tr eat  the  under l y i ng  di sease.
Tr eatm ent  w i th  ster oi ds  m ay   addr ess  both  pr obl em s.   If   the  hem ol y si s  i s  not
stopped  w i th  hi gh­dose  ster oi ds,   i m m unogl obul i n  ther apy   shoul d  be  star ted.
Once  the  hem ol y si s  i s  contr ol l ed,   a  m or e  aggr essi v e  chem other apeuti c  r egi m en
m ay   be  i m pl em ented.

9.   Wi th  the  appear ance  of   i ncr eased  spl enom egal y ,   w hat  i s  the  di f f er enti al
di agnosi s,   and  how   shoul d  y ou  conf i r m   i t?

The  i ncr eased  spl enom egal y   that  does  not  r espond  to  the  pr ev i ousl y   used
chem other apy   m ay   r epr esent  a  tr ansf or m ati on  to  a  m or e  aggr essi v e  hi stol ogi c
ty pe  of   l y m phom a.   Thi s  occur s  at  a  r ate  of   appr ox i m atel y   5%  per   y ear   i n
pati ents  w i th  f ol l i cul ar   sm al l   cl eav ed  l y m phom a;  usual l y ,   the  tr ansf or m ati on  i s
to  a  di f f use  l ar ge  cel l   l y m phom a.   Hi stol ogi c  tr ansf or m ati on  r epr esents  a
change  i n  the  natur al   hi stor y   of   the  di sease  and  si gni f i es  a  m uch  shor tened
sur v i v al .

Hi stol ogi c  tr ansf or m ati on  shoul d  be  docum ented  by   ti ssue  ex am i nati on.   Agai n,
a  l y m ph  node  or   m ass  i s  the  best  sour ce  of   ti ssue  f or   thi s  pur pose.

10.   What  ar e  the  tr eatm ent  opti ons  i n  thi s  pati ent  w hose  di sease  i s  now   i n  an
adv anced  stage?

When  hi stol ogi c  tr ansf or m ati on  occur s,   the  pr ognosi s  i s  v er y   poor   and  pati ents
r espond  poor l y   to  ev en  the  m ost  aggr essi v e  chem other apeuti c  r egi m ens
because  of   chem or esi stant  di sease.   In  ol der   pati ents  w ho  hav e  concur r ent
di sease,   i t  m ay   be  r easonabl e  to  use  onl y   pal l i ati v e  m easur es  (pai n
m anagem ent)  and  per haps  adm i ni ster   l ocal   r adi ati on  ther apy ,   i f   needed.

Suggested Readings
Bennett  CL,   Ar m i tage  JL,   Ar m i tage  GO,   et  al .   Costs  of   car e  and  outcom es  f or
hi gh­dose  ther apy   and  autol ogous  tr anspl antati on  f or   l y m phoi d  m al i gnanci es:
r esul ts  f r om   the  U ni v er si ty   of   N ebr ask a  1987  thr ough  1991.   J  Cl i n  Oncol
1995;13:969.

Canel l os  G.   Is  ther e  an  ef f ecti v e  sal v age  ther apy   f or   adv anced  Hodgk i n's
di sease?  Ann  Oncol   1991;2:1.

DeVi ta  VT  Jr ,   Hubbar d  SM,   Longo  DL.   Tr eatm ent  of   Hodgk i n's  di sease.   J  N atl
Cancer   Inst  1990;10:19.

Hancock   SL,   Hoppe  RT.   Long­ter m   com pl i cati ons  of   tr eatm ent  and  causes  of
m or tal i ty   af ter   Hodgk i n's  di sease.   Sem i n  Radi at  Oncol   1996;6:225.

Koh  HK,   Foss  FM.   Cutaneous  T­cel l   l y m phom a.   Hem atol   Oncol   Cl i n  N or th  Am
1995;9:943.

N ati onal   Cancer   Insti tute.   Sum m ar y   and  descr i pti on  of   a  w or k i ng  f or m ul ati on  f or
cl i ni cal   usage:  the  N on­Hodgk i n's  Ly m phom a  Pathol ogi c  Cl assi f i cati on  Pr oject.
Cancer   1982;49:2112.

Wal dm ann  TA,   Dav i s  MM,   Bongi ov anni   KF,   et  al .   Rear r angem ents  of   genes  f or
the  anti gen  r eceptor   on  T  cel l s  as  m ar k er s  of   l i neage  and  cl onal i ty   i n  hum an
l y m phoi d  neopl asm s.   N   Engl   J  Med  1985;313:776.

Young  RC,   Longo  DL,   Gl atstei n  E,   et  al .   The  tr eatm ent  of   i ndol ent  l y m phom as:
w atchf ul   w ai ti ng  v   aggr essi v e  com bi ned  m odal i ty   tr eatm ent.   Sem i n  Hem atol
1988;25:11.

P. 323

Lung Cancer
1.   What  ar e  the  appr ox i m ate  i nci dence,   death  r ate,   and  r i sk   f actor s  f or   l ung
cancer ?

2.   What  ar e  the  tw o  pathol ogi c  categor i es  of   l ung  cancer ,   and  thei r   hi stol ogi c
f eatur es?

3.   What  ar e  som e  cl i ni cal   f eatur es  that  m ay   suggest  a  cor r el ati on  w i th  a  cer tai n
pathol ogi c  subty pe?

4.   What  i s  essenti al l y   the  m ai nstay   of   cur ati v e  ther apy   i n  non–sm al l   cel l   l ung
cancer   (N SCLC)?

5.   How   does  the  stagi ng  and  tr eatm ent  of   sm al l   cel l   l ung  cancer   (SCLC)  di f f er
f r om   that  of   N SCLC?

6.   What  ar e  the  tw o  m ajor   deter m i nants  of   pr ognosi s  f or   both  N SCLC  and  SCLC?
Discussion
1.   What  ar e  the  appr ox i m ate  i nci dence,   death  r ate,   and  r i sk   f actor s  f or   l ung
cancer ?

Each  y ear ,   l ung  cancer   k i l l s  m or e  m en  and  w om en  i n  the  U ni ted  States  than
any   other   cancer ,   and  ther e  ar e  appr ox i m atel y   150, 000  new   cases  of   l ung
cancer   di agnosed  each  y ear .   At  the  ti m e  of   di agnosi s,   onl y   35%  of   pati ents
hav e  l ocal   di sease;  ther ef or e,   the  di sease  has  spr ead  to  r egi onal   nodes  or
di stant  si tes  i n  65%.   How ev er ,   ev en  i n  pati ents  w i th  nonm etastati c  (l ocal )
di sease,   com pl ete  cur e  i s  the  ex cepti on;  ther ef or e,   the  y ear l y   m or tal i ty   r ate
appr oaches  the  annual   i nci dence,   and  thi s  w as  esti m ated  to  be  172, 000  i n
2005.

Appr ox i m atel y   90%  of   al l   pati ents  di agnosed  w i th  l ung  cancer   hav e  a  hi stor y   of
sm ok i ng,   and  the  causal   r el ati onshi p  betw een  tobacco  use  and  l ung  cancer
m ak es  i t  a  m ajor   publ i c  heal th  pr obl em   and  one  of   the  m ost  potenti al l y
pr ev entabl e  di seases.   Other   i m por tant  r i sk   f actor s  account  f or   l ess  than  10%
of   cases  of   l ung  cancer   di agnosed,   and  these  i ncl ude  ur ani um   and  r adon
ex posur e  and  passi v e  sm ok i ng  (another   r eason  f or   sm ok i ng  cessati on
pr ogr am s).   The  r i sk   of   acqui r i ng  l ung  cancer   f al l s  si gni f i cantl y   i n  the  f i r st  5
y ear s  af ter   the  cessati on  of   sm ok i ng,   and,   ev en  af ter   20  y ear s,   the  r i sk   i s
hi gher   than  i n  peopl e  w ho  hav e  nev er   sm ok ed.

To  i l l ustr ate  the  ser i ousness  of   the  publ i c  heal th  pr obl em ,   the  l ung  cancer
i nci dence  betw een  1940  and  2000  has  r i sen  by   60%  i n  w om en,   f r om   7  to  45  per
100, 000,   and  thi s  i s  m ai nl y   due  to  the  i ncr eased  use  of   tobacco  i n  the  f em al e
popul ati on.   In  addi ti on,   a  hi gher ­than­ex pected  i nci dence  of   l ung  cancer   has
been  seen  i n  w om en  i n  the  l esser   pack ­y ear   categor i es,   suggesti ng  that  w om en
ar e  acqui r i ng  l ung  cancer   at  a  y ounger   age  and  af ter   sm ok i ng  f ew er   y ear s  than
m en.

Fur ther m or e,   al though  ov er al l   tobacco  use  i s  decr easi ng  i n  the  U ni ted  States,
sm ok i ng  m ay   be  i ncr easi ng  am ong  cer tai n  gr oups  of   m i nor i ti es  and
adol escents,   and  r ecent  tobacco  com pany   adv er ti si ng  cam pai gns  hav e  been
P. 324
di r ected  tow ar d  these  gr oups.   The  y ear s  of   potenti al   l ost  l i f e  r esul ti ng  f r om
l ung  cancer   m or tal i ty   i n  these  gr oups  i s  pr obabl y   tw i ce  that  seen  f or   the
r em ai ni ng  popul ati on.

2.   What  ar e  the  tw o  pathol ogi c  categor i es  of   l ung  cancer ,   and  thei r   hi stol ogi c
f eatur es?

For   both  tr eatm ent  and  pr ognosti c  pur poses,   m ost  l ung  cancer s  ar e  di v i ded  i nto
tw o  cl i ni cal l y   usef ul   categor i es:  SCLC,   w hi ch  accounts  f or   15%  of   al l   l ung
cancer s,   and  N SCLC,   w hi ch  consi sts  of   squam ous  cel l   car ci nom as  (40%  of   the
l ung  cancer s),   adenocar ci nom as  (40%  of   the  l ung  cancer s),   and  l ar ge  cel l
car ci nom as  and  other s  (5%  of   the  l ung  cancer s).

3.   What  ar e  som e  cl i ni cal   f eatur es  that  m ay   suggest  a  cor r el ati on  w i th  a  cer tai n
pathol ogi c  subty pe?

Pati ents  m ay   pr esent  w i th  a  v ar i ety   of   sy m ptom s,   i ncl udi ng  cough,   hem opty si s,
shor tness  of   br eath,   chest  pai n,   or   unex pl ai ned  w ei ght  l oss  (Tabl e  7­12).
Abnor m al i ti es  r ev eal ed  by   the  phy si cal   ex am i nati on  m ay   suggest  the  di agnosi s
and  i ncl ude  si gns  of   l ung  consol i dati on  r esul ti ng  f r om   an  obstr ucted  br onchus,
supr acl av i cul ar   adenopathy   stem m i ng  f r om   the  l ocal   and  r egi onal   spr ead  of   the
cancer ,   or   Hor ner   sy ndr om e,   w hi ch  i s  due  to  tum or   i m pi ngem ent  on  the
sy m patheti c  ner v e  f i ber s  that  cour se  near   the  apex   of   the  l ung.

Labor ator y   ex am i nati on  m ay   r ev eal   the  pr esence  of   hy ponatr em i a  due  to  the
sy ndr om e  of   i nappr opr i ate  anti di ur eti c  hor m one  secr eti on  (SIADH);  thi s  i s  a
par aneopl asti c  sy ndr om e  caused  by   i nappr opr i ate  v asopr essi n  secr eti on,   and  i s
seen  m ost  of ten  i n  the  setti ng  of   SCLC.   The  hy ponatr em i a  that  pr esum abl y
r esul ts  f r om   SIADH  can  be  dem onstr ated  i n  up  to  60%  of   the  pati ents  w i th
SCLC,   by   adm i ni ster i ng  a  w ater   l oad.   Cushi ng's  sy ndr om e  m ay   dev el op
secondar y   to  the  ex cessi v e  pr oducti on  of   adr enocor ti cotr opi c  hor m one  by   the
tum or ,   and,   agai n,   i s  m ost  com m onl y   seen  i n  SCLC.   Both  the  absol ute  and
i oni zed  ser um   cal ci um   l ev el s  m ay   be  hi gh,   and  thi s  can  be  due  to  m ul ti pl e
r easons,   i ncl udi ng  m etastases  to  bone  as  w el l   as  the  pr oducti on  of   a
par athy r oi d  hor m one–l i k e  substance  f r om   the  cancer .   Hy per cal cem i a  i s  m ost
of ten  seen
P. 325
i n  pati ents  w i th  squam ous  (epi der m oi d)  l ung  cancer ,   but  m ay   be  associ ated
w i th  any   hi stol ogi c  subty pe.   The  l ocati on  of   the  tum or   by   chest  r adi ogr aphi c
studi es  as  w el l   as  speci f i c  l abor ator y   f i ndi ngs  can  suggest  cer tai n  hi stol ogi c
ty pes.   Squam ous  cel l   car ci nom as  and  SCLCs  tend  to  be  f ound  centr al l y   on  the
chest  r adi ogr aphy .   Squam ous  cel l   l ung  cancer   tends  to  cav i tate,   and  thi s  can
be  seen  on  chest  r adi ogr aphs.   Adenocar ci nom a  tends  to  occur   per i pher al l y   i n
the  l ung,   and  thi s  i s  the  m ost  com m on  l ung  cancer   i n  those  w ho  hav e  nev er
sm ok ed.

Table 7­12 Signs and Symptoms of Lung Cancer

Signs  a nd Symptoms No.  of P a tie nts


(% )

Routi ne  chest  r adi ogr aphi c  study 16


(asy m ptom ati c)

Hem opty si s 30

Cough 25

Dy spnea 11

Pneum oni ti s 8

Pai n 6

Wheezi ng 2
Dy sphagi a 1

Hoar seness 0. 5

Sy stem i c  sy m ptom s—w ei ght  l oss 0. 5

Al though  these  cl i ni cal l y   usef ul   associ ati ons  cannot  establ i sh  a  di agnosi s,   they
can  hel p  the  oncol ogi st  di r ect  the  natur e  of   the  ev al uati on,   both  bef or e  and
af ter   the  hi stol ogi c  ty pe  i s  k now n.

4.   What  i s  essenti al l y   the  m ai nstay   of   cur ati v e  ther apy   i n  N SCLC?

Si gni f i cant  cur e  r ates  f or   N SCLC  ar e  achi ev ed  onl y   f or   com pl ete  sur gi cal
r esecti on  of   nonm etastati c  di sease.   The  stagi ng  pr ocess  def i nes  the  ex tent  of
di sease  spr ead,   and  the  stage  cor r el ates  w i th  the  r esectabi l i ty   and  hence  w i th
the  cur e  r ates  and  sur v i v al .   N o  cur es  ar e  seen  usi ng  the  cur r ent  f or m s  of
chem other apy ,   and  cur e  i s  uncom m on  w i th  r adi ati on  ther apy   al one
(appr ox i m atel y   5%  of   stage  III  cases).

In  al l   cases  i n  w hi ch  r esecti on  i s  per f or m ed  w i th  cur ati v e  i ntent,   the  deci si on
to  go  ahead  w i th  r esecti on  i s  based  on  the  ex pectati on  that  no  gr oss  r esi dual
di sease  w i l l   r em ai n  at  the  concl usi on  of   the  pr ocedur e.   Thi s  i nv ol v es  m ul ti pl e
pr eoper ati v e  deci si ons,   i ncl udi ng  (a)  v er i f y i ng  that  the  pati ent's  l ung  f uncti on
can  tol er ate  obl i gate  r esecti on  of   som e  nor m al   l ung;  (b)  ensur i ng  that  ther e  i s
no  pul m onar y   or   ex tr apul m onar y   di sease  that  w oul d  pr ecl ude  m ajor   sur ger y
and  gener al   anesthesi a;  and  (c)  accur atel y   stagi ng  the  ex tent  of   di sease  to
sel ect  onl y   those  pati ents  w i th  a  r easonabl e  chance  of   com pl ete  r esecti on.
Ev en  w i th  these  consi der ati ons,   how ev er ,   ov er al l   onl y   20%  of   al l   pati ents  w i th
N SCLC  sur v i v e  5  y ear s,   and  l ess  than  hal f   of   pati ents  w ho  under go  successf ul
r esecti on  r em ai n  f r ee  of   di sease  at  5  y ear s.

At  pr esent,   onl y   stage  I  and  II,   and  som e  stage  III,   pati ents  ar e  consi der ed
good  candi dates  f or   r esecti on  w i th  a  r easonabl e  chance  of   cur e.   Thi s  m eans
the  tum or   cannot  be  associ ated  w i th  m al i gnant  ef f usi on,   cannot  hav e  spr ead  to
contr al ater al   m edi asti nal   l y m ph  nodes,   and  cannot  hav e  m etastasi zed  to  any
di stant  si te.   The  usual   si tes  of   di stant  m etastases  i n  pati ents  w i th  l ung  cancer
i ncl ude  the  adr enal   gl ands,   bone,   br ai n,   l ung,   l y m ph  nodes,   and  pl eur a.   Less
com m on  si tes  of   m etastases  ar e  the  sk i n  and  bone  m ar r ow .

The  standar d  tr eatm ent  f or   stage  IIIB  pati ents  (i . e. ,   those  w i thout  di stant
m etastases  but  w i th  unr esectabl e  and  l ocal l y   adv anced  di sease)  i s  r adi ati on
ther apy   to  a  total   dose  of   55  to  60  Gy   al ong  w i th  chem other apy .   Onl y   20%  of
these  pati ents  ar e  al i v e  at  5  y ear s.   Stage  IV  pati ents  ar e  those  w i th  di stant
m etastases  and,   by   and  l ar ge,   tr eatm ent  i n  thi s  setti ng  i s  f or   pal l i ati on  onl y
and  consi sts  m ostl y   of   chem other apy   usi ng  pl ati num ­based  r egi m ens.

5.   How   does  the  stagi ng  and  tr eatm ent  of   SCLC  di f f er   f r om   that  of   N SCLC?

Sm al l   cel l   l ung  car ci nom a  di f f er s  m ar k edl y   f r om   the  other   pathol ogi c  ty pes  of
l ung  cancer   i n  ter m s  of   i ts  natur al   hi stor y ,   cel l   bi ol ogy ,   and  r esponse  to
ther apy ,   and  i s  di sti nct  f r om   N SCLC.   SCLC  has  a  r api d  cl i ni cal   cour se,   and
P. 326
the  m edi an  sur v i v al   i n  untr eated  pati ents  i s  onl y   2  m onths  f or   m etastati c  and  6
m onths  f or   l ocal i zed  di sease.   On  the  basi s  of   autopsy   data,   80%  of   pati ents
w i th  SCLC  hav e  di stant  m etastases  at  the  ti m e  of   di agnosi s.   Thi s  i ncl udes  the
40%  w ho  at  di agnosi s  hav e  no  ev i dence  of   di stant  m etastases  accor di ng  to  the
r esul ts  of   the  usual   stagi ng  pr ocedur es.

For   pr acti cal   r easons,   SCLC  i s  f ur ther   def i ned  as  ei ther   l i m i ted  or   ex tensi v e.   In
l i m i ted  di sease,   the  tum or   i s  conf i ned  to  the  hem i thor ax   of   or i gi n  and  r egi onal
l y m ph  nodes,   and  can  be  encom passed  i n  a  tol er abl e  r adi ati on  ther apy   f i el d.   In
ex tensi v e  di sease,   tum or   ex i sts  bey ond  these  bounds,   usual l y   di stant
m etastases.   Sur ger y   (other   than  f or   di agnosti c  bi opsy )  has  no  r ol e  i n  the
m anagem ent  of   SCLC  because  ev en  m ost  of   the  pati ents  w i th  l i m i ted  di sease
hav e  subcl i ni cal   di stant  m etastases;  com pl ete  r esponse  r ates  to  chem other apy
i n  SCLC  ar e  as  hi gh  as  40%  to  70%,   w i th  com pl ete  pl us  par ti al   r esponse  r ates
as  hi gh  as  80%  to  85%,   and  sur v i v al   i n  the  setti ng  of   untr eated  di sease  i s
di sm al .   Instead,   m ost  pati ents  shoul d  r ecei v e  com bi nati on  chem other apy .   Wi th
si x   or   m or e  cour ses  of   chem other apy   and  r adi ati on,   appr ox i m atel y   20%  of
pati ents  w i th  l i m i ted  di sease  can  ex pect  to  be  al i v e  at  5  y ear s,   and  these
r epr esent  pr obabl e  cur es.   The  cur r ent  standar d  of   ther apy   i ncl udes  i r r adi ati on
of   the  i nv ol v ed  ar eas  as  w el l   as  chem other apy   to  decr ease  the  chances  of
r el apse,   l ocal l y .

In  pati ents  w i th  ex tensi v e  di sease  w ho  under go  chem other apy ,   the  par ti al   pl us
com pl ete  r esponse  r ates  can  r ange  f r om   50%  to  85%  and  the  m edi an  sur v i v al
can  r ange  f r om   7  to  11  m onths;  how ev er ,   ther e  ar e  onl y   anecdotal   r epor ts  of
cur e  at  5  y ear s.

6.   What  ar e  the  tw o  m ajor   deter m i nants  of   pr ognosi s  f or   both  N SCLC  and  SCLC?

For   both  SCLC  and  N SCLC,   the  pr ognosi s  depends  on  (a)  the  tum or   ex tent
(gr aded  as  l i m i ted  or   ex tensi v e  i n  SCLC  and  as  stage  I  to  IV  i n  N SCLC)  and  (b)
the  per f or m ance  status  of   the  pati ent.   Better   r esponses  to  ther apy   and
cer tai nl y   m or e  si gni f i cant  cur e  r ates  ar e  seen  i n  pati ents  w i th  l ow er ­stage  l ung
cancer s.   In  addi ti on,   the  per f or m ance  status  [usual l y   m easur ed  by   the  Easter n
Cooper ati v e  Oncol ogy   Gr oup  (ECOG)  scal e]  i s  a  m ajor   pr edi ctor   of   r esponse  to
ther apy   and  sur v i v al .   Pati ents  w ho  ar e  sy m ptom ati c  or ,   m or e  si gni f i cantl y ,
nonam bul ator y   (as  i ndi cated  by   thei r   per f or m ance  status)  ar e  l ess  l i k el y   to
r espond  to  ther apy   and  hav e  shor ter   sur v i v al s.   The  per f or m ance  status  scal es
m ost  of ten  used  by   cl i ni ci ans  ar e  gi v en  i n  Tabl e  7­13.

Case 1
A  56­y ear ­ol d  r eal   estate  br ok er   w i th  a  76  pack ­y ear   hi stor y   of   tobacco  use  (he  has
sm ok ed  tw o  pack s  of   ci gar ettes  per   day   si nce  18  y ear s  of   age)  has  been  f ol l ow ed
up  r egul ar l y   by   hi s  phy si ci an.   He  under goes  y ear l y   chest  r adi ogr aphi c  studi es,   and
the  m ost  r ecent  r adi ogr aphs  obtai ned  8  m onths  ear l i er   w er e  nor m al .   He  i s  seen  by
hi s  phy si ci an  because  of   10  day s  of   hem opty si s,   consi sti ng  of   bl ood­ti nged  sputum
pr oducti on,   i n  the  setti ng  of   a  chr oni c  cough.   He  deni es  w ei ght  l oss,   chest  pai n,   and
bone  pai n,   and  he  ex per i ences  no  i ncr eased  dy spnea  on  ex er ti on.   On  ex am i nati on,
hi s  l ungs  ar e  f ound  to  be  cl ear ;  ther e  i s  nei ther   hepatospl enom egal y   nor   cl ubbi ng
and  the  neur ol ogi c  f i ndi ngs  ar e  gr ossl y   nonf ocal .   Labor ator y   studi es  show   nor m al
l i v er   f uncti on,   a  cal ci um   l ev el   of
P. 327
11. 1  m g/dL,   and  an  al bum i n  l ev el   of   3. 9  g/dL.   Hi s  com pl ete  bl ood  count  i s  nor m al .
A  chest  r adi ogr aphi c  study   dem onstr ates  a  new ,   2  ×  3  cm ,   r i ght  hi l ar   m ass.

Table 7­13 Karnofsky and Zubrod Performance
Status Scales

De finition Ka rnofs k y Sc a le Zubrod Sc a le


(% ) (Old (SW OG ,  ECOG
Comple x a re
Cla s s ific a tion) Va ria tions )

N or m al ;  no  com pl ai nts;  no  ev i dence 100 0


of   di sease

Abl e  to  car r y   on  nor m al   acti v i ty ; 90 1


m i nor   si gns  or   sy m ptom s  of   di sease

N or m al   acti v i ty   w i th  ef f or t;  som e 80 1


si gns  or   sy m ptom s  of   di sease  (no
speci al   car e  needed;  f ul l y
am bul ator y )

Car es  f or   sel f   but  unabl e  to  car r y   on 70 2


nor m al   acti v i ty   (unabl e  to  w or k ;  i n
bed  < 50%/d)

Requi r es  occasi onal   assi stance  but  i s 60 2


abl e  to  car e  f or   m ost  needs  (i n  bed
> 50%/d  but  not  bedr i dden)

Requi r es  consi der abl e  assi stance  and 50 3


f r equent  m edi cal   car e

Di sabl ed;  r equi r es  speci al   car e  and 40 3


assi stance

Bedr i dden 30 4

Ver y   si ck ;  hospi tal i zati on  necessar y 20 4

Mor i bund 10 4
Dead 0 5

SWOG,   Southw est  Oncol ogy   Gr oup;  ECOG,   Easter n  Cooper ati v e  Oncol ogy
Gr oup.

1.   What  di agnosti c  tests  shoul d  be  per f or m ed  i n  thi s  pati ent?
2.   Was  a  y ear l y   chest  r adi ogr aphi c  study   a  r easonabl e  scr eeni ng  pr ocedur e  f or
thi s  m an  w ho  sm ok es,   or   shoul d  thi s  hav e  been  done  m or e  f r equentl y ?
3.   What  cl i ni cal   f i ndi ng  suggests  (al though  does  not  establ i sh)  the  hi stol ogi c  ty pe
of   thi s  pati ent's  l ung  cancer ,   and  w hat  ty pe  i s  i t?
4.   What  stage  of   l ung  cancer   i s  thi s  pati ent  i n,   and  w hat  f ur ther   studi es  shoul d  be
per f or m ed?
5.   If   the  pati ent  has  stage  II  N SCLC,   w hat  i s  the  appr opr i ate  tr eatm ent?
6.   What  f ur ther   m easur es  can  be  tak en  to  r educe  the  pati ent's  r i sk   of   death  f r om
l ung  cancer ?

Case Discussion
1.   What  di agnosti c  tests  shoul d  be  per f or m ed  i n  thi s  pati ent?

Most  l ung  cancer s  ar e  di agnosed  on  the  basi s  of   the  f i ndi ngs  y i el ded  by
br onchoscopi c  bi opsy   of   a  l ung  m ass.   In  som e  si tuati ons,   how ev er ,   ther e  i s  an
easi er
P. 328
sour ce  of   ti ssue.   For   ex am pl e,   bi opsy   of   an  abnor m al   (pal pabl e)
supr acl av i cul ar   l y m ph  node,   i f   pr esent,   w i l l   l i k el y   y i el d  ti ssue  adequate  f or
hi stol ogi c  study   as  w el l   as  f or   stagi ng  pur poses,   and  the  i nf or m ati on  gai ned  i s
i m por tant  i n  pl anni ng  tr eatm ent.

The  cl i ni ci an  m ust  be  w ar y   of   i ni ti al   di agnoses  of   m al i gnancy   based  on  the


f i ndi ngs  r ev eal ed  by   the  f i ne­needl e  aspi r ati on  (FN A)  techni que.   Thi s  m ethod
obtai ns  ei ther   onl y   si ngl e  cel l s  or   sm al l   cl um ps  of   cel l s  f or   cy tol ogi c  study
w i thi n  the  aspi r ate,   and  i t  i s  al m ost  al w ay s  di f f i cul t  or   i m possi bl e  to  deter m i ne
an  ex act  pathol ogi c  di agnosi s  based  on  the  i nf or m ati on  r ev eal ed.   FN A  bi opsy
shoul d  usual l y   be  r eser v ed  f or   conf i r m ati on  of   m etastases  or   r ecur r ences  af ter
an  i ni ti al   di agnosi s  has  al r eady   been  m ade.   In  m ost  cases,   br onchoscopi c
bi opsy   obtai ns  ti ssue  adequate  f or   hi stol ogi c  ex am i nati on.

Som eti m es,   pati ents  pr esent  w i th  sy m ptom s  stem m i ng  f r om   m etastati c
di sease,   and  the  sour ce  of   a  pr i m ar y   tum or   i s  not  as  cl ear   as  i t  i s  i n  thi s
pati ent.   In  such  an  ev ent,   cor r el ati on  of   the  pathol ogi c  char acter i sti cs  i ncl udi ng
speci al   stai ns  w i th  the  l i k el y   sour ces  of   the  pr i m ar y   cancer   can  suggest  the
f ur ther   r adi ol ogi c  and  di agnosti c  tests  to  be  per f or m ed  to  deter m i ne  the  or i gi n
of   the  tum or .

2.   Was  a  y ear l y   chest  r adi ogr aphi c  study   a  r easonabl e  scr eeni ng  pr ocedur e  f or
thi s  m an  w ho  sm ok es,   or   shoul d  thi s  hav e  been  done  m or e  f r equentl y ?

Because  l ung  cancer   (both  N SCLC  and  SCLC)  cur e  r ates  ar e  si gni f i cant  onl y   i n
the  setti ng  of   ear l i er ­stage  di sease  (stage  I  and  II  N SCLC  and  l i m i ted­stage
SCLC),   i t  seem s  r easonabl e  that  ear l i er   detecti on  w oul d  i ncr ease  the
per centage  of   cur es.   In  sev er al   r andom i zed,   contr ol l ed  tr i al s,   m ass  scr eeni ng
usi ng  r oentgenogr aphi c  and  sputum   cy tol ogy   has  been  done  i n  31, 360  hi gh­r i sk
pati ents  (m en  45  y ear s  of   age  or   ol der   w ho  sm ok ed  at  l east  one  pack   of
ci gar ettes  per   day ).   Al l   these  studi es  hav e  show n  that  i ntensi v e  scr eeni ng  can
detect  ear l y   l ung  cancer ,   but  45%  to  60%  of   pati ents  so  i denti f i ed  actual l y
hav e  stage  II  or   III  di sease,   f or   w hom   the  5­y ear   sur v i v al   r ate  i s  35%.
Inv esti gator s  at  al l   thr ee  center s  contend  that  the  m or tal i ty   r ates  f or   l ung
cancer   do  not  di f f er   si gni f i cantl y   betw een  the  scr eeni ng  gr oup  and  the  contr ol
gr oup.   Ther ef or e,   at  thi s  ti m e  ther e  i s  no  justi f i cati on  f or   the  l ar ge­scal e
appl i cati on  of   these  scr eeni ng  m ethods,   ev en  i n  hi gh­r i sk   popul ati ons.

Year l y   scr eeni ng  chest  r adi ogr aphi c  studi es  i n  the  sm ok i ng  popul ati on  i s  not
r ecom m ended  and  no  study   f i ndi ngs  suppor t  m or e  f r equent  scr eeni ng  chest
r oentgenogr aphy   to  decr ease  the  m or tal i ty .

3.   What  cl i ni cal   f i ndi ng  suggests  (al though  does  not  establ i sh)  the  hi stol ogi c  ty pe
of   thi s  pati ent's  l ung  cancer ,   and  w hat  ty pe  i s  i t?

Thi s  pati ent's  chest  r adi ogr aph  r ev eal ed  the  l ung  m ass  to  be  centr al   (hi l ar )  i n
l ocati on.   The  tw o  com m on  tum or   ty pes  ty pi cal l y   f ound  i n  thi s  l ocati on  ar e
squam ous  (epi der m oi d)  cel l   car ci nom a  (a  ty pe  of   N SCLC)  and  SCLC,   al though
bi opsy   f i ndi ngs  ar e  needed  to  establ i sh  the  di agnosi s.   The  hi gh  cal ci um   v al ue
i s  m ost  of ten  seen  w i th  squam ous  cel l   car ci nom a.

4.   What  stage  of   l ung  cancer   i s  thi s  pati ent  i n,   and  w hat  f ur ther   studi es  shoul d  be
per f or m ed?

Accor di ng  to  the  cur r ent  Am er i can  Joi nt  Com m i ttee  on  Cancer   (TN M)  stagi ng
sy stem ,   the  stage  of   thi s  pati ent's  cancer   i s  T2  N X  MX  (tum or   3  cm   i n  gr eatest
di m ensi on,   nodal   status  unk now n,   and  unk now n  m etastases).

P. 329
In  the  setti ng  of   N SCLC,   nor m al   l i v er   f uncti on  test  r esul ts  (i ncl udi ng
am i notr ansf er ases,   al k al i ne  phosphatase,   LDH,   and  γ­gl utam y l   tr anspepti dase)
pr edi ct  r easonabl y   w el l   that  no  l i v er   m etastases  w i l l   be  f ound  on  a
r adi onucl i de  or   CT  scan.   In  thi s  pati ent,   a  CT  scan  of   the  chest  shoul d  be
obtai ned  to  ev al uate  the  hi l ar   and  m edi asti nal   nodes,   and  the  ex am i nati on  can
easi l y   be  ex tended  thr ough  the  l i v er   and  adr enal s.   If   the  CT  f i ndi ngs  and  the
al k al i ne  phosphatase  l ev el   ar e  nor m al ,   and  because  the  pati ent  has  no  bone
pai n,   these  w oul d  pr edi ct  no  bone  m etastases,   m ak i ng  a  bone  scan
unnecessar y .   If   f i ndi ngs  f r om   a  thor ough  neur ol ogi c  ex am i nati on  per f or m ed  by
a  consul ti ng  neur ol ogi st  ar e  nor m al ,   CT  scanni ng  of   the  head  can  be  def er r ed.
In  thi s  setti ng,   the  pati ent  w oul d  cl i ni cal l y   be  i n  stage  II,   gr aded  T2  N 0  M0.   A
PET  scan  w oul d  better   conf i r m   thi s  stagi ng  i f   sur ger y   i s  consi der ed  (see
pr ecedi ng  tex t).

5.   If   the  pati ent  has  stage  II  N SCLC,   w hat  i s  the  appr opr i ate  tr eatm ent?

For   stage  II  N SCLC,   sur gi cal   ex ci si on  (l obectom y   or   pneum onectom y )  w i th
i ntent  of   cur e  i s  the  tr eatm ent  of   choi ce.   If   the  ar ter i al   bl ood  gas  v al ues,
pul m onar y   f uncti on  tests,   el ectr ocar di ogr am ,   and  past  m edi cal   hi stor y   i ndi cate
that  the  pati ent  has  no  ex cess  r i sk   f or   m ajor   sur ger y ,   a  thor aci c  sur geon  i s
then  consul ted.   Because  the  pati ent  m ay   sti l l   hav e  spr ead  of   cancer   to  the
m edi asti nal   or   hi l ar   nodes  (and  not  seen  on  a  CT  or   PET  scan  as  bei ng
abnor m al ),   w hi ch  w oul d  m ak e  hi s  cancer   unr esectabl e,   m edi asti noscopy   m i ght
be  per f or m ed  especi al l y   i f   the  pati ent  i s  not  the  best  sur gi cal   candi date.

If   these  f i ndi ngs  ar e  negati v e,   a  r i ght  upper   l obectom y   thr ough  a  l ater al
thor acotom y   can  be  per f or m ed  w i th  subsequent  pathol ogi c  study .   If ,   f or
i nstance,   a  2. 8  ×  3. 2­cm   tum or   w i th  tw o  per i br onchi al   nodes  posi ti v e  f or
squam ous  cel l   l ung  cancer   i s  encounter ed  at  sur ger y ,   the  pati ent  i s  i n
pathol ogi c  stage  II,   desi gnated  T2  N 1  M0,   and  hi s  ov er al l   chance  of   5­y ear
sur v i v al   r anges  f r om   35%  to  45%  (w i th  postoper ati v e  chem other apy ).   If   the
pati ent  i s  f ound  to  hav e  a  pathol ogi c  stage  I  tum or ,   the  5­y ear   sur v i v al   w oul d
appr oach  50%  to  60%,   m ostl y   r epr esenti ng  cur es.

6.   What  f ur ther   m easur es  can  be  tak en  to  r educe  the  pati ent's  r i sk   of   death  f r om
l ung  cancer ?

Ev en  pati ents  sur gi cal l y   cur ed  of   thei r   l ung  cancer   hav e  a  si gni f i cant  chance  of
acqui r i ng  subsequent  l ung  cancer s,   as  w el l   as  other   tum or s.   As  w i th  those  i n
w hom   tum or s  hav e  not  y et  dev el oped,   cessati on  of   sm ok i ng  can  si gni f i cantl y
r educe  subsequent  r i sk ,   and  thi s  pati ent  shoul d  be  str ongl y   adv i sed  to  do  so.

Case 2
A  68­y ear ­ol d  w om an  pr esents  to  the  em er gency   r oom   because  of   a  new ­onset
gr and  m al   sei zur e.   She  i s  l ethar gi c,   but  neur ol ogi c  f i ndi ngs  ar e  other w i se  nor m al .   A
head  CT  scan  r ev eal s  a  2­cm   r i ght  par i etal   and  a  0. 5­cm   l ef t  occi pi tal   enhanci ng
m ass,   and  a  chest  r adi ogr aphi c  study   r ev eal s  a  4­cm   l ef t  hi l ar   m ass  w i th  di stal
atel ectasi s.   She  has  sm ok ed  one  pack   of   ci gar ettes  per   day   f or   40  y ear s,   but  qui t  1
m onth  ago.   Anti conv ul sants  ar e  adm i ni ster ed,   the  pati ent  i s  adm i tted  to  the
hospi tal ,   and  br onchoscopy   i s  per f or m ed,   w hi ch  show s  a  m ass  i n  the  r i ght  m ai nstem
br onchus.   Bi opsy   i s  done  and  pathol ogi c  ex am i nati on  r ev eal s  a  sm al l   cel l   cancer .
A  bone  scan  show s  an  abnor m al i ty   i n  her   l ef t  f em ur ,   and  her   al k al i ne  phosphatase
l ev el   i s  i ncr eased  at  214  m U /m L.   A  CT  scan  of   the  abdom en  i s  nor m al .
P. 330

1.   What  stage  i s  thi s  pati ent's  cancer ?


2.   How   shoul d  thi s  pati ent's  cancer   be  tr eated?
3.   If ,   6  w eek s  af ter   di agnosi s,   phy si cal   ex am i nati on  r ev eal s  onl y   al opeci a,   and  a
chest  r adi ogr aph  and  bone  scan  ar e  nor m al ,   w hat  i s  the  l i k el i hood  of   cur e?

Case Discussion
1.   What  stage  i s  thi s  pati ent's  cancer ?

The  pati ent  has  ex tensi v e  SCLC  w i th  m etastases  to  the  br ai n.

2.   How   shoul d  thi s  pati ent's  cancer   be  tr eated?

Most  chem other apy   tr av er ses  the  bl ood–br ai n  bar r i er   poor l y .   Ther ef or e,   the
pati ent  shoul d  be  star ted  on  w hol e­br ai n  r adi ati on  ther apy .   Because  SCLC  i s  a
r api dl y   gr ow i ng  tum or ,   and  r adi ati on  ther apy   l asts  4  to  6  w eek s,   com bi nati on
chem other apy   shoul d  al so  be  i ni ti ated.

3.   If ,   6  w eek s  af ter   di agnosi s,   phy si cal   ex am i nati on  r ev eal s  onl y   al opeci a,   and  a
chest  r adi ogr aph  and  bone  scan  ar e  nor m al ,   w hat  i s  the  l i k el i hood  of   cur e?
Sm al l   cel l   l ung  cancer   i s  r esponsi v e  to  both  r adi ati on  ther apy   and
chem other apy .   Com pl ete  r esponses  ar e  w i tnessed  i n  15%  to  30%  of   pati ents
w i th  ex tensi v e  di sease.   How ev er ,   the  5­y ear   sur v i v al   i s  onl y   4%  to  8%  i n  thi s
setti ng,   and  tum or   usual l y   r ecur s  qui ck l y .

Suggested Readings
Bunn  PA,   Li chter   AS,   Mak uch  RW,   et  al .   Chem other apy   al one  or   chem other apy
w i th  chest  r adi ati on  ther apy   i n  l i m i ted  stage  sm al l   cel l   l ung  cancer :  a
pr ospecti v e  r andom i zed  tr i al .   Ann  Inter n  Med  1987;106:655.

Car ney   DN ,   de  Lei j  L.   Lung  cancer   bi ol ogy .   Sem i n  Oncol   1988;15:199.

El der l y   Lung  Cancer   Vi nor el bi ne  Ital i an  Study   Gr oup.   Ef f ects  of   v i nor el bi ne  on
qual i ty   of   l i f e  and  sur v i v al   of   el der l y   pati ents  w i th  adv anced  non­sm al l ­cel l   l ung
cancer .   J  N atl   Cancer   Inst  1999;91:66.

Far r ay   D,   Mi r k ov i c  N ,   Al bai n  KS.   Mul ti m odal i ty   ther apy   f or   stage  III  non­sm al l ­
cel l   l ung  cancer .   J  Cl i n  Oncol   2005;23:3257.

Lababede  O,   Mezi ane  MA,   Ri ce  TW.   TN M  stagi ng  of   l ung  cancer :  a  qui ck
r ef er ence  char t.   Chest  1999;115:233.

Mountai n  CF,   Luck m an  JM,   Ham m er   SP,   et  al .   Lung  cancer   cl assi f i cati on:  the
r el ati onshi p  of   di sease  ex tent  and  cel l   ty pe  to  sur v i v al   i n  a  cl i ni cal   tr i al s
popul ati on.   J  Sur g  Oncol   1987;35:147.

Prostate Cancer
1.   How   com m on  i s  pr ostate  cancer ?

2.   How   does  pr ostate  cancer   ar i se  and  spr ead?

3.   How   i s  pr ostate  cancer   gr aded  and  staged,   and  w hy   i s  thi s  i m por tant?

P. 331
4.   What  i s  the  ty pi cal   cl i ni cal   pr esentati on  of   pr ostate  cancer ?

5.   Af ter   a  phy si cal   ex am i nati on,   how   i s  a  suspi ci on  of   pr ostate  cancer   conf i r m ed?

6.   Once  pr ostate  cancer   i s  di agnosed,   how   i s  a  stagi ng  ev al uati on  per f or m ed?

7.   What  tr eatm ent  i s  r ecom m ended  f or   stage  Al   pr ostate  cancer ?

8.   Whi ch  categor y   of   pati ents  i s  consi der ed  f or   a  r adi cal   pr ostatectom y ?

9.   Is  any   al ter nati v e  ther apy   av ai l abl e  f or   pati ents  w i th  stage  A2  or   B  pr ostate
cancer ?

10.   What  ther apy   m ay   be  adm i ni ster ed  to  pati ents  w i th  stage  C  or   D  di sease?
11.   In  ter m s  of   the  r ecom m ended  tr eatm ents,   w hat  sur v i v al   can  be  ex pected  i n
pati ents  w i th  stage  A1,   A2,   B,   or   C  pr ostate  cancer ?

12.   What  com pl i cati ons  attend  the  use  of   r adi cal   pr ostatectom y   and  i r r adi ati on
w hi ch  pati ents  shoul d  be  aw ar e  of   i n  adv ance?

13.   What  i s  the  tr eatm ent  f or   di ssem i nated  pr ostate  cancer ?

14.   Is  the  pr ostate­speci f i c  anti gen  l ev el   usef ul   i n  scr eeni ng  asy m ptom ati c  m en  f or
pr ostate  cancer ?

Discussion
1.   How   com m on  i s  pr ostate  cancer ?

Appr ox i m atel y   90, 000  new   cases  of   car ci nom a  of   the  pr ostate  ar e  i denti f i ed
ev er y   y ear .   Of   m en  i n  thei r   ei ghti es  and  ol der   than  90  y ear s,   50%  and  90%  of
the  pr ostates,   r especti v el y ,   ar e  f ound  to  hav e  car ci nom a  at  autopsy .   It  i s  the
second  m ost  com m on  m al i gnancy   i n  Am er i can  m en  and  the  m ost  com m on
cancer ous  cause  of   death  i n  m en  ol der   than  75  y ear s.   The  cl i ni cal   i nci dence  i s
hi gher   i n  bl ack s  than  i n  w hi tes,   and  i t  i s  m uch  l ow er   f or   Japanese  m en  l i v i ng  i n
Japan.   Ther ef or e,   r aci al   and  env i r onm ental   f actor s  m ay   be  i nv ol v ed  i n  i ts
dev el opm ent.

2.   How   does  pr ostate  cancer   ar i se  and  spr ead?

Pr ostate  adenocar ci nom a  ar i ses  f r om   the  epi thel i um   of   the  per i pher al   aci nar
gl ands.   It  i s  sl ow   gr ow i ng,   and  hal f   of   al l   cases  of   ear l y ­stage  (A)  pr ostate
cancer   ar e  f ound  onl y   at  autopsy .   Pr ostate  cancer   f i r st  ex tends  l ocal l y   thr ough
i ts  capsul e  and  then  i nv ades  the  l y m phati cs  and  bl ood  v essel s.   Ly m ph  node
i nv ol v em ent  occur s  sequenti al l y ,   w i th  i ni ti al   spr ead  to  the  per i pr ostati c  and
obtur ator   nodes,   and  l ater   the  i l i ac  and  per i aor ti c  nodes.   Di stant  hem atogenous
spr ead  tends  to  occur   l ate  i n  the  di sease  and  f r equentl y   i nv ol v es  the  ax i al   and
pr ox i m al   sk el eton,   l i v er ,   and  l ungs.   How ev er ,   m or e  than  hal f   of   al l
sy m ptom ati c  pati ents  w i th  pr ostate  cancer   pr esent  w i th  m etastases.

3.   How   i s  pr ostate  cancer   gr aded  and  staged,   and  w hy   i s  thi s  i m por tant?

Pr ostate  cancer   i s  br oadl y   staged  i nto  categor i es  A,   B,   C,   and  D.   Stage  A  i s


car ci nom a  detected  by   pathol ogi c  ex am i nati on  of   gl ands  r em ov ed  because  of
cl i ni cal l y   beni gn  di sease.   Stage  A1  r epr esents  w el l ­di f f er enti ated  tum or
i nv ol v i ng  l ess  than  5%  of   the  sur gi cal   speci m en,   and  stage  A2  consti tutes
tum or   ei ther   i nv ol v i ng  m or e  than  5%  of   the  sur gi cal   speci m en  or   a  poor l y
di f f er enti ated  tum or .   Stage  B  i s  cl i ni cal l y   detectabl e  di sease  that  i s
i ntr acapsul ar   and  i nv ol v es  one  l obe  (B1)  or   m or e  than  one  l obe  (B2).   In  stage
A  and  B  cancer ,   the  pr ostate­speci f i c  anti gen  l ev el s  ar e  el ev ated  15%  and  25%
of   the  i nstances,
P. 332
r especti v el y .   Stage  C  i s  cancer   that  has  i nv aded  thr ough  the  pr ostate  capsul e
but  i s  conf i ned  to  the  pel v i s.   Stage  D,   or   m etastati c  di sease,   i ncl udes  tw o
categor i es:  D1  (l ocal   pel v i c  nodal   i nv ol v em ent)  and  D2  (di stant,   nodal ,   bone,
or   v i scer al   di sease),   and  i s  associ ated  w i th  el ev ated  ser um   tum or   m ar k er s.

4.   What  i s  the  ty pi cal   cl i ni cal   pr esentati on  of   pr ostate  cancer ?

Cl assi cal l y ,   the  di sease  i s  detected  i n  el der l y   m en  w i th  the  occur r ence  of   back
pai n,   w ei ght  l oss,   anem i a,   ur i nar y   f r equency ,   and  noctur i a,   and,   l ess  of ten,
dy sur i a,   a  sl ow   ur i nar y   str eam ,   ur i nar y   r etenti on,   and  r ar el y   hem atur i a.

Pr ostate  cancer   can  be  f ound  dur i ng  a  r outi ne  phy si cal   ex am i nati on  or
scr eeni ng,   or   w hen  i nv esti gati ng  f or   a  pathol ogi c  f r actur e,   bone  pai n,   pal pabl e
l y m phadenopathy ,   chr oni c  r enal   i nsuf f i ci ency ,   anem i a,   cachex i a,   or   a  num ber
of   other   seem i ngl y   unr el ated  si gns  and  sy m ptom s.   Appr ox i m atel y   50%  of   the
i ndur ated  pr ostati c  l esi ons  f el t  on  r ectal   ex am i nati on  tur n  out  to  be
adenocar ci nom a  on  bi opsy .   The  m ost  ef f ecti v e  scr eeni ng  test  r em ai ns  a  car ef ul
r ectal   ex am i nati on  that  shoul d  be  per f or m ed  star ti ng  f r om   age  50.

5.   Af ter   a  phy si cal   ex am i nati on,   how   i s  a  suspi ci on  of   pr ostate  cancer   conf i r m ed?

Most  of ten,   the  di agnosi s  of   pr ostate  cancer   i s  conf i r m ed  by   needl e  bi opsy
f i ndi ngs.   The  tr ansr ectal   r oute  has  been  m ade  accur ate  and  saf e  thr ough  the
use  of   tr ansr ectal   ul tr asonogr aphy   and  the  bi opsy   gun.   Open  bi opsy   and
tr ansur ethr al   bi opsy   ar e  l ess  f r equentl y   used  ow i ng  to  the  hi gher   m or bi di ty
associ ated  w i th  thei r   use.

6.   Once  pr ostate  cancer   i s  di agnosed,   how   i s  a  stagi ng  ev al uati on  per f or m ed?

A  thor ough  phy si cal   ex am i nati on  i s  per f or m ed  to  deter m i ne  the  si ze  of   the
pr i m ar y   tum or   and  i f   any   ev i dence  of   m etastati c  tum or   i s  pr esent.   Ser um
pr ostate­speci f i c  anti gen  l ev el   and  l i v er   f uncti on  ar e  assessed.   A  chest
r adi ogr aphi c  study   and  r adi oi sotope  bone  scan  ar e  r outi nel y   per f or m ed.
Dependi ng  on  the  pati ent,   pel v i c  or   other   CT  scans,   ul tr asonogr aphy ,   bi pedal   or
pel v i c  l y m phadenectom y ,   and  a  sk el etal   r adi ogr aphi c  sur v ey   m ay   be  r equi r ed.
Intr av enous  py el ogr aphy   i s  per f or m ed  to  assess  ur eter al   status.

A  com pl ete  bl ood  count  as  w el l   as  BU N   and  ser um   cr eati ni ne  m easur em ents
ar e  r outi nel y   done  to  detect  anem i a  or   obstr ucti v e  ur opathy .   The  pr ostate­
speci f i c  anti gen  l ev el   i s  sensi ti v e  (95%)  and  equal l y   speci f i c  (> 95%).   A  chest
r adi ogr aphi c  study   m ay   r ev eal   m etastases  to  the  r i bs,   l ungs,   or   hi l ar   nodes.
The  bone  scan  i s  m or e  sensi ti v e  and  has  l ar gel y   suppl anted  the  sk el etal
r adi ogr aphi c  sur v ey ,   but  r equi r es  car ef ul   i nter pr etati on.   The  abdom i nal   CT
scan,   together   w i th  posi ti v e  FN A  f i ndi ngs  i n  the  pel v i c  nodes,   can  of ten
el i m i nate  the  need  f or   a  pel v i c  l y m phadenectom y   f or   stagi ng  pur poses.   The
l atter   i s  not  usef ul   f or   stage  A1  l esi ons,   w hi ch  al m ost  nev er   m etastasi ze  to
pel v i c  nodes,   or   f or   poor l y   di f f er enti ated  stage  C  pr ostate  adenocar ci nom a  w i th
el ev ated  tum or   m ar k er s,   because  m or e  than  90%  of   these  ar e  m etastati c.
How ev er ,   pel v i c  l y m phadenectom y   i s  f r equentl y   usef ul   i n  stagi ng  i nter m edi ate
di sease  to  deter m i ne  the  pr ognosi s  and  pl an  the  tr eatm ent.

7.   What  tr eatm ent  i s  r ecom m ended  f or   stage  A1  pr ostate  cancer ?

Onl y   obser v ati on  i s  r ecom m ended  by   m ost  ur ol ogi sts  f or   pati ents  w i th  stage  A1
pr ostate  cancer   because  of   the  v er y   sl ow   and  i nf r equent  (< 10%)
P. 333
pr ogr essi on  of   the  tum or   at  thi s  stage.   The  data  f r om   f ol l ow ­up  studi es  hav e
i ndi cated  that  the  sur v i v al   f or   thi s  gr oup  i s  si m i l ar   to  that  of   age­m atched
contr ol   subjects.

8.   Whi ch  categor y   of   pati ents  i s  consi der ed  f or   a  r adi cal   pr ostatectom y ?

A  r adi cal   pr ostatectom y   i s  ef f ecti v e  i n  pati ents  w i th  tum or   conf i ned  to  the
pr ostate—  stages  A2  and  B  car ci nom a.   The  r etr opubi c  appr oach  w i th  a  ner v e­
spar i ng  techni que  to  m i ni m i ze  the  r i sk   of   i m potence  and/or   i nconti nence  i s
com m onl y   used.   Thi s  i s  usual l y   pr eceded  by   bi l ater al   pel v i c  node  di ssecti on
and  f r ozen­secti on  ex am i nati on  to  conf i r m   noni nv ol v em ent  by   the  tum or .

9.   Is  any   al ter nati v e  ther apy   av ai l abl e  f or   pati ents  w i th  stage  A2  or   B  pr ostate
cancer ?

If   a  stagi ng  l y m phadenectom y   r ev eal s  no  pel v i c  node  m etastases  or   i f   pati ents
ar e  poor   oper ati v e  r i sk s,   pr ostate  cancer   m ay   be  successf ul l y   tr eated  w i th
ex ter nal ­beam   i r r adi ati on.   In  m ost  center s,   i nter nal   r adi ati on  com bi ned  w i th
som e  ex ter nal ­beam   r adi ati on  ther apy   i s  the  tr eatm ent  used.   In  gener al ,   the
l ong­ter m   contr ol   r ate  and  sur v i v al   f or   l ocal i zed  pr ostate  cancer   ar e  si m i l ar
r egar dl ess  of   w hether   sur ger y   or   i r r adi ati on  i s  the  ther apy   sel ected.

10.   What  ther apy   m ay   be  adm i ni ster ed  to  pati ents  w i th  stage  C  or   D  di sease?

Pati ents  w i th  stage  C  or   D  di sease  of ten  r ecei v e  ex ter nal ­beam   i r r adi ati on
del i v er ed  to  the  pel v i c  and  per i aor ti c  nodes,   but  thi s  has  not  been  obser v ed
consi stentl y   and  cl ear l y   to  enhance  sur v i v al .   In  som e  sy m ptom ati c  pati ents
w ho  hav e  pel v i c  pai n,   l ow er   ex tr em i ty   edem a,   hem atur i a,   or   ur ethr al
obstr ucti on,   pal l i ati v e  ex ter nal ­beam   r adi ati on  ther apy   m ay   be  used.   An
al ter nati v e  i s  to  pr oceed  di r ectl y   to  hor m onal   abl ati on  ther apy .

11.   In  ter m s  of   the  r ecom m ended  tr eatm ents,   w hat  sur v i v al   can  be  ex pected  i n
pati ents  w i th  stage  A1,   A2,   B,   or   C  pr ostate  cancer ?

Onl y   1%  of   pati ents  w i th  stage  A1  di sease  di e  of   pr ostate  cancer ,   and
m etastases  dev el op  i n  onl y   5%  w i thi n  5  to  10  y ear s  w i thout  tr eatm ent.   The
natur al   hi stor y   of   thi s  gr oup  i s  deter m i ned  i n  the  el der l y   by   coex i stent  di sease.

Wi thout  tr eatm ent,   35%  of   pati ents  i n  stage  A2  hav e  m etastases  and  20%  di e
w i thi n  5  to  10  y ear s  of   di agnosi s.   The  r esul ts  of   r adi cal   pr ostatectom y   cl osel y
appr ox i m ate  the  cl i ni cal   pi ctur e  seen  i n  untr eated  pati ents  w i th  stage  A1
di sease.   The  sam e  appl i es  to  stage  B1  and  B2  di sease,   and  hence  i t  m ay   be
concl uded  that,   w i th  appr opr i ate  tr eatm ent,   sur v i v al   i n  pati ents  w i th  ear l y ­
stage  pr ostate  cancer   i s  si m i l ar   to  that  of   age­m atched  heal thy   contr ol
subjects.   Ther ef or e,   m ost  pati ents  w i th  stage  A  or   B  di sease,   par ti cul ar l y   those
ol der   than  65  y ear s,   di e  of   other   causes.

12.   What  com pl i cati ons  attend  the  use  of   r adi cal   pr ostatectom y   and  i r r adi ati on,
w hi ch  pati ents  shoul d  be  aw ar e  of   i n  adv ance?

Radi cal   pr ostatectom y ,   ev en  usi ng  m oder n  ner v e­spar i ng  techni ques,   can  cause
i m potence  i n  20%  to  50%  of   pati ents  and  ur i nar y   i nconti nence  i n  5%.   Radi ati on
ther apy   i s  associ ated  w i th  sy m ptom s  of   r adi ati on  pr octi ti s  and  cy stour ethr i ti s.
N one  of   the  i m m edi ate  postoper ati v e  com pl i cati ons  occur s,   but  r adi ati on
ther apy   has  a  si m i l ar   i nci dence  of   the  other   si de  ef f ects,   al though  thei r   onset
m ay   tak e  l onger .

P. 334
13.   What  i s  the  tr eatm ent  f or   di ssem i nated  pr ostate  cancer ?

Hor m onal   m ani pul ati on  i s  the  standar d  appr oach  f or   the  tr eatm ent  of
di ssem i nated  pr ostate  cancer .   Chem other apy   i s  m i ni m al l y   ef f ecti v e  and  i s
consi der ed  onl y   i f   hor m onal   tr eatm ent  f ai l s  (hor m one  “r ef r actor y â€​
cancer ).

The  r esponse  to  tr eatm ent  i n  the  setti ng  of   pr ostate  cancer   i s  ty pi cal l y   di f f i cul t
to  assess.   It  i ncl udes  phy si cal   f i ndi ngs,   per f or m ance  status,   bone  pai n,   w ei ght
change,   and  the  hem ogl obi n  l ev el .   Bone  scans  and  other   i m agi ng  techni ques
and,   m ost  i m por tant,   pr ostate­speci f i c  anti gen  l ev el s  al so  pr edi ct  r esponse.

Or chi ectom y   l eads  di r ectl y   to  l ow   testoster one  l ev el s.   Lutei ni zi ng  hor m one­
r el easi ng  hor m one  agoni sts  (l eupr ol i de  and  goser el i n)  br i ng  about  suppr essed
testoster one  l ev el s  i n  2  to  3  w eek s  and  can  be  adm i ni ster ed  conv eni entl y   at
m onthl y   i nter v al s  or   l onger   as  a  subcutaneous  depot  pr epar ati on.   Testoster one
r eceptor   l ev el   antagoni sm   can  be  achi ev ed  by   the  m or e  r ecentl y   i ntr oduced
nonster oi dal   agent,   f l utam i de.

Choosi ng  am ong  the  com m onl y   used  i ni ti al   hor m onal   appr oaches  (i . e. ,
l eupr ol i de  or   f l utam i de)  of ten  depends  on  i ndi v i dual   pati ent  pr ef er ence  w i th
r espect  to  tox i ci ti es  and  cost.

The  com bi nati on  of   a  l utei ni zi ng  hor m one­r el easi ng  hor m one  agoni st  and  an
anti andr ogen  m ay   br i ng  about  an  i ncr eased  r esponse  r ate  i n  pati ents  w i th  l ow ­
v ol um e  stage  D1  pr ostate  cancer .   How ev er ,   i n  gener al ,   no  one  hor m onal
tr eatm ent  has  been  concl usi v el y   pr ov ed  to  be  super i or   i n  ter m s  of   r esponse.
Appr ox i m atel y   10%  of   pati ents  ex per i ence  com pl ete  or   par ti al   r esponses  and,
i n  another   20%,   the  di sease  r em ai ns  stabl e  f or   at  l east  3  m onths.   The
tr eatm ent  of   m etastati c  pr ostate  cancer   doses  pr ol ong  sur v i v al   but  ther e  ar e
no  def i ni ti v e  cur es.   If   i ni ti al   hor m onal   m ani pul ati on  f ai l s,   a  second  hor m onal
tr eatm ent  achi ev es  a  r esponse  i n  onl y   10%  to  20%  of   pati ents.

Chem other apy   achi ev es  r esponses  (usual l y   par ti al )  i n  40%  of   pati ents;  i n
addi ti on,   i t  can  of ten  pr oduce  tox i ci ty   but  does  pr ol ong  sur v i v al   i n  ter m s  of
m onths.   Pati ents  w ho  f ai l   i ni ti al   ther apy   ar e  usual l y   of f er ed  pal l i ati v e  or
ex per i m ental   chem other apy   appr oaches.   Som e  ev i dence  now   ex i sts  f or   the
ef f i cacy   of   sur am i n,   w hi ch  bl ock s  gr ow th  f actor   ef f ects  i n  pr ostate  cancer .

14.   Is  the  pr ostate­speci f i c  anti gen  l ev el   usef ul   i n  scr eeni ng  asy m ptom ati c  m en  f or
pr ostate  cancer ?

An  el ev ated  pr ostate­speci f i c  anti gen  v al ue  i s  usef ul   i n  the  detecti on  of   ear l y
pr ostate  cancer   i n  asy m ptom ati c  m en.   How ev er ,   because  ear l y   pr ostate  cancer
i s  of ten  not  associ ated  w i th  cl i ni cal   di sease,   studi es  ex am i ni ng  the  uti l i ty   of
m ass  pr ostate­speci f i c  anti gen  scr eeni ng  ar e  sti l l   under   w ay   to  deter m i ne  i f
thi s  scr eeni ng  ul ti m atel y   i m pr ov es  sur v i v al .   Most  cl i ni ci ans  agr ee  that  a  pr oper
pr ostate  ex am i nati on  on  r ectal   ev al uati on  f or   m en  ol der   than  50  y ear s  i s  a
cost­ef f ecti v e  and  potenti al l y   hel pf ul   scr eeni ng  m ethod.

Case
A  65­y ear ­ol d  m an  pr esents  to  the  em er gency   r oom   because  of   a  back ache  that  has
l asted  f or   sev er al   day s,   w hi ch  becam e  sev er e  af ter   a  f al l .   Si x   y ear s  bef or e,   the
pati ent
P. 335
began  to  hav e  ur i nar y   f r equency   and  dr i bbl i ng  on  m i ctur i ti on,   and  w as  noted  to
hav e  a  har d,   nontender   pr ostate  nodul e  w i th  obl i ter ati on  of   the  l ater al   sul cus  and  a
pal pabl e  sem i nal   v esi cl e  on  one  si de,   f or   w hi ch  he  r ecei v ed  ex ter nal ­beam
i r r adi ati on.   He  has  been  on  l eupr ol i de  i njecti ons  si nce  that  ti m e.   Hi s  pr ostate­
speci f i c  anti gen  l ev el   w as  sl i ghtl y   el ev ated  bef or e  ther apy   but  r etur ned  to  nor m al
ther eaf ter ,   and  w as  nor m al   4  m onths  ago.
Phy si cal   ex am i nati on  r ev eal s  a  di f f use  tender ness  ov er   the  l ow er   thor aci c  and
l um bar   spi nes.   Rectal   ex am i nati on  r ev eal s  a  har d,   i r r egul ar   pr ostate.   The
r em ai nder   of   the  phy si cal   ex am i nati on  f i ndi ngs  ar e  unr em ar k abl e.   Labor ator y
w or k up  show s  the  f ol l ow i ng:  hem ogl obi n,   11. 5  g/dL;  hem atocr i t,   32%;  w hi te  bl ood
cel l   count,   9. 8  ×  10 9 /L;  pl atel ets,   162  ×  10 9 /L;  w hi te  bl ood  cel l   di f f er enti al —
neutr ophi l s,   68%;  l y m phocy tes,   26%;  m onocy tes,   4%;  band  f or m s,   2%;  er y thr ocy te
sedi m entati on  r ate,   87  m m   i n  the  f i r st  hour ;  and  al k al i ne  phosphatase,   320  U
(nor m al ,   up  to  150  U ).   Tw o  nucl eated  r ed  bl ood  cel l s  ar e  seen  per   100  w hi te  bl ood
cel l s.
A  chest  r adi ogr aphi c  study   show s  i ncr eased  bone  densi ti es  i n  sev er al   r i bs,   and  a
r adi ogr aph  of   the  l um bar   spi ne  show s  m ul ti pl e  ar eas  of   bone  scl er osi s  but  no
f r actur es.   A  bone  scan  show s  m ul ti pl e  ar eas  of   i ncr eased  acti v i ty   scatter ed  ov er   the
ax i al   sk el eton.

1.   What  w as  the  stage  of   thi s  pati ent's  pr ostate  cancer   3  y ear s  ago?
2.   Was  the  tr eatm ent  gi v en  at  that  ti m e  appr opr i ate?
3.   How   w oul d  y ou  go  about  pr ov i ng  a  di agnosi s  of   pr ostate  cancer   now ?
4.   What  ar e  the  tr eatm ent  opti ons  and  pr ognosi s  i n  thi s  pati ent  now ?
5.   What  ar e  the  com pl i cati ons  of   ex ter nal ­beam   i r r adi ati on  f or   the  tr eatm ent  of
stage  C  pr ostate  cancer ?

Case Discussion
1.   What  w as  the  stage  of   thi s  pati ent's  pr ostate  cancer   3  y ear s  ago?

The  pati ent  had  stage  C  pr ostate  cancer   6  y ear s  ago  based  on  pal pabl e
ex tensi on  of   the  tum or   acr oss  the  pr ostati c  capsul e  and  a  negati v e  bone  scan.
A  pal pabl e  sem i nal   v esi cl e  i s  abnor m al .

2.   Was  the  tr eatm ent  gi v en  at  that  ti m e  appr opr i ate?

The  tr eatm ent  f or   stage  C  pr ostate  cancer   i s  ex ter nal ­beam   i r r adi ati on,   usual l y
w i th  i nter nal   r adi ati on  or ,   i n  sel ected  i nstances,   r adi cal   pr ostatectom y .
Adjuv ant  hor m onal   tr eatm ent  does  pr ol ong  ti m e  to  r ecur r ence  and  pr obabl y
i m pr ov es  sur v i v al .

3.   How   w oul d  y ou  go  about  pr ov i ng  a  di agnosi s  of   pr ostate  cancer   now ?

The  pr esence  of   i m m atur e  w hi te  and  r ed  bl ood  cel l s  i n  the  ci r cul ati on  together
w i th  the  anem i a  (a  l euk oer y thr obl asti c  pi ctur e)  suggests  the  ex i stence  of   bone
m ar r ow   i nf i l tr ati on.   A  bone  m ar r ow   bi opsy   and  stai ni ng  f or   aci d  phosphatase
and  pr ostate­speci f i c  anti gen  m i ght  cl i nch  the  di agnosi s.   An  el ev ated  ser um
pr ostate­speci f i c  anti gen  l ev el   i n  the  pr esence  of   scl er oti c  bone  l esi ons
establ i shes  the  di agnosi s  of   m etastati c  pr ostate  cancer .

4.   What  ar e  the  tr eatm ent  opti ons  and  pr ognosi s  i n  thi s  pati ent  now ?

The  tr eatm ent  opti on  f or   m etastati c  pr ostate  cancer   w i th  pr edom i nant  bone
l esi ons  i s  to  change  to  hor m onal   ther apy .   Radi ati on  ther apy   can  be  used  f or
the  m anagem ent  of   pai nf ul   bone  l esi ons  not  am enabl e  to  andr ogen  depr i v ati on.
A
P. 336
change  i n  andr ogen  depr i v ati on  ther apy   can  be  ef f ected  by   bl ock i ng  andr ogen
r eceptor   f uncti on.   Thi s  change  gi v es  a  m edi an  ti m e  to  f ur ther   pr ogr essi on  of
di sease  of   appr ox i m atel y   15  m onths  and  a  m edi an  sur v i v al   of   30  m onths.
Bi l ater al   or chi ectom y   i s  an  opti on  but  m ay   not  be  acceptabl e  to  som e  pati ents
f or   psy chol ogi cal   r easons.

5.   What  ar e  the  com pl i cati ons  of   ex ter nal ­beam   i r r adi ati on  f or   the  tr eatm ent  of
stage  C  pr ostate  cancer ?

The  com m on  com pl i cati ons  of   ex ter nal ­beam   i r r adi ati on  ar e  i m potence,
supr apubi c  or   per i neal   edem a,   pr octi ti s,   per si stent  tum or ,   and  f i br osi s  of   the
pr ostate,   m ak i ng  i t  har d  and  i ts  substance  cl i ni cal l y   i ndi sti ngui shabl e  f r om   that
of   m al i gnancy .

Suggested Readings
Al ber tsen  PC,   Fr y back   DG,   Stor er   BE,   et  al .   Long­ter m   sur v i v al   am ong  m en  w i th
conser v ati v el y   tr eated  l ocal i zed  pr ostate  cancer .   JAMA  1995;274:626.

Car v al hal   GF,   Sm i th  DS,   Mager   DE,   et  al .   Di gi tal   r ectal   ex am i nati on  f or
detecti ng  pr ostate  cancer   at  pr ostate  speci f i c  anti gen  l ev el s  of   4  ng/m L  or   l ess.
J  U r ol   1999;161:835.

Cr aw f or d  ED,   Ei senber ger   MA,   McLeod  DG,   et  al .   A  contr ol l ed  tr i al   of   l eupr ol i de
w i th  and  w i thout  f l utam i de  i n  pr ostati c  car ci nom a.   N   Engl   J  Med  1989;321:419.

Geor ge  N JR.   N atur al   hi stor y   of   l ocal i zed  pr ostati c  cancer   m anaged  by
conser v ati v e  ther apy   al one.   Lancet  1988;1:494.

Ger ber   GS,   Chodak   GW.   Routi ne  scr eeni ng  f or   cancer   of   the  pr ostate.   J  N atl
Cancer   Inst  1991;83:329.

Gl eason  DF.   Hi stol ogi c  gr ade,   cl i ni cal   stage,   and  pati ent  age  i n  pr ostate  cancer .
N CI  Monogr   1988;7:15.

Gw ede  CK,   Pow ­Sang  J,   Sei gne  J,   et  al .   Tr eatm ent  deci si on­m ak i ng  str ategi es
and  i nf l uences  i n  pati ents  w i th  l ocal i zed  pr ostate  car ci nom a.   Cancer
2005;104:1381.
Editors :  Sc hrie r,  Robe rt W .
Title :  Inte rna l Me dic ine  Ca s e book ,  The : Re a l P a tie nts ,  Re a l Ans w e rs ,
3rd Edition
Copy r i ght  ©2007  Li ppi ncott  Wi l l i am s  &  Wi l k i ns

>  T a b le   o f   C o nte nts   >  C ha p te r   8  ­  P ulm o no lo g y

Chapter 8
Pulmonology

Ma rvin I.  Sc hw a rz

Acute Respiratory Distress Syndrome
1.   What  i s  the  def i ni ti on  of   acute  r espi r ator y   di str ess  sy ndr om e  (ARDS)?

2.   What  ar e  the  pr i nci pl es  of   m anagem ent  of   the  pati ent  w i th  ARDS?

Discussion
1.   What  i s  the  def i ni ti on  of   ARDS?

Many   acute  m edi cal   condi ti ons  such  as  congesti v e  hear t  f ai l ur e,
pneum oni a,   or   the  acute  noni nf ecti ous  i nter sti ti al   pneum oni as  can
m i m i c  the  cl i ni cal
P. 338
pi ctur e  of   ARDS.   A  usef ul   def i ni ti on  i ncl udes  the  f ol l ow i ng:  r el ati v el y
acute  appear ance  of   di f f use  pul m onar y   i nf i l tr ates,   pr of ound  hy pox em i a
(usual l y   r equi r i ng  m echani cal   v enti l ati on),   pul m onar y   com pl i ance  l ess
than  20  m L/cm   H 2 O  (sti f f   l ungs),   and  a  pul m onar y   capi l l ar y   w edge
pr essur e  l ess  than  18  m m   Hg  (noncar di ogeni c  edem a).   Causes  i ncl ude
bacter i al ,   v i r al ,   and  pr otozoan  pneum oni as,   al l   f or m s  of   shock ,
aspi r ati on,   hi gh­al ti tude  and  neur ogeni c  pul m onar y   edem a,   tr ansf usi on­
r el ated  acute  l ung  i njur y   (TRALI)  w hi ch  i s  of ten  postsur gi cal ,   tr aum a,
and  bur ns.   The  under l y i ng  hi stol ogi c  appear ance  i s  di f f use  al v eol ar
dam age.   Thi s  consi sts  of   tw o  phases:  the  ear l y   ex udati v e  phase
dem onstr ati ng  al v eol ar   edem a  and  i ntr aal v eol ar   f i br i n  col l ecti v e  k now n
as  hy al i ne  m em br anes  and  the  f i br opr ol i f er ati v e  phase  consi sti ng  of
f i br obl asti c  pr ol i f er ati v e  and  col l agen  deposi ti on.

2.   What  ar e  the  pr i nci pl es  of   m anagem ent  i n  the  pati ent  w i th  ARDS?

Ther e  ar e  f ew   speci f i c  m edi cal   ther api es  f or   ARDS  other   than  the
tr eatm ent  of   the  under l y i ng  cause.   Tr i al s  of   bi ol ogi c  m odi f i er s  hav e
been  gener al l y   di sappoi nti ng,   ex cept  i n  sev er e  sepsi s  i n  w hi ch  hum an
r ecom bi nant  acti v ated  pr otei n  C  (acti v ated  dr otr ecogi n  α )  r educes
m or tal i ty   r ates.   Managem ent  i s  m ai nl y   suppor ti v e  and  tr eati ng  the
under l y i ng  i ni ti ati ng  cause  i n  the  hope  that  the  l ung  can  r etur n  to
nor m al .   The  speci f i c  goal s  of   ther apy   ar e  to  m ai ntai n  ti ssue
ox y genati on  (m ax i m i ze  ox y gen  del i v er y )  w hi l e  pr ev enti ng
com pl i cati ons  r esul ti ng  f r om   m echani cal   v enti l ati on,   such  as
bar otr aum a  (pneum othor ax ),   and  l ung  i njur y   stem m i ng  f r om   hi gh
ai r w ay   pr essur es  or   ox y gen  tox i ci ty .   A  r ecent  N ati onal   Insti tutes  of
Heal th  (N IH)­suppor ted  study   of   m echani cal l y   v enti l ated  ARDS  subjects
i ndi cated  a  l ow er   m or tal i ty   f or   those  v enti l ated  w i th  ti dal   v ol um es  of   6
m L/k g  v er sus  the  standar d  12  m L/k g.   Ther e  i s  ev i dence  to  suppor t  that
hi gher   r espi r ator y   pr essur es  aggr av ate  the  l ung  i njur y .

Case
A  27­y ear ­ol d  w hi te  m al e  m otor cy cl i st  i s  tr anspor ted  to  the  em er gency   r oom
af ter   he  w as  i nv ol v ed  i n  a  hi gh­speed,   head­on  col l i si on  w i th  an  oncom i ng
autom obi l e.   At  the  acci dent  scene,   he  w as  poor l y   r esponsi v e;  hi s  i ni ti al
bl ood  pr essur e  w as  70/40  m m   Hg  and  i njur i es  i ncl uded  a  f l ai l   chest  on  the
r i ght,   sev er al   pel v i c  f r actur es,   an  open  f r actur e  of   the  f em ur ,   and  a  cl osed,
di spl aced  f r actur e  of   the  l ef t  ti bi a.   A  centr al   and  tw o  per i pher al   catheter s
ar e  i nser ted,   and  nor m al   sal i ne  i s  adm i ni ster ed  at  m ax i m al   r ates.   Hi s  bl ood
i s  ty ped  and  cr ossm atched.   He  i s  al so  i ntubated  i n  the  em er gency   r oom ,   and
a  chest  tube  i nser ted  on  the  r i ght  y i el ds  a  bl oody   r etur n.   Sucti on  i s  appl i ed
and  no  ai r   l eak   i s  noted.   Sev er al   uni ts  of   bl ood  ar e  adm i ni ster ed,   and
abdom i nal   l av age  f l ui d  pr ov es  bl oody .   He  i s  r ushed  to  the  oper ati ng  r oom   to
under go  a  l apar otom y ,   and  a  l i v er   l acer ati on  i s  f ound  and  r epai r ed.   Dur i ng
sur ger y   he  r ecei v es  8  uni ts  of   w hol e  bl ood,   5  uni ts  of   pl atel ets,   and  8  uni ts
of   f r esh  f r ozen  pl asm a.   Hi s  or thopaedi c  i njur i es  ar e  appr opr i atel y   tr eated
and  he  i s  tr ansf er r ed  to  the  sur gi cal   i ntensi v e  car e  uni t  i n  cr i ti cal   but  stabl e
condi ti on.   Chest  r adi ogr aphi c  study   conf i r m s  that  the  endotr acheal   and  chest
tubes  ar e  i n  good  posi ti on  and  r ev eal s  a  r i ght  l ow er   l obe  i nf i l tr ate  thought
to  be  secondar y   to  a  pul m onar y   contusi on.   The  v enti l ator   i s  i ni ti al l y   set  at
an  i nspi r ed  ox y gen  concentr ati on  (FIO 2 )  of   40%,   r espi r ator y
P. 339
r ate  of   12  per   m i nute,   and  ti dal   v ol um e  of   90  m L  i n  an  assi st­contr ol   m ode.
Ar ter i al   bl ood  gas  m easur em ent  r ev eal s  a  pH  of   7. 46,   a  par ti al   pr essur e  of
car bon  di ox i de  (PCO 2 )  of   3  m m   Hg,   and  a  par ti al   pr essur e  of   ox y gen  (PO 2 )
of   59  m m   Hg,   w i th  an  ox y gen  satur ati on  of   92%.   On  the  second  hospi tal
day ,   12  hour s  af ter   adm i ssi on,   he  becom es  agi tated  w hi l e  on  the  v enti l ator ,
hi s  r espi r ator y   r ate  r i ses  to  25  per   m i nute,   hi s  m i nute  v enti l ati on  i ncr eases
f r om   8. 5  to  18  L  per   m i nute,   and  ai r w ay   pr essur e  r i ses  f r om   20  to  60  cm
H 2 O.   A  r epeat  chest  r adi ogr aph  now   show s  di f f use  ai r space  patter n.   Repeat
ar ter i al   bl ood  gas  anal y si s  r ev eal s  a  PO 2   of   39  m m   Hg.

1.   What  i s  the  di f f er enti al   di agnosi s  of   thi s  pati ent's  cl i ni cal


deter i or ati on?
2.   What  ar e  the  r i sk   f actor s  f or   ARDS  i n  thi s  pati ent?
3.   How   w oul d  y ou  m anage  thi s  pati ent's  hy pox em i a?
4.   What  ar e  the  potenti al   pr obl em s  associ ated  w i th  posi ti v e  end­
ex pi r ator y   pr essur e  (PEEP)?
5.   What  i s  the  m or tal i ty   r ate  associ ated  w i th  ARDS?

Case Discussion
1.   What  i s  the  di f f er enti al   di agnosi s  of   thi s  pati ent's  cl i ni cal   deter i or ati on?

Sev er al   possi bi l i ti es  need  to  be  consi der ed  i n  thi s  setti ng.   Fi r st,   an
i nf ecti on  (pneum oni a)  m ust  al w ay s  be  r ul ed  out.   It  i s  possi bl e  that  the
pati ent  aspi r ated  gastr i c  contents  at  the  acci dent  scene  w hi l e  hi s  l ev el
of   consci ousness  w as  i m pai r ed,   and  thi s  coul d  hav e  i njur ed  the  l ung
di r ectl y   ow i ng  to  aci d  aspi r ati on  or   set  the  stage  f or   an  ov er w hel m i ng
pneum oni a.   Al ter nati v el y ,   a  nosocom i al   (hospi tal ­acqui r ed)  pneum oni a
coul d  hav e  been  acqui r ed  i n  the  sur gi cal   i ntensi v e  car e  uni t,   al though
the  ear l y   onset  of   hi s  ARDS  m ak es  thi s  unl i k el y .   The  m assi v e  f l ui d
r esusci tati on  coul d  l ead  to  a  f l ui d­ov er l oad  congesti v e  hear t  f ai l ur e
sy ndr om e,   ev en  despi te  a  nor m al ­f uncti oni ng  hear t  bef or e  the  acci dent.
If   a  car di ac  contusi on  occur r ed,   thi s  w oul d  m ak e  hi m   m or e  suscepti bl e
to  thi s  com pl i cati on.   Pul m onar y   contusi ons  ar e  w or th  consi der i ng,   but
ar e  usual l y   m or e  l ocal i zed  and  dev el op  w i thi n  sev er al   hour s.   Ai r w ay
hem or r hage,   per haps  stem m i ng  f r om   ei ther   br onchi al   f r actur e  or   a
tr aum ati c  i ntubati on,   can  occur ,   but  i s  ty pi cal l y   associ ated  w i th  bl oody
secr eti ons  w hen  sev er e.   The  ARDS  coul d  al so  hav e  r esul ted  f r om
pr ol onged  hy potensi on  or   r epl acem ent  of   bl ood  pr oducts  or   TRALI.
Regar dl ess,   thi s  pati ent  has  ARDS  and  under   these  ci r cum stances,
pathol ogi c  study   w oul d  show   di f f use  al v eol ar   dam age  consi sti ng  of
hy al i ne  m em br ane  f or m ati on,   al v eol ar   w al l   edem a,   and  i nf l am m ati on.

2.   What  ar e  the  r i sk   f actor s  that  put  thi s  pati ent  at  r i sk   f or   ARDS?

Thi s  pati ent's  r i sk   f actor s  ar e:  hy potensi on—usual l y   pr ol onged  and


sev er e  (sy stol i c  bl ood  pr essur e  < 90  m m   Hg);  hy per tr ansf usi on—m or e
than  10  uni ts  of   bl ood  pr oducts  i n  a  24­hour   per i od;  aspi r ati on—any
pati ent  w i th  depr essed  m ental   status  i s  at  gr eat  r i sk   f or   gastr i c
aspi r ati on  (the  r esul ti ng  chem i cal   i njur y   i s  a  com m on  pr ecur sor   f or
ARDS).   Fat  em bol i   sy ndr om e—thi s  sy ndr om e  consi sts  of   di f f use
pul m onar y   i nf i l tr ates,   m ental   status  changes,   thr om bocy topeni a,   and
conjuncti v al   or   ax i l l ar y   petechi ae.   It  occur s  m ost  of ten  i n  the  pr esence
of   sev er e  and  m ul ti pl e  l ong
P. 340
bone  f r actur es,   and  i s  thought  to  r esul t  f r om   f at  em bol i   m i gr ati ng  f r om
the  bone  m ar r ow   to  the  l ungs.   How ev er ,   i t  occur s  usual l y   24  to  72
hour s  af ter   adm i ssi on,   m ak i ng  i t  unl i k el y   i n  thi s  pati ent.

Other   r i sk   f actor s  i ncl ude  sepsi s,   pneum oni a,   dr ugs,   pancr eati ti s,   l ung
contusi on,   tox i c  f um e  i nhal ati on,   and  ox y gen  tox i ci ty .
3.   How   w oul d  y ou  m anage  thi s  pati ent's  hy pox em i a?

Acutel y ,   the  FIO 2   shoul d  be  i ncr eased  to  100%.   How ev er ,   the  pr ol onged
adm i ni str ati on  of   100%  ox y gen  f or   3  or   m or e  day s  i s  l i k el y   to  l ead  to
ox y gen  tox i ci ty   and  f ur ther   w or seni ng  of   ARDS.   Accor di ngl y ,   the  FIO 2
shoul d  be  decr eased  to  the  l ow est  l ev el   that  achi ev es  an  ox y gen
satur ati on  of   90%  to  92%.   Ther e  appear s  to  be  a  thr eshol d  of   50%  to
60%,   bel ow   w hi ch  ox y gen  tox i ci ty   i s  r ar e.   The  v enti l ator   show s  that
ti dal   v ol um e  shoul d  be  set  at  6  m L/k g.   If   an  FIO 2   abov e  thi s  r ange  i s
necessar y ,   then  a  tr i al   of   PEEP  i s  i ndi cated.   PEEP  appear s  to  i m pr ov e
ox y genati on  by   r ecr ui ti ng  col l apsed  gas­ex change  uni ts  (atel ectasi s).
Data  i ndi cate  that  pl aci ng  the  pati ent  i n  a  pr one  posi ti on  al so  i m pr ov es
ox y genati on,   but  not  necessar i l y   the  outcom e.

4.   What  ar e  the  potenti al   pr obl em s  associ ated  w i th  PEEP?

PEEP  m ay   be  l i f esav i ng  by   i m pr ov i ng  ox y genati on  and  al l ow i ng  the  FIO 2
to  be  l ow er ed  to  “saf eâ€​   l ev el s;  how ev er ,   i t  i s  associ ated  w i th
sev er al   potenti al   pr obl em s.   The  f i r st  i s  hy potensi on.   Hi gh  l ev el s  of
posi ti v e  i ntr athor aci c  pr essur e  i m pede  v enous  r etur n  to  the  hear t  and
m ay   be  tr ansm i tted  to  the  pul m onar y   ar ter i es,   causi ng  pul m onar y
hy per tensi on.   Both  these  f actor s  ser v e  to  decr ease  car di ac  output,
w hi ch  pr eci pi tates  hy potensi on.

The  r i sk   of   bar otr aum a  i s  gr eatl y   i ncr eased  w i th  PEEP  because  of   the
posi ti v e  ai r w ay   pr essur es  and  m ay   r esul t  i n  pneum othor ax   or
pneum om edi asti num .   Pneum othor ax   i n  a  v enti l ated  pati ent  i s  of ten  a
m edi cal   em er gency   because  a  tensi on  pneum othor ax   m ay   ev ol v e.   PEEP
l ev el s  abov e  15  cm   H 2 O  ar e  par ti cul ar l y   r i sk y .   The  pr ophy l acti c
adm i ni str ati on  of   PEEP,   bef or e  the  onset  of   ARDS,   has  been  show n  to
be  of   no  v al ue.

5.   What  i s  the  m or tal i ty   r ate  associ ated  w i th  ARDS?

In  1967,   the  m or tal i ty   r ate  obser v ed  f or   ARDS  w as  60%.   Thi s  has
decr eased  to  30%  to  40%,   m ai nl y   due  to  i m pr ov ed  v enti l ator y
m anagem ent  as  opposed  to  the  tr eatm ent  of   the  ARDS  i tsel f .   In  the
sur v i v or s,   pul m onar y   f uncti on  can  r etur n  to  nor m al ,   and  thi s  usual l y
occur s  by   6  m onths.   Per si stent  abnor m al i ti es  past  thi s  ti m e  i ndi cate
pul m onar y   f i br osi s  and  pul m onar y   i m pai r m ent.

Suggested Readings
Cal f ee  CS,   Matthay   MM.   Recent  adv ances  i n  m echani cal   v enti l ati on.   Am   J
Med  2005;118:584–591.

Choc  MK.   Acute  l ung  i njur y /adul t  r espi r ator y   di str ess  sy ndr om e:  the
thi r d  Pi ttsbur gh  i nter nati onal   l ung  conf er ence.   Pr oc  Am   Thor ac  Soc
2005;2:181–245.

Vi nant  JL,   Abr aham   E.   The  l ast  100  y ear s  of   sepsi s.   Am   J  Respi r   Cr i t
Car e  Med  2006;173:256–263.

P. 341

Asthma
1.   What  i s  asthm a,   and  how   i s  i t  cl assi f i ed?

2.   How   i s  asthm a  di agnosed?

3.   What  condi ti ons  ar e  associ ated  w i th  or   m ay   com pl i cate  asthm a?

Discussion
1.   What  i s  asthm a,   and  how   i s  i t  cl assi f i ed?

Asthm a  i s  not  a  si ngl e  enti ty ,   but  r ather   a  cl i ni cal   sy ndr om e  consi sti ng
of   (a)  an  i ncr ease  i n  ai r w ay   r esi stance  to  a  v ar i ety   of   sti m ul i ;  (b)
v ar i abl e  ai r f l ow   obstr ucti on,   w hi ch  i s  usual l y   r ev er si bl e,   ei ther
spontaneousl y   or   w i th  tr eatm ent;  and  (c)  a  chr oni c,   m ul ti cel l ul ar
i nf l am m ator y   r esponse  w i thi n  the  ai r w ay s  that  pr oduces  patchy
br onchi al   epi thel i al   denudati on,   subm ucosal   edem a,   hy per secr eti on  of
m ucus,   and  subbasem ent  m em br ane  col l agen  deposi ti on.   The  asthm ati c
r esponse  to  sti m ul i   m ay   be  i m m edi ate,   occur r i ng  w i thi n  m i nutes  and
ter m ed  the  ear l y   asthm ati c  r esponse,   or   del ay ed,   ar i si ng  sev er al   hour s
af ter   ex posur e  and  ter m ed  the  l ate  asthm ati c  r esponse.   The  ear l y
asthm ati c  r esponse  pr i m ar i l y   r esul ts  f r om   br onchi al   sm ooth  m uscl e
constr i cti on  and  the  l ate  asthm ati c  r esponse  i s  char acter i zed  by
i nf l am m ator y   cel l   i nf i l tr ati on  and  acti v ati on.   Both  patter ns  m ay   be
tr i gger ed  by   ex posur e  to  the  sam e  sti m ul i ,   and  m ay   w or k   i n  concer t  to
pr oduce  sustai ned  nar r ow i ng  of   the  ai r w ay   l um en.

Asthm a  m ay   be  cl assi f i ed  on  the  basi s  of   ei ther   the  pr esum pti v e
eti ol ogy   or   sy m ptom   sev er i ty   and  the  patter n  of   ai r f l ow   obstr ucti on.
Hi stor i cal l y ,   attem pts  hav e  been  m ade  to  cl assi f y   asthm ati c  subjects  as
hav i ng  ei ther   i ntr i nsi c  or   ex tr i nsi c  di sease.   Intr i nsi c  asthm ati cs  hav e  no
per sonal   or   f am i l y   hi stor y   of   al l er gi es,   thei r   i m m unogl obul i n  E  (IgE)
l ev el s  ar e  nor m al ,   and  they   hav e  no  easi l y   i denti f i abl e  env i r onm ental
pr eci pi tants  of   thei r   sy m ptom s.   In  contr ast,   ex tr i nsi c  asthm ati cs  hav e
al l er gi c  or   atopi c  hi stor i es,   thei r   IgE  l ev el s  ar e  ty pi cal l y   el ev ated,   and
they   hav e  speci f i c  anti geni c  tr i gger s  to  thei r   asthm a.   Thi s  tr adi ti onal
eti ol ogi c  cl assi f i cati on  i s  now   pr obabl y   obsol ete  because  i ndi v i dual
asthm ati c  subjects  com m onl y   ex hi bi t  both  IgE­  and  non–IgE­m edi ated
r esponses  to  br oncho  pr ov ocati v e  sti m ul i .   Ther ef or e,   a  cl assi f i cati on
schem e  that  i s  i nstead  based  on  the  sev er i ty   of   sy m ptom s  and  on  l ung
f uncti on  i s  m or e  cl i ni cal l y   r el ev ant,   and  pr ov i des  a  f r am ew or k   on  w hi ch
to  base  a  stepw i se  tr eatm ent  appr oach.   One  pr oposed  cl assi f i cati on
schem e  i s  pr esented  i n  Tabl e  8­1.

Because  the  sev er i ty   of   an  acute  asthm a  attack   m ay   be  under esti m ated
by   both  pati ents  and  thei r   f am i l i es,   pati ents  ar e  encour aged  to  use
hom e  ex pi r ator y   f l ow   r ate  dev i ces,   w hi ch  can  m or e  objecti v el y
m easur e  asthm a  sev er i ty .   Factor s  that  hav e  been  associ ated  w i th  an
i ncr eased  r i sk   of   asthm a  m or tal i ty   i ncl ude  f r equent  em er gency   r oom
v i si ts,   hospi tal i zati on  w i thi n  the  pr ev i ous  y ear ,   pr i or   l i f e­thr eateni ng
epi sodes,   a  pr ev i ous  need  f or   i ntubati on,   a  r ecent
P. 342
r educti on  i n  the  cor ti coster oi d  dosage  or   cessati on  of   use,
noncom pl i ance  w i th  m edi cal   ther apy ,   the  pr esence  of   ser i ous
depr essi on  or   psy chosoci al   behav i or al   pr obl em s,   and  a  l ow er
soci oeconom i c  status.

Table 8­1 Asthma Classification Scheme

As thma Clinic a l Fe a ture s P ulmona ry Func tion


Se ve rity

Mi l d Inter m i ttent,   br i ef Ex pi r ator y   f l ow


sy m ptom s  (< 1­2  ti m es r ates  > 80%  of
per   w eek ) pr edi cted

Rar e  noctur nal Ex pi r ator y   f l ow   r ate


  sy m ptom s  (< 2  ti m es  per v ar i abi l i ty   < 20%
w eek )

Moder ate Ex acer bati ons  (> 2  ti m es Ex pi r ator y   f l ow


per   w eek ) r ates  60%­80%  of
pr edi cted

N octur nal   sy m ptom s  (> 2 Ex pi r ator y   f l ow   r ate


 
ti m es  per   w eek ) v ar i abi l i ty   20%­30%

Sev er e Al m ost  dai l y Ex pi r ator y   f l ow   r ate


br onchodi l ator   use < 60%  of   pr edi cted

  Fr equent  conti nuous Ex pi r ator y   f l ow   r ate


sy m ptom s v ar i abi l i ty   > 30%

Fr equent  noctur nal


   
aw ak eni ngs

Phy si cal   acti v i ti es


   
l i m i ted  by   sy m ptom s

  Hospi tal i zati on  f or  


asthm a  w i thi n  the
pr ev i ous  y ear

2.   How   i s  asthm a  di agnosed?

Because  pati ents  w i th  asthm a  ar e  a  heter ogeneous  gr oup,   the  di agnosi s
r equi r es  assessm ent  of   a  pati ent's  pul m onar y   f uncti on  and  attenti on  to
sel ect  detai l s  r ev eal ed  by   the  m edi cal   hi stor y ,   phy si cal   ex am i nati on,
and  l abor ator y   tests.   Hi stor i cal   f eatur es  i m por tant  i n  establ i shi ng  the
di agnosi s  of   asthm a  i ncl ude  the  epi sodi c  and  v ar i abl e  natur e  of   the
ai r f l ow   obstr ucti on  and  the  r ev er si bi l i ty   of   the  obstr ucti on.   The  m ost
com m on  sy m ptom s—cough,   w heezi ng,   chest  ti ghtness,   shor tness  of
br eath,   and  sputum   pr oducti on—ar e  nonspeci f i c  and  by   them sel v es
nondi agnosti c.   The  patter n  of   sy m ptom s  m ay   be  suggesti v e,   i n  that
noctur nal   (and  ear l y   m or ni ng)  sy m ptom s  ar e  par ti cul ar l y   char acter i sti c
of   asthm a.   Com m onl y   r epor ted  pr eci pi tants  of   br onchospasm   i ncl ude
ex er ci se,   col d  ai r ,   env i r onm ental   al l er gens,   ex posur e  to  occupati onal
or   chem i cal   i r r i tants,   and  upper   r espi r ator y   tr act  i nf ecti ons.   The
di f f er enti al   di agnosi s  of   adul t  w heezi ng  or   cough  m ay   i ncl ude
m echani cal   obstr ucti on  of   the  ai r w ay   (e. g. ,   f or ei gn  body ,   tum or   m ass,
or   gr anul om atous  nar r ow i ng),   v ocal   cor d  dy sf uncti on,   congesti v e  hear t
f ai l ur e,   pul m onar y   em bol us,   aspi r ati on  i njur y ,   pul m onar y   eosi nophi l i a
sy ndr om es,   and  other   f or m s  of   chr oni c  obstr ucti v e  pul m onar y   di sease
(COPD)  (e. g. ,   cy sti c  f i br osi s,   chr oni c  br onchi ti s,   and  em phy sem a).

The  phy si cal   ex am i nati on  f i ndi ngs  m ay   be  ei ther   unr em ar k abl e  or
suggest  the  pr esence  of   ai r   tr appi ng  and  hy per i nf l ati on,   w i th  an
i ncr eased  anter oposter i or   thor aci c  di am eter   and  a  l ow   di aphr agm .
Wheezi ng  i s  the  m ost  char acter i sti c  br eath  sound  of   asthm a  but  i s  an
unr el i abl e  i ndi cator   of   sev er i ty .   Br onchospasm   m ay   pr oduce  a
pr ol onged  ex pi r ator y   phase  w i th  r educed  ti dal   v ol um es  and  m i ni m al   ai r
m ov em ent.   In  thi s  setti ng,   f ai nt  w heezi ng  par adox i cal l y   i ntensi f i es  as
ai r f l ow   i m pr ov es.   Rhonchi   and  other   adv enti ti ous
P. 343
sounds  m ay   suggest  the  pr esence  of   secr eti ons  i n  the  ai r w ay s.   Si gns  of
sev er e  ai r f l ow   obstr ucti on  m ay   i ncl ude  an  i ncr eased  pul sus  par adox us,
supr acl av i cul ar   r etr acti ons  w i th  accessor y   m uscl e  use
(ster nocl ei dom astoi d  and  i nter costal s),   and  thor acoabdom i nal   par adox
(the  par adox i cal   r etr acti on  of   abdom i nal   m uscul atur e  w i th  i nspi r ati on).

Pul m onar y   f uncti on  testi ng  shoul d  be  pur sued  i n  al l   pati ents  w i th
suspected  asthm a.   Spi r om etr i c  f i ndi ngs  of   r educed  ex pi r ator y   f l ow
r ates  w i th  a  nor m al   i nspi r ator y   f l ow –v ol um e  cur v e,   l ung  v ol um es
suggesti ng  i ncr eased  thor aci c  gas  and  r esi dual   v ol um es,   and  i ncr eased
ai r w ay   r esi stance  ar e  al l   char acter i sti c  si gns  of   asthm a  and  m ay   be
al l ev i ated  by   br onchodi l ator   tr eatm ent.   Af ter   an  acute  ex acer bati on  of
asthm a,   how ev er ,   pul m onar y   f uncti on  m ay   r em ai n  abnor m al   l ong  af ter
the  sy m ptom s  hav e  r etur ned  to  thei r   basel i ne  status.

Addi ti onal   studi es  and  si gns  that  m ay   be  usef ul   i n  the  ev al uati on  of
asthm a  i ncl ude  (a)  br onchopr ov ocati on  testi ng  w i th  m ethachol i ne,
hi stam i ne,   or   ex er ci se  to  docum ent  i ncr eased  ai r w ay   r esponsi v eness  to
sti m ul i ;  (b)  per i pher al   eosi nophi l i a;  (c)  i ncr eased  IgE  l ev el s;  (d)
Char cot­Ley den  cr y stal s  (cr y stal l i zed  cati oni c  pr otei ns),   eosi nophi l s,   or
Cur schm ann's  spi r al s  (br onchi ol ar   casts  of   m ucus  and  cel l ul ar   debr i s)
i n  the  sputum ;  and  (e)  a  chest  r adi ogr aph  show i ng  hy per i nf l ati on  or   the
pr esence  of   bar otr aum a.   N o  si ngl e  test  or   batter y   of   tests  i s
appr opr i ate  f or   ev er y   suspected  case.   Sel ected  studi es  m ay   pr ov i de  the
objecti v e  ev i dence  needed  to  conf i r m   the  di agnosi s  of   asthm a  w hen  the
hi stor y   and  phy si cal   ex am i nati on  f i ndi ngs  ar e  onl y   suggesti v e.

3.   What  condi ti ons  ar e  associ ated  w i th  or   m ay   com pl i cate  asthm a?

Sev er al   condi ti ons  m ay   com pl i cate  the  asthm a  sy ndr om e,   and  they
r equi r e  speci al   consi der ati on.

Al though  a  per son's  cl i ni cal   cour se  i s  not  pr edi ctabl e,   unstabl e  asthm a
dev el ops  dur i ng  pre gna nc y  i n  appr ox i m atel y   one  thi r d  of   asthm ati c
w om en,   one  thi r d  ex per i ence  no  change,   and  sy m ptom s  ar e  actual l y
l ess  sev er e  i n  one  thi r d.   Poor l y   contr ol l ed  asthm a  dur i ng  pr egnancy
m ay   contr i bute  to  pr enatal   m or tal i ty ,   an  i ncr eased  l i k el i hood  of
pr em atur i ty ,   and  l ow   bi r th  w ei ght.   Ther ef or e,   usi ng  m edi cati ons  to
obtai n  opti m al   contr ol   of   asthm a  i s  appr opr i ate,   ev en  i f   thei r   saf ety   i n
pr egnancy   has  not  been  unequi v ocal l y   pr ov ed.   An  i nhal ed  cor ti coster oi d
pr epar ati on,   sel ecti v e  β 2   agoni sts,   appr opr i atel y   m oni tor ed
theophy l l i ne  use,   and  ev en  sy stem i c  cor ti coster oi ds  can  be  used  w hen
necessar y   to  pr ev ent  f etal   hy pox i a.   Medi cati ons  that  shoul d  be  av oi ded
i ncl ude  α ­adr ener gi c  com pounds,   br om pheni r am i ne,   epi nephr i ne,   and
som e  decongestants  (or al   α   agoni sts),   anti bi oti cs  (tetr acy cl i ne  and
ci pr of l ox aci n),   and  l i v e  v i r us  v acci nes.

The  l i k el i hood  of   asthm a­r el ated  pos tope ra tive  c omplic a tions
depends  on  the  sev er i ty   of   the  pati ent's  ai r w ay   hy per r esponsi v eness,
the  degr ee  of   ai r f l ow   obstr ucti on,   and  the  am ount  of   ex cess  ai r w ay
secr eti ons  at  the  ti m e  of   sur ger y .   In  addi ti on,   endotr acheal   i ntubati on
and  the  ty pe  of   pr ocedur e  per f or m ed  (thor aci c  and  upper   abdom i nal )
m ay   pose  an  addi ti onal   r i sk .   Pr eoper ati v e  cor ti coster oi ds  m ay   be
i ndi cated  i f   ex pi r ator y   f l ow   r ates  ar e  r educed  (< 80%  of   per sonal   best)
or   i f   cor ti coster oi ds  hav e  been  r equi r ed  to  contr ol   asthm a  i n  the
pr ev i ous  6  m onths.

P. 344
Mai ntenance  of   nasal   patency   m ay   i m pr ov e  l ow er   ai r w ay   f uncti on  and
asthm a  contr ol .   Al though  the  m echani sm s  i nv ol v ed  i n  thi s  r el ati onshi p
ar e  not  com pl etel y   under stood,   na s a l obs truc tion,   such  as  that  caused
by   r hi ni ti s,   si nusi ti s,   and  nasal   pol y ps,   m ay   l ead  to  asthm a  i nstabi l i ty
and  w or seni ng  of   sy m ptom s.   N asal   β 2   agoni sts  and  cor ti coster oi ds  ar e
som eti m es  usef ul   i n  tr eati ng  nasal   obstr ucti on.

Appr ox i m atel y   2%  of   al l   cases  of   asthm a  ar e  due  to  oc c upa tiona l


e x pos ure   to  speci f i c  sensi ti zi ng  substances.   Pr otei ns,   or gani c
com pounds,   and  som e  i nor gani c  chem i cal s  (m etal   sal ts)  hav e  been
i m pl i cated.   Once  the  di agnosi s  i s  establ i shed,   com pl ete  av oi dance  of
ex posur e  i s  m andator y   because  conti nued  ex posur e  to  ev en  m i nute
concentr ati ons  m ay   pr ov ok e  sev er e  and  potenti al l y   f atal   br onchospasm .
Al so,   once  w el l   establ i shed,   occupati onal   asthm a  m ay   not  be
com pl etel y   r ev er si bl e.   The  phar m acol ogi c  ther apy   used  f or   thi s  ty pe  of
asthm a  i s  si m i l ar   to  that  used  f or   other   f or m s  of   asthm a,   but  i s  no
substi tute  f or   di l i gent  av oi dance  of   ex posur e  to  the  of f endi ng  agents.

Che mic a l s e ns itivity  m ay   al so  pr ov ok e  asthm a  attack s.   Appr ox i m atel y


5%  to  20%  of   adul ts  w i th  asthm a  sustai n  sev er e  and  potenti al l y   f atal
ex acer bati ons  of   asthm a  af ter   tak i ng  aspi r i n  or   other   nonster oi dal
anti i nf l am m ator y   dr ugs  (tr i ad  asthm a).   Phy si cal   ex am i nati on  i n  these
pati ents  m ay   r ev eal   nasal   pol y ps,   and  sy m ptom s  of   v asom otor   r hi ni ti s
m ay   pr ecede  the  dev el opm ent  of   aspi r i n­i nduced  br onchospasm .   Less
com m onl y ,   sul f i tes,   w hi ch  m ay   be  used  as  a  f ood  pr eser v ati v e,   and
tar tr azi ne,   a  y el l ow   dy e  that  m ay   be  used  as  a  f ood  col or i ng,   hav e
been  l i nk ed  to  the  occur r ence  of   acute  br onchospasm .

Al though  ga s troe s opha ge a l re flux   i s  m or e  com m on  i n  peopl e  w i th


asthm a,   i ts  r el ati onshi p  to  br onchospasm   i s  contr ov er si al .   Most  peopl e
w i th  asthm a  and  sy m ptom ati c  gastr oesophageal   r ef l ux   hav e  hi atal
her ni as,   and  the  associ ati on  betw een  the  tw o  condi ti ons  m ay   be  best
dem onstr ated  by   si m ul taneousl y   m oni tor i ng  the  esophageal   pH  and
pul m onar y   f uncti on.   Medi cal   m anagem ent  consi sti ng  of   pr oton  pum p
i nhi bi tor s  i s  usual l y   ef f ecti v e  i n  these  pati ents.

Case
A  26­y ear ­ol d  w om an  pr esents  to  the  em er gency   r oom   at  3:00  a. m .
com pl ai ni ng  of   w or seni ng  cough  w i th  y el l ow ­gr een  sputum ,   shor tness  of
br eath,   and  w heezi ng  of   5  day s'  dur ati on.   Her   sy m ptom s  began  af ter   an
upper   r espi r ator y   tr act  i nf ecti on  that  w as  m ani f ested  as  a  l ow ­gr ade  f ev er ,
r hi nor r hea  w i th  postnasal   dr i p,   and  nasal   congesti on.   She  r epor ts  poor   sl eep
qual i ty   f or   the  l ast  2  day s  because  of   sev er e  coughi ng  and  has  used  ov er ­
the­counter   nasal   spr ay s  and  cough  suppr essants,   but  w i thout  r el i ef .   She  i s
18  w eek s  pr egnant,   but  has  no  si gni f i cant  past  m edi cal   hi stor y .   Her   phy si cal
ex am i nati on  r ev eal s  that  she  i s  di aphor eti c  and  unabl e  to  speak   i n
sentences.   Her   v i tal   si gns  r ev eal   a  r espi r ator y   r ate  of   30  br eaths  per
m i nute,   a  hear t  r ate  of   120  beats  per   m i nute,   a  tem per atur e  of   37°C
(98. 6°F),   and  a  pul sus  par adox us  of   22  m m   Hg.   Spi r om etr y   i s  attem pted
but  pr ov es  poor l y   r epr oduci bl e,   w i th  a  “best  ef f or tâ€​   f or ced  ex pi r ator y
v ol um e  i n  1  m i nute  (FEV 1 )  of   30%  of   pr edi cted.   The  r em ai nder   of   her
ex am i nati on  f i ndi ngs  ar e  notew or thy   f or   the  pr esence  of   supr acl av i cul ar
r etr acti ons  w i th  i nspi r ati on,   di f f usel y   di m i ni shed  br eath  sounds  w i th
P. 345
scatter ed,   hi gh­pi tched  i nspi r ator y   and  ex pi r ator y   w heezes,   and  a  pal pabl e
subcutaneous  cr epi tati on  ov er   her   anter i or   thor ax .   She  i s  qui te  anx i ous,   but
al er t  and  cooper ati v e.

1.   What  addi ti onal   studi es  m ay   be  i m por tant  f or   the  pr oper   m anagem ent
of   thi s  pati ent?
2.   What  ar e  the  i ni ti al   m anagem ent  consi der ati ons  i n  thi s  pati ent?
3.   What  ar e  the  tr eatm ent  consi der ati ons  f or   ongoi ng  m anagem ent  i n  thi s
pati ent?

Case Discussion
1.   What  addi ti onal   studi es  m ay   be  i m por tant  f or   the  pr oper   m anagem ent
of   thi s  pati ent?

The  pati ent's  cl i ni cal   pr esentati on  suggests  acute,   sev er e


br onchospasm ,   and  the  i m m edi ate  f ocus  of   the  em er gency   r oom   ef f or t
shoul d  be  ther apeuti c  r ather   than  di agnosti c.   Al though  thi s  i s  the  i ni ti al
epi sode  of   asthm a  f or   thi s  pati ent,   num er ous  f actor s  suggest  i t  i s  a
danger ousl y   sev er e  attack .   Dy spnea  at  r est,   an  i nabi l i ty   to  speak ,   and
the  use  of   accessor y   m uscl es  ar e  i m por tant  obser v ati ons.   Objecti v e
m easur es  of   attack   sev er i ty   ar e  an  i ncr eased  pul sus  par adox us  and
ex pi r ator y   f l ow   r ates  l ess  than  40%  of   pr edi cted.   The  i ntensi ty   of
w heezi ng  i s  an  unr el i abl e  i ndi cator .   The  pr esence  of   subcutaneous
em phy sem a  suggests  an  associ ated  pneum othor ax   or
pneum om edi asti num .   On  the  basi s  of   thi s  pr esentati on,   chest
r adi ogr aphy   and  ar ter i al   bl ood  gas  m easur em ent  ar e  i ndi cated,
al though  tr eatm ent  shoul d  not  be  del ay ed  to  do  these.   The  chest
r adi ogr aphi c  f i ndi ngs  m ay   ex cl ude  the  di agnosi s  of   pneum oni a  and
del i neate  the  sour ce  of   the  subcutaneous  em phy sem a.   A
pneum om edi asti num   can  ty pi cal l y   be  w atched  w i thout  speci f i c  ther apy ,
w her eas  a  pneum othor ax   w oul d  l i k el y   r equi r e  i nser ti on  of   a  chest  tube
w i th  w ater ­seal   sucti on  to  br i ng  about  r eex pansi on.   The  ar ter i al   bl ood
gas  studi es  w oul d  l i k el y   show   hy pox em i a  w i th  hy pocapni a.   Hy pox em i a
w i th  an  el ev ated  al v eol ar ­ar ter i al   ox y gen  gr adi ent  i s  the  r esul t  of
m i sm atched  v enti l ati on  and  per f usi on.   Acute  br onchospasm   r esul ts  i n
hy per v enti l ati on,   and  the  ar ter i al   bl ood  chem i str y   data  shoul d  r ef l ect  a
r espi r ator y   al k al osi s  w i th  a  r educed  PaCO 2 .   If   the  attack   i s  sev er e  and
pr ol onged,   the  PaCO 2   m ay   r i se  as  a  r esul t  of   i ncr eased  dead  space
v enti l ati on  (hi gh  v enti l ati on–per f usi on  r ati o)  and  r espi r ator y   m uscl e
f ati gue.   A  nor m al   or   el ev ated  PaCO 2   i n  the  setti ng  of   sev er e  ai r w ay
obstr ucti on  suggests  i m pendi ng  r espi r ator y   f ai l ur e  and  w ar r ants
i ntensi v e  car e  uni t  obser v ati on,   w i th  consi der ati on  gi v en  to  m echani cal
v enti l ati on.

2.   What  ar e  the  i ni ti al   m anagem ent  consi der ati ons  i n  thi s  pati ent?

The  i m m edi ate  goal s  of   ther apy   ar e  to  ensur e  adequate  ox y genati on
and  gas  ex change  w hi l e  r educi ng  the  br onchospasm   and  the  w or k   of
br eathi ng.   In  thi s  case,   the  pati ent  i s  “br eathi ng  f or   tw oâ€​
  and  f etal
hy pox i a  i s  an  i m por tant  concer n.   At  a  m i ni m um ,   adequate  suppl em ental
ox y gen  shoul d  be  gi v en  i m m edi atel y   to  ensur e  a  PaO 2   ex ceedi ng  65
m m   Hg  and  an  ox y gen  satur ati on  gr eater   than  90%.   The  deci si on  to  use
v enti l ator y   suppor t  consi sti ng  of   i ntubati on  and  m echani cal   v enti l ati on
i s  a  di f f i cul t  one,   but  m ay   be  l i f esav i ng  i n  pati ents  w i th  m ental   status
deter i or ati on,   w or seni ng  r espi r ator y   di str ess  f r om   ex hausti on,   or
pr ogr essi v el y   i ncr easi ng  PaCO 2   l ev el s  w i th  r espi r ator y   aci dosi s.

Fr equent  dosi ng  w i th  an  i nhal ed  β 2 ­adr ener gi c  agoni st  del i v er ed  by
nebul i zer   or   m eter ed­dose  i nhal er   i s  the  m ost  ef f ecti v e  br onchodi l ator
ther apy   f or   acute,   sev er e
P. 346
asthm a  (status  asthm ati cus).   Asthm ati c  pati ents  w ho  ar e  i ni ti al l y
unr esponsi v e  to  i ntensi v e  i nhal ed  ther apy   m ay   r espond  to  the
subcutaneous  del i v er y   of   β 2   agoni sts,   but  or al   adm i ni str ati on  i s  not
i ndi cated  f or   acute  m anagem ent.   Epi nephr i ne  shoul d  be  av oi ded  i n  thi s
pati ent  because  i t  i s  a  ter atogen.

In  addi ti on  to  i nhal ed  β 2   agoni sts  and  suppl em ental   ox y gen,   sy stem i c
cor ti coster oi ds  shoul d  be  i nsti tuted  ear l y   i n  the  em er gency   r oom
m anagem ent.   Cor ti coster oi ds  r educe  ai r w ay   obstr ucti on  by   i nter r upti ng
the  i nf l am m ator y   cascade  at  one  or   m or e  cr i ti cal   steps  i n  i ts  genesi s,
and  m ay   al so  hav e  a  sy ner gi sti c  ef f ect  on  β­adr ener gi c  r eceptor
acti v i ty .   In  gener al ,   sy stem i c  cor ti coster oi ds  shoul d  be  consi der ed  i f
si gni f i cant  i m pr ov em ent  i s  not  seen  w i thi n  the  f i r st  30  to  60  m i nutes  of
i ntensi v e  br onchodi l ator   tr eatm ent.   Ear l y   cor ti coster oi d  use  has  been
show n  to  l ead  to  a  r educti on  i n  both  the  r ate  of   hospi tal i zati on  and  the
r ate  of   r etur n  to  the  em er gency   r oom   af ter   di schar ge.   Inhal ed
cor ti coster oi ds  ar e  not  i ndi cated  f or   the  m anagem ent  of   acute,   sev er e
asthm a.   Theophy l l i ne  pr epar ati ons  of f er   l i ttl e  addi ti onal   benef i t  w hen
added  to  i nhal ed  β 2   agoni st  tr eatm ent  i n  the  em er gency   r oom ,   but
they   m ay   augm ent  r espi r ator y   m uscl e  f uncti on  dur i ng  hospi tal i zati on.
The  use  of   i nhal ed  β 2   agoni sts,   sy stem i c  cor ti coster oi ds,   and  ev en
theophy l l i ne  pr epar ati ons  (w i th  ser um   l ev el s  k ept  at  < 12  µg/m L)  m ay
be  consi der ed  appr opr i ate  i n  the  setti ng  of   pr egnancy   and  unstabl e
asthm a.   Cauti ous  hy dr ati on  i s  al so  appr opr i ate  because  i nsensi bl e
w ater   l osses  i ncr ease  w i th  hy per v enti l ati on.   The  use  of   anti bi oti cs  i s
com m onl y   r eser v ed  f or   objecti v el y   docum ented  i nf ecti ons.   The  sputum
pr oducti on,   al though  i t  i s  y el l ow ­gr een,   does  not  m andate  anti bi oti c
tr eatm ent  unl ess  ther e  i s  Gr am 's  stai n  ev i dence  of   a  dom i nant
or gani sm .

3.   What  ar e  the  tr eatm ent  consi der ati ons  f or   ongoi ng  m anagem ent  i n  thi s
pati ent?

The  opti m al   m anagem ent  of   chr oni c  asthm a  r el i es  on  f our   i nter r el ated
pr i nci pl es:  objecti v e  assessm ent  of   l ung  f uncti on,   phar m acol ogi c
ther apy ,   env i r onm ental   contr ol ,   and  pati ent  educati on.   The  goal s  of
ef f ecti v e  m anagem ent  ar e  to  m ai ntai n  near ­nor m al   pul m onar y   f uncti on
and  phy si cal   acti v i ty   l ev el s,   m i ni m i ze  sy m ptom s  and  pr ev ent
ex acer bati ons,   and  av oi d  the  adv er se  ef f ects  of   asthm a  m edi cati ons.
Spi r om etr y ,   based  on  the  peak   ex pi r ator y   f l ow   r ates  or   FEV 1 ,   pr ov i des
an  objecti v e  m easur e  of   asthm a  contr ol   and  can  be  usef ul   i n  adjusti ng
m edi cati ons  (par ti cul ar l y   taper i ng  sy stem i c  cor ti coster oi ds)  and
assessi ng  the  need  f or   i nter v enti on.   Phar m acol ogi c  ther apy   i s  ty pi cal l y
pr escr i bed  i n  a  stepw i se  m anner .   In  r ecogni ti on  that  asthm a  i s  a
chr oni c  i nf l am m ator y   di sease,   tr ends  i n  ther apy   hav e  pl aced  a  gr eater
em phasi s  on  the  use  of   i nhal ed  cor ti coster oi ds  or   cr om ol y n  as  f i r st­l i ne
m edi cati ons,   w i th  i nhal ed  β 2   agoni sts  used  to  br i ng  about  acute  r el i ef
of   br onchospasm ,   as  needed.   Theophy l l i ne  pr epar ati ons  and  or al   β­
adr ener gi c  agoni sts  ar e  of ten  used  as  second­l i ne  agents,   and  ar e
par ti cul ar l y   usef ul   f or   contr ol l i ng  the  noctur nal   w or seni ng  of   asthm a.
Shor t  “bur stsâ€​   of   or al   cor ti coster oi ds  ar e  best  used  i n  the  ear l y
tr eatm ent  of   acute,   sev er e  ex acer bati ons,   and  ev er y   ef f or t  shoul d  be
m ade  to  av oi d  chr oni c  dependence  on  or al   cor ti coster oi ds  once  the
acute  attack   i s  contr ol l ed.

In  sel ected  cases,   the  i denti f i cati on  and  av oi dance  of   speci f i c  tr i gger s
of   br onchospasm   m ay   hav e  si gni f i cant  i m pact  on  asthm a  contr ol .
Av oi dance  of   aer oal l er gens  (dust  m i tes,   cat  dander ,   pol l ens,   and
m ol ds),   chem i cal s  (sul f i tes  and  tar tr azi ne),
P. 347
cer tai n  m edi cati ons  (aspi r i n,   β­bl ock er s,   and  acety l chol i nester ase
i nhi bi tor s),   and  str ong  aer oi r r i tants  (tobacco  sm ok e,   househol d  spr ay s,
and  w ood  sm ok e)  m ay   be  hel pf ul   f or   cer tai n  pati ents.   Al though
ex er ci se  i s  a  com m on  pr eci pi tati ng  f actor ,   the  use  of   i nhal ed  β 2
agoni sts  or   cr om ol y n  bef or e  ex er ci se  m ay   m i ni m i ze  the  associ ated
br onchospasm .   Last,   pati ent  educati on  shoul d  begi n  at  the  ti m e  of
di agnosi s  and  be  encour aged  thr oughout  the  conti nued  car e.   Lear ni ng  to
i denti f y   i m por tant  si gns  and  sy m ptom s  of   asthm a,   the  cor r ect  use  of
the  peak   ex pi r ator y   f l ow   r ate  m eter   and  m eter ed­dose  i nhal er ,   and
addr essi ng  i ssues  r el ated  to  m edi cati on  ef f ects  and  env i r onm ental
contr ol   m ay   m i ni m i ze  pati ent  m i sunder standi ngs  r egar di ng  the  ongoi ng
m anagem ent  of   asthm a.   In  thi s  pati ent,   a  w ar ni ng  r egar di ng  the
av oi dance  of   α ­adr ener gi c  agoni sts  unti l   the  com pl eti on  of   the
pr egnancy   i s  al so  w ar r anted.

Suggested Readings
Busse  WW,   Lem ansk er   F.   Asthm a.   N   Engl   J  Med  2001;344:350–362.

Inter nati onal   r epor t:  i nter nati onal   consensus  r epor t  on  di agnosi s  and
tr eatm ent  of   asthm a.   Publ i cati on  no.   923091.   Washi ngton,   DC:  U . S.
Depar tm ent  of   Heal th  and  Hum an  Ser v i ces,   Publ i c  Heal th  Ser v i ce,
N ati onal   Insti tutes  of   Heal th,   June  1992.

McFadden  ER.   Acute  sev er e  asthm a.   Am   J  Respi r   Cr i t  Car e  Med


2003;168:740–759.

N ati onal   Asthm a  Educati on  Pr ogr am .   Ex ecuti v e  sum m ar y :  gui del i nes  f or
the  di agnosi s  and  m anagem ent  of   asthm a.   Publ i cati on  no.   913042  A.
Bethesda,   MD:  Of f i ce  of   Pr ev enti on,   Educati on,   and  Contr ol ,   N ati onal
Hear t,   Lung  and  Bl ood  Insti tute,   N ati onal   Insti tutes  of   Heal th,   Jul y   1991.

N ati onal   Asthm a  Educati on  Pr ogr am .   Ex ecuti v e  sum m ar y :  m anagem ent
of   asthm a  dur i ng  pr egnancy .   Publ i cati on  no.   933279  A.   Bethesda,   MD:
Of f i ce  of   Pr ev enti on,   Educati on,   and  Contr ol ,   N ati onal   Hear t,   Lung  and
Bl ood  Insti tute,   N ati onal   Insti tutes  of   Heal th,   October   1992.

Chronic Obstructive Pulmonary Disease
1.   What  i s  chr oni c  obstr ucti v e  pul m onar y   di sease  (COPD)?

2.   What  ar e  the  epi dem i ol ogi c  tr ends  i n  COPD?

3.   What  i s  the  m ost  com m onl y   hel d  theor y   ex pl ai ni ng  the  dev el opm ent  of
em phy sem a?

4.   What  ar e  the  com m on  si gns  and  sy m ptom s  of   COPD?

5.   What  ar e  the  com m on  l abor ator y   and  r adi ogr aphi c  f i ndi ngs  i n  the
setti ng  of   COPD?

Discussion
1.   What  i s  COPD?
The  ter m   COPD  i s  com m onl y   appl i ed  to  tw o  di sor der s:  em phy sem a  and
chr oni c  br onchi ti s.   Most  pati ents  w i th  COPD  hav e  a  com bi nati on  of
these
P. 348
tw o  condi ti ons.   Som e  author s  al so  i ncl ude  chr oni c  obstr ucti v e  asthm a
and  other   di sor der s  associ ated  w i th  chr oni c  ai r f l ow   l i m i tati on  (e. g. ,
br onchi ol i ti s  obl i ter ans  and  br onchi ectasi s)  under   the  headi ng  of   COPD.

2.   What  ar e  the  epi dem i ol ogi c  tr ends  i n  COPD?

Ther e  has  been  an  appr ox i m ate  60%  i ncr ease  i n  the  pr ev al ence  of
COPD  si nce  the  l ate  1970s.   Al though  em phy sem a  i s  a  com m on
postm or tem   f i ndi ng  i n  adul ts,   i ts  pr ev al ence  i s  str ongl y   cor r el ated  w i th
sm ok i ng.   COPD  i s  m or e  com m onl y   di agnosed  i n  m en  than  w om en,   but
as  m or e  adol escent  gi r l s  than  boy s  ar e  begi nni ng  to  sm ok e,   thi s  tr end
m ay   change.   A  heav y   sm ok er   ex hi bi ts  an  av er age  decl i ne  i n  FEV 1   of   40
to  45  m L  per   y ear ;  thi s  decl i ne  i s  onl y   20  m L  per   y ear   i n  a  nonsm ok i ng
adul t.

3.   What  i s  the  m ost  com m onl y   hel d  theor y   ex pl ai ni ng  the  dev el opm ent  of
em phy sem a?

In  par t,   on  the  basi s  of   obser v ati ons  gl eaned  i n  peopl e  w i th  α 1 ­
anti tr y psi n  def i ci ency ,   m ost  author i ti es  bel i ev e  that  the  destr ucti on  of
the  al v eol ar   w al l   and  the  ai r space  enl ar gem ent  seen  i n  the  setti ng  of
em phy sem a  ar e  due  to  an  i m bal ance  betw een  the  pr oteases  and
anti pr oteases  i n  the  l ow er   r espi r ator y   tr ee  (α 1 ­anti tr y psi n  bei ng  the
m ajor   pr otei n  i n  thi s  categor y ).   Ci gar ette  sm ok e  i nacti v ates  the  nor m al
anti pr oteases  i n  peopl e  w ho  do  not  hav e  α 1 ­anti tr y psi n  def i ci ency .

4.   What  ar e  the  com m on  si gns  and  sy m ptom s  of   COPD?

Al though  the  i ni ti al   com pl ai nt  i s  usual l y   dy spnea,   som e  pati ents  seek
m edi cal   car e  because  of   chr oni c  cough  or   sputum   pr oducti on,   w heezi ng,
r ecur r ent  pul m onar y   i nf ecti ons,   or ,   i n  r ar e  ci r cum stances,   w ei ght  l oss
or   l ow er   ex tr em i ty   sw el l i ng.   Ear l y   i n  the  di sease,   phy si cal   ex am i nati on
f i ndi ngs  m ay   be  nor m al .   Later ,   auscul tati on  of   the  chest  m ay   r ev eal
w heezi ng,   r honchi ,   or ,   i n  pati ents  w i th  pr edom i nant  em phy sem a,
decr eased  br eath  sounds.   Per cussi on  of   the  chest  ty pi cal l y   r ev eal s
hy per i nf l ati on  and  l ow   di aphr agm s.   In  adv anced  cases,   the  poi nt  of
m ax i m al   car di ac  i m pul se  m ay   be  f el t  i n  the  subx i phoi d  ar ea.   Cy anosi s,
a  r i ght­si ded  thi r d  hear t  sound  (S 3 ),   jugul ov enous  di stenti on,   and  l ow er
ex tr em i ty   edem a  ar e  l ate  f i ndi ngs.

5.   What  ar e  the  com m on  l abor ator y   and  r adi ogr aphi c  f i ndi ngs  i n  the
setti ng  of   COPD?

Ther e  ar e  no  speci f i c  l abor ator y   v al ues  seen  i n  the  setti ng  of   COPD.
The  r outi ne  bl ood  count  i s  nor m al ,   al though  COPD  pati ents  w i th  chr oni c
hy pox i a  m ay   show   an  el ev ated  hem atocr i t.   The  f i ndi ng  of   eosi nophi l i a
shoul d  r ai se  the  possi bi l i ty   of   concom i tant  asthm a.   Ty pi cal l y   i n  COPD
the  f l ow   r ates  ar e  r educed,   the  l ung  v ol um es  ar e  i ncr eased  due  to
hy per i nf l ati on  as  m easur ed  by   i ncr eased  thor aci c  gas  v ol um e  and
f uncti onal   r esi dual   capaci ty ,   and  the  di f f usi ng  capaci ty   i s  decr eased  i n
em phy sem a.   Reducti ons  i n  both  the  FEV 1   and  f or ced  v i tal   capaci ty
(FVC)  ar e  r outi nel y   seen,   al though  the  FEV 1   i s  r educed  out  of
pr opor ti on  to  the  FVC.   Ear l y   on,   the  chest  r adi ogr aph  i s  usual l y   nor m al .
As  em phy sem a  dev el ops,   the  l ungs  show   hy per i nf l ati on,   f l atteni ng  of
the  di aphr agm s,   and  an  i ncr eased  r etr oster nal   ai r space.   Bul l ae  can  be
seen.   The  el ectr ocar di ogr am   tends  to  be  nor m al ,   ex cept  i n  adv anced
di sease,   w hen  i t  m ay   show   l ow   v ol tage  i n  the  l i m b  l eads,   ear l y   R
w av es  i n  V 1   and  V 2 ,   and  peak ed  P  w av es  (P  pul m onal e).

P. 349
Case
A  65­y ear ­ol d  m an  i s  seen  because  of   a  5­day   hi stor y   of   pr ogr essi v e
shor tness  of   br eath  and  dy spnea  on  ex er ti on.   He  al so  com pl ai ns  of   a  cough
pr oducti v e  of   gr een  sputum ,   as  w el l   as  v ague  r i ght­si ded  chest  pai n.   He  has
f el t  f ev er i sh  at  hom e,   but  deni es  any   shak i ng  chi l l s,   sor e  thr oat,   nausea,
v om i ti ng,   di ar r hea,   edem a,   or   ex posur e  to  any one  w i th  a  si m i l ar   i l l ness.
The  pati ent  has  been  sm ok i ng  tw o  pack s  of   ci gar ettes  per   day   f or   the  l ast
30  y ear s.   How ev er ,   he  r ecentl y   decr eased  hi s  habi t  to  one  pack   per   day .   He
w as  seen  by   a  phy si ci an  appr ox i m atel y   hal f   a  y ear   ago  and  w as  tol d  that  he
had  em phy sem a.   He  has  not  been  hospi tal i zed  pr ev i ousl y .   He  i s  a  r eti r ed
bus  dr i v er   and  l i v es  at  hom e  w i th  hi s  w i f e.   They   hav e  no  pets.   Al though  he
has  noted  som e  dy spnea  on  ex er ti on  ov er   the  l ast  3  to  4  y ear s,   he  conti nues
to  m ai ntai n  an  acti v e  l i f esty l e  and  can  sti l l   m ow   the  l aw n  w i thout  m uch
di f f i cul ty .   He  can  w al k   1  to  2  m i   on  a  f l at  sur f ace  at  a  m odest  pace.   The
pati ent  r ar el y   dr i nk s  al cohol .   He  deni es  any   other   si gni f i cant  past  m edi cal
hi stor y ,   i ncl udi ng  a  hi stor y   of   chi l dhood  asthm a  or   al l er gi c  di seases,
si gni f i cant  cough,   sputum   pr oducti on,   or   ex posur e  to  asbestos.   Hi s
m edi cati ons  i ncl ude  sustai ned­r el ease  theophy l l i ne  and  ov er ­the­counter
v i tam i ns.
On  phy si cal   ex am i nati on,   the  pati ent  i s  f ound  to  be  a  som ew hat  thi n  but
w el l   dev el oped  and  i n  m oder ate  r espi r ator y   di str ess.   Hi s  bl ood  pr essur e  i s
150/98  m m   Hg  w i th  a  pul sus  par adox us  of   20  m m   Hg,   hi s  pul se  i s  110  beats
per   m i nute,   hi s  tem per atur e  i s  37. 9°C  (100. 22°F)  or al l y ,   and  hi s
r espi r ator y   r ate  i s  24  br eaths  per   m i nute  and  l abor ed.   Head,   ey e,   ear s,
nose,   and  thr oat  f i ndi ngs  ar e  unr em ar k abl e.   N o  adenopathy   i s  f ound  i n  hi s
neck ,   and  the  neck   v ei ns  ar e  f l at.   Hi s  chest  i s  hy per ex panded,   and  ther e  i s
use  of   the  accessor y   m uscl es  of   r espi r ati on.   Hy per r esonance  to  per cussi on
i s  noted.   Hi s  br eath  sounds  ar e  di stant  w i th  an  occasi onal   scatter ed  w heeze.
Dur i ng  the  car di ac  ev al uati on,   the  poi nt  of   m ax i m al   i m pul se  i s  l ocated  i n
the  epi gastr i c  ar ea.   Ther e  i s  a  r egul ar   tachy car di a  w i th  a  sy stol i c  f our th
sound  (S 4 )  hear d  best  at  the  r i ght  l ow er   ster nal   bor der .   N o  m ur m ur s  or   r ubs
ar e  noted.   Hi s  abdom en  i s  scaphoi d,   bow el   sounds  ar e  nor m al ,   and  no
tender ness  or   or ganom egal y   i s  noted.   Hi s  ex tr em i ti es  ar e  f r ee  of   cl ubbi ng,
cy anosi s,   and  edem a.   Pul se  ox i m etr y   show s  a  91%  satur ati on  on  r oom   ai r .

1.   What  tests  and  studi es  w oul d  y ou  or der   i n  thi s  pati ent?
A  chest  r adi ogr aphi c  study   r ev eal s  the  pr esence  of   hy per ex panded  l ung
f i el ds,   a  sm al l   car di ac  si l houette,   ev i dence  of   bul l ous  di sease  i n  both
l ungs,   and  an  al v eol ar   i nf i l tr ate  i n  the  r i ght  m i ddl e  l obe  w i th  som e
degr ee  of   v ol um e  l oss.   N o  ef f usi ons  ar e  seen.
Ar ter i al   bl ood  gas  m easur em ent  per f or m ed  on  r oom   ai r   r ev eal s  a  pH  of
7. 50,   a  PaCO 2   of   23  m m   Hg,   a  PaO 2   of   51  m m   Hg,   and  an  ox y gen
satur ati on  of   92%.   Respi r ator y   al k al osi s  i s  pr esent  w i th
hy per v enti l ati on.   Resul ts  of   a  com pl ete  bl ood  count  ar e  as  f ol l ow s:
w hi te  bl ood  cel l s,   14, 300/m m 3   w i th  8%  band  f or m s  and  8. 4%
pol y m or phonucl ear   l euk ocy tes;  and  the  hem atocr i t  r eadi ng  i s  44%.   A
chem i str y   panel   r ev eal s  the  f ol l ow i ng  f i ndi ngs:  sodi um ,   139  m Eq/L;
potassi um ,   4. 1  m Eq/L;  chl or i de,   108  m Eq/L;  bi car bonate,   20  m Eq/L;
bl ood  ur ea  ni tr ogen,   21  m g/dL;  and  cr eati ni ne,   0. 9  m g/dL.   Hi s
theophy l l i ne  l ev el   i s  3. 7  µg/m L.   The  el ectr ocar di ogr am   r ev eal s  si nus
tachy car di a  w i th  l ow   v ol tage  i n  the  l i m b  l eads,   and  no  acute  changes.
2.   What  i s  y our   di agnosi s  based  on  the  i nf or m ati on  y ou  hav e,   and  how
w oul d  y ou  m anage  thi s  pati ent?
P. 350
3.   What  ther apy   shoul d  y ou  i nsti tute  w hi l e  the  pati ent  i s  i n  the  hospi tal ?
The  pati ent  i s  star ted  on  i nhal ed  β 2   agoni sts  and  i ntr av enous
am pi ci l l i n.   Af ter   2  day s  of   tr eatm ent,   hi s  condi ti on  f ai l s  to  i m pr ov e  and
r espi r ator y   f ati gue  r equi r i ng  em er gent  endotr acheal   i ntubati on  and
v enti l ati on  dev el ops.   Hi s  w i f e  states  that  she  does  not  w ant  to  pr ol ong
the  pati ent's  l i f e  “by   ar ti f i ci al   m eansâ€​   and  i s  w or r i ed  that  the
pati ent  w i l l   r equi r e  i ndef i ni te  m echani cal   v enti l ati on.   Another   opti on
w oul d  be  the  use  of   noni nv asi v e  v enti l ati on  w i th  Bi PAP  (i . e. ,   nasal
posi ti v e  ai r w ay   pr essur e  dur i ng  i nspi r ati on  and  ex pi r ati on).
4.   How   w oul d  y ou  r espond  to  hi s  w i f e's  concer n  about  the  need  f or
i ndef i ni te  m echani cal   v enti l ati on?
The  pati ent's  sputum   cul tur e  gr ow s  Haem ophi l us  i nf l uenzae  that  i s
r esi stant  to  am pi ci l l i n.   Hi s  anti bi oti cs  ar e  changed,   and  4  day s  l ater   he
i s  successf ul l y   ex tubated.   Af ter   14  day s  i n  the  hospi tal ,   he  i s  r eady   to
be  di schar ged.
5.   Af ter   the  pati ent  i s  di schar ged,   how   w oul d  y ou  pr ov i de  f ol l ow ­up,   and
w hat  ar e  y our   tr eatm ent  opti ons  now ?

Case Discussion
1.   What  tests  and  studi es  w oul d  y ou  or der   i n  thi s  pati ent?
A c he s t ra diogra phic  s tudy  shoul d  be  obtai ned.   Al though  the  v al ue  of
a  r outi ne  chest  r adi ogr aphi c  study   i n  pati ents  w i th  an  ex acer bati on  of
COPD  has  been  debated,   thi s  pati ent  has  a  pr oducti v e  cough,   l ow ­gr ade
tem per atur e,   and  l ocal i zi ng  chest  pai n,   al l   of   w hi ch  i ndi cate  the
ex i stence  of   an  i ntr athor aci c  abnor m al i ty ,   str essi ng  the  i m por tance  of   a
chest  r adi ogr aph.

Despi te  a  pul se  ox i m etr y   r eadi ng  of   91%,   a rte ria l blood ga s
me a s ure me nts   ar e  i ndi cated  f or   i n  thi s  pati ent.   Ther e  ar e  sev er al
f actor s  that  can  cause  a  poor   cor r el ati on  betw een  the  pul se  ox i m etr y
v al ue  and  the  PaO 2 ,   as  m easur ed  by   ar ter i al   bl ood  gas  deter m i nati ons.
It  i s  poor   i n  pati ents  w i th  jaundi ce  or   dar k   sk i n  pi gm entati on,   as  w el l
as  i n  those  w i th  poor   per i pher al   ci r cul ati on.   Fur ther m or e,   under   v ar i ous
phy si ol ogi c  and  pathol ogi c  condi ti ons  (e. g. ,   changes  i n  the  pH  or   2, 3­
di phosphogl y cer i c  aci d  l ev el ),   the  ox y hem ogl obi n  di ssoci ati on  cur v e  can
be  shi f ted  to  the  r i ght  or   l ef t.   Ther ef or e,   the  ox i m eter   can  ei ther
under pr edi ct  or   ov er pr edi ct  the  actual   PaO 2 .   Fi nal l y ,   i n  a  pati ent  w i th  a
m oder atel y   sev er e  pul m onar y   pr ocess,   k now l edge  of   the  PaCO 2   and  pH
i s  i m per ati v e.

A  c omple te  blood c ount  and  c he mis try pa ne l  shoul d  be  obtai ned.   The
com pl ete  bl ood  count  can  pr ov i de  usef ul   i nf or m ati on  r egar di ng  the
sev er i ty   of   the  i nf ecti ous  pr ocess  (e. g. ,   l euk ocy tosi s).   Fur ther m or e,
si gni f i cant  pol y cy them i a  m ay   i ndi cate  the  ex i stence  of   l ong­standi ng
hy pox i a,   w hi ch  si gni f i es  the  chr oni ci ty   and  sev er i ty   of   the  di sease.   The
chem i str y   pr of i l e  can  pr ov i de  v al uabl e  i nf or m ati on  concer ni ng
el ectr ol y te  i m bal ance  (e. g. ,   hy ponatr em i a  i n  the  sy ndr om e  of
i nappr opr i ate  anti di ur eti c  hor m one  secr eti on)  or   v ol um e  depl eti on.
Know l edge  of   the  ser um   bi car bonate  l ev el   i s  usef ul   i n  conjuncti on  w i th
the  ar ter i al   bl ood  gas  f i ndi ngs  to  assess  the  chr oni ci ty   of   any
r espi r ator y   aci d–base  di sor der s.

P. 351
In  a  pati ent  of   adv anced  age  w i th  r i sk   f actor s  f or   cor onar y   ar ter y
di sease  (tobacco  abuse  and  hy per tensi on)  and  chest  pai n,   an
e le c troc a rdiogra phy  i s  i ndi cated.   Fur ther m or e,   m any   ty pes  of
ar r hy thm i as  (e. g. ,   m ul ti f ocal   atr i al   tachy car di a)  ar e  seen
pr edom i nantl y   i n  the  setti ng  of   decom pensated  pul m onar y   di sease.

The  the ophylline  le ve l  m ust  be  deter m i ned.   Because  thi s  dr ug  has  a
nar r ow   ther apeuti c  i ndex ,   cl ose  m oni tor i ng  of   the  ser um   l ev el s  i s
essenti al   i n  acutel y   i l l   pati ents.

2.   What  i s  y our   di agnosi s  based  on  the  i nf or m ati on  y ou  hav e,   and  how
w oul d  y ou  m anage  thi s  pati ent?

The  pati ent  has  a  r i ght  m i ddl e  l obe  pneum oni a  and,   as  a  r esul t,   an
ex acer bati on  of   hi s  COPD.   Gi v en  the  l ack   of   cough  and  sputum
pr oducti on  i n  hi s  past  hi stor y ,   as  w el l   as  the  bul l ae  noted  i n  the  chest
r adi ogr aphi c  study ,   hi s  cl i ni cal   pi ctur e  i s  consi stent  w i th  em phy sem a,
as  opposed  to  chr oni c  br onchi ti s.   Most  pati ents  hav e  a  com bi nati on  of
both  di sor der s.   He  shoul d  be  adm i tted  to  the  hospi tal .

3.   What  ther apy   shoul d  y ou  i nsti tute  w hi l e  the  pati ent  i s  i n  the  hospi tal ?

Blood a nd s putum c ulture s   shoul d  be  obtai ned.   A  Gr am 's­stai ned
sputum   speci m en  shoul d  be  ex am i ned  both  by   the  pr i m ar y   phy si ci ans
and  by   the  l abor ator y   techni ci an.

Inha le d Î²­a dre ne rgic  a gonis ts   (e. g. ,   0. 5  m L  of   al buter ol   i n  1. 5  m L


of   sal i ne)  ar e  the  m ai nstay   of   tr eatm ent  f or   a  COPD  ex acer bati on.   The
i ni ti al   dosi ng  f r equency   of   thi s  m edi cati on  depends  on  the  sev er i ty   of
the  di sease;  i t  can  be  adm i ni ster ed  ev er y   1  to  3  hour s.   As  the  pati ent's
condi ti on  i m pr ov es,   the  dosi ng  f r equency   can  be  r educed  to  ev er y   4  to
6  hour s.   Al though  m eter ed­dose  i nhal er s  can  be  used  w i th  a  si m i l ar
degr ee  of   success,   thei r   ef f i cacy   depends  on  the  abi l i ty   of   the  pati ent
to  coor di nate  the  ti m i ng  of   the  i nhal ati on  and  the  acti v ati on  of   the
i nhal er ,   m ak i ng  them   a  l ess­than­opti m al   tool   i n  an  acutel y   i l l   pati ent.

The  r ol e  of   the ophylline   i n  the  m anagem ent  of   an  acute  ex acer bati on
of   COPD  r em ai ns  contr ov er si al .   Most  author i ti es  agr ee  that  theophy l l i ne
i s  a  w eak   br onchodi l ator   w i th  a  l ow   ther apeuti c  i ndex .   In  a  r andom i zed,
contr ol l ed  study ,   the  addi ti on  of   am i nophy l l i ne  to  a  w el l   f or m ul ated
ther apeuti c  r egi m en  i n  hospi tal i zed  pati ents  w i th  COPD  f ai l ed  to  show
any   benef i t  i n  ter m s  of   i m pr ov em ent  i n  l ung  f uncti on  or   on  the  dy spnea
scal e.   If   used,   theophy l l i ne  l ev el s  shoul d  be  m oni tor ed  cl osel y   and  the
pati ent  obser v ed  f or   any   si gns  or   sy m ptom s  of   tox i ci ty .

Inha le d a ntic holine rgic s   m ay   al so  be  usef ul .   Ipr atr opi um   br om i de  i s
the  agent  of   choi ce.   It  i s  av ai l abl e  i n  a  m eter ed­dose  i nhal er
f or m ul ati on,   and  can  be  gi v en  i n­l i ne  i nto  v enti l ator   tubi ng.   Occasi onal
bl ur r ed  v i si on  or   ur i nar y   r etenti on  has  been  noted  i n  pati ents  usi ng  i t.
For   the  tr eatm ent  of   a  pati ent  w i th  stabl e  COPD,   ti otr opi um   br om i de  i s
super i or   to  β­agoni sts.   A  com bi nati on  of   the  tw o  can  m odestl y   i m pr ov e
the  br onchodi l ati on  achi ev ed,   as  w el l   as  pr ol ong  the  ef f ecti v e  dur ati on
of   acti on  of   each  agent.   The  star ti ng  dose  i s  one  i nhal ati on  tw i ce  dai l y .

The  use  of   s ys te mic  c ortic os te roids ,   l i k e  that  of   theophy l l i ne,   i s


contr ov er si al .   A  str onger   case  f or   thei r   use  can  be  m ade  i f   the  pati ent
has  ex hi bi ted  a  pr ev i ousl y   docum ented  ster oi d  r esponse,   has
eosi nophi l i a,   or   has  show n  a  si gni f i cant  br onchodi l ator   r esponse  to  the
i nhal ed  agents.   A  r easonabl e  star ti ng  dose  i s  40  to
P. 352
60  m g  of   i ntr av enous  m ethy l pr edni sol one  ev er y   6  hour s.   Thi s  r egi m en
i s  changed  to  an  or al   f or m   (e. g. ,   pr edni sone,   40  to  60  m g  per   day )  w i th
r api d  taper i ng.   Moni tor i ng  of   the  si de  ef f ects  (e. g. ,   hy per gl y cem i a,
m ental   status  changes,   and  gastr i ti s)  i s  cr uci al .   Inhal ed  cor ti coster oi ds
hav e  no  r ol e  i n  the  acute  m anagem ent  of   thi s  pati ent.

Anti bi oti cs  ar e  i ndi cated  ev en  w hen  an  i nf i l tr ate  i s  not  f ound  on  the
chest  r adi ogr aph.   In  a  w el l ­desi gned,   contr ol l ed  cl i ni cal   tr i al ,   pati ents
w i th  COPD  tr eated  w i th  br oad­spectr um   or al   anti bi oti cs  (the  new er
cephal ospor i ns,   m acr ol i des,   and  f l uor oqui nol ones)  di d  better   than  the
contr ol   pati ents.   In  thi s  si tuati on,   the  choi ce  of   anti bi oti c  depends  on
the  Gr am 's  stai n  f i ndi ngs  of   the  sputum .   Bacter i a  com m onl y   r esponsi bl e
f or   l ow er   r espi r ator y   tr act  i nf ecti ons  i n  thi s  pati ent  popul ati on  i ncl ude
Str eptococcus  pneum oni ae,   H.   i nf l uenzae,   and  Br anham el l a  catar r hal i s.
The  l atter   tw o  usual l y   pr oduce  β­l actam ase.

4.   How   w oul d  y ou  r espond  to  hi s  w i f e's  concer n  about  the  need  f or
i ndef i ni te  m echani cal   v enti l ati on?

The  condi ti on  of   pati ents  w i th  COPD  f r equentl y   deter i or ates  to  the  poi nt
w her e  they   r equi r e  v enti l ator y   suppor t.   The  pr ognosi s  f or   w eani ng  the
pati ent  f r om   the  v enti l ator ,   as  w el l   as  the  f utur e  qual i ty   of   l i f e,
depends  on  the  pati ent's  pr em or bi d  l ung  f uncti on  and  f uncti onal   state.
Al though  the  status  of   thi s  pati ent's  pul m onar y   f uncti on  i s  unk now n,
gi v en  hi s  hi gh  qual i ty   of   l i f e  and  f uncti onal   status  bef or e  thi s  epi sode,
the  odds  ar e  ov er w hel m i ngl y   i n  hi s  f av or   that  he  can  be  successf ul l y
ex tubated.   Ther ef or e,   v er y   aggr essi v e  tr eatm ent  i s  i ndi cated.

5.   Af ter   the  pati ent  i s  di schar ged,   how   w oul d  y ou  pr ov i de  f ol l ow ­up,   and
w hat  ar e  y our   tr eatm ent  opti ons  now ?

Ev er y   attem pt  shoul d  be  m ade  to  encour age  hi m   to  stop  sm ok i ng.   The
r ate  of   pul m onar y   f uncti on  l oss,   i n  sm ok er s  w ho  qui t  sm ok i ng,   r ev er ts
gr adual l y   tow ar d  the  r ate  seen  i n  the  nor m al   popul ati on.   The  r i sk   of
l ung  cancer   and  hear t  di sease  al so  decl i nes  si gni f i cantl y .

Am ong  the  ther apeuti c  i nter v enti ons  now   av ai l abl e  ar e  w el l ­desi gned
sm ok i ng  cessati on  ther apy   gr oups,   such  as  the  one  of f er ed  by   the
Am er i can  Lung  Associ ati on,   as  w el l   as  the  super v i sed  use  of   ni coti ne
gum .   It  i s  al so  i m por tant  that  a  f ol l ow ­up  chest  r adi ogr aphi c  study   be
obtai ned  w i thi n  4  to  6  w eek s  to  dem onstr ate  di sappear ance  of   the
i nf i l tr ate.   An  unr esol v ed  i nf i l tr ate  coul d  be  due  to  l ung  cancer
(especi al l y   w i th  the  v ol um e  l oss  seen  ear l i er   on  hi s  chest  r adi ogr aph).
The  i nci dence  of   l ung  cancer   i s  m uch  hi gher   i n  sm ok er s  w i th  COPD  than
i n  those  w i thout.   Thi s  r i sk   al so  di m i ni shes  si gni f i cantl y   w i th  the
cessati on  of   sm ok i ng.   The  pati ent  shoul d  al so  r ecei v e  annual   i nf l uenza
v acci nes  and,   al though  a  contr ov er si al   m easur e,   a  one­ti m e  pol y v al ent
pneum ococcal   v acci ne.   Hi s  m edi cati ons  shoul d  i ncl ude  an  i nhal ed  β
agoni st  or   i pr atr opi um   br om i de  (Atr ov ent),   or   both.   The  judi ci ous  use
of   sustai ned­dose  or al   theophy l l i nes  and  ster oi ds  (i nhal ed  i f   possi bl e)
m ay   be  i ndi cated.   Fi nal l y ,   a  r epeat  ar ter i al   bl ood  gas  m easur em ent  on
r oom   ai r   shoul d  be  per f or m ed  w hen  the  pati ent's  condi ti on  i s  cl i ni cal l y
stabl e.   Suppl em ental   ox y gen  f or   pati ents  w i th  a  PaO 2   of   55  m m   Hg  or
l ess  can  i m pr ov e  a  pati ent's  cogni ti v e  abi l i ty ,   ex er ci se  tol er ance,   and
r i ght  hear t  f uncti on,   as  w el l   as  pr ev ent  the  dev el opm ent  of   pul m onar y
hy per tensi on.   U l ti m atel y ,   i t  can  l engthen  the  pati ent's  l i f e  span.

P. 353
Suggested Readings
Chr oni c  obstr ucti v e  pul m onar y   di sease:  di sor der   of   the  car di ov ascul ar
and  r espi r ator y   sy stem s.   Pr oc  Am   Thor ac  Soc  2005;2:1–94.

Snow   V,   Lasher   S,   Mottur ­Pi l son  C.   Ev i dence  f or   m anagem ent  of   acute


ex acer bati ons  of   chr oni c  obstr ucti v e  pul m onar y   di sease.   Ann  Int  Med
2001;134:595–599.

Idiopathic Pulmonary Fibrosis
1.   What  ar e  the  basi c  pathol ogi c  ev ents  that  l ead  to  i nter sti ti al   l ung
di sease  (ILD)?

2.   What  ar e  the  ty pi cal   pul m onar y   f uncti on  test  (PFT)  abnor m al i ti es
obser v ed  i n  the  setti ng  of   i di opathi c  pul m onar y   f i br osi s  (IPF)?

3.   What  i s  the  outcom e  i f   IPF  i s  l ef t  untr eated,   and  i s  the  di agnosi s  one  of
ex cl usi on?

4.   What  ar e  the  pr esenti ng  sy m ptom s  of   IPF?

Discussion
1.   What  ar e  the  basi c  pathol ogi c  ev ents  that  l ead  to  ILD?

Regar dl ess  of   the  under l y i ng  cause  of   ILD,   the  m or phol ogi c  patter n  of
pr ogr essi on  i s  si m i l ar .   A  k now n  or   unk now n  sti m ul us  or   i m m unol ogi c
ev ent  causes  al v eol ar   epi thel i al /endothel i al   i njur y   r esul ti ng  i n
m i gr ati on  of   i nf l am m ator y   cel l s  to  the  al v eol ar   str uctur es.   U nl i k e  acute
i nsul ts,   as  seen  i n  bacter i al   pneum oni a  (w hi ch  r esul ts  i n  a  tr ansi ent
i nf l am m ator y   i nf i l tr ate),   the  i njur y   of   ILD  i s  per si stent  or   r epeti ti v e.
The  per si stence  of   the  i njur y   and  i nf l am m ati on  dam ages  the
par enchy m al   cel l s  and  causes  di sr upti on  of   the  al v eol ar   capi l l ar y
m em br ane.   Inf l am m ati on  and  abnor m al   r epai r   then  l ead  to
m esenchy m al   cel l   pr ol i f er ati on  (f i br obl asts),   w i th  the  attendant
pr oducti on  of   ex cess  col l agen  and  connecti v e  ti ssue  el em ents
ex pandi ng  the  ex tr acel l ul ar   m atr i x .   U l ti m atel y ,   the  nor m al   ar chi tectur e
of   the  l ung  i s  r epl aced  by   f i br oti c  bands  and  cy sti c  spaces  k now n  as
honey com b  l ung.   IPF  i s  the  pr ototy pi c  ILD  w hose  under l y i ng  pathol ogi c
pr ocess  i s  usual l y   i nter sti ti al   pneum oni a.

2.   What  ar e  the  ty pi cal   PFT  abnor m al i ti es  obser v ed  i n  the  setti ng  of   IPF?

The  char acter i sti c  PFT  abnor m al i ti es  i ncl ude  a  gr adual   r educti on  of   l ung
v ol um es  and  ai r f l ow   (FEV 1   and  FVC),   w i th  pr eser v ati on  of   the  FEV 1 /FVC
r ati o.   The  FEV 1 /FVC  can  be  nor m al   or   i ncr eased  due  to  the  i ncr eased
el asti c  r ecoi l   of   the  sti f f   l ung  par enchy m a.   Pati ents  w i th  som e  ILD
(e. g. ,   chr oni c  hy per sensi ti v i ty   pneum oni ti s,   Langer hans  cel l
gr anul om atosi s  of   the  l ung,   and  cy sti c  f i br osi s)  m ay   i ni ti al l y   ex hi bi t
nor m al   or   i ncr eased  l ung  v ol um es  secondar y   to  sm al l   ai r w ay
i nv ol v em ent  w i th  ai r f l ow   obstr ucti on  (a  r educed  FEV 1 /FVC  r ati o)  and  ai r
tr appi ng.   In  addi ti on,   the  dev el opm ent  of   em phy sem a  i n  conjuncti on
w i th  any   ty pe  of   ILD  m ay   be  i ni ti al l y   associ ated  w i th  r el ati v el y   m i l d
abnor m al i ti es  on  r outi ne  PFTs.   It  i s,   ther ef or e,   i m por tant  to  r em em ber
P. 354
that  the  absence  of   ty pi cal   PFT  abnor m al i ti es  does  not  ex cl ude  IPF  or
any   other   ILD.   The  gas  ex change  at  r est  i s  i ni ti al l y   nor m al   i n  m any
pati ents  w i th  IPF;  how ev er ,   ex er ci se­i nduced  desatur ati on  i s  one  of   the
ear l i est  si gns  and  the  m ost  sensi ti v e  m eans  to  detect  the  di sease.   The
di f f usi on  capaci ty   i s  ty pi cal l y   r educed  but  m ay   be  nor m al ,   especi al l y   i n
the  ear l y   stages  of   the  di sease.

3.   What  i s  the  outcom e  i f   IPF  i s  l ef t  untr eated,   and  i s  the  di agnosi s  one  of
ex cl usi on?

IPF  steadi l y   pr ogr esses  ev en  w hen  tr eated.   Wi th  ti m e,   the  chr oni c
i nf l am m ator y   r esponse  (al v eol i ti s)  pr oduces  f i br osi s,   al ong  w i th  the
cl assi c  phy si ol ogi c  abnor m al i ti es  al r eady   descr i bed.   Al though  the
eti ol ogy   i s  unk now n,   the  cl i ni copathol ogi c  m ani f estati ons  ar e  speci f i c,
and,   ther ef or e,   i t  i s  not  a  di agnosi s  of   ex cl usi on.   U p  to  50%  of   pati ents
di e  betw een  2  and  3  y ear s  af ter   di agnosi s.   At  pr esent  onl y   l ung
tr anspl antati on  i s  a  v i abl e  tr eatm ent  i n  el i gi bl e  subjects.   Cur r entl y ,
nov el   bi ol ogi c  agents  ar e  bei ng  tested.

4.   What  ar e  the  pr esenti ng  sy m ptom s  of   IPF?

The  i nsi di ous  onset  of   br eathl essness  and  nonpr oducti v e  cough  i s
com m on  to  m ost  cases.   Fati gue,   l ow ­gr ade  f ev er i shness,   ar thr al gi as,
and  m y al gi as  ar e  al so  r el ati v el y   com m on,   but  nonspeci f i c,   sy m ptom s.
The  occur r ence  of   f r ank   ar thr i ti s,   m y osi ti s  (m uscl e  tender ness  and
w eak ness),   photosensi ti v i ty ,   Ray naud's  phenom enon,   v i sual   pr obl em s,
and  so  on,   suggests  the  ex i stence  of   other   sy stem i c  pr ocesses,   such  as
col l agen  v ascul ar   di sease,   v ascul i ti s,   or   sar coi dosi s.   Dr y   i nspi r ator y
cr ack l es  m ay   be  the  onl y   phy si cal   f i ndi ng,   al though  di gi tal   cl ubbi ng  i s
seen  i n  40%  to  70%  of   cases.   A  chest  r adi ogr aph  usual l y   r ev eal s
r eti cul ar   (l i near )  or   r eti cul onodul ar   opaci ti es  i n  the  l ow er   l ung  zones.
The  hi gh­r esol uti on  CT  scan  i ndi cates  per i pher al   and  basi l ar   i nter sti ti al
i nf i l tr ates  and  honey com bi ng.

Case
A  58­y ear ­ol d  w om an  i s  r ef er r ed  f or   the  ev al uati on  of   br eathl essness  and
cough.   She  f i r st  began  noti ci ng  dy spnea  on  ex er ti on  appr ox i m atel y   3  to  4
y ear s  ago  w hen  usi ng  her   f l oor   sw eeper .   How ev er ,   she  noted  no  l i m i tati on
w hen  per f or m i ng  any   of   her   other   usual   acti v i ti es.   Her   dy spnea  w or sened
sl i ghtl y   ov er   the  ensui ng  y ear s  w i thout  any   other   sy m ptom s  unti l   9  m onths
ago,   w hen  a  nonpr oducti v e  cough  dev el oped.   Thi s  w as  tr eated  w i th
anti bi oti cs  and  i nhal ed  br onchodi l ator s,   w i thout  i m pr ov em ent.   Ov er   the  l ast
9  m onths,   her   br eathl essness  has  w or sened  and  she  now   has  tr oubl e
cl i m bi ng  one  f l i ght  of   stai r s.   She  ti r es  easi l y   and  occasi onal l y   f eel s
f ev er i sh,   but  has  not  ex per i enced  ar thr al gi as,   m y al gi as,   ni ght  sw eats,   or
other   consti tuti onal   sy m ptom s.   She  has  ex per i enced  no  chest  pai n  or
hem opty si s  and  has  no  hi stor y   of   car di opul m onar y   di sease.   Her   past  m edi cal
hi stor y   i s  unr em ar k abl e.   Medi cati ons  i ncl ude  i nhal ed  br onchodi l ator s.   She  i s
m ar r i ed  and  has  nev er   sm ok ed.   She  has  w or k ed  as  a  r etai l   sal es  cl er k   f or
17  y ear s  w i thout  ex posur es.   She  has  had  no  pet  bi r ds,   l eak y   pi pes,   or   m ol dy
condi ti ons  i n  her   hom e.
Phy si cal   ex am i nati on  r ev eal s  a  r espi r ator y   r ate  of   20  unl abor ed  br eaths  per
m i nute  w i th  dr y   i nspi r ator y   r al es  hear d  ov er   the  l ow er   thi r d  of   the  poster i or
l ung  f i el ds.   She  has  no  cl ubbi ng  or   edem a.   Labor ator y   ev al uati on  r ev eal s  a
nor m al   hem ogr am   and  bi ochem i cal
P. 355
pr of i l e.   Anti nucl ear   anti bodi es  show   w eak   posi ti v i ty   at  1:80.   Testi ng  f or
r heum atoi d  f actor   i s  negati v e.
A  chest  r adi ogr aph  show s  r eti cul ar   opaci ti es  that  ar e  m ost  pr om i nent  i n  the
l ow er   l ung  zones  as  w el l   as  r educed  l ung  v ol um es  and  the  hi gh­r esol uti on
com puted  tom ogr aphy   scan  (HRCT)  show s  per i pher al   and  basi l ar   r eti cul ar
i nf i l tr ates  as  w el l   as  honey com b  change  i n  a  si m i l ar   di str i buti on.   PFTs
r ev eal   a  total   l ung  capaci ty   of   70%  of   pr edi cted  and  a  f uncti onal   r esi dual
capaci ty   that  i s  66%  of   pr edi cted.   The  FEV 1   i s  50%  of   pr edi cted  w i th  an  FVC
that  i s  58%  of   pr edi cted.   The  FEV 1 /FVC  r ati o  i s  88%.   Her   di f f usi on  capaci ty
i s  65%  of   pr edi cted.   The  PaO 2   on  r oom   ai r   w hi l e  r esti ng  i s  60  m m   Hg,   w hi ch
dr ops  to  38  m m   Hg  w i th  ex er ci se.

1.   How   w oul d  the  di agnosi s  of   IPF  best  be  conf i r m ed  i n  thi s  pati ent?
2.   If   the  thor acoscopi c  l ung  bi opsy   speci m en  r ev eal s  the  pr esence  of   the
usual   i nter sti ti al   pneum oni ti s,   w hat  ar e  the  tr eatm ent  opti ons  f or   thi s
pati ent?

Case Discussion
1.   How   w oul d  the  di agnosi s  of   IPF  best  be  conf i r m ed  i n  thi s  pati ent?

Al though  the  cl i ni cal ,   r adi ol ogi c,   and  phy si ol ogi c  pi ctur e  i n  thi s  pati ent
i s  m ost  suggesti v e  of   IPF,   other   i nter sti ti al   l ung  pr ocesses,   such  as
chr oni c  hy per sensi ti v i ty   pneum oni ti s,   nonspeci f i c  i nter sti ti al   pneum oni a
(N SIP),   asbestosi s,   stage  III  sar coi dosi s,   and  col l agen  v ascul ar
di sease,   m ay   pr esent  i n  an  i denti cal   m anner .   How ev er ,   the  ty pi cal
HRCT  and  l ack   of   ev i dence  f or   one  of   the  af or em enti oned  ILDs,
establ i shes  the  di agnosi s.   In  cases  w her e  the  r adi ol ogi c  pr esentati on  i s
not  ty pi cal ,   sur gi cal   l ung  bi opsy   by   the  v i deo­assi sted  thor acoscope
(VATS)  i s  i ndi cated  f or   the  di agnosi s.   The  hi stol ogi c  ex pr essi on  of   IPF
i s  usual   i nter sti ti al   pneum oni a  (U IP).   Thi s  i s  char acter i zed  by   tem por al
heter ogenei ty   i n  w hi ch  end  stage  honey com b  i s  adjacent  to  nor m al   l ung
and  ther e  i s  al v eol ar   w al l   f i br osi s  of   v ar y i ng  degr ees.   Another
i m por tant  f eatur e  i s  the  pr esence  of   f i br obl asti c  f oci ,   w hi ch  ar e
subepi thel i al   col l ecti ons  of   m y of i br obl asts  i n  l oose  connecti v e  ti ssue
str om a.

2.   If   the  thor acoscopi c  l ung  bi opsy   speci m en  r ev eal s  the  pr esence  of   the
usual   i nter sti ti al   pneum oni ti s,   w hat  ar e  the  tr eatm ent  opti ons  f or   thi s
pati ent?

Cor ti coster oi ds  and  i m m unosuppr essi v e  dr ugs  (azathi opr i ne  and
cy dophospham i de)  hav e  been  r ecom m ended  f or   IPF.   How ev er ,   at
pr esent  ther e  i s  no  ev i dence  that  thi s  tr eatm ent  i m pr ov es  outcom e.   For
el i gi bl e  pati ents,   tr anspl antati on  i s  an  opti on.   The  addi ti on  of   N ­
acety l cy stei ne  to  pr edni sone–azathi opr i ne  r egi m en  show s  pr om i se  f or
a  f ew   pati ents.   N ew er   bi ol ogi cs  that  potenti al l y   i nhi bi t
f i br opr ol i f er ati on  ar e  i n  tr i al s.

Suggested Readings
Ki ng  TE.   Idi opathi c  pul m onar y   f i br osi s.   In:  Schw ar z  MI,   Ki ng  TE,   eds.
Inter sti ti al   l ung  di sease,   4th  ed.   Tor onto:  BC  Deck er ,   2003.

P. 356

Pleural Disease
1.   What  i s  a  pl eur al   ef f usi on?

2.   What  ar e  the  phy si cal   f i ndi ngs  associ ated  w i th  a  pl eur al   ef f usi on?

3.   What  i s  the  si gni f i cance  of   di sti ngui shi ng  betw een  a  tr ansudati v e  and
an  ex udati v e  pl eur al   ef f usi on?

4.   What  testi ng  di sti ngui shes  betw een  pl eur al   tr ansudates  and  ex udates?

5.   What  i s  an  em py em a?

6.   How   do  y ou  dev el op  a  tr eatm ent  pl an  i n  the  pati ent  w i th  a  pl eur al
ef f usi on?

Discussion
1.   What  i s  a  pl eur al   ef f usi on?

A  pl eur al   ef f usi on  i s  an  abnor m al   col l ecti on  of   f l ui d  i n  the  potenti al
space  betw een  the  v i scer al   and  par i etal   pl eur a.   N or m al l y ,   thi s  space
contai ns  onl y   a  f ew   m i l l i l i ter s  of   f l ui d,   w hi ch  ser v es  to  l ubr i cate  these
sur f aces.

2.   What  ar e  the  phy si cal   f i ndi ngs  associ ated  w i th  a  pl eur al   ef f usi on?

Pl eur al   ef f usi ons  can  be  detected  on  phy si cal   ex am i nati on  i f   they   ar e  of
suf f i ci ent  v ol um e  to  pr oduce  a  f l ui d  l ev el   i n  the  chest  and  com pr ess
under l y i ng  l ung  ti ssue.   Dul l ness  to  per cussi on  w i th  decr eased  or   absent
br eath  sounds  i n  a  dependent  anatom i c  l ocati on  i s  ty pi cal   of   pl eur al
ef f usi on.   Egophony   i s  an  i m por tant  f i ndi ng  that  di sti ngui shes  an
ef f usi on  stem m i ng  f r om   atel ectasi s  secondar y   to  br onchi al   obstr ucti on.
Lobar   consol i dati on  w i th  a  patent  br onchus,   as  occur s  i n  pneum oni a,
m ay   be  di f f i cul t  to  di sti ngui sh  f r om   an  ef f usi on,   because  these  tw o
abnor m al i ti es  of ten  coex i st.   Whi sper i ng  pector i l oquy   m ay   be  hear d  i n
the  pr esence  of   a  consol i dati on  but  i s  absent  ov er   a  pl eur al   ef f usi on.   In
addi ti on,   phy si cal   f i ndi ngs  that  suggest  a  sy stem i c  i l l ness,   such  as
congesti v e  hear t  f ai l ur e,   ci r r hosi s,   and  l upus  er y them atosus,   pr ov i de
i m por tant  cl ues  to  the  potenti al   cause  and  natur e  of   a  pl eur al   ef f usi on
di scov er ed  on  phy si cal   ex am i nati on  or   a  r adi ogr aphi c  study .

3.   What  i s  the  si gni f i cance  of   di sti ngui shi ng  betw een  a  tr ansudati v e  and
an  ex udati v e  pl eur al   ef f usi on?

Thi s  di v i si on  i s  an  i m por tant  f i r st  step  i n  the  di agnosti c  ev al uati on  of   a
pl eur al   ef f usi on.   In  the  contex t  of   a  tr ansudati v e  pl eur al   ef f usi on,   the
pl eur a  i tsel f   i s  not  di seased  but  f l ui d  i s  accum ul ati ng  because  of   the
ef f ect  of   abnor m al   Star l i ng  f or ces  stem m i ng  f r om   a  sy stem i c  i l l ness,
such  as  congesti v e  hear t  f ai l ur e,   ci r r hosi s,   or   nephr oti c  sy ndr om e.   In
these  condi ti ons,   pl eur al   f l ui d  accum ul ates  f or   the  sam e  r easons  that
per i pher al   edem a  and  asci tes  dev el op.   In  the  setti ng  of   an  ex udati v e
ef f usi on,   the  pl eur a  i s  pr i m ar i l y   i nv ol v ed  by   the  di sease.   Ex am pl es
i ncl ude  m al i gnanci es  i n  the  pl eur a  (usual l y   m etastati c),   i nf ecti ons,
col l agen  v ascul ar   di seases,   and  pul m onar y   i nf ar cti ons.   In  these
condi ti ons,   the  pl eur al   sur f ace  i s  i njur ed  and  f l ui d  accum ul ates
i ndependent  of   Star l i ng  f or ces.

P. 357
4.   What  testi ng  di sti ngui shes  betw een  pl eur al   tr ansudates  and  ex udates?

Ther e  ar e  a  num ber   of   si m pl e  l abor ator y   tests  that  can  hel p  i n


di sti ngui shi ng  tr ansudati v e  and  ex udati v e  pl eur al   ef f usi ons.   Ex udates
hav e  a  hi gher   pr otei n  content  and  l actate  dehy dr ogenase  (LDH)  l ev el
than  tr ansudates  because  the  m esothel i al   cel l s  ar e  i njur ed  by   the
di sease  pr ocess.   A  pl eur al   f l ui d  pr otei n  l ev el   gr eater   than  50%  of   the
pati ent's  cor r espondi ng  pl asm a  pr otei n  l ev el ,   an  LDH  l ev el   gr eater   than
180  IU ,   or   an  LDH  l ev el   gr eater   than  60%  of   the  pati ent's
cor r espondi ng  pl asm a  l ev el   di sti ngui shes  pl eur al   ex udates  f r om
tr ansudates.   If   any   of   these  ar e  pr esent,   i t  i s  m or e  than  90%  l i k el y
that  the  ef f usi on  i s  an  ex udate.   The  f i ndi ng  of   a  hi gh  w hi te  bl ood  cel l
count  i n  the  pl eur al   f l ui d  suggests  the  pr esence  of   an  ex udate  but  i s  a
l ess  r el i abl e  i ndi cator   than  the  pr otei n  and  LDH  v al ues.   Once  an
ex udate  has  been  i denti f i ed,   other   studi es  can  be  per f or m ed  to  hel p
el uci date  i ts  cause.   These  i ncl ude  cy tol ogi c  anal y si s,   m i cr obi ol ogi c
stai ns  and  cul tur es,   pH  deter m i nati on,   and  ser ol ogi c  testi ng  f or
col l agen  v ascul ar   di sease.

5.   What  i s  an  em py em a?

Em py em a  i s  an  i nf ecti on  i n  the  pl eur al   space.   Thi s  i nf ecti on  m ay   be


bacter i al ,   m y cobacter i al ,   or   f ungal   i n  or i gi n.   In  addi ti on  to  m or e
com m on  bacter i al   speci es,   Acti nom y ces  and  N ocar di a  can  al so  cause
em py em a.   Em py em as  m ust  be  di sti ngui shed  f r om   par apneum oni c
ef f usi ons.   In  thi s  case,   a  pneum oni a  abutti ng  the  pl eur a  can  r esul t  i n
an  i nf l am m ator y   ex udate.   The  di agnosti c  hal l m ar k   of   em py em a  i s
i denti f i cati on  of   the  causati v e  or gani sm   ei ther   by   bacter i al   stai ni ng  or
cul tur e  of   the  f l ui d.   Of ten  thi s  i s  not  possi bl e  due  to  pr i or   anti bi oti c
ther apy .   For   ex am pl e,   a  cl i ni cal   di agnosi s  of   em py em a  i s  m ade  on  the
basi s  of   ov er al l   cl i ni cal   pr esentati on.   How ev er ,   a  l ow   pl eur al   gl ucose
l ev el   (< 50  m g/dL)  or   a  pH  (< 7. 30),   or   both,   suggest  the  pr esence  of
em py em a,   but  ar e  al so  seen  i n  m al i gnant  ef f usi ons,   esophageal
r uptur e,   and  r heum atoi d  ar thr i ti s.   Em py em as  ar e  di f f i cul t  i nf ecti ons  to
cur e  w i th  anti m i cr obi al   ther apy   al one,   par ti cul ar l y   w hen  bacter i al   i n
or i gi n.   These  em py em as  ar e  essenti al l y   i ntr apl eur al   abscesses  and,
l i k e  m ost  abscesses,   r equi r e  dr ai nage  to  achi ev e  r esol uti on.

6.   How   do  y ou  dev el op  a  tr eatm ent  pl an  i n  the  pati ent  w i th  a  pl eur al
ef f usi on?

Tr eatm ent  depends  on  the  eti ol ogy   of   the  ef f usi on.   If   i t  i s  a  tr ansudate,
ef f ecti v e  tr eatm ent  of   the  congesti v e  hear t  f ai l ur e,   nephr oti c
sy ndr om e,   or   ci r r hosi s,   i f   possi bl e,   of ten  r esul ts  i n  r esol uti on.   If   the
ef f usi on  i s  associ ated  w i th  an  i nf ecti on,   such  as  a  par apneum oni c
ef f usi on  or   an  em py em a,   def i ni ti v e  tr eatm ent  of   the  i nf ecti on  i s
i ndi cated.   Mal i gnant  ef f usi ons  m ust  be  addr essed  i n  the  contex t  of   the
under l y i ng  m al i gnancy .   Ther ef or e,   the  tr eatm ent  of   a  pl eur al   ef f usi on
depends  on  the  i nf or m ati on  gl eaned  dur i ng  an  appr opr i ate  cl i ni cal
ev al uati on  of   the  pati ent  as  a  w hol e.

Case
A  43­y ear ­ol d  m an  w i th  l ong­standi ng  ser oposi ti v e  r heum atoi d  ar thr i ti s
pr esents  to  the  em er gency   r oom   com pl ai ni ng  of   r i ght  pl eur i ti c  chest  pai n.
He  w as  star ted  on  pr edni sone
P. 358
by   hi s  phy si ci an  1  w eek   ear l i er   f or   an  acute  f l ar e­up  of   sy nov i ti s  i n  hi s
w r i sts  and  hands.   For   sev er al   day s  bef or e  the  onset  of   the  pl eur i ti c  pai n,
the  pati ent  noted  m al ai se,   anor ex i a,   and  f ev er s.   The  ni ght  bef or e
pr esentati on,   he  noted  the  onset  of   shar p,   nonr adi ati ng  pai n  i n  the  r i ght
chest,   w hi ch  w or sened  w i th  coughi ng  or   deep  br eathi ng.   Hi s  cough  i s
nonpr oducti v e.   He  deni es  ci gar ette  or   al cohol   use.
On  phy si cal   ex am i nati on,   he  i s  f ound  to  be  i n  m oder ate  di str ess  because  of
hi s  chest  pai n.   Hi s  bl ood  pr essur e  i s  120/70  m m   Hg,   pul se  i s  120  beats  per
m i nute  and  r egul ar ,   the  r espi r ator y   r ate  i s  20  br eaths  per   m i nute,   and
tem per atur e  i s  38°C  (100. 4°F).   Hi s  phy si cal   ex am i nati on  i s  r em ar k abl e
f or   good  denti ti on;  a  nor m al   jugul ar   v enous  pr essur e;  a  r egul ar   tachy car di a
w i thout  m ur m ur s,   gal l ops,   or   r ubs;  and  no  per i pher al   edem a.   Lung
ex am i nati on  r ev eal s  dul l ness  to  per cussi on  at  the  r i ght  base  w i th  absent
br eath  sounds  i n  that  ar ea.   Egophony   i s  pr esent  at  the  r i ght  base  but
w hi sper i ng  pector i l oquy   i s  absent.   The  l ef t  l ung  i s  cl ear   ex cept  f or   a  sm al l
ar ea  of   decr eased  br eath  sounds  at  the  base.
A  com pl ete  bl ood  count  r ev eal s  a  m i l d  nor m ocy ti c  anem i a  w i th  a  hem ogl obi n
l ev el   of   12. 5  g/dL.   The  w hi te  bl ood  cel l   count  i s  13, 000/m m 3   w i th  an
i ncr ease  i n  the  num ber   of   band  f or m s.   A  chest  r adi ogr aph  show s  a  m oder ate
r i ght  pl eur al   ef f usi on,   a  sm al l   l ef t  pl eur al   ef f usi on,   and  a  nor m al   car di ac
si l houette.
Thor acentesi s  of   the  r i ght  pl eur al   ef f usi on  y i el ds  250  m L  of   y el l ow ,   sl i ghtl y
cl oudy   f l ui d.   The  w hi te  bl ood  cel l   count  i n  the  f l ui d  i s  3, 500/m m 3   w i th  90%
neutr ophi l s.   The  r ed  bl ood  cel l   count  i s  1, 000/m m 3 .   The  pr otei n  l ev el   i s  4. 0
g/dL,   the  LDH  l ev el   i s  400  IU ,   the  gl ucose  content  i s  10  m g/dL,   and  the  pH
i s  7. 12.

1.   What  i s  the  m ost  l i k el y   cause  of   the  r i ght  pl eur i ti c  chest  pai n?


2.   What  f ur ther   tests  w oul d  y ou  do  to  v er i f y   y our   di agnosi s?
3.   How   w oul d  y ou  m anage  thi s  pati ent's  acute  pr obl em ?
4.   What  ar e  the  i ntr athor aci c  m ani f estati ons  of   r heum atoi d  ar thr i ti s?

Case Discussion
1.   What  i s  the  m ost  l i k el y   cause  of   the  r i ght  pl eur i ti c  chest  pai n?

The  pr esentati on  consi sti ng  of   f ev er ,   acute  pl eur i ti c  pai n,   and  an
ex udati v e  pl eur al   ef f usi on  w i th  a  pr edom i nance  of   neutr ophi l s  i s  m ost
consi stent  w i th  a  bacter i al   i nf ecti on  (em py em a)  of   the  pl eur al   space,   or
em py em a.   Em py em a  i s  usual l y   the  r esul t  of   a  pneum oni a  that  ex tends
to  and  i nv ol v es  the  adjacent  pl eur a.   The  pl eur al   f l ui d  m ay   hav e  a  l ow
gl ucose  l ev el   and  usual l y   has  a  l ow   pH  (< 7. 30).   The  m ost  com m on
causes  i ncl ude  anaer obi c  bacter i a  (of ten  r esul ti ng  f r om   an  aspi r ati on
pneum oni a),   Staphy l ococcus  aur eus,   pneum ococcus,   and  tuber cul osi s.
In  i m m unocom pr om i sed  hosts,   the  di f f er enti al   di agnosi s  i ncl udes  f ungi
and  other   oppor tuni sti c  pathogens.

In  thi s  pati ent,   the  di agnosi s  of   pul m onar y   em bol i sm   w i th  subsequent
pul m onar y   i nf ar cti on  cannot  be  ex cl uded.   Thi s  can  pr esent  w i th  f ev er
and  pl eur i ti c  pai n  as  w el l .   How ev er ,   the  pl eur al   f l ui d  i s  usual l y   bl oody
as  a  r esul t  of   l ocal   ti ssue  i nf ar cti on.   In  addi ti on,   hem opty si s  m ay   be  an
associ ated  f i ndi ng.   Thi s  pati ent's  ef f usi on  had  a  l ow   r ed  bl ood  cel l
count  and  he  had  no  r i sk   f actor s  f or   pul m onar y   em bol i ,   such  as
pr ol onged  bed  r est  or   r ecent  tr aum a.

P. 359
Rheum atoi d  i nv ol v em ent  of   the  pl eur a  i s  com m on.   Most  pati ents  w i th
r heum atoi d  ar thr i ti s  hav e  a  pl eur al   ef f usi on  at  som e  ti m e  dur i ng  the
cour se  of   the  di sease.   The  ty pi cal   char acter i sti cs  of   the  f l ui d  ar e  an
ex udati v e  w i th  a  l ow   gl ucose  l ev el   and  a  hi gh  r heum atoi d  f actor   l ev el .
The  pr esentati on  i s  subacute  w hen  the  ef f usi on  becom es  l ar ge  enough
to  cause  sy m ptom s  or ,   i n  m ost  cases,   i s  noted  on  ex am i nati on  or   a
r adi ogr aphi c  study   i n  an  asy m ptom ati c  pati ent.

Constr i cti v e  per i car di ti s  m ay   be  a  r ar e  consequence  of   r heum atoi d


i nv ol v em ent  of   the  per i car di um .   How ev er ,   i t  i s  usual l y   m ani f ested  by
r i ght­si ded  congesti on  w i th  an  el ev ated  jugul ar   v enous  pr essur e
(usual l y   w i th  a  Kussm aul 's  si gn),   hepatom egal y ,   and  per i pher al   edem a.

2.   What  f ur ther   tests  w oul d  y ou  do  to  v er i f y   y our   di agnosi s?

Gr am 's  stai ni ng  and  cul tur e  of   the  pl eur al   f l ui d  i n  a  pati ent  w i th
em py em a  i s  i m por tant  to  i denti f y   a  causati v e  or gani sm   and  di r ect  the
choi ce  of   anti bi oti c  ther apy .   The  Gr am 's  stai n  f i ndi ngs  ar e  of ten
posi ti v e  and  can  nar r ow   the  di f f er enti al   di agnosi s  and  the  anti bi oti c
choi ces.   When  tuber cul ous  em py em a  i s  suspected,   aci d­f ast  stai ni ng  of
a  sam pl e  of   pl eur al   f l ui d  i s  i ndi cated.   Pl eur al   ti ssue  cul tur e  i s  opti m al .
Subsequent  cul tur es  and  sensi ti v i ti es  ar e  usef ul   i n  adjusti ng  dr ug
ther apy .   In  the  ev ent  of   em py em a  caused  by   anaer obi c  bacter i a,   the
Gr am 's  stai n  f i ndi ngs  ar e  m or e  of ten  than  not  negati v e,   and  cul tur es
m ay   y i el d  negati v e  r esul ts  unl ess  the  f l ui d  i s  car ef ul l y   handl ed  and
pr ocessed  anaer obi cal l y .   When  anaer obi c  em py em a  i s  suspected,
em pi r i c  ther apy   i s  of ten  necessar y   i f   an  or gani sm   cannot  be  i denti f i ed.

A  CT  angi ogr am   i s  i ndi cated  i n  cases  of   suspected  pul m onar y   em bol i
and  w i l l   show   segm ental   or   subsegm ental   f i l l i ng  def ects  i n  pul m onar y
ar ter i es.

Deter m i nati on  of   the  r heum atoi d  f actor   l ev el   i n  the  pl eur al   space  can
be  per f or m ed  to  ascer tai n  w hether   a  pl eur al   ef f usi on  i n  a  pati ent  w i th
r heum atoi d  ar thr i ti s  i s  r el ated  to  the  under l y i ng  r heum atoi d  pr ocess.
How ev er ,   the  pr esence  of   a  hi gh  r heum atoi d  f actor   does  not  ex cl ude  a
secondar y   com pl i cati on  of   a  r heum atoi d  ef f usi on  such  as  i nf ecti on.

An  echocar di ogr am   i s  a  usef ul   w ay   to  assess  the  per i car di um   w hen
per i car di al   di sease,   such  as  constr i cti v e  per i car di ti s,   i s  suspected.

3.   How   w oul d  y ou  m anage  thi s  pati ent's  acute  pr obl em ?

The  ther apy   f or   em py em a  r equi r es  appr opr i ate  anti bodi es  di r ected
tow ar d  the  k now n  or   pr esum ed  causati v e  or gani sm ,   or   or gani sm s.   In
pati ents  w i th  bacter i al   em py em as,   tr adi ti onal   ther apy   al so  i nv ol v es
chest  tube  pl acem ent  and,   som eti m es,   sur gi cal   dr ai nage  of   the  pl eur al
space  because  thi s  essenti al l y   r epr esents  an  abscess  cav i ty   and
anti bi oti c  ther apy   al one  i s  usual l y   i nef f ecti v e.   Dr ai nage  i s  not  done  i n
the  ev ent  of   tuber cul ous  em py em a,   how ev er .   The  ther apy   f or   thi s  i s
pr ol onged  (6  to  12  m onths)  tr eatm ent  w i th  anti tuber cul ous  dr ugs.   Som e
cl i ni ci ans  hav e  r ecom m ended  that,   i n  the  ev ent  of   pneum ococcal
em py em a,   anti bi oti c  ther apy   al one  i s  of ten  ef f ecti v e  w hen  the  f l ui d  i s
not  l ocul ated  and  the  pati ent  i s  cl i ni cal l y   doi ng  w el l .   In  thi s  pati ent,   the
l ow   pH  of   the  f l ui d  and  si gni f i cant  pl eur i ti c  pai n  w oul d  pr om pt
pl acem ent  of   a  sm al l   per cutaneous  catheter   i nto  the  r i ght  pl eur al   space
to  dr ai n  the  em py em a  com pl etel y .   Per i car di ocentesi s  i s  the  tr eatm ent
f or   per i car di al   tam ponade,   but  i s  not  i ndi cated  f or   constr i cti v e
per i car di ti s.   Pr edni sone  can  be  adm i ni ster ed  to
P. 360
suppr ess  a  f l ar e­up  of   r heum atoi d  di sease  i nv ol v i ng  the  joi nts,   al though
i t  does  not  appear   to  hav e  any   ef f ect  on  pl eur al   or   per i car di al
i nv ol v em ent.

4.   What  ar e  the  i ntr athor aci c  m ani f estati ons  of   r heum atoi d  ar thr i ti s?

Rheum atoi d  ar thr i ti s  i s  a  sy stem i c  i l l ness.   As  al r eady   noted,   m ost


pati ents  w i l l   hav e  pl eur al   i nv ol v em ent.   Per i car di al   i nv ol v em ent  i s  l ess
com m on  and,   f or tunatel y ,   i s  r ar el y   cl i ni cal l y   si gni f i cant.   Pul m onar y
nodul es  can  al so  f or m ,   par ti cul ar l y   i n  pati ents  w i th  r heum atoi d  nodul es
on  thei r   ex tr em i ti es.   These  nodul es  ar e  si m i l ar   hi stol ogi cal l y   to  the
per i pher al   nodul es,   they   w ax   and  w ane  w i th  the  i ntensi ty   of   the
sy stem i c  di sease,   and  ar e  r ar el y   si gni f i cant  cl i ni cal l y .   In  addi ti on  to
these  m ani f estati ons,   r heum atoi d  di sease  can  cause  an  ILD—ei ther
usual   i nter sti ti al   pneum oni a,   N SIP,   or gani zi ng  pneum oni a,   or   acute
i nter sti ti al   pneum oni a.   Rar el y ,   br onchi ol i ti s  obl i ter ans  m ay   occur   i n
r heum atoi d  ar thr i ti s.   Thi s  i s  char acter i zed  by   a  pr ogr essi v e,
i r r ev er si bl e  i l l ness  si m i l ar   to  em phy sem a  i n  i ts  r adi ogr aphi c
appear ance  and  phy si ol ogi c  abnor m al i ti es,   and  i s  usual l y   f atal   w i thi n  5
y ear s  of   pr esentati on.

Suggested Readings
Ham m   H,   Li ght  RW.   Par apneum oni c  ef f usi on  and  em py em a.   Eur   Respi r   J
1997;10:1150–1156.

Pulmonary Complications of Human
Immunodeficiency Virus Infection
1.   What  ar e  the  pul m onar y   com pl i cati ons  i n  a  pati ent  w i th  hum an
i m m unodef i ci ency   v i r us  (HIV)  i nf ecti on?

2.   What  tests  w oul d  hel p  y ou  establ i sh  a  speci f i c  di agnosi s?


Discussion
1.   What  ar e  the  pul m onar y   com pl i cati ons  i n  a  pati ent  w i th  HIV  i nf ecti on?

HIV  i s  a  l y m photr opi c  r etr ov i r us  that  i nf ects  T4  (hel per )  l y m phocy tes,
B  cel l s,   and  m onocy tes,   l eadi ng  to  a  def ect  i n  cel l ­m edi ated  i m m uni ty ,
w hi ch  then  pr edi sposes  to  the  dev el opm ent  of   a  v ar i ety   of   neopl asm s
and  oppor tuni sti c  i nf ecti ons.   The  l ung  i s  one  of   the  pr i m ar y   tar get
or gans  i n  HIV  di sease,   and  pul m onar y   com pl i cati ons  ar e  the  l eadi ng
cause  of   hospi tal i zati on  and  death  i n  HIV­i nf ected  pati ents.   The
spectr um   of   pul m onar y   di sor der s  associ ated  w i th  HIV  i nf ecti on  i ncl udes
both  i nf ecti ous  and  noni nf ecti ous  di seases.   The  i nf ecti ous  causes  of
pul m onar y   di sease  i ncl ude  both  oppor tuni sti c  and  nonoppor tuni sti c
agents.   The  m ost  com m on  oppor tuni sti c  or gani sm s  ar e  Pneum ocy sti s
ji r ov eci ,   cy tom egal ov i r us,   and  My cobacter i um   av i um ­i ntr acel l ul ar e
(MAI).   Al though  oppor tuni sti c  i nf ecti ons  ar e  com m on  i n  HIV­i nf ected
pati ents,   these  pati ents  ar e  al so  m or e  suscepti bl e  to  nonoppor tuni sti c
i nf ecti ons,
P. 361
i ncl udi ng  py ogeni c  or gani sm s  (S.   pneum oni ae  and  H.   i nf l uenzae),
My cobacter i um   tuber cul osi s,   and  f ungal   i nf ecti ons.   The  speci f i c
i nf ecti on  the  pati ent  acqui r es  depends  on  the  degr ee  of   i m m une
def i ci ency   and  hi s  or   her   ex posur e  to  speci f i c  or gani sm s.   The
noni nf ecti ous  pul m onar y   di sor der s  i ncl ude  Kaposi 's  sar com a,   nonâ
€“Hodgk i n's  l y m phom a,   l y m phocy ti c  i nter sti ti al   pneum oni ti s,
nonspeci f i c  i nter sti ti al   pneum oni a,   al v eol ar   pr otei nosi s,   br onchi ol i ti s
obl i ter ans  or gani zi ng  pneum oni a,   pr i m ar y   pul m onar y   hy per tensi on,   and
em phy sem a.

2.   What  tests  w oul d  hel p  y ou  establ i sh  a  speci f i c  di agnosi s?

Chest  i m agi ng,   ar ter i al   bl ood  gas  deter m i nati ons,   a  com pl ete  bl ood
count,   ser um   LDH  m easur em ent,   and  sputum   studi es  shoul d  al l   be
obtai ned  i n  an  HIV­i nf ected  pati ent  pr esenti ng  w i th  f ev er   and
i ncr easi ng  shor tness  of   br eath.   Al though  the  chest  r adi ogr aph  or   CT
scan  can  be  hel pf ul ,   the  r adi ogr aphi c  m ani f estati ons  of   pul m onar y
di sease  ov er l ap  si gni f i cantl y   i n  thi s  gr oup  of   pati ents.   Most  pati ents
w i th  Pneum ocy sti s  pneum oni a  (PCP)  ar e  hy pox i c  and  hy pocapni c,   and
ex hi bi t  a  w i dened  al v eol ar ­ar ter i al   gr adi ent,   of ten  bef or e  any
abnor m al i ty   i s  detected  on  a  chest  r adi ogr aph.   The  com pl ete  bl ood
count  i n  HIV­i nf ected  pati ents  ty pi cal l y   dem onstr ates  an  absol ute
l y m phopeni a,   w hi ch  pr i m ar i l y   stem s  f r om   a  decr ease  i n  the  num ber   of
T4  cel l s.   The  LDH  l ev el   i s  el ev ated  i n  95%  of   the  pati ents  and  has  been
show n  to  i ncr ease  w i th  w or seni ng  sy m ptom s  and  decl i ne  i n  r esponse  to
ther apy .   Sputum   i n  pati ents  w i th  py ogeni c  bacter i al   i nf ecti ons  i s  of ten
pur ul ent,   and  the  Gr am 's  stai n  and  cul tur e  f i ndi ngs  shoul d  di ctate  the
anti bi oti c  choi ce.   Pati ents  w i th  PCP  of ten  hav e  a  nonpr oducti v e  cough,
m ak i ng  i t  necessar y   to  obtai n  a  sputum   speci m en  that  i s  i nduced  by   the
i nhal ati on  of   hy per toni c  sal i ne.   Sputum   and  bl ood  cul tur es  ar e  al so
i ndi cated  f or   MAI  i nf ecti on  and  tuber cul osi s.   Thi s  has  a  y i el d  of   25%  to
85%,   dependi ng  on  the  ex per i ence  of   the  per son  per f or m i ng  the  test.
Methenam i ne  si l v er   stai ni ng  i s  used  to  i denti f y   the  cy sts.   If   sputum
studi es  ar e  unr ev eal i ng,   stai ni ng  of   br onchoal v eol ar   cel l s  (BAL)  has  a
hi gher   posi ti v i ty   r ate.

Case
A  37­y ear ­ol d  hom osex ual   m an,   k now n  to  be  HIV  posi ti v e,   i s  seen  f or
ev al uati on  of   pr ogr essi v e  dy spnea  and  f ev er .   He  w as  w el l   unti l   7  to  8
m onths  ago,   w hen  he  noted  the  onset  of   w ei ght  l oss,   di f f use  adenopathy ,
and  ni ght  sw eats.   Ov er   the  l ast  m onth,   he  has  noted  pr ogr essi v e  dy spnea,   a
dr y ,   nonpr oducti v e  cough,   and  dai l y   f ev er   spi k es.   He  has  sm ok ed  20
ci gar ettes  a  day   f r om   the  age  of   18  y ear s,   deni es  al cohol   or   dr ug  abuse,
and  has  l i v ed  i n  the  Ohi o  Ri v er   Val l ey   as  w el l   as  the  Southw est.   Phy si cal
ex am i nati on  r ev eal s  a  bl ood  pr essur e  of   110/65  m m   Hg,   pul se  of   100  beats
per   m i nute,   r espi r ator y   r ate  of   32  br eaths  per   m i nute,   and  tem per atur e  of
39°C  (102. 2°F).   Thr oat  ex am i nati on  f i ndi ngs  ar e  r em ar k abl e  f or   a  w hi te
ex udate  on  the  poster i or   phar y ngeal   w al l .   The  l y m ph  nodes  ar e  di f f usel y
enl ar ged  and  nontender .   Dur i ng  chest  ex am i nati on,   bi l ater al   cr ack l es  ar e
noted.   The  r em ai nder   of   the  ex am i nati on  f i ndi ngs  ar e  unr em ar k abl e.   A  chest
r adi ogr aph  r ev eal s  the  pr esence  of   di f f use  bi l ater al   i nter sti ti al   i nf i l tr ates.

1.   What  i s  the  di f f er enti al   di agnosi s  i n  thi s  pati ent,   and  do  the  chest
r adi ogr aph  f i ndi ngs  i nf l uence  thi s?
P. 362
2.   If   the  i ni ti al   test  r esul ts  do  not  conf i r m   y our   di agnosi s,   w hat  test  w oul d
y ou  do  nex t?
3.   What  ther apy   w oul d  y ou  i ni ti ate,   and  i s  ther e  a  r ol e  f or   pr ophy l acti c
ther apy ?

Case Discussion
1.   What  i s  the  di f f er enti al   di agnosi s  i n  thi s  pati ent,   and  do  the  chest
r adi ogr aph  f i ndi ngs  i nf l uence  thi s?

Al though  m al i gnancy   and  a  chr oni c  i nf ecti ous  pr ocess  such  as


tuber cul osi s  coul d  account  f or   these  chr oni c  and  subacute  sy m ptom s,
thi s  pati ent's  cl i ni cal   pi ctur e  i s  m ost  consi stent  w i th  a  com pl i cati on  of
HIV  i nf ecti on.   Besi des  the  spectr um   of   pul m onar y   di sor der s  associ ated
w i th  HIV  i nf ecti ons,   pr i m ar y   car di ac  di sor der s  shoul d  al so  be
consi der ed  w hen  dy spnea  and  bi l ater al   r al es  ar e  encounter ed.   The
f ev er   and  chi l l s  i n  thi s  pati ent  i ndi cate  an  i nf ecti ous  cause.   The
di f f er enti al   di agnosi s  i n  thi s  pati ent  w oul d  i ncl ude  pneum ocy sti s,
tuber cul osi s,   and  i nf ecti on  w i th  nonoppor tuni sti c  pul m onar y   pathogens,
such  as  S.   pneum oni ae,   H.   i nf l uenzae,   and  S.   aur eus.
As  al r eady   m enti oned,   the  chest  r adi ogr aph  can  be  hel pf ul   i n  m ak i ng
the  di agnosi s,   al though  ther e  i s  si gni f i cant  ov er l ap  i n  the  r adi ogr aphi c
m ani f estati ons  of   the  v ar i ous  pul m onar y   di seases.   In  PCP,   the  m ost
com m on  f i ndi ng  i s  a  di f f use  i ncr ease  i n  the  i nter sti ti al   and  al v eol ar
m ar k i ngs,   al though  nodul ar   i nf i l tr ates,   cav i ti es,   pneum atocel es  and
pneum othor aces,   and  pl eur al   ef f usi ons  hav e  al l   been  obser v ed  i n  thi s
setti ng.   In  addi ti on,   20%  of   pati ents  w i th  PCP  can  hav e  a  nor m al
r adi ogr aph.   Kaposi 's  sar com a  of ten  pr esents  w i th  nodul ar   i nf i l tr ates,
w i th  or   w i thout  a  pl eur al   ef f usi on.   The  ef f usi on  i s  ei ther
ser osangui neous  or   hem or r hagi c  and  i s  due  to  pl eur al   i nv ol v em ent  by
the  sar com a.   The  pr esence  of   i ntr athor aci c  adenopathy   suggests
tuber cul osi s,   non–Hodgk i n's  l y m phom a,   Kaposi 's  sar com a,   or   MAI
i nf ecti on.

2.   If   the  i ni ti al   test  r esul ts  do  not  conf i r m   y our   di agnosi s,   w hat  test  w oul d
y ou  do  nex t?

If   the  sputum   speci m en  f i ndi ngs  ar e  nondi agnosti c,   the  nex t  di agnosti c
pr ocedur e  w oul d  be  f i ber opti c  br onchoscopy   w i th  br onchoal v eol ar
l av age  and,   i n  som e  cases,   a  tr ansbr onchi al   bi opsy .   Thi s  al l ow s  the
al v eol ar   ti ssue  to  be  di r ectl y   sam pl ed.   Br onchoal v eol ar   l av age  i s
per f or m ed  by   pl aci ng  the  br onchoscope  i n  the  di stal   ai r w ay ,   i nsti l l i ng
100  to  200  m L  of   sal i ne  i nto  the  ai r w ay ,   and  then  i m m edi atel y
r em ov i ng  the  sol uti on.   The  l av age  techni que  has  a  y i el d  of   75%  to  95%
i n  the  setti ng  of   PCP,   and  tr ansbr onchi al   bi opsy   has  a  y i el d  of   85%  to
95%.   Kaposi 's  sar com a,   how ev er ,   i s  di f f i cul t  to  di agnose  usi ng
br onchoscopy .   If   br onchoscopi c  f i ndi ngs  ar e  nondi agnosti c,   an  open
l ung  bi opsy   shoul d  be  done.   Thi s  i nv ol v es  an  open  thor acotom y   and  i s
r ar el y   needed.

3.   What  ther apy   w oul d  y ou  i ni ti ate,   and  i s  ther e  a  r ol e  f or   pr ophy l acti c
ther apy ?

Tr eatm ent  w i th  tr i m ethopr i m –sul f am ethox azol e  shoul d  be  star ted  i n
conjuncti on  w i th  i ntr av enous  cor ti coster oi ds.   The  r ol e  of   pr ophy l acti c
ther apy   i s  under   i nv esti gati on.   It  al so  appear s  that  pr ophy l acti c  or al
ther apy   w i th  tr i m ethopr i m –sul f am ethox azol e  i s  hi ghl y   ef f i caci ous  i n
pr ev enti ng  r ecur r ence  of   PCP.   How ev er ,   m any   pati ents  ar e  unabl e  to
endur e  pr ol onged  ther apy   because  of   adv er se  si de  ef f ects.   Aer osol i zed
pentam i di ne  i s  al so  used  f or   PCP  pr ophy l acti c  tr eatm ent,   and  has  been
show n  to  be  ef f ecti v e.

P. 363

Suggested Readings
Hopew el l   PC,   Luce  JM.   Pul m onar y   m ani f estati ons  of   the  acqui r ed
i m m unodef i ci ency   sy ndr om e.   Cl i n  Im m unol   Al l er gy   1986;6:489.
Mur r ay   JF,   Mi l l s  J.   Pul m onar y   i nf ecti ous  com pl i cati ons  of   hum an
i m m unodef i ci ency   v i r us  i nf ecti on:  par t  I.   Am   Rev   Respi r   Di s
1990;141:1356.

Mur r ay   JF,   Mi l l s  J.   Pul m onar y   i nf ecti ous  com pl i cati ons  of   hum an
i m m unodef i ci ency   v i r us  i nf ecti on:  par t  II.   Am   Rev   Respi r   Di s
1990;141:1582.

Solitary Pulmonary Nodule
1.   What  i s  a  sol i tar y   pul m onar y   nodul e  (SPN )?

2.   What  per centage  of   SPN s  i s  beni gn?

3.   What  cl i ni cal   and  r adi ol ogi c  f i ndi ngs  ar e  associ ated  w i th  a  hi gher
i nci dence  of   m al i gnancy   i n  an  SPN ?

4.   What  i s  the  m ost  com m on  cause  of   an  SPN ?

Discussion
1.   What  i s  an  SPN ?

Sol i tar y   pul m onar y   nodul es  ar e  si ngl e  opaci ti es  l ocated  enti r el y   w i thi n
the  l ung  par enchy m a  and  usual l y   l ess  than  4  cm   i n  di am eter .   They   ar e
not  associ ated  w i th  atel ectasi s  or   hi l ar   adenopathy   on  pl ai n  chest
r oentgenogr am s.

2.   What  per centage  of   SPN s  i s  beni gn?

Sev enty ­f i v e  to  85%  of   SPN s  ar e  beni gn,   and  15%  to  25%  ar e
m al i gnant  (ei ther   pr i m ar y   or   m etastati c  di sease).   The  phy si ci an's  r ol e
i s  to  ex pedi te  the  w or k up  and  r esecti on  of   potenti al l y   cur abl e
m al i gnant  SPN s,   w hi l e  av oi di ng  costl y   ev al uati ons  and  pai nf ul
thor acotom i es  f or   SPN s  that  ar e  beni gn  or   al r eady   unr esectabl e  (i . e. ,
m etastati c).

3.   What  cl i ni cal   and  r adi ol ogi c  f i ndi ngs  ar e  associ ated  w i th  a  hi gher
i nci dence  of   m al i gnancy   i n  an  SPN ?

Ther e  ar e  sev er al   f eatur es  that  suggest  m al i gnancy :

Age .   Most  SPN s  i n  adul ts  y ounger   than  35  y ear s  ar e  beni gn.   The
r i sk   of   m al i gnant  di sease  i ncr eases  w i th  i ncr easi ng  age.

Nodule  s ize .   Mor e  than  80%  of   the  SPN s  l ar ger   than  3  cm   i n


di am eter   ar e  m al i gnant;  20%  or   f ew er   of   the  SPN s  l ess  than  2  cm
i n  di am eter   ar e  m al i gnant.   Spi cul ati on  on  CT  scan  of   the  chest  i s
m or e  l i k el y   m al i gnant.
P re s e nc e  a nd pa tte rn of c a lc ific a tion.   Cal ci f i cati on,   par ti cul ar l y
that  w i th  a  centr al ,   l am i nated,   or   di f f use  patter n,   i s  suggesti v e  of
beni gn  di sease.   Mal i gnant  di sease  onl y   r ar el y   show s  ev i dence  of
cal ci f i cati on,   and  m or e  f r equentl y   ex hi bi ts  an  eccentr i c  patter n.

His tory of prior ma ligna nc y.   As  m any   as  30%  of   m al i gnant  SPN s


ar e  m etastases  f r om   ex tr athor aci c  m al i gnanci es.

Smok ing his tory.   Al though  the  ef f ect  of   sm ok i ng  on  m al i gnancy


i n  the  setti ng  of   SPN s  has  not  been  speci f i cal l y   deter m i ned,   ther e
i s  a
P. 364
w el l ­k now n  associ ati on  betw een  sm ok i ng  and  the  dev el opm ent  of
pr i m ar y   br onchogeni c  car ci nom a.   Thi s  r i sk   i s  si m i l ar   to  that  of   the
gener al   nonsm ok i ng  popul ati on  betw een  10  and  15  y ear s  af ter
sm ok i ng  cessati on.

4.   What  i s  the  m ost  com m on  cause  of   an  SPN ?

Mor e  than  hal f   of   al l   SPN s  ar e  f ound  to  be  gr anul om as  on  pathol ogi c
ex am i nati on.   Ham ar tom as  r epr esent  the  nex t  m ost  com m on  beni gn
cause,   but  consti tute  l ess  than  10%  of   al l   SPN s.

Case
A  40­y ear ­ol d  w om an  i s  seen  f or   a  pr eoper ati v e  ev al uati on  bef or e
under goi ng  l apar oscopi c  chol ecy stectom y .   Her   m edi cal   hi stor y   i s  r em ar k abl e
f or   m i l d  untr eated  hy per tensi on.   She  has  under gone  no  pr ev i ous  sur gi cal
pr ocedur es.   She  had  a  5­pack ­y ear   hi stor y   of   sm ok i ng,   but  qui t  8  y ear s  ago.
She  w as  bor n  and  r ai sed  i n  Ci nci nnati .   She  w or k s  as  a  par al egal   i n  a
dow ntow n  l aw   f i r m .   She  tak es  no  m edi cati ons  other   than  an  occasi onal
aspi r i n  f or   headache.
A  thor ough  r ev i ew   r ev eal s  sy m ptom s  r ef er abl e  to  her   chol el i thi asi s.
Speci f i cal l y ,   she  deni es  any   sy stem i c  or   chest  com pl ai nts,   i ncl udi ng  f ev er ,
m al ai se,   w ei ght  change,   m y al gi as  or   ar thr al gi as,   chest  pai n,   shor tness  of
br eath  or   dy spnea  on  ex er ti on,   cough,   and  hem opty si s.
Phy si cal   ex am i nati on  r ev eal s  a  m i l dl y   obese  w om an  i n  no  di str ess.   Vi tal
si gns  ar e  nor m al   ex cept  f or   a  bl ood  pr essur e  of   170/90  m m   Hg.   Head  and
neck   f i ndi ngs  ar e  nor m al ,   and  her   l ungs  ar e  cl ear .   Hear t  f i ndi ngs  ar e  al so
nor m al .   Abdom i nal   ex am i nati on  r ev eal s  r i ght  upper   quadr ant  tender ness  i n
r esponse  to  deep  pal pati on  w i thout  r ebound  or   guar di ng.   Ther e  ar e  no
m asses  or   hepatospl enom egal y .   Stool   i s  guai ac  negati v e.   Her   ex tr em i ti es
ar e  nor m al ,   as  ar e  the  f i ndi ngs  f r om   a  thor ough  neur ol ogi c  ex am i nati on.
Ther e  i s  no  adenopathy .
Labor ator y   ex am i nati on  f i ndi ngs,   i ncl udi ng  a  com pl ete  bl ood  count,   r outi ne
chem i str i es,   ar ter i al   bl ood  gases  m easur em ent,   and  ur i nal y si s,   ar e  w i thi n
nor m al   l i m i ts.   An  el ectr ocar di ogr am   i s  nor m al .   A  chest  r adi ogr aphi c  study
r ev eal s  a  1. 8­cm ,   r ound  opaci ty   i n  the  l ef t  l ow er   l obe,   but  i s  other w i se
nor m al .
1.   What  i s  the  m ost  i m por tant  nex t  di agnosti c  step  at  thi s  ti m e?
2.   What  ar e  the  m ajor   possi bl e  di agnoses  of   thi s  pati ent's  nodul e?
3.   What  noni nv asi v e  di agnosti c  test  m ay   hel p  i n  di sti ngui shi ng  betw een
the  possi bl e  di agnoses  i n  thi s  pati ent?
4.   If   the  test  r esul ts  up  to  thi s  poi nt  hav e  been  nondi agnosti c  or
i ndeter m i nate,   w hat  opti ons  shoul d  be  pr esented  to  the  pati ent  at  thi s
ti m e?

Case Discussion
1.   What  i s  the  m ost  i m por tant  nex t  di agnosti c  step  at  thi s  ti m e?

If   possi bl e,   an  ol d  chest  r adi ogr aphi c  study   shoul d  be  obtai ned.   If   the
nodul e  w as  pr esent  and  i s  unchanged  i n  si ze  on  a  study   f r om   at  l east  2
y ear s  bef or e,   i t  i s  v er y   l i k el y   that  thi s  r epr esents  a  beni gn  l esi on,   and
no  f ur ther   w or k up  i s  necessar y .   Mal i gnant  l esi ons  usual l y   hav e  a
doubl i ng  ti m e  of   w eek s  to  m onths.   In  other   w or ds,   a  l esi on  that  gr ow s
ei ther   v er y   r api dl y   (day s)  or   v er y   sl ow l y   (y ear s)  i s  l i k el y   to  be  beni gn.
P. 365
In  the  ev ent  of   v er y   r api d  gr ow th,   the  pati ent  usual l y   has  other
pul m onar y   sy m ptom s  consi stent  w i th  a  beni gn  di agnosi s,   such  as
i nf ecti on  or   pul m onar y   i nf ar cti on.

2.   What  ar e  the  m ajor   possi bl e  di agnoses  of   thi s  pati ent's  nodul e?

The  di f f er enti al   di agnosi s  f or   thi s  pati ent  i ncl udes  gr anul om atous
di sease,   br onchogeni c  car ci nom a,   ham ar tom a,   pul m onar y   m etastasi s
f r om   an  unk now n  pr i m ar y   tum or ,   and  “r oundâ€​   pneum oni a.

Inf ecti ous  gr anul om atous  di seases  r esul ti ng  i n  SPN s  i ncl ude
hi stopl asm osi s,   cocci di oi dom y cosi s,   and  tuber cul osi s.   Gr anul om as  can
al so  appear   as  SPN s  i n  the  setti ngs  of   sar coi dosi s,   r heum atoi d
ar thr i ti s,   and  v ascul i ti des  such  as  Wegener 's  gr anul om atosi s.   Pr i m ar y
br onchogeni c  car ci nom a  i s  the  m ost  f r equent  sour ce  of   r esected
m al i gnant  SPN s.   Metastati c  di sease,   of ten  or i gi nati ng  f r om   pr i m ar y
adenocar ci nom as  of   the  br east,   pr ostate,   or   col on,   f r equentl y   pr esent
as  SPN s.   “Roundâ€​   pneum oni a  i s  an  uncom m on  pr esentati on  of   an
acute  pul m onar y   i nf ecti on  i n  w hi ch  the  al v eol ar   space­f i l l i ng  di sease
assum es  a  m or e  r ounded,   nodul ar   appear ance.   In  the  absence  of   other
si gns  or   sy m ptom s,   thi s  can  be  conf used  w i th  an  SPN .   Less  com m on
causes  of   SPN s  i ncl ude  ar ter i ov enous  m al f or m ati ons,   br onchogeni c
cy sts,   pul m onar y   i nf ar cti on,   and  par asi ti c  di sease.

3.   What  noni nv asi v e  di agnosti c  test  m ay   hel p  di sti ngui sh  betw een  the
possi bl e  di agnoses  i n  thi s  pati ent?

HRCT  i s  i ndi cated  f or   thi s  pati ent.   SPN s  ar e  of ten  f ound  to  be  m ul ti pl e
on  CT,   suggesti ng  the  pr esence  of   ei ther   gr anul om atous  di sease  or
pul m onar y   m etastases.   Less  than  1%  of   pr i m ar y   l ung  cancer s  pr esent
as  m ul ti pl e  and  sy nchr onous  l esi ons.   CT  i s  al so  v er y   sensi ti v e  i n
def i ni ng  the  densi ty   and  conf i gur ati on  of   an  SPN .   The  f i ndi ng  of   a  f at
densi ty   i n  the  nodul e  str ongl y   suggests  the  di agnosi s  of   ham ar tom a.
Centr al ,   l am i nated,   or   di f f use  patter ns  of   cal ci f i cati on  al so  suggest  a
beni gn  di agnosi s  (par ti cul ar l y   gr anul om atous  di sease  or   ham ar tom a),
w her eas  eccentr i c  cal ci f i cati on  can  be  f ound  i n  ei ther   beni gn  or
m al i gnant  di sease.   Less  hel pf ul   i s  the  conf i gur ati on  of   the  SPN .   Poor l y
m ar gi nated  or   spi cul ated  nodul es  ar e  of ten  m al i gnant,   but  w el l ­
m ar gi nated  spher i cal   nodul es  can  be  ei ther   beni gn  or   m al i gnant.
Ipsi l ater al   m edi asti nal   or   hi l ar   adenopathy   (def i ned  by   m ost
r adi ol ogi sts  as  l y m ph  nodes  > 1  cm   i n  tr ansv er se  di am eter )  can  be
associ ated  w i th  ei ther   beni gn  or   m al i gnant  l esi ons.   How ev er ,
adenopathy   i nv ol v i ng  the  hem i thor ax   contr al ater al   to  the  SPN   i s  hi ghl y
suggesti v e  of   nonr esectabl e  m al i gnant  di sease.

4.   If   the  test  r esul ts  up  to  thi s  poi nt  hav e  been  nondi agnosti c  or
i ndeter m i nate,   w hat  opti ons  shoul d  be  pr esented  to  the  pati ent  at  thi s
ti m e?

Ther e  ar e  thr ee  opti ons  at  thi s  poi nt.

Obs e rva tion  i s  appr opr i ate  i n  m any   pati ents,   par ti cul ar l y   those  w i th  a
v er y   l ow   l i k el i hood  of   m al i gnancy   or   those  f or   w hom   an  i nv asi v e
di agnosti c  pr ocedur e  w oul d  car r y   an  unacceptabl y   hi gh  r i sk   of
m or bi di ty   and  m or tal i ty .   The  cour se  of   the  SPN   can  be  m oni tor ed  w i th
ser i al   i m agi ng  ev er y   3  m onths  f or   the  f i r st  y ear ,   and  ev er y   6  m onths
f or   the  second  y ear .

Biops y  can  be  per f or m ed  usi ng  ei ther   CT  or   f l uor oscopy ­gui ded
tr ansthor aci c  f i ne­needl e  aspi r ati on  (FN A)  or   f i ber opti c  br onchoscopy
w i th  tr ansbr onchi al   bi opsy .   The  l atter   pr ocedur e  i s  associ ated  w i th  a
l ow er   di agnosti c  y i el d,   par ti cul ar l y   f or   sm al l   (< 2  cm )  per i pher al   SPN s.
In  any   ev ent,   i f   the  di agnosi s  i s  not  establ i shed,   m or e
P. 366
aggr essi v e  attem pts  to  obtai n  def i ni ti v e  ti ssue  m ust  be  pur sued.
N ondi agnosti c  ti ssue  f i ndi ngs  shoul d  not  be  constr ued  as  ev i dence  of   a
beni gn  l esi on.

Surgic a l lung biops y  i s  a  thi r d  opti on.   Thi s  has  the  adv antage  of   bei ng
both  a  di agnosti c  and  a  ther apeuti c  pr ocedur e.   It  i s  al so  associ ated
w i th  hi gher   m or bi di ty .   Many   sur geons  per f or m   m edi asti noscopi c  l y m ph
node  bi opsy   bef or e  open  thor acotom y ,   especi al l y   i n  cases  of   CT­pr ov en
m edi asti nal   adenopathy ,   to  av oi d  the  m or e  ex tensi v e  pr ocedur e  i f
possi bl e.

Suggested Readings
Li l l i ngton  GA,   Cask ey   CI.   Ev al uati on  and  m anagem ent  of   sol i tar y   and
m ul ti pl e  pul m onar y   nodul es.   Cl i n  Chest  Med  1993;14:111.
Mi dthun  DE,   Sw ensen  SJ,   Jett  JR.   Cl i ni cal   str ategi es  f or   sol i tar y
pul m onar y   nodul e.   Annu  Rev   Med  1992;43:195.

Mi dthun  DE,   Sw ensen  SJ,   Jett  JR.   Appr oach  to  the  sol i tar y   pul m onar y
nodul e.   May o  Cl i n  Pr oc  1993;68:378.

Webb  WR.   Radi ol ogi c  ev al uati on  of   the  sol i tar y   pul m onar y   nodul e.   AJR
Am   J  Roentgenol   1990;154:701.

Acute Pulmonary Embolism
1.   What  i s  a  pul m onar y   em bol i sm ?

2.   What  ar e  the  com m on  sour ces  of   pul m onar y   em bol i ?

3.   What  ar e  the  r i sk   f actor s  f or   pul m onar y   em bol i ?

4.   Ar e  al l   acute  pul m onar y   em bol i   si m i l ar ?

Discussion
1.   What  i s  a  pul m onar y   em bol i sm ?

A  pul m onar y   em bol i sm   r esul ts  f r om   the  m i gr ati on  of   v enous  thr om bi
f r om   the  sy stem i c  v ei ns  to  pul m onar y   ar ter i al   sy stem ,   r esul ti ng  i n
v ar y i ng  degr ees  of   obstr ucti on  of   pul m onar y   ar ter i al   bl ood  f l ow .   The
i nci dence  of   pul m onar y   em bol i   i n  the  U ni ted  States  ex ceeds  500, 000
per   y ear ,   w i th  a  m or tal i ty   appr oachi ng  10%.   If   not  di agnosed  or   i f
i m pr oper l y   tr eated,   the  m or tal i ty   r ate  can  r each  30%.

2.   What  ar e  the  com m on  sour ces  of   pul m onar y   em bol i ?

U p  to  90%  of   pul m onar y   em bol i   or i gi nate  f r om   the  deep  v enous  sy stem
of   the  l egs.   The  upper   ex tr em i ti es  can  al so  be  a  sour ce  of   v enous
thr om bi .   U sual l y   r el ated  to  tr aum a,   congeni tal   f i br om uscul ar   bands,   or
the  use  of   centr al   v enous  catheter s,   12%  of   al l   upper   ex tr em i ty
thr om bi   r esul t  i n  pul m onar y   em bol i .   In  addi ti on,   bl ood  cl ot  f or m ati on  i n
the  pel v i c  v ei ns  m ay   cause  ei ther   septi c  or   bl and  pul m onar y   em bol i ,
especi al l y   i n  the  setti ng  of   com pl i cated  obstetr i c  pr ocedur es  or
gy necol ogi c  sur ger y .

Other   causes  of   pul m onar y   ar ter i al   obstr ucti v e  em bol i   i ncl ude  ai r
i ntr oduced  dur i ng  i ntr av enous  i njecti ons,   hem odi al y si s,   or   the
pl acem ent
P. 367
of   centr al   v enous  catheter s;  am ni oti c  f l ui d  secondar y   to  v i gor ous
uter i ne  contr acti ons;  f at  as  a  r esul t  of   m ul ti pl e  l ong  bone  f r actur es;
par asi tes;  tum or   cel l s;  or   i njected  f or ei gn  m ater i al   (tal c,   m er cur y ).

Table 8­2 Risk Factors for Venous
Thrombosis

Sta s is Hype rc oa gula bility Endothe lia l


Injury

Congesti v e Def i ci ency   of Ex tensi v e  pel v i c


hear t  f ai l ur e anti thr om bi n  III sur ger y

Obesi ty Def i ci ency   of   pr otei ns Pr i or   i njur y


C  and  S

Pr ol onged  bed Mal i gnanci es Tr aum a


r est

Pr ol onged Or al   contr acepti v es


 
tr av el

Pr esence  of   a  l upus
   
anti coagul ant

  Factor   V  Lei den  


def i ci ency

3.   What  ar e  the  r i sk   f actor s  f or   pul m onar y   em bol i ?

Thr ee  basi c  r i sk   f actor s,   k now n  col l ecti v el y   as  Vi r chow 's  tr i ad,   ar e
associ ated  w i th  thr om bus  f or m ati on  and  subsequent  pul m onar y   em bol i :
stasi s,   hy per coagul abi l i ty ,   and  endothel i al   i njur y .   Most  cl i ni cal   r i sk
f actor s  ar e  der i v ed  f r om   one  of   these  pathogeni c  m echani sm s,   and
these  ar e  l i sted  i n  Tabl e  8­2.

4.   Ar e  al l   acute  pul m onar y   em bol i   si m i l ar ?

Pul m onar y   em bol i   pr oduce  sev er al   cl i ni cal   sy ndr om es.   The  r ar est,
a c ute  ma s s ive  oc c lus ion,   i s  def i ned  as  an  em bol us  that  occl udes
enough  of   the  pul m onar y   ci r cul ati on  to  pr oduce  ci r cul ator y   col l apse.   In
pati ents  w ho  do  not  sur v i v e  thi s  ev ent,   autopsy   r ev eal s  occl usi on,
usual l y   at  the  bi f ur cati on  of   the  m ai n  pul m onar y   ar ter y ,   and  the
f or m ati on  of   saddl e  em bol i .  P ulmona ry infa rc tion  r ef er s  to  an
em bol i sm   that  obstr ucts  enough  bl ood  f l ow   to  a  por ti on  of   the  l ung,
causi ng  l oss  of   v i abi l i ty   of   the  l ung  ti ssue.   Thi s  occur s  i n  10%  of   cases
of   acute  pul m onar y   em bol i sm .   The  thi r d  and  m ost  com m on  cl i ni cal
occur r ence  i s  pulmona ry e mbolis m w ithout infa rc tion.   These  ar e  the
m ost  di f f i cul t  to  di agnose  because  they   m i m i c  other   pul m onar y   and
car di ac  condi ti ons.   Because  m ost  em bol i   ar e  m ul ti pl e,   both  i nf ar cted
and  noni nf ar cted  ar eas  i n  the  l ung  can  coex i st.

Case
A  27­y ear ­ol d  w om an  pr esents  to  the  em er gency   r oom   af ter   24  hour s  of
r i ght­si ded  chest  pai n,   w hi ch  i s  w or se  w i th  i nspi r ati on.   She  i s  shor t  of
br eath  and  anx i ous.   The  pati ent  deni es  sputum   pr oducti on,   hem opty si s,
cough  or   w heezi ng  but  states  that  she  f el t  w ar m   at  hom e  but  di d  not  tak e
her   tem per atur e.   She  deni es  any   r ecent  i njur y   or   sw el l i ng  of   her   l egs,   and
i s  a  v er y   acti v e  per son.   The  pati ent  has  no  pr i or   hi stor y   of   l ung  or   hear t
di sease.
She  tak es  or al   contr acepti v es,   and  has  no  k now n  dr ug  al l er gi es.   She  has
under gone  no  sur gi cal   pr ocedur es.
P. 368
She  sm ok es  one  pack   of   ci gar ettes  per   day ,   and  does  not  consum e  al cohol .
She  deni es  i ntr av enous  dr ug  use  and  has  no  r i sk   f actor s  f or   HIV  di sease.
She  w or k s  as  an  accountant.   Her   f am i l y   hi stor y   i s  negati v e  f or   asthm a  and
hear t  di sease.
Phy si cal   ex am i nati on  r ev eal s  a  m i l dl y   obese  w om an  i n  m oder ate  r espi r ator y
di str ess.   Her   tem per atur e  i s  38. 0°C  (100. 4°F),   her   pul se  i s  115  beats  per
m i nute,   her   bl ood  pr essur e  i s  140/80  m m   Hg,   and  her   r espi r ator y   r ate  i s  26
br eaths  per   m i nute.   N o  jugul ar   v enous  di stenti on  i s  obser v ed.   Her   chest  i s
cl ear .
Car di ac  ex am i nati on  r ev eal s  r egul ar   r ate  and  r hy thm ,   w i th  nor m al   i ntensi ty
of   the  f i r st  and  second  hear t  sounds.   Ther e  ar e  no  thi r d  or   f our th  sounds,
m ur m ur s,   or   r ubs.   Abdom i nal   ex am i nati on  r ev eal s  posi ti v e  bow el   sounds
and  no  hepatospl enom egal y .   Her   ex tr em i ti es  show   no  cy anosi s,   cl ubbi ng,   or
edem a.
Her   l abor ator y   v al ues  ar e  as  f ol l ow s:  hem ogl obi n,   14. 5  g/dL;  hem atocr i t,
42%;  w hi te  bl ood  cel l s,   6, 000/m m 3   w i th  74%  segm ented  neutr ophi l s  and
26%  l y m phocy tes.   Peak   ex pi r ator y   f l ow   i s  450  L  per   m i nute,   w hi ch  i s
nor m al .
A  chest  r adi ogr aphi c  study   r ev eal s  a  nor m al   car di ac  si l houette  and  cl ear
l ung  f i el ds,   ex cept  f or   a  sm al l   per i pher al   i nf i l tr ate  i n  the  l ow er   l ef t  l obe.   An
el ectr ocar di ogr am   show s  si nus  tachy car di a  w i thout  i schem i c  changes.
Ar ter i al   bl ood  gas  m easur em ent  per f or m ed  on  r oom   ai r   r ev eal s  a  pH  of
7. 49,   a  PCO 2   of   32  m m   Hg,   a  PO 2   of   60  m m   Hg,   and  an  al v eol ar ­ar ter i al
ox y gen  gr adi ent  of   40  m m   Hg.
1.   What  i s  the  di f f er enti al   di agnosi s?
2.   What  addi ti onal   tests  shoul d  be  done  to  hel p  nar r ow   the  di f f er enti al
di agnosi s?
3.   How   do  y ou  i nter pr et  the  addi ti onal   test  r esul ts?
4.   What  i s  the  nex t  step  i n  di agnosi ng  an  acute  pul m onar y   em bol i sm ?
5.   What  i s  the  acute  m anagem ent  of   pul m onar y   em bol i sm ?
6.   How   l ong  shoul d  anti coagul ati on  ther apy   be  conti nued?
7.   What  r ol e  w oul d  thr om bol y ti c  ther apy   hav e  i n  thi s  pati ent?
8.   When  shoul d  a  v ena  cav al   f i l ter   be  pl aced?

Case Discussion
1.   What  i s  the  di f f er enti al   di agnosi s?

The  di f f er enti al   di agnosi s  i n  thi s  y oung  w om an  w i th  acute  onset  of


shor tness  of   br eath  and  chest  pai n  i s  l engthy .   N ot  al l   br eathi ng
di sor der s  ar e  due  to  pul m onar y   di sease  because  i schem i c  car di ac
di sease  can  pr esent  w i th  dy spnea  w hen  associ ated  w i th  l ef t  v entr i cul ar
f ai l ur e.   How ev er ,   the  natur e  and  l ocati on  of   the  pai n,   the  l ack   of
substanti al   car di ac  r i sk   f actor s,   and  the  pati ent's  age  m ak e  car di ac
i schem i a  unl i k el y .

Sev er al   pul m onar y   di sor der s  can  hav e  a  si m i l ar   pr esentati on.   Pati ents
w i th  acute  bacter i al   pneum oni a  com pl ai n  of   shor tness  of   br eath,   l ow ­
gr ade  f ev er ,   and  chest  pai n.   How ev er ,   pneum oni a  al so  usual l y   causes
sputum   pr oducti on  and  an  el ev ated  w hi te  bl ood  cel l   count,   w hi ch  w er e
not  pr esent  i n  thi s  pati ent.   Asthm a  can  al so  pr esent  i nsi di ousl y   w i th
acute  shor tness  of   br eath.   How ev er ,   the  l ack   of   a  hi stor y   of   asthm a,
ex posur e  to  k now n  tr i gger s  of   asthm a,   and  a  nor m al   peak   ex pi r ator y
f l ow   m ak e  thi s  di agnosi s  unl i k el y .   A  spontaneous  pneum othor ax   can
cause  sy m ptom s,   y et  i t  w oul d  be  unusual   f or   thi s  to  be  accom pani ed  by
a  l ow ­gr ade  f ev er .

P. 369
The  m ost  com m on  sy m ptom s  of   pul m onar y   em bol i   i ncl ude  shor tness  of
br eath,   pl eur i ti c  pai n,   cough,   and  hem opty si s.   Pul m onar y   em bol i   shoul d
al w ay s  be  consi der ed  i n  a  pati ent  w i th  acute  shor tness  of   br eath  and  a
k now n  r i sk   f actor   f or   thr om bosi s  (or al   contr acepti v es).   Pl eur i ti c  chest
pai n,   as  seen  i n  thi s  pati ent,   and  hem opty si s  occur   onl y   i f   the
em bol i sm   causes  a  pul m onar y   i nf ar cti on.   Fev er s  as  hi gh  as  39°C
(102. 2°F)  hav e  al so  been  r epor ted  i n  the  setti ng  of   i nf ar cti on  or   a
concur r ent  i nf ecti on.   On  phy si cal   ex am i nati on,   the  m ost  com m on  si gn
i s  i sol ated  si nus  tachy car di a;  how ev er ,   i n  those  pati ents  w i th  m assi v e
em bol i sm ,   ev i dence  of   acute  r i ght  v entr i cul ar   f ai l ur e  m ay   be  f ound.
When  consi der i ng  pul m onar y   em bol i sm ,   ther e  ar e  no  uni v er sal   cl i ni cal
f i ndi ngs  and  the  absence  of   speci f i c  f i ndi ngs  does  not  ex cl ude  the
di agnosi s.

2.   What  addi ti onal   tests  shoul d  be  done  to  hel p  nar r ow   the  di f f er enti al
di agnosi s?

To  hel p  di f f er enti ate  betw een  the  v ar i ous  di agnoses,   a  chest
r adi ogr aphi c  study ,   el ectr ocar di ogr am ,   ar ter i al   bl ood  gas  anal y si s,   and
Gr am 's  stai ni ng  of   a  sputum   sam pl e  shoul d  be  done.

3.   How   do  y ou  i nter pr et  the  addi ti onal   test  r esul ts?

In  the  setti ng  of   an  acute  pul m onar y   em bol i sm ,   the  ar ter i al   bl ood  gas
m easur em ent  cl assi cal l y   r ev eal s  a  l ow   PCO 2 ,   l ow   PO 2 ,   and  a  w i dened
al v eol ar ­ar ter i al   ox y gen  gr adi ent.   How ev er ,   m any   other   di sor der s
cause  si m i l ar   abnor m al   ar ter i al   bl ood  gas  r esul ts  and  10%  to  15%  of
pati ents  w i th  pr ov en  pul m onar y   em bol i   m ai ntai n  a  nor m al   al v eol ar ­
ar ter i al   ox y gen  gr adi ent.

The  chest  r adi ogr aph  f i ndi ngs  of   pul m onar y   em bol i   ar e  nonspeci f i c.
Ty pi cal l y ,   i nf i l tr ates,   atel ectasi s,   ef f usi ons,   or   any   com bi nati on  of
these  ar e  encounter ed.   It  i s  not  usual   f or   the  chest  r adi ogr aph  to  be
nor m al .   A  per i pher al   w edge­shaped  i nf i l tr ate,   som eti m es  r ef er r ed  to  as
a  Ham pton's  hum p,   occur s  w hen  the  em bol i sm   i s  associ ated  w i th
i nf ar cti on,   and  occasi onal l y   decr eased  pul m onar y   v ascul ar   m ar k i ngs
ar e  noted  (Wester m ar k 's  si gn),   i ndi cati v e  of   decr eased  bl ood  f l ow   to  a
secti on  of   the  l ung.

The  el ectr ocar di ogr am   i s  hel pf ul   i n  r ul i ng  out  i schem i c  hear t  di sease.
In  pati ents  w i th  pul m onar y   em bol i ,   the  el ectr ocar di ogr am   usual l y
dem onstr ates  si nus  tachy car di a  or   i s  nor m al .   Onl y   i n  the  pr esence  of
m assi v e  em bol i zati on  i s  a  r i ght  ax i s  dev i ati on  and  an  S 1 ,   Q 3 ,   T 3
patter n  seen.

These  test  r esul ts  hel p  nar r ow   the  di f f er enti al   di agnosi s.   The  chest
r adi ogr aph  f i ndi ngs  r ul e  out  a  pneum othor ax ,   and  a  tr ue  bacter i al
pneum oni a  i s  l ess  l i k el y   i n  l i ght  of   the  nor m al   sputum   f i ndi ngs.   The
l ack   of   i schem i a  on  the  el ectr ocar di ogr am   m ak es  a  pr i m ar y   car di ac
abnor m al i ty   unl i k el y .   Wi th  the  pr esentati on  of   shor tness  of   br eath,   a
w i dened  al v eol ar ­ar ter i al   ox y gen  gr adi ent,   and  chest  r adi ogr aph
f i ndi ngs  consi stent  w i th  an  i nf ar cti on,   a  pul m onar y   em bol i sm   i s  now   the
m ost  l i k el y   di agnosi s.

4.   What  i s  the  nex t  step  i n  di agnosi ng  an  acute  pul m onar y   em bol i sm ?

CT  angi ogr aphy   i s  i ndi cated  and  i f   posi ti v e  show s  f i l l i ng  def ects  i n
l ar ge­  and  m edi um ­si zed  pul m onar y   ar ter i es.   Venti l ati on/per f usi on
scans  ar e  now   r eser v ed  f or   pati ents  w ho  cannot  tol er ate  a  dy e  l oad  due
to  r enal   i nsuf f i ci ency   or   hav e  a  k now n  i odi ne  al l er gy .   Measur em ent  of
the  ser um   D­di m er ,   a  f i br i n  degr adati on  pr oduct  that  dem onstr ates  a
l ev el   bel ow   500  µg/L,   ex cl udes  the  di agnosi s  of   pul m onar y   em bol i sm .
Pul m onar y   angi ogr aphy   i s  r ar el y   i ndi cated.
P. 370
If   the  CT  angi ogr am   i s  i nconcl usi v e  and  the  suspi ci on  sti l l   hi gh  Doppl er
v enous  studi es  of   the  l ow er   ex tr em i ti es,   i f   posi ti v e,   m ay   substanti ate
the  need  f or   anti coagul ati on.

5.   What  i s  the  acute  m anagem ent  of   pul m onar y   em bol i sm ?

The  goal   of   ther apy   i s  to  pr ev ent  f ur ther   em bol i c  epi sodes,   and  hepar i n
i s  the  i ni ti al   dr ug  of   choi ce  f or   accom pl i shi ng  thi s.   Fi r st,   a  l ar ge
i ntr av enous  l oadi ng  bol us  shoul d  be  gi v en,   f ol l ow ed  by   conti nuous­dr i p
i nf usi on,   m ai ntai ned  f or   at  l east  5  and  of ten  7  to  10  day s.
Anti coagul ati on  shoul d  not  be  w i thhel d  pendi ng  the  r esul ts  of   f ur ther
studi es  unl ess  the  pati ent's  r i sk   of   bl eedi ng  com pl i cati ons  i s  gr eater
than  the  cl i ni cal   suspi ci on  of   pul m onar y   em bol i .   The  par ti al
thr om bopl asti n  ti m e  shoul d  be  m oni tor ed  and  the  hepar i n  dosage
adjusted  to  k eep  the  ti m e  betw een  1. 5  to  2. 0  ti m es  the  contr ol .

War f ar i n  i s  star ted  24  to  48  hour s  af ter   hepar i n  ther apy   has  been
i ni ti ated.   Dur i ng  the  f i r st  3  day s  of   w ar f ar i n  ther apy ,   the  pr othr om bi n
ti m e  or   IN R  i s  i ncr eased  bef or e  the  onset  of   tr ue  anti coagul ati on.
Ther ef or e,   bef or e  di sconti nui ng  the  hepar i n,   the  pr othr om bi n  ti m e  or
IN R  shoul d  be  ther apeuti c  (1. 5  to  2  ti m es  nor m al )  f or   appr ox i m atel y   2
to  3  day s.   Low ­m ol ecul ar ­w ei ght  hepar i ns  ar e  i ndi cated  f or   pr ophy l ax i s
i n  postoper ati v e  pati ents  and  pr obabl y   hav e  a  r ol e  i n  the  m anagem ent
of   acute  pul m onar y   em bol i sm   and  deep  v enous  thr om bosi s  because
they   do  not  r equi r e  m oni tor i ng  of   the  anti coagul ati on  ef f ects.

6.   How   l ong  shoul d  anti coagul ati on  ther apy   be  conti nued?

Long­ter m   anti coagul ati on  i s  usual l y   achi ev ed  w i th  w ar f ar i n,   al though
l ow ­m ol ecul ar ­w ei ght  hepar i ns  can  al so  be  used.   Pati ents  w i th
r ev er si bl e  r i sk   f actor s  that  ar e  subsequentl y   el i m i nated  shoul d  under go
anti coagul ati on  f or   a  total   of   3  m onths.   If   thi s  i s  an  i ni ti al   epi sode  of
em bol i sm   and  the  pati ent  has  no  cl ear   r i sk   f actor s,   tr eatm ent  shoul d
pr obabl y   be  m ai ntai ned  f or   3  to  6  m onths.   Fi nal l y ,   those  pati ents  w i th
r ecur r ent  em bol i   and  nonr ev er si bl e  r i sk   f actor s  (e. g. ,   adenocar ci nom a,
anti phosphol i pi d  sy ndr om e,   or   f actor   V  Lei den  def i ci ency )  shoul d  be
tr eated  f or   l i f e.   When  i t  i s  uncer tai n  how   l ong  to  m ai ntai n  ther apy ,
i m pedance  pl ethy sm ogr aphy   can  hel p  i n  i denti f y i ng  r ecur r ent  deep  v ei n
thr om bosi s.

7.   What  r ol e  w oul d  thr om bol y ti c  ther apy   hav e  i n  thi s  pati ent?

The  r ol e  of   thr om bol y ti c  agents  (str eptok i nase,   ur ok i nase,   and  ti ssue
pl asm i nogen  acti v ator )  i s  y et  to  be  el uci dated  i n  the  m anagem ent  of
acute  pul m onar y   em bol i sm .   Ther e  appear   to  be  no  si gni f i cant
di f f er ences  betw een  the  thr ee  agents  i n  the  tr eatm ent  of   pul m onar y
em bol i ,   ex cept  f or   thei r   r especti v e  costs.   Thr om bol y ti c  agents  do
accel er ate  the  r esol uti on  of   the  pul m onar y   ar ter y   cl ot,   but  they   hav e
not  been  cl ear l y   show n  to  i m pr ov e  sur v i v al   as  com par ed  w i th  the
r esul ts  obser v ed  f or   conv enti onal   hepar i n  ther apy .   The  onl y   adopted
use  of   these  agents  i s  f or   pati ents  w i th  m assi v e  em bol i sm   and  sy stem i c
hy potensi on.   When  used,   thr om bol y ti c  ther apy   m ust  be  f ol l ow ed  by   a
standar d  cour se  of   hepar i n.

8.   When  shoul d  a  v ena  cav al   f i l ter   be  pl aced?

The  pur pose  of   v ena  cav al   f i l ter s  i s  both  to  tr ap  em bol i   and  m ai ntai n
the  patency   of   the  i nf er i or   v ena  cav a.   These  f i l ter s  ar e  l ar gel y   v i ew ed
as  an  al ter nati v e  ther apy   f or   thr om boem bol i sm   w hen  anti coagul ati on  i s
unacceptabl e.   The  thr ee  m ost  com m on  i ndi cati ons  f or   f i l ter   pl acem ent
ar e  (a)  a  contr ai ndi cati on  to  anti coagul ati on,
P. 371
(b)  f ai l ur e  of   pr oper   anti coagul ati on  to  pr ev ent  the  f or m ati on  of   f ur ther
em bol i ,   and  (c)  a  com pl i cati on  of   anti coagul ati on  ther apy .

Suggested Readings
Fi shm an  AP,   Kel l ey   MA.   Pul m onar y   thr om boem bol i sm   (i ncl udi ng
pr ophy l ax i s,   tr eatm ent,   si ck l e  cel l   di sease,   and  m ul ti pl e  pul m onar y
thr om bi ).   In:  Fi shm an  AP,   ed.   Pul m onar y   di seases  and  di sor der s,   2nd  ed.
N ew   Yor k :  McGr aw ­Hi l l ,   1987.

Hur ew i tz  AN ,   Ber gof sk y   EH.   Pul m onar y   em bol i sm .   In:  Cher ni ack   RM,   ed.
Cur r ent  ther apy   of   r espi r ator y   di sease.   Tor onto:  BC  Deck er ,   1989:259.

Per r i er   A,   Desm ar i s  S,   Goehr i ng  C,   et  al .   D­di m er   testi ng  f or   suspected


pul m onar y   em bol i sm   i n  outpati ents.   Am   J  Respi r   Cr i t  Car e  Med
1997;136:492.

PIOPED  Inv esti gator s.   Val ue  of   the  v enti l ati on/per f usi on  scan  i n  acute
pul m onar y   em bol i sm :  r esul ts  of   the  pr ospecti v e  i nv esti gati on  of
pul m onar y   em bol i sm   di agnosi s  (PIOPED).   JAMA  1990;263:2753.

Sarcoidosis
1.   What  ar e  the  sy m ptom s  and  si gns  of   sar coi dosi s?

2.   What  tests  ar e  used  to  establ i sh  the  di agnosi s?

3.   What  ar e  the  ther apeuti c  opti ons?

Discussion
1.   What  ar e  the  sy m ptom s  and  si gns  of   sar coi dosi s?
Sar coi dosi s  i s  a  sy stem i c  di sor der   char acter i zed  hi stol ogi cal l y   by   the
pr esence  of   noncaseati ng  gr anul om as.   The  gr anul om as  can  be  f ound  i n
any   ti ssue,   such  as  the  l ung,   sk i n,   m y ocar di um ,   centr al   ner v ous
sy stem ,   and  k i dney s.   The  sy m ptom s  and  si gns  m ost  com m onl y   seen
stem   f r om   the  i nv ol v em ent  of   the  r eti cul oendothel i al   sy stem   and  the
l ung.   Pati ents  m ay   pr esent  w i th  one  or   m or e  of   the  f ol l ow i ng:  f ati gue;
a  pi gm ented  papul onodul ar   sk i n  r ash;  spl enom egal y ;  ar thr i ti s;  and
chest  r adi ogr aphi c  f i ndi ngs  i ndi cati ng  bi l ater al   hi l ar   adenopathy   or
patchy   nodul ar   pul m onar y   i nf i l tr ates,   or   both.   Labor ator y   abnor m al i ti es
i ncl ude  anem i a,   l euk openi a,   hy per cal cem i a,   el ev ati on  of   the  l i v er
enzy m e  l ev el s  i n  a  chol estati c  patter n,   and  a  pol y cl onal   gam m opathy .

2.   What  tests  ar e  used  to  establ i sh  the  di agnosi s?

The  m ost  def i ni ti v e  test  to  establ i sh  the  di agnosi s  of   sar coi dosi s  i s
ti ssue  bi opsy .   Si tes  f or   bi opsy   i ncl ude  the  sk i n  (i f   a  r ash  ex i sts)  or
l ung.   The  sensi ti v i ty   of   br onchoscopy   w i th  tr ansbr onchi al   bi opsy
ex ceeds  90%  i n  obtai ni ng  noncaseati ng  gr anul om as  i n  pati ents  w i th
sar coi dosi s  w ho  pr esent  w i th  hi l ar   adenopathy   and  pul m onar y
i nf i l tr ates.   How ev er ,   noncaseati ng  gr anul om as  ar e  onl y   suggesti v e,   but
not  pathognom oni c,   ev i dence  f or   the  di sease.   Other   di seases  that
pr oduce  gr anul om as,   such  as  m y cobacter i al   and  f ungal   di seases,   m ust
al so  be  consi der ed.   These  enti ti es  can  be  r ul ed  out  by   br onchoscopy
w i th  bi opsy   and  br onchoal v eol ar   l av age.

P. 372
The  ser um   l ev el   of   the  angi otensi n­conv er ti ng  enzy m e  (ACE)  i s
el ev ated  i n  som e  pati ents  w i th  sar coi dosi s,   but  thi s  i s  nei ther   a
sensi ti v e  nor   speci f i c  enough  f i ndi ng  f or   i t  to  ser v e  as  a  di agnosti c
test.   Thi s  l ev el   i s  el ev ated  i n  appr ox i m atel y   66%  of   pati ents  w i th
sar coi dosi s,   but  al so  occur s  i n  a  v ar i ety   of   di sor der s  such  as
tuber cul osi s,   cocci di oi dom y cosi s,   hy per thy r oi di sm ,   and  di abetes
m el l i tus.   How ev er ,   the  ACE  l ev el   has  been  show n  to  decr ease  w i th
ther apy ,   and  thi s  m ay ,   ther ef or e,   be  a  usef ul   objecti v e  m easur e  f or
m oni tor i ng  the  ef f ecti v eness  of   tr eatm ent.

3.   What  ar e  the  ther apeuti c  opti ons?

Sar coi dosi s  i s  a  v er y   heter ogeneous  di sease,   w i th  appr ox i m atel y   one
thi r d  of   pati ents  i m pr ov i ng  w i thout  tr eatm ent,   one  thi r d  pr ogr essi ng
cl i ni cal l y ,   and  one  thi r d  r em ai ni ng  i n  r el ati v el y   stabl e  condi ti on.
U nf or tunatel y ,   ther e  i s  no  r el i abl e  w ay   to  pr edi ct  i n  w hi ch  gr oup
pati ents  w i l l   f al l .   Factor s  that  suggest  an  unf av or abl e  pr ognosi s  i ncl ude
ex tensi v e  pul m onar y   par enchy m al   i nv ol v em ent,   r estr i cti v e  phy si ol ogy
on  pul m onar y   f uncti on  testi ng,   an  el ev ated  ACE  l ev el ,   i nv ol v em ent  of   at
l east  thr ee  or gan  sy stem s,   and  bl ack   r ace.   Or gan  i nv ol v em ent  that
m andates  the  i nsti tuti on  of   ther apy   i ncl udes  ey e,   centr al   ner v ous
sy stem ,   or   car di ac  i nv ol v em ent,   as  w el l   as  hy per cal cem i a.

Tr eatm ent  consi sts  of   cor ti coster oi ds,   usual l y   pr edni sone  or   i ts  equi v al ent
i ni ti ated  at  a  dosage  of   30  to  40  m g  per   day .   Ther e  i s  no  ev i dence  that  any
one  ster oi d  pr epar ati on  i s  super i or   to  another .   Inhal ed  cor ti coster oi ds  hav e
not  been  f ound  to  be  benef i ci al   i n  the  tr eatm ent  of   sar coi dosi s.   Betw een
80%  and  90%  of   pati ents  r espond  to  ster oi d  ther apy .   When  ef f ecti v e,   a
cl i ni cal   and  r adi ogr aphi c  r esponse  i s  usual l y   w i tnessed  w i thi n  2  to  4  w eek s.
Thi s  dosage  i s  usual l y   conti nued  f or   1  to  2  m onths,   then  gr adual l y   taper ed
ov er   the  cour se  of   the  nex t  1  to  6  m onths.   Many   pati ents  can  then
di sconti nue  tak i ng  ster oi ds,   but  other s  r equi r e  ongoi ng  ster oi d  ther apy   at  a
dosage  of   10  to  15  m g  dai l y   or   ev er y   other   day .   The  r esponse  to  ther apy   i s
conf i r m ed  by   sy m ptom ati c  and  r adi ogr aphi c  i m pr ov em ent,   suppor ted  by   a
decr easi ng  ACE  l ev el   and  stabl e  or   i m pr ov i ng  pul m onar y   f uncti on.   Al though
ther e  i s  sy m ptom ati c  and  r adi ogr aphi c  i m pr ov em ent  w i th  cor ti coster oi ds,
ther e  i s  l i ttl e  ev i dence  that  they   i nf l uence  the  natur al   cour se  of   the  di sease.

Case
A  34­y ear ­ol d  bl ack   w om an  i s  r ef er r ed  to  the  pul m onar y   cl i ni c  f or
ev al uati on  of   a  2­m onth  hi stor y   of   dr y   cough,   a  r ash  on  her   f or ehead  and
ar m s,   a  5­pound  (2. 25­k g)  w ei ght  l oss,   and  an  abnor m al   chest  r adi ogr aphi c
study .   She  has  an  8­pack ­y ear   sm ok i ng  hi stor y   and  a  hi stor y   of   pr i or
i ntr av enous  cocai ne  use,   and  6  m onths  ago  tr av el ed  to  Bak er sf i el d,
Cal i f or ni a,   f or   a  v acati on.
Phy si cal   ex am i nati on  r ev eal s  a  thi n  w om an  i n  no  di str ess.   Her   tem per atur e
i s  99°F  (37. 2°C),   pul se  i s  80  beats  per   m i nute,   bl ood  pr essur e  i s  110/70
m m   Hg,   and  r espi r ator y   r ate  i s  20  br eaths  per   m i nute.   A  pi gm ented,
papul onodul ar   r ash  i s  pr esent  on  her   f or ehead  and  upper   ar m s.   Funduscopi c
f i ndi ngs  ar e  nor m al .   Bi basi l ar   cr ack l es  ar e  hear d  on  chest  ex am i nati on.
Abdom i nal   ex am i nati on  r ev eal s  an  8­cm   l i v er   and  pal pabl e  spl een  ti p.   Ther e
i s  no  cy anosi s,   cl ubbi ng,   or   edem a  on  ex am i nati on  of   her   ex tr em i ti es.
P. 373
The  chest  r adi ogr aphi c  study   r ev eal s  bi l ater al   hi l ar   adenopathy   and  di f f use
al v eol ar   and  nodul ar   i nf i l tr ates.   Labor ator y   f i ndi ngs  ar e  as  f ol l ow s:  w hi te
bl ood  cel l   count,   4, 000/m m 3   w i th  70%  pol y m or phonucl ear   l euk ocy tes,   10%
m onocy tes,   2%  eosi nophi l s,   and  17%  l y m phocy tes;  hem ogl obi n,   11  g/dL;
hem atocr i t,   33%;  pl atel et  count,   300, 000/µL;  nor m al   ser um   el ectr ol y te
l ev el s;  cal ci um ,   10  m g/dL;  al bum i n,   3. 8  g/dL;  and  total   pr otei n,   8. 0  g/dL.

1.   What  i s  the  di f f er enti al   di agnosi s  i n  thi s  pati ent?


2.   What  tests  shoul d  be  done  to  establ i sh  the  di agnosi s  i n  thi s  pati ent?
Pul m onar y   f uncti on  testi ng  r ev eal s  the  f ol l ow i ng  l ung  v ol um es:  total
l ung  capaci ty ,   3. 32  L  (72%  of   pr edi cted);  thor aci c  gas  v ol um e,   1. 63  L
(64%  of   pr edi cted);  and  r esi dual   v ol um e,   0. 72  L  (59%  of   pr edi cted).
Spi r om etr y   show s  an  FVC  of   2. 72  L  (70%  of   pr edi cted)  and  FEV 1   of
2. 12  L  (75%  of   pr edi cted).   The  di f f usi ng  capaci ty   f or   car bon  m onox i de
(DLCO)  i s  15. 6  (46%  of   pr edi cted)  and  the  DLCO/al v eol ar   v enti l ati on
(VA)  i s  4. 97  (85%  of   pr edi cted).   Ar ter i al   bl ood  gas  m easur em ents  on
r oom   ai r   r ev eal   a  pH  of   7. 41,   PaCO 2   of   32  m m   Hg,   PaO 2   of   68  m m   Hg,
and  ox y gen  satur ati on  of   94%.
3.   How   w oul d  y ou  i nter pr et  the  r esul ts  of   the  PFTs?

Case Discussion
1.   What  i s  the  di f f er enti al   di agnosi s  i n  thi s  pati ent?

As  i s  tr ue  of   m any   pul m onar y   di sor der s,   the  r adi ogr aphi c  patter n
com bi ned  w i th  the  pati ent's  cl i ni cal   hi stor y   and  phy si cal   ex am i nati on
f i ndi ngs  nar r ow s  the  di f f er enti al   di agnosi s.   In  a  pati ent  w ho  pr esents
w i th  thi s  cl i ni cal   scenar i o  and  nor m al   cel l ul ar   i m m uni ty ,   the  di f f er enti al
di agnosi s  i s  br oad  and  i ncl udes  v ar i ous  i ndol ent  i nf ecti ous  pr ocesses
such  as  tuber cul osi s;  f ungal   i nf ecti ons  such  as  hi stopl asm osi s,
cocci di oi dom y cosi s,   and  Cr y ptococcus  neof or m ans  i nf ecti on;  i di opathi c
i m m unol ogi c  di sor der s  such  as  sar coi dosi s;  and,   l ess  com m onl y ,
m etastati c  neopl asti c  di sease,   Hodgk i n's  di sease,   non–Hodgk i n's
l y m phom a,   and  occupati onal   l ung  di seases  such  as  ber y l l i osi s  and
si l i cosi s.   How ev er ,   i n  a  pati ent  i nf ected  w i th  HIV,   the  di f f er enti al
di agnosi s  i ncl udes  a  hi gher   pr obabi l i ty   of   m y cobacter i al   i nf ecti on,
f ungal   i nf ecti on,   and  Kaposi 's  sar com a.

2.   What  tests  shoul d  be  done  to  establ i sh  the  di agnosi s  i n  thi s  pati ent?

A  ser um   HIV  test  shoul d  be  done  i n  thi s  pati ent  because  of   her   hi stor y
of   i ntr av enous  dr ug  abuse.   For   m ost  peopl e  w i th  possi bl e  sar coi dosi s,
how ev er ,   thi s  test  i s  not  necessar y .

A  sputum   sam pl e  shoul d  be  obtai ned  f or   aci d­f ast  stai ni ng  and
m y cobacter i al   cul tur e.   In  pati ents  w i th  ex tensi v e  pul m onar y   i nf i l tr ates
due  to  M.   tuber cul osi s  i nf ecti on,   thr ee  separ ate  m or ni ng  sputum
sam pl es  ar e  hi ghl y   sensi ti v e  f or   detecti ng  the  pathogen.   In  m ost
heal thy   hosts  w i th  acti v e  pul m onar y   tuber cul osi s,   the  PPD  (pur i f i ed
pr otei n  der i v ati v e)  sk i n  test  r esul t  i s  posi ti v e.   Pati ents  w i th  sar coi dosi s
ar e  f r equentl y   aner gi c  i n  r esponse  to  a  v ar i ety   of   sk i n  tests,   i ncl udi ng
the  PPD  test.

Sputum   speci m ens  f or   f ungal   stai ni ng  and  cul tur e  shoul d  al so  be
obtai ned.   The  f ungi   that  m i m i c  sar coi dosi s  ar e  r estr i cted  to  cer tai n
endem i c  ar eas.   For   i nstance,
P. 374
hi stopl asm osi s  i s  f ound  i n  the  m i dw ester n  and  southeaster n  U ni ted
States.   Cocci di oi dom y cosi s  i s  endem i c  to  the  deser t  southw est  and  ar i d
r egi ons  of   Cal i f or ni a,   such  as  the  Mojav e  Deser t  and  the  San  Joaqui n
Val l ey .   Gi v en  the  pati ent's  r ecent  tr i p  to  Bak er sf i el d,   Cal i f or ni a,   i t  i s
necessar y   to  ex cl ude  possi bl e  i nf ecti on  w i th  cocci di oi dom y cosi s.   A
thor ough  tr av el   and  occupati onal   hi stor y   shoul d  al w ay s  be  tak en  to
ex cl ude  any   aty pi cal   f ungal   ex posur e.

Ti ssue  shoul d  al so  be  obtai ned  f or   the  pur pose  of   ex cl udi ng  i nf ecti on
and  neopl asm ,   and  to  suppor t  the  di agnosi s  of   sar coi dosi s.   As  al r eady
di scussed,   sk i n  bi opsy   or   br onchoscopy   w i th  tr ansbr onchi al   bi opsy
w oul d  be  hel pf ul   i f   the  r esul ts  r ev eal ed  noncaseati ng  gr anul om as.   Aci d­
f ast  and  si l v er   stai ni ng  can  be  per f or m ed  on  the  bi opsy   speci m ens  to
ex cl ude  m y cobacter i al   and  f ungal   i nf ecti ons.   For   thi s  pati ent,   m ost
cl i ni ci ans  w oul d  r ecom m end  f i ber opti c  br onchoscopy   w i th
br onchoal v eol ar   l av age  and  tr ansbr onchi al   bi opsy .   In  the  setti ng  of
sar coi dosi s,   the  br onchoal v eol ar   l av age  f l ui d  char acter i sti cal l y   show s
an  i ncr eased  per centage  of   l y m phocy tes  w i th  a  pr edom i nant  CD4
phenoty pe.   In  addi ti on,   stai ns  and  cul tur es  f or   m y cobacter i al   and
f ungal   di seases  can  be  per f or m ed  on  the  br onchoal v eol ar   l av age  f l ui d.

3.   How   w oul d  y ou  i nter pr et  the  r esul ts  of   the  PFTs?

The  PFTs  show   r estr i cted  l ung  v ol um es.   Spi r om etr y   show s  a  m i l d
degr ee  of   obstr ucti on,   par ti cul ar l y   gi v en  the  under l y i ng  r estr i cti v e
phy si ol ogy .   The  DLCO  i s  r educed.

PFTs,   i ncl udi ng  l ung  v ol um es,   DLCO,   spi r om etr y ,   and  ar ter i al   bl ood  gas
m easur em ent,   shoul d  be  per f or m ed  f or   ev er y   pati ent  w i th  sar coi dosi s
and  pul m onar y   par enchy m al   i nv ol v em ent.   The  m ost  f r equent
abnor m al i ti es  encounter ed  ar e  a  r educti on  i n  l ung  v ol um es  (r estr i cti v e
phy si ol ogy ),   of ten  accom pani ed  by   a  r educti on  i n  DLCO.   It  i s  al so  not
uncom m on  to  f i nd  r educed  ex pi r ator y   f l ow   r ates,   i ndi cati ng  ai r w ay
i nv ol v em ent  w i th  sar coi dosi s.   The  ar ter i al   ox y gen  satur ati on  usual l y
r em ai ns  r el ati v el y   nor m al   at  r est,   unl ess  adv anced  di sease  i s  pr esent.
Wi th  ex er ci se,   the  PaO 2   f r equentl y   f al l s.

Suggested Readings
Gi l m an  MJ,   Wang  KP.   Tr ansbr onchi al   l ung  bi opsy   i n  sar coi dosi s.   Am   Rev
Respi r   Di s  1980;122:721.

Hi l l er dal   G,   N ou  E,   Oster m an  K,   et  al .   Sar coi dosi s:  epi dem i ol ogy   and
pr ognosi s:  a  15­y ear   Eur opean  study .   Am   Rev   Respi r   Di s  1984;130:29.

Rust  M,   Ber gm ann  L,   Kuhn  T.   Pr ognosti c  v al ue  of   chest  r adi ogr aph,
ser um   angi otensi n­conv er ti ng  enzy m e  and  T  hel per   cel l   count  i n  bl ood
and  i n  br onchoal v eol ar   l av age  of   pati ents  w i th  pul m onar y   sar coi dosi s.
Respi r ati on  1985;48:231.

Tuberculosis
1.   What  i s  the  contem por ar y   epi dem i ol ogy   of   tuber cul osi s?

2.   What  sy m ptom s  and  r adi ogr aphi c  f eatur es  ar e  associ ated  w i th
tuber cul osi s?

3.   Who  shoul d  r ecei v e  tr eatm ent  (pr ophy l ax i s)  f or   tuber cul osi s  i nf ecti on?

P. 375

Discussion
1.   What  i s  the  contem por ar y   epi dem i ol ogy   of   tuber cul osi s?

Despi te  num er ous  m edi cal   adv ances  i n  the  past  centur y ,   tuber cul osi s  i s
sti l l   the  cause  of   at  l east  1  m i l l i on  deaths  w or l dw i de  each  y ear ,   and  i ts
i nci dence,   w hi ch  w as  i ncr easi ng  i n  the  1980s  and  ear l y   1990s  pr i m ar i l y
because  of   AIDS,   i s  agai n  decr easi ng.   El der l y   pati ents  now   consti tute
near l y   hal f   of   the  new l y   di agnosed  cases  of   tuber cul osi s  i n  the  U ni ted
States  because  these  peopl e  w er e  ex posed  to  the  tuber cul osi s  epi dem i c
i n  the  f i r st  quar ter   of   the  20th  centur y   and  hav e  been  har bor i ng  l atent
i nf ecti on  f or   m any   decades.   The  case  f atal i ty   r ate  i n  the  el der l y   i s  al so
di spr opor ti onatel y   hi gh,   and  they   f ace  a  hi gher   r i sk   of   com pl i cati ons
w i th  tr eatm ent.   Other s  at  r i sk   f or   tuber cul osi s  i ncl ude  m edi cal l y
under ser v ed,   l ow ­i ncom e,   ethni c  m i nor i ty   popul ati ons,   especi al l y
Af r i can  Am er i cans,   N ati v e  Am er i cans,   and  Hi spani cs;  i nsti tuti onal i zed
peopl e;  pati ents  w i th  chr oni c  r enal   f ai l ur e,   si l i cosi s,   di abetes  m el l i tus,
or   l y m phor eti cul ar   m al i gnanci es;  al cohol i cs  or   those  w i th  other
substance  abuse  habi ts;  those  w i th  m al nutr i ti on;  those  w ho  hav e
under gone  gastr ectom y ;  and  those  under goi ng  i m m unosuppr essi v e  or
l ong­ter m   cor ti coster oi d  ther apy .

2.   What  sy m ptom s  and  r adi ogr aphi c  f eatur es  ar e  associ ated  w i th
tuber cul osi s?

Di v er si ty   char acter i zes  the  cl i ni cal   m ani f estati ons  of   tuber cul osi s.
Al though  m any   pati ents  hav e  consti tuti onal   sy m ptom s  consi sti ng  of
w ei ght  l oss,   f ati gue,   f ev er ,   and  ni ght  sw eats,   as  w el l   as  pul m onar y
sy m ptom s  such  as  cough,   i nter m i ttent  hem opty si s,   chest  pai n,   and
dy spnea,   none  of   these  i s  uni f or m l y   pr esent.   In  addi ti on,   the  el der l y
and  pati ents  w i th  AIDS  of ten  hav e  ex tr apul m onar y   di sease  and  the
sy m ptom s  and  si gns  ar e  aty pi cal .

The  cl assi c  chest  r adi ogr aph  i n  an  adul t  w i th  pul m onar y   tuber cul osi s
dem onstr ates  f i br onodul ar   i nf i l tr ati on  of   the  poster i or   or   api cal
segm ents  of   the  upper   l obe.   Ther e  m ay   al so  be  cav i tati on.   Tuber cul osi s
can,   how ev er ,   pr oduce  al m ost  any   f or m   of   pul m onar y   r adi ogr aphi c
abnor m al i ty .   Mor eov er ,   nor m al   r adi ogr aphi c  f i ndi ngs  do  not  ex cl ude  a
di agnosi s  of   di ssem i nated  tuber cul osi s  i n  an  el der l y   or
i m m unocom pr om i sed  pati ent.   Hi l ar   adenopathy   on  a  chest  r adi ogr aph  i n
a  pati ent  ser oposi ti v e  f or   HIV  i s  consi der ed  tuber cul osi s  unti l   pr ov ed
other w i se.

3.   Who  shoul d  r ecei v e  tr eatm ent  (pr ophy l ax i s)  f or   tuber cul osi s  i nf ecti on?
The  tuber cul i n  sk i n  test  i s  the  tr adi ti onal   m ethod  of   dem onstr ati ng
i nf ecti on  w i th  M.   tuber cul osi s,   and  i s  based  on  the  pr i nci pl e  that
i nf ecti on  el i ci ts  del ay ed­ty pe  hy per sensi ti v i ty   to  cer tai n  anti gens  i n
cul tur e  ex tr acts  cal l ed  tuber cul i ns.  The  tuber cul i n  m ost  com m onl y   used
i s  PPD;  i t  i s  i njected  i ntr acutaneousl y   on  the  v ol ar   aspect  of   the
f or ear m   i n  a  dose  of   5  tuber cul i n  uni ts.   Indur ati on  of   the  si te  at  48  to
72  hour s  i ndi cates  del ay ed  hy per sensi ti v i ty   to  i nf ecti on  w i th  M.
tuber cul osi s,   but  does  not  necessar i l y   si gni f y   the  pr esence  of   acti v e
di sease,   onl y   i nf ecti on.

Pr ev enti v e  ther apy   w i th  i soni azi d  (IN H)  gi v en  f or   6  to  12  m onths  cl ear l y
decr eases  the  r i sk   of   f utur e  tuber cul osi s—i n  other   w or ds,   the  pr ogr essi on
f r om   an  i nf ected  state  to  an  acti v el y   di seased  state  m ani f esti ng  the  cl i ni cal ,
r adi ogr aphi c,   and
P. 376
m i cr obi ol ogi c  pr of i l e.   The  goal   of   IN H  m onother apy   i s  ther ef or e  to  tr eat
subcl i ni cal   i nf ecti on  br ought  to  l i ght  by   the  posi ti v e  r esul t  of   tuber cul i n  sk i n
test.   By   str i ct  def i ni ti on,   i t  i s  not  tr ue  pr ophy l ax i s,   but  i t  i s  of ten  r ef er r ed  to
as  such.   The  dosage  of   IN H  i n  adul ts  i s  5  m g/k g,   up  to  a  total   of   300  m g
or al l y   per   day .   Peopl e  w ho  hav e  contact  w i th  a  pati ent  hav i ng  new l y
di agnosed  pul m onar y   tuber cul osi s,   and  w hose  tuber cul i n  sk i n  test  r esul t  i s
posi ti v e,   shoul d  under go  IN H  pr ev enti v e  ther apy .   If   a  chest  r adi ogr aph
show s  i nacti v e  par enchy m al   tuber cul osi s  (upper   l obe  scar r i ng),   the  sk i n  test
r esul t  i s  posi ti v e,   and  acti v e  di sease  has  been  ex cl uded  by   negati v e  sputum
f i ndi ngs,   such  pati ents  shoul d  r ecei v e  IN H  ther apy .   Peopl e  y ounger   than  35
y ear s  w i th  a  posi ti v e  r esul t  on  sk i n  test  and  a  nor m al   chest  r adi ogr aph
shoul d  al so  be  tr eated.   Pati ents  w hose  sk i n  test  r esul t  i s  posi ti v e  and  i n
w hom   the  f ol l ow i ng  cl i ni cal   si tuati ons  appl y   shoul d  r ecei v e  6  to  12  m onths
of   pr ev enti v e  ther apy :  HIV  posi ti v i ty ;  si l i cosi s;  di abetes  m el l i tus,   especi al l y
poor l y   contr ol l ed  i nsul i n­dependent  di abetes;  ster oi d  ther apy ,   especi al l y
m or e  than  15  m g  of   pr edni sone  per   day ;  chr oni c  r enal   f ai l ur e;
l y m phor eti cul ar   m al i gnanci es:  l euk em i a,   l y m phom a,   and  Hodgk i n's  di sease;
i m m unosuppr essi v e  ther apy ;  gastr ectom y ;  jejunoi l eal   by pass;  and  w ei ght
l oss  of   10%  or   m or e  of   the  i deal   body   w ei ght.

Case
A  68­y ear ­ol d  Af r i can­Am er i can  m an  w i th  a  l ong  hi stor y   of   tobacco  and
ethanol   abuse  i s  br ought  i n  by   hi s  f am i l y   f or   ev al uati on  of   w ei ght  l oss,   l ow ­
gr ade  f ev er s,   and  f ai l ur e  to  thr i v e.   The  pati ent  r epor ts  a  2­  to  3­m onth
hi stor y   of   pr ogr essi v e  20­pound  (9­k g)  w ei ght  l oss,   as  w el l   as  f ev er s,
nonpr oducti v e  cough,   and  gener al i zed  w eak ness.   Hi s  cough  becam e
pr oducti v e  of   w hi te  sputum   2  day s  ear l i er .   The  pati ent  deni es  chest  pai n,
hem opty si s,   i l l   contacts,   r ecent  tr av el ,   or   HIV  r i sk   f actor s.   He  has  sm ok ed
one  pack   of   ci gar ettes  per   day   f or   40  y ear s.   He  dr i nk s  appr ox i m atel y   one
pi nt  (hal f   a  l i ter )  of   al cohol   a  day   and  has  done  so  f or   m any   y ear s.   Car ef ul
r ev i ew   of   hi s  ol d  r ecor ds  r ev eal s  that  a  PPD  test  w as  posi ti v e  appr ox i m atel y
12  y ear s  ago.
Phy si cal   ex am i nati on  di scl oses  a  cachecti c,   i l l ­appear i ng,   el der l y   m an  i n
m oder ate  r espi r ator y   di str ess.   Hi s  or al   tem per atur e  i s  38. 5°C  (101. 3°F),
w i th  a  r espi r ator y   r ate  of   30  br eaths  per   m i nute,   hear t  r ate  of   126  beats
per   m i nute,   and  bl ood  pr essur e  of   100/60  m m   Hg.   Phy si cal   ex am i nati on
f i ndi ngs  ar e  r em ar k abl e  f or   poor   denti ti on,   and  r al es  and  r honchi   thr oughout
the  r i ght  chest,   but  other w i se  negati v e.   Ini ti al   l abor ator y   ev al uati on  r ev eal s
a  w hi te  bl ood  cel l   count  of   16, 000/m m 3   w i th  a  l ef tw ar d  shi f t  and  a
hem atocr i t  of   35%.   Hi s  ox y gen  satur ati on  i s  80%  on  r oom   ai r .   A  chest
r adi ogr aph  show s  a  l ar ge  i nter sti ti al   and  al v eol ar   i nf i l tr ate  w i th  ai r
br onchogr am s  i n  the  r i ght  upper   l obe;  no  cav i tati on,   pl eur al   ef f usi on,   or
v ol um e  l oss  i s  noted.   Ar ter i al   bl ood  gas  m easur em ents  on  the  ni ght  of
adm i ssi on  show   w or seni ng  hy pox em i a  and  hy per capni a.
The  pati ent  i s  pl aced  i n  r espi r ator y   i sol ati on  and  i s  el ecti v el y   i ntubated  on
the  ni ght  of   adm i ssi on.   Ex am i nati on  of   hi s  sputum ,   done  the  nex t  m or ni ng,
r ev eal s  a  pathogen.

1.   What  i s  the  di f f er enti al   di agnosi s  of   thi s  pati ent's  r espi r ator y   f ai l ur e?


2.   What  i s  the  di agnosti c  str ategy   at  thi s  poi nt  i n  hi s  i l l ness?
3.   What  i s  the  cor r ect  ther apeuti c  pl an?

P. 377

Case Discussion
1.   What  i s  the  di f f er enti al   di agnosi s  of   thi s  pati ent's  r espi r ator y   f ai l ur e?

Thi s  pati ent's  hi stor y   i s  consi stent  w i th  an  i nf ecti ous  pr ocess,   per haps
super i m posed  on  an  under l y i ng  m al i gnancy .   Bacter i al   pathogens  such
as  S.   pneum oni ae,   Legi onel l a  pneum ophi l a,   and  H.   i nf l uenzae  ar e
possi bl e  causes.   Because  of   hi s  hi stor y   of   ethanol   abuse  and  poor
denti ti on,   aspi r ati on  pneum oni a  (w i th  anaer obi c  pathogens)  i s  al so  a
possi bi l i ty ,   but  the  l ocati on  of   the  i nf i l tr ates  i n  the  upper   l obe  m ak es
thi s  l ess  l i k el y   as  aspi r ati on  pneum oni a  tends  to  f av or   dependent
por ti ons  of   the  l ung,   such  as  the  super i or   segm ents  of   both  l ow er
l obes.   Bacter i al   pneum oni a  di stal   to  an  obstr ucti ng  endothel i al   l esi on  i s
cl ear l y   a  possi bi l i ty   gi v en  hi s  l ong­standi ng  tobacco  use  and  age;
how ev er ,   the  absence  of   v ol um e  l oss  and  pr esence  of   ai r   br onchogr am s
on  hi s  r adi ogr aph  som ew hat  m i l i tate  agai nst  thi s  di agnosi s.   Vi r al
i nf ecti on  i s  l ess  l i k el y ,   gi v en  hi s  hi stor y   and  the  l obar   i nf i l tr ate.   Fungal
i nf ecti on,   par ti cul ar l y   w i th  Hi stopl asm a  capsul atum ,   i s  a  possi bi l i ty ,   but
l ess  l i k el y   gi v en  hi s  negati v e  geogr aphi c  hi stor y   and  the  absence  of
adenopathy   on  the  r adi ogr aph.   He  has  no  HIV  r i sk   f actor s.

An  i m por tant  cl ue  to  the  di agnosi s  i s  the  posi ti v e  r esul t  of   PPD  sk i n
test.   Because  m ost  acti v e  cases  of   pul m onar y   tuber cul osi s  i n  adul ts
consti tute  ei ther   postpr i m ar y   di sease  or   the  r eacti v ati on  of   a
pr otr acted,   ev en  l i f el ong,   i nf ecti on  w i th  the  tuber cl e  baci l l us,   thi s  pl us
hi s  posi ti v e  PPD  test  r esul t  poi nt  to  the  di agnosi s.   The  i m por tant  poi nt
her e  i s  that  the  cl i ni ci an  shoul d  suspect  r eacti v ati on  of   tuber cul osi s
ev en  w i thout  a  hi stor y   of   a  posi ti v e  PPD  test  r esul t.   Thi s  pati ent  has
sev er al   cur r entl y   r ecogni zed  epi dem i ol ogi c  r i sk   f actor s  f or
tuber cul osi s:  he  i s  el der l y ,   Af r i can  Am er i can,   and  a  substance  abuser
(ethanol ),   and  he  has  poor   nutr i ti onal   status.   He  has  an  upper   l obe
i nf i l tr ate.   The  absence  of   an  upper   l ob  i nf i l tr ate,   how ev er ,   does  not
r ul e  out  tuber cul osi s,   because  the  cl assi c  f i br onodul ar   i nf i l tr ati on  of   the
poster i or   or   api cal   segm ents  of   the  upper   l obe  (r i ght  gr eater   than  l ef t)
m ay   not  be  pr esent  i n  the  el der l y   or ,   especi al l y ,   i n  HIV­i nf ected
pati ents.   Tuber cul osi s  pr obabl y   needs  to  be  consi der ed  i n  the
di f f er enti al   di agnosi s  of   pneum oni a  i n  ev er y   pati ent  ol der   than  60
y ear s.   Thi s  pati ent's  r espi r ator y   f ai l ur e  w i th  hy pox em i a  and  r espi r ator y
m uscl e  f ati gue  (hy per capni a)  coul d  be  due  to  any   one  of   the  pr ocesses
di scussed,   but  tuber cul osi s  i s  the  m ost  l i k el y   di agnosi s.

2.   What  i s  the  di agnosti c  str ategy   at  thi s  poi nt  i n  hi s  i l l ness?

A  car ef ul   sear ch  f or   the  pathogen,   or   pathogens,   shoul d  be  under tak en.
Bl ood  cul tur es,   Gr am 's  stai ni ng  and  cul tur e  of   sputum   sam pl es,   pl us
sputum   stai ns  f or   aci d­f ast  or gani sm s  (m y cobacter i a)  and  the
possi bi l i ty   of   Legi onel l a  i ndi cate  that  a  f l uor escent  anti body   test  shoul d
be  per f or m ed.   The  v al ue  of   sputum   ex am i nati on  cannot  be
ov er em phasi zed.   Gr am 's  stai ni ng,   especi al l y   of   a  tr acheal   aspi r ate  or
sputum   pr oduced  by   a  str ong  deep  cough,   can  hel p  i n  the  di agnosi s  of
v i r tual l y   any   bacter i al   i nf ecti ous  agent  i n  the  di f f er enti al   di agnosi s.
(Legi onel l a  i s  di f f i cul t  to  see,   but  a  l ar ge  num ber   of   neutr ophi l s  w i thout
or gani sm s  suggest  thi s  di agnosi s. )  In  addi ti on  to  the  stai ns  f or   aci d­
f ast  baci l l i   (AFB),   tuber cul osi s  can  be  di agnosed  w i th  a  f l uor ochr om e
techni que  cal l ed  aur am i ne  O.   Thi s  pati ent's  sputum   w as  posi ti v e  f or
AFB.   Appr ox i m atel y   10 4   or gani sm s/m L  of   sputum   ar e  r equi r ed  f or   an
AFB  sm ear   to  be
P. 378
posi ti v e,   and  onl y   50%  to  80%  of   pati ents  w i th  pul m onar y   tuber cul osi s
hav e  posi ti v e  sputum   sm ear   f i ndi ngs.   A  r api d  r adi om etr i c  techni que
k now n  as  BACTEC  al l ow s  the  r ecov er y   and  i denti f i cati on  of   tuber cul osi s
i n  10  day s,   w hi ch  i s  an  adv antage  ov er   conv enti onal   cul tur e  m ethods
that  can  tak e  3  to  6  w eek s  to  gr ow   m y cobacter i a.

A  PPD  test  w i th  tw o  contr ol s  shoul d  be  pl aced  and  an  i ndur ati on  of   10
m m   or   m or e  i s  consi der ed  a  posi ti v e  r esul t.   How ev er ,   i n  the  setti ng  of
HIV  i nf ecti on—HIV  r i sk   f actor s,   r ecent  cl ose  contact  w i th  i nf ecti ous
peopl e,   or   chest  r adi ogr aphi c  f i ndi ngs  consi stent  w i th  ol d  heal ed
tuber cul osi s  (upper   l obe  scar r i ng)—an  i ndur ati on  of   5  m m   or   m or e  i s
consi der ed  a  posi ti v e  r eacti on.   How ev er ,   as  al r eady   m enti oned,   a
negati v e  r esul t  does  not  ex cl ude  tuber cul osi s  because  up  to  25%  to
30%  of   new l y   di agnosed  pati ents  w i th  tuber cul osi s  hav e  a  negati v e  (â
‰¤9  m m )  sk i n  test  r esul t.   A  booster   ef f ect  i s  m or e  pow er f ul   (≥6
m m   i ncr ease)  i n  the  cutaneous  r eacti on,   and  i s  achi ev ed  by   per f or m i ng
a  second  PPD  test  7  to  10  day s  af ter   the  f i r st.   A  second  PPD  shoul d
ther ef or e  be  consi der ed  i n  thi s  pati ent,   but  especi al l y   i n  an  el der l y
pati ent  w i th  a  negati v e  PPD,   i n  w hom   the  cl i ni cal   suspi ci on  f or
tuber cul osi s  i s  hi gh.   How ev er ,   the  si ne  qua  non  test  f or   tuber cul osi s
r em ai ns  the  sputum   sm ear   and  cul tur e.

Addi ti onal   di agnosti c  tests  that  shoul d  be  par t  of   the  ev al uati on  of   thi s
pati ent  i ncl ude  an  HIV  test  and  com par i son  of   pr ev i ous  and  cur r ent
chest  r adi ogr aphs,   i f   possi bl e.   The  l atter   w oul d  consti tute  an  i m por tant
par t  of   the  ev al uati on  i n  thi s  pati ent.   Fi ber opti c  br onchoscopy   coul d
al so  hel p  i n  deter m i ni ng  i f   ther e  i s  an  endobr onchi al   l esi on,   or   ex tr i nsi c
com pr essi on  f r om   a  m ass,   as  w el l   as  enabl e  sam pl i ng  of   secr eti ons  and
bi opsi es  f or   m i cr obi ol ogi c  ev al uati on.

3.   What  i s  the  cor r ect  ther apeuti c  pl an?

Af ter   appr opr i ate  cul tur e  r esul ts  ar e  obtai ned,   tr eatm ent  needs  to  be
di r ected  tow ar d  the  m ost  l i k el y   pathogens  i n  thi s  m an's  i l l ness.   On  the
basi s  of   the  di f f er enti al   di agnosi s,   y ou  w oul d  need  to  pr ov i de  anti bi oti c
cov er age  f or   S.   pneum oni ae,   H.   i nf l uenzae,   possi bl y   anaer obes,   as  w el l
as  Legi onel l a  speci es.   The  sev er i ty   of   thi s  pati ent's  i l l ness  m andates
aggr essi v e,   but  w el l ­consi der ed,   ther apy .   Hi s  i ni ti al   r egi m en  shoul d
i ncl ude  cov er age  f or   com m uni ty ­acqui r ed  pneum oni a  and  Legi onel l a.
Anti tuber cul ous  ther apy   shoul d  be  i ni ti ated  onl y   af ter   hi s  AFB  sm ear s
pr ov e  posi ti v e.   It  w as  al so  v er y   i m por tant  that  on  adm i ssi on  the
pati ent  w as  pl aced  i n  r espi r ator y   i sol ati on  to  pr ev ent  the  spr ead  of
tuber cul osi s,   w hi ch  i s  tr ansm i tted  al m ost  ex cl usi v el y   by   m eans  of
aer osol i zed  r espi r ator y   secr eti ons,   not  onl y   to  other   pati ents  but  to
heal th  car e  w or k er s  as  w el l .   Hi s  sputum   sm ear   posi ti v i ty   f or   M.
tuber cul osi s  consti tutes  a  state  of   i nf ecti ousness.   Peopl e  r ecei v i ng
ther apy   pr om ptl y   becom e  noni nf ecti ous  as  thei r   cough  subsi des  and  the
concentr ati on  of   or gani sm s  i n  thei r   sputum   decr eases.   Most  author i ti es
bel i ev e  that  tr eatm ent  r ev er ses  i nf ecti ousness  w i thi n  appr ox i m atel y   2
w eek s  of   the  star t  of   ther apy ;  unti l   then,   i sol ati on  m easur es  shoul d  be
m ai ntai ned.

Hi s  anti tuber cul ous  chem other apy   shoul d  consi st  of   a  f our ­dr ug
r egi m en,   com pr i si ng  IN H,   r i f am pi n,   py r azi nam i de,   and  str eptom y ci n,
and  be  m ai ntai ned  f or   6  m onths.   Many   studi es  hav e  conv i nci ngl y
dem onstr ated  the  cur ati v e  ef f i cacy   of   m ul ti dr ug  r egi m ens  gi v en  f or   6  to
9  m onths.   A  standar d  tr eatm ent  i s  ef f ecti v e  w hen  gi v en  f or   6  m onths  i n
a  super v i sed  setti ng.   Com pl i ance  w i th  anti tuber cul ous  ther apy   i s
absol utel y   cr i ti cal   f or   cur e.   N oncom pl i ance  i s  al so  one  of   the  m ajor
r easons  f or   the  em er gence  of   dr ug  r esi stance.   Mul ti dr ug  ther apy   i s
al w ay s  used  i n  the  tr eatm ent  of
P. 379
tuber cul osi s  because  of   the  potenti al   f or   pr i m ar y   or   spontaneous
r esi stance,   w hi ch  occur s  i n  1  ×  10 6   or gani sm s.   Ex tr apul m onar y
tuber cul osi s  i s  tr eated  l i k e  pul m onar y   tuber cul osi s,   and  tuber cul osi s
contr acted  i n  the  setti ng  of   AIDS  i s  al so  tr eated  w i th  standar d
r egi m ens,   and  i s  sti l l   cur abl e.

Suggested Readings
Am er i can  Thor aci c  Soci ety /Center s  f or   Di sease  Contr ol   and
Pr ev enti on/Inf ecti ous  Di sease  Soci ety   of   Am er i ca:  Contr ol l i ng
tuber cul osi s  i n  the  U ni ted  States.   Am   J  Respi r   Cr i t  Car e  Med
2005;172:1169–1227;  Tr eatm ent  of   tuber cul osi s.   Am   J  Respi r   Cr i t  Car e
m ed  2003;167:603–662.

Bass  JB,   Far er   LS,   Hopew el l   PC,   et  al .   Am er i can  Thor aci c  Soci ety ,
Medi cal   Secti on  of   the  Am er i can  Lung  Associ ati on.   Tr eatm ent  of
tuber cul osi s  and  tuber cul ous  i nf ecti on  i n  adul ts  and  chi l dr en.   Am   Rev
Respi r   Di s  1986;134:355.

Bass  JB,   Far er   LS,   Hopew el l   PC,   et  al .   Am er i can  Thor aci c  Soci ety ,
Medi cal   Secti on  of   the  Am er i can  Lung  Associ ati on.   Di agnosti c  standar ds
and  cl assi f i cati on  of   tuber cul osi s.   Am   Rev   Respi r   Di s  1990;142:725.

Cohn  DL,   Catl i n  BJ,   Peter son  KL,   et  al .   A  62­dose,   6­m onth  ther apy   f or
pul m onar y   and  ex tr apul m onar y   tuber cul osi s:  a  tw i ce­w eek l y ,   di r ectl y
obser v ed,   and  cost­ef f ecti v e  r egi m en.   Ann  Inter n  Med  1990;112:407.

Hef f ner   JE,   Str ange  C,   Sahn  SA.   The  i m pact  of   r espi r ator y   f ai l ur e  on  the
di agnosi s  of   tuber cul osi s.   Ar ch  Inter n  Med  1988;148:1103.

Isem an  M.   Tuber cul osi s.   In:  Sy nopsi s  of   cl i ni cal   pul m onar y   di sease,   4th
ed.   St.   Loui s:  Mosby –Year   Book ,   1989.
Editors :  Sc hrie r,  Robe rt W .
Title :  Inte rna l Me dic ine  Ca s e book ,  The : Re a l P a tie nts ,  Re a l Ans w e rs ,
3rd Edition

Copy r i ght  ©2007  Li ppi ncott  Wi l l i am s  &  Wi l k i ns

>  T a b le   o f   C o nte nts   >  C ha p te r   9  ­  Ne p hr o lo g y

Chapter 9
Nephrology

Toma s  Be rl
Is a a c  Te ite lba um

Acute Renal Failure
1.   U nder   w hat  ci r cum stances  i s  ser um   cr eati ni ne  a  r easonabl e  m ar k er   f or
gl om er ul ar   f i l tr ati on  r ate  (GFR)?  How   i s  the  cr eati ni ne  cl ear ance
esti m ated  f r om   the  ser um   cr eati ni ne?

2.   What  cl i ni cal   f i ndi ngs  m ost  com m onl y   suggest  the  pr esence  of   acute  r enal
f ai l ur e?

3.   What  pr ocesses  need  to  be  consi der ed  w hen  attem pti ng  to  ascer tai n  the
cause  of   acute  r enal   f ai l ur e?

4.   What  ar e  the  m ost  com m on  causes  of   acute  r enal   f ai l ur e  i n  hospi tal i zed
pati ents  and  i n  outpati ents?

P. 381
5.   What  ar e  the  ur i nar y   f i ndi ngs  that  assi st  i n  di f f er enti ati ng  pr er enal
azotem i a  f r om   i ntr ar enal   acute  r enal   f ai l ur e?

6.   What  ar e  the  com pl i cati ons  of   acute  r enal   f ai l ur e?

Discussion
1.   U nder   w hat  ci r cum stances  i s  ser um   cr eati ni ne  a  r easonabl e  m ar k er   f or
GFR?  How   i s  the  cr eati ni ne  cl ear ance  esti m ated  f r om   the  ser um
cr eati ni ne?

The  ser um   cr eati ni ne  i s  a  r easonabl e  m ar k er   f or   cr eati ni ne  cl ear ance  and
GFR  onl y   i n  the  steady   state,   that  i s,   w hen  the  ser um   cr eati ni ne  i s  nei ther
i ncr easi ng  nor   decr easi ng.   In  the  steady   state,   the  cr eati ni ne  cl ear ance
(C Cr )  m ay   be  esti m ated  f r om   the  ser um   cr eati ni ne  (S Cr )  by   the  Cock r of t­
Gaul t  equati on:
Another   equati on  der i v ed  as  a  r esul t  of   the  Modi f i cati on  of   Di et  i n  Renal
Di sease  (MDRD)  study   has  r ecentl y   been  v al i dated  as  a  m or e  r el i abl e
pr edi ctor   of   GFR  i n  som e  ci r cum stances,   such  as  chr oni c  k i dney   di seases:

A  GFR  cal cul ator   uti l i zi ng  thi s  equati on  m ay   be  f ound  at  the  w ebsi te,
http://w w w . nephr on. com /cgi ­bi n/MDRD. cgi ,   and  i s  al so  av ai l abl e  on  m any
“handhel dâ€​   dev i ces.

2.   What  cl i ni cal   f i ndi ngs  m ost  com m onl y   suggest  the  pr esence  of   acute  r enal
f ai l ur e?

A  r i se  i n  the  bl ood  ur ea  ni tr ogen  (BU N )  and  ser um   cr eati ni ne  l ev el s  and
dev el opm ent  of   ol i gur i a  (< 400  m L  per   day )  ar e  the  com m on  cl i ni cal
f i ndi ngs  that  suggest  the  pr esence  of   acute  r enal   f ai l ur e.   How ev er ,   the
absence  of   ol i gur i a  does  not  ex cl ude  acute  r enal   f ai l ur e  because  the
pr ocess  m ay   al so  be  nonol i gur i c.   In  f act,   20%  to  30%  of   pati ents  w i th
acute  r enal   f ai l ur e  ar e  nonol i gur i c  (> 400  m L  per   day ).

3.   What  pr ocesses  need  to  be  consi der ed  w hen  attem pti ng  to  ascer tai n  the
cause  of   acute  r enal   f ai l ur e?

In  pati ents  w i th  acute  r enal   f ai l ur e,   pr er enal ,   postr enal ,   and  i ntr ar enal
pr ocesses  need  to  be  consi der ed.   The  r especti v e  causes  of   pr er enal   and
postr enal   azotem i a  as  w el l   as  i ntr i nsi c  r enal   di sease  ar e  l i sted  i n  Tabl es
9­1, 9­2, 9­3.

4.   What  ar e  the  m ost  com m on  causes  of   acute  r enal   f ai l ur e  i n  hospi tal i zed
pati ents  and  i n  outpati ents?

In  hospi tal i zed  pati ents,   the  m ost  com m on  cause  of   acute  r enal   f ai l ur e
(45%)  i s  acute  tubul ar   necr osi s,   f ol l ow ed  by   pr er enal   azotem i a  and
obstr ucti on.   Gl om er ul onephr i ti s,   v ascul i ti s,   i nter sti ti al   nephr i ti s,   and
ather oem bol i c
P. 382
di sease  com pr i se  m ost  of   the  r em ai ni ng  causes.   In  contr ast,   acute  r enal
f ai l ur e  i n  outpati ents  i s  m ost  com m onl y   due  to  pr er enal   azotem i a  (70%),
f ol l ow ed  by   obstr ucti on.   Dr ug  nephr otox i ci ty   [e. g. ,   am i nogl y cosi des,
angi otensi n­conv er ti ng  enzy m e  (ACE)  i nhi bi tor s,   angi otensi n  r eceptor
bl ock er s  (ARBs),   and  nonster oi dal   anti i nf l am m ator y   dr ugs  (N SAIDs)]
accounts  f or   m ost  of   the  r em ai ni ng  cases.

Table 9­1 Causes of Prerenal Azotemia

Reduced  ex tr acel l ul ar   and  i ntr av ascul ar   v ol um e


Gastr oi ntesti nal   l osses  (v om i ti ng,   di ar r hea,
nasogastr i c  sucti on)
Dehy dr ati on
Bur ns
Hem or r hage
Reduced  i ntr av ascul ar   v ol um e  but  i ncr eased  ex tr acel l ul ar
v ol um e
Ci r r hosi s
N ephr oti c  sy ndr om e
Congesti v e  hear t  f ai l ur e—car di ogeni c  shock
Thi r d­space  f l ui d  accum ul ati on  (postoper ati v e  f r om
abdom i nal   sur ger y ,   sev er e  pancr eati ti s,   per i toni ti s)
Hem ody nam i cal l y   m edi ated  acute  r enal   f ai l ur e
Anesthesi a
N onster oi dal   anti i nf l am m ator y   agents  (due  to  r enal
pr ostagl andi n  i nhi bi ti on)
Inhi bi tor s  of   the  r eni n­angi otensi n  sy stem   (due  to  a
decr ease  i n  ef f er ent  ar ter i ol ar   tone)
Hepator enal   sy ndr om e
Vasoconstr i ctor   agents
Cal ci neur i n  i nhi bi tor s
Contr ast  agents

5.   What  ar e  the  ur i nar y   f i ndi ngs  that  assi st  i n  di f f er enti ati ng  pr er enal
azotem i a  f r om   i ntr ar enal   acute  r enal   f ai l ur e?

The  ur i nar y   f i ndi ngs  that  can  be  used  to  hel p  di f f er enti ate  betw een
pr er enal   azotem i a  and  i ntr ar enal   acute  r enal   f ai l ur e  ar e  l i sted  i n  Tabl e  9­
4.

6.   What  ar e  the  com pl i cati ons  of   acute  r enal   f ai l ur e?

The  v ar i ous  com pl i cati ons  of   acute  r enal   f ai l ur e  ar e  l i sted  by   categor y   i n


Tabl e  9­5.

Case
A  65­y ear ­ol d  di abeti c  w om an  pr esents  to  the  em er gency   r oom   w i th  r i ght
upper   quadr ant  pai n  that  r adi ates  ar ound  to  the  back ,   together   w i th  nausea,
v om i ti ng,   anor ex i a,   l i ghtheadedness,   and  a  di m i ni shed  ur i ne  output  dur i ng  the
l ast  24  hour s.   She  has  no  pr ev i ous  hi stor y   of   r enal   dy sf uncti on.   Her
tem per atur e  i s  37. 5°C  (99. 5°F);  supi ne,   her   bl ood  pr essur e  i s  110/70  m m
Hg  and  pul se  i s  80  beats  per   m i nute;  upr i ght,   her   bl ood  pr essur e  i s  85/60  m m
Hg  and  pul se  i s  110  beats  per   m i nute.   The  phy si cal   ex am i nati on  f i ndi ngs  ar e
other w i se  r em ar k abl e  f or   the  pr esence  of   decr eased  sk i n  tur gor ,   dr y   m ucosal
P. 383
m em br anes,   f l at  neck   v ei ns,   and  absence  of   ax i l l ar y   sw eat.   Her   l ungs  ar e
cl ear   and  the  car di ac  f i ndi ngs  ar e  nor m al .   Ther e  i s  ex qui si te  r i ght  upper
quadr ant  abdom i nal   tender ness  that  w or sens  w i th  i nspi r ati on,   her   stool   i s
guai ac  negati v e,   and  no  edem a  i s  noted.   N eur ol ogi c  ex am i nati on  r ev eal s
nonf ocal   f i ndi ngs.
Table 9­2 Causes of Postrenal Azotemia

U r ethr al   obstr ucti on


Val v es
Str i ctur es
Bl adder   neck   obstr ucti on
Pr ostati c  hy per tr ophy
Bl adder   car ci nom a
Bl adder   i nf ecti on
Functi onal
Autonom i c  neur opathy
a­Adr ener gi c  bl ock er s
Obstr ucti on  of   ur eter s,   bi l ater al
U ni l ater al   obstr ucti on  i n  sol i tar y   k i dney
Intr aur eter al
Sul f onam i de,   ur i c  aci d,   acy cl ov i r ,   anti r etr ov i r al   agent
cr y stal s
Bl ood  cl ots
Stones
N ecr oti zi ng  papi l l i ti s
Ex tr aur eter al
Tum or   of   cer v i x ,   pr ostate,   bl adder
Endom etr i osi s
Per i ur eter al   f i br osi s
Acci dental   ur eter al   l i gati on
Pel v i c  abscess  or   hem atom a

The  f ol l ow i ng  l abor ator y   data  ar e  obtai ned:  hem atocr i t,   50. 2%;  w hi te  bl ood
cel l   count,   19, 500/m m 3   w i th  82%  pol y m or phonucl ear   l euk ocy tes,   16%  band
f or m s,   and  2%  l y m phocy tes;  pl atel ets,   312, 000/m m 3 ;  sodi um ,   146  m Eq/L;
potassi um ,   4. 1  m Eq/L;  chl or i de,   111  m Eq/L;  car bon  di ox i de,   22  m Eq/L;
gl ucose,   195  m g/dL;  BU N ,   35  m g/dL;  cr eati ni ne,   1. 6  m g/dL;  total   bi l i r ubi n,   1. 8
m g/dL;  al k al i ne  phosphatase,   289  IU ;  and  aspar tate  am i notr ansf er ase  (AST),
35  U /L.
U r i nal y si s  r ev eal s  a  pH  of   5,   a  speci f i c  gr av i ty   of   1. 028;  1+   gl ucose,   tr ace
k etones,   occasi onal   nonpi gm ented  gr anul ar   casts,   and  no  cel l ul ar   casts  or
bacter i a.   The  ur i ne  sodi um   l ev el   i s  10  m Eq/L  and  the  ur i ne  cr eati ni ne  l ev el   i s
80  m g/dL.
Abdom i nal   ul tr asonogr aphy   r ev eal s  the  ex i stence  of   gal l stones  and  di l atati on
of   the  bi l i ar y   tr ee.   The  k i dney s  m easur e  11  cm   but  ex hi bi t  no  hy dr onephr osi s
or   i ncr eased  echogeni ci ty .
Whi l e  i n  the  em er gency   r oom ,   the  pati ent's  f ev er   spi k es  to  39°C  (102. 2°F),
w hi ch  i s  accom pani ed  by   3  m i nutes  of   r i gor s  and  a  decr ease  i n  bl ood  pr essur e
to
P. 384
P. 385
80/50  m m   Hg.   She  i s  adm i tted  to  the  hospi tal   w i th  a  di agnosi s  of   acute
chol ecy sti ti s  f or   the  pur pose  of   obser v ati on  and  ev entual   chol ecy stectom y .
She  i s  gi v en  gentam i ci n  [2  m g/k g  i ntr av enousl y   (IV)]  and  am pi ci l l i n  (2  g  IV
ev er y   6  hour s).   Her   ur i ne  output  ov er   12  hour s  i s  100  m L.   The  nex t  m or ni ng,
the  f ol l ow i ng  l abor ator y   v al ues  ar e  r epor ted:  sodi um ,   140  m Eq/L;  potassi um ,
5  m Eq/L;  chl or i de,   100  m Eq/L;  car bon  di ox i de,   15  m Eq/L;  gl ucose,   130  m g/dL;
BU N ,   40  m g/dL;  and  cr eati ni ne,   2. 5  m g/dL.   U r i nal y si s  now   r ev eal s  a  pH  of   5
and  a  speci f i c  gr av i ty   of   1. 010  w i th  occasi onal   r enal   tubul ar   epi thel i al   cel l s
and  a  r ar e,   m uddy ­br ow n  gr anul ar   cast.   The  ur i ne  sodi um   l ev el   i s  80  m Eq/L
and  the  ur i ne  cr eati ni ne  l ev el   i s  40  m g/dL.   Bl ood  cul tur es  ar e  posi ti v e  f or   a
gr am ­negati v e  baci l l us.

Table 9­3 Causes of Intrarenal Acute Renal
Failure

Gl om er ul ar   di seases
Rapi dl y   pr ogr essi v e  gl om er ul onephr i ti s
Posti nf ecti ous  gl om er ul onephr i ti s
Focal   gl om er ul oscl er osi s  associ ated  w i th  acqui r ed
i m m unodef i ci ency   sy ndr om e
Tubul oi nter sti ti al   nephr i ti s
Hy per sensi ti v i ty   r eacti ons:  peni ci l l i ns,   sul f onam i des,
f l uor oqui nol ones,   and  m any   other   dr ugs
Associ ated  w i th  sy stem i c  i nf ecti ons  (Legi onel l a,
Tox opl asm a)
Acute  tubul ar   necr osi s
Ischem i a,   hy potensi on,   septi cem i a
Di r ect  dr ug  tox i ci ty :  am i nogl y cosi des,   ci spl ati n,
am photer i ci n,   contr ast  agents
My ogl obi n  or   hem ogl obi n
Acute  tubul ar   necr osi s  i n  pr egnancy
Vascul ar   di seases
Renal   ar ter y   occl usi on
Acute  v ascul i ti s
Mal i gnant  hy per tensi on
Ather oem bol i c  di sease,   m ul ti pl e  chol ester ol   em bol i
sy ndr om e
Thr om boti c  m i cr oangi opathy
Other s
Acute  ur i c  aci d  nephr opathy
Hy per cal cem i c  nephr opathy
Table 9­4 Urine Findings in Prerenal Azotemia
and Acute Renal Failure

La bora tory Te s t P re re na l Azote mia Intra re na l Ac ute


Re na l Fa ilure

U r i nar y   osm ol al i ty > 500 < 400


(m Osm /k g)

U r i nar y   sodi um < 20 > 40


(m Eq/L)

U r i ne­pl asm a > 40 < 20


cr eati ni ne  r ati o

Renal   f ai l ur e <1 >2


i ndex :
U N a/U Cr /PCr

Fr acti onal <1 >2


ex cr eti on  of
sodi um :
U   N a/PN a
÷ U Cr /PCr   ×  100

U r i nar y   sedi m ent N or m al   or   occasi onal Br ow n  gr anul ar   casts,


gr anul ar   casts cel l ul ar   debr i s

U N a,   ur i nar y   sodi um   l ev el ;  U Cr ,   ur i nar y   cr eati ni ne  l ev el ;  PCr ,   ser um


cr eati ni ne  l ev el ;  PN a,   ser um   sodi um   l ev el .

Fr om   Edel stei n  CH,   Schr i er   RW.   Acute  r enal   f ai l ur e:  pathogenesi s,


di agnosi s,   and  m anagem ent.   In:

Schr i er   RW,   ed.   Renal   and  el ectr ol y te  di sor der s,   6th  ed.   Phi l adel phi a:
Li ppi ncott  Wi l l i am s  &  Wi l k i ns

2003.   Repr i nted  w i th  per m i ssi on.


Table 9­5 Complications of Acute Renal Failure

Metabol i c
Hy ponatr em i a,   hy per k al em i a,   hy pocal cem i a,
hy per phosphatem i a,   hy per m agnesem i a,   hy per ur i cem i a
Car di ov ascul ar
Pul m onar y   edem a,   ar r hy thm i as,   hy per tensi on,   per i car di ti s
N eur ol ogi c
Aster i x i s,   neur om uscul ar   i r r i tabi l i ty ,   som nol ence,   com a,
sei zur es
Hem atol ogi c
Anem i a,   coagul opathi es,   hem or r hagi c  di athesi s
Gastr oi ntesti nal
N ausea,   v om i ti ng,   bl eedi ng
Inf ecti ous
Pneum oni a,   ur i nar y   tr act  i nf ecti on,   w ound  i nf ecti on,
septi cem i a

Fr om   Edel stei n  CH,   Schr i er   RW.   Acute  r enal   f ai l ur e:  pathogenesi s,


di agnosi s,   and  m anagem ent.   In:  Schr i er   RW,   ed.   Renal   and  el ectr ol y te
di sor der s,   6th  ed.   Phi l adel phi a:  Li ppi ncott  Wi l l i am s  &  Wi l k i ns  2003.
Repr i nted  w i th  per m i ssi on.

Dur i ng  the  nex t  3  day s,   the  pati ent  r em ai ns  ol i gur i c  and  m i l d  congesti v e  hear t
f ai l ur e  dev el ops.   The  BU N   and  cr eati ni ne  l ev el s  r i se  steadi l y   to  100  and  5. 5
m g/dL,   r especti v el y .

1.   At  the  ti m e  of   ar r i v al   i n  the  em er gency   r oom ,   w hat  i s  the  m ost  l i k el y


ex pl anati on  f or   thi s  pati ent's  acute  r enal   dy sf uncti on,   and  w hy ?
2.   At  the  ti m e  of   the  pati ent's  ar r i v al   i n  the  em er gency   r oom ,   w hat
tr eatm ent  w oul d  y ou  pr escr i be,   and  w hy ?
3.   What  i s  the  cause  of   the  conti nui ng  r i se  i n  the  ser um   cr eati ni ne  l ev el
af ter   the  pati ent  i s  adm i tted  to  the  hospi tal ,   and  w hy ?
4.   What  i s  the  r ol e  f or   di ur eti cs  i n  thi s  pati ent,   and  w hat  i s  the  pr oper
dosage?
5.   What  i s  the  appr opr i ate  appr oach  to  f l ui d  m anagem ent  w hen  the  pati ent
becom es  ol i gur i c?
6.   What  ar e  the  i ndi cati ons  f or   acute  di al y si s  i n  acute  r enal   f ai l ur e,   and
w hat  al ter nati v e  ex tr acor por eal   pr ocedur es  coul d  be  consi der ed?
P. 386

Case Discussion
1.   At  the  ti m e  of   ar r i v al   i n  the  em er gency   r oom ,   w hat  i s  the  m ost  l i k el y
ex pl anati on  f or   thi s  pati ent's  acute  r enal   dy sf uncti on,   and  w hy ?

Ther e  i s  no  ev i dence  f or   a  postr enal   cause  of   the  acute  r enal   f ai l ur e  i n


thi s  pati ent,   gi v en  the  r enal   ul tr asound  study   show i ng  no  obstr ucti on.   Thi s
l eav es  pr er enal   and  i ntr ar enal   causes  as  the  sour ce  of   the  acute  r enal
f ai l ur e.   The  hi stor y   and  phy si cal   ex am i nati on  f i ndi ngs  suggest  pr er enal
azotem i a  stem m i ng  f r om   v ol um e  depl eti on.   The  l abor ator y   data  that
cor r obor ate  thi s  di agnosi s  i ncl ude  a  BU N –cr eati ni ne  r ati o  that  ex ceeds
20  and  a  f r acti onal   ex tr acti on  of   sodi um   (FEN a)  of   0. 13%.   The  FEN a  i s
cal cul ated  as  f ol l ow s:  U N a /P N a   ÷   U Cr /P Cr   ×  100%  =   10/146  ÷   80/1. 6  Ã
—  100%  =   0. 13%,   w her e  U N a   and  P N a   ar e  the  ur i ne  and  ser um   sodi um
l ev el s,   r especti v el y ,   and  U Cr   and  P Cr   ar e  the  ur i ne  and  ser um   l ev el s  of
cr eati ni ne,   r especti v el y .   In  the  setti ng  of   ol i gur i a  (< 400  m L  of   ur i ne  per
day ),   an  FEN a  of   l ess  than  1%  i m pl i es  pr er enal   azotem i a,   w her eas  an
FEN a  of   gr eater   than  2%  i m pl i es  an  i ntr ar enal   pr ocess.   In  pati ents  w ho
ar e  v ol um e  contr acted  due  to  di ur eti c  use,   the  FEN a  i s  of ten  el ev ated.   In
such  pati ents  the  f r acti onal   ex cr eti on  of   ur ea  (FEur ea)  m ay   be  m or e
usef ul ,   cal cul ated  as  U u rea /P u rea   ÷   U Cr /P Cr   ×  100.   A  v al ue  of   l ess  than
35%  suggests  pr er enal   azotem i a.

2.   At  the  ti m e  of   the  pati ent's  ar r i v al   i n  the  em er gency   r oom ,   w hat


tr eatm ent  w oul d  y ou  pr escr i be,   and  w hy ?

In  thi s  cl i ni cal   setti ng,   r epl eti on  of   the  ex tr acel l ul ar   f l ui d  v ol um e  i s  the
m ost  cr i ti cal   el em ent  of   ther apy .   Thi s  can  be  accom pl i shed  by   the
adm i ni str ati on  of   ei ther   nor m al   sal i ne  or   l actated  Ri nger 's  sol uti on;  250
to  500  m L  can  be  gi v en  r api dl y   ov er   1  to  2  hour s.   These  sol uti ons,   w hi ch
ar e  dev oi d  of   col l oi d,   di str i bute  i n  both  i ntr av ascul ar   and  ex tr av ascul ar
spaces.   Fl ui d  i nf usi on  shoul d  be  conti nued  unti l   the  bl ood  pr essur e
changes  ar e  no  l onger   ev i dent  and  a  euv ol em i c  state  has  been  r estor ed.
Thi s  w i l l   al so  be  accom pani ed  by   the  r eappear ance  of   sodi um   i n  the
ur i ne.   In  the  setti ng  of   pr er enal   azotem i a,   thi s  m aneuv er   shoul d  pr om ptl y
r etur n  r enal   f uncti on  to  basel i ne.

3.   What  i s  the  cause  of   the  conti nui ng  r i se  i n  the  ser um   cr eati ni ne  l ev el
af ter   the  pati ent  i s  adm i tted  to  the  hospi tal ,   and  w hy ?

Af ter   she  i s  adm i tted  to  the  hospi tal ,   the  pati ent's  cl i ni cal   pi ctur e
becom es  m or e  consi stent  w i th  an  i ntr ar enal   cause  of   acute  r enal   f ai l ur e,
such  as  acute  tubul ar   necr osi s.   Thi s  i s  suppor ted  by   the  pr esence  of
tubul ar   epi thel i al   cel l s  and  br ow n  gr anul ar   casts  i n  the  ur i ne.   In  addi ti on,
both  the  decr em ent  i n  the  U Cr /P Cr   to  16  and  the  i ncr ease  i n  the  FEN a  to
3. 57%  str ongl y   suppor t  thi s  di agnosi s.   As  to  the  cause  of   the  i ntr ar enal
i njur y   i tsel f ,   gr am ­negati v e  sepsi s  appear s  to  be  the  m ost  l i k el y   cul pr i t.
Am i nogl y cosi des  can  al so  cause  acute  r enal   f ai l ur e;  how ev er ,   thi s  pati ent
r ecei v ed  onl y   one  dose  of   the  anti bi oti c  and,   m or e  com m onl y ,   the
associ ated  r enal   f ai l ur e  i s  nonol i gur i c.   Am pi ci l l i n  can  cause  acute
i nter sti ti al   nephr i ti s,   w hi ch  has  been  r epor ted  f or   a  num ber   of   anti bi oti cs.
The  ur i nal y si s  w oul d  be  ex pected  to  show   w hi te  bl ood  cel l s,   r ed  bl ood
cel l s,   w hi te  bl ood  cel l   casts,   and  eosi nophi l s.

4.   What  i s  the  r ol e  f or   di ur eti cs  i n  thi s  pati ent,   and  w hat  i s  the  pr oper
dosage?

Di ur eti cs  hav e  been  used  i n  an  attem pt  to  conv er t  ol i gur i c  pati ents  w i th
acute  r enal   f ai l ur e  to  a  nonol i gur i c  state,   w hi ch  i s  associ ated  w i th  a
better   outcom e  and  si m pl er   f l ui d  m anagem ent.   Whether   thi s  â
€œconv er si onâ€​   tr ul y   al ter s  the  pr ognosi s  has
P. 387
not  been  settl ed.   Di ur eti cs  can  pl ay   a  m ajor   r ol e  i n  the  tr eatm ent  of   f l ui d
ov er l oad  that  accom pani es  the  pati ent's  di m i ni shed  ur i ne  output.   Because
l oop  di ur eti cs  need  to  r each  the  l um i nal   m em br ane  i n  thi s  setti ng,   v er y
hi gh  doses  ar e  r equi r ed  (240  to  300  m g  IV  of   f ur osem i de  or   8  to  12  m g  IV
of   bum etani de).   Doses  hi gher   than  these  hav e  been  used,   but  ar e  not
associ ated  w i th  an  i m pr ov ed  outcom e  and  can  cause  per m anent
ototox i ci ty .

5.   What  i s  the  appr opr i ate  appr oach  to  f l ui d  m anagem ent  w hen  the  pati ent
becom es  ol i gur i c?

When  a  pati ent  i s  ol i gur i c  (ur i ne  v ol um e  ≤400  m L),   f l ui d  r estr i cti on  i s
needed  and  i ntak e  shoul d  not  ex ceed  1  L  because  dai l y   i nsensi bl e  l osses
ar e  esti m ated  to  be  betw een  500  and  700  m L.   Li k ew i se,   sodi um   and
potassi um   r estr i cti on  i s  necessar y .   Ther ef or e,   the  adm i ni str ati on  of   1  L  of
0. 5  N   N aCl   (i . e. ,   appr ox i m atel y   75  m Eq  of   sodi um )  w i thout  potassi um
suppl em entati on  i s  l i k el y   to  pr ev ent  ex pansi on  of   the  ex tr acel l ul ar   f l ui d
v ol um e,   hy ponatr em i a,   and  hy per k al em i a.   If   the  epi sode  of   acute  r enal
f ai l ur e  i s  m or e  pr ol onged,   nutr i ti onal   suppor t  i s  al so  i m por tant.

6.   What  ar e  the  i ndi cati ons  f or   acute  di al y si s  i n  acute  r enal   f ai l ur e,   and


w hat  al ter nati v e  ex tr acor por eal   pr ocedur es  coul d  be  consi der ed?

Di al y si s  i s  under tak en  w henev er   any   of   the  com pl i cati ons  of   acute  r enal
f ai l ur e  ensue.   These  ar e  l i sted  i n  Tabl e  9­5.   Most  com m onl y ,   di al y si s  i s
i nsti tuted  f or   the  m anagem ent  of   f l ui d  ov er l oad  that  i s  r ef r actor y   to
di ur eti c  ther apy ,   hy per k al em i a  that  i s  r esi stant  to  ther apy ,   or   m etabol i c
aci dosi s  that  cannot  be  adequatel y   tr eated  w i th  bi car bonate.   In  ol i gur i c,
catabol i c  pati ents,   di al y si s  has  al so  been  used  to  pr ev ent  r ather   than
tr eat  ur em i c  sy m ptom s,   the  so­cal l ed  “pr ophy l acti c  di al y si s. â€​
Conti nuous  v enov enous  hem of i l tr ati on  (CVVH)  and  conti nuous  v enov enous
hem odi al y si s  (HD)  ar e  al ter nati v es  to  i nter m i ttent  HD,   and  ar e  bei ng  used
i ncr easi ngl y .

Suggested Readings
Edel stei n  CL,   Schr i er   RW.   Acute  r enal   f ai l ur e:  pathogenesi s,   di agnosi s,   and
m anagem ent.   In:  Schr i er   RW,   ed.   Renal   and  el ectr ol y te  di sor der s,   6th  ed.
Phi l adel phi a:  Li ppi ncott  Wi l l i am s  &  Wi l k i ns,   2003:  401.

Ki er an  N ,   Br ady   HR.   Cl i ni cal   ev al uati on,   m anagem ent,   and  outcom e  of
acute  r enal   f ai l ur e.   In:  Johnson  R,   Feehal l y   J,   eds.   Com pr ehensi v e  cl i ni cal
nephr ol ogy ,   2nd  ed.   Mosby ,   2003.

Metabolic Acidosis
1.   What  i s  the  def i ni ti on  of   m etabol i c  aci dosi s?

2.   What  com pensator y   m echani sm   i s  tr i gger ed  by   m etabol i c  aci dosi s?

3.   How   i s  the  ani on  gap  cal cul ated,   and  how   i s  i t  hel pf ul   i n  ev al uati ng
m etabol i c  aci dosi s?

4.   What  ar e  the  causes  of   a  m etabol i c  aci dosi s  w i th  an  i ncr eased  ani on  gap,
and  w hat  i s  the  ani on  r esponsi bl e  f or   the  i ncr eased  ani on  gap?

5.   How   i s  the  osm ol ar   gap  cal cul ated,   and  how   i s  thi s  v al ue  usef ul   i n
ev al uati ng  pati ents  w i th  a  m etabol i c  aci dosi s?

P. 388
6.   What  ar e  the  causes  of   a  m etabol i c  aci dosi s  w i th  a  nor m al   ani on  gap?

7.   What  i s  ur i nar y   ani on  gap  (U AG)  and  i n  w hat  ci r cum stances  i s  i t  usef ul ?

8.   What  i s  the  di f f er ence  betw een  pr ox i m al   and  di stal   r enal   tubul ar   aci dosi s
(RTA),   and  how   ar e  these  tw o  f or m s  of   RTA  di f f er enti ated?

Discussion
1.   What  i s  the  def i ni ti on  of   m etabol i c  aci dosi s?

Metabol i c  aci dosi s  i s  a  di sor der   that  r esul ts  f r om   ei ther   the  addi ti on  of
hy dr ogen  i on  or   the  l oss  of   bi car bonate,   w hi ch,   i f   unopposed,   r esul ts  i n
aci dem i a.   How ev er ,   m etabol i c  aci dosi s  i s  not  def i ned  ei ther   as  a
decr em ent  i n  the  ser um   bi car bonate  l ev el   or   as  any   gi v en  sy stem i c
ar ter i al   pH  because,   i n  the  setti ng  of   m i x ed  aci d–base  di sor der s.   The
ser um   bi car bonate  l ev el   or   pH,   or   both,   m ay   be  nor m al   or   ev en  el ev ated
despi te  the  pr esence  of   m etabol i c  aci dosi s.

2.   What  com pensator y   m echani sm   i s  tr i gger ed  by   m etabol i c  aci dosi s?

When  m etabol i c  aci dosi s  dev el ops,   the  decr ease  i n  pH  acti v ates  car oti d
chem or eceptor s  and  centr al   ner v ous  sy stem   r eceptor s  to  sti m ul ate
v enti l ati on.   The  i ncr ease  i n  the  m i nute  v enti l ati on  l ow er s  the  par ti al
pr essur e  of   car bon  di ox i de  (Pco 2 ),   ther eby   r etur ni ng  the  pH  tow ar d
nor m al .

3.   How   i s  ani on  gap  cal cul ated,   and  how   i s  i t  hel pf ul   i n  ev al uati ng  m etabol i c
aci dosi s?

Metabol i c  aci dosi s  i s  br oadl y   cl assi f i ed  on  the  basi s  of   the  pr esence  or
absence  of   an  i ncr eased  ani on  gap.   The  ani on  gap  (i n  m i l l i m ol es  per   l i ter )
i s  cal cul ated  usi ng  the  f ol l ow i ng  f or m ul a:  pl asm a  sodi um   ­  (pl asm a
chl or i de  +   pl asm a  bi car bonate).   In  m ost  l abor ator i es,   a  nor m al   ani on  gap
i s  consi der ed  to  be  12  ±   2  m m ol /L.   A  nor m al   ani on  gap  m etabol i c
aci dosi s  r esul ts  f r om   ei ther   the  addi ti on  of   hy dr ochl or i c  aci d  or   the  l oss
of   bi car bonate  w i th  the  concom i tant  r etenti on  of   chl or i de.   Because
chl or i de  i s  r etai ned  and  i s  i ncl uded  i n  the  cal cul ati on,   the  ani on  gap
m etabol i c  aci dosi s  i s  m ai ntai ned  i n  the  nor m al   r ange.   An  i ncr eased  ani on
gap  r esul ts  f r om   the  addi ti on  of   an  ex ogenous  or   endogenous  aci d.   The
ani ons  pr oduced  by   these  aci ds  ar e  not  m easur ed  and  chl or i de  i s  not
r etai ned.   The  ani on  gap  i ncr eases  because  bi car bonate  i s  consum ed  to
buf f er   the  or gani c  aci d.   For   ex am pl e,   or gani c  ani on  +   H +   +   N aHCO 3 ­   →
H 2 O  +   CO 2   +   N a  or gani c  ani on  +   or gani c  aci d.   Because  the  or gani c  ani on
i s  not  m easur ed  or   i ncl uded  i n  the  cal cul ati on,   the  ani on  gap  i ncr eases.

4.   What  ar e  the  causes  of   a  m etabol i c  aci dosi s  w i th  an  i ncr eased  ani on  gap,
and  w hat  i s  the  ani on  r esponsi bl e  f or   the  i ncr eased  ani on  gap?

The  v ar i ous  causes  of   m etabol i c  aci dosi s  w i th  an  i ncr eased  ani on  gap  ar e
l i sted  i n  Tabl e  9­6.

5.   How   i s  the  osm ol ar   gap  cal cul ated,   and  how   i s  thi s  v al ue  usef ul   i n
ev al uati ng  pati ents  w i th  a  m etabol i c  aci dosi s?

The  pl asm a  osm ol al i ty   i s  cal cul ated  usi ng  the  f ol l ow i ng  f or m ul a:
Cal cul ated  osm ol al i ty   =   2[N a]  +   [gl ucose]/18  +   [BU N ]/2. 8  +
[ethanol ]/4. 6.   The  osm ol ar   gap  i s  equal   to  the  m easur ed  osm ol al i ty   m i nus
the  cal cul ated  osm ol al i ty .   A  nor m al   osm ol ar   gap  i s  l ess  than  10
m Osm /k g.   When  the  osm ol ar
P. 389
gap  i s  el ev ated  i n  an  aci dem i c  pati ent,   ethy l ene  gl y col   or   m ethanol
i ntox i cati on  m ust  be  str ongl y   suspected.

Table 9­6 Causes of Metabolic Acidosis with
an Increased Anion Gap

Ca us e Anion

Incr eased  aci d  pr oducti on  

    Di abeti c  k etoaci dosi s BHB,   AcAc

    Lacti c  aci dosi s Lactate,   py r uv ate


    Star v ati on —

    Al cohol i c  k etoaci dosi s BHB  >   AcAc

    N onk etoti c  hy per osm ol ar   com a —

    Inbor n  er r or s  of   m etabol i sm —

Ingesti on  of   aci d­gener ati ng  tox i c


 
substances

    Sal i cy l ate  ov er dose  (> 30  m g/dL) Var i ety

    Methanol   i ngesti on For m ate,   l actate

    Ethy l ene  gl y col   i ngesti on Lactate,   gl y col ate,


ox al ate

    Sol v ent  i nhal ati on —

Fai l ur e  of   aci d  ex cr eti on  

    Acute  r enal   f ai l ur e Var i ety ,   SO4,   PO4

    Chr oni c  r enal   f ai l ur e —

BHB,   betahy dr ox y buty r ate;  AcAc,   acetoacetate.

6.   What  ar e  the  causes  of   a  m etabol i c  aci dosi s  w i th  a  nor m al   ani on  gap?

The  causes  of   m etabol i c  aci dosi s  w i th  a  nor m al   ani on  gap  ar e  l i sted  i n
Tabl e  9­7.

7.   What  i s  U AG  and  i n  w hat  ci r cum stances  i s  i t  usef ul ?

On  occasi on,   the  U AG  m ay   hel p  di sti ngui sh  gastr oi ntesti nal   l oss  f r om
r enal   l oss  of   HCO 3 ­   as  the  cause  of   hy per chl or em i c  m etabol i c  aci dosi s:

The  U AG  i s  an  esti m ate  of   the  ur i nar y   am m oni um   that  i s  el ev ated  i n
gastr oi ntesti nal   HCO 3 ­   l oss  but  l ow   i n  di stal   RTA.   U AG  i s  a  negati v e  v al ue
i f   ur i ne  am m oni um   i s  hi gh  (as  i n  di ar r hea;  av er age,   ­20  m Eq/L),   w her eas
i t  i s  posi ti v e  i f   ur i ne  am m oni um   i s  l ow   (as  i n  di stal   RTA;  av er age,   + 23
m Eq/L).

8.   What  i s  the  di f f er ence  betw een  pr ox i m al   and  di stal   RTA,   and  how   ar e
these  tw o  f or m s  of   RTA  di f f er enti ated?

RTA  i s  one  of   the  com m on  causes  of   m etabol i c  aci dosi s  w i th  a  nor m al
ani on  gap.   Pr ox i m al   RTA  r esul ts  f r om   a  f ai l ur e  to  r esor b  the  nor m al
am ount  of   bi car bonate  i n  the  pr ox i m al   tubul e,   w her eas  di stal   RTA  r esul ts
f r om   a  def ect  i n  hy dr ogen  i on  secr eti on  i n  the  di stal   tubul e.   These  tw o
f or m s  of   RTA  can  be  di f f er enti ated  by   deter m i ni ng  the  ur i ne  pH  dur i ng
sy stem i c  aci dosi s.   In  pr ox i m al   RTA,   w hen  the  ser um   bi car bonate,   and
ther ef or e  the  f i l ter ed
P. 390
bi car bonate  l ev el ,   i s  l ow er ed  to  one  that  al l ow s  f or   pr ox i m al   r eabsor pti on
of   al l   the  f i l ter ed  bi car bonate,   the  ur i ne  can  be  m ax i m al l y   aci di f i ed  (pH
< 5. 4)  as  ther e  i s  no  i ncr eased  di stal   del i v er y   of   unr eabsor bed
bi car bonate.   In  contr ast,   i n  di stal   RTA,   the  ur i ne  cannot  be  m ax i m al l y
aci di f i ed  (pH  > 5. 4)  i ndependent  of   the  ser um   bi car bonate  concentr ati on.

Table 9­7 The Causes of a Metabolic Acidosis
with a Normal Anion Gap

Gastr oi ntesti nal   l oss  of   HCO3


Di ar r hea
Sm al l   bow el   or   pancr eati c  dr ai nage  or   f i stul a
U r eter osi gm oi dostom y ,   l ong  or   obstr ucted  i l eal   l oop  condui t
Ani on  ex change  r esi ns
Ingesti on  of   CaCl 2  or   MgCl 2
Renal   l oss  of   HCO3  ­
Car boni c  anhy dr ase  i nhi bi tor s
Renal   tubul ar   aci dosi s
Hy per par athy r oi di sm
Hy poal doster oni sm
Mi scel l aneous
Recov er y   f r om   k etoaci dosi s
Di l uti onal   aci dosi s
Inf usi on  of   HCl   or   i ts  congener s
Par enter al   al i m entati on  aci dosi s a

a Som e  f or m ul as  contai n  ex cess  or gani c  cati ons  (bal anced  by   Cl ­),
w hi ch  y i el d  H+   on  m etabol i sm .

Case
A  29­y ear ­ol d  m an  has  been  hospi tal i zed  i n  the  psy chi atr y   ser v i ce  f or   2
m onths  because  of   depr essi on.   The  pati ent  l eav es  the  hospi tal   on  a  pass  and,
on  r etur ni ng,   com pl ai ns  of   abdom i nal   pai n  and  v om i ti ng.   Ov er   the  nex t  sev er al
hour s,   he  becom es  m or e  agi tated  and  i s  then  f ound  i n  an  unar ousabl e  state
and  postur i ng.
Phy si cal   ex am i nati on  r ev eal s  a  tem per atur e  of   102°F  (38. 8°C),   pul se  of   102
beats  per   m i nute,   r espi r ator y   r ate  of   35  br eaths  per   m i nute,   and  bl ood
pr essur e  of   160/100  m m   Hg.   The  pati ent  i s  unr esponsi v e  to  pai n.   Funduscopi c
f i ndi ngs  ar e  w i thi n  nor m al   l i m i ts.   N o  odor s  ar e  noted  on  hi s  br eath.
Labor ator y   f i ndi ngs  r ev eal   the  f ol l ow i ng:  sodi um ,   142  m Eq/L;  potassi um ,   4. 7
m Eq/L;  chl or i de,   111  m Eq/L;  bi car bonate,   10  m m ol /L;  ser um   cal ci um ,   9. 4
m g/dL;  BU N ,   12  m g/dL;  and  cr eati ni ne,   1. 3  m g/dL.   Ar ter i al   bl ood  gas
m easur em ents  per f or m ed  on  r oom   ai r   show   a  pH  of   7. 2,   PCO 2   of   17  m m   Hg,
and  par ti al   pr essur e  of   ox y gen  (PO 2 )  of   100  m m   Hg.

1.   What  i s  thi s  pati ent's  aci d–base  di stur bance,   and  w hat  ar e  the  possi bl e
causes?
2.   Why   i s  the  pati ent  tachy pnei c,   and  i s  the  com pensati on  appr opr i ate?
3.   What  other   tests  or   l abor ator y   f i ndi ngs  w oul d  be  usef ul   i n  m ak i ng  the
speci f i c  di agnosi s?
P. 391
4.   In  thi s  pati ent,   the  ser um   gl ucose  l ev el   pr ov es  to  be  nor m al   and  no
ser um   k etones  ar e  detected.   The  pl asm a  osm ol al i ty   i s  347  m Osm /k g  and
the  osm ol ar   gap  i s  cal cul ated  to  be  51  m Osm /k g.   Wi th  the  new
i nf or m ati on  y i el ded  by   these  addi ti onal   tests,   w hat  possi bl e  di agnoses
sti l l   r em ai n?
5.   How   w oul d  y ou  pr oceed  to  deter m i ne  w hi ch  substance  i s  r esponsi bl e  f or
thi s  pati ent's  pr esentati on?
6.   How   w oul d  y ou  tr eat  thi s  pati ent?

Case Discussion
1.   What  i s  thi s  pati ent's  aci d–base  di stur bance,   and  w hat  ar e  the  possi bl e
causes?

The  pati ent  has  an  aci dem i a  because  the  pH  i s  7. 2.   Thi s  coul d  r esul t  f r om
ei ther   a  m etabol i c  or   a  r espi r ator y   aci dosi s.   The  com bi nati on  of   a  l ow
PCO 2   and  a  l ow   ser um   bi car bonate  concentr ati on  conf i r m s  the  pr esence  of
a  m etabol i c  aci dosi s.   In  addi ti on,   the  ani on  gap  i s  el ev ated.   The  m ost
l i k el y   causes  of   a  m etabol i c  aci dosi s  w i th  an  i ncr eased  ani on  gap,   as
outl i ned  i n  Tabl e  9­6,   i ncl ude  di abeti c  k etoaci dosi s,   l acti c  aci dosi s,
star v ati on,   al cohol i c  k etoaci dosi s,   sal i cy l ate  ov er dose,   m ethanol   or
ethy l ene  gl y col   i ngesti on,   and  r enal   f ai l ur e.

2.   Why   i s  the  pati ent  tachy pnei c,   and  i s  the  com pensati on  appr opr i ate?

The  pati ent  i s  tachy pnei c  as  a  com pensator y   r esponse  to  the  m etabol i c
aci dosi s.   If   the  pati ent  w er e  not  tachy pnei c,   the  pH  w oul d  be  ev en  l ow er
and  thi s  w oul d  suggest  an  addi ti onal   r espi r ator y   di sor der .   Thi s  pati ent  i s
ex hi bi ti ng  an  appr opr i ate  r espi r ator y   com pensator y   r esponse.   The  ser um
bi car bonate  l ev el   i s  decr eased  by   14  m m ol /L  f r om   nor m al .   Ther ef or e,   the
PCO 2   shoul d  be  decr eased  by   14  to  21  m m   Hg  (Tabl e  9­8).   The  pati ent
has  a  PCO 2   that  i s  decr eased  by   21  m m   Hg  f r om   nor m al ,   and  thi s
com pensati on  i s  appr opr i ate  f or   the  degr ee  of   m etabol i c  aci dosi s
i nv ol v ed.   Tabl e  9­8  sum m ar i zes  the  gener al   ex pected  com pensator y
r esponses  to  aci d–base  di sor der s.

3.   What  other   tests  or   l abor ator y   f i ndi ngs  w oul d  be  usef ul   i n  m ak i ng  the
speci f i c  di agnosi s?

The  pati ent  cl ear l y   has  a  m etabol i c  aci dosi s  w i th  an  i ncr eased  ani on  gap,
but  i t  i s  necessar y   to  i denti f y   the  speci f i c  cause  w i th  f ur ther   testi ng.
Ini ti al   tests  that  m i ght  el uci date  the  cause  of   the  pr ocess  i ncl ude  (a)  the
ser um   gl ucose  l ev el   to  deter m i ne  w hether   hy per gl y cem i a  i s  pr esent;  (b)
ser um   k etone  l ev el s  to  ascer tai n  i f   acetoacetate  i s  pr esent;  (c)  ser um
sal i cy l ate  and  l actate  l ev el s  to  deter m i ne  w hether   sal i cy l ate  i ntox i cati on
or   l acti c  aci dosi s  i s  pr esent;  and  (d)  ser um   osm ol al i ty   to  deter m i ne  i f   the
osm ol ar   gap  i s  el ev ated.

4.   In  thi s  pati ent,   the  ser um   gl ucose  l ev el   pr ov es  to  be  nor m al   and  no
ser um   k etones  ar e  detected.   The  pl asm a  osm ol al i ty   i s  347  m Osm /k g  and
the  osm ol ar   gap  i s  cal cul ated  to  be  51  m Osm /k g.   Wi th  the  new
i nf or m ati on  y i el ded  by   these  addi ti onal   tests,   w hat  possi bl e  di agnoses
sti l l   r em ai n?

Wi th  thi s  addi ti onal   i nf or m ati on,   y ou  k now   that  the  pati ent  has  m etabol i c
aci dosi s  w i th  an  i ncr eased  ani on  and  osm ol ar   gap.   Thi s  l i m i ts  the  possi bl e
di agnoses  to  ei ther   m ethanol   or   ethy l ene  gl y col   i ngesti on.

P. 392
5.   How   w oul d  y ou  pr oceed  to  deter m i ne  w hi ch  substance  i s  r esponsi bl e  f or
thi s  pati ent's  pr esentati on?

Table 9­8 Rules of Thumb for Bedside
Interpretation of Acid­Base Disorders

Metabol i c  aci dosi s


Paco  2  shoul d  f al l   by 1. 0­1. 5  ×  the  f al l   i n  pl asm a
[HCO3­]
Metabol i c  al k al osi s
Paco  2  shoul d  r i se  by 0. 25­1. 0  ×  the  r i se  i n  pl asm a
[HCO3­]
Acute  r espi r ator y   aci dosi s
Pl asm a  [HCO3  ­]  shoul d  r i se  by   appr ox i m atel y   1
m m ol /L  f or   each  10­m m   Hg  i ncr em ent  i n  Paco  2  (±   3
m m ol /L)
Chr oni c  r espi r ator y   aci dosi s
Pl asm a  [HCO3  ­]  shoul d  r i se  by   appr ox i m atel y   4
m m ol /L  f or   each  10­m m   Hg  i ncr em ent  i n  Paco  2  (±   4
m m ol /L)
Acute  r espi r ator y   al k al osi s
Pl asm a  [HCO3  ­]  shoul d  f al l   by   appr ox i m atel y   1  ­  3
m m ol /L  f or   each  10­m m   Hg  decr em ent  i n  Paco2,   but
usual l y   not  to< 18  m m ol /L
Chr oni c  r espi r ator y   al k al osi s
Pl asm a  [HCO3  ­]  shoul d  f al l   by   appr ox i m atel y   2  ­
5m m ol /L  per   10­m m   Hg  decr em ent  i n  Paco2,   but
usual l y   not  to< 14  m m ol /L

Paco2,   ar ter i al   car bon  di ox i de  tensi on;  [HCO3  ­],   bi car bonate  i on
concentr ati on.

Fr om   Shapi r o  JI,   Kaehny   WD.   Pathogenesi s  and  m anagem ent  of


m etabol i c  aci dosi s  and  al k al osi s.   In:  Schr i er   RW,   ed.   Renal   and
el ectr ol y te  di sor der s,   6th  ed.   Phi l adel phi a:  Li ppi ncott  Wi l l i am s  &
Wi l k i ns,   2003.   Repr i nted  w i th  per m i ssi on.

To  deter m i ne  w hi ch  substance  i s  r esponsi bl e  f or   thi s  pati ent's


pr esentati on,   both  m ethanol   and  ethy l ene  gl y col   l ev el s  shoul d  be  assay ed
i n  the  bl ood.   In  addi ti on,   the  ur i ne  shoul d  be  ex am i ned  f or   the  pr esence
of   cal ci um   ox al ate  cr y stal s,   w hi ch  ar e  f r equentl y   pr esent  i n  the  setti ng  of
ethy l ene  gl y col   i ngesti on  because  of   the  m etabol i c  conv er si on  of   the
ethy l ene  gl y col   to  ox al ate.   In  the  setti ng  of   m ethanol   i ntox i cati on,   v i sual
di stur bances  coul d  ensue.

6.   How   w oul d  y ou  tr eat  thi s  pati ent?

The  tr eatm ent  of   m etabol i c  aci dosi s  i nv ol v es  tr eati ng  the  under l y i ng
di sor der .   In  acute  m etabol i c  aci dosi s,   the  r api d  cor r ecti on  of   pH  thr ough
the  adm i ni str ati on  of   bi car bonate  appear s  to  pr oduce  der angem ents  i n
car di ov ascul ar   f uncti on,   pr obabl y   caused  by   a  par adox i cal   i ntr acel l ul ar
aci dosi s.   The  use  of   bi car bonate  i n  thi s  setti ng  i s  ther ef or e  contr ov er si al .
Mor e  speci f i cal l y ,   tw o  goal s  becom e  i m por tant  i n  a  pati ent  w ho  has
i ngested  ethy l ene  gl y col .   The  f i r st  i s  to  i nhi bi t  the  m etabol i sm   of
ethy l ene  gl y col .   Al though  ethy l ene  gl y col   by   i tsel f   i s  not  a  tox i c
substance,   the  m etabol i tes  pr oduced  by   the  l i v er   ar e  qui te  tox i c  and  can
pr eci pi tate  acute  r enal   f ai l ur e  and  ev en  cause  death.   Al cohol
dehy dr ogenase  (ADH)  i s  the  enzy m e  r esponsi bl e  f or   the  m etabol i sm   of
ethy l ene  gl y col ,   and  i t  can  be  com peti ti v el y   i nhi bi ted  by   ethanol .
Fom epi zol e,   a  di r ect  i nhi bi tor   of   ADH  has  al so  been  em pl oy ed.   Ethy l ene
gl y col   i ngesti on  i s  sti l l   m ost  com m onl y   tr eated  by   the  i nf usi on  of   ethanol .
The  second  goal
P. 393
i s  to  r em ov e  the  ethy l ene  gl y col   f r om   the  body .   Ethy l ene  gl y col   i s
ex cr eted  v er y   sl ow l y   by   the  k i dney s  and,   i f   the  bl ood  l ev el   i s  v er y   hi gh,
HD  m ay   becom e  necessar y   to  i m pr ov e  r em ov al   of   thi s  substance  f r om   the
bl ood.   A  si m i l ar   appr oach  i s  used  f or   m ethanol   i ngesti on.

Suggested Readings
Pal m er   BF,   Al per n  RJ.   N or m al   aci d­base  bal ance  and  m etabol i c  aci dosi s.
In:  Johnson  R,   Feehal l y   J,   eds.   Com pr ehensi v e  cl i ni cal   nephr ol ogy ,   2nd  ed.
Mosby ,   2003.

Shapi r o  JI,   Kaehny   WD.   Pathogenesi s  and  m anagem ent  of   m etabol i c


aci dosi s  and  al k al osi s.   In:  Schr i er   RW,   ed.   Renal   and  el ectr ol y te  di sor der s,
6th  ed.   Phi l adel phi a:  Li ppi ncott  Wi l l i am s  &  Wi l k i ns,   2003:  115.

Metabolic Alkalosis
1.   What  i s  the  def i ni ti on  of   m etabol i c  al k al osi s?

2.   What  ar e  the  pr ocesses  i nv ol v ed  i n  the  gener ati on  of   m etabol i c  al k al osi s?

3.   What  ar e  the  pr ocesses  i nv ol v ed  i n  the  m ai ntenance  of   m etabol i c


al k al osi s?

4.   What  ar e  the  tw o  m ajor   categor i es  of   m etabol i c  al k al osi s,   and  w hat


l abor ator y   test  i s  used  to  di f f er enti ate  betw een  the  tw o?

5.   What  ar e  the  causes  of   N aCl ­r esponsi v e  m etabol i c  al k al osi s?

6.   What  ar e  the  causes  of   N aCl ­r esi stant  m etabol i c  al k al osi s?

7.   What  ar e  the  causes  of   m etabol i c  al k al osi s  that  ar e  uncl assi f i ed?

8.   What  i s  the  com pensator y   m echani sm   that  i s  sti m ul ated  by   m etabol i c


al k al osi s?

Discussion
1.   What  i s  the  def i ni ti on  of   m etabol i c  al k al osi s?

Metabol i c  al k al osi s  i s  a  di sor der   that  r esul ts  f r om   ei ther   the  l oss  of
hy dr ogen  i ons  or   the  addi ti on  of   bi car bonate,   w hi ch,   i f   unopposed,   r esul ts
i n  al k al em i a.   Metabol i c  al k al osi s  i s  not  def i ned  ei ther   as  an  i ncr em ent  i n
the  ser um   bi car bonate  concentr ati on  or   as  a  gi v en  sy stem i c  ar ter i al   pH
because,   i n  the  setti ng  of   m i x ed  aci d–base  di sor der s.   The  ser um
bi car bonate  l ev el   or   the  pH,   or   both,   coul d  be  ei ther   nor m al   or   ev en
decr eased  i n  the  pr esence  of   m etabol i c  al k al osi s.

2.   What  ar e  the  pr ocesses  i nv ol v ed  i n  the  gener ati on  of   m etabol i c  al k al osi s?

Pathophy si ol ogi cal l y ,   the  dev el opm ent  of   m etabol i c  al k al osi s  i nv ol v es  tw o
phases  (see  Fi g.   9­1).   The  f i r st  i nv ol v es  the  gener ati on  of   m etabol i c
al k al osi s.   As  f ol l ow s  f r om   the  def i ni ti on  just  gi v en,   m etabol i c  al k al osi s
can  be  gener ated  as  a  r esul t  of   ei ther   a  net  l oss  of   hy dr ogen  i ons  f r om
the  ex tr acel l ul ar   f l ui d,   m ost  com m onl y   f r om   ei ther   the  upper
gastr oi ntesti nal   tr act  or   m or e  r ar el y   thr ough  the  k i dney s,   or   f r om   the  net
addi ti on  of   bi car bonate  or   substances  that  gener ate  bi car bonate  (e. g. ,
l actate,   ci tr ate,   and  acetate).   In  addi ti on,   the
P. 394
l oss  of   f l ui d  hav i ng  hi gh  concentr ati ons  of   chl or i de  and  l ow   concentr ati ons
of   bi car bonate,   as  occur s  w i th  di ur eti c  use  and  cer tai n  gastr oi ntesti nal
tr act  di seases  such  as  v i l l ous  adenom a,   gener ates  a  m etabol i c  al k al osi s.

Figure  9­1  The  f actor s  r esponsi bl e  f or   the  gener ati on  and  m ai ntenance  of
m etabol i c  al k al osi s.   ECF,   ex tr acel l ul ar   f l ui d;  AII,   angi otensi n  II;  Paco 2 ,
par ti al   pr essur e  of   car bon  di ox i de.   (Fr om   Shapi r o  JI,   Kaehny   WD.
Pathogenesi s  and  m anagem ent  of   m etabol i c  aci dosi s  and  al k al osi s.   In:
Schr i er   RW,   ed.   Renal   and  el ectr ol y te  di sor der s,   6th  ed.   Phi l adel phi a:
Li ppi ncott  Wi l l i am s  &  Wi l k i ns,   2003.   Repr i nted  w i th  per m i ssi on. )

3.   What  ar e  the  pr ocesses  i nv ol v ed  i n  the  m ai ntenance  of   m etabol i c


al k al osi s?

The  k i dney   pr ov i des  the  cor r ecti v e  r esponse  to  m etabol i c  al k al osi s  by
ex cr eti ng  ex cess  bi car bonate.   When  the  ser um   bi car bonate  l ev el   ex ceeds
28  m Eq/L,   the  ani on  appear s  i n  the  ur i ne,   ther eby   pr ev enti ng  a  f ur ther
i ncr ease  i n  i ts  concentr ati on.   The  m ai ntenance  of   al k al osi s  ther ef or e
r equi r es  an  al ter ati on  i n  r enal   bi car bonate  r eabsor pti on.   Sev er al   f actor s
constr ai n  the  k i dney 's  abi l i ty   to  ex cr ete  bi car bonate  and  ar e  i m por tant  i n
the  m ai ntenance  phase  of   m etabol i c  al k al osi s.   Pr obabl y ,   the  m ost
i m por tant  f actor   i n  thi s  r egar d  i s  ex tr acel l ul ar   f l ui d  v ol um e  depl eti on,
w hi ch  ser v es  to  sti m ul ate  i ncr eased  sodi um   r esor pti on  and  bi car bonate
r ecl am ati on  i n  the  pr ox i m al   tubul e.   A  decr em ent  i n  GFR  w i th  a  decr ease
i n  bi car bonate  f i l tr ati on  contr i butes  to  the  m ai ntenance  of   the  m etabol i c
al k al osi s.   Another   i m por tant  f actor   i n  the  m ai ntenance  of   m etabol i c
al k al osi s  i s  the  chl or i de  concentr ati on.   When  the  pl asm a  bi car bonate
concentr ati on  r i ses,   the  chl or i de  concentr ati on  m ust  f al l .   Because  chl or i de
i s  the  onl y   ani on  other   than  bi car bonate  that  can  accom pany   sodi um
r esor pti on,   bi car bonate  r esor pti on  i s  enhanced  i n  i ts  absence.   Ther ef or e,
chl or i de  m ust  ex i st  i n  suf f i ci ent  quanti ty   to  al l ow   f or   bi car bonate
ex cr eti on.   The  hor m one  al doster one  sti m ul ates  the  ex change  of   sodi um
r eabsor pti on  f or   hy dr ogen  or   potassi um   i on  secr eti on  i n  the  di stal   tubul e.
Wi th
P. 395
the  secr eti on  of   hy dr ogen  i ons,   bi car bonate  gener ati on  occur s  i n  the
pl asm a.   Potassi um   i on  depl eti on  di r ectl y   enhances  bi car bonate
r eabsor pti on.   An  el ev ati on  i n  the  Pco 2   al so  sti m ul ates  bi car bonate
r eabsor pti on,   and  i s  i m por tant  i n  the  com pensator y   m echani sm   that  k eeps
r espi r ator y   aci dosi s  i n  check .

Table 9­9 Causes of NaCl­Responsive
Metabolic Alkalosis

Gastr oi ntesti nal   di sor der s


Vom i ti ng
Gastr i c  dr ai nage
Vi l l ous  adenom a  of   the  col on
Chl or i de  di ar r hea
Di ur eti c  ther apy
Cor r ecti on  of   chr oni c  hy per capni a
Cy sti c  f i br osi s
4.   What  ar e  the  tw o  m ajor   categor i es  of   m etabol i c  al k al osi s,   and  w hat
l abor ator y   test  i s  used  to  di f f er enti ate  betw een  the  tw o?

Metabol i c  al k al osi s  can  be  di v i ded  i nto  tw o  gr oups:  N aCl   r esponsi v e  and
N aCl   r esi stant.   The  f or m er   i s  f ound  i n  al k al em i c  pati ents  w ho  ar e  v ol um e
depl eted,   and  the  l atter   i n  those  w i th  v ol um e  ex pansi on.   The  m ost  usef ul
l abor ator y   test  f or   di f f er enti ati ng  betw een  the  tw o  gr oups  i s  a  spot  ur i ne
chl or i de  deter m i nati on  done  bef or e  the  i ni ti ati on  of   ther apy .   In  N aCl ­
r esponsi v e  states,   the  ur i ne  chl or i de  concentr ati on  i s  usual l y   l ess  than  20
m Eq/L,   and  f r equentl y   ev en  l ess  than  10  m Eq/L;  i n  N aCl ­r esi stant  states,
the  ur i ne  chl or i de  l ev el   ex ceeds  20  m Eq/L.   How ev er ,   al though  m etabol i c
al k al osi s  i s  r outi nel y   di v i ded  i nto  these  tw o  categor i es,   ther e  ar e  sev er al
di sor der s  that  ar e  uncl assi f i ed.

5.   What  ar e  the  causes  of   N aCl ­r esponsi v e  m etabol i c  al k al osi s?

The  causes  of   N aCl ­r esponsi v e  m etabol i c  al k al osi s  ar e  l i sted  i n  Tabl e  9­9.

6.   What  ar e  the  causes  of   N aCl ­r esi stant  m etabol i c  al k al osi s?

The  causes  of   N aCl ­r esi stant  m etabol i c  al k al osi s  ar e  l i sted  i n  Tabl e  9­10.

7.   What  ar e  the  causes  of   m etabol i c  al k al osi s  that  ar e  uncl assi f i ed?

The  uncl assi f i ed  causes  of   m etabol i c  al k al osi s  ar e  l i sted  i n  Tabl e  9­11.

8.   What  i s  the  com pensator y   m echani sm   that  i s  sti m ul ated  by   m etabol i c


al k al osi s?

When  m etabol i c  al k al osi s  dev el ops,   the  al k al em i a  i s  sensed  by


chem or eceptor s  i n  the  r espi r ator y   sy stem .   Thi s  l eads  to  hy pov enti l ati on
and  an  i ncr ease
P. 396
i n  Pco 2 .   As  a  gener al   r ul e,   the  Δ  Pco 2   (m m   Hg)  =   0. 25  ­  1. 0  M  ×

[HCO 3 ­ ]  m Eq/L,   w her e  Δ  Pco 2   i s  the  change  i n  the  Pco 2 .   How ev er ,   thi s
hy pov enti l ator y   r esponse  i s  not  as  ef f i ci ent  as  the  hy per v enti l ator y
r esponses  that  accom pany   a  m etabol i c  aci dosi s.

Table 9­10 Causes of NaCl­Resistant Metabolic
Alkalosis

Ex cess  m i ner al ocor ti coi d


Hy per al doster oni sm
Cushi ng's
Bar tter   sy ndr om e
Ex cessi v e  l i cor i ce  i ntak e
Pr of ound  potassi um   depl eti on  (800­1, 000  m Eq  def i ci t)

Table 9­11 Unclassified Causes of Metabolic
Alkalosis

Al k al i   adm i ni str ati on


Recov er y   f r om   or gani c  aci dosi s
Antaci ds  and  ex change  r esi ns  adm i ni ster ed  i n  r enal   f ai l ur e
Mi l k ­al k al i   sy ndr om e
Massi v e  bl ood  or   pl asm anate  (hum an  pl asm a  pr otei n  f acti on)
tr ansf usi ons
N onpar athy r oi d  hy per cal cem i a
Gl ucose  i ngesti on  af ter   star v ati on
Lar ge  doses  of   car beni ci l l i n  or   peni ci l l i n

Case
A  25­y ear ­ol d  m an  w i th  no  pr ev i ous  m edi cal   hi stor y   pr esents  to  the
em er gency   r oom   because  of   abdom i nal   pai n  and  sev er e  v om i ti ng  of   2  day s'
dur ati on,   dur i ng  w hi ch  ti m e  he  has  been  unabl e  to  eat  or   dr i nk .   He  i s  tak i ng
no  m edi cati ons.
Phy si cal   ex am i nati on  r ev eal s  the  f ol l ow i ng:  tem per atur e,   37. 6°C  (99. 68°F);
pul se,   120  beats  per   m i nute;  r espi r ator y   r ate,   18  br eaths  per   m i nute;  and
bl ood  pr essur e,   120/80  m m   Hg.   Or thostati c  changes  i n  the  pul se  and  bl ood
pr essur e  ar e  f ound,   and  ther e  i s  m i l d,   di f f use  abdom i nal   tender ness.
The  f ol l ow i ng  l abor ator y   f i ndi ngs  ar e  r epor ted:  sodi um ,   140  m Eq/L;  potassi um ,
3. 4  m Eq/L;  chl or i de,   90  m Eq/L;  bi car bonate,   35  m m ol /L;  and  cr eati ni ne,   1. 5
m g/dL.   Ar ter i al   bl ood  gas  m easur em ents  on  r oom   ai r   r ev eal   a  pH  of   7. 55,
PCO 2   of   44  m m   Hg,   and  PO 2   of   77  m m   Hg.

1.   What  aci d–base  di stur bances  ar e  pr esent  i n  thi s  pati ent?


2.   What  ar e  the  possi bl e  causes  of   thi s  pati ent's  m etabol i c  al k al osi s,   and
w hat  l abor ator y   test  m i ght  be  usef ul   to  el uci date  the  natur e  of   the  cause?
3.   What  f actor s  ar e  r esponsi bl e  f or   the  gener ati on  and  m ai ntenance  of   the
m etabol i c  al k al osi s  i n  thi s  pati ent?
4.   If   the  pati ent's  v om i ti ng  w er e  to  stop  spontaneousl y ,   w oul d  the  aci dâ
€“base  di stur bance  al so  r esol v e?
5.   How   w oul d  y ou  tr eat  thi s  pati ent?

Case Discussion
1.   What  aci d–base  di stur bances  ar e  pr esent  i n  thi s  pati ent?

The  pati ent  i s  al k al em i c  (pH,   7. 55).   Ther ef or e,   ei ther   a  m etabol i c


al k al osi s  or   a  r espi r ator y   al k al osi s,   or   both,   ex i st.   The  ser um   bi car bonate
l ev el   i s  el ev ated  to  35  m Eq/L,   and  thi s  i ndi cates  a  m etabol i c  al k al osi s.   In
the  setti ng  of   a  r espi r ator y
P. 397
al k al osi s,   the  PCO 2   w oul d  be  decr eased,   w hi ch  i s  not  the  case  i n  thi s
pati ent.   In  the  setti ng  of   m etabol i c  al k al osi s,   the  ex pected  r espi r ator y
com pensati on  (hy pov enti l ati on)  w oul d  i ncr ease  the  PCO 2 .   Because  the
PCO 2   of   44  m m   Hg  i s  an  i ncr eased  v al ue,   thi s  f ur ther   suppor ts  the
pr esence  of   a  si m pl e  m etabol i c  al k al osi s  w i th  appr opr i ate  r espi r ator y
com pensati on.

2.   What  ar e  the  possi bl e  causes  of   thi s  pati ent's  m etabol i c  al k al osi s,   and
w hat  l abor ator y   test  m i ght  be  usef ul   to  el uci date  the  natur e  of   the  cause?

As  al r eady   di scussed,   m etabol i c  al k al osi s  can  be  di v i ded  i nto  tw o  br oad
categor i es:  N aCl ­r esponsi v e  and  N aCl ­r esi stant  states.   The  hal l m ar k   of
N aCl ­r esponsi v e  m etabol i c  al k al osi s  i s  i ntr av ascul ar   v ol um e  depl eti on.   In
thi s  pati ent,   the  hi stor y   of   sev er e  v om i ti ng  pl us  the  v i tal   si gns  that
ex hi bi t  or thostati c  changes  ar e  v er y   suggesti v e  of   an  N aCl ­r esponsi v e
m etabol i c  al k al osi s  w i th  i ntr av ascul ar   v ol um e  depl eti on.   The  other   causes
of   an  N aCl ­r esponsi v e  m etabol i c  al k al osi s  ar e  nasogastr i c  dr ai nage,
v i l l ous  adenom a  of   the  col on,   chl or i de  di ar r hea,   and  di ur eti c  ther apy .
Measur em ent  of   a  spot  ur i ne  chl or i de  concentr ati on  w oul d  hel p  conf i r m
the  di agnosi s.   In  thi s  pati ent,   i t  w oul d  l i k el y   be  l ow   (< 20  m Eq/L).

3.   What  f actor s  ar e  r esponsi bl e  f or   the  gener ati on  and  m ai ntenance  of   the
m etabol i c  al k al osi s  i n  thi s  pati ent?

In  the  m etabol i c  al k al osi s  associ ated  w i th  v om i ti ng,   the  l oss  of   hy dr ogen
i ons  i n  the  v om i tus  i s  r esponsi bl e  f or   gener ati ng  the  al k al osi s.
Mai ntenance  of   the  m etabol i c  al k al osi s  i s  per petuated  by   sev er al   f actor s.
The  N aCl   l ost  w i th  v om i ti ng  l eads  to  a  state  of   i ntr av ascul ar   v ol um e
depl eti on,   w hi ch,   i n  tur n,   sti m ul ates  pr ox i m al   tubul e  r esor pti on  of   both
N aCl   and  N aHCO 3 .   It  al so  sti m ul ates  the  r eni n­angi otensi n­al doster one

sy stem .   The  r esul tant  i ncr eased  al doster one  secr eti on  sti m ul ates  N a + /H +
and  N a + /K +   ex change  i n  the  di stal   tubul e.   The  f or m er   i ncr eases
bi car bonate  r esor pti on,   w her eas  the  l atter   l eads  to  potassi um   i on
depl eti on,   w hi ch  al so  accel er ates  pr ox i m al   bi car bonate  r esor pti on.   The
i ncr eased  PCO 2   associ ated  w i th  the  com pensati on  f or   m etabol i c  al k al osi s
al so  i ncr eases  bi car bonate  r esor pti on.   These  ev ents  ar e  depi cted  i n  Fi g.
9­1.

4.   If   the  pati ent's  v om i ti ng  w er e  to  stop  spontaneousl y ,   w oul d  the  aci dâ
€“base  di stur bance  al so  r esol v e?

Cessati on  of   v om i ti ng  w oul d  not  necessar i l y   r estor e  the  aci d–base
bal ance.   The  pati ent's  v om i ti ng  i s  onl y   the  pr eci pi tati ng  cause  of   hi s
m etabol i c  al k al osi s.   At  thi s  poi nt,   i f   hi s  v om i ti ng  w er e  to  stop,   sev er al
f actor s  w oul d  sti l l   pr ev ai l   (as  di scussed  ear l i er )  and  m ai ntai n  the
m etabol i c  al k al osi s.   Onl y   w hen  both  the  gener ati ng  and  m ai ntai ni ng
f actor s  ar e  el i m i nated  can  the  aci d–base  di stur bance  r esol v e.

5.   How   w oul d  y ou  tr eat  thi s  pati ent?

In  al l   cases,   the  tr eatm ent  of   m etabol i c  al k al osi s  i nv ol v es  m anagem ent  of
the  under l y i ng  pr ocess.   How ev er ,   the  pr ocess  that  has  been  the  sour ce  of
the  m etabol i c  al k al osi s  m ay   hav e  r esol v ed,   and  other   f actor s  m ay   be
m ai ntai ni ng  the  m etabol i c  al k al osi s.   Ther ef or e,   tr eati ng  those  f actor s  that
ar e  m ai ntai ni ng  the  m etabol i c  al k al osi s  m ay   be  m ost  i m por tant.   Thi s
pati ent  shoul d  r ecei v e  dual   ther apy .   Fi r st,   the  v om i ti ng  (w hi ch  i s  the
sour ce  of   the  m etabol i c  al k al osi s)  shoul d  be  tr eated  usi ng  an  anti em eti c
agent.   Second,   the  i ntr av ascul ar   v ol um e  and  potassi um   depl eti on  m ust  be
cor r ected.   Thi s  i s  accom pl i shed  by   the  adm i ni str ati on  of   nor m al   sal i ne
P. 398
pl us  suppl em ental   potassi um .   The  nor m al   sal i ne  i s  adm i ni ster ed  unti l   the
or thostati c  changes  i n  the  pul se  and  bl ood  pr essur e  r esol v e.

Suggested Readings
Gennar i   FJ.   Metabol i c  al k al osi s.   In:  Johnson  R,   Feehal l y   J,   eds.
Com pr ehensi v e  cl i ni cal   nephr ol ogy ,   2nd  ed.   Mosby ,   2003.

Sel di n  D,   Rector   F.   The  gener ati on  and  m ai ntenance  of   m etabol i c  al k al osi s.
Ki dney   Int  1972;1:306.

Shapi r o  JI,   Kaehny   WD.   Pathogenesi s  and  m anagem ent  of   m etabol i c


aci dosi s  and  al k al osi s.   In:  Schr i er   RW,   ed.   Renal   and  el ectr ol y te  di sor der s,
6th  ed.   Phi l adel phi a:  Li ppi ncott  Wi l l i am s  &  Wi l k i ns,   2003:115.

Secondary Hypertension
1.   What  ar e  the  m ajor   causes  of   hy per tensi on,   and  w hat  i s  the  natur e  of   the
pathophy si ol ogi c  m echani sm ,   or   m echani sm s,   r esponsi bl e  f or   causi ng  the
el ev ati on  i n  bl ood  pr essur e?
2.   What  shoul d  the  i ni ti al   ev al uati on  of   a  pati ent  w ho  pr esents  w i th  an
el ev ati on  i n  bl ood  pr essur e  consi st  of ,   and,   based  on  the  ev al uati on
f i ndi ngs,   w hat  speci f i c  cl i ni cal   f eatur es  w oul d  poi nt  tow ar d  a  par ti cul ar
secondar y   cause  of   hy per tensi on?

3.   If   a  secondar y   cause  of   hy per tensi on  i s  suspected,   w hat  w oul d  the  f ur ther
di agnosti c  ev al uati on  com pr i se,   and  w hat  w oul d  be  the  l i k el y   f i ndi ngs  f or
each  cause?

4.   What  ar e  the  r especti v e  tr eatm ent  opti ons  f or   r enal   ar ter y   stenosi s,
pheochr om ocy tom a,   Cushi ng's  sy ndr om e,   and  pr i m ar y
hy per al doster oni sm ?

Discussion
1.   What  ar e  the  m ajor   causes  of   hy per tensi on,   and  w hat  i s  the  natur e  of   the
pathophy si ol ogi c  m echani sm ,   or   m echani sm s,   r esponsi bl e  f or   causi ng  the
el ev ati on  i n  bl ood  pr essur e?

Es s e ntia l hype rte ns ion  i s  the  m ost  com m on  cause  of   hy per tensi on  and
accounts  f or   appr ox i m atel y   90%  of   al l   cases.   It  i s  usual l y   asy m ptom ati c.
The  usual   age  of   onset  i s  betw een  30  and  50  y ear s  and  pati ents  usual l y
hav e  a  geneti c  pr edi sposi ti on  f or   acqui r i ng  i t.   Other   f or m s  of
hy per tensi on  m ust  be  r ul ed  out  by   an  i ni ti al   scr eeni ng  ev al uati on  bef or e
thi s  di agnosi s  i s  conf i dentl y   assi gned.   The  r egul ati on  of   ar ter i al   pr essur e
i nv ol v es  a  com pl ex ,   and  as  y et  not  f ul l y   under stood,   i nter acti on  am ong
neur ohum or al   m echani sm s,   sodi um   ex cr eti on,   and  bar or eceptor   r ef l ex es.
Ther e  i s  ev i dence  to  suggest  that  the  m echani sm   r esponsi bl e  f or   the
el ev ati on  of   bl ood  pr essur e  i n  essenti al   hy per tensi on  m ay   i nv ol v e
i nher i ted  abnor m al i ti es  i n  sodi um   ex cr eti on.   Thi s  l i m i tati on  i n  the
P. 399
abi l i ty   to  ex cr ete  sodi um   m ay   am pl i f y   the  m echani sm s  that  cause  a  r i se
i n  ar ter i al   pr essur e,   ther eby   pr oduci ng  an  abnor m al   r esponse.   These
m echani sm s  i ncl ude  (a)  an  i ncr em ent  i n  the  ex tr acel l ul ar   f l ui d  v ol um e
and  car di ac  output,   w i th  secondar y   autor egul ati on  causi ng  an  i ncr em ent
i n  per i pher al   v ascul ar   r esi stance;  (b)  an  i ncr ease  i n  the  v ascul ar
r esponse  to  v asoconstr i cti on  and  (c)  an  i ncr ease  i n  a  putati v e  ci r cul ati ng
N a + /K + ­adenosi ne  tr i phosphatase  i nhi bi tor ,   w hi ch  el ev ates  the
i ntr acel l ul ar   sodi um   and  cal ci um   l ev el s,   ther eby   al so  augm enti ng
per i pher al   v ascul ar   r esi stance.

Table 9­12 Identifiable Causes of
Hypertension

Metabol i c  sy ndr om e  (obesi ty ,   i nsul i n  r esi stance,   i m pai r ed


gl ucose  tol er ance,   dy sl i pi dem i a,   hy per tensi on)
Obstr ucti v e  sl eep  apnea
Dr ug­i nduced  hy per tensi on
Decongestants
Adr ener gi c  agents
Cal ci neur i n  i nhi bi tor s
N SAIDs
Chr oni c  k i dney   di sease
Pr i m ar y   hy per al doster oni sm
Renov ascul ar   di sease
Chr oni c  ster oi d  use  or   Cushi ng's
Pheochr om ocy tom a
Coar ctati on  of   the  aor ta
Thy r oi d  or   par athy r oi d  di sease
N SAIDs,   nonster oi dal   anti i nf l am m ator y   dr ugs.

Modi f i ed  f r om   N ol an  CR.   The  pati ent  w i th  hy per tensi on.   In:
Schr i er   RW,   ed.   Manual   of   nephr ol ogy ,   6th  ed.   Phi l adel phi a:
Li ppi ncott  Wi l l i am s  &  Wi l k i ns,   2005.   Repr i nted  w i th  per m i ssi on.

The  m ajor   secondar y   causes  of   hy per tensi on  ar e  l i sted  i n  Tabl e  9­12.

The  ex act  pr ev al ence  of   re na l a rte ry s te nos is   i s  not  k now n,   but  i t
pr obabl y   accounts  f or   appr ox i m atel y   5%  of   the  gener al   hy per tensi v e
popul ati on.   It  i s  an  i m por tant  di agnosi s  to  m ak e  because  i t  i s  the  m ost
com m on  tr eatabl e  f or m   of   secondar y   hy per tensi on  at  any   age,   and  i t  i s
one  of   the  f ew   potenti al l y   r ev er si bl e  causes  of   chr oni c  r enal   f ai l ur e.   The
di agnosi s  m ust  be  consi der ed  i n  any   pati ent  w i th  sev er e  hy per tensi on
r ef r actor y   to  ther apy   or   i n  any   pati ent  w ho  ex per i ences  the  onset  of
hy per tensi on  ei ther   w hen  v er y   y oung  or   v er y   ol d.   Ather oscl er oti c  pl aques
on  the  r enal   ar ter i es  ar e  the  cause  i n  m ost  cases,   par ti cul ar l y   i n  pati ents
ol der   than  50  y ear s.   Fi br om uscul ar   dy spl asi a,   an  enti ty   seen  i n  y ounger
pati ents,   par ti cul ar l y   w om en,   i s  the  second  m ost  com m on  cause  of
r enov ascul ar   hy per tensi on.   Ther e  i s  ev i dence  to  suggest  that  both  r eni n­
and  v ol um e­dependent  m echani sm s  pl ay   a  r ol e  i n  the  pathophy si ol ogy   of
r enov ascul ar   hy per tensi on  i n  hum ans.   The  f ol l ow i ng  ev i dence  suppor ts
the  i nter pl ay   of   both  m echani sm s:  (a)  the  pl asm a  r eni n  acti v i ty   i s  usual l y
nor m al   or   hi gh  i n  pati ents  w i th  r enal   ar ter y   stenosi s,   but  nev er   l ow ;  (b)
ther e  i s
P. 400
uni l ater al   hy per secr eti on  of   r eni n  f r om   the  af f ected  k i dney   w i th
contr al ater al   suppr essi on;  (c)  i n  pati ents  w i th  uni l ater al   r enal   ar ter y
stenosi s,   r em ov al   of   the  constr i cti on  or   tr eatm ent  w i th  an  i nhi bi tor   of   the
r eni n–angi otensi n  sy stem   usual l y   r estor es  the  bl ood  pr essur e  to  nor m al
or   near ­nor m al   v al ues;  and  (d)  the  ef f ect  of   angi otensi n  bl ock ade  and
sal t  r estr i cti on  on  bl ood  pr essur e  i n  pati ents  w i th  bi l ater al   r enal   ar ter y
stenosi s  i s  f r equentl y   addi ti v e.

P rima ry hype ra ldos te ronis m  i s  an  uncom m on  cause  of   secondar y


hy per tensi on,   w i th  a  pr ev al ence  of   appr ox i m atel y   1%  i n  the  hy per tensi v e
popul ati on.   Thi s  di sease  can  occur   at  any   age.   The  cl assi c  f or m   (Conn's
sy ndr om e)  r esul ts  f r om   a  uni l ater al   adr enocor ti cal   adenom a,   and
accounts  f or   appr ox i m atel y   hal f   the  cases  of   hy per al doster oni sm .   The
other   hal f   of   the  pati ents  hav e  bi l ater al   adr enal   hy per pl asi a.   A  sm al l
per centage  has  ov er pr oducti on  that  can  be  suppr essed  w i th
gl ucocor ti coi ds.   As  i n  other   f or m s  of   hy per tensi on,   the  ex act  pathogenesi s
i s  uncl ear .   The  f i ndi ngs  f r om   ear l y   studi es  suggested  that  the  ex pected
sal t  and  w ater   r etenti on  secondar y   to  the  al doster one  ex cess  r ai ses  the
i ntr av ascul ar   v ol um e  and  subsequentl y   car di ac  output,   ther eby   r ai si ng  the
bl ood  pr essur e.   How ev er ,   hy per v ol em i a  i s  not  a  uni v er sal   f i ndi ng  i n
pati ents  w i th  pr i m ar y   hy per al doster oni sm .   The  r esul ts  of   studi es  i n
ani m al s  hav e  suggested  that  the  m or e  i m por tant  m echani sm   i s  an
i ncr ease  i n  sodi um   stor es  and  total   per i pher al   v ascul ar   r esi stance.   The
m echani sm   r esponsi bl e  f or   thi s  i s  uncer tai n,   but  som e  study   f i ndi ngs
suggest  that  ex cess  m i ner al ocor ti coi ds  i nduce  m em br ane  changes  i n
v ascul ar   sm ooth  m uscl e,   l eadi ng  to  abnor m al   cati on  tur nov er   (possi bl y
sodi um   and  cal ci um ),   w hi ch,   i n  tur n,   augm ents  v asoconstr i cti on  and
i ncr eases  per i pher al   v ascul ar   r esi stance.

P he oc hromoc ytoma   i s  al so  a  r ar e  cause  of   hy per tensi on.   It  i s  esti m ated
to  af f ect  0. 1%  of   pati ents  w i th  hy per tensi on.   Pheochr om ocy tom a  can
occur   at  any   age,   but  i t  ar i ses  m ost  f r equentl y   i n  the  f our th  and  f i f th
decades.   In  adul ts,   m ost  pheochr om ocy tom as  af f ect  w om en.
Pheochr om ocy tom as  ar e  tum or s  of   neur oectoder m al   or i gi n.   If   they   go
undi agnosed,   they   car r y   a  hi gh  r i sk   of   causi ng  m or bi di ty   and  m or tal i ty
secondar y   to  hy per tensi v e  cr i si s,   shock ,   ar r hy thm i as,   car di ac  ar r est,   and
str ok e.   The  hy per tensi on  of   pheochr om ocy tom a  i s  a  f uncti on  of   the
nor epi nephr i ne  r el eased  i nto  the  sy napti c  cl ef t.   Ci r cul ati ng  l ev el s  of
nor epi nephr i ne  hav e  l i ttl e  di r ect  i nv ol v em ent  i n  the  cause  or   m ai ntenance
of   the  hy per tensi on.

Hy per tensi on  com pl i cates  both  a c ute  a nd c hronic  re na l pa re nc hyma l
dis e a s e s ,   and  af f ects  appr ox i m atel y   80%  to  90%  of   pati ents  on  di al y si s.
Ther e  ar e  sev er al   m echani sm s  that  m ay   be  i nv ol v ed  i n  pr oduci ng  the
hy per tensi on  i n  thi s  setti ng,   and  these  i ncl ude  (a)  a  m ar k edl y   i m pai r ed
abi l i ty   of   the  di seased  k i dney   to  ex cr ete  sal t  and  w ater ;  (b)  the
pr oducti on  of   an  uni denti f i ed  v asopr essor   substance  by   the  k i dney ;  (c)
absent  pr oducti on  of   a  necessar y   hum or al   v asodi l ator   substance  by   the
k i dney ;  (d)  f ai l ur e  of   the  k i dney s  to  i nacti v ate  ci r cul ati ng  v asopr essor
substances;  and  (e)  acti v ati on  of   the  r eni n–angi otensi n  sy stem .

The  bl ood  pr essur e  i n  the  upper   ex tr em i ti es  i s  el ev ated  i n  80%  of


chi l dr en  and  adul ts  w i th  c oa rc ta tion of the  a orta .   The  m echani sm
r esponsi bl e  f or   thi s
P. 401
hy per tensi on  i s  an  i nappr opr i ate  acti v ati on  of   the  r eni n–angi otensi n
sy stem   i n  the  pr esence  of   an  ex panded  body   f l ui d  v ol um e.

Hy per tensi on  af f ects  80%  of   pati ents  w i th  i di opathi c  Cus hing's
s yndrome .   Other   cl i ni cal   f eatur es  of   the  di sor der   i ncl ude  gl ucose
i ntol er ance,   m enstr ual   di sor der s,   ster i l i ty ,   l oss  of   l i bi do,   acne,   str i ae,
osteopor osi s,   m uscl e  w eak ness  and  w asti ng,   edem a,   pol y ur i a,   and  r enal
stones.   How ev er ,   the  m echani sm   w her eby   adr enocor ti cotr opi c  hor m one
and  cor ti sol   r ai se  bl ood  pr essur e  i n  hum ans  has  not  been  el uci dated,
al though  ther e  i s  ev i dence  to  suggest  that  gl ucocor ti coi ds  possess  a  â
€œhy per tensi nogeni câ€​   acti on  that  i s  separ ate  f r om   thei r   gl ucocor ti coi d
acti v i ty .

In  the  setti ng  of   re nin­produc ing tumors ,   hy per tensi on  r esul ts  f r om   the
ex cess  secr eti on  of   r eni n  by   ei ther   a  jux tagl om er ul ar   cel l   tum or   or
nephr obl astom a.   Thi s  causes  the  per i pher al   r eni n  l ev el s  to  be  el ev ated,
w hi ch  m edi ates  the  hy per tensi on.

2.   What  shoul d  the  i ni ti al   ev al uati on  of   a  pati ent  w ho  pr esents  w i th  an
el ev ati on  i n  bl ood  pr essur e  consi st  of ,   and,   based  on  the  ev al uati on
f i ndi ngs,   w hat  speci f i c  cl i ni cal   f eatur es  w oul d  poi nt  tow ar d  a  par ti cul ar
secondar y   cause  of   hy per tensi on?

The  i ni ti al   ev al uati on  of   pati ents  w i th  hy per tensi on  shoul d  i ncl ude  hi stor y
tak i ng,   phy si cal   ex am i nati on,   and  l abor ator y   tests  di r ected  tow ar d
uncov er i ng  a  cor r ectabl e  f or m   of   secondar y   hy per tensi on.

In  ter m s  of   the  his tory,   a  str ong  f am i l y   hi stor y ,   as  w el l   as  past
obser v ati ons  of   i nter m i ttent  bl ood  pr essur e  el ev ati ons,   suggest  essenti al
hy per tensi on.   Secondar y   hy per tensi on  of ten  dev el ops  ei ther   bef or e  30  or
af ter   55  y ear s  of   age.   Other   per ti nent  gener al   questi ons  shoul d  el i ci t
i nf or m ati on  about  ster oi d  use,   use  of   dr ugs,   i ncl udi ng  or al   contr acepti v es,
and  w hether   ther e  hav e  been  r ecur r ent  ur i nar y   tr act  i nf ecti ons  or   a
hi stor y   of   pr otei nur i a,   noctur i a,   tr aum a,   or   w ei ght  gai n  or   l oss.

P hys ic a l e x a mina tion  shoul d  di v ul ge  f ur ther   di agnosti c  cl ues  as  to  the
possi bl e  cause  of   the  hy per tensi on.   The  ex am i nati on  shoul d  f ocus  on  the
pati ent's  gener al   appear ance,   m uscul ar   dev el opm ent,   bl ood  pr essur e  and
pul ses  i n  both  upper   ex tr em i ti es  and  a  l ow er   ex tr em i ty ,   the  supi ne  and
standi ng  bl ood  pr essur e,   f unduscopy ,   pal pati on  and  auscul tati on  of   the
car oti d  ar ter i es,   car di ac  and  pul m onar y   ex am i nati on,   auscul tati on  of   the
abdom en  f or   br ui ts  and  pal pati on  f or   an  abdom i nal   aneur y sm   and
enl ar ged  k i dney s,   and  ex am i nati on  of   the  l ow er   ex tr em i ti es  f or   edem a.

La bora tory e va lua tion  at  the  i ni ti al   w or k up  shoul d  i ncl ude  ur i nal y si s  f or
the  pr esence  of   pr otei n,   bl ood,   and  gl ucose,   together   w i th  a  m i cr oscopi c
ex am i nati on;  the  ser um   cr eati ni ne  and  BU N   l ev el s;  hem atocr i t;  the  ser um
potassi um   l ev el ;  the  w hi te  bl ood  cel l   count;  the  ser um   gl ucose,
chol ester ol ,   tr i gl y cer i de,   cal ci um ,   phosphate,   and  ur i c  aci d  l ev el s;
el ectr ocar di ogr aphy ;  and  a  chest  r adi ogr aphi c  study .

The  cl i ni cal   f eatur es  that  suggest  r enal   v ascul ar   hy per tensi on  ar e  l i sted
i n  Tabl e  9­13.   The  cl i ni cal   f eatur es  suggesti ng  other   secondar y   causes  of
hy per tensi on  ar e  l i sted  i n  Tabl e  9­14.

P. 402
Table 9­13 Clinical Features Suggestive of
Renal Vascular Hypertension

Epi dem i ol ogi c  f eatur es


Hy per tensi on  i n  the  absence  of   f am i l y   hi stor y
Age  < 25  y   or   > 55  y
Ci gar ette  sm ok i ng
Whi te  r ace
Featur es  of   the  hy per tensi on
Abr upt  onset  of   m oder ate  to  sev er e  hy per tensi on
Sudden  onset  of   hy per tensi on  af ter   abdom i nal   tr aum a
Recent  accel er ati on  of   sev er i ty   of   hy per tensi on
Headaches
Resi stance  or   f ai l ur e  of   bl ood  pr essur e  contr ol   w i th
usual   ther apy
Dev el opm ent  of   sev er e  or   m al i gnant  hy per tensi on
Reti nopathy   out  of   pr opor ti on  to  sev er i ty   of   bl ood
pr essur e
Ex cel l ent  anti hy per tensi v e  r esponse  to  angi otensi n­
conv er ti ng  enzy m e  i nhi bi tor
Deter i or ati on  i n  r enal   f uncti on  i n  r esponse  to
angi otensi n­conv er ti ng  enzy m e  i nhi bi tor
Bl ood  pr essur e  unaf f ected  or   i ncr eased  w i th  di ur eti c
ther apy
Associ ated  f eatur es
U npr ov ok ed  hy pok al em i a
Hy pok al em i a  i n  r esponse  to  a  thi azi de  di ur eti c
Abdom i nal   or   f l ank   sy stol i c­di astol i c  br ui ts
Car oti d  br ui ts  or   other   ev i dence  of   l ar ge­v essel
di sease
El ev ated  per i pher al   pl asm a  r eni n  acti v i ty   i n  absence  of
al ter nati v e  ex pl anati on

Modi f i ed  f r om   Pl oth  DW.   Renov ascul ar   hy per tensi on.   In:
Jacobson  HR,   Str i k er   GE,   Kl ahr   S,   eds.   The  pr i nci pl es  and
pr acti ce  of   nephr ol ogy .   Phi l adel phi a:  BC  Deck er ,   1991:379.
Repr i nted  w i th  per m i ssi on.

3.   If   a  secondar y   cause  of   hy per tensi on  i s  suspected,   w hat  w oul d  the  f ur ther
di agnosti c  ev al uati on  com pr i se,   and  w hat  w oul d  be  the  l i k el y   f i ndi ngs  f or
each  cause?

A  num ber   of   tests  hav e  ev ol v ed  to  assess  the  l i k el i hood  of   re na l


va s c ula r hype rte ns ion.   Magneti c  r esonance  angi ogr aphy   (MRA)  or
Doppl er   ul tr asonogr aphy   of   the  r enal   ar ter i es  hav e  been  used  f or   the
ev al uati on  of   r enal   ar ter y   stenosi s.   How ev er ,   these  tests  hav e  v ar i abl e
degr ees  of   sensi ti v i ty   and  speci f i ci ty ,   l ar gel y   due  to  v ar y i ng  degr ees  of
ex per ti se  w i th  these  techni ques  at  di f f er ent  center s.   Ther ef or e,
conv enti onal   r enal   ar ter i ogr aphy   r em ai ns  the  gol d  standar d.   It  m ust  be
r ecogni zed,   how ev er ,   that  the  f i ndi ng  of   r enal   ar ter y   stenosi s  pr ov i des  no
i nf or m ati on  concer ni ng  the  pathophy si ol ogy   of   the  v ascul ar   l esi on.   A
postcaptopr i l   (25  m g)  el ev ati on  i n  pl asm a  r eni n  acti v i ty   or   a  decr ease  i n
r enal   per f usi on  postcaptopr i l   as  assessed  by   sci nti l l ati on  techni ques  or
r enal   v ei n  r eni ns  can  pr ov i de  pathophy si ol ogi c  i nf or m ati on.

If   ther e  ar e  cl i ni cal   f eatur es  hi ghl y   suggesti v e  of   a  phe oc hromoc ytoma ,


the  ev al uati on  shoul d  begi n  w i th  an  assay   of   the  total   pl asm a
catechol am i ne  l ev el ,   as  m easur ed  thr ough  an  i ndw el l i ng  21­gauge
butter f l y   needl e  i n  a  pati ent  w ho  has  been  r esti ng  supi ne  f or   30  m i nutes.
Val ues  m or e  than  2, 000  pg/m L  w ar r ant  per f or m ance  of   abdom i nal
com puted  tom ogr aphy   (CT).
P. 403
P. 404
Val ues  betw een  1, 000  and  2, 000  pg/m L  r equi r e  per f or m ance  of   the
cl oni di ne  suppr essi on  test  to  deter m i ne  w hether   a  pheochr om ocy tom a  i s
pr esent.   Cl oni di ne  does  not  suppr ess  the  r el ease  of   catechol am i nes  i n
pati ents  w i th  a  pheochr om ocy tom a,   as  i t  does  i n  pati ents  w i th  essenti al
hy per tensi on.   If   the  pl asm a  catechol am i ne  v al ues  ar e  bel ow   1, 000  pg/m L,
and  the  pati ent  i s  hy per tensi v e,   the  cl oni di ne  suppr essi on  test  shoul d  be
per f or m ed,   but,   i f   the  pati ent  i s  nor m otensi v e,   the  gl ucagon  sti m ul ati on
test  m ay   be  hel pf ul .   For   the  gl ucagon  test  to  be  posi ti v e,   the  pl asm a
catechol am i ne  l ev el   m ust  i ncr ease  by   thr eef ol d,   or   to  gr eater   than  2, 000
pg/m L,   1  to  3  m i nutes  af ter   adm i ni str ati on  of   the  dr ug.   If   any   of   these
test  r esul ts  ar e  posi ti v e,   abdom i nal   CT  shoul d  be  per f or m ed.   In  pati ents
w hose  cl i ni cal   pr esentati on  suggests  a  pheochr om ocy tom a  but  w ho  hav e
onl y   a  sl i ght  or   m oder ate  r i se  i n  the  catechol am i ne  l ev el   (< 1, 000  pg/m L),
r epeat  testi ng,   i ncl udi ng  m easur em ent  of   the  ur i nar y   catechol am i ne
l ev el s,   shoul d  be  per f or m ed.

Table 9­14 Clinical Features of Other
Secondary Causes of Hypertension

Pr i m ar y   hy per al doster oni sm


Hi stor y
Pr ox i m al   m uscl e  w eak ness,   pol y ur i a,   noctur i a,
pol y di psi a,   par esthesi a,   tetany ,   m uscl e  par al y si s,
f r ontal   headaches
Labor ator y   f eatur es
The  di agnosti c  hal l m ar k   of   thi s  di sease  i s
hy pok al em i c  m etabol i c  al k al osi s
Hy per gl y cem i a  m ay   al so  be  pr esent
Pheochr om ocy tom a
Sy m ptom s
Pati ents  m ay   pr esent  i n  a  w i de  v ar i ety   of   cl i ni cal
setti ngs,   i ncl udi ng  tr ansi ent  i schem i c  attack s,
str ok e,   headache  (usual l y   poundi ng  and  sev er e),
pal pi tati ons  w i th  or   w i thout  tachy car di a,   and
ex cessi v e  sw eati ng;  l ess  com m on  sy m ptom s
i ncl ude  tr em or ,   pal l or ,   nausea,   w eak ness,
f ati gue,   w ei ght  l oss,   and  chest  or   abdom i nal   pai n
Phy si cal   ex am i nati on
Postur al   hy potensi on  occur s  i n  50%­75%  of
pati ents;  par ox y sm al   epi sodes  of   hy per tensi on
occur   i n  appr ox i m atel y   one  thi r d  of   pati ents;
sw eati ng  and  m uscul ar   w eak ness  m ay   be  ev i dent
Labor ator y   f eatur es
Hy per gl y cem i a  or   hy per cal cem i a  m ay   be  pr esent
Coar ctati on  of   the  aor ta
Sy m ptom s
Epi stax i s,   thr obbi ng  headache,   l eg  f ati gue,   col d
ex tr em i ti es,   and  occasi onal   cl audi cati on
Phy si cal   ex am i nati on
Di spar i ty   i n  the  pul sati ons  and  bl ood  pr essur e
betw een  the  ar m s  and  l egs—the  pul sati ons  i n
the  upper   ex tr em i ti es  ar e  poundi ng;  those  i n  the
l ow er   ex tr em i ti es  ar e  w eak ,   del ay ed,   or   absent;
the  bl ood  pr essur e  i n  the  ar m s  ex ceeds  that  i n
the  l egs;  ther e  i s  col l ater al   ar ter i al   ci r cul ati on;
m ur m ur s  ar e  usual l y   pr esent  but  v ar y   i n  l ocati on
Labor ator y   f eatur es
Chest  r adi ogr aph  m ay   show   pr om i nence  of   the
l ef t  v entr i cl e,   notchi ng  of   the  i nf er i or   bor der   of
the  r i bs  f r om   col l ater al   v essel s,   and  poststenoti c
di l atati on  of   the  aor ta
Cushi ng's  sy ndr om e
Sy m ptom s
Menstr ual   di sor der s,   l oss  of   l i bi do,   hi r suti sm ,
acne,   str i ae,   m uscl e  w eak ness,   easy   br ui si ng,
edem a,   pol y ur i a
Phy si cal   ex am i nati on
Hi r suti sm ,   acne,   str i ae,   m uscl e  w eak ness  and
w asti ng,   pur pur a,   br ui si ng,   edem a,   and  poor
w ound  heal i ng
Labor ator y   f eatur es
Hy per gl y cem i a,   i m pai r ed  gl ucose  tol er ance,
neutr ophi l i a,   l y m phopeni a,   and  hy pok al em i a
Renal   par enchy m al   di sease
Sy m ptom s
U r em i a  and  anem i a;  associ ated  w i th  r enal   f ai l ur e
Phy si cal   ex am i nati on
If   any   f i ndi ngs,   those  associ ated  w i th  r enal
f ai l ur e
Labor ator y   f eatur es
Sev er al   l abor ator y   abnor m al i ti es  m ay   be
pr esent—these  i ncl ude  el ev ati on  of   the  BU N   and
cr eati ni ne  l ev el s,   anem i a,   hy pocal cem i a,
hy per phosphatem i a,   hy per k al em i a,   m etabol i c
aci dosi s,   pr otei nur i a,   and  hem atur i a

BU N ,   bl ood  ur ea  ni tr ogen.

Echocar di ogr aphy   can  v i sual i ze  the  ar ea  of   a ortic  c oa rc ta tion,   but  thi s  i s
best  conf i r m ed  by   car di ac  catheter i zati on.

Hi stor i cal l y ,   Cus hing's  s yndrome   has  been  di agnosed  on  the  basi s  of   the
f ol l ow i ng  f i ndi ngs:  el ev ated  l ev el s  of   ur i nar y   17­hy dr ox y cor ti coster oi ds
and  ur i nar y ­f r ee  cor ti sol ,   l oss  of   di ur nal   r hy thm   i n  the  pl asm a  cor ti sol
concentr ati ons,   and  f ai l ur e  of   pl asm a  cor ti sol   l ev el s  to  suppr ess  ov er ni ght
af ter   a  si ngl e  1­m g  dose  of   dex am ethasone.   Because  the  ov er ni ght
dex am ethasone  suppr essi on  test  m ay   not  el i ci t  suppr essi on  i n  obese  and
acr om egal i c  pati ents,   the  l ow ­dose  dex am ethasone  suppr essi on  test  (0. 5
m g  ev er y   6  hour s  f or   2  day s)  shoul d  be  done  to  di sti ngui sh  pati ents  w i th
Cushi ng's  sy ndr om e  f r om   heal thy   subjects.   The  hi gh­dose  dex am ethasone
suppr essi on  test  (2  m g  ev er y   6  hour s  f or   2  day s)  can  di sti ngui sh
Cushi ng's  di sease  f r om   an  adr enal   tum or ,   w hi ch  does  not  suppr ess.

If   the  cause  of   re na l pa re nc hyma l dis e a s e   cannot  be  i denti f i ed  w i th


cer tai nty   on  the  basi s  of   the  hi stor y ,   phy si cal   ex am i nati on,   and
l abor ator y   f i ndi ngs,   r enal   bi opsy   m ay   be  i ndi cated.   The  bi opsy   r esul ts
m ay   shed  l i ght  on  w hether   the  pr ocess  i s  r ev er si bl e,   and  ther eby   poi nt
tow ar d  tr eatm ent  opti ons,   i f   any .

P. 405
In  the  setti ng  of   re nin­produc ing tumors ,   deter m i nati on  of   the  pl asm a
r eni n  acti v i ty   by   r enal   v ei n  sam pl i ng  usual l y   show s  a  uni l ater al   i ncr ease
i n  the  absence  of   a  r enal   ar ter y   l esi on.

4.   What  ar e  the  r especti v e  tr eatm ent  opti ons  f or   r enal   ar ter y   stenosi s,
pheochr om ocy tom a,   Cushi ng's  sy ndr om e,   and  pr i m ar y
hy per al doster oni sm ?

The  tr eatm ent  opti ons  f or   re na l a rte ry s te nos is   ar e  ei ther   sur gi cal   or
m edi cal ,   and  the  choi ce  depends  on  the  pati ent  i nv ol v ed.   The  sur gi cal
opti ons  i ncl ude  r ev ascul ar i zati on  of   the  af f ected  k i dney   usi ng  saphenous
v ei n,   autogenous  ar ter y ,   or   sy ntheti c  (Dacr on  or   pol y tetr af l uor oethy l ene)
gr af ts.   A  r enal   ar ter y   endar ter ectom y   m ay   be  per f or m ed  i n  pati ents  w i th
osti al   ather om atous  l esi ons.   The  m ost  popul ar   m ethod  of   tr eatm ent,   at
l east  i ni ti al l y ,   i s  per cutaneous  tr ansl um i nal   bal l oon  angi opl asty   w i th
pl acem ent  of   stents.   If   these  pr ocedur es  ar e  ei ther   unsuccessf ul   or
cannot  be  under tak en,   m edi cal   m anagem ent  m ust  be  i nsti tuted.

Cur e  of   a  phe oc hromoc ytoma   consi sts  of   sur gi cal   r em ov al   of   the  tum or ,
and  pr oper   pr eoper ati v e  pr epar ati on  hel ps  r educe  the  attendant  m or bi di ty
and  m or tal i ty .   In  the  pr esence  of   hy per tensi on,   adm i ni str ati on  of   an
adr ener gi c­bl ock i ng  agent  such  as  phenox y benzam i ne  (10  to  20  m g  tw i ce
per   day ,   i ncr easi ng  to  100  m g  per   day   i f   tol er ated)  i s  r ecom m ended.
Pr azosi n  i s  not  as  ef f ecti v e.   How ev er ,   i f   the  l ocati on  of   the  tum or   i s  i n
doubt  or   i f   m ul ti pl e  tum or s  ar e  suspected,   i t  i s  best  not  to  adm i ni ster   α ­
adr ener gi c  bl ock i ng  agents  bef or e  sur ger y .   The  i ntr av ascul ar   v ol um e
shoul d  be  ex panded  both  bef or e  and  af ter   sur ger y .   In  pati ents  w i th
i noper abl e  m al i gnant  pheochr om ocy tom as,   dr ug  ther apy   i s  needed.   α ­
and  β­Bl ock er s  m ay   be  used  to  contr ol   ar r hy thm i as,   or   m ethy l ty r osi ne
m ay   be  pr escr i bed  to  i nhi bi t  catechol am i ne  sy nthesi s.

The  best  sur gi cal   appr oach  i n  a  pati ent  w i th  Cushi ng's  di sease  i s  sel ected
ex ci si on  of   the  pituita ry a de noma   thr ough  a  tr anssphenoi dal   appr oach.
Sur gi cal   r em ov al   i s  som eti m es  f ol l ow ed  by   pi tui tar y   i r r adi ati on  to  pr ev ent
r ecur r ence.   A  v ar i ety   of   dr ugs  hav e  al so  been  used  to  tr eat  pati ents  w i th
Cushi ng's  di sease.   Adr enal   tum or s  ar e  best  tr eated  sur gi cal l y .

Hy per al doster oni sm   can  be  tr eated  by   ei ther   m edi cal   or   sur gi cal   m eans.
Mi l d  al doster one  ex cess  due  to  an  adenom a,   and  al l   cases  of   bi l ater al
hy per pl asi a,   shoul d  be  m anaged  w i th  al doster one  antagoni sts  such  as
spi r onol actone  because  thi s  di sor der   i s  not  am enabl e  to  sur gi cal
tr eatm ent.   Al doster one­pr oduci ng  adenom as  can  be  r em ov ed  to  ef f ect
cur e  once  they   hav e  been  appr opr i atel y   l ocal i zed  by   r adi ol ogi c  (CT)
techni ques.

Case
A  38­y ear ­ol d  adopted  w hi te  m an  i s  seen  by   hi s  f am i l y   phy si ci an  f or   the
m anagem ent  of   hy per tensi on  of   2  y ear s'  dur ati on.   Cur r ent  m edi cati ons  i ncl ude
am i l or i de  (5  m g)  and  hy dr ochl or othi azi de  (50  m g),   w i th  good  bl ood  pr essur e
contr ol   unti l   now .   Rev i ew   of   sy stem s  r ev eal s  i ncr easi ng  f ati gue,   headaches,
and  m uscl e  cr am ps.   Phy si cal   ex am i nati on  r ev eal s  a  bl ood  pr essur e  of   140/100
m m   Hg  i n  the  l ef t  ar m   and  136/100  m m   Hg  i n  the  r i ght  ar m .   N o  di spar i ty   i n
the  bl ood  pr essur e  betw een  the  ar m s  and  the  l egs  i s  f ound.   The  r em ai nder   of
the  ex am i nati on  f i ndi ngs  ar e  other w i se  unr em ar k abl e.
P. 406
The  f ol l ow i ng  l abor ator y   data  ar e  r epor ted:  sodi um ,   145  m Eq/L;  potassi um ,
2. 7  m Eq/L;  chl or i de,   109  m Eq/L;  bi car bonate,   29  m Eq/L;  BU N ,   10  m Eq/L;
cr eati ni ne,   1. 2  m g/dL;  cal ci um ,   9. 1  m g/dL;  chol ester ol ,   213  m g/dL;
tr i gl y cer i des,   163  m g/dL;  ur i c  aci d,   6. 1  m g/dL;  phosphate,   2. 1  m g/dL;  and
gl ucose,   99  m g/dL.   Resul ts  of   ur i nal y si s,   i ncl udi ng  m i cr oscopi c  ex am i nati on,
ar e  nor m al .
The  di ur eti cs  ar e  stopped  and  the  pati ent  i s  pl aced  on  potassi um   suppl em ents.
Repeat  l abor ator y   w or k   r ev eal s  that  hi s  sodi um   l ev el   i s  147  m Eq/L,   potassi um
l ev el   i s  3  m Eq/L,   and  bl ood  pr essur e  i s  146/104  m m   Hg.

1.   What  i s  the  di f f er enti al   di agnosi s  of   thi s  pati ent's  hy per tensi on?
2.   What  sy m ptom s  ar e  r el ated  to  the  pati ent's  hy pok al em i a?
3.   What  di agnosti c  steps  w oul d  hel p  conf i r m   the  di agnosi s  i n  thi s  pati ent?
4.   What  ar e  the  tr eatm ent  opti ons  i n  thi s  pati ent?

Case Discussion
1.   What  i s  the  di f f er enti al   di agnosi s  of   thi s  pati ent's  hy per tensi on?

The  di f f er enti al   di agnosi s  i ncl udes  essenti al   hy per tensi on,   pr i m ar y


hy per al doster oni sm ,   pheochr om ocy tom a,   Cushi ng's  sy ndr om e,   a  r eni n­
pr oduci ng  tum or ,   and  r enal   ar ter y   stenosi s.   Renal   par enchy m al   di sease
and  coar ctati on  of   the  aor ta  can  be  l ar gel y   ex cl uded  as  a  cause  of   thi s
pati ent's  hy per tensi on  because  the  ser um   cr eati ni ne  l ev el   and  ur i nal y si s
f i ndi ngs  ar e  nor m al ,   as  ar e  the  phy si cal   ex am i nati on  f i ndi ngs.   The
str i k i ng  f eatur e  of   thi s  pati ent's  hy per tensi on  i s  the  hy pok al em i a  despi te
tr eatm ent  w i th  a  potassi um ­spar i ng  di ur eti c  pl us  potassi um
suppl em entati on.   Hy pok al em i a  m ay   be  a  f eatur e  of   pr i m ar y
hy per al doster oni sm ,   Cushi ng's  sy ndr om e,   r enal   ar ter y   stenosi s,   and
r eni n­pr oduci ng  tum or s.   Pheochr om ocy tom a  i s  consi der ed  a  possi bi l i ty
because  of   the  pati ent's  com pl ai nts  of   headache  and  f ati gue,   al though  the
cl i ni cal   suspi ci on  f or   thi s  i s  l ow .   Al though  hy pok al em i a  occur s  i n
Cushi ng's  sy ndr om e,   the  other   cl i ni cal   f eatur es  of   the  di sor der   appear   to
be  l ack i ng.   Renal   ar ter y   stenosi s  i s  al so  unl i k el y   unl ess  the  pati ent  has
f i br om uscul ar   dy spl asi a.   Because  the  pati ent's  f am i l y   hi stor y   i s  unk now n,
hi s  geneti c  pr opensi ty   f or   ather oscl er osi s  i s  not  k now n,   but  he  does  not
appear   to  hav e  other   ev i dence  of   ar ter i oscl er oti c  di sease  (e. g. ,   br ui ts,
angi na,   and  cl audi cati on).   Ther ef or e,   the  m ost  l i k el y   causes  i ncl ude
pr i m ar y   al doster oni sm   and  a  r eni n­pr oduci ng  tum or .   Essenti al
hy per tensi on  can  be  di agnosed  onl y   af ter   the  m ost  l i k el y   secondar y
causes  hav e  been  ex cl uded.

2.   What  sy m ptom s  ar e  r el ated  to  the  pati ent's  hy pok al em i a?

Hy pok al em i a  coul d  ex pl ai n  thi s  pati ent's  headaches,   m uscl e  cr am ps,   and


f ati gue.   Addi ti onal   sy m ptom s  m ay   i ncl ude  m uscl e  w eak ness,   pol y ur i a,   and
par esthesi as.

3.   What  di agnosti c  steps  w oul d  hel p  conf i r m   the  di agnosi s  i n  thi s  pati ent?

Pati ents  w i th  a  hi stor y   of   spontaneous  hy pok al em i a,   m ar k ed  sensi ti v i ty   to


potassi um ­w asti ng  di ur eti cs,   and  r ef r actor y   hy per tensi on  shoul d  be
ev al uated  f or   pr i m ar y   hy per al doster oni sm .   The  i ni ti al   scr eeni ng  test  i s  to
deter m i ne  the  status  of   al doster one  ex cr eti on  dur i ng  pr ol onged  sal t
l oadi ng.   To  per f or m   thi s,   10  to  12  g  of   N aCl   i s  added  to  the  pati ent's
dai l y   i ntak e.   Af ter   5  to  7  day s  of   i ncr eased  sal t  i ntak e,   the  ser um
potassi um   concentr ati ons  and  a  24­hour   ur i ne  ex cr eti on  of
P. 407
sodi um ,   potassi um ,   and  al doster one  ar e  m easur ed.   The  ser um   and  ur i ne
potassi um   v al ues  i ndi cate  w hether   ther e  i s  i nappr opr i ate  k al i ur esi s  (a
ser um   potassi um   l ev el   of   < 3  m Eq/L  w i th  a  ur i ne  potassi um   l ev el   > 30
m Eq/24  hour s).   The  24­hour   ur i ne  sodi um   l ev el   v er i f i es  com pl i ance  w i th
the  pr escr i bed  sal t  i ntak e  (≥250  m Eq  per   day ).   If ,   under   these
condi ti ons,   the  pati ent's  r ate  of   al doster one  ex cr eti on  f ai l s  to  show
suppr essi on  bel ow   14  µg  per   24  hour s,   thi s  m ak es  hi m   a  pr i m e
candi date  f or   addi ti onal   studi es.   The  pr esence  of   hy pok al em i a  and
suppr essed  pl asm a  r eni n  acti v i ty   f ur ther   suppor ts  the  di agnosi s  of
pr i m ar y   hy per al doster oni sm .   Thi s  can  be  f ur ther   conf i r m ed  by   hi gh
al doster one/r eni n  r ati o  of   gr eater   than  100.   If   a  r eni n­pr oduci ng  tum or
w er e  the  cause  of   thi s  pati ent's  hy per tensi on,   the  pl asm a  r eni n  acti v i ty
w oul d  be  el ev ated.   If   pr i m ar y   hy per al doster oni sm   i s  suspected,   adr enal
CT  scanni ng  shoul d  be  per f or m ed.   The  f i ndi ng  of   an  adr enal   m ass  w oul d
establ i sh  the  di agnosi s.   Adr enal   sci nti gr aphy   shoul d  be  done  i f   the  CT
f i ndi ngs  ar e  i nconcl usi v e.   If   the  r esul ts  of   sci nti gr aphy   ar e  al so
am bi guous,   then  adr enal   v ei n  sam pl i ng  shoul d  be  per f or m ed  to  m easur e
the  al doster one  l ev el s.   Adr enal   v ei n  sam pl i ng  i s  sti l l   the  m ost  accur ate
test  to  l ocal i ze  al doster one­pr oduci ng  tum or s.

4.   What  ar e  the  tr eatm ent  opti ons  i n  thi s  pati ent?

The  hy per tensi on  associ ated  w i th  pr i m ar y   hy per al doster oni sm   can  be
m anaged  adequatel y   i n  m ost  cases  by   m eans  of   sal t  and  w ater   depl eti on.
The  com bi nati on  of   spi r onol actone  w i th  hy dr ochl or othi azi de  or   f ur osem i de
has  been  used  successf ul l y .   How ev er ,   i f   the  adr enal   adenom a  i s  conf i ned
to  one  gl and  and  ther e  ar e  no  contr ai ndi cati ons,   the  tum or   shoul d  be
r em ov ed.   Onl y   appr ox i m atel y   hal f   of   pati ents  ar e  nor m otensi v e  5  y ear s
af ter   sur ger y ,   but  nor m al   potassi um   hom eostasi s  i s  r estor ed
per m anentl y .   If   pr i m ar y   hy per al doster oni sm   stem s  f r om   bi l ater al
hy per pl asi a  of   the  adr enal   gl and,   thi s  i s  best  m anaged  m edi cal l y   because
sur gi cal   r em ov al   of   too  m uch  of   the  adr enal   gl and  can  r esul t  i n  adr enal
i nsuf f i ci ency .

Suggested Readings
N ol an  CR.   The  pati ent  w i th  hy per tensi on.   In:  Schr i er   RW,   ed.   Manual   of
nephr ol ogy ,   6th  ed.   Phi l adel phi a:  Li ppi ncott  Wi l l i am s  &  Wi l k i ns,   2005:242.

Tex tor   SC.   Renov ascul ar   hy per tensi on.   In:  Johnson  R,   Feehal l y   J,   eds.
Com pr ehensi v e  cl i ni cal   nephr ol ogy ,   2nd  ed.   Mosby ,   2003.

Nephrolithiasis
1.   What  ar e  the  f our   m ajor   ty pes  of   k i dney   stones,   and  w hi ch  ar e
r adi opaque?

2.   What  i s  the  shar ed  pathogenesi s  f or   the  f or m ati on  of   al l   ty pes  of   k i dney
stones?
3.   What  ar e  the  f undam ental   causes  of   ov er satur ati on  of   the  ur i ne?

4.   What  ar e  the  acute  and  chr oni c  sequel ae  of   k i dney   stones?

5.   In  the  setti ng  of   ur i c  aci d  k i dney   stones,   i s  the  ov er satur ati on  of   ur i ne
w i th  ur i c  aci d  condi ti oned  pr i m ar i l y   by   the  ur i ne  pH  or   by   the  am ount  of
ur i c  aci d  ex cr eted?

P. 408
6.   What  ar e  the  thr ee  ty pes  of   k i dney   stones  that  m ay   pr esent  i n  the  f or m
of   staghor n  cal cul i ,   and  w hat  ar e  the  r especti v e  m echani sm s  r esponsi bl e
f or   thei r   f or m ati on?

7.   What  ar e  the  pr i nci pal   causes  of   cal ci um   stones?

8.   What  ar e  the  r outi ne  outpati ent  studi es  that  shoul d  be  per f or m ed  i n
pati ents  w i th  r ecur r ent  stones?

9.   What  i s  the  i ndi cati on  f or   m easur i ng  the  ex cr eti on  of   ur i c  aci d  i n  the
setti ng  of   hy per cal ci ur i a?

10.   What  ar e  the  potenti al   causes  of   hy per cal ci ur i a  and  how   shoul d  i t  be
tr eated?

Discussion
1.   What  ar e  the  f our   m ajor   ty pes  of   k i dney   stones,   and  w hi ch  ar e
r adi opaque?

The  pr i nci pal   ty pes  of   k i dney   stones  ar e  com posed  of   cal ci um   sal ts,   ur i c
aci d,   cy sti ne,   and  str uv i te.   Al l   ex cept  ur i c  aci d  stones  ar e  r adi opaque.
Cal ci um ­contai ni ng  stones  account  f or   80%  of   al l   stones,   15%  ar e
com posed  of   str uv i te,   5%  ar e  m ade  up  of   ur i c  aci d,   and  cy sti ne  stones
ar e  v er y   r ar e.

2.   What  i s  the  shar ed  pathogenesi s  f or   the  f or m ati on  of   al l   ty pes  of   k i dney
stones?

Al l   k i dney   stones  r esul t  f r om   an  ex cessi v e  super satur ati on  of   the  ur i ne.
The  i on  concentr ati on  pr oduct  at  w hi ch  sal ts  i n  sol uti on  ar e  i n  equi l i br i um
w i th  thei r   sol i d  phase  i s  cal l ed  the  equi l i br i um   sol ubi l i ty   pr oduct.   In  the
absence  of   a  sol i d  phase,   sal ts  m ay   ex i st  i n  a  super satur ated  state,
abov e  the  equi l i br i um   sol ubi l i ty   pr oduct.   In  thi s  setti ng,   cr y stal s
com posed  of   other   com pounds  m ay   act  as  heter ogeneous  seed  nucl ei   that
f oster   the  f or m ati on  of   stones.   If   the  i on  pr oduct  i s  suf f i ci entl y   hi gh,   then
new   cr y stal s  f or m .   Because  an  i ncr ease  i n  ur i ne  v ol um e  l eads  to  a
decr ease  i n  the  concentr ati on  of   al l   sol utes  i n  the  ur i ne,   an  i ncr eased
f l ui d  i ntak e  of   2. 5  to  3  L  per   day   i s  par t  of   the  tr eatm ent  f or   al l   k i dney
stones.

3.   What  ar e  the  f undam ental   causes  of   ov er satur ati on  of   the  ur i ne?

Ther e  ar e  thr ee  m ajor   r easons  f or   the  ov er satur ati on  of   ur i ne:  (a)
hy per ex cr eti on  of   a  substance  that  i s  r el ati v el y   i nsol ubl e  i n  ur i ne,   (b)  l ow
ur i ne  v ol um e,   and  (c)  an  abnor m al   ur i ne  pH.   Ci tr ate  i s  a  natur al l y
occur r i ng  i nhi bi tor   of   stone  f or m ati on.   Ther ef or e,   l ow   ur i nar y   ci tr ate
ex cr eti on  has  al so  been  i m pl i cated  as  an  i ndependent  cause  of   cal ci um
stone  f or m ati on.

4.   What  ar e  the  acute  and  chr oni c  sequel ae  of   k i dney   stones?

The  acute  consequences  of   k i dney   stones  ar e  ur i nar y   tr act  obstr ucti on,
i nf ecti on,   hem atur i a,   pai n,   and,   uncom m onl y ,   acute  r enal   f ai l ur e.   Chr oni c
consequences  of   nephr ol i thi asi s  ar e  i nf ecti on,   RTA,   and  chr oni c  r enal
i nsuf f i ci ency .

5.   In  the  setti ng  of   ur i c  aci d  k i dney   stones,   i s  the  ov er satur ati on  of   ur i ne
w i th  ur i c  aci d  condi ti oned  pr i m ar i l y   by   the  ur i ne  pH  or   by   the  am ount  of
ur i c  aci d  ex cr eted?

Because  m onosodi um   ur ate  i s  m or e  sol ubl e  than  ur i c  aci d,   ur ate  stones


ar e  r ar e.   Ther e  i s  a  si gni f i cant  r i sk   f or   such  stones  onl y   w hen  the  ur i nar y
f or m   i s  m ai nl y   ur i c  aci d.   U r i c  aci d  i s  a  w eak   aci d  that  has  one  pr oton  that
i s  di ssoci abl e  under   phy si ol ogi c  condi ti ons  w i th  a  pK  (the  negati v e
l ogar i thm   of   the  i oni zati on  constant  of   an  aci d)  of   5. 3.   Ther ef or e,   ur ate
m ay   ex i st  i n  ur i ne  as
P. 409
ei ther   m onosodi um   ur ate  or   as  ur i c  aci d.   The  concentr ati on  r ati os  of
these  tw o  f or m s  i s  a  f uncti on  of   the  am bi ent  pH.   A  change  i n  the  ur i nar y
pH  f r om   5  to  6. 5  al ter s  the  undi ssoci ated  aci d  concentr ati on  ei ghtf ol d,
w her eas  the  ur i nar y   ex cr eti on  of   ur i c  aci d  can  i ncr ease  onl y   up  to
thr eef ol d.   Ther ef or e,   changes  i n  the  ur i ne  pH  pl ay   a  gr eater   r ol e  i n  ur i c
aci d  stone  f or m ati on  than  do  changes  i n  the  am ount  of   ur i c  aci d  ex cr eted.

6.   What  ar e  the  thr ee  ty pes  of   k i dney   stones  that  m ay   pr esent  i n  the  f or m
of   staghor n  cal cul i ,   and  w hat  ar e  the  r especti v e  m echani sm s  r esponsi bl e
f or   thei r   f or m ati on?

U r i c  aci d,   cy sti ne,   and  str uv i te  k i dney   stones  m ay   f or m   i n  the  r enal
col l ecti ng  sy stem   and  assum e  a  staghor n  conf i gur ati on.

Str uv i te  k i dney   stones,   w hi ch  ar e  the  m ost  com m on  staghor n  cal cul i ,   ar e
a  consequence  of   i nf ecti on  of   the  ur i nar y   tr act  w i th  bacter i a,   usual l y
Pr oteus  speci es,   w hi ch  contai n  ur ease.   Thi s  causes  ur ea  to  be  br ok en
dow n  to  2N H 3   +   H 2 O  +   CO 2 .   Am m oni a  r eacts  w i th  a  pr oton,   f or m i ng
am m oni um .   Thi s  r eacti on  r ai ses  the  ur i ne  pH,   r esul ti ng  i n  an  i ncr eased
concentr ati on  of   phosphate  i ons.   These  condi ti ons  spaw n  the  f or m ati on  of
str uv i te  (MgN H 4 PO 4   ·   6H 2 O),   and  m ay   al so  l ead  to  the  f or m ati on  of
car bonate  apati te  (Ca 1 0 [PO 4 ] 6   ·   CO 3 )  cr y stal s;  ther ef or e,   str uv i te  stones
m ay   contai n  v ar i abl e  pr opor ti ons  of   car bonate  apati te  and  str uv i te.

Cy sti ne  stones  ar e  a  m ani f estati on  of   cy sti nur i a,   a  r ar e  her edi tar y
di sor der   that  i s  char acter i zed  by   def ects  i n  di basi c  am i no  aci d  tr anspor t.
N or m al l y ,   am i no  aci ds  ar e  al m ost  com pl etel y   r eabsor bed  by   the  pr ox i m al
tubul e.   The  ur i nar y   ex cr eti on  of   cy sti ne  i s  abnor m al l y   hi gh  i n  peopl e  w i th
cy sti ne  stones,   how ev er ,   and  thi s  pr edi sposes  them   to  the  f or m ati on  of
cy sti ne  stones.   The  ur i ne  pH  has  l i ttl e  ef f ect  on  the  sol ubi l i ty   of   cy sti ne.
The  m echani sm   r esponsi bl e  f or   the  f or m ati on  of   ur i c  aci d  stones  i s
di scussed  i n  the  pr ecedi ng  questi on.

7.   What  ar e  the  pr i nci pal   causes  of   cal ci um   stones?

Ther e  ar e  num er ous  speci f i c  causes  of   cal ci um   k i dney   stones,   but  the
m ajor   causes  can  be  gr ouped  i nto  the  f ol l ow i ng  categor i es:  l ow   ur i nar y
v ol um e,   hy per cal ci ur i a,   hy per ox al ur i a,   hy per ur i cosur i a,   and  al k al i ne
ur i ne.   Hy poci tr atur i a  m ay   al so  be  an  i ndependent  cause  of   cal ci um   stone
f or m ati on,   al though  a  l ow   ur i nar y   ex cr eti on  of   ci tr ate  m ay   actual l y   be  a
consequence  of   an  al k al i ne  ur i ne.

8.   What  ar e  the  r outi ne  outpati ent  studi es  that  shoul d  be  per f or m ed  i n
pati ents  w i th  r ecur r ent  stones?

The  ur i ne  pH  and  v ol um e  shoul d  be  assessed,   and  the  24­hour   ur i nar y
ex cr eti on  of   sodi um ,   cal ci um ,   ur i c  aci d,   ci tr ate,   ox al ate,   phosphate,   and
cr eati ni ne  shoul d  be  deter m i ned.

9.   What  i s  the  i ndi cati on  f or   m easur i ng  the  ex cr eti on  of   ur i c  aci d  i n  the
setti ng  of   hy per cal ci ur i a?

In  the  setti ng  of   hy per cal ci ur i a,   ur i c  aci d  cr y stal s  m ay   act  as  seed
cr y stal s  that  i ni ti ate  the  pr eci pi tati on  of   cal ci um   ox al ate  f r om   the  ur i ne.
If   pati ents  ar e  f ound  to  be  hy per ur i cosur i c,   al l opur i nol   tr eatm ent  m i ght
be  w ar r anted.

10.   What  ar e  the  potenti al   causes  of   hy per cal ci ur i a  and  how   shoul d  i t  be
tr eated?

Most  com m onl y ,   hy per cal ci ur i a  i s  i di opathi c  i n  or i gi n.   Bef or e  m ak i ng  such


a  di agnosi s,   how ev er ,   other   causes  of   hy per cal ci ur i a  (i . e. ,   sar coi dosi s,
P. 410
i m m obi l i zati on,   v i tam i n  D  ex cess,   hy per thy r oi di sm ,   Paget's  di sease,   and
m al i gnant  tum or s  w i th  m etastasi s)  need  to  be  ex cl uded  (Tabl e  9­15).   The
pr i m ar y   appr oach  to  tr eatm ent  i nv ol v es  the  attenti on  to  the  under l y i ng
di sor der   w hen  i denti f i ed.   For   pati ents  w i th  i di opathi c  hy per cal ci ur i a,
tr eatm ent  i s  di r ected  at  l ow er i ng  ur i nar y   cal ci um   ex cr eti on.   Thi s  i s  best
achi ev ed  w i th  thi azi de  di ur eti cs,   acti ng  on  the  di stal   tubul e.   Thi s
appr oach  needs  to  be  coupl ed  w i th  a  decr ease  i n  sodi um   i ntak e,   w hi ch
w i l l   enhance  pr ox i m al   cal ci um   r eabsor pti on.

Table 9­15 Causes of Hypercalciuria

Ca us e Se rum Othe r Se rum U s ua l Stone


Ca lc ium Va lue s Type
Le ve l
Idi opathi c N or m al N or m al Cal ci um
hy per cal ci ur i a a   ox al ate  or
cal ci um
phosphate

Pr i m ar y Hi gh Hy pophosphatem i a, Cal ci um
hy per par athy r oi di sm occasi onal l y ox al ate  or
hy per chl or em i c cal ci um
aci dosi s phosphate

Renal   tubul ar N or m al Hy per chl or em i c


aci dosi s aci dosi s  Cal ci um  
phosphate

a Sar coi dosi s,   Cushi ng's  m m obi l i zati on,   v i tam i n  D  ex cess,

hy per thy r oi di sm ,   sy ndr om e,   al k al i   Paget's  nd  m al i gnant  tum or s  (w hi ch


cause  hy per cal ci ur i a,   di sease,   r api dl y   pr ogr essi v e  bone  al though  not
stones)  m ust  be  ex cl uded  on  cl i ni cal   gr ounds.

Case
A  48­y ear ­ol d  m an  pr esents  to  a  l ocal   em er gency   r oom   because  of   r i ght  f l ank
pai n  r adi ati ng  to  hi s  r i ght  testi cl e  that  has  l asted  f or   2  hour s.   The  pai n  w as
i ni ti al l y   m i l d  and  then  becam e  pr ogr essi v el y   sev er e  ov er   an  hour .   He  has  no
nausea  or   v om i ti ng,   f ev er   or   chi l l s,   dy sur i a,   hesi tancy ,   or   decr eased  ur i nar y
str eam .   He  has  no  hi stor y   of   pr ev i ous  k i dney   stones  or   ur i nar y   tr act
i nf ecti ons.   Hi s  past  m edi cal   hi stor y   i s  r em ar k abl e  onl y   f or   a  hi stor y   of   Cr ohn's
di sease,   w hi ch  r equi r ed  r esecti on  of   a  por ti on  of   hi s  i l eum .   He  tak es  no
m edi cati ons.
On  ex am i nati on,   he  i s  f ound  to  be  i n  obv i ous  di scom f or t.   Hi s  abdom en  i s  sof t
and  nontender   w i th  no  m asses.   Ther e  i s  m i l d  costov er tebr al   angl e  tender ness.
Hi s  testi cl es  ar e  nor m al .   The  r em ai nder   of   hi s  ex am i nati on  f i ndi ngs  ar e
unr em ar k abl e.   The  ur i ne  pH  i s  6,   and  ur i nal y si s  show s  1+   pr otei n  and  2+
hem e.   The  sedi m ent  contai ns  10  to  15  r ed  bl ood  cel l s,   0  to  5  w hi te  bl ood  cel l s
per   hi gh­pow er   f i el d,   and  a  m oder ate  am ount  of   am or phous  cr y stal s.   Ther e
ar e  no  casts.   Hi s  com pl ete  bl ood  count  and  el ectr ol y te  l ev el s  ar e  nor m al .   A
chest  r adi ogr aphi c  study   and  k i dney ,   ur eter ,   and  bl adder   (KU B)  f i l m   ar e
i nter pr eted  as  nor m al .
The  f ol l ow i ng  l abor ator y   data  ar e  r epor ted:  cal ci um ,   10  m g/dL;  phosphor us,
3. 7  m g/dL;  al bum i n,   4. 1  g/dL;  cr eati ni ne,   1  m g/dL;  and  BU N ,   12  m g/dL.   Hi s
bl ood  pr essur e  i s  140/85  m m   Hg,   pul se  i s  95  beats  per   m i nute,   r espi r ator y
r ate  i s  20  br eaths  per   m i nute,   and  tem per atur e  i s  37. 2°C  (98. 96°F).

1.   What  ar e  som e  of   the  possi bl e  r enal   causes  of   thi s  pati ent's  sy m ptom s?
2.   What  i s  the  si gni f i cance  of   the  cr y stal l ur i a?
P. 411
3.   Does  the  absence  of   a  col i c­l i k e  pai n  suggest  that  thi s  pati ent's  pai n  i s
not  due  to  a  k i dney   stone?
4.   What  w oul d  be  the  appr opr i ate  test  f or   conf i r m i ng  the  di agnosi s  of   a
k i dney   stone  i n  thi s  pati ent?
5.   Once  the  di agnosi s  of   a  k i dney   stone  i s  establ i shed,   w hat  i s  the
appr opr i ate  m anagem ent  that  shoul d  be  i m pl em ented  i n  the  em er gency
r oom ?
N oncontr ast  hel i cal   CT  scan  r ev eal s  a  r adi opaque  stone  at  the  l ef t
ur eter opel v i c  juncti on.   Subsequentl y ,   the  pati ent  passes  the  stone  i n  hi s
ur i ne  w hi l e  i n  the  em er gency   r oom .   Labor ator y   anal y si s  r ev eal s  that  the
stone  i s  com posed  pr i m ar i l y   of   cal ci um   ox al ate.   Subsequentl y ,   a  24­hour
ur i ne  col l ecti on  r ev eal ed  an  i ncr ease  i n  ur i nar y   ox al ate  ex cr eti on  (> 50
m g  per   24  hour s).
6.   What  ar e  the  possi bl e  causes  and  the  tr eatm ents  of   hy per ox al ur i a  as
seen  i n  thi s  pati ent?

Case Discussion
1.   What  ar e  som e  of   the  possi bl e  r enal   causes  of   thi s  pati ent's  sy m ptom s?

Ki dney   stones,   r enal   i nf ar cti on,   and  papi l l ar y   necr osi s  m ay   al l   pr esent
w i th  the  acute  onset  of   f l ank   pai n  together   w i th  hem atur i a.   How ev er ,
r enal   i nf ar cti on  usual l y   occur s  i n  a  pati ent  w ho  has  ei ther   a  l ocal   or
sy stem i c  cause  f or   thr om bosi s  (e. g. ,   tr aum a,   aneur y sm ,   or   v ascul i ti s
i nv ol v i ng  the  r enal   ar ter y )  or   thr om boem bol i sm   (e. g. ,   endocar di ti s,   m ur al
thr om bi ,   or   f at  em bol i ).   Papi l l ar y   necr osi s  ty pi cal l y   occur s  i n  pati ents
w i th  ei ther   adv anced  di abeti c  nephr opathy   or   si ck l e  cel l   di sease.   In
contr ast,   k i dney   stones  of ten  ar i se  i n  peopl e  w ho  hav e  no  k now n
contr i butor y   m edi cal   i l l ness.

2.   What  i s  the  si gni f i cance  of   the  cr y stal l ur i a?

Ex cept  f or   the  f i ndi ng  of   cy sti ne  cr y stal s,   w hi ch  i ndi cates  cy sti nur i a,
cr y stal l ur i a  i s  of   no  di agnosti c  v al ue  w hen  ev al uati ng  a  pati ent  f or
nephr ol i thi asi s,   as  cr y stal s  can  appear   i n  nor m al   ur i ne.

3.   Does  the  absence  of   a  col i c­l i k e  pai n  suggest  that  thi s  pati ent's  pai n  i s
not  due  to  a  k i dney   stone?

N o.   Ty pi cal l y ,   the  pai n  associ ated  w i th  k i dney   stones  i s  a  steady   pai n  that
gr adual l y   w or sens;  i t  does  not  f l uctuate,   as  the  ter m   r enal   col i c  suggests.

4.   What  w oul d  be  the  appr opr i ate  test  f or   conf i r m i ng  the  di agnosi s  of   a
k i dney   stone  i n  thi s  pati ent?

Al though  i n  som e  cases  nephr ol i thi asi s  can  be  di agnosed  on  the  basi s  of
the  KU B  r adi ogr aphi c  f i ndi ngs,   i t  i s  usual l y   necessar y   to  per f or m
ex cr etor y   ur ogr aphy ,   as  i n  thi s  pati ent,   to  establ i sh  the  di agnosi s.   It
al l ow s  the  l ocati on,   si ze,   shape,   and  r adi ol ucency   of   k i dney   stones  to  be
deter m i ned.   Al though  r etr ogr ade  py el ogr aphy   can  y i el d  the  sam e
i nf or m ati on,   i t  i s  a  m or e  ex pensi v e  and  i nv asi v e  pr ocedur e.
U l tr asonogr aphy   i s  not  as  sensi ti v e  as  ex cr etor y   ur ogr aphy   f or   detecti ng
k i dney   stones.   Mor e  r ecentl y ,   noncontr ast  hel i cal   CT  has  becom e  the
pr ocedur e  of   choi ce  i n  m ost  center s.

5.   Once  the  di agnosi s  of   a  k i dney   stone  i s  establ i shed,   w hat  i s  the


appr opr i ate  m anagem ent  that  shoul d  be  i m pl em ented  i n  the  em er gency
r oom ?

The  pati ent  shoul d  be  k ept  w el l   hy dr ated,   usual l y   w i th  i ntr av enous  f l ui ds,
to  m ai ntai n  a  br i sk   ur i ne  f l ow ,   w hi ch  m ay   pr om ote  passage  of   the  stone,
and  to
P. 412
di m i ni sh  the  r i sk   of   nephr otox i ci ty   f r om   the  r adi ocontr ast  agent.   Al l   of
the  pati ent's  ur i ne  shoul d  be  str ai ned  to  deter m i ne  i f   the  pati ent  has
passed  any   stones.   If   any   stones  ar e  obtai ned,   they   shoul d  be  sent  f or
anal y si s.   Pati ents  al m ost  al w ay s  r equi r e  nar coti c  anal gesi cs  f or
m anagem ent  of   the  pai n.   Pati ents  shoul d  be  adm i tted  to  the  hospi tal   i f
i nadequate  pai n  r el i ef   i s  obtai ned  w i th  or al   anal gesi cs,   or   i n  the  ev ent  of
ur i nar y   tr act  i nf ecti on  or   acute  r enal   f ai l ur e.

6.   What  ar e  the  possi bl e  causes  and  the  tr eatm ents  of   hy per ox al ur i a  as  seen
i n  thi s  pati ent?

Hy per ox al ur i a  can  r esul t  i n  suf f i ci ent  super satur ati on  of   the  ur i ne  w i th
cal ci um   ox al ate  to  cause  the  pr eci pi tati on  of   k i dney   stones.   Mor e  than
80%  of   ur i nar y   ox al ate  i s  der i v ed  f r om   endogenous  pr oducti on,   pr i m ar i l y
as  a  br eak dow n  pr oduct  of   gl y ox y l ate.   The  r em ai nder   of   ur i nar y   ox al ate
i s  obtai ned  f r om   di etar y   sour ces.   Ther ef or e,   hy per ox al ur i a  can  be  caused
by   pr i m ar y   ov er pr oducti on,   i ntesti nal   di sease,   and  di et.   Ov er pr oducti on  of
ox al ate  (pr i m ar y   hy per ox al ur i a)  i s  her edi tar y   and  sev er e,   but  r ar e.   Injur y
of   the  bow el   w al l   i nf l i cted  by   f atty   aci ds  or   bi l e  sal ts  can  r esul t  i n  an
i ncr eased  per m eabi l i ty   to  ox al ate.   The  m ost  usual   cl i ni cal   setti ng,   as  i n
thi s  pati ent,   i s  Cr ohn's  di sease,   i l eal   r esecti on,   or   jejunoi l eal   by pass.   A
hi gh  di etar y   i ntak e  of   ox al ate  m ay   be  due  to  the  i ngesti on  of   f oods  such
as  chocol ate,   nuts,   r hubar b,   tea,   and  som e  f r ui t  jui ces,   as  w el l   as  the
i ntak e  of   v i tam i n  C  i n  ex cess  of   1, 000  m g  per   day .   Tr eatm ent  usual l y
i nv ol v es  the  com bi nati on  of   a  l ow ­ox al ate  and  l ow ­f at  di et  together   w i th
adm i ni str ati on  of   or al   cal ci um   or   chol esty r am i ne  to  “bi ndâ€​   ox al ate  i n
the  i ntesti ne.   Contr ar y   to  pr ev i ousl y   hel d  noti ons,   r estr i cti on  of   cal ci um
i ntak e  coul d  be  del eter i ous.

Suggested Readings
Coe  FL.   The  pati ent  w i th  r enal   stones.   In:  Schr i er   RW,   ed.   Manual   of
nephr ol ogy ,   6th  ed.   Phi l adel phi a:  Li ppi ncott  Wi l l i am s  &  Wi l k i ns,   2005:90.

Monk   RD,   Bushi nsk y   DA.   N ephr ol i thi asi s  and  nephr ocal ci nosi s.   In:  Johnson
R,   Feehal l y   J,   eds.   Com pr ehensi v e  cl i ni cal   nephr ol ogy ,   2nd  ed.   Mosby ,
2003.

Nephrotic Syndrome
1.   What  i s  the  def i ni ti on  of   nephr oti c  sy ndr om e?

2.   What  ar e  the  causes  of   nephr oti c  sy ndr om e?

3.   What  ar e  the  possi bl e  com pl i cati ons  of   nephr oti c  sy ndr om e?

4.   What  ar e  the  tr eatm ent  opti ons  f or   nephr oti c  sy ndr om e?

Discussion
1.   What  i s  the  def i ni ti on  of   nephr oti c  sy ndr om e?

N ephr oti c  sy ndr om e  i s  a  cl i ni cal   enti ty   char acter i zed  by   (a)  pr otei nur i a  i n
ex cess  of   3. 5  g/1. 73  m 2   of   body   sur f ace  ar ea  (or   50  m g/k g  of   body
w ei ght)  per   day ;  (b)  hy poal bum i nem i a  (< 3  g/dL),   w hi ch  i s  a  consequence
of   the  r enal   l osses
P. 413
coupl ed  w i th  i nadequate  hepati c  com pensator y   sy nthesi s;  (c)  edem a,
w hi ch  i s  a  consequence  of   both  the  hy poal bum i nem i a  and  the  sodi um
r etenti on;  (d)  hy per l i pi dem i a,   w hi ch  i s  pr obabl y   due  to  the  i ncr eased
hepati c  sy nthesi s  of   v er y –l ow ­densi ty   l i popr otei ns  w hi ch  ar e  conv er ted
to  chol ester ol ­car r y i ng  l ow ­densi ty   l i popr otei ns;  and  (e)  pr esence  of
l i pi dur i a.   Im pai r ed  r em ov al   pl ay s  an  i m por tant  but  pr obabl y   secondar y
r ol e  i n  thi s  setti ng.

2.   What  ar e  the  causes  of   nephr oti c  sy ndr om e?

The  causes  of   nephr oti c  sy ndr om e  can  be  easi l y   di v i ded  i nto  tw o  br oad
categor i es.   The  pr i m ar y ,   or   i di opathi c,   f or m s  of   nephr oti c  sy ndr om e  ar e
those  f or   w hi ch  a  speci f i c  cause  cannot  be  i denti f i ed  despi te  a  r easonabl y
thor ough  ev al uati on.   The  f i v e  m ajor   hi stol ogi c  subty pes  of   pr i m ar y
nephr oti c  sy ndr om e  i ncl ude  m i ni m al ­change  di sease  (al so  cal l ed  l i poi d
nephr osi s  or   ni l   di sease),   m em br anous  gl om er ul onephr i ti s,
m em br anopr ol i f er ati v e  gl om er ul onephr i ti s  (al so  cal l ed  m esangi ocapi l l ar y
gl om er ul onephr i ti s),   f ocal   segm ental   gl om er ul ar   scl er osi s  (FSGS),   and
pr ol i f er ati v e  gl om er ul onephr i ti s.   The  cl i ni cal   and  hi stol ogi c
char acter i sti cs  of   pr i m ar y   nephr oti c  sy ndr om e  ar e  l i sted  i n  Tabl e  9­16.

Table 9­16 The Clinical and Histologic Features
of the Primary (Idiopathic) Nephrotic
Syndrome
G lome rula r Dis e a s e Dis tinguis hing Cha ra c te ris tic
Clinic a l a nd Morphologic
La bora tory Findings Fe a ture s

Mi ni m al ­change Most  com m on  cause LM:  nor m al


di sease i n  chi l dr en  (75%); IF:  negati v e
20%  of   adul ts; EM:  podocy te
ster oi d­  or ef f acem ent;  no
cy cl ophospham i de­ i m m une
sensi ti v e  (80%); deposi ts
nonpr ogr essi v e;
nor m al   r enal
f uncti on;  scant
hem atur i a

Focal   segm ental Most  com m on  cause LM:  ear l y â


gl om er ul oscl er osi s i n  adul ts €”segm ental
(40%­50%); scl er osi s  i n
m i cr oscopi c som e  gl om er ul i
hem atur i a; w i th  tubul ar
pr ogr essi v e  r enal atr ophy ;  l ateâ
f ai l ur e  (75%) €”scl er osi s  of
m ost  gl om er ul i

Mem br anous Peak   i nci dence, LM:  ear l y â


nephr opathy f our th  and  si x th €”nor m al ;  l ateâ
decades;  m al e­ €”GBM
f em al e,   2­3:1;  ear l y thi ck eni ng
hy per tensi on  (30%); IF:  gr anul ar   IgG
spontaneous and  C3
r em i ssi on  (20%); EM:
pr ogr essi v e  r enal subepi thel i al
f ai l ur e  (30%­40%) deposi ts  and
GBM  ex pansi on

Mem br anopr ol i f er ati v e Peak   i nci dence, LM:


gl om er ul onephr i ti s second  thr ough  thi r d hy per cel l ul ar
decades;  m i x ed gl om er ul i   w i th
nephr oti c­nephr i ti c dupl i cated  GBM
f eatur es;  sl ow l y EM:  ty pe  Iâ
pr ogr essi v e  i n  m ost, €”subendothel i al
r api d  i n  som e; i m m une
hy pocom pl em entem i a deposi ts;  ty pe
II—dense
deposi t  GBM
Pr ol i f er ati v e See  Tabl e  9­19  

LM,   l i ght  m i cr oscopy ;  IF,   i m m unof l uor escence;  IgG,   i m m u


nogl obul i n  G;  EM,   el ectr on  m i cr oscopy ;  GBM,   gl om er ul ar   basem ent
m em br ane.

P. 414
The  secondar y   f or m s  of   the  nephr oti c  sy ndr om e  ar e  those  associ ated  w i th
speci f i c  eti ol ogi c  ev ents  or   i n  w hi ch  gl om er ul ar   di sease  ar i ses  as  a
com pl i cati on  of   another   di sease  or   sy stem i c  pr ocess.   These  m ay   be
br oadl y   categor i zed  i nto  those  stem m i ng  f r om   i nf ecti ons,   neopl asi a,
m edi cati ons,   al l er gens,   m ul ti sy stem   di seases,   and  her edof am i l i al
di seases,   and  al so  i ncl ude  v ar i ous  m i scel l aneous  causes  (Tabl e  9­17).
Secondar y   nephr oti c  sy ndr om e  m ay   be  associ ated  w i th  any   of   the  m ajor
hi stol ogi c  subty pes  f ound  i n  i di opathi c  nephr oti c  sy ndr om e.   The  i di opathi c
nephr oti c  sy ndr om e  i s  m or e  com m on  than  the  secondar y   f or m .

3.   What  ar e  the  possi bl e  com pl i cati ons  of   nephr oti c  sy ndr om e?

The  com pl i cati ons  of   nephr oti c  sy ndr om e  i ncl ude  accel er ated
ather oscl er osi s,   i ncr eased  suscepti bi l i ty   to  i nf ecti ons,   osteom al aci a,   and
an  i ncr eased  i nci dence  of   thr om boem bol i c  ev ents.

4.   What  ar e  the  tr eatm ent  opti ons  f or   nephr oti c  sy ndr om e?

The  tr eatm ent  of   nephr oti c  sy ndr om e  depends  on  i ts  cause.   Cer tai nl y ,   i n
the  case  of   the  secondar y   nephr oti c  sy ndr om e,   i f   the  pr i m ar y   di sor der   i s
tr eated  ef f ecti v el y ,   the  nephr oti c  sy ndr om e  tends  to  r esol v e  as  w el l .   In
the  case  of   the  pr i m ar y   nephr oti c  sy ndr om e,   cer tai n  hi stol ogi c  subty pes
(i . e. ,   m i ni m al ­change  di sease  and  possi bl y   m em br anous  nephr opathy )
r espond  to  tr eatm ent  w i th  ster oi ds,   w i th  or   w i thout  cy totox i c  agents.
Di scussi on  of   the  potenti al   r ol e  f or   other   agents  such  as  cy cl ospor i ne  or
m y cophenol ate  i s  bey ond  the  scope  of   thi s  book .   Other   l esi ons  m ay   be
r ef r actor y   to  any   ty pe  of   ther apy .   Dr ugs  such  as  the  ACE  i nhi bi tor s  or
ARBs  m ay   be  usef ul   i n  r educi ng  the  pr otei nur i a  by   af f ecti ng  i ntr ar enal
hem ody nam i cs,   but  they   cannot  i n  any   w ay   al ter   the  pr i m ar y   gl om er ul ar
abnor m al i ty   i nv ol v ed.

Case
A  40­y ear ­ol d  w om an  i s  r ef er r ed  f or   ev al uati on  of   pr otei nur i a.   Apar t  f r om
occasi onal   ar thr al gi as,   she  has  f el t  w el l   but  i s  concer ned  about  pr ogr essi v e
w ei ght  gai n  and  m ar k ed  sw el l i ng  of   her   l ow er   ex tr em i ti es.   She  has  no
per sonal   or   f am i l y   hi stor y   of   r enal   di sease,   no  k now n  chr oni c  sy stem i c  i l l ness,
nor   i s  she  tak i ng  any   m edi cati ons.   Phy si cal   ex am i nati on  f i ndi ngs,   i ncl udi ng
bl ood  pr essur e,   ar e  nor m al ,   ex cept  f or   the  pr esence  of   edem a  that  i s  m ost
notabl e  i n  dependent  ar eas.   Labor ator y   ev al uati on  r ev eal s  a  nor m al
hem atocr i t,   as  w el l   as  ser um   gl ucose,   BU N ,   and  cr eati ni ne  l ev el s,   but  she  has
pr of ound  hy poal bum i nem i a  (1. 9  g/dL)  and  hy per chol ester ol em i a  (490  m g/dL).
U r i nal y si s  show s  4+   pr otei nur i a,   ov al   f at  bodi es,   and  f r ee  f at  dr opl ets,   but  no
cel l ul ar   el em ents  or   casts.   Her   24­hour   ur i nar y   ex cr eti on  of   pr otei n  i s  f ound  to
be  8. 6  g.

1.   What  i s  the  m ost  com m on  cause  of   the  secondar y   nephr oti c  sy ndr om e  i n
adul ts  i n  the  U ni ted  States?  In  pati ents  w i th  thi s  di sor der ,   w hi ch  ear l y
f i ndi ng  ser v es  as  a  har bi nger   f or   the  subsequent  dev el opm ent  of
nephr oti c  sy ndr om e  and  r enal   i nsuf f i ci ency ?
2.   What  f eatur es  of   the  hi stor y   and  phy si cal   ex am i nati on  ar e  i m por tant  i n
deter m i ni ng  i f   thi s  pati ent  has  a  pr i m ar y   (i di opathi c)  or   secondar y   f or m
of   the  nephr oti c  sy ndr om e?
3.   What  addi ti onal   l abor ator y   tests  w oul d  y ou  or der   ei ther   to  establ i sh  or
r ef ute  a  secondar y   cause  of   the  nephr oti c  sy ndr om e?
4.   How   shoul d  thi s  pati ent's  ev al uati on  pr oceed?

P. 415

Table 9­17 Disorders Associated with Secondary
Nephrotic Syndrome

Inf ecti ous  di seases


Bacter i al :  poststr eptococcal   gl om er ul onephr i ti s,   i nf ecti v e
endocar di ti s,   nephr i ti s,   “shuntâ€​   sy phi l i s,   l epr osy
Vi r al :  hepati ti s  B  and  C,   cy tom egal ov i r us,   Epstei n­Bar r
v i r us,   her pes  zoster ,   hum an  i m m unodef i ci ency   v i r us
i nf ecti ons
Pr otozoal :  m al ar i a,   tox opl asm osi s
Hel m i nthi c:  schi stosom i asi s,   tr y panosom i asi s,   f i l ar i asi s
N eopl asti c  di seases
Sol i d  tum or s  (car ci nom a  and  sar com a):  col on,   l ung,   br east,
stom ach,   k i dney
Hem atol ogi c  m al i gnanci es  (l euk em i as  and  l y m phom as)
Medi cati ons
N onster oi dal   anti i nf l am m ator y   agents
Or gani c,   i nor gani c,   el em ental   m er cur y
Or gani c  gol d
Peni ci l l am i ne
“Str eetâ€​   her oi n
Pr obeneci d
Bi sm uth
Captopr i l
Mul ti sy stem   di seases
Sy stem i c  l upus  er y them atosus
Mi x ed  connecti v e  ti ssue  di sease
Der m atom y osi ti s
Der m ati ti s  her peti f or m i s
Sar coi dosi s
Henoch­Schönl ei n  pur pur a
Goodpastur e's  sy ndr om e
Rheum atoi d  ar thr i ti s
Am y l oi dosi s
Pol y ar ter i ti s
Al l er gi c  r eacti ons
Bee  sti ng
Pol l ens
Poi son  i v y   and  poi son  oak
Ser um   si ck ness  (anti tox i ns)
Metabol i c  di seases
Di abetes  m el l i tus
My x edem a
Hy per thy r oi di sm
Her edof am i l i al   di seases
Al por t's  sy ndr om e
Fabr y 's  di sease
N ai l ­patel l a  sy ndr om e
Si ck l e  cel l   di sease
a1­Anti tr y psi n  def i ci ency
Congeni tal   nephr oti c  sy ndr om e  (Fi nni sh  ty pe)
Her edi tar y   am y l oi dosi s  (f am i l i al   Medi ter r anean  f ev er )
Mi scel l aneous
Chr oni c  r enal   al l ogr af t  r ejecti on
Pr egnancy ­associ ated  (pr eecl am psi a,   r ecur r ent  or   tr ansi ent)
Vesi cour eter i c  r ef l ex

P. 416

Case Discussion
1.   What  i s  the  m ost  com m on  cause  of   the  secondar y   nephr oti c  sy ndr om e  i n
adul ts  i n  the  U ni ted  States?  In  pati ents  w i th  thi s  di sor der ,   w hi ch  ear l y
f i ndi ng  ser v es  as  a  har bi nger   f or   the  subsequent  dev el opm ent  of
nephr oti c  sy ndr om e  and  r enal   i nsuf f i ci ency ?

Di abetes  m el l i tus  i s  the  m ost  com m on  cause  of   secondar y   nephr oti c
sy ndr om e  i n  adul ts  i n  the  U ni ted  States.   In  pati ents  w i th  ei ther   ty pe  1  or
ty pe  2  di abetes,   the  onset  of   m i cr oal bum i nur i a  (al bum i n  ex cr eti on  of   20
to  200  µg  per   m i nute  or   30  to  300  m g/g  Cr   per   day )  pr edi cts  the
subsequent  dev el opm ent  of   nephr oti c  sy ndr om e  and  r enal   i nsuf f i ci ency .
These  pati ents  shoul d  begi n  tr eatm ent  w i th  an  ACE  i nhi bi tor   or   ARBs.

2.   What  f eatur es  of   the  hi stor y   and  phy si cal   ex am i nati on  ar e  i m por tant  i n
deter m i ni ng  i f   thi s  pati ent  has  a  pr i m ar y   (i di opathi c)  or   secondar y   f or m
of   the  nephr oti c  sy ndr om e?
Di f f er enti ati ng  betw een  the  pr i m ar y   and  secondar y   f or m s  of   the  nephr oti c
sy ndr om e  depends  on  a  car ef ul   r ev i ew   of   the  pati ent's  hi stor y   and
phy si cal   ex am i nati on  f i ndi ngs  and  the  per f or m ance  of   sel ected  l abor ator y
tests  that  can  i denti f y   under l y i ng  di sease  states.   It  i s  i m per ati v e  to
deter m i ne  i f   ther e  i s  a  f am i l y   or   per sonal   hi stor y   of   di abetes  m el l i tus  or
connecti v e  ti ssue  di sease,   her edi tar y   condi ti ons  such  as  si ck l e  cel l
di sease  or   Al por t's  sy ndr om e,   al l er gen  ex posur e,   and  so  f or th.   A
com pl ete  m edi cati on  l i st  m ust  be  obtai ned,   i ncl udi ng  the  use  of
nonpr escr i pti on  m edi ci nes  such  as  N SAIDs.   A  hi stor y   of   i l l i ci t  dr ug  use  i s
equal l y   i m por tant  because  her oi n  nephr opathy   i s  not  r ar e  i n  dr ug
abuser s.   In  addi ti on,   a  tr av el   hi stor y   i s  a  cr uci al   par t  of   the  hi stor y
tak i ng  because,   f or   ex am pl e,   m al ar i a  i s  a  w el l ­k now n  cause  of   the
nephr oti c  sy ndr om e  and  shoul d  be  consi der ed  i n  those  pati ents  w ho  hav e
tr av el ed  to  endem i c  ar eas.   Ri sk   f actor s  f or   hepati ti s  and  hum an
i m m unodef i ci ency   v i r us  (HIV)  i nf ecti on  m ust  al so  be  sought  because  hi gh­
r i sk   popul ati ons  shoul d  be  scr eened  f or   these  di sor der s.   In  thi s  par ti cul ar
pati ent  (a  y oung  w om an),   the  hi stor y   of   occasi onal   ar thr al gi as  br i ngs  up
the  possi bi l i ty   of   a  m ul ti sy stem   di sease  as  the  sour ce  of   the  nephr oti c
sy ndr om e.

3.   What  addi ti onal   l abor ator y   tests  w oul d  y ou  or der   ei ther   to  establ i sh  or
r ef ute  a  secondar y   cause  of   the  nephr oti c  sy ndr om e?

Labor ator y   tests  that  ar e  usef ul   i n  establ i shi ng  a  secondar y   cause  of   the
nephr oti c  sy ndr om e  i ncl ude  the  ser um   gl ucose  l ev el ,   an  anti nucl ear
anti body   (AN A)
P. 417
deter m i nati on,   com pl em ent  l ev el s,   hepati ti s  scr eeni ng,   v ener eal   di sease
r esear ch  l abor ator y   test,   HIV  test,   si ck l e  cel l   pr epar ati on,   an
anti str eptol y si n  ti ter ,   thr oat  cul tur e,   and  ser um   and  ur i nar y   pr otei n
el ectr ophor esi s.   The  f i ndi ngs  y i el ded  by   the  hi stor y   and  phy si cal
ex am i nati on  di ctate  w hi ch  of   these  tests  shoul d  be  per f or m ed  i n  a
par ti cul ar   pati ent.   In  thi s  pati ent,   the  AN A  test  i s  posi ti v e  and  the
com pl em ent  l ev el s  ar e  l ow ,   i ndi cati ng  that  she  m ay   hav e  sy stem i c  l upus
er y them atosus  (SLE)  as  the  cause  of   her   nephr oti c  sy ndr om e.

4.   How   shoul d  thi s  pati ent's  ev al uati on  pr oceed?

In  the  setti ng  of   SLE,   a  k i dney   bi opsy   shoul d  be  per f or m ed  i n  an  ef f or t  to
establ i sh  the  natur e  of   the  under l y i ng  di sor der   r esponsi bl e  f or   the
nephr oti c  sy ndr om e.   Thi s  pati ent  m ost  l i k el y   has  ei ther   di f f use
pr ol i f er ati v e  gl om er ul onephr i ti s  or   m em br anous  nephr opathy   w i th  SLE.
The  ther apy   f or   the  f or m er   cal l s  f or   tr eatm ent  w i th  ster oi ds  and  cy totox i c
agents,   al though  the  l atter   does  not.

Suggested Readings
Ber nar d  DB.   Ex tr ar enal   com pl i cati ons  of   the  nephr oti c  sy ndr om e.   Ki dney
Int  1988;33:1184.
Kay sen  GA.   Pr otei nur i a  and  the  nephr oti c  sy ndr om e.   In:  Schr i er   RW,   ed.
Renal   and  el ectr ol y te  di sor der s,   6th  ed.   Phi l adel phi a:  Li ppi ncott  Wi l l i am s  &
Wi l k i ns,   2003:580.

Glomerulonephritis
1.   What  i s  the  def i ni ti on  of   hem atur i a?

2.   What  ar e  the  m ajor   causes  of   hem atur i a?

3.   What  can  hel p  poi nt  tow ar d  a  gl om er ul ar   or i gi n  as  the  sour ce  of   the
hem atur i a?

4.   What  i s  the  def i ni ti on  of   the  nephr i ti c  sy ndr om e?

5.   What  ar e  the  pr i m ar y   di seases  of   the  k i dney   associ ated  w i th  gl om er ul ar


hem atur i a  (nephr i ti c  sy ndr om e)?

6.   What  sy stem i c  di seases  ar e  associ ated  w i th  gl om er ul ar   hem atur i a?

7.   How   i s  r api dl y   pr ogr essi v e  gl om er ul onephr i ti s  (RPGN )  def i ned?

8.   What  cl i ni cal   di sor der s  cause  RPGN ?

Discussion
1.   What  i s  the  def i ni ti on  of   hem atur i a?

Hem atur i a  r ef er s  to  the  pr esence  of   an  abnor m al l y   hi gh  num ber   of   r ed
bl ood  cel l s  (> 5  per   hi gh­pow er   f i el d)  i n  the  ur i ne.   Thi s  i s  m ost  com m onl y
detected  by   a  di psti ck   (Hem asti x )  m ethod,   w hi ch  i denti f i es  the  pr esence
of   hem ogl obi n.   The  hem atur i a  i s  consi der ed  m acr oscopi c  w hen  the  ur i ne
i s  obv i ousl y   r ed  due  to  the  pr esence  of   bl ood,   and  i t  i s  deem ed
m i cr oscopi c  w hen  the  ur i ne  gr ossl y   appear s  nor m al .   A  num ber   of   f oods
(such  as  beets)  and  som e  dr ugs  (such  as  phenazopy r i di ne  hy dr ochl or i de)
as  w el l   as  por phy r i a  can  tur n  the  ur i ne  r ed.   In  these  ci r cum stances,   the
di psti ck   r esul t  i s  negati v e.

P. 418
2.   What  ar e  the  m ajor   causes  of   hem atur i a?

The  causes  of   hem atur i a  ar e  best  appr oached  i n  ter m s  of   thei r   bei ng
ei ther   ex tr ar enal   or   r enal   i n  or i gi n.   Ex tr ar enal   bl eedi ng  can  occur   i n  the
ur eter s  due  to  cal cul i   or   car ci nom a;  i n  the  bl adder   due  to  hem or r hagi c
cy sti ti s  stem m i ng  f r om   i nf ecti on  (i ncl udi ng  Schi stosom a  haem atobi um   i n
endem i c  ar eas),   as  w el l   as  f r om   cy cl ophospham i de  use,   car ci nom a,
catheter i zati on,   or   cal cul i ;  i n  the  pr ostate  due  to  hy per tr ophy ,   car ci nom a,
or   pr ostati ti s;  and  i n  the  ur ethr a  due  to  ur ethr i ti s  or   tr aum a.   Renal
causes  of   hem atur i a  can  be  cl assi f i ed  as  ei ther   gl om er ul ar   or
nongl om er ul ar   and  ar e  l i sted  i n  Tabl e  9­18.
3.   What  can  hel p  poi nt  tow ar d  a  gl om er ul ar   or i gi n  as  the  sour ce  of   the
hem atur i a?

The  f ol l ow i ng  f i ndi ngs  poi nt  tow ar d  a  gl om er ul ar   cause  as  the  sour ce  of
hem atur i a:  (a)  the  pr esence  of   dy sm or phi c  r ed  bl ood  cel l s  on  phase­
contr ast  m i cr oscopy ;  (b)  the  pr esence  of   r ed  bl ood  cel l   casts,   w hi ch  i s
v i r tual l y   a  di agnosti c  f i ndi ng;  and  (c)  pr otei nur i a  ex ceedi ng  500  m g  per
day .

4.   What  i s  the  def i ni ti on  of   the  nephr i ti c  sy ndr om e?

The  nephr i ti c  sy ndr om e  i s  def i ned  by   a  constel l ati on  of   ur i nar y   f i ndi ngs
that  i ncl ude  the  pr esence  of   hem atur i a,   pr otei nur i a,   and  r ed  bl ood  cel l
casts.   These  f i ndi ngs  i ndi cate  the  pr esence  of   a  gl om er ul ar   l esi on  and  ar e
f r equentl y   accom pani ed  by   azotem i a,   hy per tensi on,   and  edem a.

5.   What  ar e  the  pr i m ar y   di seases  of   the  k i dney   associ ated  w i th  gl om er ul ar


hem atur i a  (nephr i ti c  sy ndr om e)?

The  pr i m ar y   di seases  associ ated  w i th  gl om er ul ar   hem atur i a  ar e


i m m unogl obul i n  A  (IgA)  nephr opathy ,   poststr eptococcal
gl om er ul onephr i ti s,   m em br anopr ol i f er ati v e  gl om er ul onephr i ti s,   and
i di opathi c  RPGN .

6.   What  sy stem i c  di seases  ar e  associ ated  w i th  gl om er ul ar   hem atur i a?

SLE,   Henoch­Schönl ei n  pur pur a,   Goodpastur e's  sy ndr om e,   v ascul i ti s


(i ncl udi ng  pol y ar ter i ti s  nodosa  and  Wegener 's  gr anul om atosi s),   and
essenti al   m i x ed  cr y ogl obul i nem i a  ar e  al l   associ ated  w i th  gl om er ul ar
hem atur i a.

7.   How   i s  RPGN   def i ned?

RPGN   i s  pr i m ar i l y   def i ned  i n  cl i ni cal   ter m s  as  a  gl om er ul ar   di sease


char acter i zed  by   pr ogr essi on  to  end­stage  r enal   di sease  w i thi n  w eek s  to
m onths.   The  pathol ogi c  cor r el ate  i s  ex tensi v e  cr escent  f or m ati on  i n  the
gl om er ul i ,   as  seen  i n  k i dney   bi opsy   speci m ens.

8.   What  cl i ni cal   di sor der s  cause  RPGN ?

A  num ber   of   di sor der s  cause  RPGN .   These  ar e  best  def i ned  i n
i m m unopathol ogi c  ter m s,   dependi ng  on  the  absence  or   pr esence  (and
patter n)  of   i m m une  deposi ts  (Tabl e  9­19).

Case
A  21­y ear ­ol d  col l ege  student  i s  r ef er r ed  to  the  r enal   cl i ni c  f or   f ur ther
ev al uati on  of   m i cr oscopi c  hem atur i a,   w hi ch  w as  di scov er ed  dur i ng  a
pr eem pl oy m ent  phy si cal   ex am i nati on.   Ther e  i s  no  hi stor y   of   r ecent  i nf ecti ons,
tr aum a,   or   i ntr av enous  dr ug  abuse.   She  deni es  any   hi stor y   of   r ashes,
ar thr al gi a,   m y al gi as,   f ev er s,   or   epi sodes  of   gr oss  hem atur i a.
Phy si cal   ex am i nati on  r ev eal s  a  w el l ­dev el oped,   w el l ­nour i shed  w om an  w ho  i s
i n  no  acute  di str ess.   Her   bl ood  pr essur e  i s  125/85  m m   Hg;  pul se,   72  beats  per
m i nute;  and
P. 419
P. 420
r espi r ator y   r ate,   16  br eaths  per   m i nute.   N o  r ashes,   l y m phadenopathy ,   or   joi nt
tender ness  i s  noted.   The  r em ai nder   of   the  phy si cal   ex am i nati on  f i ndi ngs  ar e
w i thi n  nor m al   l i m i ts.

Table 9­18 Glomerular and Nonglomerular Renal
Parenchymal Causes of Hematuria

Gl om er ul ar
Pr ol i f er ati v e  gl om er ul onephr i ti s
Pr i m ar y
Secondar y
Fam i l i al   di seases  of   the  gl om er ul us
Al por t's  sy ndr om e
Recur r ent  beni gn  hem atur i a  (thi n  basem ent  m em br ane
di sease)
Mal i gnant  hy per tensi on
N ongl om er ul ar
N eopl asm s
Renal   cel l   car ci nom a
Wi l m s'  tum or
Beni gn  cy sts
Vascul ar
Renal   i nf ar ct
Renal   v ei n  thr om bosi s
Mal i gnant  hy per tensi on
Ar ter i ov enous  m al f or m ati on
Capi l l ar y   necr osi s
Loi n  pai n­hem atur i a  sy ndr om e
Metabol i c
Hy per cal ci ur i a
Hy per ur i cosur i a
Fam i l i al
Pol y cy sti c  k i dney   di sease
Medul l ar y   sponge  k i dney
Papi l l ar y   necr osi s
Anal gesi c  abuse
Si ck l e  cel l   di sease  and  tr ai t
Renal   tuber cul osi s
Di abetes
Obstr ucti v e  ur opathy
Dr ugs
Anti coagul ants  (hepar i n,   coum ar i n)
Dr ug­i nduced  acute  i nter sti ti al   nephr i ti s
Tr aum a
Adapted  f r om   Li eber thal   W.   Hem atur i a  and  the  acute  nephr i ti c
sy ndr om e.   In:  Jacobson  HR,   Str i k er   GE,   Kl ahr   S,   eds.   The  pr i nci pl es
and  pr acti ce  of   nephr ol ogy .   Phi l adel phi a:  BC  Deck er ,   1991.

Table 9­19 Immunopathogenetic Classification of
Rapidly Progressive Glomerulonephritis

Anti ­GBM  anti body   (l i near   i m m une  deposi ts)


Wi th  l ung  hem or r hage  (Goodpastur e's
Wi thout  l ung  hem or r hage  (i di opathi c)
Im m une  com pl ex   (gr anul ar   i m m une  deposi ts)
Pr edom i nantl y   IgA
IgA  nephr opathy
Henoch­Schonl ei n  pur pur a
Pr edom i nantl y   IgG  (other s  m ay   be  pr esent)
Posti nf ecti ous
Vi scer al   abscess
Bacter i al   endocar di ti s
Lupus  nephr i ti s
Cr y ogl obul i nem i a
Mem br anopr ol i f er ati v e  gl om er ul onephr i ti s
Pauci i m m une  (no  i m m une  deposi ts)
Vascul i ti s
Mi cr oscopi c  pol y ar ter i ti s
Wegener 's
Hy per sensi ti v i ty   v ascul i ti des  (e. g. ,   Chur g­Str auss
sy ndr om e)
Idi opathi c

GBM,   gl om er ul ar   basem ent  m em br ane;  IgA,   i m m unogl obul i n  A;  IgG,


i m m unogl obul i n  G.

The  f ol l ow i ng  l abor ator y   data  ar e  r epor ted:  ser um   sodi um ,   135  m Eq/L;
potassi um ,   4. 5  m Eq/L;  chl or i de,   105  m Eq/L;  car bon  di ox i de,   25  m Eq/L;
gl ucose,   98  m g/dL;  BU N ,   12  m g/dL;  and  cr eati ni ne,   0. 8  m g/dL.   U r i nal y si s
show s  a  speci f i c  gr av i ty   of   1. 015,   pH  of   5. 0,   1+   hem e,   and  1+   pr otei n  on
di psti ck   ex am i nati on.   Mi cr oscopi c  ex am i nati on  of   the  ur i ne  r ev eal s  5  to  10  r ed
bl ood  cel l s  per   hi gh­pow er   f i el d,   and  possi bl y   one  r ed  bl ood  cel l   cast  i s  noted
on  cl ose  scr uti ny   of   the  enti r e  sl i de.   The  24­hour   ur i ne  ex cr eti on  i s  of   1. 5  L
total   v ol um e,   w i th  1, 200  m g  of   cr eati ni ne  and  1, 200  m g  of   pr otei n.
On  f ur ther   l abor ator y   ex am i nati on,   no  secondar y   sy stem i c  cause  f or   the
nephr i ti c  sy ndr om e  i s  i denti f i ed.   Speci f i cal l y ,   AN A  and  anti neutr ophi l
cy topl asm i c  anti body   tests  ar e  negati v e,   as  ar e  tests  f or   hepati ti s  B  and  C.
Li k ew i se,   both  the  C3  and  C4  com pl em ent  l ev el s  ar e  nor m al .   Consequentl y ,   a
per cutaneous  r enal   bi opsy   i s  per f or m ed.   The  hi stol ogi c,   i m m unof l uor escence,
and  el ectr on  m i cr oscopy   f i ndi ngs  ar e  al l   consi stent  w i th  IgA  nephr opathy .

1.   What  ar e  the  cl i ni cal   enti ti es  that  hav e  been  associ ated  w i th  pr om i nent
m esangi al   IgA  deposi ts?
2.   What  cl i ni cal   f i ndi ngs  i ndi cate  a  poor   pr ognosi s  i n  IgA  nephr opathy ?
3.   What  i s  the  cl i ni cal   cour se  of   IgA  nephr opathy ?
P. 421
4.   What  w oul d  y ou  adv i se  thi s  pati ent  i f   she  w er e  to  contem pl ate
pr egnancy ?
5.   What  tr eatm ent  opti ons  ar e  av ai l abl e  f or   thi s  pati ent?

Case Discussion
1.   What  ar e  the  cl i ni cal   enti ti es  that  hav e  been  associ ated  w i th  pr om i nent
m esangi al   IgA  deposi ts?

Henoch­Schönl ei n  pur pur a,   chr oni c  l i v er   di sease,   der m ati ti s


her peti f or m i s,   ax i al   ar thr opathi es,   and  Ber ger 's  di sease  hav e  al l   been
f ound  i n  the  setti ng  of   m esangi al   IgA  deposi ts.

2.   What  cl i ni cal   f i ndi ngs  i ndi cate  a  poor   pr ognosi s  i n  IgA  nephr opathy ?

The  cl i ni cal   f i ndi ngs  that  por tend  a  poor   pr ognosi s  i n  IgA  nephr opathy   ar e
per si stent  pr otei nur i a  of   gr eater   than  1  g  per   day ,   el ev ated  bl ood
pr essur e,   m al e  gender ,   an  el ev ated  ser um   cr eati ni ne  l ev el ,   and  the
absence  of   m acr oscopi c  hem atur i a.

3.   What  i s  the  cl i ni cal   cour se  of   IgA  nephr opathy ?

Pati ents  w i th  IgA  nephr opathy   m ay   ex per i ence  i nter m i ttent  epi sodes  of
gr oss  hem atur i a,   and  5%  to  10%  of   the  pati ents  m ay   hav e  ear l y   nephr oti c
sy ndr om e.   End­stage  r enal   di sease  dev el ops  i n  appr ox i m atel y   10%  of
af f ected  pati ents  by   10  y ear s,   and  by   20  y ear s  i n  20%  of   af f ected
pati ents.   In  addi ti on,   another   20%  to  30%  m ay   ex per i ence  som e  decl i ne
i n  r enal   f uncti on  w i thi n  20  y ear s.

4.   What  w oul d  y ou  adv i se  thi s  pati ent  i f   she  w er e  to  contem pl ate  pr egnancy ?

Despi te  ear l y   r epor ts  to  the  contr ar y ,   l ar ge  r etr ospecti v e  sur v ey s  r ev eal
no  ev i dence  i ndi cati ng  that  IgA  nephr opathy   unf av or abl y   al ter s  the  cour se
of   pr egnancy .   In  addi ti on,   the  chances  f or   a  successf ul   pr egnancy   ar e
ex cel l ent  i f   the  pati ent  r em ai ns  f r ee  of   hy per tensi on  or   r enal
i nsuf f i ci ency .

5.   What  tr eatm ent  opti ons  ar e  av ai l abl e  f or   thi s  pati ent?


Ther e  i s  no  pr ov en  tr eatm ent  f or   IgA  nephr opathy .   The  r esul ts  of   som e
tr i al s  of   ster oi ds  hav e  suggested  that  they   ar e  som ew hat  ef f ecti v e  i n
pati ents  w i th  per si stent  pr otei nur i a,   w hen  r enal   f uncti on  i s  sti l l   w el l
pr eser v ed  (S Cr   < 1. 4  m g/dL).

Suggested Readings
Adl er   SG,   Fai r l ey   K.   The  pati ent  w i th  hem atur i a,   pr otei nur i a,   or   both,   and
abnor m al   f i ndi ngs  on  ur i nar y   m i cr oscopy .   In:  Schr i er   RW,   ed.   Manual   of
nephr ol ogy ,   6th  ed.   Phi l adel phi a:  Li ppi ncott  Wi l l i am s  &  Wi l k i ns,   2005:  116.

Gl assock   RJ.   The  gl om er ul opathi es.   In:  Schr i er   RW,   ed.   Renal   and
el ectr ol y te  di sor der s,   6th  ed.   Phi l adel phi a:  Li ppi ncott  Wi l l i am s  &  Wi l k i ns,
2003:623.

Hyperkalemia
1.   What  ar e  the  causes  of   spur i ous  hy per k al em i a?

2.   What  ar e  the  pr i m ar y   m echani sm s  that  under l i e  hy per k al em i a,   and  w hat


ar e  the  causes  of   each?

P. 422
3.   At  w hat  l ev el   of   r enal   i nsuf f i ci ency   does  hy per k al em i a  occur ?

4.   What  ar e  the  cl i ni cal   consequences  of   hy per k al em i a?

5.   What  ther apeuti c  opti ons  ar e  av ai l abl e  f or   hy per k al em i c  pati ents,   and
how   r api dl y   do  they   r ev er se  the  pr ocess?

Discussion
1.   What  ar e  the  causes  of   spur i ous  hy per k al em i a?

The  causes  of   spur i ous  hy per k al em i a  (pseudohy per k al em i a)  com pr i se


hem ol y si s  of   the  bl ood  sam pl e,   a  m ar k ed  l euk ocy tosi s  (w hi te  bl ood  cel l
count  > 50, 000/m m 3 ),   thr om bocy tosi s  (pl atel et  count  > 800, 000/m m 3 ),   and
an  ex cessi v el y   ti ght  tour ni quet.

2.   What  ar e  the  pr i m ar y   m echani sm s  that  under l i e  hy per k al em i a,   and  w hat


ar e  the  causes  of   each?

The  pr i m ar y   m echani sm s  that  br i ng  about  hy per k al em i a  ar e  an  i ncr eased


potassi um   i nput  f r om   ei ther   endogenous  (e. g. ,   hem atom as  or
r habdom y ol y si s)  or   ex ogenous  sour ces,   a  tr anscel l ul ar   r edi str i buti on  of
potassi um ,   and  decr eased  ur i nar y   ex cr eti on  of   potassi um   as  occur s  i n
r enal   i nsuf f i ci ency .   The  causes  of   these  potassi um ­r el ated  abnor m al i ti es
ar e  l i sted  i n  Tabl e  9­20.
3.   At  w hat  l ev el   of   r enal   i nsuf f i ci ency   does  hy per k al em i a  occur ?

In  the  absence  of   other   f actor s,   hy per k al em i a  super v enes  i n  pati ents  w i th
r enal   di sease  w hen  the  GFR  i s  l ess  than  10  m L  per   m i nute.   The  adapti v e
r esponse  to  decr eased  r enal   m ass  i nv ol v es  the  i ncr eased  ex cr eti on  of
potassi um   per   nephr on;  thi s  m ai ntai ns  nor m ok al em i a  despi te  an
unchanged  potassi um   i ntak e  (usual l y   60  to  80  m Eq  per   day ).   How ev er ,   i n
the  pr esence  of   the  pr ocesses  l i sted  i n  the  answ er   to  the  pr ev i ous
questi on,   hy per k al em i a  ar i ses  w hen  the  GFR  i s  hi gher   (as  hi gh  as  40  m L
per   m i nute).

4.   What  ar e  the  cl i ni cal   consequences  of   hy per k al em i a?

The  m ost  i m m edi ate  and  i m por tant  i m pact  of   hy per k al em i a  i s  on  the  cel l s
possessi ng  ex ci tabl e  m em br anes  (ner v e  and  m uscl e)  because  i t
depol ar i zes  such  cel l s.   The  m ost  si gni f i cant  ef f ect  of   hy per k al em i a  i s  on
the  hear t.   The  ty pi cal   sequence  of   el ectr ocar di ogr aphi c  changes  seen  w i th
i ncr easi ng  degr ees  of   hy per k al em i a  i ncl ude  tal l ,   peak ed  T  w av es;  P­w av e
abnor m al i ti es  (i ncl udi ng  l oss  of   the  P  w av e);  pr ol ongati on  of   the  QRS
com pl ex ;  si nus  ar r est;  atr i ov entr i cul ar   di ssoci ati on;  v entr i cul ar
f i br i l l ati on;  and  car di ac  ar r est.

5.   What  ther apeuti c  opti ons  ar e  av ai l abl e  f or   hy per k al em i c  pati ents,   and
how   r api dl y   do  they   r ev er se  the  pr ocess?

The  v ar i ous  ther apeuti c  opti ons  f or   hy per k al em i a  ar e  l i sted  i n  Fi g.   9­2.   As


show n,   cal ci um   gl uconate  has  the  m ost  r api d  onset  and  shoul d  ther ef or e
be  the  f i r st­l i ne  tr eatm ent  to  pr otect  agai nst  the  neur om uscul ar   ef f ects  of
hy per k al em i a.   N ote  al so  that  the  use  of   cal ci um   gl uconate,   i nsul i n  w i th
gl ucose,   or   sodi um   bi car bonate  does  not  decr ease  total ­body   potassi um
content;  unl ess  a  decr ease  i n  total ­body   potassi um   i s  achi ev ed  (e. g. ,   w i th
k al i ur esi s,   k ay ex al ate,   or   di al y si s),   hy per k al em i a  w i l l   r ecur   w hen  the
ther apeuti c  ef f ect  of   these  agents  di ssi pates.

P. 423

Table 9­20 The Causes of Hyperkalemia

Causes  of   i ncr eased  potassi um   i nput


Ex ogenous  potassi um   l oads
Rapi d  i ntr av enous  potassi um   adm i ni str ati on
Hi gh  potassi um   i ntak e  w i th  sev er e  sodi um   r estr i cti on
Endogenous  potassi um   l oads
Rhabdom y ol y si s
Hem ol y si s
Tum or   l y si s  sy ndr om e
Hem atom as
Incr eased  catabol i sm
Bur ns
Causes  of   tr anscel l ul ar   shi f t
Insul i n  def i ci ency
Metabol i c  aci dosi s  due  to  m i ner al   aci d  r etenti on
Hy per toni ci ty   (gl ucose  or   m anni tol )
Ex er ci se
Hy per k al em i c  per i odi c  par al y si s
Di gi tal i s  i ntox i cati on
β­Adr ener gi c  antagoni sts
Causes  of   i m pai r ed  r enal   ex cr eti on
Di f f use  adr enal   i nsuf f i ci ency   (Addi son's
Sel ecti v e  m i ner al ocor ti coi d  (al doster one)  def i ci ency
Pr i m ar y   r enal   tubul ar   secr etor y   def ect
Obstr ucti v e  ur opathy
Si ck l e  cel l   di sease
Sy stem i c  l upus  er y them atosus
Renal   tr anspl antati on
Tubul oi nter sti ti al   nephr opathy
Dr ug  i nduced
Spi r onol actone
Tr i am ter ene
Am i l or i de
Inhi bi tor s  of   the  r eni n­angi otensi n  sy stem
Pentam i di ne
N onster oi dal   anti i nf l am m ator y   dr ugs
Cal ci neur i n  i nhi bi tor s
Tr i m ethopr i m
Hepar i n

P. 424
Figure  9­2  Tr eatm ent  of   hy per k al em i a.   GFR,   gl om er ul ar   f i l tr ati on  r ate;  K,
potassi um .   (Fr om   Kel l eher   CL,   Li nas  S.   The  pati ent  w i th  hy pok al em i a  or
hy per k al em i a.   In:  Schr i er   RW,   ed.   Manual   of   N ephr ol ogy ,   6th  ed.
Phi l adel phi a:  Li ppi ncott  Wi l l i am s  &  Wi l k i ns,   2005.   Repr i nted  w i th
per m i ssi on. )

Case
A  30­y ear ­ol d  w hi te  m an  has  both  di abetes  m el l i tus  and  hy per tensi on.   The
di abetes  w as  di agnosed  at  8  y ear s  of   age  w hen  k etoaci dosi s  dev el oped.   He
has  si nce  had  pr ol i f er ati v e  r eti nopathy ,   nephr opathy ,   and  per i pher al   and
autonom i c  neur opathy .   The  nephr opathy   w as  r ecogni zed  w hen  the  nephr oti c
sy ndr om e  dev el oped  3  y ear s  ago,   and  ther e  has  al so  been  a  gr adual   i ncr ease
i n  hi s  ser um   cr eati ni ne  l ev el   ov er   the  l ast  18  m onths.   Hy per tensi on  w as  f i r st
detected  a  y ear   ago.   Al though  hi s  ser um   gl ucose  l ev el s  hav e  i n  gener al   been
w el l   contr ol l ed  w i th  the  tw i ce­dai l y   adm i ni str ati on  of   i nsul i n,   bl ood  pr essur e
contr ol   has  been  subopti m al   despi te  tr eatm ent  w i th  l osar tan  and
hy dr ochl or othi azi de.
The  f ol l ow i ng  phy si cal   ex am i nati on  f i ndi ngs  ar e  noted:  supi ne  hear t  r ate  of   76
beats  per   m i nute  and  bl ood  pr essur e  of   160/110  m m   Hg;  standi ng  hear t  r ate
of   80  beats  per   m i nute  and  bl ood  pr essur e  of   130/90  m m   Hg.   Funduscopy
r ev eal s  the  pr esence  of   hem or r hages,   ex udates,   and  neov ascul ar i zati on.   Hi s
l ung  f i el ds  ar e  cl ear ,   no  car di ac  m ur m ur   i s  pr esent,   and  ther e  i s  tr ace  l ow er
ex tr em i ty   edem a,   decr eased  sensati on  to  pi npr i ck   and  v i br ati on  i n  the  di stal
l ow er   ex tr em i ti es,   and  absent  deep  tendon  r ef l ex es  i n  the  l ow er   ex tr em i ti es.
The  f ol l ow i ng  l abor ator y   v al ues  ar e  r epor ted:  sodi um ,   138  m Eq/L;  potassi um ,
7. 2  m Eq/L;  chl or i de,   110  m Eq/L;  car bon  di ox i de,   20  m Eq/L;  gl ucose,   129
m g/dL;
P. 425
cr eati ni ne,   2. 4  m g/dL;  BU N ,   30  m g/dL;  and  hem ogl obi n  A Ic ,   8. 8%.
El ectr ocar di ogr aphy   dem onstr ates  r egul ar   si nus  r hy thm   at  76  beats  per   m i nute
w i th  a  nor m al   ax i s.   The  P  w av es  ar e  f l attened,   the  QRS  com pl ex   i s  0. 12
seconds  i n  dur ati on,   and  ther e  ar e  peak ed  T  w av es  i n  the  pr ecor di al   l eads.
U r i nal y si s  r ev eal s  a  speci f i c  gr av i ty   of   1. 015,   pH  of   5. 0,   3+   pr otei n,   and
hy al i ne  casts.   A  24­hour   ur i ne  sam pl e  show s  a  cr eati ni ne  cl ear ance  (C Cr )  of   35
m L  per   m i nute  and  4. 6  g  of   pr otei n.

1.   What  do  the  el ectr ocar di ogr aphi c  f i ndi ngs  si gni f y ?  How   shoul d  the  pati ent
be  tr eated?
2.   What  ar e  the  m ost  l i k el y   f actor s  contr i buti ng  to  thi s  pati ent's
hy per k al em i a?
3.   What  ar e  the  dr ugs  that  can  cause  hy poal doster oni sm ?
4.   What  i s  the  appr opr i ate  subsequent  ther apy   f or   thi s  pati ent?

Case Discussion
1.   What  do  the  el ectr ocar di ogr aphi c  f i ndi ngs  si gni f y ?  How   shoul d  the  pati ent
be  tr eated?

The  el ectr ocar di ogr aphi c  f i ndi ngs  ar e  char acter i sti c  of   hy per k al em i a.   The
pati ent  shoul d  be  tr eated  i m m edi atel y   w i th  cal ci um   gl uconate  f ol l ow ed  by
m easur es  to  l ow er   the  ser um   potassi um ,   as  outl i ned  i n  Fi g.   9­2.

2.   What  ar e  the  m ost  l i k el y   f actor s  contr i buti ng  to  thi s  pati ent's
hy per k al em i a?

The  m ajor   contr i butor y   f actor s  r esponsi bl e  f or   the  hy per k al em i a  i n  thi s


pati ent  i ncl ude  a  decr em ent  i n  the  GFR,   the  use  of   l osar tan,   and
hy por eni nem i c  hy poal doster oni sm .   Di etar y   potassi um   ex cess  m ay   be
oper ant  as  w el l .

The  pati ent  al so  has  a  m etabol i c  aci dosi s  that  i s  pr obabl y   contr i buti ng  to
the  hy per k al em i a.   The  dev el opm ent  of   hy per k al em i a  w hen  the  r enal
i nsuf f i ci ency   i s  onl y   m oder ate  i s  l i k el y   because  other   f actor s  ar e  i nv ol v ed
i n  the  pr ocess.   The  sy ndr om e  of   hy por eni nem i c  hy poal doster oni sm   i s
com m on  i n  pati ents  w i th  di abetes,   and  the  pr esence  of   hy per chl or em i c
aci dosi s  f ur ther   suppor ts  thi s  possi bi l i ty .

3.   What  ar e  the  dr ugs  that  can  cause  hy poal doster oni sm ?


Inhi bi tor s  of   the  r eni n–angi otensi n  sy stem ,   hepar i n,   N SAIDs,   and
spi r onol actone  can  al l   pr eci pi tate  hy poal doster oni sm .   β­Adr ener gi c
bl ock er s  m ay   contr i bute  to  hy poal doster oni sm   by   i m pai r i ng  r eni n
secr eti on.   Spi r onol actone  i s  a  com peti ti v e  i nhi bi tor   of   al doster one's
cy tosol i c  r eceptor ,   w her eas  am i l or i de  i nhi bi ts  potassi um   secr eti on
thr ough  the  oper ati on  of   an  al doster one­i ndependent  m echani sm .   Cal ci um
channel   bl ock er s  hav e  not  been  r epor ted  to  i nhi bi t  al doster one  sy nthesi s,
but  spi r onol actone  i s  k now n  to  i nhi bi t  al doster one  acti on.   Tr i m ethopr i m
has  been  r epor ted  to  hav e  an  am i l or i de­l i k e  ef f ect  i n  pati ents  w i th  the
acqui r ed  i m m unodef i ci ency   sy ndr om e;  pentam i di ne  has  si m i l ar   ef f ects  i n
these  pati ents.   Cal ci neur i n  i nhi bi tor s  al so  cause  hy per k al em i a,   pr obabl y
by   an  al doster one­m edi ated  m echani sm .

4.   What  i s  the  appr opr i ate  subsequent  ther apy   f or   thi s  pati ent?

Thi s  pati ent  shoul d  r estr i ct  hi s  di etar y   potassi um   i ntak e  and  tak e  l oop
di ur eti cs  to  m anage  the  hy por eni nem i c  hy poal doster oni sm .

Hi s  l osar tan  (an  ARB)  dose  needs  to  be  decr eased.   Mi ner al ocor ti coi d
r epl acem ent  can  w or sen  the  hy per tensi on  and  sodi um   r etenti on,   and
shoul d  ther ef or e
P. 426
be  av oi ded.   Sodi um   r estr i cti on  shoul d  al so  be  av oi ded  because  i t
attenuates  the  k al i ur eti c  ef f ect  of   the  di ur eti c;  sodi um   del i v er y   i s
i m por tant  to  potassi um   ex cr eti on.

Suggested Readings
Kel l eher   CL,   Li nas  S.   The  pati ent  w i th  hy pok al em i a  or   hy per k al em i a.   In:
Schr i er   RW,   ed.   Manual   of   nephr ol ogy ,   6th  ed.   Phi l adel phi a:  Li ppi ncott
Wi l l i am s  &  Wi l k i ns,   2005:  37.

Peter son  LN ,   Lev i   M.   Di sor der s  of   potassi um   m etabol i sm .   In:  Schr i er   RW,
ed.   Renal   and  el ectr ol y te  di sor der s,   6th  ed.   Phi l adel phi a:  Li ppi ncott
Wi l l i am s  &  Wi l k i ns,   2003:  171.

Hyponatremia
1.   What  does  the  ser um   sodi um   concentr ati on  r ef l ect,   and  w hat  f actor s  can
al ter   the  w ay   i n  w hi ch  i t  i s  i nter pr eted?  In  w hat  setti ng  i s
pseudohy ponatr em i a  obser v ed?

2.   What  i s  the  under l y i ng  pathogenesi s  of   hy ponatr em i a?

3.   What  i s  the  di agnosti c  appr oach  to  hy ponatr em i a,   and  w hat  ar e  i ts  m ajor
causes?

4.   What  ar e  som e  of   the  dr ugs  that  pr oduce  hy ponatr em i a?


5.   What  ar e  the  m ost  com m on  di sor der s  associ ated  w i th  the  sy ndr om e  of
i nappr opr i ate  anti di ur eti c  hor m one  secr eti on  (SIADH)?

Discussion
1.   What  does  the  ser um   sodi um   concentr ati on  r ef l ect,   and  w hat  f actor s  can
al ter   the  w ay   i n  w hi ch  i t  i s  i nter pr eted?  In  w hat  setti ng  i s
pseudohy ponatr em i a  obser v ed?

Hy ponatr em i a  r epr esents  a  decr ease  i n  the  concentr ati on  of   sodi um
r el ati v e  to  that  of   w ater   i n  the  ser um .   Total ­body   sodi um   content  m ay   be
decr eased,   unchanged,   or   ev en  i ncr eased.   The  ser um   sodi um
concentr ati on  i s  a  m easur e  of   the  toni ci ty   of   body   f l ui ds,   and  i t  i s  the
m ajor   contr i butor   to  the  ser um   osm ol al i ty ,   as  show n  by   the  equati on:
P osm   =   2  ×  P N a   +   (gl ucose/18)  +   (ur ea/2. 8),   w her e  P osm   i s  the  ser um
osm ol al i ty   and  P N a   i s  the  ser um   sodi um   concentr ati on.

Hy per gl y cem i a  can  cause  a  decr em ent  i n  the  ser um   sodi um   l ev el   by


shi f ti ng  i ntr acel l ul ar   w ater   out  of   cel l s.   Because  gl ucose  i s  not  f r eel y
m ov abl e  acr oss  cel l   m em br anes,   w hen  the  ex tr acel l ul ar   gl ucose
concentr ati on  i s  el ev ated  i n  i nsul i n­def i ci ent  or   ­r esi stant  pati ents,   w ater
m ov es  out  of   the  cel l s  to  equal i ze  osm ol al i ty   on  both  si des  of   the
m em br ane.   The  m ov em ent  of   w ater   di l utes  the  ser um   sodi um
concentr ati on,   but  the  ser um   osm ol al i ty   i s  m ai ntai ned.   Cl i ni cal l y ,
hy per gl y cem i a­i nduced  hy ponatr em i a  i s  f r equentl y   encounter ed  i n  the
setti ngs  of   di abeti c  k etoaci dosi s  and  nonk etoti c  hy per osm ol ar   com a.   To
deter m i ne  w hether   a  pati ent  has  a  sodi um   or   w ater   def i ci t,   the  ser um
sodi um   l ev el   shoul d
P. 427
be  esti m ated  as  i f   the  pati ent  w er e  nor m ogl y cem i c.   The  cor r ecti on  f actor
i s  as  f ol l ow s:  f or   each  100­m g/dL  i ncr ease  i n  the  ser um   gl ucose  l ev el ,
the  ser um   sodi um   concentr ati on  decr eases  by   1. 6  m Eq/L.   For   ex am pl e,   i f
the  sodi um   concentr ati on  i s  109  m Eq/L  and  the  ser um   gl ucose  content  i s
1, 600  m g/dL,   the  cor r ected  sodi um   concentr ati on  (N a c )  w oul d  be
cal cul ated  as  f ol l ow s:

Ther ef or e,   the  ser um   sodi um   concentr ati on  al w ay s  needs  to  be


i nter pr eted  i n  l i ght  of   the  gl ucose  concentr ati on.   Ev ents  i denti cal   to  these
occur   w i th  ex ogenous  m anni tol   adm i ni str ati on.

In  pseudohy ponatr em i a,   the  ser um   sodi um   concentr ati on  i s  l ow   but  the
ser um   osm ol al i ty   i s  nor m al .   It  occur s  i n  setti ngs  of   sev er e  hy per l i pi dem i a
and  hy per pr otei nem i a,   and  i s  r ar e.   The  m echani sm   r esponsi bl e  f or   the
l ow   ser um   sodi um   concentr ati on  caused  by   hy per l i pi dem i a  and
hy per pr otei nem i a  di f f er s  f r om   that  of   hy per gl y cem i a.   At  ex tr em el y
el ev ated  concentr ati ons,   both  l i pi d  and  pr otei n  cause  the  sodi um
di str i buti on  space  (i . e. ,   pl asm a  w ater   space)  to  be  decr eased.   Al though
the  sodi um   concentr ati on  i n  pl asm a  w ater   i s  nor m al ,   i t  i s  decr eased  i n
the  total   pl asm a  because  of   ex cess  l i pi d  or   pr otei n.

2.   What  i s  the  under l y i ng  pathogenesi s  of   hy ponatr em i a?

Hy ponatr em i a  ar i ses  w hen  ur i nar y   di l uti on  i s  abnor m al .   The  abi l i ty   to


ex cr ete  a  l ar ge  v ol um e  of   sol ute­f r ee  w ater   depends  on  thr ee  f actor s:  (a)
nor m al   f l ui d  del i v er y   to  the  di stal   nephr on  (i . e. ,   nor m al   GFR  and  nor m al
pr ox i m al   tubul e  r eabsor pti on);  (b)  nor m al   f uncti oni ng  of   the  thi ck
ascendi ng  l i m b  of   Henl e  and  the  cor ti cal   di l uti ng  segm ents,   w hi ch  ar e
si tes  of   ur i nar y   di l uti on;  and  (c)  the  absence  of   v asopr essi n  i n  the
ci r cul ati on,   ther eby   al l ow i ng  the  col l ecti ng  duct  to  r em ai n  w ater
i m per m eabl e.   In  the  pr esence  of   v asopr essi n,   the  tubul ar   f l ui d
equi l i br ates  osm oti cal l y   w i th  the  i sotoni c  or   hy per toni c  ur i ne,   ther eby
pr ev enti ng  the  ex cr eti on  of   m ax i m al l y   di l ute  ur i ne.

3.   What  i s  the  di agnosti c  appr oach  to  hy ponatr em i a,   and  w hat  ar e  i ts  m ajor
causes?

Once  hy ponatr em i a  i s  conf i r m ed,   the  nex t  step  i s  to  deter m i ne  w hether   i t
i s  associ ated  w i th  a  l ow ,   nor m al ,   or   hi gh  total ­body   sodi um
concentr ati on.   U sual l y ,   a  phy si cal   ex am i nati on  can  di sti ngui sh  am ong
these  possi bi l i ti es.   Or thostati c  hy potensi on  and  f l at  neck   v ei ns  ar e  seen
i n  pati ents  w i th  a  l ow   total ­body   sodi um   content.   Edem a  and  asci tes  ar e
com m on  f i ndi ngs  i n  pati ents  w i th  a  hi gh  total ­body   sodi um   content.
Pati ents  w i th  nor m al   total ­body   sodi um   ex hi bi t  nei ther   or thostati c
changes  nor   edem a.   The  m ajor   causes  of   each  categor y   of   sodi um
concentr ati on  ar e  sum m ar i zed  i n  Tabl e  9­21.

4.   What  ar e  som e  of   the  dr ugs  that  pr oduce  hy ponatr em i a?

Dr ugs  can  i m pai r   w ater   ex cr eti on  ei ther   by   enhanci ng  the  r enal   acti on  of
v asopr essi n  or   by   causi ng  r el ease  of   the  hor m one.   Som e  of   the  m or e
com m on  agents  ar e  l i sted  i n  Tabl e  9­22.

P. 428
5.   What  ar e  the  m ost  com m on  di sor der s  associ ated  w i th  SIADH?

In  hospi tal i zed  pati ents,   SIADH  i s  the  m ost  com m on  cause  of
hy ponatr em i a.   Thi s  i s  br oadl y   due  to  a  m al i gnancy ,   pul m onar y   di sor der ,
or   centr al   ner v ous  sy stem   di sor der ,   as  show n  i n  Tabl e  9­23.

Table 9­21 Causes of Hyponatremia

Hypovole mia  (De c re a s e d Euvole mia  (Ne a r­ Hype rvole mia


Tota l­Body Sodium) Norma l Tota l­Body (Inc re a s e d Tota l­
Sodium) Body Sodium)
Ex tr ar enal   sodi um Di ur eti cs Ex tr ar enal
l osses Hy pothy r oi di sm di sor der s
Vom i ti ng Gl ucocor ti coi d Congesti v e
(steady   state) def i ci ency hear t
Di ar r hea Dr ugs f ai l ur e
Fl ui d Pai n  or Hepati c
sequestr ati on em oti onal ci r r hosi s
in str ess Renal   di sor der s
Per i toni ti s Respi r ator y N ephr oti c
Pancr eati ti s f ai l ur e sy ndr om e
Rhabdom y ol y si s Posi ti v e­ Acute
Bur ns pr essur e r enal
Renal   sodi um   l osses br eathi ng f ai l ur e
Di ur eti cs Sy ndr om e  of Chr oni c
Osm oti c  di ur esi s i nappr opr i ate r enal
(gl ucose,   ur ea, anti di ur eti c f ai l ur e
m anni tol ) hor m one
Mi ner al ocor ti coi d secr eti on
def i ci ency
Sal t­l osi ng  nephr i ti s

Case
A  68­y ear ­ol d  m an  i s  hospi tal i zed  because  of   a  per si stent  cough  and  20­l b  (9­
k g)  w ei ght  l oss  dur i ng  the  l ast  3  m onths.   He  has  a  40­pack ­y ear   sm ok i ng
hi stor y .   On  phy si cal   ex am i nati on,   he  i s  f ound  to  be  sl i ghtl y   conf used  and  sl ow
to  r espond.   Ther e  ar e  no  or thostati c  changes  i n  hi s  bl ood  pr essur e  or   pul se.
Chest  ex am i nati on  r ev eal s  f i ndi ngs  com pati bl e  w i th  a  l ef t  pl eur al   ef f usi on.
Abdom i nal   ex am i nati on  r ev eal s  no  m asses  or   or ganom egal y .   Ther e  i s  no
edem a.   He  w ei ghs  60  k g.
The  f ol l ow i ng  l abor ator y   v al ues  ar e  r epor ted:  sodi um ,   109  m Eq/L;  potassi um ,
3. 4  m Eq/L;  chl or i de,   78  m Eq/L;  bi car bonate,   24  m Eq/L;  BU N ,   4  m g/dL;
gl ucose,   85  m g/dL;  ur i c  aci d,   3. 5  m g/dL;  ser um   osm ol al i ty ,   230  m Osm ;  and
ur i ne  osm ol al i ty ,   300  m Osm .   A  chest  r adi ogr aphi c  study   show s  a  l ef t  pl eur al
ef f usi on.   Pur i f i ed  pr otei n  der i v ati v e  (PPD)  testi ng  i s  posi ti v e.
The  pati ent's  ser um   sodi um   concentr ati on  i ncr eases  to  133  m Eq/L  w i thi n  24
hour s.   At  that  ti m e,   the  pati ent  i s  noted  to  be  al er t  and  hi s  behav i or
appr opr i ate.   How ev er ,   by   the  nex t  day ,   he  has  becom e  uncom m uni cati v e  and
agi tated.

1.   What  ar e  the  m ost  l i k el y   causes  of   hy ponatr em i a  i n  thi s  pati ent,   and


w hy ?
2.   How   do  the  ser um   potassi um ,   BU N ,   and  ur i c  aci d  l ev el s  hel p  i n  the
assessm ent  of   thi s  pati ent?
P. 429
3.   What  ar e  the  pr i m ar y   consi der ati ons  i n  tr eati ng  pati ents  w i th
hy ponatr em i a,   and  how   shoul d  thi s  pati ent's  condi ti on  be  m anaged?

Table 9­22 Drugs Associated with
Hyponatremia

Anti di ur eti c  hor m one  anal ogs


Deam i no­d­ar gi ni ne  v asopr essi n
Ox y toci n
Dr ugs  that  enhance  anti di ur eti c  hor m one  r el ease
Chl or pr opam i de
Cl of i br ate
Car bam azepi ne­ox car bazepi ne
Vi ncr i sti ne
N i coti ne
N ar coti cs  (µ­opi oi d  r eceptor s)
Anti psy choti cs  or   anti depr essants a
Dr ugs  that  potenti ate  r enal   acti on  of   anti di ur eti c  hor m one
Chl or pr opam i de
Cy cl ophospham i de
N onster oi dal   anti i nf l am m ator y   dr ugs
Acetam i nophen
Dr ugs  that  cause  hy ponatr em i a  by   unk now n  m echani sm s
Hal oper i dol
Fl uphenazi ne
Am i tr i pty l i ne
Ser otoni n  uptak e  i nhi bi tor s
“Ecstacy â€​   (am phetam i ne  r el ated)

a Anti di ur eti c  hor m one  r el ease  m ay   be  secondar y   to  under l y i ng

psy chosi s.

Fr om   Ber l   T,   Schr i er   RW.   Di sor der s  of   w ater   m etabol i sm .   In:


Schr i er   RW,   ed.   Renal   and  el ectr ol y te  di sor der s,   6th  ed.
Phi l adel phi a:  Li ppi ncott  Wi l l i am s  &  Wi l k i ns,   2003:45.   Repr i nted
w i th  per m i ssi on.

4.   What  coul d  account  f or   thi s  pati ent's  neur ol ogi c  deter i or ati on  af ter   hi s
i ni ti al   i m pr ov em ent?

Case Discussion
1.   What  ar e  the  m ost  l i k el y   causes  of   hy ponatr em i a  i n  thi s  pati ent,   and
w hy ?

Thi s  pati ent  appear s  to  hav e  hy ponatr em i a  associ ated  w i th  a  nor m al
total ­body   sodi um   concentr ati on  because  ther e  ar e  nei ther   or thostati c
changes  nor   edem a.   He  ther ef or e  has  euv ol em i c  hy ponatr em i a.   Pi tui tar y
i nsuf f i ci ency   appear s  cl i ni cal l y   unl i k el y   and  no  w ater ­r etai ni ng
m edi cati ons  ar e  pr esent,   ther eby   m ak i ng  SIADH  the  m ost  l i k el y   cause  of
the  hy ponatr em i a.   The  tw o  l eadi ng  di agnoses  ar e  l ung  cancer   or
pul m onar y   tuber cul osi s.   In  SIADH,   a  pati ent  i s  sl i ghtl y   v ol um e  ex panded.
Ther ef or e,   as  i n  thi s  pati ent,   the  BU N   and  ur i c  aci d  l ev el s  tend  to  be  l ow .
Fr om   the  cl i ni cal
P. 430
poi nt  of   v i ew ,   SIADH  i s  the  m ost  l i k el y   di agnosi s  i n  thi s  pati ent,   but
hy pothy r oi di sm   shoul d  al so  be  consi der ed.

Table 9­23 The Most Common Disorders
Associated with the Syndrome of
Inappropriate Secretion of Antidiuretic
Hormone

Mal i gnancy
Lung
Duodenum
Pancr eas
Ly m phom a
Pul m onar y   di sor der s
Pneum oni a
Abscess
Asper gi l l osi s
Respi r ator y   f ai l ur e
Posi ti v e­pr essur e  br eathi ng
Centr al   ner v ous  sy stem   di sor der s
N eopl asm
Encephal i ti s
Meni ngi ti s
Br ai n  abscess
Head  tr aum a
Gui l l ai n­Bar r é  sy ndr om e
Subdur al   or   subar achnoi d  hem or r hage
Acute  i nter m i ttent  por phy r i a
Acute  psy chosi s
Str ok e
Other
AIDS
Pr ol onged  ex er ci se
Idi opathi c  (el der l y )
AIDS,   acqui r ed  i m m unodef i ci ency   sy ndr om e.

Modi f i ed  f r om   Ber l   T,   Schr i er   RW.   Di sor der s  of   w ater


m etabol i sm .   In:  Schr i er   RW,   ed.   Renal   and  el ectr ol y te  di sor der s,
6th  ed.   Phi l adel phi a:  Li ppi ncott  Wi l l i am s  &  Wi l k i ns,   2003:47.
Repr i nted  w i th  per m i ssi on.

2.   How   do  the  ser um   potassi um ,   BU N ,   and  ur i c  aci d  l ev el s  hel p  i n  the


assessm ent  of   thi s  pati ent?

The  ser um   potassi um   concentr ati on  of   3. 4  m Eq/L  and  the  BU N   v al ue  of   5
m g/dL  v i r tual l y   r ul e  out  adr enal   i nsuf f i ci ency   because  thi s  i s
char acter i zed  by   a  hy per k al em i c  aci dosi s  and  an  el ev ati on  i n  the  BU N   and
ser um   cr eati ni ne  l ev el s  as  a  consequence  of   v ol um e  contr acti on.   Al though
the  l ow   ser um   potassi um   concentr ati on  br i ngs  i nto  questi on  the  use  of
di ur eti cs,   the  l ow   ur i c  aci d  l ev el   m ak es  thi s  unl i k el y .   A  l ow   ur i c  aci d
l ev el   i s  com m onl y   obser v ed  i n  the  setti ng  of   SIADH.

P. 431
3.   What  ar e  the  pr i m ar y   consi der ati ons  i n  tr eati ng  pati ents  w i th
hy ponatr em i a,   and  how   shoul d  thi s  pati ent's  condi ti on  be  m anaged?

The  opti m al   tr eatm ent  f or   sev er e  hy ponatr em i a  i s  sti l l   contr ov er si al


because  al though  pr of ound  hy ponatr em i a  i s  associ ated  w i th  hi gh  m or tal i ty
and  m or bi di ty ,   i ts  r api d  cor r ecti on  m ay   cause  the  f or m ati on  of   neur ol ogi c
l esi ons,   w hi ch  ar e  usual l y   i r r ev er si bl e.   The  pr i m ar y   consi der ati ons  i n  the
ther apy   ar e  the  acuteness  or   chr oni ci ty   of   the  pr ocess  and  the  pr esence
or   absence  of   neur ol ogi c  sy m ptom s  attr i butabl e  to  hy ponatr em i a.   The
f ol l ow i ng  ar e  gener al   tr eatm ent  gui del i nes.

In  the  setti ng  of   a c ute  s ymptoma tic  hypona tre mia   w i th  a  change  i n
m ental   status  or   sei zur es,   the  r i sk   f or   com pl i cati ons  stem m i ng  f r om
cer ebr al   edem a  ex ceeds  the  r i sk   of   com pl i cati ons  due  to  r api d  tr eatm ent.
The  pati ent  shoul d  r ecei v e  f ur osem i de  and  hy per toni c  sal i ne  unti l
conv ul si ons  subsi de.

As ymptoma tic  hypona tre mia   i s  al m ost  al w ay s  chr oni c,   and  r api d
cor r ecti on  i s  l i k el y   to  do  m or e  har m   than  good.   The  tr eatm ent  i n  these
pati ents  shoul d  consi st  of   w ater   r estr i cti on  r egar dl ess  of   thei r   ser um
sodi um   status.

In  the  setti ng  of   s ymptoma tic  hypona tre mia   of   chr oni c  or   unk now n
dur ati on,   the  ser um   sodi um   l ev el   shoul d  be  r ai sed  pr om ptl y   by
appr ox i m atel y   10  m Eq/L  thr ough  the  adm i ni str ati on  of   hy per toni c  sal i ne,
and  then  w ater   r estr i cti on.   A  cor r ecti on  r ate  of   1  to  2  m Eq/L  per   hour   at
any   gi v en  ti m e  or   an  i ncr ease  i n  the  ser um   sodi um   l ev el   by   m or e  than  12
m Eq  per   day   shoul d  not  be  ex ceeded.

In  the  pr esent  case,   because  the  pati ent  i s  sy m ptom ati c,   i t  i s  pr udent  to
cor r ect  the  ser um   sodi um   l ev el   to  appr ox i m atel y   120  m Eq/L  i n  8  to  12
hour s.   The  sol ute­f r ee  w ater   l oss  needed  to  accom pl i sh  thi s  m ay   be
esti m ated  by   m ul ti pl y i ng  total ­body   w ater   ×  (1  ­  actual   ser um
sodi um /desi r ed  ser um   sodi um ).   Ther ef or e,   to  cor r ect  the  ser um   sodi um   i n
thi s  60­k g  m an  f r om   109  to  120  m Eq/L,   he  m ust  hav e  a  negati v e  w ater
bal ance  of   60  ×  0. 6  ×  (1  ­  109/120)  =   3. 3  L.   Thi s  m ay   be
accom pl i shed  by   i nf usi ng  nor m al   sal i ne  at  a  r ate  of   250  m L  per   hour
w hi l e  r epl aci ng  ur i nar y   sodi um   l osses  w i th  3%  sal i ne  so  as  to  achi ev e  a
net  sol ute­f r ee  w ater   l oss.   A  si ngl e  i njecti on  of   f ur osem i de  (20  m g  IV)
m ay   be  adm i ni ster ed  to  pr om ote  di ur esi s;  ur i nar y   potassi um   l osses
shoul d  be  r epl aced.   The  ser um   sodi um   concentr ati on  m ay   be  r ai sed  by
1. 0  to  1. 5  m Eq/L  per   hour .   Once  the  ser um   sodi um   l ev el   has  i ncr eased  by
appr ox i m atel y   10  m Eq/L,   thi s  r egi m en  shoul d  be  di sconti nued.

As  f or   the  l ong­ter m   m anagem ent  of   thi s  pati ent,   w ater   r estr i cti on  to
1, 000  m L  per   day   i s  the  tr eatm ent  of   choi ce.   How ev er ,   because
com pl i ance  m ay   be  di f f i cul t  to  achi ev e,   dem ecl ocy cl i ne  can  be  gi v en.   Thi s
dr ug  i nter f er es  w i th  the  anti di ur eti c  hor m one  ef f ect  on  the  k i dney   and
r esul ts  i n  m or e  di l ute  ur i ne.   If   the  pati ent's  pr i m ar y   di sease,   l ung  cancer ,
or   tuber cul osi s  r esponds  to  tr eatm ent,   thi s  w oul d  l i k el y   pr om ote
r esol uti on  of   the  SIADH.   N ov el   v asopr essi n  antagoni sts  that  hav e
aquar eti c  pr oper ti es  ar e  l i k el y   to  be  pr ef er abl e  to  dem ecl ocy cl i ne.

4.   What  coul d  account  f or   thi s  pati ent's  neur ol ogi c  deter i or ati on  af ter   hi s
i ni ti al   i m pr ov em ent?

Thi s  pati ent's  ser um   sodi um   l ev el   i ncr eased  by   24  m Eq/L  i n  the  f i r st  24
hour s.   Thi s,   ther ef or e,   puts  hi m   at  r i sk   f or   dev el opm ent  of   osm oti c
dem y el i nati on  (OD)  that  i s  char acter i zed  by   a  f l acci d  quadr i par esi s,
i m pai r ed  speech  and  sw al l ow i ng,
P. 432
f aci al   w eak ness,   and  poor   r esponse  to  pai nf ul   sti m ul i .   Pathol ogi cal l y ,   l oss
of   m y el i n  ar ound  ner v e  sheaths  can  be  seen  i n  ponti ne  as  w el l   as
ex tr aponti ne  ar eas.   The  pathogenesi s  of   thi s  l esi on  r em ai ns  unk now n.
Ther e  ar e  sev er al   r i sk   f actor s  f or   the  dev el opm ent  of   OD,   i ncl udi ng
al cohol i sm ,   m al nutr i ti on,   and  bur ns,   and  i t  i s  al so  seen  i n  w om en  tak i ng
thi azi de  di ur eti cs.   The  r esul ts  of   hum an  and  ani m al   studi es  suggest  that
r api d  cor r ecti on  of   sev er e  chr oni c  hy ponatr em i a  m ay   be  associ ated  w i th
OD,   w her eas  the  hy ponatr em i a  i tsel f   i s  unr el ated.

The  f i ndi ngs  f r om   studi es  of   osm oti cal l y   i nacti v e  sol utes  (such  as  am i no
aci ds,   m y oi nosi tol ,   sor bi tol ,   and  m ethy l am i ne)  m ay   hav e  i m pl i cati ons  f or
the  pathogenesi s  of   OD.   The  i ntr acel l ul ar   l ev el s  of   these  sol utes  decr ease
sl ow l y   dur i ng  the  adaptati on  to  changes  i n  ex tr acel l ul ar   osm ol al i ty   so  that
the  cel l   v ol um e  i s  m ai ntai ned.   Ther ef or e,   i n  the  setti ng  of   chr oni c
hy ponatr em i a,   the  r api d  i ncr ease  i n  ex tr acel l ul ar   osm ol al i ty   m ay   shr i nk
br ai n  cel l s,   w hi ch  hav e  di m i ni shed  osm oti cal l y   acti v e  sol utes  as  a
consequence  of   adaptati on  to  the  chr oni c  hy ponatr em i a.

Suggested Readings
Ber l   T,   Schr i er   RW.   Di sor der s  of   w ater   m etabol i sm .   In:  Schr i er   RW,   ed.
Renal   and  el ectr ol y te  di sor der s,   6th  ed.   Phi l adel phi a:  Li ppi ncott  Wi l l i am s  &
Wi l k i ns,   2003:1.

Par i k h  C,   Kum ar   S,   Ber l   T.   Di sor der s  of   w ater   bal ance.   In:  Johnson  R,
Feehal l y   J,   eds.   Com pr ehensi v e  cl i ni cal   nephr ol ogy ,   2nd  ed.   Mosby ,   2003.
Editors :  Sc hrie r,  Robe rt W .
Title :  Inte rna l Me dic ine  Ca s e book ,  The : Re a l P a tie nts ,  Re a l Ans w e rs ,
3rd Edition
Copy r i ght  ©2007  Li ppi ncott  Wi l l i am s  &  Wi l k i ns

>  T a b le   o f   C o nte nts   >  C ha p te r   10  ­  R he um a to lo g y

Chapter 10
Rheumatology

Robe rt W .  Ja ns on

Ankylosing Spondylitis
1.   What  ar e  thr ee  possi bl e  causes  of   l ow   back   pai n  (LBP)  i n  y oung  m en?

2.   What  f eatur es  i n  the  hi stor y   and  phy si cal   ex am i nati on  ar e  hel pf ul   i n
di f f er enti ati ng  i nf l am m ator y   LBP  i n  ank y l osi ng  spondy l i ti s  (AS)  f r om
m echani cal   LBP?

3.   What  f i v e  di seases  ar e  cl assi f i ed  as  ser onegati v e


spondy l oar thr opathi es?

4.   What  i s  the  def i ni ti on  of   sci ati ca  and  w hat  ar e  thr ee  possi bl e  causes  of
i t?

P. 434

Discussion
1.   What  ar e  thr ee  possi bl e  causes  of   LBP  i n  y oung  m en?

Thr ee  possi bl e  causes  of   back   pai n  i n  y oung  m en  i ncl ude  l um bosacr al
m uscl e  spasm ,   a  r uptur ed  i nter v er tebr al   di sc,   and  AS  or   another
ser onegati v e  spondy l oar thr opathy .   For m s  of   com m on  autoi m m une  and
chr oni c  i nf l am m ator y   di seases,   such  as  r heum atoi d  ar thr i ti s  (RA)  or
sy stem i c  l upus  er y them atosus  (SLE),   r ar el y   i nv ol v e  the  joi nts  of   the
l ow   back .   Ther ef or e,   LBP  i s  not  one  of   the  i ni ti al   sy m ptom s  of   these
di sor der s.

2.   What  f eatur es  i n  the  hi stor y   and  phy si cal   ex am i nati on  ar e  hel pf ul   i n
di f f er enti ati ng  i nf l am m ator y   LBP  i n  AS  f r om   m echani cal   LBP?

  Infla mma tory LBP Me c ha nic a l LBP

Age  at  onset < 40  y Any   age


Ty pe  of   onset Insi di ous Acute

Sy m ptom   dur ati on > 3  m o < 4  w k

Mor ni ng  sti f f ness > 60  m i n < 30  m i n

N octur nal   pai n Fr equent Absent

Ef f ect  of   ex er ci se Im pr ov em ent Ex acer bati on

Sacr oi l i ac  joi nt  tender ness Fr equent Absent

Back   m obi l i ty Loss  i n  al l   pl anes Abnor m al   f l ex i on

Chest  ex pansi on Of ten  decr eased N or m al

N eur ol ogi c  def i ci ts U nusual Possi bl e

LBP,   l ow   back   pai n.

3.   What  f i v e  di seases  ar e  cl assi f i ed  as  ser onegati v e


spondy l oar thr opathi es?

The  spondy l oar thr opathi es  consi st  of   AS,   r eacti v e  ar thr i ti s  (f or m er l y
k now n  as  Rei ter 's  sy ndr om e),   psor i ati c  ar thr i ti s,   ar thr i ti s  secondar y   to
i nf l am m ator y   bow el   di sease,   and  undi f f er enti ated  spondy l oar thr opathy .

4.   What  i s  the  def i ni ti on  of   sci ati ca,   and  w hat  ar e  thr ee  possi bl e  causes  of
i t?

Sci ati ca  i s  def i ned  as  back   pai n  that  r adi ates  l ater al l y   dow n  one  l eg
bel ow   the  k nee.   The  pai n  i s  usual l y   shar p  or   bur ni ng.   Sci ati ca  usual l y
occur s  as  a  consequence  of   l um bar   spondy l osi s  (degener ati v e  di sc  or
f acet  joi nt  di sease)  and  can  be  associ ated  w i th  a  r uptur ed
i nter v er tebr al   di sc  or   an  i di opathi c  sci ati c  ner v e  i r r i tati on.   Inf ecti ous,
neopl asti c,   and  i nf i l tr ati v e  di sor der s  shoul d  al w ay s  be  consi der ed.

Case
A  34­y ear ­ol d  w hi te  m an  com pl ai ns  of   neck   pai n.   At  the  age  of   22,   the
pati ent  f i r st  noted  l ow   back ,   buttock ,   and  spi ne  pai n.   He  had  been  i nv ol v ed
i n  a  m otor   v ehi cl e  acci dent  to  w hi ch  he  attr i buted  som e  of   hi s  back   pai n.   At
that  ti m e,   he  saw   a  num ber   of   phy si ci ans  w ho  di agnosed  m echani cal   LBP
and  r ecom m ended  bed  r est.   How ev er ,   he  f ound  thi s  onl y   seem ed  to  m ak e
hi s  back   and  buttock   pai n  w or se.   Ty pi cal l y ,   he  w as  v er y   sti f f   i n  the
P. 435
m or ni ng  f or   m or e  than  2  hour s  but  i n  the  af ter noon  he  f el t  better   w i th
m ov em ent  and  ex er ci se.   He  al so  noted  i ncr easi ng  f ati gue  and  som e  m i l d
w ei ght  l oss.   Ten  y ear s  ago,   hi s  r i ght  hi p  star ted  hur ti ng.   Ei ght  y ear s  ago,
pai n  suddenl y   dev el oped  i n  hi s  r i ght  ey e.   He  saw   an  ophthal m ol ogi st  w ho
di agnosed  acute  i r i ti s  and  pl aced  hi m   on  ster oi d  ey e  dr ops.   Tw o  y ear s  ago,
hi s  k nees  star ted  to  sw el l   i nter m i ttentl y .   Hi s  l um bar   and  thor aci c  spi ne
r egi ons  becam e  f used  and  to  stand  up  and  l ook   str ai ght  ahead  he  had  to
bend  hi s  k nees.   He  f i nal l y   had  to  qui t  hi s  job  as  a  tr uck   dr i v er   because  i t
r equi r ed  pr ol onged  si tti ng  that  m ade  hi s  back   pai n  and  sti f f ness  w or se.
Muscul osk el etal   ex am i nati on  r ev eal s  no  obv i ous  sw el l i ng  i n  any   joi nt.   N o
m ov em ent  i n  the  l um bar   or   thor aci c  spi ne  i s  noted  w hi l e  the  pati ent  i s
bendi ng  ov er .   Hi s  r i ght  hi p  i s  f ound  to  be  pai nf ul   on  f l ex i on  w i th  i nter nal
r otati on.
Radi ogr aphi c  studi es  of   the  l um bosacr al   spi ne  ar e  obtai ned  and  i nter pr eted
to  show   al m ost  com pl ete  obl i ter ati on  of   both  sacr oi l i ac  joi nt  spaces.   The
poster i or   el em ents  i n  the  di stal   l um bar   ar ea  ar e  al so  f ound  to  be
obl i ter ated,   together   w i th  br i dgi ng  or   “bam booi ngâ€​   of   the  spi ne.   A  chest
r adi ogr aphi c  study   show s  squar i ng  of   the  thor aci c  v er tebr ae  w i th  si gni f i cant
sy ndesm ophy te  f or m ati on.

1.   Wher e  i s  the  pr i m ar y   si te  of   di sease  i n  AS?


2.   What  or gans  can  be  i nv ol v ed  i n  AS,   and  w hat  ar e  the  cl i ni cal
m ani f estati ons?
3.   What  ar e  thr ee  char acter i sti c  cl i ni cal   f i ndi ngs  i n  pati ents  w i th  AS  that
hel p  di sti ngui sh  i t  f r om   RA?
4.   What  i s  the  char acter i sti c  f am i l y   hi stor y ,   gender   i nci dence,   and  hum an
l y m phocy te  anti gen  (HLA)  patter n  f ound  i n  the  contex t  of   AS?
5.   What  ty pes  of   tr eatm ent  ar e  hel pf ul   i n  AS?

Case Discussion
1.   Wher e  i s  the  pr i m ar y   si te  of   di sease  i n  AS?

In  AS,   i nf l am m ati on  occur s  at  the  i nser ti on  of   a  l i gam ent,   tendon,   or
ar ti cul ar   capsul e  i nto  bone,   a  str uctur e  k now n  as  the  enthesi s.   The
cause  of   thi s  l ocal i zed  i nf l am m ati on  r em ai ns  unk now n.   Si tes  of
enthesopathy   i n  AS  i ncl ude  the  sacr oi l i ac  joi nts;  l i gam entous  str uctur es
of   the  i nter v er tebr al   di scs,   m anubr i oster nal   joi nts,   and  sy m phy si s
pubi s;  l i gam entous  attachm ents  i n  the  spi nous  pr ocesses,   the  i l i ac
cr ests  (w hi sk er i ng),   tr ochanter s,   patel l ae,   cl av i cl es,   and  cal canei
(Achi l l es  enthesi ti s  or   pl antar   f asci i ti s);  and  capsul es  and  i ntr acapsul ar
l i gam ents  of   l ar ge  sy nov i al   joi nts.   Inf l am m ati on  can  al so  be  seen  i n  the
sy nov i um ,   the  ti ssue  l i ni ng  the  joi nts.

2.   What  or gans  can  be  i nv ol v ed  i n  AS,   and  w hat  ar e  the  cl i ni cal
m ani f estati ons?

Ocul ar   i nv ol v em ent  pr esents  as  anter i or   uv ei ti s  (25%  to  40%  of


pati ents);  secondar y   gl aucom a  and  catar acts  can  al so  occur .   Car di ac
i nv ol v em ent  i ncl udes  aor ti c  i nsuf f i ci ency ,   aor ti ti s,   conducti on
abnor m al i ti es,   di astol i c  dy sf uncti on,   and  per i car di ti s.   Pul m onar y
i nv ol v em ent  i ncl udes  upper   l obe  f i br osi s  and  r estr i cti v e  changes.   Renal
i nv ol v em ent  i ncl udes  IgA  nephr opathy ,   secondar y   am y l oi dosi s,   and
chr oni c  pr ostati ti s.   Per i pher al   joi nt  i nv ol v em ent  (par ti cul ar l y   hi ps  and
shoul der s)  can  occur   i n  appr ox i m atel y   30%  of   pati ents.   Si gni f i cant
spi nal   osteopor osi s  can  occur .   N eur ol ogi c  i nv ol v em ent  i ncl udes
atl antoax i al   subl ux ati ons  and  cauda  equi na  sy ndr om e.

P. 436
3.   What  ar e  thr ee  char acter i sti c  cl i ni cal   f i ndi ngs  i n  pati ents  w i th  AS  that
hel p  di sti ngui sh  i t  f r om   RA?

The  thr ee  cl i ni cal   m ani f estati ons  char acter i sti c  of   AS  ar e  i nf l am m ator y
ar thr i ti s  of   the  spi ne,   Achi l l es  tendi ni ti s,   and  pl antar   f asci i ti s.   These
thr ee  f i ndi ngs  ar e  ex tr em el y   r ar e  i n  pati ents  w i th  RA.

4.   What  i s  the  char acter i sti c  f am i l y   hi stor y ,   gender   i nci dence,   and  HLA
patter n  f ound  i n  the  contex t  of   AS?

Ty pi cal l y ,   ther e  i s  a  f am i l y   hi stor y   of   AS,   par ti cul ar l y   i n  m al e  f am i l y


m em ber s.   In  f act,   i t  occur s  m or e  com m onl y   i n  m en  than  w om en  (3:1).
Thi s  di sease  i s  v er y   hi ghl y   associ ated  w i th  the  pr esence  of   HLA­B27.
Tw o  per cent  of   HLA­B27–posi ti v e  per sons  dev el op  AS.   Am ong  those
HLA­B27–posi ti v e  per sons  w i th  an  af f ected  f i r st­degr ee  r el ati v e,   the
r ate  r i ses  to  15%  to  20%.

5.   What  ty pes  of   tr eatm ent  ar e  hel pf ul   i n  AS?

The  tr eatm ent  of   AS  i ncl udes  nonster oi dal   anti i nf l am m ator y   dr ugs
(N SAIDs),   ex tensi on  ex er ci ses  f or   the  back ,   and  phy si cal   ther apy .   It  i s
r ecom m ended  that  al l   thr ee  f or m s  of   ther apy   be  used  i n  af f ected
pati ents.   It  i s  thought  that  ex tensi on  ex er ci ses  f or   the  back   m ay   hel p
pati ents  m ai ntai n  a  m or e  nor m al   upr i ght  postur e  as  the  back   f uses  ov er
ti m e.   Sul f asal azi ne  or   l ow ­dose  w eek l y   m ethotr ex ate  (MTX)  ther apy
m ay   be  benef i ci al   i n  pati ents  hav i ng  pr ogr essi v e  di sease  w i th
per i pher al   ar thr i ti s  but  does  not  al ter   the  sacr oi l i i ti s.   Or al
cor ti coster oi ds  ar e  of   no  v al ue.   Local   cor ti coster oi d  i njecti ons  m ay   be
usef ul   i n  the  tr eatm ent  of   enthesopathi es  and  r ecal ci tr ant  per i pher al
sy nov i ti s.   The  tum or   necr osi s  f actor   α   (TN F­Î± )  bl ock i ng  dr ugs  ar e
v er y   ef f ecti v e  i n  AS,   act  on  both  spi nal   and  per i pher al   joi nts,   and  m ay
possi bl y   del ay   or   pr ev ent  spi nal   ank y l osi s  (tr eatm ent  r esul ts  i n
i m pr ov em ent  i n  m agneti c  r esonance  i m agi ng  (MRI)  appear ance  of
enthesi ti s  and  sacr oi l i i ti s).   The  use  of   anti ­TN F  agents  shoul d  be
consi der ed  i n  pati ents  w i th  acti v e  AS  w ho  hav e  f ai l ed  to  r espond  to  tw o
or   m or e  N SAIDs  f or   ax i al   di sease  and  one  or   m or e  di sease­m odi f y i ng
anti r heum ati c  dr ug  (DMARD)  f or   per i pher al   ar thr i ti s.

Suggested Readings
Hasl ock   I.   Ank y l osi ng  spondy l i ti s:  m anagem ent.   In:  Hochber g  MC,
Si l m an  AJ,   Sm ol en  JS,   et  al .   eds.   Rheum atol ogy ,   3r d  ed.   Edi nbur gh:
Mosby ,   2003:1211–1224.

Janson  RW.   Ank y l osi ng  spondy l i ti s.   In:  West  SG,   ed.   Rheum atol ogy
secr ets,   2nd  ed.   Phi l adel phi a:  Hanl ey   &  Bel f us,   2002:255–261.

Khan  MA.   Cl i ni cal   f eatur es  of   ank y l osi ng  spondy l i ti s.   In:  Hochber g  MC,
Si l m an  AJ,   Sm ol en  JS,   et  al .   eds.   Rheum atol ogy ,   3r d  ed.   Edi nbur gh:
Mosby ,   2003:1161–1182.

Van  der   Li nden  S,   Van  der   Hei jde  D,   Br aun  J.   Ank y l osi ng  spondy l i ti s.   In:
Har r i s  ED  Jr ,   Budd  RC,   Fi r estei n  GS,   et  al .   eds.   Kel l ey 's  tex tbook   of
r heum atol ogy ,   7th  ed.   Phi l adel phi a:  El sev i er   Saunder s,   2005:1125â
€“1141.

Crystal­Induced Arthritis
1.   What  ar e  thr ee  di f f er ent  f or m s  of   cr y stal ­i nduced  ar thr i ti s,   and  w hat
ar e  the  cr y stal s  i nv ol v ed?

P. 437
2.   What  ar e  f our   di f f er ent  di seases  that  char acter i sti cal l y   pr esent  w i th
ar thr i ti s  of   a  si ngl e  joi nt?

3.   What  thr ee  joi nts  ar e  m ost  com m onl y   i nv ol v ed  i n  acute  attack s  of   gout?

4.   What  ar e  som e  hi stor i cal   f eatur es  of ten  f ound  i n  pati ents  w i th  gout?

5.   What  ar e  thr ee  l abor ator y   test  f i ndi ngs  that  m ay   be  abnor m al   i n  the
setti ng  of   gout?

Discussion
1.   What  ar e  thr ee  di f f er ent  f or m s  of   cr y stal ­i nduced  ar thr i ti s,   and  w hat
ar e  the  cr y stal s  i nv ol v ed?

Gout  i s  a  cr y stal ­i nduced  ar thr i ti s  due  to  the  deposi ti on  of   m onosodi um
ur ate  (MSU )  cr y stal s.   Pseudogout  r esul ts  f r om   the  f or m ati on  and
r el ease  of   cal ci um   py r ophosphate  di hy dr ate  (CPPD)  cr y stal s.   The
deposi ti on  of   hy dr ox y apati te  cr y stal s  can  i nduce  acute  i nf l am m ator y
ar thr i ti des  such  as  cal ci f i c  per i ar thr i ti s/tendi ni ti s  and  the  Mi l w auk ee
shoul der   sy ndr om e,   a  destr ucti v e  ar thr opathy   of   the  shoul der
associ ated  w i th  r otator   cuf f   def ects.

2.   What  ar e  f our   di f f er ent  di seases  that  char acter i sti cal l y   pr esent  w i th
ar thr i ti s  of   a  si ngl e  joi nt?
Ar thr i ti s  of   a  si ngl e  joi nt  (m onoar ti cul ar   ar thr i ti s)  m ay   be  the  i ni ti al
sy m ptom   of   septi c  ar thr i ti s,   cr y stal   deposi ti on  di seases,   tr aum ati c
ar thr i ti s,   and  other   causes  such  as  osteoar thr i ti s  (OA),   coagul opathy ,
av ascul ar   necr osi s,   and  pi gm ented  v i l l onodul ar   sy nov i ti s.   Other
hi stor i cal   and  cl i ni cal   f eatur es  m ay   be  used  to  di sti ngui sh  am ong  these
thr ee  di agnoses.   A  def i ni ti v e  di agnosi s  i s  m ade  on  the  basi s  of   the
f i ndi ngs  y i el ded  by   sy nov i al   f l ui d  ex am i nati on  i ncl udi ng  cel l   count  w i th
di f f er enti al ,   Gr am 's  stai n  and  cul tur e,   and  pol ar i zed  l i ght  m i cr oscopy
f or   cr y stal   anal y si s.

3.   What  thr ee  joi nts  ar e  m ost  com m onl y   i nv ol v ed  i n  acute  attack s  of   gout?

Acute  gout  m ost  com m onl y   ar i ses  i n  the  f i r st  m etatar sophal angeal
(MTP)  joi nt;  thi s  i s  k now n  as  podagr a.   The  nex t  m ost  com m onl y
i nv ol v ed  si tes  ar e  the  i nstep  and  ank l e.   Knees,   w r i sts,   f i nger s,   and
el bow s  can  al so  be  i nv ol v ed.   Gout  has  a  pr edi l ecti on  f or   cool ,
per i pher al   joi nts  w her e  the  sol ubi l i ty   of   MSU   cr y stal s  m ay   be
di m i ni shed  as  a  r esul t  of   the  cool er   tem per atur e.

4.   What  ar e  som e  hi stor i cal   f eatur es  of ten  f ound  i n  pati ents  w i th  gout?

Pati ents  w i th  gout  m ay   hav e  a  posi ti v e  f am i l y   hi stor y   f or   the  di sease,


par ti cul ar l y   i n  m al e  m em ber s.   Gout  i s  al so  m or e  com m on  i n  peopl e
w ho  hav e  a  hi stor y   of   obesi ty ,   m etabol i c  sy ndr om e,   or   al cohol i sm .
Acute  attack s  of   gout  m ay   occur   dur i ng  or   af ter   an  epi sode  of
ex cessi v e  al cohol   i ngesti on,   ex cess  di etar y   pur i ne  i ntak e,   tr aum a,
acute  m edi cal   i l l ness,   or   sur ger y .   The  attack s  com m onl y   begi n  abr uptl y
dur i ng  the  ni ght  or   ear l y   m or ni ng  hour s.

5.   What  ar e  the  thr ee  l abor ator y   test  f i ndi ngs  that  m ay   be  abnor m al   i n
the  setti ng  of   gout?

Pati ents  w i th  acute  attack s  of   gout  of ten  hav e  a  m i l d  l euk ocy tosi s  and
an  el ev ated  er y thr ocy te  sedi m entati on  r ate  (ESR).   To  dev el op  gout,
these  pati ents  hav e  to  be  chr oni cal l y   hy per ur i cem i c,   def i ned  as  a
ser um   ur i c  aci d  l ev el   gr eater
P. 438
than  7. 0  m g/dL  i n  m en  and  6. 0  m g/dL  i n  w om en.   At  the  ti m e  of   an
acute  attack ,   up  to  30%  of   pati ents  m ay   hav e  a  nor m al   ser um   ur i c  aci d
l ev el .

Case
A  52­y ear ­ol d  m an  com es  to  the  em er gency   r oom   com pl ai ni ng  of   pai n  i n  hi s
bi g  toe.   He  w as  w el l   unti l   5:00  thi s  m or ni ng,   w hen  he  w as  aw ak ened  by   an
achi ng  pai n  i n  hi s  r i ght  gr eat  toe.   Wi thi n  a  f ew   hour s,   the  joi nt  w as  dusk y
r ed  and  hot,   and  w as  ex qui si tel y   tender   to  the  poi nt  that  ev en  the  w ei ght  of
the  beddi ng  hur t  hi s  toe.   By   8:00  a. m . ,   the  pati ent  coul d  bear   onl y   par ti al
w ei ght  on  the  f oot.   The  pati ent  r epor ts  a  f ew   sel f ­l i m i ted,   tr i v i al   epi sodes  of
tw i nges  of   pai n  i n  thi s  toe  ov er   the  past  y ear .   The  pati ent  descr i bes  f eel i ng
f ev er i sh  w i thout  r i gor s  or   chi l l s.   Ther e  i s  no  hi stor y   of   tr aum a  to  the  f oot,
nor   i s  ther e  a  f am i l y   hi stor y   of   ar thr i ti s  or   si m i l ar   attack s.   He  i s  tak i ng
hy dr ochl or othi azi de  f or   contr ol   of   hy per tensi on.
On  phy si cal   ex am i nati on,   the  pati ent  i s  f ound  to  be  a  stock y ,   ov er w ei ght,
and  r ed­f aced  m an.   Hi s  bl ood  pr essur e  i s  170/100  m m   Hg,   hi s  pul se  i s  90
beats  per   m i nute  and  r egul ar ,   and  hi s  tem per atur e  i s  38°C  (100. 4°F).
Sk i n  ex am i nati on  di scl oses  no  l esi ons  or   nodul es.   On  ex am i nati on  of   hi s
joi nts,   al l   show   a  nor m al   r ange  of   m oti on  w i thout  sy nov i ti s  or   def or m i ty ,
ex cept  f or   the  r i ght  f i r st  MTP  joi nt,   w hi ch  show s  sy nov i ti s,   2+ /4;  w ar m th,
4+ /4;  tender ness,   4+ /4;  and  er y them a  at  the  base  of   the  toe  ex tendi ng  onto
the  dor sal   aspect  of   the  f or ef oot  w i th  sl i ght  edem a.
The  f ol l ow i ng  l abor ator y   v al ues  ar e  r epor ted:  w hi te  bl ood  cel l   (WBC)  count
12, 500  cel l s/m m 3   w i th  92%  pol y m or phonucl ear   l euk ocy tes  and  2%  band
f or m s;  ur i c  aci d  9. 0  m g/dL;  cr eati ni ne  1. 0  m g/dL.   U r i nal y si s  r ev eal s  no  r ed
bl ood  cel l s  or   pr otei n.   A  r adi ogr aphi c  study   of   the  r i ght  f oot  di scl oses  sof t
ti ssue  sw el l i ng  ar ound  the  r i ght  f i r st  MTP  joi nt,   but  no  er osi ons.

1.   How   i s  the  di agnosi s  of   gout  establ i shed?


2.   Why   ar e  hum ans  pr edi sposed  to  dev el opi ng  gout?
3.   What  ar e  the  f our   r ev er si bl e  secondar y   causes  of   hy per ur i cem i a?
4.   What  ar e  the  f our   cl i ni cal   stages  of   gout?
5.   What  ar e  the  appr opr i ate  ther api es  f or   an  acute  attack   of   gout  and
chr oni c  sy m ptom ati c  hy per ur i cem i a?

Case Discussion
1.   How   i s  the  di agnosi s  of   gout  establ i shed?

The  di agnosi s  of   gout  r equi r es  aspi r ati on  of   sy nov i al   f l ui d  or   a  tophus
f or   cr y stal   anal y si s  by   pol ar i zed  m i cr oscopy .   MSU   cr y stal s  ar e  needl e­
shaped  and  negati v el y   bi r ef r i ngent.   In  contr ast,   CPPD  cr y stal s
(pseudogout)  ar e  r hom boi d­shaped  and  posi ti v el y   bi r ef r i ngent.   In  gout,
sy nov i al   f l ui d  i s  i nf l am m ator y   (ty pi cal l y   20, 000  to  100, 000
l euk ocy tes/m m 3 ).   The  sy nov i al   f l ui d  shoul d  be  sent  f or   Gr am 's  stai n
and  cul tur e  as  i n  r ar e  cases,   septi c  joi nt  f l ui ds  can  contai n  MSU
cr y stal s.   El ev ated  ser um   ur i c  aci d  l ev el s  ar e  not  di agnosti c  of   gout  as
m any   i ndi v i dual s  hav e  asy m ptom ati c  hy per ur i cem i a  and  nev er   dev el op
gout.

P. 439
2.   Why   ar e  hum ans  pr edi sposed  to  dev el opi ng  gout?

U r i c  aci d  i s  the  end  pr oduct  of   the  degr adati on  of   pur i nes.   Hum ans  l ack
the  enzy m e  ur i case,   w hi ch  ox i di zes  ur i c  aci d  to  the  hi ghl y   sol ubl e
com pound  al l antoi n.   The  l ack   of   thi s  enzy m e  subjects  hum ans  to  the
potenti al   r i sk   of   dev el opi ng  hy per ur i cem i a  and  gout.   Al though  hum ans
possess  the  ur i case  gene,   i t  i s  i nacti v e.   U r i c  aci d  m ay   hav e  anti ox i dant
and  f r ee  r adi cal   scav enger   pr oper ti es.

3.   What  ar e  the  f our   r ev er si bl e  secondar y   causes  of   hy per ur i cem i a?

The  r ev er si bl e  secondar y   causes  of   hy per ur i cem i a  i ncl ude  al cohol


consum pti on,   di ets  contai ni ng  pur i ne­r i ch  f oods  (m eats  and  or gan
m eats;  seaf ood,   par ti cul ar l y   shel l f i sh),   m edi cati ons  that  decr ease  the
r enal   ex cr eti on  of   ur i c  aci d  (cy cl ospor i ne,   ni coti ni c  aci d,   di ur eti cs,
etham butol ,   l ow ­dose  aspi r i n,   py r azi nam i de),   and  obesi ty   (w ei ght  l oss
can  i m pr ov e  hy per ur i cem i a).   The  cur r ent  di etar y   r ecom m endati ons  ar e
consum pti on  of   m eat,   seaf ood,   and  al cohol   hav e  to  be  i n  m oder ati on;
pur i ne­r i ch  v egetabl es  ar e  acceptabl e;  and  l ow ­f at  dai r y   pr oducts  and
w i ne  m ay   be  pr otecti v e  f r om   gout.

4.   What  ar e  the  f our   cl i ni cal   stages  of   gout?

The  f our   stages  of   gout  ar e  asy m ptom ati c  hy per ur i cem i a,   acute  gouty
ar thr i ti s,   i nter cr i ti cal   gout  (the  asy m ptom ati c  i nter v al   betw een
attack s),   and  chr oni c  tophaceous  gout.   Many   pati ents  w i th
asy m ptom ati c  hy per ur i cem i a  do  not  pr ogr ess  to  gouty   ar thr i ti s.   Ther e
m ay   not  be  shar p  dem ar cati ons  betw een  the  l ast  thr ee  stages  of   gout
because  som e  pati ents  hav e  both  chr oni c  tophaceous  gout  as  w el l   as
i nter m i ttent  acute  attack s.

5.   What  ar e  the  appr opr i ate  ther api es  f or   an  acute  attack   of   gout  and
chr oni c  sy m ptom ati c  hy per ur i cem i a?

The  pr ef er r ed  tr eatm ent  f or   an  acute  attack   of   gout  i s  an  or al   N SAID,   i f
not  contr ai ndi cated.   Thi s  shoul d  be  gi v en  i n  hi gh  doses  f or   a  f ew   day s
f ol l ow ed  by   a  taper i ng,   w i th  di sconti nuati on  by   7  to  10  day s.   Or al
col chi ci ne  can  onl y   be  used  i n  y ounger   pati ents  w i th  nor m al   r enal   and
hepati c  f uncti on.   Its  use  i s  l i m i ted  by   the  hi gh  i nci dence  of   acute
gastr oi ntesti nal   si de  ef f ects.   Intr av enous  col chi ci ne  shoul d  be  av oi ded
because  of   the  potenti al   f or   ex cess  dosi ng  i n  hi gh­r i sk   pati ents,   l i k el y
r esul ti ng  i n  death.   Both  or al l y   and  i ntr aar ti cul ar l y   adm i ni ster ed
cor ti coster oi ds  ar e  ef f ecti v e  i n  the  m anagem ent  of   acute  attack s  of
gout  i n  pati ents  w ho  ar e  i ntol er ant  of   or   hav e  contr ai ndi cati ons  to  the
af or em enti oned  m edi cati ons.   Pati ents  w i th  chr oni c  sy m ptom ati c
hy per ur i cem i a  r equi r e  l i f el ong  ther apy   w i th  a  ur ate­l ow er i ng
m edi cati on.   Pr obeneci d,   a  ur i cosur i c,   can  be  used  i f   they   ar e  r enal
under ex cr etor s  of   ur i c  aci d  (< 700  m g  per   24  hour s),   hav e  a  cr eati ni ne
cl ear ance  gr eater   than  50  m L  per   m i nute,   and  ar e  not  tak i ng  m or e  than
81  m g  of   aspi r i n  per   day .   Al l opur i nol ,   a  x anthi ne  ox i dase  i nhi bi tor ,   i s
i ndi cated  i f   they   ar e  ov er pr oducer s  (> 700  m g  per   24  hour s),   hav e  ur i c
aci d  or   cal ci um   stones,   or   tophaceous  di sease.   Al l opur i nol   i s  m or e
com m onl y   used  as  i t  w or k s  f or   both  under ex cr etor s  and  ov er pr oducer s
of   ur i c  aci d,   and  i s  tak en  onl y   once  dai l y   w hi ch  i ncr eases  com pl i ance.
N ew   ther api es  under   i nv esti gati on  i ncl ude  i ntr av enous  pol y ethy l ene
gl y col   (PEG)­ur i case  and  f ebux ostat  (a  nonpur i ne,   sel ecti v e  i nhi bi tor   of
x anthi ne  ox i dase).
P. 440

Suggested Readings
Gi bson  T.   Cl i ni cal   f eatur es  of   gout.   In:  Hochber g  MC,   Si l m an  AJ,   Sm ol en
JS,   et  al .   eds.   Rheum atol ogy ,   3r d  ed.   Edi nbur gh:  Mosby ,   2003:1919â
€“1928.

Janson  RW.   Gout.   In:  West  SG,   ed.   Rheum atol ogy   secr ets,   2nd  ed.
Phi l adel phi a:  Hanl ey   &  Bel f us,   2002:325–333.

Ter k el taub  R.   Di seases  associ ated  w i th  the  ar ti cul ar   deposi ti on  of
cal ci um   py r ophosphate  dehy dr ate  and  basi c  cal ci um   phosphate  cr y stal s.
In:  Har r i s  ED  Jr ,   Budd  RC,   Fi r estei n  GS,   et  al .   eds.   Kel l ey 's  tex tbook   of
r heum atol ogy ,   7th  ed.   Phi l adel phi a:  El sev i er   Saunder s,   2005:1430â
€“1448.

Wor tm ann  RL,   Kel l ey   WN .   Gout  and  hy per ur i cem i a.   In:  Har r i s  ED  Jr ,
Budd  RC,   Fi r estei n  GS,   et  al .   eds.   Kel l ey 's  tex tbook   of   r heum atol ogy ,   7th
ed.   Phi l adel phi a:  El sev i er   Saunder s,   2005:1402–1429.

Fibromyalgia
1.   What  i s  the  def i ni ti on  of   nonar ti cul ar   r heum ati sm   and  w hat  ar e  the  f our
f or m s  of   the  di sor der ?

2.   N am e  f our   com m on  ty pes  of   tendi ni ti s  and  bur si ti s,   and  the  m ajor
str uctur e  i nv ol v ed  i n  each  ty pe?

3.   What  ar e  the  cr i ter i a  f or   di agnosi s  of   f i br om y al gi a  sy ndr om e  (FMS)?

4.   What  ar e  f i v e  m edi cal   i l l nesses  that  m ay   ex hi bi t  sy m ptom s  si m i l ar   to


those  of   FMS?

Discussion
1.   What  i s  the  def i ni ti on  of   nonar ti cul ar   r heum ati sm ,   and  w hat  ar e  the
f our   f or m s  of   the  di sor der ?

N onar ti cul ar   r heum ati sm   r ef er s  to  aches  and  pai ns  that  ar i se  f r om
str uctur es  outsi de  of   joi nts,   so  i t  i s  not  actual l y   a  tr ue  f or m   of
ar thr i ti s.   Four   f or m s  of   nonar ti cul ar   r heum ati sm   ar e  tendi ni ti s,   bur si ti s,
FMS,   and  the  m y of asci al   pai n  sy ndr om e.   Tendi ni ti s  i nv ol v es
i nf l am m ati on  and  pai n  i n  speci f i c  tendons  and  i s  usual l y   due  to  str ess
or   ov er use.   Bur sae  ar e  sy nov i um ­l i ned  sacs  that  ei ther   ov er l i e  or   ar e
adjacent  to  joi nts  and  m ay   al so  becom e  i nf l am ed  secondar y   to  ov er use.
FMS  i s  a  di f f use  chr oni c  pai n  di sor der   that  i s  di scussed  i n  l ater
questi ons.   The  m y of asci al   pai n  sy ndr om e,   som eti m es  ter m ed  r epeti ti v e
str ai n  sy ndr om e,   consi sts  of   l ocal i zed  (one  anatom i c  r egi on)  tender   and
pai nf ul   m uscl es  i n  the  absence  of   any   ev i dence  of   an  i nf l am m ator y
m uscl e  di sease  or   FMS.

2.   N am e  f our   com m on  ty pes  of   tendi ni ti s  and  bur si ti s,   and  the  m ajor
str uctur e  i nv ol v ed  i n  each  ty pe?

“Tenni s  el bow â€​   i s  pai n  ov er   the  l ater al   epi condy l e  of   the  el bow   due
to  i nf l am m ati on  of   the  tendons  of   the  w r i st  ex tensor   m uscl es  that
i nser t  at  thi s  l ocati on.   “Gol f er 's  el bow â€​   i s  pai n  ov er   the  m edi al
epi condy l e  due  to  i nf l am m ati on  of   the  w r i st  f l ex or   tendons  that  i nser t
at  thi s  l ocati on.   The
P. 441
“shoul der   i m pi ngem ent  sy ndr om eâ€​   r esul ts  f r om   i m pi ngem ent  of   the
tendons  of   the  r otator   cuf f   w i th  shoul der   abducti on  or   f l ex i on  and  can
be  associ ated  w i th  supr aspi natus  tendi ni ti s,   subacr om i al   bur si ti s,   or
r otator   cuf f   tear s.   “Housem ai d's  k neeâ€​   i s  pr epatel l ar   bur si ti s
br ought  about  by   r epeti ti v e  tr aum a  or   ov er use  such  as  k neel i ng.
Another   com m on  ar ea  f or   bur si ti s  i s  ov er   the  gr eater   tr ochanter   of   the
l ater al   hi p.

3.   What  ar e  the  cr i ter i a  f or   di agnosi s  of   FMS?

The  di agnosti c  cr i ter i a  f or   FMS  i ncl ude  at  l east  3  m onths  of   w i despr ead
pai n  that  i s  bi l ater al ,   abov e  and  bel ow   the  w ai st,   and  i ncl udes  ax i al
sk el etal   pai n,   and  pai n  to  pal pati on  at  a  m i ni m um   of   11  of   18
pr edef i ned  tender   poi nts  (di scussed  i n  subsequent  tex t).   The  di agnosi s
of   other   di seases  does  not  ex cl ude  the  di agnosi s  of   FMS.

4.   What  ar e  f i v e  m edi cal   i l l nesses  that  m ay   ex hi bi t  sy m ptom s  si m i l ar   to


those  of   FMS?

Il l nesses  that  m ay   ex hi bi t  sy m ptom s  si m i l ar   to  those  of   FMS  i ncl ude


cel i ac  spr ue,   hepati ti s  C,   hy per par athy r oi di sm ,   hy pothy r oi di sm ,   and
pol y m y al gi a  r heum ati ca  (PMR).   How ev er ,   each  of   these  i l l nesses  i s
associ ated  w i th  char acter i sti c  hi stor i cal ,   cl i ni cal ,   and  l abor ator y
abnor m al i ti es  that  di sti ngui sh  i t  f r om   FMS.   In  addi ti on,   i t  i s  of ten
di f f i cul t  to  di f f er enti ate  the  sy m ptom s  of   FMS  f r om   those  of   chr oni c
f ati gue  sy ndr om e.   The  di f f er enti al   di agnosi s  f or   FMS  al so  i ncl udes  RA,
SLE,   i nf l am m ator y   m y opathi es,   obstr ucti v e  sl eep  apnea,   par aneopl asti c
di sor der s,   and  ser onegati v e  spondy l oar thr opathi es.

Case
A  38­y ear ­ol d  w om an  i s  r ef er r ed  f or   ev al uati on  because  of   di f f use  pai n  and
f ati gue.   She  com pl ai ns  of   6  m onths  of   f ati gue,   gener al i zed  pai n,   di f f i cul ty
sl eepi ng,   m or ni ng  sti f f ness,   and  i nter m i ttent  sw el l i ng  of   her   f i nger s.   The
sti f f ness  i s  w or se  i n  the  m or ni ng,   but  she  cannot  put  a  def i ni te  ti m e  l i m i t  on
i t.   She  has  a  hi stor y   of   m i gr ai ne  headaches  and  i r r i tabl e  bow el   sy ndr om e.
She  w as  f i r st  seen  by   her   f am i l y   phy si ci an  com pl ai ni ng  of   “pai n  al l
ov er . â€​
  She  w as  i ni ti al l y   tr eated  w i th  i ndom ethaci n  w i thout  r el i ef .
Subsequentl y ,   she  has  tr i ed  sev er al   di f f er ent  N SAIDs  w i thout  r el i ef   of   her
sy m ptom s.
She  i s  a  di v or ced  m other   of   thr ee  chi l dr en,   w ho  w or k s  f ul l   ti m e  as  a
l i censed  pr acti cal   nur se.   She  has  no  hi stor y   of   a  r ash,   or al   ul cer s,   sei zur es,
bl ood  di sor der ,   or   k now n  k i dney   di sease.
Phy si cal   ex am i nati on  r ev eal s  nor m al   v i tal   si gns,   as  w el l   as  nor m al   head,
ear ,   ey es,   nose,   thr oat,   neck ,   sk i n,   chest,   and  abdom i nal   f i ndi ngs.   Her
f i nger s  and  joi nts  ar e  nor m al   w i thout  any   sw el l i ng  or   sy nov i ti s.   Her
m uscul ar   and  neur ol ogi c  ex am i nati ons  ar e  nonf ocal .   Sev er al   tender   poi nts
ar e  i denti f i ed.

1.   What  ar e  tw o  char acter i sti cs  of   the  sl eep  di sor der   that  com m onl y
accom pani es  FMS?
2.   What  ar e  the  char acter i sti c  phy si cal   f i ndi ngs  i n  FMS?
3.   Ar e  ther e  any   l abor ator y   test  abnor m al i ti es  char acter i sti c  of   FMS?
4.   What  i s  the  ther apy   f or   FMS?
5.   Whi ch  psy chol ogi cal   di sor der s  ar e  of ten  associ ated  w i th  FMS?

P. 442

Case Discussion
1.   What  ar e  tw o  char acter i sti cs  of   the  sl eep  di sor der   that  com m onl y
accom pani es  FMS?

The  sl eep  di sor der   seen  i n  the  contex t  of   FMS  i s  char acter i zed  by   ear l y
m or ni ng  aw ak eni ng  and  unr ef r eshi ng  or   nonr estor ati v e  sl eep.
Di sr upti on  of   del ta­w av e  sl eep  (non­REM  stage  IV  sl eep)  occur s  due  to
al pha­w av e  i ntr usi on,   and  i s  ter m ed  the  al pha­del ta  sl eep  patter n  of
FMS.   Obstr ucti v e  sl eep  apnea  and  r estl ess  l eg  sy ndr om e  shoul d  al so  be
consi der ed  i n  pati ents  pr esenti ng  w i th  FMS.

2.   What  ar e  the  char acter i sti c  phy si cal   f i ndi ngs  i n  f i br om y al gi a?

Pati ents  w i th  FMS  hav e  a  nor m al   phy si cal   ex am i nati on  ex cept  f or
tender   poi nts  i n  pr eci se  l ocati ons.   These  tender   poi nts  ar e  ty pi cal l y
l ocated  at  the  occi put,   at  the  m i dpor ti on  of   the  tr apezi us,   the  or i gi n  of
the  supr aspi natus,   l ow   anter i or   cer v i cal   r egi on,   second  costochondr al
juncti on,   l ater al   epi condy l e,   outer   upper   quadr ant  of   the  buttock s,
gr eater   tr ochanter   r egi on,   and  m edi al   k nee  ar ea.   These  ar eas  ar e
usual l y   tender   bi l ater al l y   i n  pati ents  w i th  FMS.   Contr ol   poi nts  such  as
the  m i df or ear m   and  anter i or   m i dthi gh  ar e  not  nor m al l y   pai nf ul   i n
pati ents  w i th  FMS.

3.   Ar e  ther e  any   l abor ator y   test  abnor m al i ti es  char acter i sti c  of   FMS?
Al l   l abor ator y   test  r esul ts  i n  the  setti ng  of   FMS  ar e  usual l y   com pl etel y
nor m al .   To  i ni ti al l y   ex cl ude  di sor der s  that  m ay   m i m i c  FMS,   a  com pl ete
bl ood  count,   ESR,   cr eati ni ne,   l i v er   f uncti on  tests,   thy r oi d­sti m ul ati ng
hor m one,   cr eati ne  phosphok i nase  (CPK),   cal ci um ,   phosphor us,   and
ur i nal y si s  shoul d  be  per f or m ed.   Anti nucl ear   anti body   (AN A)  testi ng
shoul d  not  be  per f or m ed  unl ess  ther e  i s  pr etest  pr obabi l i ty   of   a
connecti v e  ti ssue  di sease  (CTD)  si nce  a  substanti al   num ber   of
i ndi v i dual s  w i th  FMS  (12%  to  30%)  can  hav e  a  l ow   ti ter ,   nonspeci f i c
posi ti v e  AN A.

4.   What  i s  the  ther apy   f or   FMS?

The  appr opr i ate  ther apy   f or   FMS  i ncl udes  pati ent  educati on,   anal gesi cs
such  as  acetam i nophen  or   tr am adol ,   l ow ­dose  tr i cy cl i c  anti depr essants
or   cy cl obenzapr i ne  at  bedti m e  to  i m pr ov e  the  sl eep  cy cl e,   and  l ow ­
i m pact  aer obi c  ex er ci ses.   Anti i nf l am m ator y   m edi cati ons  ar e  not
gener al l y   hel pf ul .   Sel ecti v e  ser otoni n  r euptak e  i nhi bi tor s  (SSRIs)  and
pr egabal i n  m ay   hav e  som e  ef f i cacy   i n  FMS.   Thi s  i s  a  v er y   f r ustr ati ng
di sor der   f or   both  the  pati ent  and  phy si ci an.   Many   pati ents  m ay   be
hel ped  by   thi s  appr oach  to  ther apy ,   the  m ost  i m por tant  el em ent  of
w hi ch  i s  an  ex er ci se  pr ogr am .

5.   Whi ch  psy chol ogi cal   di sor der s  ar e  of ten  associ ated  w i th  FMS?

Functi onal   psy chi atr i c  di sor der s,   such  as  the  som atof or m   di sor der s,   and
or gani c  psy chi atr i c  di sor der s,   such  as  m ajor   depr essi on  and  anx i ety
di sor der s,   hav e  been  associ ated  w i th  FMS  i n  appr ox i m atel y   30%  of
pati ents.   The  anx i ety   and  m i l d  depr essi on  that  of ten  pr esent  i n  FMS
m ay   be  secondar y   to  chr oni c  pai n  and  concer ns  r egar di ng  per sonal
i ndependence  and  debi l i ty .

Suggested Readings
Bur k ham   J,   Har r i s  ED  Jr .   Fi br om y al gi a:  a  chr oni c  pai n  sy ndr om e.   In:
Har r i s  ED  Jr ,   Budd  RC,   Fi r estei n  GS,   et  al .   eds.   Kel l ey 's  tex tbook   of
r heum atol ogy ,   7th  ed.   Phi l adel phi a:  El sev i er ,   Saunder s,   2005:522–536.

P. 443

Gol denber g  DL.   Fi br om y al gi a  and  r el ated  sy ndr om es.   In:  Hochber g  MC,
Si l m an  AJ,   Sm ol en  JS,   et  al .   eds.   Rheum atol ogy ,   3r d  ed.   Edi nbur gh:
Mosby ,   2003:701–712.

Mal y ak   M.   Fi br om y al gi a.   In:  West  SG,   ed.   Rheum atol ogy   secr ets,   2nd  ed.
Phi l adel phi a:  Hanl ey   &  Bel f us,   2002:428–440.
Osteoarthritis
1.   What  i s  the  joi nt  str uctur e  that  i s  pr i m ar i l y   i nv ol v ed  i n  OA?

2.   Why   i s  pai n  at  the  base  of   the  thum b  and  the  gr adual   onset  of   pai n  i n  a
k nee  w i th  m i ni m al   sw el l i ng  m or e  char acter i sti c  of   OA  than  of   RA?

3.   What  ar e  the  r i sk   f actor s  f or   dev el opi ng  OA?

4.   What  ar e  som e  of   the  char acter i sti c  f i ndi ngs  encounter ed  dur i ng
phy si cal   ex am i nati on  i n  pati ents  w i th  OA?

Discussion
1.   What  i s  the  joi nt  str uctur e  that  i s  pr i m ar i l y   i nv ol v ed  i n  OA?

OA  i s  the  m ost  com m on  joi nt  di sor der   i n  the  w or l d.   It  i s  a  di sor der   of
ar ti cul ar   car ti l age  w i th  secondar y   changes  i n  the  adjacent  bone.

2.   Why   i s  pai n  at  the  base  of   the  thum b  and  the  gr adual   onset  of   pai n  i n  a
k nee  w i th  m i ni m al   sw el l i ng  m or e  char acter i sti c  of   OA  than  of   RA?

Pai n  at  the  base  of   the  thum b  r epr esents  ar thr i ti s  of   the  f i r st
car pom etacar pal   (CMC)  joi nt.   Thi s  joi nt  i s  com m onl y   i nv ol v ed  i n  the
setti ng  of   OA  because  of   f r equent  m echani cal   dam age  i ncur r ed  dur i ng
nor m al   use  of   the  hand.   Ear l y   OA  m ay   be  char acter i zed  by   joi nt  pai n
w i th  use,   w i thout  si gns  of   i nf l am m ati on;  m or ni ng  sti f f ness  i s  ty pi cal l y
f or   l ess  than  30  m i nutes.   OA  i s  noni nf l am m ator y   and  can  i nv ol v e  the
di stal   i nter phal angeal s  (DIPs)  w i th  associ ated  Heber den's  nodes;
pr ox i m al   i nter phal angeal s  (PIPs)  w i th  associ ated  Bouchar d's  nodes;  the
f i r st  CMC  of   the  hand;  the  f i r st  MTP  joi nts;  the  spi ne;  hi ps;  and  k nees.
RA  i s  an  i nf l am m ator y   ar thr i ti s  and  i nv ol v es  bi l ater al
m etacar pophal angeal s  (MCPs)  and  PIPs  i n  a  sy m m etr i c  m anner   and  can
al so  i nv ol v e  the  MTPs  and  other   sy nov i um ­l i ned  joi nts;  m or ni ng
sti f f ness  i s  ty pi cal l y   f or   m or e  than  60  m i nutes.

3.   What  ar e  the  r i sk   f actor s  f or   dev el opi ng  OA?

The  r i sk   f actor s  f or   dev el opi ng  OA  ar e  age,   obesi ty ,   abnor m al   joi nt
m echani cs,   pr ev i ous  joi nt  tr aum a  or   i nf l am m ator y   joi nt  di sease,
her edi ty   (especi al l y   OA  of   the  DIP  joi nts),   and  cer tai n  occupati ons  that
r equi r e  r epeti ti v e  use  of   joi nt  gr oups,   bendi ng,   or   car r y i ng  heav y
l oads.   Metabol i c  di sor der s  associ ated  w i th  OA  i ncl ude  cr y stal   deposi ti on
di seases,   Paget's  di sease,   ochr onosi s,   acr om egal y ,   hem ochr om atosi s,
and  Wi l son's  di sease.

4.   What  ar e  som e  of   the  char acter i sti c  f i ndi ngs  encounter ed  dur i ng
phy si cal   ex am i nati on  i n  pati ents  w i th  OA?

Ty pi cal   f i ndi ngs  encounter ed  dur i ng  phy si cal   ex am i nati on  i n  pati ents
w i th  OA  i ncl ude  bony   ov er gr ow th  (osteophy tes),   joi nt  l i ne  tender ness,
cr epi tus  on
P. 444
passi v e  m oti on,   and  l i m i tati on  of   m oti on  w i th  pai n  on  ex tr em es  of
m oti on.   The  end  r esul t  m ay   be  joi nt  def or m i ty .

Case
A  56­y ear ­ol d  m al e  constr ucti on  w or k er   com pl ai ns  of   chr oni c  pai n  i n  hi s
k nees  and  i nter m i ttent  pai n  at  the  base  of   hi s  thum b.   When  gr i ppi ng
som ethi ng  f or cef ul l y ,   the  pai n  at  the  base  of   the  thum b  (f i r st  CMC)  i s
som eti m es  so  shar p  that  he  i s  f or ced  m om entar i l y   to  stop  w hat  he  i s  doi ng.
Hi s  k nees  ache  di f f usel y   af ter   ex cessi v e  use.   These  com pl ai nts  k eep  hi m
f r om   w or k i ng  as  of ten  as  he  w oul d  l i k e.   He  r epor ts  no  si gni f i cant  m or ni ng
sti f f ness.   Hi s  f am i l y   hi stor y   i s  unr em ar k abl e.   Past  m edi cal   hi stor y   i s
si gni f i cant  f or   m i l d  essenti al   hy per tensi on  f or   w hi ch  he  has  been  tak i ng
hy dr ochl or othi azi de  f or   8  y ear s.
Phy si cal   ex am i nati on  r ev eal s  sl i ght  quadr i ceps  atr ophy   on  the  r i ght  w i th
sl i ght  genu  v ar um   and  a  pes  anser i nus  bur si ti s,   f l attened  ar ches,   and
m oder ate  obesi ty .   Ther e  i s  m i l d  cr epi tus  i n  both  k nees  w i thout  l i gam entous
i nstabi l i ty   or   ef f usi ons.   Ther e  i s  m oder ate  tender ness  of   the  f i r st  CMC  joi nts
bi l ater al l y .   Ther e  ar e  no  Heber den's  or   Bouchar d's  nodes.

1.   What  ar e  som e  of   the  char acter i sti c  changes  that  af f ect  the  ar ti cul ar
car ti l age  i n  pati ents  w i th  OA?
2.   What  ar e  f our   char acter i sti c  r adi ogr aphi c  f i ndi ngs  encounter ed  i n
pati ents  w i th  OA?
3.   Di scuss  the  nonphar m acol ogi c  m anagem ent  of   OA?
4.   Di scuss  the  phar m acol ogi c  opti ons  f or   the  tr eatm ent  of   OA?

Case Discussion
1.   What  ar e  som e  of   the  char acter i sti c  changes  that  af f ect  the  ar ti cul ar
car ti l age  i n  pati ents  w i th  OA?

Abnor m al   joi nt  m echani cal   f actor s  r esul t  i n  pi ts,   cl ef ts,   and  ul cer ati ons
i n  the  gr oss  ar ti cul ar   car ti l age  sur f ace  i n  OA.   Mi cr oscopi cal l y ,
osteoar thr i ti c  car ti l age  r ev eal s  i ni ti al   chondr ocy te  pr ol i f er ati on
f ol l ow ed  by   ev entual   chondr ocy te  death;  decr eased  pr oteogl y can  and
col l agen  concentr ati ons  w i th  r esul tant  i ncr eased  w ater   content  of   the
car ti l age;  i ncr eased  am ounts  of   m atr i x   m etal l opr otei nases  (MMPs)  and
i nf l am m ator y   m edi ator s;  and  decr eased  am ounts  of   ti ssue  i nhi bi tor s  of
m etal l opr otei nases  (TIMPs).   Thi s  r esul ts  i n  car ti l age  l oss  w i th
secondar y   thi ck eni ng  of   the  subchondr al   bone  and  f or m ati on  of
osteophy tes.

2.   What  ar e  f our   char acter i sti c  r adi ogr aphi c  f i ndi ngs  encounter ed  i n
pati ents  w i th  OA?
Radi ogr aphi c  f i ndi ngs  ty pi cal l y   encounter ed  i n  pati ents  w i th  OA  i ncl ude
l oss  of   joi nt  space,   cy sts  i n  subchondr al   bone,   subchondr al   scl er osi s  or
ebur nati on,   and  osteophy tes  (bony   spur s)  at  the  joi nt  m ar gi ns.

3.   Di scuss  the  nonphar m acol ogi c  m anagem ent  of   OA?

N onphar m acol ogi c  m odal i ti es  hel pf ul   i n  the  m anagem ent  of   OA  consi st
of   pati ent  educati on,   heat  or   col d  appl i cati on,   w ei ght  r educti on,
phy si cal   ther apy   that  f ocuses  on  m uscl e­str engtheni ng  ex er ci ses,
or thoti cs  and  br aci ng,   and  or thopaedi c  sur gi cal   opti ons  i n  sel ect
pati ents.

P. 445
4.   Di scuss  the  phar m acol ogi c  opti ons  f or   the  tr eatm ent  of   OA?

Acetam i nophen,   an  anal gesi c,   shoul d  be  the  f i r st­l i ne  ther apy   f or   OA.   If
thi s  i s  unsuccessf ul ,   N SAIDs  can  be  used.   N ar coti c  anal gesi cs  shoul d  be
consi der ed  i n  pati ents  w i th  r ef r actor y   pai n.   Topi cal   appl i cati on  of
capsai ci n  cr eam   or   i ntr aar ti cul ar   i njecti on  of   hy al ur onate  or
cor ti coster oi ds  m ay   be  benef i ci al   i n  som e  pati ents.   The  nutr aceuti cal s,
gl ucosam i ne  and  chondr oi ti n  sul f ate,   m ay   hav e  som e  benef i t  i n  tr eati ng
OA  sy m ptom s.   Long­ter m   chondr opr otecti v e  ef f ects  of   these  agents
hav e  not  been  establ i shed.

Suggested Readings
Al tm an  RD,   Lozada  CJ.   Osteoar thr i ti s  and  r el ated  di sor der s:  cl i ni cal
f eatur es.   In:  Hochber g  MC,   Si l m an  AJ,   Sm ol en  JS,   et  al .   eds.
Rheum atol ogy ,   3r d  ed.   Edi nbur gh:  Mosby ,   2003:1793–1800.

Dougados  M.   Cl i ni cal   f eatur es  of   osteoar thr i ti s.   In:  Har r i s  ED  Jr ,   Budd
RC,   Fi r estei n  GS,   et  al .   eds.   Kel l ey 's  tex tbook   of   r heum atol ogy ,   7th  ed.
Phi l adel phi a:  El sev i er ,   Saunder s,   2005:1514–1527.

Lozada  CJ.   Managem ent  of   osteoar thr i ti s.   In:  Har r i s  ED  Jr ,   Budd  RC,
Fi r estei n  GS,   et  al .   eds.   Kel l ey 's  tex tbook   of   r heum atol ogy ,   7th  ed.
Phi l adel phi a:  El sev i er ,   Saunder s,   2005:1528–1540.

Vogel gesang  S.   Osteoar thr i ti s.   In:  West  SG,   ed.   Rheum atol ogy   secr ets,
2nd  ed.   Phi l adel phi a:  Hanl ey   &  Bel f us,   2002:365–374.

Polymyositis and Dermatomyositis
1.   What  thr ee  gener al   categor i es  of   joi nt  or   m uscl e  di sease  need  to  be
consi der ed  i n  a  pati ent  pr esenti ng  w i th  di f f use  aches  and  m uscl e
w eak ness?

2.   What  ar e  the  f i v e  subgr oups  of   i nf l am m ator y   m uscl e  di sease?

3.   What  hi stor i cal   i nf or m ati on  w oul d  suggest  the  pr esence  of


i nf l am m ator y   m uscl e  di sease?

4.   What  tw o  l abor ator y   test  r esul ts  m i ght  be  abnor m al   i n  pati ents  w i th
i nf l am m ator y   m uscl e  di sease?

5.   What  f our   di agnosti c  tests  or   pr ocedur es  shoul d  be  per f or m ed  i n  any
pati ent  w i th  suspected  i nf l am m ator y   m uscl e  di sease?

Discussion
1.   What  thr ee  gener al   categor i es  of   joi nt  or   m uscl e  di sease  need  to  be
consi der ed  i n  a  pati ent  pr esenti ng  w i th  di f f use  aches  and  m uscl e
w eak ness?

A  pati ent  w i th  di f f use  aches  and  m uscl e  w eak ness  m ay   hav e  a  f or m   of
i nf l am m ator y   ar thr i ti s,   par ti cul ar l y   RA;  an  endocr i nopathy ,   par ti cul ar l y
thy r oi d  or   par athy r oi d  di sease;  or   a  f or m   of   i nf l am m ator y   m uscl e
di sease.   The  di f f er enti al   di agnosi s  al so  i ncl udes  neur opathi c  di seases,
m edi cati ons,   i nf ecti ons,   m etabol i c  m y opathi es,   and  neopl asi a.

P. 446
2.   What  ar e  the  f i v e  subgr oups  of   i nf l am m ator y   m uscl e  di sease?

Inf l am m ator y   m uscl e  di sease  can  be  di v i ded  i nto  the  f ol l ow i ng


di sor der s:  pr i m ar y   i di opathi c  pol y m y osi ti s,   pr i m ar y   i di opathi c
der m atom y osi ti s,   chi l dhood  der m atom y osi ti s  associ ated  w i th  v ascul i ti s,
pol y m y osi ti s  and  der m atom y osi ti s  associ ated  w i th  col l agen  v ascul ar
di sease  such  as  SLE  or   scl er oder m a,   and  pol y m y osi ti s  and
der m atom y osi ti s  associ ated  w i th  m al i gnancy .

3.   What  hi stor i cal   i nf or m ati on  w oul d  suggest  the  pr esence  of


i nf l am m ator y   m uscl e  di sease?

Inf l am m ator y   m uscl e  di sease  has  an  i nsi di ous  onset  ov er   3  to  6  m onths
usual l y   w i th  no  i denti f i abl e  pr eci pi tati ng  ev ent.   The  w eak ness  i ni ti al l y
af f ects  the  m uscl es  of   the  shoul der   and  pel v i c  gi r dl e.   The  pati ents  m ay
ex per i ence  di f f i cul ty   i n  cl i m bi ng  stai r s,   getti ng  out  of   chai r s,   or
com bi ng  thei r   hai r .   Weak ness  of   neck   f l ex or s  occur s  i n  appr ox i m atel y
50%  of   pati ents.   Phar y ngeal   m uscl e  i nv ol v em ent  m ay   cause  dy sphoni a,
dy sphagi a,   or   aspi r ati on.   Ocul ar ,   f aci al ,   and  bul bar   m uscl e  w eak ness  i s
ex tr em el y   r ar e.

4.   What  tw o  l abor ator y   test  r esul ts  m i ght  be  abnor m al   i n  pati ents  w i th
i nf l am m ator y   m uscl e  di sease?

Tw o  abnor m al   l abor ator y   test  f i ndi ngs  i n  pati ents  w i th  pol y m y osi ti s  or
der m atom y osi ti s  ar e  el ev ati ons  i n  the  ESR  and  the  ser um   CPK  l ev el .
Appr ox i m atel y   50%  of   pati ents  hav e  a  posi ti v e  AN A.   My osi ti s­speci f i c
anti bodi es,   such  as  anti –Jo­1,   can  occur   i n  a  subset  of   pati ents  and
can  pr edi ct  cl i ni cal   m ani f estati ons  (m y osi ti s,   i nter sti ti al   l ung  di sease,
noner osi v e  ar thr i ti s,   Ray naud's  phenom enon,   and  m echani c's  hands)
and  pr ognosi s.

5.   What  f our   di agnosti c  tests  or   pr ocedur es  shoul d  be  per f or m ed  i n  any
pati ent  w i th  suspected  i nf l am m ator y   m uscl e  di sease?

The  di agnosti c  ev al uati on  of   pati ents  w i th  suspected  i nf l am m ator y


m uscl e  di sease  shoul d  i ncl ude  ser ol ogi c  testi ng  f or   AN A  subty pes  to
r ul e  out  a  m y osi ti s  ov er l ap  sy ndr om e,   el ectr ocar di ogr aphy   to  scr een
f or   car di ac  i nv ol v em ent,   el ectr om y ogr aphy   (EMG)  to  conf i r m   a
m y opathi c  pr ocess,   and  m uscl e  bi opsy   to  conf i r m   the  suspected
di agnosi s.

Case
A  47­y ear ­ol d  w om an  i s  seen  by   her   pr i m ar y   car e  phy si ci an  w i th  a  chi ef
com pl ai nt  of   a  3­m onth  hi stor y   of   m uscl e  w eak ness  al ong  w i th  v ague
com pl ai nts  of   decr eased  ener gy   and  di f f use  aches  and  pai ns.   Routi ne
phy si cal   ex am i nati on  f i ndi ngs  ar e  unr em ar k abl e.   The  r esul ts  of   a  basel i ne
bi ochem i cal   scr een  i ncl udi ng  thy r oi d  f uncti on  studi es  ar e  w i thi n  nor m al
l i m i ts.   El ectr ocar di ogr aphy ,   a  chest  r adi ogr aphi c  study ,   and  pul m onar y
f uncti on  test  r esul ts  ar e  al so  unr ev eal i ng.   She  i s  gi v en  an  em pi r i c  tr i al   of
napr ox en.
Tw o  m onths  l ater ,   she  begi ns  to  ex per i ence  actual   m uscl e  tender ness  and
di f f i cul ty   cl i m bi ng  the  tw o  f l i ghts  of   stai r s  to  her   apar tm ent.   On  questi oni ng,
she  al so  com pl ai ns  of   pai n,   di f f i cul ty   i n  chew i ng  m eats,   and  an  8­l b  (3. 6­k g)
w ei ght  l oss.   She  deni es  f ev er s,   chest  pai n,   shor tness  of   br eath,   a  change  i n
bow el   habi ts,   or   sk i n  r ashes.
Phy si cal   ex am i nati on  r ev eal s  gr ade  4/5  str ength  i n  the  pr ox i m al   m uscl e
gr oups  of   both  the  upper   and  l ow er   ex tr em i ti es  w i thout  atr ophy .   Ther e  i s
al so  gr ade  4/5  w eak ness
P. 447
of   the  neck   f l ex or s.   Her   di stal   str ength  i s  nor m al .   Her   r ef l ex es  ar e
sy m m etr i c.   Her   sk i n  i s  cl ear .   Br east  and  pel v i c  ex am i nati on  f i ndi ngs  ar e
unr em ar k abl e.
The  f ol l ow i ng  l abor ator y   r esul ts  ar e  r epor ted:  hem atocr i t  34%;  ESR  63  m m
per   hour ;  AN A  1:256  f i ne  speck l ed  patter n;  r heum atoi d  f actor   (RF)  negati v e;
CPK  1, 850  U /L  (nor m al   < 150  U /L).
She  i s  schedul ed  to  under go  r i ght­si ded  EMG  and  m uscl e  bi opsy   of   the  l ef t
tr i ceps.

1.   What  other   or gans  besi de  m uscl e  m ay   be  i nv ol v ed  i n  pati ents  w i th
pol y m y osi ti s  or   der m atom y osi ti s?
2.   What  f our   di f f er ent  sk i n  l esi ons  ar e  seen  i n  pati ents  w i th
der m atom y osi ti s?
3.   What  di agnosti c  ev al uati on  i s  i ndi cated  to  sear ch  f or   a  possi bl e
m al i gnancy   i n  pati ents  w i th  pol y m y osi ti s  or   der m atom y osi ti s,   and  w hat
m ay   happen  to  the  m uscl e  di sease  w hen  the  m al i gnancy   i s  tr eated?
4.   What  i s  the  appr oach  to  tr eatm ent  of   pol y m y osi ti s/der m atom y osi ti s?

Case Discussion
1.   What  other   or gans  besi de  m uscl e  m ay   be  i nv ol v ed  i n  pati ents  w i th
pol y m y osi ti s  or   der m atom y osi ti s?

The  l ungs,   hear t,   and  joi nts  m ay   al so  be  i nv ol v ed  i n  pati ents  w i th
pol y m y osi ti s  or   der m atom y osi ti s.   Pul m onar y   i nv ol v em ent  i ncl udes
i nter sti ti al   l ung  di sease,   aspi r ati on  pneum oni a,   r espi r ator y   m uscl e
w eak ness,   and  pul m onar y   hy per tensi on.   Car di ac  m ani f estati ons  ar e
dy sr hy thm i as,   conducti on  bl ock s,   and  m y ocar di ti s.   Pati ents  m ay
ex per i ence  pol y ar thr al gi a  or   an  i nf l am m ator y   ar thr i ti s.   Char acter i sti c
sk i n  f i ndi ngs  (di scussed  i n  subsequent  tex t)  ar e  r equi r ed  f or   a
di agnosi s  of   der m atom y osi ti s.

2.   What  f our   di f f er ent  sk i n  l esi ons  ar e  seen  i n  pati ents  w i th


der m atom y osi ti s?

The  sk i n  l esi ons  seen  i n  pati ents  w i th  der m atom y osi ti s  i ncl ude  an
er y them atous  r ash  ov er   the  anter i or   chest  and  neck   (V­si gn  r ash),   an
er y them atous  r ash  ov er   the  shoul der s  and  pr ox i m al   ar m s  (shaw l ­si gn
r ash),   er y them atous  r ai sed  l esi ons  ov er   the  k nuck l es  (Gottr on's
papul es),   and  a  per i or bi tal   l i l ac  col or ed  r ash  (hel i otr ope  r ash).
Gottr on's  papul es  and  the  hel i otr ope  r ash  ar e  consi der ed
pathognom oni c  cutaneous  f eatur es  of   der m atom y osi ti s.   Mechani c's
hands  (cr ack i ng  and/or   f i ssur i ng  of   the  sk i n  of   the  f i nger   pads)  can  be
seen  i n  the  anti sy nthetase  sy ndr om e  of ten  associ ated  w i th  anti –Jo­1
anti bodi es.

3.   What  di agnosti c  ev al uati on  i s  i ndi cated  to  sear ch  f or   a  possi bl e
m al i gnancy   i n  pati ents  w i th  pol y m y osi ti s  or   der m atom y osi ti s,   and  w hat
m ay   happen  to  the  m uscl e  di sease  w hen  the  m al i gnancy   i s  tr eated?

Mal i gnanci es  m ay   dev el op  i n  pati ents  w i th  pol y m y osi ti s  or
der m atom y osi ti s,   ei ther   bef or e  or   af ter   (3  to  5  y ear s)  the  onset  of
i nf l am m ator y   m uscl e  di sease.   The  di agnosti c  ev al uati on  f or   a  possi bl e
m al i gnancy   i n  thi s  setti ng  shoul d  be  age  appr opr i ate  and  usual l y
i ncl udes  a  good  hi stor y   and  phy si cal   ex am i nati on  (i ncl udi ng  br east,
pel v i s,   and  pr ostate),   a  chest  r adi ogr aphi c  study ,   m am m ogr aphy ,   stool
guai ac  testi ng,   and  r outi ne  l abor ator y   tests.   The  m al i gnanci es  f ound  i n
these  pati ents  i ncl ude  am ong  other s  car ci nom as  of   the  l ung,
gastr oi ntesti nal   tr act,   br east,   ov ar i es,
P. 448
and  pancr eas,   and  Hodgk i n's  l y m phom a.   If   the  m al i gnancy   i s  tr eated,
the  m uscl e  di sease  m ay   i m pr ov e.
4.   What  i s  the  appr oach  to  tr eatm ent  of   pol y m y osi ti s/der m atom y osi ti s?

The  tr eatm ent  of   pol y m y osi ti s  and  der m atom y osi ti s  shoul d  f i r st  consi st
of   sy stem i c  cor ti coster oi ds  gi v en  i n  hi gh  doses.   If   pati ents  show   a  poor
r esponse  to  ster oi ds  or   i f   the  dosage  cannot  be  decr eased,
i m m unosuppr essi v e  dr ug  tr eatm ent  w i th  such  agents  as  azathi opr i ne  or
MTX  m ay   be  i nsti tuted.   Hy dr ox y chl or oqui ne  can  be  used  to  tr eat  the
cutaneous  m ani f estati ons  of   der m atom y osi ti s.   Ref r actor y   cases  of
i nf l am m ator y   m y opathi es  m ay   r espond  to  i ntr av enous  i m m une  gl obul i n.
Pr ogr essi v e  phy si cal   ther apy   i s  r ecom m ended  to  m ai ntai n  r ange  of
m oti on,   pr ev ent  contr actur es,   and,   as  m uscl e  i nf l am m ati on  subsi des,   to
r egai n  m uscl e  str ength.

Suggested Readings
Oddi s  CV,   Medsger   TA  Jr .   Inf l am m ator y   m uscl e  di sease:  cl i ni cal
f eatur es.   In:  Hochber g  MC,   Si l m an  AJ,   Sm ol en  JS,   et  al .   eds.
Rheum atol ogy ,   3r d  ed.   Edi nbur gh:  Mosby ,   2003:1537–1554.

Spencer   RT.   Inf l am m ator y   m uscl e  di sease.   In:  West  SG,   ed.
Rheum atol ogy   secr ets,   2nd  ed.   Phi l adel phi a:  Hanl ey   &  Bel f us,   2002:167â
€“173.

Wor tm ann  RL.   Inf l am m ator y   di sease  of   m uscl e  and  other   m y opathi es.   In:
Har r i s  ED  Jr ,   Budd  RC,   Fi r estei n  GS,   et  al .   eds.   Kel l ey 's  tex tbook   of
r heum atol ogy ,   7th  ed.   Phi l adel phi a:  El sev i er ,   Saunder s,   2005:1309â
€“1335.

Reactive Arthritis
1.   What  i s  r eacti v e  ar thr i ti s?

2.   In  w hat  tw o  di seases  i s  ar thr i ti s  associ ated  w i th  di ar r hea?

3.   What  tw o  possi bl e  di agnoses  ar e  suggested  w hen  acute  ar thr i ti s  occur s


i n  a  pati ent  w i th  ur ethr al   di schar ge?

4.   What  ar e  the  hi stor y   and  phy si cal   ex am i nati on  f i ndi ngs  ty pi cal l y
obser v ed  i n  pati ents  w i th  r eacti v e  ar thr i ti s?

Discussion
1.   What  i s  r eacti v e  ar thr i ti s?

Reacti v e  ar thr i ti s  i s  a  ster i l e  i nf l am m ator y   sy nov i ti s  f ol l ow i ng  a  di stant


i nf ecti on  by   an  or gani sm   that  i nf ects  m ucosal   sur f aces,   par ti cul ar l y
ur ogeni tal   or   enter i c  i nf ecti ons.   Reacti v e  ar thr i ti s  has  r epl aced  the
ter m   Rei ter 's  sy ndr om e  as  m ost  pati ents  do  not  hav e  the  Rei ter 's
sy ndr om e's  cl assi c  tr i ad  of   ar thr i ti s,   conjuncti v i ti s,   and  ur ethr i ti s.

2.   In  w hat  tw o  di seases  i s  ar thr i ti s  associ ated  w i th  di ar r hea?

Ar thr i ti s  associ ated  w i th  di ar r hea  m ay   be  seen  i n  the  setti ng  of   ei ther
r eacti v e  ar thr i ti s  or   i nf l am m ator y   bow el   di sease.   In  r eacti v e  ar thr i ti s,
the
P. 449
di ar r hea  m ay   pr ecede  the  ar thr i ti s  by   a  f ew   w eek s.   In  i nf l am m ator y
bow el   di sease,   the  per i pher al   ar thr i ti s  and  di ar r hea  of ten  ar i se  at  the
sam e  ti m e  and  the  cl i ni cal   acti v i ty   of   the  ar thr i ti s  m ay   cor r el ate  w i th
the  acti v i ty   of   the  i nf l am m ator y   bow el   di sease.

3.   What  tw o  possi bl e  di agnoses  ar e  suggested  w hen  acute  ar thr i ti s  occur s


i n  a  pati ent  w i th  ur ethr al   di schar ge?

Acute  ar thr i ti s  occur r i ng  i n  a  pati ent  w i th  a  ur ethr al   di schar ge  suggests
a  di agnosi s  of   ei ther   di ssem i nated  gonococcal   i nf ecti on  or   r eacti v e
ar thr i ti s.   These  di agnoses  can  be  di f f er enti ated  on  the  basi s  of
char acter i sti c  cl i ni cal   f eatur es  as  w el l   as  by   a  posi ti v e  ur ethr al   or
cer v i cal   cul tur e  f or   N ei sser i a  gonor r hoeae.   The  ur ethr i ti s  associ ated
w i th  r eacti v e  ar thr i ti s  can  pr esent  as  an  asepti c  py ur i a  or   be  secondar y
to  an  i nf ecti on  w i th  Chl am y di a  or   U r eapl asm a.

4.   What  ar e  the  hi stor y   and  phy si cal   ex am i nati on  f i ndi ngs  ty pi cal l y
obser v ed  i n  pati ents  w i th  r eacti v e  ar thr i ti s?

Reacti v e  ar thr i ti s  i s  di agnosed  on  the  basi s  of   hi stor y   and  phy si cal
ex am i nati on  f i ndi ngs  and  not  on  the  basi s  of   any   l abor ator y   r esul t.
These  cl i ni cal   f i ndi ngs  i ncl ude  the  dev el opm ent  of   an  acute  ar thr i ti s  i n
one  or   a  f ew   joi nts,   of ten  of   the  l ow er   ex tr em i ti es,   af ter   an  epi sode  of
ei ther   di ar r hea,   or   pai nl ess  ur ethr i ti s  or   cer v i ci ti s.   The  di agnosi s  m ay
be  f ur ther   str engthened  by   the  pr esence  of   or al   ul cer s,   conjuncti v i ti s,
or   anter i or   uv ei ti s,   as  w el l   as  char acter i sti c  sk i n  f i ndi ngs  of   ci r ci nate
bal ani ti s  or   k er atoder m a  bl ennor r hagi cum .   Enthesopathi es  (dacty l i ti s,
pl antar   f asci i ti s,   and  Achi l l es  tendi ni ti s)  and  tenosy nov i ti s  can  al so  be
com m on  cl i ni cal   f eatur es  of   r eacti v e  ar thr i ti s.

Case
A  32­y ear ­ol d  m an  i s  seen  because  of   i ncr easi ng  r i ght  k nee  pai n  and
sw el l i ng  ov er   the  l ast  3  day s.   On  f ur ther   questi oni ng,   i t  i s  di scov er ed  that  2
w eek s  ago,   the  pati ent  had  an  epi sode  of   m i l d  dy sur i a  associ ated  w i th  a
m ucous  di schar ge.   Thi s  i l l ness  r esol v ed  spontaneousl y   af ter   4  day s.   Si x
day s  ago,   pai nl ess,   shal l ow   ul cer ati ons  of   the  gl ans  peni s  dev el oped.   Dur i ng
thi s  per i od,   he  al so  noted  the  onset  of   bi l ater al   r edness  and  pr ur i tus  of   the
ey es  al ong  w i th  a  cl ear   di schar ge.   Thr ee  day s  ago,   acute  sw el l i ng  of   the
r i ght  k nee  associ ated  w i th  pai n  ar ose  spontaneousl y   and  has  steadi l y
w or sened.
Phy si cal   ex am i nati on  r ev eal s  m i l d  i njecti on  of   the  conjuncti v al   v essel s
bi l ater al l y .   Hi s  v i sual   acui ty   and  r eti na  ar e  nor m al .   Sl i t­l am p  ex am i nati on
by   ophthal m ol ogy   dem onstr ates  no  ev i dence  of   anter i or   uv ei ti s.   Ex am i nati on
of   the  sk i n  r ev eal s  di scr ete  hy per k er atoti c  nodul es  ov er   the  sol es  of   hi s  f eet
bi l ater al l y   and  ther e  ar e  thr ee  shal l ow   ul cer s  on  the  gl ans  peni s.   Hi s  r i ght
k nee  i s  w ar m   and  tender ,   and  ther e  i s  a  si gni f i cant  am ount  of   pal pabl e
sy nov i al   f l ui d.   The  r em ai nder   of   the  ex am i nati on  f i ndi ngs  ar e  unr em ar k abl e.

1.   What  other   f or m s  of   r heum ati c  di sease  need  to  be  consi der ed  w hen
r eacti v e  ar thr i ti s  i s  suspected,   and  w hat  di agnosti c  tests  or   pr ocedur es
shoul d  be  per f or m ed  to  ex cl ude  them ?
2.   What  ar e  som e  of   the  cl i ni cal   or   l abor ator y   char acter i sti cs  of   r eacti v e
ar thr i ti s  that  hel p  di f f er enti ate  i t  f r om   RA?
3.   What  ar e  the  thr ee  ty pes  of   sk i n  l esi ons  seen  i n  pati ents  w i th  r eacti v e
ar thr i ti s?
P. 450
4.   The  back   di sease  i n  pati ents  w i th  r eacti v e  ar thr i ti s  i s  char acter i zed  by
w hat  r adi ogr aphi c  f i ndi ngs?
5.   What  i s  the  ther apy   f or   r eacti v e  ar thr i ti s?

Case Discussion
1.   What  other   f or m s  of   r heum ati c  di sease  need  to  be  consi der ed  w hen
r eacti v e  ar thr i ti s  i s  suspected,   and  w hat  di agnosti c  tests  or   pr ocedur es
shoul d  be  per f or m ed  to  ex cl ude  them ?

In  a  pati ent  suspected  of   hav i ng  r eacti v e  ar thr i ti s,   septi c  ar thr i ti s
needs  to  be  ex cl uded.   The  f i ndi ngs  y i el ded  by   joi nt  aspi r ati on,   w hi ch
i ncl udes  ex am i nati on  of   the  f l ui d  f or   cel l   count  w i th  di f f er enti al ,
together   w i th  Gr am 's  stai ni ng  and  cul tur e,   can  cl i nch  the  di agnosi s  of   a
nongonococcal   bacter i al   septi c  joi nt.   Gonococcal   ar thr i ti s  i s  another
possi bl e  di agnosi s.   Besi de  ex am i nati on  and  cul tur e  of   sy nov i al   f l ui d,
the  ev al uati on  shoul d  i ncl ude  ur ethr al   or   cer v i cal ,   bl ood,   phar y ngeal ,
and  per i r ectal   cul tur es.   Cr y stal ­i nduced  ar thr i ti s  i s  di agnosed  by   the
f i ndi ng  of   cr y stal s  i n  the  sy nov i al   f l ui d  by   pol ar i zed  m i cr oscopy .   Both
SLE  and  RA  need  to  be  consi der ed.   Ser ol ogi c  testi ng  f or   AN A,   RF,   and
anti cy cl i c  ci tr ul l i nated  pepti de  (anti ­CCP)  anti bodi es  i s  per f or m ed  to  ai d
i n  the  di agnosi s  of   these  condi ti ons.   A  r outi ne  com pl ete  bl ood  count
and  a  f ul l   chem i str y   pr of i l e,   i ncl udi ng  l i v er   f uncti on  tests,   shoul d  al so
be  per f or m ed  to  sear ch  f or   a  sy stem i c  di sease  that  m ay   pr esent  w i th
joi nt  f i ndi ngs  si m i l ar   to  those  seen  i n  the  setti ng  of   r eacti v e  ar thr i ti s.
Reacti v e  ar thr i ti s  can  be  associ ated  w i th  hum an  i m m unodef i ci ency
v i r us  (HIV)  i nf ecti on.   The  m ost  usef ul   l abor ator y   tests  i n  r eacti v e
ar thr i ti s  ar e  sw abs  or   cul tur es  that  conf i r m   the  pr esence  of
ar thr i togeni c  or gani sm s  such  as  Chl am y di a,   U r eapl asm a,   Sal m onel l a,
Shi gel l a,   Yer si ni a,   Cam py l obacter ,   and  Cl ostr i di um   di f f i ci l e  i n  the
ur ogeni tal   or   gastr oi ntesti nal   tr acts.

2.   What  ar e  som e  of   the  cl i ni cal   or   l abor ator y   char acter i sti cs  of   r eacti v e
ar thr i ti s  that  hel p  di f f er enti ate  i t  f r om   RA?

In  contr ast  to  RA,   an  asy m m etr i c  ar thr i ti s  that  pr edom i nates  i n  the
l ow er   ex tr em i ti es  i s  char acter i sti c  of   r eacti v e  ar thr i ti s.   In  addi ti on,
sacr oi l i i ti s  (of ten  uni l ater al   or   asy m m etr i c)  af f ects  20%  to  30%  of
pati ents,   the  sy ndr om e  i s  associ ated  w i th  HLA­B27  (80%  of   pati ents),
and  pati ents  f r equentl y   hav e  an  enthesopathy .   Si nce  r eacti v e  ar thr i ti s
i s  one  of   the  ser ol ogi cal l y   negati v e  spondy l oar thr opathi es,   the  AN A,   RF,
and  anti ­CCP  anti bodi es  ar e  negati v e.

3.   What  ar e  the  thr ee  ty pes  of   sk i n  l esi ons  seen  i n  pati ents  w i th  r eacti v e
ar thr i ti s?

The  sk i n  l esi ons  of   r eacti v e  ar thr i ti s  i ncl ude  pai nl ess  m outh  ul cer s,
k er atoder m a  bl ennor r hagi cum   (psor i af or m   l esi ons  on  the  sol es  of   the
f eet;  m ay   al so  i nv ol v e  the  scr otum ,   peni s,   pal m s,   tr unk ,   and  scal p),
and  ci r ci nate  bal ani ti s  (ser pi gi nous  ul cer ati on  of   the  gl ans  peni s).   The
l atter   tw o  condi ti ons  ar e  pr edom i nantl y   associ ated  w i th  ur ogeni tal
r eacti v e  ar thr i ti s.

4.   The  back   di sease  i n  pati ents  w i th  r eacti v e  ar thr i ti s  i s  char acter i zed  by
w hat  r adi ogr aphi c  f i ndi ngs?

The  l um bosacr al   spi ne  f i l m   of   a  pati ent  w i th  r eacti v e  ar thr i ti s  m ay


show   asy m m etr i c  and  bul k y   sy ndesm ophy tes.   Thi s  i s  i n  contr ast  w i th
AS,   i n  w hi ch  the
P. 451
sy ndesm ophy tes  ar e  usual l y   sy m m etr i c  and  f l ow i ng.   In  r eacti v e
ar thr i ti s,   sacr oi l i i ti s,   i f   pr esent,   i s  of ten  uni l ater al   and  asy m m etr i c.

5.   What  i s  the  ther apy   f or   r eacti v e  ar thr i ti s?

Pati ents  w i th  r eacti v e  ar thr i ti s  ar e  i ni ti al l y   tr eated  w i th  N SAIDs


(ty pi cal l y   i ndom ethaci n)  together   w i th  appr opr i ate  anti bi oti cs  dur i ng
the  acute  phase,   par ti cul ar l y   i f   ur ethr i ti s  or   cer v i ci ti s  i s  pr esent.   If   the
di sease  pr ogr esses  despi te  N SAID  tr eatm ent,   sul f asal azi ne  or   MTX  m ay
be  of   v al ue  f or   m anagi ng  the  i nf l am m ator y   ar thr i ti s.   Intr aar ti cul ar
cor ti coster oi ds  m ay   be  hel pf ul   but  sy stem i c  cor ti coster oi ds  ar e  usual l y
i nef f ecti v e.   The  TN F­Î±   bl ock i ng  dr ugs  ar e  v er y   ef f ecti v e  i n  r ef r actor y
cases  of   r eacti v e  ar thr i ti s.   Topi cal   cor ti coster oi ds  and  k er atol y ti c
agents  ar e  usef ul   f or   k er atoder m a  bl ennor r hagi cum .   Phy si cal   ther apy
consi sti ng  of   heat,   ul tr asound,   and  r ange­of ­m oti on  ex er ci ses  m ay   be
hel pf ul   i n  pati ents  w i th  r eacti v e  ar thr i ti s.

Suggested Readings
Meehan  RT.   Rei ter 's  sy ndr om e  and  r eacti v e  ar thr i ti des.   In:  West  SG,   ed.
Rheum atol ogy   secr ets,   2nd  ed.   Phi l adel phi a:  Hanl ey   &  Bel f us,   2002:269â
€“275.

Tak   Yan  Yu  D,   Fan  PT.   Rei ter 's  sy ndr om e,   undi f f er enti ated
spondy l oar thr opathy ,   and  r eacti v e  ar thr i ti s.   In:  Har r i s  ED  Jr ,   Budd  RC,
Fi r estei n  GS,   et  al .   eds.   Kel l ey 's  tex tbook   of   r heum atol ogy ,   7th  ed.
Phi l adel phi a:  El sev i er ,   Saunder s,   2005:1142–1154.

Toi v anen  A.   Reacti v e  ar thr i ti s:  cl i ni cal   f eatur es  and  tr eatm ent.   In:
Hochber g  MC,   Si l m an  AJ,   Sm ol en  JS,   et  al .   eds.   Rheum atol ogy ,   3r d  ed.
Edi nbur gh:  Mosby ,   2003:1233–1240.

Rheumatoid Arthritis
1.   What  f our   char acter i sti cs  of   RA  hel p  di sti ngui sh  i t  f r om   OA?

2.   What  consti tuti onal   sy m ptom s  m ay   be  seen  i n  RA?

3.   What  ar e  thr ee  char acter i sti c  phy si cal   f i ndi ngs  i n  RA?

4.   What  f i v e  di seases  m ay   m i m i c  RA?

5.   Whi ch  ser ol ogi c  tests  m ay   be  usef ul   i n  the  di agnosi s  of   RA?

Discussion
1.   What  f our   char acter i sti cs  of   RA  hel p  di sti ngui sh  i t  f r om   OA?

U nl i k e  pati ents  w i th  OA  (noni nf l am m ator y ),   those  w i th  RA


(i nf l am m ator y )  ex per i ence  m or ni ng  sti f f ness  l asti ng  m or e  than  30
m i nutes  pl us  gel   phenom enon  (w or se  sti f f ness  af ter   r est);  sy m m etr i c
joi nt  di sease;  char acter i sti c  bi l ater al   sy nov i ti s  of   the  hands  and  f eet
(PIPs,   MCPs,   and  MTPs);  and  an  i nter m i ttent  or   w ax i ng  and  w ani ng
cour se.

2.   What  consti tuti onal   sy m ptom s  m ay   be  seen  i n  RA?

Most  pati ents  ex per i ence  gener al i zed  m al ai se  or   f ati gue.   Occasi onal l y
w ei ght  l oss,   l ow ­gr ade  f ev er ,   sl eep  di stur bance,   or   m i l d
l y m phadenopathy   m ay   be  pr esent.   These  sy m ptom s  m ay   be  the  end
r esul t  of   ci r cul ati ng
P. 452
i nf l am m ator y   cy tok i nes  pr oduced  i n  the  i nf l am ed  sy nov i al   ti ssue  of   the
af f ected  joi nts.

3.   What  ar e  thr ee  char acter i sti c  phy si cal   f i ndi ngs  i n  RA?
Phy si cal   f i ndi ngs  encounter ed  i n  the  setti ng  of   RA  m ay   i ncl ude  sw el l i ng
and  w ar m th  of   one  or   m or e  joi nts  ty pi cal l y   i n  a  sy m m etr i c  di str i buti on,
tender ness  on  pal pati on  of   the  sw ol l en  joi nts,   and  the  pr esence  of
nontender   subcutaneous  nodul es  (r heum atoi d  nodul es)  ov er   the
ex tensor   sur f ace  of   the  f or ear m ,   Achi l l es  tendon,   and  di gi ts  of   the
hands.

4.   What  f i v e  di seases  m ay   m i m i c  RA?

RA  m ay   be  m i m i ck ed  by   SLE  and  other   CTDs  such  as  m i x ed  connecti v e
ti ssue  di sease  (MCTD),   scl er oder m a,   and  PMR;  pol y ar ti cul ar   gout  or
pseudogout;  the  ar thr i ti s  of   subacute  bacter i al   endocar di ti s;  the
ar thr i ti s  secondar y   to  m al i gnancy ;  and  the  ser onegati v e
spondy l oar thr opathi es.   The  di agnosi s  of   RA  i s  based  on  the  hi stor y ,
phy si cal   ex am i nati on,   and  l abor ator y   f i ndi ngs.

5.   Whi ch  ser ol ogi c  tests  m ay   be  usef ul   i n  the  di agnosi s  of   RA?

RFs  ar e  autoanti bodi es  di r ected  agai nst  the  Fc  por ti on  of   IgG.   In  RA,   RF
has  a  sensi ti v i ty   of   appr ox i m atel y   80%  and  speci f i ci ty   of   80%.
Ther ef or e,   RF  i s  detected  i n  appr ox i m atel y   80%  of   pati ents  w i th  RA  but
i t  i s  nonspeci f i c  and  can  be  detected  i n  m any   other   di sor der s  such  as
other   CTDs  and  chr oni c  v i r al   or   bacter i al   i nf ecti ons.   Anti ­CCP
anti bodi es  ar e  di r ected  agai nst  ci tr ul l i ne­m odi f i ed  ar gi ni ne  r esi dues  i n
a  pr otei n.   In  RA,   anti ­CCP  anti bodi es  hav e  a  sensi ti v i ty   of   60%  to  75%
and  a  hi gh  speci f i ci ty   of   90%  to  96%.   Ther ef or e,   anti ­CCP  anti bodi es
ar e  usual l y   detected  onl y   i n  RA.   Pati ents  w i th  RA  w ho  hav e  a  posi ti v e
RF  and/or   anti ­CCP  anti bodi es  ar e  at  a  hi gher   r i sk   of   dev el opi ng
er osi v e  joi nt  destr ucti on  and  debi l i ty .   An  el ev ated  ESR  or   C­r eacti v e
pr otei n  (CRP)  l ev el   suggests  the  pr esence  of   an  acute  i nf l am m ator y
di sease.   A  com pl ete  bl ood  count  m ay   show   an  anem i a  of   chr oni c
(i nf l am m ator y )  di sease.   AN As  ar e  f ound  i n  30%  of   pati ents  w i th  RA,
usual l y   i n  a  l ow   ti ter   w i th  a  negati v e  AN A  pr of i l e,   and  ar e  of   l i ttl e
di agnosti c  v al ue.

Case
A  38­y ear ­ol d  w om an  i s  seen  because  of   pai n  and  sw el l i ng  i n  the  joi nts  of
her   hands,   as  w el l   as  i n  her   w r i sts,   el bow s,   and  k nees.   Her   sy m ptom s  hav e
been  i nter m i ttent  ov er   the  l ast  8  m onths  but  hav e  w or sened  r ecentl y   and
becom e  m or e  pr ol onged.   The  pai n  and  sw el l i ng  hav e  been  accom pani ed  by
hand  sti f f ness  i n  the  m or ni ng,   f r equentl y   l asti ng  f or   2  hour s  or   m or e,   and
she  has  noted  r etur n  of   the  sti f f ness  l ater   i n  the  day   af ter   per i ods  of
i nacti v i ty .   She  al so  com pl ai ns  of   pr ogr essi v el y   w or seni ng  f ati gue  and  l ack
of   ener gy .   She  deni es  r ash,   photosensi ti v i ty ,   al opeci a,   or al   ul cer s,   or
sy m ptom s  of   Ray naud's  phenom enon.   She  ex per i ences  l ef t  w r i st  pai n  that
r adi ates  to  her   el bow   and  i nto  her   f i nger s,   w hi ch  i s  w or se  i n  the  m or ni ng
and  occasi onal l y   aw ak ens  her   at  ni ght.
On  phy si cal   ex am i nati on,   sw el l i ng,   w ar m th,   and  tender ness  ar e  noted  i n
sev er al   MCP  and  PIP  joi nts  bi l ater al l y .   Her   w r i sts  ar e  sl i ghtl y   sw ol l en  and
tender   to  pal pati on  especi al l y   i n  the  r egi on  of   the  ul nar   sty l oi d  pr ocesses.
Her   el bow s  ex hi bi t  sl i ght  tender ness  to  pal pati on  and  m i l d  f l ex i on
contr actur es  bi l ater al l y .   Sm al l   ef f usi ons  ar e  pr esent  i n  both  k nees.
Tender ness  i s  el i ci ted  ov er   sev er al   MTP  joi nts  i n  both  f eet.   Ti nel 's  si gn
(tappi ng
P. 453
ov er   the  v ol ar   car pal   l i gam ent  w i th  the  w r i st  i n  ex tensi on)  i s  el i ci ted  ov er
the  l ef t  w r i st  and  Phal en's  test  (posi ti oni ng  the  w r i st  at  f ul l   v ol ar   f l ex i on  f or
60  seconds)  r epr oduces  the  pati ent's  l ef t  w r i st  and  f or ear m   pai n.
Ex am i nati on  of   the  sk i n  r ev eal s  the  pr esence  of   sev er al   subcutaneous
nodul es  ov er   the  pr ox i m al   ex tensor   aspects  of   both  f or ear m s.

1.   What  i s  the  pr i m ar y   pathophy si ol ogi c  pr ocess  i n  RA?


2.   What  ar e  f our   char acter i sti c  r adi ogr aphi c  f i ndi ngs  i n  RA,   and  w hat  ar e
the  m echani sm s  r esponsi bl e  f or   thei r   dev el opm ent?
3.   What  ar e  the  f our   m ost  com m on  ex tr aar ti cul ar   m ani f estati ons  of   RA?
4.   The  natur al   hi stor y   of   the  joi nt  di sease  i n  pati ents  w i th  RA  assum es
w hat  thr ee  patter ns?
5.   What  i s  the  tr eatm ent  f or   RA?

Case Discussion
1.   What  i s  the  pr i m ar y   pathophy si ol ogi c  pr ocess  i n  RA?

The  joi nt  di sease  i n  RA  begi ns  as  i nf l am m ati on  i n  the  sy nov i um   and
i nv ol v es  the  i nf i l tr ati on  of   m acr ophages,   T  cel l s,   and  B  cel l s.   The
sy nov i al   ti ssue  pr ol i f er ates  and  can  gr ow   ov er   the  car ti l age  and  bone.
Thi s  i nf l am m ator y   pr ol i f er ati v e  sy nov i ti s  i s  k now n  as  pannus.  The
pr oducts  of   m acr ophages,   i nter l euk i n  1  (IL­1)  and  TN F­Î± ,   and
f i br obl asts  ar e  abundant  i n  the  r heum atoi d  sy nov i um .   The  ov er al l
pr ocess  can  r esul t  i n  car ti l age  l oss  and  er osi v e  joi nt  destr ucti on.

2.   What  ar e  f our   char acter i sti c  r adi ogr aphi c  f i ndi ngs  i n  RA,   and  w hat  ar e
the  m echani sm s  r esponsi bl e  f or   thei r   dev el opm ent?

The  sof t  ti ssue  sw el l i ng  seen  on  r adi ogr aphi c  studi es  i n  pati ents  w i th
RA  i s  due  to  the  i nf l am ed,   pr ol i f er ati v e  sy nov i ti s.   Joi nt  space  nar r ow i ng
r esul ts  f r om   the  l oss  of   ar ti cul ar   car ti l age;  the  r esul t  of   destr ucti v e
enzy m es  pr oduced  by   sy nov i al   f i br obl asts,   and  chondr ocy tes.
Jux taar ti cul ar   osteopeni a  i s  due  to  the  l oss  of   cal ci um   i n  bones
sur r oundi ng  the  i nf l am m ator y   ar thr i ti s  and  r esul ts  f r om   the  ef f ects  of
pr ostagl andi ns,   IL­1,   and  TN F­Î± ,   w hi ch  ar e  r el eased  by   the  i nf l am ed
sy nov i um .   Mar gi nal   er osi ons  ar e  pr oduced  by   the  pr ol i f er ati v e  sy nov i ti s
as  i t  ex tends  i nto  the  subchondr al   bone  at  the  joi nt  m ar gi ns.

3.   What  ar e  the  f our   m ost  com m on  ex tr aar ti cul ar   m ani f estati ons  of   RA?

The  f our   m ost  com m on  ex tr aar ti cul ar   m ani f estati ons  of   RA  ar e
subcutaneous  nodul es  (r heum atoi d  nodul es),   car pal   tunnel   sy ndr om e,
i nter sti ti al   l ung  di sease,   and  Fel ty 's  sy ndr om e  (spl enom egal y   and
neutr openi a  i n  the  setti ng  of   RA).   Other   ex tr aar ti cul ar   f eatur es  i ncl ude
ocul ar   i nv ol v em ent  (k er atoconjuncti v i ti s  si cca,   epi scl er i ti s,   and
scl er i ti s),   addi ti onal   pul m onar y   i nv ol v em ent  (pl eur al   di sease,   nodul es,
br onchi ol i ti s,   and  pul m onar y   hy per tensi on),   car di ac  i nv ol v em ent
(per i car di ti s  and  r ar e  m y ocar di ti s),   and  r heum atoi d  v ascul i ti s.

4.   The  natur al   hi stor y   of   the  joi nt  di sease  i n  pati ents  w i th  RA  assum es
w hat  thr ee  patter ns?

The  natur al   hi stor y   of   RA  m ay   consi st  of   a  m onocy cl i c  patter n  (20%  of


pati ents),   al though  i n  r etr ospect  som e  of   these  cases  m ay   hav e  been  a
v i r al ­i nduced,   sel f ­l i m i ted  pol y ar thr i ti s;  a  pol y cy cl i c  patter n  (70%  of
pati ents)  w i th  r epeated  epi sodes  of
P. 454
acti v e  di sease  i nter sper sed  w i th  per i ods  of   i nacti v i ty ;  or   a  pr ogr essi v e
patter n  (10%  of   pati ents)  w i th  i ncr easi ng  joi nt  i nv ol v em ent  and  no
di sease­f r ee  i nter v al s.

5.   What  i s  the  tr eatm ent  f or   RA?

Ear l y   detecti on  and  suppr essi on  of   i nf l am m ator y   sy nov i ti s  w i l l   l i k el y


pr ev ent  the  pr ogr essi on  of   car ti l age  and  bony   destr ucti on  al ong  w i th
f uncti onal   i m pai r m ent.   N SAIDs  and  l ow ­dose  cor ti coster oi ds  can
pr ov i de  r api d  r el i ef   of   pai n  and  sti f f ness.   Ear l y   i n  the  di sease  (w i thi n  3
m onths),   tr eatm ent  w i th  DMARDs  shoul d  be  i ni ti ated  i n  m ost  pati ents.
These  agents  i ncl ude  hy dr ox y chl or oqui ne,   sul f asal azi ne,   MTX,   and
l ef l unom i de.   The  choi ce  of   DMARD  i s  a  cl i ni cal   deci si on  based  on
sev er i ty   of   di sease  and  pr ognosi s.   MTX  i s  the  m ost  com m onl y
pr escr i bed  DMARD.   An  appr oach  to  tr eatm ent  of   RA  w oul d  be  to  i ni ti ate
MTX  w i th  r api d  dose  escal ati on  or   MTX  i n  com bi nati on  w i th  other
DMARDs  such  as  hy dr ox y chl or oqui ne  and/or   sul f asal azi ne.   If   the
di sease  i s  r ef r actor y   to  thi s  tr eatm ent,   an  anti ­TN F  bi ol ogi c  agent
shoul d  be  consi der ed  w i th  conti nuati on  of   MTX.   The  av ai l abl e  anti ­TN F
agents  ar e  etaner cept,   i nf l i x i m ab,   and  adal i m um ab.   Another   opti on  i s
anak i nr a,   an  IL­1  r eceptor   antagoni st.   N ew   ther api es  f or   r ef r actor y   RA
i ncl ude  abatacept,   cy totox i c  T­l y m phocy te–associ ated  anti gen  4­Ig,
and  the  B­cel l   depl eti ng  m onocl onal   anti body ,   r i tux i m ab.   When  joi nts
becom e  sev er el y   dam aged  because  of   chr oni c  RA,   r econstr ucti v e
or thopaedi c  sur gi cal   pr ocedur es  m ay   be  per f or m ed  to  hel p  r estor e
f uncti on.

Suggested Readings
El l i ott  JR,   O'Del l   J.   Rheum atoi d  ar thr i ti s.   In:  West  SG,   ed.   Rheum atol ogy
secr ets,   2nd  ed.   Phi l adel phi a:  Hanl ey   &  Bel f us,   2002:117–128.
Genov ese  MC,   Har r i s  ED  Jr .   Tr eatm ent  of   r heum atoi d  ar thr i ti s.   In:  Har r i s
ED  Jr ,   Budd  RC,   Fi r estei n  GS,   et  al .   eds.   Kel l ey 's  tex tbook   of
r heum atol ogy ,   7th  ed.   Phi l adel phi a:  El sev i er ,   Saunder s,   2005:1079â
€“1100.

Gor don  DA,   Hasti ngs  DE.   Cl i ni cal   f eatur es  of   r heum atoi d  ar thr i ti s.   In:
Hochber g  MC,   Si l m an  AJ,   Sm ol en  JS,   et  al .   eds.   Rheum atol ogy ,   3r d  ed.
Edi nbur gh:  Mosby ,   2003:765–780.

Har r i s  ED  Jr .   Cl i ni cal   f eatur es  of   r heum atoi d  ar thr i ti s.   In:  Har r i s  ED  Jr ,
Budd  RC,   Fi r estei n  GS,   et  al .   eds.   Kel l ey 's  tex tbook   of   r heum atol ogy ,   7th
ed.   Phi l adel phi a:  El sev i er ,   Saunder s,   2005:1043–1078.

Scleroderma
1.   What  thr ee  di f f er ent  r heum ati c  di seases  ar e  suggested  by   a
pr edom i nance  of   sk i n  f i ndi ngs?

2.   Ray naud's  phenom enon  m ay   occur   i n  associ ati on  w i th  w hat  f our
r heum ati c  di seases?

3.   Dy sphagi a  or   hear tbur n  m ay   pr edom i nate  i n  w hat  tw o  r heum ati c


di seases?

4.   What  f eatur es  char acter i ze  CREST  sy ndr om e?

5.   What  i s  the  di f f er ence  betw een  l i m i ted  and  di f f use  scl er oder m a
(sy stem i c  scl er osi s)?

P. 455

Discussion
1.   What  thr ee  di f f er ent  r heum ati c  di seases  ar e  suggested  by   a
pr edom i nance  of   sk i n  f i ndi ngs?

A  pr edom i nance  of   sk i n  f i ndi ngs  i n  a  pati ent  w i th  a  suspected


r heum ati c  di sease  suggests  a  di agnosi s  of   SLE,   der m atom y osi ti s,   or
scl er oder m a.   The  sk i n  f i ndi ngs  i n  each  of   these  di seases,   how ev er ,   ar e
di sti nct,   w hi ch  al l ow s  thei r   di f f er enti ati on.

2.   Ray naud's  phenom enon  m ay   occur   i n  associ ati on  w i th  w hat  f our
r heum ati c  di seases?

Ray naud's  phenom enon  (a  col d­i nduced  bl anchi ng  or   cy anosi s  of   the
f i nger s  or   toes)  m ay   be  seen  i n  the  setti ngs  of   scl er oder m a  (90%),
MCTD  (70%),   SLE  (20%),   or   pol y m y osi ti s/der m atom y osi ti s  (20%).
When  the  phenom enon  occur s  al one,   w i thout  an  associ ated  CTD,   i t  i s
cal l ed  Ray naud's  di sease.

3.   Dy sphagi a  or   hear tbur n  m ay   pr edom i nate  i n  w hat  tw o  r heum ati c


di seases?

Dy sphagi a  (di scom f or t  w hen  sw al l ow i ng  f ood)  and  hear tbur n  ar e


esophageal   abnor m al i ti es  that  m ay   occur   i n  the  setti ng  of   ei ther
scl er oder m a  or   pol y m y osi ti s  and  der m atom y osi ti s.   In  scl er oder m a,   the
l ow er   por ti on  of   the  esophagus  i s  i nv ol v ed.   In  der m atom y osi ti s  and
pol y m y osi ti s,   the  m uscl es  i n  the  phar y nx   and  upper   thi r d  of   the
esophagus  m ay   be  i nv ol v ed.

4.   What  f eatur es  char acter i ze  CREST  sy ndr om e?

CREST  sy ndr om e  i s  a  cl i ni cal   v ar i ant  of   l i m i ted  scl er oder m a  that  i s


char acter i zed  by   cal ci nosi s,   Ray naud's  phenom enon,   esophageal
dy sm oti l i ty ,   scl er odacty l y ,   and  tel angi ectases.   Pati ents  w i th  the
sy ndr om e  m ay   ex per i ence  a  m or e  beni gn  cour se  than  those  w i th  m or e
w i despr ead  scl er oder m a  that  i nv ol v es  other   i nter nal   or gans.   Ther e  i s
an  i ncr eased  r i sk   f or   the  dev el opm ent  of   pul m onar y   hy per tensi on  w i th
l i m i ted  scl er oder m a.

5.   What  i s  the  di f f er ence  betw een  l i m i ted  and  di f f use  scl er oder m a
(sy stem i c  scl er osi s)?

In  l i m i ted  sy stem i c  scl er osi s,   f i br oti c  sk i n  di sease  i s  l i m i ted  to  the
hands  and  f or ear m s,   f eet,   neck ,   and  f ace.   Pul m onar y   hy per tensi on  can
occur .   Pati ents  w i th  l i m i ted  sy stem i c  scl er osi s  hav e  a  hi gh  i nci dence  of
anti centr om er e  anti bodi es.   In  di f f use  sy stem i c  scl er osi s,   f i br oti c  sk i n
i nv ol v es  the  f i nger s,   hands,   ar m s,   l egs,   and  ty pi cal l y   the  tr unk   and
f ace.   Pul m onar y   (i nter sti ti al   l ung  di sease),   r enal ,   gastr oi ntesti nal ,   and
car di ac  i nv ol v em ent  can  occur .   Pati ents  w i th  di f f use  sy stem i c  scl er osi s
ar e  m or e  l i k el y   to  hav e  anti bodi es  to  topoi som er ase  1  (anti –Scl ­70).

Case
A  45­y ear ­ol d  w om an  seek s  m edi cal   attenti on  because  of   pr ogr essi v e
sy m m etr i c  sk i n  ti ghteni ng  that  has  i nv ol v ed  the  di gi ts,   hands,   and  f or ear m s
dur i ng  the  l ast  6  m onths.   These  sk i n  changes  ar e  pai nl ess  and  ar e
associ ated  w i th  m i l d  pr ur i tus.   Dur i ng  the  l ast  12  m onths,   she  has  al so  noted
the  onset  of   col d  sensi ti v i ty   of   the  hands,   especi al l y   w hen  handl i ng  objects
i n  the  r ef r i ger ator ,   w i th  m ul ti pl e  f i nger s  becom i ng  col d,   pal e,   and  num b.
She  al so  r epor ts  gener al i zed  f ati gue,   dy spnea  on  ex er ti on,   and  a  decr ease
i n  ex er ci se  tol er ance.   She  deni es  chest  pai n,   pal pi tati ons,   or   par ox y sm al
noctur nal   dy spnea,   but  has
P. 456
noti ced  sy m m etr i c  sw el l i ng  i n  both  l ow er   ex tr em i ti es.   She  has  noted  a  10­l b
(4. 5­k g)  w ei ght  l oss  i n  the  l ast  6  m onths,   w hi ch  she  has  attr i buted  to
decr eased  f ood  i ntak e  because  of   her   hear tbur n  and  dy sphagi a.
On  phy si cal   ex am i nati on,   the  w om an  appear s  y ounger   than  her   stated  age
as  she  l ack s  the  nor m al   f or ehead  w r i nk l i ng  and  has  a  “pur sed­l i psâ€​
appear ance.   A  f ew   scatter ed  f aci al   tel angi ectases  ar e  noted.   Her   sk i n  i s
v er y   ti ght  and  cannot  be  easi l y   l i f ted  f r om   ov er   the  dor sum   of   the  hands,
f i nger s,   and  l ow er   f or ear m s.   Ther e  ar e  v er y   sm al l   punctate  heal ed
ul cer ati ons  on  sev er al   f i nger ti ps.   N ai l   f i ndi ngs  ar e  unr em ar k abl e.   Her
m uscl e  str ength  i s  nor m al   and  ther e  i s  no  ev i dence  of   sy nov i ti s.   Chest
ex am i nati on  r ev eal s  cl ear   l ung  f i el ds.   On  car di ac  ex am i nati on,   no  gal l ops,
m ur m ur s,   or   r ubs  ar e  hear d  but  the  pul m oni c  second  sound  (P 2 )  i s  l oud.   Her
jugul ar   v enous  pr essur e  i s  sl i ghtl y   el ev ated,   and  ther e  i s  1  +   pi tti ng  edem a
ov er   both  l ow er   ex tr em i ti es.

1.   What  i s  the  pr i m ar y   pathophy si ol ogi c  pr ocess  i n  sy stem i c  scl er osi s?


2.   What  f our   r adi ogr aphi c  f i ndi ngs  m ay   be  seen  i n  pati ents  w i th  sy stem i c
scl er osi s?
3.   What  ar e  som e  of   the  com pl i cati ons  associ ated  w i th  esophageal   and
sm al l   i ntesti nal   i nv ol v em ent  i n  sy stem i c  scl er osi s?
4.   What  car di ac  and  r enal   pr obl em s  m ay   ar i se  i n  pati ents  w i th  sy stem i c
scl er osi s?
5.   What  i s  the  ther apy   f or   pati ents  w i th  sy stem i c  scl er osi s?

Case Discussion
1.   What  i s  the  pr i m ar y   pathophy si ol ogi c  pr ocess  i n  sy stem i c  scl er osi s?

Sy stem i c  scl er osi s  i s  a  sy stem i c  f i br oti c  di sor der .   In  the  sk i n,   ther e  i s
ear l y   CD4 +   T­cel l   i nf i l tr ati on  and  m assi v e  nor m al   ty pe  I  col l agen
deposi ti on  by   der m al   f i br obl asts  l i k el y   i nduced  by   tr ansf or m i ng  gr ow th
f actor   β  (TGF­Î²).   Ar ter i al   endothel i al   cel l   dam age  w i th  m y oi nti m al   cel l
pr ol i f er ati on  (oni on  sk i nni ng)  occur s,   r esul ti ng  i n  nar r ow i ng  of   the
v ascul ar   l um en.   Ischem i c  dam age  and  f i br osi s  can  occur   i n  v i scer al
or gans  as  a  r esul t  of   thi s  v ascul opathy .

2.   What  f our   r adi ogr aphi c  f i ndi ngs  m ay   be  seen  i n  pati ents  w i th  sy stem i c
scl er osi s?

Radi ogr aphi c  abnor m al i ti es  that  m ay   be  f ound  i n  pati ents  w i th
scl er oder m a  i ncl ude  w i dem outh  di v er ti cul a  of   the  tr ansv er se  and
descendi ng  col on  on  bar i um   enem a,   pul m onar y   i nter sti ti al   f i br osi s,   l oss
of   di stal   di gi tal   tuf ts,   and  subcutaneous  cal ci nosi s  par ti cul ar l y   i n  the
hands.

3.   What  ar e  som e  of   the  com pl i cati ons  associ ated  w i th  esophageal   and
sm al l   i ntesti nal   i nv ol v em ent  i n  sy stem i c  scl er osi s?

The  l ow er   esophageal   i nv ol v em ent  that  can  occur   i n  pati ents  w i th


sy stem i c  scl er osi s  m ay   l ead  to  sev er e  esophageal   r ef l ux ,   dy sphagi a,
and  ul ti m atel y   esophageal   str i ctur es  m ay   dev el op.   Inv ol v em ent  of   the
sm al l   i ntesti ne  m ay   l ead  to  l oss  of   m oti l i ty   w i th  m al absor pti on
secondar y   to  bacter i al   ov er gr ow th.   Other   com pl i cati ons  of
gastr oi ntesti nal   i nv ol v em ent  w i th  sy stem i c  scl er osi s  i ncl ude
w ater m el on  stom ach  (gastr i c  antr al   v ascul ar   ectasi a)  and  pneum atosi s
cy stoi des  i ntesti nal i s.

P. 457
4.   What  car di ac  and  r enal   pr obl em s  m ay   ar i se  i n  pati ents  w i th  sy stem i c
scl er osi s?

The  hear ts  of   pati ents  w i th  sy stem i c  scl er osi s  m ay   be  af f ected  by
patchy   f i br osi s,   w hi ch  can  cause  conducti on  di stur bances  and
ar r hy thm i as.   Per i car di ti s  and  congesti v e  hear t  f ai l ur e  can  al so  occur .
In  the  ev ent  of   r enal   i nv ol v em ent,   pati ents  can  hav e  hy per tensi on  w i th
m i l d  pr otei nur i a  that  som eti m es  l eads  to  scl er oder m a  r enal   cr i si s
(accel er ated  hy per tensi on  and  r api d  l oss  of   k i dney   f uncti on  pr ogr essi ng
to  r enal   f ai l ur e).   Most  pati ents  w ho  dev el op  scl er oder m a  r enal   cr i si s
hav e  di f f use  cutaneous  i nv ol v em ent.   Mi cr oangi opathi c  hem ol y ti c
anem i a  and  thr om bocy topeni a  can  be  pr esent  i n  the  setti ng  of   r enal
cr i si s.

5.   What  i s  the  ther apy   f or   pati ents  w i th  sy stem i c  scl er osi s?

Ther e  ar e  cur r entl y   no  k now n  m edi cati ons  that  can  al ter   the  natur al
cour se  of   scl er oder m a.   Aggr essi v e  sk i n  car e  i s  hel pf ul   i n  pr ev enti ng
br eak dow n  and  l ocal   i nf ecti on.   Ray naud's  phenom enon  i s  tr eated  w i th
pr otecti on  f r om   the  col d  and  cal ci um   channel   bl ock er s.
Gastr oesophageal   r ef l ux   r equi r es  aggr essi v e  ther apy   w i th  a  pr oton
pum p  i nhi bi tor .   Br oad­spectr um   anti bi oti cs  m ay   be  used  i f   di ar r hea
ar i ses  as  a  r esul t  of   sm al l   i ntesti nal   i nv ol v em ent.   An  angi otensi n­
conv er ti ng  enzy m e  i nhi bi tor   shoul d  be  used  i n  hy per tensi v e  pati ents
w i th  sy stem i c  scl er osi s  i n  an  ef f or t  to  pr ev ent  f ur ther   r enal   dam age
and  possi bl e  r enal   cr i si s  by   r ev er si ng  the  under l y i ng  hy per r eni nem i a.
Pati ents  w i th  ear l y   pr ogr essi v e  i nter sti ti al   l ung  di sease  m ay   benef i t
f r om   tr eatm ent  w i th  cy cl ophospham i de.   Si gni f i cant  pul m onar y   ar ter i al
hy per tensi on,   the  l eadi ng  cause  of   death  i n  pati ents  w i th  l i m i ted
scl er oder m a,   of ten  r equi r es  aggr essi v e  ther apy   w i th  ox y gen,
anti coagul ati on,   and  agents  such  as  bosentan,   si l denaf i l ,   or
pr ostanoi ds.

Suggested Readings
Col l i er   DH.   Sy stem i c  Scl er osi s.   In:  West  SG,   ed.   Rheum atol ogy   secr ets,
2nd  ed.   Phi l adel phi a:  Hanl ey   &  Bel f us,   2002:151–161.

Sei bol d  JR.   Scl er oder m a.   In:  Har r i s  ED  Jr ,   Budd  RC,   Fi r estei n  GS,   et  al .
eds.   Kel l ey 's  tex tbook   of   r heum atol ogy ,   7th  ed.   Phi l adel phi a:  El sev i er ,
Saunder s,   2005:1279–1308.
Wi gl ey   FM,   Hum m er s  LK.   Cl i ni cal   f eatur es  of   sy stem i c  scl er osi s.   In:
Hochber g  MC,   Si l m an  AJ,   Sm ol en  JS,   et  al .   eds.   Rheum atol ogy ,   3r d  ed.
Edi nbur gh:  Mosby ,   2003:1463–1479.

Synovial Fluid Analysis and Septic Arthritis
1.   When  shoul d  ar thr ocentesi s  be  per f or m ed?

2.   What  di agnosti c  tests  shoul d  be  per f or m ed  on  al l   sy nov i al   f l ui d


aspi r ates  r egar dl ess  of   the  suspected  di agnosi s?

3.   What  ar e  the  char acter i sti cs  of   nor m al ,   noni nf l am m ator y ,


i nf l am m ator y ,   and  septi c  sy nov i al   ef f usi ons?

4.   What  ar e  the  causes  of   bl oody   or   hem or r hagi c  sy nov i al   f l ui d?

P. 458

Discussion
1.   When  shoul d  ar thr ocentesi s  be  per f or m ed?

The  m ost  i m por tant  r eason  to  per f or m   an  ar thr ocentesi s  i s  to  ex cl ude  a
joi nt  i nf ecti on.   Sy nov i al   f l ui d  anal y si s  i s  of ten  hel pf ul   di agnosti cal l y   i n
a  pati ent  w i th  joi nt  pai n  and  sw el l i ng  of   uncl ear   eti ol ogy .   Sy nov i al   f l ui d
anal y si s  w i l l   deter m i ne  i f   the  f l ui d  i s  nor m al ,   noni nf l am m ator y ,
i nf l am m ator y   i ncl udi ng  cr y stal   di sease,   or   septi c.

2.   What  di agnosti c  tests  shoul d  be  per f or m ed  on  al l   sy nov i al   f l ui d


aspi r ates  r egar dl ess  of   the  suspected  di agnosi s?

Sy nov i al   f l ui d  shoul d  be  r outi nel y   sent  f or   cel l   count  w i th  di f f er enti al ,
cr y stal   anal y si s,   and  Gr am 's  stai n  and  cul tur e.   Chem i str y
deter m i nati ons  ar e  unl i k el y   to  y i el d  addi ti onal   usef ul   i nf or m ati on  and
shoul d  not  be  or der ed  r outi nel y .

3.   What  ar e  the  char acter i sti cs  of   nor m al ,   noni nf l am m ator y ,


i nf l am m ator y ,   and  septi c  sy nov i al   ef f usi ons?

Spe c ia l
Type  of Fluid Le uk oc yte s /µL
Fe a ture s

N or m al Cl ear , < 200  (< 25%  PMN s)


col or l ess,
v i scous

N oni nf l am m ator y Cl ear ,   y el l ow , 200­2, 000  (< 25%  PMN s)


(ty pe  I  f l ui d) v i scous
Inf l am m ator y Cl oudy ,   y el l ow , > 2, 000  (> 50%  PMN s)
(ty pe  II  f l ui d) l ow   v i scosi ty ,
cul tur e
negati v e

Septi c  (ty pe  III Pur ul ent, > 50, 000  (> 95%  PMN s)  but  not
f l ui d) cul tur e  posi ti v e al l   f l ui ds  > 50, 000  ar e  septi c,
they   m ay   be  i nf l am m ator y

PMN s,   pol y m or phonucl ear   l euk ocy tes.

4.   What  ar e  the  causes  of   bl oody   or   hem or r hagi c  sy nov i al   f l ui d?

The  causes  of   hem or r hagi c  sy nov i al   f l ui d  ar e  tr aum a  w i th  or   w i thout


f r actur e;  bl eedi ng  di sor der s  i ncl udi ng  anti coagul ati on,   hem ophi l i a,   v on
Wi l l ebr and's  di sease,   scur v y ,   and  thr om bocy topeni a;  cr y stal l i ne
ar thr opathy ,   par ti cul ar l y   acute  pseudogout  and  hy dr ox y apati te
deposi ti on  di sease;  Char cot's  ar thr opathy ;  tum or s  i ncl udi ng  pi gm ented
v i l l onodul ar   sy nov i ti s;  hem angi om a;  and  si ck l e  cel l   ar thr opathy .

Case
A  52­y ear ­ol d  w om an  i s  seen  i n  the  em er gency   r oom   because  of   an  acutel y
pai nf ul   and  sw ol l en  r i ght  k nee.   The  pati ent  has  a  10­y ear   hi stor y   of   RA  that
has  not  r esponded  w el l   to  m ul ti pl e  m edi cati ons.   For   the  l ast  6  m onths,   she
has  been  tak i ng  i bupr of en,   azathi opr i ne  100  m g  dai l y ,   and  pr edni sone  10  m g
dai l y .   Despi te  thi s  r egi m en,   she  conti nues  to
P. 459
ex per i ence  2  hour s  of   m or ni ng  sti f f ness  w i th  sw el l i ng,   er y them a,   and  pai n
i n  m ul ti pl e  sm al l   joi nts  of   her   hands,   w r i sts,   k nees,   and  ank l es.   She  i s  now
unabl e  to  bear   w ei ght  on  the  r i ght  l eg.   A  l ow ­gr ade  f ev er   al so  dev el oped.
On  phy si cal   ex am i nati on,   the  pati ent's  tem per atur e  i s  f ound  to  be  38. 5°C
(101. 3°F)  and  her   bl ood  pr essur e  i s  150/100  m m   Hg.   She  appear s  both
acutel y   and  chr oni cal l y   i l l   w i th  m i l d  sw el l i ng  of   m ul ti pl e  MCP  and  PIP  joi nts
as  w el l   as  both  w r i sts  and  ank l es.   Her   r i ght  k nee  i s  hel d  i n  10  degr ees  of
f l ex i on  and  i t  cannot  be  m ov ed  because  of   sev er e  pai n.   The  k nee  ex hi bi ts  a
l ar ge  ef f usi on  and  i s  di f f usel y   tender   w i th  er y them a  ar ound  the  enti r e  joi nt.
Joi nt  aspi r ati on  i s  per f or m ed  and  20  m L  of   opaque,   y el l ow   f l ui d  i s  r em ov ed
that  has  l ow   v i scosi ty .   The  WBC  count  i n  the  sy nov i al   f l ui d  aspi r ate  i s
75, 000/µL  w i th  98%  pol y m or phonucl ear   l euk ocy tes.   Sy nov i al   f l ui d  cr y stal
anal y si s  i s  negati v e.   Ther e  ar e  gr am ­posi ti v e  cocci   on  Gr am 's  stai n  of   the
sy nov i al   f l ui d.   The  f l ui d  i s  cul tur ed  f or   or gani sm s.

1.   How   do  bacter i a  r each  the  sy nov i um   to  cause  a  septi c  ar thr i ti s?
2.   What  ar e  the  r i sk   f actor s  f or   dev el opi ng  a  septi c  ar thr i ti s?
3.   How   do  nongonococcal   bacter i al   septi c  ar thr i ti s  and  di ssem i nated
gonococcal   ar thr i ti s  di f f er ?
4.   What  i s  “pseudosepti câ€​
  ar thr i ti s?

Case Discussion
1.   How   do  bacter i a  r each  the  sy nov i um   to  cause  a  septi c  ar thr i ti s?

Inf ecti ous  or gani sm s  r each  the  sy nov i al   m em br ane  thr ough
hem atogenous  spr ead  due  to  a  r em ote  i nf ecti on  (m ost  com m on),
di ssem i nati on  f r om   an  adjacent  sof t  ti ssue  i nf ecti on  or   osteom y el i ti s,
di agnosti c  or   ther apeuti c  m easur es,   or   penetr ati ng  punctur e  f r om
tr aum a.   The  m ost  com m on  or gani sm   causi ng  septi c  ar thr i ti s  i n  y oung
sex ual l y   acti v e  adul ts  i s  N .   gonor r hoeae  and  i n  pati ents  ol der   than  50
y ear s  i s  Staphy l ococcus  aur eus  f ol l ow ed  by   gr am ­negati v e  or gani sm s.

2.   What  ar e  the  r i sk   f actor s  f or   dev el opi ng  a  septi c  ar thr i ti s?

The  r i sk   f actor s  f or   dev el opi ng  a  septi c  ar thr i ti s  i ncl ude  abnor m al
joi nts  due  to  ar thr i ti s;  pr ostheti c  joi nts;  i m pai r ed  host  def ense
m echani sm s  i ncl udi ng  ex tr em es  of   age,   i m m unosuppr essi v e  dr ugs,
al cohol i sm ,   neopl asti c  di seases,   and  chr oni c  di seases  such  as  di abetes,
chr oni c  k i dney   di sease,   ci r r hosi s,   hem ogl obi nopathi es,   and  HIV;  and
host  phagocy ti c  def ects  such  as  i m pai r ed  chem otax i s  and  com pl em ent
def i ci enci es.   Intr av enous  dr ug  abuse  i s  al so  a  pr edi sposi ng  r i sk   f or
dev el opi ng  a  septi c  ar thr i ti s  of ten  w i th  aty pi cal   joi nt  i nv ol v em ent.   In
addi ti on  to  joi nts  of   the  l ow er   ex tr em i ti es,   i ntr av enous  dr ug  abuser s
can  dev el op  septi c  ar thr i ti s  of   the  ax i al   sk el eton,   v er tebr al   di sc  spaces,
sacr oi l i ac  joi nts,   acr om i ocl av i cul ar   joi nts,   and  ster nocl av i cul ar   joi nts.

3.   How   do  nongonococcal   bacter i al   septi c  ar thr i ti s  and  di ssem i nated


gonococcal   ar thr i ti s  di f f er ?

P. 460

Nongonoc oc c a l
  G onoc oc c a l Arthritis
Ba c te ria l Arthritis

Host Ex tr em es  of   age, Young,   heal thy   adul ts


i m m unosuppr essed

Joi nt  patter n Monoar ti cul ar Mi gr ator y   pol y ar thr al gi as,


ar thr i ti s

Der m ati ti s Rar e Com m on

Tenosy nov i ti s Rar e Com m on

Posi ti v e  joi nt > 95% < 25%


cul tur es
Posi ti v e 50% < 10%
bl ood
cul tur es

Tr eatm ent Ar thr oscopi c  or Cef tr i ax one  dai l y   unti l   cl i ni cal


open  joi nt  l av age i m pr ov em ent  f ol l ow ed  by   7­d
and  pr ol onged tr eatm ent  w i th  or al   cef i x i m e
i ntr av enous or   a  f l uor oqui nol one
anti bi oti cs

Mechani sm Bacter em i c  seedi ng Im m une  com pl ex   or


of   the  joi nt hy per sensi ti v i ty   r eacti on

4.   What  i s  “pseudosepti câ€​


  ar thr i ti s?

Pseudosepti c  ar thr i ti s  ty pi cal l y   occur s  i n  the  setti ng  of   poor l y


contr ol l ed  RA.   The  pati ent  pr esents  w i th  acute  onset  of   one  or   m or e
sw ol l en  joi nts  w i th  sy nov i al   f l ui d  WBC  count  gr eater   than  100, 000  cel l s/
µL  and  a  negati v e  Gr am 's  stai n  and  cul tur e  of   the  f l ui d.   Af ter   joi nt
i nf ecti on  has  been  ex cl uded,   the  pati ent  r esponds  to  i ncr eased  doses  of
cor ti coster oi ds  r ather   than  anti bi oti cs.   Pseudosepti c  ar thr i ti s  can  al so
occur   i n  acute  cr y stal ­i nduced  ar thr i ti s,   par ti cul ar l y   acute  pseudogout,
and  i n  ser onegati v e  spondy l oar thr opathi es,   especi al l y   r eacti v e
ar thr i ti s.

Suggested Readings
Ger l ag  M,   Tak   PP.   Sy nov i al   f l ui d  anal y si s,   sy nov i al   bi opsy ,   and  sy nov i al
pathol ogy .   In:  Har r i s  ED  Jr ,   Budd  RC,   Fi r estei n  GS,   et  al .   eds.   Kel l ey 's
tex tbook   of   r heum atol ogy ,   7th  ed.   Phi l adel phi a:  El sev i er ,   Saunder s,
2005:675–690.

Gi l l i l and  WR.   Bacter i al   septi c  ar thr i ti s.   In:  West  SG,   ed.   Rheum atol ogy
secr ets,   2nd  ed.   Phi l adel phi a:  Hanl ey   &  Bel f us,   2002:281–289.

Li dgr en  l .   Septi c  ar thr i ti s  and  osteom y el i ti s.   In:  Hochber g  MC,   Si l m an


AJ,   Sm ol en  JS,   et  al .   eds.   Rheum atol ogy ,   3r d  ed.   Edi nbur gh:  Mosby ,
2003:1055–1065.

Mahow al d  ML.   Gonococcal   ar thr i ti s.   In:  Hochber g  MC,   Si l m an  AJ,   Sm ol en


JS,   et  al .   eds.   Rheum atol ogy ,   3r d  ed.   Edi nbur gh:  Mosby ,   2003:1067â
€“1075.

Spencer   RT.   Ar thr ocentesi s  and  sy nov i al   f l ui d  anal y si s.   In:  West  SG,   ed.
Rheum atol ogy   secr ets,   2nd  ed.   Phi l adel phi a:  Hanl ey   &  Bel f us,   2002:63â
€“67.

P. 461

Systemic Lupus Erythematosus
1.   What  cl i ni cal   f eatur es  suggest  a  di agnosi s  of   SLE?

2.   What  abnor m al   l abor ator y   r esul ts  suggest  a  di agnosi s  of   SLE?

3.   Besi des  SLE,   AN As  ar e  com m onl y   f ound  i n  w hat  other   di seases?

Discussion
1.   What  cl i ni cal   f eatur es  suggest  a  di agnosi s  of   SLE?

For   the  pur poses  of   cl i ni cal   studi es,   any   per son  hav i ng  4  or   m or e  of   the
f ol l ow i ng  11  cr i ter i a  i s  consi der ed  to  hav e  SLE:  m al ar   r ash,   di scoi d
r ash,   photosensi ti v i ty ,   or al   ul cer s,   ar thr i ti s,   ser osi ti s  (pl eur i ti s  or
per i car di ti s),   r enal   di sor der   (per si stent  pr otei nur i a  > 0. 5  g  per   day   or
cel l ul ar   casts),   neur ol ogi c  di sor der   (sei zur es  or   psy chosi s),
hem atol ogi c  di sor der   (hem ol y ti c  anem i a,   l euk openi a,   l y m phopeni a,   or
thr om bocy topeni a),   i m m unol ogi c  di sor der   (anti ­  DN A  anti bodi es,   anti ­
Sm i th  [Sm ]  anti bodi es,   or   posi ti v e  f i ndi ngs  of   anti phosphol i pi d
anti bodi es),   and  AN A.

2.   What  abnor m al   l abor ator y   r esul ts  suggest  a  di agnosi s  of   SLE?

Al m ost  al l   pati ents  w i th  SLE  dem onstr ate  el ev ated  ser um   l ev el s  of   AN A.
How ev er ,   thi s  test  i s  not  speci f i c  f or   SLE.   Other   l abor ator y
abnor m al i ti es  i n  SLE  can  i ncl ude  anti –doubl e­str anded  DN A
anti bodi es,   anti ­Sm   anti bodi es,   f al se­posi ti v e  test  f or   sy phi l i s,   l ow
ser um   com pl em ent  l ev el s,   pr ol onged  par ti al   thr om bopl asti n  ti m e,
anti phosphol i pi d  anti bodi es,   cy topeni as,   and  acti v e  ur i ne  sedi m ent.

3.   Besi des  SLE,   AN As  ar e  com m onl y   f ound  i n  w hat  other   di seases?

In  SLE,   AN As  hav e  a  sensi ti v i ty   of   93%  to  100%  but  a  l ow er   speci f i ci ty
of   appr ox i m atel y   50%  si nce  AN As  can  occur   i n  m any   other   di seases.
Condi ti ons  associ ated  w i th  a  posi ti v e  AN A  i ncl ude  other   autoi m m une
di seases  (scl er oder m a:  60%  to  85%,   MCTD:  90%  to  100%,
i nf l am m ator y   m y opathi es:  50%,   RA:  30%  to  50%,   Sjögr en's
sy ndr om e:  40%  to  70%,   and  dr ug­i nduced  l upus:  100%),   or gan­speci f i c
autoi m m une  di seases  (such  as  Hashi m oto's  thy r oi di ti s:  46%,   Gr av es'
di sease:  50%,   autoi m m une  hepati ti s:  63%  to  91%,   pr i m ar y   bi l i ar y
ci r r hosi s:  10%  to  40%,   i di opathi c  thr om bocy topeni c  pur pur a:  10%  to
40%,   and  m ul ti pl e  scl er osi s:  25%),   chr oni c  i nf ecti ons  (such  as
m ononucl eosi s,   hepati ti s  C,   HIV  i nf ecti on,   par v ov i r us  B19  i nf ecti on,
bacter i al   endocar di ti s,   and  tuber cul osi s),   l y m phopr ol i f er ati v e  di seases,
FMS:  12%  to  30%,   and  heal thy   w om en  and  el der l y   pati ents:  5%  to
30%.   In  SLE,   the  posi ti v e  and  negati v e  pr edi cti v e  v al ues  of   an  AN A  ar e
11%  to  30%,   and  95%,   r especti v el y .   Ther ef or e,   an  AN A  shoul d  be
tested  onl y   w hen  the  pati ent  has  a  hi gh  cl i ni cal   pr etest  pr obabi l i ty   of
hav i ng  a  CTD.

Case
A  28­y ear ­ol d  w om an  pr esents  w i th  a  2­m onth  hi stor y   of   pai nf ul   joi nts  and
f ati gue.   She  states  that  the  joi nt  pai n  af f ects  her   hands,   w r i sts,   f eet,
ank l es,   and  k nees  and  i s
P. 462
associ ated  w i th  som e  joi nt  sw el l i ng  and  2  to  3  hour s  of   m or ni ng  sti f f ness.
Ov er   the  l ast  3  to  4  m onths,   the  pati ent  has  noted  gr adual l y   i ncr easi ng
f ati gue  and  has  had  thr ee  or   f our   epi sodes  of   r ash  ov er   her   f ace  and  neck .
Dur i ng  the  l ast  sum m er ,   she  states  that  she  had  a  si m i l ar   r ash  that  w as
pr eci pi tated  by   ex posur e  to  the  sun.   She  has  al so  noted  that  pr ol onged  sun
ex posur e  r esul ts  i n  i ncr easi ng  f ati gue  and  a  f l u­l i k e  sy ndr om e.   Tw o  w eek s
ago,   she  noted  her   ank l es  tend  to  sw el l   at  the  end  of   the  day .   Past  m edi cal
hi stor y   r ev eal s  that  8  m onths  ago  she  had  an  epi sode  of   pl eur i ti c  chest  pai n
that  l asted  8  to  10  day s  and  w as  tr eated  by   her   f am i l y   doctor   w i th
i ndom ethaci n  f ol l ow ed  by   gr adual   r esol uti on.
Phy si cal   ex am i nati on  r ev eal s  a  ti r ed­l ook i ng  w om an  w ho  i s  i n  no  acute
di str ess.   Her   tem per atur e  i s  38. 2°C  (100. 8°F),   bl ood  pr essur e  i s  140/100
m m   Hg,   and  pul se  i s  96  beats  per   m i nute  and  r egul ar .   On  ex am i nati on  of
the  sk i n,   an  er y them atous  r ash  i s  noted  ov er   her   nose  and  cheek s  that
spar es  the  nasol abi al   f ol ds.   Sev er al   shal l ow   pai nl ess  ul cer s  ar e  f ound  i n  her
m outh.   Joi nt  ex am i nati on  r ev eal s  m i ni m al   sw el l i ng  of   the  w r i sts  and  MCP
joi nts.   Pul m onar y   and  car di ac  f i ndi ngs  ar e  nor m al   ex cept  f or   2+   pi tti ng
edem a  i n  the  pr eti bi al   ar ea,   bi l ater al l y .

1.   What  ar e  the  tw o  m ost  com m on  m echani sm s  of   ti ssue  dam age  i n


pati ents  w i th  SLE?
2.   Besi des  the  sk i n  and  joi nts,   w hat  other   or gans  ar e  com m onl y   af f ected
i n  pati ents  w i th  SLE?
3.   What  ser ol ogi c  tests  and  di agnosti c  pr ocedur es  m ay   be  hel pf ul   i n  the
m anagem ent  of   l upus  nephr i ti s?
4.   What  ar e  the  f our   possi bl e  causes  of   per i pher al   edem a  i n  pati ents  w i th
SLE?
5.   What  i s  the  ther apy   f or   SLE?

Case Discussion
1.   What  ar e  the  tw o  m ost  com m on  m echani sm s  of   ti ssue  dam age  i n
pati ents  w i th  SLE?
Ti ssue  dam age  i n  pati ents  w i th  SLE  m ay   be  caused  by   anti bodi es  to  cel l
sur f ace  com ponents  or   by   the  pr esence  of   sol ubl e  i m m une  com pl ex es  i n
the  ci r cul ati on.   Anti bodi es  to  pl atel ets,   WBCs,   or   r ed  bl ood  cel l s  m ay
i nduce  thr om bocy topeni a,   l euk openi a,   or   anem i a,   r especti v el y .
Anti phosphol i pi d  anti bodi es  m ay   i nduce  v enous  or   ar ter i al   thr om boses,
r ecur r ent  f etal   l oss,   or   thr om bocy topeni a.   Sol ubl e  i m m une  com pl ex es
i n  the  ci r cul ati on  m ay   deposi t  i n  bl ood  v essel s  or   al ong  basem ent
m em br anes  i n  the  sk i n  or   k i dney s,   r esul ti ng  i n  v ascul i ti s,   der m ati ti s,
or   gl om er ul onephr i ti s.

2.   Besi des  the  sk i n  and  joi nts,   w hat  other   or gans  ar e  com m onl y   af f ected
i n  pati ents  w i th  SLE?

Other   or gans  that  m ay   be  af f ected  i n  the  setti ng  of   SLE  i ncl ude  the
centr al   and  per i pher al   ner v ous  sy stem s,   l ungs  (pl eur i ti s,   capi l l ar i ti s,
pneum oni ti s,   pul m onar y   hy per tensi on,   and  “shr i nk i ng  l ung
sy ndr om eâ€​ ) ,   hear t  (per i car di ti s,   m y ocar di ti s,   and  v al v ul ar   di sease),
k i dney s  (m esangi al   nephr i ti s,   di f f use  pr ol i f er ati v e  gl om er ul onephr i ti s,
and  m em br anous  nephr opathy ),   and  gastr oi ntesti nal   sy stem
(pancr eati ti s  and  m esenter i c  v ascul i ti s),   as  w el l   as  the  f or m ed
el em ents  of   the  bl ood  and  ser ous  m em br anes.

P. 463
3.   What  ser ol ogi c  tests  and  di agnosti c  pr ocedur es  m ay   be  hel pf ul   i n  the
m anagem ent  of   l upus  nephr i ti s?

Low   ser um   com pl em ent  l ev el s  and/or   hi gh  ti ter s  of   anti bodi es  to
doubl e­str anded  DN A  m ay   pr ecede  f l ar es  of   r enal   di sease.   A  k i dney
bi opsy   m ay   ai d  i n  the  m anagem ent  of   pati ents  w i th  l upus  nephr i ti s
par ti cul ar l y   w hen  the  sev er i ty   of   the  di sease  appear s  to  be  changi ng,
the  di sease  i s  r ef r actor y   to  hi gh­dose  pr edni sone  ther apy ,   and
cy totox i c  ther apy   w i th  i ntr av enous  bol us  cy cl ophospham i de  ther apy   i s
bei ng  consi der ed.

4.   What  ar e  the  f our   possi bl e  causes  of   per i pher al   edem a  i n  pati ents  w i th
SLE?

Per i pher al   edem a  i n  a  pati ent  w i th  SLE  m ay   be  due  to  r enal   di sease
w i th  si gni f i cant  pr otei nur i a,   congesti v e  hear t  f ai l ur e  secondar y   to
car di ac  i nv ol v em ent,   pr otei n­l osi ng  enter opathy   due  to  m esenter i c
v ascul i ti s,   or   per i pher al   v enous  thr om bosi s  stem m i ng  f r om   the
f or m ati on  of   anti car di ol i pi n  anti bodi es.

5.   What  i s  the  ther apy   f or   SLE?

Pati ents  w i th  SLE  ar e  m anaged  accor di ng  to  the  ex tent  and  sev er i ty   of
thei r   or gan  i nv ol v em ent.   Pati ents  w i th  m i l d  di sease  consi sti ng  of
ar thr i ti s,   sk i n,   and  non–l i f e­thr eateni ng  bl ood  or   other   or gan
i nv ol v em ent  m ay   be  tr eated  w i th  N SAIDs,   anti m al ar i al s  such  as
hy dr ox y chl or oqui ne,   and  l ow ­dose  cor ti coster oi ds  i f   necessar y .   Pati ents
w i th  m or e  sev er e  or gan  i nv ol v em ent,   par ti cul ar l y   of   the  centr al
ner v ous  sy stem   and  k i dney s,   m ay   be  tr eated  w i th  hi gh  doses  of
cor ti coster oi ds  and  or al   azathi opr i ne  or   i ntr av enous  cy cl ophospham i de.
Recent  ev i dence  suggests  that  m y cophenol ate  m of eti l   m ay   be  usef ul   i n
som e  pati ents  w i th  l upus  nephr i ti s.   Other   ther api es  m ay   be  used  f or
the  am el i or ati on  of   speci f i c  or gan  i nv ol v em ent.

Suggested Readings
Edw or thy   SM.   Cl i ni cal   m ani f estati ons  of   sy stem i c  l upus  er y them atosus.
In:  Har r i s  ED  Jr ,   Budd  RC,   Fi r estei n  GS,   et  al .   eds.   Kel l ey 's  tex tbook   of
r heum atol ogy ,   7th  ed.   Phi l adel phi a:  El sev i er ,   Saunder s,   2005:1201â
€“1224.

Gl adm an  DD,   U r ow i tz  MB.   Sy stem i c  l upus  er y them atosus:  cl i ni cal
f eatur es.   In:  Hochber g  MC,   Si l m an  AJ,   Sm ol en  JS,   et  al .   eds.
Rheum atol ogy ,   3r d  ed.   Edi nbur gh:  Mosby ,   2003:1359–1393.

Hahn  BH.   Managem ent  of   sy stem i c  l upus  er y them atosus.   In:  Hochber g
MC,   Si l m an  AJ,   Sm ol en  JS,   et  al .   eds.   Rheum atol ogy ,   3r d  ed.   Edi nbur gh:
Mosby ,   2003:1225–1247.

Kotzi n  BL.   Sy stem i c  l upus  er y them atosus.   In:  West  SG,   ed.
Rheum atol ogy   secr ets,   2nd  ed.   Phi l adel phi a:  Hanl ey   &  Bel f us,   2002:128â
€“147.

Vasculitis
1.   Vascul i ti s  shoul d  be  suspected  i n  pati ents  pr esenti ng  w i th  any
com bi nati on  of   w hat  cl i ni cal   m ani f estati ons?

2.   N am e  the  pr i m ar y   v ascul i ti c  di sor der s  based  on  the  dom i nant  v essel
si ze  and  anti neutr ophi l   cy topl asm i c  anti bodi es  (AN CA).

3.   What  ser ol ogi c  tests  or   di agnosti c  pr ocedur es  shoul d  be  per f or m ed  i n
pati ents  w i th  suspected  v ascul i ti s?

P. 464
4.   What  m or e  ex tensi v e  pr ocedur es  m ay   be  of   v al ue  i n  hel pi ng  to
establ i sh  the  di agnosi s  of   a  speci f i c  f or m   of   v ascul i ti s?

Discussion
1.   Vascul i ti s  shoul d  be  suspected  i n  pati ents  pr esenti ng  w i th  any
com bi nati on  of   w hat  cl i ni cal   m ani f estati ons?
Vascul i ti s  com pr i ses  a  heter ogeneous  gr oup  of   di seases  char acter i zed
by   i nf l am m ator y   changes  i n  the  bl ood  v essel s  w i th  subsequent
i m pai r m ent  of   f l ow   and  ti ssue/or gan  i schem i a.   Pati ents  pr esent  w i th  a
m ul ti sy stem   i nf l am m ator y   di sease  of ten  w i th  f ev er   of   unk now n  or i gi n
and/or   unex pl ai ned  consti tuti onal   sy m ptom s;  suspi ci ous  sk i n  l esi ons
such  as  ul cer s,   l i v edo  r eti cul ar i s,   and  pal pabl e  pur pur a;  i schem i c
neur opathi es;  and  r api dl y   pr ogr essi v e  or gan  dy sf uncti on  such  as
str ok es,   pul m onar y –r enal   sy ndr om es,   and  other   or gan  i schem i a.

2.   N am e  the  pr i m ar y   v ascul i ti c  di sor der s  based  on  the  dom i nant  v essel
si ze  and  AN CA.

Vascul i ti des  af f ecti ng  l ar ge  ar ter i es:

Tak ay asu's  ar ter i ti s:  aor ti c  ar ch  and  i ts  br anches,   can  i nv ol v e  any
par t  of   the  aor ta;  m or e  cl audi cati on  of   upper   than  l ow er
ex tr em i ti es,   centr al   ner v ous  sy stem   ev ents;  gr anul om atous
panar ter i ti s.

Gi ant  cel l   (tem por al )  ar ter i ti s  (GCA):  tem por al   ar ter i es,   v essel s
or i gi nati ng  f r om   the  aor ti c  ar ch,   other   ar ter i es  l ess  com m on;
tem por al   headache,   jaw   cl audi cati on,   scal p  tender ness,   v i sual
l oss;  ar ter i ti s  w i th  gi ant  cel l s  and  di sr upti on  of   the  i nter nal   el asti c
l am i na.

Vascul i ti des  af f ecti ng  pr edom i nantl y   m edi um ­si zed  ar ter i es:

Pol y ar ter i ti s  nodosa  (PAN ):  sm al l ­  and  m edi um ­si zed  ar ter i es;
m ay   af f ect  any   or gan,   but  sk i n,   joi nts,   per i pher al   ner v es,   gut,   and
k i dney   ar e  m ost  com m onl y   i nv ol v ed;  f ocal   but  panm ur al
necr oti zi ng  ar ter i ti s  w i th  a  pr edi l ecti on  f or   i nv ol v em ent  at  the
v essel   bi f ur cati on.

Kaw asak i   di sease:  sm al l ­  and  m edi um ­si zed  ar ter i es;  acute
f ebr i l e  i l l ness  pr i m ar i l y   af f ecti ng  i nf ants  and  y oung  chi l dr en;
f ev er ,   pr om i nent  m ucocutaneous  changes,   cer v i cal
l y m phadenopathy ,   pol y m or phous  r ash,   er y them a  and  edem a  of
hands  and  f eet,   desquam ati on,   m y ocar di ti s,   cor onar y   v ascul i ti s;
pr obabl e  i nf ecti ous  v ector   r esul ti ng  i n  cy tok i ne­m edi ated
endothel i al   dam age.

Vascul i ti des  af f ecti ng  pr edom i nantl y   sm al l   v essel s  (AN CA­posi ti v e):

Wegener 's  gr anul om atosi s:  sm al l ­  and  m edi um ­si zed  ar ter i es;
upper   r espi r ator y   tr act  (si nuses),   l ungs,   and  k i dney s,   m ay   af f ect
other   or gans;  pauci i m m une,   necr oti zi ng,   gr anul om atous  ar ter i ti s
usual l y   associ ated  w i th  ser um   cy topl asm i c–anti neutr ophi l
cy topl asm i c  anti bodi es  (c­AN CA)  usual l y   di r ected  agai nst
pr otei nase  3  i n  the  pr i m ar y   gr anul es  of   neutr ophi l s.

Mi cr oscopi c  pol y angi i ti s  (MPA):  ar ter i ol es,   capi l l ar i es,   and


v enul es;  pul m onar y   hem or r hage,   gl om er ul onephr i ti s,   pal pabl e
pur pur a,   per i pher al   neur opathy ,   joi nt  and  abdom i nal   pai n;
pauci i m m une,   necr oti zi ng  v ascul i ti s,   ser um   per i nucl ear â
€“anti neutr ophi l   cy topl asm i c  anti bodi es
P. 465
(p­AN CA)  usual l y   di r ected  agai nst  m y el oper ox i dase  i n  the  pr i m ar y
gr anul es  of   neutr ophi l s.

Chur g­Str auss  sy ndr om e:  sm al l   ar ter i es  and  v enul es;  asthm a,
eosi nophi l i a,   m ul ti or gan  i nv ol v em ent  [l ungs,   sk i n,   per i pher al
ner v es,   gut,   hear t,   and  k i dney s  (r ar e)];  necr oti zi ng  ex tr av ascul ar
gr anul om as  and  v ascul i ti s  of   sm al l   ar ter i es  and  v enul es,
eosi nophi l s  pr esent  i n  ear l y   stage.

Vascul i ti des  af f ecti ng  pr edom i nantl y   sm al l   v essel s  (AN CA­negati v e):

Henoch­Schönl ei n  pur pur a  (HSP):  ar ter i ol es  and  v enul es;


pal pabl e  pur pur i c  sk i n  l esi ons  on  l ow er   ex tr em i ti es,   ar thr i ti s,
abdom i nal   pai n,   hem atur i a;  l euk ocy tocl asti c  (neutr ophi l i c
per i v ascul ar /tr ansm ur al   i nf i l tr ate)  or   necr oti zi ng  v ascul i ti s  of ten
w i th  IgA  deposi ti on.

Cutaneous  l euk ocy tocl asti c  angi i ti s:  ar ter i ol es  and  v enul es;
pal pabl e  pur pur i c  sk i n  l esi ons,   ar thr al gi as,   sy stem i c  sy m ptom s
m ay   be  pr esent,   usual l y   secondar y   to  i m m une  com pl ex es  [dr ugs,
bugs  (i nf ecti ons),   CTD  or   m al i gnancy ];  l euk ocy tocl asti c  v ascul i ti s.

Cr y ogl obul i nem i c  v ascul i ti s:  cr y ogl obul i ns  ar e  i m m unogl obul i ns
that  ar e  r ev er si bl y   pr eci pi tated  by   r educed  tem per atur es;
cr y ogl obul i ns  ar e  deposi ted  i n  sm al l   v essel s  i ncl udi ng
gl om er ul ocapi l l ar i es;  pur pur a,   ar thr al gi as,   per i pher al   neur opathy ,
Ray naud's  phenom enon,   pul m onar y   hem or r hage,
gl om er ul onephr i ti s  ar e  possi bl e;  of ten  RF  and  hepati ti s  C  anti body
posi ti v e.

3.   What  ser ol ogi c  tests  or   di agnosti c  pr ocedur es  shoul d  be  per f or m ed  i n
pati ents  w i th  suspected  v ascul i ti s?

The  di agnosti c  ev al uati on  of   a  pati ent  w i th  suspected  v ascul i ti s  shoul d
be  based  on  the  cl i ni cal   si tuati on  but  of ten  i ncl udes  a  chest
r adi ogr aphi c  study ,   ESR,   CRP,   a  com pl ete  bl ood  count  w i th  di f f er enti al ,
l i v er   f uncti on  tests,   CPK,   cr eati ni ne  and  ur i nal y si s,   tests  f or   the
pr esence  of   AN As,   AN CAs  and  RF,   cr y ogl obul i ns,   and  bi opsy   of   a  sk i n
l esi on  or   an  i nv ol v ed  or gan.   In  som e  ty pes  of   v ascul i ti s,   com pl em ent
l ev el s  m ay   be  l ow   secondar y   to  consum pti on.   An  ESR  gr eater   than  100
m m   per   hour   and  a  CRP  gr eater   than  10  m g/dL  i n  the  absence  of   a
w i despr ead  m al i gnancy   or   bacter i al   i nf ecti on  shoul d  suggest  a
v ascul i ti c  pr ocess.

4.   What  m or e  ex tensi v e  pr ocedur es  m ay   be  of   v al ue  i n  hel pi ng  to


establ i sh  the  di agnosi s  of   a  speci f i c  f or m   of   v ascul i ti s?

Mor e  ex tensi v e  di agnosti c  pr ocedur es  f or   establ i shi ng  the  di agnosi s  of
a  speci f i c  f or m   of   v ascul i ti s  i ncl ude  ar ter i ogr aphy   of   the  m esenter i c
v essel s  i f   a  ti ssue  bi opsy   i s  i naccessi bl e,   and  an  el ectr om y ogr aphy   w i th
ev al uati on  of   ner v e  conducti on  v el oci ti es  to  ev al uate  a  per i pher al
neur opathy   or   a  m ononeur i ti s  m ul ti pl ex .   A  com puted  tom ogr aphy   (CT)
scan  of   the  si nuses  and  chest  i s  i ndi cated  i f   a  di agnosi s  of   Wegener 's
gr anul om atosi s  i s  bei ng  consi der ed.

Case
A  45­y ear ­ol d  w hi te  m an  seek s  m edi cal   car e  because  of   hem opty si s  of   1­
w eek   dur ati on.   He  has  not  f el t  w el l   f or   appr ox i m atel y   4  m onths  and  has  l ost
10  l b  (4. 5  k g)  dur i ng  thi s  ti m e.   He  has  been  r ecei v i ng  v ar i ous  anti bi oti cs  f or
the  tr eatm ent  of   chest  r adi ogr aphi c
P. 466
abnor m al i ti es  thought  to  r epr esent  pneum oni a.   Al though  these  changes  hav e
v ar i ed  i n  pr esentati on,   they   hav e  not  di sappear ed.   A  f ew   w eek s  ear l i er ,   he
noted  som e  bl oody   nasal   di schar ge.   He  star ted  coughi ng  up  bl ood  1  w eek
ago  but  attr i buted  i t  to  hi s  bl oody   nose.   The  pati ent  al so  com pl ai ns  that  hi s
l ef t  k nee  has  been  hur ti ng  and  that  r ed  spots  hav e  appear ed  on  hi s  ar m s
and  l egs.   He  deni es  f ev er ,   pur ul ent  sputum ,   al l er gi es  or   asthm a,   k now n
tuber cul osi s,   or   chest  pai n.
On  phy si cal   ex am i nati on,   ther e  i s  a  cur i ous  depr essi on  i n  hi s  upper   nose
(saddl e­nose  def or m i ty ),   bl oody   di schar ge  i n  hi s  nasal   cav i ty ,   a  pai nl ess
ul cer   on  hi s  sof t  pal ate,   and  a  sl i ghtl y   w ar m   and  sw ol l en  l ef t  k nee.   Chest
f i ndi ngs  ar e  nor m al .   Ther e  ar e  m any   sm al l ,   pur pur i c,   r ai sed  l esi ons  on  the
sk i n  of   hi s  l ow er   ex tr em i ti es  that  ar e  pai nl ess.

1.   What  ar e  f our   possi bl e  di agnoses  i n  thi s  pati ent?


2.   What  di agnosti c  studi es  or   pr ocedur es  m i ght  be  of   v al ue  i n  thi s
pati ent?
3.   Whi ch  di sor der s  ar e  associ ated  w i th  p­AN CA?
4.   What  consti tutes  appr opr i ate  ther apy   f or   thi s  pati ent  w i th  Wegener 's
gr anul om atosi s?

Case Discussion
1.   What  ar e  f our   possi bl e  di agnoses  i n  thi s  pati ent?

Four   possi bl e  di agnoses  i n  thi s  pati ent  ar e  Wegener 's  gr anul om atosi s,
Chur g­Str auss  sy ndr om e  (al l er gi c  gr anul om atosi s),   i ntr anasal   dr ug
abuse,   or   a  l ung  tum or .   Chur g­Str auss  sy ndr om e  occur s  pr i m ar i l y   i n
pati ents  w i th  a  hi stor y   of   al l er gi es  or   asthm a  and  i s  of ten  associ ated
w i th  per i pher al   eosi nophi l i a.   AN CA  r eacti ng  w i th  hum an  neutr ophi l
el astase  can  occur   i n  cocai ne­i nduced  m i dl i ne  destr ucti v e  l esi ons.   The
saddl e­nose  def or m i ty   and  pal pabl e  pur pur a  w oul d  be  uncom m on
m ani f estati ons  of   a  pr i m ar y   l ung  car ci nom a.

2.   What  di agnosti c  studi es  or   pr ocedur es  m i ght  be  of   v al ue  i n  thi s  pati ent?
N asophar y ngeal   ex am i nati on  w i th  bi opsy ,   CT  scan  of   the  si nuses  and
chest,   cr eati ni ne  and  ur i nal y si s,   and  br onchoscopy   w i th  bi opsy   or   open
l ung  bi opsy   w oul d  al l   be  hel pf ul   i n  the  ev al uati on  of   thi s  pati ent's
di sor der .   An  AN CA  shoul d  be  or der ed  because  m ost  pati ents  w i th
sy stem i c  Wegener 's  gr anul om atosi s  ar e  c­AN CA  posi ti v e  and  hav e
anti pr otei nase  3  anti bodi es.   In  appr ox i m atel y   60%  of   pati ents,   c­AN CA
ti ter s  cor r el ate  w i th  Wegener 's  di sease  acti v i ty .

3.   Whi ch  di sor der s  ar e  associ ated  w i th  p­AN CA?

A  p­AN CA  m ay   be  pr esent  due  to  a  v ar i ety   of   di f f er ent  anti bodi es
di r ected  agai nst  m y el oper ox i dase,   el astase,   cathepsi n,   and  l actof er r i n,
and  can  occur   i n  m any   di f f er ent  di seases.   Di seases  associ ated  w i th  p­
AN CA  di r ected  agai nst  m y el oper ox i dase  i ncl ude  Wegener 's
gr anul om atosi s  (10%),   Chur g­Str auss  sy ndr om e  (50%),   MPA  (50%  to
80%),   and  i di opathi c  cr escenti c  gl om er ul onephr i ti s  (65%).   N onspeci f i c
p­AN CAs  di r ected  agai nst  other   v ar i ous  pr otei ns  can  occur   i n  CTDs,
Cr ohn's  di sease,   ul cer ati v e  col i ti s,   scl er osi ng  chol angi ti s,   cy sti c
f i br osi s,   chr oni c  i nf ecti ons,   and  r ar e  dr ug­i nduced  v ascul i ti c  sy ndr om es
associ ated  w i th  pr opy l thi our aci l ,   hy dr al azi ne,   and  m i nocy cl i ne.

P. 467
4.   What  consti tutes  appr opr i ate  ther apy   f or   thi s  pati ent  w i th  Wegener 's
gr anul om atosi s?

Standar d  ther apy   f or   Wegener 's  gr anul om atosi s  i ncl udes  both  hi gh
doses  of   cor ti coster oi ds  and  or al   cy cl ophospham i de.   Or al
tr i m ethopr i m /sul f am ethox azol e  pr ophy l ax i s  agai nst  Pneum ocy sti s
car i ni i   shoul d  be  consi der ed  w hi l e  on  the  abov e  ther apy .

Suggested Readings
Cohen  MD.   Appr oach  to  the  pati ent  w i th  suspected  v ascul i ti s.   In:  West
SG,   ed.   Rheum atol ogy   secr ets,   2nd  ed.   Phi l adel phi a:  Hanl ey   &  Bel f us,
2002:201–207.

Hel l m ann  DB,   Hunder   GG.   Gi ant  cel l   ar ter i ti s  and  pol y m y al gi a
r heum ati ca.   In:  Har r i s  ED  Jr ,   Budd  RC,   Fi r estei n  GS,   et  al .   eds.   Kel l ey 's
tex tbook   of   r heum atol ogy ,   7th  ed.   Phi l adel phi a:  El sev i er ,   Saunder s,
2005:1343–1356.

Stone  JH.   The  cl assi f i cati on  and  epi dem i ol ogy   of   sy stem i c  v ascul i ti s.   In:
Har r i s  ED  Jr ,   Budd  RC,   Fi r estei n  GS,   et  al .   eds.   Kel l ey 's  tex tbook   of
r heum atol ogy ,   7th  ed.   Phi l adel phi a:  El sev i er ,   Saunder s,   2005:1336â
€“1342.

Watts  RA,   Scott  DGI.   Ov er v i ew   of   the  i nf l am m ator y   v ascul ar   di seases.


In:  Hochber g  MC,   Si l m an  AJ,   Sm ol en  JS,   et  al .   eds.   Rheum atol ogy ,   3r d
ed.   Edi nbur gh:  Mosby ,   2003:1583–1591.

You might also like